Management accounting 9788131731789, 8131731782

2,631 184 5MB

English Pages xiii, 781 pages; 23 cm [808] Year 2011

Report DMCA / Copyright

DOWNLOAD FILE

Polecaj historie

Management accounting
 9788131731789, 8131731782

Table of contents :
Cover......Page 1
Management Accounting......Page 4
Copyright......Page 5
Contents......Page 8
Preface......Page 16
Definition of Management Accounting......Page 18
Scope of Management Accounting......Page 19
Objectives of Management Accounting......Page 20
Role or Functions of Management Accounting......Page 21
Utility of Management Accounting......Page 22
Limitations of Management Accounting......Page 23
Tools and Techniques of Management Accounting......Page 24
Management Accounting and Financial Accounting......Page 25
Chapter Review Summary......Page 26
Chapter Review Quiz......Page 27
Exercise......Page 28
Characteristics of Financial Statements......Page 29
Income Statement or Profit & Loss Account......Page 30
Statement of Changes in Financial Position......Page 31
Relationship Between Income Statement and Balance Sheet......Page 32
Objectives of Financial Statement Analysis......Page 33
Traditional Approach to Financial Statement Analysis......Page 34
Types of Financial Statement Analysis......Page 35
Techniques/Tools of Financial Statement Analysis......Page 36
Concept of Comparative Financial Statement Analysis......Page 37
Concept of Common-size Financial Statement Analysis......Page 38
Disadvantages of Common-size Statement......Page 39
Disadvantages of Trend Analysis......Page 40
Worked-out Problems......Page 41
Chapter Review Quiz......Page 65
Exercise......Page 66
What is a Ratio?......Page 74
Role or Importance of Ratio Analysis......Page 75
Limitations of Ratio Analysis......Page 76
Different Kinds of Ratios or Classification of Ratios......Page 77
Computation of Proprietors’ Fund, Capital Employed, Working Capital......Page 78
Balance Sheet Ratios......Page 82
Revenue Statement Ratio......Page 85
Mixed or Composite Ratio......Page 88
DU Pont Analysis......Page 92
Ratios to be used in Determining Solvency, Performance, Managerial Efficiency etc. of an Enterprise......Page 93
Different Ratios at Glance......Page 94
Worked-out Problems......Page 96
Chapter Review Summary......Page 194
Chapter Review Quiz......Page 195
Exercise......Page 196
Difference Between Cash and Fund......Page 216
Uses of Cash Flow Statement......Page 217
Limitations of Cash Flow Statement......Page 218
Differences Between Cash Flow Statement and Fund Flow Statement......Page 219
Cash Flow Statement as Prescribed in AS-3......Page 220
Cash Flows from Investing Activities......Page 221
Cash Flows from Financing Activities......Page 222
Proforma of Cash Flow Statement under Indirect Method......Page 223
Fundamental Differences Between Cash Flow Statement as per AS-3 and Traditional Method......Page 224
Worked-out Problems......Page 225
Chapter Review Summary......Page 331
Chapter Review Quiz......Page 332
Exercise......Page 334
Generally Accepted Concept of Fund......Page 358
Limitations of Fund Flow Statement......Page 359
What is Fund from Operation?......Page 360
Direct Approach of Computation of Fund from Operation......Page 361
Is Depreciation a Source of Fund?......Page 363
Distinction between Fund Flow Statement and Cash Flow Statement......Page 364
Worked-out Problems......Page 365
Chapter Review Summary......Page 422
Chapter Review Quiz......Page 423
Exercise......Page 424
Need for Working Capital......Page 437
Classification of Working Capital on the Basis of Concept......Page 438
Sources of Temporary Working Capital......Page 439
Working Capital or Operating Cycle......Page 440
Procedure for Estimation/Forecasting of Working Capital......Page 441
Different Approaches of Estimation/Forecasting of Working Capital......Page 442
Worked-out Problems......Page 444
Chapter Review Summary......Page 476
Exercise......Page 477
Budgetary Control......Page 484
Limitations of Budgetary Control......Page 485
Zero-Based Budgeting......Page 486
Classification on the Basis of Period......Page 487
Classification on the Basis of Function or Coverage......Page 488
Worked-out Problems......Page 490
Chapter Review Summary......Page 538
Chapter Review Quiz......Page 539
Exercise......Page 540
Features of Standard Costing......Page 554
Advantages of Standard Costing......Page 555
Comparison between Standard Costing and Budgetary Control......Page 556
Purposes of Variance Analysis......Page 557
Material Variances......Page 558
Labour Variances......Page 560
Variable Overhead Variances......Page 561
Fixed Overhead Variances......Page 563
Sales Value Variances......Page 565
Sales Margin (Profit) Variances......Page 566
Formulae of Various Variances at a Glance......Page 568
Worked-out Problems......Page 573
Chapter Review Summary......Page 619
Chapter Review Quiz......Page 620
Exercise......Page 621
Features of Absorption Costing......Page 632
Variable Costing......Page 633
Limitations of Variable Costing......Page 634
Income Determination under Absorption Costing and Variable Costing......Page 635
Distinction between Absorption Costing and Variable Costing......Page 636
Worked-out Problems......Page 637
Chapter Review Summary......Page 644
Chapter Review Quiz......Page 645
Exercise......Page 646
Concept of Marginal Cost......Page 649
Advantages of Marginal Costing......Page 650
Objectives of CVP Analysis......Page 651
Elements of CVP Analysis......Page 652
Presentation of BE Chart......Page 657
Mathematical Representation of CVP Analysis......Page 658
Different Formulae of CVP Analysis at a Glance......Page 659
Worked-out Problems......Page 661
Chapter Review Summary......Page 706
Chapter Review Quiz......Page 708
Exercise......Page 709
Cost Concepts in Decision Making......Page 720
Concept of Relevant Cost and Relevant Revenue......Page 721
Concept of Key or Limiting Factor......Page 722
Make or Buy Decision......Page 723
Worked-out Problems......Page 724
Chapter Review Summary......Page 749
Exercise......Page 750
Features of Capital Budgeting......Page 760
Types of Capital Budgeting Decisions......Page 761
Concept of Discounted Cash Flow......Page 762
Non-discounted Techniques......Page 763
Discounted Cash Flow Techniques......Page 764
Worked-out Problems......Page 766
Chapter Review Summary......Page 784
Chapter Review Quiz......Page 785
Exercise......Page 786
Meaning and Concept of Responsibility Accounting......Page 792
Advantages or Benefits or Purposes of Responsibility Accounting......Page 793
Profit Centre......Page 794
Responsibility Reporting......Page 795
Chapter Review Summary......Page 796
Exercise......Page 797
Appendix......Page 800

Citation preview

Project: Management Accounting_Debarshi Bhattacharyya File: X:\Pearson\Management Accounting_Debarshi Bhattacharyya\MAIN\A01\A01_DEBA_ISBN_EN_SE_PREL.indd

ACE Pro India Pvt. Ltd.

MANAGEMENT ACCOUNTING

Modified Date: Tue, Jul 06, 2010 12:44:26 PM

Output Date: Tue, Jul 06, 2010 12:44:56 PM REVISES

Project: Management Accounting_Debarshi Bhattacharyya File: X:\Pearson\Management Accounting_Debarshi Bhattacharyya\MAIN\A01\A01_DEBA_ISBN_EN_SE_PREL.indd

ACE Pro India Pvt. Ltd.

This page intentionally left blank

Modified Date: Tue, Jul 06, 2010 12:44:26 PM

Output Date: Tue, Jul 06, 2010 12:44:56 PM REVISES

Project: Management Accounting_Debarshi Bhattacharyya File: X:\Pearson\Management Accounting_Debarshi Bhattacharyya\MAIN\A01\A01_DEBA_ISBN_EN_SE_PREL.indd

ACE Pro India Pvt. Ltd.

MANAGEMENT ACCOUNTING

Debarshi Bhattacharyya Assistant Professor S. R. Fatepuria College University of Kalyani, West Bengal

Modified Date: Tue, Jul 06, 2010 12:44:26 PM

Output Date: Tue, Jul 06, 2010 12:44:56 PM REVISES

Project: Management Accounting_Debarshi Bhattacharyya File: X:\Pearson\Management Accounting_Debarshi Bhattacharyya\MAIN\A01\A01_DEBA_ISBN_EN_SE_PREL.indd

ACE Pro India Pvt. Ltd.

Assistant Acquisitions Editor: Anshul Yadav Assistant Production Editor: Barun Kumar Sarkar Compositor: Ace Pro India Pvt. Ltd Copyright © 2011 Dorling Kindersley (India) Pvt. Ltd. This book is sold subject to the condition that it shall not, by way of trade or otherwise, be lent, resold, hired out, or otherwise circulated without the publisher’s prior written consent in any form of binding or cover other than that in which it is published and without a similar condition including this condition being imposed on the subsequent purchaser and without limiting the rights under copyright reserved above, no part of this publication may be reproduced, stored in or introduced into a retrieval system, or transmitted in any form or by any means (electronic, mechanical, photocopying, recording or otherwise), without the prior written permission of both the copyright owner and the publisher of this book. ISBN: 978-81-317-3178-9 10 9 8 7 6 5 4 3 2 1 Published by Dorling Kindersley (India) Pvt. Ltd., licensees of Pearson Education in South Asia. Head Office: 7th Floor, Knowledge Boulevard, A-8(A), Sector-62, Noida 201309, India Registered Office: 11 Community Centre, Panchsheel Park, New Delhi 110017, India

Modified Date: Tue, Jul 06, 2010 12:44:26 PM

Output Date: Tue, Jul 06, 2010 12:44:56 PM REVISES

Project: Management Accounting_Debarshi Bhattacharyya File: X:\Pearson\Management Accounting_Debarshi Bhattacharyya\MAIN\A01\A01_DEBA_ISBN_EN_SE_PREL.indd

ACE Pro India Pvt. Ltd.

This book is dedicated at the feet of my parents the late Dhurjati Prasad Bhattacharyya and the late Aparna Bhattacharyya who are in front of me round the clock with their blessings.

Modified Date: Tue, Jul 06, 2010 12:44:26 PM

Output Date: Tue, Jul 06, 2010 12:44:56 PM REVISES

Project: Management Accounting_Debarshi Bhattacharyya File: X:\Pearson\Management Accounting_Debarshi Bhattacharyya\MAIN\A01\A01_DEBA_ISBN_EN_SE_PREL.indd

ACE Pro India Pvt. Ltd.

This page intentionally left blank

Modified Date: Tue, Jul 06, 2010 12:44:26 PM

Output Date: Tue, Jul 06, 2010 12:44:56 PM REVISES

Project: Management Accounting_Debarshi Bhattacharyya File: X:\Pearson\Management Accounting_Debarshi Bhattacharyya\MAIN\A01\A01_DEBA_ISBN_EN_SE_PREL.indd

ACE Pro India Pvt. Ltd.

Contents Preface

1

xv

Introduction to Management Accounting 1 1.1 Definition of Management Accounting 1 1.2 Nature or Features of Management Accounting 2 1.3 Scope of Management Accounting 2 1.4 Objectives of Management Accounting 3 1.5 Role or Functions of Management Accounting 4 1.6 Utility of Management Accounting 5 1.7 Limitations of Management Accounting 6 1.8 Tools and Techniques of Management Accounting 7 1.9 Requisites for Installation of Management Accounting System 8 1.10 How Does Management Accounting Differ from the other Branches of Accounting 8 1.10.1 Management Accounting and Financial Accounting 8 1.10.2 Management Accounting and Cost Accounting 9 Chapter Review Summary 9 • Chapter Review Quiz 10 • Exercise 11

2 Analysis and Interpretation of Financial Statements 12 2.1 Meaning of Financial Statements 12 2.2 Characteristics of Financial Statements 12 2.3 Anatomy/Components of Financial Statements 13 2.3.1 Income Statement or Profit & Loss Account 13 2.3.2 Balance Sheet 14 2.3.3 Statement of Changes in Financial Position 14 2.4 Relationship Between Income Statement and Balance Sheet 15 2.5 Concept of Financial Statement Analysis 16 2.6 Objectives of Financial Statement Analysis 16 2.7 Requisites of Financial Statement Analysis 17 2.8 Different Approaches to Financial Statement Analysis 17 2.8.1 Traditional Approach to Financial Statement Analysis 17 2.8.2 Modern Approach to Financial Statement Analysis 18 2.9 Types of Financial Statement Analysis 18 2.10 Techniques/Tools of Financial Statement Analysis 19 2.11 Comparative Financial Statement Analysis 20 2.11.1 Concept of Comparative Financial Statement Analysis 20 2.11.2 Advantages of Comparative Financial Statement Analysis 21 2.11.3 Disadvantages of Comparative Financial Statement Analysis 21

Modified Date: Tue, Jul 06, 2010 12:44:26 PM

Output Date: Tue, Jul 06, 2010 12:44:56 PM REVISES

Project: Management Accounting_Debarshi Bhattacharyya File: X:\Pearson\Management Accounting_Debarshi Bhattacharyya\MAIN\A01\A01_DEBA_ISBN_EN_SE_PREL.indd

viii

ACE Pro India Pvt. Ltd.

CONTENTS

2.12 Common-size Financial Statement Analysis 21 2.12.1 Concept of Common-size Financial Statement Analysis 21 2.12.2 Advantages of Common-size Statement 22 2.12.3 Disadvantages of Common-size Statement 22 2.12.4 Comparative Statement vs. Common-size Statement 23 2.13 Trend Analysis 23 2.13.1 Concept of Trend Analysis 23 2.13.2 Steps in Computing Trend Values (or Percentages) 23 2.13.3 Advantages of Trend Analysis 23 2.13.4 Disadvantages of Trend Analysis 23 2.14 Worked-out Problems 24 Chapter Review Summary 48 • Chapter Review Quiz 48 • Exercise

49

3 Accounting Ratios for Financial Statement Analysis 57 3.1 Financial Statements 57 3.2 What is a Ratio? 57 3.3 What is an Accounting Ratio? 58 3.4 What is Ratio Analysis? 58 3.5 Steps in Ratio Analysis 58 3.6 Role or Importance of Ratio Analysis 58 3.7 Uses of Ratio Analysis 59 3.8 Advantages of Ratio Analysis 59 3.9 Limitations of Ratio Analysis 59 3.10 Parties Interested in Ratio Analysis 60 3.11 Different Kinds of Ratios or Classification of Ratios 60 3.12 Standard or Ideal Ratio 61 3.13 Computation of Proprietors’ Fund, Capital Employed, Working Capital 61 3.14 Calculation and Interpretation of Various Ratios 65 3.14.1 Balance Sheet Ratios (Ratios as ascertained from various items of the Balance Sheet) 65 3.14.2 Revenue Statement Ratio (Ratios as ascertained from the various items of the Profit & Loss A/c) 68 3.14.3 Mixed or Composite Ratio (Ratio ascertained taking some items from the Balance Sheet and some other items from the Profit & Loss A/c) 71 3.15 Du Pont Analysis 75 3.16 Ratios to be used in Determining Solvency, Performance, Managerial Efficiency etc. of an Enterprise 76 3.17 Different Ratios at Glance 77 3.18 Worked-out Problems 79 Chapter Review Summary 177 • Chapter Review Quiz 178 • Exercise 179

Modified Date: Tue, Jul 06, 2010 12:44:26 PM

Output Date: Tue, Jul 06, 2010 12:44:56 PM REVISES

Project: Management Accounting_Debarshi Bhattacharyya File: X:\Pearson\Management Accounting_Debarshi Bhattacharyya\MAIN\A01\A01_DEBA_ISBN_EN_SE_PREL.indd

ACE Pro India Pvt. Ltd.

CONTENTS

4

Cash Flow Analysis 199 4.1 What is Cash? 199 4.2 Difference Between Cash and Fund 199 4.3 What is Cash Flow Statement? 200 4.4 Why is Cash Flow Statement Prepared? 200 4.5 Importance of Cash Flow Statement 200 4.6 Uses of Cash Flow Statement 200 4.7 Advantages of Cash Flow Statement 201 4.8 Limitations of Cash Flow Statement 201 4.9 Differences Between Cash Book and Cash Flow Statement 202 4.10 Differences Between Cash Flow Statement and Fund Flow Statement 202 4.11 Proforma of Cash Flow Statement Prepared under Traditional/Conventional Method 203 4.12 Cash Flow Statement as Prescribed in AS-3 203 4.12.1 Cash Flows from Operating Activities 204 4.12.2 Cash Flows from Investing Activities 204 4.12.3 Cash Flows from Financing Activities 205 4.13 Proforma of Cash Flow Statement as Prescribed in AS-3 206 4.13.1 Proforma of Cash Flow Statement under Direct Method [Paragraph (a) Page II – 910] 206 4.13.2 Proforma of Cash Flow Statement under Indirect Method [Paragraph (b) Page II – 910] 206 4.14 Fundamental Differences Between Cash Flow Statement as per AS-3 and Traditional Method 207 4.15 Worked-out Problems 208 Chapter Review Summary 314 • Chapter Review Quiz 315 • Exercise 317

5

Fund Flow Analysis 341 5.1 Meaning and Concept of Fund 341 5.2 Generally Accepted Concept of Fund 341 5.3 Meaning of Flow of Fund 342 5.4 What is Fund Flow Statement? 342 5.5 Importance or Purposes of Fund Flow Statement 342 5.6 Limitations of Fund Flow Statement 342 5.7 Proforma of Presentation of Fund Flow Statement 343 5.8 Different Sources of Fund 343 5.9 Different Applications of Fund 343 5.10 What is Fund from Operation? 343 5.11 Different Approaches of Computation of Fund from Operation 344 5.11.1 Direct Approach of Computation of Fund from Operation 344 5.11.2 Indirect Approach of Computation of Fund from Operation 344

Modified Date: Tue, Jul 06, 2010 12:44:26 PM

ix

Output Date: Tue, Jul 06, 2010 12:44:56 PM REVISES

Project: Management Accounting_Debarshi Bhattacharyya File: X:\Pearson\Management Accounting_Debarshi Bhattacharyya\MAIN\A01\A01_DEBA_ISBN_EN_SE_PREL.indd

x

ACE Pro India Pvt. Ltd.

CONTENTS

5.12 5.13 5.14 5.15

Is Depreciation a Source of Fund? 346 Comparison between Fund Flow Statement, Income Statement and Balance Sheet 347 Distinction between Fund Flow Statement and Cash Flow Statement 347 Worked-out Problems 348 Chapter Review Summary 405 • Chapter Review Quiz 406 • Exercise 407

6 Working Capital 420 6.1 Meaning and Concept of Working Capital 420 6.2 Importance of Working Capital 420 6.3 Need for Working Capital 420 6.4 Different Concepts and Classification of Working Capital 421 6.4.1 Classification of Working Capital on the Basis of Concept 421 6.4.2 Classification of Working Capital on the Basis of Time 422 6.5 Sources of Working Capital 422 6.5.1 Sources of Permanent Working Capital 422 6.5.2 Sources of Temporary Working Capital 422 6.6 Determinants of Working Capital 423 6.7 Components of Working Capital 423 6.8 Positive and Negative Working Capital 423 6.9 Working Capital or Operating Cycle 423 6.10 Estimation or Forecasting of Working Capital 424 6.11 Procedure for Estimation/Forecasting of Working Capital 424 6.12 Valuation of Stock of Work-In-Progress (WIP) 425 6.13 Different Approaches of Estimation/Forecasting of Working Capital 425 6.14 Worked-out Problems 427 Chapter Review Summary 459 • Chapter Review Quiz 460 • Exercise 460 7

Budget and Budgetary Control 467 7.1 Meaning of Budget 467 7.2 Meaning of Budgeting 467 7.3 Features of Budget 467 7.4 Budgetary Control 467 7.5 Objectives of Budgetary Control 468 7.6 Advantages of Budgetary Control 468 7.7 Limitations of Budgetary Control 468 7.8 Comparison between Budget and Budgetary Control 469 7.9 Comparison between Standard Costing and Budgetary Control 469 7.10 Zero-Based Budgeting (ZBB) 469

Modified Date: Tue, Jul 06, 2010 12:44:26 PM

Output Date: Tue, Jul 06, 2010 12:44:56 PM REVISES

Project: Management Accounting_Debarshi Bhattacharyya File: X:\Pearson\Management Accounting_Debarshi Bhattacharyya\MAIN\A01\A01_DEBA_ISBN_EN_SE_PREL.indd

ACE Pro India Pvt. Ltd.

CONTENTS

7.11 Classification or Types of Budget 470 7.11.1 Classification on the Basis of Period 470 7.11.2 Classification on the Basis of Flexibility of Production 471 7.11.3 Classification on the Basis of Function or Coverage 471 Worked-out Problems 473 Chapter Review Summary 521 • Chapter Review Quiz 522 •

Exercise

xi

523

8 Standard Costing and Variance Analysis 537 8.1 What is Standard Cost? 537 8.2 What is Standard Costing? 537 8.2.1 Features of Standard Costing 537 8.3 Preliminary Steps for Establishing Standard Costing System 538 8.4 Types of Standards 538 8.5 Distinction between Standard Cost, Estimated Cost and Actual/Historical Cost 538 8.6 Advantages of Standard Costing 538 8.7 Limitations of Standard Costing 539 8.8 Comparison between Standard Costing and Budgetary Control 539 8.9 Analysis of Variance 540 8.9.1 Direction of Variances 540 8.9.2 Nature of Variances 540 8.9.3 Purposes of Variance Analysis 540 8.10 Classification of Variances 541 8.10.1 Material Variances 541 8.10.2 Labour Variances 543 8.10.3 Variable Overhead Variances 544 8.10.4 Fixed Overhead Variances 546 8.10.5 Sales Value Variances 548 8.10.6 Sales Margin (Profit) Variances 549 8.11 Formulae of Various Variances at a Glance 551 8.12 Worked-out Problems 556 Chapter Review Summary 602 • Chapter Review Quiz 603 • Exercise 604 9

Absorption Costing and Variable Costing 9.1 Absorption Costing 615 9.2 Features of Absorption Costing 615 9.3 Advantages of Absorption Costing 616 9.4 Limitations of Absorption Costing 616 9.5 Variable Costing 616

615

Modified Date: Tue, Jul 06, 2010 12:44:26 PM

Output Date: Tue, Jul 06, 2010 12:44:56 PM REVISES

Project: Management Accounting_Debarshi Bhattacharyya File: X:\Pearson\Management Accounting_Debarshi Bhattacharyya\MAIN\A01\A01_DEBA_ISBN_EN_SE_PREL.indd

xii

ACE Pro India Pvt. Ltd.

CONTENTS

9.6 9.7 9.8 9.9

Features of Variable Costing 617 Advantages of Variable Costing 617 Limitations of Variable Costing 617 Income Determination under Absorption Costing and Variable Costing 618 9.9.1 Income Determination under Absorption Costing Technique 619 9.9.2 Income Determination under Variable Costing Technique 619 9.10 Distinction between Absorption Costing and Variable Costing 619 9.11 Worked-out Problems 620 Chapter Review Summary 627 • Chapter Review Quiz 628 • Exercise

10

11

Marginal Costing and Cost–Volume–Profit Analysis 632 10.1 Concept of Marginal Cost 632 10.2 Definition of Marginal Costing 633 10.3 Features of Marginal Costing 633 10.4 Advantages of Marginal Costing 633 10.5 Limitations of Marginal Costing 634 10.6 Meaning of Cost–Volume–Profit (CVP) Analysis 634 10.7 Features of CVP Analysis 634 10.8 Objectives of CVP Analysis 634 10.9 Assumptions Underlying CVP Analysis 635 10.10 Classification of Cost Under CVP Analysis 635 10.11 Elements of CVP Analysis 635 10.12 Break-Even Analysis (BE Analysis) 640 10.13 Break-Even Chart (BE Chart) 640 10.13.1 Presentation of BE Chart 640 10.14 Angle of Incidence 641 10.15 Mathematical Representation of CVP Analysis 641 10.16 Different Formulae of CVP Analysis at a Glance 642 10.17 Worked-out Problems 644 Chapter Review Summary 689 • Chapter Review Quiz 691



Exercise

629

692

Decision Making 703 11.1 Concept of Decision Making 703 11.2 Steps in Decision Making 703 11.3 Cost Concepts in Decision Making 703 11.4 Concept of Relevant Cost and Relevant Revenue 704 11.5 Comparison of Differential Cost Analysis and Marginal Costing 705 11.6 Concept of Key or Limiting Factor 705 11.7 Different Situations of Decision Making 706 11.7.1 Determination of Most Profitable Product/Sales Mix 706

Modified Date: Tue, Jul 06, 2010 12:44:26 PM

Output Date: Tue, Jul 06, 2010 12:44:56 PM REVISES

Project: Management Accounting_Debarshi Bhattacharyya File: X:\Pearson\Management Accounting_Debarshi Bhattacharyya\MAIN\A01\A01_DEBA_ISBN_EN_SE_PREL.indd

ACE Pro India Pvt. Ltd.

CONTENTS

11.7.2 Accept or Reject Decision 706 11.7.3 Make or Buy Decision 706 11.7.4 Operate or Shut-Down Decision 707 11.8 Worked-out Problems 707 Chapter Review Summary 732 • Chapter Review Quiz

733



Exercise

xiii

733

12

Capital Budgeting 743 12.1 Concept of Capital Budgeting 743 12.2 Features of Capital Budgeting 743 12.3 Importance of Capital Budgeting 744 12.4 Types of Investments 744 12.5 Types of Capital Budgeting Decisions 744 12.6 Evaluation Criteria of Capital Budgeting Decisions 745 12.7 Concept of Present Value (PV) 745 12.8 Determination of PV Factor or Discounting Factor 745 12.9 Concept of Discounted Cash Flow 745 12.10 Capital Budgeting Techniques 746 12.10.1 Non-discounted Techniques (Without Considering the Time Value of Money) 746 12.10.2 Discounted Cash Flow Techniques (Time-Adjusted Techniques) 747 12.11 Worked-out Problems 749 Chapter Review Summary 767 • Chapter Review Quiz 768 • Exercise 769

13

Responsibility Accounting 775 13.1 Meaning and Concept of Responsibility Accounting 775 13.2 Prerequisites of Responsibility Accounting 776 13.3 Assumptions of Responsibility Accounting 776 13.4 Advantages or Benefits or Purposes of Responsibility Accounting 776 13.5 Limitations of Responsibility Accounting 777 13.6 Responsibility Centre 777 13.7 Types of Responsibility Centres 777 13.7.1 Cost Centre 777 13.7.2 Profit Centre 777 13.7.3 Investment Centre 778 13.8 Controllable and Non-controllable Costs 778 13.9 Responsibility Reporting 778 13.10 Types of Responsibility Reports 779 Chapter Review Summary 779 • Chapter Review Quiz 780 • Exercise 780 Appendix 783

Modified Date: Tue, Jul 06, 2010 12:44:26 PM

Output Date: Tue, Jul 06, 2010 12:44:56 PM REVISES

Project: Management Accounting_Debarshi Bhattacharyya File: X:\Pearson\Management Accounting_Debarshi Bhattacharyya\MAIN\A01\A01_DEBA_ISBN_EN_SE_PREL.indd

ACE Pro India Pvt. Ltd.

This page intentionally left blank

Modified Date: Tue, Jul 06, 2010 12:44:26 PM

Output Date: Tue, Jul 06, 2010 12:44:56 PM REVISES

Project: Management Accounting_Debarshi Bhattacharyya File: X:\Pearson\Management Accounting_Debarshi Bhattacharyya\MAIN\A01\A01_DEBA_ISBN_EN_SE_PREL.indd

ACE Pro India Pvt. Ltd.

Preface I am delighted to introduce Management Accounting before the undergraduate students of commerce. This book covers the latest syllabi of B.Com. courses of almost all Indian universities. It presents conceptual and theoretical aspects of Management Accounting in a simple, lucid and comprehensive manner, together with a variety of problems, their solutions, and exercises. I have explained the fundamental aspects of the subject for beginners with mathematical examples. Every topic has been discussed from the basic level of the concept to the highest possible standard as permissible for students at this level. The highlight of the book is the pedagogical elements such as chapter review quiz and exercises at the end of every chapter. It also contains numerous theoretical questions and practical problems, so that students can assess their levels of performance. I firmly believe that this book will meet the varied requirements of students. My apologies for any error that might have crept into the book in spite of my best efforts. I assure you that I will amend any such errors, if any, at the earliest opportunity. I welcome suggestions for the improvement of the quality of the book and may be reached at [email protected]. I express my deepest sense of gratitude to innumerable teachers, friends and well-wishers who inspired me round the clock with their valuable guidance and suggestions. I express my gratitude to Anshul Yadav of Pearson Education (India), without whose constant support and cooperation this book would never have seen the light of day. I also express my indebtedness to Raza Khan, Praveen Tiwari and Barun Kumar Sarkar, also of Pearson Education (India), who extended their cooperation for the early publication of the book. My heartiest gratitude to Sarbani Roy Bhattacharyya and Archisman Bhattacharyya, without whose great sacrifice in my personal life this book would not have been possible. I believe that my honest endeavour will be amply rewarded if this book proves useful to students of all merits and qualities for whom it is intended.

Debarshi Bhattacharyya

Modified Date: Tue, Jul 06, 2010 12:44:26 PM

Output Date: Tue, Jul 06, 2010 12:44:56 PM REVISES

Project: Management Accounting_Debarshi Bhattacharyya File: X:\Pearson\Management Accounting_Debarshi Bhattacharyya\MAIN\A01\A01_DEBA_ISBN_EN_SE_PREL.indd

ACE Pro India Pvt. Ltd.

This page intentionally left blank

Modified Date: Tue, Jul 06, 2010 12:44:26 PM

Output Date: Tue, Jul 06, 2010 12:44:56 PM REVISES

Project: Management Accounting_Debarshi Bhattacharyya ACE Pro India Pvt. Ltd. File: X:\Pearson\Management Accounting_Debarshi Bhattacharyya\MAIN\M01\LAYOUT_M01\M01_DEBA_ISBN_EN_SE_C01.indd

Introduction to Management Accounting

1

LEARNING OBJECTIVES On completion of the study of the chapter, you should be able to understand: What is Management Accounting? Nature, scope, objectives, functions, utilities and limitations of Management Accounting. Tools and techniques of Management Accounting. Requisites for installation of Management Accounting System. Distinction between Management Accounting and Financial Accounting. Distinction between Management Accounting and Cost Accounting.

1.1 DEFINITION OF MANAGEMENT ACCOUNTING Management Accounting is that branch of accounting which deals with presenting and providing accounting information to the management in such a systematic way so that it can perform its managerial functions of planning, controlling and decision-making in an effective and efficient manner. It acts as a ‘decision-making support system’ to the management. Many distinguished accountants and accounting institutions have defined the term Management Accounting in different languages. Some of them are enumerated as follows: According to Robert N. Anthony, ‘Management Accounting is concerned with accounting information which is useful to the management’. ‘Management Accounting is a term used to describe the accounting methods, system and techniques which, coupled with special knowledge and ability, assists management in its task of maximizing profits or minimizing losses’, according to J. Batty. As per Institute of Cost and Works Accountants of India (ICWAI), ‘Management Accounting is a system of collection and presentation of relevant economic information relating to an enterprise for planning, controlling and decision-making’. According to American Accounting Association (AAA), ‘Management Accounting is the application of appropriate techniques and concepts in processing historical and projected economic data of an entry to assist management in establishing plans for reasonable economic objectives in the making of rational decisions with a view towards these objectives’. As per Certified Institute of Management Accountants (CIMA), United Kingdom, ‘Management Accounting is an integral part of management concerned with identifying, presenting and interpreting information used for formulating strategy, planning and controlling activities, decision-making, optimizing the use of resources, disclosure to shareholders and other external to the entity, disclosure to employees, and safeguarding assets’. Analysing the above definitions, it can be concluded that Management Accounting is concerned with collection of data from both internal as well as external sources and communication of relevant information to the management, after processing, analysing and interpreting those, to perform its managerial functions of planning, controlling and decision-making in an effective and efficient manner.

Modified Date: Tue, Jun 01, 2010 06:12:23 PM

Output Date: Tue, Jul 06, 2010 11:37:07 AM

Rev-III

Project: Management Accounting_Debarshi Bhattacharyya ACE Pro India Pvt. Ltd. File: X:\Pearson\Management Accounting_Debarshi Bhattacharyya\MAIN\M01\LAYOUT_M01\M01_DEBA_ISBN_EN_SE_C01.indd

2

MANAGEMENT ACCOUNTING

1.2 NATURE OR FEATURES OF MANAGEMENT ACCOUNTING Management Accounting is basically the most effective tool to the management for the purpose of its planning, controlling and decision-making. Various distinguished nature or features of Management Accounting are discussed as follows: i. Management Accounting deals with the collection of accounting and other data, and it analyses, interprets and communicates all relevant information to the management which are effectively required for planning, controlling and decision-making for the organization. Hence, in a nutshell, one of the most important nature of Management Accounting is to analyse and interpret accounting and other data to make them understandable and usable to the management. ii. It interprets the analysed data as obtained from the operational and non-operational activities of the enterprise and makes necessary comments and conclusions on them. iii. It provides necessary information to the management to judge the effectiveness of its managerial functions. iv. It provides necessary information to the management to perform its managerial functions of decisionmaking, planning and controlling. v. It provides necessary information to the management to review whether the performance of the enterprise has been achieved towards its goals and objectives. vi. It acts as a yardstick for measuring the level of performance of the operational and non-operational activities of the enterprise. vii. It serves as a yardstick for measuring the effectiveness of managerial performance of the different activities of the enterprise. viii. It is a forward-looking tool to the management. It analyses and interprets historical data for projecting the future trends of the different activities of the enterprise. ix. It prepares necessary plans to implement various financial decisions of the management. It also develops a system of feedback reporting and monitoring performance against the plan of the management. 1.3 SCOPE OF MANAGEMENT ACCOUNTING Management Accounting has a very widespread scope. It covers a very wide area of accounting system, which is discussed as follows: i. Financial Accounting: Financial Accounting provides the basic historical data to the Management Accounting which analyses and interprets those data and provides necessary information to the management for its planning, controlling and decision-making. As Management Accounting does not maintain the basic financial records, the success of an effective and efficient Management Accounting System depends on the existence of an effective Financial Accounting System. Therefore, Management Accounting System can be introduced into an organization where there exists a well-designed Financial Accounting System. Management Accounting applies the principles and practices of Financial Accounting. ii. Cost Accounting: On the one hand, Cost Accounting provides cost-related basic data to the Management Accounting, which analyses and interprets those costing data and provides necessary information to the management for the purpose of its controlling and decision-making. On the other hand, most of the Cost Accounting techniques like Standard Costing, Budgetary Control, Marginal Costing, Cost–Volume–Profit (CVP) Analysis, Differential Cost Analysis and Inventory Controlling, are used by Management Accounting in its process of planning, controlling and decision-making. Management Accounting uses the principles and practices of Cost Accounting. iii. Forecasting and budgeting: Management Accounting exercises the tool of forecasting and budgeting in the process of planning, controlling and decision-making. Forecasting makes an estimate of the probable event with a set of given or assumed information. Budgeting prepares a number of plans for any future project by setting definite goals. Forecasting helps to prepare the budget and budgeting

Modified Date: Tue, Jun 01, 2010 06:12:23 PM

Output Date: Tue, Jul 06, 2010 11:37:07 AM

Rev-III

Project: Management Accounting_Debarshi Bhattacharyya ACE Pro India Pvt. Ltd. File: X:\Pearson\Management Accounting_Debarshi Bhattacharyya\MAIN\M01\LAYOUT_M01\M01_DEBA_ISBN_EN_SE_C01.indd

INTRODUCTION TO MANAGEMENT ACCOUNTING

iv.

v.

vi.

vii.

viii.

ix.

x.

xi.

xii.

3

helps to exercise the budgetary control technique on future projects. Both these tools are frequently used in Management Accounting. Statistical tools: Various statistical tools like graphs, charts, diagrams, time series, sampling, index numbers and Regression Analysis are used in Management Accounting in the process of planning, controlling and decision-making. Operational research techniques: Various operational research techniques like Linear Programming, Transportation Theory, Games Theory and Simulation Method are used in Management Accounting to resolve various problems prevailing under the existing situation in the process of decision-making. Financial analysis and interpretation: Various financial analysis techniques such as Ratio Analysis, Fund Flow Analysis, Cash Flow Analysis, Comparative Financial Statement, Common-Size Statement and Trend Analysis are widely used in Management Accounting to analyse and interpret financial data to make them easily understandable and useable to the management. Successful application of Management Accounting depends a lot on these financial analysis and interpretation works. Tax accounting and tax planning: Determination of taxable income and tax liability of the enterprise fall within the purview of the Management Accounting. In the process of decision-making, the analysis of implication of tax provisions on future projects also falls within the purview of Management Accounting. On the other hand, the management accountant must have a vast knowledge of tax laws and their accounting procedures, and also tax planning, to minimize the tax burden of the enterprise. Management Information System (MIS): Management Information System (MIS) is a modern computerized information system, by which accurate processing and analysis of a large volume of data can be done within a very short time. This information system is used in Management Accounting to provide necessary and relevant information to the management in the process of its planning, controlling and decision-making. Internal control and internal audit: Management Accounting highly depends on internal control system existing in the organization, like internal check and internal audit, to appraise the targeted performance and to identify the weaker area of the organization. Office system: Management Accounting System should also be well conversant with the modern office management system like filing, indexing, copying, electronic data processing, information network system, and email and fax system. Legal provisions: Management Accounting System should also be well informed about relevant and necessary legal provisions like Companies Act, Foreign Exchange Act, Securities Act, and Direct and Indirect Tax Laws. In the process of decision-making, management accountants should restrict their plan and action within the periphery of such legal provisions. Other areas: Apart from the aforementioned areas, Management Accounting also includes various newly developed areas of accounting like Human Resource Accounting, Social Accounting, Environmental Accounting and Inflation Accounting, within the purview of its scope.

1.4 OBJECTIVES OF MANAGEMENT ACCOUNTING The prime objective of Management Accounting is to provide necessary information to the management for an effective and efficient execution of managerial functions. Various objectives of Management Accounting are enumerated as follows: i. Analysis and interpretation of financial statements: Management Accounting collects, analyses and interprets the necessary data from the results shown by the Financial and Cost Accounting System, and also provides necessary and relevant information to the management in a systematic and useful manner which are to be applied by the management in the process of its planning, controlling and decision-making. Various tools like Ratio Analysis, Fund Flow Analysis, Cash Flow Analysis, Comparative

Modified Date: Tue, Jun 01, 2010 06:12:23 PM

Output Date: Tue, Jul 06, 2010 11:37:07 AM

Rev-III

Project: Management Accounting_Debarshi Bhattacharyya ACE Pro India Pvt. Ltd. File: X:\Pearson\Management Accounting_Debarshi Bhattacharyya\MAIN\M01\LAYOUT_M01\M01_DEBA_ISBN_EN_SE_C01.indd

4

MANAGEMENT ACCOUNTING

ii.

iii.

iv.

v.

vi.

vii.

viii.

Financial Statement, Common-Size Statement and Trend Analysis are widely used in Management Accounting for analysing and interpreting those data so as to make them easily understandable and useable to the management. Planning and policy-making: Management Accounting provides necessary and relevant information to the management in the process of its planning and policy-making to achieve organizational goals. Various statistical forecasting techniques like Time-Series Analysis and Regression Analysis are used in Management Accounting to guide proper planning and policy-making. Decision-making: Management Accounting provides necessary and relevant information to the management in the process of its decision-making. The success of the management highly depends upon a perfect decision-making. Such decision-making broadly depends on the effectiveness of information network. Management Accounting provides the above information to the management by applying Marginal Costing Technique, Differential Costing Technique and Absorption Costing Technique, for an effective and accurate decision-making. Controlling: Management Accounting applies various useful techniques such as Standard Costing, Budgetary Control, Responsibility Accounting and Management Audit, to ensure an effective managerial control over the use of resources of the enterprise. Management control is a control system which assures that the resources of the enterprise are effectively and efficiently used for achieving its goals and objectives. Management Accounting plays a significant role to the management in ensuring the existence of a proper managerial control system. Communicating: Proper communication of the performance of various sections of an enterprise to different levels of management is essentially required for planning, controlling and decision-making. Management Accounting does such communication by preparing reports of performance of various sections of the enterprise with the help of management information system. Coordinating: Management Accounting helps in coordinating various business activities of an enterprise. Its techniques of planning make a very good coordination between various activities of a concern. A master budget of the concern for a given period is prepared through coordination between various business activities of the concern. Proper reporting of different business activities are also made through coordination between various sections of the enterprise. Tax planning: Determination of tax liability of the enterprise after availing various tax rebates and reliefs falls within the purview of Management Accounting System. Management Accounting helps the management in the process of tax planning by availing various tax rebates and reliefs and, thus, reduces the burden of tax of the enterprise, on the whole. Advisory Service: Management Accounting renders valuable advice to the management for resolving any financial or other problems of the enterprise. To overcome any existing financial and other problems, various Management Accounting techniques are applied according to the nature of the problem. Management Accounting also plays a very important role as an advisor to the management.

1.5 ROLE OR FUNCTIONS OF MANAGEMENT ACCOUNTING The function of Management Accounting is to assist the management to perform its functions of planning, organizing, directing, controlling and decision-making. The major functions of Management Accounting are as follows: i. Collection of data: Management Accounting does not maintain records of financial and cost data but it collects the basic financial data mainly from the records as maintained by financial and Cost Accounting for the purpose of preparing plans and actions of the management. ii. Supply of modified data: It modifies the collected raw data by classifying and compiling them for making them suitable for the purpose of their analysis and interpretation. It extracts the necessary and effective information from those basic data for the purpose of their analysis and interpretation.

Modified Date: Tue, Jun 01, 2010 06:12:23 PM

Output Date: Tue, Jul 06, 2010 11:37:07 AM

Rev-III

Project: Management Accounting_Debarshi Bhattacharyya ACE Pro India Pvt. Ltd. File: X:\Pearson\Management Accounting_Debarshi Bhattacharyya\MAIN\M01\LAYOUT_M01\M01_DEBA_ISBN_EN_SE_C01.indd

INTRODUCTION TO MANAGEMENT ACCOUNTING

5

iii. Analysis and interpretation of data: It analyses and interprets those modified data and extracts the necessary and effective information for making them understandable and useable to the management in the process of its planning, controlling and decision-making. After analysing and interpreting all those data, it presents the results with necessary comments to the management. iv. Planning and forecasting: It formulates some definite plans for implementing policies of the management. It helps the management for formulating different short- and long-term policies by providing necessary and relevant information in relation thereto. In the process of formulating policies and for their proper execution, it makes the forecasting of some probable future happenings. v. Communication: It provides a means of communicating plans and actions of the management over all areas of activities of the organization. It provides necessary and relevant information to all levels of management. vi. Ensuring control: It ensures a control over the performance of different sections of an enterprise. It uses various techniques like Budgetary Control, Standard Costing and Responsibility Accounting, and to identify the weaker areas of performance of activity and suggests appropriate remedial measures to overcome the prevailing problems. vii. Helping in decision-making: It helps the management in the process of its effective decision-making by providing necessary and relevant information in the relation thereto. viii. Performance evaluation: It evaluates the performance of activities of different divisions as well as the business as a whole of an enterprise by using its various tools and techniques. ix. Preparation of reports: It prepares reports of performances of different activities of the enterprise and provides to the management on regular intervals. These managerial reports are prepared as per the requirement of the management. 1.6 UTILITY OF MANAGEMENT ACCOUNTING Management Accounting provides very valuable services to the management in the course of its functioning. Different utilities of Management Accounting are discussed as follows: i. Planning: It formulates policies and programmes by setting definite goals and prepares a systematic plan for achieving these goals. It makes such plans for achieving organizational goals and targets. ii. Controlling: It plays a most significant role in the process of controlling. Management Accounting, in the process of controlling, involves framing of budgets, comparison of actual results with budgeted estimates, ascertainment of any deviation of actual results from budgeted estimates by computation of variances and adoption of necessary remedial measures against such deviation. iii. Coordinating: It plays the most vital role in the process of coordinating of different divisions of an enterprise. Its techniques of planning make a very good coordination between the various activities of a concern. Proper reporting of different business activities are also made by Management Accounting through coordination between the various sections of the enterprise. iv. Performance Evaluation: It plays a very important role in the process of evaluation of performance of the different activities of an enterprise. It evaluates the performances of different divisions and employees of the organization by comparing the target estimates with the actual performances of the divisions and employees. v. Organizing: It plays an important role in the process of organizing of the different activities of an enterprise. It divides the whole organization into suitable cost or profit centres. A sound system of internal control and internal audit is assigned to each cost or profit centre for ensuring a planned organizing system. vi. Motivating: It helps the management in the process of motivating the employees by setting various targets to achieve the organizational goals.

Modified Date: Tue, Jun 01, 2010 06:12:23 PM

Output Date: Tue, Jul 06, 2010 11:37:07 AM

Rev-III

Project: Management Accounting_Debarshi Bhattacharyya ACE Pro India Pvt. Ltd. File: X:\Pearson\Management Accounting_Debarshi Bhattacharyya\MAIN\M01\LAYOUT_M01\M01_DEBA_ISBN_EN_SE_C01.indd

6

MANAGEMENT ACCOUNTING

vii. Communicating: It communicates the performances of the various divisions and employees of the enterprise with the help of the management information system to the different levels of its management by preparing reports of performance of those sections and employees of the enterprise. Such communication is essentially required for planning, controlling and decision-making of the enterprise. viii. Decision-making: It plays the most important role in the process of decision-making of a concern. The success of the management highly depends upon the perfect decision-making and such decisionmaking broadly depends on the effectiveness of information network. It provides necessary and relevant information to the management for effective and accurate decision-making. 1.7 LIMITATIONS OF MANAGEMENT ACCOUNTING Despite the fact that Management Accounting acts as a very useful tool to the management in the process of performing its managerial functions, it suffers from the following limitations: i. Reliance on accounting data: Management Accounting collects the basic data mainly from the records as maintained by financial and Cost Accounting. Hence, it starts to work on the basis of the data as supplied by the other branches of accounting. If those basic data are incorrect, then the entire effort of the management accountant becomes useless. ii. Based on historical data: It guides the management in the process of decision-making for the future activities on the basis of the historical data as supplied by Financial Accounting and Cost Accounting. Therefore, Management Accounting uses historical data for making future decisions, which may not always result in a correct decision. iii. Wide scope: It covers a very vast area and also includes a number of related fields such as Financial Accounting, Cost Accounting, Statistics, Operational Research, Law and Economics, to become more effective to the management. It is really very difficult to develop such a Management Accounting System where all the related people are not well-equipped with full knowledge of all these related areas. iv. Highly expensive: The installation of a sound Management Accounting System in a concern is highly expensive as it essentially requires a wide network of management information system. Moreover, the operating expense of the Management Accounting department is also very high. As a result, small concerns generally cannot afford to install this system. v. Complicated application: It is such a system where a number of different tools and techniques are applied. It also uses a number of accounting and non-accounting subjects for analysis and interpretation of data. Accordingly, its proper application is really complicated when compared to other branches of accounting. A management accountant may try to avoid such a complicated approach of decision-making. vi. Lack of objectivity: It uses both quantitative as well as qualitative data for analysis and interpretation, and also prepares reports on the basis of such interpretation. The interpretation of information as provided by Management Accounting in the form of reports may be influenced by a personal bias of the interpreter, which may reduce the utility of Management Accounting. vii. Not a substitute of management: It merely provides necessary and relevant information to the management to perform their managerial functions of planning, controlling and decision-making in an effective and efficient manner, but certainly is not a substitute for the management itself. It acts only as a decision-making tool to assist the management, but cannot take the ultimate decision on its own. viii. Developing stage: It is still in its developing stage as it is relatively a recent area in the field of accounting. It follows some concepts and conventions which are not yet generally accepted. Many experiments have been still in progress on this area of accounting.

Modified Date: Tue, Jun 01, 2010 06:12:23 PM

Output Date: Tue, Jul 06, 2010 11:37:07 AM

Rev-III

Project: Management Accounting_Debarshi Bhattacharyya ACE Pro India Pvt. Ltd. File: X:\Pearson\Management Accounting_Debarshi Bhattacharyya\MAIN\M01\LAYOUT_M01\M01_DEBA_ISBN_EN_SE_C01.indd

INTRODUCTION TO MANAGEMENT ACCOUNTING

7

1.8 TOOLS AND TECHNIQUES OF MANAGEMENT ACCOUNTING Management Accounting uses various tools and techniques for providing necessary and effective information to the management for performing its managerial functions. Various tools and techniques that are commonly used in Management Accounting are discussed as follows: i. Financial Statement Analysis: It is a methodical and systematic analysis and interpretation of the data as disclosed in the balance sheet and income statement with a view to extract necessary and relevant information for proving them to the management for determining liquidity, solvency, profitability, activity and the managerial performance of the enterprise. Various tools of Financial Statement Analysis such as Ratio Analysis, Comparative Financial Statement, Common-Size Statement and Trend Analysis are frequently used in Management Accounting for analysis and interpretation of financial statements. ii. Fund Flow Analysis: It is a detailed analysis of inflows and outflows of fund (i.e., the working capital) of an enterprise during a particular accounting period. Such analysis is done by preparing a Fund Flow Statement at the end of an accounting period. The Fund Flow Statement exhibits inflows and outflows of fund from various activities of the enterprise during an accounting period. As working capital is considered as the life-blood of every business concern, efficient management of working capital is highly effective for the smooth running of all operating activities of the concern. For an effective and efficient management of the working capital of a concern, Fund Flow Analysis is frequently used as a tool of the Management Accounting. iii. Cash Flow Analysis: It is a detailed analysis of inflows and outflows of cash and cash equivalents (i.e., cash in hand, cash at bank and short-term investments) of an enterprise during a particular accounting period. Such analysis is done by preparing a Cash Flow Statement at the end of an accounting period. The Cash Flow Statement so prepared exhibits the inflows and outflows of cash from various activities of the enterprise during an accounting period. As the movement of cash is very much significant to every business concern, an efficient management of cash is highly effective for the liquidity planning of the concern. For an effective and efficient management of cash of a concern, Cash Flow Analysis is frequently used as a tool of Management Accounting. iv. Costing techniques: Various costing techniques such as Marginal Costing, Standard Costing and Differential Costing are frequently used as tools of Management Accounting in its process of cost control and decision-making. v. Budgetary control: Budgetary control involves framing of budgets, comparison of actual results with budgeted estimates, ascertainment of any deviation of actual results from budgeted estimates by computation of variances and adoption of necessary remedial measures against such deviation. It is an essential tool widely used in the Management Accounting in the process of its controlling, planning and performance evaluation of an enterprise. vi. Statistical and operational research techniques: Various statistical and operational research techniques such as charts, graphs, index number, sampling, time series, Regression Analysis, Linear Programming, Games Theory, and Programme Evaluation and Review Technique (PERT) are frequently used as tools of Management Accounting in its process of performance evaluation and decision-making. vii. Responsibility Accounting: It involves preparation of budget for various responsibility centres and assignment of specific responsibilities to the concerned individual managers for carrying out the budget directions. In the process of cost control, responsibility accounting is widely used as a tool of Management Accounting. viii. Management Reporting: It involves preparation and submission of reports of performance of various activities of a concern to the management on regular intervals for its effective planning, controlling, performance evaluation and decision-making. Management Reporting is widely used as an essential tool in Management Accounting.

Modified Date: Tue, Jun 01, 2010 06:12:23 PM

Output Date: Tue, Jul 06, 2010 11:37:07 AM

Rev-III

Project: Management Accounting_Debarshi Bhattacharyya ACE Pro India Pvt. Ltd. File: X:\Pearson\Management Accounting_Debarshi Bhattacharyya\MAIN\M01\LAYOUT_M01\M01_DEBA_ISBN_EN_SE_C01.indd

8

MANAGEMENT ACCOUNTING

1.9 REQUISITES FOR INSTALLATION OF MANAGEMENT ACCOUNTING SYSTEM Following are the requisites for installation of an effective and efficient Management Accounting System in an organization: i. Introduction of appropriate organization manual defining therein power, functions, responsibilities and scope of the employees of the organization. ii. Recruitment of adequate number of employees and arrangement of time-to-time proper training for those employees. iii. Classification and codification of accounts. iv. Introduction of sound systems of internal control and internal audit in the organization. v. Setting up of suitable systems of budgetary control and standard costing technique. vi. Setting up of a suitable system for integrating cost and financial data. vii. Setting up of suitable cost centres and profit centres. viii. Setting up of a suitable system of responsibility accounting. ix. Developing of a sound management information system. x. Developing of an operational research system in the organization. xi. Preparation of an effective proforma for feedback receiving and managerial report. 1.10 HOW DOES MANAGEMENT ACCOUNTING DIFFER FROM THE OTHER BRANCHES OF ACCOUNTING Management Accounting differs significantly from the other branches of accounting such as Financial Accounting and Cost Accounting. Factors that distinguish Management Accounting from Financial Accounting and Cost Accounting are separately discussed as follows: 1.10.1 Management Accounting and Financial Accounting Financial Accounting records all monetary transactions in the books of accounts and ascertains the results of the financial activities of the concern for an accounting period by preparation of financial statements at the end of every accounting period. On the other hand, Management Accounting collects the basic data mainly from the Financial Accounting System and provides necessary information to the management after analysing and interpreting those data. Points of difference between the Management Accounting and Financial Accounting are enumerated as follows: Management Accounting 1. Management Accounting is primarily based on the data as obtained from Financial Accounting. 2. Its main function is to assist the management in the process of its planning, controlling, performance evaluation and decision-making by proving necessary information to the management. 3. Reports as prepared in Management Accounting may contain both objective as well as subjective figures. 4. Reports as prepared in Management Accounting are exclusively meant for the management of the concern. 5. Reports are prepared as per the requirement of the management.

Financial Accounting 1. Financial Accounting is based on the monetary transactions of the enterprise. 2. Its main functions are recording and classifying monetary transactions in the books of accounts and preparation of financial statements at the end of every accounting period. 3. Reports as prepared in Financial Accounting should always be supported by relevant figures. It lays emphasis on the objectivity of the data. 4. Reports as prepared in Financial Accounting are meant for the management as well as for shareholders and creditors of the concern. 5. Reports are prepared at the end of every accounting period.

(Continued)

Modified Date: Tue, Jun 01, 2010 06:12:23 PM

Output Date: Tue, Jul 06, 2010 11:37:07 AM

Rev-III

Project: Management Accounting_Debarshi Bhattacharyya ACE Pro India Pvt. Ltd. File: X:\Pearson\Management Accounting_Debarshi Bhattacharyya\MAIN\M01\LAYOUT_M01\M01_DEBA_ISBN_EN_SE_C01.indd

INTRODUCTION TO MANAGEMENT ACCOUNTING

Management Accounting 6. Reports as prepared in Management Accounting are not subject to statutory audit. 7. It evaluates the sectional as well as the entire performance of the business. 8. Its success depends on the existence of a sound Financial Accounting System.

9

Financial Accounting 6. Reports as prepared in Financial Accounting are always subject to statutory audit. 7. It ascertains the results and exhibits the financial strength of the business as a whole. 8. Its success does not depend, in any way, on the existence of a sound Management Accounting System.

1.10.2 Management Accounting and Cost Accounting Management Accounting collects the basic data from the Financial Accounting and Cost Accounting systems and provides necessary information to the management after analysing and interpreting those data. On the other hand, Cost Accounting records all cost data in the cost book as obtained from the Financial Accounting, ascertains costs and reveals all cost-related information of the concern at the end of every accounting period. Points of difference between Management Accounting and Cost Accounting are stated as follows: Management Accounting 1. The main objective of Management Accounting is to assist the management in the process of its planning, controlling, performance evaluation and decisionmaking by providing necessary information on time. 2. It uses both quantitative as well as qualitative data, measurable and even not measurable in monetary terms. 3. Primary emphasis given in Management Accounting is on effective and efficient performance of the business. 4. Its success depends on the existence of a sound Cost Accounting System. 5. It is based on the data as obtained from Financial Accounting and Cost Accounting. 6. Management Accounting provides historical as well as predictive information for future decision-making. 7. Reports as prepared in Management Accounting are exclusively meant for the management of the concern. 8. It uses the principles and practices of financial as well as Cost Accounting. 9. Reports as prepared in Management Accounting are not subject to statutory audit. 10. It deals with cost- as well as finance-related data of an enterprise.

Cost Accounting 1. The main objective of Cost Accounting is to ascertain, allocate and do accounting for costs and to assist the management in the process of its cost control and cost-related decision-making. 2. It uses only quantitative cost data measurable in monetary terms. 3. Primary emphasis given in Cost Accounting is on cost determination and cost control of the business. 4. Its success does not depend, in any way, on the existence of a Management Accounting System. 5. It is based on the cost-related data as obtained from Financial Accounting. 6. Cost Accounting provides historical cost information for future cost-related decision-making. 7. Reports as prepared in Cost Accounting are mainly meant for the management, but also useful to the shareholders and creditors of the concern. 8. It uses the principles and practices of Cost Accounting alone. 9. Reports as prepared in Cost Accounting are subject to statutory audit (i.e., cost audit) in many countries. 10. It deals only with cost-related data of an enterprise.

CHAPTER REVIEW SUMMARY  Management Accounting is concerned with the collection of data from both internal as well as external sources and communicates relevant information to the management, after processing, analysing and interpreting those, to perform their managerial functions of planning, controlling and decision-making in an effective and efficient manner.  Nature or features of Management Accounting are as follows: (a) It deals with the collection of accounting and other data, analysing them, interpretting them and communicating all relevant information to the management; (b) It provides necessary information to the management to perform their managerial functions of decision-making, planning and control; (c) It acts as a yardstick for measuring the effectiveness of managerial

Modified Date: Tue, Jun 01, 2010 06:12:23 PM

Output Date: Tue, Jul 06, 2010 11:37:07 AM

Rev-III

Project: Management Accounting_Debarshi Bhattacharyya ACE Pro India Pvt. Ltd. File: X:\Pearson\Management Accounting_Debarshi Bhattacharyya\MAIN\M01\LAYOUT_M01\M01_DEBA_ISBN_EN_SE_C01.indd

10







 









MANAGEMENT ACCOUNTING

performance of the different activities of the enterprise; and (d) It prepares necessary plans to implement the various financial decisions of the management. Management Accounting has a very wide spread scope. It covers the following areas: (a) Financial Accounting; (b) Cost Accounting; (c) forecasting and budgeting; (d) statistical tools; (e) operational research techniques; (f) financial analysis and interpretation; (g) tax accounting and tax planning; (h) management information system; (i) internal control and internal audit; (j) office system; (k) legal provisions; and (l) other areas like social accounting, human resource accounting, inflation accounting, and environmental accounting. Objectives of Management Accounting are: (a) analysis and interpretation of financial statements; (b) planning and policy-making; (c) decision-making and controlling; (e) communicating; (f) coordinating; (g) tax planning; and (h) advisory service. Role or functions of Management Accounting are: (a) collection of data; (b) supply of modified data; (c) analysis and interpretation of data; (d) planning and forecasting; (e) communication; (f) ensuring control; (g) helping in decision-making; (h) performance evaluation; and (i) preparation of managerial reports. Utilities of Management Accounting are: (a) planning; (b) controlling; (c) coordinating; (d) performance evaluation; (e) organizing; (f) motivating; (g) communicating; and (h) decision-making. Limitations of Management Accounting are: (a) reliance on accounting data; (b) based on historical data; (c) vast area coverage; (d) highly expensive; (e) complicated application; (f) lack of objectivity; (g) not a substitute of management; and (h) developing stage. Tools and techniques of Management Accounting are: (a) Financial Statement Analysis; (b) Fund Flow Analysis; (c) Cash Flow Analysis; (d) costing techniques; (e) budgetary control; (f) statistical and operational research techniques; (g) Responsibility Accounting; and (h) Management Reporting. Requisites for installation of Management Accounting System are: (a) introduction of appropriate organization manual; (b) recruitment of adequate number of employees; (c) introduction of systems of internal control and internal audit; (d) setting up of suitable systems of budgetary control and standard costing; (e) development of a management information system; (f) setting up of a suitable system for integrating cost and financial data; and (g) setting up suitable cost centres and profit centres. Distinctions between Management Accounting and Financial Accounting are: (a) Management Accounting is primarily based on the data from Financial Accounting whereas Financial Accounting is based on the monetary transactions of the enterprise; (b) the main function of Management Accounting is to assist the management in its planning, controlling, performance evaluation and decision-making whereas the main function of Financial Accounting is recording of monetary transactions and preparation of financial statements; (c) Management Accounting evaluates both sectional as well as the whole performance of the business whereas Financial Accounting ascertains the results and exhibits the financial strength of the business as a whole; and (d) success of Management Accounting depends on the existence of a sound Financial Accounting System whereas the success of Financial Accounting does not depend, in any way, on the existence of Management Accounting System. Distinctions between Management Accounting and Cost Accounting are as follows: (a) the main objective of Management Accounting is to assist the management in the process of its planning, controlling, performance evaluation and decision-making whereas the main objective of Cost Accounting is to ascertain, allocate and do accounting for costs, and to assist the management in the process of cost control; (b) primary emphasis given in Management Accounting is on the effective and efficient performance of the business whereas the primary emphasis given in Cost Accounting is on cost determination and cost control of the business; (c) the success of Management Accounting depends on the existence of a sound Cost Accounting system whereas the success of Cost Accounting does not depend, in any way, on the existence of Management Accounting System; and (d) Management Accounting is based on the data as obtained from Financial Accounting and Cost Accounting whereas Cost Accounting is based on the cost-related data as obtained from Financial Accounting.

CHAPTER REVIEW QUIZ 1. State whether the following statements are true or false: a. Various statistical and operational research tools are used in Management Accounting in the process of planning, controlling and decision-making. b. Management Accounting interprets the analysed data as obtained from the non-operational activities alone of the enterprise.

Modified Date: Tue, Jun 01, 2010 06:12:23 PM

Output Date: Tue, Jul 06, 2010 11:37:07 AM

Rev-III

Project: Management Accounting_Debarshi Bhattacharyya ACE Pro India Pvt. Ltd. File: X:\Pearson\Management Accounting_Debarshi Bhattacharyya\MAIN\M01\LAYOUT_M01\M01_DEBA_ISBN_EN_SE_C01.indd

INTRODUCTION TO MANAGEMENT ACCOUNTING

11

c. Management Accounting prepares the necessary plans to implement various financial decisions of the management. d. Management Accounting is a backward-looking tool to the management. e. Management Accounting helps in coordinating various business activities of an enterprise. f. Management Accounting System should be well conversant with the modern office management system. g. Management Accounting is a branch of Financial Accounting. h. Cost Accounting does not provide cost-related basic information to the Management Accounting. i. Installation of Management Accounting System is highly expensive to every concern. Ans.: True: (a), (c), (e), (f), (i); False: (b), (d), (g), (h). 2. Fill in the blanks in the following statements: . a. Success of the management highly depends upon perfect b. Management Accounting System can be introduced into an organization where there exists a well-desystem. signed for projecting the future trends of c. Management Accounting analyses and interprets different activities of the enterprise. of the enterprise. d. Management Accounting acts as a yardstick for measuring of e. Management Accounting is concerned with the accounting information which is useful to the . f. Management Accounting communicates relevant information to the management to perform their and . managerial functions of , Ans.: (a) decision-making; (b) Financial Accounting; (c) historical data; (d) level of performance; (e) management; (f) decision-making; planning; controlling. EXERCISE I. Theoretical Questions A. Short Answer Type Questions

1. 2. 3. 4. 5. 6. 7.

What is Management Accounting? Write any two features of Management Accounting. Write any two objectives of Management Accounting. Write any three utilities of Management Accounting. Write any three limitations of Management Accounting. Give any two points of differences between Management Accounting and Financial Accounting. Give any two points of differences between Management Accounting and Cost Accounting.

B. Essay Type Questions

1. 2. 3. 4. 5. 6. 7. 8. 9.

Define Management Accounting. Discuss its nature. Discuss the scope of Management Accounting. Discuss the objectives of Management Accounting. Discuss the role of Management Accounting. Discuss the advantages of Management Accounting. Discuss the limitations of Management Accounting. Discuss the various tools and techniques used in Management Accounting. Enumerate various requisites for installation of Management Accounting System. Distinguish between a. Management Accounting and Financial Accounting. b. Management Accounting and Cost Accounting.

Modified Date: Tue, Jun 01, 2010 06:12:23 PM

Output Date: Tue, Jul 06, 2010 11:37:07 AM

Rev-III

Project: Management Accounting_Debarshi Bhattacharyya ACE Pro India Pvt. Ltd. File: X:\Pearson\Management Accounting_Debarshi Bhattacharyya\MAIN\M02\LAYOUT_M02\M02_DEBA_ISBN_EN_SE_C02.indd

Analysis and Interpretation of Financial Statements

2

LEARNING OBJECTIVES On completion of the study of the chapter, you should be able to understand: What are Financial Statements? Components of Financial Statements. Relationship between Income Statement and Balance Sheet. Concept of Financial Statement Analysis. Objectives and requisites of Financial Statement Analysis. Different approaches to Financial Statement Analysis. Different types of Financial Statement Analysis. Different techniques of Financial Statement Analysis.

2.1 MEANING OF FINANCIAL STATEMENTS Financial Statements are compilation of financial data, arranged and organized in a systematic and summarized manner according to the accounting principles, to assess the financial position of an enterprise as regards to its profitability, operational efficiency, long- and short-term solvency and growth potential. On the basis of the information as disclosed in the Financial Statements, users of them come to know about the growth, profitability, solvency and financial strength of an enterprise. These are prime tools in the hands of the management of an enterprise by which they can present the financial position of the enterprise before the interested parties such as shareholders, lenders and creditors. A number of statements prepared at the end of every accounting period are collectively called ‘Financial Statements.’ Financial Statements include the following: i. Income Statement or Profit & Loss A/c. ii. Balance Sheet. iii. Fund Flow Statement. iv. Cash Flow Statement. 2.2 CHARACTERISTICS OF FINANCIAL STATEMENTS Financial Statements indicate the financial strength of an enterprise and communicate financial information to the different interested parties. According to the American Institute of Certified Public Accountants (AICPA), ‘Financial Statements reflect a combination of recorded facts, accounting conventions and personal judgments and the judgments and conventions applied affect them materially.’ Therefore, this statement of AICPA implies that the data as contained in the Financial Statements are affected by the recorded facts, accounting conventions and personal judgements. Accordingly, the nature of Financial Statements may be explained as follows: i. Recorded Facts: Financial transactions that take place are recorded in the books of accounts of the concern in monetary value. Financial Statements are prepared taking those recorded transactions. These recorded facts reflect the results of the past activities of the enterprise; and accordingly, they are

Modified Date: Mon, Jul 05, 2010 04:56:11 PM

Output Date: Tue, Jul 06, 2010 11:38:25 AM

Rev II

Project: Management Accounting_Debarshi Bhattacharyya ACE Pro India Pvt. Ltd. File: X:\Pearson\Management Accounting_Debarshi Bhattacharyya\MAIN\M02\LAYOUT_M02\M02_DEBA_ISBN_EN_SE_C02.indd

13

ANALYSIS AND INTERPRETATION OF FINANCIAL STATEMENTS

ii.

iii.

iv.

v.

vi.

vii.

called ‘historical evidences.’ On the basis of these historical evidences as recorded during an accounting period, Financial Statements are prepared at the end of the accounting period. Hence, one of the characteristics of the Financial Statements is that they are prepared on the basis of past recorded facts. Accounting Conventions: Some accounting principles, concepts, conventions and postulates are applied while preparing the Financial Statements, in order to make them reliable, meaningful and comparable. Concepts of materiality, consistency, periodicity, conservatism and full disclosures are the examples of such accounting principles, concepts, conventions and postulates that are applied in Financial Statements. Periodic Information: Financial Statements exhibit a periodical financial information of an entity. As Financial Statements are prepared generally at the end of every accounting year, these provide relevant financial information for that accounting year. Summary of Activities: Financial Statements should incorporate the summary of the financial information of all the activities of the concern. It includes the financial information of operating, investing and financing activities of the concern. Prime aid for Financial Analysis: Financial Statements provide the prime information for the financial analysis of a concern. These act as the prime aid for financial analysis by disclosing all the material information before the users. Legal Conformity: Financial Statements must disclose all the information as required by the law of the country. It should be presented in conformity with the applicable legal provisions. They should also be presented in conformity with the relevant accounting standards of the country. Personal Judgements: Financial Statements are not only prepared on the basis of the established accounting principles and conventions and legal requirements, personal judgements of the accountants and management are also applied in some cases for preparing the Financial Statements. For example, in case of valuation of inventories, personal judgement is essentially required as regards to the application of method of such valuation (i.e., LIFO or FIFO or Weighted Average). Thus, Financial Statements are also influenced by personal judgements of the accountant or the management of the concern.

2.3 ANATOMY/COMPONENTS OF FINANCIAL STATEMENTS Financial Statements comprise a number of statements generally prepared at the end of every financial year to assess the various financial activities and strength of an enterprise. Components of Financial Statements are as shown in Figure 2.1. Financial Statements

Income Statement

Balance Sheet

Statement of changes in financial position

Fund Flow Statement

Figure 2.1

Cash Flow Statement

Components of Financial Statements

These Financial Statements are generally prepared at the end of every accounting year by a concern to evaluate the financial activities and the strength of the concern, as well as due to legal obligation, as all the above statements are the carriers of financial information that are required by different users of Financial Statements. Each component of the Financial Statements is separately described as follows. 2.3.1 Income Statement or Profit & Loss Account Income Statement (or Profit & Loss A/c—P&L A/c) is a statement (or an account) prepared at the end of every accounting year to ascertain the revenue profit earned (or the revenue loss incurred) from the business

Modified Date: Mon, Jul 05, 2010 04:56:11 PM

Output Date: Tue, Jul 06, 2010 11:38:25 AM

Rev II

Project: Management Accounting_Debarshi Bhattacharyya ACE Pro India Pvt. Ltd. File: X:\Pearson\Management Accounting_Debarshi Bhattacharyya\MAIN\M02\LAYOUT_M02\M02_DEBA_ISBN_EN_SE_C02.indd

14

MANAGEMENT ACCOUNTING

during that accounting year. Such revenue profit is ascertained by deducting various costs incurred during an accounting year to generate revenue into the business from the revenue generated for the business, for that accounting year, by way of sale of goods and other activities. Such profit is ascertained in three parts as follows: First, the excess of revenue generated from the operating activities (i.e., sale of goods, discount received, commission received) of the enterprise during an accounting period over the direct revenue expenses incurred to carry on the operating activities (i.e., materials consumed, wages, factory expenses) of the enterprise for that accounting year represents the Gross Profit for that accounting year. Secondly, the Operating Profit is ascertained by deducting the indirect revenue operating expenses incurred (i.e., salary to staff, general expenses, selling expenses) for that accounting year from the Gross Profit as ascertained in the above. Finally, the Net Profit for the accounting year is ascertained by adjusting the nonoperating incomes (i.e., interest, dividend received from investment) and non-operating expenses (i.e., interest on loan) for that accounting year with the Operating Profit as ascertained in the above. Hence, the profit earned (or loss incurred) during an accounting year is ascertained through the Income Statement by matching the total revenue expenses incurred during that accounting year with the revenue incomes earned during that accounting year. Therefore, the Income Statement exhibits the profit-earning capacity of an enterprise for a particular accounting year. It is prepared generally at the end of every accounting year mainly to evaluate the regular operating performance of a concern for that accounting period. 2.3.2 Balance Sheet Balance Sheet is a summarized statement, generally prepared at the end of every accounting period, showing therein the values of assets and liabilities with a view to exhibit the financial position of the concern. The Balance Sheet of every concern consists of two parts—assets and liabilities. In the ‘asset side’ of the Balance Sheet, the values of assets owned by the concern as on that date are exhibited. An Asset is defined as the claim or right acquired by the concern as a result of its past transaction, which is supposed to provide the future economic benefit to the concern. The asset side of the Balance Sheet consists of four parts—Fixed Assets, investments, Current Assets and miscellaneous expenditure. Fixed Assets are those that have been acquired by the concern not for resale, rather acquired for long-term use for the operating activities of the business (e.g., Land & Building, Plant & Machinery). Investments represent the amount of long-term fund invested outside the business (e.g., investment in shares or debentures of other companies). Current Assets are those that have been acquired by the firm with the expectation to consume or convert them into cash within a year from the date of their acquisition (Stock of goods, Sundry Debtors). Miscellaneous expenditure represents the deferred loss to the extent not written off (i.e., Preliminary expenses, Discount on issue of shares or debentures). In the ‘liability side’ of the Balance Sheet, the values of liabilities payable by the concern as on that date are exhibited. A Liability is defined as the obligation incurred by the concern as a result of its past transaction, against which the concern is liable to pay money or other assets to another party. Liability side of the Balance Sheet consists of three parts—owners’ fund, outsiders’ long-term loans and Current Liabilities. ‘Owners’ fund’ refers to the long-term fund invested by the owner into the business, which is comprised of Share Capital, undistributed profits and reserves and so on (i.e., Share Capital, Reserves & Surplus). ‘Outsiders’ long-term loan’ refers to the long-term fund invested by the outsiders (i.e., not the owner) into the business in the form of loan of a long-term nature (e.g., Debentures, other long-term secured and unsecured loans). ‘Current Liabilities’ are those obligations of the concern which are supposed to pay within a period of 1 year from the date of their occurrence (e.g., Sundry Creditors, Bills Payable). 2.3.3 Statement of Changes in Financial Position Traditionally, Financial Statements refer to the Income Statement and the Balance Sheet. But, as per modern concept, further extension of the periphery of Financial Statements is essentially needed in the modern business world to disclose more financial information of an entity to the different users of the Financial Statements.

Modified Date: Mon, Jul 05, 2010 04:56:11 PM

Output Date: Tue, Jul 06, 2010 11:38:25 AM

Rev II

Project: Management Accounting_Debarshi Bhattacharyya ACE Pro India Pvt. Ltd. File: X:\Pearson\Management Accounting_Debarshi Bhattacharyya\MAIN\M02\LAYOUT_M02\M02_DEBA_ISBN_EN_SE_C02.indd

ANALYSIS AND INTERPRETATION OF FINANCIAL STATEMENTS

15

As a consequence, some other statements, which exhibit changes in the financial position of an entity between two accounting years, are incorporated in the composition of Financial Statements. They are: Fund Flow Statement and Cash Flow Statement. 2.3.3.1 Fund Flow Statement Fund Flow Statement is a summarized statement generally prepared at the end of an accounting year showing therein the inflows and outflows of the fund of an enterprise during that accounting year. In this context, ‘fund’ refers to the working capital (i.e., Current Assets—Current Liabilities). Therefore, the Fund Flow Statement is a summarized statement generally prepared at the end of an accounting year, which exhibits various sources of the working capital and its uses made by a concern during that particular accounting year. This statement shows the flows of fund (i.e., working capital) of a concern during an accounting year. A Fund Flow Statement is prepared on the basis of the published data as disclosed by the traditional Financial Statements (i.e., Income Statement and Balance Sheet) of two different accounting years. It summarizes all the fund flows between two accounting years. It is an essential tool for the managerial decision-making. 2.3.3.2 Cash Flow Statement Cash Flow Statement is a summarized statement generally prepared at the end of an accounting year showing therein the inflows and outflows of the cash of an enterprise during that accounting year. As per Accounting Standard 3 (Revised) as issued by the Institute of Chartered Accountants of India (ICAI), ‘cash’ refers to the cash-in-hand, cash-at-bank and short-term investments (i.e., cash & cash equivalents). Therefore, a Cash Flow Statement exhibits various sources of cash and cash equivalents and their uses made by a concern during the particular accounting year. This statement shows the flows of cash and cash equivalents of a concern during an accounting year. A Cash Flow Statement is prepared on the basis of the published data as disclosed by the traditional Financial Statements (i.e., Income Statement and Balance Sheet) of two different accounting years. It summarizes all cash flows between the two accounting years. It is an essential tool for the managerial decision-making. Cash Flow Statement reports the management the net cash flow (i.e., cash inflow less the cash outflow or vice versa) from each activity of the enterprise as well as of the overall business of the enterprise. 2.4 RELATIONSHIP BETWEEN INCOME STATEMENT AND BALANCE SHEET Income Statement and the Balance Sheet, the two most important components of Financial Statements, are prepared under two different statements with two different objectives. While the Income Statement is prepared with a view to ascertain the net financial result for an accounting year from the operating and other activities of a concern, the Balance Sheet is prepared with a view to assess the financial position of the concern at the end of an accounting year. Yet, these two statements are closely related to each other. The relationship between them is explained as follows: Stock and flow relationship: In every business, there is a continuous flow of resources with the occurrences of transactions. These flows of resources are having two directions, such as inflows as well as outflows. Inflow of resources occurs as soon as the revenue is generated in the business by way of sale. On the other hand, the outflow of resources takes place when costs are incurred to generate revenue in the business. Income Statement and Balance Sheet reveal the flows and stock of these resources. Income Statement reveals the flows of resources that take place during an accounting year and ascertains the profit as excess of inflows over outflows of resources for that period. On the other hand, Balance Sheet exhibits the stock of resources possessed by a concern at the end of an accounting period. Complementary relationship: Although Income Statement and Balance Sheet provide different financial information to the users of Financial Statements, the truth is—either of the two statements alone cannot provide a full disclosure about the financial condition of a concern. On the one hand, Balance Sheet cannot be drawn without preparing the Income Statement. On the other hand, income

Modified Date: Mon, Jul 05, 2010 04:56:11 PM

Output Date: Tue, Jul 06, 2010 11:38:25 AM

Rev II

Project: Management Accounting_Debarshi Bhattacharyya ACE Pro India Pvt. Ltd. File: X:\Pearson\Management Accounting_Debarshi Bhattacharyya\MAIN\M02\LAYOUT_M02\M02_DEBA_ISBN_EN_SE_C02.indd

16

MANAGEMENT ACCOUNTING

statement alone discloses the results of the operating and other activities of a concern, but not the financial strength of the concern. Therefore, for a full disclosure of financial information of a concern, preparation of both—the Income Statement as well as the Balance Sheet—are essentially needed for every concern. As either of these two statements cannot provide a full financial information to the different users, these are complementary to each other to provide the full disclosure about the financial condition of a concern. Link relationship: Balance Sheet acts as a link between the Income Statements of successive accounting years. While the benefits that are derivable from the expenses incurred or income earned during an accounting year (i.e., revenue expenses or incomes) may be exhausted during that accounting year, the benefits that may not be exhausted during that accounting year (i.e., capital expenditures or incomes) may be carried forward to the subsequent accounting years. While such revenue expenses and incomes are considered in the Income Statement for that accounting year, those capital expenditures and incomes are carried forward through the Balance Sheet to the next accounting year. If the benefits derivable from those carried forward incomes and expenditures are exhausted during the next accounting year, those would be considered in the Income Statement for the next accounting year; if are not exhausted during the next accounting year, then the residual values of those unexpired expenses or incomes would appear in the Balance Sheet of the subsequent year and so on. Therefore, the Balance Sheet serves as a link between the Income Statements of the successive accounting years. 2.5 CONCEPT OF FINANCIAL STATEMENT ANALYSIS Financial Statement Analysis is an analysis which critically examines the relationship between various elements of the Financial Statements with a view to obtain the necessary and effective information from them. It is a process of scanning the Financial Statements for evaluating the relationship between the items as disclosed in them. According to John N. Myer, ‘Financial Statement Analysis is largely a study of relationships among the various financial factors in a business, as disclosed by a single set of statements, and study of these factors as shown in a series of statements.’ Therefore, Financial Statement Analysis involves a systematic and critical examination of the information contained in the Financial Statements with a view to provide an effective and more meaningful information to its different users. It is basically a postmortem of the transactional activities of an enterprise as disclosed in the Financial Statements so as to judge the operational ability, profitability, solvency, managerial efficiency and the overall performance of the enterprise more clearly. 2.6 OBJECTIVES OF FINANCIAL STATEMENT ANALYSIS Financial Statement Analysis is a very important and effective tool to the management of an enterprise. The objectives of a Financial Statement Analysis are as follows: i. To assess the real meaning and significance of financial data as disclosed in the Financial Statements. ii. To assess the liquidity and short-term solvency position of the enterprise. iii. To assess the long-term solvency position of the enterprise. iv. To assess the present and future profitability of the enterprise. v. To evaluate the operational efficiency of the enterprise. vi. To evaluate the managerial efficiency of the enterprise. vii. To assess the growth potentiality of the enterprise. viii. To evaluate the different activities of the concern. ix. To assess the financial stability of the enterprise. x. To assess the financial as well as the business risk of the concern. xi. To assess the overall performance of the concern.

Modified Date: Mon, Jul 05, 2010 04:56:11 PM

Output Date: Tue, Jul 06, 2010 11:38:25 AM

Rev II

Project: Management Accounting_Debarshi Bhattacharyya ACE Pro India Pvt. Ltd. File: X:\Pearson\Management Accounting_Debarshi Bhattacharyya\MAIN\M02\LAYOUT_M02\M02_DEBA_ISBN_EN_SE_C02.indd

ANALYSIS AND INTERPRETATION OF FINANCIAL STATEMENTS

17

2.7 REQUISITES OF FINANCIAL STATEMENT ANALYSIS Financial Statement Analysis is done for many purposes. It may be done for a general purpose or for a specific purpose. Requisites for such Financial Statement Analysis are as follows: i. Study of internal and external environment: A financial analyst is required to be well acquainted with the internal as well as the external environment of the enterprise before commencing the analysis of its Financial Statements. For the said purpose, the internal environment refers to the employee morale, manager–subordinate relationship, corporate governance, relationship with customers and suppliers, structural change and so on. On the other hand, the external environment refers to applicable laws, competitors’ strength, customers’ profile, suppliers’ profile and so on. A study of these factors enables a financial analyst to make a better interpretation of the financial position and performance of the concern. ii. Objective of the analysis: A financial analyst should be well acquainted with the objectives of such analysis so that he can locate the area of his study. iii. Study of accounting procedure: The analyst should have a clear idea of the accounting procedures, principles and policies that are being followed by the enterprise. This helps the analyst to make an appropriate interpretation of the Financial Statements of the enterprise. iv. Rearrangement of financial data: Based on the objectives and the area of coverage of the analysis, the analyst should rearrange the primary financial data as extracted from the Financial Statements before commencing the analysis. v. Selection of techniques for analysis: The analyst should select the most appropriate tools and techniques for interpretation and analysis of Financial Statements. This enables the analyst to make an effective interpretation of the financial position and performance of the enterprise. vi. Interpretation of findings: The analyst should interpret his findings, those he gathered during the course of his analysis, in a lucid and effective manner so as to make them meaningful and easily understandable to the users of the Financial Statements. vii. Submission of report: The analyst must prepare a report on the basis of the interpretation of his findings which he gathered during the course of the analysis and submit the same before the management/ client for their controlling, planning and decision-making purposes. 2.8 DIFFERENT APPROACHES TO FINANCIAL STATEMENT ANALYSIS Generally, two approaches are followed for the analysis of Financial Statements. They are: i. Traditional approach to Financial Statement Analysis. ii. Modern approach to Financial Statement Analysis. These two approaches of Financial Statement Analysis are separately discussed as follows. 2.8.1 Traditional Approach to Financial Statement Analysis Traditional approach to Financial Statement Analysis is the conventional approach of financial analysis which came into existence at the end of the 19th century. This approach of analysis is based on the financial data contained in the Financial Statements. It takes into consideration the traditional Financial Statements comprising of the Income Statement and the Balance Sheet. Traditionally, financial ratios were used as the basic tool for assessing the financial position of the enterprise. For example, current ratio was widely used to assess the short-term solvency position of an entity. As soon as it was realised that current ratio alone cannot be a true indicator of the short-term solvency position of an entity, a more realistic measure was developed by way of introduction of ‘Acid-Test (or Quick or Liquid) Ratio’ for assessing the short-term solvency position of an entity. For measuring the profitability of a concern, this approach developed Gross Profit Ratio, Net Profit Ratio and so on. For measuring the long-term solvency position of the business, it developed Debt–Equity Ratio.

Modified Date: Mon, Jul 05, 2010 04:56:11 PM

Output Date: Tue, Jul 06, 2010 11:38:25 AM

Rev II

Project: Management Accounting_Debarshi Bhattacharyya ACE Pro India Pvt. Ltd. File: X:\Pearson\Management Accounting_Debarshi Bhattacharyya\MAIN\M02\LAYOUT_M02\M02_DEBA_ISBN_EN_SE_C02.indd

18

MANAGEMENT ACCOUNTING

Various turnover ratios were also developed for measuring various activities, efficiency, utilization of resources and managerial performance of the enterprise. Gradually, the traditional approach to Financial Statement Analysis further extended its scope by developing an inter-firm comparison of ratios and by accepting ‘industry average ratio’ as the standard ratio for evaluation of short-term solvency position, long-term solvency position, managerial efficiency and performance of the business. Traditional approach reached a landmark when it developed the concept of preparing ‘Common-size Financial Statements’ to study the comparison of various elements of Financial Statements. Recently, another development was made by the traditional approach to Financial Statement Analysis by introducing the Index Number Trend Analysis of Financial Statements. This analysis is used to study the Financial Statements of an enterprise over a series of years. In spite of the above development, the traditional approach does not give any emphasis on the social desirability factors like human resource, environmental issues and other external factors, in the Financial Statements. Though this approach ignores non-financial items, it considers financial items alone for the purpose of Financial Statement Analysis. 2.8.2 Modern Approach to Financial Statement Analysis Modern approach to Financial Statement Analysis has been introduced in order to overcome the various limitations of the traditional approach. This approach gives emphasis on both financial as well as non-financial factors of the Financial Statements. It includes social desirability factors in the corporate Financial Statements. In this approach of Financial Statement Analysis, both internal as well as external environmental factors of the business are taken into consideration. It incorporates various non-financial factors such as employee morale, applicable regulations, relationship with customers and suppliers, business risk and financial risk involved with the concern, and competitive environment. Modern approach to Financial Statement Analysis is a forward-looking phenomenon. Accordingly, it gives more emphasis on future. It provides relevant and effective information for future decision-making. It processes past and present information as disclosed in the Financial Statements to make the prediction on the future issues of the concern. This analysis uses modern statistical and mathematical tools and techniques for interpretation of Financial Statements. Under this analysis, conclusions are made after empirical verification. It provides all the necessary information as regards to short-term- as well as long-term financial condition of the concern to all users. 2.9 TYPES OF FINANCIAL STATEMENT ANALYSIS Analysis of Financial Statements can be made on different basis. Different types of Financial Statement Analysis are shown in Figure 2.2. Types of Financial Statement Analysis

From the viewpoint of its users

Internal analysis

External analysis

Figure 2.2

From the viewpoint of its objectivity

Short-term analysis

Long-term analysis

From the viewpoint of time span

Horizontal/dynamic analysis

Vertical/static analysis

Types of Financial Statement Analysis

i. From the viewpoint of its users: From the viewpoint of the users of the Financial Statements, analysis of such may be of two types as the following: a. Internal Analysis: As the management of an enterprise has the right to access its books and accounts (i.e., internal sources), such an analysis is done by the management of the enterprise. As such an analysis is based on the factual data recorded in the books of accounts, it provides a reliable and dependable information of the enterprise to the users.

Modified Date: Mon, Jul 05, 2010 04:56:11 PM

Output Date: Tue, Jul 06, 2010 11:38:25 AM

Rev II

Project: Management Accounting_Debarshi Bhattacharyya ACE Pro India Pvt. Ltd. File: X:\Pearson\Management Accounting_Debarshi Bhattacharyya\MAIN\M02\LAYOUT_M02\M02_DEBA_ISBN_EN_SE_C02.indd

ANALYSIS AND INTERPRETATION OF FINANCIAL STATEMENTS

19

b. External Analysis: Such an analysis is done by the external parties, like creditors, investors, bankers, government and research scholars, who do not have any access to the internal books and accounts of the enterprise. Accordingly, such an analysis is based on the published reports and statements of the enterprise. ii. From the viewpoint of its objectivity: From the viewpoint of the users of the Financial Statements, analysis of Financial Statements may be of two types: a. Short-term Analysis: Such an analysis of Financial Statements is done to examine liquidity, profitearning capacity and short-term solvency of an enterprise. This analysis is made taking into account the various elements of Current Assets and Current Liabilities with a view to judge the short-term financial position of the enterprise. It is very much useful for the short-term financial planning. b. Long-term Analysis: Such an analysis of Financial Statements is done to examine the long-term financial stability and long-term solvency as well as the future profit-earning capacity of an enterprise. This analysis is made taking various items of long-term assets and liabilities with a view to judge long-term financial position of the enterprise. It is very much useful for long-term financial planning. iii. From the viewpoint of time span: From the viewpoint of the users of the Financial Statements, analysis of Financial Statements may be of two types: a. Vertical/Static Analysis: Such an analysis is done to examine the Financial Statements of one particular year alone of an enterprise. Therefore, this analysis is based on the financial data of one accounting year alone; and that is why, it is also called as ‘static analysis.’ It presents the structural relationship of different items contained in the Financial Statements. Accounting ratios and Common-size Statements are two tools that are generally used for this analysis. b. Horizontal/Dynamic Analysis: Such an analysis is done to examine the Financial Statements of a number of years of an enterprise. Therefore, this analysis is based on the financial data of several accounting years; and that is why it is also called ‘dynamic analysis.’ It is a time-series analysis of the data contained in the Financial Statements. It indicates the growth of the enterprise over a number of years by a comparative time-series analysis. Comparative Financial Statements and TrendPercentage Analysis are two tools that are commonly used for this analysis. 2.10 TECHNIQUES/TOOLS OF FINANCIAL STATEMENT ANALYSIS Techniques of Financial Statement Analysis can be defined as the tool or method of analysing Financial Statements, evaluating its performance, and identifying the strength and weakness of the enterprise by interpreting the results and taking corrective measures. These methods would help the analyst to present the information from the Financial Statements in such a form that would help the users to understand the core areas of the functioning of the business of an enterprise. There are several techniques or tools which are generally used for analysing the Financial Statements, but none of them alone are sufficient for interpreting the overall performance of the enterprise and for decision-making as well. Rather, a combination of few techniques is generally used for an effective interpretation and analysis of the overall performance of the enterprise. Various techniques that are used for interpretation and analysis of Financial Statements are shown as follows: i. Comparative Financial Statement. ii. Common-size Statement. iii. Trend Analysis. iv. Ratio Analysis. v. Cash Flow Statement. vi. Fund Flow Statement. vii. Working Capital Analysis. viii. Cost–Volume–Profit Analysis. In this chapter, we shall study in detail the first three methods alone of the above-mentioned Financial Statement Analysis.

Modified Date: Mon, Jul 05, 2010 04:56:11 PM

Output Date: Tue, Jul 06, 2010 11:38:25 AM

Rev II

Project: Management Accounting_Debarshi Bhattacharyya ACE Pro India Pvt. Ltd. File: X:\Pearson\Management Accounting_Debarshi Bhattacharyya\MAIN\M02\LAYOUT_M02\M02_DEBA_ISBN_EN_SE_C02.indd

20

MANAGEMENT ACCOUNTING

2.11 COMPARATIVE FINANCIAL STATEMENT ANALYSIS 2.11.1 Concept of Comparative Financial Statement Analysis Comparative Financial Statement Analysis is a form of horizontal analysis where the Financial Statements of two or more years, or of two or more different companies, or of a company and its industry, are compared, analysed and interpreted. That is why, this technique of analysis is also called ‘inter-period analysis’ (when the Financial Statements of two or more years are taken into consideration) or ‘inter-firm analysis’ (when Financial Statements of two or more companies are taken into consideration). The two most commonly used forms of such analysis are: i. Comparative Balance Sheet. ii. Comparative Income Statement. In case of inter-period analysis, it should be kept in mind that uniformity in accounting concepts and conventions is maintained during all the years that are taken into consideration for comparison. On the other hand, in case of inter-firm analysis, the size of the firms taken into consideration for comparison must be more or less the same; as otherwise, meaningful conclusions cannot be drawn. To have more accurate results, external factors like market conditions and business risk should also be considered. Under year-to-year change analysis, all the elements of a Balance Sheet or Income Statement are compared and absolute changes as well as the percentage changes are calculated on the basis of the previous year as the base year. The changes in Fixed Assets, investments, Current Assets, proprietors’ fund and Current Liabilities are compared to determine the long-term solvency position of the firm and growth of the firm. From the Comparative Income Statement, we are able to know the absolute and percentage changes in the Gross Profit, Operating Profit and Net Profit. (A) Comparative statement exhibits the following pertinent information: i. Absolute figures for two or more years of the items appearing in the Financial Statements (i.e., in the Balance Sheet and in the Income Statement). ii. Changes in the above absolute figures of the current year as compared to the previous year is taken as the base year (or firm-wise changes). iii. Percentage changes in the absolute figures of the current year on the basis of the base year (or percentage of the firm-wise changes). Illustration Say, the debtors and stock of a company as on 31 December 2008 are Rs. 50,000 and Rs. 90,000, respectively, and as on 31 December 2009 are Rs. 60,000 and Rs. 75,000, respectively. Then, the Comparative Balance Sheet would exhibit the following: Absolute Figure 2008 (Rs.) 2009 (Rs.) Debtors Stock

50,000 90,000

Changes in Absolute Figure (Rs.)

% Changes

10,000 (15,000)

20 (16.67)

60,000 75,000

% change in the absolute figure is calculated in the following manner: % change =

Change in the absolute figure × 100 Absolute figure in the base year

% change in Debtors = 10,000 ÷ 50,000 × 100 = 20% % change in Stock =15,000 ÷ 90,000 × 100 = 16.67%

Modified Date: Mon, Jul 05, 2010 04:56:11 PM

Output Date: Tue, Jul 06, 2010 11:38:25 AM

Rev II

Project: Management Accounting_Debarshi Bhattacharyya ACE Pro India Pvt. Ltd. File: X:\Pearson\Management Accounting_Debarshi Bhattacharyya\MAIN\M02\LAYOUT_M02\M02_DEBA_ISBN_EN_SE_C02.indd

ANALYSIS AND INTERPRETATION OF FINANCIAL STATEMENTS

21

2.11.2 Advantages of Comparative Financial Statement Analysis i. Changes in the components of assets and liabilities can easily be known at a glance through a Comparative Balance Sheet. ii. Comparative Income Statements identify the trend of changes in sales, Gross Profit, Operating Profit and Net Profit. iii. Inter-firm comparison helps the management to evaluate the performance of the firm in relation to other firms in the same industry. Thus, the management can identify the strength and weakness of the firm and can adopt appropriate corrective measures too. iv. Helps to determine the long- and short-term solvency position of the firm. v. Trend of changes in the working capital, capital employed and proprietors’ fund can be easily known. 2.11.3 Disadvantages of Comparative Financial Statement Analysis i. When a particular item of Financial Statements does not appear in the base year but appears in the current year alone, then the meaningful computation of percentage change in the absolute figure of that item cannot be done. ii. Comparative analysis cannot exhibit a meaningful disclosure when uniformity in accounting concepts and conventions is not maintained over the years under analysis. iii. Such comparison becomes meaningless when the sizes of the firms differ. iv. Comparative analysis might express misleading disclosure if certain external factors like market conditions and business risk are not taken into consideration. v. It does not take into account the effect of changes in the price level. 2.12 COMMON-SIZE FINANCIAL STATEMENT ANALYSIS 2.12.1 Concept of Common-size Financial Statement Analysis Through horizontal or comparative analysis of Financial Statements, a comparative study of different items or elements of Financial Statements of two or more years can be made, but the relative position of each item of the Financial Statements cannot be known. To study the relative significance of every item in the Financial Statements, a vertical analysis is proved to be very much effective. An important tool to such analysis is the ‘Common-size Statements.’ There are two forms of such statements: i. Common-size Income Statement. ii. Common-size Balance Sheet. These statements are termed as Common-size as each item in such statements is represented as a percentage of one chosen base item. More clearly, every item as appearing in such Income Statement is represented as a percentage of a base item. In case of Common-size Income Statement, generally, net sales is considered as the base item. Accordingly, net sales is taken as 100 and the other items in the Income Statement are expressed as a percentage of the said net sales. On the other hand, in case of Common-size Balance Sheet, generally, the total of the Balance Sheet is considered as the base item. Accordingly, the total of the Balance Sheet is taken as 100 and all the elements of the Balance Sheet are expressed as the percentage of the said total of the Balance Sheet. A Common-size Statement exhibits the following pertinent information: i. Absolute figures of the items appearing in the Financial Statements of two or more years or two or more firms. ii. Percentage figure of every item in the Financial Statements as a percentage of the pre-selected base item.

Modified Date: Mon, Jul 05, 2010 04:56:11 PM

Output Date: Tue, Jul 06, 2010 11:38:25 AM

Rev II

Project: Management Accounting_Debarshi Bhattacharyya ACE Pro India Pvt. Ltd. File: X:\Pearson\Management Accounting_Debarshi Bhattacharyya\MAIN\M02\LAYOUT_M02\M02_DEBA_ISBN_EN_SE_C02.indd

22

MANAGEMENT ACCOUNTING

Illustration Say, Sales, Cost of Goods Sold, operating expenses and non-operating expenses of X Ltd for the year that ended on 31 December 2009 are Rs. 5,00,000, Rs. 3,00,000, Rs. 1,00,000 and Rs. 50,000 respectively, and those of Y Ltd for the year that ended on 31 December 2009 are Rs. 4,00,000, Rs. 2,80,000, Rs. 40,000 and Rs. 40,000 respectively. Then, the Common-size Income Statement for the year that ended on 31 December 2009 of X Ltd and Y Ltd would exhibit the following:

Less: Less: Less:

Sales Cost of Goods Sold Gross Profit Operating Expenses Operating Profit Non-operating Expenses Net Profit

Absolute Figures X Ltd (Rs.) Y Ltd (Rs.) 5,00,000 4,00,000 3,00,000 2,80,000 2,00,000 1,20,000 1,00,000 40,000 1,00,000 80,000 50,000 40,000 50,000 40,000

% of Sales X Ltd Y Ltd 100 100 60 70 40 30 20 10 20 20 10 10 10 10

% figures of the absolute figures are calculated in the following manner: Here, sales are considered as the base item and taken at 100. In case of X Ltd, Sales = Rs. 5,00,000 (which is considered as 100) % of the Cost of Goods Sold (as a percentage on sales) = 3,00,000 ÷ 5,00,000 × 100 = 60% % of the operating expenses (as a percentage on sales) = 1,00,000 ÷ 5,00,000 × 100 = 20%, and so on 2.12.2 Advantages of Common-size Statement i. It exhibits a relative significance of every item in the Financial Statements in percentage terms of its base figure. ii. In Common-size Income Statement, all items are converted in the percentage terms of sales. Such conversion helps the firm to ascertain how much portion of its sales is used up in cost of sales, operating expenses, interest and tax. iii. In a Common-size Balance Sheet, every item is presented in terms of the total of assets or liabilities, which helps an analyst to know how much percentage of total asset has been apportioned between the current and fixed asset and so on and how much percentage of the total liabilities is payable to the owners and the outsiders. iv. Nature of capital structure can be known at a glance, that is, the proportion of own capital and debt capital in total capital structure. v. It makes the inter-company comparison easier. It facilitates the comparison of firms with different sizes. vi. It helps to know the short- and long-term financial position of an enterprise. 2.12.3 Disadvantages of Common-size Statement i. Under this statement, the relative importance of each component of Financial Statements, on the whole, is exhibited, but no comparative study is done between the years. ii. In such statement, the time value of money is not considered. iii. This analysis does not prove to be very useful for any important decision-making. iv. It does not provide any meaningful disclosure while comparing the Common-size Statements of two or more years. v. It is not useful when it comes to making any segment-wise analysis.

Modified Date: Mon, Jul 05, 2010 04:56:11 PM

Output Date: Tue, Jul 06, 2010 11:38:25 AM

Rev II

Project: Management Accounting_Debarshi Bhattacharyya ACE Pro India Pvt. Ltd. File: X:\Pearson\Management Accounting_Debarshi Bhattacharyya\MAIN\M02\LAYOUT_M02\M02_DEBA_ISBN_EN_SE_C02.indd

ANALYSIS AND INTERPRETATION OF FINANCIAL STATEMENTS

23

2.12.4 Comparative Statement vs. Common-size Statement Comparative Statement 1. It is a form of horizontal analysis. 2. It is a comparison of different elements of Financial Statements between two or more years. 3. It fails to exhibit the relative importance of an individual component in the total. 4. Changes in values of different items in two different years are shown both in absolute as well as percentage terms. 5. It facilitates both intra- as well as inter-firm comparison. 6. It is very popular technique of Financial Statement Analysis.

Common-size Statement 1. It is a form of vertical analysis. 2. It is a comparison of different elements of Financial Statements with one chosen base item in the same year. 3. It shows the relative importance of each item of such statement in the total. 4. Changes in values of different items are shown as a percentage of a base item. 5. It mainly facilitates inter-firm comparison only. 6. It is not so popular when compared with comparative statement.

2.13 TREND ANALYSIS 2.13.1 Concept of Trend Analysis Trend Analysis is a form of comparative analysis of Financial Statements between two or more years. It indicates the trend of an individual item of the Financial Statements over a period of time. It helps to analyse the trend of each such item and thus, helps the management in the process of present and future policy-making. It is a useful tool for inter-firm comparison. When year-to-year change analysis of a comparative statement becomes cumbersome, more meaningful long-term trend comparison is done through Trend Analysis. 2.13.2 Steps in Computing Trend Values (or Percentages) i. A year (generally, a normal year, that is, a year where no abnormal incident has occurred) is chosen as the base year and the figures as appearing in the Financial Statements of that year are assigned as 100. ii. Necessary adjustments for price level should be made in the figures of other years when compared to this base year. iii. Now, trend percentages are to be computed in the following manner: Trend percentage =

Current year’s figure × 100 Base year’s figure

iv. Conclusion is then to be drawn studying the trend patterns of various items and also taking into account the absolute values of the concerned items. 2.13.3 Advantages of Trend Analysis i. Trend Analysis helps in analysing the growth in the financial activities of the firm at a glance. ii. Graphical presentation of trend line helps the management to take a quick decision on the concerned issue. iii. Trend values not only help the management in any important decision-making but in the controlling process as well. iv. Trend Analysis proves to be very useful for taking rational investment decisions. 2.13.4 Disadvantages of Trend Analysis i. Choosing of the base year is a difficult task. ii. It is not easy to identify the impact of inflation on the trend data and thus, the result might be misleading. iii. Changes in market conditions are not considered in Trend Analysis.

Modified Date: Mon, Jul 05, 2010 04:56:11 PM

Output Date: Tue, Jul 06, 2010 11:38:25 AM

Rev II

Project: Management Accounting_Debarshi Bhattacharyya ACE Pro India Pvt. Ltd. File: X:\Pearson\Management Accounting_Debarshi Bhattacharyya\MAIN\M02\LAYOUT_M02\M02_DEBA_ISBN_EN_SE_C02.indd

24

MANAGEMENT ACCOUNTING

Tutorial Notes to Students for Solving Problems i. For preparing a Comparative Balance Sheet, firstly take the absolute figures of the items appearing in the given Balance Sheets of both the years under two separate columns. Thereafter, calculate the increase or decrease in the absolute figures (i.e., the Absolute Change) of every item and write the said changes in a separate column. Then, calculate the percentage change in absolute figures on the basis of the base year’s figures under a separate column. ii. For preparing a Comparative Income Statement, firstly take the absolute figures of the items appearing in the given Income Statements of both the years under two separate columns. Thereafter, calculate the increase or decrease in the absolute figures (i.e., the absolute change) of every item and write the said changes in a separate column. Then, calculate the percentage change in the absolute figures on the basis of the base year’s figures under a separate column. iii. For preparing a Common-size Balance Sheet, firstly take the absolute figures of the items appearing in the given Balance Sheets of both the years (or companies) under two separate columns. Then, calculate the percentage of each absolute figure in the given Balance Sheets on the basis of the respective Balance Sheet Total of each year (or company), considering each Balance Sheet Total as 100. iv. For preparing a Common-size Income Statement, firstly take the absolute figures of the items appearing in the given Income Statements of both the years (and companies) under two separate columns. Then, calculate the percentage of each absolute figure of the given Income Statements on the basis of the respective sales figure of each year (or company), considering each Sales Figure as 100. v. For calculating the Trend Percentage of an item in the Financial Statements, firstly arrange the years chronologically with the corresponding absolute figures of the item taken into consideration. In this arrangement, the base year with the corresponding absolute figure of the given item is to be arranged first. Trend percentage of this base year is to be taken at 100. Then, the trend percentages of the absolute figures of the other years are to be calculated on the basis of the trend percentage of the base year in the following manner: Trend percentage of the other years =

Absolute figure of the other year × 100 Absolute figure of base year

2.14 WORKED-OUT PROBLEMS Problem 1 From the following Balance Sheet of New India Ltd, prepare a Comparative Balance Sheet: Liabilities Share Capital Reserves Debentures

2004 Rs. 30,000 12,000 60,000 1,02,000

2005 Rs. 60,000 10,000 60,000 1,30,000

Assets Fixed Assets Debtors Bank

2004 Rs. 80,000 10,000 12,000 1,02,000

2005 Rs. 1,02,000 20,000 8,000 1,30,000

[B.Com. (Hons), Calcutta University—2007] Solution Comparative Balance Sheet of New India Ltd as on 31 December 2004 and 31 December 2005 31 December 2004 Rs. Sources of Fund: (A) Proprietors’ Fund: Share Capital

31 December 2005 Rs.

30,000

60,000

Absolute Change Rs.

30,000

% Change

100.00 (Continued)

Modified Date: Mon, Jul 05, 2010 04:56:11 PM

Output Date: Tue, Jul 06, 2010 11:38:25 AM

Rev II

Project: Management Accounting_Debarshi Bhattacharyya ACE Pro India Pvt. Ltd. File: X:\Pearson\Management Accounting_Debarshi Bhattacharyya\MAIN\M02\LAYOUT_M02\M02_DEBA_ISBN_EN_SE_C02.indd

25

ANALYSIS AND INTERPRETATION OF FINANCIAL STATEMENTS

Reserves Less:

Miscellaneous Expenditure

(B) Loan Fund: Debentures Total (A + B) Applications of Fund: (C) Fixed Assets (D) Working Capital: Current Assets, Loans & Advances: Debtors Bank Total (E) Less: Current Liabilities & Provisions Total (F) D=E−F Total (C + D)

31 December 2004 Rs. 12,000 42,000 Nil 42,000

31 December 2005 Rs. 10,000 70,000 Nil 70,000

Absolute Change Rs. (2,000) 28,000 Nil 28,000

60,000 1,02,000

60,000 1,30,000

Nil 28,000

Nil 27.45

80,000

1,02,000

22,000

27.50

10,000 12,000 22,000 Nil Nil 22,000 1,02,000

20,000 8,000 28,000 Nil Nil 28,000 1,30,000

10,000 (4,000) 6,000 Nil Nil 6,000 28,000

100.00 (33.33) 27.27 Nil Nil 27.27 27.45

% Change (16.67) 66.67 Nil 66.67

Alternative Solution Comparative Balance Sheet of New India Ltd as on 31 December 2004 and 31 December 2005

(a) Fixed Assets (b) Current Assets, Loans and Advances: Debtors Bank Total Assets (a + b) (c) Proprietors’ Fund: Share Capital Reserves Less:

Miscellaneous Expenditure

(d) Loan Fund: Debentures (e) Current Liabilities & Provisions Total Liabilities (c + d + e)

31 December 2004 Rs. 80,000

31 December 2005 Rs. 1,02,000

Absolute Change Rs. 22,000

10,000 12,000 22,000 1,02,000

20,000 8,000 28,000 1,30,000

10,000 (4,000) 6,000 28,000

100.00 (33.33) 27.27 27.45

30,000 12,000 42,000 Nil 42,000

60,000 10,000 70,000 Nil 70,000

30,000 (2,000) 28,000 Nil 28,000

100.00 (16.67) 66.67 Nil 66.67

60,000 Nil 1,02,000

60,000 Nil 1,30,000

Nil Nil 28,000

Nil Nil 27.45

% Change 27.50

Problem 2 From the following figures of the Balance Sheet of X & Co., prepare a comparative Balance Sheet: Particulars Equity Share Capital Preference Share Capital 10% Debentures

1 January 2007 (Rs.) 4,00,000 2,00,000 1,50,000

1 January 2008 (Rs.) 5,00,000 1,00,000 1,00,000 (Continued)

Modified Date: Mon, Jul 05, 2010 04:56:11 PM

Output Date: Tue, Jul 06, 2010 11:38:25 AM

Rev II

Project: Management Accounting_Debarshi Bhattacharyya ACE Pro India Pvt. Ltd. File: X:\Pearson\Management Accounting_Debarshi Bhattacharyya\MAIN\M02\LAYOUT_M02\M02_DEBA_ISBN_EN_SE_C02.indd

26

MANAGEMENT ACCOUNTING

Particulars

1 January 2007 (Rs.) 40,000 2,00,000 2,20,000 5,70,000 2,80,000 80,000

Reserves & Surplus Long-term Loan Investment Fixed Assets Current Assets Current Liabilities

1 January 2008 (Rs.) 70,000 3,00,000 2,50,000 6,30,000 3,10,000 1,20,000

[B.Com. (Hons), Calcutta University—2009] Solution Comparative Balance Sheet of X & Co. as on 01 January 2007 and 01 January 2008

Sources of Fund: (A) Proprietors’ Fund: Equity Share Capital Preference Share Capital Reserves & Surplus Less:

Miscellaneous Expenditure

(B) Loan Fund: 10% Debentures Long-term Loans Total (A + B) Applications of Fund: (C) Fixed Assets (D) Investments (E) Working Capital: Current Assets Less: Current Liabilities Total (C + D + E)

1 January 2007 Rs.

1 January 2008 Rs.

Absolute Change Rs.

4,00,000 2,00,000 40,000 6,40,000 Nil 6,40,000

5,00,000 1,00,000 70,000 6,70,000 Nil 6,70,000

1,00,000 (1,00,000) 30,000 30,000 Nil 30,000

25 (50) 75 4.69 Nil 4.69

1,50,000 2,00,000 9,90,000

1,00,000 3,00,000 10,70,000

(50,000) 1,00,000 80,000

(33.33) 50 8.08

5,70,000 2,20,000

6,30,000 2,50,000

60,000 30,000

10.53 13.64

2,80,000 80,000 2,00,000 9,90,000

3,10,000 1,20,000 1,90,000 10,70,000

30,000 40,000 (10,000) 80,000

10.71 50 (5) 8.08

% Change

Problem 3 Following are the Balance Sheets of Rawdon Ltd as on 31 March 2008 and 31 March 2009: Liabilities Equity Share Capital Reserves & Surplus Secured Loan Current Liabilities

31 March 2008 Rs. 2,50,000 1,25,000 2,50,000 2,75,000

31 March 2009 Rs. 4,00,000 2,00,000 3,50,000 50,000

9,00,000

10,00,000

Assets Fixed Assets Investment Stock Debtors Other Current Assets

31 March 2008 Rs. 5,00,000 20,000 3,00,000 50,000 30,000 9,00,000

31 March 2009 Rs. 7,00,000 50,000 2,00,000 20,000 30,000 10,00,000

Prepare a Comparative Balance Sheet and interpret the financial position of the company.

Modified Date: Mon, Jul 05, 2010 04:56:11 PM

Output Date: Tue, Jul 06, 2010 11:38:25 AM

Rev II

Project: Management Accounting_Debarshi Bhattacharyya ACE Pro India Pvt. Ltd. File: X:\Pearson\Management Accounting_Debarshi Bhattacharyya\MAIN\M02\LAYOUT_M02\M02_DEBA_ISBN_EN_SE_C02.indd

27

ANALYSIS AND INTERPRETATION OF FINANCIAL STATEMENTS

Solution Comparative Balance Sheet of Rawdon Ltd as on 31 March 2008 and 31 March 2009

Sources of Fund: (A) Proprietors’ Fund: Equity Share Capital Reserves & Surplus Less:

Miscellaneous Expenditure

(B) Loan Fund: Secured Loan Total (A + B) Applications of Fund: (C) Fixed Assets (D) Investments (E) Working Capital: Current Assets, Loans & Advances: Stock Debtors Other Current Assets Total (F) Less: Current Liabilities & Provisions: Current Liabilities Total (G) E=F−G Total (C + D + E)

31 March 2008 Rs.

31 March 2009 Rs.

Absolute Change Rs.

% Change

2,50,000 1,25,000 3,75,000 Nil 3,75,000

4,00,000 2,00,000 6,00,000 Nil 6,00,000

1,50,000 75,000 2,25,000 Nil 2,25,000

60 60 60 Nil 60

2,50,000 6,25,000

3,50,000 9,50,000

1,00,000 3,25,000

40 52

5,00,000 20,000

7,00,000 50,000

2,00,000 30,000

40 150

3,00,000 50,000 30,000 3,80,000

2,00,000 20,000 30,000 2,50,000

(1,00,000) (30,000) Nil (1,30,000)

(33.33) (60) Nil (34.21)

2,75,000 2,75,000 1,05,000 6,25,000

50,000 50,000 2,00,000 9,50,000

(2,25,000) (2,25,000) 95,000 3,25,000

(81.82) (81.82) 90.48 52

Alternative Solution Comparative Balance Sheet of Rawdon Ltd as on 31 March 2008 and 31 March 2009

(a) Fixed Assets (b) Investments (c) Current Assets, Loans & Advances: Stock Debtors Other Current Assets Total Assets (a + b + c) (d) Proprietors’ Fund: Equity Share Capital Reserves & Surplus Less:

Miscellaneous Expenditure

(e) Loan Fund: Secured Loan

31 March 2008 Rs. 5,00,000 20,000

31 March 2009 Rs. 7,00,000 50,000

Absolute Change Rs. 2,00,000 30,000

3,00,000 50,000 30,000 3,80,000 9,00,000

2,00,000 20,000 30,000 2,50,000 10,00,000

(1,00,000) (30,000) Nil (1,30,000) 1,00,000

2,50,000 1,25,000 3,75,000 Nil 3,75,000

4,00,000 2,00,000 6,00,000 Nil 6,00,000

1,50,000 75,000 2,25,000 Nil 2,25,000

2,50,000

3,50,000

1,00,000

% Change 40 150 (33.33) (60) Nil (34.21) 11.11 60 60 60 Nil 60 40 (Continued)

Modified Date: Mon, Jul 05, 2010 04:56:11 PM

Output Date: Tue, Jul 06, 2010 11:38:25 AM

Rev II

Project: Management Accounting_Debarshi Bhattacharyya ACE Pro India Pvt. Ltd. File: X:\Pearson\Management Accounting_Debarshi Bhattacharyya\MAIN\M02\LAYOUT_M02\M02_DEBA_ISBN_EN_SE_C02.indd

28

MANAGEMENT ACCOUNTING

(f ) Current Liabilities & Provisions: Current Liabilities Total Liabilities (d + e + f)

31 March 2008 Rs.

31 March 2009 Rs.

Absolute Change Rs.

2,75,000 9,00,000

50,000 10,00,000

(2,25,000) 1,00,000

% Change

(81.82) 11.11

Comments on the Financial Position i. Proprietors’ fund has been increased more (by 60%) than the increase in the loan fund (by 40%) in the current year. It shows that the long-term solvency position of the company has improved during the current year. ii. Investments in Fixed Assets, Working Capital and outside the business—all are remarkably increased during the current year out of the increase in the capital employed in the business during the current year. It shows a sign of growth and prosperity of the company over the years. iii. As the Net Working Capital of the business has been increased a lot (by 90.48%), it indicates a sign of shortterm solvency of the company. iv. Overall financial condition of the business is very sound and it possesses the sign of growth potentiality.

Problem 4 Balance Sheets of Star Ltd for the years that ended on 31 December 2006 and 31 December 2007 are as follows: Liabilities Equity Share Capital 10% Preference Share Capital General Reserve 15% Debentures Creditors Bills Payable Tax Payable

31 December 2006 Rs. 8,00,000 6,00,000 4,00,000 2,00,000 3,00,000 1,00,000 2,00,000 26,00,000

31 December 2007 Rs. 8,00,000 6,00,000 4,90,000 3,00,000 4,00,000 1,50,000 3,00,000 30,40,000

Assets Building Land Plant Furniture Stock Debtors Cash

31 December 2006 Rs. 6,00,000 2,00,000 6,00,000 2,00,000 4,00,000 4,00,000 2,00,000 26,00,000

31 December 2007 Rs. 5,40,000 2,00,000 5,40,000 2,80,000 6,00,000 6,00,000 2,80,000 30,40,000

Prepare a Comparative Balance Sheet in a vertical form and offer your comments in brief on Fixed Assets. [B.Com. (Hons), Mumbai University—April 2008] Solution Comparative Balance Sheet of Star Ltd as on 31 December 2006 and 31 December 2007

Sources of Fund: (A) Proprietors’ Fund: Equity Share Capital 10% Preference Share Capital Reserves & Surplus: General Reserve

31 December 2006 Rs.

31 December 2007 Rs.

Absolute Change Rs.

% Change

8,00,000 6,00,000

8,00,000 6,00,000

Nil Nil

Nil Nil

4,00,000

4,90,000

90,000

22.5 (Continued)

Modified Date: Mon, Jul 05, 2010 04:56:11 PM

Output Date: Tue, Jul 06, 2010 11:38:25 AM

Rev II

Project: Management Accounting_Debarshi Bhattacharyya ACE Pro India Pvt. Ltd. File: X:\Pearson\Management Accounting_Debarshi Bhattacharyya\MAIN\M02\LAYOUT_M02\M02_DEBA_ISBN_EN_SE_C02.indd

29

ANALYSIS AND INTERPRETATION OF FINANCIAL STATEMENTS

Less:

Miscellaneous Expenditure

(B) Loan Fund: 15% Debentures Total (A + B) Applications of Fund: (C) Fixed Assets: Building Land Plant Furniture (D) Investments (E) Working Capital: Current Assets, Loans & Advances: Stock Debtors Cash Total (F) Less: Current Liabilities & Provisions: Creditors Bills Payable Tax Payable Total (G) E=F−G Total (C + D + E)

31 December 2006 Rs. 18,00,000 Nil 18,00,000

31 December 2007 Rs. 18,90,000 Nil 18,90,000

Absolute Change Rs. 90,000 Nil 90,000

2,00,000 20,00,000

3,00,000 21,90,000

1,00,000 1,90,000

50 9.50

6,00,000 2,00,000 6,00,000 2,00,000 16,00,000 Nil

5,40,000 2,00,000 5,40,000 2,80,000 15,60,000 Nil

(60,000) Nil (60,000) 80,000 (40,000) Nil

(10) Nil (10) 40 (2.5) Nil

4,00,000 4,00,000 2,00,000 10,00,000

6,00,000 6,00,000 2,80,000 14,80,000

2,00,000 2,00,000 80,000 4,80,000

50 50 40 48

3,00,000 1,00,000 2,00,000 6,00,000 4,00,000 20,00,000

4,00,000 1,50,000 3,00,000 8,50,000 6,30,000 21,90,000

1,00,000 50,000 1,00,000 2,50,000 2,30,000 1,90,000

33.33 50 50 41.67 57.50 9.50

% Change 5 Nil 5

Comment on the Fixed Assets The total investment in the Fixed Assets has come down (by 2.5%) during the current year. This happens due to a fall in the book values of Building and Plant, may be due to depreciation or sale, though the investment in furniture has been substantially increased (by 40%) during the current year. Although the total capital employed in the business has been increased (by 9.5%) during the current year, such an increase in the capital has not been invested in the Fixed Assets.

Problem 5 From the following information, prepare a Comparative Balance Sheet of Rezala Ltd as on 31 March 2008 and 31 March 2009 and interpret the financial position of the company: Liabilities Equity Shares of Rs. 100 each 12% Preference Shares of Rs. 100 each Statutory Reserve Profit & Loss A/c

31 March 2008 Rs. in ’000 800 300 50 50

31 March 2009 Rs. in ’000 900 200 50 30

Assets Land & Building Plant & Machinery Investment Stock

31 March 2008 Rs. in ’000 550 350 150 350

31 March 2009 Rs. in ’000 500 300 50 250 (Continued)

Modified Date: Mon, Jul 05, 2010 04:56:11 PM

Output Date: Tue, Jul 06, 2010 11:38:25 AM

Rev II

Project: Management Accounting_Debarshi Bhattacharyya ACE Pro India Pvt. Ltd. File: X:\Pearson\Management Accounting_Debarshi Bhattacharyya\MAIN\M02\LAYOUT_M02\M02_DEBA_ISBN_EN_SE_C02.indd

30

MANAGEMENT ACCOUNTING

31 March 2008 Rs. in ’000 150 170 60 270 150 2,000

Liabilities Revaluation Reserve General Reserve 10% Debentures (Rs. 100 each) Sundry Creditors Bills Payable

31 March 2009 Rs. in ’000 100 150 30 120 70 1,650

Assets Sundry Debtors Bills Receivable Cash at Bank

31 March 2008 Rs. in ’000 250 50 300

31 March 2009 Rs. in ’000 300 50 200

2,000

1,650

Solution Comparative Balance Sheet of Rezala Ltd as on 31 March 2008 and 31 March 2009

Sources of Fund: (A) Proprietors’ Fund: Equity Share Capital 12% Preference Share Capital Reserves & Surplus: Revaluation Reserve General Reserve Statutory Reserve Profit & Loss A/c Less:

Miscellaneous Expenditure

(B) Loan Fund: 10% Debentures Total (A + B) Applications of Fund: (C) Fixed Assets: Land & Building Plant & Machinery (D) Investment (E) Working Capital: Current Assets, Loans & Advances: Stock Sundry Debtors Bills Receivable Cash and Bank Total (F) Less: Current Liabilities & Provisions: Sundry Creditors Bills Payable Total (G) E=F−G Total (C + D + E)

Modified Date: Mon, Jul 05, 2010 04:56:11 PM

31 March 2008 Rs.

31 March 2009 Rs.

Absolute Change Rs.

% Change

800 300

900 200

100 (100)

12.50 (33.33)

150 170 50 50 1520 – 1520

100 150 50 30 1430 – 1430

(50) (20) – (20) (90) – (90)

(33.33) (11.76) – (40) (5.92) – (5.92)

60 1580

30 1460

(30) (120)

(50) (7.59)

550 350 900 150

500 300 800 50

(50) (50) (100) (100)

(9.09) (14.29) (11.11) (66.67)

350 250 50 300 950

250 300 50 200 800

(100) 50 – (100) (150)

(28.57) 20 – (33.33) (15.79)

270 150 420 530 1,580

120 70 190 610 1,460

(150) (80) (230) 80 (120)

(55.56) (53.33) (54.76) 15.09 (7.59)

Output Date: Tue, Jul 06, 2010 11:38:25 AM

Rev II

Project: Management Accounting_Debarshi Bhattacharyya ACE Pro India Pvt. Ltd. File: X:\Pearson\Management Accounting_Debarshi Bhattacharyya\MAIN\M02\LAYOUT_M02\M02_DEBA_ISBN_EN_SE_C02.indd

31

ANALYSIS AND INTERPRETATION OF FINANCIAL STATEMENTS

Comments on the Financial Position i. The total capital employed in the business has been reduced (by 7.59%) during the current year, as a result of which the investments in the Fixed Assets and outside the business have been reduced. It seems that the company has used the proceeds received from the new issue of equity share to finance the redemption of preference shares. ii. Working Capital of the company has been slightly increased during the current year. iii. The financial position of the company seems to be deteriorating as the total capital employed in the business has been reduced in the current year accompanied by a reduction in the value of Fixed Assets and Current Assets.

Problem 6 From the following information of Lawdon Ltd for the years that ended on 31 March 2008 and 31 March 2009, prepare a Comparative Balance Sheet as on 31 March 2008 and 31 March 2009 and interpret the financial position of the company: Liabilities Equity Share Capital General Reserve Sundry Creditors Bills Payable Outstanding Expenses

31 March 2008 Rs. 5,00,000 40,000 2,40,000 50,000

31 March 2009 Rs. 6,00,000 50,000 70,000 25,000

50,000



8,80,000

7,45,000

Assets Land & Building Plant & Machinery Investment Stock Debtors Bills Receivable Profit & Loss A/c Discount on Issue of Shares

31 March 2008 Rs. 2,00,000 1,50,000 2,97,000 40,000 50,000 63,000 70,000 10,000 8,80,000

31 March 2009 Rs. 1,00,000 2,00,000 2,00,000 30,000 50,000 1,00,000 60,000 5,000 7,45,000

Solution Comparative Balance Sheet of Lawdon Ltd as on 31 March 2008 and 31 March 2009

Sources of Fund: (A) Proprietors’ Fund: Equity Share Capital General Reserve Less:

Miscellaneous Expenditure: Profit & Loss A/c (Dr.) Discount on Issue of Shares

(B) Loan Fund Total (A + B) Applications of Fund: (C) Fixed Assets: Land & Building Plant & Machinery

31 March 2008 Rs.

31 March 2009 Rs.

Absolute Change Rs.

% Change

5,00,000 40,000 5,40,000

6,00,000 50,000 6,50,000

1,00,000 10,000 1,10,000

20 25 20.37

70,000 10,000 4,60,000 Nil 4,60,000

60,000 5,000 5,85,000 Nil 5,85,000

(10,000) (5,000) 1,25,000 Nil 1,25,000

(14.29) (50) 27.17 Nil 27.17

2,00,000 1,50,000 3,50,000

1,00,000 2,00,000 3,00,000

(1,00,000) 50,000 (50,000)

(50) 33.33 (16.67) (Continued)

Modified Date: Mon, Jul 05, 2010 04:56:11 PM

Output Date: Tue, Jul 06, 2010 11:38:25 AM

Rev II

Project: Management Accounting_Debarshi Bhattacharyya ACE Pro India Pvt. Ltd. File: X:\Pearson\Management Accounting_Debarshi Bhattacharyya\MAIN\M02\LAYOUT_M02\M02_DEBA_ISBN_EN_SE_C02.indd

32

MANAGEMENT ACCOUNTING

(D) Investments (E) Working Capital: Current Assets, Loans and Advances: Stock Debtors Bills Receivable Total (F) Less: Current Liabilities and Provisions: Sundry Creditors Bills Payable Outstanding Expenses Total (G) E=F−G Total (C + D + E)

31 March 2008 Rs. 2,97,000

31 March 2009 Rs. 2,00,000

Absolute Change Rs. (97,000)

40,000 50,000 63,000 1,53,000

30,000 50,000 1,00,000 1,80,000

(10,000) Nil 37,000 27,000

(25) Nil 58.73 17.65

2,40,000 50,000 50,000 3,40,000 (1,87,000) 4,60,000

70,000 25,000 Nil 95,000 85,000 5,85,000

(1,70,000) (25,000) (50,000) (2,45,000) 2,72,000 1,25,000

(70.83) (50) (100) (72.06) 145.45 27.17

% Change (32.66)

Comments on the Financial Position i. The total capital employed in the business has been increased (by 27.17%) during the current year, but the major portion of which has been invested in Current Assets and plant and machinery. ii. Investments in the other Fixed Assets and outside the business have been substantially reduced during the current year. iii. Working Capital of the company has been remarkably increased (by 145.45%) during the current year due to a remarkable decrease in the Current Liabilities and an increase in the Bills Receivable, which indicates a good sign of short-term solvency of the business. iv. The entire capital employed in the business represents the owner’s capital and no debt capital. Although this indicates a sign of total long-term solvency of the company, it fails to take advantage of trading on equity due to the total absence of debt capital in the capital structure. v. The financial position of the company is sound enough, but it is operating with the owner’s capital only.

Problem 7 From the following information, prepare a Comparative Income Statement: 2004 4,00,000 2,40,000 1,20,000

Sales Cost of Goods Sold Operating Expenses

2005 5,00,000 3,50,000 1,80,000

[B.Com. (Hons), Calcutta University—2006] Solution Comparative Income Statement of _______________ for the years that ended on 31 December 2004 and 31 December 2005

Less:

Sales Cost of Goods Sold Gross Profit

2004 Rs. 4,00,000 2,40,000 1,60,000

2005 Rs. 5,00,000 3,50,000 1,50,000

Absolute Change Rs. 1,00,000 1,10,000 (10,000)

% Change 25 45.83 (6.25) (Continued)

Modified Date: Mon, Jul 05, 2010 04:56:11 PM

Output Date: Tue, Jul 06, 2010 11:38:25 AM

Rev II

Project: Management Accounting_Debarshi Bhattacharyya ACE Pro India Pvt. Ltd. File: X:\Pearson\Management Accounting_Debarshi Bhattacharyya\MAIN\M02\LAYOUT_M02\M02_DEBA_ISBN_EN_SE_C02.indd

33

ANALYSIS AND INTERPRETATION OF FINANCIAL STATEMENTS

Less: Less:

2004 Rs. 1,20,000 40,000 Nil 40,000

Operating Expenses Operating Profit Non-operating Expense Net Profit / Net Loss

2005 Rs. 1,80,000 (30,000) Nil (30,000)

Absolute Change Rs. 60,000 (70,000) Nil (70,000)

% Change 50 (175) Nil (175)

Problem 8 From the following information of Jodha Ltd, prepare a Comparative Income Statement for the years that ended on 31 December 2008 and 31 December 2009:

Less:

2008 Rs. 1,00,000 70,000 30,000 5,000 6,000 19,000 1,000 5,000 13,000

Net Sales Cost of Goods Sold

Less:

Office & Administration Expenses Selling & Distribution Expenses

Less:

Interest Paid Tax Paid

2009 Rs. 1,10,000 75,000 35,000 7,000 5,000 23,000 2,000 6,000 15,000

Solution Comparative Income Statement of Jodha Ltd for the years that ended on 31 December 2008 and 31 December 2009

Less: Less:

Less: Less:

Net Sales Cost of Goods Sold Gross Profit Operating Expenses: Office & Administration Expenses Selling & Distribution Expenses Operating Profit Interest Paid Net Profit before Tax Tax Paid Net Profit after Tax

2008 Rs. 1,00,000 70,000 30,000

2009 Rs. 1,10,000 75,000 35,000

Absolute Change Rs. 10,000 5,000 5,000

5,000 6,000 19,000 1,000 18,000 5,000 13,000

7,000 5,000 23,000 2,000 21,000 6,000 15,000

2,000 (1,000) 4,000 1,000 3,000 1,000 2,000

% Change 10.00 7.14 16.67 40 16.67 21.05 100 16.67 20.00 15.38

Problem 9 From the following information of Lalbaba Ltd for the years that ended on 31 March 2008 and 31 March 2009, prepare a Comparative Income Statement and comment on the performance of the company. 2008 (Rs.) 4,75,000 25,000 3,00,000

Gross Sales Sales Return Purchase During the Year

2009 (Rs.) 5,20,000 10,000 3,20,000 (Continued)

Modified Date: Mon, Jul 05, 2010 04:56:11 PM

Output Date: Tue, Jul 06, 2010 11:38:25 AM

Rev II

Project: Management Accounting_Debarshi Bhattacharyya ACE Pro India Pvt. Ltd. File: X:\Pearson\Management Accounting_Debarshi Bhattacharyya\MAIN\M02\LAYOUT_M02\M02_DEBA_ISBN_EN_SE_C02.indd

34

MANAGEMENT ACCOUNTING

2008 (Rs.) 50,000 10,000 600 15,000 30,000 7,000 15,000

Opening Stock Closing Stock Direct Wages Paid Office & Administrative Expenses Selling & Distribution Expenses Interest Paid Tax Paid

2009 (Rs.) 60,000 12,000 800 25,000 25,000 8,000 20,000

Solution Comparative Income Statement of Lalbaba Ltd for the years that ended on 31 March 2008 and 31 March 2009

Less: Less: Less:

Less: Less:

Gross Sales Sales Return Net Sales Cost of Goods Sold1 Gross Profit Operating Expenses: Office & Administration Expenses Selling & Distribution Expenses Operating Profit Interest Paid Net Profit before Tax Tax Paid Net Profit after Tax

31 March 2008 Rs. 4,75,000 25,000 4,50,000 3,40,600 1,09,400

31 March 2009 Rs. 5,20,000 10,000 5,10,000 3,68,800 1,41,200

Absolute Change Rs. 45,000 (15,000) 60,000 28,200 31,800

15,000 30,000 64,400 7,000 57,400 15,000 42,400

25,000 25,000 91,200 8,000 83,200 20,000 63,200

10,000 5,000 26,800 1,000 25,800 5,000 20,800

% Change 9.47 (60) 13.33 8.28 29.07 66.67 16.67 41.61 14.29 44.95 33.33 49.06

Working Notes 1. Calculation of Cost of Goods Sold Cost of Goods Sold = Opening Stock + Purchases + Direct Expenses − Closing Stock. Cost of Goods Sold for the year that ended on 31 March 2008 = 50,000 + 3,00,000 + 600 − 10,000 = Rs. 3,40,600. Cost of Goods Sold for the year that ended on 31 March 2009 = 60,000 + 3,20,000 + 800 − 12,000 = Rs. 3,68,800.

Comments on the Financial Position i. Gross Profit for the current year has increased by 29.07% as a result of a greater increase in Sales by 9.47% over a lower increase in Cost of Goods Sold by 8.28%. ii. Operating Profit has also been increased remarkably by 41.61% due to a lower increase in the Operating Expenses. iii. Net Profit has also been increased substantially by 49.06% during the current year due to a marginal increase in the interest and constant tax rate. iv. The overall performance of the company is progressing strongly.

Modified Date: Mon, Jul 05, 2010 04:56:11 PM

Output Date: Tue, Jul 06, 2010 11:38:25 AM

Rev II

Project: Management Accounting_Debarshi Bhattacharyya ACE Pro India Pvt. Ltd. File: X:\Pearson\Management Accounting_Debarshi Bhattacharyya\MAIN\M02\LAYOUT_M02\M02_DEBA_ISBN_EN_SE_C02.indd

35

ANALYSIS AND INTERPRETATION OF FINANCIAL STATEMENTS

Problem 10 From the following information, prepare a Comparative Income Statement for the years that ended on 31 March 2008 and 31 March 2009: Trading and Profit & Loss A/c for the years that ended on 31 March 2008 and 31 March 2009

To Opening Stock To Purchases To Carriage Inward To Gross Profit c/d To Salaries of Staff To Salary of Manager To Insurance To Advertisement Expenses To General Expenses To Interest To Tax Payable To Net Profit b/d

31 March 2008 Rs. 1,50,000 7,70,000 20,000 2,85,000 12,25,000 20,000 5,000 4,000

31 March 2009 Rs. 3,00,000 9,80,000 25,000 5,75,000 18,80,000 45,000 15,000 3,500

50,000 75,000 5,000 600 1,40,400 3,00,000

95,000 85,000 6,000 700 3,39,800 5,90,000

By Sales Less: Sales Return By Closing Stock By Gross Profit b/d By Other Incomes

31 March 2008 Rs. 10,00,000 50,000 9,50,000 2,75,000 12,25,000 2,85,000 15,000

31 March 2009 Rs. 15,00,000 20,000 14,80,000 4,00,000 18,80,000 5,75,000 15,000

3,00,000

5,90,000

Solution Comparative Income Statement of _______________ for the years that ended on 31 March 2008 and 31 March 2009

Less: Less: Less:

Add: Less: Less:

Gross Sales Sales Return Net Sales Cost of Goods Sold1 Gross Profit Operating Expenses: Salaries to Staff Salary to Manager Insurance Advertisement Expenses General Expenses Operating Profit Other Incomes Net Profit before Interest & Tax Interest Net Profit before Tax Tax Net Profit after Tax

Modified Date: Mon, Jul 05, 2010 04:56:11 PM

31 March 2008 Rs. 10,00,000 50,000 9,50,000 6,65,000 2,85,000

31 March 2009 Rs. 15,00,000 20,000 14,80,000 9,05,000 5,75,000

Absolute Change Rs. 5,00,000 (30,000) 5,30,000 2,40,000 2,90,000

20,000 5,000 4,000 50,000 75,000 1,31,000 15,000 1,46,000 5,000 1,41,000 600 1,40,400

45,000 15,000 3,500 95,000 85,000 3,31,500 15,000 3,46,500 6,000 3,40,500 700 3,39,800

25,000 10,000 (500) 45,000 10,000 2,00,500 Nil 2,00,500 1,000 1,99,500 100 1,99,400

Output Date: Tue, Jul 06, 2010 11:38:25 AM

% Change 50 60 55.79 36.09 101.75 125 200 (12.50) 90 13.33 153.05 Nil 137.39 20 141.49 16.67 142.02

Rev II

Project: Management Accounting_Debarshi Bhattacharyya ACE Pro India Pvt. Ltd. File: X:\Pearson\Management Accounting_Debarshi Bhattacharyya\MAIN\M02\LAYOUT_M02\M02_DEBA_ISBN_EN_SE_C02.indd

36

MANAGEMENT ACCOUNTING

Working Notes 1. Calculation of Cost of Goods Sold Cost of Goods Sold = Opening Stock + Purchases + Direct Expenses − Closing Stock. Cost of Goods Sold for the year that ended on 31 March 2008 = 1,50,000 + 7,70,000 + 20,000 − 2,75,000 = Rs. 6,65,000. Cost of Goods Sold for the year that ended on 31 March 2009 = 3,00,000 + 9,80,000 + 25,000 − 4,00,000 = Rs. 9,05,000.

Problem 11 Following are the Income Statements and Balance Sheets of Discovery Ltd: Income Statement for the years that ended on 31 December 2008 and 31 December 2009 Particulars

2008 (Rs.) 4,00,000 60,000 50,000 1,90,000 60,000 20,000 25,000 35,000 10,000 24,000

Net Sales Opening Stock of Goods Closing Stock of Goods Purchases for the Year Direct Wages Factory Expenses Office & Administration Expenses Selling & Distribution Expenses Interest on Debentures Income Tax

2009 (Rs.) 5,00,000 70,000 40,000 2,10,000 70,000 30,000 35,000 25,000 20,000 32,000

Balance Sheets as on 31 December 2008 and 31 December 2009 Liabilities Equity Share Capital (of Rs. 100 each) Preference Share Capital Reserves & Surplus Debentures Creditors Bills Payable

31 December 2008 Rs. 2,00,000

31 December 2009 Rs. 3,00,000

1,00,000 1,60,000 80,000 50,000 10,000 6,00,000

2,00,000 2,20,000 1,30,000 40,000 10,000 9,00,000

Assets Fixed Assets Investment Stock Debtors Cash & Bank Preliminary Expense

31 December 2008 Rs. 2,80,000

31 December 2009 Rs. 4,50,000

1,00,000 70,000 80,000 50,000 20,000 6,00,000

1,80,000 1,20,000 1,00,000 40,000 10,000 9,00,000

Prepare the Comparative Income Statement and Balance Sheet of 2 years of the company. Solution Comparative Income Statement of Discovery Ltd for the years that ended on 31 December 2008 and 31 December 2009

Less: Less:

Net Sales Cost of Goods Sold1 Gross Profit Operating Expenses: Office & Administration Expenses Selling & Distribution Expenses Operating Profit

2008 Rs. 4,00,000 2,80,000 1,20,000

2009 Rs. 5,00,000 3,40,000 1,60,000

Absolute Change Rs. 1,00,000 60,000 40,000

25,000 35,000 60,000

35,000 25,000 1,00,000

10,000 (10,000) 40,000

% Change 25.00 21.43 33.33 40 (28.57) 66.67 (Continued)

Modified Date: Mon, Jul 05, 2010 04:56:11 PM

Output Date: Tue, Jul 06, 2010 11:38:25 AM

Rev II

Project: Management Accounting_Debarshi Bhattacharyya ACE Pro India Pvt. Ltd. File: X:\Pearson\Management Accounting_Debarshi Bhattacharyya\MAIN\M02\LAYOUT_M02\M02_DEBA_ISBN_EN_SE_C02.indd

ANALYSIS AND INTERPRETATION OF FINANCIAL STATEMENTS

Less: Less:

2008 Rs. 10,000 50,000 24,000 26,000

Interest on Loan Net Profit before Tax Income Tax Net Profit after Tax

2009 Rs. 20,000 80,000 32,000 48,000

Absolute Change Rs. 10,000 30,000 8,000 22,000

37

% Change 100 60 33.33 84.62

Comparative Balance Sheet of Discovery Ltd as on 31 December 2008 and 31 December 2009

Sources of Fund: (A) Proprietors’ Fund: Equity Share Capital Preference Share Capital Reserves & Surplus Less:

Miscellaneous Expenditure: Preliminary Expenses

(B) Loan Fund: Debentures Total (A + B) Applications of Fund: (C) Fixed Assets (D) Investments (E) Working Capital: Current Assets, Loans & Advances: Stock Debtors Cash & Bank Total (F) Less: Current Liabilities & Provisions: Creditors Bills Payable Total (G) E=F−G Total (C + D + E)

31 December 2008 Rs.

31 December 2009 Rs.

Absolute Change Rs.

% Change

2,00,000 1,00,000 1,60,000 4,60,000

3,00,000 2,00,000 2,20,000 7,20,000

1,00,000 1,00,000 60,000 2,60,000

50 100 37.50 56.52

20,000 4,40,000

10,000 7,10,000

(10,000) 2,70,000

(50) 61.36

80,000 5,20,000

1,30,000 8,40,000

50,000 3,20,000

62.50 61.54

2,80,000 1,00,000

4,50,000 1,80,000

1,70,000 80,000

60.71 80

70,000 80,000 50,000 2,00,000

1,20,000 1,00,000 40,000 2,60,000

50,000 20,000 (10,000) 60,000

71.43 25 (20) 30

50,000 10,000 60,000 1,40,000 5,20,000

40,000 10,000 50,000 2,10,000 8,40,000

(10,000) Nil (10,000) 70,000 3,20,000

(20) Nil (16.67) 50 61.54

Working Notes 1. Calculation of Cost of Goods Sold Cost of Goods Sold = Opening Stock + Purchases + Direct Expenses − Closing Stock. Cost of Goods Sold for the year that ended on 31 December 2008 = 60,000 + 1,90,000 + 60,000 + 20,000 − 50,000 = Rs. 2,80,000. Cost of Goods Sold for the year that ended on 31 December 2009 = 70,000 + 2,10,000 + 70,000 + 30,000 − 40,000 = Rs. 3,40,000.

Modified Date: Mon, Jul 05, 2010 04:56:11 PM

Output Date: Tue, Jul 06, 2010 11:38:25 AM

Rev II

Project: Management Accounting_Debarshi Bhattacharyya ACE Pro India Pvt. Ltd. File: X:\Pearson\Management Accounting_Debarshi Bhattacharyya\MAIN\M02\LAYOUT_M02\M02_DEBA_ISBN_EN_SE_C02.indd

38

MANAGEMENT ACCOUNTING

Problem 12 From the following information, prepare a Common-size Income Statement: 2008 (Rs.) 2,00,000 40,000 85,000 10,000 40%

Sales Cost of Goods Sold Operating Expenses Non-operating Expenses Tax Rate

2009 (Rs.) 2,25,000 55,000 66,000 6,000 40%

Solution Common-size Income Statement for the years that ended on 31 December 2008 and 31 December 2009

Less: Less: Less: Less:

Net Sales Cost of Goods Sold Gross Profit Operating Expenses Operating Profit Non-operating Expenses Profit before Tax Tax @ 40% on Profit Before Tax Profit after Tax

Absolute Figures 2008 (Rs.) 2009 (Rs.) 2,00,000 2,25,000 40,000 55,000 1,60,000 1,70,000 85,000 66,000 75,000 1,04,000 10,000 6,000 65,000 98,000 26,000 39,200 39,000 58,800

% of Net Sales 2008 2009 100 100 20 24.44 80 75.56 42.5 29.34 37.5 46.22 5 2.67 32.5 43.55 13 17.42 19.5 26.13

Problem 13 From the following information of a company for the year 2008 and 2009, prepare a Common-size Income Statement and make your comment on the performance of the company: 2008 (Rs.) 1,00,000 60,000 30,000 3,000 2,000 50%

Net Sales Cost of Goods Sold Selling, General & Administrative Expenses Interest Paid Interest Received Income Tax

2009 (Rs.) 1,10,000 65,700 37,000 6,200 3,300 50%

Solution Common-size Income Statement for the years 2008 and 2009

Less: Less:

Add:

Net Sales Cost of Goods Sold Gross Profit Operating Expenses: Selling, General & Administrative Expenses Operating Profit Interest Income

Absolute Figures 2008 (Rs.) 2009 (Rs.) 1,00,000 1,10,000 60,000 65,700 40,000 44,300

30,000 10,000 2,000

37,000 7,300 3,300

% of Net Sales 2008 2009 100 100 60 59.73 40 40.27

30 10 2

33.64 6.63 3 (Continued)

Modified Date: Mon, Jul 05, 2010 04:56:11 PM

Output Date: Tue, Jul 06, 2010 11:38:25 AM

Rev II

Project: Management Accounting_Debarshi Bhattacharyya ACE Pro India Pvt. Ltd. File: X:\Pearson\Management Accounting_Debarshi Bhattacharyya\MAIN\M02\LAYOUT_M02\M02_DEBA_ISBN_EN_SE_C02.indd

ANALYSIS AND INTERPRETATION OF FINANCIAL STATEMENTS

Less: Less:

Profit before Interest & Tax Interest Profit before Tax Tax @ 50% on Profit before Tax Net Profit

Absolute Figures 2008 (Rs.) 2009 (Rs.) 12,000 10,600 3,000 6,200 9,000 4,400 4,500 2,200 4,500 2,200

39

% of Net Sales 2008 2009 12 9.67 3 5.67 9 4 4.50 2 4.50 2

Comments 1. Both sales as well as the Cost of Goods Sold have been increased in 2009 as compared to 2008, but sales have been increased at a higher rate than the Cost of Goods Sold. As a consequence, the Gross Profit for 2009 has been increased from 40% of sales to 40.27% of sales, indicating a marginal increase in the profit. 2. Operating expenses have been increased from 30% of sales to 33.64% of sales in 2009 as compared to 2008 resulting in a decrease in the Operating Profit from 10% to 6.63% of sales. 3. Interest cost has been increased from 3% to 5.67% of sales in 2009 resulting in a decline in the Net Profit before tax from 9% to 4% of sales in 2009. 4. A marginal decrease in the tax amount could not stop the Net Profit to fall from 4.5% to 2% of sales in 2009. 5. Therefore, we can conclude that profitability of the concern is declining over the period of time.

Problem 14 From the following information, make a comparable Profit & Loss A/c of A Ltd and B Ltd in terms of percentage alone: A Ltd Rs. in Lakhs 15 12 0.7 0.8 0.2

Sales Cost of Goods Sold Administrative Expenses Selling Expenses Financial Charge

B Ltd Rs. in Lakhs 80 60 5 7 2

[B.Com. (Hons), Calcutta University—2008] Solution Comparable Profit & Loss A/c of A Ltd and B Ltd for the period A Ltd Absolute Figure

Less: Less:

Less:

Sales Cost of Goods Sold Gross Profit Operating Expenses: Administrative Expenses Selling Expenses Operating Profit Non-operating Expenses: Financial Charge Net Profit

Modified Date: Mon, Jul 05, 2010 04:56:11 PM

B Ltd % of Sales

Absolute Figure

% of Sales 100 75 25

Rs. in lakhs 15 12 3

100 80 20

Rs. in lakhs 80 60 20

0.7 0.8 1.5

4.7 5.3 10

5 7 8

6.25 8.75 10

0.2 1.3

1.3 8.7

2 6

2.5 7.5

Output Date: Tue, Jul 06, 2010 11:38:25 AM

Rev II

Project: Management Accounting_Debarshi Bhattacharyya ACE Pro India Pvt. Ltd. File: X:\Pearson\Management Accounting_Debarshi Bhattacharyya\MAIN\M02\LAYOUT_M02\M02_DEBA_ISBN_EN_SE_C02.indd

40

MANAGEMENT ACCOUNTING

Problem 15 Following data are available from M & Co. for 2006 and 2007. You are required to prepare a Common-size Income Statement. Particulars Add: Add:

2006 (Rs.) 10,00,000 50,000 1,00,000 11,50,000 8,00,000 2,00,000 50,000 1,00,000 11,50,000

Sales Commission Received Discount Received Cost of Goods Sold Operating Expenses Non-operating Expenses Profit

2007 (Rs.) 12,50,000 75,000 1,25,000 14,50,000 9,00,000 3,00,000 75,000 1,75,000 14,50,000

[B.Com. (Hons), Calcutta University—2009] Solution Common-size Income Statement for the years that ended on 31 December 2006 and 31 December 2007

Less: Add: Add: Less: Less:

Sales Cost of Goods Sold Gross Profit Commission Received Discount Received Operating Expenses Operating Profit Non-operating Expenses Net Profit

Absolute Figures 2006 (Rs.) 2007 (Rs.) 10,00,000 12,50,000 8,00,000 9,00,000 2,00,000 3,50,000 50,000 75,000 1,00,000 1,25,000 3,50,000 5,50,000 2,00,000 3,00,000 1,50,000 2,50,000 50,000 75,000 1,00,000 1,75,000

% of Net Sales 2006 2007 100 100 80 72 20 28 5 6 10 10 35 44 20 24 15 20 5 6 10 14

Problem 16 With the help of the following information for the year that ended in 2006, prepare a Common-size Income Statement: Selling and Distribution Expense Administration Expense Cost of Sales Income Tax Net Income after Tax Other Income

Rs. 10,000 Rs. 20,000 75% of net sales 20% of Net Profit before tax Rs. 48,000 Rs. 10,000

[B.Com. (Hons), Calcutta University—2007] Solution Common-size Income Statement for the year that ended on 31 December 2006

Less:

Absolute Figure (Rs.) 3,20,000 2,40,000

Net Sales Cost of Sales

% of Net Sales 100.00 75.00 (Continued)

Modified Date: Mon, Jul 05, 2010 04:56:11 PM

Output Date: Tue, Jul 06, 2010 11:38:25 AM

Rev II

Project: Management Accounting_Debarshi Bhattacharyya ACE Pro India Pvt. Ltd. File: X:\Pearson\Management Accounting_Debarshi Bhattacharyya\MAIN\M02\LAYOUT_M02\M02_DEBA_ISBN_EN_SE_C02.indd

41

ANALYSIS AND INTERPRETATION OF FINANCIAL STATEMENTS

Less:

Add:

Less:

Gross Profit Operating Expenses: Administration Expense Selling and Distribution Expense Operating Profit Non-operating Income: Other Income Profit before Tax Tax @ 20% on Profit before Tax Profit after Tax

Absolute Figure (Rs.) 80,000

% of Net Sales 25.00

20,000 10,000 50,000

6.25 3.12 15.62

10,000 60,000 12,000 48,000

3.12 18.75 3.75 15.00

Working Notes Given, Tax = 20% of Net Profit before Tax. If Profit before Tax = 100, then Tax = 20. Net Profit after Tax = Profit before tax − Tax = 100 − 20 = 80. Again, given Net Profit after Tax = Rs. 48,000. Net Profit before Tax = 100 ÷ 80 × 48,000 = Rs. 60,000. Tax = 20 ÷ 80 × 48,000 = Rs. 12,000.

Less:

Add:

Rs. 60,000

Net Profit before Tax Non-operating Income Included in the above: Other Income Operating Profit Operating Expenses: Administration Expense Selling & Distribution Expense Gross Profit

10,000 50,000 20,000 10,000 80,000

Again, Cost of Sales = 75% of Net Sales. If Net Sales = 100, then Cost of Sales = 75. Gross Profit = Net Sales − Cost of Sales = 100 − 75 = 25. Net Sales = 100 ÷ 25 × 80,000 = Rs. 3,20,000. Cost of Sales = 75 ÷ 25 × 80,000 = Rs. 2,40,000.

Problem 17 From the following information, prepare a Common-size Balance Sheet for 2008 and 2009: 2008 (Rs.) 1,00,000 70,000 80,000 2,50,000 1,00,000 20,000 20,000 20,000 60,000 30,000 2,50,000

Fixed Assets Investment Current Assets Total Equity Share Capital 8% Preference Share Capital Reserves & Surplus Secured Loan Unsecured Loan Current Liabilities & Provisions Total

Modified Date: Mon, Jul 05, 2010 04:56:11 PM

Output Date: Tue, Jul 06, 2010 11:38:25 AM

2009 (Rs.) 1,20,000 70,000 90,000 2,80,000 1,00,000 20,000 30,000 30,000 70,000 30,000 2,80,000

Rev II

Project: Management Accounting_Debarshi Bhattacharyya ACE Pro India Pvt. Ltd. File: X:\Pearson\Management Accounting_Debarshi Bhattacharyya\MAIN\M02\LAYOUT_M02\M02_DEBA_ISBN_EN_SE_C02.indd

42

MANAGEMENT ACCOUNTING

Solution Common-size Balance Sheet as on 31 December 2008 and as on 31 December 2009 Absolute Figures 2008 (Rs.) 2009 (Rs.) Assets: Fixed Assets Investment Current Assets Total Assets Liabilities: Equity Share Capital 8% Preference Share Capital Reserves & Surplus Secured Loan Unsecured Loan Current Liabilities & Provisions Total Liabilities

% of Balance Sheet Total 2008 2009

1,00,000 70,000 80,000 2,50,000

1,20,000 70,000 90,000 2,80,000

40 28 32 100

42.86 25 32.14 100

1,00,000 20,000 20,000 20,000 60,000 30,000 2,50,000

1,00,000 20,000 30,000 30,000 70,000 30,000 280,000

40 8 8 8 24 12 100

35.71 7.14 10.71 10.72 25 10.72 100

Problem 18 From the following information as furnished by Aaj Ltd and Kal Ltd as on 31 March 2009, prepare a Commonsize Balance Sheet as on that date and make your comments on the position of the companies:

Less:

Aaj Ltd Rs. 3,00,000 1,05,000 1,95,000 50,000 30,000 15,000 6,000 22,000 3,18,000 1,00,000 60,000 35,000 70,000 50,000 3,000 3,18,000

Fixed Assets (at Cost) Provision for Depreciation Investment Sundry Debtors Stock Prepaid Expenses Cash & Bank Share Capital General Reserve Revenue Reserve Long-term Loan Sundry Creditors Provision for Tax

Kal Ltd Rs. 5,00,000 2,80,000 2,20,000 – 80,000 29,000 18,000 67,000 4,14,000 1,50,000 79,000 40,000 85,000 50,000 10,000 4,14,000

Solution Common-size Balance Sheet of Aaj Ltd and Kal Ltd as on 31 March 2009 Absolute Figures

Less:

Assets: Fixed Assets (at Cost) Provision for Depreciation

% of Balance Sheet Total

Aaj Ltd Rs.

Kal Ltd Rs.

Aaj Ltd

Kal Ltd

3,00,000 1,05,000

5,00,000 2,80,000

94.33 33.02

120.77 67.63 (Continued)

Modified Date: Mon, Jul 05, 2010 04:56:11 PM

Output Date: Tue, Jul 06, 2010 11:38:25 AM

Rev II

Project: Management Accounting_Debarshi Bhattacharyya ACE Pro India Pvt. Ltd. File: X:\Pearson\Management Accounting_Debarshi Bhattacharyya\MAIN\M02\LAYOUT_M02\M02_DEBA_ISBN_EN_SE_C02.indd

43

ANALYSIS AND INTERPRETATION OF FINANCIAL STATEMENTS

Absolute Figures

Investments Current Assets: Sundry Debtors Stock Prepaid Expenses Cash & Bank Total Assets Liabilities: Share Capital Reserves & Surplus: General Reserve Revenue Reserve Long-term Loan Current Liabilities & Provisions: Sundry Creditors Provision for Taxation Total Liabilities

Aaj Ltd Rs. 1,95,000 50,000

Kal Ltd Rs. 2,20,000 –

30,000 15,000 6,000 22,000 3,18,000

% of Balance Sheet Total Aaj Ltd

Kal Ltd

61.31 15.73

53.14 –

80,000 29,000 18,000 67,000 4,14,000

9.43 4.72 1.89 6.92 100

19.32 7 4.35 16.18 100

1,00,000

1,50,000

31.45

36.23

60,000 35,000 70,000

79,000 40,000 85,000

18.87 11.01 22.01

19.08 9.66 20.53

50,000 3,000 3,18,000

50,000 10,000 4,14,000

15.72 0.94 100

12.08 2.42 100

Comments 1. Investments in Fixed Assets and outside the business are more than three times than the investment in the Current Assets of Aaj Ltd. On the other hand, investments in Fixed Assets and outside the business are almost equal to the investment in the Current Assets of Kal Ltd. This indicates that a major portion of the capital employed in Aaj Ltd has been invested in the Long-term Assets, whereas the total capital employed in Kal Ltd has been almost evenly invested in the Long-term as well as Short-term assets. 2. The proportion of equity and debt capital in the capital structure of both the companies is almost the same. Debt–equity ratio of both the companies indicates a sound long-term solvency position. 3. The liquidity position of both the companies seems to be prudent as the ratio of Current Assets to Current Liabilities in terms of percentage to total assets is around 2:1. 4. The financial position of both the companies is sound.

Problem 19 From the following information as obtained from the books of Explorer Ltd, prepare a Common-size Income Statement and a Common-size Balance Sheet of 2 years: Particulars Sales Cost of Goods Sold Operating Expenses (other) Non-operating Expenses Non-operating Incomes Tax Rate Fixed Assets Investments Current Assets

2008 Rs. 4,00,000 1,80,000 70,000 40,000 10,000 50% 3,00,000 1,00,000 2,00,000

2009 Rs. 6,00,000 2,40,000 80,000 60,000 20,000 50% 5,00,000 2,00,000 3,00,000 (Continued)

Modified Date: Mon, Jul 05, 2010 04:56:11 PM

Output Date: Tue, Jul 06, 2010 11:38:25 AM

Rev II

Project: Management Accounting_Debarshi Bhattacharyya ACE Pro India Pvt. Ltd. File: X:\Pearson\Management Accounting_Debarshi Bhattacharyya\MAIN\M02\LAYOUT_M02\M02_DEBA_ISBN_EN_SE_C02.indd

44

MANAGEMENT ACCOUNTING

2008 Rs. 10,000 1,50,000 50,000 1,40,000 1,20,000 50,000 1,00,000

Particulars Miscellaneous Expenditure Equity Share Capital Preference Share Capital Reserves & Surplus Secured Loan Unsecured Loan Current Liabilities & Provisions

2009 Rs. 20,000 3,00,000 2,00,000 2,20,000 1,50,000 30,000 1,20,000

Solution Common-size Income Statement of Explorer Ltd for the years that ended on 31 December 2008 and 31 December 2009

Less: Less: Add: Less: Less:

Sales Cost of Goods Sold Gross Profit Operating Expenses Operating Profit Non-operating Incomes Non-operating Expenses Profit before Tax Tax @ 50% on Profit before Tax Profit after Tax

Absolute Figures 2008 (Rs.) 2009 (Rs.) 4,00,000 6,00,000 1,80,000 2,40,000 2,20,000 3,60,000 70,000 80,000 1,50,000 2,80,000 10,000 20,000 1,60,000 3,00,000 40,000 60,000 1,20,000 2,40,000 60,000 1,20,000 60,000 1,20,000

% of Net Sales 2008 2009 100 100 45 40 55 60 17.50 13.33 37.50 46.67 2.50 3.33 40 50 10 10 30 40 15 20 15 20

Common-size Balance Sheet of Explorer Ltd as on 31 December 2008 and as on 31 December 2009 Absolute Figures 2008 (Rs.) 2009 (Rs.) Assets: Fixed Assets Investment Current Assets Total Assets Liabilities: Proprietors’ Fund: Equity Share Capital 8% Preference Share Capital Reserves & Surplus Less:

Miscellaneous Expenditure

% of Balance Sheet Total 2008 2009

3,00,000 1,00,000 2,00,000 6,00,000

5,00,000 2,00,000 3,00,000 10,00,000

50 16.67 33.33 100

50 20 30 100

1,50,000 50,000 1,40,000 3,40,000 10,000 3,30,000

3,00,000 2,00,000 2,20,000 7,20,000 20,000 7,00,000

25 8.33 23.34 56.67 1.67 55

30 20 22 72 2 70

1,20,000 50,000 1,00,000 6,00,000

1,50,000 30,000 1,20,000 10,00,000

20 8.33 16.67 100

15 3 12 100

Loan Fund: Secured Loan Unsecured Loan Current Liabilities & Provisions Total Liabilities

Modified Date: Mon, Jul 05, 2010 04:56:11 PM

Output Date: Tue, Jul 06, 2010 11:38:25 AM

Rev II

Project: Management Accounting_Debarshi Bhattacharyya ACE Pro India Pvt. Ltd. File: X:\Pearson\Management Accounting_Debarshi Bhattacharyya\MAIN\M02\LAYOUT_M02\M02_DEBA_ISBN_EN_SE_C02.indd

45

ANALYSIS AND INTERPRETATION OF FINANCIAL STATEMENTS

Problem 20 From the following figures, calculate the trend percentages taking 2001 as the base year and make your comment: Year 2001 2002 2003 2004 2005 2006

Sales 6,00,000 7,00,000 8,20,000 9,50,000 10,00,000 11,00,000

Cost of Goods Sold (Rs.) 4,00,000 4,75,000 6,00,000 7,90,000 8,15,000 9,05,000

Inventory (Rs.) 2,00,000 2,20,000 2,40,000 2,50,000 2,60,000 2,80,000

Solution Statement of Trend Analysis (Base year = 2001) Sales Amount Rs. 6,00,000 7,00,000 8,20,000 9,50,000 10,00,000 11,00,000

Year 2001 2002 2003 2004 2005 2006

Trend %

Year

100 116.7 136.7 158.3 166.7 183.33

2001 2002 2003 2004 2005 2006

Cost of Goods Sold Amount Trend % Rs. 4,00,000 100 4,75,000 118.75 6,00,000 150 7,90,000 197.5 8,15,000 203.75 9,05,000 226.25

Year 2001 2002 2003 2004 2005 2006

Inventory Amount Rs. 2,00,000 2,20,000 2,40,000 2,50,000 2,60,000 2,80,000

Trend % 100 110 120 125 130 140

Comments 1. Sales have steadily increased over the years, especially in the recent times, but the corresponding costs of goods sold have also been alarmingly increased at a higher rate. 2. Stock of goods has also been gradually increased over the years. Accumulation of more stock than the required quantity might hinder the profitability position of the firm. Therefore, further investigation is required in this respect.

Problem 21 From the following data as obtained from the Financial Statements of Kailash Ltd, calculate the trend percentage taking 2007 as the base year: 2007 (Rs.) 1,80,000 80,000 10,000 90,000

Sales Purchases Expenses Profit

2008 (Rs.) 2,00,000 1,00,000 15,000 85,000

2009 (Rs.) 2,60,000 1,10,000 20,000 1,30,000

Solution Statement of Trend Analysis (Base year = 2007) Year 2004 2005 2006

Sales Amount

Trend

Rs. 1,80,000 2,00,000 2,60,000

% 100 111.11 144.44

Year 2004 2005 2006

Modified Date: Mon, Jul 05, 2010 04:56:11 PM

Purchases Amount Rs. 80,000 1,00,000 1,10,000

Trend % 100 125 137.5

Year 2004 2005 2006

Expenses Amount Trend Rs. % 10,000 100 15,000 150 85,000 850

Output Date: Tue, Jul 06, 2010 11:38:25 AM

Year 2004 2005 2006

Profit Amount Rs. 90,000 85,000 1,30,000

Trend % 100 94.44 144.4

Rev II

Project: Management Accounting_Debarshi Bhattacharyya ACE Pro India Pvt. Ltd. File: X:\Pearson\Management Accounting_Debarshi Bhattacharyya\MAIN\M02\LAYOUT_M02\M02_DEBA_ISBN_EN_SE_C02.indd

46

MANAGEMENT ACCOUNTING

Problem 22 From the following supplied trend percentage, prepare a comparative statement of Current Assets in absolute value taking 2003 as the base year: 2004 120 130 160 175 110

Trend Percentage 2005 130 140 220 250 150

Corresponding Value of Current Assets 2006 3,600 – Cash & Bank 6,800 – Debtors 4,000 – Finished Goods 4,500 – Work in Progress 1,750 – Raw Materials

2006 150 200 250 300 175

[B.Com. (Hons), Calcutta University—2007] Solution Statement of Trend Analysis of Current Assets (Base year = 2003) Year 2003 2004 2005 2006

Cash & Bank Amount Trend Rs. % 2,400 100 2,880 120 3,120 130 3,600 150

Debtors Amount Trend Rs. % 3,400 100 4,420 130 4,760 140 6,800 200

Finished Goods Amount Trend Rs. % 1,600 100 2,560 160 3,520 220 4,000 250

WIP Amount Rs. 1,500 2,625 3,750 4,500

Trend % 100 175 250 300

Raw Materials Amount Trend Rs. % 1,000 100 1,100 110 1,500 150 1,750 175

Working Notes Value of Current Assets in the Base Year (i.e., in 2003) Cash & Bank = 100 ÷ 150 × 3,600 = Rs. 2,400. Debtors = 100 ÷ 200 × 6,800 = Rs. 3,400. Finished Goods = 100 ÷ 250 × 4,000 = Rs. 1,600. Work in Progress = 100 ÷ 300 × 4,500 = Rs. 1,500. Raw Materials = 100 ÷ 175 × 1,750 = Rs. 1,000.

Problem 23 The following is the financial information of ZN Ltd for 3 years that ended on 31 December every year: Particulars Share Capital Gross Profit Current Liabilities Fixed Assets Long-term Loan Cost of Goods Sold Working Capital Net Worth Current Assets Sales

2005 (Rs.) 1,50,000 3,50,000 40,000 2,40,000 1,00,000 ? 60,000 2,00,000 ? 5,50,000

2006 (Rs.) 1,80,000 3,50,000 ? 2,50,000 ? 4,00,000 50,000 2,20,000 1,20,000 7,50,000

2007 (Rs.) 1,90,000 4,00,000 ? 2,35,000 1,20,000 3,00,000 1,40,000 2,55,000 2,00,000 ? (Continued)

Modified Date: Mon, Jul 05, 2010 04:56:11 PM

Output Date: Tue, Jul 06, 2010 11:38:25 AM

Rev II

Project: Management Accounting_Debarshi Bhattacharyya ACE Pro India Pvt. Ltd. File: X:\Pearson\Management Accounting_Debarshi Bhattacharyya\MAIN\M02\LAYOUT_M02\M02_DEBA_ISBN_EN_SE_C02.indd

47

ANALYSIS AND INTERPRETATION OF FINANCIAL STATEMENTS

Particulars

2005 (Rs.) 3,00,000 ?

Capital Employed Reserves & Surplus

2006 (Rs.) ? 40,000

2007 (Rs.) ? 65,000

You are required to prepare a Vertical Trend Financial Statement taking 2005 as the base. [B.Com. (Hons), Mumbai University—October 2008] Solution Vertical Trend Income Statement (Base year = 2005)

Less:

Sales Cost of Goods Sold Gross Profit

2005 Amount Rs. 5,50,000 2,00,000 3,50,000

Trend % 100 100 100

2006 Amount Rs. 7,50,000 4,00,000 3,50,000

Trend % 136.36 200 100

2007 Amount Rs. 7,00,000 3,00,000 4,00,000

Trend % 127.27 150 114.28

Vertical Trend Balance Sheet (Base year = 2005)

Liabilities: Share Capital Reserves & Surplus Net Worth Long-term Loan Capital Employed Current Liabilities Total Liabilities Assets: Fixed Assets Current Assets Total Assets

2005 Amount Trend Rs. %

2006 Amount Trend Rs. %

2007 Amount Trend Rs. %

1,50,000 50,000 2,00,000 1,00,000 3,00,000 40,000 3,40,000

100 100 100 100 100 100 100

1,80,000 40,000 2,20,000 80,000 3,00,000 70,000 3,70,000

120 80 110 80 100 175 108.82

1,90,000 65,000 2,55,000 1,20,000 3,75,000 60,000 4,35,000

127.67 130 127.50 120 125 150 127.94

2,40,000 1,00,000 3,40,000

100 100 100

2,50,000 1,20,000 3,70,000

104.17 120 108.82

2,35,000 2,00,000 4,35,000

97.97 200 127.94

Working Notes 1. Computation of Missing Figures for 2005 Cost of Goods Sold = Sales − Gross Profit = 5,50,000 − 3,50,000 = Rs. 2,00,000 Reserves & Surplus = Net Worth − Share Capital = 2,00,000 − 1,50,000 = Rs. 50,000 Current Assets = Working Capital + Current Liabilities = 60,000 + 40,000 = Rs. 1,00,000 2. Computation of Missing Figures for 2006 Current Liabilities = Current Assets − Working Capital = 1,20,000 − 50,000 = Rs. 70,000 Capital Employed = Fixed Assets + Working Capital = 2,50,000 + 50,000 = Rs. 3,00,000 Again, Capital Employed = Net Worth + Long-term Loan Long-term Loan = Capital Employed − Net Worth = 3,00,000 − 2,20,000 = Rs. 80,000 Current Liabilities = Current Assets − Working Capital = 1,20,000 − 50,000 = Rs. 70,000 3. Computation of Missing Figures for 2007 Sales = Cost of Goods Sold + Gross Profit = 3,00,000 + 4,00,000 = Rs. 7,00,000 Current Liabilities = Current Assets − Working Capital = 2,00,000 − 1,40,000 = Rs. 60,000 Capital Employed = Net Worth + Long-term Loan = 2,55,000 + 1,20,000 = Rs. 3,75,000

Modified Date: Mon, Jul 05, 2010 04:56:11 PM

Output Date: Tue, Jul 06, 2010 11:38:25 AM

Rev II

Project: Management Accounting_Debarshi Bhattacharyya ACE Pro India Pvt. Ltd. File: X:\Pearson\Management Accounting_Debarshi Bhattacharyya\MAIN\M02\LAYOUT_M02\M02_DEBA_ISBN_EN_SE_C02.indd

48

MANAGEMENT ACCOUNTING

CHAPTER REVIEW SUMMARY Financial Statements are a compilation of financial data, arranged and organized in a systematic and summarized manner according to the accounting principles, to assess the financial position of an enterprise as regards its profitability, operational efficiency, long- and short-term solvency and growth potential. A number of statements prepared at the end of every accounting period are collectively called ‘Financial Statements.’  Characteristics of Financial Statements are: (a) they are prepared on the basis of past recorded facts; (b) some accounting principles, concepts, conventions and postulates are applied while preparing the Financial Statements; (c) they provide periodical information; (d) they exhibit a summary of all activities; and (e) they are the prime tool for financial analysis.  Components of Financial Statements are: (a) Income Statement; (b) Balance Sheet; (c) Fund Flow Statement; and (d) Cash Flow Statement.  Relationship between the items in the Balance Sheet and the Income Statement are: (a) Balance Sheet discloses the stock of resources while the Income Statement reveals the flows of resources; (b) Balance Sheet and Income Statement are complementary to each other; and (c) Balance Sheet serves as a link between the Income Statements of successive years.  Financial Statement Analysis is an analysis which critically examines the relationship between various components of Financial Statements with a view to obtain necessary and effective information from them.  Objectives of Financial Statement Analysis are: (a) to assess the real meaning and significance of financial data as disclosed in the Financial Statements; (b) to assess the liquidity, short- and long-term solvency position, profitability and growth potentiality of the enterprise; (c) to assess the operational efficiency of the enterprise; (d) to assess the financial stability of the enterprise; and (e) to assess the overall performance of the enterprise.  Requisites of Financial Statement Analysis are: (a) study of internal and external environment; (b) objective of the analysis; (c) study of accounting procedure; (d) rearrangement of financial data; (e) selection of techniques for analysis; (f) interpretation of findings; and (g) submission of report.  Generally, two approaches are followed for the analysis of Financial Statements. They are: (a) traditional approach to Financial Statement Analysis; and (b) modern approach to Financial Statement Analysis.  Different types of Financial Statement Analysis are: (a) internal and external analysis; (b) short- and long-term analysis; and (c) horizontal/dynamic analysis and vertical/static analysis.  Different techniques of Financial Statement Analysis are: (a) Comparative Financial Statement; (b) Commonsize Statement; (c) Trend Analysis; (d) Ratio Analysis; (e) Cash Flow Analysis; (f) Fund Flow Analysis; (g) Working Capital Analysis; and (h) Cost–Volume–Profit Analysis.  Comparative Financial Statement Analysis is a form of horizontal analysis where the Financial Statements of two or more years, or of two or more different companies, or of a company and its industry, are compared, analysed and interpreted.  To study the relative significance of every item of the Financial Statements, vertical analysis is useful. An important tool for such analysis is ‘Common-size Statement.’  Trend Analysis is a form of comparative analysis of Financial Statements between two or more years. It indicates the trend of individual items of the Financial Statements over a period of time. 

CHAPTER REVIEW QUIZ 1. State whether the following statements are true or false: a. Financial Statement Analysis is a tool which can be used by the management alone. b. The management decides which technique of Financial Statement Analysis fits best in a particular situation. c. Analysis of Financial Statements ignores the issue of price-level changes. d. Financial Statement Analysis should be restricted to Financial Statements alone. e. Horizontal analysis is used to compare the Financial Statements over a number of years. f. Fund Flow Statement is a part of Financial Statements. g. The choice of base year for Trend Analysis depends on the sales volume. h. Comparative Statement does not take into account the changes in the price level. i. Financial Statements include Cash Flow Statement too.

Modified Date: Mon, Jul 05, 2010 04:56:11 PM

Output Date: Tue, Jul 06, 2010 11:38:25 AM

Rev II

Project: Management Accounting_Debarshi Bhattacharyya ACE Pro India Pvt. Ltd. File: X:\Pearson\Management Accounting_Debarshi Bhattacharyya\MAIN\M02\LAYOUT_M02\M02_DEBA_ISBN_EN_SE_C02.indd

ANALYSIS AND INTERPRETATION OF FINANCIAL STATEMENTS

49

j. Common-size Statement considers the time value of money. k. Both comparative and Common-size Statements can show relative importance of each individual item in the total structure. Ans.: True: (c), (e), (f), (h), (i); False: (a), (b), (d), (g), (j), (k). 2. Choose the correct alternative from the following: a. Financial Statements exhibit: (i) anticipated facts; (ii) recorded facts; (iii) estimates of facts. b. Internal analysis of Financial Statements is done by the: (i) management; (ii) bankers; (iii) customers. c. Common-size Statement is a form of: (i) sectional analysis; (ii) horizontal analysis; (iii) vertical analysis. d. Comparative statement is a form of: (i) sectional analysis; (ii) horizontal analysis; (iii) vertical analysis. e. Financial Statement Analysis to examine the Financial Statements of one particular year alone is called: (i) short-term analysis; (ii) horizontal analysis; (iii) vertical analysis. f. Financial Statement Analysis to examine the Financial Statements of a number of years is called: (i) longterm analysis; (ii) horizontal analysis; (iii) vertical analysis. Ans.: (a) (ii); (b) (i); (c) (iii); (d) (ii); (e) (iii); (f) (ii). 3. Fill in the blanks of the following statements: . a. Vertical analysis is also called . b. Horizontal analysis is also called items. c. Traditional approach to Financial Statement Analysis ignores d. Under traditional approach to Financial Statement Analysis, are used as a tool of assessing the financial position of the enterprise. or trend of any particular series of data. e. Trend Analysis indicates the approach to Financial Statement Analysis includes the social desirability factors in the f. corporate Financial Statements. Ans.: (a) static analysis; (b) dynamic; (c) non-financial; (d) financial ratios; (e) increasing; decreasing; (f) Modern. EXERCISE I. Theoretical Questions A. Short Answer Type Questions

1. 2. 3. 4. 5. 6. 7. 8. 9. 10. 11. 12. 13. 14. 15. 16.

What is meant by a Financial Statement? What are the different types of Financial Statements? What is Financial Statement Analysis? What are the different approaches to Financial Statement Analysis? What are the different types of Financial Statement Analysis? What is the vertical analysis of Financial Statements? What is the horizontal analysis of Financial Statements? What is Trend Analysis? What is a comparative Financial Statement? What is a Common-size Financial Statement? What is an asset? What is a liability? What is meant by an Income Statement? What is a Balance Sheet? What is a Fund Flow Statement? What is a Cash Flow Statement?

B. Essay Type Questions

1. 2. 3. 4. 5.

Define Financial Statements. What are the characteristics of Financial Statements? Explain the various components of Financial Statements. Discuss the relationship between Income Statement and Balance Sheet. What do you understand by Financial Statement Analysis? Discuss their objectives and requisites. Discuss the different approaches to Financial Statement Analysis.

Modified Date: Mon, Jul 05, 2010 04:56:11 PM

Output Date: Tue, Jul 06, 2010 11:38:25 AM

Rev II

Project: Management Accounting_Debarshi Bhattacharyya ACE Pro India Pvt. Ltd. File: X:\Pearson\Management Accounting_Debarshi Bhattacharyya\MAIN\M02\LAYOUT_M02\M02_DEBA_ISBN_EN_SE_C02.indd

50

MANAGEMENT ACCOUNTING

6. 7. 8. 9. 10. 11.

Discuss the different types of Financial Statement Analysis. Discuss the different techniques of Financial Statement Analysis. What is Comparative Financial Statement? What are its advantages and disadvantages? What is Common-size Financial Statement? What are its advantages and disadvantages? Compare between Comparative Financial Statement and Common-size Financial Statement. What is Trend Analysis? What are its advantages and disadvantages?

II.

Practical Problems

1. From the following Balance Sheet of Jai Hind Ltd, prepare a Comparative Balance Sheet: 2008 Rs. 50,000 20,000 30,000 1,00,000

Liabilities Share Capital Reserves Debentures

2009 Rs. 80,000 30,000 40,000 1,50,000

Assets Fixed Assets Investment Current Assets

2008 Rs. 70,000 10,000 20,000 1,02,000

2009 Rs. 95,000 20,000 35,000 1,50,000

[B.Com. (Hons), Calcutta University—Adapted] 2. From the following figures of the Balance Sheet of Shikara Ltd, prepare a Comparative Balance Sheet: 31 December 2008 (Rs.) 2,00,000 1,50,000 1,00,000 50,000 60,000 50,000 4,50,000 1,20,000 60,000

Particulars Equity Share Capital Preference Share Capital 10% Debentures Reserves & Surplus Long-term Loan Investment Fixed Assets Current Assets Current Liabilities

31 December 2009 (Rs.) 3,00,000 1,00,000 60,000 90,000 80,000 60,000 5,10,000 1,80,000 1,20,000

[B.Com. (Hons), Calcutta University—Adapted] 3. Following are the Balance Sheets of Raga Ltd as on 31 March 2008 and 31 March 2009: 31 March 2008 Rs. 3,00,000 1,00,000 1,00,000 1,00,000

Liabilities Equity Share Capital Reserves & Surplus Long-term Loan Current Liabilities

6,00,000

31 March 2009 Rs. 5,00,000 1,50,000 2,00,000 1,50,000

Assets Fixed Assets Investment Stock Debtors Cash & Bank

10,00,000

31 March 2008 Rs. 4,00,000 50,000 70,000 50,000 30,000 6,00,000

31 March 2009 Rs. 6,50,000 1,20,000 1,00,000 80,000 50,000 10,00,000

Prepare a Comparative Balance Sheet and interpret the financial position of the company. 4. Balance Sheets of Mumbaikar Ltd for the years that ended on 31 December 2008 and 31 December 2009 are as follows: Liabilities Equity Share Capital 12% Preference Share Capital General Reserve

31 December 2008 Rs. 4,00,000 2,00,000 2,00,000

31 December 2009 Rs. 6,00,000 1,00,000 3,00,000

Assets Land Building Machinery

31 December 2008 Rs. 3,80,000 2,00,000 3,50,000

31 December 2009 Rs. 5,30,000 3,00,000 4,50,000 (Continued)

Modified Date: Mon, Jul 05, 2010 04:56:11 PM

Output Date: Tue, Jul 06, 2010 11:38:25 AM

Rev II

Project: Management Accounting_Debarshi Bhattacharyya ACE Pro India Pvt. Ltd. File: X:\Pearson\Management Accounting_Debarshi Bhattacharyya\MAIN\M02\LAYOUT_M02\M02_DEBA_ISBN_EN_SE_C02.indd

51

ANALYSIS AND INTERPRETATION OF FINANCIAL STATEMENTS

Liabilities 15% Debentures Creditors Bills Payable Tax Payable

31 December 2008 Rs. 1,50,000 1,20,000 40,000 90,000 12,00,000

31 December 2009 Rs. 2,50,000 1,80,000 60,000 1,10,000 16,00,000

Assets Furniture Stock Debtors Cash

31 December 2008 Rs. 1,00,000 90,000 60,000 20,000 12,00,000

31 December 2009 Rs. 70,000 1,10,000 1,00,000 40,000 16,00,000

Prepare a Comparative Balance Sheet in vertical form and offer your comments in brief on Fixed Assets. [B.Com. (Hons), Mumbai University—Adapted] 5. From the following information, prepare a Comparative Balance Sheet of Jakariya Ltd as on 31 March 2008 and 31 March 2009 and interpret the financial position of the company: Liabilities Equity Share Capital 11% Preference Share Capital Statutory Reserve Profit & Loss A/c Revaluation Reserve General Reserve 9% Debentures Sundry Creditors Bills Payable

31 March 2008 Rs. in ’000 320 180 30 40 80 60 50 110 30 900

31 March 2009 Rs. in ’000 400 100 40 90 50 80 75 145 20 1000

Assets Land & Building Plant & Machinery Investment Stock Sundry Debtors Bills Receivable Cash & Bank

31 March 2008 Rs. in ’000 430 220 50 75 65 20 40

900

31 March 2009 Rs. in ’000 520 250 30 90 50 30 30

1000

6. From the following information of Wajood Ltd for the years that ended on 31 March 2008 and 31 March 2009, prepare a Comparative Balance Sheet as on 31 March 2008 and 31 March 2009 and interpret the financial position of the company: Liabilities Equity Share Capital General Reserve Sundry Creditors Bills Payable Outstanding Expenses

31 March 2008 Rs. 2,50,000 70,000 90,000 25,000 15,000

31 March 2009 Rs. 4,00,000 1,20,000 1,40,000 35,000 5,000

4,50,000

7,00,000

Assets Land & Building Plant & Machinery Investment Stock Debtors Bills Receivable Profit & Loss A/c Discount on Issue of Shares

31 March 2008 Rs. 1,20,000 1,00,000 70,000 50,000 30,000 20,000 40,000

31 March 2009 Rs. 2,80,000 2,20,000 50,000 70,000 50,000 10,000 10,000

20,000 4,50,000

10,000 7,00,000

2008 8,00,000 4,80,000 2,40,000

2009 10,00,000 7,00,000 2,00,000

7. From the information given below, prepare a Comparative Income Statement: Sales Cost of Goods Sold Operating Expenses

[B.Com. (Hons), Calcutta University—Adapted]

Modified Date: Mon, Jul 05, 2010 04:56:11 PM

Output Date: Tue, Jul 06, 2010 11:38:25 AM

Rev II

Project: Management Accounting_Debarshi Bhattacharyya ACE Pro India Pvt. Ltd. File: X:\Pearson\Management Accounting_Debarshi Bhattacharyya\MAIN\M02\LAYOUT_M02\M02_DEBA_ISBN_EN_SE_C02.indd

52

MANAGEMENT ACCOUNTING

8. From the following information of Laskar Ltd, prepare a Comparative Income Statement for the years that ended on 31 December 2008 and 31 December 2009:

Less:

2008 Rs. 1,00,000 70,000 30,000 5,000 6,000 19,000 1,000 5,000 13,000

Net Sales Cost of Goods Sold

Less:

Office & Administration Expenses Selling & Distribution Expenses

Less:

Interest Paid Tax Paid

2009 Rs. 1,10,000 75,000 35,000 7,000 5,000 23,000 2,000 6,000 15,000

9. From the following information of Lalbaba Ltd for the years that ended on 31 March 2008 and 31 March 2009, prepare a Comparative Income Statement and comment on the performance of the company. 2008 (Rs.) 6,30,000 30,000 60,000 4,20,000 80,000 60,000 40,000 60,000 20,000 30,000

Gross Sales Sales Return Opening Stock Purchase During the Year Closing Stock Direct Wages Paid Office & Administrative Expenses Selling & Distribution Expenses Interest Paid Tax Paid

2009 (Rs.) 8,40,000 40,000 80,000 5,80,000 70,000 80,000 70,000 1,00,000 40,000 50,000

10. From the following information, prepare a Comparative Income Statement for the years that ended on 31 March 2008 and 31 March 2009:

Trading and Profit & Loss A/c for the years that ended on 31 March 2008 and 31 March 2009

To Opening Stock To Purchases To Carriage Inward To Gross Profit c/d To Salaries of Staff To Depreciation To Insurance To Advertisement To General Expenses To Interest To Provision for Tax To Net Profit b/d

31 March 2008 Rs. 75,000 4,25,000 20,000 1,80,000 7,00,000 30,000 15,000 5,000 25,000 15,000 20,000 30,000 70,000 2,10,000

31 March 2009 Rs. 1,20,000 6,35,000 35,000 3,50,000 11,40,000 45,000 25,000 20,000 55,000 35,000 40,000 50,000 1,30,000 4,00,000

By Sales Less: Sales Return By Closing Stock By Gross Profit b/d By Other Incomes

31 March 2008 Rs. 6,40,000 40,000 6,00,000 1,00,000 7,00,000 1,80,000 30,000

31 March 2009 Rs. 10,50,000 50,000 10,00,000 1,40,000 11,40,000 3,50,000 50,000

2,10,000

4,00,000

11. Following are the Income Statements and Balance Sheets of Trozen Ltd:

Income Statement for the years that ended on 31 December 2008 and 31 December 2009 Particulars Net Sales Opening Stock of Goods

2008 (Rs.) 5,00,000 40,000

2009 (Rs.) 7,50,000 60,000 (Continued)

Modified Date: Mon, Jul 05, 2010 04:56:11 PM

Output Date: Tue, Jul 06, 2010 11:38:25 AM

Rev II

Project: Management Accounting_Debarshi Bhattacharyya ACE Pro India Pvt. Ltd. File: X:\Pearson\Management Accounting_Debarshi Bhattacharyya\MAIN\M02\LAYOUT_M02\M02_DEBA_ISBN_EN_SE_C02.indd

53

ANALYSIS AND INTERPRETATION OF FINANCIAL STATEMENTS

Particulars

2008 (Rs.) 60,000 2,90,000 70,000 30,000 20,000 40,000 20,000 25,000

Closing Stock of Goods Purchases for the Year Direct Wages Factory Expenses Office & Administration Expenses Selling & Distribution Expenses Interest on Debentures Income Tax

2009 (Rs.) 80,000 4,10,000 90,000 50,000 40,000 50,000 30,000 50,000

Balance Sheets as on 31 December 2008 and 31 December 2009 Liabilities Equity Share Capital Preference Share Capital Reserves & Surplus Debentures Creditors Bills Payable

31 December 2008 Rs. 3,00,000 2,00,000 1,50,000 1,50,000 1,50,000 50,000 10,00,000

31 December 2009 Rs. 5,00,000 1,00,000 2,00,000 1,00,000 2,60,000 40,000 12,00,000

Assets Fixed Assets Investment Stock Debtors Cash & Bank Preliminary Expense

31 December 2008 Rs. 4,80,000 1,80,000 90,000 1,80,000 40,000 30,000 10,00,000

31 December 2009 Rs. 5,50,000 2,30,000 1,30,000 2,40,000 30,000 20,000 12,00,000

Prepare the Comparative Income Statement and Balance Sheet of 2 years of the company. 12. From the following information, prepare a Common-size Income Statement: 2008 (Rs.) 4,00,000 2,00,000 40,000 40,000 40%

Sales Cost of Goods Sold Operating Expenses Non-operating Expenses Tax Rate

2009 (Rs.) 6,00,000 3,60,000 90,000 30,000 50%

13. From the following information of a company for the year 2008 and 2009, prepare a Common-size Income Statement: 2008 (Rs.) 2,00,000 1,20,000 40,000 4,000 6,000 50%

Net Sales Cost of Goods Sold Selling, General & Administrative Expenses Interest Paid Interest Received Income Tax

2009 (Rs.) 3,00,000 1,50,000 45,000 9,000 6,000 40%

14. From the following information, make a comparable Profit & Loss A/c of X Ltd and Y Ltd in terms of percentage alone: X Ltd Rs. in lakhs 50 30 2 3 2

Sales Cost of Goods Sold Administrative Expenses Selling Expenses Financial Charge

Y Ltd Rs. in lakhs 100 50 3 5 4

[B.Com. (Hons), Calcutta University—Adapted]

Modified Date: Mon, Jul 05, 2010 04:56:11 PM

Output Date: Tue, Jul 06, 2010 11:38:25 AM

Rev II

Project: Management Accounting_Debarshi Bhattacharyya ACE Pro India Pvt. Ltd. File: X:\Pearson\Management Accounting_Debarshi Bhattacharyya\MAIN\M02\LAYOUT_M02\M02_DEBA_ISBN_EN_SE_C02.indd

54

MANAGEMENT ACCOUNTING

15. Following data are available from Jojila Ltd for 2008 and 2009. You are required to prepare a Common-size Income Statement. Particulars Add: Add:

Sales Commission Received Discount Received Cost of Goods Sold Operating Expenses Non-operating Expenses Profit

2008 Rs. 5,00,000 75,000 1,25,000 7,00,000 3,00,000 1,00,000

2009 Rs. 9,00,000 90,000 1,35,000 11,25,000 4,50,000 2,25,000

25,000 2,75,000 7,00,000

45,000 4,05,000 11,25,000

[B.Com. (Hons), Calcutta University—Adapted] 16. With the help of the following information for the year that ended in 2006, prepare a Common-size Income Statement: Selling and Distribution Expense Administration Expense Cost of Sales Income Tax Net Profit after Tax Other Income

Rs. 20,000 Rs. 10,000 60% of Net Sales 40% of Net Profit before Tax Rs. 60,000 Rs. 10,000

[B.Com. (Hons), Calcutta University—Adapted] Ans.: Sales—Rs. 1,25,000; Cost of sales—Rs. 75,000; Gross Profit—Rs. 50,000; Net Profit before tax—Rs. 1,00,000. 17. From the following information, prepare a Common-size Balance Sheet for 2008 and 2009: 2008 (Rs.) 2,00,000 50,000 1,50,000 4,00,000 2,00,000 60,000 30,000 50,000 20,000 40,000 4,00,000

Fixed Assets Investment Current Assets Total Equity Share Capital 8% Preference Share Capital Reserves & Surplus Secured Loan Unsecured Loan Current Liabilities & Provisions Total

2009 (Rs.) 3,50,000 80,000 1,70,000 6,00,000 3,00,000 1,00,000 50,000 60,000 30,000 60,000 6,00,000

18. From the following information as furnished by Heera Ltd and Panna Ltd as on 31 March 2009, prepare a Common-size Balance Sheet as on that date and make your comment on the position of the companies:

Less:

Heera Ltd Rs. 4,00,000 1,60,000 2,40,000 40,000 50,000 30,000 10,000

Fixed Assets (at Cost) Provision for Depreciation Investment Sundry Debtors Stock Prepaid Expenses

Panna Ltd Rs. 6,00,000 2,40,000 3,60,000 70,000 80,000 50,000 20,000 (Continued)

Modified Date: Mon, Jul 05, 2010 04:56:11 PM

Output Date: Tue, Jul 06, 2010 11:38:25 AM

Rev II

Project: Management Accounting_Debarshi Bhattacharyya ACE Pro India Pvt. Ltd. File: X:\Pearson\Management Accounting_Debarshi Bhattacharyya\MAIN\M02\LAYOUT_M02\M02_DEBA_ISBN_EN_SE_C02.indd

55

ANALYSIS AND INTERPRETATION OF FINANCIAL STATEMENTS

Heera Ltd Rs. 30,000 4,00,000 1,80,000 50,000 20,000 50,000 70,000 30,000 4,00,000

Cash & Bank Share Capital General Reserve Capital Reserve Long-term Loan Sundry Creditors Provision for Tax

Panna Ltd Rs. 40,000 6,20,000 2,50,000 80,000 40,000 90,000 90,000 70,000 6,20,000

19. From the following information as obtained from the books of Chomoski Ltd, prepare a Common-size Income Statement and a Common-size Balance Sheet of 2 years: 2008 Rs. 3,00,000 1,50,000 60,000 40,000 20,000 50% 1,80,000 70,000 1,30,000 20,000 1,20,000 80,000 70,000 50,000 20,000 60,000

Particulars Sales Cost of Goods Sold Operating Expenses Non-operating Expenses Non-operating Incomes Tax Rate Fixed Assets Investments Current Assets Miscellaneous Expenditure Equity Share Capital Preference Share Capital Reserves & Surplus Secured Loan Unsecured Loan Current Liabilities & Provisions

2009 Rs. 8,00,000 4,40,000 2,00,000 50,000 30,000 50% 4,20,000 1,50,000 1,10,000 20,000 3,50,000 50,000 1,30,000 80,000 40,000 50,000

20. From the following figures, calculate the trend percentages taking 2004 as the base year: Year 2004 2005 2006 2007 2008 2009

Sales 4,00,000 4,50,000 5,20,000 6,00,000 6,60,000 7,00,000

Cost of Goods Sold 2,40,000 3,15,000 3,90,000 4,80,000 3,96,000 4,90,000

Inventory 1,00,000 1,20,000 1,50,000 1,60,000 1,80,000 2,10,000

21. From the following data as obtained from the Financial Statements of Mount Abu Ltd, calculate the trend percentage taking 2007 as the base year: Sales Purchases Expenses Profit

Modified Date: Mon, Jul 05, 2010 04:56:11 PM

2007 (Rs.) 2,50,000 1,50,000 50,000 50,000

2008 (Rs.) 3,00,000 2,10,000 45,000 45,000

Output Date: Tue, Jul 06, 2010 11:38:25 AM

2009 (Rs.) 4,00,000 2,00,000 1,00,000 1,00,000

Rev II

Project: Management Accounting_Debarshi Bhattacharyya ACE Pro India Pvt. Ltd. File: X:\Pearson\Management Accounting_Debarshi Bhattacharyya\MAIN\M02\LAYOUT_M02\M02_DEBA_ISBN_EN_SE_C02.indd

56

MANAGEMENT ACCOUNTING

22. From the following supplied trend percentage, prepare a comparative statement of Current Assets in absolute value, taking 2006 as the base year: 2007 120 130 160 175 110

Trend Percentage 2008 130 140 220 250 150

Corresponding Value of Current Assets 2009 (Rs.) 15,000 – Cash & Bank 40,000 – Debtors 20,000 – Finished Goods 7,500 – Work in Progress 7,000 – Raw Materials

2009 150 200 250 300 175

[B.Com. (Hons), Calcutta University—Adapted] Ans.: Value in 2006: Cash & Bank—Rs. 10,000; Debtors – Rs. 20,000; Finished goods—Rs. 8,000; WIP—Rs. 2,500; and Raw materials—Rs. 4,000. 23. Lehman Brothers’ Income Statement for the last 4 years that were ending on 31 March of 2005, 2006, 2007 and 2008 shows the following position. Prepare a trend revenue statement in vertical form (taking 31 December 2005 as the base year) which is suitable for analysis and comment on the administrative expenses and nonoperating income.

Year ending on 31 March 2005 (Rupees in crores) Particulars Sales Materials Consumed Wages Gross Profit Office Rent Office Salary Other Administrative Expenses Sales Promotion Expenses Interest on Debentures Dividend Received

2005 70 21 7 ? 7 14 17.50 3.50 10 0.50

2006 60 18 6 ? 7 14 15 3 10 0.40

2007 50 15 2 ? 7 14 12.50 2.50 10 0.30

2008 30 9 1 ? 7 3.50 7.50 1.50 10 0.20

[B.Com. (Hons), Mumbai University—April 2009] Ans.: Gross Profit (Rs. in crore): 2005 – 42; 2006 – 36; 2007 – 33; 2008 – 20. 24. The following is the financial information of Vision Ltd for 3 years that ended on 31 December every year: Particulars Share Capital Gross Profit Current Liabilities Fixed Assets Long-term Loan Cost of Goods Sold Working Capital Net Worth Current Assets Sales Capital Employed Reserves & Surplus

2007 (Rs.) 3,00,000 1,50,000 1,20,000 4,00,000 80,000 ? 80,000 4,00,000 ? 4,00,000 4,80,000 ?

2008 (Rs.) 3,30,000 2,00,000 ? 4,60,000 ? 3,00,000 1,00,000 4,40,000 2,40,000 5,00,000 ? 1,10,000

2009 (Rs.) 3,80,000 2,20,000 ? 5,00,000 1,50,000 3,80,000 1,50,000 5,00,000 3,00,000 ? ? 1,20,000

You are required to prepare a vertical-trend Financial Statement taking 2007 as the base year. [B.Com. (Hons), Mumbai University—Adapted] Ans.: In 2007: Cost of Goods Sold—Rs. 2,50,000; Current Assets—Rs. 2,00,000; Reserves & Surplus—Rs. 1,00,000. In 2008: Current Liabilities—Rs. 1,40,000; Long-term loan—Rs. 1,20,000; Capital employed—Rs. 5,60,000. In 2009: Sales—Rs. 6,00,000; Current Liabilities—Rs. 1,50,000; Capital employed—Rs. 6,50,000.

Modified Date: Mon, Jul 05, 2010 04:56:11 PM

Output Date: Tue, Jul 06, 2010 11:38:25 AM

Rev II

Project: Management Accounting_Debarshi Bhattacharyya ACE Pro India Pvt. Ltd. File: X:\Pearson\Management Accounting_Debarshi Bhattacharyya\MAIN\M03\LAYOUT_M03\M03_DEBA_ISBN_EN_SE_C03_Part-1.indd

Accounting Ratios for Financial Statement Analysis

3

LEARNING OBJECTIVES On completion of the study of the chapter, you should be able to understand: What is Accounting Ratio? What is meant by Ratio Analysis? Importance, uses, advantages and limitations of Ratio Analysis. Structural Analysis of Financial Statements for the purpose of computation of different Ratios. Classification of Ratios on the basis of their sources. Computation of different Ratios from Financial Statements. Uses of various Ratios to analyse liquidity, solvency, profitability, Capital Structure, activity and the overall performance of an enterprise. How Ratios are applied to interpret Financial Statements of an enterprise. Du Pont Analysis of Financial Statements.

3.1 FINANCIAL STATEMENTS Financial Statements are compilation of financial data, arranged and organized in a systematic and summarized manner according to the accounting principles, to assess financial position of an enterprise as regards to its profitability, operational efficiency, long- and short-term solvency, and growth potential. On the basis of the information as disclosed in the Financial Statements, users of them come to know about the growth, profitability, solvency and the financial strength of an enterprise. It is a tool in the hands of the management of an enterprise by which they present the financial position of the enterprise before the interested parties such as shareholders, lenders, creditors and so on. A number of statements prepared at the end of every Accounting Period are collectively called Financial Statements. Financial Statements include the following: i. Income Statement or Profit & Loss A/c. ii. Balance Sheet. iii. Fund Flow Statement. iv. Cash Flow Statement. 3.2 WHAT IS A RATIO? A Ratio is a relationship between two or more items expressed in mathematical terms. It is an expression of quantitative relationship between two amounts or items which is expressed in numbers or percentage. Say, there are 50 computers in an office where 200 employees are working. For identifying a quantitative relationship between the number of employees working and the number of computers used in the office, we should calculate Employee—Computer Ratio, which is 200 : 50 = 4 : 1. Therefore, a ‘ratio’ is a quantitative expression which exhibits a comparative relationship of one item with the other.

Modified Date: Tue, Jul 06, 2010 11:23:17 AM

Output Date: Tue, Jul 06, 2010 11:24:06 AM

Rev II

Project: Management Accounting_Debarshi Bhattacharyya ACE Pro India Pvt. Ltd. File: X:\Pearson\Management Accounting_Debarshi Bhattacharyya\MAIN\M03\LAYOUT_M03\M03_DEBA_ISBN_EN_SE_C03_Part-1.indd

58

MANAGEMENT ACCOUNTING

3.3 WHAT IS AN ACCOUNTING RATIO? A Financial Ratio is a relationship between two or more Accounting Data expressed in mathematical terms. Therefore, Ratio calculated from different Accounting Data for exhibiting a meaningful and useful relationship between them is called ‘Accounting Ratio.’ Accounting information is obtained from the process of Financial Accounting, that is, from the Profit & Loss A/c and the Balance Sheet. Taking relevant data from these Financial Statements, Accounting Ratios are calculated and expressed either in terms of Ratio (e.g., 3:2) or in terms of number (e.g., 5 or 5 times) or in terms of percentage (e.g., 20%). Therefore, Accounting Ratio is a quantitative expression of various accounting information and exhibits a meaningful and useful relationship between them. 3.4 WHAT IS RATIO ANALYSIS? A single Accounting Data by itself may not communicate any meaningful and useful information, but when expressed in relation to some other data, it definitely conveys some significant information to the interested parties. Accordingly, Accounting Ratio is computed to obtain a relationship between two or more Accounting Data expressed in mathematical terms. The process of comparative analysis and interpretation of all these Accounting Ratios are called Ratio Analysis. Ratio Analysis a tool adopted for determining the quantitative relationship between two or more financial data as obtained from the Financial Statements. Analysing various Accounting Ratios in a systematic and meaningful manner, financial strength, weakness and growth potentiality of an enterprise can be identified. As every Accounting Ratio possesses its distinctive feature, some relevant Ratios of the same functional area are calculated and analysed for getting a conclusive idea about that functional area. Therefore, Ratio Analysis is a tool of Financial Statement Analysis for identifying financial strength, weakness and growth potentiality of an enterprise by means of determination, analysis and interpretation of relevant various Accounting Ratios. 3.5 STEPS IN RATIO ANALYSIS The Ratio Analysis requires three steps. They are: i. Selection of relevant various Accounting Data from the Financial Statements. ii. Computation of related Accounting Ratios using those selected Accounting Data. iii. Analysis and interpretation of those Accounting Ratios in a very significant, logical and useful manner with an objective to obtain some conclusive financial information of the enterprise. In the phase of Ratio Analysis, Computed Ratios are generally compared with some predetermined standards to make any comment as regards to the financial position of the enterprise. 3.6 ROLE OR IMPORTANCE OF RATIO ANALYSIS As Ratio Analysis is a very powerful tool of analysing Financial Statements, it plays a vital role to the facts of determining the financial strength and weakness of an enterprise. Its importance lies in the fact that it presents various information as obtained from the Financial Statements in a comparative way, and helps in analysing the performance of the enterprise. Ratio Analysis, as an analytical tool of Financial Statement Analysis, plays the following role: i. It measures and evaluates the financial condition of an enterprise. ii. It measures and evaluates liquidity, solvency, profitability, managerial efficiency, Capital Structure and activity of an enterprise. iii. It measures and evaluates the operating effectiveness of an enterprise. iv. It provides adequate information as regards to the direction of financial changes of an enterprise. v. It identifies the functional areas within the business where adoption of remedial measures are needed. vi. It helps the management in the course of Decision Making process. vii. It helps in predicting the future condition of the business. viii. It is essential for coordinating the various functional machineries of an enterprise. ix. It serves as integral part of Budgetary Control System.

Modified Date: Tue, Jul 06, 2010 11:23:17 AM

Output Date: Tue, Jul 06, 2010 11:24:06 AM

Rev II

Project: Management Accounting_Debarshi Bhattacharyya ACE Pro India Pvt. Ltd. File: X:\Pearson\Management Accounting_Debarshi Bhattacharyya\MAIN\M03\LAYOUT_M03\M03_DEBA_ISBN_EN_SE_C03_Part-1.indd

ACCOUNTING RATIOS FOR FINANCIAL STATEMENT ANALYSIS

59

x. It works as the most effective technique for interpretation of Financial Statement Analysis. xi. It serves as a management control system. 3.7 USES OF RATIO ANALYSIS As Ratio Analysis is a means of communicating relevant information to different parties, who are interested, in and around the enterprise, it is very much useful to for improvement of their needs according to their respective interest in the enterprise. Various uses of Ratio Analysis are given as follows: i. It is useful for identification of financial strengths and weaknesses of an enterprise. ii. It is useful to measure liquidity, solvency, profitability, managerial efficiency and activity of an enterprise. iii. It is useful for inter- and intra-firm comparison of performance. iv. It is useful to measure the proper utilization of the various assets of the business. v. It is useful to measure the operating efficiency of the business. vi. It is useful to assess the level of efficiency of the management. vii. It is useful for a reasonable prediction of the future plans of the business. viii. It is useful to optimize the Capital Structure of the business. ix. It helps the management for reviewing the past years’ activity of the business by means of Trend Analysis. x. It is useful to the management for a perfect Budget Preparation. xi. It is useful to determine the corporate sickness. 3.8 ADVANTAGES OF RATIO ANALYSIS Following are the advantages of Ratio Analysis: i. It is treated as the most powerful tool for measuring short-and long-term solvency of a concern. ii. It is also treated as an essential tool for measuring the profitability and managerial efficiency of a concern. iii. It is considered as the most important tool for measuring the operating activity of a concern. iv. It is treated as one of the most important tool for the Capital Structure Analysis of a concern. v. It summarizes the large number of quantitative Accounting Data by calculating the different Ratios. vi. It relates the present accounting information with the past. vii. It helps the management to prepare the necessary Budget and to formulate the future policies. viii. An effective inter- and intra-firm comparison of Accounting Data can be done with the help of Ratio Analysis. ix. It helps the management to analyse the probable relationship between different accounting items through proper scrutiny and also to analyse the past performances. x. It is considered essential for coordinating the various functional machineries of an enterprise. xi. It is very much useful for estimating the Working Capital requirement and also for maintaining a reasonable balance between the purchases and sales of a business. xii. It provides the necessary information to all the interested parties in and around the concern. xiii. It provides necessary information to the management for future Decision Making. xiv. It indicates the efficiency level as regards to the proper utilization of assets of the business. 3.9 LIMITATIONS OF RATIO ANALYSIS Following are the drawbacks or limitations of Ratio Analysis: i. It depends on the data supplied from the process of Financial Accounting, and if those data are incorrect, information obtained from the Ratio Analysis cannot be relied upon. ii. Ratio Analysis may tend to interpret the wrong direction if it is based on unauthenticated data.

Modified Date: Tue, Jul 06, 2010 11:23:17 AM

Output Date: Tue, Jul 06, 2010 11:24:06 AM

Rev II

Project: Management Accounting_Debarshi Bhattacharyya ACE Pro India Pvt. Ltd. File: X:\Pearson\Management Accounting_Debarshi Bhattacharyya\MAIN\M03\LAYOUT_M03\M03_DEBA_ISBN_EN_SE_C03_Part-1.indd

60

MANAGEMENT ACCOUNTING

iii. As the Ratios are calculated on the basis of the past results, a proper prediction for future may not always be dependable. iv. A Single Ratio calculated for any functional area of a business does not convey its conclusive state. A series of Ratios is needed to get any conclusive idea about that area of the business. v. It is formulated on the basis of historical figures as obtained from the Financial Statements. Performance of a concern based on the historical data does not always indicate its present real condition. vi. The Price Level changes very often distort the Trend Analysis process, which is done with the help of various Ratios calculated in the process of Ratio Analysis. vii. Generally, different firms follow different accounting principles and policies. Under such a circumstance, inter-firm comparison with the help of Ratio Analysis process does not exhibit a real picture at all. Inter-firm comparison with the help of Ratio Analysis process only becomes effective when the concerned firms follow the uniform accounting principles and policies. viii. Intra-firm comparison through the process of Ratio Analysis may also be proved worthless if the uniform accounting principles and policies are not observed in different units of the same concern. ix. Some special events may not be disclosed through the process of Ratio Analysis, for example, debentures might be maturing very shortly for repayment, which cannot be identified from the Ratios. x. The Ratios should be studied in the proper context. To have a practical experience and thorough knowledge about the particular industry are very much essential for an effective Ratio Analysis. Otherwise, it may prove to be absolutely worthless. xi. Ratio Analysis may be a very useful tool in the hands of an expert analyst, but is not so in the hands of common persons. 3.10 PARTIES INTERESTED IN RATIO ANALYSIS Following parties are generally interested in Ratio Analysis, in and around a concern, for different reasons as follows: i. Shareholders: Interested in the rate of return on capital, that is, Gross Profit (GP) Ratio, Net Profit Ratio and Earnings per Share (EPS), which are again sub-divided as: (i) Primary EPS; (ii) Diluted EPS; (iii) Dividends per Share (DPS); etc. ii. Debentureholders: Interested in the Profit Earning Capacity and solvency of the business. iii. Creditors: Interested in the interest cover, solvency and liquidity of the business. iv. Bankers: Interested in solvency, liquidity and interest cover, i.e., in Current Ratio, Liquid Ratio, Interest Coverage Ratio, etc. v. Management: Interested in profitability, efficiency, production, capacity utilization, etc.—i.e., in Stock Turnover Ratio, Turnover to Capital Employed, Turnover to Fixed Assets, etc. vi. Government: Interested in profitability, capacity utilization, solvency, etc. 3.11 DIFFERENT KINDS OF RATIOS OR CLASSIFICATION OF RATIOS Accounting Ratios may be broadly classified into the following categories: i. Balance Sheet Ratios: Ratios calculated from the different items as appearing in the Balance Sheet of a concern are called Balance Sheet Ratios, e.g., Current Ratio, Liquid Ratio, Proprietary Ratio, Capital Gearing Ratio, Debt-Equity Ratio, etc. ii. Revenue Statement Ratios: Ratios calculated from the different items as appearing in the Profit & Loss A/c of a concern are called Revenue Statement Ratios, e.g., GP Ratio, Net Profit Ratio, Operating Ratio, Stock Turnover Ratio, etc. iii. Mixed or Composite Ratios: Ratios calculated taking some items as appearing in the Balance Sheet and taking some items as appearing in the Profit & Loss A/c are called Mixed or Composite Ratios, e.g., Return on Net Worth, Return on Investment (ROI), Capital Turnover Ratio, Debtors’/Creditors’ Turnover/Velocity Ratio, Fixed Asset Turnover Ratio, Turnover of Total Assets, etc.

Modified Date: Tue, Jul 06, 2010 11:23:17 AM

Output Date: Tue, Jul 06, 2010 11:24:06 AM

Rev II

Project: Management Accounting_Debarshi Bhattacharyya ACE Pro India Pvt. Ltd. File: X:\Pearson\Management Accounting_Debarshi Bhattacharyya\MAIN\M03\LAYOUT_M03\M03_DEBA_ISBN_EN_SE_C03_Part-1.indd

61

ACCOUNTING RATIOS FOR FINANCIAL STATEMENT ANALYSIS

3.12 STANDARD OR IDEAL RATIO For the purpose of Ratio Analysis, it is essential to compare the actual Ratios as computed with some predetermined Standard or Ideal Ratios, any deviation from which is identified as the positive or negative feature of the business, as the case may be. Therefore, a Standard or Ideal Ratio is a predetermined calculated Ratio, which is considered to be normal or standard, or which is the most meaningful or significant, or which is the most effectively applicable universally to different industries under different times. For identifying the strength or weakness of the business, the actual Ratios are compared with these standard or Ideal Ratios. Hence, the Standard Ratios are the yardstick for the determination of strength or weakness of a firm, and the significance of an Accounting Ratio is highly increased when it is compared with the predetermined relevant standard Ratio. It should be noted that calculation of such Standard/Ideal Ratio is really a very difficult task due to a number of external and internal factors. Tutorial Note Ratios are computed taking various data from the Financial Statements. For this purpose, Financial Statements mean the Balance Sheet and the Profit & Loss A/c (i.e., Revenue Statement). Ratios calculated taking different data from the Balance Sheet are called ‘Balance Sheet Ratios,’ Ratios calculated taking different data from the Profit & Loss A/c are called Revenue Statement Ratios and Ratios calculated taking some data from the Profit & Loss A/c and the remaining data from the Balance Sheet are called Mixed or Composite Ratios. For an easy understanding of the topic, students should have a crystal-clear idea about the composition of the Balance Sheet. Different forms of a company Balance Sheet, its composition and its miniature presentation are discussed in the following under Para 3.13. Students are advised to follow this part of the topic very carefully to gather a conceptual idea about Ratio Analysis.

3.13 COMPUTATION OF PROPRIETORS’ FUND, CAPITAL EMPLOYED, WORKING CAPITAL Ratios are calculated taking different Accounting Data from the Financial Statements. For a better understanding of Accounting Ratios, it is very much essential to have a clear concept as regards to the Balance Sheet of a company. The proforma of a company Balance Sheet as given in the Schedule VI of the Companies Act is shown as follows: Balance Sheet of a company Amount Rs.

Liabilities Issued & Subscribed Share Capital:

      

Equity Share Capital Preference Share Capital Reserves & Surplus Securities Premium Capital Reserve General Reserve Profit & Loss A/c Secured Loan Debentures Mortgage Loan Unsecured Loan Loan from Bank [without Security]

  

Assets Fixed Assets Goodwill Land & Building Plant & Machinery Furniture & Fixture Investments Investment in Government Securities Investment in Shares of Other Companies Current Assets, Loans & Advances Stock Sundry Debtors Cash at Bank Cash in Hand

Amount Rs.    

 

    (Continued)

Modified Date: Tue, Jul 06, 2010 11:23:17 AM

Output Date: Tue, Jul 06, 2010 11:24:06 AM

Rev II

Project: Management Accounting_Debarshi Bhattacharyya ACE Pro India Pvt. Ltd. File: X:\Pearson\Management Accounting_Debarshi Bhattacharyya\MAIN\M03\LAYOUT_M03\M03_DEBA_ISBN_EN_SE_C03_Part-1.indd

62

MANAGEMENT ACCOUNTING

Amount Rs.

Liabilities Current Liabilities & Provisions Bills Receivable Sundry Creditors Bills Payable Provision for Taxation

   

Amount Rs.

Assets

Miscellaneous Expenditure Preliminary Expenses Discount on Issue of Shares Profit & Loss A/c

   



While analysing the Accounting Ratios, the investments are not separately identified. If it is of a long termnature, then it can be classified under Fixed Assets. On the other hand, if it is of a short-term nature, then it can be classified under Current Assets. Accordingly, investments are shown either as a part of Fixed Assets or as a part of Current Assets, while ascertaining and analysing the Accounting Ratio, depending upon their nature. As a consequence of the above consideration, the portrait of a company Balance Sheet is slightly changed as compared to the proforma as given under the Schedule VI of the Companies Act. For the purpose of ascertaining and better analysing the Accounting Ratio, the changed portrait of the company Balance Sheet is shown as follows: Balance Sheet of a company Amount Rs.

Liabilities Share Capital: Equity Share Capital Preference Share Capital Reserves & Surplus Secured Loan Unsecured Loan Current Liabilities

Amount Rs. 

Assets Fixed Assets

      

Current Assets



Miscellaneous Expenditure Profit & Loss A/c

  

Now, for better understanding of analysis of Accounting Ratio, another form of company Balance Sheet is depicted below, interchanging the sides of some elements of the above Balance Sheet: Amount

Liabilities Add: Less:

Add:

Rs.

Equity Share Capital Reserves & Surplus Miscellaneous Expenditure Profit & Loss A/c [Dr Balance] Equity Shareholders’ Fund Preference-Share Capital Proprietors’ Fund / Net worth Long-term Loans: Secured Loans Unsecured Loan Total Capital Employed

 

 

Assets

Rs.    Less:

Fixed Assets Working Capital: Current Assets Current Liabilities

   

 

Amount Rs.

Amount Rs. 

  

Total Capital Employed



Analysing the above form of Balance Sheet, we find some important information as written in the following: i. Equity Shareholders’ Fund = Equity Share Capital + Reserves & Surplus – Miscellaneous Expenditure – Profit & Loss A/c (Debit Balance). ii. Proprietors’ Fund or Net worth or Shareholders’ Fund or Owners’ Fund

Modified Date: Tue, Jul 06, 2010 11:23:17 AM

Output Date: Tue, Jul 06, 2010 11:24:06 AM

Rev II

Project: Management Accounting_Debarshi Bhattacharyya ACE Pro India Pvt. Ltd. File: X:\Pearson\Management Accounting_Debarshi Bhattacharyya\MAIN\M03\LAYOUT_M03\M03_DEBA_ISBN_EN_SE_C03_Part-1.indd

63

ACCOUNTING RATIOS FOR FINANCIAL STATEMENT ANALYSIS

iii. iv. v. vi. vii. viii.

= Equity Shareholders’ Fund + Preference Share Capital = [Equity Share Capital + Reserves & Surplus – Miscellaneous Expenditure – Profit & Loss A/c (Dr Balance)] + Preference Share Capital. = (Equity Share Capital + Preference Share Capital) + Reserves & Surplus – Miscellaneous Expenditure – Profit & Loss A/c = (Total) Share Capital + Reserves & Surplus – Miscellaneous Expenditure – Profit & Loss A/c (Dr) Total Long-term Loan or Debt Capital or Loan Capital = Secured Loan + Unsecured Loan. Total Capital Employed = Proprietors’ Fund or Net worth + Total long-term Loan or Debt Capital As in a Balance Sheet, the Total of Asset side = Total of Liability side Total Capital Employed = Fixed Assets (FAs) + Working Capital (WC) Working Capital (WC) = Current Assets (CAs) – Current Liabilities (CLs) Total Capital Employed = Proprietors’ Fund + Long-term loan = Fixed Assets + Working Capital Gist of a company Balance Sheet (i.e., Ultimate Balance Sheet Equation): Proprietors’ Fund + Long-term Loan = Fixed Asset + Working Capital

Stop and Think Ultimate Balance Sheet equation (i.e. Net worth + Long-term Loan = Fixed Assets + Working Capital) as shown earlier is very much effective for interpretation of various Balance Sheet Ratios and also for preparation of Balance Sheet from the given ratios.

Illustration Following is the Balance Sheet of a company as on 31 March 2009: Liabilities Equity Share Capital Preference Share Capital Reserves & Surplus 11% Mortgage Loan 10% Debentures Sundry Creditors Bank Overdraft Prereceived Incomes Outstanding Expenses Bills Payable

Rs. 4,00,000 2,00,000 2,50,000 1,00,000 2,00,000 80,000 50,000 15,000 25,000 30,000

Assets Goodwill Land & Building Plant & Machinery Furniture & Fittings Investment [Long Term] Stock in Trade Sundry Debtors Cash at Bank Cash in Hand Bills Receivable Accrued Incomes Prepaid Expenses Discount on Issue of Shares Preliminary Expenses

Rs. 1,00,000 3,50,000 2,50,000 2,00,000 1,00,000 70,000 80,000 60,000 30,000 35,000 15,000 10,000 30,000 20,000 13,50,000

13,50,000

Calculate different information from the above Balance Sheet as required for Ratio Analysis and also summarize the Balance Sheet. Solution From the above Balance Sheet, following relevant information can be extracted: 1 . Proprietors’ Fund Rs. Equity Share Capital Preference Share Capital Total Share Capital

Rs. 4,00,000 2,00,000 6,00,000 (Continued)

Modified Date: Tue, Jul 06, 2010 11:23:17 AM

Output Date: Tue, Jul 06, 2010 11:24:06 AM

Rev II

Project: Management Accounting_Debarshi Bhattacharyya ACE Pro India Pvt. Ltd. File: X:\Pearson\Management Accounting_Debarshi Bhattacharyya\MAIN\M03\LAYOUT_M03\M03_DEBA_ISBN_EN_SE_C03_Part-1.indd

64

MANAGEMENT ACCOUNTING

Rs. Add:

Reserves & Surplus

Less:

Miscellaneous Expenditure: Discount on Issue of Shares Preliminary Expenses

30,000 20,000

Rs. 2,50,000 8,50,000

Proprietors’ Fund

50,000 8,00,000

Total Long-term Loan

Rs. 1,00,000 2,00,000 3,00,000

2. Long-term Loan 11% Mortgage Loan 10% Debentures

3. Fixed Assets Rs. 1,00,000 3,50,000 2,50,000 2,00,000 1,00,000 10,00,000

Goodwill Land & Building Plant & Machinery Furniture & Fittings Investment [Long Term] Total Fixed Assets

4. Working Capital Rs. Total Current Assets: Stock in Trade Sundry Debtors Cash at Bank Cash in Hand Bills Receivable Accrued Incomes Prepaid Expenses Less:

Rs. 70,000 80,000 60,000 30,000 35,000 15,000 10,000 3,00,000

Total Current Liabilities: Sundry Creditors Bank Overdraft Prereceived Incomes Outstanding Expenses Bills Payable

80,000 50,000 15,000 25,000 30,000 Working Capital

2,00,000 1,00,000

Now, the given Balance Sheet can be summarized as follows: Balance Sheet as on 31 March 2009 Liabilities Proprietors’ Fund Long-term Loan Total Capital Employed

Modified Date: Tue, Jul 06, 2010 11:23:17 AM

Rs. 8,00,000 3,00,000 11,00,000

Assets Fixed Assets Working Capital Total Capital Employed

Output Date: Tue, Jul 06, 2010 11:24:06 AM

Rs. 10,00,000 1,00,000 11,00,000

Rev II

Project: Management Accounting_Debarshi Bhattacharyya ACE Pro India Pvt. Ltd. File: X:\Pearson\Management Accounting_Debarshi Bhattacharyya\MAIN\M03\LAYOUT_M03\M03_DEBA_ISBN_EN_SE_C03_Part-1.indd

ACCOUNTING RATIOS FOR FINANCIAL STATEMENT ANALYSIS

65

3.14 CALCULATION AND INTERPRETATION OF VARIOUS RATIOS 3.14.1 Balance Sheet Ratios (Ratios as ascertained from various items of the Balance Sheet) 1. Current Ratio Current Assets (CA) Current Ratio = Current Liabilities (CL) Notes: i. This Ratio indicates whether an enterprise possesses sufficient Current Assets to pay off its Current Liabilities. This Ratio is an indicator of short-term solvency or liquidity position of an enterprise. ii. Conventional standard or Ideal Ratio is 2 : 1, i.e., the enterprise should have twice the Current Assets than the Current Liabilities, to exhibit ideal short-term solvency position. iii. As CAs – CLs = WC. This Ratio is also called Working Capital Ratio and it indicates the quantum of the Working Capital (WC) of an enterprise. iv. Current Assets = Stock + Debtors + Cash at Bank + Cash in Hand + Prepaid Expenses + Accrued incomes, etc v. Current Liabilities = Creditors + Bills Payable + Outstanding Expenses + Prereceived Incomes + Bank Overdraft, etc 2. Liquid/Quick/Acid Test Ratio Liquid/Quick/Acid Test Ratio = =

Liquid/Quick Assets Liquid/Quick Liabilities Current Asset–Stock Current Liabilities–Bank Overdraft

Notes: i. This Ratio indicates whether an enterprise possesses sufficient liquid/quick assets to pay off its liquid/quick liabilities. This Ratio is an indicator of the liquidity position of an enterprise. ii. Liquid or Quick asset means the asset which is receivable within a financial year during which it comes into existence and more easily convertible into cash. ∴ Liquid/Quick Assets = Current Assets − Stock iii. Liquid or Quick Liability means the Liability which is payable within a financial year and more quickly payable in cash. ∴ Liquid/Quick Liabilities = Current Liabilities − Bank Overdraft iv. Conventional standard or Ideal Ratio is at least 1:1, i.e., an enterprise should have at least an equal amount of liquid/quick assets to pay off its liquid/quick liabilities. 3. Proprietary Ratio or Net Worth to Total Assets Proprietary Ratio =

Proprietors’ Fund Total Assets

Modified Date: Tue, Jul 06, 2010 11:23:17 AM

Output Date: Tue, Jul 06, 2010 11:24:06 AM

Rev II

Project: Management Accounting_Debarshi Bhattacharyya ACE Pro India Pvt. Ltd. File: X:\Pearson\Management Accounting_Debarshi Bhattacharyya\MAIN\M03\LAYOUT_M03\M03_DEBA_ISBN_EN_SE_C03_Part-1.indd

66

MANAGEMENT ACCOUNTING

Notes: i. The Standard or Ideal Ratio may be considered in between 0.75. and 1. ii. Proprietors’ Fund = Net Worth = Share Capital + Reserves & Surplus – Miscellaneous Expenditure – Profit & Loss A/c (Dr) iii. Total Assets = Fixed Assets + Current Assets iv. This Ratio indicates the portion of Total Assets acquired out of Shareholders’ Fund. This is an indicator of the efficiency of the management regarding formulation of financial planning. 4. Fixed Asset Proprietorship Ratio or FAs to Net Worth Fixed Asset Proprietorship Ratio =

Fixed Assets Proprietors’ Fund

Notes: i. This Ratio indicates the portion of Proprietors’ Fund or Shareholders’ Fund invested in Fixed Assets. It is also an indicator of the efficiency of the management regarding the formulation of the financial planning. ii. Proprietors’ Fund = Net Worth = Share Capital + Reserves & Surplus – Miscellaneous Expenditure – Profit & Loss A/c (Dr) 5. Debt-Equity Ratio This Ratio may be expressed in different manner as written in the following: Debt i. Debt-Equity Ratio = Equity Debt Capital Long-term Loan = = Shareholders’ equity Proprietors’ Fund ii. Debt-Equity Ratio = = = iii. Debt-Equity Ratio = =

Debt Debt + Equity Long-term Loan Long-term Loan + Proprietors’ Fund Long-term Loan Capital Employed Total Debt Shareholders’ Equity Long-term Debt + Short-term Debt Proprietors’ Fund

Notes: i. Out of the above three expressions of Debt-Equity Ratio, the expression written under (a) above is commonly followed. ii. This Ratio indicates the proportion of Debt Capital and Owners’ Capital included in the Capital Structure. This is an indicator of the Capital Structure of an enterprise. It also shows the efficiency of the management in financial planning.

Modified Date: Tue, Jul 06, 2010 11:23:17 AM

Output Date: Tue, Jul 06, 2010 11:24:06 AM

Rev II

Project: Management Accounting_Debarshi Bhattacharyya ACE Pro India Pvt. Ltd. File: X:\Pearson\Management Accounting_Debarshi Bhattacharyya\MAIN\M03\LAYOUT_M03\M03_DEBA_ISBN_EN_SE_C03_Part-1.indd

ACCOUNTING RATIOS FOR FINANCIAL STATEMENT ANALYSIS

67

iii. Standard or Ideal Ratio (for the expression (a) above) is 1:2, i.e., an enterprise should have twice their own capital than the debt capital. iv. Long-term Debt = Secured & Unsecured long-term Loans v. Short-term Debt = Current Liabilities 6. Capital Gearing Ratio This Ratio may also be expressed in different manner as written in the following: i. Capital Gearing Ratio =

=

Fixed Interest Bearing Securities Ordinary Securities Preference Share Capital + Long-term Loan Equity Shareholders’ Fund

ii. Capital Gearing Ratio =

Preference Share Capital + Debentures Equity Share Capital

iii. Capital Gearing Ratio =

Preference Share Capital Equity Share Capital

Notes: i. Equity Shareholders’ Fund = Equity Share Capital + Reserves & Surplus – Miscellaneous Expenditure – Profit & Loss A/c (Dr) ii. This Ratio indicates the proportion of debentures and preference Share Capital (i.e., fixed interest bearing securities) included in the Capital Structure of an enterprise. It is also an indicator of the degree of risk involved in the total Capital Employed in the business. iii. Standard or Ideal Ratio (for the express written in (a) above) is 1:2, i.e., an enterprise should have twice the ordinary securities than the Fixed Interest Bearing securities. iv. Out of the above three expressions, the expression as written in (a) is commonly used. 7. Fixed Asset Ratio Fixed Asset Ratio =

Fixed Assets Capital Employed

Notes: i. This Ratio indicates the proportion of the total Capital Employed that is invested in the Fixed Assets of an enterprise. This is also an indicator of the managerial efficiency of the enterprise. ii. Capital Employed = Proprietors’ Fund + Long-term Loan = Fixed Assets + Working Capital 8. Share Capital to Reserves and Surplus Share Capital to Reserves and Surplus =

Modified Date: Tue, Jul 06, 2010 11:23:17 AM

Share Capital Reserve & Surplus

Output Date: Tue, Jul 06, 2010 11:24:06 AM

Rev II

Project: Management Accounting_Debarshi Bhattacharyya ACE Pro India Pvt. Ltd. File: X:\Pearson\Management Accounting_Debarshi Bhattacharyya\MAIN\M03\LAYOUT_M03\M03_DEBA_ISBN_EN_SE_C03_Part-1.indd

68

MANAGEMENT ACCOUNTING

Notes: i. This Ratio indicates the proportion of Reserves and Surplus that is included in the fund invested by the shareholders (i.e. Proprietors’ Fund) in the business. ii. Higher Ratio indicates a lesser amount of Reserves and Surplus that is included in the Proprietors’ Fund. 3.14.2 Revenue Statement Ratio (Ratios as ascertained from the various items of the Profit & Loss A/c) 1. Gross Profit Ratio: Gross Profit Ratio (GP Ratio) =

Gross Profit × 100 Net Sales

Notes: i. ii. iii. iv.

GP = Sales – Cost of Goods Sold (CGS) Net Sales = Total Sales – Sales Return Normally, this Ratio is expressed as a percentage. This Ratio indicates the direct or the trading profitability of an enterprise. By means of GP Ratio, the amount and rate of profit earned by an enterprise from its trading activity can be known. Higher Ratio expresses a better direct profitability of the concern.

2. Net Profit Ratio Net Profit Ratio (NP Ratio) =

Net Profit after Tax × 100 Net Sales

Notes: i. ii. iii. iv.

Net Profit after Tax (profit after tax) = Net Profit before Tax (Profit before Tax) – Provision for Tax Net Sales = Total Sales – Sales Return Normally, this Ratio is expressed as a percentage. This Ratio indicates the total profitability of an enterprise. By means of NP Ratio, the amount and rate of total profit earned by an enterprise from its all activities including trading activity can be known. The higher Ratio exhibits the better profitability position of the enterprise.

3. Operating Ratio Notes: i. This Ratio is also expressed as a percentage. ii. This Ratio indicates the percentage of the total operating expenses included in the sales. In other words, this is an indicator of the total operating expenses incurred for production and sales of goods with respect to the selling price of those goods. 4. Operating Profit Ratio Operating Profit Ratio =

Operating Profit × 100 Net Sales

Notes: i. Operating Profit (i.e., EBIT) = Net Profit before Interest and Tax + Non-operating Expenses Debited to Profit & Loss A/c − Non-operating Incomes credited to Profit & Loss A/c ii. Net Sales = Total Sales – Sales Return

Modified Date: Tue, Jul 06, 2010 11:23:17 AM

Output Date: Tue, Jul 06, 2010 11:24:06 AM

Rev II

Project: Management Accounting_Debarshi Bhattacharyya ACE Pro India Pvt. Ltd. File: X:\Pearson\Management Accounting_Debarshi Bhattacharyya\MAIN\M03\LAYOUT_M03\M03_DEBA_ISBN_EN_SE_C03_Part-1.indd

ACCOUNTING RATIOS FOR FINANCIAL STATEMENT ANALYSIS

69

iii. Normally, this Ratio is expressed as a percentage. iv. This Ratio indicates the operating profitability of an enterprise. By means of Operating Ratio, the amount and rate of profit earned by an enterprise from its operating/trading activity can be known. Higher Ratio exhibits a better operating profitability position of the enterprise. 5. Stock Turnover Ratio or Stock Velocity Ratio Stock Turnover/Stock Velocity Ratio =

Cost of Goods Sold Average Stock

Notes: i. Cost of Goods Sold (CGS) = Sales – GP Opening Stock + Closing Stock 2 iii. Where the CGS cannot be ascertained, it may be replaced by sales, i.e., in the absence of adequate Sales . information regarding the CGS, the Ratio may be taken as Average Stock iv. In the absence of an adequate information as regards to the Opening Stock, it should be assumed that, Closing Stock = Opening Stock. In that circumstance, ii. Average Stock =

Closing Stock + Closing Stock 2 2 Closing Stock = = Closing Stocks 2 v. Where this is given in terms of Ratio or number or times, the problem can directly be solved with the help of the formula as given above. But, where this is given in terms of period (i.e., months or weeks or days), the problem can be solved with the help of the formula as given above after converting the given period in terms of Ratio or number or times as follows: 12 months 52 weeks 365 days or or Given months Given weeks Given days Average Stock =

vi. This Ratio indicates the movement of stock during a particular period. In other words, it indicates how fast goods are sold out from the stock of those goods. Higher Ratio indicates a faster movement of stock. 6. Contribution–Sales Ratio (i.e., Profit–Volume Ratio or P/V Ratio) Contribution–Sales Ratio =

Contribution × 100 Sales

Notes: i. Contribution = Sales − Variable Cost = Fixed Operating Cost + Operating Profit (or − Operating Loss) ii. It indicates the percentage of contribution included in the sales. iii. Higher ratio indicates a faster profit earning capacity of the business.

Modified Date: Tue, Jul 06, 2010 11:23:17 AM

Output Date: Tue, Jul 06, 2010 11:24:06 AM

Rev II

Project: Management Accounting_Debarshi Bhattacharyya ACE Pro India Pvt. Ltd. File: X:\Pearson\Management Accounting_Debarshi Bhattacharyya\MAIN\M03\LAYOUT_M03\M03_DEBA_ISBN_EN_SE_C03_Part-1.indd

70

MANAGEMENT ACCOUNTING

7. Operating Leverage Operating Leverage =

Contribution EBIT

Notes: i. Contribution = Sales − Variable Cost = Fixed Operating Cost + Operating Profit (i.e., EBIT) ii. EBIT/PBIT = Operating Earnings/Profit before Interest and Tax iii. Operating Leverage measures the relationship between the sales revenue and the Earnings/ Profits before Interest and Tax (EBIT/PBIT), i.e., it measures the effect of change in the Sales Revenue on the level of EBIT (i.e., operating profit). iv. If the Degree of Operating Leverage (DOL) = 1, there is no operating leverage, i.e., no fixed operating cost. The value of DOL must be greater than 1 for identifying the fixed operating cost. Higher the Fixed Costs in relation to the Variable Costs, greater would be the DOL. 8. Financial Leverage Financial Leverage =

EBIT EBT

Notes: i. EBT/PBT = Operating Earnings/Profit after Interest but before Tax ii. EBIT/PBIT = Operating Earnings/Profits before Interest and Tax iii. Financial leverage measures the relationship between the EBIT and the EPS, and it reflects the effect of changes in EBIT on the level of EPS, i.e., it measures the responsiveness of the EPS to a change in EBIT. iv. If the Degree of Financial Leverage (DFL) = 1, there is no Financial Leverage, i.e., no Fixed Financial charges in the form of interest on loan. The value of DFL must be greater than 1 for identifying the fixed financial charges. Higher the fixed financial charges, greater would be the DFL. 9. Combined Leverage Combined Leverage =

Contribution EBT

Notes: i. EBT = Operating Earnings/Profits after Interest but before Tax ii. Contribution = Sales − Variable Cost = Fixed Operating Cost + Operating Profit (i.e., EBIT) iii. The total risk involved in a firm can be determined by combining the operating and financial leverage. Hence, the combined effect of operating and financial leverage provides a risk profile of the firm. Higher Degree of Combined Leverage (DCL) indicates a greater risk of the business, i.e., existence of greater amount of fixed operating and financial charges. 10. Interest Coverage Ratio Interest Coverage Ratio =

Modified Date: Tue, Jul 06, 2010 11:23:17 AM

EBIT Interest

Output Date: Tue, Jul 06, 2010 11:24:06 AM

Rev II

Project: Management Accounting_Debarshi Bhattacharyya ACE Pro India Pvt. Ltd. File: X:\Pearson\Management Accounting_Debarshi Bhattacharyya\MAIN\M03\LAYOUT_M03\M03_DEBA_ISBN_EN_SE_C03_Part-1.indd

ACCOUNTING RATIOS FOR FINANCIAL STATEMENT ANALYSIS

71

Notes: i. Interest = Annual interest paid/Payable against Debt Capital ii. EBIT /PBIT = Earnings/Profits before Interest and Tax iii. This ratio indicates the proportion of interest paid out of the profit earned. Higher ratio signifies a lesser amount of profit paid as interest and lower ratio signifies a higher amount of profit paid as interest. 11. Dividend Coverage Ratio Dividend Coverage Ratio =

EAT Total Dividend

Notes: i. EAT = Earnings/Profits after Tax ii. Total Dividend = Preference and Equity Dividend paid/payable for a year iii. This Ratio indicates the proportion of total dividend paid out of the profit earned. Higher ratio signifies a lesser amount of profit paid as dividend and Lower ratio signifies a higher amount of profit paid as dividend. 3.14.3 Mixed or Composite Ratio (Ratio ascertained taking some items from the Balance Sheet and some other items from the Profit & Loss A/c) 1. Debtors’ Velocity Ratio or Debtors’ Turnover Ratio Debtors’ Velocity/Debtors’ Turnover Ratio =

Net Credit Sales Receivables

Notes: i. Receivables = Trade Debtors + Bills Receivable ii. Net Credit Sales = Total Credit Sales – Sales Return iii. Where this is given in terms of ratio or number or times, the problem can directly be solved with the help of the formula as given above. But, where this is given in terms of period (i.e., months or weeks or days, which is called ‘average collection period’), the problem can be solved with the help of the same formula as given above, by converting the given period in terms of Ratio as shown in the following: 365 days 12 months 52 weeks or or Given months Given months Given months iv. This ratio indicates the debt collection (against the credit sales) policy of an enterprise. This is an indicator of the credit period allowed to Trade Debtors. Generally, the credit period allowed to Trade Debtors should not be beyond 3 months to avoid the risk of bad debt. 2. Creditors’ Velocity Ratio / Creditors’ Turnover Ratio Creditors’ Velocity/Creditors’ Turnover Ratio =

Modified Date: Tue, Jul 06, 2010 11:23:17 AM

Net Credit Purchases Payables

Output Date: Tue, Jul 06, 2010 11:24:06 AM

Rev II

Project: Management Accounting_Debarshi Bhattacharyya ACE Pro India Pvt. Ltd. File: X:\Pearson\Management Accounting_Debarshi Bhattacharyya\MAIN\M03\LAYOUT_M03\M03_DEBA_ISBN_EN_SE_C03_Part-1.indd

72

MANAGEMENT ACCOUNTING

Notes: i. Net Credit Purchases = Total Credit Purchases – Purchases Return ii. Payables = Trade Creditors – Bills Payable iii. Where this is given in terms of Ratio or number or times, the problem can directly be solved with the help of the formula as given above. But where this is given in terms of period (i.e., months or weeks or days, which is called ‘average payment period’), the problem can be solved with the help of the same formula as given above by converting the given period in terms of Ratio as shown in the following: 12 months 52 weeks 365 days or or Given months Given months Given months iv. This ratio indicates the debt payment (against the credit purchases) policy of an enterprise. This is an indicator of the credit period received from the trade creditors. Generally, the credit period received from trade creditors will be 2 months. 3. Fixed Asset Turnover Ratio or Turnover of Fixed Asset Turnover Fixed Asset Turnover Ratio = Fixed Assets Notes: i. Turnover = Sales ii. This ratio indicates the extent of the utilization of Fixed Assets in the trading activities of an enterprise. It is an indicator of the managerial efficiency in the operating activities. The higher Ratio exhibits a longer stability of the trading activity of the enterprise. 4. Working Capital Turnover Ratio or Turnover of Working Capital Cost of Good Sold Working Capital Turnover Ratio = Working Capital Notes: i. Cost of Goods Sold (CGS) = Sales – Gross Profit (GP) ii. Working Capital = Current Assets – Current Liabilities iii. This ratio indicates the level of efficiency of the management in the operating activity of an enterprise. It exhibits the relationship between the CGS during a particular period and the WC in the hands of the enterprise at the end of that period. This ratio measures the extent of the utilization of the WC in the operating activity. 5. Total Asset Turnover Ratio or Turnover of Total Assets Turnover (i.e. Sales) Total Asset Turnover Ratio= Total Assets Notes: i. Total Assets = Fixed Assets + Current Assets ii. This ratio indicates the extent of utilization of Total Assets in the operating activity of an enterprise. 6. Capital Turnover Ratio or Turnover of Capital Employed Turnover Capital Turnover Ratio = Capital Employed

Modified Date: Tue, Jul 06, 2010 11:23:17 AM

Output Date: Tue, Jul 06, 2010 11:24:06 AM

Rev II

Project: Management Accounting_Debarshi Bhattacharyya ACE Pro India Pvt. Ltd. File: X:\Pearson\Management Accounting_Debarshi Bhattacharyya\MAIN\M03\LAYOUT_M03\M03_DEBA_ISBN_EN_SE_C03_Part-1.indd

ACCOUNTING RATIOS FOR FINANCIAL STATEMENT ANALYSIS

73

Notes: i. Capital Employed = Proprietors’ Fund + Long-term Loan ii. This ratio indicates the extent of utilization of the Capital Employed in the business in its operating activity. Higher the Ratio, larger is the return on the Capital Employed in the trading/ operating activity. 7. Return on Capital Employed or Return on Investment Net Profit before Interest & Tax Return on Capital Employed = × 100 Capital Employed Notes: i. It is normally expressed as a percentage. ii. It indicates the rate of return earned by an enterprise from its total Capital Employed in the business. It is also an indicator of the profit earning capacity of an enterprise. A higher return reveals a better profitability on the total Capital Employed in the business. iii. Net Profit before interest and tax = NP as ascertained after comparing all the revenue incomes with all the Revenue Expenses except interest on loan and provision for Income Tax iv. Capital Employed = Proprietors’ Fund + Long-term Loan 8. Return on Shareholders’ Fund or Return on Net Worth Net Profit after Interest & Tax Return on Net Worth = × 100 Net Worth Notes: i. It is normally expressed as a percentage. ii. It indicates the rate of return earned by an enterprise on the capital invested by its owners. This is an indicator of the rate of return on the shareholders’ fund invested in the business. The higher return reveals the better profitability position to the shareholders’ of the enterprise. iii. Net worth = Proprietors’ Fund = Shareholders’ Fund = Share Capital + Reserves & Surplus – Miscellaneous Expenses – Profit & Loss A/c (Dr) 9. Return on Equity Shareholders’ Fund Return on Equity Shareholders’ Fund =

Net Profit after Interest, Tax & Preference Dividend × 100 Equity Shareholders’ Fund

Notes: i. Net Profit after Interest, Tax and Preference Dividend = Net Profit after Interest and Tax – Preference Dividend ii. Equity Shareholders’ Fund = Equity Share Capital + Reserves & Surplus – Miscellaneous Expenses – Profit & Loss A/c (Dr) iii. It is normally expressed as a percentage iv. It indicates the rate of return earned by an enterprise on the capital that is invested by the Equity Shareholders of the enterprise. This is an indicator of the rate of Return on Equity Shareholders’ Fund. The higher return reveals the better profitability position to the Equity Shareholders. 10. Earnings per Share Earning per Share (EPS) =

Modified Date: Tue, Jul 06, 2010 11:23:17 AM

Net Profit after Tax & Preference Dividend No. of Equity Shares

Output Date: Tue, Jul 06, 2010 11:24:06 AM

Rev II

Project: Management Accounting_Debarshi Bhattacharyya ACE Pro India Pvt. Ltd. File: X:\Pearson\Management Accounting_Debarshi Bhattacharyya\MAIN\M03\LAYOUT_M03\M03_DEBA_ISBN_EN_SE_C03_Part-1.indd

74

MANAGEMENT ACCOUNTING

Notes: i. Net Profit after Tax and Preference Dividend = Earnings after Tax (EAT) – Preference Dividend. ii. This Ratio is an indicator of the amount of revenue profit of the concern that goes to each equity share. More clearly, it indicates the quantum of earnings of the company that is receivable by each equity share. iii. As per Accounting Standard 20 (AS 20), EPS is divided into two parts: (a) Primary/Basic EPS and (b) Diluted EPS. Primary NP Attributable to Equity Shareholders a. = Basic EPS Average Number of Equity Shares Outstanding b. Diluted EPS =

Adjusted NP attributable to Equity Shareholders Adjusted Weighted Average Number of Shares Outstanding

11. Dividends per Share Dividend per Share (DPS) =

Total Equity Dividend for a year No. of Equity Shares

Notes: i. This Ratio is an indicator of the amount of dividend payable against each equity share. More clearly, it indicates the quantum of dividend distributed/distributable to each equity share. 12. Dividend Payout Ratio Dividend Payout Ratio (D/P Ratio) =

DPS × 100 EPS

Notes: i. DPS = Dividends per Share (equity) ii. EPS = Earnings per Share (equity) iii. This Ratio is an indicator of the percentage of earning of the concern distributed as dividend among the shareholders (equity). D/P Ratio varies from 0% to 100%. When the D/P Ratio is 0%, it indicates that no dividend is declared out of the earning for a year. On the other hand, when the D/P Ratio is 0%, it indicates that the entire earning for a year is distributed as a dividend. Therefore, a higher ratio signifies a higher amount of dividend and a lower ratio indicates a lesser amount of dividend. 13. Price Earning Ratio Price Earning Ratio (P/E Ratio) =

MPS × 100 EPS

Notes: i. MPS = Market Price per Share (equity) ii. EPS = Earnings per Share (Equity) iii. This Ratio is an indicator of the relationship between the prevailing MPS and the earning of a concern that currently goes to each share. In other words, it indicates the relationship between the investment against each share and the earning from each share. 14. Debt Service Coverage Ratio Debt Service Coverage Ratio =

Modified Date: Tue, Jul 06, 2010 11:23:17 AM

Cash Profit before Interest & Tax Interest on Loan + Instalment of Loan Due

Output Date: Tue, Jul 06, 2010 11:24:06 AM

Rev II

Project: Management Accounting_Debarshi Bhattacharyya ACE Pro India Pvt. Ltd. File: X:\Pearson\Management Accounting_Debarshi Bhattacharyya\MAIN\M03\LAYOUT_M03\M03_DEBA_ISBN_EN_SE_C03_Part-1.indd

ACCOUNTING RATIOS FOR FINANCIAL STATEMENT ANALYSIS

75

Notes: i. Cash Profit before interest and tax = Net Profit (NP) + All non-cash items debited to Profit & Loss A/c − All non-cash items credited to Profit & Loss A/c + Interest on Loan + Tax ii. This Ratio is an indicator of firm’s ability to pay the instalment due on loan and interest on loan, out of its cash profit earned during a year. Higher Ratio signifies the greater ability of the firm to pay the instalment due on loan and interest on loan, out of its cash profit earned during a year. 3.15 DU PONT ANALYSIS Return on Investment (ROI) or Return on Total Assets represents the earning capacity of an enterprise and is the most vital Ratio for evaluating the profit earning position of the enterprise. ROI or Return on Total Assets is nothing but the combination of two other very important Ratios—one is the Net Profit (NP) Ratio and the other is the Total Asset Turnover Ratio. If either of these two Ratios changes, there would be a consequent change in ROI. ROI is expressed as follows: Return on Investment (ROI) or Return on Total Assets = NP Ratio × Total Asset Turnover Ratio Net Profit after Tax (EAT) Sales = × Sales Total Assets =

Net Profit after Tax (EAT) Total Assets

On the other hand, financial leverage multiplier (FLM) represents the extent of the Total Assets of an enterprise that is contributed by the equity shareholders. In other words, FLM shows the proportion of the Total Assets of an enterprise that are acquired out of Equity Shareholders’ Fund. FLM is expressed as follows: Total Assets Financial Leverage Multiplier (FLM) = Equity Shareholders’Fund Combining ROI and FLM, return on equity (ROE) can be derived as follows: ROE = ROI × FLM = =

Net Profit after Tax (EAT) Total Assets

×

Total Assets Equity Shareholders’ Fund

Net Profit after Tax (EAT) Equity Shareholders’ Fund

Du Pont system of financial analysis is one of the most effective technique of earning capacity or profitability analysis of an enterprise. Under this system of financial analysis, ROI is defined as the product of the NP Ratio and the Total Asset Turnover Ratio (as shown in the above equation). On the other hand, FLM is defined as the Total Assets divided by Equity Shareholders’ Fund (as shown above). Combining these two, ROE is derived (as also shown in the same). Under Du Pont Analysis, various factors which influence ROE (as well as ROI) are diagrammatically exhibited with the help of a chart, which is popularly known as Du Pont Chart, as it was firstly developed and applied by Du Pont Company of the United States, in its annual report. Du Pont Analysis enables the enterprise to break its ROE into three major components, in order to identify the area that is responsible for the profit-earning capacity of the enterprise. These three components are: profit on sales (i.e., Net Profit Ratio), efficient use of assets (i.e., Total Asset Turnover Ratio) and use of leverage in the Capital Structure (i.e., Financial Leverage Multiplier). Effect of these three components on ROE of an enterprise is expressed as follows: ROE = (Net Profit Ratio × Total Asset Turnover Ratio) × FLM = ROI × FLM

Modified Date: Tue, Jul 06, 2010 11:23:17 AM

Output Date: Tue, Jul 06, 2010 11:24:06 AM

Rev II

Project: Management Accounting_Debarshi Bhattacharyya ACE Pro India Pvt. Ltd. File: X:\Pearson\Management Accounting_Debarshi Bhattacharyya\MAIN\M03\LAYOUT_M03\M03_DEBA_ISBN_EN_SE_C03_Part-1.indd

76

MANAGEMENT ACCOUNTING

Break-up representation of ROE, as exhibited in detail by the Du Pont Chart, is depicted in Figure 3.1. Du Pont Chart Return on Equity (ROE)

×

Return on Investment (ROI)

×

Net Profit Ratio

Profit after Tax (PAT)

÷

Sales

Sales – (Cost of Goods Sold + Operating Expenses + Interest + Tax)

Financial Leverage Multiplier (FLM)

Total Assets Turnover Ratio

÷

Sales

Fixed Assets

Total Assets = Total Liabilities

÷

Equity Shareholders’ Fund

Total Outside Liabilities

+

Equity Shareholders’ Fund

Total Assets

+

Current Assets

Long-term Liabilities

+

Current Liabilities

Balance Sheet

Income/Revenue Statement

Figure 3.1

Du Pont Chart

Du Pont system of Financial Analysis breaks down the Financial Statements into two parts, namely, Income/ Revenue Statement and Balance Sheet, for the purpose of the assessment of the earning capacity of an enterprise. For this purpose, two different profitability Ratios, namely ROE and ROI, are calculated by taking various financial informations from the Financial Statements (i.e., Income/Revenue Statement and Balance Sheet). 3.16 RATIOS TO BE USED IN DETERMINING SOLVENCY, PERFORMANCE, MANAGERIAL EFFICIENCY ETC. OF AN ENTERPRISE i. For Liquidity or Short-term Solvency: Current Ratio and Liquid Ratio. ii. For Profitability: Gross Profit Ratio, Net Profit Ratio, Operating Ratio, Operating Profit Ratio, Return on Capital Employed, Return on Net Worth, Return on Equity Shareholders’ Fund, Earnings per Share, Dividend Coverage Ratio, Financial Leverage and Interest Coverage Ratio. iii. For Capital Structure Analysis: Debt-Equity Ratio, Capital Gearing Ratio, Fixed Asset Proprietorship Ratio, Proprietary Ratio, Net Worth to Total Liabilities, and Share Capital to Reserves and Surplus. iv. For Long-term Solvency: Proprietary Ratio, Fixed Asset Proprietorship Ratio, Debt-Equity Ratio, Total Outside Liabilities to Net Worth, Interest Coverage Ratio and Debt Service Coverage Ratio.

Modified Date: Tue, Jul 06, 2010 11:23:17 AM

Output Date: Tue, Jul 06, 2010 11:24:06 AM

Rev II

Project: Management Accounting_Debarshi Bhattacharyya ACE Pro India Pvt. Ltd. File: X:\Pearson\Management Accounting_Debarshi Bhattacharyya\MAIN\M03\LAYOUT_M03\M03_DEBA_ISBN_EN_SE_C03_Part-1.indd

ACCOUNTING RATIOS FOR FINANCIAL STATEMENT ANALYSIS

77

v. To measure the activity of the enterprise: Stock Turnover Ratio, Debtors’ Turnover Ratio, Creditors’ Turnover Ratio, Fixed Asset Turnover Ratio, Capital Turnover Ratio, Working Capital Turnover Ratio and Operating Leverage. vi. To evaluate the managerial efficiency: GP Ratio, NP Ratio, return on the Capital Employed, Fixed Asset Turnover Ratio, Stock Turnover Ratio, debtors’ turnover Ratio, creditors’ turnover Ratio, Capital Turnover Ratio, return on net worth, ROE Shareholders’ Fund, EPS, DPS, D/P Ratio and Price Earning Ratio. 3.17 DIFFERENT RATIOS AT GLANCE i. Balance Sheet Ratios: a. Current Ratio =

Current Assets Current Liabilities

Current Assets − Stock b. Liquid/Quick/Acid Test Ratio= Current Liabilities − Bank Overdraft c. Proprietary Ratio =

Proprietors’ Fund Total Sssets

d. Fixed Asset Proprietorship ratio = e. Debt-Equity Ratio =

Fixed Assets Proprietors’ Fund

Long-term Loan Proprietors’ Fund

f. Capital Gearing Ratio = Preference Share Capital + Long-term Loans Equity Shareholders’ Fund g. Fixed Asset Ratio =

Fixed Assets Proprietors’ Fund

h. Share Capital to Reserves and Surplus =

Share Capital Reserves & Surplus

ii. Revenue Statement Ratios: Gross Profit a. Gross Profit Ratio = × 100 Sales Net Profit after Tax × 100 b. Net Profit Ratio = Sales (Cost of Good Sold + Other Operating Expenses) × 100 Sales Cost of Goods Sold d. Stock Turnover/Velocity Ratio = Average Stock Operating Profit e. Operating Profit Ratio = × 100 Net Sales Contribution f. Contribution Sales Ratio (i.e., P/V Ratio) = × 100 Sales c. Operating Ratio =

g. Operating Leverage =

Modified Date: Tue, Jul 06, 2010 11:23:17 AM

Contribution EBIT

Output Date: Tue, Jul 06, 2010 11:24:06 AM

Rev II

Project: Management Accounting_Debarshi Bhattacharyya ACE Pro India Pvt. Ltd. File: X:\Pearson\Management Accounting_Debarshi Bhattacharyya\MAIN\M03\LAYOUT_M03\M03_DEBA_ISBN_EN_SE_C03_Part-1.indd

78

MANAGEMENT ACCOUNTING

h. Financial Leverage =

EBIT EBT

Contribution EBT EBT j. Interest Coverage Ratio = Interest EAT k. Dividend Coverage Ratio = Total dividend i. Combined Leverage =

iii. Mixed/Composite Ratio: a. Debtors’ Velocity/Turnover Ratio= Credit Sales Receivables b. Creditors’ Velocity/Turnover Ratio= Credit Purchases Payables Turnover c. Fixed Asset Turnover Ratio = Fixed Assets d. Working Capital Turnover Ratio = e. Total Asset Turnover Ratio = f. Capital Turnover Ratio =

Cost of Goods Sold Working Capital

Turnover Total Assets

Turnover Capital Employed

g. Return on Capital Employed/Investment= Net Profit before Interest & Tax ×100 Capital Employed Net Profit after interest & Tax × 100 h. Return on Net Worth = Net Worth i. Return on Equity Shareholders’ Fund (ROE) Net Profit after interest & Tax − Preference Dividend = × 100 Equity Shareholders’ Fund Net Profit after Tax & Preference Dividend No. of Equity Shares Total Equity Dividend for a year k. Dividends per Share (DPS) = No. of Equity Shares j. Earnings per Share (EPS) =

l. Dividend Payout Ratio (D/P Ratio) = DPS × 100 EPS MPS m. Price Earning Ratio (P/E Ratio) = × 100 EPS n. Debt Service Coverage Ratio =

Modified Date: Tue, Jul 06, 2010 11:23:17 AM

Cash Profit before Interest & Tax Interest on Loan + Instalment of Loan Due

Output Date: Tue, Jul 06, 2010 11:24:06 AM

Rev II

Project: Management Accounting_Debarshi Bhattacharyya ACE Pro India Pvt. Ltd. File: X:\Pearson\Management Accounting_Debarshi Bhattacharyya\MAIN\M03\LAYOUT_M03\M03_DEBA_ISBN_EN_SE_C03_Part-1.indd

79

ACCOUNTING RATIOS FOR FINANCIAL STATEMENT ANALYSIS

Tutorial Notes to Students for Solving Problems i. Problems from this chapter may be of two types: (i) Computation of different Ratios and interpretations of results from the given Profit & Loss A/c and Balance Sheet; (ii) Preparation of Profit & Loss A/c and Balance Sheet from the given Ratios. Students should identify first their target on the basis of the given information in the problem. ii. For solving problems from this chapter, students should specially remember the following: a. In any Balance Sheet, the Total of the Asset side = Total of the Liability side b. Gist of a company Balance Sheet or ultimate Balance Sheet equation is: Net Worth + Long-term Loans = Fixed Assets + Working Capital iii. If not otherwise stated in the problem, it must be assumed that there is no deferred expense or Fictitious Asset (i.e., losses under the head ‘Miscellaneous Expenditure’). iv. Difference between the GP and NP in a Profit & Loss A/c represents the ‘Indirect Operating Expenses’ charged to Profit & Loss A/c. v. As many detailed items as practicable of the Profit & Loss A/c and Balance Sheet are to be computed out of the given information in the problem, for the purpose of preparation of the Profit & Loss A/c and the Balance Sheet.

3.18 WORKED-OUT PROBLEMS Problem 1 From the following information as extracted from the books of P Ltd, ascertain the amount of Capital Employed in the business: Rs. 5,00,000 3,00,000 40,000 1,30,000 90,000 30,000 2,00,000 1,00,000 55,000 25,000 30,000 20,000

Equity Share Capital Preference Share Capital Securities Premium General Reserve Profit & Loss A/c (Cr) Capital Reserve 10% Debentures 12% Bank Loan Sundry Creditors Bills Payable Preliminary Expenses Discount on Issue of Shares

Solution Books of P Ltd Statement showing ascertainment of Capital Employed

Add:

Share Capital: Equity Share Capital Preference Share Capital Reserves Surplus: Securities Premium General Reserve Profit & Loss A/c (Cr) Capital Reserve

Rs.

Rs.

5,00,000 3,00,000

8,00,000

40,000 1,30,000 90,000 30,000

2,90,000 10,90,000 (Continued)

Modified Date: Tue, Jul 06, 2010 11:23:17 AM

Output Date: Tue, Jul 06, 2010 11:24:06 AM

Rev II

Project: Management Accounting_Debarshi Bhattacharyya ACE Pro India Pvt. Ltd. File: X:\Pearson\Management Accounting_Debarshi Bhattacharyya\MAIN\M03\LAYOUT_M03\M03_DEBA_ISBN_EN_SE_C03_Part-1.indd

80

MANAGEMENT ACCOUNTING

Rs. Less:

Add:

Miscellaneous Expenditure: Preliminary Expenses Discount on Issue of Shares Proprietors’ Fund Long-term Loans: 10% Debentures 12% Bank Loan Capital Employed

30,000 20,000

2,00,000 1,00,000

Rs.

50,000 10,40,000

3,00,000 13,40,000

Problem 2 From the following information, ascertain the amount of Capital Employed in the business: Rs. 1,20,000 3,00,000 1,80,000 90,000 80,000 72,000 57,000 43,000 14,000 18,000 6,000 47,000 12,000 11,000 30,000

Goodwill Land & Building Plant & Machinery Furniture & Fixtures 10% Investment [Long Term] Stock in Trade Sundry Debtors Cash at Bank Cash in Hand Bills Receivable Prepaid Expenses Sundry Creditors Bills Payable Outstanding Expenses Bank Overdraft

Solution Books of S Ltd Statement showing ascertainment of Capital Employed Rs.

Rs.

Fixed Assets: Goodwill Land & Building Plant & Machinery Furniture & Fixture 10% Investment [Long Term] Add:

Working Capital: Current Assets: Stock in Trade Sundry Debtors Cash at Bank Cash in Hand Bills Receivable Prepaid Expenses

Rs. 1,20,000 3,00,000 1,80,000 90,000 80,000 7,70,000

72,000 57,000 43,000 14,000 18,000 6,000 2,10,000 (Continued)

Modified Date: Tue, Jul 06, 2010 11:23:17 AM

Output Date: Tue, Jul 06, 2010 11:24:06 AM

Rev II

Project: Management Accounting_Debarshi Bhattacharyya ACE Pro India Pvt. Ltd. File: X:\Pearson\Management Accounting_Debarshi Bhattacharyya\MAIN\M03\LAYOUT_M03\M03_DEBA_ISBN_EN_SE_C03_Part-1.indd

81

ACCOUNTING RATIOS FOR FINANCIAL STATEMENT ANALYSIS

Rs. Less:

Current Liabilities: Sundry Creditors Bank Overdraft Bills Payable Outstanding Expenses

Rs.

Rs.

47,000 30,000 12,000 11,000 1,00,000

Capital Employed

1,10,000 8,80,000

Problem 3 Following is the Balance Sheet of Hum Ltd as on 31 March 2006: Liabilities Equity Share Capital Preference Share Capital Reserves & Surplus 12% Mortgage Loan 10% Debentures Sundry Creditors Bills Payable Bank Overdraft Prereceived Incomes Outstanding Expenses

Rs. 2,00,000 1,00,000 1,50,000 80,000 1,20,000 85,000 10,000 75,000 5,000 25,000

Assets Goodwill Land & Building Plant & Machinery Furniture & Fittings Investment [Long-term] Stock in Trade Short-term Investment Sundry Debtors Cash at Bank Cash in Hand Bills Receivable Accrued Incomes Prepaid Expenses Discount on Issue of Shares Preliminary Expenses

Rs. 50,000 2,00,000 1,50,000 30,000 70,000 80,000 40,000 90,000 40,000 25,000 15,000 4,000 6,000 20,000 30,000 8,50,000

8,50,000

Compute: (a) Working Capital; (b) Proprietors’ Fund; (c) Capital Employed; (d) Current Ratio; (e) Acid Test Ratio; (f) Proprietary Ratio; (g) Fixed Asset Proprietorship Ratio; (h) Debt-Equity Ratio; (i) Capital Gearing Ratio and (j) Fixed Asset Ratio. Solution i. Statement showing computation of Working Capital Rs. Total Current Assets: Stock in Trade Short-term Investment Sundry Debtors Cash at Bank Cash in Hand Bills Receivable Accrued Incomes Prepaid Expenses Less:

Total Current Liabilities: Sundry Creditors Bills Payable

Rs. 80,000 40,000 90,000 40,000 25,000 15,000 4,000 6,000 3,00,000

85,000 10,000 (Continued)

Modified Date: Tue, Jul 06, 2010 11:23:17 AM

Output Date: Tue, Jul 06, 2010 11:24:06 AM

Rev II

Project: Management Accounting_Debarshi Bhattacharyya ACE Pro India Pvt. Ltd. File: X:\Pearson\Management Accounting_Debarshi Bhattacharyya\MAIN\M03\LAYOUT_M03\M03_DEBA_ISBN_EN_SE_C03_Part-1.indd

82

MANAGEMENT ACCOUNTING

Rs. 75,000 5,000 25,000

Bank Overdraft Prereceived Incomes Outstanding Expenses Working Capital

Rs.

2,00,000 1,00,000

ii. Statement showing computation of Proprietors’ Fund Rs.

Add: Less:

Rs. 2,00,000 1,00,000 1,50,000 4,50,000

Equity Share Capital Preference Share Capital Reserves & Surplus Miscellaneous Expenditure: Discount on Issue of Shares Preliminary Expenses Proprietors’ Fund

20,000 30,000

50,000 4,00,000

iii. Statement showing computation of Capital Employed Rs. Add:

Proprietors’ Fund as computed in (b) above Long-term Loans: 12% Mortgage Loan 10% Debentures Capital Employed

80,000 1,20,000

Rs. 4,00,000

2,00,000 6,00,000

Note: Alternatively, Capital Employed may also be computed following a different approach as computed in the following:

Statement showing computation of Capital Employed Rs. Fixed Assets: Goodwill Land & Building Plant & Machinery Furniture & Fitting Investment [Long Term] Add:

Working Capital [as computed in (a) above] Capital Employed

50,000 2,00,000 1,50,000 30,000 70,000 5,00,000 1,00,000 6,00,000

iv. Computation of Current Ratio Current Assets Current Ratio = Current Liabilities Here, Current Assets = Rs. 3,00,000 [as ascertained in (a) above] Current Liabilities = Rs. 2,00,000 [as ascertained in (a) above] ∴Current Ratio = v. Acid Test Ratio =

Rs. 3,00,000 3 = = 1.5 Rs. 2,00,000 2

Current Assets − Stock Current Liabilities − Bank Overdraft

Modified Date: Tue, Jul 06, 2010 11:23:17 AM

Output Date: Tue, Jul 06, 2010 11:24:06 AM

Rev II

Project: Management Accounting_Debarshi Bhattacharyya ACE Pro India Pvt. Ltd. File: X:\Pearson\Management Accounting_Debarshi Bhattacharyya\MAIN\M03\LAYOUT_M03\M03_DEBA_ISBN_EN_SE_C03_Part-1.indd

ACCOUNTING RATIOS FOR FINANCIAL STATEMENT ANALYSIS

=

Rs.3,00,000 − Rs.80,000 Rs.2,00,000 − Rs.75,000

=

Rs.2,20,000 Rs.1,25,000

83

= 1.76

Proprietors’ Fund Total Assets Here, Proprietors’ Fund = Rs. 4,00,000 [as ascertained in (b) above] Total Assets = FAs + CAs = Rs. 5,00,000 [as ascertained in (c) above] + Rs. 3,00,000 = Rs. 8,00,000 Rs. 4,00,000 1 ∴ Proprietary Ratio= = = 0.5 Rs. 8,00,000 2

vi. Proprietary Ratio =

Fixed Assets Proprietors’ Fund Here, FAs = Rs. 5,00,000 [as ascertained in (c) above] Proprietors’ Fund = Rs. 4,00,000 [as ascertained in (b) above] Rs. 5,00,000 5 ∴ Fixed Asset Proprietorship Ratio = = = 1.25 Rs. 4,00,000 4

vii. Fixed Asset Proprietorship Ratio =

Long-term Loan Debt = Equity Proprietors’ Fund Here, Long-term Loan = Rs. 2,00,000 [as ascertained in (h) above] Proprietors’ Fund = Rs. 4,00,000 [as ascertained in (b) above] Rs. 2,00,000 1 Debt-Equity Ratio = = = 0.5 Rs. 4,00,000 2

viii. Debt-Equity Ratio =

Fixed Interest Bearing Securities Ordinary Securities Preference Share Capital + Long-term Loan = Equity Shareholders’ Fund Here, Fixed Interest Bearing securities = Preference Share Capital + 10% Debentures + 12% Mortgage loan = Rs. 1,00,000 + Rs. 1,20,000 + Rs. 80,000 = Rs. 3,00,000 Ordinary securities = Equity Shareholders’ Fund = Proprietors’ Fund – Preference Share Capital = Rs. 4,00,000 [as computed in (b) above] – Rs. 1,00,000 = 3,00,000 Rs. 3,00,000 ∴ Capital Gearing Ratio = =1 Rs. 3,00,000

ix. Capital Gearing Ratio =

x. Fixed Asset Ratio =

Fixed Assets Capital Employed

Here, FAs = Rs. 5,00,000 [as computed in (c) above] Capital Employed = Rs. 6,00,000 [as computed in (c) above] 5,00,000 ∴ Fixed Asset Ratio = = 5:6 6,00,000

Modified Date: Tue, Jul 06, 2010 11:23:17 AM

Output Date: Tue, Jul 06, 2010 11:24:06 AM

Rev II

Project: Management Accounting_Debarshi Bhattacharyya ACE Pro India Pvt. Ltd. File: X:\Pearson\Management Accounting_Debarshi Bhattacharyya\MAIN\M03\LAYOUT_M03\M03_DEBA_ISBN_EN_SE_C03_Part-1.indd

84

MANAGEMENT ACCOUNTING

Problem 4 Following are the Profit & Loss A/c for the year that ended on 31 March 2008 and a Balance Sheet as on that date of company: Profit & Loss A/c for the year that ended on 31 March 2008 Dr.

Cr. Rs. 70,000 8,00,000 1,20,000 2,90,000 3,00,000 15,80,000 25,000 70,000 15,000 15,000 25,000 30,000 10,000 10,000 1,00,000 3,00,000 40,000 20,000 1,50,000 2,10,000

To Opening Stock To Purchases To Carriage Inward To Wages To Gross Profit c/d To Establishment Charges To Salaries To Rent & Rates To General Expenses To Depreciation To Selling Expanses To Interest on Debentures To Provision for Taxation To Net Profit c/d To General Reserve To Proposed Dividend To Balance c/f

By Sales By Closing Stock

Rs. 15,00,000 80,000

By Gross Profit b/d

15,80,000 3,00,000

By Balance b/d By Net Profit b/d

3,00,000 1,10,000 1,00,000 2,10,000

Balance Sheet as on 31 March 2006 Liabilities Equity Share Capital Preference Share Capital General Reserve Profit & Loss A/c 10% Debentures Sundry Creditors Bank Overdraft Bills Payable Provision for Taxation Proposed Dividend

Rs. 4,00,000 1,00,000 40,000 1,50,000 1,00,000 60,000 50,000 10,000 10,000 20,000 9,40,000

Assets Fixed Assets Stock in Trade Sundry Debtors Cash & Bank Bills Receivable Discount on Issue of Shares

Rs. 7,00,000 80,000 90,000 40,000 10,000 20,000

9,40,000

From the above information, calculate: (a) GP Ratio; (b) NP Ratio; (c) Operating Ratio; (d) Return on Net Worth; (e) Return on Capital Employed; (f) Capital Turnover Ratio; (g) Fixed Asset Turnover Ratio; (h) Working Capital Turnover Ratio; (i) Stock Turnover Ratio; (j) Debtors’ Velocity Ratio; (k) Creditors’ Velocity Ratio. Solution i. Gross Profit Ratio =

Gross Profit

Modified Date: Tue, Jul 06, 2010 11:23:17 AM

Sales

× 100 =

3,00,000 × 100 = 20% 15,00,000

Output Date: Tue, Jul 06, 2010 11:24:06 AM

Rev II

Project: Management Accounting_Debarshi Bhattacharyya ACE Pro India Pvt. Ltd. File: X:\Pearson\Management Accounting_Debarshi Bhattacharyya\MAIN\M03\LAYOUT_M03\M03_DEBA_ISBN_EN_SE_C03_Part-1.indd

ACCOUNTING RATIOS FOR FINANCIAL STATEMENT ANALYSIS

ii. Net Profit Ratio =

85

Net Profit after Tax 1,00,000 × 100 = × 100 = 6.67% Sales 15,00,000

(Cost of Goods Sold + Other Operating Expenses) × 100 Sales 12,00,0001 + 1,80,0002 = × 100 = 92% 15,00,000

iii. Operating Ratio =

Net Profit after Tax 1,00,000 × 100 = × 100 = 14.92% Net Worth 6,70,000 4

iv. Return on Net Worth =

v. Return of Capital Employed = = vi. Capital Turnover Ratio =

Net Profit before interest & Tax × 100 Capital Employed

1,20,0003 × 100 = 15.58% 7,70,0005

Turnover 15,00,000 = = 1.95 Capital Employed 7,70,000

vii. Fixed Asset Turnover Ratio =

Turnover 15,00,000 = = 2.14 Fixed Assets 7,00,000

viii. Working Capital Turnover Ratio = ix. Stock Turnover Ratio =

Turnover 15,00,000 = = 21,4285 Working Capital 70,00011

Cost of Goods Sold 12,00,0001 = = 16 Average Stock 75,0006

Credit Sales 15,00,0007 = = 15 Receivables 1,00,0008 365 days 365 ∴ Average Collection Period = = = 24 days (approx.) Debtors’ Velocity Ratio 15

x. Debtors’ Velocity Ratio =

xi. Creditors’ Velocity Ratio =

Credit Purchases 8,00,00010 = = 11.4285 Payables 70,0009

∴ Average Payment Period =

365 days 365 = = 32 days (approx.) Creditors’ Velocity Ratio 11.4285

Working Notes 1. Calculation of Cost of Goods Sold (CGS)

Add:

Opening Stock Purchases

Less:

Closing Stock

Rs. 70,000 8,00,000 8,70,000 80,000 (Continued)

Modified Date: Tue, Jul 06, 2010 11:23:17 AM

Output Date: Tue, Jul 06, 2010 11:24:06 AM

Rev II

Project: Management Accounting_Debarshi Bhattacharyya ACE Pro India Pvt. Ltd. File: X:\Pearson\Management Accounting_Debarshi Bhattacharyya\MAIN\M03\LAYOUT_M03\M03_DEBA_ISBN_EN_SE_C03_Part-1.indd

86

MANAGEMENT ACCOUNTING

Add: Add:

Materials Consumed Carriage Inward Wages Cost of Goods Sold

Rs. 7,90,000 1,20,000 2,90,000 12,00,000

2. Calculation of other Operating Expenses Establishment Charges Salaries Rent & Rates General Expenses Depreciation Selling Expenses Total other Operating Expenses

Rs. 25,000 70,000 15,000 15,000 25,000 30,000 1,80,000

3. Calculation of Net Profit before Interest and Tax (Net PBIT)

Add: Add:

Net Profit after Interest & Tax Interest on Debentures Provision for Taxation Net Profit before Interest and Tax

Rs. 1,00,000 10,000 10,000 1,20,000

4. Calculation of Net Worth

Add:

Less:

Rs. 4,00,000 1,00,000 5,00,000

Equity Share Capital Preference Share Capital Total Share Capital Reserves & Surplus: General Reserve Profit & Loss A/c

40,000 1,50,000 6,90,000

Miscellaneous Expenditure: Discount on Issue of Shares Net Worth or Proprietors’ Fund

20,000 6,70,000

5. Calculation of Capital Employed

Add:

Net Worth as calculated in (4) above Long-term Loans: 10% Debentures Capital Employed

Rs. 6,70,000 1,00,000 7,70,000

6. Calculation of Average Stock Average Stock =

Opening Stock + Closing Stock Rs. 70,000 + Rs. 80,000 = = Rs. 75.000 2 2

7. Calculation of Credit Sales In the absence of adequate information, it is assumed that the entire sales have been made on credit. Total Sales = Credit Sales = Rs. 15,00,000 (Continued)

Modified Date: Tue, Jul 06, 2010 11:23:17 AM

Output Date: Tue, Jul 06, 2010 11:24:06 AM

Rev II

Project: Management Accounting_Debarshi Bhattacharyya ACE Pro India Pvt. Ltd. File: X:\Pearson\Management Accounting_Debarshi Bhattacharyya\MAIN\M03\LAYOUT_M03\M03_DEBA_ISBN_EN_SE_C03_Part-1.indd

87

ACCOUNTING RATIOS FOR FINANCIAL STATEMENT ANALYSIS

8. Calculation of Receivables Rs. 90,000 10,000 1,00,000

Sundry Debtors Bills Receivable Receivables

9. Calculation of Payables Rs. 60,000 10,000 70,000

Sundry Creditors Bills Payable Payables

10. Calculation of Credit Purchases In the absence of adequate information, it is assumed that the entire purchases have been made on credit. Total Purchases = Credit Purchases = Rs. 8,00,000 11. Calculation of Working Capital Rs. Total Current Assets: Stock in Trade Sundry Debtors Cash & Bank Bills Receivable Less:

Rs. 80,000 90,000 40,000 10,000 2,20,000

Total Current Liabilities: Sundry Creditors Bank Overdraft Bills Payable Provision for Taxation Proposed Dividend Working Capital

60,000 50,000 10,000 10,000 20,000

1,50,000 70,000

Problem 5 From the following Balance Sheet of Y Ltd, calculate the different turnover Ratios: Liabilities Equity Share Capital General Reserve 8% Debentures Creditors

Rs. 6,00,000 2,50,000 1,50,000 2,00,000 12,00,000

Assets Fixed Assets Stock Debtors Cash at Bank

Rs. 6,50,000 2,25,000 2,75,000 50,000 12,00,000

Given Capital Turnover Ratio is 1.5 times. [B.Com. (Hons), Calcutta University—2008] Solution Here, Capital Employed = Equity Share Capital + General Reserve + 8% Debentures = Rs. 6,00,000 + Rs. 2,50,000 + Rs. 1,50,000 = Rs. 10,00,000 Again, Capital Turnover Ratio =

Modified Date: Tue, Jul 06, 2010 11:23:17 AM

Turnover = 1.5 Capital Employed

Output Date: Tue, Jul 06, 2010 11:24:06 AM

Rev II

Project: Management Accounting_Debarshi Bhattacharyya ACE Pro India Pvt. Ltd. File: X:\Pearson\Management Accounting_Debarshi Bhattacharyya\MAIN\M03\LAYOUT_M03\M03_DEBA_ISBN_EN_SE_C03_Part-1.indd

88

MANAGEMENT ACCOUNTING

or

Turnover = 1.5 10,00,000 ∴ Turnover = Rs. 15,00,000

Total Asset Turnover Ratio = Inventory Turnover Ratio =

Turnover 15,00,000 = = 6.67 times Closing Inventory 2,25,000

Fixed Asset Turnover Ratio = Debtors’ Turnover Ratio =

Turnover 15,00,000 = = 1.25 times Total Assets 12,00,000

Turnover 15,00,000 = = 2.31 times Fixed Assets 6,50,000

Turnover 15,00,000 = = 5.4545 times Debtors 2,75,000

Creditors’ Turnover Ratio =

Turnover 15,00,000 = = 7.5 times Creditors 2,00,000

Problem 6 From the following details, calculate the average collection period: Sundry Debtors Bills Receivable Average Stock Inventory Turnover Ratio GP Ratio Credit Sales to Total Sales Assume, 1 year = 360 days

Rs. 2,10,000 Rs. 30,000 Rs. 3,60,000 6 10% 80%

Solution Debtors’ Turnover Ratio =

Credit Sales 19,20,0002 = =8 Receivables 2,40,0001

∴ Average Collection Period =

360 days 360 days = = 45 days Debtors’ Turnover Ratio 8

Working Notes 1. Calculation of Receivables Sundry Debtors Bills Receivable Receivables

Rs. 2,10,000 30,000 2,40,000

2. Calculation of Credit Sales Inventory Turnover Ratio =

Cost of Goods Sold (CGS) Average Stock (Continued)

Modified Date: Tue, Jul 06, 2010 11:23:17 AM

Output Date: Tue, Jul 06, 2010 11:24:06 AM

Rev II

Project: Management Accounting_Debarshi Bhattacharyya ACE Pro India Pvt. Ltd. File: X:\Pearson\Management Accounting_Debarshi Bhattacharyya\MAIN\M03\LAYOUT_M03\M03_DEBA_ISBN_EN_SE_C03_Part-1.indd

ACCOUNTING RATIOS FOR FINANCIAL STATEMENT ANALYSIS

Here, 6 =

89

Cost of Goods Sold 3,60,000

CGS = 3,60,000 × 6 = Rs. 21,60,000 Gross Profit Again, GP Ratio = × 100 = 10% Sales ∴ GP rate = 10% on sales = 1/10 on sales = 10 = 1 = 9 Gross Profit ∴Rate of GP on Cost = = 1/9 (on cost) Cost of Goods Sold i.e., If sales then, GP CGS

∴ Total Sales = CGS + GP or Total Sales = CGS + 1/9 on CGS Total Sales = Rs. 21,60,000 + 1/9 on Rs. 21,60,000 = Rs. 24,00,000 Again, Credit Sales to Total Sales = 80% Credit Sales 80 or = Total Sales 100 Credit Sales 4 or = 24,00,000 5 24,00,000 ∴ Credit Sales = × 4 = Rs.19,20,000 5

Problem 7 i. Cost of Goods Sold of a company was Rs. 1,00,000 in 2005. Inventory Turnover was 4 times. Inventory at the end was 1.5 times of that at the beginning. Calculate the Closing Inventory. ii. From the information given, calculate the average collection period: Total Sales (including cash sales – Rs. 20,000)—Rs. 1,00,000; Sales Return—Rs. 7,000; Debtors—Rs. 9,000; Bills Receivable—Rs. 2,000; Creditors—Rs. 10,000. [B.Com. (Hons), Kalyani University—2006] Solution i. Inventory Turnover = Here, 4 =

Cost of Goods Sold Average Stock

1,00,000 Average Stock

∴ Average Stock = Rs. 25,000 Let the Opening Stock be x. ∴ Closing Stock = 1.5 x Opening Stock + Closing Stock ∴ Average Stock = 2

Modified Date: Tue, Jul 06, 2010 11:23:17 AM

Output Date: Tue, Jul 06, 2010 11:24:06 AM

Rev II

Project: Management Accounting_Debarshi Bhattacharyya ACE Pro India Pvt. Ltd. File: X:\Pearson\Management Accounting_Debarshi Bhattacharyya\MAIN\M03\LAYOUT_M03\M03_DEBA_ISBN_EN_SE_C03_Part-1.indd

90

MANAGEMENT ACCOUNTING

x + 1.5x 2 or 2.5x = 50,000 ∴ x = Rs. 20,000 ∴ Closing Inventory = 1.5x = 1.5 × 20,000 = Rs. 30,000 or 25,000 =

Receivables × 365 days Net Credit Sales Debtors + Bills Receivable = × 365 Credit Sales − Sales Return 9,000 + 2,000 = × 365 = 55days 80,000 − 7,000

ii. Average Collection Period =

Problem 8 Given: Current Assets = Rs. 1,50,000 Current Liabilities = Rs. 1,29,000 Fixed Assets/Net Worth = 0.70 No Debt Capital. Calculate: Value of Fixed Assets. [B.Com. (Hons), Calcutta University—2006] Solution List of a company Balance Sheet is as follows: Net Worth + Long-term Loan = Fixed Assets + Working Capital Here, NW + Nil = FAs + (1,50,000 − 1,29,000) [As Working Capital = CAs − CLs] ∴ Net Worth = FAs + 21,000 FAs Again, = 0.70 NW FAs or = 0.70 FAs+21,000 or FAs = 0.7 FAs + 14,700 or 0.3 FAs = 14,700 ∴ Fixed Assets = Rs. 49,000 Problem 9 The following data have been extracted from the Financial Statements of a company: Rs. 4,00,000 1,00,000 1,20,000 1,30,000 40,000 30,000 2,00,000

Equity Share Capital 12% Preference Share Capital General Reserve Profit & Loss A/c (Cr) Securities Premium Capital Reserve 15% Debentures

(Continued)

Modified Date: Tue, Jul 06, 2010 11:23:17 AM

Output Date: Tue, Jul 06, 2010 11:24:06 AM

Rev II

Project: Management Accounting_Debarshi Bhattacharyya ACE Pro India Pvt. Ltd. File: X:\Pearson\Management Accounting_Debarshi Bhattacharyya\MAIN\M03\LAYOUT_M03\M03_DEBA_ISBN_EN_SE_C03_Part-1.indd

91

ACCOUNTING RATIOS FOR FINANCIAL STATEMENT ANALYSIS

Rs. 2,70,000 25,000 32,000 20,000

Net Profit before Tax Provision for Taxation Tax actually Paid during the year Discount on Issue of Shares

From the above information, calculate: (a) Return on Equity Shareholders’ Fund. (b) Return on Net Worth. (c) Return on Capital Employed. Solution i. Return on Equity Shareholders’ Fund Net Profit after Tax & Preference Dividend = × 100 Equity Shareholders’ Fund =

2,33,0006 × 100 = 33.2857% 7,00,000

ii. Return on Net Worth =

Net Profit after Tax Net Worth

× 100

2,45,0005 × 100 = 30.625% 8,00,0002 iii. Return on Capital Employed =

=

Net Profit before Interest & Tax × 100 Capital Employed

=

3,00,0004 × 100 = 30% 10,00,0003

Working Notes 1. Calculation of Equity Shareholders’ Fund Rs. Add:

Less:

Equity Share Capital Reserves & Surplus: General Reserve Securities Premium Capital Reserve Profit & Loss A/c (Cr)

1,20,000 40,000 30,000 1,30,000

Miscellaneous Expenditure: Discount on Issue of Shares Equity Shareholders’ Fund

Rs. 4,00,000

3,20,000 7,20,000 20,000 7,00,000 (Continued)

Modified Date: Tue, Jul 06, 2010 11:23:17 AM

Output Date: Tue, Jul 06, 2010 11:24:06 AM

Rev II

Project: Management Accounting_Debarshi Bhattacharyya ACE Pro India Pvt. Ltd. File: X:\Pearson\Management Accounting_Debarshi Bhattacharyya\MAIN\M03\LAYOUT_M03\M03_DEBA_ISBN_EN_SE_C03_Part-1.indd

92

MANAGEMENT ACCOUNTING

2. Calculation of Net Worth

Add:

Rs. 7,00,000 1,00,000 8,00,000

Equity Shareholders’ Fund as Calculated in (1) above 12% Preference Share Capital Net Worth

3. Calculation of Capital Employed

Add:

Rs. 8,00,000

Net worth as Calculated in (2) above Long-term Loans: 15% Debentures

2,00,000 10,00,000

Capital Employed

4. Calculation of Net PBIT

Add:

Net Profit before Tax as given Interest on Debentures already charged [15% on Rs. 2,00,000] Net PBIT

Rs. 2,70,000 30,000 3,00,000

5. Calculation of Net Profit after tax

Less:

Rs. 2,70,000 25,000 2,45,000

Net Profit before Tax as given Provision for Taxation Net Profit after Tax

6. Calculation of Net Profit after tax and preference dividend

Less:

Net Profit after Tax as Calculated in (5) above Preference Dividend [12% on Rs. 1,00,000] Net Profit after Tax & Preference Dividend

Rs. 2,45,000 12,000 2,33,000

Problem 10 A company has a profit margin of 20% and asset turnover of 3 times. What is the company’s ROI? How will this ROI vary if: i. Profit Margin is increased by 5%? ii. Asset Turnover is decreased to 2 times? iii. Profit Margin is decreased by 5% and Asset Turnover is increased to 4 times? [B.Com. (Hons), Calcutta University—2008] Solution Here, Profit Margin = 20%. i.e., if Sales = 100, then Profit = 20 Sales Again, Asset Turnover = =3 Total Assets

Modified Date: Tue, Jul 06, 2010 11:23:17 AM

Output Date: Tue, Jul 06, 2010 11:24:06 AM

Rev II

Project: Management Accounting_Debarshi Bhattacharyya ACE Pro India Pvt. Ltd. File: X:\Pearson\Management Accounting_Debarshi Bhattacharyya\MAIN\M03\LAYOUT_M03\M03_DEBA_ISBN_EN_SE_C03_Part-1.indd

93

ACCOUNTING RATIOS FOR FINANCIAL STATEMENT ANALYSIS

or

100 = 3 ∴ Total Assets = 33.33 Total Assets

∴ Return on Investment (ROI) =

Profit 20 × 100 = × 100 = 60% Total Assets 33.33

i. New Profit Margin = 20% + 5% = 25%. i.e., if Sales = 100, then Profit = 25 Profit 25 ∴ New ROI = × 100 = × 100 = 75% Total Assets 33.33 ii. New Asset Turnover = or

Sales =2 Total Assets

100 = 2. ∴ Total Assets = 50. And, Profit Margin = 20% Total Assets

∴ New ROI =

Profit 25 × 100 = × 100 = 75% Total Assets 33.33

iii. New Profit Margin = 20% − 5% = 15% i.e., if Sales = 100, then Profit = 15 Sales New Asset Turnover = =4 Total Assets or

100 = 4. ∴ Total Assets = 25 Total Assets

New ROI =

Profit 15 × 100 = × 100 = 60% Total Assets 25

Problem 11 JKL Ltd has the following Balance Sheets as on 31 March 2006 and 31 March 2005: Rs. in lakhs 31 March 31 March 2006 2005 Sources of Funds: Shareholders’ Funds Loan Funds

Less:

Applications of Funds: Fixed Assets Cash & Bank Debtors Stock Other Current Assets Current Liabilities

Modified Date: Tue, Jul 06, 2010 11:23:17 AM

Output Date: Tue, Jul 06, 2010 11:24:06 AM

2,377 3,570 5,947

1,472 3,083 4,555

3,466 489 1,495 2,867 1,567 (3,937) 5,947

2,900 470 1,168 2,407 1,404 (3,794) 4,555

Rev II

Project: Management Accounting_Debarshi Bhattacharyya ACE Pro India Pvt. Ltd. File: X:\Pearson\Management Accounting_Debarshi Bhattacharyya\MAIN\M03\LAYOUT_M03\M03_DEBA_ISBN_EN_SE_C03_Part-1.indd

94

MANAGEMENT ACCOUNTING

The Income Statement of the JKL Ltd for the year that ended is as follows:

Less: Less: Less: Less:

Rs. in lakhs 31 March 31 March 2006 2005 22,165 13,882 20,860 12,544 1,305 1,338 1,135 752 170 586 113 105 57 481 23 192 34 289

Sales CGS GP Selling, General & Administrative Expenses Earnings before Interest and Tax (EBIT) Interest Expense Profit before Tax Tax Profit after Tax

Required: i. Calculate for the year 2005–06: a. Inventory Turnover Ratio. b. Financial Leverage. c. ROI. d. ROE. e. Average Collection Period. ii. Give a brief comment on the financial position of JKL Ltd. [C.A. (PE II)—May 2006] Solution i. a. Inventory Turnover Ratio (for the year 2005 − 06) = b. Financial Leverage =

Cost of Goods Sold 20,860 = = 7.91 Average Inventory 2,637

EBIT EBT

586 = 1.22 481 170 For the year 2005−06, Financial Leverage = = 2.98 57 For the year 2004 −05, Financial Leverage =

c. ROI (for the year 2005− 06) = = d. ROE (for the year 2005−06) = =

Net Profit before Interest but after Tax × 100 Average Capital Employed 102 × 100 = 1.94% 5,251 Net Profit available to Equity Shareholders × 100 Average Equity Shareholders’ Fund 34 × 100 = 1.77% 1,924.50

e. Inventory Turnover Ratio (for the year 2005−06) =

Modified Date: Tue, Jul 06, 2010 11:23:17 AM

Average Debtors 1,331.50 = = 22 days Average Sales per day 60.73

Output Date: Tue, Jul 06, 2010 11:24:06 AM

Rev II

Project: Management Accounting_Debarshi Bhattacharyya ACE Pro India Pvt. Ltd. File: X:\Pearson\Management Accounting_Debarshi Bhattacharyya\MAIN\M03\LAYOUT_M03\M03_DEBA_ISBN_EN_SE_C03_Part-1.indd

95

ACCOUNTING RATIOS FOR FINANCIAL STATEMENT ANALYSIS

ii. Profitability of operation of the company remarkably declines from 586 (Rs. in lakhs) to 170 (Rs. in lakhs), due to a huge increase in the operating expenses during the year 2005–06. NP of the company also reduces due to an increase in the interest expenses. During the year 2005–06, both Fixed Operating Expense as well as fixed financial expense have increased, as a consequence of which the NP of the company radically reduced. During 2005–06, both operating and Financial Leverages have become adverse, as a result of which the company has been crucially suffering from a liquidity crisis during the year 2005–06. Working Notes 1. Calculation of Average Inventory Average Inventory =

Opening Inventory + Closing Inventory 2,407 + 2,867 = = 2,637 (Rs. in lakhs) 2 2

2. Calculation of Average Equity Shareholders’ Fund Average Equity Shareholder’s Fund =

Opening Equity Shareholder’s Fund + Closing Equity Shareholder’s Fund 2

1,472 + 2,377 = 1,924.5 (Rs. in lakhs) 2 3. Calculation of Average Capital Employed =

Opening Capital Employed + Closing Capital Employed 2 (1,472 + 3,083) + (2,377 + 3,570) = = 5,251 (Rs. in lakhs) 2 4. NP available to Equity Shareholders Average Capital Employed =

Rs. in lakhs 34 Nil 34

Profit after Tax Preference Dividend NP Available to Equity Shareholders

Less:

5. Calculation of Average Debtors Average Debtors =

Opening Debtors + Closing Debtors 1,168 + 1,495 = = 1,331.5 (Rs. in lakhs) 2 2

6. Calculation of Average Sales per day Average Sales per day for the year 2005−06 =

22,165 = 60.73 (Rs. in lakhs) 365

7. Net Profit before interest but after tax

Less:

EBIT for the year 2005–06 Tax @ 40% Net Profit before Interest but after Tax

Rs. in lakhs 170 68 102

Note: Tax Rate =

Tax Amount 23 × 100 = × 100 = 40% (approx.) EBT 57

Modified Date: Tue, Jul 06, 2010 11:23:17 AM

Output Date: Tue, Jul 06, 2010 11:24:06 AM

Rev II

Project: Management Accounting_Debarshi Bhattacharyya ACE Pro India Pvt. Ltd. File: X:\Pearson\Management Accounting_Debarshi Bhattacharyya\MAIN\M03\LAYOUT_M03\M03_DEBA_ISBN_EN_SE_C03_Part-1.indd

96

MANAGEMENT ACCOUNTING

Problem 12 Following information is available from the Financial Statements of K Ltd for the year that ended on 31 March 2006: i. Inventory Turnover Ratio is 6 times, Closing Debtors are outstanding for 2 months and Closing Creditors are outstanding for 73 days. ii. Ratio of CGS to: Proprietors’ Fund is 2 : 1. Fixed Asset is 4 : 1. iii. GP Ratio is 20%. iv. Closing Stock is greater than the opening stock by Rs. 10,000. v. GP for the year that ended on 31 March 2006 is Rs. 1,20,000. vi. Reserves and surplus appearing in the Balance Sheet as on 31 March 2006 is Rs. 40,000. You are asked to calclaute the following: (i) Proprietors’ Fund; (ii) Fixed Asset; (iii) Closing Debtors; (iv) Closing Creditors; (v) Closing Stock; (vi) Share Capital. Solution i. Ratio of CGS to Proprietors’ Fund = 2:1 Cost of Goods Sold 2 or = Proprietors’ Fund 1 or

4,80,0001 2 = Proprietors’ Fund 1

∴ Proprietors’ Fund =

4,80,000 = Rs. 2,40,000 4

ii. Ratio of CGS to Fixed Asset = 4:1 or

Cost of Goods Sold 4 = Fixed Asset 1

or

4,80,0001 4 = Fixed Asset 1

∴ Fixed Asset =

4,80,000 = Rs. 1,20,000 4

iii. Debt Collection Period = 2 months. ∴ Debtors’ Turnover Ratio = or

12 months Debt Collection Period

Credit Sales 12 months = Receivables 2 months

6,00,000 =6 Rs. 1,00,000 ∴ Receivables = Rs. 1,00,000 Again, Receivables = Debtors + Bills Receivable or

Modified Date: Tue, Jul 06, 2010 11:23:17 AM

Output Date: Tue, Jul 06, 2010 11:24:06 AM

Rev II

Project: Management Accounting_Debarshi Bhattacharyya ACE Pro India Pvt. Ltd. File: X:\Pearson\Management Accounting_Debarshi Bhattacharyya\MAIN\M03\LAYOUT_M03\M03_DEBA_ISBN_EN_SE_C03_Part-1.indd

ACCOUNTING RATIOS FOR FINANCIAL STATEMENT ANALYSIS

97

In the absence of adequate information, it is assumed that Bills Receivable = Nil ∴ Receivables = Debtors + Nil or 1,00,000 = Debtors ∴ Closing Debtors = Rs. 1,00,000 iv. Average Payment Period = 73 days ∴ Creditors’ Turnover Ratio = or

Credit Purchases =5 Payables

or

4,90,000 =5 Payables

365 Days 365 Days = =5 Average Payment Period 73 Days

4,90,0001 = Rs.98,000 5 Again, Payables = Creditors + Bills payable Here, it is assumed that Bills payable = Nil ∴ Payables = Creditors + Nil or 98,000 = Creditors ∴ Closing Creditors = Rs. 98,000 ∴ Payables =

v. Inventory Turnover Ratio = or 6 =

Cost of Goods Sold Average Stock

4,80,0001 =9 Average Stock

∴ Average Stock =

4,80,000 = Rs. 80,000 6

But, Average Stock =

Opening Stock + Closing Stock 2

Again, according to the question, Closing stock − Opening stock = 10,000 ∴ Opening stock = Closing stock – 10,000 (Closing Stock − 10,000) + Closing Stock 2 2 Closing Stock − 10,000 or 80,000 = 2 or 2 Closing Stock – 10,000 = 1,60,000 ∴ Average Stock =

2 Closing Stock = 1,60,000 + 10,000 1,70,000 ∴ Closing Stock = = Rs. 85,000 2

or

vi. Proprietors’ Fund = Share Capital + Reserves & Surplus – Miscellaneous Expenditure Here, 2,40,000 = Share Capital + 40,000 – Nil [Assumed that Miscellaneous expenditure = Nil] ∴ Share Capital = 2,40,000 – 40,000 = Rs. 2,00,000

Modified Date: Tue, Jul 06, 2010 11:23:17 AM

Output Date: Tue, Jul 06, 2010 11:24:06 AM

Rev II

Project: Management Accounting_Debarshi Bhattacharyya ACE Pro India Pvt. Ltd. File: X:\Pearson\Management Accounting_Debarshi Bhattacharyya\MAIN\M03\LAYOUT_M03\M03_DEBA_ISBN_EN_SE_C03_Part-1.indd

98

MANAGEMENT ACCOUNTING

Working Notes 1. Calculation of CGS GP ratio =

Gross Profit × 100 = 20% Sales

∴ GP rate = 20% or = 1÷ 5 on Sales i.e., If Sales = 5, then GP = 1 CGS = 4 ∴ Rate of GP on CGS Gross Profit 1 = = Cost of Goods Sold 4 1,20,000 1 or = Cost of Goods Sold 4

∴ CGS = Rs. 4,80,000 2. Calculation of Credit Sales Gross Profit GP ratio = × 100 Sales or 20 =

1,20,000 × 100 Sales

1,20,000 × 100 = Rs. 6,00,000 20 In the absence of adequate information, it is assumed that the entire sales have been made on credit. ∴ Total Sales = Credit Sales = Rs. 6,00,000 ∴ Sales =

3. Calculation of Credit Purchases We know that Opening Stock + Purchases = Materials Consumed + Closing Stock ∴ Purchases = Materials Consumed + Closing Stock − Opening Stock In the absence of adequate information regarding seggragation of the CGS, it is assumed that the CGS includes materials consumed only. ∴ Materials Consumed = CGS = Rs. 4,80,000 Again, Closing Stock is greater than the Opening Stock by Rs. 10,000 ∴ Closing Stock − Opening Stock = Rs. 10,000 ∴ Purchases = 4,80,0000 + 10,000 = Rs. 4,90,000 In the absence of adequate information, it is assumed that the entire purchases have been made on credit. ∴ Total Purchases = Credit Purchases = Rs. 4,90,000

Problem 13 From the following information, prepare a Balance Sheet: Current Ratio Quick Ratio GP Ratio Stock Turnover Ratio Debtor's Turnover Turnover of Fixed Assets

1.5 1.25 25% 9 times 1.5 months 1.2 (Continued)

Modified Date: Tue, Jul 06, 2010 11:23:17 AM

Output Date: Tue, Jul 06, 2010 11:24:06 AM

Rev II

Project: Management Accounting_Debarshi Bhattacharyya ACE Pro India Pvt. Ltd. File: X:\Pearson\Management Accounting_Debarshi Bhattacharyya\MAIN\M03\LAYOUT_M03\M03_DEBA_ISBN_EN_SE_C03_Part-1.indd

99

ACCOUNTING RATIOS FOR FINANCIAL STATEMENT ANALYSIS

Capital Gearing Ratio Reserves to Share Capital Fixed Asset to Net Worth Sales

0.375 0.25 1.25 Rs. 24,00,000

[C.S. (Inter)—Adapted] Solution Balance Sheet of

as on Amount Rs. 12,80,000 3,20,000 6,00,000

Liabilities Share Capital2 Reserves & Surplus2 Long-term Loans3 Current Liabilities: Crediltors Bank Overdraft

3,20,0007 80,0007 26,00,000

Assets Fixed Assets1 Current Assets: Stock5 Debtors6 Cash & Bank

Amount Rs. 20,00,000 2,00,000 3,00,000 1,00,000 26,00,000

Working Notes 1. Turnover of Fixed Assets = 1.2 Turnover or = 1.2 Fixed Assets 24,00,000 or = 1.2 Fixed Assets 24,00,000 ∴ Fixed Assets = = Rs. 20,00,000 1.2 2. Fixed Assets to net worth = 1.25 or

Fixed Assets = 1.25 Net Worth

or

20,00,000 = 1.25 Net Worth

∴ Net Worth =

20,00,000 1.25

= Rs. 16, 00, 000

Again, Reserves to Share Capital = 0.25 Reserves = 0.25 Share Capital ∴ Reserves = 0.25 × Share Capital Again, Net Worth = Share Capital + Reserves & Surplus – Miscellaneous Expenditure Here, 16,00,000 = Share Capital + 0.25 × Share Capital – Nil [assumed as Miscellaneous Expenditure = Nil] or 1.25 × Share Capital = 16,00,000 16,00,000 ∴ Share Capital = = Rs.12,80,000 1.25 or

∴ Reserves & Surplus = 0.25 × Rs. 12,80,000 = Rs. 3,20,000 (Continued)

Modified Date: Tue, Jul 06, 2010 11:23:17 AM

Output Date: Tue, Jul 06, 2010 11:24:06 AM

Rev II

Project: Management Accounting_Debarshi Bhattacharyya ACE Pro India Pvt. Ltd. File: X:\Pearson\Management Accounting_Debarshi Bhattacharyya\MAIN\M03\LAYOUT_M03\M03_DEBA_ISBN_EN_SE_C03_Part-1.indd

100

MANAGEMENT ACCOUNTING

3. Capital Gearing Ratio =

Fixed Interest Bearing Securities Ordinary Securities

Long-term Loan + Preference Share Capital Equity Shareholders ’ Fund In the absence of adequate information, it is assumed that there is no Preference Share Capital. In such a condition, Equity Shareholders’ Fund = Net Worth = Rs. 16,00,000 or 0.375 =



Long-term Loans + Nil = 0.375 16,00,000

∴ Long-term loan = 0.375 × 16,00,000 = Rs. 6,00,000 4. In a company Balance Sheet, we know that Net Worth + Long-term Loans = Fixed Assets + Working Capital Here, 16,00,000 + 6,00,000 = 20,00,000 + Working Capital ∴ Working Capital = Rs. 2,00,000 CA Again, Current Ratio = CL CA or 1.5 = CL ∴ CA = 1.5 CL. Again, Working Capital = CAs − CLs Here, 2,00,000 = 1.5 CLs − CLs = 0.5 CL ∴ Current Liabilities (CLs) = 2,00,000/0.5 = Rs. 4,00,000

∴ Current Assets (CAs) = 1.5×4,00,000 = Rs. 6,00,000 Gross Profit × 100 Sales Gross Profit Here, 25 = × 100 24,00,000 24,00,000 × 25 ∴ GP = = Rs. 6,00,000 100 Again, CGS = Sales – GP Here, CGS = 24,00,000 – 6,00,000 = Rs. 18,00,000 Now, Stock Turnover Ratio = 9 Cost of Goods Sold or =9 Average Stock 18,00,000 or =9 Average Stock

5. GP Ratio =

18,00,000 = Rs. 2,00,000 9 Opening Stock + Closing Stock Again, Average Stock = 2 In the absence of adequate information as regards to the Opening Stock, it is assumed that Opening Stock = Closing Stock. Closing Stock + Closing Stock 2 Closing Stocks ∴ Average Stock = = 2 2 = Closing Stock = Rs. 2,00,000 ∴ Average Stock =

Modified Date: Tue, Jul 06, 2010 11:23:17 AM

Output Date: Tue, Jul 06, 2010 11:24:06 AM

Rev II

Project: Management Accounting_Debarshi Bhattacharyya ACE Pro India Pvt. Ltd. File: X:\Pearson\Management Accounting_Debarshi Bhattacharyya\MAIN\M03\LAYOUT_M03\M03_DEBA_ISBN_EN_SE_C03_Part-1.indd

ACCOUNTING RATIOS FOR FINANCIAL STATEMENT ANALYSIS

101

6. Debtors’ Turnover = 1.5 months 12 months or Debtors’ Turnover Ratio = =8 1.5 months or Credit Sales = 8 Receivables 24,00,000 = 8 [Assumed that entire sales have been made on credit] Receivables 24,00,000 ∴ Receivables (i.e., Debtors) = = Rs. 3,00,000 8 Again, Current Assets = Stock + Debtors + Cash & Bank Here, 6,00,0004 = 2,00,0005 + 3,00,0006 + Cash & Bank ∴ Cash & Bank = 6,00,000 − 5,00,000 = Rs. 1,00,000 7. Quick Ratio = CA − Stock CL − Bank O/D 6,00,000 − 2,00,000 Here, 1.25 = 4,00,000 − Bank O/D or 4,00,000 = 5,00,000 – 1.25 Bank Overdraft or 1.25 Bank Overdraft = 5,00,000 – 4,00,000 1,00,000 or Bank Overdraft = = Rs. 80,000 1.25 Again, Current Liabilities = Payables (i.e., Creditors) + Bank overdraft Here, 4,00,0004 = Creditors + 80,000 ∴ Creditors = 4,00,000 – 80,000 = Rs. 3,20,000 or,

Problem 14 From the following Financial Data, make out a statement of Proprietors’ Fund with as many details as possible: (i) Proprietary Ratio [FA to Proprietors’ Equity] (ii) Current Ratio (iii) Liquid Ratio (iv) Capital Gearing [Equity Capital to Preference Capital] (v) Reserves & Surplus to Equity Capital (vi) Working Capital (vii) Bank Overdraft

0.75 2.5 1.5 2:1 0.30 Rs. 90,000 Rs. 20,000

There is no Long-term Loan or Fictitious Assets. [B.Com. (Hons), Calcutta University—2007] Solution Statement of Proprietors’ Fund Rs. Sources of Fund: Equity Share Capital Preference Share Capital Add:

Reserves & Surplus

Less:

Miscellaneous Expenditure

Rs.

Rs. 2,00,000 1,00,000 3,00,000 60,000 3,60,000 Nil 3,60,000 (Continued)

Modified Date: Tue, Jul 06, 2010 11:23:17 AM

Output Date: Tue, Jul 06, 2010 11:24:06 AM

Rev II

Project: Management Accounting_Debarshi Bhattacharyya ACE Pro India Pvt. Ltd. File: X:\Pearson\Management Accounting_Debarshi Bhattacharyya\MAIN\M03\LAYOUT_M03\M03_DEBA_ISBN_EN_SE_C03_Part-1.indd

102

MANAGEMENT ACCOUNTING

Rs.

Add:

Less:

Applications of Fund: Fixed Assets Working Capital: Current Assets: Stock Liquid Assets

Rs.

Rs. 2,70,000

90,000 60,000 1,50,000

Current Liabilities: Bank Overdraft Liquid Liabilities

20,000 40,000 60,000

Less:

90,000 3,60,000 Nil 3,60,000

Long-term Loan

Working Notes Current Assets 5 = 2.5 = Current Liabilities 2 ∴ Working Capital = CAs − CLs = 5 − 2 = 3 Again, given Working Capital = Rs. 90,000 ∴ Current Liabilities (CLs) = Rs. 90,000 × 2 ÷ 3 = Rs. 60,000 ∴ Current Assets (CAs) = Rs. 90,000 × 5 ÷ 3 = Rs. 1,50,000 Liquid Assets CA − Stock Again, Liquid Ratio = = = 1.5 Liquid Liabilities CL − Bank Overdraft 1,50,000 − Stock = 1.5 or 60,000 − 20,000 or 60,000 = 1,50,000 − Stock ∴ Stock = Rs. 90,000 ∴ Liquid Assets = CAs − Stock = 1,50,000 − 90,000 = Rs. 60,000 ∴Liquid Liabilities = CLs − Bank O/D = 60,000 − 20,000 = Rs. 40,000 2. Again, as per the company Balance Sheet, we know, Proprietors’ Fund + Long-term Loan = Fixed Asset + Working Capital Here, Proprietors Fund (PF) + Nil = Fixed Asset (FA) + 90,000 ∴PF = FAs + 90,000 Fixed Assets Again, given Proprietary Ratio = = 0.75 Proprietor’s Fund FA = 0.75 or FA + 90,000 or FAs = 0.75 FAs + 67,500 or 0.25 FAs = 67,500 ∴Fixed Asset = Rs. 2,70,000 ∴Proprietors’ Fund = FAs + 90,000 = 2,70,000 + 90,000 = Rs. 3,60,000 Equity Capital 2 = 3. Capital Gearing = Preference Capital 1 ∴ Equity Share Capital : Preference Share Capital = 2 : 1 = 10 : 5 Again, Reserves & Surplus : Equity Share Capital = 0.30 = 3:10 ∴Equity Share Capital: Preference Share Capital: Reserves & Surplus = 10 : 5 : 3 1.

Current Ratio =

(Continued)

Modified Date: Tue, Jul 06, 2010 11:23:17 AM

Output Date: Tue, Jul 06, 2010 11:24:06 AM

Rev II

Project: Management Accounting_Debarshi Bhattacharyya ACE Pro India Pvt. Ltd. File: X:\Pearson\Management Accounting_Debarshi Bhattacharyya\MAIN\M03\LAYOUT_M03\M03_DEBA_ISBN_EN_SE_C03_Part-1.indd

ACCOUNTING RATIOS FOR FINANCIAL STATEMENT ANALYSIS

103

Again, Proprietors’ Fund = Equity Share Capital + Preference Share Capital + Reserves & Surplus = 10 + 5 + 3 = 18 Again, Proprietors’ Fund = Rs. 3,60,000 ∴Equity Share Capital = 10/18 × Rs. 3,60,000 = Rs. 2,00,000 ∴Preference Share Capital = 5/18 × Rs. 3,60,000 = Rs. 1,00,000 ∴Reserves & Surplus = 3/18 × Rs. 3,60,000 = Rs. 60,000

Problem 15 From the following information of X Engineering Co., complete the proforma Balance Sheet, if its sales are Rs. 16,00,000: Sales to Net Worth Current Liabilities to Net Worth Total Liabilities to Net Worth Current Ratio Sales to Closing Inventory Average Collection Period

2.3 times 42% 75% 2.9 times 4.5 times 64 days

Proforma Balance Sheet Liabilities Net Worth Long-term Liabilities Current Liabilities

Rs. ? ? ?

Assets Fixed Assets Stock Debtors Cash

Rs. ? ? ? ?

[C.A. (Inter)—Adapted] Solution Balance Sheet of X Engineering Co. as on Amount Rs. 6,40,000 2,11,200 2,68,800

Liabilities Net Worth1 Long-term Liabilities3 Current Liabilities2

Assets Fixed Assets7 Current Assets: Stock4 Debtors5 Cash & Bank6

11,20,000

Amount Rs. 3,40,480 3,55,555 3,20,000 1,03,965 11,20,000

Working Notes 1.

Given Sales = Rs. 16,00,000 Again, Sales to Net worth (NW) = 2.5 Sales or = 2.5 NW 16,00,000 or = 2.5 NW ∴Net Worth = Rs. 6,40,000 (Continued)

Modified Date: Tue, Jul 06, 2010 11:23:17 AM

Output Date: Tue, Jul 06, 2010 11:24:06 AM

Rev II

Project: Management Accounting_Debarshi Bhattacharyya ACE Pro India Pvt. Ltd. File: X:\Pearson\Management Accounting_Debarshi Bhattacharyya\MAIN\M03\LAYOUT_M03\M03_DEBA_ISBN_EN_SE_C03_Part-1.indd

104

MANAGEMENT ACCOUNTING

Current Liabilities to Net Worth = 42% CL or = 42% NW CL or = 42% 6, 40,000 ∴ Current Liabilities = Rs. 2,68,800 3. Total Liabilities (TL) to Net Worth (NW) = 75% TL or = 75% NW TL = 75% or 6, 40,000 ∴ Total Liabilities = Rs. 4,80,000 Again, Total Liabilities = Long-term Liabilities + Current Liabilities Here, 4,80,000 = Long-term Liabilities + 2,68,800 ∴ Long-term Liabilities = Rs. 2,11,200 4. Sales to Closing Inventory = 4.5 times Sales = 4.5 or Closing Inventory 16,00,000 = 4.5 or Closing Inventory ∴ Closing Inventory = Rs. 3,55,555 5. Average Collection Period = 73 days ∴ Debtors’ Turnover Ratio = 365 days = 5 73 days Credit Sales =5 or Receivables

2.

16,00,000 = 5 [ assumed that entire sales were made on credit ] Receivables 16,00,000 ∴Receivables = = Rs. 3,20,000 5 Again, Receivables = Debtors + Bills Receivable Here, 3,20,000 = Debtors + Nil ∴ Debtors = Rs. 3,20,000 Current Assets ( CAs) 6. Current Ratio = = 2.9 Current Liabilities ( CLs) CAs = 2.9 or 2,68,800 ∴Current Assets = 2,68,800 × 2.9 = Rs. 7,79,520 Here, CAs = Stock + Debtors + Cash or 7,79,520 = 3,55,555 + 3,20,000 + Cash ∴ Cash = Rs. 1,03,965 7. In every Balance Sheet, Total Assets = Total Liabilities Here, Total Liabilities = Net Worth + Long-term Liabilities + Current Liabilities or TL = 6,40,0001 + 2,11,2003 + 2,68,8002 ∴ Total Assets = Rs. 11,20,000 Again, Total Assets = Fixed Assets + Current Assets or

(Continued)

Modified Date: Tue, Jul 06, 2010 11:23:17 AM

Output Date: Tue, Jul 06, 2010 11:24:06 AM

Rev II

Project: Management Accounting_Debarshi Bhattacharyya ACE Pro India Pvt. Ltd. File: X:\Pearson\Management Accounting_Debarshi Bhattacharyya\MAIN\M03\LAYOUT_M03\M03_DEBA_ISBN_EN_SE_C03_Part-1.indd

ACCOUNTING RATIOS FOR FINANCIAL STATEMENT ANALYSIS

105

Here, 11,20,000 = FAs + 7,79,5206 ∴ Fixed Assets = Rs. 3,40,480

Problem 16 Assume that firm has an Owners’ Equity of Rs. 1,00,000. The Ratio for the firm are: Short-term Debts to Total Debts Total Debt to Owners’ Equity Fixed Assets to Owners’ Equity Total Asset Turnover Inventory Turnover

0.40 0.60 0.60 2 Times 8 Times

Complete the following Balance Sheet from the information given above: Capital & Liabilities Short-term Debt Long-term Debt Total Debt Owners’ Equity Total Capital and Liabilities

Rs. – – – – –

Assets Cash Inventory Total Current Assets Fixed Assets Total Assets

Rs. – – – – –

[C.A. (Inter)—Adapted] Solution Balance Sheet of Capital & Liabilities Short-term Debt2 Long-term Debt2 Total Debt Owners’ Equity Total Capital and Liabilities

as on Rs. 24,000 36,000 60,000 1,00,000 1,60,000

Assets Cash4 Inventory3 Total Current Assets Fixed Assets1 Total Assets

Rs. 60,000 40,000 1,00,000 60,000 1,60,000

Working Notes 1. Fixed Assets (FA) to Owners’ Equity (OE) = 0.60 FAs or = 0.60 OE FAs = 0.60 or 1,00,000 ∴ Fixed Assets = Rs. 60,000 2. Total Debt (TD) to Owners’ Equity (OE) = 0.60 TD or = 0.60 OE TD = 0.60 or 1,00,000 ∴Total Debt = Rs. 60,000 Again, Short-term Debt (STD) to Total Debt (TD) = 0.40 STD or = 0.40 TD

Modified Date: Tue, Jul 06, 2010 11:23:17 AM

Output Date: Tue, Jul 06, 2010 11:24:06 AM

Rev II

Project: Management Accounting_Debarshi Bhattacharyya ACE Pro India Pvt. Ltd. File: X:\Pearson\Management Accounting_Debarshi Bhattacharyya\MAIN\M03\LAYOUT_M03\M03_DEBA_ISBN_EN_SE_C03_Part-1.indd

106

MANAGEMENT ACCOUNTING

STD = 0.40 60,000 ∴ Short-term Debt = Rs. 24,000 ∴ Long-term Debt (LTD) = TD − STD = 60,000 − 24,000 = Rs. 36,000 3. In every Balance Sheet, Total Assets = Total Liabilities Here, Total Liabilities (TL) = STD + LTD + OE or TL = 24,0002 + 36,0002 + 1,00,000 ∴ Total Liabilities = Rs. 1,60,000 ∴ Total Assets = Rs. 1,60,000 Turnover Again, Total Asset Turnover Ratio = =2 Total Assets or

or

Turnover =2 1,60,000

∴ Turnover = Rs. 3,20,000 Again, Inventory turnover =

CGS =8 Average Stock

In the absence of adequate information, in the given problem, as regards to Opening Stock and GP Ratio, it is assumed that, Turnover Inventory Turnover = =8 Closing Stock

or

3,20,000 =8 Closing Stock

∴ Closing Stock =

3,20,000 = Rs. 40,000 8

4. Here, Total Assets = Fixed Assets + Inventory + Cash or 1,60,000 = 60,0001 + 40,0003 + Cash ∴ Cash = Rs. 60,000

Problem 17 Following are the Ratios relating to the trading activities of an organization: Debtors’ Velocity Stock Velocity Creditors’ Velocity GP Ratio Capital Turnover Ratio Fixed Asset Turnover Ratio

3 months 4 months 2 months 25% 3 4

GP for the year that ended on 31 March 2007 was Rs. 7,50,000. Stock as on 31 March 2007 was Rs. 30,000 more than it was on 1 April 2006. At the end of the year, Bills Payable and Bills Receivable were Rs. 45,000 and Rs. 50,000, respectively, and Bank Overdraft was Rs. 1,10,000. Prepare the Statement of Proprietors’ Fund as on 31 March 2007. (Make necessary assumptions that you think fit.) [B.Com. (Hons), Calcutta University—Adapted]

Modified Date: Tue, Jul 06, 2010 11:23:17 AM

Output Date: Tue, Jul 06, 2010 11:24:06 AM

Rev II

Project: Management Accounting_Debarshi Bhattacharyya ACE Pro India Pvt. Ltd. File: X:\Pearson\Management Accounting_Debarshi Bhattacharyya\MAIN\M03\LAYOUT_M03\M03_DEBA_ISBN_EN_SE_C03_Part-1.indd

ACCOUNTING RATIOS FOR FINANCIAL STATEMENT ANALYSIS

107

Solution Statement of Proprietors’ Fund of an Organization as on 31 March 2007 Rs.

Rs.

Sources of Fund: Proprietors’ Fund5

Add:

Less:

Less:

Rs. 10,00,000 10,00,000

Applications of Fund: Fixed Assets1 Working Capital: Current Assets: Stock3 Debtors2 Bills Receivable

7,50,000

7,65,000 7,00,000 50,000 15,15,000

Current Liabilities: Creditors4 Bills Payable Bank Overdraft

3,35,000 45,000 1,10,000

4,90,000 10,25,000 17,25,000 7,75,000 10,00,000

Long-term Loan6

Working Notes 1. GP Ratio = 25% GP or = 25% Sales 7,50,000 = 25% or Sales 7,50,000 ∴ Sales = = Rs. 30,00,000 25% Again, Fixed Asset Turnover Ratio = 4 Sales = 4 Fixed Assets (FA ) 30,00,000 =4 or FAs 30,00,000 ∴Fixed Assets = = Rs. 7,50,000 4 or

2. Debtors’ Velocity = 3 months 12 months ∴ Debtors’ Velocity Ratio = =4 3 months Credit Sales =4 or Receivables 30,00,000 = 4 [ assumed that entire sales were made on credit ] or Receivables ∴ Receivables = 30,00,000 = Rs. 7,50,000 4 (Continued)

Modified Date: Tue, Jul 06, 2010 11:23:17 AM

Output Date: Tue, Jul 06, 2010 11:24:06 AM

Rev II

Project: Management Accounting_Debarshi Bhattacharyya ACE Pro India Pvt. Ltd. File: X:\Pearson\Management Accounting_Debarshi Bhattacharyya\MAIN\M03\LAYOUT_M03\M03_DEBA_ISBN_EN_SE_C03_Part-1.indd

108

MANAGEMENT ACCOUNTING

Again, Receivables = Debtors + Bills Receivable Here, 7,50,000 = Debtors + 50,000 ∴Debtors = Rs. 7,00,000 3. Let the value of Stock as on 01 April 2006 (i.e., Opening Stock) be x. ∴Value of Stock as on 31 March 2007 (i.e., Closing stock) = x + 30,000 ∴ Average Stock = ( Opening Stock + Closing Stock ) = ⎡⎣ x + ( x + 30,000 )⎤⎦ = x + 15,000 2 2 Again, CGS = Sales − GP = 30,00,000 – 7,50,000 = Rs. 22,50,000 Here, Stock Velocity = 4 months ∴ Stock − Velocity Ratio = 12 months = 3 4 months CGS =3 or Average Stock 22,50,000 =3 or x ( + 15,000) or 3x + 45,000 = 22,50,000 or 3x = 22,50,000 − 45,000 ∴ x = 22,05,000 = 7,35,000 3 ∴ Opening Stock = x = Rs. 7,35,000 ∴ Closing Stock = x + 30,000 = Rs. 7,65,000 4. We know, in case of a trading concern, Opening Stock + Purchases = CGS + Closing Stock Here, 7,35,000 + Purchases = 22,50,000 + 7,65,000 ∴ Purchases = Rs. 22,80,000 Again, Creditors’ Velocity = 2 months Credit Purchases 12 months ∴ Creditors’ Velocity Ratio = =6 = 6‚ or Payables 2 months 22,80,000 = 6 [ assumed that the entire purchases were made on credit ] or Payables ∴ Payables = 22,80,000 = Rs. 3,80,000 6 Again, Payables = Creditors + Bills Payable Here, 3,80,000 = Creditors + 45,000 ∴ Creditors = Rs. 3,35,000 Turnover 5. Generally,Capital Turnover Ratio = Capital Employed As in the given problem, no relationship between the Proprietors’ Fund and the Long-term Loan is given, we should assume that (in such a situation only), Capital Turnover Ratio =

Here, 3 = 30,00,000

Turnover Proprietor’s Fund (PF)

PF

∴ Proprietor’s Fund = 30,00,000 = Rs. 10,00,000 3 (Continued)

Modified Date: Tue, Jul 06, 2010 11:27:29 AM

Output Date: Tue, Jul 06, 2010 01:33:40 PM

Rev II

Project: Management Accounting_Debarshi Bhattacharyya ACE Pro India Pvt. Ltd. File: X:\Pearson\Management Accounting_Debarshi Bhattacharyya\MAIN\M03\LAYOUT_M03\M03_DEBA_ISBN_EN_SE_C03_Part-1.indd

ACCOUNTING RATIOS FOR FINANCIAL STATEMENT ANALYSIS

109

6. Working Capital = Current Assets − Current Liabilities Here, Working Capital = (Stock + Debtors + Bills Receivable) − (Creditors + Bills Payable + Bank Overdraft) or Working Capital = (7,65,000 + 7,00,000 + 50,000) − (3,35,000 + 45,000 + 1,10,000) = Rs. 10,25,000 Again, as per company Balance Sheet, we know that Proprietors’ Fund + Long-term Loan = Fixed Asset + Working Capital Here, 10,00,000 + Long-term Loan = 7,50,000 + 10,25,000 ∴ Long-term Loan = Rs. 7,75,000

Problem 18 From the following information relating to Moonlight Ltd, prepare a Balance Sheet as on 31 December 1997. Current Ratio Liquid Ratio Net Working Capital Cost of Sales/Closing Stock GP Ratio Average Debt Collection Period Fixed Assets /Shareholders’ Net Worth Reserves & Surplus/Share Capital

2:5 1:5 Rs. 3,00,000 8 times 25% 1.5 months 0.75 0.50

[B.Com. (Hons), Calcutta University—1998] Solution Balance Sheet of Moonlight Ltd as on 31 December 1997 Liabilities Share Capital5 Reserves & Surplus5 Current Liabilities1

Amount Rs. 8,00,000 4,00,000 2,00,000

Assets Fixed Assets4 Current Assets: Stock1 Debtors2 Cash & Bank3

14,00,000

Amount Rs. 9,00,000 2,00,000 2,66,667 33,333 14,00,000

Working Notes 1. Current Ratio =

Current Assets ( CAs) 5 = 2.5 = Current Liabilities ( CLs) 2

∴Net Working Capital = CAs − CLs = 5 − 2 = 3 Again, given Working Capital = Rs. 3,00,000 ∴ Current Assets = 3,00,000 × 5 ÷ 3 = Rs. 5,00,000 ∴ Current Liabilities = 3,00,000 × 2 ÷ 3 = Rs. 2,00,000 (Continued)

Modified Date: Tue, Jul 06, 2010 11:23:17 AM

Output Date: Tue, Jul 06, 2010 11:24:06 AM

Rev II

Project: Management Accounting_Debarshi Bhattacharyya ACE Pro India Pvt. Ltd. File: X:\Pearson\Management Accounting_Debarshi Bhattacharyya\MAIN\M03\LAYOUT_M03\M03_DEBA_ISBN_EN_SE_C03_Part-1.indd

110

MANAGEMENT ACCOUNTING

Now, Liquid Ratio =

(Current Assets − Stock ) (Current Liabilities − Bank Overdraft )

If it is assumed that Bank Overdraft = Nil, then (Current Assets − Stock ) Liquid Ratio = CL 5,00,000 − Stock ) ( Here, 1.5 = 2,00,000 or 3,00,000 = 5,00,000 − Stock ∴ Stock = Rs. 2,00,000 2.

Cost of Sales =8 Closing Stock Cost of Sales =8 2,00,000 ∴ Cost of Sales = Rs. 16,00,000 Now, GP Ratio = 25% (on Sales) ∴ GP = 1/4 on Sales = 1/3 on Cost of Sales = 1/3 on 16,00,000 = Rs. 5,33,333

or

∴ Sales = Cost of Sales + GP = 16,00,000 + 5,33,333 = Rs. 21,33,333 Again, Average Collection Period = 1.5 months 12 months ∴ Debtors’ Velocity Ratio = =8 1.5 months Credit Sales =8 or Receivables 21,33,333 = 8 [ assumed that the entire sales were made on credit ] or Receivables ∴ Receivables = 21,33,333 = Rs. 2,66,667 8 As there is no Bills Receivable, then here Receivables = Debtors = Rs. 2,66,667 3. Normally, Current Assets = Stock + Debtors + Cash Here, 5,00,000 = 2,00,000 + 2,66,667 + Cash ∴ Cash = Rs. 33,333. 4. Fixed Assets (FA) to Net Worth (NW) = 0.75 FAs or = 0.75 NW ∴FA = 0.75 NW Again, as per company Balance Sheet, we know that Net Worth (NW) + Long-term Loan (LTL) = Fixed Asset (FA) + Working Capital (WC) Here, NW + Nil = 0.75 NW + 3,00,000 [assumed that there is no Long-term Loan] or NW − 0.75 NW = 3,00,000 or NW = 3,00,000 ÷ 0.25 ∴ Net worth = Rs. 12,00,000 ∴Fixed Assets = 0.75 × 12,00,000 = Rs. 9,00,000 5. Reserves & Surplus (RS) to Share Capital (Sh. Cap.) = 0.50 RS = 0.50 or Sh.Cap. ∴ RS = 0.50 Sh. Cap. Again, Net Worth = Share Capital + Reserves & Surplus − Miscellaneous Expenditure (Continued)

Modified Date: Tue, Jul 06, 2010 11:23:17 AM

Output Date: Tue, Jul 06, 2010 11:24:06 AM

Rev II

Project: Management Accounting_Debarshi Bhattacharyya ACE Pro India Pvt. Ltd. File: X:\Pearson\Management Accounting_Debarshi Bhattacharyya\MAIN\M03\LAYOUT_M03\M03_DEBA_ISBN_EN_SE_C03_Part-1.indd

ACCOUNTING RATIOS FOR FINANCIAL STATEMENT ANALYSIS

111

Here, 12,00,000 = Sh. Cap. + 0.5 Sh. Cap. − Nil [assumed that Miscellaneous expenditure = Nil] or 1.5 Sh. Cap. = 12,00,000 ∴ Share Capital = 12,00,000÷1.5 = Rs. 8,00,000 ∴ Reserves & Surplus = 0.5 × 8,00,000 = Rs. 4

Problem 19 From the following data, complete the following Balance Sheet: GP Shareholders’ Fund GP Margin Credit Sales to Total Sales Total Asset Turnover Inventory Turnover Average Collection Period [a 360-day year] Current Ratio Long-term Debt to Equity

Rs. 54,000 Rs. 6,00,000 20% 80% 0.3 times 4 times 20 days 1.8 40%

Balance Sheet Liabilities Creditors Long-term Debt Shareholders’ Fund

Rs. – – –

Assets Cash Debtors Inventory Fixed Assets



Rs. – – – – –

[C.A. (PE II)—November 2005]

Solution Balance Sheet as on Liabilities

Rs. 60,000 2,40,000 6,00,000

Creditors Long-term Debt Shareholders’ Fund

Assets Cash Debtors Inventory Fixed Assets

9,00,000

Rs. 42,000 12,000 54,000 7,92,000 9,00,000

Working Notes 1. GP = Rs. 54,000 Again, GP Margin =

GP × 100 = 20 Sales

54,000 × 100 = 20 Sales ∴ Sales = 54,000 × 100 ÷ 20 = Rs. 2,70,000 Again, Credit Sales to Total Sales = 80% = 80 ÷ 100 = 4/5 Credit Sales 4 = or Total Sales 5 Credit Sales 4 = or 2,70,000 5 or

(Continued)

Modified Date: Tue, Jul 06, 2010 11:23:17 AM

Output Date: Tue, Jul 06, 2010 11:24:06 AM

Rev II

Project: Management Accounting_Debarshi Bhattacharyya ACE Pro India Pvt. Ltd. File: X:\Pearson\Management Accounting_Debarshi Bhattacharyya\MAIN\M03\LAYOUT_M03\M03_DEBA_ISBN_EN_SE_C03_Part-1.indd

112

MANAGEMENT ACCOUNTING

∴Credit Sales = 2,70,000 × 4 ÷ 5 = Rs. 2,16,000 Again, Average Collection Period = 20 days ∴ Debtors’ Velocity Ratio = or

Credit sales = 18 Receivables

or

2,16,000 = 18 Receivables

360 days = 18 20 days

2,16,000 = Rs. 12,000 18 As there is no bills receivable, then here Receivables = Debtors = Rs. 12,000 2. CGS = Sales − GP Here, CGS = 2,70,000 – 54,000 = Rs. 2,16,000 CGS Again, Inventory Turnover Ratio = Average Stock 2,16,000 Here, 4 = Average Stock ∴ Receivables =

2,16,000 = Rs. 54,000 4 In the absence of adequate information, in the given problem, as regards to Opening stock, it is assumed that Opening stock = Closing stock Then, Closing Stock = Average Stock = Rs. 54,000 3. Given, Shareholders’ Fund (i.e., here, Equity) = Rs. 6,00,000 Again, Long-term Debt (LTD) to Equity = 40% ∴ Average Stock =

or

LTD = 40% 6,00,000

∴Long-term Debt = 40% of 6,00,000 = Rs. 2,40,000 4. Total Asset Turnover = 0.3 or

Turnover = 0.3 Total Assets

or

2,70,0001 = 0.3 Total Assets

2,70,000 = Rs. 9,00,000 0.3 Again, in every Balance Sheet, Total Assets = Total Liabilities ∴Total Liabilities = Rs. 9,00,000 Here, Total Liabilities = Creditors + Long-term Loan + Shareholders’ Fund or 9,00,000 = Creditors + 2,40,0003 + 6,00,000 ∴Creditors = Rs. 60,000 Current Assets (CAs) CAs 5. Current Ratio = = 1.8 or = 1.8 Current Liabilities (CLs) 60,000 ∴ Total Assets =

∴Current Assets = 60,000 × 1.8 = Rs. 1,08,000 (Continued)

Modified Date: Tue, Jul 06, 2010 11:23:17 AM

Output Date: Tue, Jul 06, 2010 11:24:06 AM

Rev II

Project: Management Accounting_Debarshi Bhattacharyya ACE Pro India Pvt. Ltd. File: X:\Pearson\Management Accounting_Debarshi Bhattacharyya\MAIN\M03\LAYOUT_M03\M03_DEBA_ISBN_EN_SE_C03_Part-1.indd

ACCOUNTING RATIOS FOR FINANCIAL STATEMENT ANALYSIS

113

Here, Current Assets = Inventory + Debtors + Cash or 1,08,000 = 54,0002 + 12,0001 + Cash ∴Cash = Rs. 42,000 6. Here, Total Assets = Fixed Assets + Inventory + Debtors + Cash or 9,00,0004 = Fixed Assets + 54,0002 + 12,0001 + 42,0005 ∴Fixed Assets = Rs. 7,92,000

Problem 20 Prepare a Profit & Loss A/c and a Balance Sheet (with as many details as possible) from the following information for the year that ended on 31 December 2006: GP Ratio NP Ratio Stock Turnover NP to Proprietary Fund Total Outside Liability to Proprietary Fund Total Assets Closing Stock

25% 15% 10 times 1:1 2:1 Rs. 12,00,000 Rs. 1,00,000

[B.B.A. (Hons), Calcutta University—2007] Solution Profit & Loss A/c of

for the year that ended on 31 December 2006

Dr.

Cr. Particulars

To Opening Stock3 To Purchases (Bal. fig.) To GP c/d2 To Other Operating Expenses (Bal. fig.) To NP1

Amount Rs. 3,00,000 18,00,000 6,66,667 27,66,667 2,66,667 4,00,000 6,66,667

Balance Sheet of Liabilities Proprietors’ Fund1 Total Outside Liabilities1

By Sales2 By Closing Stock

Amount Rs. 26,66,667 1,00,000

By GP b/d

27,66,667 6,66,667

Particulars

6,66,667

as on 31 December 2006 Amount Rs. 4,00,000 8,00,000 12,00,000

Assets Total Assets

Amount Rs. 12,00,000 12,00,000

Working Notes 1. Total Assets = Total Liabilities = Rs. 12,00,000 Again, Total Liabilities = Total Outside Liabilities + Proprietors’ Fund Total Outside Liabilities 2 Here, = Proprietors’ Fund 1 (Continued)

Modified Date: Tue, Jul 06, 2010 11:23:17 AM

Output Date: Tue, Jul 06, 2010 11:24:06 AM

Rev II

Project: Management Accounting_Debarshi Bhattacharyya ACE Pro India Pvt. Ltd. File: X:\Pearson\Management Accounting_Debarshi Bhattacharyya\MAIN\M03\LAYOUT_M03\M03_DEBA_ISBN_EN_SE_C03_Part-1.indd

114

MANAGEMENT ACCOUNTING

∴ Total Liabilities = 2 + 1 = 3 ∴ Total Outside Liabilities =

2 × 12,00,000 = Rs. 8,00,000 3

1 × 12,00,000 = Rs. 4,00,000 3 1 Again, Net Profit to Proprietor’s Fund = 1 ∴Proprietors’ Fund =

or

NP 1 = 4,00,000 1

∴NP = Rs. 4,00,000 2. NP Ratio = NP = 15% Sales or

4,00,000 = 15% Sales

∴ Sales =

4,00,000 15%

Again, GP Ratio = or

= Rs. 26,66,667

GP = 25% Sales

GP = 25% 26,66,667

∴ GP = 26,66,667 × 25% = Rs. 6,66,667 3. CGS = Sales − GP = 26,66,667 − 6,66,667 = Rs. 20,00,000

∴ Stock Turnover Ratio = or

20,00,000 = 10 Average Stock

∴ Average Stock =

20,00,000 = Rs. 2,00,000 10

Again, Average Stock = or or

CGS = 10 Average Stock

2,00,000 =

(Opening Stock + Closing Stock ) 2

(Opening Stock + 1,00,000)

2 2,00,000 = (Opening Stock + 1,00,000) = 4,00,000

∴ Opening Stock = 3,00,000

Modified Date: Tue, Jul 06, 2010 11:23:17 AM

Output Date: Tue, Jul 06, 2010 11:24:06 AM

Rev II

Project: Management Accounting_Debarshi Bhattacharyya ACE Pro India Pvt. Ltd. File: X:\Pearson\Management Accounting_Debarshi Bhattacharyya\MAIN\M03\LAYOUT_M03\M03_DEBA_ISBN_EN_SE_C03_Part-2.indd

ACCOUNTING RATIOS FOR FINANCIAL STATEMENT ANALYSIS

115

Problem 21 From the following Ratios and further information given below, prepare a trading and Profit & Loss A/c and a Balance Sheet of Mr Green: Fixed Assets/Capital Fixed Assets Capital/Liabilities NP/Capital GP Ratio Stock Turnover Ratio Fixed Assets/Total Current Assets NP to Sales Closing Stock

5/4 Rs. 5,00,000 1/2 1/5 25% 10 5/7 20% Rs. 50,000

Out of the Current Assets, Sundry Debtors are Rs. 6,00,000, and the balance represents Cash and Closing Stock. [C.A. (Inter)—May 1992]

Solution Books of Mr Green Trading and Profit & Loss A/c for the year that ended on Dr.

Cr. Particulars

To Opening Stock6 To Purchases7 To Gross Profit c/d5 To Other Operating Expenses8 To Net Profit3

Amount Rs. 10,000 3,40,000 1,00,000 4,50,000 20,000 80,000 1,00,000

Particulars

Amount Rs.

By Sales4 By Closing Stock

4,00,000 50,000

By Gross Profit b/d

4,50,000 1,00,000 1,00,000

Balance Sheet as on Liabilities Capital1 Total Outside Liabilities2

Amount Rs. 4,00,000 8,00,000

Assets Fixed Assets Current Assets: Stock Debtors Cash9

12,00,000

Modified Date: Sat, Jul 03, 2010 12:42:12 PM

Output Date: Tue, Jul 06, 2010 11:40:08 AM

Amount Rs. 5,00,000 50,000 6,00,000 50,000 12,00,000

Rev II

Project: Management Accounting_Debarshi Bhattacharyya ACE Pro India Pvt. Ltd. File: X:\Pearson\Management Accounting_Debarshi Bhattacharyya\MAIN\M03\LAYOUT_M03\M03_DEBA_ISBN_EN_SE_C03_Part-2.indd

116

MANAGEMENT ACCOUNTING

Working Notes 1.

Fixed Assets 5 = Capital 4 5,00,000 5 = or Capital 4 Capital = Rs. 4,00,000

2.

Capital 1 = Liabilities 2 or

4,00,000 1 = Liabilities 2

∴ Total Outside Liabilities = Rs. 8,00,000 NP 1 3. = Capital 5 or

NP 1 = 4,00,000 5

∴ NP = Rs. 80,000 4.

NP = 20% Sales 80,000 = 20% or Sales ∴ Sales = Rs. 4,00,000

5. GP ratio = or

GP = 25% Sales

GP = 25% 4,00,000

∴ GP = 4,00,000 × 25% = Rs. 1,00,000 6. In a trading concern, CGS = Sales − GP Here, CGS = 4,00,000 − 1,00,000 = Rs. 3,00,000 Again, Stock Turnover Ratio = or

3,00,000 = 10 Average stock

∴ Average Stock =

3,00,000 = Rs. 30,000 10

Again, Average Stock = or

CGS = 10 Average stock

30,000 =

(Opening Stock + Closing Stock) 2

(Opening Stock + 50,000) 2 (Continued)

Modified Date: Sat, Jul 03, 2010 12:42:12 PM

Output Date: Tue, Jul 06, 2010 11:40:08 AM

Rev II

Project: Management Accounting_Debarshi Bhattacharyya ACE Pro India Pvt. Ltd. File: X:\Pearson\Management Accounting_Debarshi Bhattacharyya\MAIN\M03\LAYOUT_M03\M03_DEBA_ISBN_EN_SE_C03_Part-2.indd

ACCOUNTING RATIOS FOR FINANCIAL STATEMENT ANALYSIS

117

or Opening Stock + 50,000 = 60,000 ∴Opening Stock = Rs. 10,000 7. In a trading concern, Opening Stock + Purchases = CGS + Closing stock Here, 10,000 + Purchases = 3,00,000 + 50,000 ∴ Purchases = Rs. 3,40,000 8. Other operating expenses debited to Profit & Loss A/c = GP − NP = 1,00,000 − 80,000 = Rs. 20,000 9.

Fixed Assets 5 = Total Current Assets 7 5,00,000 5 = Total Current Assets 7 ∴ Total Current Assets = 5,00,000 × 7 ÷ 5 = Rs. 7,00,000 Again, Total Current Assets = Stock + Debtors + Cash Here, 7,00,000 = 50,000 + 6,00,000 + Cash ∴ Cash = Rs. 50,000 or

Problem 22 From the following information, prepare the trading and Profit & Loss A/c for the year that ended on 31 March 2008 and a Balance Sheet as on that date of Mr Teem: Gross Profit Ratio Net Profit Ratio Net Profit to Capital Capital to Outside Liabilities Long Term Debt to Short-term Debt Fixed Assets to Capital Fixed Assets to Current Assets Stock Turnover Ratio Closing Stock Fixed Assets Debtors

40% 25% 1:5 3:2 1:1 2:3 2:3 1:5 Rs. 6,00,000 Rs. 10,00,000 Rs. 7,00,000

Solution Books of Mr Teem Trading and Profit & Loss A/c for the year that ended on 31 March 2008 Dr.

Cr. Particulars 8

To Opening Stock To Purchases (Bal. fig.) To Gross Profit c/d6 To Other Operating Expenses7 (Bal. fig.) To Net Profit4

Modified Date: Sat, Jul 03, 2010 12:42:12 PM

Amount Rs. 3,60,000 9,60,000 4,80,000 18,00,000 1,80,000 3,00,000 4,80,000

By Sales By Closing Stock

Amount Rs. 12,00,000 6,00,000

By Gross Profit b/d

18,00,000 4,80,000

Particulars 5

Output Date: Tue, Jul 06, 2010 11:40:08 AM

4,80,000

Rev II

Project: Management Accounting_Debarshi Bhattacharyya ACE Pro India Pvt. Ltd. File: X:\Pearson\Management Accounting_Debarshi Bhattacharyya\MAIN\M03\LAYOUT_M03\M03_DEBA_ISBN_EN_SE_C03_Part-2.indd

118

MANAGEMENT ACCOUNTING

Balance Sheet as on 31 March 2008 Amount Rs.

Liabilities Capital2 Long-term Debt3 Short-term Debt3

15,00,000 5,00,000 5,00,000

Assets Fixed Assets Current Assets: Stock Debtors Cash & Bank1

25,00,000

Amount Rs. 10,00,000 6,00,000 7,00,000 2,00,000 25,00,000

Working Notes 1. Fixed Assets to Current Assets = 2/3 FA 2 or = CA 3 or

10,00,000 2 = CA 3

or

2 CAs = 30,00,000

30,00,000 = Rs. 15,00,000 2 Again, Current Assets = Stock + Debtors + Cash & Bank ∴ Current Assets (CAs) =

Here, 15,00,000 = 6,00,000 +7,00,000 + Cash & Bank ∴Cash & Bank = 15,00,000 – 13,00,000 = Rs. 2,00,000 2. Fixed Assets to Capital = 2/3 or

FA 2 = Capital 3

or

10,00,000 2 = Capital 3

or

2 Capital = 30,00,000

∴ Capital =

30,00,000 = Rs. 15,00,000 2

3. Capital to Outside Liabilities = 3/2 or

Capital 3 = Outside Liabilities 2

or

15,00,000 3 = Outside Liabilities 2

2 × 15,00,000 = Rs. 10,00,000 3 Again, Long-term Debt to Short-term Debt = 1:1

or

Outside Liabilities =

(Continued)

Modified Date: Sat, Jul 03, 2010 12:42:12 PM

Output Date: Tue, Jul 06, 2010 11:40:08 AM

Rev II

Project: Management Accounting_Debarshi Bhattacharyya ACE Pro India Pvt. Ltd. File: X:\Pearson\Management Accounting_Debarshi Bhattacharyya\MAIN\M03\LAYOUT_M03\M03_DEBA_ISBN_EN_SE_C03_Part-2.indd

ACCOUNTING RATIOS FOR FINANCIAL STATEMENT ANALYSIS

or

119

Long-term Debt 1 = Short-term Debt 1

∴Long-term Debt = Short-term Debt Again, Total Outside Liabilities (i.e., Debt) = Long-term Debt + Short-term Debt Here, 10,00,000 = Short-term Debt + Long-term Debt or 2 Short-term debts = 10,00,000 ∴Short-term Debt =

10,00,000 = Rs. 5,00,000 2

∴Long-term Debt = Short-term Debt = Rs. 5,00,000 4. NP to Capital = 1/5 or

Net Profit 1 = Capital 5

or

Net Profit 1 = 15,00,000 5

∴NP =

1 × 15,00,000 = Rs. 3,00,000 5

5. NP Ratio = 25% or

Net Profit = 25% Sales

or

3,00,000 25 1 = = Sales 100 4

∴ Sales = 4 × 3,00,000 = Rs. 12,00,000 6. GP Ratio = 40% Gross Profit × 100 = 40 or Sales Gross Profit 40 2 = = 12,00,000 100 5 12,00,000 × 2 ∴ Gross Profit = = Rs. 4,80,000 5 or

7. Other operating expenses charged to Profit & Loss A/c = GP – NP = Rs. 4,80,000 – Rs. 3,00,000 = Rs. 1,80,000 8. Stock Turnover Ratio = 1.5 or

Cost of Goods Sold = 1.5 Average Stock (Continued)

Modified Date: Sat, Jul 03, 2010 12:42:12 PM

Output Date: Tue, Jul 06, 2010 11:40:08 AM

Rev II

Project: Management Accounting_Debarshi Bhattacharyya ACE Pro India Pvt. Ltd. File: X:\Pearson\Management Accounting_Debarshi Bhattacharyya\MAIN\M03\LAYOUT_M03\M03_DEBA_ISBN_EN_SE_C03_Part-2.indd

120

MANAGEMENT ACCOUNTING

or or

Sales – Gross Profit = 1.5 Average Stock 12,00,000 – 4,80,000 = 1.5 Average Stock

or 1.5 Average Stock = 7,20,000 7,20,000 = Rs. 4,80,000 1.5 Opening Stock + Closing Stock Again, Average Stock = 2 Opening Stock + 6,00,000 Here, 4,80,000 = 2

∴ Average Stock =

or Opening Stock + 6,00,000 = 9,60,000 ∴Opening Stock = 9,60,000 − 6,00,000 = Rs. 3,60,000

Problem 23 From the following Ratios and further information given below, prepare a Balance Sheet as on 31 March 2009: Liquid Ratio Return on Capital employed Fixed Asset Turnover Ratio Closing Stock Owner’s Equity to Fixed Asset Debtors’ Turnover Debt-Equity Ratio

1.2 10% 8:5 12.5% on Sales 8:15 1 month 5:4

For the year that ended on 31 March 2009, the company made a profit of Rs. 1,00,000 after paying an interest of Rs. 1,20,000 on term loan, but before tax. Tax paid for the year was Rs. 40,000. Bank balance stood at Rs. 1,00,000, besides stock and debtors of the concern. [C.A. (Inter)—Adapted]

Solution Books of a concern Balance Sheet as on 31 March 2009 Liabilities Owner’s Equity1 Term Loan1 Current Liabilities: Bank Overdraft5 Liquid Liabilities5

Modified Date: Sat, Jul 03, 2010 12:42:12 PM

Amount Rs. 8,00,000 10,00,000 50,000 2,50,000 21,00,000

Assets Fixed Assets2 Current Assets: Stock3 Debtors4 Bank

Output Date: Tue, Jul 06, 2010 11:40:08 AM

Amount Rs. 15,00,000 3,00,000 2,00,000 1,00,000 21,00,000

Rev II

Project: Management Accounting_Debarshi Bhattacharyya ACE Pro India Pvt. Ltd. File: X:\Pearson\Management Accounting_Debarshi Bhattacharyya\MAIN\M03\LAYOUT_M03\M03_DEBA_ISBN_EN_SE_C03_Part-2.indd

ACCOUNTING RATIOS FOR FINANCIAL STATEMENT ANALYSIS

121

Working Notes 1.

Add:

Net Profit after Interest but before Tax Interest on Term Loan

Less:

Tax

Net PBIT Net Profit before Interest but after Tax

Now, Return on Capital Employed = or

10 =

Rs. 1,00,000 1,20,000 2,20,000 40,000 1,80,000

Net Profit before Interest but after Tax × 100 Capital Employed

1,80,000 × 100 Capital Employed

∴ Capital Employed = Rs. 18,00,000 Again, Debt-Equity Ratio =

Debt 5 = Equity 4

∴ Capital Employed = Equity + Debt = 5 + 4 = 9 ∴Equity = 4/9 × 18,00,000 = Rs. 8,00,000 ∴ Debt (i.e., Term Loan) = 5/9 × 18,00,000 = Rs. 10,00,000 2. Owner’s Equity to Fixed Asset = 8:15 Owner’s Equity 8 = or Fixed Asset 15 or

8,00,000 8 = Fixed Asset 15

∴ Fixed Asset = 8,00,000 × 15 ÷ 8 = Rs. 15,00,000 3. Fixed Asset Turnover Ratio = or

Turnover Fixed Asset

Turnover 8 = 15,00,000 5

∴ Turnover = 15,00,000 × 8 ÷ 5 = Rs. 24,00,000 ∴ Closing stock = 12.5% of sales = 12.5% of 24,00,000 = Rs. 3,00,000 4. Debtors’ Turnover = 1 month ∴ Debtors’ Turnover Ratio =

12 months = 12 1 month

or

Credit Sales = 12 Debtors

or

24,00,000 = 12 [ assumed that all sales were made on credit ] Debtors

∴ Debtors = Rs. 2,00,000 (Continued)

Modified Date: Sat, Jul 03, 2010 12:42:12 PM

Output Date: Tue, Jul 06, 2010 11:40:08 AM

Rev II

Project: Management Accounting_Debarshi Bhattacharyya ACE Pro India Pvt. Ltd. File: X:\Pearson\Management Accounting_Debarshi Bhattacharyya\MAIN\M03\LAYOUT_M03\M03_DEBA_ISBN_EN_SE_C03_Part-2.indd

122

MANAGEMENT ACCOUNTING

5. In a Balance Sheet, Owner’s Equity + Long-term Loan = Fixed Asset + Working Capital Here, 8,00,000 + 10,00,000 = 15,00,000 + Working Capital ∴Working Capital = Rs. 3,00,000 Again, in the given problem, Current Assets = Stock + Debtors + Bank = 3,00,000 + 2,00,000 + 1,00,000 + Rs. 6,00,000 Now, Working Capital = Current Assets (CAs) − Current Liabilities (CLs) or

3,00,000 = 6,00,000 − CL

∴Current Liabilities = Rs. 3,00,000 Again, Liquid Ratio = Here, 1.2 = or

CAs − Stock CLs − Bank Overdraft

(6,00,000 – 3,00,000) (3,00,000 − Bank overdraft)

3,00,000 = 3,60,000 − 1.2 Bank Overdraft

∴ Bank overdraft = 60,000 ÷ 1.2 = Rs. 50,000 Again, CLs = Liquid Liabilities + Bank Overdraft Here, 3,00,000 = Liquid Liabilities + 50,000 ∴ Liquid Liabilities = Rs. 2,50,000

Problem 24 A company gives you the following information in respect of the year 2008–09: Gross Profit Ratio Net Profit Ratio Stock Turnover Ratio Average Debt Collection Period Credit Period Allowed by Suppliers Current Ratio Depreciation on Fixed Asset @ 10% Long-term Loan Opening Stock Closing Stock

20% 15% 10 2 months 1 month 4 Rs. 20,000 Rs. 1,00,000 Rs. 90,000 Rs. 1,02,000

On 31 March 2009, Current Asset consisted of Stock, Debtors and cash only. There was no Bank Overdraft. All purchases were made on credit. Cash sales were 25% of the total sales. Prepare a Profit & Loss A/c of the company for the year that ended on 31 March 2009 and a Balance Sheet as on that date.

Modified Date: Sat, Jul 03, 2010 12:42:12 PM

Output Date: Tue, Jul 06, 2010 11:40:08 AM

Rev II

Project: Management Accounting_Debarshi Bhattacharyya ACE Pro India Pvt. Ltd. File: X:\Pearson\Management Accounting_Debarshi Bhattacharyya\MAIN\M03\LAYOUT_M03\M03_DEBA_ISBN_EN_SE_C03_Part-2.indd

ACCOUNTING RATIOS FOR FINANCIAL STATEMENT ANALYSIS

123

Solution Books of a Company Profit & Loss A/c for the year that ended on 31 March 2009 Dr.

Cr. Particulars

To Cost of Goods Sold1 To Gross Profit c/d1 To Other Operating Expenses (Bal. fig.) To Depreciation on Fixed Asset To Net Profit2

Amount Rs. 9,60,000 2,40,000 12,00,000 40,000 20,000 1,80,000 2,40,000

By Sales1

Amount Rs. 12,00,000

By Gross Profit b/d

12,00,000 2,40,000

Particulars

2,40,000

Balance Sheet as on 31 March 2009 Liabilities Net Worth7 Long-term Debt Current Liabilities: Bank Overdraft Payables5

Amount Rs. 3,23,000 1,00,000 50,000 Nil 81,000 4,04,000

Assets Fixed Assets3 Current Assets: Stock4 Debtors4 Cash6

Amount Rs. 1,80,000 1,02,000 1,50,000 72,000 4,04,000

Working Notes (Opening Stock + Closing Stock) (90,000 + 1,02,000) = = Rs. 96,000 2 2 CGS Now, Stock Turnover Ratio = Average Stock CGS or 10 = 96,000 ∴ CGS = 96,000 × 10 = Rs. 9,60,000 ∴ GP = 20% on Sales = 1/5 on Sales = 1/4 on CGS = 1/4 on 9,60,000 = Rs. 2,40,000 ∴ Sales = CGS + GP = 9,60,000 + 2,40,000 = Rs. 12,00,000 NP 2. NP Ratio = × 100 Sales NP or 15% = 12,00,000 ∴ Net Profit after Tax = 15% × 12,00,000 = Rs. 1,80,000 1. Average Stock =

3. Depreciation on Fixed Asset @ 10% = Rs. 20,000 ∴ Net Fixed Asset = 20,000 × 90 ÷ 10 = Rs. 1,80,000 4. Total Sales = Rs. 12,00,000 Cash Sales = 25% of Total Sales ∴ Credit Sales = 75% of Total Sales = 75% of 12,00,000 = Rs. 9,00,000 Average Debt Collection Period = 2 months 12 months ∴ Debtors’ Turnover Ratio = =6 2 months (Continued)

Modified Date: Sat, Jul 03, 2010 12:42:12 PM

Output Date: Tue, Jul 06, 2010 11:40:08 AM

Rev II

Project: Management Accounting_Debarshi Bhattacharyya ACE Pro India Pvt. Ltd. File: X:\Pearson\Management Accounting_Debarshi Bhattacharyya\MAIN\M03\LAYOUT_M03\M03_DEBA_ISBN_EN_SE_C03_Part-2.indd

124

MANAGEMENT ACCOUNTING

or

Credit Sales =6 Debtors

9,00,000 =6 Debtors 9,00,000 ∴ Debtors = = Rs. 1,50,000 6 5. Generally, CGS = Materials Consumed + Wages & Manufacturing Overheads In the absence of adequate information regarding wages and manufacturing overheads, it may be assumed that CGS = Materials Consumed = Rs. 9,60,000 Now, Opening Stock + Purchases = Materials Consumed + Closing Stock ∴ Purchases = Materials Consumed + Closing Stock − Opening Stock = 9,60,000 + 1,02,000 – 90,000 ∴ Purchases = Rs. 9,72,000 Now, Credit period allowed by suppliers = 1 month 12 months ∴ Creditors’ Turnover Ratio = = 12 1 month Credit Purchases or = 12 Payables 9,72,000 = 12 or Payables 9,72,000 ∴ Payables = = Rs. 81,000 12 6. Here, Current Liabilities (CLs) = Payables + Bank Overdraft = 81,000 + Nil = Rs. 81,000 or

Again, Current ratio =

Current Asset (CAs) =4 Current Liability (CLs)

CAs =4 81,000 ∴ Current Asset = 81,000 × 4 = Rs. 3,24,000 Here, Current Asset = Stock + Debtors + Cash or 3,24,000 = 1,02,000 + 1,50,000 + Cash ∴ Cash = Rs. 72,000 7. Working Capital = Current Assets (CAs) − Current Liabilities (CLs) = 3,24,000 − 81,000 = Rs. 2,43,000 In a Balance Sheet, Net Worth + Long-term Loan = Fixed Asset + Working Capital Here, Net worth + 1,00,000 = 1,80,000 + 2,43,000 ∴Net Worth = Rs. 3,23,000 or

Problem 25 From the following information of S Ltd, prepare its trading, Profit & Loss A/c and Balance Sheet: Sales Working Capital Bank Overdraft Share Capital

Rs. 7,30,000 Rs. 1,20,000 Rs. 15,000 Rs. 2,50,000

Quick Ratio Current Ratio Proprietary Ratio Fixed Assets/Proprietary Fund GP Ratio

1.3 2.5 0.6 10%

Net Profit is 10% of Proprietary fund. There are no long-term liabilities and fictitious assets. Closing stock is 10% more than the opening stock. [B.Com. (Hons), Calcutta University—2008]

Modified Date: Sat, Jul 03, 2010 12:42:12 PM

Output Date: Tue, Jul 06, 2010 11:40:08 AM

Rev II

Project: Management Accounting_Debarshi Bhattacharyya ACE Pro India Pvt. Ltd. File: X:\Pearson\Management Accounting_Debarshi Bhattacharyya\MAIN\M03\LAYOUT_M03\M03_DEBA_ISBN_EN_SE_C03_Part-2.indd

ACCOUNTING RATIOS FOR FINANCIAL STATEMENT ANALYSIS

125

Solution Books of S Ltd Trading and Profit & Loss A/c for the year that ended on Dr.

Cr. Particulars 2

To Opening Stock To Purchases (Bal. fig.) To Gross Profit c/d1 To Indirect Expenses7 To Net Profit6

Amount Rs. 1,05,000 6,67,500 73,000 8,45,500 43,000 30,000 73,000

By Sales By Closing Stock

Amount Rs. 7,30,000 1,15,500

By Gross Profit b/d

8,45,500 73,000

Particulars

73,000

Balance Sheet as on Liabilities Share Capital Reserves & Surplus5 Current Liabilities: Bank Overdraft Quick Liabilities3

Amount Rs. 2,50,000 50,000 15,000 65,000 3,80,000

Assets Fixed Assets4 Current Assets: Stock2 Quick Assets3

Amount Rs. 1,80,000 1,15,500 84,500 3,80,000

Working Notes GP Sales G.P Here, 10% = 7,30,000 ∴ GP = Rs. 73,000

1. GP Ratio =

2. Current Ratio =

Current Assets (CAs) 2.5 = 2.5 = Current Liabilities (CLs) 1

∴ Working Capital = CAs − CLs = 2.5 − 1 = 1.5 Again, given Working Capital = Rs. 1,20,000 ∴ Current Assets (CAs) = 2.5 / 1.5 × 1,20,000 = Rs. 2,00,000 ∴ Current Liabilities (CLs) = 1/1.5 × 1,20,000 = Rs. 80,000 (CAs − Stock) Again, Quick Ratio = (CLs − Bank Overdraft) (2,00,000 − Stock) (80,000 –15,000) or 84,500 = 2,00,000 − Stock ∴ Closing Stock = Rs. 1,15,500 Again, Closing Stock = 10% more than Opening Stock ∴Opening Stock = 100/110 × 1,15,500 = Rs. 1,15,500 or

1.3 =

3. Total Current Assets = Quick Assets + Stock Here, 2,00,000 = Quick Assets + 1,15,500 ∴ Quick Assets = Rs. 84,500 (Continued)

Modified Date: Sat, Jul 03, 2010 12:42:12 PM

Output Date: Tue, Jul 06, 2010 11:40:08 AM

Rev II

Project: Management Accounting_Debarshi Bhattacharyya ACE Pro India Pvt. Ltd. File: X:\Pearson\Management Accounting_Debarshi Bhattacharyya\MAIN\M03\LAYOUT_M03\M03_DEBA_ISBN_EN_SE_C03_Part-2.indd

126

MANAGEMENT ACCOUNTING

Again, Total Current Liabilities = Quick Liabilities + Bank Overdraft Here, 80,000 = Quick Liabilities + 15,000 ∴Quick Liabilities = Rs. 65,000 4. Proprietary Ratio =

Fixed Assets (FA) = 0.6 Proprietors’ Fund (PF)

∴FA = 0.6 PF Again, as per company Balance Sheet, we know that Proprietors’ Fund (PF) + Long-term Loan (LTL) = Fixed Asset (FA) + Working Capital (WC) Here, PF = FAs + WC [as here, LT Loans = Nil] or PF = 0.6 PF + 1,20,000 or 0.4 PF = 1,20,000 ∴ Proprietors’ Fund (PF) = Rs. 3,00,000 ∴ Fixed Asset (FA) = 0.6 × 3,00,000 = Rs. 1,80,000 5. Proprietors’ Fund (PF) = Share Capital (SC) + Reserves & Surplus (RS) − Miscellaneous Expenditure (ME) Here, 3,00,0004 = 2,50,000 + RS – Nil [as here, Miscellaneous expenditure = Nil] ∴Reserves & Surplus (RS) = Rs. 50,000 6. NP = 10% of PF = 10% of 3,00,000 = Rs. 30,000 7. Indirect Expenses Debited to Profit & Loss A/c = GP − NP= 73,000 – 30,000 = Rs. 43,000

Problem 26 From the following information, prepare a trading and Profit & Loss A/c for the year that ended on 31 March 2009 and a Balance Sheet as on that date: Gross Profit Margin Net Profit Margin Return on Investment Rate of Tax Interest on Debt Fixed Asset Turnover Ratio Debtors’ Turnover Inventory Turnover Ratio Current Ratio Debt Asset Ratio Short-term Debt Net Sales

25% 5% 5% 50% Rs. 5,000 0.80 6 months 1.25 2.5 0.60 Rs. 50,000 Rs. 1,00,000

Solution Books of _______________ Trading and Profit & Loss A/c for the year that ended on 31 March 2009 Dr.

Cr. Particulars

To Cost of Goods Sold1 To Gross Profit c/d1 To Other Operating Expenses (Bal. fig.) To Interest on Debt

Amount Rs. 75,000 25,000 1,00,000 10,000 5,000

Particulars By Sales

By Gross Profit b/d

Amount Rs. 1,00,000 1,00,000 25,000 (Continued)

Modified Date: Sat, Jul 03, 2010 12:42:12 PM

Output Date: Tue, Jul 06, 2010 11:40:08 AM

Rev II

Project: Management Accounting_Debarshi Bhattacharyya ACE Pro India Pvt. Ltd. File: X:\Pearson\Management Accounting_Debarshi Bhattacharyya\MAIN\M03\LAYOUT_M03\M03_DEBA_ISBN_EN_SE_C03_Part-2.indd

ACCOUNTING RATIOS FOR FINANCIAL STATEMENT ANALYSIS

127

Dr.

Cr. Amount Rs. 5,000 5,000 25,000

Particulars To Tax2 To Net Profit after Tax2

Balance Sheet of Liabilities Net Worth8 Long-term Debt7 Short-term Debt

Particulars

Amount Rs.

25,000

as on 31 March 2009 Amount Rs. 1,00,000 1,00,000 50,000

Assets Fixed Assets3 Current Assets: Stock4 Debtors5 Cash & Bank6

2,50,000

Amount Rs. 1,25,000 60,000 50,000 15,000 2,50,000

Working Notes GP Sales GP 25% = 1,00,000

1. GP Margin = or

∴GP = Rs. 25,000 ∴CGS = Sales – GP = 1,00,000 – 25,000 = Rs. 75,000 Net PAT Sales Net PAT 5% = or 1,00,000 ∴ Net Profit after tax = Rs. 5,000 As the Rate of Tax = 50% ∴Tax = 50/50 × 5,000 = Rs. 5,000 Sales 3. Fixed Asset Turnover ratio = Fixed Asset 1,00,000 or 0.8 = Fixed Asset 1,00,000 ∴ Fixed Asset = = Rs. 1,25,000 0.8 CGS 4. Inventory Turnover Ratio = Average Stock 75,000 or 1.25 = Average Stock 2. NP Margin =

75,000 = Rs. 60,000 1.25 In the absence of adequate information regarding opening stock, it may be assumed that Opening Stock = Closing Stock Then, Closing Stock = Average Stock = Rs. 60,000 ∴ Average Stock =

(Continued)

Modified Date: Sat, Jul 03, 2010 12:42:12 PM

Output Date: Tue, Jul 06, 2010 11:40:08 AM

Rev II

Project: Management Accounting_Debarshi Bhattacharyya ACE Pro India Pvt. Ltd. File: X:\Pearson\Management Accounting_Debarshi Bhattacharyya\MAIN\M03\LAYOUT_M03\M03_DEBA_ISBN_EN_SE_C03_Part-2.indd

128

MANAGEMENT ACCOUNTING

5. Debtors’ Turnover Ratio =

Credit Sales Debtors

1,00,000 [Assumed all sales were made on credit] Debtors 1,00,000 ∴ Debtors = = Rs. 50,000 2

or

2=

6. Current Ratio =

Current Asset (CAs) Current Liability [i.e., Short-term Debt ]

CAs 50,000 ∴Current Asset = 50,000 × 2.5 = Rs. 1,25,000

or

2.5 =

Again, Current Asset = Stock + Debtors + Cash & Bank or

1,25,000 = 60,000 + 50,000 + Cash & Bank

∴Cash & Bank = Rs. 15,000 7. Here, Total Asset = Fixed Asset + Current Asset = 1,25,000 + 1,25,000 = Rs. 2,50,000 Now, Debt Asset Ratio = or

0.6 =

Total Debt 2,50,000

∴ Total Debt =

Total Debt Total Asset

2,50,000 = Rs. 1,50,000 0.6

Again, Total Debt = Long-term Debt + Short-term Debt or

1,50,000 = Long-term Debt + 50,000

∴Long-term Debt = Rs. 1,00,000 8.

Net Profit after Tax2 Interest on Debt

Add:

Net Profit before Interest but after Tax

Rs. 5,000 5,000 10,000

Net Profit before Interest but after Tax × 100 Capital Employed 10,000 5% = Capital Employed

Now, ROI = or

∴ Capital Employed =

10,000 = Rs. 2,00,000 5%

Again, Capital Employed = Net Worth + Long-term Debt or

2,00,000 = Net Worth + 1,00,000

∴Net Worth = Rs. 1,00,000

Modified Date: Sat, Jul 03, 2010 12:42:12 PM

Output Date: Tue, Jul 06, 2010 11:40:08 AM

Rev II

Project: Management Accounting_Debarshi Bhattacharyya ACE Pro India Pvt. Ltd. File: X:\Pearson\Management Accounting_Debarshi Bhattacharyya\MAIN\M03\LAYOUT_M03\M03_DEBA_ISBN_EN_SE_C03_Part-2.indd

129

ACCOUNTING RATIOS FOR FINANCIAL STATEMENT ANALYSIS

Problem 27 A & Co. started its business on 1 April 2004. The following information was received from the company for the year 2004–05: Capital Introduced on 1 April 2004 Drawings during the year Depreciation @ 20% on Fixed Assets General Expenses [Excluding Depreciation] Other Figures at the year-end: Net Working Capital Current Ratio Acid Test Ratio [Cash & Debtors to Current Liabilities] Capital Employed Turnover [Sales to Net Assets]

Rs. 5,00,000 Rs. 50,000 Rs. 25,000 Rs. 68,000 Rs. 50,000 2:1 1.5:1 2:1

Prepare an Income Statement for the year 2004–05 and a Balance Sheet as on 31 March 2005, with as many details as possible. [B.B.A. (Hons), Calcutta University—2005] Solution Income Statement of A & Co. for the year that ended on 31 March 2005 Rs. Less: Less:

Sales5 Cost of Goods Sold (Bal. fig.) Gross Loss General Expenses Depreciation Net Loss for the year4

68,000 25,000

Rs. 3,00,000 5,07,000 2,07,000 93,000 3,00,000

Balance Sheet of A & Co. as on 31 March 2005

Less: Less:

Liabilities Capital: Opening Balance Drawings Net Loss Current Liabilities1

Rs. 5,00,000 50,000 4,50,000 3,00,000

Rs. Less:

1,50,000 50,000 2,00,000

Assets Fixed Assets3 Depreciation Current Assets: Stock2 Cash & Debtors2

Rs. 1,25,000 25,000

Rs. 1,00,000 25,000 75,000 2,00,000

Working Notes 1. Net Working Capital = Current Assets (CAs) − Current Liabilities (CLs) = Rs. 50,000 CAs 2 Again, Current Ratio = = CLs 1 ∴Net Working Capital = 2 − 1 = 1 ∴Current Assets = 2/1 × Rs. 50,000 = Rs. 1,00,000 ∴ Current Liabilities = 1/1 × Rs. 50,000 = Rs. 50,000 2. Here, Acid Test Ratio =

Cash & Debtors 1.5 = CLs 1 (Continued)

Modified Date: Sat, Jul 03, 2010 12:42:12 PM

Output Date: Tue, Jul 06, 2010 11:40:08 AM

Rev II

Project: Management Accounting_Debarshi Bhattacharyya ACE Pro India Pvt. Ltd. File: X:\Pearson\Management Accounting_Debarshi Bhattacharyya\MAIN\M03\LAYOUT_M03\M03_DEBA_ISBN_EN_SE_C03_Part-2.indd

130

MANAGEMENT ACCOUNTING

(CAs − Stock) = 1.5 CLs or (1,00,000 − Stock) = 1.5 50,000 ∴Stock = 1,00,000 − 75,000 = Rs. 25,000 ∴Cash & Debtors = 1,00,000 − 25,000 = Rs. 75,000

or

3. Depreciation on Fixed Assets @ 20% = Rs. 25,000 ∴Value of Fixed Assets before Charging Depreciation = 100/20 × 25,000 = Rs. 1,25,000 4. Here, the Balance Sheet equation is Capital + CLs = FAs + CA ∴ Closing Capital = FAs + CAs − CLs = (1,25,000 − 25,000) + 1,00,000 − 50,000 = Rs. 1,50,000 ∴Net Loss for the year = Opening capital – Closing capital − Drawings = 5,00,000 − 1,50,000 − 50,000 = Rs. 3,00,000 5. Capital Turnover Ratio =

Sales 2 = Closing Capital 1

Sales =2 1,50,000 ∴ Sales = Rs. 3,00,000

or

Problem 28 From the following information, prepare a trading and Profit & Loss A/c for the year that ended on 31 March 2008 and a Balance Sheet as on that date of K Ltd: Gross Profit Ratio Current Ratio Net Profit to Equity Capital Stock Turnover Ratio Average Debt Collection Period Creditors’ Velocity Proprietary Ratio [Fixed Assets to Capital Employed] Capital Gearing Ratio [Preference Shares & Debentures to Equity) General Reserves and Profit & Loss A/c to Issued Equity Capital Preference Share Capital to Debentures

25% 2 10% 5 times 2 months 3 months 80% 25% 25% 3

Cost of sales consists of 40% for materials, and balance for wages and overheads. GP is Rs. 6,00,000. Solution Books of _______________ Trading and Profit & Loss A/c for the year that ended on 31 March 2008 Dr. Particulars To Materials Consumed2 To Wages & Overheads2 To Gross Profit c/d To Other Operating Expenses & Interest on Debentures (Bal. fig.) To Net Profit

Modified Date: Sat, Jul 03, 2010 12:42:12 PM

Amount Rs. 7,20,000 10,80,000 6,00,000 24,00,000 4,78,400

Particulars By Sales1

By Gross Profit b/d

1,21,600 6,00,000

Output Date: Tue, Jul 06, 2010 11:40:08 AM

Cr. Amount Rs. 24,00,000

24,00,000 6,00,000

6,00,000

Rev II

Project: Management Accounting_Debarshi Bhattacharyya ACE Pro India Pvt. Ltd. File: X:\Pearson\Management Accounting_Debarshi Bhattacharyya\MAIN\M03\LAYOUT_M03\M03_DEBA_ISBN_EN_SE_C03_Part-2.indd

ACCOUNTING RATIOS FOR FINANCIAL STATEMENT ANALYSIS

131

Balance Sheet as on 31 March 2008 Amount Rs. 12,16,000 2,85,000 1,82,400 1,21,600 95,000

Liabilities Equity Share Capital8 Preference Share Capital8 General Reserve9 Profit & Loss A/c9 Debentures8 Current Liabilities: Payables4 Bank Overdraft6

Assets Fixed Assets7 Current Assets: Stock3 Receivables5

1,80,000 2,00,000 2,32,300

Amount Rs. 15,20,000 3,60,000 4,00,000

2,32,300

Working Notes 1. Gross Profit Ratio = Gross Profit = 25% Sales 6,00,000 = 25% or Sales ∴Sales = 6,00,000 × 25% = Rs. 24,00,000 2. Cost of Sales = Sales − Gross Profit = 24,00,000 – 6,00,000 = Rs. 18,00,000 ∴Materials Consumed = 40% of Cost of Sales = 40% of 18,00,000 = Rs. 7,20,000 ∴Wages & Overheads = 60% of Cost of Sales = 60% of 18,00,000 = Rs. 10,80,000 3. Stock Turnover Ratio =

Cost of Goods Sold =5 Average Stock

18,00,000 = 5 Average Stock 18,00,000 ∴ Average Stock = = Rs. 3,60,000 5 Here, in the absence of adequate information, it is considered that Opening Stock = Closing Stock ∴Average Stock = Closing Stock = Rs. 3,60,000 ∴Opening Stock = Closing Stock = Average Stock = Rs. 3,60,000

or

4. Materials Consumed = Rs. 7,20,000 We know, Opening Stock + Purchases + Wages & Overheads = Cost of Sales + Closing Stock Here, 3,60,000 + Purchases + 10,80,000 = 18,00,000 + 3,60,000 ∴ Purchases = Rs. 7,20,000 Again, Creditors’ Velocity = 3 months ∴ Creditors’ Velocity Ratio =

12 months =4 3 months

or

Credit Purchases =4 Payables

or

7,20,000 = 4 [ assumed that the entire purchases were made on credit ] Payables

∴Payables = 7,20,000 ÷ 4 = Rs. 1,80,000 (Continued)

Modified Date: Sat, Jul 03, 2010 12:42:12 PM

Output Date: Tue, Jul 06, 2010 11:40:08 AM

Rev II

Project: Management Accounting_Debarshi Bhattacharyya ACE Pro India Pvt. Ltd. File: X:\Pearson\Management Accounting_Debarshi Bhattacharyya\MAIN\M03\LAYOUT_M03\M03_DEBA_ISBN_EN_SE_C03_Part-2.indd

132

MANAGEMENT ACCOUNTING

5. Average Collection Period = 2 months ∴ Debtors’ Velocity Ratio = or

12 months =6 2 months

Credit Sales =6 Receivables

24,00 ,000 = 6 [ assuming that the total sales were made on credit ] Receivables ∴Receivables = Rs. 4,00,000.

or

Current Assets (CAs) =2 Current Liabilities (CLs) Here, it is assumed that there is no cash or bank balance or prepaid expense. Then, CAs = Stock + Receivables = 3,60,000 + 4,00,000 = Rs. 7,60,000 7,60,000 ∴ =2 CLs ∴Current Liabilities = Rs. 3,80,000 Again, Current Liabilities = Payables + Bank Overdraft Here, 3,80,000 = 1,80,000 + Bank Overdraft ∴Bank Overdraft = Rs. 2,00,000 ∴Working Capital = CAs − CLs = 7,60,000 – 3,80,000 = Rs. 3,80,000

6. Current Ratio =

7. Here, Proprietary Ratio =

Fixed Assets (FA) = 80% Capital Employed

∴ FA = 0.8 Capital Employed Again, Capital Employed = Fixed Asset + Working Capital Here, Capital Employed = 0.8 Capital Employed + 3,80,000 or 0.2 Capital Employed = 3,80,000 ∴ Capital Employed = 3,80,000 ÷ 0.2 = Rs. 19,00,00 ∴Fixed Asset = 0.8 × 19,00,000 = Rs. 15,20,00 8. Capital Gearing Ratio = or

25% =

Preference Share Capital & debentures Equity (i.e., Equity Shareholders’ Fund)

Preference Share Capital & Debentures Equity

∴ Preference Share Capital & Debentures = 0.25 Equity Again, Capital Employed = Equity Share Capital + Reserves & Surplus + Preference Share Capital + Debentures Again, Equity (i.e., Equity Shareholders’ Fund) = Equity Share Capital + Reserves & Surplus ∴Capital Employed = Equity + (Preference Share Capital + Debentures) or 19,00,000 = Equity + 0.25 Equity or 1.25 Equity = 19,00,000 ∴Equity = 19,00,000 ÷ 1.25 = Rs. 15,20,000 ∴Preference Share Capital + Debentures = 0.25 × 15,20,000 = Rs. 3,80,000 Preference Share Capital 3 = Debentures 1 ∴Preference Share Capital + Debentures = 3 + 1 = 4 ∴Preference Share Capital = 3/4 × 3,80,000 = Rs. 2,85,000 ∴ Debentures = 1/4 × 3,80,000 = Rs. 95,000 Again,

Modified Date: Sat, Jul 03, 2010 12:42:12 PM

Output Date: Tue, Jul 06, 2010 11:40:08 AM

Rev II

Project: Management Accounting_Debarshi Bhattacharyya ACE Pro India Pvt. Ltd. File: X:\Pearson\Management Accounting_Debarshi Bhattacharyya\MAIN\M03\LAYOUT_M03\M03_DEBA_ISBN_EN_SE_C03_Part-2.indd

133

ACCOUNTING RATIOS FOR FINANCIAL STATEMENT ANALYSIS

Here, Equity (i.e., Equity Shareholders’ Fund) = Equity Share Capital + General Reserves + Profit & Loss A/c = 15,20,000 Again,

(General Reserves + Profit & Loss A/c) 25 1 = 25% = = Equity Share Capital 100 4

∴Equity Shareholders’ Fund = Equity Share Capital + General Reserves + Profit & Loss A/c = 4 + 1 = 5 ∴Equity Share Capital = 4/5 × 15,20,000 = Rs. 12,16,000 ∴General Reserves × Profit & Loss A/c = 1/5 × 15,20,000 = Rs. 3,04,000 9. NP to Equity capital = 10% NP = 10% or Equity Capital or

NP = 10% 12,16,000

∴NP (i.e., Profit & Loss A/c) = 10% × 12,16,000 = Rs. 1,21,600 Again, General Reserves + Profit & Loss A/c = 3,04,000 or General Reserve + 1,21,600 = 3,04,000 ∴General Reserve = Rs. 1,82,400

Problem 29 Construct trading and Profit & Loss A/c for the year that ended on 31 March 2006 from the following details: Stock Velocity Debtors’ Velocity Operating Ratio Creditors’ Velocity Office Overhead to Selling & Distribution Overhead Depreciation on Fixed Assets Goods are Sold at Cost Plus Dividend Received on Investments Loss on Sale of Machinery Bank Interest Accrued Interest on Loan Taken for Investment CGS includes ‘Chargeable Expenses’ Debtors Creditors Stock in Trade [31 March 2006] Cash Purchase

1.2 months 73 days 0.85 3 months 1÷2 Rs. 15,000 33 13 % Rs. 15,000 Rs. 7,000 Rs. 3,000 Rs. 4,500 Rs. 1,44,000 Rs. 1,12,000 Rs. 58,000 Rs. 70,000

[B.Com. (Hons), Calcutta University—2007] Solution Trading and Profit & Loss A/c of _______________ for the year that ended on 31 March 2006 Dr. Particulars To Opening Stock3 To Purchases: Cash Credit1

Cr. Rs.

70,000 4,48,000

Rs. 50,000

5,18,000

Particulars By Sales: Cash Credit2 By Closing Stock

Rs. Nil 7,20,000

Rs.

7,20,000 58,000 (Continued)

Modified Date: Sat, Jul 03, 2010 12:42:12 PM

Output Date: Tue, Jul 06, 2010 11:40:08 AM

Rev II

Project: Management Accounting_Debarshi Bhattacharyya ACE Pro India Pvt. Ltd. File: X:\Pearson\Management Accounting_Debarshi Bhattacharyya\MAIN\M03\LAYOUT_M03\M03_DEBA_ISBN_EN_SE_C03_Part-2.indd

134

MANAGEMENT ACCOUNTING

Dr.

Cr.

Particulars To Wages (Bal. fig.) To Gross Profit c/d

Rs.

To Depreciation on Fixed To Office Overheads4

Rs. 30,000 1,80,000 7,78,000 15,000 19,000 38,000

To Selling & Distribution Overheads4 To Interest on Loan Taken for Investments To Loss on sale of Machinery To NP4

Particulars

By Gross Profit b/d By Dividend Received on Investments By Accrued Bank Interest

4,500 7,000 1,14,500 1,98,000

Rs.

Rs.

7,78,000 1,80,000 15,000 3,000

1,98,000

Working Notes 1. Given, Creditors’ Velocity = 3 months 12 months ∴ Creditors’ Velocity Ratio = =4 3 months or Credit Purchases = 4 Payables Here, Payables = Creditors = Rs. 1,12,000 [as there is no bills payable] Credit Purchases or =4 1,12,000 ∴ Credit Purchases = Rs. 4,48,000 2. Debtors’ Velocity = 73 days 365 days ∴ Debtors’ Velocity Ratio = =5 73 days Credit Sales or =5 Receivables Here, Receivables = Debtors = Rs. 1,44,000 [as there is no bills receivable] Credit Sales or =5 1, 44,000 ∴Credit Sales = Rs. 7,20,000 3. Goods Sold at Cost Plus 33 13 %. i.e., GP = 33 13 % on Cost = 1/3 on Cost = 1/4 on Sales = 1/4 on Rs. 7,20,000 = Rs. 1,80,000 [assumed that there is no cash sales] ∴CGS = Sales − GP = 7,20,000 – 1,80,000 = Rs. 5,40,000 12 months Again, Stock Velocity = 1.2 months = = 10 1.2 months CGS = 10 or Average Stock 5, 40,000 = 10 or Average Stock ∴ Average stock = Rs. 54,000

Modified Date: Sat, Jul 03, 2010 12:42:12 PM

Output Date: Tue, Jul 06, 2010 11:40:08 AM

Rev II

Project: Management Accounting_Debarshi Bhattacharyya ACE Pro India Pvt. Ltd. File: X:\Pearson\Management Accounting_Debarshi Bhattacharyya\MAIN\M03\LAYOUT_M03\M03_DEBA_ISBN_EN_SE_C03_Part-2.indd

ACCOUNTING RATIOS FOR FINANCIAL STATEMENT ANALYSIS

135

(Opening Stock + Closing Stock) 2 (Opening Stock + 58,000) or 54,000 = 2 or Opening Stock + 58,000 = 1,08,000 ∴Opening Stock = Rs. 10,000 Again, Average Stock =

(Cost of Goods Sold + Other Operating Expenses) Sales (5, 40,000 + Other Operating Expenses) or 0.85 = 7,20,000 or 5,40,000 + Other Operating Expenses = 6,12,000 ∴Other Operating Expenses = Rs. 72,000 Here, Other Operating Expenses = Depreciation on Fixed Assets + Office Overheads + Selling & Distribution Overheads or 72,000 = 15,000 + Office Overheads + Selling & Distribution Overheads ∴Office Overheads + Selling & Distribution Overheads = Rs. 57,000 Office Overheads 1 Given, = Selling & Distribution Overheads 2 ∴ Office overheads + Selling & Distribution Overheads = 1 + 2 = 3 ∴ Office Overheads = 1/3 × 57,000 = Rs. 19,000 ∴Selling & Distribution Overheads = 2/3 × 57,000 = Rs. 38,000

4. Operating Ratio =

Problem 30 From the following particulars, prepare a statement showing Proprietors’ Fund as on 31 March 2009: Gross Profit Gross Profit Ratio Capital Turnover Ratio Fixed Asset Turnover Ratio Long-term Loan Reserves & Surplus Gearing Ratio [Preference Capital to Equity Capital] Stock Velocity Debtors’ Velocity Creditors’ Velocity Bank Overdraft Bills Receivable Bills Payable Miscellaneous Expenditure Closing Stock

Rs. 7,50,000 25% 1.6901408 4 Rs. 7,75,000 Rs. 2,00,000 1: 3 4 months 3 months 6 times Rs. 1,10,000 Rs. 50,000 Rs. 45,000 Nil Rs. 30,000 more than the Opening Stock

Solution Statement of Proprietors’ Fund of an organization as on 31 March 2009 Rs. Sources of Fund: Equity Share Capital2 Preference Share Capital2

Rs.

Rs. 6,00,000 2,00,000 (Continued)

Modified Date: Sat, Jul 03, 2010 12:42:12 PM

Output Date: Tue, Jul 06, 2010 11:40:08 AM

Rev II

Project: Management Accounting_Debarshi Bhattacharyya ACE Pro India Pvt. Ltd. File: X:\Pearson\Management Accounting_Debarshi Bhattacharyya\MAIN\M03\LAYOUT_M03\M03_DEBA_ISBN_EN_SE_C03_Part-2.indd

136

MANAGEMENT ACCOUNTING

Rs.

Rs.

Total Share Capital Add:

Reserves & Surplus

Less:

Miscellaneous Expenditure Applications of Fund: Fixed Assets1 Working Capital: Current Assets: Stock4 Debtors3 Bills Receivable

Add:

Less:

Rs. 8,00,000 2,00,000 10,00,000 Nil 10,00,000 7,50,000

7,65,000 7,00,000 50,000 15,15,000

Current Liabilities: Creditors5 Bills Payable Bank Overdraft

3,35,000 45,000 1,10,000 4,90,000 10,25,000 17,25,000 7,75,000 10,00,000

Long-term Loan6

Less:

Working Notes GP × 100 Sales 7,50,000 25% = Sales

1. GP Ratio = or

∴Sales = 7,50,000 ÷ 25% = Rs. 30,00,000 Sales Again, Fixed Asset Turnover Ratio = Fixed Asset 30,00,000 or 4 = Fixed Asset ∴Fixed asset = 30,00,000 ÷ 4 = Rs. 7,50,000 Sales Capital Employed 30,00,000 or 1.6901408 = Capital Employed 30,00,000 ∴ Capital Employed = = Rs. 17,75,000 1.6901408 Again, Capital Employed = Proprietors’ Fund + Long-term Loan or 17,75,000 = Proprietors’ Fund + 7,75,000 ∴Proprietors' Fund = Rs. 10,00,000

2. Capital Turnover Ratio =

Again, Gearing Ratio =

Preference Share Capital Equity Share Capital

Modified Date: Sat, Jul 03, 2010 12:42:12 PM

Output Date: Tue, Jul 06, 2010 11:40:08 AM

Rev II

Project: Management Accounting_Debarshi Bhattacharyya ACE Pro India Pvt. Ltd. File: X:\Pearson\Management Accounting_Debarshi Bhattacharyya\MAIN\M03\LAYOUT_M03\M03_DEBA_ISBN_EN_SE_C03_Part-2.indd

ACCOUNTING RATIOS FOR FINANCIAL STATEMENT ANALYSIS

or

137

1 Preference Share Capital = 3 Equity Share Capital

∴Equity Share Capital = 3 × Preference Share Capital Now, Proprietors’ Fund = Equity Share Capital + Preference Share Capital + Reserves & Surplus − Miscellaneous Expenditure Here, 10,00,000 = 3 × Preference Share Capital + Preference Share Capital + 2,00,000 − Nil or 4 × Preference Share Capital = 8,00,000 ∴Preference Share Capital = 8,00,000 ÷ 4 = Rs. 2,00,000 ∴Equity Share Capital = 3 × 2,00,000 = Rs. 6,00,000 3. Debtors’ Velocity = 3 months = or

12 months =4 3 months

Credit Sales =4 Receivables

30,00,000 = 4 [assumed that all sales were made on credit] Receivables 30,00,000 ∴ Receivables = = Rs. 7,50,000 4 Again, Receivables = Debtors + Bills Receivable or 7,50,000 = Debtors + 50,000 ∴Debtors = Rs. 7,00,000

or

4. CGS = Sales – GP = 30,00,000 – 7,50,000 = Rs. 22,50,000 12 months Now, Stock Velocity = 4 months = =3 4 months CGS or =3 Average Stock or

22,50,000 =3 Average Stock

∴ Average Stock =

22,50,000 = Rs. 7,50,000 3

Now, let the Opening Stock be x. Closing Stock = x + 30,000 (Opening Stock + Closing Stock) x + (x + 30,000) ∴ Average Stock = = = x + 15,000 2 2 or 7,50,000 = x + 15,000 ∴x = Opening Stock = Rs. 7,35,000 ∴Closing Stock = 7,35,000 + 30,000 = Rs. 7,65,000 5. We know that CGS = Materials Consumed + Wages & Manufacturing Overheads In the absence of adequate information regarding Wages & Manufacturing overheads, here it may be assumed that Materials consumed = CGS = 22,50,0004 Now, Opening Stock + Purchases = Materials Consumed + Closing Stock ∴Purchases = Materials Consumed + Closing Stock − Opening Stock = 22,50,000 + 7,65,000 – 7,35,000 ∴Purchases = Rs. 22,80,000 (Continued)

Modified Date: Sat, Jul 03, 2010 12:42:12 PM

Output Date: Tue, Jul 06, 2010 11:40:08 AM

Rev II

Project: Management Accounting_Debarshi Bhattacharyya ACE Pro India Pvt. Ltd. File: X:\Pearson\Management Accounting_Debarshi Bhattacharyya\MAIN\M03\LAYOUT_M03\M03_DEBA_ISBN_EN_SE_C03_Part-2.indd

138

MANAGEMENT ACCOUNTING

Now, Creditors’ Velocity = 6 or

Credit Purchases =6 Payables

or

22,80,000 = 6 [ assumed that all purchases were made on credit ] Payables

∴ Payables =

22,80,000 = Rs. 3,80,000 6

Again, Payables = Creditors + Bills Payable or 3,80,000 = Creditors + 45,000 ∴Creditors = Rs. 3,35,000 6. In a Balance Sheet, Proprietors’ Fund + Long-term Loan = Fixed Asset + Working Capital Here, 10,00,000 + 7,75,000 = 7,50,000 + Working Capital ∴Working Capital = Rs. 10,25,000 Again, in the given problem, Rs. Current Assets: Stock Debtors Bills Receivable Bank (Bal. fig.) Less:

Rs. 7,65,000 7,00,000 50,000 Nil 15,15,000

Current Liabilities: Bank Overdraft Creditors Bills Payable

1,10,000 3,35,000 45,000

Working Capital

4,90,000 10,25,000

Problem 31 From the following information, prepare a Trading and Profit & Loss A/c for the year that ended on 31 March 2009 and a Balance Sheet as on that date of Arzoo Ltd: Gross Profit Ratio Net Profit Ratio [to Average Capital Employed] Stock velocity Debtors’ velocity Current Ratio Quick Ratio Proprietary Ratio [Fixed Assets to Proprietors’ Fund] Share Capital Working Capital Bank Overdraft

20% 10% 4 36.5 days 2.5 1.5 7 Rs. 1,80,000 Rs. 63,000 Rs. 10,000

There is no fictitious asset. In the current asset, there is no asset other than stock, debtors and cash. Closing Stock is 20% higher than the Opening Stock. [M.Com. Calcutta University—Adapted]

Modified Date: Tue, Jul 06, 2010 11:42:59 AM

Output Date: Tue, Jul 06, 2010 01:35:12 PM

Rev II

Project: Management Accounting_Debarshi Bhattacharyya ACE Pro India Pvt. Ltd. File: X:\Pearson\Management Accounting_Debarshi Bhattacharyya\MAIN\M03\LAYOUT_M03\M03_DEBA_ISBN_EN_SE_C03_Part-2.indd

ACCOUNTING RATIOS FOR FINANCIAL STATEMENT ANALYSIS

139

Solution Books of Arzoo Ltd Trading and Profit & Loss A/c for the year that ended on 31 March 2009 Dr.

Cr. Particulars 1

To Opening Stock To Purchases2 To Gross Profit c/d2 To Other Operating Expenses (Bal. fig.) To Net Profit5

Amount Rs. 47,500 2,18,500 52,250 3,18,250 32,250 20,000 52,250

By Sales By Closing Stock1

Amount Rs. 2,61,250 57,000

By Gross Profit b/d

3,18,250 52,250

Particulars 2

52,250

Balance Sheet as on 31 March 2009 Liabilities Share Capital Reserves & Surplus4 Long-term Loan3 Current Liabilities: Bank Overdraft Liquid Liabilities6

Amount Rs. 1,80,000 30,000 Nil 10,000 32,000 2,52,000

Assets Fixed Assets3 Current Assets: Stock1 Debtors7 Cash8

Amount Rs. 1,47,000

57,000 26,125 21,875 2,52,000

Working Notes 1. Current Ratio =

Current Assets (CAs) 5 = 2.5 = Current Liabilities (CLs) 2

∴ Working Capital = CAs – CLs = 5 – 2 = 3 Again, given Working Capital = Rs. 63,000 ∴Current Liabilities (CLs) = 2/3 × Rs. 63,000= Rs. 42,000 ∴Current Assets (CAs) = 5/3 × Rs. 63,000 = Rs. 1,05,000 Liquid Assets (CAs – Stock) Again, Quick Ratio = = = 1.5 Liquid Liabilities (CLs – Bank Overdraft) (1,05,000 – Stock) Here, = 1.5 (42,000 –10,000) or 48,000 = 1,05,000 – Stock ∴Stock (i.e., Closing Stock) = Rs. 57,000 Now, let Opening Stock = 100 Then, Closing Stock = 100 + 20% of 100 = 120 ∴Opening Stock = 100/120 × 57,000 = Rs. 47,500 (Opening Stock + Closing Stock) (47,500 + 57,000) ∴ Average Stock = = = Rs. 52,250 2 2 CGS 2. Stock Velocity = Average Stock CGS or 4 = 52,250 (Continued)

Modified Date: Sat, Jul 03, 2010 12:42:12 PM

Output Date: Tue, Jul 06, 2010 11:40:08 AM

Rev II

Project: Management Accounting_Debarshi Bhattacharyya ACE Pro India Pvt. Ltd. File: X:\Pearson\Management Accounting_Debarshi Bhattacharyya\MAIN\M03\LAYOUT_M03\M03_DEBA_ISBN_EN_SE_C03_Part-2.indd

140

MANAGEMENT ACCOUNTING

∴CGS = 52,250 × 4 = Rs. 2,09,000 ∴GP = 20% on Sales =

1

5

on Sales =

1

4

on CGS =

1

4

on 2,09,000 = Rs. 52,250

∴Sales = CGS + GP = 2,09,000 + 52,250 = Rs. 2,61,250 We know, CGS = Materials Consumed + Wages & Overheads In the absence of adequate information regarding wages and overheads, here it may be assumed that Materials Consumed = CGS = 2,09,000 Now, Opening Stock + Purchases = Materials Consumed + Closing Stock ∴Purchases = Materials Consumed + Closing Stock – Opening Stock = 2,09,000 + 57,000 – 47,500 ∴Purchases = Rs. 2,18,500 3. Proprietary Ratio =

Fixed Assets (FA) = 0.7 Proprietors’ Fund (PF)

∴FA = 0.7 PF Again, as per company Balance Sheet, we know that Proprietors’ Fund (PF) + Long-term Loan (LTL) = Fixed Asset (FA) + Working Capital (WC) Here, PF = FAs + WC [assuming that LT Loans = Nil] or PF = 0.7 PF + 63,000 or 0.3 PF = 63,000 ∴Proprietors’ Fund (PF) = 63,000 ÷ 0.3 = Rs. 2,10,000 ∴Fixed Asset (FA) = 0.7 × 2,10,000 = Rs. 1,47,000 4. Proprietors’ Fund = Share Capital + Reserves & Surplus − Miscellaneous Expenditure Here, 2,10,000 = 1,80,000 + Reserves & Surplus − Nil ∴Reserves & Surplus = Rs. 30,000 5. Closing Capital Employed = Closing Proprietors’ Fund + Closing Long-term Loan = 2,10,000 + Nil = Rs. 2,10,000 Opening Capital Employed = Closing Capital Employed – NP for the year [assuming that neither shares were issued nor long-term loan was repaid during the year] Let NP for the year = x ∴Opening Capital Employed = 2,10,000 − x (Opening Capital Employed + Closing Capital Employed) ∴ Average Capital Employed = 2 ⎡⎣(2,10,000 − x ) + 2,10,000 ⎤⎦ or Average Capital Employed = 2 4,20,000 − x ) 2,10,000 − x ( ∴ Average Capital Employed = = = 2,10,000 − 0.5x 2 2 NP × 100 Now, NP Ratio = Average Capital Employed x

or

10% =

or

0.1 =

or or

x = 21,000 − 0.05x 1.05x = 21,000

∴ x=

(2,10,000 − 0.5x ) x

(2,10,000 − 0.5x )

21,000 = 20,000 1.05

∴NP for the year = Rs. 20,000

Modified Date: Sat, Jul 03, 2010 12:42:12 PM

Output Date: Tue, Jul 06, 2010 11:40:08 AM

Rev II

Project: Management Accounting_Debarshi Bhattacharyya ACE Pro India Pvt. Ltd. File: X:\Pearson\Management Accounting_Debarshi Bhattacharyya\MAIN\M03\LAYOUT_M03\M03_DEBA_ISBN_EN_SE_C03_Part-2.indd

ACCOUNTING RATIOS FOR FINANCIAL STATEMENT ANALYSIS

141

6. Current Liabilities = Liquid Liabilities + Bank Overdraft Here, 42,000 = Liquid Liabilities + 10,000 ∴Liquid Liabilities = Rs. 32,000 7. Debtors’ velocity = 36.5 days = or

365 days = 10 36.5 days

Credit Sales = 10 Debtors

2,61,250 = 10 [ assumed that all sales were made on credit ] Debtors 2,61,250 ∴ Debtors = = Rs. 26,125 10

or

8. Current Asset = Stock + Debtors + Cash or 1,05,000 = 57,000 + 26,125 + Cash ∴Cash = Rs. 21,875

Problem 32 From the following information, prepare a Profit & Loss A/c for the year that ended on 31 March 2008 and a Balance Sheet as on that date of K Ltd: Current Asset to Stock Current Ratio Acid Test Ratio Financial Leverage Earning per Share Book Value per Share Average Collection Period [Assume 360 days in a year] Stock Turnover Ratio Fixed Asset Turnover Ratio Total Liabilities to Net Worth Net Working Capital Net Profit to Sales Variable Cost Interest on Long-term Loan Tax

3:2 3 1 2.2 Rs. 40 Rs. 100 30 days 5 5:6 3.75 Rs. 10,00,000 10% 60% @ 12% Nil

[C.A. (Inter)—Adapted] Solution Books of K Ltd Profit & Loss A/c for the year that ended on 31 March 2008 Dr.

Cr. Particulars

To Variable Cost5 To Fixed Cost5 To EBIT c/d4 To Interest on Loan To Tax

Amount Rs. 23,40,000 15,60,000 11,00,000 50,00,000 6,00,000 Nil

By Sales2

Amount Rs. 50,00,000

By EBIT b/d

50,00,000 11,00,000

Particulars

(Continued)

Modified Date: Sat, Jul 03, 2010 12:42:12 PM

Output Date: Tue, Jul 06, 2010 11:40:08 AM

Rev II

Project: Management Accounting_Debarshi Bhattacharyya ACE Pro India Pvt. Ltd. File: X:\Pearson\Management Accounting_Debarshi Bhattacharyya\MAIN\M03\LAYOUT_M03\M03_DEBA_ISBN_EN_SE_C03_Part-2.indd

142

MANAGEMENT ACCOUNTING

Dr.

Cr. Amount Rs. 5,00,000 11,00,000

Particulars To EAT

Particulars

Amount Rs. 11,00,000

Balance Sheet as on 31 March 2008 Amount Rs. 12,50,000 7,50,000 50,00,000 5,00,000 75,00,000

Liabilities Share Capital9 [of Rs. 100] Reserves & Surplus9 Long-term Loan6 Current Liabilities1

Assets Fixed Assets7 Current Assets: Stock11 Liquid Assets1

Amount Rs. 60,00,000 10,00,000 5,00,000 75,00,000

Working Notes 1. Current Ratio =

Current Assets ( CAs) 3 =3= Current Liabilities ( CLs) 1

∴Net Working Capital = CAs − CLs = 3 − 1 = 2 Again, given Net Working Capital = Rs. 10,00,000 ∴ Current Assets (CAs) = 3/2 × 10,00,000 = Rs. 2,00,000 ∴Current Liabilities (CLs) = 1/2 × 10,00,000 = Rs. 5,00,000 Again,

CAs 3 = Stock 2

15,00,000 3 = Stock 2 ∴Stock = Rs. 10,00,000 Now, Liquid Assets = CAs − Stock = 15,00,000 – 10,00,000 = Rs. 5,00,000 or

CGS Average Stock Here, in the absence of adequate information, it is considered that Turnover Stock Turnover Ratio = Closing Stock Turnover or 5 = 10,00,000

2. Stock Turnover Ratio =

∴Turnover (i.e., Sales) = Rs. 50,00,000 3.

NP = 10% Sales NP = 10% or 50,00,000 ∴NP (i.e., EAT) = Rs. 5,00,000

4. As Tax = Nil, EBT = EAT = Rs. 5,00,000 EBIT Now, Financial Leverage = EBT

Modified Date: Sat, Jul 03, 2010 12:42:12 PM

Output Date: Tue, Jul 06, 2010 11:40:08 AM

Rev II

Project: Management Accounting_Debarshi Bhattacharyya ACE Pro India Pvt. Ltd. File: X:\Pearson\Management Accounting_Debarshi Bhattacharyya\MAIN\M03\LAYOUT_M03\M03_DEBA_ISBN_EN_SE_C03_Part-2.indd

ACCOUNTING RATIOS FOR FINANCIAL STATEMENT ANALYSIS

143

EBIT 5,00,000 ∴EBIT = Rs. 11,00,000

or 2.2 =

5. CGS = Sales – EBIT = 50,00,000 – 11,00,000 = Rs. 39,00,000 ∴Variable Cost = 60% of 39,00,000 = Rs. 23,40,000 ∴Fixed Cost = 40% of 39,00,000 = Rs. 15,60,000 6. Interest on Loan = EBIT − EBT = 11,00,000 – 5,00,000 = Rs. 6,00,000 Again, Rate of Interest on Long-term Loan = 12% ∴Long-term Loan = 100/12 × 6,00,000 = Rs. 50,00,000 Turnover 7. Fixed Asset Turnover Ratio = Fixed Assets 5 50,00,000 = or 6 Fixed Assets ∴Fixed Assets = Rs. 60,00,000 8. Here, Total outside Liabilities = Long-term Loan + Current Liabilities = 50,00,000 + 5,00,000 = Rs. 55,00,000 And, Total Liabilities = Total Outside Liabilities + Net Worth (NW) Now,

Total Liabilities = 3.75 Net Worth (NW )

or

( Total Outside Liabilities + NW ) = 3.75

or

(55,00,000 + NW ) = 3.75

NW

NW or 3.75 NW = 55,00,000 + NW or 2.75 NW = 55,00,000 ∴ Net worth = 55,00,000 ÷ 2.75 = Rs. 20,00,000 EAT No. of Shares 5,00,000 Here, 40 = No. of shares ∴No. of shares = 5,00,000 ÷ 40 = 12,500 ∴Share Capital = 12,500 shares @ Rs. 100 each = Rs. 12,50,000 Now, Net Worth = Share Capital + Reserves & Surplus (RS) – Miscellaneous Expenditure Here, 20,00,000 = 12,50,000 + RS – Nil [assumed that Miscellaneous Expenditure = Nil] ∴Reserves & Surplus (RS) = Rs. 7,50,000

9. EPS =

Problem 33 From the following information, prepare the Profit & Loss A/c for the year that ended on 31 March 2009 and a Balance Sheet as on that date: Fixed Asset [Net after writing off 12.5%] Fixed Asset Turnover Ratio Gross Profit Ratio Net Profit [before Interest] to Sales Fixed Charges Cover [Debenture Interest @ 7%]

Rs. 10,50,000 2 25% 8% 8 (Continued)

Modified Date: Sat, Jul 03, 2010 12:42:12 PM

Output Date: Tue, Jul 06, 2010 11:40:08 AM

Rev II

Project: Management Accounting_Debarshi Bhattacharyya ACE Pro India Pvt. Ltd. File: X:\Pearson\Management Accounting_Debarshi Bhattacharyya\MAIN\M03\LAYOUT_M03\M03_DEBA_ISBN_EN_SE_C03_Part-2.indd

144

MANAGEMENT ACCOUNTING

Finished Goods Turnover Ratio Current Ratio Quick Ratio Reserve to Capital Working Capital Debt Collection Period Materials Consumed to Sales Stock of Raw Materials

5 2.5 1.5 0.20 Rs. 9,00,000 2 months 30% 8 months’ consumption

Solution Books of _____________ Profit & Loss A/c for the year that ended on 31 March 2009 Dr.

Cr. Particulars

To Opening Stock of Finished Goods2 To Materials Consumed1 To Wages & Manufacturing Overheads1 To Gross Profit c/d1 To Other Operating Expenses (Bal. fig.) To Depreciation on Fixed Asset3 To Debenture Interest5 To Net Profit5

Amount Rs. 3,15,000 6,30,000 9,45,000 5,25,000 24,15,000 2,07,000 1,50,000 21,000 1,47,000 5,25,000

By Sales1 By Closing Stock of Finished Goods2

Amount Rs. 21,00,000 3,15,000

By Gross Profit b/d

24,15,000 5,25,000

Particulars

5,25,000

Balance Sheet as on 31 March 2009 Amount Rs. 13,75,000 2,75,000 3,00,000

Liabilities Share Capital6 Reserves & Surplus6 7% Debenture5 Current Liabilities: Bank Overdraft7 Quick Liabilities7

90,000 5,10,000 25,50,000

Assets Fixed Assets Current Assets: Stock of Finished Goods2 Stock of Raw Materials2 Receivables8 Cash & Bank9

Amount Rs. 10,50,000 3,15,000 4,20,000 3,50,000 4,15,000 25,50,000

Working Notes 1. Fixed Asset Turnover Ratio = or

Turnover Fixed Asset

Turnover =2 10,50,000

∴Turnover = 10,50,000 × 2 = Rs. 21,00,000 ∴GP Ratio = 25% on Sales = ¼ on Sales = ¼ on 21,00,000 = Rs. 5,25,000 ∴CGS = Sales − GP = 21,00,000 – 5,25,000 = Rs. 15,75,000 (Continued)

Modified Date: Sat, Jul 03, 2010 12:42:12 PM

Output Date: Tue, Jul 06, 2010 11:40:08 AM

Rev II

Project: Management Accounting_Debarshi Bhattacharyya ACE Pro India Pvt. Ltd. File: X:\Pearson\Management Accounting_Debarshi Bhattacharyya\MAIN\M03\LAYOUT_M03\M03_DEBA_ISBN_EN_SE_C03_Part-2.indd

ACCOUNTING RATIOS FOR FINANCIAL STATEMENT ANALYSIS

145

Now, Materials Consumed to Sales = 30% or

Materials Consumed = 30% 21,00,000

∴Materials Consumed = 21,00,000 × 30% = Rs. 6,30,000 Again, CGS = Materials Consumed + Wages & Manufacturing Overheads or

15,75,000 = 6,30,000 + Wages & Manufacturing Overheads

∴Wages & Manufacturing Overheads = Rs. 9,45,000 2. Finished Goods Turnover Ratio = or

CGS =5 Stock of Finished Goods

15,75,000 =5 Stock of Finished Goods

∴Closing Stock of Finished Goods = 15,75,000 ÷ 5 = Rs. 3,15,000 In the absence of adequate information, it may be assumed that Opening Stock of Finished Goods = Closing Stock of Finished Goods = Rs. 3,15,000 ∴Closing Stock of Raw Materials = 8 months’ Consumption = 6,30,000 × 8 ÷ 12 = Rs. 4,20,000 3. Fixed Asset after the writing off depreciation @ 12.5% = Rs. 10,50,000 ∴Depreciation on Fixed Asset = 10,50,000 × 12.5 ÷ 87.5 = Rs. 1,50,000 4. Net Profit before Interest to Sales =

Net Profit before interest = 8% Sales

Net Profit before interest = 8% 21,00,000 ∴Net Profit before interest = 8% × 21,00,000 = Rs. 1,68,000

or

5. Fixed Interest Cover = or

Net PBIT =8 Annual Interest Charges

1,68,000 = 8 [ assuming that Tax = Nil] Annual Interest Charges

∴Annual Interest Charges = 1,68,000 ÷ 8 = Rs. 21,000 Here, Debentures are already issued as fixed interest charges. As the debenture interest rate is 7%, Value of 7% Debentures = 21,000 × 100 ÷ 7 = Rs. 3,00,000 ∴Net Profit after interest & tax = Net Profit before Interest – Interest – Tax = 1,68,0004 – 21,000 – Nil = Rs. 1,47,000 6. In a Balance Sheet, Proprietors’ Fund + Long-term Loan = Fixed Asset + Working Capital Here, Proprietors’ Fund + 3,00,000 = 10,50,000 + 9,00,000 ∴Proprietors’ Fund = Rs. 16,50,000 Now, Reserve to Capital = 0.20

Modified Date: Sat, Jul 03, 2010 12:42:12 PM

Output Date: Tue, Jul 06, 2010 11:40:08 AM

Rev II

Project: Management Accounting_Debarshi Bhattacharyya ACE Pro India Pvt. Ltd. File: X:\Pearson\Management Accounting_Debarshi Bhattacharyya\MAIN\M03\LAYOUT_M03\M03_DEBA_ISBN_EN_SE_C03_Part-2.indd

146

MANAGEMENT ACCOUNTING

or

Reserves & Surplus 1 = Share Capital 5

∴Share Capital = 5 × Reserves & Surplus Again, Proprietors’ Fund = Share Capital + Reserves & Surplus – Miscellaneous Expenditure or

16,50,000 = 5 × Reserves & Surplus + Reserves & Surplus – Nil [assuming that Miscellaneous Expenditure = Nil]

∴Reserves & Surplus = 16,50,000 ÷ 6 = Rs. 2,75,000 ∴ Share Capital = 5 × 2,75,000 = Rs. 13,75,000 7. Current Ratio =

Current Assets (CAs) 5 = 2.5 = Current Liabilities (CLs) 2

∴ Working Capital = CAs − CLs = 5 − 2 = 3 Again, given Working Capital = Rs. 9,00,000 ∴Current Liabilities (CLs) = 2/3 × Rs. 9,00,000 = Rs. 6,00,000 ∴ Current Assets (CAs) = 5/3 × Rs. 9,00,000 = Rs. 15,00,000 Again, Quick Ratio =

Liquid Assets (CAs – Stock ) = = 1.5 Liquid Liabilities ( CLs – Bank Overdraft )

Here, ⎡⎣15,00,000 – (3,15,000 + 4,20,000 )⎤⎦ = 1.5 (6,00,000 − Bank Overdraft ) or

7,65,000 = 9,00,000 − 1.5 × Bank Overdraft

∴Bank Overdraft = 1,35,000 ÷ 1.5 = Rs. 90,000 Here, Current Liabilities (CLs) = Quick Liabilities + Bank Overdraft or

6,00,000 = Quick Liabilities + 90,000

∴Quick Liabilities = Rs. 5,10,000 8. Debt Collection Period = 2 months ∴ Debtors’ Turnover Ratio =

12 months =6 2 months

or

Credit Sales = 6 [ assumed that all sales were made on credit ] Receivables

or

21,00,000 =6 Receivables

∴Receivables = 21,00,000 ÷ 6 = Rs. 3,50,000 9. Current Asset = Stock of Finished Goods + Stock of Raw Materials + Receivables + Cash & Bank or

15,00,000 = 3,15,000 + 4,20,000 + 3,50,000 + Cash & Bank

∴Cash & Bank = Rs. 4,15,000

Modified Date: Sat, Jul 03, 2010 12:42:12 PM

Output Date: Tue, Jul 06, 2010 11:40:08 AM

Rev II

Project: Management Accounting_Debarshi Bhattacharyya ACE Pro India Pvt. Ltd. File: X:\Pearson\Management Accounting_Debarshi Bhattacharyya\MAIN\M03\LAYOUT_M03\M03_DEBA_ISBN_EN_SE_C03_Part-3.indd

ACCOUNTING RATIOS FOR FINANCIAL STATEMENT ANALYSIS

147

Problem 34 The financial information of Good Luck Ltd for the year 2000 are given as follows: Ratio of Current Asset to Current Liabilities Liquidity Ratio [Debtors & Bank Balance to Current Liabilities] Issued Capital [Equity Shares of Rs. 10 each] Net Current Asset [As over Current Liabilities] Fixed Assets [Net Block] percentage of Shareholders’ Equity as on Closing Date Gross Profit [% of Turnover] Annual Rate of Turnover of Stock [Based on the Cost on 31 December 2000] Average Age of Outstanding Debtors for the year 2000 Net Profit [% on Issued Share Capital]

1.75:1 1.25:1 Rs. 1,20,000 Rs. 60,600 60% 20% 5.26 times 2 months 16%

On 31 December, Current Assets consisted of stock, debtors and bank balances. You are required to prepare trading and Profit & Loss A/c and Balance Sheet for the year that ended on 31 December 2000. [B.Com. (Hons), Calcutta University Part II—2002] Solution Books of Good Luck Ltd Trading and Profit & Loss A/c for the year that ended on 31 December 2000 Dr. Amount Rs. 2,12,504 53,126 2,65,630 33,926 19,200 53,126

Particulars To Cost of goods sold To Gross Profit c/d2

2

To Other Operating Expenses3 To Net Profit3

Particulars By Sales

2

Cr. Amount Rs. 2,65,630 2,65,630 53,126

By Gross Profit b/d

53,126

Balance Sheet as on 31 December 2000 Amount Rs. 1,20,000 31,500 Nil 80,800

Liabilities Share Capital [of Rs. 10 Each] Reserves & Surplus5 Long-term Loans Current Liabilities1

Assets Fixed Assets5 Current Assets: Stock1 Debtors4 Bank4

2,32,300

Amount Rs. 90,900 40,400 44,272 56,728 2,32,300

Working Notes 1. Ratio of Current Assets to Current Liabilities = 1.75:1 ∴

Current Assets (CAs) 1.75 = Current Liabilities (CLs) 1

∴Net Current Assets [over CLs, i.e., Working Capital] = CAs – CLs = 1.75 − 1 = 0.75 Again, given Net Current Assets [over CLs, i.e., Working Capital] = Rs. 60,600 (Continued)

Modified Date: Sat, Jul 03, 2010 12:42:06 PM

Output Date: Tue, Jul 06, 2010 11:43:37 AM

Rev II

Project: Management Accounting_Debarshi Bhattacharyya ACE Pro India Pvt. Ltd. File: X:\Pearson\Management Accounting_Debarshi Bhattacharyya\MAIN\M03\LAYOUT_M03\M03_DEBA_ISBN_EN_SE_C03_Part-3.indd

148

MANAGEMENT ACCOUNTING

∴Current Assets (CAs) = 1.75/0.75 × 60,600 = Rs. 1,41,400 ∴Current Liabilities (CLs) = 1/0.75 × 60,600 = Rs. 80,800 Now, Liquidity Aatio =

(Debtors + Bank ) = 1.25 CLs

1

Here, CAs = Stock + Debtors + Bank ∴Debtors + Bank = CAs − Stock ∴

(CAs − Stock) = 1.25 CLs

or (1,41,400 − Stock) ÷ 80,800 = 1.25 or 1,01,000 = 1,41,400 − Stock ∴ Stock = Rs. 40,400 2. Average Rate of turnover of Stock [based on cost] = 5.26 Cost of Goods Sold or = 5.26 Average Stock Here, in the absence of adequate information, it is considered that Opening Stock = Closing Stock ∴ Average Stock = Closing Stock = Rs. 40,400 or

Cost of Goods Sold = 5.26 Closing Stock

or

Cost of Goods Sold = 5.26 40,400

∴ Cost of Goods Sold = Rs. 2,12,504 Again, Gross Profit = 20% on Sales = 1/5 on Sales = 1/4 on Cost of Goods Sold = 1/4 on 2,12,504 = Rs. 53,126 ∴ Sales (i.e., Turnover) = Cost of Goods Sold + Gross Profit = 2,12,504 + 53,126 = Rs. 2,65,630 3. Net Profit = 16% on Share Capital = 16% on 1,20,000 = Rs. 19,200 ∴ Other Operating Expenses Debited to Profit & Loss A/c = Gross Profit − Net Profit = 53,126 – 19,200 = Rs. 33,926

4. Average Collection Period = 2 months Debtors’ Velocity Ratio =

12 months =6 2 months

or

Credit Sales =6 Debtors

or

2,65,630 = 6 [ assuming that the total sales were made on credit ] Debtors

∴ Debtors = Rs. 44,272 Here, CAs = Stock + Debtors + Bank or 1,41,400 = 40,400 + 44,272 + Bank ∴ Bank = Rs. 56,728 5. Fixed Asset (FA)% of Shareholders’ Equity (i.e., Net Worth or NW) = 60% or or

FAs = 60% NW FAs = 0.60 NW (Continued)

Modified Date: Sat, Jul 03, 2010 12:42:06 PM

Output Date: Tue, Jul 06, 2010 11:43:37 AM

Rev II

Project: Management Accounting_Debarshi Bhattacharyya ACE Pro India Pvt. Ltd. File: X:\Pearson\Management Accounting_Debarshi Bhattacharyya\MAIN\M03\LAYOUT_M03\M03_DEBA_ISBN_EN_SE_C03_Part-3.indd

ACCOUNTING RATIOS FOR FINANCIAL STATEMENT ANALYSIS

149

∴ Fixed Asset = 0.60 NW Again, as per company Balance Sheet, we know Net Worth (NW) + Long-term Loan (LTL) = Fixed Asset (FA) + Working Capital (WC) Here, NW + Nil = 0.60 NW + 60,600 [assumed that there is no Long-term loan] or NW − 0.60 NW = 60,600 or NW = 60,600 ÷ 0.40 ∴ Net Worth = Rs. 1,51,500 ∴ Fixed Assets = 0.60 × 1,51,500 = Rs. 90,900 Again, Net Worth = Equity Share Capital + Reserves & Surplus [as there is no Preference Capital or Miscellaneous Expenditure] or 1,51,500 = 1,20,000 + Reserves & Surplus ∴ Reserves & Surplus = Rs. 31,500

Problem 35 From the following information, prepare Profit & Loss A/c for the year that ended on 31 March 2008 and Balance Sheet as on that date of Nimbus Ltd: Quick Ratio Current Ratio Gross Profit Ratio Stock Turnover Ratio Debtors’ Turnover Creditors’ Velocity Operating Ratio Fixed Asset Proprietorship Ratio Earning per Share Earnings for the year as a Perentage of Share Capital Nominal Value Per Share No. of Shares Allotted No. of Working Days in a year Working Capital

1.4 2.5 40% 5 times 36 days 54 days 90% 70% Re. 1 25% Rs. 5 25,000 360 Rs. 45,000

Opening Stock was Rs. 4,000 less than the Closing Stock. There were no long-term loans, deferred expenses and prepaid expenses. All sales and purchases were made on credit. Solution Books of Nimbus Ltd Profit & Loss A/c for the year that ended on 31 March 2008 Dr. Particulars To Materials Consumed1 To Purchases3 To Gross Profit c/d To Indirect Operating Expenses5 To Non-operating Expenses (Bal. fig.) To Net Profit6

Modified Date: Sat, Jul 03, 2010 12:42:06 PM

Amount Rs. 29,000 1,59,000 1,03,333 2,91,333 77,500 833 25,000 1,03,333

By Sales2 By Closing Stock1

Cr. Amount Rs. 2,58,333 33,000

By Gross Profit b/d

2,91,333 1,03,333

Particulars

Output Date: Tue, Jul 06, 2010 11:43:37 AM

1,03,333

Rev II

Project: Management Accounting_Debarshi Bhattacharyya ACE Pro India Pvt. Ltd. File: X:\Pearson\Management Accounting_Debarshi Bhattacharyya\MAIN\M03\LAYOUT_M03\M03_DEBA_ISBN_EN_SE_C03_Part-3.indd

150

MANAGEMENT ACCOUNTING

Balance Sheet as on 31 March 2008 Amount Rs.

Liabilities Share Capital: 25,000 Shares of Rs. 5 Each, Paid up to Rs. 4 Per Share6 Reserves & Surplus: Reserve7 Profit & Loss A/c7 Current Liabilities: Payables3 Other Current Liabilities3

1,00,000 25,000 25,000

Assets Fixed Assets7 Current Assets: Stock1 Receivables4 Cash & Bank4

23,850 6,150 1,80,000

Amount Rs. 1,05,000

33,000 25,833 16,167

1,80,000

Working Notes 1. Current ratio =

Current Assets (CAs) 5 = 2.5 = Current Liabilities (CLs) 2

∴ Working Capital = CAs − CLs = 5 − 2 = 3 Again, given Working Capital = Rs. 45,000 ∴ Current Liabilities (CLs) = 2/3 × Rs. 45,000 = Rs. 30,000 ∴ Current Assets (CAs) = 5/3 × Rs. 45,000 = Rs. 75,000 Liquid Assets (CAs − Stock) Again, Quick Ratio = = = 1.4 Liquid Liabilities (CLs − Bank Overdraft) (75,000 − Stock) Here, = 1.4 [ assuming that Bank Overdraft = Nil] (30,000 − Nil) or 42,000 = 75,000 – Stock ∴ Stock (i.e., Closing Stock) = Rs. 33,000 ∴ Opening Stock = 33,000 – 4,000 = Rs. 29,000 (Opening Stock + Closing Stock) (29,000 + 33,000) = = Rs. 31,000 2 2 Cost of Goods Sold Now, Stock Turnover Ratio = Average Stock Cost of Goods Sold Here, 5 = 31,000 ∴ Cost of Goods Sold = Rs. 1,55,000. ∴ Sales = Cost of Goods Sold + Gross Profit = Cost of Goods Sold + (40% on Sales) = Cost of Goods Sold + (2/5 on Sales) = Cost of Goods Sold + (2/3 on CGS) = 1,55,000 + (2/3 of 1,55,000) = Rs. 2,58,333

2. Average Stock =

3. We know, Opening Stock + Purchases = CGS + Closing Stock Here, 29,000 + Purchases = 1,55,000 + 33,000 ∴ Purchases = Rs. 1,59,000 Again, Creditors’ Velocity = 54 days 360 days 20 ∴ Creditors’ Velocity Ratio = = 54 days 3 Credit Purchases 20 or = Payables 3 (Continued)

Modified Date: Sat, Jul 03, 2010 12:42:06 PM

Output Date: Tue, Jul 06, 2010 11:43:37 AM

Rev II

Project: Management Accounting_Debarshi Bhattacharyya ACE Pro India Pvt. Ltd. File: X:\Pearson\Management Accounting_Debarshi Bhattacharyya\MAIN\M03\LAYOUT_M03\M03_DEBA_ISBN_EN_SE_C03_Part-3.indd

ACCOUNTING RATIOS FOR FINANCIAL STATEMENT ANALYSIS

or

151

1,59,000 20 = [as here, Total Purchases = Credit Purchases] Payables 3

∴ Payables = 1,59,000 ÷ 20 × 3 = Rs. 23,850 Here, Current Liabilities = Payables + Other Current Liabilities or 30,000 = 23,850 + Other Current Liabilities ∴ Other Current Liabilities = Rs. 6,150 4. Debtors’ Turnover = 36 days 360 days ∴Debtors’ Velocity Ratio = = 10 36 days or

Credit Sales = 10 Receivables

or

2,58,333 = 4 [ as here, Total Sales = Credit Sales] Receivables

∴ Receivables = 2,58,333 ÷ 10 = Rs. 25,833 Here, Current Assets = Stock + Receivables + Cash & Bank or 75,000 = 33,000 + 25,833 + Cash & Bank ∴ Cash & Bank = Rs. 16,167 5. Operating Ratio = Here, 90 =

(CGS + Indirect operating Expenses) × 100 Sales

(1,55,000 + Indirect Operating Expenses) × 100 2,58,333

or Indirect Operating Expenses + 1,55,000 = 2,58,333 × 90 ÷ 100 ∴ Indirect Operating Expenses = 2,32,500 – 1,55,000 = Rs. 77,500 6. No. of Shares Allotted = 25,000 Again, EPS (EPS) = Here,

1=

NP ( i.e., Earnings) for the year No. of Shares

Net Profit for the year 25,000

∴ Net Profit (i.e., Earnings) for the year = Rs. 25,000 Again, Earnings for the year = 25% of Share Capital or 25,000 = 25/100 × Share Capital ∴ (Paid up) Share Capital = 25,000 × 100 ÷ 25 = Rs. 1,00,000 Paid up Share Capital Rs. 1,00,000 = = Rs. 4 No. of Shares 25,000 Fixed Assets (FA) 7. Fixed Asset Proprietorship Ratio = Proprietors’ Fund (PF) Paid up Value per Share =

FAs PF ∴ FA = 0.70 PF Again, as per company Balance Sheet, we know Proprietors’ Fund (PF) + Long-term Loan (LTL) = Fixed Asset (FA) + Working Capital (WC) Here, PF + Nil = 0.7 PF + 45,000 [as there is no Long-term Loan] or 0.30 PF = 45,000

Here, 70% =

(Continued)

Modified Date: Sat, Jul 03, 2010 12:42:06 PM

Output Date: Tue, Jul 06, 2010 11:43:37 AM

Rev II

Project: Management Accounting_Debarshi Bhattacharyya ACE Pro India Pvt. Ltd. File: X:\Pearson\Management Accounting_Debarshi Bhattacharyya\MAIN\M03\LAYOUT_M03\M03_DEBA_ISBN_EN_SE_C03_Part-3.indd

152

MANAGEMENT ACCOUNTING

∴ Proprietors’ Fund = 45,000 ÷ 0.30 = Rs. 1,50,000 ∴ Fixed Assets = 0.70 × 1,50,000 = Rs. 1,05,000 Again, Proprietors’ Fund = Paid-up Share Capital + Reserves & Surplus [as there is no miscellaneous expenditure] or 1,50,000 = 1,00,000 + Reserves & Surplus ∴ Reserves & Surplus = Rs. 50,000 Here, Reserves & Surplus = Reserves + Profit & Loss A/c or 50,000 = Reserves + 25,0006 Reserves = Rs. 25,000

Problem 36 Akash Ltd commenced the manufacture of personal computers on 1 April 2007, with an equity capital of Rs. 5,00,000 in shares of Rs. 10 each. The following details are gathered from the accounting records of the company for the year that ended on 31 March 2008: Gross Profit Ratio Current Ratio Net Profit Ratio Stock Turnover Ratio [Based on Closing Stock] Debtors’ Turnover Ratio Interest Coverage Ratio Sales to Working Capital Sales to Net Fixed Assets Credit Sales Long-term Debt/Equity Ratio

30% 2 10% 7 times 6 times 5 times 8 times 4 times 75% 1:2 1 33 % 3

Provision for Income Tax Proposed Dividend [Assume that it is not taxable] Investments as on 31 March 2008 Selling & Distribution Expenses [50% was Outstanding on 31 March 2008] Depreciation Rate [Depreciation was not part of the CGS]

20% Rs. 1,50,000 Rs. 1,00,000 20%

You are required to prepare: (a) Profit & Loss A/c for the year that ended on 31 March 2008. (b) Balance Sheet of the company as on that date. [C.A. (Final)—Adapted] Solution Books of Akash Ltd Profit & Loss A/c for the year that ended on 31 March 2008 Rs. Less: Less:

Sales1 Cost of Goods Sold3 Gross Profit3 Selling & Distribution Expenses Depreciation

1,00,000 1,25,000

Rs. 20,00,000 14,00,000 6,00,000 2,25,000 (Continued)

Modified Date: Sat, Jul 03, 2010 12:42:06 PM

Output Date: Tue, Jul 06, 2010 11:43:37 AM

Rev II

Project: Management Accounting_Debarshi Bhattacharyya ACE Pro India Pvt. Ltd. File: X:\Pearson\Management Accounting_Debarshi Bhattacharyya\MAIN\M03\LAYOUT_M03\M03_DEBA_ISBN_EN_SE_C03_Part-3.indd

153

ACCOUNTING RATIOS FOR FINANCIAL STATEMENT ANALYSIS

Rs.

Rs. 3,75,000 75,000 3,00,000 1,00,000 2,00,000 1,00,000 1,00,000

Rs. 6,25,000 1,25,000

Rs.

Profit before Interest & Tax Interest on Loan4 Profit before Tax Provision for Tax [33 13 % on Rs. 3,00,000] Profit after Tax Proposed Dividend [20% on Share Capital] Balance of Profit carried over to Balance Sheet

Less: Less: Less:

Balance Sheet as on 31 March 2008 Liabilities Equity Share Capital [50,000 Shares of Rs. 10 each] Reserves & Surplus: Profit & Loss A/c 9 Long-term Debt9 Current Liabilities & Provisions: Outstanding Selling & Distribution Overheads Provision for Tax Proposed Dividend

Rs. 5,00,000

1,00,000 3,00,000

Assets Fixed Assets2 Less: Depreciation Investment Current Assets: Stock7 Receivables6 Cash & Bank8

50,000 1,00,000 1,00,000 11,50,000

5,00,000 1,50,000 2,00,000 2,50,000 50,000

11,50,000

Working Notes 1. Let the Sales be 100x. Again, Sales to Net Fixed Assets = 4 Sales =4 or Net FAs 100x =4 or Net FAs 100x = 25x ∴ Net FAs = 4 Again, Depreciation = 20% on Fixed Assets ∴Depreciation = 20/80 on Net FAs = 20 ÷ 80 × 25x = 6.25

Less:

Now, GP [30% of x] Selling & Distribution Expenses

Less:

Depreciation PBIT

Again, Interest Coverage Ratio = Here, 5 =

Rs. 30x 1,00,000 30x − 1,00,000 6.25x 23.75x − 1,00,000

PBIT Interest on Debt(I)

(23.75x –1,00,000) I

Interest on Debt (I) =

(23.75x –1,00,000) = 4.75x − 20,000 5 (Continued)

Modified Date: Sat, Jul 03, 2010 12:42:06 PM

Output Date: Tue, Jul 06, 2010 11:43:37 AM

Rev II

Project: Management Accounting_Debarshi Bhattacharyya ACE Pro India Pvt. Ltd. File: X:\Pearson\Management Accounting_Debarshi Bhattacharyya\MAIN\M03\LAYOUT_M03\M03_DEBA_ISBN_EN_SE_C03_Part-3.indd

154

MANAGEMENT ACCOUNTING

Less:

Now, PBIT Interest on Debt Profit before Tax

Less:

⎡1 ⎤ Tax @ 33 13 % on PBT ⎢ % on (1x − 80, 000)⎥ 3 ⎣ ⎦

Rs. 23.75x – 1,00,000 4.75x − 20,000 19x − 80,000 1 (19x − 80,000) 3 2 Profit after Tax

Again, NP Ratio =

3

(19x − 80,000)

PAT Sales

⎡2 ⎤ ⎢⎣ 3 (19x − 80,000)⎥⎦ or 10% = 100x or 10x = 2 / 3 (19x − 80,000) or 38x − 1,60,000 = 30x or 8x = 1,60,000 ∴ x = 1,60,000 ÷ 8 = 20,000 ∴ Sales = 100x = 100 × 20,000 = Rs. 20,00,000 2.

Add:

Net Fixed Assets [25x = 25 × 20,000] Depreciation [6.25x = 6.25 × 20,000] Fixed Assets before Depreciation

Rs. 5,00,000 1,25,000 6,25,000

3. GP = 30% of Sales = 30% of 20,00,000 = Rs. 6,00,000 ∴ CGS = Sales − GP = 20,00,000 − 6,00,000 = Rs. 14,00,000 4. Interest on debt = 4.75x − 20,000 = (4.75 × 20,000) − 20,000 = Rs. 75,000 5. Sales to Working Capital (WC) = 8 or

Sales =8 WC

20,00,000 =8 WC ∴ WC = 20,00,000 ÷ 8 = Rs. 2,50,000 CAs 2 =2= Again, Current Ratio = CLs 1 ∴ WC = CAs − CLs = 2 − 1 = 1 ∴ CAs = 2/1 × 2,50,000 = Rs. 5,00,000 ∴ CLs = 1/1 × 2,50,000 = Rs. 2,50,000 6. Credit Sales = 75% [of Total Sales] = 75% of 20,00,000 = Rs. 15,00,000 or

Now, Debtors’ Turnover Ratio = Here, 6 =

Credit Sales Receivables

15,00,000 Receivables

∴Receivables = 15,00,000 ÷ 6 = Rs. 2,50,000 (Continued)

Modified Date: Sat, Jul 03, 2010 12:42:06 PM

Output Date: Tue, Jul 06, 2010 11:43:37 AM

Rev II

Project: Management Accounting_Debarshi Bhattacharyya ACE Pro India Pvt. Ltd. File: X:\Pearson\Management Accounting_Debarshi Bhattacharyya\MAIN\M03\LAYOUT_M03\M03_DEBA_ISBN_EN_SE_C03_Part-3.indd

155

ACCOUNTING RATIOS FOR FINANCIAL STATEMENT ANALYSIS

CGS 7. Stock Turnover Ratio [based on Closing Stock ] = Closing Stock or

7=

14,00,000 Closing Stock

∴ Closing Stock = 14,00,000 ÷ 7 = Rs. 2,00,000 8. Current Assets5 = Closing Stock + Receivables + Cash & Bank or 5,00,000 = 2,00,000 + 2,50,000 + Cash & Bank ∴ Cash & Bank = Rs. 50,000 9. Here, Reserves & Surplus = Balance of Profit & Loss A/c = Rs. 1,00,000 Long-term Debt 1 Again, = Equity Ratio 2 or

Long-term Debt 1 = (Equity Share Capital + Reserves & Surplus) 2

or

Long-term Debt 1 = (5,00,000 + 1,00,000) 2

∴ Long-term Debt = 6,00,000 ÷ 2 = Rs. 3,00,000

Problem 37 Compute the following annual Financial Statements on the basis of the Ratios given as follows: Trading and Profit & Loss A/c for the year that ended on 31 March 2008 Dr. Amount Rs. 6,00,000 ? ? 10,000 ? ?

Particulars To Cost of Goods Sold To Operating Expenses To Earnings before Interest & Tax To Debenture Interest To Income Tax To Earnings after Tax

Particulars By Sales

By Earnings before Interest & Tax

Cr. Amount Rs. 20,00,000

?

Balance Sheet as on 31 March 2008 Amount Rs.

Liabilities Net Worth: Share Capital Reserves & Surplus 10% Debentures Sundry Creditors

i. ii. iii. iv. v. vi.

? ? ? 60,000

Assets Fixed Assets Current Assets: Stock Debtors Cash

Amount Rs. ? ? 35,000 ?

Net Profit to Sales—5%. Current Ratio—1.5. Return on Net Worth—20%. Inventory turnover—15 times. Share Capital to Reserves 4:1. Rate of Income Tax—50%.

Modified Date: Sat, Jul 03, 2010 12:42:06 PM

Output Date: Tue, Jul 06, 2010 11:43:37 AM

Rev II

Project: Management Accounting_Debarshi Bhattacharyya ACE Pro India Pvt. Ltd. File: X:\Pearson\Management Accounting_Debarshi Bhattacharyya\MAIN\M03\LAYOUT_M03\M03_DEBA_ISBN_EN_SE_C03_Part-3.indd

156

MANAGEMENT ACCOUNTING

Solution Trading and Profit & Loss A/c for the year that ended on 31 March 2008 Dr. Amount Rs. 6,00,000 11,90,000 2,10,000 20,00,000 10,000 1,00,000 1,00,000 2,10,000

Particulars To Cost of Goods Sold To Operating Expenses2 To Earnings before Interest & Tax1 To Debenture Interest To Income Tax1 To Earnings after Tax1

By Sales

Cr. Amount Rs. 20,00,000

By Earnings before Interest & Tax

20,00,000 2,10,000

Particulars

2,10,000

Balance Sheet as on 31 March 2008 Amount Rs.

Liabilities Net Worth: Share Capital4 Reserves & Surplus4 10% Debentures3 Sundry Creditors

4,40,000 1,10,000 1,00,000 60,000 7,10,000

Assets Fixed Assets7 Current Assets: Stock5 Debtors Cash6

Amount Rs. 6,20,000 40,000 35,000 15,000 7,10,000

Working Notes 1. NP to Sales = 5% NP = 5% or Sales or

NP = 5% 20,00,000

∴ NP (i.e., EAT) = 5% of 20,00,000 = Rs. 1,00,000 Here, Rate of Income Tax = 50% [of Earnings before Tax] ∴ Income Tax = 50/50 × EAT = 50/50 × 1,00,000 = Rs. 1,00,000 ∴ Earnings before Interest & Tax (EBIT) = EAT + Income Tax + Debenture Interest = 1,00,000 + 1,00,000 + 10,000 = Rs. 2,10,000 2. Operating Expenses = Sales − (CGS + EBIT) = 20,00,000 − (6,00,000 + 2,10,000) = Rs. 11,90,000 3. Debenture Interest @ 10% = Rs. 10,000 ∴ 10% Debentures = 100/10 × 10,000 = Rs. 1,00,000 4. Return on Net Worth = Here, 20 =

(EBIT − Tax) × 100 Net Worth

(2,10,000 –1,00,000) × 100 NW

∴ Net Worth = 1,10,000 ÷ 20 × 100 = Rs. 5,50,000 (Continued)

Modified Date: Sat, Jul 03, 2010 12:42:06 PM

Output Date: Tue, Jul 06, 2010 11:43:37 AM

Rev II

Project: Management Accounting_Debarshi Bhattacharyya ACE Pro India Pvt. Ltd. File: X:\Pearson\Management Accounting_Debarshi Bhattacharyya\MAIN\M03\LAYOUT_M03\M03_DEBA_ISBN_EN_SE_C03_Part-3.indd

ACCOUNTING RATIOS FOR FINANCIAL STATEMENT ANALYSIS

157

Again, Share Capital to Reserves = 4:1 or

Share Capital 4 = Reserves & Surplus 1

∴ Share Capital = 4 × Reserves & Surplus Here, Net Worth = Share Capital + Reserves & Surplus or 5,50,000 = 4 × Reserves & Surplus + Reserves & Surplus or 5 × Reserves & Surplus = 5,50,000 ∴ Reserves & Surplus = Rs. 1,10,000 ∴ Share Capital = 4 × 1,10,000 = Rs. 4,40,000 5. Inventory Turnover = 15 times or

CGS = 15 Average Stock

or

6,00,000 = 15 Average Stock

∴ Average Stock = 6,00,000 ÷ 15 = Rs. 40,000 Here, in the absence of adequate information, it is assumed that, Opening Stock = Closing Stock ∴ Closing Stock = Average Stock = Rs. 40,000 6. Current Ratio = or

CAs = 1.5 CLs

CAs = 1.5 [Here, CLs = Sundry Creditors only ] 60,000

∴ CA = 60,000 × 1.5 = Rs. 90,000 Here, CAs = Stock + Debtors + Cash or 90,000 = 40,000 + 35,000 + Cash ∴Cash = Rs. 15,000 7. In every Balance Sheet, Total Liabilities = Total Assets Here, Total Liabilities = Rs. 7,10,000 ∴ Total Assets = Rs. 7,10,000 Again, Total Assets = Fixed Assets + Current Assets or 7,10,000 = FAs + 90,000 ∴ Fixed Assets = Rs. 6,20,000

Problem 38 Given below are Cash Position Ratios of X Ltd and the industry average. Industry average is arrived at by taking the average position of 25 companies of the similar trade: Absolute Liquid Ratios X Ltd Industry Average

0.36 0.30

Cash Position to Total Asset Ratio 12.5% 15%

Cash Interval 25 days 33 days

How do you feel about the cash position of X Ltd? [B.Com. (Hons), Calcutta University—2008]

Modified Date: Sat, Jul 03, 2010 12:42:06 PM

Output Date: Tue, Jul 06, 2010 11:43:37 AM

Rev II

Project: Management Accounting_Debarshi Bhattacharyya ACE Pro India Pvt. Ltd. File: X:\Pearson\Management Accounting_Debarshi Bhattacharyya\MAIN\M03\LAYOUT_M03\M03_DEBA_ISBN_EN_SE_C03_Part-3.indd

158

MANAGEMENT ACCOUNTING

Solution Absolute Liquid Ratio =

Cash in hand & at bank + Short-termInvestment

Liquid Liabilities This Ratio indicates the position of cash and cash equivalents of a concern to pay off its liquid liabilities. Cash Position to Total Asset Ratio =

Cash in hand & at bank + Short-term Investments Total Assets

This Ratio indicates the proportion of Liquid Assets out of the Total Assets deployed by a concern. Cash interval expresses an idea about the time length that can be covered by the available cash and cash equivalents for meeting its operating expenses. After analysing the above comparative data of X Ltd and the industry average, the following observations can be identified: i. Absolute Liquid Ratio of X Ltd is better than the industry average. ii. Cash position to Total Asset Ratio of X Ltd is lower than the industry average. iii. Cash interval of X Ltd is also lower than that of the industry average. Hence, it can be concluded by an overall analysis of the above comparative data that X Ltd maintains a comparatively lower cash position than the industry average. Therefore, it is advised to increase the cash position of X Ltd. Problem 39 The operating performance of three divisions of a company for the year that ended on 31 March 2006 is given as follows: Sales Operating Profit Investment

Division I Rs. 2,00,000 25,000 2,50,000

Division II Rs. 2,50,000 25,000 2,00,000

Division III Rs. 3,00,000 27,000 2,00,000

i. Identify the most profitable division on the basis of: (a) Operating Profit Margin; and (b) ROI. ii. Explain with reasons which of the above two measures provides a better indication of the overall operating performance of all the divisions. [B.B.A. (Hons), Calcutta University—2007] Solution

Sales Operating Profit Investment (a) Operating Profit Margin ⎡ Operating Profit ⎤ × 100 ⎥ ⎢ Sales ⎣ ⎦ ⎡ Operating Profit ⎤ × 100 ⎥ (b) ROI ⎢ Sales ⎣ ⎦

Modified Date: Sat, Jul 03, 2010 12:42:06 PM

Division I Rs. 2,00,000 25,000 2,50,000

Division II Rs. 2,50,000 25,000 2,00,000

Division III Rs. 3,00,000 27,000 2,00,000

25,000 × 100 =12.50% 2,00,000

25,000 × 100 =10% 2,50,000

27,000 × 100 = 9% 3,00,000

25,000 × 100 = 12.50% 2,00,000

27,000 × 100 = 13.50% 2,00,000

25,000 × 100 =10% 2,50,000

Output Date: Tue, Jul 06, 2010 11:43:37 AM

Rev II

Project: Management Accounting_Debarshi Bhattacharyya ACE Pro India Pvt. Ltd. File: X:\Pearson\Management Accounting_Debarshi Bhattacharyya\MAIN\M03\LAYOUT_M03\M03_DEBA_ISBN_EN_SE_C03_Part-3.indd

ACCOUNTING RATIOS FOR FINANCIAL STATEMENT ANALYSIS

159

i. a. On the basis of Operating Profit Margin, Division I is the most profitable division as it gives the highest profit margin on sales. b. On the basis of ROI, Division III is the most profitable division as it gives the highest return on the total investment. ii. Success of a concern is measured by the return to its owner from the business at the end of an accounting period. Although the operating success of the concern is reflected through its Operating Profit Margin, but its overall operating performance is evaluated by the return that it has been earning from its total Capital Employed in the business and return to the owner of the business. Accordingly, ROI (i.e., Return on the Total Capital Employed) provides a better indication of the overall operating performance of a concern, as it reflects the return against the total long-term fund invested into the business. In spite of earning a good Operating Profit Margin, a concern cannot sustain in the long run if it does not get a return on its total investment.

Problem 40 From the following information of Sunshine Ltd, for the year that ended on 31 March 2009, examine the details from the point of view of: (i) Solvency position; (ii) Profitability position; and (iii) Activity position of the company. Also comment on the condition of the business. Profit & Loss A/c for the year that ended on 31 March 2009 Dr. Particulars To Opening Stock To Purchases To Gross Profit c/d To Trade Expenses To Interest on debenture To Provision for Tax To Net Profit

Amount Rs. 4,00,000 16,00,000 7,00,000 27,00,000 3,35,000 25,000 50,000 3,00,000 7,10,000

By Sales By Closing Stock

Cr. Amount Rs. 21,00,000 6,00,000

By Gross Profit b/d By Interest

27,00,000 7,00,000 10,000

Particulars

7,10,000

Balance Sheet as on 31 March 2009 Liabilities Equity Share Capital 8% Preference Share Capital General Reserve Profit & Loss A/c 5% Debenture Sundry Creditors Bank Overdraft Provision for Taxation

Amount Rs. 12,00,000 8,00,000 1,00,000 2,00,000 5,00,000 9,00,000 50,000 50,000 38,00,000

Assets Plant & Machinery Land & Building Furniture & Fixture Stock in Trade Debtors Bills Receivable Cash

Amount Rs. 10,00,000 10,00,000 6,00,000 6,00,000 3,80,000 2,00,000 20,000 38,00,000

[B.Com. (Hons), Calcutta University—Adapted]

Modified Date: Sat, Jul 03, 2010 12:42:06 PM

Output Date: Tue, Jul 06, 2010 11:43:37 AM

Rev II

Project: Management Accounting_Debarshi Bhattacharyya ACE Pro India Pvt. Ltd. File: X:\Pearson\Management Accounting_Debarshi Bhattacharyya\MAIN\M03\LAYOUT_M03\M03_DEBA_ISBN_EN_SE_C03_Part-3.indd

160

MANAGEMENT ACCOUNTING

Solution A. Test of Solvency Positions: I. Short-term solvency position: i. Current Ratio = ii. Liquid Ratio =

CAs 12,00,0001 = = 1.2:1 CLs 10,00,0002

(CAs − Stock ) Liquid Assets 6,00,0001 = = = 0.63 :1 Liquid Liabilities (CLs – Bank Overdraft ) 9,50,0002

 Comment Both Current Ratio and Liquid Ratio of the company are lower than the Ideal Ratios of 2:1 and at least 1:1, respectively. Here, the company does not have sufficient Liquid Assets to pay off its liquid liabilities (as the Liquid Ratio is less than 1). On the other hand, the Current Assets of the company are marginally exceeding its Current Liabilities, not the the double as it should be. Hence, the short-term solvency position of the company is well below the ideal level and the company should try to improve its short-term solvency position.

II. Long-term solvency position: i.

Proprietary Ratio =

ii. Debt-Equity Ratio =

Proprietors’ Fund 23,00,0003 = = 0.6053 :1 Total Assets 38,00,000 Long-term Debt Debt = Equity Proprietors’ Fund

= 5,00,000 ÷ 23,00,0003 = 0.2174 : 1 iii. Capital Gearing Ratio = Fixed Interest Bearing Securities Ordinary Securities =

(Debenture + Preference Share Capital ) Equity Shareholders’ Fund

= (5,00,000 + 8,00,000) ÷ 15,00,0003 = 0.87 : 1 iv. Interest Coverage Ratio =

Net PBIT 3,75,0004 = = 15 Interest 25,000

 Comment Proprietary Ratio of the company shows that more than 60% of its assets are funded by its own capital. Debt-Equity Ratio expresses that the own capital invested in the business is almost 5 times of the debt capital. The Capital Gearing Ratio exhibits that it is a low-geared company, as the said Ratio is just below 1. The Interest Coverage Ratio shows that the annual interest of the company is covered by its profit by as high as 15 times. Therefore, as far as the long-term solvency is concerned, it can be concluded that all the above Ratios indicate the sound solvency position of the company, even though the company is unable to reap the benefit of trading on equity through more long-term borrowings.

Modified Date: Sat, Jul 03, 2010 12:42:06 PM

Output Date: Tue, Jul 06, 2010 11:43:37 AM

Rev II

Project: Management Accounting_Debarshi Bhattacharyya ACE Pro India Pvt. Ltd. File: X:\Pearson\Management Accounting_Debarshi Bhattacharyya\MAIN\M03\LAYOUT_M03\M03_DEBA_ISBN_EN_SE_C03_Part-3.indd

ACCOUNTING RATIOS FOR FINANCIAL STATEMENT ANALYSIS

161

B. Test of Profitability Positions: 1. GP Ratio =

GP 7,00,000 × 100 = × 100 = 33 1 3 % Sales 21,00,000

2. NP Ratio =

Net PAT 3,00,000 4 × 100 = × 100 = 14.29% Sales 21,00,000

3. Operating Profit Ratio =

Operating Profit before Interest & Tax ( EBIT)

Sales = 3,65,0004 ÷ 21,00,000 × 100 = 17.38%

4. Return on Net Worth = 5. ROI =

× 100

Net PAT 3,00,0004 × 100 = × 100 = 13.04% Net Worth 23,00,0003

Net Profit before Interest but after Tax 3,25,0004 × 100 = × 100 = 11.61% Capital Employed 28,00,0003

6. ROE Shareholders’ Fund = 7. Financial Leverage =

Net PAT − Preference Dividend 2,36,0005 × 100 = × 100 = 15.73% Equity Shareholders’ Fund 15,00,0003

PBIT 3,65,0004 = = 1.0735 PBT 3,40,0004

 Comment Gross Profit Ratio shows a high rate of Gross Profit earned on sales, but the rates of Net Profit and operating profit are considerably lower than the Gross Profit rate. This is because of the huge amount of indirect operating expenses that the company has incurred during the year. Due to a similar reason, Return on Investment is also lower. Return on Net Worth is higher than the Return on Investment due to a negligible amount of interest on the borrowed capital. Return on Equity Shareholders’ Fund is also higher than the Return on Net Worth due to the low rate of preference dividend. Financial leverage equal to almost 1 indicates a negligible amount of interest on the borrowed capital. Hence, the overall profitability of the company is satisfactory with the exception of the fact that it has currently incurred a huge amount of indirect operating expenses. The company should try to reduce its indirect operating expenses so as to increase its overall return.

C. Test of Activity Positions: 1. Stock Turnover Ratio =

CGS 14,00,0006 = = 2.80 times Average Stock 5,00,0007

2. Debtors’ Turnover Ratio =

Credit Sales 21,00,000 = = 3.62 times Receivables 5,80,0008

3. Creditors’ Turnover Ratio = 4. Capital Turnover Ratio =

Modified Date: Sat, Jul 03, 2010 12:42:06 PM

Credit Purchases 16,00,000 = = 1.77 times Payables 9,00,0009

Sales 21,00,000 = = 0.75 Capital Employed 28,00,0003

Output Date: Tue, Jul 06, 2010 11:43:37 AM

Rev II

Project: Management Accounting_Debarshi Bhattacharyya ACE Pro India Pvt. Ltd. File: X:\Pearson\Management Accounting_Debarshi Bhattacharyya\MAIN\M03\LAYOUT_M03\M03_DEBA_ISBN_EN_SE_C03_Part-3.indd

162

MANAGEMENT ACCOUNTING

5. Fixed Asset Turnover Ratio =

Sales 21,00,000 = = 0.81 Fixed Asset 26,00,00011

6. Working Capital Turnover Ratio =

Sales 21,00,000 = = 10.50 WC 2,00,00010

 Comment Stock Turnover Ratio of 2.80 indicates that the company’s stock-holding period is too long. Debtors’ Turnover Ratio of 3.62 also indicates the company’s longer Debt Collection Period. Accordingly, a substantial amount of WC is being blocked for a prolonged period in stock and debtors. On the other hand, the Creditors’ Turnover Ratio of 1.77 indicates that the company’s creditors’ payment period is abnormally long, which may cast a stigma on its reputation in the market. As a combination of all the above facts, the company is having a very little amount of net WC in its hand. High Working Capital Turnover Ratio indicates that the company has the ability to generate sales in the market using a lower amount of net WC. Capital Turnover Ratio and Fixed Asset Turnover Ratio, both are near to 1, and thus support in favour of the company’s sales generating capacity with respect to the total capital invested into the business and to the amount invested into its Fixed Assets, respectively. Hence, the company has been facing with a longer-period stock-holding problem, longer-period debt-collection problem and also, abnormally longer-period creditors’ payment period, in spite of having a very satisfactory sales-generating capacity. Comment on the overall condition of the business: 1. 2. 3. 4.

Short-term solvency position is not satisfactory at all. Long-term solvency position is quite sound. Profitability position is satisfactory. As far as the sales-generating capacity is concerned, the company is performing at a highly satisfactory level, but its stock-holding period, Debt Collection Period and creditors’ payment period are too long.

Working Notes 1. Calculation of Current Assets and Liquid Assets Stock in Trade Debtors Bills Receivable Cash Current Assets Less:

Stock in Trade Liquid Assets

Rs. 6,00,000 3,80,000 2,00,000 20,000 12,00,000 6,00,000 6,00,000

2. Calculation of Current Liabilities and Liquid Liabilities Sundry Creditors Bank Overdraft Provision for Taxation Current Liabilities Less:

Bank Overdraft Liquid Liabilities

Rs. 9,00,000 50,000 50,000 10,00,000 50,000 9,50,000 (Continued)

Modified Date: Sat, Jul 03, 2010 12:42:06 PM

Output Date: Tue, Jul 06, 2010 11:43:37 AM

Rev II

Project: Management Accounting_Debarshi Bhattacharyya ACE Pro India Pvt. Ltd. File: X:\Pearson\Management Accounting_Debarshi Bhattacharyya\MAIN\M03\LAYOUT_M03\M03_DEBA_ISBN_EN_SE_C03_Part-3.indd

ACCOUNTING RATIOS FOR FINANCIAL STATEMENT ANALYSIS

163

3. Calculation of Equity Shareholders’ Fund, Proprietors’ Fund and Capital Employed Rs. 12,00,000

Equity Share Capital Reserves & Surplus: General Reserve Profit & Loss A/c

Proprietors' Fund/Net Worth

1,00,000 2,00,000 15,00,000 8,00,000 23,00,000

Capital Employed

5,00,000 28,00,000

Equity Shareholders' Fund 8% Preference Share Capital Long-term Loan: 5% Debenture

4. Calculation of Operating Profit

Add: Add: Less: Less:

Rs. 3,00,000 25,000 3,25,000 50,000 3,75,000 10,000 3,65,000 25,000 3,40,000

Net Profit after Tax as per Profit & Loss A/c [EAT] Interest on Debenture Net Profit before Interest but after Tax Provision for Tax Net PBIT Interest Earned [Being Non-operating Income] Operating PBIT Interest on Debenture Operating Profit before Tax (PBT)

5. Calculation of Profit available to Equity Shareholders

Less:

Rs. 3,00,000 64,000 2,36,000

Net Profit after Tax as per Profit & Loss A/c [EAT] Preference Dividend [8% on Rs. 8,00,000] Profit Available to Equity Shareholders

6. CGS = Sales − GP = 21,00,000 − 7,00,000 = Rs. 14,00,000 7. Average Stock =

(Opening Stock + Closing Stock) (4,00,000 + 6,00,000) = = Rs. 5,00,000 2 2

8. Receivables = Debtors + Bills Receivable = 3,80,000 + 2,00,000 = Rs. 5,80,000 9. Payables = Creditors + Bills Payable = 9,00,000 + Nil = Rs. 9,00,000 10. WC = CAs − CLs = 12,00,0001 − 10,00,0002 = Rs. 2,00,000 11. Fixed Assets = Plant & Machinery + Land & Building + Furniture & Fixture = 10,00,000 + 10,00,000 + 6,00,000 = Rs. 26,00,000

Problem 41 Calculate liquidity and WC Ratios from the following accounts of a manufacturer of products for the construction industry, and comment on the Ratios. 2008 Rs. in ’000 2,065.00 1,478.60 586.40

Turnover Cost of Sales Gross Profit

2007 Rs. in ’000 1,788.70 1,304.00 484.70 (Continued)

Modified Date: Sat, Jul 03, 2010 12:42:06 PM

Output Date: Tue, Jul 06, 2010 11:43:37 AM

Rev II

Project: Management Accounting_Debarshi Bhattacharyya ACE Pro India Pvt. Ltd. File: X:\Pearson\Management Accounting_Debarshi Bhattacharyya\MAIN\M03\LAYOUT_M03\M03_DEBA_ISBN_EN_SE_C03_Part-3.indd

164

MANAGEMENT ACCOUNTING

2008 Rs. in ’000

2007 Rs. in ’000

119.00 400.90 4.20 48.20 572.30

109.00 347.40 18.80 48.00 523.20

49.10 62.00 19.20 370.70 501.00 71.30

35.30 46.70 14.30 324.00 420.30 102.90

2008 Rs. in ’000 329.80 236.20

2007 Rs. in ’000 285.40 210.80

Current Assets: Stocks Debtors [Note 1] Short-term Investments Cash at Bank and in Hand

Creditors: Amounts falling due within one year: Loans and Overdrafts Provision for Taxation Dividend Payable Creditors [Note 2] Net Current Assets Notes: 1. Trade Debtors 2. Trade Creditors Note: Industry average: Current Ratio – 1.5 and Quick Ratio – 1.0.

Solution Calculation of Liquidity and Working Capital Ratios 2008

2007

⎡ CAs ⎤ Current Ratio ⎢ ⎥ ⎣ CLs ⎦

572.30 ÷ 501.00 = 1.14

523.20 ÷ 420.30 = 1.24

⎡ Quick Assets ⎤ Quick Ratio ⎢ ⎥ ⎣ Quick Liabilities ⎦

453.30 ÷ 501.00 = 0.90

414.20 ÷ 420.30 = 0.99

⎡ Receivables ⎤ Average Collection Period ⎢ × 365 days ⎥ ⎣ Credit Sales ⎦

329.80 ÷ 2,065.00 × 365 = 58 days

285.40 ÷ 1,788.70 × 365 = 58 days

Payables ⎡ ⎤ × 365 days⎥ Average Payment Period ⎢ Credit Purchases ⎣ ⎦

236.20 ÷ 1,478.60 × 365 = 58 days

210.80 ÷ 1,304.00 × 365 = 59 days

⎡ Average Stock ⎤ × 365 days⎥ Stock Turnover Period ⎢ CGS ⎣ ⎦

119.00 ÷ 1,478.60 × 365 = 29 days

109.00 ÷ 1,304.00 × 365 = 31 days

 Comment After a close comparative scrutiny of the Ratios as computed earlier, it has been observed that the company’s both Current Ratio as well as Quick Ratio for both the years are a little lower than the industry average, but they are close to each other. This indicates that the company does not hold a huge stock in its hand, which is also duly supported by its low Stock Turnover Period. The given company is a manufacturing company that is engaged in the construction industry, and, therefore, it is generally expected to have a comparatively longer Debtors’ Turnover Period due to a relatively poor cash flow in the construction industry. Under such a circumstance, such (Continued)

Modified Date: Sat, Jul 03, 2010 12:42:06 PM

Output Date: Tue, Jul 06, 2010 11:43:37 AM

Rev II

Project: Management Accounting_Debarshi Bhattacharyya ACE Pro India Pvt. Ltd. File: X:\Pearson\Management Accounting_Debarshi Bhattacharyya\MAIN\M03\LAYOUT_M03\M03_DEBA_ISBN_EN_SE_C03_Part-3.indd

ACCOUNTING RATIOS FOR FINANCIAL STATEMENT ANALYSIS

165

company should strictly control its stock level, as the stock level of the given company is strictly controlled. On the other hand, the longer creditors’ turnover period of the company shows that it pays for raw materials only after collecting the dues from the credit sales. Reviewing the overall condition, it seems that the Working Capital of the company is strongly managed to avoid a poor liquidity condition, which could be caused due to the longer Debtors’ Turnover Period.

Problem 42 The Balance Sheets of Sandakfu Ltd for the last 3 years read as follows: As on 31 March 2007

Less:

Sources of Fund: Share Capital [Share of Rs. 10 each] Securities Premium Reserves [After 10% Dividend] Long-term Loan Total Funds Represented by: Fixed Assets Depreciation Capital WIP [work-in-progress] Investment A. Net Current Assets: Current Assets: Debtors Stock Cash & Bank Others

Less:

Current Liabilities B. Total Assets [A + B] Sales [excluding Excise Duty and Sales Tax @ 20%]

Rs. in Lakhs As on 31 March 2008

As on 31 March 2009

2,000 1,500 1,500 1,000 6,000

2,000 1,500 1,700 800 6,000

3,000 500 1,800 800 6,100

2,000 700 1,300 800 200 2,300

2,500 950 1,550 900 200 2,650

3,000 1,250 1,750 700 200 2,650

1,700 1,800 500 400 4,400 700 3,700 6,000 3,900

1,800 1,900 500 600 4,800 1,450 3,350 6,000 4,000

1,850 2,400 500 1,400 6,150 2,700 3,450 6,100 5,000

Calculate for the year 2007–08 and 2008–09: i. Fixed Asset Turnover Ratio. ii. Stock Turnover Ratio. iii. Debtors’ Turnover Ratio in terms of number of days’ sales. iv. EPS. Briefly comment on the performance of the company. [C.A. (Inter)—Adapted] Solution Calculation of Ratios for the year 2007–08 and 2008–09 2007–08

2008–09

(a) Fixed Asset Turnover Ratio ⎡ Net Sales Excluding Excise Duty & Sales Tax ⎤ ⎢ ⎥ Average Fixed Asset ⎣ ⎦

4, 000 = 2.80 times 1, 425

5, 000 = 3.03 times 1, 650 (Continued)

Modified Date: Sat, Jul 03, 2010 12:42:06 PM

Output Date: Tue, Jul 06, 2010 11:43:37 AM

Rev II

Project: Management Accounting_Debarshi Bhattacharyya ACE Pro India Pvt. Ltd. File: X:\Pearson\Management Accounting_Debarshi Bhattacharyya\MAIN\M03\LAYOUT_M03\M03_DEBA_ISBN_EN_SE_C03_Part-3.indd

166

MANAGEMENT ACCOUNTING

2007–08

2008–09

(b) Stock Turnover Ratio ⎡ Net sales Excluding Excise Duty & Sales Tax ⎤ ⎢ ⎥ Average Fixed Asset ⎣ ⎦

4, 000 = 2.16 times 1, 850

5, 000 = 2.33 times 2,150

1,750 × 366 = 133 days 4, 800

1, 825 × 365 = 111days 6, 000

400 = Rs. 2 per share 200

400 = Rs. 1.33 per share 300

(c) Debtors’ Turnover Ratio [in terms of no. of days’ sales] ⎡ ⎤ Average Receivables × No. of days in the year ⎥ ⎢ ⎣ Credit Sales Including Excise ⎦ Duty & Equity Shares ⎡ Earning available to Equity Shareholders ⎤ (d) EPS ⎢ ⎥ No. of Equity Shares ⎣ ⎦

Comments on the performance of the company: Fixed Asset Turnover Ratio indicates the level of efficiency of uses or utilizations of Fixed Assets. Here, this Ratio has increased in the year 2008–09 as compared to that of in 2007–08, and, thus, shows a better efficient use or utilization in Fixed Assets in the year 2008–09. Stock Turnover Ratio is an indicator of the movement of stock. Higher Ratio indicates a faster movement of stock. Here, this Ratio has increased in 2008–09 as compared to that of in 2007–08, and, thus, shows a faster movement of stock in 2008–09 than in 2007–08. Yet, the inventory-holding period of the company is still high. Therefore, this Ratio should be compared with the industry average to draw a final conclusion about the efficiency of the inventory management of the company. Debt Collection Period indicates the efficiency of the collection department as regards to the collection of credit sales. Here, the Debt Collection Period in 2008–09 is shorter that of in 2007–08, and, thus, reflects a more efficient collection process in 2008–09 than in 2007–08. But, to draw a final conclusion about the efficiency of debtors’ management of the company, this Ratio should be compared with the industry average and the credit period received by the company from its creditors. In spite of an increase in the Reserve balance in the year 2008–09, the EPS has decreased in 2008–09 from Rs. 2 to Rs. 1.33. This happens, perhaps, due to the increase in the number of equity shares in the year 2008–09 on account of the issue of fully paid bonus shares out of Securities premium. Working Notes 1. Calculation of Sales Including Excise Duty and Sales Tax

Add:

Sales Excluding Excise Duty & Sales Tax 20% Excise Duty & Sales Tax Sales Including Excise Duty & Sales Tax

2007–08 Rs. in Lakhs 4,000 800 4,800

2008–09 Rs. in Lakhs 5,000 1,000 6,00

Note: While calculating the Fixed Asset Turnover Ratio and Stock Turnover Ratio, ‘sales excluding excise duty & sales tax’ is considered. But, while calculating Debtors’ Turnover Ratio, ‘sales including excise duty & sales tax’ is considered as sales to debtors include excise duty and sales tax.

2. Calculation of Average Fixed Assets (Net) In 2007–08: 1,300 + 1,550 = 1, 425(Rs. in Lakhs) Average Fixed Assets (Net) = 2 (Continued)

Modified Date: Sat, Jul 03, 2010 12:42:06 PM

Output Date: Tue, Jul 06, 2010 11:43:37 AM

Rev II

Project: Management Accounting_Debarshi Bhattacharyya ACE Pro India Pvt. Ltd. File: X:\Pearson\Management Accounting_Debarshi Bhattacharyya\MAIN\M03\LAYOUT_M03\M03_DEBA_ISBN_EN_SE_C03_Part-3.indd

ACCOUNTING RATIOS FOR FINANCIAL STATEMENT ANALYSIS

In 2008–09: Average Fixed Assets (Net) =

167

1,550 + 1,750 = 1,650(Rs. in Lakhs) 2

3. Calculation of Average Stock In 2007–08: 1,800 + 1,900 = 1,850(Rs. in Lakhs) Average Stock = 2 In 2008–09: Average Stock =

1,900 + 2, 400 = 2,150(Rs. in Lakhs) 2

4. Calculation of Average Receivables In 2007–08: 1,700 + 1,800 = 1,750(Rs. in Lakhs) Average Receivables = 2 In 2008–09: 1,800 + 1,850 = 1,825(Rs. in Lakhs) Average Receivables = 2 5. Calculation of earnings available to Equity Shareholders 2007–08 Rs. in Lakhs 200 200 400

Increase in Reserves Dividend @ 10% Earnings Available to Equity Shareholders

2008–09 Rs. in Lakhs 100 300 400

6. Debtors’ Turnover Ratio in terms of number of days Credit Sales Average Receivables Here, Debtors’ Turnover Ratio in terms of number of days = Average Collection Period We know, Debtors’ Turnover Ratio =

=

No. of days in a year No. of days in a year = Debtors turnover Ratio Credit Sales ÷ Average Receivables

=

Average Receivables × No. of days in a year Credit Sales

Note: Being 2008 a leap year, the number of days in 2007–08 is taken at 366.

Problem 43 Following are the summarized accounts of Bee Ltd and Zee Ltd for the 2 years 2007 and 2008: Rs. in Lakhs Particulars Sales Manufacturing & Other Expenses Depreciation Profit before Tax

Bee Ltd 2007 54.12 51.04 0.56 2.52 54.12

Zee Ltd 2008 45.75 43.56 0.51 1.68 45.75

2007 17.52 14.96 0.60 1.96 17.52

2008 14.47 11.82 0.35 2.30 14.47 (Continued)

Modified Date: Sat, Jul 03, 2010 12:42:06 PM

Output Date: Tue, Jul 06, 2010 11:43:37 AM

Rev II

Project: Management Accounting_Debarshi Bhattacharyya ACE Pro India Pvt. Ltd. File: X:\Pearson\Management Accounting_Debarshi Bhattacharyya\MAIN\M03\LAYOUT_M03\M03_DEBA_ISBN_EN_SE_C03_Part-3.indd

168

MANAGEMENT ACCOUNTING

Rs. in Lakhs Particulars

Bee Ltd 2007 1.65 8.36 11.24 7.28 0.93 29.46 9.47 0.56 4.24 2.54 12.65 29.46

Miscellaneous Expenditure Fixed Assets Stock Debtors Bank Creditors Taxation [Less Advance Tax] Short-term Borrowings Long-term Borrowings Capital & Reserves

Zee Ltd 2008 1.69 9.41 12.19 8.24 0.33 31.86 9.26 0.68 8.00 2.10 11.82 31.86

2007 – 3.51 1.77 5.82 4.64 15.74 2.33 0.87 4.64 0.10 7.80 15.74

2008 – 2.75 2.26 4.02 2.46 11.49 1.75 0.58 2.16 – 7.00 11.49

You are required to: i. Indicate and calculate five Ratios which in your opinion are relevant in determining the stability of the two companies. ii. Compare the Ratios so determined for the two companies. Indicate what conclusions can be drawn therefrom? [C.S. (Inter)—Adapted] Solution i. In my opinion, the following five Ratios are very much relevant in determining the stability of the given two companies as growing concerns: Computation of five Relevant Ratios in determining the stability of two companies Bee Ltd 2007 ⎡ CAs ⎤ (i) Current Ratio ⎢ ⎥ ⎣ CLs ⎦

Zee Ltd 2008

2007

2008

19.45 ÷ 14.27 = 1.36

20.76 ÷ 17.94 = 1.16

12.23 ÷ 7.84 = 1.56

8.74 ÷ 4.49 = 1.95

⎡ Total Outside Liabilities ⎤ (ii) Total Debts to Net Worth ⎢ ⎥ 16.81 ÷ 11.00 Net Worth ⎣ ⎦ = 1.582

20.04 ÷ 10.13 = 1.978

7.94 ÷ 7.80 = 1.017

4.49 ÷ 7.00 = 0.641

54.12 ÷ 27.81 = 1.946

45.75 ÷ 30.17 = 1.516

17.52 ÷ 15.74 = 1.113

14.47 ÷ 11.49 = 1.259

PBT ⎡ ⎤ × 100 ⎥ (iv) ROI ⎢ Total Assets ⎣ ⎦

2.52 ÷ 27.81 = 0.090

1.68 ÷ 30.17 = 0.055

1.96 ÷ 15.74 = 0.124

2.30 ÷ 11.49 = 0.20

(v) Liquid Assets to Operating Expenses [excluding Depreciation] per day

8.21 ÷ 0.1398 = 59

8.57 ÷ 0.1190 = 72

10.46 ÷ 0.0409 6.48 ÷ 0.0323 = 256 = 201

⎡ Turnover ⎤ (iii) Total Asset Turnover Ratio ⎢ ⎥ ⎣ Total Assets ⎦

ii. From the different Ratios as calculated in (i) above, it has been observed that the results of Zee Ltd for the year 2008 were better than that of for 2007. On the other hand, the results of Bee Ltd for the year 2008 were weaker than that of for 2007. Short-term liquidity or solvency Ratios (i.e., Current Ratio and Liquid Assets to Operating-expenses Ratio) of Zee Ltd were improved in 2008 than 2007, whereas these Ratios of Bee Ltd were deteriorated in 2008 than it was in 2007. Zee Ltd had improved its longterm solvency Ratios (i.e., Total debts to Net Worth and Total Asset Turnover Ratio) in 2008 than in

Modified Date: Sat, Jul 03, 2010 12:42:06 PM

Output Date: Tue, Jul 06, 2010 11:43:37 AM

Rev II

Project: Management Accounting_Debarshi Bhattacharyya ACE Pro India Pvt. Ltd. File: X:\Pearson\Management Accounting_Debarshi Bhattacharyya\MAIN\M03\LAYOUT_M03\M03_DEBA_ISBN_EN_SE_C03_Part-3.indd

ACCOUNTING RATIOS FOR FINANCIAL STATEMENT ANALYSIS

169

2007, whereas these Ratios were also deteriorated in 2008 than in 2007 in the case of Bee Ltd. As far as profitability was concerned, Zee Ltd had remarkably increased its ROI in 2008 than it was in 2007, whereas ROI of B Ltd was remarkably declined in 2008 than 2007. Therefore, Ratios as computed in the above are showing a stronger position in 2008 as compared to 2007 in the case of Zee Ltd, whereas they are showing a deteriorating position in 2008 as compared to 2007 in the case of Bee Ltd. Working Notes Rs. in Lakhs Bee Ltd

1. Net Worth

Less:

2007 12.65 1.65 11.00

Capital & Reserves Miscellaneous Expenditure Net Worth

Zee Ltd 2008 11.82 1.69 10.13

2007 7.80  7.80

2008 7.00  7.00

Rs. in Lakhs

2. Total Outside Liabilities Bee Ltd

Zee Ltd

2007 2.54 4.24 9.47 0.56 16.81

2008 2.10 8.00 9.26 0.68 20.04

2007 0.10 4.64 2.33 0.87 7.94

2008  2.16 1.75 0.58 4.49

11.24 7.28 0.93 19.45

12.19 8.24 0.33 20.76

1.77 5.82 4.64 12.23

2.26 4.02 2.46 8.74

9.47 0.56 4.24 14.27

9.26 0.68 8.00 17.94

2.33 0.87 4.64 7.84

1.75 0.58 2.16 4.49

Fixed Assets Current Assets [as computed in (3) above] Total Assets

8.36 19.45 27.81

9.41 20.76 30.17

3.51 12.23 15.74

2.75 8.74 11.49

6. Liquid Assets Current Assets [as computed in (3) above] Less: Stock Liquid Assets

19.45 11.24 8.21

20.76 12.19 8.57

12.23 1.77 10.46

8.74 2.26 6.48

Long-term Borrowings Short-term Borrowings Creditors Taxation [less Advance Tax] Total outside Liabilities 3. Current Assets Stock Debtors Bank Current Assets 4. Current Liabilities Creditors Taxation [less Advance Tax] Short-term Borrowings Current Liabilities 5. Total Assets

7. Operating Expenses (excluding Depreciation) per day 51.04 /365 = 0.1398

Modified Date: Sat, Jul 03, 2010 12:42:06 PM

Output Date: Tue, Jul 06, 2010 11:43:37 AM

43.56 /366 = 0.1190

14.96 /365 = 0.0409

11.82 /365 = 0.0323

Rev II

Project: Management Accounting_Debarshi Bhattacharyya ACE Pro India Pvt. Ltd. File: X:\Pearson\Management Accounting_Debarshi Bhattacharyya\MAIN\M03\LAYOUT_M03\M03_DEBA_ISBN_EN_SE_C03_Part-3.indd

170

MANAGEMENT ACCOUNTING

Problem 44 The following accounting information and Financial Ratios have been obtained in respect of Meghna Ltd, relating to the year that ended on 31 March 2009: A. Accounting information: Gross Profit Net Profit Raw Materials Consumed Direct Wages Stock of Raw Materials Stock of Finished Goods Debt Collection Period All sales are Made on Credit B. Financial Ratios: Fixed Assets to Sales Fixed Assets to Current Assets Current Ratio Long-term Loans to Current Liabilities Capital to Reserves & Surplus

15% of sales 8% of sales 20% of work’s cost 10% of work’s cost 3 months’ usage 6% of work’s cost 60 days

1:3 13:11 2:1 2:1 1:4

If the value of the Fixed Assets as on 31 March 2009 amounted to Rs. 26 lakhs, prepare a summarized Profit & Loss A/c of the company for the year that ended on 31 March 2009 and a Balance Sheet as on that date. Solution Books of Meghna Ltd Profit & Loss A/c for the year that ended on 31 March 2009 Dr. Particulars 7

To Raw Materials Consumed To Direct Wages7 To Works Overheads (Bal. fig.) To Gross Profit c/d4 To Indirect Operating Expenses (Bal. fig.) To Net Profit5

Amount Rs. 13,26,000 6,63,000 46,41,000 11,70,000 78,00,000 5,46,000 6,24,000 11,70,000

Particulars By Sales

1

By Gross Profit b/d

Cr. Amount Rs. 78,00,000

78,00,000 11,70,000 11,70,000

Balance Sheet as on 31 March 2008 Liabilities Share Capital11 Reserves & Surplus11 Long-term Loan6 Current Liabilities3

Amount Rs. 3,00,000 12,00,000 22,00,000 11,00,000

Assets Fixed Assets1 Current Assets: Stock of Raw Materials8 Stock of Finished Goods8 Debtors9 Cash10

48,00,000

Amount Rs. 26,00,000 3,31,500 3,97,800 12,82,192 1,88,508 48,00,000 (Continued)

Modified Date: Sat, Jul 03, 2010 12:42:06 PM

Output Date: Tue, Jul 06, 2010 11:43:37 AM

Rev II

Project: Management Accounting_Debarshi Bhattacharyya ACE Pro India Pvt. Ltd. File: X:\Pearson\Management Accounting_Debarshi Bhattacharyya\MAIN\M03\LAYOUT_M03\M03_DEBA_ISBN_EN_SE_C03_Part-3.indd

ACCOUNTING RATIOS FOR FINANCIAL STATEMENT ANALYSIS

171

Working Notes 1. Fixed Assets (FA) to Sales = 1:3 or

Fixed Assets 1 = Sales 3

or

26,00,000 1 = Sales 3

∴ Sales = 26,00,000 × 3 = Rs. 78,00,000 2. Fixed Assets (FA) to Current Assets (CAs) = 13:11 or

FAs 13 = CAs 11

or

26,00,000 13 = CAs 11

∴CA = 26,00,000 × 11 ÷ 13 = Rs. 22,00,000 3. Current Ratio =

Current Assets ( CAs) 2 = Current Liabilities ( CLs) 1

22,00,000 =2 CLs ∴ CL = 22,00,000 ÷ 2 = Rs. 11,00,000 4. GP = 15% of Sales = 15% of 78,00,000 = Rs. 11,70,000 or

5. NP = 8% of Sales = 8% of 78,00,000 = Rs. 6,24,000 6. Long-term Loan to CLs = 2/1 Long-term Loan =2 11,00,000 ∴ Long-term Loan = 11,00,000 × 2 = Rs. 22,00,000 or

7. Work’s Cost = Sales − GP = 78,00,000 – 11,70,000 = Rs. 66,30,000 ∴ Direct Wages = 10% of Work’s Cost = 10% of 66,30,000 = Rs. 6,63,000 8. Stock of Raw Materials = 3 months’ usage = 3/12 × 13,26,000 = Rs. 3,31,500 Stock of Finished Goods = 6% of Work’s Cost = 6% of 66,30,000 = Rs. 3,97,800 9. Debt Collection Period = 60 days or

Debtors × 365 = 60 78,00,000

∴ Debtors = 78,00,000 × 60 ÷ 365 = Rs. 12,82,192 10. Here, Current Assets = Stock of Raw Materials + Stock of Finished Goods + Debtors + Cash or 22,00,000 = 3,31,500 + 3,97,800 + 12,82,192 + Cash ∴ Cash = Rs. 1,88,508 11. In a Balance Sheet, Net Worth (NW) + Long-term loans = FAs + WC Here, NW + 22,00,000 = 26,00,000 + (22,00,000 – 11,00,000) ∴ NW = Rs. 15,00,000 Now given, Share Capital to Reserves & Surplus (R&S) = 1: 4 (Continued)

Modified Date: Sat, Jul 03, 2010 12:42:06 PM

Output Date: Tue, Jul 06, 2010 11:43:37 AM

Rev II

Project: Management Accounting_Debarshi Bhattacharyya ACE Pro India Pvt. Ltd. File: X:\Pearson\Management Accounting_Debarshi Bhattacharyya\MAIN\M03\LAYOUT_M03\M03_DEBA_ISBN_EN_SE_C03_Part-3.indd

172

MANAGEMENT ACCOUNTING

or

Share Capital 1 = R&S 4

∴ Reserves & Surplus = 4 × Share Capital Again, Net Worth = Share Capital + Reserves & Surplus [assuming no Miscellaneous expenditure] or 15,00,000 = Share Capital + 4 × Share Capital or 5 × Share Capital = 15,00,000 ∴ Share Capital = Rs. 3,00,000 ∴ Reserves & Surplus = 4 × 3,00,000 = Rs. 12,00,000

Problem 45 Following is the incomplete information of X Ltd: Trading and Profit & Loss A/c for the year that ended on 31 March 2008 To Opening Stock To Purchases To Direct Expenses To GP c/d To Establishment Expenses To Interest on Loan To Provision for Taxation To Net Profit c/d To Proposed Dividends To Transfer to General Reserve To Balance Transferred to Balance Sheet

Rs. in ’000 700 ? 175 ? ? 740 60 ? ? ? ? ? ? ?

By Sales By Closing Stock

Rs. in ’000 ? ?

By Gross Profit b/d By Commission

? ? 100

By Balance b/f By Net Profit b/d

? 140 ? ?

Balance Sheet as on 31 March 2008 Liabilities Paid-up Capital General Reserve: Balance at the Beginning of the Year Proposed Addition Profit & Loss A/c 10% Loan A/c Current Liabilities

Amount Rs. in ’000 1,000 ? ? ? ? ? ?

Assets Fixed Assets: Plant & Machinery Other Fixed Assets Current Assets: Stock Sundry Debtors Cash at Bank

Amount Rs. in ’000 1,400 ? ? ? 125 ?

Other information: i. Current Ratio is 2:1. ii. Closing Stock is 25% of sales. iii. Proposed dividends to paid up capital Ratio is 2:3. iv. GP Ratio is 60% of turnover. v. Loan is half of the Current Liabilities. vi. Transfer to general reserves to proposed dividends Ratio is 1:1. vii. Profit carried forward is 10% of the proposed dividends.

Modified Date: Sat, Jul 03, 2010 12:42:06 PM

Output Date: Tue, Jul 06, 2010 11:43:37 AM

Rev II

Project: Management Accounting_Debarshi Bhattacharyya ACE Pro India Pvt. Ltd. File: X:\Pearson\Management Accounting_Debarshi Bhattacharyya\MAIN\M03\LAYOUT_M03\M03_DEBA_ISBN_EN_SE_C03_Part-3.indd

ACCOUNTING RATIOS FOR FINANCIAL STATEMENT ANALYSIS

173

viii. Provision for taxation is equal to the amount of Net Profit of the year. ix. Balance to credit of the general reserve at the beginning of the year is twice the amount transferred to that account from the current year’s profits. All working notes should be part of your answer. You are required to complete: x. (i) Trading and Profit & Loss A/c for the year that ended on 31 March 2008. xi. (ii) A Balance Sheet as on that date. [C.A. (PE—II)—May 2008] Solution Books of X Ltd Trading and Profit & Loss A/c for the year that ended on 31 March 2008 Dr. To Opening Stock To Purchases (Bal. fig.) To Direct Expenses To Gross Profit c/d To Establishment Expenses To Interest on Loan To Provision for Taxation4 To Net Profit c/d To Proposed Dividends1 To Transfer to General Reserve2 To Balance Transferred to Balance Sheet3

Rs. in ’000 700.00 2,613.33 175.00 3,220.00 6,708.33 740.00 60.00 1,260.00 1,260.00 3,320.00 666.67 666.67 66.66 1,400.00

Cr. Rs. in ’000 5,366.66 1,341.67

By Sales5 By Closing Stock6

By Gross Profit b/d (Bal. fig.) By Commission

6,708.33 3,220.00 100.00

By Balance b/f By Net Profit b/d

3,320.00 140.00 1,260.00 1,400.00

Balance Sheet as on 31 March 2008 Amount

Liabilities

Rs. in '000

Paid-up Capital General Reserve: Balance at the beginning of the year Add: Addition during the year Profit & Loss A/c 10% Loan A/c7 Current Liabilities8

1,333.34 666.67

Amount Rs. in ’000 1,000.00

2,000.01 66.66 600.00 1,200.00 4,866.67

Assets Fixed Assets: Plant & Machinery Other Fixed Assets (bal. fig.) Current Assets: Stock6 Sundry Debtors9 Cash at Bank

Amount Rs. in ’000 1,400.00 1,066.67 1,347.67 933.33 125.00 4,866.67

Working Notes 1. Paid-up Capital = 1,000 [Rs. in ’000]. Again, Proposed Dividend to Paid-up Capital = 2:3 or

Proposed Dividend 2 = 1,000 3 (Continued)

Modified Date: Sat, Jul 03, 2010 12:42:06 PM

Output Date: Tue, Jul 06, 2010 11:43:37 AM

Rev II

Project: Management Accounting_Debarshi Bhattacharyya ACE Pro India Pvt. Ltd. File: X:\Pearson\Management Accounting_Debarshi Bhattacharyya\MAIN\M03\LAYOUT_M03\M03_DEBA_ISBN_EN_SE_C03_Part-3.indd

174

MANAGEMENT ACCOUNTING

Proposed Dividend =

(1,000 × 2) = 666.67 3

[Rs.in ’000]

2. Transfer to general reserve to Proposed Dividend = 1:1 or,

Transfer to General Reserve =1 666.67

∴ Transfer to General Reserve = 666.67 [Rs. in ’000] Again, the Balance of general Reserve at the beginning of the year = Twice the amount transferred to general reserve from the current years’ profit = 2 × 666.67 = 1,333.34 [Rs. in ’000] 3. Profit carried forward = 10% of Proposed Dividend = 10% of 666.67 = 66.66 [Rs. in ’000] 4. Provision for tax = Amount of NP for the year = 1,260 [Rs. in ’000] 5. GP Ratio = 60% (of Turnover) or

GP = 60% Sales

or

3,220 = 60% Sales

∴ Sales = 3,220 ÷ 60% = 5,366.66 [Rs. in ’000] 6. Closing Stock = 25% of Sales = 25% of 5,366.66 = 1,341.67 [Rs. in ’000] 7. Interest on Loan = 60 (Rs. ’000) at an interest @ 10% ∴ 10% Loan = 100 ÷ 10 × 60 = 600 [Rs. in ’000] 8. Loan = 1/2 of CL or 600 = 1/2 × CL ∴ CL = 1,200 [Rs. in ’000] 9.

Current Ratio =

CAs 2 = CLs 1

CAs =2 1,200 ∴ CA = 2,400 [Rs. in ’000] Again, CAs = Stock + Debtors + Bank or 2,400 = 1,341.67 + Debtors + 125 ∴ Debtors = 933.33 [Rs. in ’000]

or

Problem 46 Following is the abridged Balance Sheet of Xenos Ltd as on 31 March 2008: Balance Sheet as on 31 March 2008 Liabilities Paid-up Share Capital Profit & Loss A/c Current Liabilities

Amount Rs. 5,00,000 85,000 2,00,000

Freehold property Plant & Machinery Less: Depreciation Stock Debtors Cash

7,85,000

Modified Date: Sat, Jul 03, 2010 12:42:06 PM

Amount Rs. 4,00,000

Assets

Output Date: Tue, Jul 06, 2010 11:43:37 AM

2,50,000 75,000

1,75,000 1,05,000 1,00,000 5,000 7,85,000

Rev II

Project: Management Accounting_Debarshi Bhattacharyya ACE Pro India Pvt. Ltd. File: X:\Pearson\Management Accounting_Debarshi Bhattacharyya\MAIN\M03\LAYOUT_M03\M03_DEBA_ISBN_EN_SE_C03_Part-3.indd

ACCOUNTING RATIOS FOR FINANCIAL STATEMENT ANALYSIS

175

From the following information, you are required to prepare a Profit & Loss A/c for the year that ended on 31 March 2009 and a Balance Sheet as on that date: i. The composition of the total of the ‘liability side’ of the Balance Sheet as on 31 March 2009 (the Paidup Share Capital remains the same as on 31 March 2008) was as follows: Share Capital—50%; Profit & Loss A/c—15%; 10% Debentures—10%; Creditors—25%. Debentures were issued on 1 April 2008, interest of which was being paid on 30 September 2008 and 31 March 2009. ii. During the year that ended on 31 March 2009, additional Plant & Machinery had been bought and a further Rs. 25,000 depreciation was written off. Freehold property remained unchanged. Total Fixed Assets then consisted 60% of the total Fixed and Current Assets. iii. The current Ratio was 1.6:1. The quick Ratio was 1:1. iv. Debtors (4/5th of the Quick assets) to Sales Ratio revealed a credit period of 2 months. v. Gross Profit was @ 15% of the selling price and Return on Net Worth as on 31 March 2009 was 10%. Ignore taxation. [C.A. (Inter)—Adapted] Solution Books of Xenos Ltd Profit & Loss A/c for the year that ended on 31 March 2008 Dr.

Cr. Particulars

To Opening Stock To Purchases (Bal. fig.) To Gross Profit c/d6 To Depreciation on Machinery To Other operating Expenses (Bal. fig.) To Debenture Interest7 To Net Profit

Amount Rs. 1,05,000 10,65,000 1,80,000 13,50,000 25,000 80,000 10,000 65,000 1,80,000

By Sales By Closing Stock

Amount Rs. 12,00,000 1,50,000

By Gross Profit b/d

13,50,000 1,80,000

Particulars 5

1,80,000

Balance Sheet as on 31 March 2008 Liabilities Paid-up Share Capital1 Reserves & Surplus Profit & Loss A/c8 Secured Loan 10% Debentures1 Current Liabilities: Creditors1

Amount Rs. 5,00,000 1,50,000 1,00,000 2,50,000 10,00,000

Assets Fixed Assets Freehold Property Plant & Machinery2 Current Assets: Stock3 Debtors4 Bank4

Amount Rs. 4,00,000 2,00,000 1,50,000 2,00,000 50,000 10,00,000

Working Notes 1. The Paid-up Share Capital as on 31 March 2008 remains the same as it was on 31 March 2007 (i.e., Rs. 5,00,000). Again, Share Capital (i.e., Rs. 5,00,000) = 50% of the total of the ‘liability side’ of the Balance Sheet. ∴ Profit & Loss A/c = 15% of the total of liability side = 15/50 × 5,00,000 = Rs. 1,50,000 (Continued)

Modified Date: Sat, Jul 03, 2010 12:42:06 PM

Output Date: Tue, Jul 06, 2010 11:43:37 AM

Rev II

Project: Management Accounting_Debarshi Bhattacharyya ACE Pro India Pvt. Ltd. File: X:\Pearson\Management Accounting_Debarshi Bhattacharyya\MAIN\M03\LAYOUT_M03\M03_DEBA_ISBN_EN_SE_C03_Part-3.indd

176

MANAGEMENT ACCOUNTING

∴ 10% Debentures = 10% of the total of liability side = 10/50 × 5,00,000 = Rs. 1,00,000 ∴ Creditors = 25% of the total of liability side = 25 ÷ 50 × 5,00,000 = Rs. 2,50,000 2. Here, Current Liability = Creditors = Rs. 2,50,000 Current Asset ( CAs) 1.6 Again, Current Ratio = = Current Liability ( CLs) 1 or

Current Asset =

1.6 × 2,50,000 = Rs. 4,00,000 1

Again, Total Fixed Asset (FA) = 60% of [Total FAs + Total CA] or FAs = 60/100 × (FA + 4,00,000) or 10 × FAs = 6 × FAs + 24,00,000 or 4 × FAs = 24,00,000 ∴ Total FAs = 24,00,000 ÷ 4 = Rs. 6,00,000 Again, Total FAs = Freehold Property + Plant & Machinery (P&M) or, 6,00,000 = 4,00,000 + P&M [as Freehold property remains unchanged] ∴ Plant & Machinery (Closing Balance) = Rs. 2,00,000 Now, in case of Plant & Machinery, Opening Balance + Addition during the year − Depreciation for the year = Closing Balance or 1,75,000 + Addition during the year – 25,000 = 2,00,000 ∴ Addition of Plant & Machinery during the year = Rs. 50,000 3.

Quick Ratio = or

Quick Assets (CAs − Stock ) = 1: 1 = Quick Liabilities ( CLs − Bank O/D)

( 4,00,000 − Stock ) = 1 assumed that there is no Bank overdraft [ ] (2,50,000 − Nil)

or 2,50,000 = 4,00,000 − Stock ∴ Stock = Rs. 1,50,000 4. Quick Assets = CAs − Stock = 4,00,000 – 1,50,000 = Rs. 2,50,000 ∴Debtors = 4/5 of Quick assets = 4/5 × 2,50,000 = Rs. 2,00,000 Here, CAs = Stock + Debtors + Bank or, 4,00,000 = 1,50,000 + 2,00,000 + Bank ∴ Bank = Rs. 50,000 5. Debtors to Sales Ratio = 2 months Debtors or × 12 months = 2 months Sales or or

Debtors 2 months 1 = = Sales 12 months 6 2,00,000 1 = Sales 6

∴ Sales = Rs. 12,00,000 6. GP = 15% of Sales = 15% of 12,00,000 = Rs. 1,80,000 7. Annual Debenture Interest = (10% on Rs. 1,00,000 for 1 year) × 2 = Rs. 10,000 2 NP = 10% 8. Return on Net Worth (NW ) = NW Here, NW = Share Capital + Profit & Loss A/c = 5,00,000 + 1,50,000 = Rs. 6,50,000 (Continued)

Modified Date: Sat, Jul 03, 2010 12:42:06 PM

Output Date: Tue, Jul 06, 2010 11:43:37 AM

Rev II

Project: Management Accounting_Debarshi Bhattacharyya ACE Pro India Pvt. Ltd. File: X:\Pearson\Management Accounting_Debarshi Bhattacharyya\MAIN\M03\LAYOUT_M03\M03_DEBA_ISBN_EN_SE_C03_Part-3.indd

177

ACCOUNTING RATIOS FOR FINANCIAL STATEMENT ANALYSIS



NP = 10% 6,50,000 ∴ NP = Rs. 65,000

Add:

Balance of Profit & Loss A/c as on 31 March 2007 NP for the year Balance of Profit & Loss A/c as on 31 March 2008 carried over to Balance Sheet

Rs. 85,000 65,000 1,50,000

CHAPTER REVIEW SUMMARY  Financial Statements are a compilation of financial data, arranged and organized in a systematic and summarized manner, according to the accounting principles, to assess the financial position of an enterprise as regards to its profitability, operational efficiency, long- and short-term solvency, and growth potential. A number of statements prepared at the end of every Accounting Period are collectively called ‘Financial Statements,’ for example, Balance Sheet, Profit & Loss A/c, Cash Flow Statement and Fund Flow Statement.  A Ratio is a relationship between two or more items expressed in mathematical terms. It is an expression of quantitative relationship between two amounts or items which is/are expressed in numbers or percentage.  Ratio calculated from different Accounting Data for exhibiting a meaningful and useful relationship between them is called Accounting Ratio.  Ratio Analysis is a tool for identifying financial strength, weakness and growth potentiality of an enterprise by means of determination, analysis and interpretation of relevant various accounting Ratios.  Ratio Analysis, as an analytical tool of Financial Statement Analysis, performs important functions: (a) measures and evaluates the financial condition of an enterprise; (b) measures and evaluates the liquidity, solvency, profitability, managerial efficiency, Capital Structure and activity of an enterprise; (c) measures and evaluates the operating effectiveness of an enterprise; (d) identifies the functional areas within the business where the adoption of remedial measures are needed; (e) helps the management in the course of Decision Making process; and (f) serves as a management control system.  It is very much useful to all the interested parties for improvement of their needs according to their respective interest in the enterprise, such as: (a) useful for identification of financial strengths and weaknesses of an enterprise; (b) useful to measure the liquidity, solvency, profitability, managerial efficiency and activity of an enterprise; (c) useful for inter- and intra-firm comparison of performance; and (d) useful to determine the corporate sickness.  It has many advantages, such as: (a) most powerful tool for measuring the short- and long-term solvency, profitability, operating activity, Capital Structure Analysis and the managerial efficiency of a concern; (b) summarizes a large number of quantitative Accounting Data by calculating the different Ratios; (c) helps the management to prepare the necessary budget and to formulate future policies; (d) relates the present accounting information with the past and (e) formulates an effective inter- and intra-firm comparison of accounting data.  In spite of immense advantages and usefulness of Ratio Analysis, it suffers from some limitations, such as: (a) it may tend to interpret a wrong direction if it is based on unauthenticated data; (b) as the Ratios are calculated on the basis of the past results, proper prediction for future may not always be dependable; (c) a single Ratio calculated for any functional area of a business does not convey its conclusive state; (d) it is formulated on the basis of historical figures as obtained from the Financial Statements. Performance of a concern based on the historical data does not always indicate its present real condition; (e) the price level changes very often distort the trend analysis process which is done with the help of various Ratios calculated in the process of Ratio Analysis.  Shareholders, moneylenders, creditors, government, bankers, management and so on are the different interested parties in Ratio Analysis.  Accounting Ratios may be broadly classified into three categories: (i) Balance Sheet Ratio; (ii) Revenue Statement Ratio; and (iii) Mixed or Composite Ratio.  A Standard or Ideal Ratio is a predetermined calculated Ratio, which is considered to be normal or standard, or which is the most meaningful or significant, or which is most effectively applicable universally to different industries under different times.

Modified Date: Sat, Jul 03, 2010 12:42:06 PM

Output Date: Tue, Jul 06, 2010 11:43:37 AM

Rev II

Project: Management Accounting_Debarshi Bhattacharyya ACE Pro India Pvt. Ltd. File: X:\Pearson\Management Accounting_Debarshi Bhattacharyya\MAIN\M03\LAYOUT_M03\M03_DEBA_ISBN_EN_SE_C03_Part-3.indd

178 

MANAGEMENT ACCOUNTING

Du Pont System of financial analysis is one of the most-effective technique of earning capacity or profitability analysis of an enterprise. In Du Pont Analysis, various factors which influence ROE (as well as ROI) are diagrammatically exhibited with the help of a chart, which is popularly known as Du Pont Chart. Du Pont Analysis enables the enterprise to break its ROE into three major components in order to identify the area responsible for profit-earning capacity of the enterprise. These three components are profit on sales (i.e., NP Ratio), efficient use of assets (i.e., Total Asset Turnover Ratio) and use of leverage in the Capital Structure (i.e., FLM).

CHAPTER REVIEW QUIZ 1. State whether the following statements are true or false: a. Current Ratio measures the liquidity of the business. b. Debt-Equity Ratio measures the short-term financial position of the business. c. Ratio Analysis measures the profitability, efficiency and financial soundness of the business. d. Activity Ratio and Turnover Ratios are same. e. Current Ratio improves with increase in credit purchase. f. Ideal Ratio between Current Assets and Current Liabilities is 2:1. g. Acid Test Ratio indicates the liquidity position of a concern. Ans.: (a) True; (b) False; (c) True; (d) True; (e) False; (f) True; (g) True. 2. State which statements are true and which are false: a. Working Capital is the excess of Current Assets over Current Liabilities. b. The Ideal Ratio between Liquid Assets and Current Liabilities is 1:1. c. Liquidity Ratio improves with increase in credit sales. d. 45% or .45 times have the same meaning. e. Amount from Current Assets is realized within 3 years. f. Debt-Equity Ratio measures the profitability of the business. g. Liquid Ratio is also known as Acid Test Ratio. Ans.: (a) True; (b) True; (c) True; (d) False; (e) False; (f) False; (g) True. 3. Choose the correct alternative from the following: a. Quick Ratio is a: (i) Balance Sheet Ratio; (ii) Revenue Statement Ratio; (iii) Mixed Ratio. b. Quick assets: (i) Include cash & bank only; (ii) Exclude sundry debtors; (iii) Exclude stock. c. Total long-term fund invested into the business is called: (i) Net Worth; (ii) Working Capital; (iii) Total Liabilities; (iv) Capital Employed. d. Net Worth includes: (i) Working Capital; (ii) Long-term loan; (iii) Total Share Capital; (iv) None of these. e. Liquid liabilities exclude: (i) Bills Payable; (ii) Bank Overdraft; (iii) Sundry Creditors; (iv) Accrued Expense. f. Proprietary Ratio measures: (i) Activity; (ii) Profitability; (iii) Liquidity; (iv) Long-term financial position of the business. g. Acid Test Ratio measures: (i) Activity; (ii) Profitability; (iii) Liquidity; (iv) Long-term financial position of the business. h. Stock Velocity Ratio is a: (i) Balance Sheet Ratio; (ii) Revenue Statement Ratio; (iii) Mixed Ratio. i. Return on Capital Employed measures: (i) Activity; (ii) Profitability; (iii) Liquidity; (iv) Long-term financial position of the business. Ans.: (a) (i); (b) (iii); (c) (iv); (d) (iii); (e) (ii); (f) (iv); (g) (iii); (h) (iii); (i) (ii). 4. Select the correct alternative from the following: a. Liquid Assets = (i) Current Assets + Stock; (ii) Current Assets − Stock; (iii) Current Assets − Stock − Prepaid expenses. b. Ideal Proprietary Ratio should be: (i) 75%; (ii) 50%; (iii) 90%; (iv) 25%. c. CGS = (i) Sales—GP; (ii) Sales proceeds; (iii) Sales—NP. d. The satisfactory Ratio between the internal and external equity is (i) 1:1; (ii) 4:1; (iii) 2:1; (iv) 3:1. e. The Ideal Ratio between long-term funds and long-term loans is (i) 1:1; (ii) 3:1; (iii) 2:1; (iv) 4:1. f. Debt-Equity Ratio measures: (i) Activity; (ii) Profitability; (iii) Liquidity; (iv) Long-term financial position of the business.

Modified Date: Sat, Jul 03, 2010 12:42:06 PM

Output Date: Tue, Jul 06, 2010 11:43:37 AM

Rev II

Project: Management Accounting_Debarshi Bhattacharyya ACE Pro India Pvt. Ltd. File: X:\Pearson\Management Accounting_Debarshi Bhattacharyya\MAIN\M03\LAYOUT_M03\M03_DEBA_ISBN_EN_SE_C03_Part-3.indd

ACCOUNTING RATIOS FOR FINANCIAL STATEMENT ANALYSIS

179

g. Acid Test Ratio is also known as: (i) Liquid Ratio; (ii) GP Ratio; (iii) Current Ratio. h. Quick Ratio is also known as: (i) Operating Ratio; (ii) Liquid Ratio; (iii) Debt-Equity Ratio. Ans.: (a) (iii); (b) (ii); (c) (i); (d) (i); (e) (iii); (f) (iv); (g) (i); (h) (ii). 5. Assuming the Current Ratio as 2, state in each of the following cases whether the Ratio will improve or decline or will have no change: a. Payment of a current liability. b. Payment of Fixed Assets. c. Cash collected from customers. d. Bills Receivable dishonoured. e. Issue of new equity shares. f. Redemption of preference shares. g. Bills Receivable drawn upon customers. h. Bills Payable accepted from suppliers. i. Long-term loan raised. [C.S. (Inter)—Adapted] Ans.: (a) no change; (b) decline; (c) no change; (d) no change; (e) improve; (f) decline; (g) no change; (h) no change; (i) improve. 6. Indicate the important accounting Ratios that would be used by each of the following cases: a. A long-term creditor is interested in determining whether his claim is adequately secured. b. A bank that has been approached by a company for short-term loan/overdraft. c. A shareholder who is examining his portfolio and who is to decide whether he should hold or sell his shares in a company. [C.S. (Inter)—May 1993] Ans.: (a) Debt service coverage Ratio; (b) Current/Quick Ratio; (c) Earning per Share. 7. Identify the correct answer in each case from the following: a. If Average Collection Period is 3 months, then Debtors’ Turnover Ratio is: (i) 3; (ii) 4; (iii) 12; (iv) 6. b. If Creditors’ Turnover Ratio is 6, then Average Payment Period is: (i) 6 months; (ii) 3 months; (iii) 2 months. c. If Current Assets and Current Liabilities are Rs. 4,50,000 and Rs. 5,00,000 respectively, then Working Capital is: (i) Rs. 50,000; (ii) Rs. 9,50,000; (iii) Rs. 50,000; (iv) none of these. d. Increase in Total Assets: (i) decreases Proprietary Ratio; (ii) increases Proprietary Ratio; (iii) does not affect Proprietary Ratio. e. Increase in Liquid Assets: (i) decreases Liquid Ratio; (ii) increases Liquid Ratio; (iii) does not affect Liquid Ratio. f. If Sales, GP and Average Stock are Rs. 5,00,000, Rs. 1,00,000 and Rs. 80,000 respectively, then StockTurnover Ratio is: (i) 6.25; (ii) 5; (iii) 1.25; (iv) 5. Ans.: (a) (ii); (b) (iii); (c) (iii); (d) (i); (e) (ii); (f) (iv). EXERCISE I. Theoretical Questions A. Objective Type Questions:

1. 2. 3. 4. 5. 6. 7. 8. 9.

Give four examples of Financial Statements. What is a Ratio? What is Accounting Ratio? What do you mean by receivables? What do you mean by payables? Give the formula for calculating Proprietors’ Fund. Give the formula for calculating Capital Employed. Mention the formula for calculating Return on Investment. Mention the formula for calculating Acid Test Ratio.

Modified Date: Sat, Jul 03, 2010 12:42:06 PM

Output Date: Tue, Jul 06, 2010 11:43:37 AM

Rev II

Project: Management Accounting_Debarshi Bhattacharyya ACE Pro India Pvt. Ltd. File: X:\Pearson\Management Accounting_Debarshi Bhattacharyya\MAIN\M03\LAYOUT_M03\M03_DEBA_ISBN_EN_SE_C03_Part-3.indd

180 10. 11. 12. 13. 14.

MANAGEMENT ACCOUNTING

Name three Ratios for measuring profitability. Name three non-operating expenses. Give the formula for calculating average debtors. Give the formula for calculating the Debt-Equity Ratio. Give the gist of a company Balance Sheet.

B. Short Answer Type Questions

1. 2. 3. 4. 5. 6. 7. 8. 9. 10. 11. 12. 13.

What are Financial Statements? What is Ratio Analysis? What are the steps in Ratio Analysis? Who are the parties interested in Ratio Analysis? What is Ideal or Standard Ratio? What is Net Worth? What is Equity Shareholder’s fund? What is Capital Employed? What is Working Capital? What is Quick Asset? What is Quick Liability? What is Cost of Goods Sold? Write short notes on: (i) Acid Test Ratio; (ii) Debt-Equity Ratio; (iii) Capital Gearing Ratio; (iv) Current Ratio; (v) Operating Profit Ratio; (vi) Dividend Payout Ratio; (vii) Operating Ratio; (viii) Gross Profit Ratio; (ix) Net Profit Ratio; (x) Stock Turnover Ratio; (xi) Capital Turnover Ratio; (xii) Fixed Asset Turnover Ratio; 14. Name the Ratios to be used to study the long-term solvency, short-term solvency, liquidity, profitability, activity and managerial efficiency of a concern? C. Essay Type Questions

1. 2. 3. 4. 5. 6. 7.

8. 9. 10. 11. 12. 13.

What is a Ratio? What is meant by accounting Ratio? What is Ratio Analysis? What is the importance of Ratio Analysis in accounting? What are the uses of Ratio Analysis in accounting? Discuss the advantages and limitations of accounting Ratio. What do you mean by Ratio Analysis? Explain briefly how accounting Ratios are classified with suitable examples. What do you mean by Ratio Analysis? Explain the following Ratios: (a) Capital Gearing Ratio; (b) Gross Profit Ratio; (c) Return on Investment (ROI); (d) Operating Ratio; (e) Operating Leverage; (f) Dividends per Share (DPS). Discuss the role of the following Ratios in the interpretation of Financial Statements: (i) Debtors’ Turnover Ratio; (ii) Creditors’ Turnover Ratio; (iii) Capital Gearing Ratio; (iv) Debt-Equity Ratio; (v) Proprietary Ratio; (vi) Fixed Asset Proprietorship Ratio; (vii) EPS; (viii) Return on Net Worth; (ix) Return on Capital Employed; (x) Financial Leverage; (xi) Return on Equity; (xii) Price Earning Ratio; (xiii) Dividend Payout Ratio; (xiv) Contribution Ratio; (xv) Debt service coverage Ratio. What Ratios are to be worked out to study the long- and short-term solvency position of a concern? What Ratios are to be worked out to study the liquidity, profitability, activity and managerial efficiency of a concern? ‘Accounting Ratios are mere guides and complete reliance on them in Decision Making is suicidal.’ Elucidate the statement. What is Return on Equity? How does it differ from Return on Investment? What is Du Pont Analysis of Financial Statements? What is Du Pont Chart? Explain the various components of return on equity with the help of Du Pont Chart. Distinguish between the following: (a) Current Ratio and Liquid Ratio; (b) Current Asset and Liquid Asset; (c) Current Liability and Liquid Liability; (d) Net Worth and Capital Employed; (e) Proprietors’ Fund and Equity Shareholder’s fund; (f) Return on Net Worth and Return on Capital Employed; (g) Net Profit Ratio and Gross Profit Ratio; (h) Operating Ratio and Operating Profit Ratio; (i) Capital Turnover Ratio and Total Asset Turnover Ratio; (j) Debtors’ Velocity Ratio and Creditors’ Velocity Ratio; (k) EPS and DPS; (l) Price Earning Ratio and Dividend Payout Ratio; (m) Proprietary Ratio and Fixed Asset Proprietorship Ratio; (n) Financial Leverage and Operating Leverage.

Modified Date: Sat, Jul 03, 2010 12:42:06 PM

Output Date: Tue, Jul 06, 2010 11:43:37 AM

Rev II

Project: Management Accounting_Debarshi Bhattacharyya ACE Pro India Pvt. Ltd. File: X:\Pearson\Management Accounting_Debarshi Bhattacharyya\MAIN\M03\LAYOUT_M03\M03_DEBA_ISBN_EN_SE_C03_Part-3.indd

ACCOUNTING RATIOS FOR FINANCIAL STATEMENT ANALYSIS

181

II. Practical Problems:

1. From the following information as extracted from the books of P Ltd, ascertain the amount of Capital Employed in the business: Rs. 4,00,000 2,00,000 50,000 1,10,000 20,000 1,00,000 50,000 47,000 23,000 20,000 10,000 30,000

Equity Share Capital Preference Share Capital Securities Premium General Reserve Capital Reserve 10% Debentures 12% Bank Loan Sundry Creditors Bills Payable Preliminary Expenses Discount on Issue of Shares Profit & Loss A/c

Ans.: Rs. 8,70,000. 2. From the following information as furnished by Z Ltd, compute the amount of Capital Employed in the business. Rs. 4,00,000 3,00,000 1,00,000 2,00,000 1,50,000 80,000 60,000 30,000 40,000 20,000 10,000 50,000 20,000 20,000 30,000

Land & Building Plant & Machinery Furniture & Fixture Investment [Long-term] Patents & Trade Mark Stock in Trade Sundry Debtors Bills Receivable Cash & Bank Preliminary Expenses Share Issue Expenses Sundry Creditors Bills Payable Outstanding Expenses Securities Premium

Ans.: Rs. 12,70,000. 3. Following is the Balance Sheet of Parajoy Ltd as on 31 December 2006: Liabilities Equity Share Capital Preference Share Capital Capital Reserve General Reserve Profit & Loss A/c 10% Debentures 7% Bank Loan Bank Overdraft Sundry Creditors Bills Payable Outstanding Expenses

Amount Rs. 4,00,000 2,00,000 80,000 1,80,000 2,20,000 3,00,000 1,00,000 80,000 85,000 10,000 5,000

Assets Goodwill Land & Building Plant & Machinery Furniture & Fittings Investments [long term] Stock in Trade Marketable securities Sundry Debtors Cash & Bank Bills receivable Prepaid Expenses Discount on Shares Advertisement Suspense

16,40,000

Modified Date: Sat, Jul 03, 2010 12:42:06 PM

Output Date: Tue, Jul 06, 2010 11:43:37 AM

Amount Rs. 2,00,000 3,00,000 5,00,000 80,000 1,20,000 1,50,000 50,000 1,10,000 60,000 20,000 10,000 20,000 20,000 16,40,000

Rev II

Project: Management Accounting_Debarshi Bhattacharyya ACE Pro India Pvt. Ltd. File: X:\Pearson\Management Accounting_Debarshi Bhattacharyya\MAIN\M03\LAYOUT_M03\M03_DEBA_ISBN_EN_SE_C03_Part-3.indd

182

MANAGEMENT ACCOUNTING

From the above Balance Sheet, compute: (a) Working Capital; (b) Proprietors’ Fund; (c) Capital Employed; (d) Current Ratio; (e) Liquid Ratio; (f) Proprietary Ratio; (g) Asset Proprietors’ Ratio; (h) Debt-Equity Ratio; (i) Gearing Ratio. Ans.: (a) Rs. 2,40,000; (b) Rs. 12,00,000; (c) Rs. 14,40,000; (d) 2.5:1; (e) 3.125:1; (f) 0.65:1; (g) 15:13; (h) 5:13; and (i) 2:1 or 5:7. 4. From the following information, calculate: (i) Working Capital; (ii) Capital Employed; (iii) Current Ratio; (iv) Acid Test Ratio; (v) Debt-Equity Ratio; (vi) Asset Proprietorship Ratio. Balance Sheet of X company Ltd as on 31 December 1995 Amount Rs. 1,50,000 30,000 24,000 48,000 12,000 16,000 2,000 10,000 2,92,000

Liabilities Equity Share Capital 12% Preference Share Capital Reserves & Surplus 15% Debentures Bank Loan Sundry Creditors Proposed Dividend Provision for Taxation

Assets Fixed Assets Stores Stock in trade Sundry Debtors Cash at Bank Cash in Hand Preliminary Expenses Discount on Issue of Shares

Amount Rs. 1,80, 000 12,000 32,000 10,000 4,000 2,000 48,000 4,000 2,92,000

[B.Com. (Hons), Calcutta University—1996]

Hints: (i) Here, Debt-Equity Ratio =

Debt Long-term Debt = Equity Proprietors’ Fund considered

(ii) Here, Asset Proprietorship Ratio =

Fixed Assets Proprietors’ Fund considered

Ans.: (i) Rs. 20,000; (ii) Rs. 2,00,000; (iii) 1.5:1; (iv) 4:7 or 0.57; (v) 6:19 or 0.315; and (vi) 45:38 or 1.184. 5. Calculate the following Ratios from the given information: a. Operating Profit Ratio. b. Working Capital Turnover Ratio. c. Debt-Equity Ratio. d. GP Ratio. e. Operating Ratio. f. Current Ratio. Information: Particulars Net sales CGS Operating Expenses Current Assets Current Liabilities Capital Employed Long-term Debts

Rs. 2,00,000 1,20,000 30,000 60,000 30,000 2,40,000 1,60,000

[ISC Exam—2006] Ans.: (a) 25%; (b) 6.67 times; (c) 2:3; (d) 40%; (e) 75%; and (f) 2:1.

Hints: Here, Debt-Equity Ratio =

Modified Date: Sat, Jul 03, 2010 12:42:06 PM

Debt Long-term Debt + Equity = Debt ( Long-term Debt + Proprietors’ Fund )

Output Date: Tue, Jul 06, 2010 11:43:37 AM

Rev II

Project: Management Accounting_Debarshi Bhattacharyya ACE Pro India Pvt. Ltd. File: X:\Pearson\Management Accounting_Debarshi Bhattacharyya\MAIN\M03\LAYOUT_M03\M03_DEBA_ISBN_EN_SE_C03_Part-3.indd

ACCOUNTING RATIOS FOR FINANCIAL STATEMENT ANALYSIS

183

6. The following is the Balance Sheet of a company: Liabilities Share Capital Reserves & Surplus Debentures Bank Overdraft Sundry Creditors

Rs. 15,00,000 6,00,000 5,00,000 2,00,000 12,00,000 40,00,000

Assets Fixed Assets Stock Book Debts Investment [short-term] Cash

Rs. 16,50,000 9,10,000 12,40,000 1,60,000 40,000 40,00,000

Annual sales—Rs. 74,40,000 and GP—Rs. 7,44,000. Bank overdraft is payable on demand. From the above information, you are required to calculate the following Ratios: (a) Debt-Equity Ratio; (b) Current Ratio; (c) Proprietary Ratio; (d) GP Ratio; (e) Debtors’ Turnover; (f) Stock Turnover Ratio. Ans.: (a) 0.23:1; (b) 1.61:1; (c) 0.525:1; (d) 10%; (e) 2 months; and (f) 7 times (approx.). Hints: Bank Overdraft payable on demand is to be treated as a quick liability. 7. Following are the trading and Profit & Loss A/c for the year that ended on 31 March 2007 and the Balance Sheet as on that date of a concern: Trading and Profit & Loss A/c for the year that ended on 31 March 2007 To Opening Stock To Purchases To Carriage inward To Wages To Gross Profit c/d To Salaries To Rent & Rates To Insurance To General Expenses To Depreciation To Selling & Distribution Expenses To Interest on Debentures To Provision for Taxation To Net Profit

Rs. 30,000 1,80,000 40,000 1,30,000 1,10,000 4,90,000 25,000 10,000 5,000 3,000 10,000 7,000 3,000 5,000 45,000 1,10,000

By Sales By Closing Stock

Rs. 4,40,000 50,000

By Gross Profit b/d

4,90,000 1,10,000

1,10,000

Balance Sheet as on 31 March 2007 Liabilities Equity Share Capital Preference Share Capital Reserves & Surplus 10% Debentures Bank Overdraft Sundry Creditors Bills Payable Provision for Taxation

Rs. 1,00,000 40,000 70,000 30,000 20,000 30,000 15,000 5,000 3,10,000

Assets Fixed Assets Stock in Trade Sundry Debtors Cash & Bank Bills Receivable Preliminary Expenses

Rs. 1,60,000 50,000 80,000 7,500 2,500 10,000

3,10,000

From the above information, calculate: (a) GP Ratio; (b) NP Ratio; (c) Operating Ratio; (d) Return on Capital Employed; (e) Return on shareholders’ fund; (f) Capital Turnover Ratio; (g) Fixed Asset Turnover Ratio; (h) Average Collection Period; (i) Average Payment period; (j) Stock Turnover Ratio.

Modified Date: Sat, Jul 03, 2010 12:42:06 PM

Output Date: Tue, Jul 06, 2010 11:43:37 AM

Rev II

Project: Management Accounting_Debarshi Bhattacharyya ACE Pro India Pvt. Ltd. File: X:\Pearson\Management Accounting_Debarshi Bhattacharyya\MAIN\M03\LAYOUT_M03\M03_DEBA_ISBN_EN_SE_C03_Part-3.indd

184

MANAGEMENT ACCOUNTING

Ans.: (a) 25%; (b) 11.36 36%; (c) 13.63 63%; (d) 23.04%; (e) 22.50%; (f) 44:23 or 1.1913; (g) 11:4 or 2.75; (h) 2.25 months; (i) 3 months; and (j) 8.25 times. 8. The following data have been adstructed from the annual accounts of a company: Rs. 3,00,000 1,00,000 50,000 1,20,000 1,50,000 2,00,000 2,10,000 60,000 55,000 21,000 12,000 20,000

Equity Share Capital 12% Preference Share Capital Capital Reserve General Reserve Profit & Loss A/c (Cr) 15% Debentures Profit after Tax Provision for Tax Tax already Paid during the Year Proposed Equity Dividend Proposed Preference Dividend Preliminary Expenses From the above information, calculate: (a) Return on Net Worth; (b) Return on Capital Employed.

Ans.: (a) 30% and (b) 33.33%. 9. The Balance Sheet of R Ltd stood as follows as on: Liabilities Capital Reserves Loans Creditors & Others Current Liabilities

31 March 31 March 2009 2008 Rs. in Lakhs Rs. in Lakhs 250 250 116 100 Less: 100 120 129

25

595

495

Assets Fixed Assets Depreciation Investment Stock Debtors Cash & Bank Other Current Assets Miscellaneous Expenditure

31 March 31 March 2009 2008 Rs. in Lakhs Rs. in Lakhs 400 300 140 100 260 200 40 30 120 100 70 50 20 20 25 25 60 70 595 495

You are given the following information for the year 2008–09: Rs. in Lakhs 600 150 24 60 50

Sales Profit before Interest & Tax Interest Provision for Tax Proposed Dividend

From the above particulars, calculate for the year 2008–09: i. Return on Capital Employed. ii. Stock Turnover Ratio. iii. Return on Net Worth. iv. Current Ratio. v. Proprietary Ratio [C.A. (Inter)—Adapted] Ans.: (a) 32.05% [on the basis of Average Capital Employed]; (b) 5.45 times [on the basis of Sales/Average stock]; (c) 22.53% [on the basis of Average Net Worth]; (d) 1.82 times; and (e) 0.57.

Modified Date: Sat, Jul 03, 2010 12:42:06 PM

Output Date: Tue, Jul 06, 2010 11:43:37 AM

Rev II

Project: Management Accounting_Debarshi Bhattacharyya ACE Pro India Pvt. Ltd. File: X:\Pearson\Management Accounting_Debarshi Bhattacharyya\MAIN\M03\LAYOUT_M03\M03_DEBA_ISBN_EN_SE_C03_Part-3.indd

ACCOUNTING RATIOS FOR FINANCIAL STATEMENT ANALYSIS

185

10. ABC Ltd has made a plan for the next year. It is estimated that the company will employ Total Assets of Rs. 10,00,000, with 50% of the assets being financed by loan capital at an interest of 10% p.a. The direct costs for the year are estimated at Rs. 6,00,000 and all other operating costs for the year are estimated at Rs. 1,00,000. The goods will be sold to customers at 150% of the direct costs. Rate of income tax is assumed to be 50%. You are required to calculate: (a) NP margin and (b) Return on Assets. [B.Com. (Hons), Kalyani University—2009 (O.P.)] Ans.: (a) 33 1 3 % and (b) 10%. 11. From the following details, calculate the Average Collection period: Sundry Debtors Bills Receivable Average Stock Inventory Turnover Ratio GP Ratio Credit Sales to Cash Sales Assume, 1 year = 360 Days

Rs. 3,20,000 Rs. 40,000 Rs. 5,40,000 9 10% 4:1

Ans.: 30 days. 12. Calculate the Average Collection Period from the following details: Average Inventory Debtors Inventory Turnover Ratio GP Ratio Credit Sales to Total Sales Bills Receivable

Rs. 50,000 Rs. 35,000 3 25% 60% Rs. 5,000

Ans.: 4 months. 13. From the following information, calculate the average-payment period: Rs. 45,000 5,000 40,000 50,000 3,90,000

Sundry Creditors Bills Payable Opening Stock of Raw Materials Closing Stock of Raw Materials Raw Materials Consumed

Ans.: 1.5 months. 14. From the following information, calculate the average-payment period: Sundry Creditors Bills Payable Opening Stock of Raw Materials Closing Stock of Raw Materials CGS Wages & Overheads included in CGS Cash purchases to Total Purchases [of Raw Materials]

Rs. 60,000 15,000 45,000 75,000 5,55,000 331 ÷ 3% 25%

Ans.: 3 months. 15. Following are the Ratios relating to the trading activities of a concern: Debtors’ Velocity Stock Velocity Creditors’ Velocity GP Ratio

Modified Date: Sat, Jul 03, 2010 12:42:06 PM

3 months 6 months 2 months 20%

Output Date: Tue, Jul 06, 2010 11:43:37 AM

Rev II

Project: Management Accounting_Debarshi Bhattacharyya ACE Pro India Pvt. Ltd. File: X:\Pearson\Management Accounting_Debarshi Bhattacharyya\MAIN\M03\LAYOUT_M03\M03_DEBA_ISBN_EN_SE_C03_Part-3.indd

186

MANAGEMENT ACCOUNTING

GP for the year that ended on 31 December 2006 was Rs. 5,00,000. Stock at the end of 2006 was Rs. 20,000 more than what it was at the begining of the year. Bills Payable and Bills Receivable at the end of 2006 were Rs. 36,667 and Rs. 60,000, respectively. You are required to ascertain: (a) Sales; (b) Sundry Debtors; (c) Sundry Creditors; (d) Closing Stock. [C.A. (Inter)—Adapted] Ans.: (a) Rs. 25,00,000; (b) Rs. 5,65,000; (c) Rs. 3,00,000; and (d) Rs. 10,10,000. 16. From the following information relating to a limited company, prepare a Balance Sheet and a statement of Proprietors’ Fund: Current Ratio Liquid Ratio Fixed Assets ÷ Proprietary Fund Working Capital Reserves & Surplus Bank Overdraft

2 1.5 3÷4 Rs. 75,000 Rs. 50,000 Rs. 10,000

There were no long-term loans or fictitious assets. [C.A. (Inter)—Adapted] Ans.: Share Capital—Rs. 2,50,000; Fixed Assets—Rs. 2,25,000; Stock—Rs. 52,500; Liquid Assets—Rs. 97,500; Liquid Liabilities—Rs. 65,000; Total of Balance Sheet—Rs. 3,75,000; and Total of Proprietors’ Fund—Rs. 3,00,000. 17. From the following information, prepare the Balance Sheet of P Ltd as on 31 March 2006: Current Ratio Liquid Ratio Fixed Asset Proprietorship Ratio Gearing Ratio Working Capital Reserves & Surplus Bank Overdraft Long-term Loan Miscellaneous Expenditure

2:1 1.25 0.75 1:3 Rs. 8,000 Rs. 4,000 Rs. 2,000 Nil Nil

Ans.: Equity Share Capital—Rs. 21,000; Preference Share Capital—Rs. 7,000; Fixed Assets—Rs. 24,000; Stock— Rs. 8,500; Liquid Assets—Rs. 7,500; Liquid liabilities—Rs. 6,000; and Total of Balance Sheet—Rs. 32,000. 18. WC of a company is Rs. 1,35,000, Current Ratio is 2.5, Liquid Ratio is 1.5, Proprietory Ratio is 0.75 and Gearing Ratio (Equity Capital/Preference Capital) is 2. Reserves and Surplus and Bank Overdraft are Rs. 90,000 and Rs. 30,000 respectively. There are no long-term loans and fictitious assets. From the above information, calculate the following: (a) Current Assets; (b) Current Liabilities; (c) Net Block (i.e., Fixed Assets); (d) Proprietors’ Fund; (e) Quick Liabilities; (f) Quick Assets; (g) Stock; (h) Preference and Equity capital. Also draw up the statement of proprietors’ fund. [C.A. (Inter)—Adapted] Ans.: (a) Rs. 2,25,000; (b) Rs. 90,000; (c) Rs. 1,35,000; (d) Rs. 2,70,000; (e) Rs. 60,000; (f) Rs. 90,000; (g) Rs. 1,35,000; and (h) Rs. 60,000, Rs. 1,20,000. Total of proprietors’ fund—Rs. 2,70,000. 19. From the following information, prepare a summarized Balance Sheet as on 31 December 2006: Stock Velocity Fixed Asset Turnover Ratio

6 4 (Continued)

Modified Date: Sat, Jul 03, 2010 12:42:06 PM

Output Date: Tue, Jul 06, 2010 11:43:37 AM

Rev II

Project: Management Accounting_Debarshi Bhattacharyya ACE Pro India Pvt. Ltd. File: X:\Pearson\Management Accounting_Debarshi Bhattacharyya\MAIN\M03\LAYOUT_M03\M03_DEBA_ISBN_EN_SE_C03_Part-3.indd

ACCOUNTING RATIOS FOR FINANCIAL STATEMENT ANALYSIS

Capital Turnover Ratio Gross Profit Ratio Debt Collection Period Creditors’ Payment Period Gross Profit

187

2 20% 2 months 73 days Rs. 60,000

Closing Stock was Rs. 5,000 in excess of Opening Stock. Ans.: Capital—Rs. 1,50,000; Fixed Assets—Rs. 75,000; Closing Stock—Rs. 42,500; Receivables—Rs. 50,000; Payables—Rs. 49,000; Cash & Bank—Rs. 31,500; and Total of Balance Sheet—Rs. 1,99,000.

Note: It is assumed that there is no long-term loan. 20. (a) From the following information, compute the GP Ratio: Opening Stock is Rs. 50,000; Closing Stock is 1.2 times of the Opening Stock; Stock Turnover is 4 times; Sales is Rs. 2,50,000. (b) From the following information, compute the Current Assets and Current Liabilities: Current Ratio—3:1; Quick Ratio—1:1; Closing Stock—Rs. 60,000; Bank Overdraft—Nil. [B.Com. (Hons), Calcutta University—2004] Ans.: (a) 12% and (b) CAs—Rs. 90,000, CLs—Rs. 30,000. 21. From the following information relating to Zebra Ltd, you are required to prepare its Balance Sheet: Current Ratio Acid Test Ratio Gross Profit sales Net Working Capital/Net Worth Sales/Fixed Assets Sales/Net Worth Sales/Debtors Reserve/Capital Net Worth/Long-term Loan Stock Velocity Ratio Share Capital

2.5 1.5 0.2 0.3 2 1.5 6 1 20 6 times Rs. 10,00,000

Ans.: Reserves—Rs. 10,00,000; Long-term loan—Rs. 1,00,000; Fixed Assets—Rs. 15,00,000; Stock—Rs. 4,00,000; Debtors—Rs. 5,00,000; Other Current Assets—Rs. 1,00,000; Current Liabilities—Rs. 4,00,000; and Total of Balance Sheet—Rs. 25,00,000. 22. From the following details, prepare a Balance Sheet: Current Ratio Liquid Ratio Stock Turnover Ratio Gross Profit Ratio Debt Collection Period Reserves to Capital Turnover of Fixed Assets Capital Gearing Ratio Fixed Assets to Net Worth Sales for the year

1.75 1.25 9 times 25% 1.5 months 0.2 1.2 0.6 1.25 Rs. 12,00,000

[C.S. (Inter)—Adapted] Ans.: Share Capital—Rs. 5,00,000; Reserves & Surplus—Rs. 1,00,000; Long-term loan—Rs. 3,00,000; Current Liabilities—Rs. 2,00,000; Fixed Assets—Rs. 7,50,000; Stock—Rs. 1,00,000; Debtors—Rs. 1,50,000; Cash— Rs. 1,00,000; and Total of Balance Sheet—Rs. 11,00,000.

Modified Date: Sat, Jul 03, 2010 12:42:06 PM

Output Date: Tue, Jul 06, 2010 11:43:37 AM

Rev II

Project: Management Accounting_Debarshi Bhattacharyya ACE Pro India Pvt. Ltd. File: X:\Pearson\Management Accounting_Debarshi Bhattacharyya\MAIN\M03\LAYOUT_M03\M03_DEBA_ISBN_EN_SE_C03_Part-3.indd

188

MANAGEMENT ACCOUNTING

23. From the following particulars, prepare a summarized Balance Sheet with details, as on 31 December 2006: Fixed Assets to Net Worth Current Ratio Reserve included in Proprietors’ Fund Acid Test Ratio Long-term Loan Current Liabilities

8:5 3:1 25% 3:2 Rs. 5,00,000 Rs. 1,00,000

Ans.: Share Capital—Rs. 3,75,000; Reserves—Rs. 1,25,000; Fixed Assets—Rs. 8,00,000; Stock—Rs. 1,50,000; Liquid assests—Rs. 1,50,000; and Total of Balance Sheet—Rs. 10,00,000.

Note: It is assumed that there is no Bank Overdraft. 24. From the following details available, prepare a summarized Balance Sheet of ABC Ltd, as on 31 December 2006: Fixed Assets to Net Worth Current Ratio Acid Test Ratio Reserves included in Proprietors’ Fund Current Liabilities Cash & Bank Balances Fixed Assets

0.75:1 5: 2 3: 2 1: 4 Rs. 2,00,000 Rs. 10,000 Rs. 6,00,000

Ans.: Share Capital—Rs. 6,00,000; Reserves—Rs. 2,00,000; Stock—Rs. 2,00,000; Receivables—Rs. 2,90,000; Longterm loan—Rs. 1,00,000; and Total of Balance Sheet—Rs. 11,00,000.

Note: It is assumed that there is no bank overdraft. 25. Assets of a company consist of Current Asset while its current liability comprises of bank credit and trade credit in the Ratio of 2:1. From the following information, prepare the Balance Sheet of the company as on 31 March 2009: Gross Margin Inventory Turnover Current Ratio Quick Ratio Reserves & Surplus to Cash Average Collection Period Working Capital Share Capital

20% 6 1.5 0.9 3 2 months Rs. 45,000 Rs. 1,99,500

Ans.: Fixed Asset—Rs. 1,95,000; Reserves & Surplus—Rs. 40,500; Stock—Rs. 54,000; Debtors—Rs. 67,500; Cash— Rs. 13,500; Bank credit—Rs. 60,000; Trade credit—Rs. 30,000; and Total of Balance Sheet—Rs. 3,30,000. 26. Following figures have been extracted from the books of Zara Ltd as on 31 March 2009: Equity Share Capital [of Rs. 10 each] 9% Preference Share Capital [of Rs. 10 each] Profit after Tax Equity Dividend Paid Market Price per Equity Share

Rs. 8,00,000 Rs. 3,00,000 Rs. 2,70,000 @ 20% Rs. 40

Calculate: (a) Dividend yield on equity share; (b) Earning per equity share; (c) Price Earning Ratio; (d) Cover for Preference and Equity dividend. Ans.: (a) 5%; (b) Rs. 3.0375; (c) 13.17; and (d) 1.44 times. 27. Calculate the WC requirement from the following information: Debtors’ Velocity Stock Velocity Creditors’ Velocity

60 days 90 days 75 days (Continued)

Modified Date: Sat, Jul 03, 2010 12:42:06 PM

Output Date: Tue, Jul 06, 2010 11:43:37 AM

Rev II

Project: Management Accounting_Debarshi Bhattacharyya ACE Pro India Pvt. Ltd. File: X:\Pearson\Management Accounting_Debarshi Bhattacharyya\MAIN\M03\LAYOUT_M03\M03_DEBA_ISBN_EN_SE_C03_Part-3.indd

ACCOUNTING RATIOS FOR FINANCIAL STATEMENT ANALYSIS

Gross Profit Ratio Sales Credit purchases Cash & Bank Balance

189

25% Rs. 20,00,000 1 ÷ 3 of CGS 2.5% of Sales

The company expects a 50% sales increment during the next year. Assume 1 year = 360 days. [C.A. (Inter)—Adapted] Ans.: Total Current Assets—Rs. 11,37,500; Total Current Liabilities—Rs. 1,56,250; and WC—Rs. 9,81,250. 28. From the following information, prepare a trading and Profit & Loss A/c for the year that ended on 31 March 2009 and a Balance Sheet as on that date: Gross Profit Ratio Net Profit Ratio Fixed Assets/Total Current Assets Fixed Assets/Capital Stock Turnover Ratio Net Profit to Capital Capital/Total Liabilities Fixed Assets Closing Stock Long-term Loan to Current Liabilities

25% 20% 5÷7 5÷4 10 1÷5 1÷2 Rs. 10,00,000 Rs. 1,00,000 1 ÷ .3

[C.A. (Inter)—Adapted] Ans.: GP—Rs. 2,00,000; Opening Stock—Rs. 20,000; Purchases—Rs. 6,80,000; Sales—Rs. 8,00,000; Other Operating expenses—Rs. 40,000; NP—Rs. 1,60,000; Capital—Rs. 8,00,000; Long-term loan—Rs. 4,00,000; Current Liabilities—Rs. 12,00,000; Liquid Assets—Rs. 13,00,000; and Balance Sheet total—Rs. 24,00,000. 29. A company having a net WC of Rs. 2,80,000 as on 30 June 2008 indicates the following Financial Ratios and performance figures: Current Ratio—2.4; Liquid Ratio—1.6; GP Ratio—20%; Inventory turnover (on cost of sales)—8; Credit allowed—1.5 months. The company’s Fixed Asset is equivalent to 90% of its Net Worth (Share Capital plus reserve), while reserve is 40% of the Share Capital. Prepare the Balance Sheet of the company as on 30 June 2008. [B.Com. (Hons), Kalyani University—2009 (N.P.)] Ans.: Share Capital—Rs. 20,00,000; Reserves—Rs. 8,00,000; Fixed Assets—Rs. 25,20,000; Stock—Rs. 1,60,000; Debtors—Rs. 2,00,000; Cash & Bank—Rs. 1,20,000; Current Liabilities—Rs. 2,00,000; and Balance Sheet total—Rs. 30,00,000. 30. Based on the following information of the Financial Ratios, prepare the Balance Sheet of a company as on 31 March 2009: Current Ratio Liquidity Ratio Stock Turnover Ratio Gross Profit Ratio Average Debt Collection Period Turnover Ratio to Net Fixed Assets Long-term Debt to Net Worth Fixed Assets to Net Worth Net WC

2.5 1.5 5 20% 2.4 months 2.5 7/25 0.80 Rs. 6,00,000

Ans.: Net Worth—Rs. 12,50,000; Long-term loan—Rs. 3,50,000; Current Liabilities—Rs. 4,00,000; Fixed Assets— Rs. 10,00,000; Stock—Rs. 4,00,000; Debtors—Rs. 5,00,000; Cash & Bank—Rs. 1,00,000; and Total of Balance Sheet—Rs. 20,00,000.

Modified Date: Sat, Jul 03, 2010 12:42:06 PM

Output Date: Tue, Jul 06, 2010 11:43:37 AM

Rev II

Project: Management Accounting_Debarshi Bhattacharyya ACE Pro India Pvt. Ltd. File: X:\Pearson\Management Accounting_Debarshi Bhattacharyya\MAIN\M03\LAYOUT_M03\M03_DEBA_ISBN_EN_SE_C03_Part-3.indd

190

MANAGEMENT ACCOUNTING

31. From the following particulars, prepare the Balance Sheet of Sand Co. Ltd as on 31 March 2009: Current Ratio Gross Profit Ratio Stock Velocity Debtors’ Velocity Creditors’ Velocity Cash Sales to Credit Sales Fixed Assets to Turnover Capital Block [employed] to Current Asset Capital Block: Net Profit Reserve Debentures to Share Capital Working Capital

2 25% 2 months 3 months 2 months 1:2 1:3 3:2 10% of Turnover 2.5% of Turnover 1:2 Rs. 4,00,000

Ans.: Share Capital—Rs. 6,00,000; Reserves—Rs. 60,000; Profit & Loss A/c—Rs. 2,40,000; Debentures— Rs. 3,00,000; Fixed Assets—Rs. 8,00,000; Stock—Rs. 3,00,000; Receivable—Rs. 4,00,000; Payable—Rs. 3,00,000; Other Current Liabilities—Rs. 1,00,000; Cash—Rs. 1,00,000; and Total of Balance Sheet—Rs. 16,00,000. 32. From the following information, prepare the Profit & Loss A/c for the year that ended on 31 March 2009 and a Balance Sheet as on that date: Current Ratio Quick Ratio Gross Profit Ratio Stock Velocity Debtors’ Velocity Net Profit Ratio [on Average Capital Employed] Proprietary Ratio Fixed Asset ÷ Proprietors’ Fund Capital Working Capital Bank overdraft

2.5 2 20% 5 73 days 10% 6 Rs. 4,00,000 Rs. 1,80,000 Rs. 30,000

Closing Stock is 20% higher than the Opening Stock. Current Assets contain stock, debtors and cash only. There is no fictitious asset. Ans.: GP—Rs. 1,37,500; Opening Stock—Rs. 1,00,000; Closing Stock—Rs. 1,20,000; Purchases—Rs. 5,70,000; Sales—Rs. 6,87,500; Other Operating expenses—Rs. 94,500; NP—Rs. 43,000; Creditors—Rs. 90,000; Debtors—Rs. 1,37,500; Cash & Bank—Rs. 42,500; and Balance Sheet total—Rs. 5,70,000. 33. Following information relates to the operation of Ekta Ltd: Current Ratio Inventory Turnover Ratio Gross Profit Margin Average Debt Collection Period Total Asset Turnover Total Debt to Equity Ratio Gross Profit Net Worth

2.20 5 20% 50 days 2 2 Rs. 36,00,000 Rs. 30,00,000

You are required to complete the following Balance Sheet of the company: Liabilities Net Worth Long-term Loan Current Liabilities

Modified Date: Sat, Jul 03, 2010 12:42:06 PM

Rs.

Assets

Rs.

Fixed Assets Inventories Receivables Cash & Bank

Output Date: Tue, Jul 06, 2010 11:43:37 AM

Rev II

Project: Management Accounting_Debarshi Bhattacharyya ACE Pro India Pvt. Ltd. File: X:\Pearson\Management Accounting_Debarshi Bhattacharyya\MAIN\M03\LAYOUT_M03\M03_DEBA_ISBN_EN_SE_C03_Part-3.indd

ACCOUNTING RATIOS FOR FINANCIAL STATEMENT ANALYSIS

191

Ans.: Long-term loan—Rs. 30,00,000; Fixed Assets—Rs. 24,00,000; Inventories—Rs. 36,00,000; Receivables Rs. 25,00,000; Cash & Bank—Rs. 5,00,000; and Total of Balance Sheet—Rs. 90,00,000. 34. From the following information of Jay Bharat Ltd, prepare trading and Profit & Loss A/c and Balance Sheet: Current Ratio Liquid Ratio Gross Profit Ratio Stock Turnover Debtors’ Turnover Net Profit Ratio Proprietors’ Fund to Equity Share Capital Working Capital Equity Share Capital

2.2:1 1.7:1 16% 5.6 times 6 times 7% 1.2:1 Rs. 1,44,000 65,000 shares of Rs. 10 each

[B.Com. (Hons), Calcutta University—2005 ] Ans.: GP—Rs. 64,000; Sales—Rs. 4,00,000; Stock—Rs. 60,000; CGS—Rs. 3,36,000; Other Operating expenses—Rs. 36,000; NP—Rs. 28,000; Current Liabilities—Rs. 1,20,000; Debtors—Rs. 66,667; Cash & Bank—Rs. 1,37,333; Reserves & Surplus—Rs. 1,30,000; Long-term loan—Nil; and Balance Sheet total—Rs. 9,00,000. 35. From the following information, prepare a Balance Sheet as on 31 March 2009: Current Ratio Acid Test Ratio Gross Profit Ratio Net Working Capital to Net Worth Ratio Fixed Asset Turnover Ratio Sales to Net Worth Ratio Sales to Debtors Ratio Net Worth to Long-term Loan Ratio Stock Velocity Paid-up Share Capital Reserve to Capital Ratio

2.5 1.5 0.2 0.3 2.0 1.5 6.0 20.0 2 months Rs. 10,00,000 1.0

[C.A. (Inter)—Adapted] Ans.: Fixed Asset—Rs. 15,00,000; Stock—Rs. 4,00,000; Debtors—Rs. 5,00,000; Cash & Bank—Rs. 1,00,000; Reserves—Rs. 10,00,000; Long-term loan—Rs. 1,00,000; Current Liabilities—Rs. 4,00,000; and Balance Sheet total—Rs. 25,00,000. 36. From the following information, prepare a Balance Sheet as on 31 March 2009: Gross Profit Gross Profit to Cost of Goods Sold Stock Velocity Opening Stock Accounts Receivable Velocity Accounts Payable Velocity Current Assets Bills Receivable Bills Payable Fixed Asset Turnover Ratio Assume 1 year = 360 days

Rs. 80,000 1/3 6 times Rs. 36,000 72 days 90 days Rs. 1,50,000 Rs. 20,000 Rs. 5,000 8 times

Ans.: Fixed Asset—Rs. 30,000; Closing Stock—Rs. 44,000; Debtors—Rs. 44,000; Creditors—Rs. 57,000; Proprietors’ Fund—Rs. 76,000; and Balance Sheet total—Rs. 1,38,000.

Modified Date: Sat, Jul 03, 2010 12:42:06 PM

Output Date: Tue, Jul 06, 2010 11:43:37 AM

Rev II

Project: Management Accounting_Debarshi Bhattacharyya ACE Pro India Pvt. Ltd. File: X:\Pearson\Management Accounting_Debarshi Bhattacharyya\MAIN\M03\LAYOUT_M03\M03_DEBA_ISBN_EN_SE_C03_Part-3.indd

192

MANAGEMENT ACCOUNTING

37. From the following information of a company, prepare its Profit & Loss A/c for the year that ended on 31 March 2009 and the Balance Sheet, as on that date: Current Ratio Quick Ratio [Ratio of Debtors & Bank balance to Current Liabilities] Gross Profit Ratio Fixed Asset to Shareholders’ equity NP to Issued Share Capital Net Current Asset Issued Share Capital Stock Turnover Ratio [CGS to Closing Stock] Average age of outstanding for the year

1.8 1.35 25% 80% 20% Rs. 2,00,000 Rs. 6,00,000 5 times 36.5 days

Hints: Net Current Asset means WC. Ans.: GP—Rs. 1,87,500; Sales—Rs. 7,50,000; Stock—Rs. 1,12,500; CGS—Rs. 5,62,500; Other Operating expenses— Rs. 67,500; NP—Rs. 1,20,000; Current Liabilities—Rs. 2,50,000; Debtors—Rs. 75,000; Bank—Rs. 2,62,500; Current Asset—Rs. 4,50,000; Fixed Asset—Rs. 8,00,000; and Balance Sheet total—Rs. 12,50,000. 38. From the following information, prepare a Profit & Loss A/c for the year that ended on 31 March 2009 and a Balance Sheet, as on that date: Share Capital Gross WC Long-term Loan Current Ratio Liquid Ratio

Rs. 5,00,000 Rs. 4,00,000 Rs. 1,00,0 00 2:1 1:1 1 33 % 3 6 times 20% 20% of GP 1 month 0.20

Fixed Asset to Turnover Stock Turnover Ratio Gross Profit Ratio Net Profit Debt Collection Period Reserves & Surplus to Share Capital

Hints: Gross WC means Current Asset. Ans.: GP—Rs. 3,00,000; Sales—Rs. 15,00,000; Stock—Rs. 2,00,000; CGS—Rs. 11,00,000; Other Operating expenses—Rs. 2,40,000; NP—Rs. 60,000; Current Liabilities—Rs. 2,00,000; Debtors—Rs. 1,15,000; Bank—Rs. 75,000; Fixed Asset—Rs. 5,00,000; and Balance Sheet total—Rs. 9,00,000. 39. From the following information, prepare a Balance Sheet as on 31 March 2009: Fixed Asset Working Capital Gross Profit Ratio Current Ratio Stock velocity Debtors’ Velocity Creditors’ Velocity Fixed Asset to Turnover Net Profit

Rs. 6,00,000 Rs. 4,00,000 25% 2 2 months 1.5 months 2 months 4 5% of Turnover 2 of NP 3

Reserves Capital Gearing

1:1

Ans.: Stock—Rs. 18,750; Debtors—Rs. 18,750; Creditors—Rs. 18,750; Cash & Bank—Rs. 7,62,500; Share capital—Rs. 4,87,500; Reserve—Rs. 5,000; Profit & Loss A/c—Rs. 7,500; Long-term loan—Rs. 5,00,000; Liquid liabilities—Rs. 3,81,250; and Balance Sheet total—Rs. 14,00,000.

Modified Date: Sat, Jul 03, 2010 12:42:06 PM

Output Date: Tue, Jul 06, 2010 11:43:37 AM

Rev II

Project: Management Accounting_Debarshi Bhattacharyya ACE Pro India Pvt. Ltd. File: X:\Pearson\Management Accounting_Debarshi Bhattacharyya\MAIN\M03\LAYOUT_M03\M03_DEBA_ISBN_EN_SE_C03_Part-3.indd

ACCOUNTING RATIOS FOR FINANCIAL STATEMENT ANALYSIS

193

40. Complete the following Balance Sheet, assuming that only the equity capital and retained profit figures are given: Liabilities Equity Capital Retained Earnings Creditors

Rs.

Assets

Rs.

Fixed Assets Inventories Debtors Cash

You are given the following further information: Total debt is two-third of Net Worth; Turnover of Total Assets is 1.8; 30 days’ sales are in the form of Debtors; Turnover of Inventory is 5; CGS in the year is Rs. 9,00,000; and Acid-test Ratio is 1:1. Ans.: Fixed Asset—Rs. 4,20,000; Inventory—Rs. 1,80,000; Debtors—Rs. 1,50,000; Creditors—Rs. 4,00,000; Cash— Rs. 2,50,000; and Balance Sheet total—Rs. 10,00,000. 41. Using the following information, complete the Balance Sheet of Tee Ltd: Owner’s Equity Debt-Equity Ratio Gross Profit Margin [based on Cost of Goods Sold] Average Debt Collection Period Total Asset Turnover Ratio Share Capital to Reserves & Surplus Credit Sales Stock Turnover Acid Test Ratio Owner’s Equity

Rs. 5,00,000 1:2 20% 3 months 2:1 4:1 60% 5 times 2.5:1 Rs. 5,00,000

Balance Sheet as on_______________ Liabilities Share Capital Reserves & Surplus Long-term Loan Creditors

Rs.

1,50,000

Assets

Rs.

Fixed Assets Stock Debtors Cash & Bank

Ans.: Share Capital—Rs. 4,00,000; Long-term loan—Rs. 2,50,000; Reserves & Surplus—Rs. 1,00,000; Fixed Assets—Rs. 2,25,000; Stock—Rs. 3,00,000; Debtors—Rs. 2,70,000; Cash & Bank—Rs. 1,05,000; and Total of Balance Sheet—Rs. 9,00,000. 42. From the following information, prepare a trading and Profit & Loss A/c for the year that ended on 31 March 2009 and a Balance Sheet as on that date: Gross Profit Ratio NP Ratio Return on Investment Rate of Tax Interest on Debt Fixed Asset Turnover Ratio Debtors’ Turnover Stock Turnover Current Ratio Debt Asset Ratio Current Liabilities Net sales

20% 6% 5% 50% Rs. 20,000 0.80 4.8 months 6 months 4.75 0.50 Rs. 1,00,000 Rs. 5,00,000

Ans.: GP—Rs. 1,00,000; CGS—Rs. 4,00,000; Tax—Rs. 30,000; Other Operating expenses—Rs. 20,000; NP— Rs. 30,000; Net Worth—Rs. 5,50,000; Long-term debt—Rs. 4,50,000; Stock—Rs. 2,00,000; Debtors—Rs. 2,00,000; Cash & Bank—Rs. 75,000; Fixed Asset—Rs. 6,25,000; and Balance Sheet total—Rs. 11,00,000.

Modified Date: Sat, Jul 03, 2010 12:42:06 PM

Output Date: Tue, Jul 06, 2010 11:43:37 AM

Rev II

Project: Management Accounting_Debarshi Bhattacharyya ACE Pro India Pvt. Ltd. File: X:\Pearson\Management Accounting_Debarshi Bhattacharyya\MAIN\M03\LAYOUT_M03\M03_DEBA_ISBN_EN_SE_C03_Part-3.indd

194

MANAGEMENT ACCOUNTING

43. From the following information, prepare a trading and Profit & Loss A/c for the year that ended on 31 March 2009 and a Balance Sheet, as on that date: Fixed Asset Fixed Asset Turnover Ratio Gross Profit Ratio Net Profit Ratio Fixed Asset to Current Asset Long-term Loan to Current Liabilities Current Ratio Share Capital to Reserves & Surplus Gearing Ratio Debtors’ Velocity Creditors’ Velocity Consumption of Raw Materials Stock of Raw Materials Stock of Finished Goods Bills Receivable Bills Payable

Rs. 10,50,000 2:1 25% 15% 1:1 1:3 2:1 5:2 4:1 6 times 6 months 40% of cost 4 months’ consumption 20% of cost Rs. 50,000 Rs. 25,000

Ans.: GP—Rs. 5,25,000; Raw materials consumed—Rs. 6,30,000; Sales—Rs. 21,00,000; Wages & Overheads— Rs. 9,45,000; Other Operating expenses—Rs. 2,10,000; NP—Rs. 3,15,000; Equity Share Capital—Rs. 8,00,000; Preference-Share Capital—Rs. 2,00,000; Reserves & Surplus—Rs. 4,00,000; Long-term loan—Rs. 1,75,000; Stock of raw materials—Rs. 2,10,000; Stock of finished goods—Rs. 3,15,000; Debtors—Rs. 3,00,000; Cash & Bank—Rs. 1,75,000; Bank overdraft—Rs. 2,10,000; Creditors—Rs. 2,90,000; and Balance Sheet total— Rs. 21,00,000. 44. A company provides you the following information relating to the year that ended on 31 March 2009: Sales Current Ratio Share Capital to Reserves & Surplus Rate of Income Tax Return on Net Worth Net Profit to Sales Inventory Turnover Ratio Cost of Goods Sold Interest on Debenture Sundry Debtors Sundry Creditors

Rs. 60,00,000 2 7:3 50% 25% 6.25% 12 Rs. 18,00,000 Rs. 60,000 Rs. 2,00,000 Rs. 2,00,000

Required: a. Determine the other operating expenses for the year that ended on 31 March 2009. b. Complete the following Balance Sheet as on 31 March 2009: Balance Sheet as on 31 March 2009 Liabilities Share Capital Reserves & Surplus 15% Debenture Sundry Creditors

Rs.

Assets

Rs.

Fixed Assets Stock Debtors Cash

Ans.: (a) Rs. 33,90,000; (b) Share Capital—Rs. 10,50,000; 15% Debenture—Rs. 4,00,000; Reserves & Surplus— Rs. 4,50,000; Creditors—Rs. 2,00,000; Fixed Assets—Rs. 17,00,000; Stock—Rs. 1,50,000; Debtors— Rs. 2,00,000; Cash—Rs. 50,000; and Total of Balance Sheet—Rs. 21,00,000.

Modified Date: Sat, Jul 03, 2010 12:42:06 PM

Output Date: Tue, Jul 06, 2010 11:43:37 AM

Rev II

Project: Management Accounting_Debarshi Bhattacharyya ACE Pro India Pvt. Ltd. File: X:\Pearson\Management Accounting_Debarshi Bhattacharyya\MAIN\M03\LAYOUT_M03\M03_DEBA_ISBN_EN_SE_C03_Part-3.indd

ACCOUNTING RATIOS FOR FINANCIAL STATEMENT ANALYSIS

195

45. From the following information, prepare a trading and Profit & Loss A/c for the year that ended on 31 March 2009 and a Balance Sheet as on that date of Quee Ltd: Gross Profit Ratio Stock Turnover Ratio Average Debt Collection Period Creditors’ Velocity Current Ratio Proprietary Ratio [Fixed Assets to Capital Employed] Net Profit to Equity Capital Capital Gearing Ratio [Preference Shares & Debentures to Capital Employed] General Reserve & Profit & Loss A/c to Issued Equity Capital Preference Share Capital to Debentures

25% 5 times 3 months 3 months 2 80% 10% 30% 25% 2

Cost of sales consists of 50% for materials. GP is Rs. 6,00,000. Ans.: GP—Rs. 12,50,000; CGS—Rs. 37,50,000; Sales—Rs. 50,00,000; Other Operating expenses—Rs. 9,70,000; NP—Rs. 2,80,000; Equity-Share Capital—Rs. 28,00,000; Preference-Share Capital—Rs. 10,00,000; Reserves & Surplus—Rs. 7,00,000; Debenture—Rs. 5,00,000; Stock—Rs. 2,00,000; Creditors—Rs. 4,68,750; Bank overdraft—Rs. 5,31,250; Fixed Asset—Rs. 40,00,000; and Balance Sheet total—Rs. 60,00,000. 46. Following are some of the figures (ratios & percentages) relating to three companies carrying on a similar type of business for the year that ended on 31 March 2009. State by giving reasons of which company, in your opinion, has put in the best performance and has followed the best financial policy: Ratios and Percentages Net Block to Net Worth Current Assets to Current Liabilities Quick Assets to Current Liabilities Gross Profit to Net Sales (%) Net Profit to Net Sales (%) Net Sales to Capital Dividend to Profit Earned (%)

X Ltd 1.70 0.94 0.75 20 8 1.50 40

Y Ltd 0.83 1.32 0.68 15 6 2.50 25

Z Ltd 0.90 1.03 0.22 12 3 3.50 50

[C.A.I.I.B—Adapted] Ans.: Best performance—X Ltd; Best financial policy—Y Ltd. 47. The following comparative percentages are obtained from the Financial Statements of two companies, X Ltd and Y Ltd: X Ltd (%) 5 12

NP [after Tax] to Sales NP [after Tax] to Shareholders’ equity

Y Ltd (%) 10 8

Explain how you would interpret these data and which company, in your opinion, appears to be more successful. [C.A. (Inter)—Adapted] Ans.: X Ltd. 48. Following is the Balance Sheet of Mumbai Ltd as on 31 March 2009: Liabilities Equity Share Capital General reserve Profit & Loss A/c 8% Debentures Sundry Creditors Bank Overdraft

Amount Rs. 20,000 4,000 6,000 16,000 8,000 2,000

Assets Goodwill Fixed Assets Stock in Trade Sundry Debtors Short-term Investment Cash in hand

Amount Rs. 12,000 28,000 6,000 6,000 4,000 6,000 (Continued)

Modified Date: Sat, Jul 03, 2010 12:42:06 PM

Output Date: Tue, Jul 06, 2010 11:43:37 AM

Rev II

Project: Management Accounting_Debarshi Bhattacharyya ACE Pro India Pvt. Ltd. File: X:\Pearson\Management Accounting_Debarshi Bhattacharyya\MAIN\M03\LAYOUT_M03\M03_DEBA_ISBN_EN_SE_C03_Part-3.indd

196

MANAGEMENT ACCOUNTING

Amount Rs. 4,000 2,000 62,000

Liabilities Provision for Taxation Proposed Dividend

Amount Rs.

Assets

62,000

Sales for the year amounted to Rs. 1,20,000. Calculate the relevant Ratios for testing (a) Liquidity; (b) Solvency; (c) Capital Structure; and (d) Capital gearing of the company. [B.Com. (Hons), Bombay University—Adapted] Ans.: (a) Current Ratio, Quick Ratio; (b) Debt-Equity Ratio, Proprietary Ratio; (c) Fixed Asset Proprietorship Ratio, Net Worth to Total Liabilities, Proprietary Ratio, Debt-Equity Ratio; and (d) Capital Gearing Ratio. 49. The following Ratios of D Ltd and their corresponding industry averages are available: Ratios

D Ltd 1.75 0.85 25% 6.5 35 days 9.2% Rs. 3.50

Current Liquid Stock to Working Capital Inventory turnover Debt Collection Period Return on Assets Earning per Share

Industry 2.10 2.25 20% 8.2 30 days 10.7% Rs. 2.75

You are required to comment on the financial position and performance of D Ltd. [M.Com., Calcutta University—Adapted] Ans.: Financial position and activity of D Ltd is weaker the industry average, but its return to shareholders is better than the industry average. 50. Following are the Financial Statements of Fatepuria Ltd: Profit & Loss A/c for the year that ended on 31 March 2008

Less: Less: Less: Less:

Rs. 3,90,000 3,35,400 54,600 22,750 31,850 5,850 26,000 10,400 15,600

Net Sales Cost of Goods Sold Gross Profit Administrative & Selling Expenses Operating Profit Interest on Loan Profit before Tax Provision for Tax [ 33 13 % on Rs. 3,00,000] profit after Tax

Balance Sheet as on 31 March 2008 Liabilities Equity Share Capital 7% Preference Share Capital Reserves & Surplus Long-term Loan Current Liabilities & Provisions: Sundry Creditors Quick Liabilities Provision for Tax

Modified Date: Sat, Jul 03, 2010 12:42:06 PM

Rs. 1,30,000 26,000 1,04,000 1,82,000 41,600 2,600 33,800 5,20,000

Assets Fixed Assets Less: Depreciation Investment Current Assets: Stock Debtors Cash & Bank

Output Date: Tue, Jul 06, 2010 11:43:37 AM

Rs. 4,68,000 1,30,000

Rs. 3,38,000 39,000 78,000 52,000 13,000 5,20,000

Rev II

Project: Management Accounting_Debarshi Bhattacharyya ACE Pro India Pvt. Ltd. File: X:\Pearson\Management Accounting_Debarshi Bhattacharyya\MAIN\M03\LAYOUT_M03\M03_DEBA_ISBN_EN_SE_C03_Part-3.indd

ACCOUNTING RATIOS FOR FINANCIAL STATEMENT ANALYSIS

197

Dividend declared and paid during the year was as follows: a. On Preference Shares, a regular dividend was paid. b. On Equity Shares, a cash dividend of Rs. 1.30 was paid. The market price per equity share of the company was quoted at Rs. 52 during the last week of March 2009. You are required to calculate the five Ratios that are important and relevant to a banker and comment on the financial position of the company. Ans.: Calculate: Current Ratio; Debt-Equity Ratio; Interest Coverage Ratio; Dividend Earnings Ratio; Price Earnings Ratio of equity shares. 51. The Balance Sheet of Jantar Mantar Ltd as on 31 March 2009 is as follows: Liabilities Equity Share Capital [shares of Rs. 10 each] Retained Earnings 10% Long-term Loan Current Liabilities

Rs. 90,000

Assets

Rs. 2,25,000 75,000

Fixed Assets Current Assets

30,000 1,20,000 60,000 3,00,000

3,00,000

The company’s Total Asset Turnover Ratio is 3, fixed operating cost is Rs. 1,50,000 and variable operating cost Ratio is 50%. The income tax rate is 50%. You are required to: i. Calculate the different types of leverage for the company. ii. Determine the likely level of EBIT if EPS is: (a) Re. 1; (b) Rs. 2; and (c) Re. 0. Ans.: (i) DOL—1.5, DFL—1.042 and DCL—1.563; (ii) (a) Rs. 30,000, (b) Rs. 48,000 and (c) Rs. 12,000. 52. The following figures relate to the year that ended on 31 March 2009 of two companies:

Less: Less: Less:

R Ltd Rs. in Lakhs 500 200 300 150 150 50 100

Sales Variable Cost Contribution Fixed Cost EBIT [i.e., Operating Profit] Interest on Loan Earnings before Tax (EBT)

S Ltd Rs. in Lakhs 1,000 300 700 400 300 100 200

You are required to: i. Calculate the operating, financial and combined leverage for the two companies. ii. Comment on the relative risk position of two companies. [I.C.W.A. (Stage IV)—Adapted] Ans.: (i) R Ltd 2 1.5 3

DOL DFL DCL

S Ltd 2.33 1.5 3.5

ii. S Ltd possesses a higher overall risk than R Ltd. 53. (a) From the following details, prepare a comparative Balance Sheet in a vertical form, showing sources and employment of fund with the broad break-up of the components of WC: Year Current Ratio Liquid Ratio Fixed Asset to Proprietary Fund Bank Overdraft Working Capital

Modified Date: Sat, Jul 03, 2010 12:42:06 PM

Output Date: Tue, Jul 06, 2010 11:43:37 AM

2007–08 2.50 1.20 0.70 Rs. 80,000 Rs. 2,25,000

2008–09 1.80 0.90 0.80 Rs. 1,20,000 Rs. 2,40,000

Rev II

Project: Management Accounting_Debarshi Bhattacharyya ACE Pro India Pvt. Ltd. File: X:\Pearson\Management Accounting_Debarshi Bhattacharyya\MAIN\M03\LAYOUT_M03\M03_DEBA_ISBN_EN_SE_C03_Part-3.indd

198

MANAGEMENT ACCOUNTING

There was no long-term loan or fictitious asset. (b) Comment upon the utilization of funds in 2008–09 as compared to that in 2007–08. [C.A. (Final)—Adapted] Ans.: (a) 2007–08 (Rs.) 5,25,000 3,75,000 1,50,000

Fixed Asset Current Asset Current Liabilities

2008–09 (Rs.) 9,60,000 5,40,000 3,00,000

(b) More funds were invested in the Fixed Asset in 2008–09 than that in 2007–08. 54. As the manager of a financial service company, you have received a proposal seeking a term loan, from a firm that is planning an investment in a Fixed Asset of Rs. 500 lakhs, in a new project. The loan is indicated to be repayable in three annual instalments commencing from the end of the 2nd year. The following information concerning the project is available: Rs. in Lakhs Gross Profit [before depreciation] Depreciation Interest on term Loan WC borrowing Provision for Tax

Year 1 75 50 25 10 

Year 2 100 45 45 15 

Year 3 150 40 30 20 10

Year 4 150 35 15 20 30

Assuming other techno-economic criteria to be satisfactory, you are required to: a. Compute any three Ratios which, in your opinion, would guide the financing decision. b. Interpret briefly such Ratios and give views on the proposal. [C.A. (Final)—Adapted] Ans.: (a) Average ROI: Before tax 12%; After tax 10%. Debt Service Coverage Ratio Cash Interest Cover on Term Loan

Year 1 2.60 2.60

Year 2 0.60 1.89

Year 3 0.90 4.00

Year 4 0.90 26.67

(b) The project will not earn a satisfactory ROI and the term-loan borrowing will seriously create a liquidity problem of the firm.

Modified Date: Sat, Jul 03, 2010 12:42:06 PM

Output Date: Tue, Jul 06, 2010 11:43:37 AM

Rev II

Project: Management Accounting_Debarshi Bhattacharyya ACE Pro India Pvt. Ltd. File: X:\Pearson\Management Accounting_Debarshi Bhattacharyya\MAIN\M04\LAYOUT_M04\M04_DEBA_ISBN_EN_SE_C04_I.indd

Cash Flow Analysis

4

LEARNING OBJECTIVES On completion of the study of the chapter, you should be able to understand: What is meant by Cash? What is a Cash Flow Statement? What are the uses and advantages of preparation of a Cash Flow Statement? Distinction between a Cash Flow Statement and a Cash Book. Distinction between a Cash Flow Statement and a Fund Flow Statement. How to prepare a Cash Flow Statement under the Traditional Method? How to prepare a Cash Flow Statement as per Indian Accounting Standard (AS-3) (Revised), as issued by the Institute of Chartered Accountants of India? What are the purposes of computation of Net Cash Flows from Operating, Investing and Financing Activities separately as per AS-3 (Revised)? Fundamental difference between a Cash Flow Statement prepared under the Traditional Method and that prepared under the specific format as prescribed in AS-3 (Revised).

4.1 WHAT IS CASH? Generally, cash refers to cash in hand and cash at bank. But, for the purpose of preparation of a Cash Flow Statement, the Council of the Institute of Chartered Accountants of India (ICAI) issued Accounting Standard (AS-3) (Revised) in the year 1997, where it is stated that ‘cash’ includes the following: i. Cash in hand; ii. Demand Deposits with bank; and iii. Cash Equivalents. Cash Equivalents, which are considered as a part of cash as per AS-3, are defined as ‘short-term highly liquid investments that are readily convertible into known amounts of cash and which are subject to an insignificant risk of changes in value.’ Therefore, as per AS-3 (Revised), as issued for the purpose of preparation and presentation of Cash Flow Statement, cash includes: i. Cash in Hand; ii. Cash at Bank; and iii. Short-term Investments or Marketable Securities. 4.2 DIFFERENCE BETWEEN CASH AND FUND The term Fund is generally defined as Net Working Capital (i.e., Current Assets − Current Liabilities) for the purpose of preparation and presentation of Fund Flow Statement. On the other hand, the term Cash is defined by the AS-3 as Cash in hand, Cash at bank and Short-term investments for the purpose of preparation and presentation of Cash Flow Statement. Therefore, fund includes cash, but cash does not represent the whole fund, rather a part of the fund only.

Modified Date: Mon, Jul 05, 2010 05:30:16 PM

Output Date: Tue, Jul 06, 2010 11:46:14 AM

Rev II

Project: Management Accounting_Debarshi Bhattacharyya ACE Pro India Pvt. Ltd. File: X:\Pearson\Management Accounting_Debarshi Bhattacharyya\MAIN\M04\LAYOUT_M04\M04_DEBA_ISBN_EN_SE_C04_I.indd

200

MANAGEMENT ACCOUNTING

4.3 WHAT IS CASH FLOW STATEMENT? Cash Flow Statement is a summarized statement showing sources of Cash Inflows and applications of cash outflows of an enterprise during a particular period of time (generally, a year). A Cash Flow Statement is prepared on the basis of the published data as disclosed by the Financial Statement of two different financial years. It summarizes all Cash Flows between two financial years. It is an essential tool for managerial decisionmaking. Cash Flow Statement reports the management Net Cash Flow (i.e., cash inflow less cash outflow or vice versa) from each activity of the enterprise as well as of the overall business of the enterprise. 4.4 WHY IS CASH FLOW STATEMENT PREPARED? As already discussed, Cash Flow Statement is used as an essential managerial tool. It is prepared to locate the various sources of Cash Inflows within the business and to identify the various uses of cash outflows from the business during a particular period. As it is a summarized statement of Cash Inflows and Cash Outflows for different activities of an enterprise, during a particular period, the management of that enterprise gets a vivid picture of the movement of cash resources.Thus, the management can assess the stronger and weaker area of movement of cash for different activities of the business and can draw up its future planning. 4.5 IMPORTANCE OF CASH FLOW STATEMENT Cash Flow Statement is issued as an important tool of Financial Statement analysis to the management. It helps the management to assess the ability of an enterprise to generate cash resources from its different activities and the needs of the enterprise to utilize those cash resources into its different activities. Analysis of sources and application of cash for different activities of an enterprise enables its management to prepare the reliable cash flow projection for the future planning. Cash Flow Statement is very much useful for a short-term planning of an enterprise. As Cash Flow Statement locates various inflows and outflows of cash for different activities of a concern, the management of the concern can easily assess the cash position of those activities and can also easily identify the stronger and weaker area of cash position of the different activities of the concern. 4.6 USES OF CASH FLOW STATEMENT Cash Flow Statement is a very useful tool to the management. It has the following uses: i. Prediction of Future Cash Flows: On the basis of Cash Flow Statement, an enterprise can predict the flows of cash from and to the organization for the near future period. ii. Short-term Financial Planning: With the help of the information as provided through the Cash Flow Statement, an enterprise can chalk out its future short-term financial planning as regards to the investment of its cash resources. iii. Liquidity and Solvency Position: It indicates to the management about the liquidity and solvency position of the enterprise, so that the management can take appropriate measures if it finds any weakness. iv. Disclosure of Movement of Cash: It discloses the movement of cash from and to the enterprise during a particular period, and any deviation from the budgeted figure can be identified. v. Efficiency in Cash Management: Cash Flow Statement, as already prepared for the current period, helps a lot to prepare a projected Cash Flow Statement for the future period. If the management observes any cash surplus or cash deficiency, which is likely to occur in the future period, it can make out an appropriate planning for investing the projected cash surplus or for arranging the necessary cash to meet the projected cash deficiency. vi. Identification of Stronger or Weaker Activity: As per AS-3, as issued by the ICAI, Cash Flow Statement is prepared in three parts, namely, Cash Flow from Operating Activities, Cash Flow from Investing Activities and Cash Flow from Financing Activities. Therefore, by preparing a Cash Flow Statement, the management of an enterprise can identify its stronger or weaker cash position of the respective activities of the enterprise, and accordingly, it can take an appropriate measure to resolve the matter.

Modified Date: Mon, Jul 05, 2010 05:30:16 PM

Output Date: Tue, Jul 06, 2010 11:46:14 AM

Rev II

Project: Management Accounting_Debarshi Bhattacharyya ACE Pro India Pvt. Ltd. File: X:\Pearson\Management Accounting_Debarshi Bhattacharyya\MAIN\M04\LAYOUT_M04\M04_DEBA_ISBN_EN_SE_C04_I.indd

CASH FLOW ANALYSIS

201

vii. Ability of Payment of Dividend: Cash Flow Statement shows whether an enterprise possesses adequate cash for payment of dividend against its shares after paying off its all operating expenses and interest on debt. Thus, it provides an appropriate information to the management as regards to the firm’s ability to pay the cash diviadend to its shareholders. viii. Efficiency in Operating Activities: Cash Flow Statement separately shows the efficiency of an enterprise in its Operating Activities. If any weakness is observed, the management can take an appropriate remedial measure. ix. Efficiency in Managerial Performance: Fixed Assets of an enterprise are acquired out of the funds raised from the long-term sources. If such long-term funds are financed by long-term borrowings, it is prudent to pay interest on these long-term borrowings and also to repay the principal amount of these long-term borrowings out of the cash generated from the Operating Activities. All these above information are exhibited by the Cash Flow Statement, and thus, it reflects the level of performance of the management. x. A Tool of Comparative Study: Cash Flow Statement is used by the management of an enterprise as an essential tool of comparative study of different activities, like operating, investing and Financing Activities of the enterprise, with the other enterprises existing under the same industry and also with the enterprises that are existing under the other industries. Thus, the management can locate the area of activity where a greater emphasis is to be given for the growth of the enterprise. 4.7 ADVANTAGES OF CASH FLOW STATEMENT All enterprises enjoy lots of benefits through the Cash Flow Statement, as the latter is having the following advantages: i. It provides adequate information as regards to the inflows and outflows of cash resources to and from the enterprise. ii. It provides separate information as regards to the inflows and outflows of cash from the different activities of the enterprise. iii. It evaluates the level of efficiency of the management of the enterprise as regards to the uses of its cash resources. iv. It discloses the liquidity and solvency position of the enterprise. v. It helps the management a lot for future cash planning of the enterprise. vi. It provides the necessary information to the management, so that the situation of excessive surplus cash or deficiency of cash can be avoided. vii. It locates the stronger and weaker activities of an enterprise. viii. It helps to prepare the budget for the future period. 4.8 LIMITATIONS OF CASH FLOW STATEMENT In spite of the immense usefulness of the Cash Flow Statement, it has its own limitations. They are as follows: i. Cash Flow Statement shows only the inflows and outflows of cash, and thus, it does not take into consideration the non-cash transactions of the enterprise that had taken place during the same period. ii. As it considers only the inflows and outflows of cash, the Net Cash Flow of a certain period does not necessarily mean the Net Profit of the business, as Net Profit is ascertained considering both cash as well as non-cash transactions. Thus, there may be a huge difference between the Net Cash Flow and the Net Profit for the same period. iii. As it discloses the Net Cash Flow, and not the Net Profit for a certain period, it is not a substitute of the profit and loss account (Profit & Loss A/c) or Income statement. iv. As it is prepared at the end of a certain period, it is not useful for rectifying the errors that had already taken place during that period.

Modified Date: Mon, Jul 05, 2010 05:30:16 PM

Output Date: Tue, Jul 06, 2010 11:46:14 AM

Rev II

Project: Management Accounting_Debarshi Bhattacharyya ACE Pro India Pvt. Ltd. File: X:\Pearson\Management Accounting_Debarshi Bhattacharyya\MAIN\M04\LAYOUT_M04\M04_DEBA_ISBN_EN_SE_C04_I.indd

202

MANAGEMENT ACCOUNTING

v. It is also not a substitute of the Fund Flow Statement, as it discloses only the cash position of the enterprise and not the financial position of the enterprise as a whole. vi. As it is prepared at the end of a certain period, it can provide only the indications about the probable inflows and outflows of cash for the future period, and not the exact inflows and outflows. 4.9 DIFFERENCES BETWEEN CASH BOOK AND CASH FLOW STATEMENT Apparently, a Cash Flow Statement may be considered as the summarized form of a Cash Book. But actually, there are some fundamental differences between the Cash Book and the Cash Flow Statement, which are as follows: Cash Book 1. Cash Book is the Book of Primary Entry for cash transactions. 2. In Cash Book, all cash receipts and cash payments are recorded. 3. Process of recording of cash transactions in the Cash Book takes place throughout the financial year. 4. Process of recording in the Cash Book is the most essential activity under the Financial Accounting System. 5. Primary objective behind the preparation of Cash Book is the recording of all cash transactions and to find out the balance of cash. 6. Cash Book exhibits lesser information as regards to movement of cash than the Cash Flow Statement. 7. Net Cash Flows from different activities of the enterprise, like operating, investing, and Financing Activities, are not separately exhibited in the Cash Book. 8. While preparing the Cash Book, cash refers to Cash in hand and Cash at bank (demand deposits) only.

Cash Flow Statement 1. Cash Flow Statement is a statement showing the sources and utilizations of cash. 2. In Cash Flow Statement, all inflows and outflows of cash are exhibited. 3. It is generally prepared at the end of the financial year. 4. Preparation of Cash Flow Statement is one of the important activities under the Management Accounting System. 5. The objective behind preparation of Cash Flow Statement is to provide necessary information as regards to sources and utilizations of cash. 6. Cash Flow Statement exhibits more detailed information regarding the movement of cash than the Cash Book. 7. Net Cash Flows from different activities of an enterprise are separately exhibited in the Cash Flow Statement prepared as per AS-3, as issued by the ICAI. 8. While preparing the Cash Flow Statement, cash includes Cash in hand, Cash at bank (demand deposits) and short-term investments.

4.10 DIFFERENCES BETWEEN CASH FLOW STATEMENT AND FUND FLOW STATEMENT Both Cash Flow Statement as well as Fund Flow Statement are very important tools for managerial decision making. But there are some fundamental differences between these two, which are as follows: Cash Flow Statement 1. Cash Flow Statement exhibits inflows and outflows of cash and cash equivalents alone. 2. While preparing a Cash Flow Statement, cash refers to cash in hand, cash at bank (demand deposits) and shortterm investments (i.e., Cash & Cash equivalents alone). 3. Cash Flow Statement is prepared on cash basis of accounting. 4. It is the most important tool for short-term financial analysis. 5. By preparing a Cash Flow Statement, the movement of cash alone can be known and identified. 6. Net Cash Inflows from different activities of an enterprise, such as operating, investing and Financing Activities, are separately exhibited through the Cash Flow Statement.

Fund Flow Statement 1. Fund Flow Statement exhibits inflows and outflows of funds (i.e., working capital) alone. 2. While preparing a Fund Flow Statement, fund refers to the working capital (i.e., Current Assets and Current Liabilities alone). 3. Fund Flow Statement is prepared on accrual basis of accounting. 4. It is the most important tool for long-term financial analysis. 5. By preparing a Fund Flow Statement, the movement of Working Capital items (i.e., Current Assets and Current Liabilities) alone can be known and identified. 6. Net fund flows from different activities of an enterprise are not separately exhibited through the Fund Flow Statement. (Continued)

Modified Date: Mon, Jul 05, 2010 05:30:16 PM

Output Date: Tue, Jul 06, 2010 11:46:14 AM

Rev II

Project: Management Accounting_Debarshi Bhattacharyya ACE Pro India Pvt. Ltd. File: X:\Pearson\Management Accounting_Debarshi Bhattacharyya\MAIN\M04\LAYOUT_M04\M04_DEBA_ISBN_EN_SE_C04_I.indd

203

CASH FLOW ANALYSIS

Cash Flow Statement 7. It shows the brief reasons for change in cash between two Balance Sheet dates. 8. Cash Flow Statement starts with the opening of cash and cash equivalents of a closing accounting period and ends with the closing Cash & Cash equivalents of that accounting period.

Fund Flow Statement 7. It shows the brief reasons for change in the working capital between two Balance Sheet dates. 8. Fund Flow Statement shows the amount changes between the opening Working Capital balances of an accounting period.

4.11 PROFORMA OF CASH FLOW STATEMENT PREPARED UNDER TRADITIONAL/CONVENTIONAL METHOD Cash Flow Statement of for the period Sources of Cash To Opening Cash & Bank Balance To Inflows of cash: New Issue of Shares New Issue of Shares Loan Raised Sale of Fixed Assets Interest Received on Investments Net Cash Flow from Operation Payment of Interest on Debt

Rs. – – – – – – – – –

Applications of Cash By Outflows of cash: Redemption of Preference Shares Redemption of Preference Shares Purchase of Fixed Assets Further Investments Made Payment of Dividend Payment of Tax Repayment of Loan By Closing Cash & Bank Balance

Rs. – – – – – – – – –

Note: Under traditional method, ‘Cash’ refers to ‘cash in hand and at bank’ (demand deposits) only.

4.12 CASH FLOW STATEMENT AS PRESCRIBED IN AS-3 Accounting standard (Revised)–3 (AS-3) as issued by the ICAI in March 1997, prescribes a specific format for presenting a Cash Flow Statement. Preparation of Cash Flow Statement as per AS-3 is mandatory in respect of accounting periods that are commencing on or after 01 April 2001 (i.e., from the accounting year 2001–02) for the following enterprises: i. Enterprises that have a turnover of more than Rs. 50 crores during a financial year. ii. Companies whose shares or debts are listed on a recognized stock exchange in India (Cash Flow Statement of listed companies shall be presented under indirect method only as prescribed in AS-3). AS-3 prescribes two forms of presentation of Cash Flow Statement, which are as follows: i. Direct Method: Under this method, the Net Cash Flow from Operating Activities is to be calculated directly by deducting the cash outflows from the Operating Activities from Cash Inflows from the Operating Activities. ii. Indirect Method: Under this method, the Net Cash Flow from Operating Activities is to be calculated indirectly by adding back all the non-operating and non-cash items debited and by deducting all the non-operating and non-cash items credited to the Profit & Loss A/c for an accounting year with the Net Profit for that year. For both the methods, ‘Cash’ refers to cash in hand, cash at bank (demand deposits alone) and short-term investments or marketable securities. All these combinedly are termed as ‘Cash and Cash Equivalents.’ Under both the above methods, Net Cash Flows from the operating, investing and Financing Activities are to be separately shown in the Cash Flow Statement. In other words, Cash Flow Statement prepared under both the above methods is subdivided into the following three parts: i. Net Cash Flows from Operating Activities. ii. Net Cash Flows from Investing Activities. iii. Net Cash Flows from Financing Activities.

Modified Date: Mon, Jul 05, 2010 05:30:16 PM

Output Date: Tue, Jul 06, 2010 11:46:14 AM

Rev II

Project: Management Accounting_Debarshi Bhattacharyya ACE Pro India Pvt. Ltd. File: X:\Pearson\Management Accounting_Debarshi Bhattacharyya\MAIN\M04\LAYOUT_M04\M04_DEBA_ISBN_EN_SE_C04_I.indd

204

MANAGEMENT ACCOUNTING

4.12.1 Cash Flows from Operating Activities Operating or trading activities are principal revenue-producing activities of an enterprise. In other words, principal business activities of an enterprise, other than investing and Financing Activities, are called Operating Activities. Say, in case of a book-publishing company, the principal business activities are printing and publication of books and their sales. The publishing company may have huge, excess cash which are invested outside the business from where the company may receive a substantial amount of cash in the form of interest, but this activity is not the principal business activity of the company and hence, should not be treated as an operating activity of the company. Its Operating Activities are printing and publication of books and their sales. All cash receipts (i.e., inflows of cash) and cash payments (i.e., outflows of cash) for printing, publication and sale of books are to be considered as Cash Flows from Operating Activities. Examples of Cash Inflows from Operating Activities: Cash sales of goods/services, collection from credit sales, etc. Examples of Cash out Flows from Operating Activities: Cash purchases of goods/ services, payment against credit sales, payment of factory expenses, payment of office & administration expenses, payment of selling & distribution expenses, etc. Illustration Cash sales of goods—Rs. 1,50,000; Collection from debtors—Rs. 3,50,000; Cash purchases of goods—Rs. 90,000; Payment to creditors—Rs. 2,20,000; Payment of wages—Rs. 50,000; Payment of factory expenses—Rs. 40,000; Payment of office & administration expenses—Rs. 25,000; Payment of selling & distribution expenses—Rs. 35,000; and Payment of tax on operating profit—Rs. 15,000. Calculate the Net Cash Flow from Operating Activities. Solution Calculation of Net Cash Flow from Operating Activities

Less:

Less:

Cash Inflows from Operating Activities: Cash Sales of Goods Collection from Debtors Cash Outflows from Operating Activities: Cash Purchases of Goods Payment to Creditors Payment of Wages Payment of Factory Expenses Payment of Office & Administration Expenses Payment of Selling & Distribution Expenses Cash Generated from Operation Payment of Tax on Operating Profit Net Cash Flow From Operating Activities

Rs.

Rs.

1,50,000 3,50,000

5,00,000

90,000 2,20,000 50,000 40,000 25,000 35,000

4,60,000 40,000 15,000 25,000

4.12.2 Cash Flows from Investing Activities Investing activities are the acquisition and disposal of long-term assets and other investments not included in the cash equivalents. In other words, Investing Activities arise out of acquisition and sale of Fixed Assets and long-term investments made outside the enterprise. Say, a book publishing company invests its funds for acquisition of Fixed Assets, even though they are used for carrying on its Operating Activities, or into shares of another company. Any inflows or outflows of cash from these activities of the company are to be treated as Cash Flows from Investing Activities. Examples of Cash Inflows from Investing Activities: Sale proceeds of Fixed Assets and long-term investments, income received from investments, etc. Examples of Cash Outflows from Investing Activities: Purchase of Fixed Assets, additions to Fixed Assets, investments made, etc.

Modified Date: Mon, Jul 05, 2010 05:30:16 PM

Output Date: Tue, Jul 06, 2010 11:46:14 AM

Rev II

Project: Management Accounting_Debarshi Bhattacharyya ACE Pro India Pvt. Ltd. File: X:\Pearson\Management Accounting_Debarshi Bhattacharyya\MAIN\M04\LAYOUT_M04\M04_DEBA_ISBN_EN_SE_C04_I.indd

205

CASH FLOW ANALYSIS

Illustration Sale proceeds of Fixed Assets—Rs. 5,50,000; Sale proceeds of long-term investments—Rs. 2,50,000; Income received from investments—Rs. 1,00,000; Purchase of Fixed Assets—Rs. 4,20,000; Investment (long-term) made—Rs. 3,00,000; and Payment of tax on investing income—Rs. 20,000. Calculate the Net Cash Flow from Investing Activities. Solution Calculation of Net Cash Flow from Investing Activities

Less:

Cash Inflows from Investing Activities: Sale Proceeds of Fixed Assets Sale Proceeds of Long-term Investments Income Received from Investments Cash Outflows from Investing Activities: Purchase of Fixed Assets Investment (Long-term) Made Payment of Tax on Investing Income Net Cash Flow from Investing Activities

Rs.

Rs.

5,50,000 2,50,000 1,00,000

9,00,000

4,20,000 3,00,000 20,000

7,40,000 1,60,000

4.12.3 Cash Flows from Financing Activities Financing activities are the activities of an enterprise which result in changes in size and composition of the owner’s capital (including Preference Share Capital) and borrowings of that enterprise. In other words, activities related to the total capital employed (i.e., equity & Preference Share Capital and long-term loans) of an enterprise are treated as Financing Activities. Any inflows or outflows of cash from these activities of the enterprise are to be treated as ‘Cash Flows from Financing Activities.’ Say, the book-publishing company issues its equity shares, redeems its debentures, etc. Examples of Cash Inflows from Financing Activities: Proceeds received from the issue of equity and preference shares and debentures, proceeds received from the long-term loans, etc. Examples of Cash out Flows from Financing Activities: Redemption of preference shares and debentures, repayment of long-term loans, payment of equity and preference dividend, payment of interest on loans, etc. Illustration Proceeds received from the issue of equity shares—Rs. 3,00,000; Proceeds received from the issue of preference shares—Rs. 2,00,000; Proceeds received from the long-term loans—Rs. 1,50,000; Redemption of preference shares—Rs. 1,50,000; Redemption of debentures—Rs. 1,00,000; Payment of dividend—Rs. 30,000; and Payment of interest—Rs. 20,000. Calculate Net Cash Flow from the Financing Activities. Solution Calculation of Net Cash Flow from Financing Activities

Less:

Cash Inflows from Financing Activities: Proceeds Received from Issue of Equity Shares Proceeds Received from Issue of Preference Shares Proceeds Received from Long-term Loans Cash Outflows from Financing Activities: Redemption of Preference Shares Redemption of Debentures Payment of Dividend Payment of Interest Net Cash Flow from Financing Activities

Modified Date: Mon, Jul 05, 2010 05:30:16 PM

Output Date: Tue, Jul 06, 2010 11:46:14 AM

Rs.

Rs.

3,00,000 2,00,000 1,50,000

6,50,000

1,50,000 1,00,000 30,000 20,000

3,00,000 3,50,000

Rev II

Project: Management Accounting_Debarshi Bhattacharyya ACE Pro India Pvt. Ltd. File: X:\Pearson\Management Accounting_Debarshi Bhattacharyya\MAIN\M04\LAYOUT_M04\M04_DEBA_ISBN_EN_SE_C04_I.indd

206

MANAGEMENT ACCOUNTING

4.13 PROFORMA OF CASH FLOW STATEMENT AS PRESCRIBED IN AS-3 4.13.1 Proforma of Cash Flow Statement under Direct Method [Paragraph (a) Page II – 910] Cash Flow Statement of for the period ended Rs. A. Cash Flows from Operating Activities: Cash Receipts from Customers Less: Cash Paid to Suppliers & Employees Cash Generated from Operation Less: Income Tax Paid Cash Flows from Operation before Extraordinary Items Add: Proceeds from any Disaster Settlement Net Cash Flow from Operating Activities B. Cash Flows from Investing Activities: Proceeds from Sale of Fixed Assets including Investments Less: Purchase of Fixed Assets Including Investments

– – – – – –

Interest Received Dividend Received Net Cash Flow from Investing Activities C. Cash Flows from Financing Activities: Proceeds from Issuance of Share Capital Proceeds from Long-term Borrowings Repayment of Long-term Borrowings including Redemption of Preference Shares

Less:

Interest Paid Dividend Paid Net Cash Flow from Financing Activities Net Increase in Cash & Cash Equivalents Cash & Cash Equivalents at the beginning of the period Cash & Cash Equivalents at the end of the period

Rs.

– – – – – –

Add:

Less:

Rs.

– – – – – –

Add:

– –

– – – – –

4.13.2 Proforma of Cash Flow Statement under Indirect Method [Paragraph (b) Page II – 910] Cash Flow Statement of for the period ended Rs. A. Cash Flows from Operating Activities: Net Profit for the Period before Taxation & Extraordinary Items Add: Adjustment for Non-current and Non-operating Items charged to Profit & Loss A/c: Depreciation Interest Paid Foreign Exchange Loss Loss on Sale of Fixed Assets & Investments Less:

Add:

Adjustment for Non-current and Non-operating Items credited to Profit & Loss A/c: Interest Earned Dividend Earned Profit on Sale of Fixed Assets & Investments Operating Profit before Working Capital Changes Increase in Operating Current Liabilities Decrease in Operating Current Assets

Rs.

Rs.



– – – –

– – – – –

– –

– – – – (Continued)

Modified Date: Mon, Jul 05, 2010 05:30:16 PM

Output Date: Tue, Jul 06, 2010 11:46:14 AM

Rev II

Project: Management Accounting_Debarshi Bhattacharyya ACE Pro India Pvt. Ltd. File: X:\Pearson\Management Accounting_Debarshi Bhattacharyya\MAIN\M04\LAYOUT_M04\M04_DEBA_ISBN_EN_SE_C04_I.indd

207

CASH FLOW ANALYSIS

Less:

Increase in Operating Current Assets Decrease in Operating Current Liabilities Cash Generated from Operation Less: Income Tax Paid Add: Proceeds from any Disaster Settlement Net Cash Flow from Operating Activities B. Cash Flows from Investing Activities: Proceeds from Sale of Fixed Assets including Investments Less: Purchase of Fixed Assets including Investments



– – – – – –

Add:

Interest Received Dividend Received Net Cash Flow from Investing Activities C. Cash Flows from Financing Activities: Proceeds from Issuance of Share Capital Proceeds from Long-term Borrowings Less:

Repayment of Long-term Borrowings including Redemption of Preference Shares

Less:

Interest Paid Dividend Paid Net Cash Flow from Financing Activities Net increase in Cash & Cash Equivalents Cash & Cash Equivalents at the beginning of the period Cash & Cash Equivalents at the end of the period

– – – –

– – – – – –

Add:

– –

– – – – –

Stop and Think The only difference between the direct and indirect methods lies in the calculation of Net Cash Flows from Operating Activities. Under direct method, the Net Cash Flow from the Operating Activities is directly calculated by deducting the cash outflows from the Operating Activities from the Cash Inflows from the Operating Activities. On the other hand, under the indirect method, the Net Cash Flow from the Operating Activities is indirectly calculated by adding back all the non-operating and non-cash items debited and by deducting all the non-operating and noncash items credited to the Profit & Loss A/c for an accounting year with the Net Profit for that year. Under both the methods, the Net Cash Flows from investing and Financing Activities are calculated following the same approach (i.e., no difference between both the methods).

4.14 FUNDAMENTAL DIFFERENCES BETWEEN CASH FLOW STATEMENT AS PER AS-3 AND TRADITIONAL METHOD i. As per Traditional Method, Cash means cash in hand and at bank (demand deposits) only, whereas as per AS-3, Cash includes not only cash in hand and at bank (demand deposits) but also short-term investments or marketable securities. ii. Cash Flows from different activities of an enterprise, such as operating, investing and Financing Activities, can be separately known through the Cash Flow Statement prepared as per AS-3, by virtue of which the enterprise can identify its stronger and weaker area of cash generation. But, as per Traditional Method, there is no such segregation of Cash Flows from the different activities of an enterprise. iii. As per AS-3, two separate approaches, namely, Direct and Indirect Method, may be followed for the preparation of a Cash Flow Statement, whereas a single approach is followed while preparing the Cash Flow Statement under Traditional Method. iv. Preparation of a Cash Flow Statement as per AS-3 is mandatory for all enterprises having an annual turnover of more than Rs. 50 crores and also for all companies whose shares or debts are listed on a recognized Stock Exchange of India. Enterprises those are not falling under the above categories may prepare their Cash Flow Statement as per the Traditional Method.

Modified Date: Mon, Jul 05, 2010 05:30:16 PM

Output Date: Tue, Jul 06, 2010 11:46:14 AM

Rev II

Project: Management Accounting_Debarshi Bhattacharyya ACE Pro India Pvt. Ltd. File: X:\Pearson\Management Accounting_Debarshi Bhattacharyya\MAIN\M04\LAYOUT_M04\M04_DEBA_ISBN_EN_SE_C04_I.indd

208

MANAGEMENT ACCOUNTING

Stop and Think Where a detailed information as regards to cash sales, collection from debtors, cash purchases, payment to creditors, and so on, are given in the problem, or can be calculated, the Cash Flow from the operating activity can be calculated following the Direct Method as well as the Indirect Method, as prescribed in AS-3 (Revised). More clearly, where the Profit & Loss A/c for an accounting period along with the opening and closing Balance Sheets of that accounting period are given in the problem, the Cash Flow from the operating activity can be calculated following both the Direct Method as well as the Indirect Method, as prescribed in AS-3 (Revised). In such a situation, students are advised to follow the Direct Method.

 Tutorial Notes to Students for Solving Problems 1. As per conventional/traditional approach, Cash refers to ‘cash in hand’ and ‘cash at bank’ only, but as per AS-3 (Revised), it refers to ‘cash and cash equivalents’ (i.e., Cash in hand, Cash at bank and Short-term investments/Marketable securities). 2. For all examination purposes (i.e., B.Com., M.Com., C.A., I.C.W.A., C.S., M.B.A., etc.) and for some practical purposes (i.e., companies having a turnover of more than Rs. 50 crores during a financial year and whose shares or debts are listed on a recognized stock exchange in India), Cash Flow Statement is compulsorily required to be presented, following the format as prescribed in AS-3 (Revised). 3. As per AS-3 (Revised), a Cash Flow Statement may be presented under two methods, that is, Direct Method and Indirect Method. Accordingly, students should know which method of presenting Cash Flow Statement is desirable on the basis of the information given in the problem. 4. Where details of information as regards to cash sales, collection from debtors, cash purchases, payment to creditors, and so on, are given in the problem or somehow can be calculated, the cash flow from Operating Activities can be calculated following the direct method as well as the indirect method, as prescribed in AS-3 (Revised). More clearly, where the Profit & Loss A/c for an accounting period, along with opening and closing Balance Sheets of that accounting period, is given in the problem, the cash flow from Operating Activities can be calculated following both the direct method as well as the indirect method, as prescribed in AS-3 (Revised). In such a situation, students are advised to follow the direct method. 5. Where only opening and closing Balance Sheets of an accounting period are given in the problem, along with some additional information, the cash flow from Operating Activities can be calculated following the indirect method only, as prescribed in AS-3 (Revised). 6. Under both direct and indirect methods, the method of calculating the cash flow from both Investing Activities and Financing Activities is the same, that is, no separate approach is followed under both direct and indirect methods, for the purpose of calculating the cash flow from Investing Activities and Financing Activities. 7. The sum of Net Cash Flows from operating, investing and Financing Activities represents net increase/decrease in cash and cash equivalents. Sum/deduction of this net increase/ decrease in cash and cash equivalents with/ from the cash and cash equivalents at the beginning of the period should always be equal to the cash and cash equivalents at the end of the period; and thus, students should satisfy themselves about the accuracy of their solution to the problem given in the examination.

4.15 WORKED-OUT PROBLEMS Problem 1 Choose the correct alternative from the following: i. Depreciation on Fixed Assets: (a) causes cash inflow; (b) causes cash outflow; (c) has no effect on Cash Flow. ii. Interest paid on debentures causes: (a) cash outflow from operating activities; (b) cash outflow from Financing Activities; (c) cash outflow from investing activities. iii. Cash sale of goods causes: (a) cash inflow from operating activities; (b) cash inflow from financing activities; (c) cash inflow from investing activities.

Modified Date: Mon, Jul 05, 2010 05:30:16 PM

Output Date: Tue, Jul 06, 2010 11:46:14 AM

Rev II

Project: Management Accounting_Debarshi Bhattacharyya ACE Pro India Pvt. Ltd. File: X:\Pearson\Management Accounting_Debarshi Bhattacharyya\MAIN\M04\LAYOUT_M04\M04_DEBA_ISBN_EN_SE_C04_I.indd

CASH FLOW ANALYSIS

209

iv. Payment of dividend on shares causes cash outflow from: (a) operating activities; (b) financing activities; (c) investing activities. v. Purchase of Fixed Assets causes cash outflow from: (a) operating activities; (b) financing activities; (c) investing activities. vi. Sale of long-term investment at a loss: (a) increases cash; (b) decreases cash; (c) has no effect on cash. vii. Redemption of debentures by converting them into shares: (a) increases cash; (b) decreases cash; (c) has no effect on cash. viii. If the net operating profit of a business concern is Rs. 60,000 and its debtors have increased during the year by Rs. 20,000, cash from operation is equal to: (a) Rs. 60,000; (b) Rs. 80,000; (c) Rs. 40,000. ix. Payment of income tax for which a provision has been made: (a) increases Cash Flow; (b) decreases Cash Flow; (c) has no effect on Cash Flow. x. Payment of wages and salaries causes cash outflow from; (a) operating activities; (b) financing activities; (c) investing activities. xi. Issue of equity shares for cash causes cash inflow from: (a) operating activities; (b) Financing Activities; (c) investing activities. xii. Dividend received from investment causes cash inflow from: (a) operating activities; (b) financing activities; (c) investing activities. Ans. (i) (c); (ii) (b); (iii) (a); (iv) (b); (v) (c); (vi) (a); (vii) (c); (viii) (c); (ix) (b); (x) (a); (xi) (b); (xii) (c). Calculation of Net Cash Flow from Operation under Conventional/Traditional Approach Problem 2 From the following information, calculate the Net Cash Flow from the operation of a company for the year that ended on 31 March 2007 under traditional approach: 31 March 2006 Rs. 2,00,000 56,000 54,000 48,000 18,000 14,000 12,000 15,000 34,000

Balance in Profit & Loss A/c Stock in Trade Sundry Debtors Sundry Creditors Bills Receivable Bills Payable Outstanding Expenses Prepaid Expenses Cash at Bank

31 March 2007 Rs. 2,40,000 36,000 82,000 72,000 27,000 11,000 16,000 9,000 56,000

While ascertaining the Net Profit for the year that ended on 31 March 2007, the following items were taken into the Profit & Loss A/c: Rs. 40,000 30,000 20,000 10,000 20,000 25,000 30,000 15,000 30,000 10,000

Transfer to General Reserve Proposed Dividend Goodwill written off Preliminary Expenses written off Interest on Debentures Depreciation on Fixed Assets Provision for Taxation Loss on Sale of Investment Profit on Sale of Machinery Income from Investment

Modified Date: Mon, Jul 05, 2010 05:30:16 PM

Output Date: Tue, Jul 06, 2010 11:46:14 AM

Rev II

Project: Management Accounting_Debarshi Bhattacharyya ACE Pro India Pvt. Ltd. File: X:\Pearson\Management Accounting_Debarshi Bhattacharyya\MAIN\M04\LAYOUT_M04\M04_DEBA_ISBN_EN_SE_C04_I.indd

210

MANAGEMENT ACCOUNTING

Solution Profit & Loss A/c

Dr.

Cr.

Amount (Rs.)

Particulars To Non-current & Non-operating Items Charged: Transfer to General Reserve Proposed Dividend Goodwill Preliminary Expenses Depreciation on Fixed Assets Provision for Taxation Loss on Sale of Investment Interest on Debentures To Balance c/f

Amount (Rs.) 2,00,000

Particulars By Balance b/f

40,000 30,000 20,000 10,000 25,000 30,000 15,000 20,000 2,40,000 4,30,000

By Non-current & Non-operating Items Credited: Profit on Sale of Machinery Income from Investment

By Net Fund Flow from Operation (Bal. fig.)

30,000 10,000

1,90,000 4,30,000

Statement showing calculation of Net Cash Flow from operation for the year that ended on 31 March 2007 Rs. Add:

Less:

Net fund flow from Operation for the year (as ascertained in above) Decrease in Operating Current Assets: Stock in Trade Prepaid Expenses Increase in Operating Current Liabilities: Sundry Creditors Outstanding Expenses Increase in Operating Current Assets: Sundry Debtors Bills Receivable Decrease in Operating Current Liabilities: Bills Payable Net Cash Flow from Operation

Rs. 1,90,000

20,000 6,000 24,000 4,000

54,000 2,44,000

28,000 9,000 3,000

40,000 2,04,000

Calculation of Net Cash Flow from Operating Activities as per AS-3

Problem 3 From the following information relating to Y Ltd, calculate the Cash Flow from Operating Activities: Rs. 57,500.00 5,000.00 2,000.00 4,500.00 3,200.00 20,000.00

Operating Profit before Changes in Operating Assets Debtors (Decrease) Stock (Increase) Bills Payable (Decrease) Creditors (Increase) Cash at Bank (Increase)

[B.Com. (Hons), Calcutta University—2006]

Modified Date: Mon, Jul 05, 2010 05:30:16 PM

Output Date: Tue, Jul 06, 2010 11:46:14 AM

Rev II

Project: Management Accounting_Debarshi Bhattacharyya ACE Pro India Pvt. Ltd. File: X:\Pearson\Management Accounting_Debarshi Bhattacharyya\MAIN\M04\LAYOUT_M04\M04_DEBA_ISBN_EN_SE_C04_I.indd

211

CASH FLOW ANALYSIS

Solution Statement showing calculation of Cash Flow from Operating Activities (under Indirect Method) of Y Ltd for the period Rs. Add:

Operating Profit before changes in Operating Assets Increase in Operating Current Liabilities: Creditors Decrease in Operating Current Assets: Debtors

Rs. 57,500

3,200 5,000 8,200 65,700

Less:

Decrease in Operating Current Liabilities: Bills Payable Increase in Operating Current Assets: Stock

4,500 2,000 6,500 59,200

Cash Flows from Operating Activities

Stop and Think As the details of the Profit & Loss A/c for the year is not given in the problem, Net Cash Flow from the Operating Activities is to be calculated following the indirect method, as prescribed in AS-3.

Problem 4 Compute the Net Cash Flow from Operating Activities from the following details, by Indirect Method: Particulars Profit & Loss A/c Debtors Outstanding Rent Goodwill Prepaid Insurance Creditors

2006 Rs. 1,10,000 50,000 24,000 80,000 8,000 26,000

2007 Rs. 1,20,000 62,000 42,000 76,000 4,000 38,000

Solution Calculation of Net Cash Flow from Operating Activities (under Indirect Method) of for the year 2007 Rs. Add:

Add:

Net Profit for the year (Rs. 1,20,000 – Rs. 1,10,000) Adjustment for Non-current & Non-operating Items debited to Profit & Loss A/c: Goodwill written off (80,000 – 76,000) Operating Profit before Working Capital Changes Increase in Operating Current Liabilities: Outstanding Rent Creditors Decrese in Operating Current Assets: Prepaid Insurance

Rs. 10,000

4,000 14,000 18,000 12,000 4,000 34,000 (Continued)

Modified Date: Mon, Jul 05, 2010 05:30:16 PM

Output Date: Tue, Jul 06, 2010 11:46:14 AM

Rev II

Project: Management Accounting_Debarshi Bhattacharyya ACE Pro India Pvt. Ltd. File: X:\Pearson\Management Accounting_Debarshi Bhattacharyya\MAIN\M04\LAYOUT_M04\M04_DEBA_ISBN_EN_SE_C04_I.indd

212

MANAGEMENT ACCOUNTING

Rs. Less:

Increase in Operating Current Assets: Debtors Decrease in Operating Current Liabilities

Rs. 48,000

12,000 Nil 12,000 60,000

Net Cash Flow from Operating Activities

Problem 5 The following is the position of Current Assets and Current Liabilities of M Ltd: 2006 Rs. 1,000 10,000 15,000 20,000

Particulars Provision for Bad debts Short-term Loan Creditors Bills Receivable

2007 Rs. 3,000 19,000 10,000 40,000

The company incurred a loss of Rs. 45,000 during the year. Calculate the Net Cash Flows from the Operating Activities by Indirect Method. Solution Calculation of Net Cash Flow from Operating Activities (under Indirect Method) of M Ltd for the year 2007 Rs. Add:

Add:

Net Profit for the Year Adjustment for Non-current & Non-operating Items Debited to Profit & Loss A/c: Provision for Bad Debts (Rs. 3,000 – Rs. 1,000) Operating Profit before Working Capital Changes Increase in Operating Current Liabilities: Short-term Loan Decrease in Operating Current Assets

Rs. (45,000)

2,000 (43,000) 9,000 Nil 9,000 (34,000)

Less:

Increase in Operating Current Assets: Bills Receivable Decrease in Operating Current Liabilities: Creditors

20,000 5,000 25,000 (59,000)

Net Cash Flow from Operating Activities Note: Provision for bad debt is a non-cash item and therefore, it is added back to the profit for the year.

Problem 6 From the following, you are required to calculate the Net Cash Flow from the Operating Activities by Indirect Method: Particulars Balance of Profit & Loss A/c Debtors Bills Receivable

31 March 2007 Rs. 60,000 87,000 62,000

31 March 2008 Rs. 65,000 40,000 1,03,000 (Continued)

Modified Date: Mon, Jul 05, 2010 05:30:16 PM

Output Date: Tue, Jul 06, 2010 11:46:14 AM

Rev II

Project: Management Accounting_Debarshi Bhattacharyya ACE Pro India Pvt. Ltd. File: X:\Pearson\Management Accounting_Debarshi Bhattacharyya\MAIN\M04\LAYOUT_M04\M04_DEBA_ISBN_EN_SE_C04_I.indd

213

CASH FLOW ANALYSIS

31 March 2007 Rs. 2,02,000 78,000 30,000 5,000 80,000

Particulars General Reserve Dividend Equalization Fund Salary Outstanding Wages Prepaid Goodwill

31 March 2008 Rs. 2,37,000 1,00,000 12,000 7,000 70,000

Solution Calculation of Net Cash Flow from Operating Activities (under Indirect Method) of for the year that ended on 31 March 2008 Rs. Add:

Add:

Add:

Less:

Net Profit for the Year (after Appropriation) (Rs. 65,000 – Rs. 60,000) Appropriation made during the year: Transfer to General Reserve Transfer to Dividend-equalization fund Net Profit before appropriation Adjustment for Non-current & Non-operating Items charged to Profit & Loss A/c: Goodwill written off Operating Profit before Working Capital Changes Increase in Operating Current Liabilities Decrease in Operating Current Assets: Debtors Increase in Operating Current Assets: Bills Receivable Wages Prepaid Decrease in Operating Current Liabilities: Salary outstanding

Rs. 5,000

35,000 22,000

57,000 62,000

10,000 72,000 Nil 47,000

47,000 1,19,000

41,000 2,000 18,000 61,000 58,000

Net Cash Flow from Operating Activities

Working Notes Analysis of Non-current Assets and Liabilities Assets/Liabilities Goodwill General Reserve Dividend Equalization Fund

Opening Balance Rs. 80,000 2,02,000 78,000

Closing Balance Rs. 70,000 2,37,000 1,00,000

Increase/ Decrease Rs. (−) 10,000 (+) 35,000 (+) 22,000

Analysis Goodwill written off Transfer during the year Transfer during the year

Problem 7 X Ltd made a profit of Rs. 1,20,000 after charging a depreciation of Rs. 20,000 on assets and a transfer to general reserve of Rs. 30,000. The goodwill written off was Rs. 7,000 and the gain on sale of machinery was Rs. 3,000. The other information available to you is (changes in the value of Current Assets and Current Liabilities) at the end of the year. Debtors showed an increase of Rs. 6,000, Creditors an increase of Rs. 10,000,

Modified Date: Mon, Jul 05, 2010 05:30:16 PM

Output Date: Tue, Jul 06, 2010 11:46:14 AM

Rev II

Project: Management Accounting_Debarshi Bhattacharyya ACE Pro India Pvt. Ltd. File: X:\Pearson\Management Accounting_Debarshi Bhattacharyya\MAIN\M04\LAYOUT_M04\M04_DEBA_ISBN_EN_SE_C04_I.indd

214

MANAGEMENT ACCOUNTING

Prepaid Expenses an increase of Rs. 200, Bills Receivable a decrease of Rs. 3,000, Bills Payable a decrease of Rs. 4,000 and Outstanding Expenses a decrease of Rs. 2,000. Ascertain the Net Cash Flow from the Operating Activities. [CBSE Examination—Adapted]

Solution Calculation of Net Cash Flow from Operating Activities (under Indirect Method) of X Ltd for the year Rs. Add:

Less:

Add:

Less:

Net Profit for the year Adjustment for Non-current and Non-operating items Debited to Profit & Loss A/c: Transfer to General Reserve Goodwill written off Depreciation Adjustment for Non-current and Non-operating items credited to Profit & Loss A/c: Gain on Sale of Machinery Operating Profit before Working Capital Changes Increase in Operating Current Liabilities: Creditors Decrease in Operating Current Assets: Bills Receivable Increase in Operating Current Assets: Debtors Prepaid Expenses Decrease in Operating Current Liabilities: Bills Payable Outstanding Expenses Net Cash Flow from Operating Activities

30,000 7,000 20,000

Rs. 30,000

57,000 87,000 3,000 84,000

10,000 3,000

13,000 97,000

6,000 200 4,000 2,000

12,200 84,800

Problem 8 From the following information, calculate the Net Cash Flow from the Operating Activities under indirect method for the year that ended on 31 March 2008: Profit & Loss A/c for the year that ended on 31 March 2008 Dr. Particulars To Loss on Sale of Land To Discount on Issue of Shares written off To Interest on debentures To Depreciation To Goodwill written off To General Reserve To Tax Provision To Proposed Dividend To Interim Dividend To Net Profit

Modified Date: Mon, Jul 05, 2010 05:30:16 PM

Rs. 40,000 10,000 18,000 1,20,000 15,000 25,000 30,000 1,80,000 70,000 5,10,000 10,18,000

Particulars By Gross Profit By Interest on Investment By Dividend Received By Profit on Sale of Plant By Rent Received By Refund of Tax By Insurance Claim Received for Earthquake By Commission Receivable

Output Date: Tue, Jul 06, 2010 11:46:14 AM

Cr. Rs. 8,20,000 15,000 18,000 20,000 12,000 8,000 90,000 35,000

10,18,000

Rev II

Project: Management Accounting_Debarshi Bhattacharyya ACE Pro India Pvt. Ltd. File: X:\Pearson\Management Accounting_Debarshi Bhattacharyya\MAIN\M04\LAYOUT_M04\M04_DEBA_ISBN_EN_SE_C04_I.indd

CASH FLOW ANALYSIS

215

Additional information: Particulars Debtors Creditors Stock Provision for Tax Accrued Commission Outstanding Wages Prepaid Expenses

31 March 2007 (Rs.) 25,000 15,000 1,40,000 50,000 15,000 20,000 18,000

31 March 2008 (Rs.) 1,00,000 50,000 1,00,000 60,000 30,000 25,000 20,000

Solution Calculation of Net Cash Flow from Operating Activities (under Indirect Method) of for the year that ended on 31 March 2008 Rs. Add:

Less:

Add:

Less:

Less: Add:

Net Profit for the year Adjustment for Non-current and Non-operating Items debited to Profit & Loss A/c: Loss on Sale of Land Discount on Issue of Share Interest on Debentures Goodwill written off Depreciation General Reserve Tax Provision Proposed Dividend Interim Dividend Adjustment for Non-current and Non-operating Items credited to Profit & Loss A/c: Interest on Investment Dividend Received Profit on Sale of Plant Rent Received Refund of Tax Insurance Claim for Earthquake Operating Profit before Working Capital Changes Increase in Operating Current Liabilities: Creditors Outstanding Wages Decrease in Operating Current Assets: Stock Increase in Operating Current Assets: Debtors Accrued Commission Prepaid Expenses Decrease in Operating Current Liabilities Cash Generated from Operation Income Tax Paid (see Working Notes) Net Cash Flow from Operating Activities before Extraordinary Item Extraordinary Income: Insurance Claim for Earthquake Net Cash Flow from Operating Activities

Modified Date: Mon, Jul 05, 2010 05:30:16 PM

Output Date: Tue, Jul 06, 2010 11:46:14 AM

40,000 10,000 18,000 15,000 1,20,000 25,000 30,000 1,80,000 70,000

15,000 18,000 20,000 12,000 8,000 90,000

Rs. 5,10,000

5,08,000 10,18,000

1,63,000 8,55,000

35,000 5,000 40,000

75,000 15,000 2,000 Nil

80,000 9,35,000

92,000 8,43,000 20,000 8,23,000 90,000 9,13,000

Rev II

Project: Management Accounting_Debarshi Bhattacharyya ACE Pro India Pvt. Ltd. File: X:\Pearson\Management Accounting_Debarshi Bhattacharyya\MAIN\M04\LAYOUT_M04\M04_DEBA_ISBN_EN_SE_C04_I.indd

216

MANAGEMENT ACCOUNTING

Working Notes 1.

Provision for Tax Account Cr.

Dr. To Bank – Tax paid for the year (Bal. fig.) To Balance c/f

Rs. 20,000

By Balance b/f By Profit & Loss A/c – Tax provided for the year

60,000 80,000

Rs. 50,000 30,000 80,000

2. Commission received is considered as an income from Operating Activities.

Stop and Think Alternatively, the Net Cash Flow from the Operating Activities could be calculated adding the amount of commission receivable (i.e., Rs. 35,000) as credited to the Profit & Loss A/c with the Gross Profit (i.e., Rs. 8,20,000). But, in such calculation, the proforma for calculating the Net Cash Flow from Operating Activities, under Indirect Method, as prescribed in AS-3 would not have been followed.

Problem 9 From the following information, calculate the Net Cash Flow from the Operating Activities of a concern for the year that ended on 31 March 2007: Cash Sales for the year Credit Sales for the year Collection from Debtors during the year Cash Purchases for the year Credit Purchases for the year Payment to Creditors during the year Wages Paid during the year Outstanding Wages for the year Salaries Paid during the year Salaries for the year General Expenses Paid during the year Unpaid General Expenses for the year Depreciation on Fixed Assets for the year Loss of Stock due to fire Insurance Claim Received against Loss of Stock Interest Received on Investment during the year (of which Rs. 3,000 provided for Interest Income) Payment of Income Tax during the year (of which Rs. 2,000 Paid for Interest Income)

Rs. 1,20,000 6,60,000 4,40,000 80,000 3,25,000 2,40,000 60,000 18,000 30,000 40,000 20,000 6,000 26,000 28,000 10,000 20,000 32,000

Stop and Think As detailed information as regards to cash sales, collection from debtors, cash purchases, payment to creditors, and so on are given in the problem, the Cash Flow from the Operating Activities can be calculated, in this case, following the Direct Method, as prescribed in AS-3 (Revised).

Modified Date: Mon, Jul 05, 2010 05:30:16 PM

Output Date: Tue, Jul 06, 2010 11:46:14 AM

Rev II

Project: Management Accounting_Debarshi Bhattacharyya ACE Pro India Pvt. Ltd. File: X:\Pearson\Management Accounting_Debarshi Bhattacharyya\MAIN\M04\LAYOUT_M04\M04_DEBA_ISBN_EN_SE_C04_I.indd

217

CASH FLOW ANALYSIS

Solution Statement showing computation of Net Cash Flow from Operating Activities (under Direct Method) of a concern for the year that ended on 31 March 2007 Rs. Cash Receipts from Customers: Cash Sales Collection from Debtors

Rs.

1,20,000 4,40,000 5,60,000

Less:

Less: Add:

Cash Paid to Suppliers & Employees: Cash Purchases Payment to Creditors Payment of Wages Payment of Salaries Payment of General Expenses Cash Generated from Operation Payment of Income Tax (for Operating Activities) Cash Flow from Operation before Extraordinary Item Extraordinary Income: Insurance Claim Received against Abnormal Loss of Stock Net Cash Flow from Operating Activities

80,000 2,40,000 60,000 30,000 20,000

4,30,000 1,30,000 30,000 1,00,000 10,000 90,000

Tutorial Note Interest received on investment is not an operating income (rather investing income) and that is why, it is not considered while calculating the Cash Flow from Operating Activities. Due to the same reason, income tax paid on such income is not taken into consideration.

Problem 10 From the following information, calculate the Net Cash Flow from the Operating Activities of P Ltd for the year that ended on 31 March 2007. Profit & Loss A/c for the year that ended on 31 March 2007 Dr.

Cr. Rs.

To Opening Stock To Purchases: Cash Credit To Wages: Paid Outstanding To Salaries: Paid Outstanding To Rent Paid To Depreciation on Assets To Preliminary Expenses written off To Provision for Taxation To Net Profit for the year

Modified Date: Mon, Jul 05, 2010 05:30:16 PM

40,000 1,70,000 55,000 5,000 25,000 2,000

Rs. 43,000

Rs. By Sales: Cash Credit

2,10,000 By Closing Stock By Dividend from Investment

Rs.

70,000 3,42,000 4,12,000 35,000 12,000

60,000

27,000 5,000 18,000 10,000 43,000 43,000 4,59,000

Output Date: Tue, Jul 06, 2010 11:46:14 AM

4,59,000

Rev II

Project: Management Accounting_Debarshi Bhattacharyya ACE Pro India Pvt. Ltd. File: X:\Pearson\Management Accounting_Debarshi Bhattacharyya\MAIN\M04\LAYOUT_M04\M04_DEBA_ISBN_EN_SE_C04_I.indd

218

MANAGEMENT ACCOUNTING

Additional information: i. Balance of debtors and creditors were as follows: 1 April 2006 Rs. 44,000 45,000

Debtors Creditors

31 March 2007 Rs. 56,000 40,000

ii. Tax paid during the year amounted to Rs. 40,000. Stop and Think As details of the Profit & Loss A/c along with the opening and closing debtors’ and creditors’ balances are given in the problem, the Cash Flow from the Operating Activities, in this case, can be computed by following both Direct Method as well as Indirect Method, as prescribed in AS-3.

Solution i. Under Direct Method: Statement showing computation of Net Cash Flow from Operating Activities (under Direct Method) of P Ltd for the year that ended on 31 March 2007 Rs. Cash Receipts from Customers: Cash Sales Collection from Debtors (1)

Rs.

70,000 3,30,000 4,00,000

Less:

Less:

Cash Paid to Suppliers & Employees: Cash Purchases Payment to Creditors (2) Payment of Wages Payment of Salaries Payment of Rent Cash Generated from Operation Payment of Income Tax Net Cash Flow from Operating Activities

40,000 1,75,000 55,000 25,000 5,000

3,00,000 1,00,000 40,000 60,000

Working Notes 1. Dr. To Balance b/f To Sales – Credit

Debtors Account Rs. 44,000 By Bank – Collection (Bal. fig.) 3,42,000 By Balance c/f 3,86,000

Cr. Rs. 3,30,000 56,000 3,86,000

Creditors Account Rs. 1,75,000 By Balance b/f 40,000 By Purchases – Credit 2,35,000

Cr. Rs. 45,000 1,70,000 2,35,000

2. Dr. To Bank – Payment (Bal. fig.) To Balance c/f

Modified Date: Mon, Jul 05, 2010 05:30:16 PM

Output Date: Tue, Jul 06, 2010 11:46:14 AM

Rev II

Project: Management Accounting_Debarshi Bhattacharyya ACE Pro India Pvt. Ltd. File: X:\Pearson\Management Accounting_Debarshi Bhattacharyya\MAIN\M04\LAYOUT_M04\M04_DEBA_ISBN_EN_SE_C04_I.indd

CASH FLOW ANALYSIS

219

ii. Under Indirect Method: Statement showing computation of Net Cash Flow from Operating Activities (under Indirect Method) of P Ltd for the year that ended on 31 March 2007 Rs. Net Profit as per Profit & Loss A/c Add:

Less:

Add:

Less:

Less:

Non-operating & Non-current items Charged to Profit & Loss: Depreciation on Fixed Assets Preliminary Expenses written off Provision for Taxation Non-operating & Non-current Items Credited to Profit & Loss: Dividend from Investment Operating Profit before Working Capital Changes Increase in Operating Current Liabilities: Outstanding Wages Outstanding Salaries Decrease in Operating Current Assets: Stock Increase in Operating Current Assets: Debtors Decrease in Operating Current Liabilities: Creditors Cash Generated from Operation Payment of Income Tax Net Cash Flow from Operating Activities

18,000 10,000 43,000

Rs. 43,000

71,000 1,14,000 12,000 1,02,000

5,000 2,000 8,000

15,000 1,17,000

12,000 5,000

17,000 1,00,000 40,000 60,000

Problem 11 From the following particulars, calculate the Net Cash Flow from the Operating Activities of R Ltd for the year that ended on 31 March 2007: Rs. Balance of Profit & Loss A/c: As on 31 March 2007 As on 31 March 2006 Appropriation of Profit for the year 2006–07: Transfer to General Reserve Proposed Dividend Expenses and Losses for the year 2006–07: Interest on Debentures Depreciation on Fixed Assets Wages & Salaries Provision for Taxation Goodwill written off Loss on Sale of Machinery Incomes and Gains for the year 2006–07: Profit on Sale of Furniture Income from Investment Expenses Paid during the year 2006–07: Interest on Debentures Wages & Salaries Income Tax (of which Rs. 3,000 for Non-operating Income)

Modified Date: Mon, Jul 05, 2010 05:30:16 PM

Output Date: Tue, Jul 06, 2010 11:46:14 AM

3,98,000 2,72,000 24,000 20,000 16,000 18,000 37,000 40,000 14,000 12,000 13,000 7,000 18,000 39,000 53,000

Rev II

Project: Management Accounting_Debarshi Bhattacharyya ACE Pro India Pvt. Ltd. File: X:\Pearson\Management Accounting_Debarshi Bhattacharyya\MAIN\M04\LAYOUT_M04\M04_DEBA_ISBN_EN_SE_C04_I.indd

220

MANAGEMENT ACCOUNTING

Balances of Current Assets and Current Liabilities were as follows: As on 31 March 2006 (Rs.) 34,000 31,000 47,000 9,000 8,000 19,000 3,000

Sundry Debtors Sundry Creditors Stock in Trade Bills Receivable Bills Payable Cash & Bank Accrued Income from Investment

As on 31 March 2007 (Rs.) 62,000 48,000 43,000 7,000 3,000 36,000 4,000

Stop and Think As neither the details of Profit & Loss A/c for the year 2006–07 (i.e., cash sales, credit sales, cash purchases, credit purchases) are given in the problem, nor these can be ascertained out of the given information, the Cash Flow from the Operating Activities, in this case, cannot be computed by following the Direct Method, as prescribed in AS-3. So, this problem is to be solved following the indirect method, as prescribed in AS-3.

Solution Statement showing computation of Net Cash Flow from Operating Activities (under Indirect Method) of R Ltd for the year that ended on 31 March 2007

Less: Add:

Less:

Add:

Less:

Less:

Net Profit after Appropriation for the year: Balance of Profit & Loss A/c as on 31 March 2007 Balance of Profit & Loss A/c as on 31 March 2006 Non-operating and Non-current Items debited to Profit & Loss A/c: Transfer to General Reserve Proposes to Dividend Interest on Debentures Depreciation on Fixed Assets Provision for Taxation Goodwill written off Loss on Sale of Machinery Non-operating and Non-current Items Credited to Profit & Loss A/c: Profit on Sale of Furniture Income from Investment Operating Profit before Working Capital Changes Increase in Operating Current Liabilities: Sundry Creditors Decrease in Operating Current Assets: Bill Receivable Stock in Trade Increase in Operating Current Assets: Sundry Debtors Decrease in Operating Current Liabilities: Bills Payable Cash Generated from Operation Income Tax Paid for Operating Income (Rs. 53,000 – Rs. 3,000) Net Cash Flow from Operating Activities

Modified Date: Mon, Jul 05, 2010 05:30:16 PM

Output Date: Tue, Jul 06, 2010 11:46:14 AM

Rs.

Rs.

3,98,000 2,72,000

1,26,000

24,000 20,000 16,000 18,000 40,000 14,000 12,000

13,000 7,000

1,44,000 2,70,000

20,000 2,50,000

17,000 2,000 4,000

23,000 2,73,000

28,000 5,000

33,000 2,40,000 50,000 1,90,000

Rev II

Project: Management Accounting_Debarshi Bhattacharyya ACE Pro India Pvt. Ltd. File: X:\Pearson\Management Accounting_Debarshi Bhattacharyya\MAIN\M04\LAYOUT_M04\M04_DEBA_ISBN_EN_SE_C04_I.indd

CASH FLOW ANALYSIS

221

Tutorial Note i. As accrued income on investment is a non-operating current asset, it is not considered for calculating ‘Net Cash Flow from Operating Activities.’ ii. Income tax paid against operating income alone is deducted for calculating the ‘Net Cash Flow from the Operating Activities.’ iii. As Wages and Salaries are operating expenses, these are not added back.

Calculation of Net Cash Flow from Investing Activities as per AS-3. Problem 12 From the following information, calculate the Net Cash Flow from Investing Activities: Opening (Rs.) 4,00,000 1,00,000 2,80,000

Particulars Machinery (at Cost) Accumulated Depreciation Patents

Closing (Rs.) 4,20,000 1,10,000 1,60,000

Additional information: i. During the year, a machine costing Rs. 40,000 with an accumulated depreciation of Rs. 24,000 was sold for Rs. 20,000. ii. Patents were written off to the extent of Rs. 40,000 and some patents were sold at a profit of Rs. 20,000. Solution Calculation of Net Cash Flow from Investing Activities of ______________ for the period ______________

Less:

Inflows of Cash: Proceeds Received from Sale of Machinery Proceeds Received from Sale of Patents3 Outflows of cash: Purchase of Machinery1 Net Cash Flow from Investing Activities

Rs.

Rs.

20,000 1,00,000

1,20,000 60,000 60,000

Working Notes 1. Dr. To Balance b/f

To Profit & Loss A/c – Profit on Sale of Machinery [Rs. 20,000 – (Rs. 40,000 – Rs. 24,000)] To Bank – Purchase (Bal. fig.)

Machinery (at Cost) Account Rs. 4,00,000 By Provision for Depreciation A/c2 – Accumulated Depreciation on machine sold 4,000 By Bank—Sale proceeds

60,000 4,64,000

By Balance c/f

Cr. Rs. 24,000

20,000

4,20,000 4,64,000 (Continued)

Modified Date: Mon, Jul 05, 2010 05:30:16 PM

Output Date: Tue, Jul 06, 2010 11:46:14 AM

Rev II

Project: Management Accounting_Debarshi Bhattacharyya ACE Pro India Pvt. Ltd. File: X:\Pearson\Management Accounting_Debarshi Bhattacharyya\MAIN\M04\LAYOUT_M04\M04_DEBA_ISBN_EN_SE_C04_I.indd

222

MANAGEMENT ACCOUNTING

2. Dr. To Machinery A/c – Accumulated Depreciation on machine Sold & Transferred To Balance c/f

Provision for Depreciation Account Rs. 24,000 By Balance b/f By Profit & Loss A/c – Depreciation for the year (Bal. fig.) 1,10,000 1,34,000

Cr. Rs. 1,00,000 34,000

1,34,000

3. Dr.

Patents Account Rs. 2,80,000 By Profit & Loss A/c – Written off 20,000 By Bank – Sale proceeds of patents (Bal. fig.) By Balance c/f 3,00,000

To Balance b/f To Profit & Loss A/c – Profit on Sale of Patents

Cr. Rs. 40,000 1,00,000 1,60,000 3,00,000

Problem 13 From the following particulars, calculate the Cash Flow from Investing Activities of Miser Ltd for the year 2007: Purchased Rs. 1,80,000 2,00,000 4,40,000 –

Investments Goodwill Machinery Patents

Sold Rs. 1,00,000 – 1,50,000 1,00,000

Interest received on debentures held as an investment – Rs. 16,000. Dividend received on shares held as investments – Rs. 20,000. A plot of land was purchased out of surplus funds for investment purchases and was let out for commercial use and the rent received was Rs. 80,000. Solution Calculation of Net Cash Flow from the Investing Activities of Miser Ltd for the year 2007 Rs. Inflows of Cash: Sale Proceeds of Investments Sale Proceeds of Machinery Sale Proceeds of Patents Interest Received from Investments Dividend Received from Investments Rent Received from Land

Rs.

1,00,000 1,50,000 1,00,000 16,000 20,000 80,000 4,66,000

Less:

Outflows of Cash: Purchase of Investments Purchase of Goodwill Purchase of Machinery Net Cash Flow from Investing Activities

Modified Date: Mon, Jul 05, 2010 05:30:16 PM

Output Date: Tue, Jul 06, 2010 11:46:14 AM

1,80,000 2,00,000 4,40,000 8,20,000 (3,54,000)

Rev II

Project: Management Accounting_Debarshi Bhattacharyya ACE Pro India Pvt. Ltd. File: X:\Pearson\Management Accounting_Debarshi Bhattacharyya\MAIN\M04\LAYOUT_M04\M04_DEBA_ISBN_EN_SE_C04_I.indd

CASH FLOW ANALYSIS

223

Calculation of Net Cash Flow from Financing Activities as per AS-3 Problem 14 From the following information, as furnished by Arzoo Ltd, calculate its Net Cash Flow from the Financing Activities for the year that ended on 31 March 2008: 31 March 2007 Rs. 3,00,000 2,00,000 2,00,000

Equity Share Capital Preference Share Capital Debentures

31 March 2008 Rs. 5,00,000 1,00,000 2,50,000

During the year 2007–08, debentures of Rs. 1,00,000 were redeemed for cash and debenture interest paid was Rs. 30,000. Preference Dividend paid was Rs. 20,000 and equity dividend paid was Rs. 40,000. Solution Calculation of Net Cash Flow from the Financing Activities of Arzoo Ltd, for the year that ended on 31 March 2008 Rs. Inflows of Cash: Proceeds Received from Issue of Equity Shares Proceeds Received from Issue of Debentures

Rs.

2,00,000 1,50,000 3,50,000

Less:

Outflows of Cash: Redemption of Preference Shares Redemption of Debentures Payment of Debentures’ Interest Payment of Preference Dividend Payment of Equity Dividend

1,00,000 1,00,000 30,000 20,000 40,000 2,90,000 60,000

Net Cash Flow from Financing Activities

Working Notes 1. Dr.

Debentures Account Rs. 1,00,000 By Balance b/f 2,50,000 By Bank – New Issue (Bal. fig.) 3,50,000

To Bank – Redeemed To Balance c/f

Cr. Rs. 2,00,000 1,50,000 3,50,000

2. Analysis of other Non-current Liabilities Liabilities Equity Share Capital Preference Share Capital

Opening Balance Rs. 3,00,000 2,00,000

Modified Date: Mon, Jul 05, 2010 05:30:16 PM

Closing Balance Rs. 5,00,000 1,00,000

Increase/ Decrease Rs. (+)2,00,000 (−)1,00,000

Output Date: Tue, Jul 06, 2010 11:46:14 AM

Analysis New Issue of Equity Shares Redemption of Preference Shares

Rev II

Project: Management Accounting_Debarshi Bhattacharyya ACE Pro India Pvt. Ltd. File: X:\Pearson\Management Accounting_Debarshi Bhattacharyya\MAIN\M04\LAYOUT_M04\M04_DEBA_ISBN_EN_SE_C04_I.indd

224

MANAGEMENT ACCOUNTING

Problem 15 From the following particulars as furnished by Lava Ltd, calculate its Net Cash Flow from the Financing Activities for the year that ended on 31 March 2008: 31 March 2007 Rs. 3,00,000 2,00,000 2,00,000 80,000 36,000 6,000

Equity Share Capital Preference Share Capital Debentures Long-term Loan Proposed Dividend (Equity) Outstanding Interest on Debentures

31 March 2008 Rs. 5,00,000 2,00,000 1,50,000 1,50,000 50,000 5,000

During the year 2007–08, the following events took place: Rs. 1,00,000 1,00,000 50,000 20,000 10,000 15,000

New Issue of Preference Shares at 10% discount New Issue of Debentures at 20% Discount Repayment of Long-term Loan Payment of Preference Dividend Payment of Interest on long-term Loan Interim Equity Dividend paid during the year in addition to proposed Dividend Interest on Debentures for the year New Equity Shares were issued at 10% Premium

24,000

Solution Calculation of Net Cash Flow from the Financing Activities for the year that ended on 31 March 2008 Rs.

Less:

Inflows of Cash: Proceeds Received from new Issue of Equity Shares1 Proceeds Received from new Issue of Preference Shares Proceeds Received from new Issue of Debentures Raising of long-term Loan4 Premium Received on new Issue of Equity Shares (10% on Rs. 2,00,000) Outflows of Cash: Redemption of Preference Shares2 Redemption of Debentures3 Repayment of Long-term Loan Payment of Preference Dividend Payment of Proposed Equity Dividend6 Payment of Interim Equity Dividend Payment of Interest on Debentures5 Payment of Interest on Long-term Loan Net Cash Flow from Financing Activities

Modified Date: Mon, Jul 05, 2010 05:30:16 PM

Output Date: Tue, Jul 06, 2010 11:46:14 AM

Rs.

2,00,000 90,000 80,000 1,20,000 20,000 5,10,000 1,00,000 1,50,000 50,000 20,000 36,000 15,000 25,000 10,000

4,06,000 1,04,000

Rev II

Project: Management Accounting_Debarshi Bhattacharyya ACE Pro India Pvt. Ltd. File: X:\Pearson\Management Accounting_Debarshi Bhattacharyya\MAIN\M04\LAYOUT_M04\M04_DEBA_ISBN_EN_SE_C04_I.indd

CASH FLOW ANALYSIS

225

Working Notes 1. Dr.

To Balance c/f

Equity Share Capital Account Rs. By Balance b/f 5,00,000 By Bank – New Issue of Shares (Bal. fig.) 5,00,000

Cr. Rs. 3,00,000 2,00,000 5,00,000

2. Dr. To Bank – Redeemed (Bal. fig.)

To Balance c/f

Preference Share Capital Account Rs. 1,00,000 By Balance b/f By Bank – proceeds received from new issue of shares 2,00,000 By Discount on Issue of Shares A/c 3,00,000

Cr. Rs. 2,00,000 90,000 10,000 3,00,000

3. Dr. To Bank – Redeemed (Bal. fig.) To Balance c/f

Debentures Account Rs. 1,50,000 By Balance b/f By Bank – proceeds received from new issue 1,50,000 By Discount on Issue of Debentures 3,00,000

Cr. Rs. 2,00,000 80,000 20,000 3,00,000

4. Dr. To Bank – Repayment To Balance c/f

Long-term Loan Account Rs. 50,000 By Balance b/f 1,50,000 By Bank – Long-term Loan raised (Bal. fig.) 2,00,000

Cr. Rs. 80,000 1,20,000 2,00,000

5. Dr. To Bank – Interest Paid during the year (Bal. fig.) To Balance c/f

Interest on Debentures Account Rs. By Balance b/f 25,000 5,000 By Profit & Loss A/c – Interest Payable for the year 30,000

Cr. Rs. 6,000 24,000 30,000

6. Dr. To Bank – Dividend proposed in 2006–07 & paid in 2007–08 To Balance c/f

Modified Date: Mon, Jul 05, 2010 05:30:16 PM

Proposed Equity Dividend Account Rs. By Balance b/f 36,000 50,000 By Profit & Loss Appropriation A/c – Dividend proposed for 2007–08 86,000

Output Date: Tue, Jul 06, 2010 11:46:14 AM

Cr. Rs. 36,000

50,000 86,000

Rev II

Project: Management Accounting_Debarshi Bhattacharyya ACE Pro India Pvt. Ltd. File: X:\Pearson\Management Accounting_Debarshi Bhattacharyya\MAIN\M04\LAYOUT_M04\M04_DEBA_ISBN_EN_SE_C04_I.indd

226

MANAGEMENT ACCOUNTING

Preparation of Cash Flow Statement under Traditional/Conventional approach Problem 16 From the following particulars, prepare a Cash Flow Statement for the year that ended on 31 December 2007: Assets on 31 December 2006 Fixed Assets Cash in Hand Other Current Assets

Rs. 75,000 11,000 74,000

Liabilities on 31 December 2006 Bank Loan Trade Creditors Capital

Rs. 25,000 52,000 83,000

The Balance of Assets and Liabilities as on 31 December 2007 were as follows: Rs. 1,05,000 2,000 92,000

Fixed Assets Cash in Hand Other Current Assets

Bank Loan Trade Creditors Capital

Rs. 10,000 76,000 1,13,000

During the year 2007, the proprietor of the business withdrew Rs. 31,000 from the business and invested a further capital of Rs. 40,000 and also provided Rs. 15,000 as depreciation on Fixed Assets. Solution Cash Flow Statement of ______________ for the year that ended on 31 December 2007 Inflow of Cash Opening Cash Balance Add: Inflows of cash: Introduction of Fresh Capital Net Cash Flow from Operation4

Amount Rs. 11,000 40,000 42,000

Outflow of Cash Outflows of cash: Repayment of Bank Loan1 Purchase of Fixed Assets2 Proprietors’ Drawings Closing Cash Balance

93,000

Amount Rs. 15,000 45,000 31,000 2,000 93,000

Working Notes 1. Dr. To Bank – Repayment (Bal. fig.) To Balance c/f

Bank Loan Account Rs. 15,000 By Balance b/f 10,000 25,000

Cr. Rs. 25,000 25,000

2. Dr. To Balance b/f To Bank – Purchase (Bal. fig.)

Fixed Asset Account Rs. 75,000 By Profit & Loss A/c – Depreciation 45,000 By Balance c/f 1,20,000

Cr. Rs. 15,000 1,05,000 1,20,000

3. Dr. To Bank – Drawings To Balance c/f

Capital Account Rs. 31,000 By Balance b/f By Bank – Further capital invested 1,13,000 By Profit & Loss A/c – Net Profit for the year transferred (Bal. fig.) 1,44,000

Cr. Rs. 83,000 40,000 21,000 1,44,000 (Continued)

Modified Date: Mon, Jul 05, 2010 05:30:16 PM

Output Date: Tue, Jul 06, 2010 11:46:14 AM

Rev II

Project: Management Accounting_Debarshi Bhattacharyya ACE Pro India Pvt. Ltd. File: X:\Pearson\Management Accounting_Debarshi Bhattacharyya\MAIN\M04\LAYOUT_M04\M04_DEBA_ISBN_EN_SE_C04_I.indd

CASH FLOW ANALYSIS

227

4. Calculation of Net Cash Flow from operation Rs. 21,000

Net Profit for the Year3 Non-operating and Non-current items charged to Profit & Loss A/c: Depreciation on Fixed Assets Fund from operation Increase in Operating Current Liabilities: Trade creditors (Rs. 76,000 – Rs. 52,000)

Add:

Add:

Less:

15,000 36,000 24,000 60,000

Increase in Operating Current Assets: Other Current Assets (Rs. 92,000 – Rs. 74,000) Net Cash Flow from Operation

18,000 42,000

Problem 17 Balance Sheets of Bhaskar & Soumya Co. are given as follows: Liabilities Current Liabilities Loan from Bhaskar Bank Loan Capital

Year 1999 Rs. 2,40,000 – 3,20,000 9,60,000

Year 2000 Rs. 2,80,000 1,20,000 2,90,000 10,00,000

15,20,000

16,90,000

Assets Cash Debtors Stock Land Building Machinery

Year 1999 Rs. 30,000 2,50,000 1,90,000 2,00,000 3,70,000 4,80,000 15,20,000

Year 2000 Rs. 20,000 2,70,000 1,60,000 2,50,000 4,40,000 5,50,000 16,90,000

During the year 2000, Bhaskar & Soumya introduced an additional capital of Rs. 20,000 and drew Rs. 60,000. Provision for depreciation on machinery: Opening Balance—Rs. 2,00,000 and Closing Balance—Rs. 2,20,000. No depreciation was provided on other assets. The value of building was increased by Rs. 25,000 and the same was adjusted with the capital account. Prepare the Cash Flow Statement of Bhaskar & Soumya Co. for the year 2000. [B.Com. (Hons), Calcutta University—2002] Solution Cash Flow Statement of Bhaskar & Soumya Co. for the year that ended on 31 December 2000 Inflow of Cash Opening Cash Balance Add: Inflow of Cash: Loan taken from Bhaskar1 Introduction of additional capital6 Net Cash Flow from Operation7

Amount Rs. 30,000 1,20,000 20,000 1,25,000

Outflow of Cash Outflow of Cash: Partner’s drawings Repayment of Bank Loan2 Purchase of Land3 Purchase of Machinery5 Construction of Building4 Closing Cash Balance

2,95,000

Modified Date: Mon, Jul 05, 2010 05:30:16 PM

Output Date: Tue, Jul 06, 2010 11:46:14 AM

Amount Rs. 60,000 30,000 50,000 90,000 45,000 20,000 2,95,000

Rev II

Project: Management Accounting_Debarshi Bhattacharyya ACE Pro India Pvt. Ltd. File: X:\Pearson\Management Accounting_Debarshi Bhattacharyya\MAIN\M04\LAYOUT_M04\M04_DEBA_ISBN_EN_SE_C04_I.indd

228

MANAGEMENT ACCOUNTING

Working Notes 1. Dr.

To Balance c/f

Loan from Bhaskar Rs. By Balance b/f 1,20,000 By Bank – Loan taken (Bal. fig.) 1,20,000

Cr. Rs. Nil 1,20,000 1,20,000

2. Dr. To Bank – Repayment (Bal. fig.) To Balance c/f

Bank Loan Account Rs. 30,000 By Balance b/f 2,90,000 3,20,000

Cr. Rs. 3,20,000 3,20,000

3. Dr. To Balance b/f To Bank – Purchase (Bal. fig.)

Land Account Rs. 2,00,000 50,000 By Balance c/f 2,50,000

Cr. Rs. 2,50,000 2,50,000

4. Dr. To Balance b/f To Capital A/c – Revaluation To Bank – Construction of building (Bal. fig.)

Building Account Rs. 3,70,000 25,000 By Balance c/f 45,000 4,40,000

Cr. Rs.

4,40,000 4,40,000

5. Dr. To Balance b/f To Bank – Purchase (Bal. fig.)

Machinery Account Rs. 4,80,000 By Profit & Loss A/c – Depreciation for the year (Rs. 2,20,000 – Rs. 2,00,000) 90,000 By Balance c/f 5,70,000

Cr. Rs. 20,000 5,50,000 5,70,000

6. Dr. To Bank – Drawings

To Balance c/f

Partners’ Capital Account Rs. 60,000 By Balance b/f By Bank – Introduction of additional capital By Building A/c – Profit on Revaluation 10,00,000 By Profit & Loss A/c – Profit for the year transferred (Bal. fig.) 10,60,000

Cr. Rs. 9,60,000 20,000 25,000 55,000 10,60,000 (Continued)

Modified Date: Mon, Jul 05, 2010 05:30:16 PM

Output Date: Tue, Jul 06, 2010 11:46:14 AM

Rev II

Project: Management Accounting_Debarshi Bhattacharyya ACE Pro India Pvt. Ltd. File: X:\Pearson\Management Accounting_Debarshi Bhattacharyya\MAIN\M04\LAYOUT_M04\M04_DEBA_ISBN_EN_SE_C04_I.indd

CASH FLOW ANALYSIS

229

7. Calculation of Net Cash Flow from Operation Rs. Net Profit for the Year Non-current and Non-operating Items Charged to Profit & Loss A/c: Depreciation on Machinery Fund from Operation Increase in Operating Current Liabilities: Current Liabilities Decrease in Operating Current Assets: Stock

Add:

Add:

Less:

Rs. 55,000 20,000 75,000

40,000 30,000

Increase in Operating Current Assets: Debtors Net Cash Flow from Operation

70,000 1,45,000 20,000 1,25,000

Problem 18 From the following information, prepare the Cash Flow Statement for the year that ended on 31 March 2008: Balance Sheets as on ______________ Liabilities Share Capital Profit & Loss A/c Bank Loan Creditors Bills Payable

As on 31 March 2007 Rs. 1,50,000 20,000 1,50,000 80,000 50,000 4,50,000

As on 31 March 2008 Rs. 1,75,000 80,000 50,000 95,000 40,000 4,00,000

Assets Land & Building Machinery Stock Debtors Cash

As on 31 March 2007 Rs. 1,10,000 2,00,000 50,000 70,000 20,000 4,50,000

As on 31 March 2008 Rs. 1,50,000 1,40,000 45,000 80,000 25,000 4,00,000

Additional Information: i. Net Profit for the year 2007–08 amounted to Rs. 60,000. ii. During the year 2007–08, a machine costing Rs. 25,000 (accumulated depreciation was Rs. 10,000) was sold for Rs. 13,000. The provision for depreciation against machinery as on 31 March 2007 was Rs. 50,000 and on 31 March 2008 was Rs. 85,000. [B.Com. (Hons), Calcutta University—Adapted] Solution Cash Flow Statement of ______________ for the year that ended on 31 March 2008 Sources of Cash Opening Cash Balance Add: Inflow of Cash: Fresh Issue of Shares Sale proceeds of Machinery Net Cash Flow from Operation

Modified Date: Mon, Jul 05, 2010 05:30:16 PM

Amount Rs. 20,000 25,000 13,000 1,07,000 1,65,000

Applications of Cash Outflow of Cash: Repayment of Bank Loan Purchase of Land & Building Closing Cash Balance

Output Date: Tue, Jul 06, 2010 11:46:14 AM

Amount Rs. 1,00,000 40,000 25,000 1,65,000

Rev II

Project: Management Accounting_Debarshi Bhattacharyya ACE Pro India Pvt. Ltd. File: X:\Pearson\Management Accounting_Debarshi Bhattacharyya\MAIN\M04\LAYOUT_M04\M04_DEBA_ISBN_EN_SE_C04_I.indd

230

MANAGEMENT ACCOUNTING

Working Notes 1. Dr.

Cr.

Share Capital Account Rs.

To Balance c/f

By Balance b/f By Bank – Fresh Issue of Shares (Bal. fig.)

1,75,000 1,75,000

Rs. 1,50,000 25,000 1,75,000

2. Dr.

Cr.

Bank Loan Account

To Cash – Repayment (Bal. fig.) To Balance c/f

Rs. 1,00,000 50,000 1,50,000

Rs. 1,50,000

By Balance b/f

1,50,000

3. Dr.

Cr.

Land & Building Account

To Balance b/f To Cash – Purchase (Bal. fig.)

Rs. 1,10,000 40,000 1,50,000

Rs. By Balance c/f

1,50,000 1,50,000

4. Dr.

Cr.

Machinery Account

To Balance b/f

Rs. 2,00,000

By Cash – Sale proceeds By Profit & Loss A/c – Loss on sale of Machinery [(25,000 – 10,000) – 13,000] By Profit & Loss A/c – Depreciation for this year [85,000 – (50,000 – 10,000)] By Balance c/f

2,00,000

Rs. 13,000 2,000 45,000 1,40,000 2,00,000

5. Calculation of Net Cash Flow from Operation Rs. Add:

Add:

Net Profit for the year Non-current & Non-operating Items Charged to Profit & Loss A/c: Loss on Sale of Machinery Depreciation on Machinery Fund from operation Increase in Operating Current Liabilities: Creditors Decrease in Operating Current Assets: Stock

Rs. 60,000

2,000 45,000 47,000 1,07,000 15,000 5,000 20,000 1,27,000

Less:

Decrease in Operating Current Liabilities: Bills Payable Increase in Operating Current Assets: Debtors Net Cash Flow from Operation

Modified Date: Mon, Jul 05, 2010 05:30:16 PM

Output Date: Tue, Jul 06, 2010 11:46:14 AM

10,000 10,000 20,000 1,07,000

Rev II

Project: Management Accounting_Debarshi Bhattacharyya ACE Pro India Pvt. Ltd. File: X:\Pearson\Management Accounting_Debarshi Bhattacharyya\MAIN\M04\LAYOUT_M04\M04_DEBA_ISBN_EN_SE_C04_I.indd

CASH FLOW ANALYSIS

231

Problem 19 Prepare a Cash Flow Statement of Kawali Ltd for the year that ended on 31 December 2007 from the following particulars: Balance Sheets as on ______________ Liabilities Equity Share Capital 10% Preference Share Capital Profit & Loss A/c 12% Debenture Creditors Proposed Dividend Provision for Taxation

31 December 2006 Rs. 1,00,000 1,00,000 2,00,000 1,00,000 30,000 20,000 1,10,000

31 December 2007 Rs. 2,00,000 1,20,000 3,00,000 80,000 20,000 30,000 50,000

6,60,000

8,00,000

Assets Goodwill Building Machinery Stock Debtors Bank Preliminary expenses

31 December 2006 Rs. 1,00,000 2,00,000 1,00,000 80,000 1,20,000 50,000

31 December 2007 Rs. 80,000 3,20,000 1,52,000 70,000 1,50,000 20,000

10,000 6,60,000

8,000 8,00,000

Additional Information: i. A building having a book value of Rs. 20,000 was sold for Rs. 30,000. Depreciation on the building provided for 2007 was Rs. 40,000. ii. A machinery having a book value of Rs. 35,000 was sold for Rs. 30,000. Depreciation provided on machinery for the year 2007 amounted to Rs. 25,000. iii. Tax paid during the year 2007 was Rs. 90,000. iv. 12% debentures were redeemed at 10% premium. Solution Cash Flow Statement of Kawali Ltd for the year that ended on 31 December 2007 Sources of Cash Opening Bank balance Add: Inflow of Cash: Proceeds Received from Issue of Equity Shares Proceeds Received from Issue of Preference Shares Sale proceeds of Building Sale proceeds of Machinery Net Cash Flow from Operation

Amount Rs. 50,000

1,00,000 20,000 30,000 30,000 2,14,000 4,44,000

Applications of Cash Outflow of Cash: Redemption of debentures Payment of Dividend Payment of Tax Purchase of Building Purchase of Machinery Closing Bank Balance

Amount Rs. 22,000 20,000 90,000 1,80,000 1,12,000 20,000 4,44,000

Working Notes 1. Analysis of Non-current Assets and Liabilities (against which there is no adjustment) Assets/Liabilities Equity Share Capital Preference Share Capital Goodwill Preliminary Expenses

Closing Balance Rs. 2,00,000(Cr.) 1,20,000(Cr.) 80,000(Dr.) 8,000(Dr.)

Opening Balance Rs. 1,00,000(Cr.) 1,00,000(Cr.) 1,00,000(Dr.) 10,000(Dr.)

Increase/ Decrease Rs. (+) 1,00,000 (+) 20,000 (−) 20,000 (−) 2,000

Analysis Fresh issue Fresh issue Written off against Profit & Loss A/c Written off against Profit & Loss A/c (Continued)

Modified Date: Mon, Jul 05, 2010 05:30:16 PM

Output Date: Tue, Jul 06, 2010 11:46:14 AM

Rev II

Project: Management Accounting_Debarshi Bhattacharyya ACE Pro India Pvt. Ltd. File: X:\Pearson\Management Accounting_Debarshi Bhattacharyya\MAIN\M04\LAYOUT_M04\M04_DEBA_ISBN_EN_SE_C04_I.indd

232

MANAGEMENT ACCOUNTING

2. Dr.

Building Account Rs. 2,00,000 By Bank – Sale proceeds 10,000 By Profit & Loss A/c – Depreciation

To Balance b/f To Profit & Loss A/c – Profit on Sale (30,000 – 20,000) To Bank – Purchase (Bal. fig.)

1,80,000 3,90,000

By Balance c/f

Cr. Rs. 30,000 40,000 3,20,000 3,90,000

3. Dr.

Machinery Account Rs. 1,00,000 By Bank – Sale proceeds 1,12,000 By Profit & Loss A/c – Loss on sale (35,000 – 30,000) By Profit & Loss A/c – Depreciation By Balance c/f 2,12,000

To Balance b/f To Bank – purchase (Bal. fig.)

Cr. Rs. 30,000 5,000 25,000 1,52,000 2,12,000

4. Dr. To Bank – Redeemed (20,000 + 10%)

To Balance c/f

12% Debenture Account Rs. 22,000 By Balance b/f By Profit & Loss A/c – Premium Paid on redemption adjusted 80,000 (10% on Rs. 20,000) 1,02,000

Cr. Rs. 1,00,000 2,000 1,02,000

5. Dr. To Bank – Dividend Proposed in 2006 & Paid in 2007

Proposed Dividend Account Rs. 20,000 By Balance b/f By Profit & Loss A/c – Dividend proposed for 2007

By Balance c/f

30,000 50,000

Cr. Rs. 20,000 30,000

50,000

6. Dr. To Bank – Tax Paid To Balance c/f

Provision for Taxation Account Rs. 90,000 By Balance b/f 50,000 By Profit & Loss A/c – Provision made in 2007 (Bal. fig.) 1,40,000

Cr. Rs. 1,10,000 30,000 1,40,000

7. Dr. To Depreciation on Building To Depreciation on Machinery To Loss on Sale of Machinery

Profit & Loss A/c Rs. 40,000 By Balance b/f 25,000 By Profit on Sale of building 5,000 By Fund from Operation (Bal. fig.)

Cr. Rs. 2,00,000 10,000 2,44,000 (Continued)

Modified Date: Mon, Jul 05, 2010 05:30:16 PM

Output Date: Tue, Jul 06, 2010 11:46:14 AM

Rev II

Project: Management Accounting_Debarshi Bhattacharyya ACE Pro India Pvt. Ltd. File: X:\Pearson\Management Accounting_Debarshi Bhattacharyya\MAIN\M04\LAYOUT_M04\M04_DEBA_ISBN_EN_SE_C04_I.indd

CASH FLOW ANALYSIS

Dr. To Premium Paid on Redemption of Debentures To Proposed Dividend To Provision for Taxation To Goodwill written off To Preliminary Expenses written off To Balance c/f

233

Cr.

Profit & Loss A/c Rs. 2,000

Rs.

30,000 30,000 20,000 2,000 3,00,000 4,54,000

4,54,000

8.

Add:

Less:

Calculation of Net Cash Flow from Operation Fund from Operation Increase in Operating Current Liabilities Decrease in Operating Current Assets: Stock Increase in Operating Current Assets: Debtors Decrease in Operating Current Liabilities: Creditors Net Cash Flow from Operation

Rs.

Rs. 2,44,000

Nil 10,000

10,000 2,54,000

30,000 10,000

40,000 2,14,000

Stop and Think i. Here, workings for Non-current Assets and Liabilities are done in two forms. Non-current Assets and liabilities against which there is no adjustment are collectively shown under a single working in a statement form (Working Note 1). For Non-current Assets and Liabilities against which there are adjustments, separate accounts are opened for each of them. Students are advised to follow this type of working for the examination purpose to save time. ii. A premium paid on the redemption of debenture was definitely written off ; otherwise, it would have appeared in the asset side of the Balance Sheet as on 31 December 2007. As there was no securities premium, it was written off against Profit & Loss A/c. Under Working Note – 4, the adjustment of the premium paid on redemption against Profit & Loss A/c is shown in debenture account for avoiding the hazard of opening another account. A separate account for premium on redemption of debentures could have been opened alternatively. Preparation of Cash Flow Statement as per AS-3

Tutorial Note There are two methods of preparation of Cash Flow Statement as per AS-3, namely, Direct Method and Indirect Method. If, in the problem, the details of Profit & Loss A/c for the relevant year along with the Balance Sheets of the relevant year and its proceeding year are given, then the Cash Flow Statement may be prepared following either of the two methods (i.e., Direct Method or Indirect Method) as recommended in AS-3. But if, in the problem, only the Balance Sheets of two consecutive years along with some additional information are given (i.e., where the details of the Profit & Loss A/c are not given in the problem), then the Cash Flow Statement is to be prepared following the Indirect Method only as recommended in AS-3.

Modified Date: Mon, Jul 05, 2010 05:30:16 PM

Output Date: Tue, Jul 06, 2010 11:46:14 AM

Rev II

Project: Management Accounting_Debarshi Bhattacharyya ACE Pro India Pvt. Ltd. File: X:\Pearson\Management Accounting_Debarshi Bhattacharyya\MAIN\M04\LAYOUT_M04\M04_DEBA_ISBN_EN_SE_C04_I.indd

234

MANAGEMENT ACCOUNTING

Problem 20 From the following information, prepare a Cash Flow Statement as on 31 December 2007 applying the method given in AS-3: Balance Sheets as on ______________ Liabilities Share Capital Profit & Loss A/c Loan from IDBI Creditors Proposed Dividend

31 December 2006 Rs. 40,000 13,000

31 December 2007 Rs. 45,000 25,000

20,000 15,000 2,000

10,000 21,000 5,000

90,000

1,06,000

Assets

Less:

Fixed Assets at cost Provision for Depreciation

31 December 2006 Rs. 45,000 10,000 35,000 20,000 30,000 5,000 90,000

Inventory Debtors Cash

31 December 2007 Rs. 50,000 15,000 35,000 30,000 35,000 6,000 1,06,000

There was no disposal of Fixed Assets. Tax paid on 30 June 2007 amounted to Rs. 3,000.

Guidance Note As the details of the Profit & Loss A/c for the year that ended on 31 December 2007 is not given in the problem, the Cash Flow Statement for the same is to be prepared, following the Indirect Method, as recommended in AS-3.

Solution Cash Flow Statement (under Indirect Method) of ______________ for the year that ended on 31 December 2007 Rs. A. Cash Flows from Operating Activities: Net Profit for the year (Rs. 25,000 – Rs. 13,000) Add: Adjustment for Non-operating & Non-current items debited to Profit & Loss A/c: Proposed Dividend Depreciation for the Year Tax Less:

Add:

Less:

Adjustment for Non-operating & Non-current items credited to Profit & Loss A/c Operating profit before Working Capital changes (i.e., Fund from operation) Increase in Operating Current Liabilities: Creditors Decrease in Operating Current Assets Decrease in Operating Current Liabilities Increase in Operating Current Assets: Inventory Debtors Cash generated from operation

Rs.

Rs.

12,000

5,000 5,000 3,000

13,000 25,000 Nil 25,000

6,000 Nil

6,000 31,000

Nil 10,000 5,000

15,000 16,000 (Continued)

Modified Date: Mon, Jul 05, 2010 05:30:16 PM

Output Date: Tue, Jul 06, 2010 11:46:14 AM

Rev II

Project: Management Accounting_Debarshi Bhattacharyya ACE Pro India Pvt. Ltd. File: X:\Pearson\Management Accounting_Debarshi Bhattacharyya\MAIN\M04\LAYOUT_M04\M04_DEBA_ISBN_EN_SE_C04_I.indd

235

CASH FLOW ANALYSIS

Rs. Less:

Income Tax Paid Net Cash Flow from Operating Activities B. Cash Flows from Investing Activities: Proceeds from Sale of Fixed Assets & Investments Less: Purchase of Fixed Assets & Investments Net Cash Flow from Investing Activities C. Cash Flows from Financing Activities Proceeds Received from Issue of Shares Less: Repayment of Loan from IDBI Payment of Dividend Net Cash Flow from Financing Activities Net Increase in Cash & Cash Equivalents Add: Cash & Cash Equivalents at the Begining of the year Cash & Cash Equivalents at the end of the year

Rs. 3,000

Rs. 13,000

Nil 5,000 (5,000) 5,000 10,000 2,000

12,000 (7,000) 1,000 5,000 6,000

Working Notes 1. Analysis of Non-current Assets and Liabilities (against which there is no adjustment) Opening Balance Rs. 40,000 13,000 20,000 45,000 10,000

Assets/Liabilities Share Capital Profit & Loss A/c Loan from IDBI Fixed Assets (at cost) Provision for Depreciation on Fixed Assets

Closing Balance Rs. 45,000 25,000 10,000 50,000 15,000

Increase/ Decrease Rs. (+) 5,000 (+) 12,000 (−) 10,000 (+) 5,000 (+) 5,000

Analysis Fresh Issue Net Profit for the year Repayment of Loan Purchase of Fixed Assets Depreciation provided for the year

2. Dr. To Bank – Dividend proposed in 2006 & Paid in 2007 To Balance c/f

Cr.

Proposed Dividend Account Rs. 2,000 By Balance b/f

Rs. 2,000

5,000

5,000

By Profit & Loss A/c – Dividend proposed for 2007

7,000

7,000

3. Dr.

Cr.

(Income) Tax Account Rs. 3,000 3,000

To Bank – Tax Paid in 2007

By Profit & Loss A/c – Tax Paid in 2007 charged

Rs. 3,000 3,000

Problem 21 Balance Sheet of R Ltd (Rs. in ’000) Liabilities Share Capital Reserve Profit & Loss A/c

As on 31 March 2007 1,000 200 100

As on 31 March 2006 800 150 60

Assets Machinery Building Investment

As on 31 March 2007 700 600 100

As on 31 March 2006 500 400 – (Continued)

Modified Date: Mon, Jul 05, 2010 05:30:16 PM

Output Date: Tue, Jul 06, 2010 11:46:14 AM

Rev II

Project: Management Accounting_Debarshi Bhattacharyya ACE Pro India Pvt. Ltd. File: X:\Pearson\Management Accounting_Debarshi Bhattacharyya\MAIN\M04\LAYOUT_M04\M04_DEBA_ISBN_EN_SE_C04_I.indd

236

MANAGEMENT ACCOUNTING

Liabilities Debentures Tax Provision Proposed Dividend Sundry Creditors

As on 31 March 2007 200 100 200 700 2,500

As on 31 March 2006 – 70 100 820 2,000

As on 31 March 2007 500 400 200

Assets Debtors Stock Cash & Bank

2,500

As on 31 March 2006 700 200 200 2,000

Additional Details: i. Building is still under construction and no depreciation was charged. ii. Depreciation was charged @ 25% on the opening value of machinery. iii. An old machine costing Rs. 50,000 was sold for Rs. 35,000 (Written Down Value [WDV]—Rs. 20,000). iv. Income tax paid during the year—Rs. 50,000. Prepare a Cash Flow Statement as per AS-3 and interpret it. [B.Com. (Hons), Calcutta University—2008] Solution Cash Flow Statement (under Indirect Method) of R Ltd for the year that ended on 31 March 2007 Rs. A. Cash Flows from Operating Activities: Net Profit (after appropriation) for the Year4 Add: Non-operating & Non-current items debited to Profit & Loss A/c: Transfer to Reserve4 Proposed Dividend3 Depreciation on Machinery1 Tax provision2 Less:

Add:

Non-operating & Non-current items Credited to Profit & Loss A/c: Profit on Sale of Machinery1 Operating profit before Working Capital changes (i.e., fund from operation) Increase in Operating Current Liabilities Decrease in Operating Current Assets: Debtors

Decrease in Operating Current Liabilities: Sundry Creditors Increase in Operating Current Assets: Stock Cash generated from operation Less: Income Tax Paid Net Cash Flow from Operating Activities B. Cash Flows from Investing Activities: Sale proceeds of Machinery Less: Purchase of Machinery1 Further construction of building4 Investment made4 Net Cash Flow from Investing Activities C. Cash Flows from Financing Activities: Proceeds Received from New Issue of Shares4 Proceeds Received from New Issue of debentures4

Rs.

Rs.

40 50 200 125 80

455 495 15 480

Nil 200

200 680

Less:

120 200

320 360 50 310 35

345 200 100

645 (610) 200 200 400 (Continued)

Modified Date: Mon, Jul 05, 2010 05:30:16 PM

Output Date: Tue, Jul 06, 2010 11:46:14 AM

Rev II

Project: Management Accounting_Debarshi Bhattacharyya ACE Pro India Pvt. Ltd. File: X:\Pearson\Management Accounting_Debarshi Bhattacharyya\MAIN\M04\LAYOUT_M04\M04_DEBA_ISBN_EN_SE_C04_I.indd

237

CASH FLOW ANALYSIS

Rs. Less:

Add:

Payment of proposed Dividend3 Net Cash Flow from Financing Activities Net increase in Cash & Cash equivalents Cash & Cash Equivalents at the begining of the Year Cash & Cash Equivalents at the end of the Year

Rs. 100

Rs. 300 Nil 200 200

Note: Here, investment is considered as a Long-term Investment.

Interpretation of above Cash Flow Statement: From the above Cash Flow Statement, it has been observed that there is no net increase in cash and cash equivalents during the year 2006–07. In spite of the Net Cash Inflows from the Operating Activities of Rs. 3,10,000 and Net Cash Inflows from the Financing Activities of Rs. 3,00,000 during the year, a huge amount of net cash outflows from the Investing Activities (of Rs. 6,10,000), due to a huge investment in the Fixed Assets counter balancing the net increase in cash and cash equivalents during the year. Therefore, in spite of a substantial net Cash Inflows from the Operating Activities, there is no net increase in the cash and cash equivalents during the year due to a huge investment in the Fixed Assets. Working Notes 1. Dr. To Balance b/f To Profit & Loss A/c – Profit on Sale of old machine (35 – 20) To Bank – Purchase (Bal. fig.)

Machinery Account Rs. in ’000 500 By Bank – Sale proceeds of old machine 15 By Profit & Loss A/c – Depreciation (25% of 125) 345 By Balance c/f 860

Cr. Rs. in ’000 35 125

Tax Provision Account Rs. in ’000 50 By Balance b/f 100 By Profit & Loss A/c – Tax Provided for the year 150

Cr. Rs. in ’000 70 80 150

Proposed Dividend Account Rs. in ’000 100 By Balance b/f

Cr. Rs. in ’000 100

700 860

2. Dr. To Bank – Tax Paid during the Year To Balance c/f

3. Dr. To Bank – Dividend paid during the year To Balance c/f

200

By Profit & Loss A/c – Dividend proposed during the year

300

200 300

4. Analysis of other Non-current Assets and Liabilities Assets/Liabilities Share Capital Profit & Loss A/c Reserve

Opening Balance Rs. in ’000 800 60 150

Closing Balance Rs. in ’000 1,000 100 200

Increase/ Decrease Rs. in ’000 (+) 200 (+) 40 (+) 50

Analysis Fresh issue Net Profit for the year Transfer during the year (Continued)

Modified Date: Mon, Jul 05, 2010 05:30:16 PM

Output Date: Tue, Jul 06, 2010 11:46:14 AM

Rev II

Project: Management Accounting_Debarshi Bhattacharyya ACE Pro India Pvt. Ltd. File: X:\Pearson\Management Accounting_Debarshi Bhattacharyya\MAIN\M04\LAYOUT_M04\M04_DEBA_ISBN_EN_SE_C04_I.indd

238

MANAGEMENT ACCOUNTING

Opening Balance Rs. in ’000 – 500 –

Assets/Liabilities Debentures Building Investment

Closing Balance Rs. in ’000 200 700 100

Increase/ Decrease Rs. in ’000 (+) 200 (+) 200 (+) 100

Analysis New issue Further construction Further investment

Problem 22 Redraft the given Traditional Cash Flow of 31 December 2008 as per AS-3 provisions: Traditional Cash Flow as on 31 December 2008 Sources Opening Cash Balance Cash from operation: Cash from Sales Less: Cash Paid: For purchase For Expenses Dividend Received

Rs. in Lakh

Rs. in Lakh 225

Uses Purchase of Fixed Assets2 Closing Cash Balance

Rs. in Lakh 100 194

909 (588) (255)

66 3 294

294

[B.Com. (Hons), Calcutta University— 2009] Solution Cash Flow Statement (under Direct Method) of ______________ for the year that ended on 31 December 2008 Rs. in Lakh A. Cash Flows from Operating Activities: Cash Receipts from Sales Less: Cash Paid for Purchases Cash Paid for Expenses Cash generated from operation Less: Income Tax Paid5 Net Cash Flow from Operating Activities B. Cash Flows from Investing Activities: Dividend Received (from investment) Less: Purchase of Fixed Assets Net Cash Flow from Investing Activities C. Cash Flows from Financing Activities: Net increase in Cash & Cash Equivalents Add: Cash & Cash Equivalents at the beginning of the year Cash & Cash Equivalents at the end of the year

Modified Date: Mon, Jul 05, 2010 05:30:16 PM

Output Date: Tue, Jul 06, 2010 11:46:14 AM

Rs. in Lakh

Rs. in Lakh

909 588 255 842 66 Nil 66 3 (100) (97) Nil (31) 225 194

Rev II

Project: Management Accounting_Debarshi Bhattacharyya ACE Pro India Pvt. Ltd. File: X:\Pearson\Management Accounting_Debarshi Bhattacharyya\MAIN\M04\LAYOUT_M04\M04_DEBA_ISBN_EN_SE_C04_I.indd

CASH FLOW ANALYSIS

239

Problem 23 Following is the Cash Flow Abstract of Alpha Ltd for the year that ended on 31 March 2008: Cash Flow Abstract Inflows Opening Balance: Cash Bank Share Capital – Shares Issued Collection from Debtors Sale of Fixed Assets

Rs. 10,000 70,000 5,00,000 3,50,000 70,000

Outflows Payment to Creditors Salaries & Wages Payment of Overheads Fixed Assets Acquired Debentures Redeemed Bank Loan Repaid Taxation Dividends Closing Balance: Cash Bank

Rs. 90,000 25,000 15,000 4,00,000 50,000 2,50,000 55,000 1,00,000 5,000 10,000 10,00,000

10,00,000

Prepare a Cash Flow Statement for the year that ended on 31 March 2008 in accordance with AS-3. [C.A. (PE II)—November 2008]

Solution Cash Flow Statement (Direct Method) of Alpha Ltd for the year that ended on 31 March 2008 Rs. A. Cash Flows from Operating Activities: Cash Receipts from Customers Less: Cash Paid to Suppliers Cash Paid to Employees (Salaries & Wages) Other Cash Payments (Overheads) Cash Generated from Operation Less: Income Tax Paid Net Cash Flow from Operating Activities B. Cash Flows from Investing Activities: Proceeds from Sale of Fixed Assets Less: Payment for Purchase of Fixed Assets Net Cash Flow from Investing Activities C. Cash Flows from Financing Activities: Proceeds Received from New Issue of Shares Less: Payment of Dividend Payment for Redemption of Debentures Repayment of Bank Loan Net Cash Flow from Financing Activities Net increase in Cash & Cash equivalents Add: Cash & Cash Equivalents at the beginning of the year Cash & Cash Equivalents at the end of the year

Modified Date: Mon, Jul 05, 2010 05:30:16 PM

Output Date: Tue, Jul 06, 2010 11:46:14 AM

Rs.

Rs.

3,50,000 90,000 25,000 15,000

1,30,000 2,20,000 55,000 1,65,000 70,000 4,00,000 (3,30,000) 5,00,000

1,00,000 50,000 2,50,000

4,00,000 1,00,000 (65,000) 80,000 15,000

Rev II

Project: Management Accounting_Debarshi Bhattacharyya ACE Pro India Pvt. Ltd. File: X:\Pearson\Management Accounting_Debarshi Bhattacharyya\MAIN\M04\LAYOUT_M04\M04_DEBA_ISBN_EN_SE_C04_I.indd

240

MANAGEMENT ACCOUNTING

Problem 24 ABC Ltd gives you the following information. You are required to prepare a Cash Flow Statement by using Indirect Method as per AS-3 for the year that ended on 31 March 2004: Balance Sheets as on Liabilities Capital Retained Earnings Debentures Current Liabilities: Creditors Bank Loan Liability for Expenses Dividend Payable

31 March 2003 Rs. 50,00,000 26,50,000 –

31 March 2004 Rs. 50,00,000 36,90,000 9,00,000

8,80,000 1,50,000

8,20,000 3,00,000

3,30,000 1,50,000

2,70,000 3,00,000

91,60,000

1,12,80,000

Assets

Less:

Plant & Machinery Depreciation

Less:

Current Assets: Debtors Provision

Cash Marketable Securities Inventories Prepaid Expenses

31 March 2003 Rs. 27,30,000 6,10,000 21,20,000

31 March 2004 Rs. 40,70,000 7,90,000 32,80,000

23,90,000 1,50,000 22,40,000

28,30,000 1,90,000 26,40,000

15,20,000 11,80,000 20,10,000 90,000 91,60,000

18,20,000 15,00,000 19,20,000 1,20,000 1,12,80,000

Additional Information: i. Net Profit for the year that ended on 31 March 2004, after charging depreciation, is Rs. 22,40,000. ii. Debtors worth Rs. 2,30,000 were determined to be worthless and were written off against provision for Doubtful Debt Account during the year. iii. ABC Ltd declared a dividend of Rs. 12,00,000 for the year 2003–04. [C.A. (Inter)—May 2004] Solution Cash Flow Statement (under Indirect Method) of ABC Ltd for the year that ended on 31 March 2007 Rs. A. Cash Flows from Operating Activities: Net Profit for the Year after charging depreciation Add: Adjustment for Non-operating & Non-current items debited to Profit & Loss A/c: Provision for doubtful debt less bad debt adjusted Depreciation on plant & Machinery (Rs. 2,70,000 – Rs. 2,30,000)

Add:

Less:

Operating profit before Working Capital changes Increase in Operating Current Liabilities: Bank Loan Decrease in Operating Current Assets: Inventories Decrease in Operating Current Liabilities: Creditors Liability for Expenses

Rs.

Rs.

22,40,000

40,000 1,80,000 2,20,000 24,60,000 1,50,000 90,000

2,40,000 27,00,000

60,000 60,000 (Continued)

Modified Date: Mon, Jul 05, 2010 05:30:16 PM

Output Date: Tue, Jul 06, 2010 11:46:14 AM

Rev II

Project: Management Accounting_Debarshi Bhattacharyya ACE Pro India Pvt. Ltd. File: X:\Pearson\Management Accounting_Debarshi Bhattacharyya\MAIN\M04\LAYOUT_M04\M04_DEBA_ISBN_EN_SE_C04_I.indd

241

CASH FLOW ANALYSIS

Increase in Operating Current Assets: Prepaid Expenses Debtors Net Cash Flow from Operating Activities B. Cash Flows from Investing Activities: Proceeds from Sale of Fixed Assets & investment Less: Purchase of plant & Machinery3 Net Cash Flow from Investing Activities C. Cash Flows from Financing Activities Proceeds Received from Issue of debentures3 Less: Payment of Dividend1 Net Cash Flow from Financing Activities Net increase in Cash & Cash equivalents Add: Cash & Cash Equivalents at the beginning of the Year (Rs. 15,20,000 + Rs. 11,80,000) Cash & Cash Equivalents at the end of the Year (Rs. 18,20,000 + Rs. 15,00,000)

Rs.

Rs.

Rs.

30,000 4,40,000

5,90,000 21,10,000 Nil 13,40,000 (13,40,000) 9,00,000 10,50,000 (1,50,000) 6,20,000 27,00,000 33,20,000

Working Notes 1. Dr. To Bank – Dividend Paid in 2003–04 To Balance c/f

Dividend Payable Account Rs. 10,50,000 By Balance b/f By Profit & Loss appropriation A/c 3,00,000 – Dividend declared for 2003–04 13,50,000

Cr. Rs. 1,50,000 12,00,000

Retained Earning Account Rs. 12,00,000 By Balance b/f

Cr. Rs. 26,50,000

13,50,000

2. Dr. To Dividend Payable – Dividend declared for 2003–04 To Balance c/f

3.

36,90,000 48,90,000

By Profit & Loss A/c – Net Profit for 2003–04

22,40,000 48,90,000

Analysis of other Non-current Assets and Liabilities Assets/Liabilities

Plant & Machinery (at cost) Provision for Depreciation on Plant & Machinery Debentures

Opening Balance Rs. 27,30,000 6,10,000

Closing Balance Rs. 40,70,000 7,90,000

Increase/ Decrease Rs. (+) 13,40,000 (+) 1,80,000

Nil

9,00,000

(+) 9,00,000

Analysis New Purchase Depreciation for 2003–04 New Issue

4. Provision for Doubtful Debt Account Rs. To Bad Debt – Bad Debt for 2003–04 adjusted 2,30,000 By Balance b/f By Profit & Loss A/c To Balance c/f 1,90,000 – Dividend Proposed for 2007 4,20,000

Cr.

Dr.

Modified Date: Mon, Jul 05, 2010 05:30:16 PM

Output Date: Tue, Jul 06, 2010 11:46:14 AM

Rs. 1,50,000 2,70,000 4,20,000

Rev II

Project: Management Accounting_Debarshi Bhattacharyya ACE Pro India Pvt. Ltd. File: X:\Pearson\Management Accounting_Debarshi Bhattacharyya\MAIN\M04\LAYOUT_M04\M04_DEBA_ISBN_EN_SE_C04_I.indd

242

MANAGEMENT ACCOUNTING

Problem 25 Usha Ltd had the following condensed trial balance as on 31 March 2002: Debit Cash Accounts Receivable Investments Plant Assets Land

Rs. 7,500 30,000 20,000 67,500 40,000 1,65,000

Credit Current Liabilities Long-term Notes Payable Bonds Payable Capital Stock Retained Earnings

Rs. 15,000 25,500 25,000 75,000 24,500 1,65,000

Additional Information: During 2002–03, the following transactions took place: i. A tract of land was purchased for a cash of Rs. 7,750. ii. Bonds payable in the amount of Rs. 6,000 were retired for cash at face value. iii. An additional Rs. 20,000 equity shares were issued at par for cash. iv. Dividends totalling Rs. 9,375 were paid. v. Net income for 2002–03 was Rs. 28,450 after allowing for a depreciation of Rs. 9,500. vi. Land was purchased through the issuance of Rs. 22,500 in bonds. vii. Usha Ltd sold a part of its investments portfolio for Rs. 12,875 cash. The transaction resulted in a gain of Rs. 1,375 for the firm. viii. Current Liabilities increased to Rs. 18,000 on 31 March 2003. ix. Accounts receivable on 31 March 2003 totalled to Rs. 38,000. Prepare a Statement of Cash Flows for 2002–03, using indirect method, as per AS-3 (Revised). [M.Com., Delhi University—2003] Solution Cash Flow Statement (under Indirect Method) of Usha Ltd for the year that ended on 31 March 2003 Rs. A. Cash Flows from Operating Activities: Net Profit for the year after charging depreciation Add: Adjustment for Non-operating & Non-current Items debited to Profit & Loss A/c: Depreciation Less:

Add:

Adjustment for Non-operating & Non-current Items credited to Profit & Loss A/c: Gain on Sale of Investment Portfolio Operating Profit before Working Capital changes Increase in Operating Current Liabilities: Current Liabilities

Increase in Operating Current Assets: Accounts Receivable Net Cash Flow from Operating Activities B. Cash Flows from Investing Activities: Proceeds from Sale of Investment Portfolio

Rs.

Rs.

28,450

9,500 37,950

1,375 36,575 3,000 39,575

Less:

8,000 31,575 12,875 (Continued)

Modified Date: Mon, Jul 05, 2010 05:30:16 PM

Output Date: Tue, Jul 06, 2010 11:46:14 AM

Rev II

Project: Management Accounting_Debarshi Bhattacharyya ACE Pro India Pvt. Ltd. File: X:\Pearson\Management Accounting_Debarshi Bhattacharyya\MAIN\M04\LAYOUT_M04\M04_DEBA_ISBN_EN_SE_C04_I.indd

CASH FLOW ANALYSIS

Rs. Less:

Purchase of land for cash Net Cash Flow from Investing Activities C. Cash Flows from Financing Activities Proceeds Received from Issue of Equity Shares Less: Payment of Dividend Redemption of Bond Payable Net Cash Flow from Financing Activities Net increase in Cash & Cash equivalents Add: Cash & Cash Equivalents at the beginning of the year Cash & Cash Equivalents at the end of the year

Rs. 7,750

243

Rs. 5,125

20,000 9,375 6,000

15,375 4,625 41,325 7,500 48,825

Problem 26 Following are the summarized Balance Sheets of Bibhu–Ribhu Enterprise Ltd, as on 31 March 2002 and 31 March 2003: Liabilities Share Capital General Reserve Profit & Loss A/c Long-term Loan Sundry Creditors Provision for Taxation

As on 31 March 2002 Rs. 2,00,000 50,000 30,500 70,000 1,50,000 30,000 5,30,500

As on 31 March 2003 Rs. 2,50,000 60,000 30,600 – 1,35,200 35,000 5,10,800

Assets Goodwill Land & Building Machinery Inventory Sundry Debtors Bank

As on 31 March 2002 Rs. – 2,00,000 1,50,000 1,00,000 80,000 – 5,30,500

As on 31 March 2003 Rs. 5,000 1,90,000 1,69,000 74,000 64,200 8,000 5,10,800

Additional Information: During the year that ended on 31 March 2003, the following transactions took place: i. Dividends of Rs. 23,000 were paid. ii. An inventory of Rs. 20,000 and machinery of Rs. 25,000 of another company were purchased for a consideration of Rs. 45,000 payable in shares. iii. Machinery was further purchased for cash at Rs. 8,000. iv. Depreciation written off on machinery was Rs. 12,000. v. Income Tax provided during the year was Rs. 33,000. vi. Sale of old machinery of Rs. 2,000 at a loss of Rs. 300, which was written off to general reserve. Prepare a Cash Flow Statement for the year that ended on 31 March 2003 using modern method, as per the revised guidelines of AS-3. [B.Com., Guwahati University—2004] Solution Cash Flow Statement (under Indirect Method) of Bibhu–Ribhu Enterprise Ltd for the year that ended on 31 March 2003 Rs. A. Cash Flows from Operating Activities: Net Profit (after appropriation) for the year5 Add: Adjustment for Non-operating & Non-current Items debited to Profit & Loss A/c: Dividend Paid

Rs.

Rs.

100

23,000 (Continued)

Modified Date: Mon, Jul 05, 2010 05:30:16 PM

Output Date: Tue, Jul 06, 2010 11:46:14 AM

Rev II

Project: Management Accounting_Debarshi Bhattacharyya ACE Pro India Pvt. Ltd. File: X:\Pearson\Management Accounting_Debarshi Bhattacharyya\MAIN\M04\LAYOUT_M04\M04_DEBA_ISBN_EN_SE_C04_I.indd

244

MANAGEMENT ACCOUNTING

Add:

Transfer to General Reserve3 Depreciation on Machinery Income Tax Provided Depreciation on Land & Building5 Operating profit before Working Capital changes Decrease in Operating Current Assets: Sundry Debtors Inventory [1,00,000 – (74,000 – 20,000)]

Rs. 10,300 12,000 33,000 10,000

Rs.

Rs.

88,300 88,400

15,800 46,000 61,800 1,50,200

Less:

Decrease in Operating Current Liabilities: Sundry creditors Cash Generated from Operation Less: Income tax Paid4 Net Cash Flow from Operating Activities B. Cash Flows from Investing Activities: Proceeds Received from Sale of Machinery2 Less: Purchase of new Machinery for cash Net Cash Flow from Investing Activities C. Cash Flows from Financing Activities: Repayment of long-term Loan5 Payment of Dividend Net Cash Flow from Financing Activities Net increase in Cash & Cash Equivalents Add: Cash & Cash Equivalents at the beginning of the year Cash & Cash Equivalents at the end of the year

14,800 1,35,400 28,000 1,07,400 1,700 8,000 (6,300) (70,000) (23,000) (93,000) 8,100 500 8,600

Working Notes 1. Dr. To Equity Share Capital – Consideration Paid

Cr.

Vendor Company Account Rs. 50,000

By Machinery By Inventory By Goodwill (Bal. fig.)

50,000

Rs. 25,000 20,000 5,000 50,000

2. Dr. To Balance b/f To Vendor Company’s A/c – Acquired by Shares To Bank

Cr.

Machinery Account Rs. 1,50,000 25,000 8,000

Purchased for Cash

By Profit & Loss A/c – Depreciation By Bank – Sale Proceeds of old Machinery (2,000 – 300) By General Reserve – Loss on Sale of Machinery By Balance c/f

1,83,000

Rs. 12,000 1,700 300 1,69,000 1,83,000 (Continued)

Modified Date: Mon, Jul 05, 2010 05:30:16 PM

Output Date: Tue, Jul 06, 2010 11:46:14 AM

Rev II

Project: Management Accounting_Debarshi Bhattacharyya ACE Pro India Pvt. Ltd. File: X:\Pearson\Management Accounting_Debarshi Bhattacharyya\MAIN\M04\LAYOUT_M04\M04_DEBA_ISBN_EN_SE_C04_I.indd

245

CASH FLOW ANALYSIS

3. Dr.

Cr.

General Reserve Account Rs. 300

To Machinery A/c – Loss on Sale Adjusted To Balance c/f

Rs. 50,000

By Balance b/f

60,000 60,300

By Profit & Loss A/c – Transfer during the year

10,300 60,300

4. Dr.

Cr.

Provision for Tax Account Rs. 28,000 35,000 63,000

To Bank – Tax Paid (Bal. fig.) To Balance c/f

5.

By Balance b/f By Profit & Loss A/c – Provided during the year

Rs. 30,000 33,000 63,000

Analysis of other Non-current Assets and liabilities

Land & Building Share Capital

Opening Balance Rs. 2,00,000 2,00,000

Closing Balance Rs. 1,90,000 2,50,000

Increase/ Decrease Rs. (−) 10,000 (+)50,000

Long-term loan Profit & Loss A/c

70,000 30,500

– 30,600

(−) 70,000 (+) 100

Assets/Liabilities

Analysis Depreciation for the Year New Issue to the vendor company against assets Repayment Net Profit after appropriation for the Year

Problem 27 Following are the summarized Balance Sheets of Rabangla Ltd as on 31 March 2007 and 31 March 2008: (Rs. in Lakhs) Liabilities Share Capital General Reserve Profit & Loss A/c 7% Debentures Loan from Bank (pledged by freehold property) Provision for Taxation Proposed Dividend Sundry Creditors

31 March 2007 300.00 200.00 25.00 75.00 27.00

31 March 2008 325.00 219.00 20.00 50.00 14.25

21.00 22.50 60.00 730.50

37.50 23.25 127.50 816.50

Assets Freehold Property Machinery Long-term Investment Short-term Investment Stock Debtors Bank

31 March 2007 225.00 135.00 150.00 112.50 52.50 45.00 10.50

31 March 2008 240.00 165.00 162.50 100.00 75.00 70.00 4.00

730.50

816.50

The following additional information for the year 2007–08 are relevant for the purpose: i. Credit Sales – Rs. 675 lakhs. ii. Credit Purchases – Rs. 520 lakhs. iii. Overheads – Rs. 83.75 lakhs.

Modified Date: Mon, Jul 05, 2010 05:30:16 PM

Output Date: Tue, Jul 06, 2010 11:46:14 AM

Rev II

Project: Management Accounting_Debarshi Bhattacharyya ACE Pro India Pvt. Ltd. File: X:\Pearson\Management Accounting_Debarshi Bhattacharyya\MAIN\M04\LAYOUT_M04\M04_DEBA_ISBN_EN_SE_C04_I.indd

246

MANAGEMENT ACCOUNTING

iv. Depreciation on Machinery – Rs. 17.50 lakhs. v. Tax Liability for 2006–07 was settled at Rs. 22.50 lakhs. vi. Dividend for 2006–07 was paid in 2007–08. Prepare a Cash Flow Statement for the year that ended on 31 March 2008.

Stop and Think As, in this problem, the credit sales, credit purchases, overheads incurred and so on are given, the Cash Flow Statement can be prepared, under both the methods as prescribed in AS-3, that is, direct and indirect method. Here, for the better understanding of the difference between the two methods, the Cash Flow Statement is prepared separately under both the methods. However, students are advised to follow either of the two methods in the examination hall (preferably under Direct Method in such a case).

Solution Method 1: Direct Method Cash Flow Statement (under Direct Method) of Rabangla Ltd for the year that ended on 31 March 2008 Rs. in Lakhs A. Cash Flows from Operating Activities: Cash Receipts from Customers Less: Cash Paid to Suppliers & Employees: Cash Paid to Suppliers Overheads Paid in Cash Cash Generated from Operation Less: Income Tax Paid Net Cash Flow from Operating Activities B. Cash Flows from Investing Activities: Proceeds Received from Sale of Fixed Assets Less: Purchase of Machinery Purchase of Freehold Property Further Investment (Long-term) made

Rs. in Lakhs

Rs. in Lakhs

650.00 452.50 83.75 536.25 113.75 22.50 91.25 Nil 47.50 15.00 12.50 75.00

Net Cash Flow from Investing Activities C. Cash Flows from Financing Activities: New Issue of Shares Less: Redemption of Debentures Repayment of Loan from Bank Payment of Dividend Net Cash Flow from Financing Activities Net increase in Cash & Cash equivalents Add: Opening Cash & Cash equivalents: Bank Short-term Investments Closing Cash & Cash Equivalents (100 + 4)

Modified Date: Mon, Jul 05, 2010 05:30:16 PM

Output Date: Tue, Jul 06, 2010 11:46:14 AM

(75.00) 25.00 25.00 12.75 22.50 60.25

(35.25) (19.00)

10.50 112.50 123.00 104.00

Rev II

Project: Management Accounting_Debarshi Bhattacharyya ACE Pro India Pvt. Ltd. File: X:\Pearson\Management Accounting_Debarshi Bhattacharyya\MAIN\M04\LAYOUT_M04\M04_DEBA_ISBN_EN_SE_C04_I.indd

247

CASH FLOW ANALYSIS

Method 2: Indirect Method Cash Flow Statement (under Indirect Method) of Rabangla Ltd for the year that ended on 31 March 2008 Rs. in Lakhs A. Cash Flows from Operating Activities: Net Profit (after appropriation) for the year6 Add: Appropriations: Proposed Dividend3 Transfer to General Reserve6 Net Profit (before appropriation) for the year Add: Adjustment for Non-current & Non-operating Items Debited to Profit & Loss A/c: Depreciation on Machinery Provision for taxation4 Operating profit before Working Capital changes Add: Increase in Current Liabilities: Creditors Decrease in Current Assets: Stock Debtors Cash generated from operation Less: Income tax Paid4 Net Cash Flow from Operating Activities B. Cash Flows from Investing Activities: Same as Calculated in above under Direct Method C. Cash Flows from Financing Activities: Same as Calculated in above under Direct Method Net increase in Cash & Cash Equivalents Add: Opening Cash & Cash Equivalents Closing Cash & Cash Equivalents

Rs. in Lakhs

Rs. in Lakhs

(5.00) 23.25 19.00

17.50 39.00

42.25 37.25

56.50 93.75 67.50 161.25

Less:

22.50 25.00

47.50 113.75 22.50 91.25 (75.00) (35.25) (19) 132 104

Working Notes 1. Dr. To Balance b/f To Sales – On credit

Debtors Account Rs. in Lakhs 45 By Bank – Collection (Bal. fig.) 675 By Balance c/f 720

Cr. Rs. in Lakhs 650 70 720

Creditors Account Rs. in Lakhs 452.50 By Balance b/f 127.50 By Purchases – On credit 580.00

Cr. Rs. in Lakhs 60.00 520.00 580.00

2. Dr. To Bank – Payment (Bal. fig.) To Balance c/f

(Continued)

Modified Date: Mon, Jul 05, 2010 05:30:16 PM

Output Date: Tue, Jul 06, 2010 11:46:14 AM

Rev II

Project: Management Accounting_Debarshi Bhattacharyya ACE Pro India Pvt. Ltd. File: X:\Pearson\Management Accounting_Debarshi Bhattacharyya\MAIN\M04\LAYOUT_M04\M04_DEBA_ISBN_EN_SE_C04_I.indd

248

MANAGEMENT ACCOUNTING

3. Dr. To Bank – Dividend for 2006–07 paid To Balance c/f

Proposed Dividend Account Rs. in Lakhs 22.50 By Balance b/f 23.25 By Profit & Loss appropriation A/c – Dividend proposed for 2007–08 45.75

Cr. Rs. in Lakhs 22.50

Provision for Taxation Account Rs. in Lakhs 22.50 By Balance b/f 37.50 By Profit & Loss A/c – Provision made in 2007–08 60.00

Cr. Rs. in Lakhs 21.00

Machinery Account Rs. in Lakhs 135.00 By Profit & Loss A/c – Depreciation for 2007–08 By Balance c/f 47.50 182.50

Cr. Rs. in Lakhs 17.50

23.25 45.75

4. Dr. To Bank – Tax liability for 2006–07 Paid To Balance c/f

39.00 60.00

5. Dr. To Balance b/f To Bank – Machinery purchased (Bal. fig.)

165.00 182.50

6. Analysis of other Non-current Assets and Liabilities Assets/Liabilities Share Capital 7% Debentures Loan from Bank Freehold Property Long-term Investment General Reserve

Opening Balance 300 75 27 225 150 200

Closing Balance 325 50 14.25 240 162.50 219

Increase/ Decrease (+) 25 (−) 25 (−) 12.75 (+) 15 (+) 12.50 (+) 19

Analysis New Issue of Shares Redemption of Debentures Repayment of Loan Purchase of Property Further Investment Transfer to Reserve Net Loss (after Appropriation)

Stop and Think i. Under Direct Method, all workings as shown in the above are to be prepared except General Reserve and Profit & Loss A/c, under Working No. 6. ii. Under Indirect Method, all workings as shown earlier are to be prepared except Working Notes 1 & 2. iii. Here ‘cash and cash equivalents’ means cash at bank and short-term investment. iv. As there is no outstanding or prepaid overhead either at the beginning or at the end of the year, it must be considered that the entire overhead expenses had been duly paid during the year 2007–08.

Modified Date: Mon, Jul 05, 2010 05:30:16 PM

Output Date: Tue, Jul 06, 2010 11:46:14 AM

Rev II

Project: Management Accounting_Debarshi Bhattacharyya ACE Pro India Pvt. Ltd. File: X:\Pearson\Management Accounting_Debarshi Bhattacharyya\MAIN\M04\LAYOUT_M04\M04_DEBA_ISBN_EN_SE_C04_II.indd

249

CASH FLOW ANALYSIS

Problem 28 From the following particulars, prepare a Cash Flow Statement of Chalsa Ltd for the year that ended on 31 December 2007: Dr.

Profit & Loss A/c for the Year that Ended on 31 December 2007 Rs. To Opening Stock 5,500 By Sales To Purchases 80,500 By Closing Stock To Wages 16,000 By Loss of Stock To Gross Profit c/d 1,39,000 2,41,000 To Expenses 10,000 By Gross Profit b/d To Depreciation 12,000 By Income from Investment (Tax Free) To Bad Debt 1,000 To Loss on Sale of Plant 5,000 To Loss of Stock less Insurance Claim 1,000 (i.e., Net loss) To Deferred Expenses 5,000 To Income Tax 10,000 To Net Profit c/d 1,01,000 1,45,000 To Dividend 8,000 By Balance b/d To Reserve 23,000 By Net Profit b/d To Balance c/d 1,00,000 1,31,000

Cr. Rs. 2,16,000 20,000 5,000 2,41,000 1,39,000 6,000

1,45,000 30,000 1,01,000 1,31,000

Balance Sheets Liabilities Share Capital Reserve Profit & Loss A/c Creditors Liabilities for Expenses Proposed Dividend Provision for Taxation Advance Income from Investment

2006 Rs. 80,000 50,000 30,000 30,000 8,000 5,000 15,000 5,000 2,23,000

2007 Rs. 90,000 73,000 1,00,000 20,000 10,000 10,000 20,000 3,000 3,26,000

Assets Fixed Assets Investment Stock in Trade Debtors Cash & Bank Prepaid Expenses Deferred Expenses

2006 Rs. 80,000 68,500 5,500 50,000 3,500 5,000 10,500

2007 Rs. 2,10,000 30,000 20,000 55,000 1,500 4,000 5,500

2,23,000

3,26,000

[B.Com. (Hons), Calcutta University—Adapted] Tutorial Note As the details of the Profit & Loss A/c and the Balance Sheet of two consecutive years are given in the problem, Statement, in this case, can be prepared under both the methods. Here, the Cash Flow Statement is prepared under both the methods, but students are advised to follow either of the two methods in the examination hall (preferably Direct Method in such a case).

Solution Method 1: Direct Method Cash Flow Statement of Chalsa Ltd for the year that ended on 31 December 2007 Rs. A. Cash Flows from Operating Activities: Cash Receipts from Customers1

Rs.

Rs.

2,10,000 (Continued)

Modified Date: Sat, Jul 03, 2010 11:47:06 AM

Output Date: Tue, Jul 06, 2010 11:46:58 AM

Rev II

Project: Management Accounting_Debarshi Bhattacharyya ACE Pro India Pvt. Ltd. File: X:\Pearson\Management Accounting_Debarshi Bhattacharyya\MAIN\M04\LAYOUT_M04\M04_DEBA_ISBN_EN_SE_C04_II.indd

250

MANAGEMENT ACCOUNTING

Rs. Less:

Cash paid to Suppliers & Employees: Payment to Suppliers2 Payment to Wages Payment of Expenses3

Rs.

Rs.

90,500 16,000 7,000 1,13,500 96,500 5,000 91,500 4,000

Cash generated from operation Less: Income Tax paid4 Cash flow from operation before extraordinary item Add: Insurance claim Received against loss of Stock (Rs. 5,000 – Rs. 1,000) Net Cash Flow from Operating Activities B. Cash Flows from Investing Activities: Proceeds Received from Sale of Plant5 Proceeds Received from Sale of Investment8 Income Received from Investment6

95,500 23,000 38,500 4,000 65,500 1,70,000

Purchase of Fixed Assets5 Net Cash Flow from Investing Activities C. Cash Flows from Financing Activities: Proceeds Received from issue of shares8 Less: Dividend paid7 Net Cash Flow from Financing Activities Net Increase in Cash & Cash Equivalents Add: Cash & Cash Equivalents at the beginning of the year Cash & Cash Equivalents at the end of the year

Less:

(1,04,500) 10,000 3,000 7,000 (2,000) 3,500 1,500

Method 2: Indirect Method Cash Flow Statement of Chalsa Ltd for the year that ended on 31 December 2007 Rs.

Add:

Less:

Add:

A. Cash Flows from Operating Activities: Net Profit (before appropriation) for the year as ascertained in Profit & Loss A/c: Adjustment for Non-operating & Non-current items charged to Profit & Loss A/c: Income Tax Depreciation Deferred Expenses Loss on Sale of Plant Net Loss of Stock

10,000 12,000 5,000 5,000 1,000

Adjustment for Non-operating & Non-current items credited to Profit & Loss A/c: Income from Investments Loss of Stock (cost price)

6,000 5,000

Operating Profit before Working Capital changes Increase in Operating Current Liabilities: Liabilities for Expenses Decrease in Operating Current Assets: Prepaid Expenses

Rs.

Rs.

1,01,000

33,000 1,34,000

11,000 1,23,000 2,000 1,000 3,000 1,26,000 (Continued)

Modified Date: Sat, Jul 03, 2010 11:47:06 AM

Output Date: Tue, Jul 06, 2010 11:46:58 AM

Rev II

Project: Management Accounting_Debarshi Bhattacharyya ACE Pro India Pvt. Ltd. File: X:\Pearson\Management Accounting_Debarshi Bhattacharyya\MAIN\M04\LAYOUT_M04\M04_DEBA_ISBN_EN_SE_C04_II.indd

251

CASH FLOW ANALYSIS

Rs. Less:

Less:

Increase in Operating Current Assets: Stock in trade Debtors Decrease in Operating Current Liabilities: Creditors

Rs.

Rs.

14,500 5,000 10,000

Cash Generated from Operation Income Tax paid

Add:

Insurance claim Received against loss of Stock Net Cash Flow from Operating Activities B. Cash Flows from Investing Activities: Same as calculated under Direct Method in above C. Cash Flows from Financing Activities: Same as calculated under Direct Method in above Net Increase in Cash & Cash Equivalents Add: Cash & Cash Equivalents at the beginning of the year Cash & Cash Equivalents at the end of the year

29,500 96,500 5,000 91,500 4,000 95,500 (1,04,500) 7,000 (2,000) 3,500 1,500

Working Notes 1. Dr. To Balance b/f To Sales

Debtors Account Rs. 50,000 By Bank – Collection (Bal. fig.) 2,16,000 By Bad Dept By Balance c/f 2,66,000

Cr. Rs. 2,10,000 1,000 55,000 2,66,000

Creditors Account Rs. 90,500 By Balance b/f 20,000 By Purchases 1,10,500

Cr. Rs. 30,000 80,500 1,10,500

2. Dr. To Bank – Payment (Bal. fig.) To Balance c/f

3. Dr. To Balance b/f (i.e., opening prepaid) To Bank – Expenses paid during the year (Bal. fig.) To Balance c/f (i.e., closing outstanding)

Expenses Account Rs. 5,000 By Balance b/f (i.e., opening outstanding) By Profit & Loss A/c – Expenses for the year 7,000 charged to Profit & Loss A/c 10,000 By Balance c/f (i.e., closing prepaid) 22,000

Cr. Rs. 8,000 10,000 4,000 22,000

4. Dr.

Provision for Taxation Account Rs. To Bank – Tax paid during the year (Bal. fig.) 5,000 By Balance b/f By Profit & Loss A/c – Provided for during this year To Balance c/f 20,000 25.000

Cr. Rs. 15,000 10,000

25,000 (Continued)

Modified Date: Sat, Jul 03, 2010 11:47:06 AM

Output Date: Tue, Jul 06, 2010 11:46:58 AM

Rev II

Project: Management Accounting_Debarshi Bhattacharyya ACE Pro India Pvt. Ltd. File: X:\Pearson\Management Accounting_Debarshi Bhattacharyya\MAIN\M04\LAYOUT_M04\M04_DEBA_ISBN_EN_SE_C04_II.indd

252

MANAGEMENT ACCOUNTING

5. Dr.

Fixed Asset Account Rs. 80,000 By Profit & Loss A/c – Depreciation for the year 1,70,000 By Profit & Loss A/c – Loss on Sale of Plant By Bank – Sale proceeds of Plant (Bal. fig.) By Balance c/f 2,50,000

Cr. Rs. 12,000 5,000 23,000 2,10,000 2,50,000

Income from Investments Account Rs. To Profit & Loss A/c – Income Receivable for 6,000 By Balance b/f (i.e., opening pre-received) the year To Balance c/f (i.e., closing pre-received) 3,000 By Bank – Income Received during the year (Bal. fig.) 9,000

Cr. Rs. 5,000

To Balance b/f To Bank – Purchase of Fixed Assets (Bal. fig.)

6. Dr.

4,000 9,000

7. Dr.

Proposed Dividend Account Rs. 3,000 By Balance b/f

To Bank – Dividend for 2006 paid in 2007 (Bal. fig.) To Balance c/f

8.

10,000 13,000

Cr. Rs. 5,000

By Profit & Loss A/c – Dividend proposed for 2007

8,000 13,000

Analysis of other Non-current Assets and liabilities Assets/Liabilities

Share Capital Investments

Opening Balance Rs. 80,000 68,500

Closing Balance Rs. 90,000 30,000

Increase/ Decrease Rs. (+)10,000 (−)38,500

Analysis New issue of Shares Sale of Investments

Stop and Think i. As the income from Investment is tax-free, the entire amount of income Tax was paid against the Operating Profit only. ii. Under Indirect Method, no need to prepare Working Notes 1 & 2 (i.e., Debtors’ and Creditors’ Account). iii. As Bad Debt is an Operating and Current item (as debtors are reduced for Bad Debt), it is not added back to the Net Profit while calculating the Cash Flow from Operating activities under Indirect Method. But, as the loss of Stock is a loss of abnormal nature, it is not considered as the Operating Loss; and accordingly, added back to the Net Profit while calculating the Cash Flow from the Operating Activities under Indirect Method.

Problem 29 Balance Sheet of Yoyng India Ltd, as on 31 March 2006 and 31 March 2005 (Figures – Rs. in ’000) Liabilities Equity Shares (Rs. 10 each) Profit & Loss A/c 10% Debentures

31 March 2006 4,300 640 2,050

31 March 2005 4,000 980 2,200

Assets Building Machinery Land

31 March 2006 2,500 2,000 1,500

31 March 2005 2,500 1,600 1,800 (Continued)

Modified Date: Sat, Jul 03, 2010 11:47:06 AM

Output Date: Tue, Jul 06, 2010 11:46:58 AM

Rev II

Project: Management Accounting_Debarshi Bhattacharyya ACE Pro India Pvt. Ltd. File: X:\Pearson\Management Accounting_Debarshi Bhattacharyya\MAIN\M04\LAYOUT_M04\M04_DEBA_ISBN_EN_SE_C04_II.indd

CASH FLOW ANALYSIS

Liabilities Trade Creditors Provision for Taxation Depreciation on Building Depreciation on Machinery

31 March 2006 650 125 600 300 8,665

31 March 2005 800 100 500 200 8,780

31 March 2006 65 1,400 800 400 8,665

Assets Prepaid Expenses Inventory Debtors Cash & Bank

253

31 March 2005 80 1,550 650 600 8,780

Additional Information: i. Dividend paid during the year – Rs. 4,50,000. ii. Land was sold for cash at a Profit of Rs. 50,000. iii. Machinery Costing Rs. 2,00,000 (WDV – Rs. 40,000) was sold for Rs. 30,000. Also Machinery costing Rs. 6,00,000 was purchased. iv. Amount Transferred to provision for Taxation during the year – Rs. 1,60,000. Prepare a Cash Flow Statement for the year that ended on 31 March 2006. [B.Com. (Hons), Calcutta University—2006]

Solution Cash Flow Statement (under Indirect Method) of Young India Ltd for the year that ended on 31 March 2006 Rs. in ’000 A. Cash Flows from Operating Activities: Net Profit after appropriation for the year Add: Appropriations made during the year: Dividend for the year4 Net Profit (before appropriation) for the year Add: Adjustment for Non-operating and Non-current items charged to Profit & Loss A/c: Provision for Taxation Depreciation on Building1 Depreciation on Machinery6 Loss on Sale of Machinery5 Less:

Add:

Less:

Less:

Adjustment for Non-operating and Non-current items credited to Profit & Loss A/c: Profit on Sale of land Operating Profit before Working Capital changes Increase in Operating Current Liabilities Decrease in Operating Current Liabilities: Prepaid Expenses Inventory Decrease in Operating Current Liabilities: Trade creditors Increase in Operating Current Assets: Debtors Cash Generated from Operation Income Tax paid2 Net Cash Flow from Operating Activities

Rs. in ’000

Rs. in ’000

(340) 450 110

160 100 260 10

530 640

50 590 Nil 15 150

165 755

150 150

300 455 135 320 (Continued)

Modified Date: Sat, Jul 03, 2010 11:47:06 AM

Output Date: Tue, Jul 06, 2010 11:46:58 AM

Rev II

Project: Management Accounting_Debarshi Bhattacharyya ACE Pro India Pvt. Ltd. File: X:\Pearson\Management Accounting_Debarshi Bhattacharyya\MAIN\M04\LAYOUT_M04\M04_DEBA_ISBN_EN_SE_C04_II.indd

254

MANAGEMENT ACCOUNTING

Rs. in ’000 B. Cash Flows from Investing Activities: Proceeds Received from Sale of building3 Proceeds Received from Sale of Machinery Less: Purchase of Machinery Net Cash Flow from Investing Activities C. Cash Flows from Financing Activities: Proceeds Received from issue of Shares1 Less: Redemption of Debentures1 Payment of Dividend Net Cash Flow from Financing Activities Net Increase in Cash & Cash Equivalents Add: Cash & Cash Equivalents at the beginning of the year Cash & Cash Equivalents at the end of the year

Rs. in ’000

350 30

Rs. in ’000

380 600 (220) 300

150 450

600 (300) (200) 600 400

Working Notes 1.

Analysis of other Non-current Assets and Liabilities (against which there is no adjustment) Assets/Liabilities

Equity Shares 10% Debentures Depreciation on Duilding Building (at cost) Profit & Loss A/c

Opening Balance Rs. in ’000 4,000 2,200 500 2,500 980

Closing Balance Rs. in ’000 4,300 2,050 600 2,500 640

Increase/ Decrease Rs. in ’000 (+) 300 (−) 150 (+) 100 Nil (+) 340

Analysis Fresh issue of Shares Redemption of Debentures Depreciation for the year No Purchase or Sale Loss (after appropriation) for the year

2. Dr.

Provision for Taxation Account Rs. in ’000 To Bank – Tax paid during the year (Bal. fig.) 135 By Balance b/f To Balance c/f 125 By Profit & Loss A/c – Provision made for 2005–06 260

Cr. Rs. in ’000 100 160 260

3. Dr. To Balance b/f To Profit & Loss A/c – Profit on Sale

Land Account Rs. in ’000 1,800 By Bank – Sale proceeds of Land (Bal. fig.) 50 By Balance c/f 1,850

Cr. Rs. in ’000 350 1,500 1,850

Dividend Account Rs. in ’000 450 By Profit & Loss appropriation A/c – Dividend provided 450

Cr. Rs. in ’000

4. Dr. To Bank – Dividend paid

450 450 (Continued)

Modified Date: Sat, Jul 03, 2010 11:47:06 AM

Output Date: Tue, Jul 06, 2010 11:46:58 AM

Rev II

Project: Management Accounting_Debarshi Bhattacharyya ACE Pro India Pvt. Ltd. File: X:\Pearson\Management Accounting_Debarshi Bhattacharyya\MAIN\M04\LAYOUT_M04\M04_DEBA_ISBN_EN_SE_C04_II.indd

255

CASH FLOW ANALYSIS

5. Dr.

Machinery (at Cost) Account Rs. in ’000 1,600 By Provision for Depreciation A/c – Accumulated Depreciation on Machinery Sold 600 By Bank – Sale Proceeds of Machinery By Profit & Loss A/c – Loss on Sale By Balance c/f 2,200

To Balance b/f To Bank – Purchase

Cr. Rs. in ’000 160 30 10 2,000 2,200

6. Dr.

Provision for Depreciation on Machinery Rs. in ’000 To Machinery A/c – Accumulated By Balance b/f Depreciation on Machinery sold Transferred (2,00,000 – 40,000) 160 To Balance c/f 300 By Profit & Loss A/c – Depreciation for the year (Bal. fig.) 460

Cr. Rs. in ’000 200

260 460

Problem 30 From the following Balance Sheets of JP International, prepare a Cash Flow Statement for the year that ended on 31 December 2006, as per AS-3: Balance Sheets as on Liabilities Equity Share Capital Redeemable Preference Share Capital Debenture Long-term loan Reserves & Surplus Bank Overdraft Sundry Creditors Proposed Dividend Provision for Taxation

31 December 2005 Rs. 150

31 December 2006 Rs. 350

100 150 100 40 60 80 30 20 730

150 100 50 50 – 100 60 40 900

Assets Goodwill Fixed Assets Inventories Debtors Bank Prepaid Expenses Miscellaneous Expenditure

31 December 2005 Rs. 75 355 110 120 Nil 30 40

31 December 2006 Rs. 60 620 70 75 25 20 30

730

900

Additional information available on 31 December 2006: i. Accumulated Depreciation on Fixed Assets amounted to Rs. 1,60,000 and Rs. 1,85,000, as on 31 December 2005 and 31 December 2006, respectively; and a Plant Costing Rs. 30,000 (25% depreciated) was sold for Rs. 50,000. ii. A land of Rs. 1,50,000 and stock of Rs. 40,000 were purchased for a consideration of Rs. 2,00,000, paid for in shares. iii. Dividend for 2005 was paid along with an interim Dividend of 5% on the Opening Equity Capital. iv. Tax Liabilities for 2005 was settled at Rs. 28,000. [B.Com. (Hons), Calcutta University—2007]

Modified Date: Sat, Jul 03, 2010 11:47:06 AM

Output Date: Tue, Jul 06, 2010 11:46:58 AM

Rev II

Project: Management Accounting_Debarshi Bhattacharyya ACE Pro India Pvt. Ltd. File: X:\Pearson\Management Accounting_Debarshi Bhattacharyya\MAIN\M04\LAYOUT_M04\M04_DEBA_ISBN_EN_SE_C04_II.indd

256

MANAGEMENT ACCOUNTING

Solution Cash Flow Statement (under Indirect Method) of JP International for the year that ended on 31 December 2006 Rs. in ’000 A. Cash Flows from Operating Activities: Net Profit (after approprition) for the year Add: Appropriation made during the year: Dividend for 2006 provided for6 Interim Dividend paid in 2006 provided for7 Net Profit (before appropriation) for the year Add: Adjustment for Non-operating and Non-current items charged to Profit & Loss A/c: Provision for Taxtion5 Goodwill written off4 Miscellaneous Expenditure written off1 Depreciation on Fixed Assets9

Rs. in ’000

Rs. in ’000

10.00 60.00 7.50

67.50 77.50

48.00 25.00 10.00 32.50 115.50 193.00

Less:

Add:

Less:

Less:

Adjustment for Non-operating and Non-current items credited to Profit & Loss A/c: Profit on Sale of Plant8 Increase in Operating Current Liabilities: Sundry Creditors Decrease in Operating Current Assets: Inventories [110 – (70 – 40)] Debtors Prepaid Expenses Decrease in Operating Current Liabilities: Bank Overdraft Increase in Operating Current Assets

27.50 165.50 20 80 45 10 155.00 320.50 60 Nil 60.00 260.50 28.00

Cash generated from Operation Tax Paid

232.50 Net Cash Flow from Operating Activities B. Cash Flows from Investing Activities: Proceeds Received from Sale of Plant Less: Purchase of Fixed Assets8 Net Cash Flow from Investing Activities C. Cash Flows from Financing Activities: Fresh issue of Preference Shares1 Less: Redemption of Debentures1 Repayment of Long-term Loan1 Dividend for 2005 paid6 Interim Equity Dividend paid7 Net Cash Flow from Financing Activities Net Increase in Cash & Cash Equivalents Add: Cash & Cash Equivalents at the beginning of the year Cash & Cash Equivalents at the end of the year

Modified Date: Sat, Jul 03, 2010 11:47:06 AM

Output Date: Tue, Jul 06, 2010 11:46:58 AM

50 170 (120.00) 50.00 50 50 30 7.5

137.50 (87.50) 25 Nil 25

Rev II

Project: Management Accounting_Debarshi Bhattacharyya ACE Pro India Pvt. Ltd. File: X:\Pearson\Management Accounting_Debarshi Bhattacharyya\MAIN\M04\LAYOUT_M04\M04_DEBA_ISBN_EN_SE_C04_II.indd

257

CASH FLOW ANALYSIS

Working Notes 1.

Analysis of other Non-current Assets and Liabilities (against which there is no adjustment) Assets/Liabilities

Equity Shares Debentures Long-term Loans Miscellaneous Expenditure

Opening Balance Rs. in ’000 100 150 100 40

Closing Balance Rs. in ’000 150 100 50 30

Increase/ Decrease Rs. in ’000 (+) 50 (−) 50 (−) 50 (−) 10

Analysis Fresh issue of Shares Redemption of Debentures Repayment of Loan Written off against Profit & Loss A/c

2. Dr. To Equity Share Capital – Purchase consideration discharged

Vendor Company Account Rs. in ’000 By Fixed Assets A/c (land) 200 By Stock By Goodwill (Bal. fig.) –Excess Consideration paid Adjusted 200

Cr. Rs. in ’000 150 40 10 200

3. Dr.

To Balance c/f

Equity Share Capital Account Rs. in ’000 By Balance b/f 350 By Vendor company A/c – Shares issued to Discharge the Purchase Consideration 350

Cr. Rs. in ’000 150 200 350

4. Dr. To Balance b/f To Vendor company A/c – Excess Consideration Paid Adjusted

Goodwill Account Rs. in ’000 75 By Profit & Loss A/c – Written off (Bal. fig.) By Balance c/f 10 85

Cr. Rs. in ’000 25 60 85

5. Dr. To Bank – Tax liability for 2005 paid To Balance c/f

Provision for Taxation Account Rs. in ’000 28 By Balance b/f 40 By Profit & Loss A/c – Provision made in 2006 (Bal. fig.) 68

Cr. Rs. in ’000 20 48 68

6. Dr. To Bank – Dividend for 2005 paid To Balance c/f

Proposed Dividend Account Cr. Rs. in ’000 Rs. in ’000 30 By Balance b/f 30 60 By Profit & Loss appropriation A/c – Proposed Dividend for 2006 provided for (Bal. fig.) 60 90 90 (Continued)

Modified Date: Sat, Jul 03, 2010 11:47:06 AM

Output Date: Tue, Jul 06, 2010 11:46:58 AM

Rev II

Project: Management Accounting_Debarshi Bhattacharyya ACE Pro India Pvt. Ltd. File: X:\Pearson\Management Accounting_Debarshi Bhattacharyya\MAIN\M04\LAYOUT_M04\M04_DEBA_ISBN_EN_SE_C04_II.indd

258

MANAGEMENT ACCOUNTING

7. Dr.

Cr. Rs. in ’000

Interim Dividend Account

Rs. in ’000 To Bank – Interim Dividend for 2006 paid (5% on Rs. 1,50,000)

7.50 7.50

By Profit & Loss Appropriation A/c – Interim Dividend provided for

7.50 7.50

8. Dr.

Fixed Asset (at Cost) Account Rs. in ’000 515.00 By Provision for Depreciation A/c To Balance b/f (355 + 160) – Accumulated Depreciation on Plant sold To Vendor Company A/c – Land purchased 150.00 By Bank – Sale proceeds of Plant To Profit & Loss A/c – Profit on Sale of Plant By Balance c/f (620 + 185) [50,000 – (30,000 – 25%)] 27.50 To Bank – Fixed Assets Purchased for Cash (Bal. fig.) 170.00 862.50

Cr. Rs. in ’000

7.50 50.00 805.00

862.50

9. Dr.

Provision for Depreciation on Fixed Asset Account Rs. in ’000 To Fixed Asset A/c – Accumulated By Balance b/f Depreciation on Plant sold transferred (25% of Rs. 30,000) 7.50 By Profit & Loss A/c – Depreciation for 2006 (Bal. fig.) To Balance c/f 185.00 192.50

Cr. Rs. in ’000 20.00

32.50

192.50

Stop and Think i. Land and Stock were purchased from a company, the consideration payable against which discharged by issue of shares. The company which sold its Land and Stock is termed here as the ‘vendor company’ for which an account (W/N No. 2) is opened. ii. In this problem, neither the value Fixed Assets were purchased for cash, nor the Depreciation charged for the year 2006 is given. That is why, despite the value of Fixed Assets appearing in the Balance Sheet at WDV, two separate accounts are opened here namely—one is Fixed Asset (at cost) account, the opening and closing values have been derived by adding the WDV, with respective accumulated Depreciation, and the other is the provision for Depreciation on Fixed Asset Account, taking the accumulated Depreciations on opening and closing date as Opening & Closing Balances respectively. iii. While deducting the decrease in the Value of Inventories, the Value of Stock worth Rs. 40,000 bought from the Vendor Company is deducted to get the net decrease in the stock due to Operational Activities.

Problem 31 Following were the Balance Sheets of a company as on 31 December 2007 and 31 December 2006: Liabilities Share Capital Reserves

2007 Rs. in ’000 1,500 3,410

2006 Rs. in ’000 1,250 1,380

Assets Fixed Assets at cost

2007 Rs. in ’000 2,180

2006 Rs. in ’000 1,910 (Continued)

Modified Date: Sat, Jul 03, 2010 11:47:06 AM

Output Date: Tue, Jul 06, 2010 11:46:58 AM

Rev II

Project: Management Accounting_Debarshi Bhattacharyya ACE Pro India Pvt. Ltd. File: X:\Pearson\Management Accounting_Debarshi Bhattacharyya\MAIN\M04\LAYOUT_M04\M04_DEBA_ISBN_EN_SE_C04_II.indd

CASH FLOW ANALYSIS

Liabilities Long-term Loan Sundry Creditors Interest Payable Income Tax Payable

2007 Rs. in ’000 1,110

2006 Rs. in ’000 1,040

150 230 400

1,890 100 1,000

6,800

6,660

Assets Less:

Provision for Depreciation Long-term Investments Inventories Sundry Debtors Short-term Investments Cash & Bank Income Receivable on Investments

259

2007 Rs. in ’000

2006 Rs. in ’000

1,450 730 2,500 900 1,700 670 200

1,060 850 2,500 1,950 1,200 135 25

100 6,800

– 6,660

Statement of Profit & Loss A/c for the year that ended on 31 December 2007 Rs. in ’000 Less:

Less:

Add:

Less:

Sales Cost of Sales: Materials Consumed Wages & Overheads Gross Profit Depreciation on Fixed Assets Administrative & Selling Expenses Interest on Loan Loss on Sale of long-term Investment

19,000 7,000 450 850 400 100

Income from Investment Insurance Claim Received from Earthquake Disaster settlement Net Profit before Tax Income Tax Net Profit after Tax

500 180

Rs. in ’000 30,650

26,000 4,650

1,800 2,850 680 3,530 300 3,230

Additional Information: i. Plant, having an Original Cost of Rs. 80,000 and accumulated Depreciation of Rs. 60,000, was sold in 2007 for Rs. 20,000. ii. Investments (long-term) further made during 2007 was Rs. 500 (in lakhs). iii. An amount of Rs. 2,50,000 was raised from the long-term borrowing. iv. Income Tax of Rs. 3,00,000 as provided in the statement of Profit & Loss A/c included Rs. 30,000 as Tax deducted at source on income from the Long-term Investment. Prepare a Cash Flow Statement for the year that ended on 31 December 2007 separately under Direct Method and Indirect Method. Solution Method 1: Direct Method Cash Flow Statement of a company for the year that ended on 31 March 2008 Rs. in ’000 A. Cash Flows from Operating Activities: Cash Receipts from Customers1 Less: Cash paid to Suppliers & Employees: Payment to Suppliers for Goods3

Rs. in ’000

Rs. in ’000

30,150 19,690 (Continued)

Modified Date: Sat, Jul 03, 2010 11:47:06 AM

Output Date: Tue, Jul 06, 2010 11:46:58 AM

Rev II

Project: Management Accounting_Debarshi Bhattacharyya ACE Pro India Pvt. Ltd. File: X:\Pearson\Management Accounting_Debarshi Bhattacharyya\MAIN\M04\LAYOUT_M04\M04_DEBA_ISBN_EN_SE_C04_II.indd

260

MANAGEMENT ACCOUNTING

Payment of Administrative & Selling Expenses Payment of Wages and Overheads Cash generated from Operation Less: Income Tax paid on Operating Profit4 Cash Flow from operation before extraordinary item Add: Extraordinary Income: Insurance proceeds received from earthquake disaster settlement Net Cash Flow from Operating Activities B. Cash Flows from Investing Activities: Sale proceeds of Plant Sale proceeds of long-term Investments7 Income Received from Investments less TDS10 Purchase of Fixed Assets5 Purchase of Investments7 Net Cash Flow from Investing Activities C. Cash Flows from Financing Activities: Proceeds Received from New Issue of shares11 Proceeds Received from long-term loan Less:

Less:

Repayment of long-term loan13 Payment of Dividend12 Payment of Interest9

Rs. in ’000 850 7,000

Rs. in ’000

Rs. in ’000

27,540 2,610 870 1,740

180 1,920 20 400 370 790 350 500

850 (60) 250 250 500

180 1,200 270 1,650

Add:

Net Cash Flow from Financing Activities Net Increase in Cash & Cash Equivalents Opening Cash & Cash Equivalents: Cash & Bank Short-term Investment

(1,150) 710 25 135 160 870

Closing Cash & Cash Equivalents (200 + 670)

Method 2: Indirect Method Cash Flow Statement of a Company for the year that ended on 31 December 2007 Rs. in ’000 Add:

Less:

Add:

Net Profit before Tax Adjustment for Non-operating & Non-current items debited to Profit & Loss A/c: Depreciation on Fixed Assets Interest on loan Loss on Sale of Long-term Investment Adjustment for Non-operating & Non-current items credited to Profit & Loss A/c: Income from Investment Insurance claim received from earthquake disaster settlement Operating Profit before Working Capital Changes Increase in Operating Current Liabilities Decrease in Operating Current Liabilities: Inventories

450 400 100

500 180

Rs. in ’000 3,530

Rs. in ’000

950 4,480

680 3,800

Nil 1,050

1,050 4,850 (Continued)

Modified Date: Sat, Jul 03, 2010 11:47:06 AM

Output Date: Tue, Jul 06, 2010 11:46:58 AM

Rev II

Project: Management Accounting_Debarshi Bhattacharyya ACE Pro India Pvt. Ltd. File: X:\Pearson\Management Accounting_Debarshi Bhattacharyya\MAIN\M04\LAYOUT_M04\M04_DEBA_ISBN_EN_SE_C04_II.indd

261

CASH FLOW ANALYSIS

Rs. in ’000 Decrease in Operating Current Liabilities: Sundry Creditors Increase in Operating Current Assets: Sundry Debtors Cash generated from operation Less: Income Tax paid on Operating Profit Cash flow from operation before extraordinary item Add: Extraordinary item: Insurance claim Received from earthquake-disaster settlement Net Cash Flow from Operating Activities B. Cash Flows from Investing Activities: Same as computed under direct method in above C. Cash Flows from Financing Activities: Same as computed under direct method in above Net Increase in Cash & Cash Equivalents Add: Cash & Cash Equivalents at the beginning of the year Cash & Cash Equivalents at the end of the year

Rs. in ’000

Rs. in ’000

Less:

1,740 500

2,240 2,610 870 1,740 180 1,920 (60) (1,150) 710 160 870

Working Notes 1. Dr. To Balance b/f To Sales

Sundry Debtors Account Rs. in ’000 1,200 By Bank – Collection (Bal. fig.) 30,650 By Balance c/f 31,850

Cr. Rs. in ’000 30,150 1,700 31,850

2. Dr. To Balance b/f To Purchases (Bal. fig.)

Inventories Account Rs. in ’000 1,950 By Materials consumed 17,950 By Balance c/f 19,900

Cr. Rs. in ’000 19,000 900 19,900

3. Dr. To Bank – Payment (Bal. fig.) To Balance c/f

Sundry Creditors Account Rs. in ’000 19,690 By Balance b/f 150 By Purchases2 19,840

Cr. Rs. in ’000 1,890 17,950 19,840

4. Dr.

Income Tax (on Operating Profit) Account Rs. in ’000 To Bank – Tax paid on Operating Profit By Balance b/f (Bal. fig.) 870 To Balance c/f 400 By Profit & Loss A/c – Tax provided for 2007 (300 – 30) 1,270

Cr. Rs. in ’000 1,000

270 1,270 (Continued)

Modified Date: Sat, Jul 03, 2010 11:47:06 AM

Output Date: Tue, Jul 06, 2010 11:46:58 AM

Rev II

Project: Management Accounting_Debarshi Bhattacharyya ACE Pro India Pvt. Ltd. File: X:\Pearson\Management Accounting_Debarshi Bhattacharyya\MAIN\M04\LAYOUT_M04\M04_DEBA_ISBN_EN_SE_C04_II.indd

262

MANAGEMENT ACCOUNTING

5. Dr.

Fixed Asset (at Cost) Account Rs. in ’000 1,910 By Provision for Depreciation A/c6 – Accumulated Depreciation on Plant sold 350 By Bank – Sale Proceeds of Plant By Balance c/f 2,260

To Balance b/f To Bank – Purchase (Bal. fig.)

Cr. Rs. in ’000 60 20 2,180 2,260

6. Dr.

Provision for Depreciation Account Rs. in ’000 By Balance b/f 60 1,450 By Profit & Loss A/c – Depreciation provided for 2007 1,510

To Fixed Asset A/c – Accumulated Depreciation on Plant sold To Balance c/f

Cr. Rs. in ’000 1,060 450 1,510

7. Dr. To Balance b/f To Bank – Further investment made

Long-term Investment Account Cr. Rs. in ’000 Rs. in ’000 2,500 By Bank – Sale proceeds of Investment (Bal. fig.) 400 500 By Profit & Loss A/c – Loss on Sale of Investment 100 By Balance c/f 2,500 3,000 3,000

8. Dr. To Balance b/f To Profit & Loss A/c – Income Receivable for 2007

Income on Investment Account Rs. in ’000 – By Bank – Income Received in 2007 (Bal. fig.) 500 By Balance c/f 500

Cr. Rs. in ’000 400 100 500

9. Dr.

Interest Payable on Loan Account Rs. in ’000 To Bank – Interest paid in 2007 (Bal. fig.) 270 By Balance b/f To Balance c/f 230 By Profit & Loss A/c – Interest Payable for 2007 (Bal. fig.) 500

10.

Less:

Cr. Rs. in ’000 100 400 500

Net income from Investment received in 2007 Income Received from Investment in 20078 Tax Deducted from source (TDS) Net income from Investment

Rs. in ’000 400 30 370

11. Dr. To Balance c/f

Share Capital Account Rs. in ’000 1,500 By Balance b/f By Bank – New Issue of Shares (Bal. fig.) 1,500

Cr. Rs. in ’000 1,250 250 1,500

(Continued)

Modified Date: Sat, Jul 03, 2010 11:47:06 AM

Output Date: Tue, Jul 06, 2010 11:46:58 AM

Rev II

Project: Management Accounting_Debarshi Bhattacharyya ACE Pro India Pvt. Ltd. File: X:\Pearson\Management Accounting_Debarshi Bhattacharyya\MAIN\M04\LAYOUT_M04\M04_DEBA_ISBN_EN_SE_C04_II.indd

CASH FLOW ANALYSIS

263

12. Dr. To Bank – Dividend Declared & paid in 2007 To Balance c/f

Reserve Account Rs. in ’000 1,200 By Balance b/f 3,410 By Profit & Loss A/c – Net Profit for 2007 Transferred 4,610

Cr. Rs. in ’000 1,380 3,230 4,610

13. Dr. To Bank – Loan Repaid in 2007 (Bal. fig.) To Balance c/f

Long-term Loan Account Rs. in ’000 180 By Balance b/f 1,110 By Bank – Loan taken in 2007 1,290

Cr. Rs. in ’000 1,040 250 1,290

Stop and Think i. Cash and Cash equivalent refers to cash in hand, at bank and Short-term Investment. ii. Under Indirect Method, no need to prepare Working Notes 1, 2 & 3. iii. As Tax of Rs. 30,000 was already deducted from the source on income from the Long-term Investment, it is shown as a deduction from income from Investment (see Working Note 10) under the head ‘Cash Flow from Investing Activities’ and the balance amount of income Tax of Rs. 2,70,000 (i.e., Rs. 3,00,000 – Rs. 30,000), as provided in the statement of Profit & Loss A/c is considered as a Tax Provision on the Operating Profit (see Working Note 4). iv. As there are no outstanding or prepaid Wages and Overheads and Administrative and Selling Expenses, either at the beginning or at the end of the year 2007, it must be considered that the amount of such expenses as charged in the Profit & Loss A/c have been paid in full.

Problem 32 From the following summarized Balance Sheets of Sunshine Ltd and other relevant information, prepare a Cash Flow Statement for the year that ended on 31 March 2004: Liabilities Share Capital Securities premium General Reserve Capital Reserve Profit & Loss A/c 8% Debentures Provision for Taxation Proposed Dividend Sundry Creditors Outstanding Expenses

31 March 2003 Rs. 5,00,000 45,000 1,00,000 – 70,000 1,00,000 35,000 50,000 73,000 10,200 9,83,200

31 March 2004 Rs. 7,00,000 55,000 45,000 15,000 67,500 85,000 45,000 70,000 61,100 12,700 11,56,300

Assets Land & Building Plant & Machinery Stock Debtors Cash Prepaid Expenses

31 March 2003 Rs. 5,00,000 2,05,000 98,000 1,02,000 62,500 15,700

31 March 2004 Rs. 6,50,000 2,15,500 95,000 1,18,200 68,700 8,900

9,83,200

11,56,300

Additional Information: i. A Plant Costing Rs. 50,000 (WDV – Rs. 35,200) has been sold during 2003–04 for Rs. 40,000. Profit on sale has been transferred to Capital Reserve Account. ii. A piece of land has been sold for Rs. 70,000 during the year 2003–04. The Profit on sale of such land has also been transferred to Capital Reserve account.

Modified Date: Sat, Jul 03, 2010 11:47:06 AM

Output Date: Tue, Jul 06, 2010 11:46:58 AM

Rev II

Project: Management Accounting_Debarshi Bhattacharyya ACE Pro India Pvt. Ltd. File: X:\Pearson\Management Accounting_Debarshi Bhattacharyya\MAIN\M04\LAYOUT_M04\M04_DEBA_ISBN_EN_SE_C04_II.indd

264

MANAGEMENT ACCOUNTING

iii. Balance of Capital Reserve on 31 March 2004 consists of Profit on Sale of Plant and land alone. iv. During the year 2003–04, the bonus share of Rs. 1,00,000 has been issued out of general reserve. v. Debentures were redeemed at a premium of 10%. Premium on redemption has been transferred to Profit & Loss A/c. vi. Amount appropriated during 2003–04: For Proposed Dividend Rs. 70,000 For Taxation Rs. 40,000 vii. Depreciation provided during 2003–04: On Plant & Machinery Rs. 32,800 On Building Rs. 22,500 [B.Com. (Hons), Calcutta University—2005]

Solution Books of Sunshine Ltd Cash Flow Statement (under Indirect Method) for the year that ended on 31 March 2004 Rs. A. Cash Flows from Operating Activities: Balance of Profit & Loss A/c as on 31 March 2004 Balance of Profit & Loss A/c as on 31 March 2003 Net Profit for the year 2003–04 after appropriation Add: Appropriations: Transfer to General Reserve2 Proposed Dividend

Add: Add:

Net Profit before Appropriation Provision for Taxation Net Profit Before Tax Adjustment for Non-operating and Non-current items charged to Profit & Loss A/c: Depreciation on Land & Building5 Depreciation on Plant & Machinery4 Premium on Redemption of Debentures8

Rs.

Rs.

67,500 70,000 (2.500) 45,000 70,000 1,15,000 1,12,500 40,000 1,52,500

22,500 32,800 1,500 56,800 2,09,300

Less:

Add:

Adjustment for Non-operating and Non-current items Credited to Profit & Loss A/c Operating Profit before Working Capital changes Increase in Operating Current Liabilities: Outstanding Expenses (Rs. 12,700 – Rs. 10,200) Decrease in Operating Current Assets: Stock (Rs. 98,000 – Rs. 95,000) Prepaid Expenses (Rs. 15,700 – Rs. 8,900)

Nil 2,09,300 2,500 3,000 6,800 12,300 2,21,600

Less:

Increase in Operating Current Assets: Debtors (Rs. 1,18,200 – Rs. 1,02,000) Decrease in Operating Current Liabilities: Sundry Creditors (Rs. 73,000 – Rs. 61,100)

16,200 11,900 28,100 (Continued)

Modified Date: Sat, Jul 03, 2010 11:47:06 AM

Output Date: Tue, Jul 06, 2010 11:46:58 AM

Rev II

Project: Management Accounting_Debarshi Bhattacharyya ACE Pro India Pvt. Ltd. File: X:\Pearson\Management Accounting_Debarshi Bhattacharyya\MAIN\M04\LAYOUT_M04\M04_DEBA_ISBN_EN_SE_C04_II.indd

265

CASH FLOW ANALYSIS

Rs. Cash Generated from Operation Less: Income Tax paid9 Net Cash Flow from Operating Activities B. Cash Flows from Investing Activities: Sale Proceeds of Plant4 Sale Proceeds of Land3 Less:

Purchase of Land & Building3 Purchase of Plant & Machinery4

Rs. 1,93,500 30,000

Rs.

1,63,500 40,000 70,000 1,10,000 2,32,300 78,500 3,10,000

Net Cash Flow from Investing Activities C. Cash Flows from Financing Activities: Fresh issue of Shares for cash1 Premium Received on fresh issue of Shares6 Less:

Redemption of Debentures7 Premium paid on Redemption of Debentures8 Payment of Dividend10

(2,00,800) 1,00,000 10,000 1,10,000 15,000 1,500 50,000 66,500

Add:

Net Cash Flow from Financing Activities Net Increase in Cash & Cash Equivalents Cash & Cash Equivalents on 1 April 2003 Cash & Cash Equivalents on 31 March 2004

43,500 6,200 62,500 68,700

Working Notes 1. Dr.

To Balance c/f

Share Capital Account Rs. By Balance b/f By General Reserve – Bonus Shares issued 7,00,000 By Bank – Fresh issue of Shares for cash (Bal. fig.) 7,00,000

Cr. Rs. 5,00,000 1,00,000 1,00,000

General Reserve Account Rs. 1,00,000 By Balance b/f 45,000 By Profit & Loss A/c – Transfer during 2003–04 (Bal. fig.) 1,45,000

Cr. Rs. 1,00,000 45,000

7,00,000

2. Dr. To Share Capital – Bonus Shares To Balance c/f

1,45,000

3. Dr. To Balance b/f To Capital Reserve – Profit on sale of land To Bank – Purchase of Land & Building (Bal. fig.)

Land & Building Account Rs. 5,00,000 By Bank – Sale proceeds of land 10,200 By Profit & Loss A/c 2,32,300 – Depreciation By Balance c/f 7,42,500

Cr. Rs. 70,000 22,500 6,50,000 7,42,500 (Continued)

Modified Date: Sat, Jul 03, 2010 11:47:06 AM

Output Date: Tue, Jul 06, 2010 11:46:58 AM

Rev II

Project: Management Accounting_Debarshi Bhattacharyya ACE Pro India Pvt. Ltd. File: X:\Pearson\Management Accounting_Debarshi Bhattacharyya\MAIN\M04\LAYOUT_M04\M04_DEBA_ISBN_EN_SE_C04_II.indd

266

MANAGEMENT ACCOUNTING

4. Dr. To Balance b/f To Capital Reserve – Profit on Sale of Plant (Rs. 40,000 – Rs. 35,200) To Bank – Purchased of Land & Building (Bal. fig.)

Plant & Machinery Account Rs. 2,05,000 By Bank – Sale proceeds of Plant By Profit & Loss A/c – Depreciation 4,800 By Balance c/f 78,500 2,88,300

Cr. Rs. 40,000 32,800 2,15,500 2,88,300

5. Dr.

To Balance c/f

Capital Reserve Account Rs. By Balance b/f By Land & Building A/c – Profit on Sale of Land 15,000 By Plant & Machinery A/c – Profit on Sale of Plant 15,000

Cr. Rs. Nil 10,200 4,800 15,000

Securities Premium Account Rs. By Balance b/f 55,000 By Bank – Premium received on fresh issue of Shares (Bal. fig.) 55,000

Cr. Rs. 45,000

6. Dr.

To Balance c/f

10,000 55,000

7. Dr. To Bank – Redeemed (Bal. fig.) To Balance c/f

8% Debenture Account Rs. 15,000 By Balance b/f 85,000 1,00,000

Cr. Rs. 1,00,000 1,00,000

8. Dr.

Premium on Redemption of Debentures Account Rs. To Bank – Premium paid on By Profit & Loss A/c – Premium paid on redemption of Debentures 1,500 redemption adjusted (10% of Rs. 15,000) 1,500

Cr. Rs. 1,500 1,500

9. Dr. To Bank – Tax paid (Bal. fig.) To Balance c/f

Provision for Taxation Account Rs. 30,000 By Balance b/f 45,000 By Profit & Loss A/c – Provision made for 2003–04 75,000

Cr. Rs. 35,000 40,000 75,000

10. Dr. To Bank – Dividend paid (Bal. fig.) To Balance c/f

Modified Date: Sat, Jul 03, 2010 11:47:06 AM

Proposed Dividend Account Rs. 50,000 By Balance b/f 70,000 By Profit & Loss A/c – Dividend proposed for 2003–04 1,20,000

Output Date: Tue, Jul 06, 2010 11:46:58 AM

Cr. Rs. 50,000 70,000 1,20,000

Rev II

Project: Management Accounting_Debarshi Bhattacharyya ACE Pro India Pvt. Ltd. File: X:\Pearson\Management Accounting_Debarshi Bhattacharyya\MAIN\M04\LAYOUT_M04\M04_DEBA_ISBN_EN_SE_C04_II.indd

267

CASH FLOW ANALYSIS

Problem 33 KBC Co. Ltd Summarized Balance Sheets as on 31 December 2003 and 31 December 2004 Liabilities

31 March 2003 Rs.

Share Capital: Equity Shares Preference Shares Reserves & Surplus: General Reserve Profit & Loss A/c Loans Current Liabilities: Creditors Bank Overdraft Provision for Taxation Proposed Dividend

31 March 2004 Rs.

65,500 41,100

65,500 38,000

6,000 1,21,000 20,000

8,000 1,24,799 30,000

34,600 15,500 27,200 4,900 3,35,800

40,987 19,370 30,994 7,111 3,64,761

31 March 2003 Rs.

Assets Fixed Assets: Goodwill Land & Building Plant & Machinery Loose Tools Trade Investment Current Assets: Stock Short-term Investment Debtors Bills Receivable Cash

31 March 2004 Rs.

12,000 32,000 75,760 840 9,400

9,000 31,000 94,040 760 10,600

1,72,789 1,050 30,250 1,206 505 3,35,800

1,50,372 7,602 48,147 3,079 10,161 3,64,761

Summary of Profit & Loss A/c for the year that ended on 31 December 2004 Rs. Less:

Less: Less:

Trading Profit (prior to deduction of under-noted items) Depreciation on Land & Building Depreciation on Plant & Machinery Depreciation on Lost Tools Audit fee Directors’ Remuneration Others’ Remuneration Interest on Loan Net Profit before Taxation Provision for Tax Net Profit after Taxation Appropriations: Goodwill written off Transfer to General Reserve Proposed Preference Dividend Proposed Equity Dividend Net Profit for the year 2004 after appropriation Balance of Profit & Loss A/c as on 31 December 2003 Balance of Profit & Loss A/c as on 31 December 2004

1,000 15,120 360 1,200 9,400

Rs.

Rs. 64,684

16,480 1,500 10,600 1,600

3,000 2,000 3,333 8,772

30,180 34,504 13,600 20,904

17,105 3,799 1,21,000 1,24,799

Fixed Asset Schedule

Land & Building Plant & Machinery

Cost Rs. 35,000 1,01,560 1,36,560

Goodwill Loose tools

End of 2003 Depreciation Book Value Rs. Rs. 3,000 32,000 25,800 75,760 28,800 1,07,760 12,000 840 1,20,600

Cost Rs. 35,000 1,26,560 1,61,560

End of 2004 Depreciation Book Value Rs. Rs. 4,000 31,000 32,520 94,040 36,520 1,25,040 9,000 760 1,34,800

Note: During 2004, a Plant was purchased at a cost of Rs. 35,200. Some old Plant was sold for Rs. 2,000. Any gain/loss on sale is included in the trading Profit of Rs. 64,684. Prepare a Cash Flow Statement for the year that ended on 31 December 2004.

Modified Date: Sat, Jul 03, 2010 11:47:06 AM

Output Date: Tue, Jul 06, 2010 11:46:58 AM

Rev II

Project: Management Accounting_Debarshi Bhattacharyya ACE Pro India Pvt. Ltd. File: X:\Pearson\Management Accounting_Debarshi Bhattacharyya\MAIN\M04\LAYOUT_M04\M04_DEBA_ISBN_EN_SE_C04_II.indd

268

MANAGEMENT ACCOUNTING

Solution Books of KBC Ltd Cash Flow Statement (under Indirect Method) for the year that ended on 31 December 2004 Rs. A. Cash Flows from Operating Activities: Net Profit after approprition for the year Add: Appropriations: Goodwill written off Transfer to General Reserve Proposed Preference Dividend Proposed Equity Dividend Net Profit before Appropriation Add: Provision for Tax Net Profit before Tax Add: Adjustment for Non-operating and Non-current items charged to Profit & Loss A/c: Interest on Loan Depreciation on Land & Building Depreciation on Plant & Machinery Depreciation on lost tools Less:

Add:

Adjustment for Non-operating and Non-current items credited to Profit & Loss A/c: Profit on Sale of Plant3 Cash from operation before Working Capital Changes Increase in Operating Current Liabilities: Creditors (Rs. 40,987 – Rs. 34,600) Decrease in Operating Current Assets: Stock (Rs. 1,72,789 – Rs. 1,50,372)

Rs.

Rs.

3,799 3,000 2,000 3,333 8,772

17,105 20,904 13,600 34,504

1,600 1,000 15,120 360 18,080 52,584 200 52,384 6,387 22,417 28,804 81,188

Less:

Increase in Operating Current Assets: Debtors (Rs. 48,147 – Rs. 30,250) Bills Receivable (Rs. 3,079 – Rs. 1,206) Decrease in Operating Current Liabilities Cash Generated from Operation Less: Tax paid5 Net Cash Flow from Operating Activities B. Cash Flows from Investing Activities: Sale proceeds of Plant Less: Further Investment10 Purchase of Plant3 Purchase of Loose Tools9 Net Cash Flow from Investing Activities C. Cash Flows from Financing Activities: Loan Raised7 Less: Redemption of Preference Shares6 Payment of Preference & Equity Dividend8 Payment of Interest on Loan11

17,897 1,873 Nil

19,770 61,418 9,806 51,612 2,000

1,200 35,200 280

36,680 (34,680) 10,000

3,100 9,894 1,600 14,594 (Continued)

Modified Date: Sat, Jul 03, 2010 11:47:06 AM

Output Date: Tue, Jul 06, 2010 11:46:58 AM

Rev II

Project: Management Accounting_Debarshi Bhattacharyya ACE Pro India Pvt. Ltd. File: X:\Pearson\Management Accounting_Debarshi Bhattacharyya\MAIN\M04\LAYOUT_M04\M04_DEBA_ISBN_EN_SE_C04_II.indd

269

CASH FLOW ANALYSIS

Rs.

Add:

Less:

Less:

Net Cash Flow from Financing Activities Net Increase in Cash & Cash Equivalents Cash & Cash Equivalents as on 1 January 2004: Cash Short-term Investments Bank overdraft Cash & Cash Equivalents as on 31 December 2004: Cash Short-term Investments Bank Overdraft

Rs.

505 1,050 1,555 15,500

Rs. (4,594) 12,338

(13,945)

10,161 7,602 17,763 19,370 (1,607)

Working Notes 1. Dr.

Land & Building Account Rs. 35,000 By Balance c/f 35,000

To Balance b/f

Cr. Rs. 35,000 35,000

2. Dr.

To Balance c/f

Provision for Depreciation on Land & Building Account Rs. By Balance b/f 4,000 By Profit & Loss A/c – Depreciation for the year 4,000

Cr. Rs. 3,000 1,000 4,000

3. Dr.

Plant & Machinery (at Cost) Account Rs. To Balance b/f 1,01,560 To Bank – Purchased of Plant 35,200 By Provision for Depreciation on Plant & Machinery A/c4 – Accumulated Depreciation on Plant Sold To Profit & Loss A/c – Profit on Sale of Plant 200 By Bank – Sale proceeds of Plant (Bal. fig.) By Balance c/f 1,36,960

Cr. Rs. 8,400

2,000 1,26,560 1,36,960

4. Dr.

Provision for Depreciation on Plant & Machinery Account Rs. To Plant & Machinery – Accumulated 8,400 By Balance b/f Depreciation on Plant Sold To Balance c/f 32,520 By Profit & Loss A/c – Depreciation for the year 40,920

Cr. Rs. 25,800 15,120 40,920 (Continued)

Modified Date: Sat, Jul 03, 2010 11:47:06 AM

Output Date: Tue, Jul 06, 2010 11:46:58 AM

Rev II

Project: Management Accounting_Debarshi Bhattacharyya ACE Pro India Pvt. Ltd. File: X:\Pearson\Management Accounting_Debarshi Bhattacharyya\MAIN\M04\LAYOUT_M04\M04_DEBA_ISBN_EN_SE_C04_II.indd

270

MANAGEMENT ACCOUNTING

5. Dr.

Provision for Taxation Account Rs. 9,806 By Balance b/f 30,994 By Profit & Loss A/c – Provision made for this year 40,800

To Bank – Tax paid (Bal. fig.) To Balance c/f

Cr. Rs. 27,200 13,600 40,800

6. Dr.

Preference Share Capital Account Rs. 3,100 By Balance b/f 38,000 41,100

To Bank – Redeemed (Bal. fig.) To Balance c/f

Cr. Rs. 41,100 41,100

7. Dr.

Loan Account Rs. By Balance b/f 30, 000 By Bank – Loan Raised (Bal. fig.) 30,000

To Balance c/f

Cr. Rs. 20,000 10,000 30,000

8. Dr. To Bank – Preference & Equity Dividends paid during the year (Bal. fig.) To Balance c/f

Proposed Dividend Account Rs. 9,894 By Balance b/f 7,111

By Profit & Loss A/c – Proposed Equity & Preference Dividends for the year

Cr. Rs. 4,900 12,105

17,005

17,005

Loose Tool Account Rs. 840 By Profit & Loss A/c – Depreciation 280 By Balance c/f 1,120

Rs. 360 760 1,120

9. Dr. To Balance b/f To Bank – Purchase of Loose Tools (Bal. fig.)

Cr.

10. Dr. To Balance b/f To Bank – Further Investment (Bal. fig.)

Trade Investment Account Rs. 9,400 1,200 By Balance b/f 10,600

Cr.

10,600 10,600

Interest on Loan Account Rs. 1,600 By Profit & Loss A/c – Interest paid charged 1,600

Rs. 1,600 1,600

Rs.

11. Dr. To Bank – Interest paid

Modified Date: Sat, Jul 03, 2010 11:47:06 AM

Output Date: Tue, Jul 06, 2010 11:46:58 AM

Cr.

Rev II

Project: Management Accounting_Debarshi Bhattacharyya ACE Pro India Pvt. Ltd. File: X:\Pearson\Management Accounting_Debarshi Bhattacharyya\MAIN\M04\LAYOUT_M04\M04_DEBA_ISBN_EN_SE_C04_II.indd

271

CASH FLOW ANALYSIS

Problem 34 From the following information, prepare a Cash Flow Statement for the year that ended on 31 March 2005: Balance Sheet of Tawang Ltd as on 31 March 2004 and 31 March 2005 Liabilities Share Capital General Reserve Profit & Loss A/c Debentures Creditors for Machinery Creditors for Goods Creditors for Expenses: For Wages For Salaries For Trade Expenses Outstanding Interest on Debentures Provision for Taxation Proposed Dividend

31 March 2004 Rs. 1,00,000 60,000 30,000 50,000 – 40,000

31 March 2005 Rs. 2,00,000 23,000 60,000 80,000 82,000 30,000

7,000 2,000 1,000 10,000

13,000 5,000 2,000 15,000

30,000 20,000 3,50,000

40,000 25,000 5,75,000

Assets Goodwill Land & Building Plant & Machinery Investments Stock Debtors Marketable Securities Bank Cash Prepaid Trade Expenses Accrued Interest on Investment

31 March 2004 Rs. 20,000 70,000 40,000 40,000 42,000 40,000 30,000 55,000 4,000 6,000 3,000

31 March 2005 Rs. 15,000 1,23,000 1,30,000 70,000 65,000 25,000 45,000 72,000 15,000 8,000 7,000

3,50,000

5,75,000

Profit & Loss A/c of Tawang Ltd for the year t hat ended on 31 March 2005 Dr.

Cr. Rs.

To Opening Stock To Purchases: Cash Credit To Wages Add: Outstanding To Gross Profit c/d To Salaries Add: Outstanding To Insurance To Bad Debts To Trade Expenses Add: Outstanding Less: Prepaid To Depreciation To Discount Allowed To Interest on Debentures Add: Outstanding To Loss on Sale of Investment To Provision for Tax

28,000 2,10,000 52,000 6,000

63,000 3,000

Rs. 42,200

2,38,000

Rs. By Sales: Cash Credit By Closing Stock

97,000 4,33,000

58,000 2,57,000 5,95,000 66,000 8,000

By Gross Profit b/d By Discount Received By Interest on Investment

Rs.

5,30,000 65,000

5,95,000 2,57,000 4,000 8,000

6,000 40,000 1,000 41,000 2,000

10,000 5,000

Add: Accrued

4,000

12,000

39,000 32,.000 4,000 15,000 5,000 35,000 (Continued)

Modified Date: Sat, Jul 03, 2010 11:47:06 AM

Output Date: Tue, Jul 06, 2010 11:46:58 AM

Rev II

Project: Management Accounting_Debarshi Bhattacharyya ACE Pro India Pvt. Ltd. File: X:\Pearson\Management Accounting_Debarshi Bhattacharyya\MAIN\M04\LAYOUT_M04\M04_DEBA_ISBN_EN_SE_C04_II.indd

272

MANAGEMENT ACCOUNTING

Dr.

Cr. Rs.

To Net Profit c/d To Goodwill written off To Transfer to General Reserve To Proposed Dividend To Balance c/f

Rs. 63,000 2,73,000 5,000

Rs.

Rs. 2,73,000 63,000

By Net Profit b/d

73,000 15,000 36,000 63,000

63,000

Additional Information: i. During 2004–05, the Bonus Shares of Rs. 50,000 were issued out of General Reserve. ii. During 2004–05, the Debentures of Rs. 10,000 were redeemed at par. iii. Depreciation on Land and Building and Plant and Machinery for the year 2004–05 were amounted to Rs. 17,000 and Rs. 15,000, respectively iv. During 2003–04, the Investments of the Book Value of Rs. 20,000 were sold for Rs. 15,000.  Tutorial Note As the details of Profit & Loss A/c including purchases, sales, and so on, are given in the problem, it can be solved both under Direct Method as well as Indirect Method, as prescribed by the AS-3 for the Cash Flow Statement. That is why, it is solved under both the methods separately. But, for the examination purpose, either of the methods should be adopted according to the choice and efficiency of the students, if not specifically stated in the problem, as regards to the method to be followed.

Solution Method 1 (under Direct Method): Cash Flow Statement of Tawang Ltd for the year that ended on 31 March 2005 A. Cash Flows from Operating Activities: Cash Receipts from Customers: Cash Sales Collection from Credit Sales1 Less: Cash paid to Suppliers and Employees: Cash Purchases Payment for Credit Purchases2 Wages Paid3 Salaries Paid4 Trade Expenses Paid5 Insurance Paid Cash generated from operation Less: Tax Paid6 Net Cash Flows from Operating Activities B. Cash Flows from Investing Activities: Sale proceeds of Investment Interest Received on Investment14

Rs.

Rs.

97,000 4,38,000

5,35,000

Rs.

28,000 2,16,000 52,000 63,000 40,000 8,000 4,07,000 1,28,000 25,000 1,03,000 15,000 8,000

23,000 (Continued)

Modified Date: Sat, Jul 03, 2010 11:47:06 AM

Output Date: Tue, Jul 06, 2010 11:46:58 AM

Rev II

Project: Management Accounting_Debarshi Bhattacharyya ACE Pro India Pvt. Ltd. File: X:\Pearson\Management Accounting_Debarshi Bhattacharyya\MAIN\M04\LAYOUT_M04\M04_DEBA_ISBN_EN_SE_C04_II.indd

273

CASH FLOW ANALYSIS

Less:

Purchase of Land & Building7 Cash purchase of Plant & Machinery8 Further Investment9

Rs. 70,000 23,000 50,000

Rs.

Rs.

1,43,000 Net Cash Flow from Investing Activities C. Cash Flows from Financing Activities: Issue of Shares for Cash 10 Issue of Debentures for Cash11

(1,20,000) 50,000 40,000 90,000

Less:

Redemption of Debentures Payment of Interest on Debentures13 Payment of Dividend12

10,000 10,000 10,000 30,000

Add:

Net Cash Flow from Financing Activities Net Increase in Cash & Cash Equivalents Cash & Cash Equivalents as on 1 April 2004: Cash Bank Marketable Securities

60,000 43,000 4,000 55,000 30,000 89,000

Cash & Cash Equivalents as on 31 March 2005: Cash Bank Marketable Securities

15,000 72,000 45,000 1,32,000

Method 2 (under Indirect Method): Cash Flow Statement of Tawang Ltd for the year that ended on 31 March 2005 Rs. A. Cash Flows from Operating Activities: Net Profit for the year after appropriation Add: Appropriations: Goodwill written off Transfer to General Reserve Proposed Dividend

Add: Add:

Net Profit before appropriation Provision for Taxation Net Profit before Tax Adjustment for Non-operating and Non-current items debited to Profit & Loss A/c: Loss on Sale of Investment Interest on Debentures Depreciation

Rs.

Rs.

30,000 5,000 13,000 15,000 33,000 63,000 35,000 98,000

5,000 15,000 32,000 52,000

Less:

Add:

Adjustment for Non-operating and Non-current items credited to Profit & Loss A/c: Interest on Investment Operating Profit before Working Capital changes Increase in Operating Current Liabilities: Creditors for Wages Creditors for Salaries

12,000 1,38,000 6,000 3,000 (Continued)

Modified Date: Sat, Jul 03, 2010 11:47:06 AM

Output Date: Tue, Jul 06, 2010 11:46:58 AM

Rev II

Project: Management Accounting_Debarshi Bhattacharyya ACE Pro India Pvt. Ltd. File: X:\Pearson\Management Accounting_Debarshi Bhattacharyya\MAIN\M04\LAYOUT_M04\M04_DEBA_ISBN_EN_SE_C04_II.indd

274

MANAGEMENT ACCOUNTING

Creditors for trade Expenses Decrease in Operating Current Assets: Debtors Decrease in Operating Current Liabilities Creditors for goods Increase in Operating Current Assets: Stock Prepaid trade Expenses Cash generated from operation Less: Income Tax paid6 Net Cash Flow from Operating Activities B. Cash Flows from Investing Activities: Sales proceeds of Investments Interest Received on Investments14

Rs. 1,000

Rs.

15,000

25,000 1,63,000

Rs.

Less:

10,000 23,000 2,000

35,000 1,28,000 25,000 1,03,000

15,000 8,000 23,000

Less:

Purchase of Land & Building7 Cash purchase of Plant & Machinery8 Further Investment9

70,000 23,000 50,000 1,43,000

Net Cash Flow from Investing Activities C. Cash Flows from Financing Activities: Issue of Shares for Cash10 Issue of Debentures for Cash11

(1,20,000) 50,000 40,000 90,000

Less:

Redemption of Debentures Payment of interest on Debentures13 Payment of Dividend

10,000 10,000 10,000 30,000

Add:

Net Cash Flow from Financing Activities Net Increase in Cash & Cash Equivalents Cash & Cash Equivalents as on 1 April 2004: Cash Bank Marketable Securities

60,000 43,000 4,000 55,000 30,000 89,000

Cash & Cash Equivalents as on 31 March 2005: Cash Bank Marketable Securities

15,000 72,000 45,000 1,32,000

Working Notes 1. Dr. To Balance b/f To Sales – Credit

Debtors Account Rs. 40,000 By Bank – Collection from Debtors (Bal. fig.) 4,33,000 By Bad Debt By Discount Allowed By Balance c/f 4,73,000

Cr. Rs. 4,38,000 6,000 4,000 25,000 4,73,000 (Continued)

Modified Date: Sat, Jul 03, 2010 11:47:06 AM

Output Date: Tue, Jul 06, 2010 11:46:58 AM

Rev II

Project: Management Accounting_Debarshi Bhattacharyya ACE Pro India Pvt. Ltd. File: X:\Pearson\Management Accounting_Debarshi Bhattacharyya\MAIN\M04\LAYOUT_M04\M04_DEBA_ISBN_EN_SE_C04_II.indd

CASH FLOW ANALYSIS

275

2. Dr. To Bank – Payment to Creditors (Bal. fig.) To Discount Received To Balance c/f

Creditors for Goods Account Rs. 2,16,000 By Balance b/f 4,000 By Purchases – Credit 30,000 2,50,000

Cr. Rs. 40,000 2,10,000 2,50,000

3. Dr. To Bank – Wages paid during the year (Bal. fig.) To Balance c/f

Wages Account Rs. 52,000 By Balance b/f 13,000 By Trading A/c – Wages for the year 65,000

Cr. Rs. 7,000 58,000 65,000

Salaries Account Rs. 63,000 By Balance b/f 5,000 By Profit & Loss A/c – Salaries for the year 68,000

Cr. Rs. 2,000 66,000

4. Dr. To Bank – Salaries Paid during the year (Bal. fig.) To Balance c/f

68,000

5. Dr. To Balance b/f To Bank – Trade Expenses paid during the year (Bal. fig.) To Balance c/f

Trade Expense Account Rs. 6,000 By Balance b/f By Profit & Loss A/c – Trade Expenses 40,000 for the year 2,000 By Balance c/f 48,000

Cr. Rs. 1,000 39,000 8,000 48,000

6. Dr. To Bank – Tax paid (Bal. fig.) To Balance c/f

Provision for Taxation Account Rs. 25,000 By Balance b/f 40,000 By Profit & Loss A/c – Provision made for this year 65,000

Cr. Rs. 30,000 35,000 65,000

7. Dr.

Land & Building Account Rs. 70,000 By Profit & Loss A/c – Depreciation 70,000 By Balance c/f 1,40,000

Cr. Rs. 17,000 1,23,000 1,40,000

Plant & Machinery Account Rs. To Balance b/f 40,000 By Profit & Loss A/c – Depreciation To Creditors for Machinery – Credit purchase 82,000 By Balance c/f To Bank – Cash purchase (Bal. fig.) 23,000 1,45,000

Cr. Rs. 15,000 1,30,000

To Balance b/f To Bank – Purchases (Bal. fig.)

8. Dr.

1,45,000 (Continued)

Modified Date: Sat, Jul 03, 2010 11:47:06 AM

Output Date: Tue, Jul 06, 2010 11:46:58 AM

Rev II

Project: Management Accounting_Debarshi Bhattacharyya ACE Pro India Pvt. Ltd. File: X:\Pearson\Management Accounting_Debarshi Bhattacharyya\MAIN\M04\LAYOUT_M04\M04_DEBA_ISBN_EN_SE_C04_II.indd

276

MANAGEMENT ACCOUNTING

9. Dr. To Balance c/f To Bank – Further Investment (Bal. fig.)

Investment Account Rs. 40,000 By Bank – Sale proceeds 50,000 By Profit & Loss A/c – Loss on Sale By Balance c/f 90,000

Cr. Rs. 15,000 5,000 70,000 90,000

10. Dr.

To Balance c/f

Share Capital Account Rs. By Balance b/f By General Reserve – Bonus Shares issued 2,00,000 By Bank – Fresh issue of Shares for cash (Bal. fig.) 2,00,000

Cr. Rs. 1,00,000 50,000

Debentures’ Account Rs. 10,000 By Balance b/f 8,00,000 By Bank – Fresh issue (Bal. fig.) 90,000

Cr. Rs. 50,000 40,000 90,000

50,000 2,00,000

11. Dr. To Bank – Redeemed To Balance c/f

12. Dr. To Bank – Dividend paid (Bal. fig.) To Balance c/f

Proposed Dividend Account Rs. 10,000 By Balance b/f 25,000 By Profit & Loss A/c – Dividend proposed for the year 35,000

Cr. Rs. 20,000 15,000 35,000

13. Dr. To Bank – Interest paid (Bal. fig.) To Balance c/f

Interest on Debentures Account Rs. 10,000 By Balance b/f 15,000 By Profit & Loss A/c – Interest Payable for the year 25,000

Cr. Rs. 10,000 15,000 25,000

14. Dr.

Accrued Interest on Investment Account Rs. To Balance b/f 3,000 By Bank – Interest Received (Bal. fig.) To Profit & Loss A/c – Interest Receivable 12,000 By Balance c/f for the year 15,000

Cr. Rs. 8,000 7,000 15,000

Note: Under Indirect Method, Working Notes – No. 1, 2, 3, 4 and 5 need not be prepared.

Modified Date: Sat, Jul 03, 2010 11:47:06 AM

Output Date: Tue, Jul 06, 2010 11:46:58 AM

Rev II

Project: Management Accounting_Debarshi Bhattacharyya ACE Pro India Pvt. Ltd. File: X:\Pearson\Management Accounting_Debarshi Bhattacharyya\MAIN\M04\LAYOUT_M04\M04_DEBA_ISBN_EN_SE_C04_II.indd

277

CASH FLOW ANALYSIS

Problem 35 Following were the Balance Sheets of Macao Ltd, as on 31 March 2007 and 31 March 2008: Liabilities Equity Share Capital Preference Share Capital Profit & Loss A/c General Reserve Securities Premium Capital Reserve Sundry Creditors Proposed Dividend Provision for Taxation Bills Payable

As on 31 March 2007 Rs. 3,00,000 1,50,000 30,000 40,000 20,000 – 55,000 42,000 40,000 20,000 6,97,000

As on 31 March 2008 Rs. 3,50,000 1,00,000 73,000 75,000 25,000 20,000 83,000 50,000 50,000 16,000 8,42,000

Assets Goodwill Land & Building Plant & Machinery Investment Stock Debtors Bills Receivable Cash & Bank Preliminary Expenses

As on 31 March 2007 Rs. 1,00,000 2,00,000 80,000 20,000 77,000 1,40,000 20,000 25,000 35,000

As on 31 March 2008 Rs. 85,000 1,70,000 2,00,000 35,000 1,00,000 1,70,000 30,000 22,000 30,000

6,97,000

8,42,000

Additional Information: i. One piece of land was sold at a profit which was credited to Capital Reserve. ii. One machine, having a Book Value of Rs. 18,000, was sold for Rs. 15,000. iii. Depreciation charged on Plant and Machinery for the year 2007–08 was Rs. 16,000. iv. Tax Paid for the year 2006–07 amounted to Rs. 36,000. v. Dividend on Investment of Rs. 4,000 was received during the year, which included a pre-acquisition Dividend of Rs. 1,000. vi. Out of the Proposed Dividend for the year 2006–07, Rs. 35,000 was approved by the Share Holders and paid in 2007–08, along with an Interim Dividend for the year of Rs. 12,000. Prepare a Cash Flow Statement for the year that ended on 31 March 2008. Solution Cash Flow Statement (under Indirect Method) of Macao Ltd for the year that ended on 31 March 2008 Rs. A. Cash Flows from Operating Activities: Net Profit (after appropriation) for the year8 Add: Adjustment for Non-operating & Non-current items debited to Profit & Loss A/c: Proposed Dividend6 Transfer to General Reserve8 Interim Dividend7 Goodwill written off8 Preliminary Expenses written off8 Loss on Sale of Machinery2 Depreciation on Plant & Machinery2 Provision for Tax5 Less:

Adjustment for Non-operating & Non-current items credited to Profit & Loss A/c: Post-acquisition Dividend on Investment4 Operating Profit before Working Capital changes

Rs.

Rs.

43,000

43,000 35,000 12,000 15,000 5,000 3,000 16,000 46,000

1,75,000 2,18,000

3,000 2,15,000 (Continued)

Modified Date: Sat, Jul 03, 2010 11:47:06 AM

Output Date: Tue, Jul 06, 2010 11:46:58 AM

Rev II

Project: Management Accounting_Debarshi Bhattacharyya ACE Pro India Pvt. Ltd. File: X:\Pearson\Management Accounting_Debarshi Bhattacharyya\MAIN\M04\LAYOUT_M04\M04_DEBA_ISBN_EN_SE_C04_II.indd

278

MANAGEMENT ACCOUNTING

Rs. Add:

Increase in Operating Current Liabilities: Sundry Creditors Decrease in Operating Current Assets

Decrease in Operating Current Liabilities: Bills Payable Increase in Operating Current Assets: Inventory Debtors Bills Receivable Cash Generated from Operation Less: Income Tax paid5 Net Cash Flow from Operating Activities B. Cash Flows from Investing Activities: Proceeds from Sale of land1 Proceeds from Sale of Machinery2 Dividend Received from Investment3&4

28,000 Nil

Rs.

Rs.

28,000 2,43,000

Less:

4,000 23,000 30,000 10,000

67,000 1,76,000 36,000 1,40,000

50,000 15,000 4,000 69,000

Less:

Purchase of new Machinery2 New Investment Made3

1,54,000 16,000 1,70,000

Net Cash Flow from Investing Activities C. Cash Flows from Financing Activities: Proceeds Received from New Issue of Equity Shares including premium8 Less: Redemption of Preference Shares8 Payment of Proposed Dividend6 Payment of Interim Dividend7 Net Cash Flow from Financing Activities Net Increase in Cash & Cash Equivalents Add: Cash & Cash Equivalents at the beginning of the year Cash & Cash Equivalents at the end of the year

(1,01,000)

55,000 50,000 35,000 12,000

97,000 (42,000) (3,000) 25,000 22,000

Working Notes 1. Dr. To Balance b/f To Capital Reserve – Profit on Sale of land

Land & Building Account Rs. 2,00,000 By Bank – Sale proceeds 20,000 By Balance c/f 2,20,000

Cr. Rs. 50,000 1,70,000 2,20,000

2. Dr. To Balance b/f To Bank – Cost of new Machinery

Modified Date: Sat, Jul 03, 2010 11:47:06 AM

Plant & Machinery Account Rs. 80,000 By Bank – Sale proceeds 1,54,000 By Profit & Loss A/c – Loss on Sale of Machinery (Rs. 18,000 – Rs. 15,000) By Profit & Loss A/c – Depreciation for the year By Balance c/f 2,34,000

Output Date: Tue, Jul 06, 2010 11:46:58 AM

Cr. Rs. 15,000 3,000 16,000 2,00,000 2,34,000

Rev II

Project: Management Accounting_Debarshi Bhattacharyya ACE Pro India Pvt. Ltd. File: X:\Pearson\Management Accounting_Debarshi Bhattacharyya\MAIN\M04\LAYOUT_M04\M04_DEBA_ISBN_EN_SE_C04_II.indd

279

CASH FLOW ANALYSIS

3. Dr.

Investment Account Rs. 20,000 By Bank – Pre-acquisition Dividend Received & credited 16,000 By Balance c/f 36,000

To Balance b/f To Bank – New Investment made

Cr. Rs. 1,000 35,000 36,000

4. Dr.

Dividend (Post-Acquisition) on Investment Account Rs. To Profit & Loss A/c – Dividend Received & 3,000 By Bank – Post-acquisition Dividend credited Received 3,000

Cr. Rs. 3,000 3,000

5. Dr.

Provision for Tax Account Rs. 36,000 By Balance b/f

To Bank – Tax for the year 2006–07 paid in 2007–08 To Balance c/f

50,000

Cr. Rs. 40,000

By Profit & Loss A/c – Provision made in 2007–08

86,000

46,000 86,000

6. Dr. To Bank – Dividend for the year 2006–07 paid in 2007–08 To Balance c/f

Proposed Dividend Account Rs. 35,000 By Balance b/f

50,000 85,000

Cr. Rs. 42,000

By P&L Appropriation A/c – Dividend proposed in 2007–08

43,000 85,000

7. Dr.

Interim Dividend Account Rs. 12,000 By Profit & Loss appropriation A/c – Interim Dividend adjusted 12,000

To Bank – Interim Dividend paid

8.

Cr. Rs. 12,000 2,000

Analysis of other Non-current Assets and Liabilities Assets/Liabilities

Goodwill Preliminary Expenses Equity Share Capital Preference Share Capital Securities premium Capital Reserve General Reserve Profit & Loss A/c

Opening Balance Rs. 1,00,000 35,000 3,00,000 1,50,000 20,000 – 40,000 30,000

Modified Date: Sat, Jul 03, 2010 11:47:06 AM

Closing Balance Rs. 85,000 30,000 3,50,000 1,00,000 25,000 20,000 75,000 73,000

Increase/ Decrease Rs. (−)15,000 (−)5,000 (+)50,000 (−)50,000 (+)5,000 (+)20,000 (+)35,000 (+)43,000

Analysis Written off against Profit & Loss A/c Written off against Profit & Loss A/c New issue of Equity Shares Redemption of Preference Shares Premium Received on New Issue of Equity Shares Profit on Sale of land Transfer from Profit & Loss A/c Net Profit (after appropriation) for the year

Output Date: Tue, Jul 06, 2010 11:46:58 AM

Rev II

Project: Management Accounting_Debarshi Bhattacharyya ACE Pro India Pvt. Ltd. File: X:\Pearson\Management Accounting_Debarshi Bhattacharyya\MAIN\M04\LAYOUT_M04\M04_DEBA_ISBN_EN_SE_C04_II.indd

280

MANAGEMENT ACCOUNTING

Problem 36 The Balance Sheet of New Light Ltd for the year that ended on 31 March 2001 and 31 March 2002 are as follows: Liabilities Equity Share Capital 10% Preference Share Capital Capital Reserve General Reserve Profit & Loss A/c 9% Debentures Current Liabilities Proposed Dividend Provision for Tax Unpaid Dividend

31 March 2001 Rs. 12,00,000 4,00,000 – 6,80,000 2,40,000 4,00,000 4,80,000 1,20,000 3,60,000 – 38,80,000

31 March 2002 Rs. 16,00,000 2,80,000 40,000 8,00,000 3,00,000 2,80,000 5,20,000 1,44,000 3,40,000 16,000 43,20,000

Assets

Less:

Fixed Assets Depreciation Investment Cash Other Current Assets Preliminary Expenses

31 March 2001 Rs. 32,00,000 9,20,000 22,80,000 4,00,000 10,000 11,10,000 80,000

31 March 2002 Rs. 38,00,000 11,60,000 26,40,000 3,20,000 10,000 13,10,000 40,000

38,80,000

43,20,000

Additional Information: i. The company sold one Fixed Asset for Rs. 1,00,000, the cost of which was Rs. 2,00,000 and the depreciation provided on it was Rs. 80,000. ii. The company also decided to write off another Fixed Asset costing Rs. 56,000 on which a depreciation amounting to Rs. 40,000 has been provided. iii. Depreciation on Fixed Assets provided was Rs. 3,60,000. iv. Company sold some Investment at a Profit of Rs. 40,000, which was credited to the Capital Reserve. v. Debentures and Preference Shares were redeemed at a 5% premium. vi. Company decided to value the stock at cost, whereas previously the practice was to value the stock at cost less 10%. The stock according to books on 31 March 2001 was Rs. 2,16,000. The stock on 31 March 2002 was correctly valued at Rs. 3,00,000. Prepare a Cash Flow Statement as per AS-3 by Indirect Method. [C.A. (Inter)—November 2003]

Solution Cash Flow Statement (under Indirect Method) of New Light Ltd for the year that ended on 31 March 2002 Rs. A. Cash Flows from Operating Activities: Net Profit (after appropriation) for the year Add: Adjustment for Non-operating & Non-current items debited to Profit & Loss A/c: Proposed Dividend4 Transfer to General Reserve9 Premium on Redemption of Preference Shares7 Fixed asset written off 1 Preliminary Expenses written off 9 Loss on Sale of Fixed Asset1

Rs.

Rs.

60,000

1,44,000 1,20,000 6,000 16,000 40,000 20,000 (Continued)

Modified Date: Sat, Jul 03, 2010 11:47:06 AM

Output Date: Tue, Jul 06, 2010 11:46:58 AM

Rev II

Project: Management Accounting_Debarshi Bhattacharyya ACE Pro India Pvt. Ltd. File: X:\Pearson\Management Accounting_Debarshi Bhattacharyya\MAIN\M04\LAYOUT_M04\M04_DEBA_ISBN_EN_SE_C04_II.indd

281

CASH FLOW ANALYSIS

Premium on Redemption of Debentures8 Depreciation on Fixed Assets2 Provision for Tax6 Less:

Rs. 6,000 3,60,000 3,40,000

Inflated Profit due to undervaluation of Opening Stock (10/90 × Rs. 2,16,000)

Operating Profit before Working Capital changes Increase in Operating Current Assets: Stock [Rs. 3,00,00 – (Rs. 2,16,000 + Rs. 24,000)] Other Current Assets [(13,10,000 – 3,00,000) – (11,10,000 – 2,16,000)] Cash generated from operation Less: Income Tax paid6 Net Cash Flow from Operating Activities B. Cash Flows from Investing Activities: Proceeds from Sale of Fixed Assets1 Proceeds from Sale of Investment3

Rs.

Rs.

10,52,000 11,12,000 24,000

10,88,000

Less:

Purchase of Fixed Assets1 Net Cash Flow from Investing Activities C. Cash Flows from Financing Activities: Proceeds Received from New Issue of Equity Shares9 Less: Redemption of Preference Shares9 &7 Redemption of Debentures9 &8 Payment of Dividend4 Net Cash Flow from Financing Activities Net Increase in Cash & Cash Equivalents Add: Cash & Cash Equivalents at the beginning of the year Cash & Cash Equivalents at the end of the year

60,000 1,16,000

1,76,000 9,52,000 3,60,000 5,92,000

1,00,000 1,20,000 2,20,000 8,56,000

Less:

(6,36,000) 4,00,000 1,26,000 1,26,000 1,04,000

3,56,000 44,000 Nil 10,000 10,000

Working Notes 1. Dr. To Balance b/f To Bank – Cost of New Fixed Assets (Bal. fig.)

Fixed Asset (at Cost) Account Rs. 32,00,000 By Provision for Depreciation A/c – Accumulated Depreciation on Fixed Assets sold 8,56,000 By Bank – Sale proceeds By Profit & Loss A/c – Loss on Sale of Fixed Assets [(2,00,000 – 80,000) – 1,00,000] By Provision for Depreciation A/c – Accumulated Depreciation on Fixed Asset written off By Profit & Loss A/c – WDV of the Fixed Asset written off By Balance c/f 40,56,000

Cr. Rs. 80,000 1,00,000 20,000

40,000 16,000 38,00,000 40,56,000 (Continued )

Modified Date: Sat, Jul 03, 2010 11:47:06 AM

Output Date: Tue, Jul 06, 2010 11:46:58 AM

Rev II

Project: Management Accounting_Debarshi Bhattacharyya ACE Pro India Pvt. Ltd. File: X:\Pearson\Management Accounting_Debarshi Bhattacharyya\MAIN\M04\LAYOUT_M04\M04_DEBA_ISBN_EN_SE_C04_II.indd

282

MANAGEMENT ACCOUNTING

2. Dr.

Provision for Depreciation Account Rs. 80,000 By Balance b/f

To Fixed Asset A/c – Accumulated Depreciation on Fixed Assets sold To Fixed Asset A/c – Accumulated Depreciation on Fixed Asset written off To Balance c/f

40,000

By Profit & Loss A/c – Depreciation for the year

Cr. Rs. 9,20,000 3,60,000

11,60,000 12,80,000

12,80,000

Investment Account Rs. 4,00,000 By Bank – Sale proceeds of Investment 40,000 By Balance c/f

Cr. Rs. 1,20,000 3,20,000

4,40,000

4,40,000

3. Dr. To Balance b/f To Capital Reserve A/c – Profit on Sale of Investment

4. Dr.

Proposed Dividend Account Rs. To Bank – Dividend paid (1,20,000 – 16,000) 1,04,000 By Balance b/f To Unpaid Dividend A/c 16,000 By Profit & Loss appropriation A/c – Dividend not claimed – Dividend proposed in 2001–02 (Bal. fig.) To Balance c/f 1,44,000 2,64,000

Cr. Rs. 1,20,000 1,44,000

2,64,000

5. Dr.

To Balance c/f

Unpaid Dividend Account Rs. By Balance b/f 16,000 By Proposed Dividend A/c 16,000

Cr. Rs. Nil 16,000 16,000

Provision for Tax Account Rs. 3,60,000 By Balance b/f

Cr. Rs. 3,60,000

6. Dr. To Bank – Tax for the year 2000–01 paid in 2001–02 To Balance c/f

3,40,000

By Profit & Loss A/c – Provision made in 2001–02

7,00,000

3,40,000 7,00,000

7. Dr.

Premium on Redemption of Preference Share Account Rs. To Bank – Premium paid on redemption 6,000 By Profit & Loss A/c – Premium paid (5% on Rs. 1,20,000) adjusted 6,000

Modified Date: Sat, Jul 03, 2010 11:47:06 AM

Output Date: Tue, Jul 06, 2010 11:46:58 AM

Cr. Rs. 6,000 6,000

Rev II

Project: Management Accounting_Debarshi Bhattacharyya ACE Pro India Pvt. Ltd. File: X:\Pearson\Management Accounting_Debarshi Bhattacharyya\MAIN\M04\LAYOUT_M04\M04_DEBA_ISBN_EN_SE_C04_II.indd

CASH FLOW ANALYSIS

283

8. Dr.

Premium on Redemption of Debentures’ Account Rs. To Bank – Premium paid on redemption 6,000 By Profit & Loss A/c – Premium paid (5% on Rs. 1,20,000) adjusted 6,000

Cr. Rs. 6,000 6,000

9. Analysis of other Non-current Assets and liabilities Assets/Liabilities Preliminary Expenses Equity Share Capital Preference Share Capital Debentures Capital Reserve General Reserve Profit & Loss A/c

Opening Balance Rs. 80,000 12,00,000 4,00,000 4,00,000 – 6,80,000 2,40,000

Closing Balance Rs. 40,000 16,00,000 2,80,000 2,80,000 40,000 8,00,000 3,00,000

Increase/ Decrease Rs. (−)40,000 (+)4,00,000 (−)1,20,000 (−)1,20,000 (+)40,000 (+)1,20,000 (+)60,000

Analysis Written off against Profit & Loss A/c New issue of Equity Shares Redemption of Preference Shares Redemption of Debentures Profit on Sale of Investment Transfer from Profit & Loss A/c Net Profit (after appropriation) for the year

Problem 37 Balance Sheets of Honululu Ltd as on 31 March 2007 and 31 March 2008 were as follows: Liabilities Equity Share Capital (Rs. 10) 8% Preference Shares of Rs. 100 each, Rs. 75 per share called up & paid up Securities premium Capital Redemption Reserve General Reserve Profit & Loss A/c Sundry Creditors Proposed Dividend Provision for Taxation Bills Payable Unclaimed Dividend

As on 31 March 2007 Rs. 3,00,000 1,12,500

As on 31 March 2008 Rs. 4,00,000 –

20,000 – 30,000 60,000 12,500 20,000 20,000 5,000 – 5,80,000

7,000 1,10,000 50,000 40,000 50,000 30,000 30,000 – 3,000 7,20,000

Land & Building Plant & Machinery

As on 31 March 2007 Rs. 2,00,000 80,000

As on 31 March 2008 Rs. 1,70,000 2,00,000

Furniture Investment Inventory Sundry Debtors Cash & Bank Preliminary Expenses

80,000 20,000 77,000 1,40,000 25,000 35,000

2,00,000 35,000 1,00,000 1,70,000 22,000 30,000

5,80,000

7,20,000

Assets

Additional Information: i. During the year 2007–08, an old machine, whose book value was Rs. 60,000, was sold at a loss of Rs. 8,000, and it was replaced by a new machine costing Rs. 1,60,000. ii. Depreciation on furniture provided for the year 2007–08 amounted to Rs. 15,000. A part of the furniture was sold at a Profit of Rs. 3,000 and the cost of new furniture acquired during the year was Rs. 78,000. iii. A Preference Share final call @ Rs. 25 per share was made before redeeming the preference shares at a premium of 10%. The Securities Premium Account was utilized to provide the premium paid and the redemption was made partly out of profit and partly out of new issue of Equity Shares at 5% premium.

Modified Date: Sat, Jul 03, 2010 11:47:06 AM

Output Date: Tue, Jul 06, 2010 11:46:58 AM

Rev II

Project: Management Accounting_Debarshi Bhattacharyya ACE Pro India Pvt. Ltd. File: X:\Pearson\Management Accounting_Debarshi Bhattacharyya\MAIN\M04\LAYOUT_M04\M04_DEBA_ISBN_EN_SE_C04_II.indd

284

MANAGEMENT ACCOUNTING

iv. A bonus Dividend was declared during 2007–08 to the old Equity Shareholders, @ one Equity Share at par for every five Equity Shares held, out of general reserve. v. Before Redemption, a Preference Dividend for the year 2007–08 was paid by the company. Prepare a Cash Flow Statement for the year that ended on 31 March 2008.

Solution Cash Flow Statement (under Indirect Method) of Honululu Ltd for the year that ended on 31 March 2008 Rs. A. Cash Flows from Operating Activities: Net Profit (after appropriation) for the year: Balance of Profit & Loss A/c as on 31 March 2008 Less: Balance of Profit & Loss A/c as on 31 March 2007 Add:

Less:

Add:

Adjustment for Non-operating & Non-current items debited to Profit & Loss A/c: Transfer to Capital Redemption Reserve5 Transfer to General Reserve6 Preference-Dividend Declared7 Proposed-Dividend Declared8 Preliminary Expenses written off13 Loss on Sale of Machinery11 Depreciation on Plant & Machinery11 Depreciation on Furniture12 Provision for Tax10 Adjustment for Non-operating & Non-current items credited to Profit & Loss A/c: Profit on Sale of Furniture12 Operating Profit before Working Capital changes Increase in Operating Current Liabilities: Sundry Creditors Decrease in Operating Current Assets: Inventory

Decrease in Operating Current Liabilities: Bills Payable Increase in Operating Current Assets: Sundry Debtors Cash Generated from Operation Less: Income Tax paid10 Net Cash Flow from Operating Activities B. Cash Flows from Investing Activities: Proceeds from Sale of Furniture12 Proceeds from Sale of Machinery11

Rs.

Rs.

40,000 60,000 (20,000)

1,10,000 80,000 9,000 30,000 10,000 8,000 20,000 15,000 30,000

3,12,000 2,92,000

3,000 2,89,000 37,500 12,000

49,500 3,38,500

Less:

5,000 12,000

17,000 3,21,500 20,000 3,01,500

26,000 52,000 78,000

Less:

Purchase of New Machinery11 New Investment made13 Purchase of New Furniture12 Addition to Building13

1,60,000 10,000 78,000 10,000 2,58,000

Net Cash Flow from Investing Activities

(1,80,000) (Continued)

Modified Date: Sat, Jul 03, 2010 11:47:06 AM

Output Date: Tue, Jul 06, 2010 11:46:58 AM

Rev II

Project: Management Accounting_Debarshi Bhattacharyya ACE Pro India Pvt. Ltd. File: X:\Pearson\Management Accounting_Debarshi Bhattacharyya\MAIN\M04\LAYOUT_M04\M04_DEBA_ISBN_EN_SE_C04_II.indd

285

CASH FLOW ANALYSIS

Rs. C. Cash Flows from Financing Activities: Proceeds Received from New Issue of Equity Shares, including premium1 & 4 (40,000 + 2,000) Receipt of final Preference Share call2

Rs.

Rs.

42,000 37,500 79,500

Less:

Redemption of Preference Shares, including premium2 & 3 (1,50,000 + 15,000) Payment of Preference Dividend7 Payment of proposed Dividend8

1,65,000 9,000 17,000 1,91,000

Add:

Net Cash Flow from Financing Activities Net Increase in Cash & Cash Equivalents Cash & Cash Equivalents at the beginning of the year Cash & Cash Equivalents at the end of the year

(1,11,000) 10,000 20,000 30,000

Working Notes 1. Dr.

Equity Share Capital Account Rs. By Balance b/f By General Reserve – Bonus Shares issued (1/5 × 30,000 = 6,000 Shares of Rs. 10) 4,00,000 By Bank – New issue of Shares for cash 4,00,000 (Bal. fig.)

To Balance c/f

Cr. Rs. 3,00,000 60,000 40,000 4,00,000

2. Dr.

8% Preference Share Capital Account Rs. To Bank – Preference Shares Redeemed 1,50,000 By Balance b/f By Bank – Final call on Shares Received (1,500 × Rs. 25) 1,50,000

Cr. Rs. 1,12,000 37,500 1,50,000

Note: No. of Preference Shares = Rs. 1,12,500 ÷ Rs. 75 = 1,500 shares.

3. Dr.

Premium on Redemption on Preference Share Account Rs. To Bank – Premium paid on redemption 15,000 By Securities premium A/c – Premium paid on redemption adjusted 15,000

Cr. Rs. 15,000 15,000

4. Dr. To Premium on redemption on Preferenceshare A/c – Premium-paid adjusted To Balance c/f

Securities Premium Account Rs. By Balance b/f 15,000 7,000 By Bank – Premium Received on New Issue of Equity Shares (5% on Rs. 40,000) 22,000

Cr. Rs. 20,000

2,000 22,000 (Continued)

Modified Date: Sat, Jul 03, 2010 11:47:06 AM

Output Date: Tue, Jul 06, 2010 11:46:58 AM

Rev II

Project: Management Accounting_Debarshi Bhattacharyya ACE Pro India Pvt. Ltd. File: X:\Pearson\Management Accounting_Debarshi Bhattacharyya\MAIN\M04\LAYOUT_M04\M04_DEBA_ISBN_EN_SE_C04_II.indd

286

MANAGEMENT ACCOUNTING

5. Dr.

Capital Redemption Reserve Account Rs. 1,10,000 By Balance b/f By Profit & Loss A/c – Difference between face value of Preference Shares redeemed & face value of new shares issued for cash transferred 1,10,000

To Balance c/f

Cr. Rs. – 1,10,000

1,10,000

6. Dr.

General Reserve Account Rs. By Balance b/f 60,000 50,000 By Profit & Loss A/c – Transfer during the year (Bal. fig.) 1,10,000

To Equity share capital A/c – Bonus Dividend declared To Balance c/f

Cr. Rs. 30,000 80,000 1,10,000

7. Dr. To Bank – Preference Dividend paid (8% on Rs. 1,12,500)

Preference Dividend Account Rs. 9,000 By Profit & Loss A/c – Preference Dividend Declared 9,000

Cr. Rs. 9,000 9,000

8. Dr. To Bank – Dividend for 2006–07 paid (20,000 – 3,000) To Unpaid Dividend A/c9 – Dividend not claimed To Balance c/f

Proposed Dividend Account Rs. 17,000 By Balance b/f 3,000

By Profit & Loss appropriation A/c – Dividend proposed in 2007–08

30,000 50,000

Cr. Rs. 20,000 30,000

50,000

9. Dr.

To Balance c/f

Unpaid Dividend Account Rs. By Balance b/f 3,000 By Proposed Dividend A/c – Unpaid Dividend for 2006–07 3,000

Cr. Rs. Nil 3,000 3,000

10. Dr. To Bank – Tax for the year 2006–07 paid in 2007–08 To Balance c/f

Provision for Tax Account Rs. 20,000 By Balance b/f

Cr. Rs. 20,000

30,000

30,000

By Profit & Loss A/c – Provision made for 2007–08

50,000

Modified Date: Sat, Jul 03, 2010 11:47:06 AM

Output Date: Tue, Jul 06, 2010 11:46:58 AM

50,000

Rev II

Project: Management Accounting_Debarshi Bhattacharyya ACE Pro India Pvt. Ltd. File: X:\Pearson\Management Accounting_Debarshi Bhattacharyya\MAIN\M04\LAYOUT_M04\M04_DEBA_ISBN_EN_SE_C04_II.indd

287

CASH FLOW ANALYSIS

11. Dr.

Plant & Machinery Account Rs. 2,00,000 By Bank – Sale proceeds of old machine (60,000 – 8,000) 1,60,000 By Profit & Loss A/c – Loss on Sale of old Machinery By Profit & Loss A/c – Depreciation for the year (Bal. fig.) By Balance c/f 3,60,000

To Balance b/f To Bank – Cost of new Machinery

Cr. Rs. 52,000 8,000 20,000 2,80,000 3,60,000

12. Dr. To Balance b/f To Bank – Cost of new furniture To Profit & Loss A/c – Profit on Sale of furniture

Furniture Account Rs. 80,000 By Bank – Sale proceeds of furniture (Bal. fig.) 78,000 By Profit & Loss A/c – Depreciation for the year 3,000 By Balance c/f 1,61,000

Cr. Rs. 26,000 15,000 1,20,000 1,61,000

13. Analysis of other Non-current Assets and Liabilities Assets/Liabilities Land & Building Preliminary Expenses Investment

Opening Balance Rs. 1,30,000 20,000 30,000

Closing Balance Rs. 1,40,000 10,000 40,000

Increase/ Decrease Rs. (+)15,000 (−)10,000 (+)10,000

Analysis Addition to building Written off against Profit & Loss A/c New Investment made

Problem 38 From the following Balance Sheet and information, prepare a Cash Flow Statement of Ryan Ltd for the year that ended on 31 March 2003.

Liabilities Equity Share Capital 10% Redeemable Preference capital Capital Redemption Reserve Capital Reserve General Reserve Profit & Loss A/c 9% Debentures Sundry Creditors Bills Payable Liabilities for Expenses Provision for Taxation Proposed Dividend

31 March 2003 Rs.

31 March 2002 Rs.

6,00,000 – 1,00,000 1,00,000 1,00,000 70,000 2,00,000 95,000 20,000 30,000 95,000 90,000 15,00,000

5,00,000 2,00,000 – – 2,50,000 50,000 – 80,000 30,000 20,000 60,000 60,000 12,50,000 (Continued)

Modified Date: Sat, Jul 03, 2010 11:47:06 AM

Output Date: Tue, Jul 06, 2010 11:46:58 AM

Rev II

Project: Management Accounting_Debarshi Bhattacharyya ACE Pro India Pvt. Ltd. File: X:\Pearson\Management Accounting_Debarshi Bhattacharyya\MAIN\M04\LAYOUT_M04\M04_DEBA_ISBN_EN_SE_C04_II.indd

288

MANAGEMENT ACCOUNTING

Assets Land & Building Plant & Machinery Investments Inventory Bills Receivable Sundry Debtors Cash & Bank Preliminary Expenses Voluntary Separation Payments

31 March 2003 Rs.

31 March 2002 Rs.

1,50,000 7,65,000 50,000 95,000 65,000 1,75,000 65,000 10,000 1,25,000 15,00,000

2,00,000 5,00,000 80,000 90,000 70,000 1,30,000 90,000 25,000 65,000 12,50,000

Additional Information: i. A piece of land has been sold out for Rs. 1,50,000 (cost – Rs. 1,20,000) and the balance land was revalued. The capital reserve consisted of Profit on sale and Profit on revaluation. ii. On 1 April 2002, a Plant was sold for Rs. 90,000 (original cost – Rs. 70,000 & WDV – Rs. 50,000), and Debentures worth Rs. 1 lakh was issued at par as a part of consideration for a Plant of Rs. 4.5 lakh acquired. iii. Part of the Investments (cost – Rs. 50,000) was sold for Rs. 70,000. iv. Pre-acquisition Dividend received was Rs. 5,000 and it was adjusted against the Cost of Investment. v. Director has proposed a 15% Dividend for the current year. vi. Voluntary Separation cost of Rs. 50,000 was adjusted against General Reserve. vii. Income Tax liability for the current year was estimated at Rs. 1,35,000. viii. Depreciation @ 15% has been written off from the Plant account but no depreciation has been charged on land and building. [C.A. (PE II)—May 2003]

Solution Cash Flow Statement (under Indirect Method) of Ryan Ltd for the year that ended on 31 March 2003 Rs. A. Cash Flows from Operating Activities: Net Profit (after appropriation) for the year9 Add: Adjustment for Non-operating & Non-current items Debited to Profit & Loss A/c: Proposed Dividend8 Preliminary Expenses written off 9 Depreciation on Plant & Machinery2 Provision for Taxation7 Less:

Adjustment for Non-operating & Non-current items credited to Profit & Loss A/c: Profit on Sale of Plant2 Profit on Sale of Investment3 Operating Profit before Working Capital Changes

Rs.

Rs.

20,000

90,000 15,000 1,35,000 1,35,000

40,000 20,000

3,75,000 3,95,000

60,000 3,35,000 (Continued)

Modified Date: Sat, Jul 03, 2010 11:47:06 AM

Output Date: Tue, Jul 06, 2010 11:46:58 AM

Rev II

Project: Management Accounting_Debarshi Bhattacharyya ACE Pro India Pvt. Ltd. File: X:\Pearson\Management Accounting_Debarshi Bhattacharyya\MAIN\M04\LAYOUT_M04\M04_DEBA_ISBN_EN_SE_C04_II.indd

CASH FLOW ANALYSIS

Rs. Add:

Less:

Increase in Operating Current Liabilities: Sundry Creditors Liability for Expenses Decrease in Operating Current Assets: Bills Receivable Decrease in Operating Current Liabilities: Bills Payable Increase in Operating Current Assets: Sundry Debtors Inventory

Payment of voluntary separation cost5 Cash generated from operation Less: Income Tax paid7 Net Cash Flow from Operating Activities B. Cash Flows from Investing Activities: Proceeds from Sale of Land1 Proceeds from Sale of Plant2 Proceeds from Sale of Investment3 Pre-acquisition Dividend Received on Investment3

Rs.

289

Rs.

15,000 10,000 5,000

30,000 3,65,000

10,000 45,000 5,000

Less:

60,000 3,05,000 1,10,000 1,95,000 1,00,000 95,000

1,50,000 90,000 70,000 5,000 3,15,000

Less:

Plant acquired for cash2 New Investment made3

3,50,000 25,000 3,75,000

Net Cash Flow from Investing Activities C. Cash Flows from Financing Activities: New issue of Equity Shares for cash9 New issue of Debentures for cash6

(60,000) 1,00,000 1,00,000 2,00,000

Less:

Redemption of Preference Shares9 Payment of Dividend8

2,00,000 60,000 2,60,000

Add:

Net Cash Flow from Financing Activities Net Increase in Cash & Cash Equivalents Cash & Cash Equivalents at the beginning of the year Cash & Cash Equivalents at the end of the year

(60,000) (25,000) 90,000 65,000

Working Notes 1.

Dr. To Balance b/f To Capital Reserve – Profit on Sale of Land (1,50,000 – 1,20,000) To Capital Reserve – Profit on Revaluation of Land (Bal. fig.)

Land & Building Account Rs. 2,00,000 By Bank – Sale proceeds of Land By Balance c/f 30,000 70,000 3,00,000

Cr. Rs. 1,50,000 1,50,000

3,00,000 (Continued)

Modified Date: Sat, Jul 03, 2010 11:47:06 AM

Output Date: Tue, Jul 06, 2010 11:46:58 AM

Rev II

Project: Management Accounting_Debarshi Bhattacharyya ACE Pro India Pvt. Ltd. File: X:\Pearson\Management Accounting_Debarshi Bhattacharyya\MAIN\M04\LAYOUT_M04\M04_DEBA_ISBN_EN_SE_C04_II.indd

290

MANAGEMENT ACCOUNTING

2. Dr.

Plant & Machinery Account Rs. To Balance b/f 5,00,000 By Bank – Sale proceeds of Plant Machine (60,000 – 8,000) To Profit & Loss A/c – Profit on Sale of Plant 40,000 By Profit & Loss A/c – Depreciation for (90,000 – 50,000) the year [15% on {(5,00,000 – 50,000) + 4,50,000}] To 9% Debentures A/c – Part of the 1,00,000 consideration for Plant Acquired discharged To Bank – Balance of the consideration for 3,50,000 By Balance c/f Plant Acquired Discharged (4,50,000 – 1,00,000) 9,90,000

Cr. Rs. 90,000 1,35,000

7,65,000

9,90,000

3. Dr. To Balance b/f To Profit & Loss A/c – Profit on Sale of Investment (70,000 – 50,000) To Bank – New Investment made (Bal. fig.)

Investment Account Rs. 80,000 By Bank – Sale proceeds of Investment By Bank – Pre-acquisition Dividend 20,000 received 25,000 By Balance c/f 1,25,000

Cr. Rs. 70,000 5,000 50,000 1,25,000

4. Dr.

General Reserve Account Rs. To Voluntary Separation cost – Written off 50,000 By Balance b/f To Capital Redemption Reserve – Difference 1,00,000 between Face Value of Preference Shares Redeemed & Face Value of New Equity Shares issued (2,00,000 – 1,00,000) To Balance c/f 1,00,000 2,50,000

Cr. Rs. 2,50,000

2,50,000

5. Dr.

Voluntary Separation Cost Account Rs. To Balance b/f 65,000 By General Reserve – Written off To Bank – Voluntary Separation Cost Incurred 1,10,000 By Balance c/f 1,75,000

Cr. Rs. 50,000 1,25,000 1,75,000

6. Dr.

To Balance c/f

Modified Date: Sat, Jul 03, 2010 11:47:06 AM

9% Debenture Account Rs. By Balance b/f By Plant & Machinery A/c – Part of the consideration for Plant Acquired discharged 2,00,000 By Bank – New Issue of debenture for cash (Bal. fig.) 2,00,000

Output Date: Tue, Jul 06, 2010 11:46:58 AM

Cr. Rs. – 1,00,000

1,00,000 2,00,000

Rev II

Project: Management Accounting_Debarshi Bhattacharyya ACE Pro India Pvt. Ltd. File: X:\Pearson\Management Accounting_Debarshi Bhattacharyya\MAIN\M04\LAYOUT_M04\M04_DEBA_ISBN_EN_SE_C04_II.indd

291

CASH FLOW ANALYSIS

7. Dr.

Provision for Taxation Account Rs. 1,00,000 By Balance b/f 95,000 By Profit & Loss A/c – Provision made for 2002–03 1,95,000

To Bank – Tax Liability paid (Bal. fig.) To Balance c/f

Cr. Rs. 60,000 1,35,000 1,95,000

8. Dr. To Bank – Dividend for 2001–02 paid in 2002–03 To Balance c/f

Proposed-Dividend Account Rs. 60,000 By Balance b/f 90,000

Cr. Rs. 60,000

By Profit & Loss appropriation A/c – Dividend proposed in 2002–03

90,000

1,50,000

9.

1,50,000

Analysis of other Non-current Assets and liabilities Opening Balance Rs. 5,00,000 2,00,000 –

Closing Balance

Capital Redemption Reserve Profit & Loss A/c

– 50,000

1,00,000 70,000

Preliminary Expenses

25,000

10,000

Assets/Liabilities Equity Share Capital Preference Share Capital Capital Reserve

Rs. 6,00,000 – 1,00,000

Increase/ Decrease Analysis Rs. (+)1,00,000 New issue for cash (−)2,00,000 Redeemed for cash (+)1,00,000 Profit on Sale & Profit on revaluation of land (+)1,00,000 Transfer from General Reserve (+)20,000 Net Profit for the year after appropriation (−)15,000 Written off against Profit & Loss A/c

Problem 39 The Balance Sheet of JK Limited as on 31 March 2005 and 31 March 2006 are given as follows:

Liabilities Share Capital Capital Reserve General Reserve Profit & Loss A/c 9% Debenture Current Liabilities Proposed Dividend Provision for Tax Unpaid Dividend

31 March 2005 1,440 – 816 288 960 576 144 432 – 4,656

Modified Date: Sat, Jul 03, 2010 11:47:06 AM

Balance Sheet as on 31 March Assets 2006 1,920 Fixed Assets 48 Less: Depreciation 960 360 Investment 672 Cash 624 Other Current Assets (including Stock) 174 408 Preliminary Expenses 18 5,184

Output Date: Tue, Jul 06, 2010 11:46:58 AM

Rs. in ’000 31 31 March March 2005 2006 3,840 4,560 (1,104) (1,392) 2,736 3,168 480 384 210 312 1,134 1,272

96

48

4,656

5,184

Rev II

Project: Management Accounting_Debarshi Bhattacharyya ACE Pro India Pvt. Ltd. File: X:\Pearson\Management Accounting_Debarshi Bhattacharyya\MAIN\M04\LAYOUT_M04\M04_DEBA_ISBN_EN_SE_C04_II.indd

292

MANAGEMENT ACCOUNTING

Additional Information: i. During the year 2005–06, Fixed Assets with a book value of Rs. 2,40,000 (accumlated depreciation – Rs. 84,000) was sold for Rs. 1,20,000. ii. Provided Rs. 4,20,000 as depreciation. iii. Some Investments are sold at a Profit of Rs. 48,000 and the Profit was credited to Capital Reserve. iv. It was decided that the stocks be Valued at Cost, whereas previously the practice was to value the stock at a cost less 10%. The stock was Rs. 2,59,200 as on 31 March 2005. The stock as on 31 March 2006 was correctly valued at Rs. 3,60,000. v. It was decided to write off the Fixed Assets costing Rs. 60,000 on which a depreciation amouning to Rs. 48,000 has been provided. vi. Debentures are redeemed at Rs. 105. Required: Prepare a Cash Flow Statement. [C. A. (PE II)—May 2007]

Solution Cash Flow Statement (under Indirect Method) of JK Ltd for the year that ended on 31 March 2006 Rs. in ’000 A. Cash Flows from Operating Activities: Net Profit (after appropriation) for the year8 Add: Adjustment for Non-operating & Non-current items debited to Profit & Loss A/c: Transfer to General Reserve8 Proposed Dividend4 Preliminary Expenses written off8 Loss on Sale of Fixed Asset1 Depreciation on Fixed Assets Fixed Asset written off1 Provision for Tax6 Premium on Redemption of Debentures7 Less: Add:

Undervaluation of Opening Stock (10 / 90 × Rs. 2,59,200) Operating Profit before Working Capital Changes Increase in Operating Current Liabilities Decrease in Operating Current Assets

Less:

Decrease in Operating Current Liabilities Increase in Operating Current Assets [1,272 – (1,134 + 28.80)] Cash generated from operation Less: Income Tax paid Net Cash Flow from Operating Activities B. Cash Flows from Investing Activities: Proceeds from Sale of Fixed Asset1 Proceeds from Sale of Investment3 Less: Purchase of Fixed Assets1 Net Cash Flow from Investing Activities

Rs. in ’000

Rs. in ’000

72.00

144 174 48 120 420 12 408 14.40

48 Nil Nil 109.20

1,340.40 1,412.80 28.80 1,383.60 48.00 1,431.60 109.20 1,322.80 432.00 890.40

120.00 144.00

264.00 1,104.00 (840.00) (Continued)

Modified Date: Sat, Jul 03, 2010 11:47:06 AM

Output Date: Tue, Jul 06, 2010 11:46:58 AM

Rev II

Project: Management Accounting_Debarshi Bhattacharyya ACE Pro India Pvt. Ltd. File: X:\Pearson\Management Accounting_Debarshi Bhattacharyya\MAIN\M04\LAYOUT_M04\M04_DEBA_ISBN_EN_SE_C04_II.indd

293

CASH FLOW ANALYSIS

Rs. in ’000 C. Cash Flows from Financing Activities: Proceeds Received from New Issue of Shares8 Less: Redemption of Debentures including Premium7 Payment of Dividend4 Net Cash Flow from Financing Activities Net Increase in Cash & Cash Equivalents Add: Cash & Cash Equivalents at the beginning of the year Cash & Cash Equivalents at the end of the year

Rs. in ’000

Rs. in ’000

480.00 302.40 126.00

428.40 51.60 102.00 210.00 312.00

Working Notes 1. Dr. To Balance b/f

To Bank – Cost of New Fixed Assets (Bal. fig.)

Fixed Asset (at Cost) Account Rs. in ’000 3,840 By Provision for Depreciation A/c – Accumulated Depreciation on fixed assets sold 1,104 By Bank – Sale proceeds By Profit & Loss A/c – Loss on Sale of Fixed Assets (240 – 120) By Provision for Depreciation A/c – Accumulated Depreciation on Fixed Asset written off By Profit & Loss A/c – WDV of the Fixed Asset written off By Balance c/f 4,944

Cr. Rs. in ’000 84

120 120 48

12 4,560 4,944

2. Dr.

Provision for Depreciation Account Rs. in ’000 84 By Balance b/f

To Fixed Asset A/c – Accumulated Depreciation on Fixed Assets sold To Fixed Asset A/c – Accumulated Depreciation on Fixed Asset written off To Balance c/f

48

By Profit & Loss A/c – Depreciation for the year

1,392 1,524

Cr. Rs. in ’000 1,104 420

1,524

3. Dr. To Balance b/f To Capital Reserve A/c – Profit on Sale of Investment

Investment Account Rs. in ’000 480 By Bank – Sale Proceeds of Investment 48 By Balance c/f 528

Cr. Rs. in ’000 144 384 528 (Continued)

Modified Date: Sat, Jul 03, 2010 11:47:06 AM

Output Date: Tue, Jul 06, 2010 11:46:58 AM

Rev II

Project: Management Accounting_Debarshi Bhattacharyya ACE Pro India Pvt. Ltd. File: X:\Pearson\Management Accounting_Debarshi Bhattacharyya\MAIN\M04\LAYOUT_M04\M04_DEBA_ISBN_EN_SE_C04_II.indd

294

MANAGEMENT ACCOUNTING

4. Dr. To Bank – Dividend paid (144 – 18) To Unpaid Dividend A/c – Dividend not claimed To Balance c/f

Proposed Dividend Account Rs. in ’000 126 By Balance b/f 18 By Profit & Loss appropriation A/c – Dividend proposed for 2005–06 (Bal. fig.) 174 318

Cr. Rs. in ’000 144 174

Unpaid Dividend Account Rs. in ’000 By Balance b/f 18 By Proposed Dividend A/c 18

Cr. Rs. in ’000 Nil 18 18

Provision for Tax Account Rs. in ’000 432 By Balance b/f

Cr. Rs. in ’000 432

318

5. Dr.

To Balance c/f

6. Dr. To Bank – Tax for the year 2004–05 paid in 2005–06 To Balance c/f

408

By Profit & Loss A/c – Provision made in 2005–06

840

408 840

7. Dr.

9% Debenture Account Rs. in ’000 302.40 By Balance b/f

To Bank – Debentures Redeemed (288 + 14.40) To Balance c/f

672.00

Cr. Rs. in ’000 960.00

By Profit & Loss A/c – Premium paid on redemption adjusted [5% of (960 – 672)]

974.40

8.

14.40 974.40

Analysis of other Non-current Assets and liabilities Assets/Liabilities

Preliminary Expenses Share Capital Capital Reserve General Reserve Profit & Loss A/c

Opening Balance Rs. in ’000 96 1,440 – 816 288

Closing Balance Rs. in ’000 48 1,920 48 960 360

Increase/ Decrease Rs. in ’000 (−)48 (+)480 (+)48 (+)144 (+)72

Analysis Written off against Profit & Loss A/c New issue of Shares Profit on Sale of Investment Transfer from Profit & Loss A/c Net Profit (after appropriation) for the year

Guidance Note Here, the book value of Rs. 2,40,000 of the fixed asset sold [as stated in Additional Information (i) of the problem] is considered as a WDV of the Fixed Asset sold, not as its cost. Accordingly, the loss on sale of that Fixed Asset is arrived at after deducting the sale proceeds from the said WDV, that is, Rs. 2,40,000 – Rs. 1,20,000.

Modified Date: Sat, Jul 03, 2010 11:47:06 AM

Output Date: Tue, Jul 06, 2010 11:46:58 AM

Rev II

Project: Management Accounting_Debarshi Bhattacharyya ACE Pro India Pvt. Ltd. File: X:\Pearson\Management Accounting_Debarshi Bhattacharyya\MAIN\M04\LAYOUT_M04\M04_DEBA_ISBN_EN_SE_C04_III.indd

CASH FLOW ANALYSIS

295

Problem 40 From the information contained in the Income Statement and Balance Sheet of A Ltd, Prepare a Cash Flow Statement:

Income Statement for the year that ended on 31 March 2006 Rs. 2,52,00,000

Net Sales (A) Cash Cost of Sales Depreciation Salaries & Wages Operating Expenses Provision for Taxation (B) Net Operating Profit (A – B) Non-recurring Income – Profits on Sale of Equipment Retained Earnings and Profit Brought Forward Less:

Dividends Declared and Paid during the Year Profit & Loss A/c Balance as on 31 March 2006

1,98,00,000 6,00,000 24,00,000 8,00,000 8,80,000 2,44,80,000 7,20,000 1,20,000 8,40,000 15,18,000 23,58,000 7,20,000 16,38,000

Balance Sheet as on

Assets Fixed Assets: Land Buildings & Equipments Current Assets: Cash Debtors Stock Advances Liabilities and Equity Share Capital Surplus in Profit & Loss A/c Sundry Creditors Outstanding Expenses Income Tax Payable Accumulated Depreciation on Buildings and Equipment

31 March 2005 Rs.

31 March 2006 Rs.

4,80,000 36,00,000

9,60,000 57,60,000

6,00,000 16,80,000 26,40,000 78,000 90,78,000

7,20,000 18,60,000 9,60,000 90,000 1,03,50,000

36,00,000 15,18,000 24,00,000 2,40,000 1,20,000 12,00,000 90,78,000

44,40,000 16,38,000 23,40,000 4,80,000 1,32,000 13,20,000 1,03,50,000

The original cost of equipment sold during the year 2005–06 was Rs. 7,20,000. [C.A. (PE II)—November 2006]

Modified Date: Sat, Jul 03, 2010 12:44:29 PM

Output Date: Tue, Jul 06, 2010 11:47:54 AM

Rev II

Project: Management Accounting_Debarshi Bhattacharyya ACE Pro India Pvt. Ltd. File: X:\Pearson\Management Accounting_Debarshi Bhattacharyya\MAIN\M04\LAYOUT_M04\M04_DEBA_ISBN_EN_SE_C04_III.indd

296

MANAGEMENT ACCOUNTING

Solution Cash Flow Statement (under direct method) of A Ltd for the year that ended on 31 March 2006 Rs. in ’000 A. Cash Flows from Operating Activities: Cash Receipts from Customers1 Less: Cash Paid to Suppliers & Employees4 Cash Generated from Operation Less: Income Tax Paid5 Net Cash Flow from Operating Activities B. Cash Flows from Investing Activities: Proceeds from Sale of Equipment6 Less: Purchase of Building & Equipment6 Purchase of Land8 Net Cash Flow from Investing Activities C. Cash Flows from Financing Activities: Proceeds Received from New Issue of Shares8 Less: Payment of Dividend Net Cash Flow from Financing Activities Net increase in Cash & Cash Equivalents Add: Cash & Cash Equivalents at the beginning of the year Cash & Cash Equivalents at the end of the year

Rs. in ’000

Rs. in ’000

25,020 21,152 3,868 868 3,000 360 2,880 480

3,360 (3,000) 840 720 120 120 600 720

Working Notes 1. Dr. To Balance b/f To Sales

Debtors Account Rs. in ’000 1,680 By Bank – Collection (Bal. fig.) 25,200 By Balance c/f 26,880

Cr. Rs. in ’000 25,020 1,860 26,880

Stock Account Rs. in ’000 2,640 By Cost of Sales 18,120 By Balance c/f

Cr. Rs. in ’000 19,800 960

2. Dr. To Balance b/f To Purchase of goods including other Direct Cash Expenses (Bal. fig.)

20,760

20,760

3. Dr.

Operating Expenses Account Rs. in ’000 To Balance b/f (advance) 78 By Balance b/f (Outstanding) To Bank – Payment during the year (Bal. fig.) 572 By Profit & Loss A/c – Operating Expenses for the year To Balance c/f (Outstanding) 480 By Balance c/f (advance) 1,130

Cr. Rs. in ’000 240 800 90 1,130

4. Calculation of cash paid to suppliers and employees

Add:

Purchase of Goods including other Direct Expenses2 Opening Balance of Creditors

Rs. in ’000 18,120 2,400 (Continued)

Modified Date: Sat, Jul 03, 2010 12:44:29 PM

Output Date: Tue, Jul 06, 2010 11:47:54 AM

Rev II

Project: Management Accounting_Debarshi Bhattacharyya ACE Pro India Pvt. Ltd. File: X:\Pearson\Management Accounting_Debarshi Bhattacharyya\MAIN\M04\LAYOUT_M04\M04_DEBA_ISBN_EN_SE_C04_III.indd

CASH FLOW ANALYSIS

Less:

Closing Balance of Creditors

Add: Add:

Salaries & Wages Paid Operating Expenses Paid3

297

Rs. in ’000 20,520 2,340 18,180 2,400 572 21,152

Cash Paid to Suppliers and Employees

5. Dr.

Income Tax Payable Account Rs. in ’000 To Bank – Tax paid during 2005–06 (Bal. fig.) 868 By Balance b/f To Balance c/f 132 By Profit & Loss A/c – Provision made in 2005–06 1,000

Cr. Rs. in ’000 120 880 1,000

6. Dr.

Building & Equipment (at cost) Account Rs. in ’000 To Balance b/f 3,600 By Accumulated Depreciation A/c – Accumulated Depreciation on Equipment Sold To Bank – New Purchase (Bal. fig.) 2,880 By Bank – Sale proceeds of Equipment [(720 – 480) + 120] To Profit & Loss A/c – Profit on Sale of 120 By Balance c/f Fixed Assets 6,600

Cr. Rs. in ’000 480

360 5,760 6,600

7. Dr.

Accumulated Depreciation on Building & Equipment Account Cr. Rs. in ’000 Rs. in ’000 To Fixed Asset A/c – Accumulated 48 By Balance b/f 1,200 Depreciation on Equipment Sold To Balance c/f 1,320 By Profit & Loss A/c – Depreciation for the year 600 1,800 1,800

8. Analysis of other non-Current Assets and liabilities Assets/Liabilities Share Capital Land

Opening Balance Rs. in ’000 3,600 480

Closing Balance Rs. in ’000 4,440 960

Increase/ Decrease Rs. in ’000 (+) 840 (+) 480

Analysis New Issue of Shares New purchase of Building & Equipment

Problem 41 The Balance Sheet of X Limited as on 31 March 2007 is as follows: Liabilities Equity Share Capital 8% Preference Share Capital Reserves & Surplus 10% Debentures Sundry creditors

Rs. in ’000 6,000 3,250 1,400 1,950 3,250

Assets Fixed Assets (at cost) Less: Depreciation written off Stock Sundry Debtors Cash

15,850

Modified Date: Sat, Jul 03, 2010 12:44:29 PM

Output Date: Tue, Jul 06, 2010 11:47:54 AM

Rs. in ’000 16,250 5,200 11,050 1,950 2,600 250 15,850

Rev II

Project: Management Accounting_Debarshi Bhattacharyya ACE Pro India Pvt. Ltd. File: X:\Pearson\Management Accounting_Debarshi Bhattacharyya\MAIN\M04\LAYOUT_M04\M04_DEBA_ISBN_EN_SE_C04_III.indd

298

MANAGEMENT ACCOUNTING

The following additional information is available: i. The stock turnover ratio based on Cost of Goods Sold would be 6 times. ii. The cost of Fixed Assets to sales ratio would be 1:4. iii. Fixed Assets costing Rs. 30,00,000 to be installed on 1 April 2007, payment would be made on 31 March 2008. iv. In March 2008, a dividend of 7% on equity capital would be paid. v. Rs. 5,50,000 and 11% debentures would be issued on 1 April 2007. vi. Rs. 30,00,000 as equity shares would be issued on 31 March 2008. vii. Creditors would be 25% of materials consumed. viii. Debtors would be 10% of sales. ix. The Cost of Goods Sold would be 90% of sales and includes material of 40% and a depreciation of 5% of sales. x. The profit is subject to debenture interest and taxation @ 30%. Required: i. Prepare the projected Balance Sheet as on 31 March 2008. ii. Prepare the projected Cash Flow Statement in accordance with AS-3. [C.A. (PE II)—November 2007] Solution i. Projected Balance Sheet of X Ltd as on 31 March 2008 Liabilities Equity Share Capital 8% Preference Share Capital Reserves & Surplus8 Secured Loan: 10% Debentures 11% Debentures Current Liabilities & Provisions: Sundry Creditors3 Provision for Tax

Rs. in ’000 9,000.00 3,250.00 5,931.15

Add:

Assets Fixed Assets (at cost) Purchased during the year

Less:

Accumulated Depreciation (5,200 + 3,850)

1,950.00 550.00 7,700.00 2,233.35 30,614.50

Current Assets: Stock7 Sundry Debtors6 Cash (Bal. fig.)

Rs. in ’000 16,250.00 3,000.00 19,250.00 9,050.00 10,200.00 11,550.00 7,700.00 1,164.50 30,614.50

ii. Projected Cash Flow Statement (under Direct Method) of X Ltd for the year that ended on 31 March 2008 Rs. in ’000 A. Cash Flows from Operating Activities: Cash Receipts from Customers9 Less: Cash Paid to Suppliers11 Cash Paid for Wages & other Overheads5

Rs. in ’000

Rs. in ’000

71,900 35,950 34,650 70,600

Net Cash Flow from Operating Activities B. Cash Flows from Investing Activities: Cash Payment for Purchase of Fixed Assets Net Cash Flow from Investing Activities C. Cash Flows from Financing Activities: Proceeds Received from issue of Equity Shares Proceeds Received from issue of 11% Debentures

1,300 (3,000) (3,000) 3,000.00 550.00 3,550.00 (Continued)

Modified Date: Sat, Jul 03, 2010 12:44:29 PM

Output Date: Tue, Jul 06, 2010 11:47:54 AM

Rev II

Project: Management Accounting_Debarshi Bhattacharyya ACE Pro India Pvt. Ltd. File: X:\Pearson\Management Accounting_Debarshi Bhattacharyya\MAIN\M04\LAYOUT_M04\M04_DEBA_ISBN_EN_SE_C04_III.indd

CASH FLOW ANALYSIS

Less:

Payment of equity dividend Payment of preference dividend Payment of debenture interest

Rs. in ’000 420.00 260.00 235.50

Rs. in ’000

299

Rs. in ’000

935.50 Net Cash Flow from Financing Activities Net increase in Cash & Cash equivalents Cash & Cash Equivalents at the beginning of the year Cash & Cash Equivalents at the end of the year

Add:

2,614.50 914.00 250.00 1,164.50

Working Notes 1. Fixed Assets (FA) at cost on 31 March 2008 = 16,250 + 3,000 = 19,250 (Rs. in ’000). Now, Fixed Assets (FA) at cost to Sales ratio = 1 / 4 FA 1 = or Sales 4 or

2. 3. 4. 5.

6. 7.

8.

19,250 1 = Sales 4

∴ Sales = 19,250 × 4 = 77,000 (Rs. in ’000) ∴ Cost of Goods Sold = 90% of sales = 90% of 77,000 = 69,300 (Rs. in ’000) Materials Consumed = 40% of Sales = 40% of 77,000 = 30,800 (Rs. in ’000) Sundry Creditors = 25% of materials Consumed = 25% of 30,800 = 7,700 (Rs. in ’000) Depreciation on Fixed Assets for the year = 5% of sales = 5% of 77,000 = 3,850 (Rs. in ’000) Wages and other overheads included in the Cost of Goods Sold = Cost of Goods Sold – (Materials consumed + Depreciation for the year) = 69,300 – (30,800 + 3,850) = 34,650 (Rs. in ’000) As there is no outstanding or prepaid wages & other overheads, either at the beginning or at the end of the year, it implies that the entire amount of wages and other overheads included in the Cost of Goods Sold is paid in cash during the year. Sundry Debtors (Closing) = 10% of Sales = 10% of 77,000 = 7,700 (Rs. in ’000) Cost of Goods Sold Stock Turnover Ratio = Average Stock 69,3001 Here, 6 = [Here, Closing Stock is considered in place of Average Stock] Closing Stock 69,300 ∴ Closing stock = = 11,550 (Rs. in ’000 ) 6 Projected Profit & Loss A/c for the year that ended on 31 March 2008 Rs. in ’000

Less: Less: Less: Less:

Add:

Sales1 Cost of Goods Sold1 Profit before Interest & Tax Interest on debentures [(10% on 1,950) + (11% on 550)] Profit before Tax Tax @ 30% [30% on 7,444.50] Profit after Tax Preference dividend [8% on 3,250] Equity dividend [7% on 6,000] Surplus Profit after Appropriation kept as Reserves & Surplus Reserves & Surplus at the beginning of the year Reserves & Surplus at the end of the year

260.00 420.00

Rs. in ’000 77,000.00 69,300.00 7,700.00 255.50 7,444.50 2,233.35 5,211.15 680.00 4,531.15 1,400.00 5,931.15 (Continued)

Modified Date: Sat, Jul 03, 2010 12:44:29 PM

Output Date: Tue, Jul 06, 2010 11:47:54 AM

Rev II

Project: Management Accounting_Debarshi Bhattacharyya ACE Pro India Pvt. Ltd. File: X:\Pearson\Management Accounting_Debarshi Bhattacharyya\MAIN\M04\LAYOUT_M04\M04_DEBA_ISBN_EN_SE_C04_III.indd

300

MANAGEMENT ACCOUNTING

9. Dr. To Balance b/f To Sales1

Sundry Debtors Account Rs. ’000 2,600 By Bank – Collection (Bal. fig.) 77,000 By Balance c/f6 79,600

Cr. Rs. ’000 71,900 7,700 79,600

Stock Account Rs. ’000 1,950 By Materials Consumed2 40,400 By Balance c/f7 42,350

Cr. Rs. ’000 30,800 11,550 42,350

Sundry Creditors Account Rs. ’000 35,950 By Balance b/f 7,700 By Purchase of Materials10 43,650

Cr. Rs. ’000 3,250 40,400 43,650

10. Dr. To Balance b/f To Purchase of Materials (Bal. fig.)

11. Dr. To Bank – Payment (Bal. fig.) To Balance c/f3

Problem 42 X Ltd has the following balances as on 1 April 2007:

Less:

Rs. 11,40,000 3,99,000 7,41,000 4,75,000 66,500 1,14,000 76,000 5,70,000

Fixed Assets Depreciation Stocks and Debtors Bank Balance Creditors Bills Payable Capital (Shares of Rs. 100 each)

The company made the following estimates for the financial year 2007–08: i. The company will pay a free-of-tax dividend of 10%, the rate of tax being 25%. ii. The company will acquire Fixed Assets costing Rs. 1,90,000, after selling one machine for Rs. 38,000 costing Rs. 95,000, and on which a depreciation provided amounted to Rs. 66,500. iii. Stocks and debtors, creditors and bills payable at the end of the financial year are expected to be Rs. 5,60,500, Rs. 1,48,200 and Rs. 98,800, respectively. iv. The profit would be Rs. 1,04,500 after a depreciation of Rs. 1,14,000. Prepare the projected Cash Flow Statement and ascertain the bank balance of X Ltd at the end of the financial year 2007–08. [C.A.—(PE II)—May 2008] Solution Projected Cash Flow Statement (under Indirect Method) of X Ltd for the financial year 2007–08 Rs. A. Cash Flows from Operating Activities: Estimated Net Profit for the year

Rs.

Rs.

1,04,500 (Continued)

Modified Date: Sat, Jul 03, 2010 12:44:29 PM

Output Date: Tue, Jul 06, 2010 11:47:54 AM

Rev II

Project: Management Accounting_Debarshi Bhattacharyya ACE Pro India Pvt. Ltd. File: X:\Pearson\Management Accounting_Debarshi Bhattacharyya\MAIN\M04\LAYOUT_M04\M04_DEBA_ISBN_EN_SE_C04_III.indd

301

CASH FLOW ANALYSIS

Rs. Add:

Less:

Add:

Adjustment for Non-operating & Non-current Items debited to Profit & Loss A/c: Depreciation Adjustment for Non-operating & Non-current Items credited to Profit & Loss A/c: Profit on Sale of Machinery Operating Profit before Working Capital changes Increase in Operating Current Liabilities: Creditors Bills Payable

Rs.

Rs.

1,14,000 2,18,500

9,500 2,09,000 34,200 22,800

Increase in Operating Current Assets: Stock & Debtors Cash Generated from Operation Less: Income Tax paid1 Net Cash Flow from Operating Activities B. Cash Flows from Investing Activities: Proceeds Received from sale of Machinery Less: Cash Paid for purchase of new Fixed Assets Net Cash Flow from Investing Activities C. Cash Flows from Financing Activities: Payment of Tax-free Dividend to Shareholders Payment of Tax on the above Dividend2 Net Cash Flow from Financing Activities Net Increase in Cash & Cash Equivalents Add: Cash & Cash Equivalents at the beginning of the year Cash & Cash Equivalents at the end of the year

57,000 2,66,000

Less:

85,500 1,80,500 Nil 1,80,500 38,000 1,90,000 (1,52,000) (57,000) (19,000) (76,000) (47,500) 66,500 19,000

Working Notes 1. In the absence of adequate information, income tax on the company’s operating profit is ignored here. 2. Net tax-free dividend to be paid to shareholders = 10% on 5,70,000 = Rs. 57,000 Again, the given tax rate on the above dividend = 25% As the company has decided to pay a tax-free dividend to its shareholders, it has to bear the amount of tax payable by its shareholders on receipt of such dividend. Hence, the total amount to be paid for such dividend is as follows:

Add:

10% Tax-free Dividend to be paid to Shareholders Tax on above Dividend @ 25% to be borne by the Company (25 ÷ 75 × 57,000) Total Amount to be paid for such Dividend

Rs. 57,000 19,000 76,000

Problem 43 The following are the changes in the account balance taken from the Balance Sheets of Dada Ltd at the beginning and end of the year:

Equity Share Capital of 30,000 Shares of Rs. 10 each, issued and fully paid Capital Reserve 8% Debentures

Changes in Rupees in Debit (or Credit) 0 (49,200) (50,000) (Continued)

Modified Date: Sat, Jul 03, 2010 12:44:29 PM

Output Date: Tue, Jul 06, 2010 11:47:54 AM

Rev II

Project: Management Accounting_Debarshi Bhattacharyya ACE Pro India Pvt. Ltd. File: X:\Pearson\Management Accounting_Debarshi Bhattacharyya\MAIN\M04\LAYOUT_M04\M04_DEBA_ISBN_EN_SE_C04_III.indd

302

MANAGEMENT ACCOUNTING

Changes in Rupees in Debit (or Credit) 1,000 43,000 60,000 (14,400) 50,000 38,500 (11,800) (76,500) 30,000 (3,300) 47,000 (64,300) 0

Debenture Discount Freehold Property at Cost/Revaluation Plant and Machinery at Cost Depreciation on Plant and Machinery Debtors Stock and Work-in-progress (WIP) Creditors Net Profit for the year Dividend Paid in respect of earlier year Provision for Doubtful Debts Trade Investments at Cost Bank

You are informed that: i. Capital Reserve as at the end of the year represented realized profits on the sale of one freehold property together with the surplus arising on the revaluation of the balance of freehold properties. ii. During the year, the plant costing Rs. 18,000, against which a depreciation provision of Rs. 13,500 was lying, was sold for Rs. 7,000. iii. During the middle of the year, Rs. 50,000 debentures were issued for cash at a discount of Rs. 1,000. iv. The Net Profit for the year was after crediting the profit on sale of plant and charging debenture interest. You are required to prepare a statement which will explain, why bank borrowing has increased by Rs. 64,300 during the year-end. Ignore taxation. [C.A. (Final)—Adapted] Solution Cash Flow Statement (under Indirect Method) of Raj Ltd for the year that ended on__________ Rs. A. Cash Flows from Operating Activities: Net profit (after interest) for the year Add: Adjustment for Non-operating & Non-current Items Debited to Profit & Loss A/c: Depreciation on Plant & Machinery4 Interest on Debenture5 Less:

Add:

Adjustment for Non-operating & Non-current Items credited to Profit & Loss A/c: Profit on Sale of Plant2 Operating Profit before Working Capital Changes Increase in Operating Current Liabilities: Creditors Provision for Doubtful Debts

Increase in Operating Current Assets: Debtors Stock & WIP Net Cash Flow from Operating Activities B. Cash Flows from Investing Activities: Proceeds from Sale of Freehold Property3

Rs.

Rs.

76,500

27,900 2,000

29,900 1,06,400

2,500 1,03,900 11,800 3,300

15,100 1,19,000

Less:

50,000 38,500

88,500 30,500 6,200 (Continued)

Modified Date: Sat, Jul 03, 2010 12:44:29 PM

Output Date: Tue, Jul 06, 2010 11:47:54 AM

Rev II

Project: Management Accounting_Debarshi Bhattacharyya ACE Pro India Pvt. Ltd. File: X:\Pearson\Management Accounting_Debarshi Bhattacharyya\MAIN\M04\LAYOUT_M04\M04_DEBA_ISBN_EN_SE_C04_III.indd

303

CASH FLOW ANALYSIS

Rs. Proceeds from Sale of Plant1 Purchase of Plant & Machinery for Cash1 New Trade Investment Made for Cash3 Net Cash Flow from Investing Activities C. Cash Flows from Financing Activities: Proceeds Received from Issue of Debenture at Discount Less: Payment of Dividend for earlier year Payment of Debenture Interest Net Cash Flow from Financing Activities Net increase in Cash & Cash Equivalents Add: Cash & Cash Equivalents at the beginning of the year Cash & Cash Equivalents at the end of the year (i.e., Net Decrease over Opening Balance) Less:

78,000 47,000

Rs. 7,000 13,200

Rs.

1,25,000 (1,11,800) 49,000

30,000 2,000

32,000 17,000 (64,300) Nil (64,300)

Working Notes 1. Cost of Plant & Machinery Purchased

Add:

Rs. 60,000 18,000 78,000

Net Increase in Cost Cost of Plant Sold during the period Cost of Plant & Machinery Purchased

2. Profit on Sale of Plant Profit on Sale of plant = Sale proceeds – WDV = 7,000 – (18,000 – 13,500)

Rs. = Rs. 2,500.

3. Proceeds received from sale of Freehold Property

Less:

Rs. 49,200 43,000 6,200

Net increase in Capital Reserve Profit on Revaluation of Freehold Property included in above Proceeds Received from Sale of Freehold Property

4. Depreciation on Plant & Machinery provided during the period

Add:

Rs. 14,400 13,500 27,900

Increase in Provision for Depreciation Accumulated Depreciation on Plant Sold Depreciation on Plant & Machinery Provided during the period

5. Interest on Debenture Interest on debentue (on increased debentures) = 8% on 50,000 for 1/2 year

Rs. = 2,000.

Problem 44 Balance Sheets of a company as on 31 March 2007 and 2008 were as follows: Liabilities Equity Share Capital 8% Preference Share Capital General Reserve Securities premium

31 March 2007 Rs. 10,00,000 2,00,000 1,20,000 

31 March 2008 Rs. 10,00,000 3,00,000 1,45,000 25,000

Assets Goodwill Land & Building Plant & Machinery Investment (non-trading)

31 March 2007 Rs. 1,00,000 7,00,000 6,00,000 2,40,000

31 March 2008 Rs. 80,000 6,50,000 6,60,000 2,20,000 (Continued)

Modified Date: Sat, Jul 03, 2010 12:44:29 PM

Output Date: Tue, Jul 06, 2010 11:47:54 AM

Rev II

Project: Management Accounting_Debarshi Bhattacharyya ACE Pro India Pvt. Ltd. File: X:\Pearson\Management Accounting_Debarshi Bhattacharyya\MAIN\M04\LAYOUT_M04\M04_DEBA_ISBN_EN_SE_C04_III.indd

304

MANAGEMENT ACCOUNTING

Liabilities Profit & Loss A/c 11% Debentures Creditors Provision for tax Proposed dividend

31 March 2007 Rs. 2,10,000 5,00,000 1,85,000 80,000 1,36,000

31 March 2008 Rs. 3,00,000 3,00,000 2,15,000 1,05,000 1,44,000

24,31,000

25,34,000

Assets Stock Debtors Cash & Bank Prepaid Expenses Premium on redemption of debentures

31 March 2007 Rs. 4,00,000 2,88,000 88,000 15,000 

31 March 2008 Rs. 3,85,000 4,15,000 93,000 11,000 20,000

24,31,000

25,34,000

Additional Information: i. Investments were sold during the year at a profit of Rs. 15,000. ii. During the year, an old machine costing Rs. 80,000 was sold for Rs. 36,000. Its WDV was Rs. 45,000. iii. Depreciation charged on plants and machinery @ 20% on the opening balance. iv. There was no purchase or sale of land and building. v. Provision for tax made during the year was Rs. 96,000. vi. Preference shares were issued for consideration of cash during the year. You are required to prepare: i. Cash Flow Statement as per AS-3. ii. Schedule of changes in working capital. [C.A. (PCC)—November 2008] Solution i. Cash Flow Statement (under Indirect Method) of 31 March 2008

for the year that ended on Rs.

A. Cash Flows from Operating Activities: Net Profit (After Appropriation) for the Year5 Add: Adjustment for Non-operating & Non-current Items Debited to Profit & Loss A/c: Proposed Dividend4 Transfer to General Reserve7 Goodwill written Off 7 Depreciation on Plant & Machinery1 Depreciation on Land & Building7 Loss on Sale of Plant & Machinery1 Interest on Debentures6 Provision for Taxation Less:

Add:

Adjustment for Non-operating & Non-current Items Credited to Profit & Loss A/c: Profit on Sale of Investment2 Operating Profit before Working Capital Changes Increase in Operating Current Liabilities: Creditors Decrease in Operating Current Assets: Prepaid Expenses Stock

Rs.

Rs.

90,000

1,44,000 25,000 20,000 1,20,000 50,000 9,000 33,000 96,000

4,97,000 5,87,000

15,000 5,72,000 30,000 4,000 15,000

49,000 6,21,000 (Continued)

Modified Date: Sat, Jul 03, 2010 12:44:29 PM

Output Date: Tue, Jul 06, 2010 11:47:54 AM

Rev II

Project: Management Accounting_Debarshi Bhattacharyya ACE Pro India Pvt. Ltd. File: X:\Pearson\Management Accounting_Debarshi Bhattacharyya\MAIN\M04\LAYOUT_M04\M04_DEBA_ISBN_EN_SE_C04_III.indd

305

CASH FLOW ANALYSIS

Rs. Increase in Operating Current Assets: Debtors Cash Generated from Operation Less: Income Tax Paid5 Net Cash Flow from Operating Activities B. Cash Flows from Investing Activities: Proceeds from Sale of Plant & Machinery1 Proceeds from Sale of Investment2

Rs.

Rs.

Less:

1,27,000 4,94,000 71,000 4,23,000 36,000 35,000

Purchase of Plant & Machinery for Cash1 Net Cash Flow from Investing Activities C. Cash Flows from Financing Activities: New Issue of Preference Shares for Cash7 Premium Received on New Issue of Preference Shares7

Less:

Less:

Add:

Redemption of Debentures at Premium3 Payment of Dividend4 Payment of Debenture Interest6 Net Cash Flow from Financing Activities Net increase in Cash & Cash Equivalents Cash & Cash Equivalents at the beginning of the year Cash & Cash Equivalents at the end of the year

71,000 2,25,000 (1,54,000)

1,00,000 25,000

1,25,000

2,20,000 1,36,000 33,000

3,89,000 (2,64,000) 5,000 88,000 93,000

Working Notes 1. Dr.

Plant & Machinery Account Rs. 6,00,000 By Bank – Sale Proceeds of Machine 2,25,000 By Profit & Loss A/c – Loss on Sale of Machine By Profit & Loss A/c – Depreciation for the year [20% on 6,00,000] By Balance c/f 8,25,000

To Balance b/f To Bank – New Purchase (Bal. fig.)

Cr. Rs. 36,000 9,000 1,20,000 6,60,000 8,25,000

2. Dr. To Balance b/f To Profit & Loss A/c – Profit on Sale of Investment

Investment Account Rs. 2,40,000 By Bank – Sale Proceeds of Investment (Bal. fig.) 15,000 By Balance c/f

2,20,000

2,55,000

2,55,000

Cr. Rs. 35,000

3. Dr. To Bank – Debentures Redeemed at Premium (Bal. fig.) To Balance c/f

11% Debenture Account Rs. 2,20,000 By Balance b/f 3,00,000

By Premium on Redemption A/c – Premium paid on Redemption

5,20,000

Cr. Rs. 5,00,000 20,000 5,20,000 (Continued)

Modified Date: Sat, Jul 03, 2010 12:44:29 PM

Output Date: Tue, Jul 06, 2010 11:47:54 AM

Rev II

Project: Management Accounting_Debarshi Bhattacharyya ACE Pro India Pvt. Ltd. File: X:\Pearson\Management Accounting_Debarshi Bhattacharyya\MAIN\M04\LAYOUT_M04\M04_DEBA_ISBN_EN_SE_C04_III.indd

306

MANAGEMENT ACCOUNTING

4. Dr.

Proposed Dividend Account Rs. To Bank – Dividend for 2006–07 Paid in 2007–08 1,36,000 By Balance b/f To Balance c/f 1,44,000 By Profit & Loss Appropriation A/c – Dividend Proposed in 2007–08 (Bal. fig.) 2,80,000

Cr. Rs. 1,36,000 1,44,000 2,80,000

5. Dr. To Bank – Tax Paid in 2007–08 (Bal. fig.) To Balance c/f

Provision for Tax Account Rs. 71,000 By Balance b/f 1,05,000 By Profit & Loss A/c – Tax Provided in 2007–08 1,76,000

Cr. Rs. 80,000 96,000 1,76,000

6. It has been assumed that debentures were redeemed at the beginning of 2007–08. That is why, interest was paid on the remaining debentures, that is, 11% on Rs. 3,00,000 = Rs. 33,000. 7. i. Analysis of other Non-current Assets and Liabilities Assets/Liabilities 8% Preference Share Capital Securities Premium Profit & Loss A/c Goodwill Land & Building General Reserve

Opening Balance Rs. 2,00,000 

Closing Balance Rs. 3,00,000 25,000

Increase/ Decrease Rs. (+)1,00,000 (+)25,000

2,10,000 1,00,000 7,00,000 1,20,000

3,00,000 80,000 6,50,000 1,45,000

(+)90,000 (−)20,000 (−)50,000 (+)25,000

Analysis New Issue for Cash Premium Received on New Issue of Preference Shares Net Profit for the year Written off against Profit & Loss A/c Depreciation for the year Transfer from Profit

ii. Statement of changes in Working Capital

Particulars Current Assets: Stock Debtors Prepaid Expenses Cash & Bank Total (A) Current Liabilities: Creditors Total (B) Working Capital (A  B) Increase in Working Capital

Modified Date: Sat, Jul 03, 2010 12:44:29 PM

Change in Working Capital Increase Decrease Rs. Rs.

As on 31 March 2007 Rs.

As on 31 March 2008 Rs.

4,00,000 2,88,000 15,000 88,000 7,91,000

3,85,000 4,15,000 11,000 93,000 9,04,000

 1,27,000  5,000

15,000  4,000 

1,85,000 1,85,000 6,06,000 83,000 6,89,000

2,15,000 2,15,000 6,89,000  6,89,000



30,000

Output Date: Tue, Jul 06, 2010 11:47:54 AM

 1,32,000

83,000 1,32,000

Rev II

Project: Management Accounting_Debarshi Bhattacharyya ACE Pro India Pvt. Ltd. File: X:\Pearson\Management Accounting_Debarshi Bhattacharyya\MAIN\M04\LAYOUT_M04\M04_DEBA_ISBN_EN_SE_C04_III.indd

CASH FLOW ANALYSIS

307

Problem 45 XYZ Co. Ltd’s Comparative Balance Sheet for 2002 and the company’s income statement for the year are as follows: Income Statement for the year that ended on 31 December 2002

Less: Less: Less:

Less:

(Rs. in Crores) 1,000 530 470 352 118

Sales Cost of Goods Sold Gross Margin Operating Expenses Net Operating Income Non-operating Items: Loss on Sale of Equipment Income before Taxes Income Tax Net Income

4 114 48 66

Comparative Balance Sheet for the years that ended on 31 December 2002 and 31 December 2001 (Rs. in crores) 2002 Sources of Fund: Shareholder’s Fund: Share Capital Retained Earnings Loan Fund: Bonds Payable

Less:

Applications of Fund: Plant & Equipment Accumulated Depreciation Investments Current Assets: Inventory Accounts Receivable Prepaid Expenses Cash

Less:

Current Liabilities & Provisions: Accounts Payable Accrued Liabilities Deferred Tax Provision

2001

140 110 250

140 92 232

135 385

40 272

430 218 212 60

309 194 115 75

205 180 17 26 428

160 270 20 10 460

(230) (70) (15)

(310) (60) (8) 113 385

82 272

Additional Information: i. Dividends of Rs. 48 crores were paid in 2002. ii. The loss on sale of Rs. 4 crores reflects a transaction in which the equipment, with original cost of Rs. 12 crores and accumulated depreciation of Rs. 5 crores, was sold for Rs. 3 crores in cash. Required: Using the indirect method, determine the net cash provided by operating activities for 2002 and construct a statement of Cash Flows. [C.A. (PE II)—May 2003]

Modified Date: Sat, Jul 03, 2010 12:44:29 PM

Output Date: Tue, Jul 06, 2010 11:47:54 AM

Rev II

Project: Management Accounting_Debarshi Bhattacharyya ACE Pro India Pvt. Ltd. File: X:\Pearson\Management Accounting_Debarshi Bhattacharyya\MAIN\M04\LAYOUT_M04\M04_DEBA_ISBN_EN_SE_C04_III.indd

308

MANAGEMENT ACCOUNTING

Solution Cash Flow Statement (Indirect Method) of XYZ Co. Ltd for the year that ended on 31 December 2002 Rs. in Crores A. Cash Flows from Operating Activities: Net Income after Tax for the year Add: Adjustment for Non-operating & Non-current Items Debited to Profit & Loss A/c: Loss on Sale of Equipment Depreciation on Plant & Equipment2 Provision for Tax Operating Profit before Working Capital Changes Add: Increase in Operating Current Liabilities: Deferred Tax Provision Accrued Liabilities Decrease in Operating Current Assets: Prepaid Expenses Accounts Receivable Decrease in Operating Current Liabilities: Accounts Payable Increase in Operating Current Assets: Inventory Cash Generated from Operation Less: Income Tax paid Net Cash Flow from Operating Activities B. Cash Flows from Investing Activities: Proceeds from Sale of Equipment Proceeds from Sale of Investment3

66

4 29 48

81 147

7 10 3 90

110 257

Less:

Plant & Equipment Acquired for Cash1 Net Cash Flow from Investing Activities C. Cash Flows from Financing Activities: Proceeds Received from New Issue of Bonds4 Less: Payment of Cash Dividend Net Cash Flow from Financing Activities Net increase in Cash & Cash Equivalents Add: Cash & Cash Equivalents at the beginning of the year Cash & Cash Equivalents at the end of the year Less:

80 45

125 132 48 84 3 15 18 133 (115) 95 48 47 16 10 26

Working Notes 1. Dr. To Balance b/f

To Bank – New Purchase (Bal. fig.)

Plant & Equipment (at cost) Account Rs. in crore 309 By Provision for Depreciation A/c – Accumulated Depreciation on Equipment Sold 133 By Bank – Sale Proceeds of Equipment By Profit & Loss A/c – Loss on Sale of Equipment By Balance c/f 442

Cr. Rs. in crore 5

3 4 430 442 (Continued)

Modified Date: Sat, Jul 03, 2010 12:44:29 PM

Output Date: Tue, Jul 06, 2010 11:47:54 AM

Rev II

Project: Management Accounting_Debarshi Bhattacharyya ACE Pro India Pvt. Ltd. File: X:\Pearson\Management Accounting_Debarshi Bhattacharyya\MAIN\M04\LAYOUT_M04\M04_DEBA_ISBN_EN_SE_C04_III.indd

309

CASH FLOW ANALYSIS

2. Dr.

Provision for Depreciation on Plant & Equipment Account Rs. in crore To Plant & Equipment A/c – Accumulated 5 By Balance b/f Depreciation on Equipment Sold To Balance c/f 218 By Profit & Loss A/c – Depreciation for the year (Bal. fig.) 223

Cr. Rs. in crore 194 29 223

3. Dr. To Balance b/f

Investment Account Rs. in crore 75 By Bank – Sale Proceeds (Bal. fig.) By Balance c/f 75

Cr. Rs. in crore 15 60 75

Bonds Payable Account Rs. in crore By Balance b/f By Bank A/c 135 – Bonds Issued (Bal. fig.) 135

Cr. Rs. in crore 40 95

Retained Earning Account Rs. in crore 48 By Balance b/f 110 By Profit & Loss A/c – Provision made in 2005–06 158

Cr. Rs. in crore 92 66

4. Dr.

To Balance c/f

135

5. Dr. To Dividend – Dividend Declared & Paid To Balance c/f

158

Problem 46 From the following information provided, prepare a Cash Flow Statement as per AS-3 and comment on the financial position of the company. Balance Sheets as on Liabilities Equity Share Capital Securities Premium Profit & Loss A/c 10% Debentures Bank Overdraft Creditors Proposed Dividend Provision for Depreciation on: Plant & Machinery Furniture

31 March 2007 Rs. 1,00,000 15,000 28,000 70,000 14,000 34,000 15,000

45,000 13,000 3,34,000

Modified Date: Sat, Jul 03, 2010 12:44:29 PM

31 March 2008 Rs. 1,50,000 35,000 70,000 30,000 48,000 20,000

Assets Freehold Property Plant & Machinery Furniture Stock Debtors Bank Premium on Redemption of Debentures

54,000 15,000 4,22,000

Output Date: Tue, Jul 06, 2010 11:47:54 AM

31 March 2007 Rs. 1,10,000 1,20,000 24,000 37,000 43,000

31 March 2008 Rs. 1,30,000 1,51 ,000 29,000 51,000 44,000 16,000 1,000

3,34,000

4,22,000

Rev II

Project: Management Accounting_Debarshi Bhattacharyya ACE Pro India Pvt. Ltd. File: X:\Pearson\Management Accounting_Debarshi Bhattacharyya\MAIN\M04\LAYOUT_M04\M04_DEBA_ISBN_EN_SE_C04_III.indd

310

MANAGEMENT ACCOUNTING

Additional Information: i. There had been no disposal of freehold propery in the year. ii. The machine, which had cost Rs. 8,000 and in respect of which Rs. 6,000 depreciation had been provided, was sold for Rs. 3,000, and furniture, which had cost Rs. 5,000 and in respect of which a depreciation of Rs. 2,000 had been provided, was sold for Rs. 1,000. The profits and losses on these transactions had been dealt with through Profit & Loss A/c. iii. Actual premium on the redemption of debentures was Rs. 2,000, of which Rs. 1,000 had been written off to the Profit & Loss A/c. iv. No interim dividend has been paid. [B.Com. (Hons), Calcutta University—2009] Solution Cash Flow Statement (under Indirect Method) of

for the year that ended on 31 March 2008 Rs.

A. Cash Flows from Operating Activities: Net Profit (after appropriation) for the year8 Add: Adjustment for Non-operating & Non-current Items Debited to Profit & Loss A/c: Proposed Dividend7 Depreciation on Furniture4 Depreciation on Plant & Machinery3 Premium on Redemption of Debentures5 Loss on Sale of Furniture3 Less:

Add:

Rs.

42,000

20,000 4,000 15,000 1,000 2,000

Adjustment for Non-operating & Non-current Items Credited to Profit & Loss A/c: Profit on Sale of Machinery1 Operating Profit before Working Capital Changes Increase in Operating Current Liabilities: Creditors

Increase in Operating Current Assets: Debtors Stock Cash Generated from Operation Less: Income-tax Paid7 Net Cash Flow from Operating Activities B. Cash Flows from Investing Activities: Proceeds from Sale of Machinery1 Proceeds from Sale of Furniture3

Rs.

42,000 84,000

1,000 83,000 14,000 97,000

Less:

Less:

Purchase of Plant & Machinery for Cash1 Purchase of Furniture for Cash3 Purchase of Freehold Property for Cash8

1,000 14,000

15,000 82,000 Nil 82,000 3,000 1,000 4,000

39,000 10,000 20,000 69,000

Net Cash Flow from Investing Activities C. Cash Flows from Financing Activities: New Issue of Equity Shares for Cash8 Premium Received from New Issue of Equity Shares8

(65,000) 50,000 20,000 70,000 (Continued)

Modified Date: Sat, Jul 03, 2010 12:44:29 PM

Output Date: Tue, Jul 06, 2010 11:47:54 AM

Rev II

Project: Management Accounting_Debarshi Bhattacharyya ACE Pro India Pvt. Ltd. File: X:\Pearson\Management Accounting_Debarshi Bhattacharyya\MAIN\M04\LAYOUT_M04\M04_DEBA_ISBN_EN_SE_C04_III.indd

311

CASH FLOW ANALYSIS

Less:

Redemption of Debentures at Premium6 Payment of Dividend7

Rs. 42,000 15,000

Rs.

Rs.

57,000

Add:

Net Cash Flow from Financing Activities Net increase in Cash & Cash Equivalents Cash & Cash Equivalents at the beginning of the year Cash & Cash Equivalents at the end of the year

13,000 30,000 (14,000) 16,000

Working Notes 1. Dr.

Plant & Machinery (at cost) Account Rs. To Balance b/f 1,20,000 By Provision for Depreciation A/c – Accumulated Depreciation on Machinery Sold To Bank – New Purchase (Bal. fig.) 39,000 By Bank – Sale Proceeds of Machinery To Profit & Loss A/c – Profit on Sale of 1,000 By Balance c/f Machinery [3,000 – (8,000 – 6,000)] 1,60,000

Cr. Rs. 6,000 3,000 1,51,000 1,60,000

2. Dr.

Provision for Depreciation on Plant & Machinery Account Rs. To Plant-&-Machinery A/c 6,000 By Balance b/f – Accumulated Depreciation on Machinery Sold To Balance c/f 54,000 By Profit & Loss A/c – Depreciation for the year (Bal. fig.) 60,000

Cr. Rs. 45,000

15,000 60,000

3. Dr. To Balance b/f

To Bank – New Purchase (Bal. fig.)

Furniture (at cost) Account Rs. 24,000 By Provision for Depreciation A/c – Accumulated Depreciation on Furniture Sold 10,000 By Bank – Sale Proceeds of Furniture By Profit & Loss A/c – Loss on Sale of Furniture [(5,000 – 2,000) – 1,000] By Balance c/f 34,000

Cr. Rs. 2,000

1,000 2,000 29,000 34,000

4. Dr.

Provision for Depreciation on Furniture Account Rs. To Furniture A/c – Accumulated 2,000 By Balance b/f Depreciation on Furniture Sold To Balance c/f 15,000 By Profit & Loss A/c – Depreciation for the year (Bal. fig.) 17,000

Cr. Rs. 13,000 4,000 17,000 (Continued)

Modified Date: Sat, Jul 03, 2010 12:44:29 PM

Output Date: Tue, Jul 06, 2010 11:47:54 AM

Rev II

Project: Management Accounting_Debarshi Bhattacharyya ACE Pro India Pvt. Ltd. File: X:\Pearson\Management Accounting_Debarshi Bhattacharyya\MAIN\M04\LAYOUT_M04\M04_DEBA_ISBN_EN_SE_C04_III.indd

312

MANAGEMENT ACCOUNTING

5. Dr.

Premium on Redemption of Debenture Account Rs. To Balance b/f Nil By Profit & Loss A/c – Premium written off To 10% Debenture A/c – Premium paid on 2,000 By Balance c/f Redemption 2,000

Cr. Rs. 1,000 1,000 2,000

6. Dr. To Bank – Debentures Redeemed at Premium (Bal. fig.) To Balance c/f

10% Debenture Account Rs. 42,000 By Balance b/f 30,000

Cr. Rs. 70,000

By Premium on Redemption A/c – Premium paid on Redemption

72,000

2,000 72,000

7. Dr.

Proposed-Dividend Account Rs. To Bank – Dividend for 2006–07 paid in 15,000 By Balance b/f 2007–08 To Balance c/f 20,000 By Profit & Loss Appropriation A/c – Dividend Proposed in 2007–08 (Bal. fig.) 35,000

Cr. Rs. 15,000 20,000 35,000

8. Analysis of other Non-current Assets and Liabilities Assets/Liabilities Equity Share Capital Securities Premium Profit & Loss A/c Freehold property

Opening Balance Rs. 1,00,000 15,000

Closing Balance Rs. 1,50,000 35,000

Increase/ Decrease Rs. (+)50,000 (+)20,000

28,000

70,000

(+)42,000

1,10,000

1,30,000

(+)20,000

Analysis New Issue for Cash Premium Received on New Issue of Equity Shares Net Profit for the year after Appropriation New Purchase for Cash

Problem 47 The following figures have been extracted from the books of X Ltd for the year that ended on 31 March 2004. You are required to prepare a Cash Flow Statement. i. Net profit, before taking into account the income tax and income from law suits but after taking into account the following items, was Rs. 20 lakhs: a. Depreciation on Fixed Assets – Rs. 5 lakhs. b. Discount on issue of debentures written off – Rs. 30,000. c. Interest on debentures paid – Rs. 3,50,000. d. Book value of investment – Rs. 3 lakhs (sale of investment for Rs. 3,20,000).

Modified Date: Sat, Jul 03, 2010 12:44:29 PM

Output Date: Tue, Jul 06, 2010 11:47:54 AM

Rev II

Project: Management Accounting_Debarshi Bhattacharyya ACE Pro India Pvt. Ltd. File: X:\Pearson\Management Accounting_Debarshi Bhattacharyya\MAIN\M04\LAYOUT_M04\M04_DEBA_ISBN_EN_SE_C04_III.indd

313

CASH FLOW ANALYSIS

ii. iii.

iv. v. vi.

e. Interest received on investment – Rs. 60,000. f. Compensation received was Rs. 90,000 by the company in a suit filed. Income tax paid during the year – Rs. 10,50,000. 15,000, 10% preference shares of Rs. 100 each were redeemed on 31 March 2004 at a premium of 5%. Further the company issued 50,000 equity shares of Rs. 10 each at a premium of 20% on 2 April 2003. Dividend on preference shares were paid at the time of redemption. Dividends paid for the year 2002–03 was Rs. 5 lakhs and interim dividend paid was Rs. 3 lakhs for the year 2003–04. Land was purchased on 2 April 2003 for Rs. 2,40,000, for which the company issued 20,000 equity shares of Rs. 10 each at a premium of 20% to the land owner as consideration. Current Assets and Current Liabilities at the beginning and at the end of the year were detailed as follows: 31 March 2003 Rs. 12,00,000 2,08,000 1,96,300 50,000 45,000 1,66,000 75,000

Stock Sundry Debtors Cash in Hand Bills Receivable Bills Payable Sundry Creditors Outstanding Expenses

31 March 2004 Rs. 13,18,000 2,13,000 35,300 40,000 40,000 1,71,300 81,800

[C.A. (PE II)—May 2005] Solution Cash Flow Statement (under indirect method) of X Ltd for the year that ended on 31 March 2004 Rs. A. Cash Flows from Operating Activities: Net Profit before Income Tax & Extraordinary Item Add: Adjustment for Non-operating & Non-current Items debited to Profit & Loss A/c: Depreciation on Fixed Asset Interest Paid on Debentures Discount on Issue of Debentures Less:

Add:

Adjustment for Non-operating & Non-current Items credited to Profit & Loss A/c: Profit on Sale of Investment Interest Received on Investment Operating Profit before Working Capital changes Increase in Operating Current Liabilities: Sundry Creditors Outstanding Expenses Decrease in Operating Current Assets: Bills Receivable

Rs.

Rs.

20,00,000

5,00,000 3,50,000 30,000

20,000 60,000

8,80,000 28,80,000

80,000 28,00,000

5,300 6,800 10,000

22,100 (Continued)

Modified Date: Sat, Jul 03, 2010 12:44:29 PM

Output Date: Tue, Jul 06, 2010 11:47:54 AM

Rev II

Project: Management Accounting_Debarshi Bhattacharyya ACE Pro India Pvt. Ltd. File: X:\Pearson\Management Accounting_Debarshi Bhattacharyya\MAIN\M04\LAYOUT_M04\M04_DEBA_ISBN_EN_SE_C04_III.indd

314

MANAGEMENT ACCOUNTING

Rs. Decrease in Operating Current Liabilities: Bills Payable Increase in Operating Current Assets: Sundry Debtors Stock Cash Generated from Operation Less: Income Tax Paid Cash Flow from Operation before Extraordinary Item Add: Cash Flow from Extraordinary Item: Compensation Received in a suit filed Net Cash Flow from Operating Activities B. Cash Flows from Investing Activities: Proceeds Received from Sale of Investment Interest Received from Investment Net Cash Flow from Investing Activities C. Cash Flows from Financing Activities: Proceeds Received from Issue of Equity Shares at a Premium Less: Redemption of Preference Shares at a Premium Payment of Preference Dividend Payment of Debenture Interest Payment of Equity Dividend

Rs. 28,22,100

Rs.

Less:

5,000 5,100 1,18,000

1,28,100 26,94,000 10,50,000 16,44,000 90,000 17,34,000 3,20,000 60,000 3,80,000 6,00,000

15,75,000 1,50,000 3,50,000 8,00,000 28,75,000

Add:

Net Cash Flow from Financing Activities Net Increase in Cash & Cash Equivalents Cash & Cash Equivalents at the beginning of the year Cash & Cash Equivalents at the end of the year

(22,75,000) (1,61,000) 1,96,300 35,300

CHAPTER REVIEW SUMMARY  Cash Flow Statement is a summarized statement showing sources of cash inflows and applications of cash outflows of an enterprise during a particular period of time (generally, at the end of an accounting year). It is used as an essential tool for managerial tool. It is prepared to locate the various sources of cash inflows within the business and to identify the various uses of cash outflows from the business during a particular period.  It is prepared to fulfil some objectives of an enterprise, such as: (a) Prediction of future cash flows; (b) Shortterm financial planning; (c) Liquidity and solvency position; (d) Disclosure of movement of cash; (e) Efficiency in cash management; (f) Identification of stronger or weaker activity; (g) Efficiency in managerial performance; and (h) A tool of comparative study.  Cash Flow Statement has many advantages, such as: (a) It provides adequate information as regards to inflows and outflows of cash resources to and from the enterprise; (b) It provides separate information as regards to inflows and outflows of cash from the different activities of the enterprise; (c) It evaluates the level of efficiency of the management of the enterprise as regards to uses of its cash resources; (d) It discloses the liquidity and solvency position of the enterprise; (e) It helps to prepare the budget for the future period.  In spite of its immense usefulness, it has its own limitations too, such as: (a) It shows only the inflows and outflows of cash; and thus, it does not take into consideration the non-cash transactions of the enterprise;

Modified Date: Sat, Jul 03, 2010 12:44:29 PM

Output Date: Tue, Jul 06, 2010 11:47:54 AM

Rev II

Project: Management Accounting_Debarshi Bhattacharyya ACE Pro India Pvt. Ltd. File: X:\Pearson\Management Accounting_Debarshi Bhattacharyya\MAIN\M04\LAYOUT_M04\M04_DEBA_ISBN_EN_SE_C04_III.indd

CASH FLOW ANALYSIS



 

315

(b) As it discloses the Net Cash Flow, not the Net Profit for a certain period, it is not a substitute of the Profit & Loss A/c; (c) As it is prepared; at the end of a certain period, it is not useful for rectifying the errors that already took place during that period; and (d) It is not also a substitute of Fund Flow Statement as it discloses cash alone. As per conventional/traditional approach, ‘cash’ refers to Cash in hand and Cash at bank only, but as per AS-3 (Revised), ‘cash’ refers to cash and cash equivalents (i.e., Cash in hand, Cash at bank and Short-term investments/Marketable securities. As per AS-3 (Revised), a Cash Flow Statement may be presented under two methods, that is, Direct Method and Indirect Method. A Cash Flow Statement prepared under both the above methods is subdivided into the following three parts: i. Net Cash Flows from Operating Activities. ii. Net Cash Flows from Investing Activities. iii. Net Cash Flows from Financing Activities.









Cash flows to and from the principal business activities of an enterprise other than investing and Financing Activities are called ‘cash flows from Operating Activities.’ For example, cash sale of goods/services, cash purchases of goods/services and so on. Cash flows from Investing Activities arise out of acquisition and sale of Fixed Assets and long-term investments made outside the enterprise. For example, cash purchase of Fixed Assets, cash sale of Fixed Assets, and so on. Cash flows from Financing Activities are movement of cash of an enterprise which result in changes in size and composition of the owner’s capital (including Preference Share Capital) and borrowings of that enterprise. For example, issue of shares in cash, repayment of long-term loans in cash, and so on. Cash flows from different activities of an enterprise, such as operating, investing and Financing Activities, can be separately known through the Cash Flow Statement prepared as per AS-3, by virtue of which the enterprise can identify its stronger and weaker area of cash generation. But, as per traditional method, there are no such segregations of cash flows from the different activities of an enterprise.

CHAPTER REVIEW QUIZ 1. Classify the following items into: (a) Cash flow from operating activities, (b) Cash flow from investing activities; and (c) Cash flow from financing activities: (i) Purchase of machinery; (ii) Purchase of goods; (iii) Sale of long-term investments; (iv) Sale of goods; (v) Issue of equity shares; (vi) Redemption of preference shares; (vii) Receipt of interest on investment; (viii) Payment of interest on debentures; (ix) Payment of dividend; (x) Issue of debentures; (xi) Repayment of long-term loan; (xii) Payment of income tax on operational profit; (xiii) Payment of administration expenses; (xiv) Payment of wages; and (xv) Sale of building. Ans.: (a) (ii), (iv), (xii), (xiii), (xiv); (b) (i), (iii), (vii), (xv); (c) (v), (vi), (viii), (ix), (x), (xi). 2. How would you deal with the following items while calculating the Cash Flow from Operating Activities from the given figure of net profit earned during a year: a. Increase in creditors for goods. b. Increase in creditors for furniture. c. Decrease in debtors for goods. d. Increase in Stock in Trade. e. Decrease in Bills Payable. f. Decrease in Bills Receivable. g. Increase in Outstanding Expenses. h. Increase in Prepaid Expenses. i. Issue of Preference Shares.

Modified Date: Sat, Jul 03, 2010 12:44:29 PM

Output Date: Tue, Jul 06, 2010 11:47:54 AM

Rev II

Project: Management Accounting_Debarshi Bhattacharyya ACE Pro India Pvt. Ltd. File: X:\Pearson\Management Accounting_Debarshi Bhattacharyya\MAIN\M04\LAYOUT_M04\M04_DEBA_ISBN_EN_SE_C04_III.indd

316

MANAGEMENT ACCOUNTING

j. Redemption of Debentures. k. Increase in Cash Balance. l. Decrease in Short-term Investment. m. Purchase of Machinery. n. Payment of Dividend. o. Receipt of Interest on Investment. Ans.: To be added: (a), (c), (f), (g); To be deducted: (d), (e), (h); No effect: (b), (I), (j), (k), (l), (m), (n), (o). 3. Choose the correct alternative from the following: a. Cash and cash equivalent refers to: (i) Cash in hand; (ii) Cash at bank; (iii) Cash in hand & at bank; (iv) Cash in hand & at bank and Short-term Investments. b. Cash purchases of raw materials used in production causes cash outflow from: (i) operating activities; (ii) investing activities; (iii) financing activities; (iv) none of these. c. Interest received from Long-term Investment in Debentures of a company causes cash inflow from: (i) Operating Activities; (ii) Investing Activities; (iii) Financing Activities; (iv) none of these. d. Interest paid on Long-term Loan causes cash outflow from: (i) Operating Activities; (ii) Investing Activities; (iii) Financing Activities; (iv) none of these. e. Redemption of Preference Share causes cash outflow from: (i) Operating Activities; (ii) Investing Activities; (iii) Financing Activities; (iv) none of these. f. Payment of Administrative and Selling Overheads causes cash outflow from: (i) Operating Activities; (ii) Investing Activities; (iii) Financing Activities; (iv) none of these. g. Depreciation on Fixed Assets: (i) causes cash inflow; (ii) causes cash outflow; (iii) has no effect on cash flow. h. Sale of Fixed Assets causes cash inflow from: (i) Operating Activities; (ii) Investing Activities; (iii) Financing Activities; (iv) none of these. i. Repayment of borrowing causes cash outflow from: (i) Operating Activities; (ii) Investing Activities; (iii) Financing Activities; (iv) none of these. j. Amortization of patent right: (i) causes cash inflow; (ii) causes cash outflow; (iii) has no effect on cash flow. k. If net profit of a business concern is Rs. 1,40,000 and its creditors have gone up by Rs. 15,000 during the year, cash from operation is equal to: (i) Rs. 1,55,000; (ii) Rs. 1,25,000; (iii) Rs. 1,40,000. l. If the net profit of a business concern is Rs. 75,000 and its debtors have gone down by Rs. 20,000 during the year, the cash from operation is equal to: (i) Rs. 95,000; (ii) Rs. 55,000; (iii) Rs. 75,000. Ans.: (a) (iv); (b) (i); (c) (ii); (d) (iii); (e) (iii); (f) (i); (g) (iii); (h) (ii); (i) (iii); (j) (iii); (k) (i); (l) (ii). 4. State whether the following statements are true or false: a. Cash Flow Statement is a part of Financial Statements. b. Cash Flow Statement is a statement of sources and applications of cash during a particular period of time. c. Difference between sources and applications of cash may increase or decrease in a working capital. d. Cash flow signifies only the causes of cash variations. e. Cash Flow Statement is a technique of financial forecasting. f. Cash flows are inflows and outflows of cash and cash equivalents. g. Cash Flow Statement should report cash flows from operating and investing activities during a particular period of time. h. Payment of dividend on Share Capital is an example of cash outflow from investing activities. i. Fund Flow Statement presents a more complete picture than a Cash Flow Statement. j. Investing activities are the acquisition and disposal of long-term assets and other investments not included in cash equivalents. Ans.: True: (a), (b), (d), (f), (i), (j); False: (c), (e), (g), (h).

Modified Date: Sat, Jul 03, 2010 12:44:29 PM

Output Date: Tue, Jul 06, 2010 11:47:54 AM

Rev II

Project: Management Accounting_Debarshi Bhattacharyya ACE Pro India Pvt. Ltd. File: X:\Pearson\Management Accounting_Debarshi Bhattacharyya\MAIN\M04\LAYOUT_M04\M04_DEBA_ISBN_EN_SE_C04_III.indd

CASH FLOW ANALYSIS

317

EXERCISE I. Theoretical Questions A. Short Answer Type Questions

1. 2. 3. 4. 5. 6. 7. 8.

What is meant by Cash? What is Cash Equivalent? What is Cash Flow? What is the difference between Cash and Fund? What is meant by Cash Flow Statement? What is meant by Operating Activities? What is meant by Investing Activities? What is meant by Financing Activities?

B. Essay Type Questions

1. 2. 3. 4.

Define Cash Flow Statement. What are its uses? What are the advantages and limitations of Cash Flow Statement? Why is Cash Flow Statement prepared? What is its importance? Distinguish between: a. Cash Flow Statement and Fund Flow Statement. b. Cash Flow Statement and Cash Budget. c. Cash Flow Statement and Cash Book. 5. Discuss briefly the classification of activities as prescribed in AS-3 for preparation of Cash Flow Statement and give three examples of each such class of activities. 6. Give the proforma of a Cash Flow Statement as suggested in AS-3. 7. Discuss the fundamental differences between the Cash Flow Statement as prescribed in AS-3 and that under traditional approach. II. Practical Problems

I. Calculation of Net Cash Flow from operation under conventional/ traditional approach. 1. From the following information, calculate the Net Cash Flow from operation of a company for the year that ended on 31 March 2009 under traditional approach: 31 March 2008 Rs. 4,20,000 75,000 92,000 63,000 22,000 18,000 10,000 12,000 42,000

Balance in Profit & Loss A/c Stock in Trade Sundry Debtors Sundry Creditors Bills Receivable Bills Payable Outstanding Expenses Prepaid Expenses Cash at Bank

31 March 2009 Rs. 6,80,000 55,000 1,24,000 91,000 34,000 15,000 15,000 7,000 71,000

While ascertaining the Net Profit for the year that ended on 31 March 2009, the following items were taken into the Profit & Loss A/c: Rs. 36,000 24,000

Transfer to General Reserve Proposed Dividend

(Continued)

Modified Date: Sat, Jul 03, 2010 12:44:29 PM

Output Date: Tue, Jul 06, 2010 11:47:54 AM

Rev II

Project: Management Accounting_Debarshi Bhattacharyya ACE Pro India Pvt. Ltd. File: X:\Pearson\Management Accounting_Debarshi Bhattacharyya\MAIN\M04\LAYOUT_M04\M04_DEBA_ISBN_EN_SE_C04_III.indd

318

MANAGEMENT ACCOUNTING

Rs. 18,000 7,000 12,000 28,000 22,000 12,000 18,000 8,000 5,000

Goodwill written off Preliminary Expenses written off Interest on Debentures Depreciation on Fixed Assets Provision for Taxation Loss on Sale of Investment Profit on Sale of Machinery Income from Investment Discount on Issue of Shares

Ans.: Rs. 4,09,000. II. Calculation of Net Cash Flow from operating activities as per AS-3 2. From the information given in the following relating to Q Ltd, calculate cash flow from operating activities: Operating Profit before Changes in Operating Assets Stock (Decrease) Debtors (Increase) Creditors (Decrease) Bills Payable (Increase) Outstanding Expenses (Decrease) Prepaid Expenses (Increase) Cash at Bank (Increase)

Rs. 76,000 Rs. 18,000 Rs. 14,000 Rs. 22,000 Rs. 6,000 Rs. 4,000 Rs. 5,000 Rs. 37,000

[B.Com. (Hons), Calcutta University—Adapted] Ans.: Rs. 55,000. 3. Compute the Net Cash Flow from the operating activities, from the following details, by indirect method: Particulars Profit & Loss A/c Debtors Outstanding Rent Goodwill Prepaid Insurance Creditors Preliminary Expenses

2008 Rs. 1,50,000 70,000 18,000 20,000 14,000 42,000 8,000

2009 Rs. 1,40,000 94,000 34,000 10,000 9,000 56,000 6,000

2007–08 Rs. 7,000 24,000 39,000 18,000 47,000

2008–09 Rs. 12,000 42,000 33,000 21,000 29,000

Ans.: 13,000. 4. The following is the position of Current Assets and Current Liabilities of Dee Ltd: Particulars Provision for Bad Debts Short-term Loan Creditors Bills Receivable Debtors

The company incurred a loss of Rs. 27,000 during the year. Calculate the Net Cash Flows from the Operating Activities by Indirect Method. Ans.: Rs. 5,000.

Modified Date: Sat, Jul 03, 2010 12:44:29 PM

Output Date: Tue, Jul 06, 2010 11:47:54 AM

Rev II

Project: Management Accounting_Debarshi Bhattacharyya ACE Pro India Pvt. Ltd. File: X:\Pearson\Management Accounting_Debarshi Bhattacharyya\MAIN\M04\LAYOUT_M04\M04_DEBA_ISBN_EN_SE_C04_III.indd

CASH FLOW ANALYSIS

319

5. From the following you are required to calculate the Net Cash Flow from the operating activities by indirect method: Particulars Balance of Profit & Loss A/c Debtors Bills Receivable General Reserve Dividend Equalization Fund Salary Outstanding Wages Prepaid Goodwill Creditors

31 March 2008 Rs. 76,000 72,000 17,000 1,20,000 62,000 15,000 6,000 75,000 38,000

31 March 2009 Rs. 98,000 51,000 53,000 1,60,000 87,000 7,000 10,000 60,000 47,000

Ans.: Rs. 1,04,000. 6. J Ltd made a profit of Rs. 2,25,000 after charging a depreciation of Rs. 42,000 on assets and a transfer to general reserve of Rs. 40,000. The goodwill written off was Rs. 12,000 and the gain on sale of machinery was Rs. 14,000. The other information available is (changes in the value of Current Assets and Current Liabilities) at the end of the year. Debtors showed an increase of Rs. 16,000; Creditors an increase of Rs. 15,000; Prepaid expenses an increase of Rs. 3,000; Bills Receivable a decrease of Rs. 5,000; Bills Payable a decrease of Rs. 6,000; Outstanding Expenses a decrease of Rs. 1,000 and Prepaid Expenses a decrease of Rs. 1,000. Ascertain the Net Cash Flow from the Operating Activities. [CBSE Examination—Adapted] Ans.: Rs. 3,00,000. 7. From the following information, calculate the Net Cash Flow from operating activities: Cash Sales Credit Sales Opening Debtors’ Balance Closing Debtors’ Balance Cash Purchases Credit Purchases Opening Creditors’ Balance Closing Creditors’ Balance Sales Return Bad Debt Discount Allowed Return Outward Discount Received Wages & Salaries Paid Wages & Salaries Outstanding Overhead Charges Paid Depreciation Interest on Investment Received Interest on Debentures Paid Income Tax Provided Income Tax Paid (of which Rs. 45,000 was paid for Operating Income and the balance for Investing Income)

Rs. 2,40,000 4,60,000 70,000 90,000 1,40,000 2,10,000 50,000 60,000 20,000 15,000 25,000 30,000 20,000 80,000 20,000 60,000 45,000 27,000 30,000 60,000 50,000

Ans.: Rs. 1,45,000.

Modified Date: Sat, Jul 03, 2010 12:44:29 PM

Output Date: Tue, Jul 06, 2010 11:47:54 AM

Rev II

Project: Management Accounting_Debarshi Bhattacharyya ACE Pro India Pvt. Ltd. File: X:\Pearson\Management Accounting_Debarshi Bhattacharyya\MAIN\M04\LAYOUT_M04\M04_DEBA_ISBN_EN_SE_C04_III.indd

320

MANAGEMENT ACCOUNTING

8. From the following information, calculate the Net Cash Flow from the operating activities by Indirect Method for the year that ended on 31 March 2009: Dr.

Profit & Loss A/c for the Year that Ended on 31 March 2009 Particulars Rs. 26,000 By Gross Profit 5,000 By Interest on Investment 15,000 By Dividend Received 64,000 By Profit on Sale of Plant 10,000 By Rent Received 20,000 By Refund of Tax 35,000 By Insurance Claim for earthquake 75,000 By Commission Receivable 25,000 5,75,000 8,50,000

Cr.

Particulars To Loss on Sale of Land To Discount on Issue of Shares written off To Interest on Debentures To Depreciation To Goodwill written off To General Reserve To Tax Provision To Proposed Dividend To Interim Dividend To Net Profit

Rs. 6,90,000 17,000 22,000 25,000 14,000 6,000 60,000 16,000

8,50,000

Additional Information: Particulars Debtors Creditors Stock Provision for Tax Accrued Commission Outstanding Wages Prepaid Expenses Proposed Dividend

31 March 2008 Rs. 57,000 33,000 74,000 45,000 9,000 23,000 15,000 65,000

31 March 2009 Rs. 89,000 66,000 51,000 40,000 15,000 29,000 24,000 85,000

Ans.: Rs. 7,61,000. 9. From the following information, calculate the Net Cash Flow from the operating activities of a concern for the year that ended on 31 March 2009: Cash Sales for the year Credit Sales for the year Collection from Debtors During the year Cash Purchases for the year Credit Purchases for the year Payment to Creditors during the year Wages Paid during the year Outstanding Wages for the year Salaries Paid during the year Salaries for the year General Expenses Paid during the year Unpaid General Expenses for the year Depreciation on Fixed Assets for the year Loss of Stock due to Fire Insurance Claim Received against Loss of Stock Interest Received on Investment during the year (of which Rs. 5,000 provided for Interest Income) Payment of Income Tax during the year (of which Rs. 3,000 paid for Interest Income)

Rs. 1,45,000 5,85,000 3,98,000 77,000 2,96,000 1,91,000 63,000 15,000 37,000 45,000 35,000 10,000 32,000 25,000 12,000 17,000 43,000

Ans.: Rs. 1,12,000.

Modified Date: Sat, Jul 03, 2010 12:44:29 PM

Output Date: Tue, Jul 06, 2010 11:47:54 AM

Rev II

Project: Management Accounting_Debarshi Bhattacharyya ACE Pro India Pvt. Ltd. File: X:\Pearson\Management Accounting_Debarshi Bhattacharyya\MAIN\M04\LAYOUT_M04\M04_DEBA_ISBN_EN_SE_C04_III.indd

321

CASH FLOW ANALYSIS

10. From the following information, calculate the Net Cash Flow from the Operating Activities of Sosa Ltd for the year that ended on 31 March 2009: Dr. To Opening Stock To Purchases: Cash Credit To Wages: Paid Outstanding To Salaries: Paid Outstanding To Rent Paid To Depreciation on Assets To Preliminary Expenses written off To Provision for Taxation To Net Profit for the year

Profit & Loss A/c for the Year that Ended on 31 March 2009 Rs. Rs. 62,000 By Sales: Cash 60,000 Credit 2,23,000 2,83,000 By Closing Stock 62,000 By Interest from Investment 6,000 68,000 32,000 3,000

Cr. Rs.

Rs.

93,000 3,94,000 4,87,000 52,000 11,000

35,000 8,000 16,000 7,000 48,000 23,000 5,50,000

5,50,000

Additional Information: 1. Balance of debtors and creditors were as follows: 1 April 2008 Rs. 46,000 49,000

Debtors Creditors

31 March 2009 Rs. 62,000 42,000

2. Tax paid during the year amounted to Rs. 45,000. Ans.: Rs. 34,000. 11. From the following particulars, calculate the Net Cash Flow from the operating activities of Hoo Ltd for the year that ended on 31 March 2009: Rs. Balance of Profit & Loss A/c: As on 31 March 2009 As on 31 March 2008 Appropriation of Profit for the Year 2008–09: Transfer to General Reserve Proposed Dividend Expenses and Losses for the Year 2008–09: Interest on Debentures Depreciation on Fixed Assets Wages & Salaries Provision for Taxation Goodwill written off Loss on Sale of Machinery Incomes and Gains for the Year 2008–09: Profit on Sale of Furniture Income from Investment

7,30,000 4,80,000 40,000 30,000 20,000 25,000 40,000 45,000 10,000 20,000 15,000 25,000 (Continued)

Modified Date: Sat, Jul 03, 2010 12:44:29 PM

Output Date: Tue, Jul 06, 2010 11:47:54 AM

Rev II

Project: Management Accounting_Debarshi Bhattacharyya ACE Pro India Pvt. Ltd. File: X:\Pearson\Management Accounting_Debarshi Bhattacharyya\MAIN\M04\LAYOUT_M04\M04_DEBA_ISBN_EN_SE_C04_III.indd

322

MANAGEMENT ACCOUNTING

Rs. Expenses Paid during the year 2008–09: Interest on Debentures Wages & Salaries Income Tax (of which Rs. 4,000 for Non-operating Income)

25,000 36,000 55,000

Balances of Current Assets and Current Liabilities were as follows: As on 31 As on 31 March 2008 March 2009 Rs. Rs. 44,000 63,000 38,000 55,000 49,000 42,000 8,000 6,000 7,000 4,000 26,000 41,000 5,000 8,000

Sundry Debtors Sundry Creditors Stock in Trade Bills Receivable Bills Payable Cash & Bank Accrued Income from Investment

Ans.: Rs. 3,90,000. 12. The financial position of A Ltd was as follows: Liabilities Equity Share Capital General Reserve Profit & Loss A/c Creditors Bills Payable Proposed Dividend Provision for Taxation

2001 Rs. 4,50,000 40,000 30,000 55,000 20,000 42,000 40,000 6,77,000

2002 Rs. 5,00,000 70,000 48,000 83,000 16,000 50,000 50,000 8,17,000

Assets Goodwill Land & Building Plant Stock Debtors Bills Receivable Cash

2001 Rs. 1,15,000 2,00,000 80,000 77,000 1,60,000 20,000 25,000 6,77,000

2002 Rs. 90,000 1,70,000 2,00,000 1,09,000 2,00,000 30,000 18,000 8,17,000

Additional Information: i. Depreciation of Rs. 15,000 and Rs. 25,000 had been charged on plant and land & building respectively. ii. An interim dividend of Rs. 20,000 had been paid in 2002. iii. An income tax of Rs. 35,000 was paid during 2002. Required: Calculate the cash flow from the operating activities as per AS-3 (Revised). [B.Com., Delhi University—2003] Ans.: Rs. 1,35,000. III Calculation of Net Cash Flow from the investing activities as per AS-3 13. From the following information, calculate the Net Cash Flow from the investing activities: Particulars Plant & Machinery (at cost) Accumulated Depreciation on above Patents

Opening Rs. 6,00,000 1,40,000 1,40,000

Closing Rs. 7,40,000 2,00,000 80,000

Additional Information: i. During the year, a machine costing Rs. 60,000 with an accumulated depreciation of Rs. 32,000 was sold for Rs. 25,000. ii. Patents were written off to the extent of Rs. 20,000 and some patents were sold at a profit of Rs. 30,000. Ans.: (Rs. 1,05,000).

Modified Date: Sat, Jul 03, 2010 12:44:29 PM

Output Date: Tue, Jul 06, 2010 11:47:54 AM

Rev II

Project: Management Accounting_Debarshi Bhattacharyya ACE Pro India Pvt. Ltd. File: X:\Pearson\Management Accounting_Debarshi Bhattacharyya\MAIN\M04\LAYOUT_M04\M04_DEBA_ISBN_EN_SE_C04_III.indd

CASH FLOW ANALYSIS

323

14. From the following particulars, calculate the cash flow from the investing activities of Laser Ltd for the year 2008: Purchased Rs. 2,20,000 1,75,000 3,75,000 

Investments Goodwill Machinery Patents

Sold Rs. 1,30,000  1,20,000 1,10,000

Interest received on debentures held as an investment  Rs. 33,000. Dividend received on shares held as investments – Rs. 27,000. A plot of land was purchased out of the surplus funds for an investment purchased and was let out for commercial use and rent received was Rs. 50,000. Ans.: Rs. 3,00,000. IV. Calculation of Net Cash Flow from financing activities as per AS-3 15. From the following information as furnished by Zebra Ltd, calculate its Net Cash Flow from the financing activities for the year that ended on 31 March 2009: 31 March 2008 Rs. 2,00,000 1,00,000 1,00,000

Equity Share Capital Preference Share Capital Debentures

31 March 2009 Rs. 3,50,000 75,000 1,40,000

During the year 2008–09, debentures of Rs. 70,000 were redeemed for cash, while debenture interest paid was Rs. 20,000. Preference dividend paid was Rs. 15,000 and equity dividend paid was Rs. 35,000. Ans.: Rs. 95,000. 16. From the following particulars as furnished by Jhajha Ltd, calculate its Net Cash Flow from the financing activities for the year that ended on 31 March 2009: 31 March 2008 Rs. 2,00,000 1,00,000 1,20,000 1,00,000 24,000 10,000

Equity Share Capital Preference Share Capital Debentures Long-term Loan Proposed Dividend (Equity) Outstanding Interest on Debentures

31 March 2009 Rs. 3,00,000 1,50,000 1,70,000 1,20,000 38,000 8,000

During the year 2008–09, the following events took place: New Issue of Preference Shares at 10% discount New Issue of Debentures at 20% discount Repayment of Long-term Loan Payment of Preference Dividend Payment of Interest on Long-term Loan Interim Equity Dividend paid during the year in addition to Proposed Dividend Interest on Debentures for the year New Equity Shares issued at a 10% Premium

Rs. 1,00,000 1,50,000 70,000 22,000 14,000 16,000 18,000

Ans.: Rs. 94,000.

Modified Date: Sat, Jul 03, 2010 12:44:29 PM

Output Date: Tue, Jul 06, 2010 11:47:54 AM

Rev II

Project: Management Accounting_Debarshi Bhattacharyya ACE Pro India Pvt. Ltd. File: X:\Pearson\Management Accounting_Debarshi Bhattacharyya\MAIN\M04\LAYOUT_M04\M04_DEBA_ISBN_EN_SE_C04_III.indd

324

MANAGEMENT ACCOUNTING

V. Preparation of Cash Flow Statement under Traditional/Conventional Approach 17. From the following particulars, prepare a Cash Flow Statement for the year that ended on 31 December 2008: Assets on 31 December 2007 Fixed Assets Cash in Hand Other Current Assets

Rs. 1,20,000 8,000 41,000

Liabilities on 31 December 2007 Bank Loan Trade Creditors Capital

Rs. 50,000 44,000 75,000

The balance of assets and liabilities as on 31 December 2008 were as follows: Rs. 1,80,000 6,000 76,000

Fixed Assets Cash in Hand Other Current Assets

Rs. 30,000 72,000 1,60,000

Bank Loan Trade Creditors Capital

During the year 2008, the proprietor of the business withdrew Rs. 20,000 from the business and invested a further capital of Rs. 55,000 and provided Rs. 20,000 as a depreciation on the Fixed Assets. Ans.: Net Cash Flow from operation—Rs. 63,000; Cash Flow Statement total—Rs. 1,26,000. 18. Balance Sheets of Babul & Supriyo Co. are given as follows: Liabilities Current Liabilities Loan from Babul Bank Loan Capital

Year 2007 Rs. 1,20,000  1,80,000 2,40,000

Year 2008 Rs. 1,50,000 70,000 1,20,000 4,20,000

5,40,000

7,60,000

Assets Cash Debtors Stock Land Building Machinery

Year 2007 Rs. 50,000 50,000 60,000 1,00,000 1,20,000 1,60,000 5,40,000

Year 2008 Rs. 30,000 70,000 80,000 1,60,000 2,00,000 2,20,000 7,60,000

During the year 2008 Babul & Supriyo introduced an additional capital of Rs. 50,000 and drew Rs. 70,000. Provision for depreciation on Machinery: Opening balance—Rs. 60,000 and Closing balance—Rs. 90,000. No depreciation was provided on the other assets. The value of building was increased by Rs. 20,000 and the same was adjusted with the Capital Account. Prepare the Cash Flow Statement of Babul & Supriyo Co. for the year 2008. [B.Com. (Hons), Calcutta University—Adapted] Ans.: Net Cash Flow from operation—Rs. 2,00,000; Cash Flow Statement total—Rs. 3,70,000. 19. From the following information, prepare the Cash Flow Statement for the year that ended on 31 March 2009: Balance Sheets as on Liabilities Share Capital Profit & Loss A/c Bank Loan Creditors Bills Payable

31 March 2008 Rs. 2,00,000 40,000 1,00,000 50,000 40,000 4,30,000

31 March 2009 Rs. 3,00,000 90,000 60,000 75,000 25,000 5,50,000

Assets Land & Building Machinery Stock Debtors Cash

31 March 2008 Rs. 1,20,000 1,60,000 60,000 50,000 40,000 4,30,000

31 March 2009 Rs. 1,70,000 2,40,000 40,000 50,000 50,000 5,50,000

Additional Information: i. Net profit for the year 2008–09 amounted to Rs. 50,000. ii. During the year 2008–09, a machine costing Rs. 50,000 (accumulated depreciation—Rs. 20,000) was sold for Rs. 25,000. The provision for depreciation against Machinery as on 31 March 2008 was Rs. 30,000 and on 31 March 2009 was Rs. 70,000. [B.Com. (Hons), Calcutta University—Adapted] Ans.: Net Cash Flow from operation—Rs. 1,45,000; Cash Flow Statement total—Rs. 3,10,000.

Modified Date: Sat, Jul 03, 2010 12:44:29 PM

Output Date: Tue, Jul 06, 2010 11:47:54 AM

Rev II

Project: Management Accounting_Debarshi Bhattacharyya ACE Pro India Pvt. Ltd. File: X:\Pearson\Management Accounting_Debarshi Bhattacharyya\MAIN\M04\LAYOUT_M04\M04_DEBA_ISBN_EN_SE_C04_III.indd

CASH FLOW ANALYSIS

325

20. Prepare a Cash Flow Statement of Jungle Ltd for the year that ended on 31 December 2008 from the following particulars Balance Sheets as on Liabilities Equity Share Capital 9% Preference Share Capital Profit & Loss A/c 10% Debentures Creditors Proposed Dividend Provision for Taxation

31 December 2007 Rs. 2,00,000 1,00,000

31 December 2008 Rs. 2,50,000 1,50,000

1,40,000 1,50,000 50,000 30,000 50,000 7,20,000

2,20,000 1,00,000 40,000 40,000 40,000 8,40,000

Assets Goodwill Building Machinery Stock Debtors Bank Preliminary Expenses

31 December 2007 Rs. 80,000 2,50,000

31 December 2008 Rs. 50,000 3,30,000

1,60,000 70,000 1,10,000 40,000 10,000 7,20,000

2,35,000 50,000 1,40,000 30,000 5,000 8,40,000

Additional Information: i.

A building having a book value of Rs. 40,000 was sold for Rs. 50,000. Depreciation on the building provided for 2008 was Rs. 60,000. ii. A machinery having a book value of Rs. 45,000 was sold for Rs. 35,000. Depreciation provided on the Machinery for the year 2008 amounted to Rs. 40,000. iii. Tax paid during the year 2008 was Rs. 45,000. iv. 10% Debentures were redeemed at a 10% premium. Ans.: Net Cash Flow from operation—Rs. 2,75,000; Cash Flow Statement total—Rs. 5,00,000. VI. Preparation of Cash Flow Statement as per AS-3 21. From the following information prepare a Cash Flow Statement as on 31 December 2007, applying the method given in AS-3: Balance Sheets as on 31 December 2007 Rs. 70,000

31 December 2008 Rs. 90,000

Profit & Loss A/c

25,000

55,000

Loan from Bank Creditors Proposed Dividend

50,000 25,000 20,000

20,000 45,000 30,000

1,90,000

2,40,000

Liabilities Share Capital

Assets

Less:

Fixed Assets at Cost Provision for Depreciation Inventory Debtors Cash

31 December 2007 Rs. 60,000

31 December 2008 Rs. 90,000

20,000 40,000 60,000 50,000 40,000 1,90,000

30,000 60,000 70,000 60,000 50,000 2,40,000

There was no disposal of Fixed Assets. Tax paid during 2007–08 amounted to Rs. 12,000. Ans.: Net Cash Flow from operating activities—Rs. 70,000; Net Cash Flow from investing activities—Rs. 30,000; Net Cash Flow from financing activities—Rs. 30,000. 22. From the following summary cash account of X Ltd, prepare a Cash Flow Statement for the year that ended on 31 March 2001, in accordance with AS-3 (Revised), using the Direct Method. The company does not have any cash equivalents. Summary Cash Account for the year that ended on 31 March 2001 Balance on 01 April 2000 Issue of Equity Shares

Rs. in ’000 50 300

Payment to Suppliers Wages & Salaries

Rs. in ’000 2,000 100 (Continued)

Modified Date: Sat, Jul 03, 2010 12:44:29 PM

Output Date: Tue, Jul 06, 2010 11:47:54 AM

Rev II

Project: Management Accounting_Debarshi Bhattacharyya ACE Pro India Pvt. Ltd. File: X:\Pearson\Management Accounting_Debarshi Bhattacharyya\MAIN\M04\LAYOUT_M04\M04_DEBA_ISBN_EN_SE_C04_III.indd

326

MANAGEMENT ACCOUNTING

Summary Cash Account for the year that ended on 31 March 2001 Rs. in ’000 2,800 100

Receipt from Customers Sale of Fixed Asset

Rs. in ’000 200 200 250 300 50 150 3,250

Overhead Expenses Purchase of Fixed Asset Taxation Repayment of Bank Loan Dividend Balance on 31 March 2001

3,250

[C.A. (Final)—November 2001] Ans.: Net Cash Flow from Operating Activities—Rs. 2,50,000; Net Cash Flow from Investing Activities—Rs. 1,00,000; Net Cash Flow from Financing Activities—Rs. 50,000. 23. Balance Sheet of Tamta Ltd (Rs. in ’000) Liabilities Share Capital Reserve Profit & Loss A/c Debentures Tax Provision Proposed Dividend Sundry Creditors

As on 31 March 2009 1,500 350 170 100 70 60 450 2,700

As on 31 March 2008 900 280 90  50 40 640 2,000

Assets Machinery Building Investment Debtors Stock Cash & Bank

As on 31 March 2009 800 700 200 400 300 300

As on 31 March 2008 400 500  750 250 100

2,700

2,000

Additional details: i. Building is still under construction and no depreciation was charged. ii. Depreciation was charged @ 20% on the opening value of the machinery. iii. An old machine costing Rs. 70,000 was sold for Rs. 40,000 (WDV—Rs. 30,000). iv. Income Tax paid during the year—Rs. 60,000. Prepare a Cash Flow Statement as per AS-3 and interpret it. [B.Com. (Hons), Calcutta University—Adapted] Ans.: Net Cash Flow from Operating Activities—Rs. 4,10,000; Net Cash Flow from Investing Activities—Rs. 8,70,000; Net Cash Flow from Financing Activities—Rs. 6,60,000. 24. ABC Ltd gives you the following information. You are required to prepare a Cash Flow Statement by using Indirect Method as per AS-3 for the year that ended on 31 March 2004: Balance Sheets as on Liabilities Capital Retained Earnings Debentures Current Liabilities: Creditors Bank Loan Liability for Expenses

31 March 2003 Rs. 50,00,000 26,50,000 

31 March 2004 Rs. 50,00,000 36,90,000 9,00,000

8,80,000 1,50,000 3,30,000

8,20,000 3,00,000 2,70,000

Assets Plant & Machinery Less: Depreciation Current Assets: Debtors Less: Provision

31 March 2003 Rs. 27,30,000 6,10,000 21,20,000

31 March 2004 Rs. 40,70,000 7,90,000 32,80,000

23,90,000 1,50,000 22,40,000

28,30,000 1,90,000 26,40,000 (Continued)

Modified Date: Sat, Jul 03, 2010 12:44:29 PM

Output Date: Tue, Jul 06, 2010 11:47:54 AM

Rev II

Project: Management Accounting_Debarshi Bhattacharyya ACE Pro India Pvt. Ltd. File: X:\Pearson\Management Accounting_Debarshi Bhattacharyya\MAIN\M04\LAYOUT_M04\M04_DEBA_ISBN_EN_SE_C04_III.indd

327

CASH FLOW ANALYSIS

Balance Sheets as on Liabilities Dividend Payable

31 March 2003 Rs. 1,50,000

31 March 2004 Rs. 3,00,000

91,60,000

1,12,80,000

Assets Cash Marketable Securities Inventories Prepaid Expenses

31 March 2003 Rs. 15,20,000 11,80,000 20,10,000 90,000 91,60,000

31 March 2004 Rs. 18,20,000 15,00,000 19,20,000 1,20,000 1,12,80,000

Additional Information: i. ii.

Net Profit for the year that ended on 31 March 2004, after charging depreciation, is Rs. 22,40,000. Debtors worth Rs. 2,30,000 were determined to be worthless and were written off against provision for doubtful debt account during the year. iii. ABC Ltd declared a dividend of Rs. 12,00,000 for the year 2003–04. [C.A. (Inter)—May 2004] Ans.: Net Cash Flow from operating activities—Rs. 19,60,000; Net Cash Flow from investing activities—Rs. 13,40,000; Net Cash Flow from financing activities—Nil. 25. From the following details relating to the accounts of Grow More Ltd, prepare a Cash Flow Statement: 31 March 2002 Rs. Liabilities: Share Capital Reserve Profit & Loss A/c Debentures Provision for Taxation Proposed Dividend Sundry Creditors Assets: Plant & Machinery Land & Building Investments Sundry Debtors Stock Cash in Hand/at Bank

31 March 2001 Rs.

10,00,000 2,00,000 1,00,000 2,00,000 1,00,000 2,00,000 7,00,000 25,00,000

8,00,000 1,50,000 60,000  70,000 1,00,000 8,20,000 20,00,000

7,00,000 6,00,000 1,00,000 5,00,000 4,00,000 2,00,000 25,00,000

5,00,000 4,00,000  7,00,000 2,00,000 2,00,000 20,00,000

i. ii. iii. iv.

Depreciation @ 25% was charged on the opening value of Plant and Machinery. During the year, an old machine costing 50,000 (WDV – 20,000) was sold for Rs. 35,000. Rs. 50,000 was paid towards income tax during the year. The building under construction was not subject to any depreciation. [C.A. (Inter)—November 2002] Ans.: Net Cash Flow from Operating Activities—Rs. 3,10,000; Net Cash Flow from Investing Activities—Rs. 6,10,000; Net Cash Flow from Financing Activities—Rs. 3,00,000. 26. X Ltd has the following balances on 01 April 2008:

Less:

Rs. 15,00,000 5,00,000

Fixed Assets at Cost Accumulated Depreciation Bank

Rs. 10,00,000 87,500 (Continued)

Modified Date: Sat, Jul 03, 2010 12:44:29 PM

Output Date: Tue, Jul 06, 2010 11:47:54 AM

Rev II

Project: Management Accounting_Debarshi Bhattacharyya ACE Pro India Pvt. Ltd. File: X:\Pearson\Management Accounting_Debarshi Bhattacharyya\MAIN\M04\LAYOUT_M04\M04_DEBA_ISBN_EN_SE_C04_III.indd

328

MANAGEMENT ACCOUNTING

Rs. Other Current Assets Current Liabilities

Rs. 6,25,000 2,50,000

Next year’s estimates are: i. ii. iii.

Net Profit will be Rs. 1,75,000 after providing for a depreciation of Rs. 1,50,000. The company will acquire Fixed Assets costing Rs. 2,50,000 after selling one machine for Rs. 70,000, whose cost is Rs. 1,50,000, on which the depreciation provided will amount to Rs. 90,000. Current Assets and Current Liabilities (other than Bank balance) on 31 March 2009 are estimated to be Rs. 7,50,000 and Rs. 4,00,000 respectively.

Required: Calculate the cash flows from the operating activities and the investing activities for the year 2008–09. [B.Com., Delhi University—Adapted] Ans.: Cash flows from Operating Activities—Rs. 3,40,000; Cash flows from Investing Activities—Rs. 1,80,000. 27. From the following particulars, prepare a Cash Flow Statement of Gulmohor Co. Ltd for the year that ended on 31 March 2007: Dr.

Profit & Loss A/c for the Year that Ended on 31 March 2007 Rs. Particulars 45,000 By Sales (Less Return) 3,60,000 By Closing Stock 1,75,000 2,40,000 8,20,000 To Office Expenses 50,000 By Gross Profit b/d To Selling Expenses 30,000 By Interest on Deposit with Bank To Depreciation on Fixed Asset 40,000 To Income Tax 50,000 To Net Profit c/d 80,000 2,50,000 To Dividend 30,000 By Balance b/d To Balance c/d 70,000 By Net Profit b/d 1,00,000

Cr. Rs. 7,50,000 70,000

Particulars To Opening Stock To Purchases (Less Return) To Wages To Gross Profit c/d

8,20,000 2,40,000 10,000

2,50,000 20,000 80,000 1,00,000

Balance Sheets as on Liabilities Share Capital Profit & Loss A/c Loan from Bank Sundry Creditors Bills Payable Outstanding Expenses Tax Liability

31 March 2006 Rs. 3,00,000 20,000 1,00,000 60,000 30,000 10,000 15,000

31 March 2007 Rs. 3,80,000 70,000 1,25,000 55,000 40,000 15,000 25,000

5,35,000

7,10,000

Assets Plant & Machinery (net) Investment Deposit with Bank Stock Sundry Debtors Bills Receivable Cash Prepaid Expenses

31 March 2006 Rs. 3,15,000 50,000 45,000 65,000 40,000 17,000 3,000 5,35,000

31 March 2007 Rs. 4,60,000 50,000 70,000 42,500 45,000 35,000 7,500 7,10,000

[B.Com. (Hons), Kalyani University—2008] Ans.: Net Cash Flow from Operating Activities—Rs. 1,18,000; Net Cash Flow from Investing Activities—Rs. 1,75,000; Net Cash Flow from Financing Activities—Rs. 75,000.

Modified Date: Sat, Jul 03, 2010 12:44:29 PM

Output Date: Tue, Jul 06, 2010 11:47:54 AM

Rev II

Project: Management Accounting_Debarshi Bhattacharyya ACE Pro India Pvt. Ltd. File: X:\Pearson\Management Accounting_Debarshi Bhattacharyya\MAIN\M04\LAYOUT_M04\M04_DEBA_ISBN_EN_SE_C04_III.indd

CASH FLOW ANALYSIS

329

28. The summarized Balance Sheets of KLM Ltd as on 31 March 2007 and 31 March 2008 are given as follows: Liabilities Share Capital General Reserve Profit & Loss A/c Mortgage Loan Sundry Creditors Provision for Tax

As on 31 March 2007 Rs. 4,00,000 40,000 1,00,000 60,000 30,000 10,000 6,40,000

As on 31 March 2008 Rs. 4,80,000 70,000 1,80,000 50,000 40,000 15,000 8,35,000

Assets Fixed Assets Investment Stock Sundry Debtors Bills Receivable Bank

As on 31 March 2007 Rs. 3,40,000 1,25,000 45,000 65,000 25,000 40,000 6,40,000

As on 31 March 2008 Rs. 4,60,000 1,50,000 70,000 80,000 30,000 45,000 8,35,000

Additional Information: i. ii. iii. iv.

An investment costing Rs. 35,000 was sold during the year for Rs. 45,000. Provision for tax made during the year was Rs. 12,000. Dividend paid during the year amounted to Rs. 20,000. During the year, a part of the Fixed Assets costing Rs. 10,000 was sold for Rs. 12,000, and the depreciation provided on the Fixed Assets for the year 2007–08 was Rs. 25,000. You are asked to prepare a Cash Flow Statement for the year that ended on 31 March 2008 as per AS-3. Ans.: Net Cash Flow from Operating Activities—Rs. 1,13,000; Net Cash Flow from Investing Activities—Rs. 1,58,000; Net Cash Flow from Financing Activities—Rs. 50,000. 29. Jaroa Ltd has the following balances as on 1 April 2008:

Less:

Fixed Assets Depreciation Stock & Debtors Bank Balance Creditors Bills Payable Capital (Shares of Rs. 100 each)

Rs. 8,50,000 1,20,000 7,30,000 3,25,000 65,000 84,000 44,000 4,00,000

The company made the following estimates for the financial year 2008–09: i. The company will pay a free-of-tax dividend @ 10%, the rate of tax being 20%. ii. The company will acquire Fixed Assets costing Rs. 1,20,000 after selling one machine for Rs. 30,000, costing Rs. 65,000, and on which the depreciation provided amounted to Rs. 12,500. iii. Stock and Debtors, Creditors and Bills Payable at the end of the financial year are expected to be Rs. 4,30,000, Rs. 1,08,000 and Rs. 58,000 respectively. iv. The Net Profit would be Rs. 87,500 after a depreciation of Rs. 36,000. Prepare the projected Cash Flow Statement and ascertain the bank balance of R Ltd at the end of the financial year 2008–09. [C.A. (PE II)—Adapted] Ans.: Net Cash Flow from Operating Activities—Rs. 79,000; Net Cash Flow from Investing Activities—Rs. 90,000; Net Cash Flow from Financing Activities—Rs. 50,000; Closing bank balance—Rs. 4,000. 30. Raj Ltd gives you the following information for the year that ended on 31 March 2006: i. Sales for the year—Rs. 48,00,000. The company sold goods for cash only. ii. Cost of Goods Sold was 75% of sales. iii. Closing inventory was higher than the opening inventory by Rs. 50,000. iv. Trade Creditors on 31 March 2006 exceed the outstanding on 31 March 2005 by Rs. 1,00,000. v. Tax paid during the year amounts to Rs. 1,50,000. vi. Amount paid to Trade Creditors during the year was Rs. 35,50,000.

Modified Date: Sat, Jul 03, 2010 12:44:29 PM

Output Date: Tue, Jul 06, 2010 11:47:54 AM

Rev II

Project: Management Accounting_Debarshi Bhattacharyya ACE Pro India Pvt. Ltd. File: X:\Pearson\Management Accounting_Debarshi Bhattacharyya\MAIN\M04\LAYOUT_M04\M04_DEBA_ISBN_EN_SE_C04_III.indd

330 vii. viii. ix. x. xi.

MANAGEMENT ACCOUNTING

Administrative and selling expenses paid was Rs. 3,60,000. One new machinery was acquired in December 2005 for Rs. 6,00,000. Dividend paid during the year was Rs. 1,20,000. Cash in hand and at bank on 31 March 2006 was Rs. 70,000. Cash in hand and at bank on 1 April 2005 was Rs. 50,000.

Required: Prepare a Cash Flow Statement for the year that ended on 31 March 2006, as per the prescribed accounting standard. [C.A. (PE II)—May 2006] Ans.: Net Cash Flow from Operating Activities—Rs. 7,40,000; Net Cash Flow from Investing Activities—Rs. 6,00,000; Net Cash Flow from Financing Activities—Rs. 1,20,000. 31. The Balance Sheet of Bollywood Ltd as on 31 March 2008 and 31 March 2009 are as follows: Balance Sheets (Rs. in ’000) Liabilities Equity Share Capital General Reserve Profit & Loss A/c Secured Loan Sundry Creditors Bills Payable Outstanding Expenses Unpaid Dividend

As on 31 March 2008 350 20 40  30 50 10 10 510

As on 31 March 2009 400   180 45 25 30  680

Assets Fixed Assets Stock Debtors Bills Receivable Investment Cash Profit & Loss A/c

As on 31 March 2008 210 90 60 50 70 30 –

As on 31 March 2009 320 140 44 75 40 31 30

510

680

Accumulated depreciation was Rs. 60,000 on 31 March 2008 and on 31 March 2009, it was Rs. 57,000. Machinery costing Rs. 1,70,000, having a WDV of Rs. 80,000, was sold for Rs. 70,000. Prepare a Cash Flow Statement for the year that ended on 31 March 2009 as per AS-3. [B.Com., Bombay University—Adapted] Ans.: Net Cash Flow from Operating Activities—Rs. 18,000; Net Cash Flow from Investing Activities—Rs. 2,37,000; Net Cash Flow from Financing Activities—Rs. 2,20,000. 32. Following are the Balance Sheets as on 31 March 2008 and 31 March 2009: Liabilities Equity Share Capital General Reserve Profit & Loss A/c Mortgage Loan (Against Plant & Machinery) Sundry Creditors Provision for Tax Bills Payable Bank Overdraft

As on 31 March 2008 Rs. 1,00,000 60,000 5,000 –

As on 31 March 2009 Rs. 1,50,000 10,000 30,000 40,000

30,000 10,000 10,000 – 2,15,000

20,000 15,000 30,000 65,000 3,60,000

Assets Land & Building Plant & Machinery Furniture & Fittings Investment Stock Sundry Debtors Cash Preliminary Expenses

As on 31 March 2008 Rs. 80,000 42,000 7,000 6,000 27,500 46,500 2,000 4,000

As on 31 March 2009 Rs. 75,000 85,000 6,000 12,000 94,500 77,200 7,300 3,000

2,15,000

3,60,000

During the year that ended on 31 March 2009, the following transactions took place: i. Bonus shares have been issued at one for every two held out of General Reserve. ii. Company purchased plant and machinery for Rs. 60,000, out of which Rs. 20,000 was paid in cash and for the rest, plant and machinery was mortgaged to the seller. iii. Dividend Paid was Rs. 15,000.

Modified Date: Sat, Jul 03, 2010 12:44:29 PM

Output Date: Tue, Jul 06, 2010 11:47:54 AM

Rev II

Project: Management Accounting_Debarshi Bhattacharyya ACE Pro India Pvt. Ltd. File: X:\Pearson\Management Accounting_Debarshi Bhattacharyya\MAIN\M04\LAYOUT_M04\M04_DEBA_ISBN_EN_SE_C04_III.indd

CASH FLOW ANALYSIS

331

iv. v. vi. vii.

Furniture (Book value —Rs. 2,100) was sold for Rs. 3,045. Investment costing Rs. 3,000, written off in the year 2002, were sold for Rs. 5,000 on 12 March 2009. Furniture purchased during the year was for Rs. 1,500. Net Profit for the year, after charging a depreciation on land and building, plant and machinery, furniture and fittings, and Rs. 21,000 as provision for tax. Prepare a Cash Flow Statement for the year that ended on 31 March 2009, as per AS-3. [B.Com., Bombay University—Adapted] Ans.: Net Cash Flow from Operating Activities—Rs. 25,245; Net Cash Flow from Investing Activities—Rs. 19,455; Net cash flow from Financing Activities—Rs. 15,000.. 33. The Balance Sheets of KLO Ltd as on 31 March 2008 and 31 March 2009 are as follows: Liabilities Share Capital General Reserve Profit on Sale of Investment 7% Debentures Creditors for Goods

Creditors for Expenses Proposed Dividend Provision for Tax

31 March 2008 Rs. 12,00,000 4,00,000 –

31 March 2009 Rs. 14,00,000 5,00,000 20,000

6,00,000 3,20,000

4,00,000 5,00,000

20,000 60,000 1,40,000

24,000 70,000 1,50,000

29,40,000

34,64,000

Assets

Less:

Fixed Assets Accumulated Depreciation

Stock (at Cost) Sundry Debtors (Less Provision of Rs. 40,000 & Rs. 50,000 respectively) Bills Receivable Prepaid Expenses Miscellaneous Expenditure

31 March 2008 Rs. 20,00,000 4,00,000 16,00,000

31 March 2009 Rs. 24,00,000 5,00,000 19,00,000

4,00,000 4,50,000

5,40,000 4,90,000

80,000 20,000 30,000

1,30,000 24,000 20,000

29,40,000

34,64,000

Additional Information: i. During the current year, the Fixed Assets (valued at Rs. 20,000, depreciation written off —Rs. 60,000) was sold for Rs. 16,000. ii. Proposed dividend for the last year paid in the current year. iii. During the current year, investments costing Rs. 1,60,000 were sold and later in the year, investments of the same cost were purchased. iv. Debentures were redeemed at a premium of 10%. v. Liability for tax for the last year came to Rs. 1,10,000. vi. During the current year, the bad debts written off were Rs. 30,000 against provision. You are asked to prepare a Cash Flow Statement for the year that ended on 31 March 2009 as per AS-3. Ans.: Net Cash Flow from Operating Activities—Rs. 5,52,000; Net Cash Flow from Investing Activities—Rs. 4,44,000); Net Cash Flow from Financing Activities—Rs. 1,08,000. 34. The following are the summarized Balance Sheets of a company as on 31 March 2007 and 31 March 2008: Liabilities Share Capital General Reserve Profit & Loss A/c Bank Loan Sundry Creditors Provision for Tax

Rs. in ’000 As on 31 As on 31 March March 2007 2008 100.00 125.00 25.00 30.00 15.25 15.30 35.00 – 75.00 67.60 15.00 17.50 265.25

Modified Date: Sat, Jul 03, 2010 12:44:29 PM

Assets Land & Building Plant & Machinery Stock Sundry Debtors Cash Bank Goodwill

255.40

Output Date: Tue, Jul 06, 2010 11:47:54 AM

Rs. in ’000 As on 31 As on 31 March March 2007 2008 100.00 95.00 75.00 84.50 50.00 37.00 40.00 32.10 0.25 0.30 4.00 2.50 265.25 255.40

Rev II

Project: Management Accounting_Debarshi Bhattacharyya ACE Pro India Pvt. Ltd. File: X:\Pearson\Management Accounting_Debarshi Bhattacharyya\MAIN\M04\LAYOUT_M04\M04_DEBA_ISBN_EN_SE_C04_III.indd

332

MANAGEMENT ACCOUNTING

Additional Information: i. ii.

Dividends of Rs. 11,500 were paid. The following assets of another company were purchased for a consideration of Rs. 25,000 paid for in shares: Stock—Rs. 10,000 and Machinery—Rs. 12,500. iii. Machinery was further purchased for a cash of Rs. 12,500. iv. Depreciation written off: on building—Rs. 5,000 and on machinery—Rs. 7,000. v. Income tax paid during the year—Rs. 14,000. vi. Net profit for the year was Rs. 33,050. Prepare a Cash Flow Statement for the year that ended on 31 March 2008 as per AS-3. [B.Com. (Hons), Calcutta University—Adapted] Ans.: Net Cash Flow from Operating Activities—Rs. 54,550; Net Cash Flow from Investing Activities—Rs. 4,000; Net cash flow from Financing Activities—Rs. 46,500. 35. The following are the Balance Sheets of a company as on 31 March 2008 and 31 March 2009:

Liabilities Equity Share Capital 8% Preference Share Capital General Reserve Capital Reserve Securities Premium Profit & Loss A/c Sundry Creditors Bills Payable Provision for Tax Proposed Dividend

Rs. in ’000 As on 31 As on 31 March March 2008 2009 300 350 150 100 40 75 – 20 20 25 30 73 55 83 20 16 40 50 42 50 697 842

Assets Goodwill Land & Building (At Cost) Plant & Machinery (Net) Investment Stock Sundry Debtors Bills Receivable Cash & Bank Preliminary Expenses

Rs. in ’000 As on 31 As on 31 March March 2008 2009 100 89 200 170 80 200 20 35 77 100 140 170 20 30 25 18 35 30 697

842

Additional Information: i. One piece of land was sold at a profit and the profit was transferred to Capital Reserve. ii. One machine was sold for Rs. 15,000, WDV of which on the date of sale was Rs. 18,000. iii. The depreciation charged on plant and machinery amounted to Rs. 16,000. iv. A dividend of Rs. 4,000 was received from investment, of which Rs. 2,000 was credited to investment account, being a dividend declared from pre-acquisition profit. v. The actual amount of dividend and tax paid were Rs. 35,000 and Rs. 38,000 respectively. Prepare a Cash Flow Statement for the year that ended on 31 March 2009 as per AS-3. [M.Com., Calcutta University—Adapted] Ans.: Net Cash Flow from Operating Activities—Rs. 1,25,000; Net Cash Flow from Investing Activities—Rs. 1,02,000; Net Cash Flow from Financing Activities—Rs. 30,000. 36. From the following Balance Sheets and income statement of Tiku Ltd, prepare a Cash Flow Statement for the year that ended on 31 March 2009: Balance Sheets as on

Liabilities Paid-up Capital

Retained Earnings

Rs. in ’000 As on 31 As on 31 March March 2008 2009 50 50

350

Assets Fixed Assets (After Accumulated Depreciation of 100 & 175 respectively)

Rs. in ’000 As on 31 As on 31 March March 2008 2009 900 950

415 (Continued)

Modified Date: Sat, Jul 03, 2010 12:44:29 PM

Output Date: Tue, Jul 06, 2010 11:47:54 AM

Rev II

Project: Management Accounting_Debarshi Bhattacharyya ACE Pro India Pvt. Ltd. File: X:\Pearson\Management Accounting_Debarshi Bhattacharyya\MAIN\M04\LAYOUT_M04\M04_DEBA_ISBN_EN_SE_C04_III.indd

CASH FLOW ANALYSIS

Rs. in ’000 As on 31 As on 31 March March 2008 2009 500 550 80 100 80 90

Liabilities Long-term Debt Notes Payable Accounts Payable

1,060

Assets

Inventory Accounts Receivable Cash

1,205

333

Rs. in ’000 As on 31 As on 31 March March 2008 2009

100 50 10 1,060

110 60 85 1,205

Income Statement for the year that ended on 31 March 2009

Less: Less: Less: Less:

(Rs. in ’000) 1,200 800 400 150 250 50 200 100 100

Sales Cost of Goods Sold Gross Profit Selling, General & Administrative Expenses Earnings before Interest & Tax (EBIT) Interest Expenses Earnings before Tax (EBT) Tax @ 50% Net Income

Additional Information: Rs. in ’000 35 65 75

Dividend Paid Addition to retained earnings Depreciation

Ans.: Net Cash Flow from Operating Activities—Rs. 2,35,000; Net Cash Flow from Investing Activities—Rs. 1,25,000; Net Cash Flow from Financing Activities—Rs. 35,000. 37. From the following information, prepare a Cash Flow Statement for the year that ended on 31 March 2009: Balance Sheet of Queen Ltd (Rs. in Lakhs) Liabilities Equity Share Capital Preference Share Capital Capital Redemption Reserve Securities Premium Profit & Loss A/c General Reserve Current Liabilities Provision for Taxation

As on 31 March 2008 10.00 5.00 Nil 0.25 2.75 10.00 7.00 3.00 38.00

As on 31 March 2009 15.00 Nil 5.00 Nil 3.00 7.00 2.00 4.00 36.00

Assets Plant at WDV Stock Debtors Cash Balance Miscellaneous Expenditure

As on 31 March 2008 15.00 6.00 15.00 1.00 1.00

As on 31 March 2009 18.00 3.00 9.00 2.00 4.00

38.00

36.00

Additional Information: i. ii. iii.

Equity shares were of Rs. 100 each, fully paid up. Preference Shares were of Rs. 100 each, but paid up to Rs. 50 per share. During the year 2008–09, the company paid Rs. 2,00,000 as Equity Dividend and Rs. 56,250 as Preference Dividend. The company redeemed the Preference Shares at a premium of 5% after making a call of Rs. 50 per share, to make the shares fully paid.

Modified Date: Sat, Jul 03, 2010 12:44:29 PM

Output Date: Tue, Jul 06, 2010 11:47:54 AM

Rev II

Project: Management Accounting_Debarshi Bhattacharyya ACE Pro India Pvt. Ltd. File: X:\Pearson\Management Accounting_Debarshi Bhattacharyya\MAIN\M04\LAYOUT_M04\M04_DEBA_ISBN_EN_SE_C04_III.indd

334

MANAGEMENT ACCOUNTING

iv.

During the year 2008–09, one plant, whose book value was Rs. 1,00,000, was sold at a loss of Rs. 25,000, and the company purchased the plant for Rs. 6,00,000. v. Miscellaneous expenditure included Rs. 5,00,000 Share Issue Expenses paid during the year. vi. A sum of Rs. 3,50,000 was provided for taxation for the year. Ans.: Net Cash Flow from Operating Activities—Rs. 9,81,250; Net Cash Flow from Investing Activities—Rs. 5,25,000; Net Cash Flow from Financing Activities—Rs. 3,56,250. 38. The Balance Sheets of Bharbi Ltd as on 31 March 2008 and 31 March 2009 are as follows:

Liabilities Share Capital General Reserve Profit & Loss A/c Capital Reserve Debentures Provision for Tax Proposed Dividend Unpaid Dividend Current Liabilities

As on 31 March 2008 Rs. 3,00,000 1,70,000 60,000 – 2,00,000 90,000 30,000 – 1,20,000 9,70,000

As on 31 March 2009 Rs. 4,00,000 2,00,000 75,000 10,000 1,40,000 85,000 36,000 4,000 1,30,000 10,80,000

Assets

Less:

Fixed Assets at Cost Depreciation Trade Investment Current Assets Bank Preliminary Expenses

As on 31 March 2008 Rs. 8,00,000 2,30,000 5,70,000 1,00,000 2,00,000 80,000 20,000

As on 31 March 2009 Rs. 9,50,000 2,90,000 6,60,000 80,000 3,00,000 30,000 10,000

9,70,000

10,80,000

During the year 2008–09, the company: i.

Sold one machine for Rs. 25,000, the cost of which was Rs. 50,000 and the depreciation provided on it was Rs. 21,000. ii. Provided Rs. 95,000 as depreciation. iii. Redeemed 30% of the debentures @ 103. iv. Sold some trade investment at a profit which was credited to Capital Reserve. v. Decided to value the stock at cost whereas previously the practice was to value the stock at cost less 10%. The stock according to books on 31 March 2008 was Rs. 54,000. The stock on 31 March 2009 was correctly valued at cost Rs. 75,000. vi. Decided to write off Fixed Assets costing Rs. 14,000 (fully depreciated). Prepare a Cash Flow Statement for the year that ended on 31 March 2009 as per AS-3. Ans.: Net Cash Flow from Operating Activities—Rs. 96,800; Net Cash Flow from Investing Activities—Rs. 1,59,000; Net Cash Flow from Financing Activities—Rs. 12,200. 39. Following are the summary of cash transactions extracted from the books of a company: Balance as on 1 July 2008 Receipts from Customers Issue of Shares Sale of Fixed Assets Total Payment to Suppliers Payment for Fixed Assets Payment of Overheads Wages & Salaries Taxation Dividend Repayment of Bank Loan Balance on 30 June 2009 Total

Modified Date: Sat, Jul 03, 2010 12:44:29 PM

Output Date: Tue, Jul 06, 2010 11:47:54 AM

Rs. in ’000 70 5,566 600 256 6,492 4,094 460 230 138 486 160 500 424 6,492

Rev II

Project: Management Accounting_Debarshi Bhattacharyya ACE Pro India Pvt. Ltd. File: X:\Pearson\Management Accounting_Debarshi Bhattacharyya\MAIN\M04\LAYOUT_M04\M04_DEBA_ISBN_EN_SE_C04_III.indd

CASH FLOW ANALYSIS

335

Prepare a Cash Flow Statement for the year that ended on 30 June 2009, in accordance with AS-3. [C.S. (Inter)—December 2002] Ans.: Net Cash Flow from Operating Activities—Rs. 6,18,000; Net Cash Flow from Investing Activities—Rs. 2,04,000; Net Cash Flow from Financing Activities—Rs. 60,000. 40. The Balance Sheets of Jhajha Ltd as on 31 March 2008 and 31 March 2009 are as follows: Balance Sheet of Jhajha Ltd (Rs. in Lakhs) As on 31 March 2008 300.00 225.00

As on 31 March 2009 300.00 240.00

6% Debentures (Unsecured)

75.00

75.00

Mortgage Loan (by Freehold Property) Creditors

27.00

14.25

45.00

45.00

Proposed Dividend (subject to Deduction of Tax) Provision for Taxation Secured Overdraft (by Floating Charge on Assets)

22.50

23.25

Freehold Property at Cost Plant & Machinery (at Cost less Depreciation) Investment in Shares of companies under same management (unquoted) Investment in Shares of other Companies (quoted) (Market Value on 31 March 2008 – 150 Lakhs, and on 31 March 2009 – 120 Lakhs) Stock

21.00 15.00

37.50 82.50

Debtors Bank

730.50

817.50

Liabilities Share Capital Reserve

Assets

As on 31 March 2008 225.00 135.00

As on 31 March 2009 240.00 165.00

150.00

150.00

112.50

112.50

52.50

75.00

45.00 10.50

75.00

730.50

817.50

The following additional information for the year 2008–09 are relevant: i. Credit Sales Rs. 675 lakhs ii. Credit Purchases Rs. 520 lakhs iii. Overheads Rs. 83.75 lakhs iv. Depreciation on Plant & Machinery Rs. 17.50 lakhs v. Dividend for 2007–08 was paid in full. vi. Amount paid towards taxation for the year 2007–08 Rs. 21.50 lakhs In view of credit squeeze, the company has been asked by the bank to reduce the overdraft substantially within 6 months, if possible by 50%. Prepare a Cash Flow Statement for the year that ended on 31 March 2009 as per AS-3. Ans.: Net Cash Flow from Operating Activities – 26.41 (Rs. in lakhs); Net Cash Flow from Investing Activities—62.50 (Rs. in lakhs); Net Cash Flow from Financing Activities—41.90 (Rs. in lakhs). 41. Following are the Balance Sheets of a company as on 31 March 2008 and 31 March 2009: Liabilities Equity Share Capital (Rs. 10 each, fully paid) General Reserve Capital Reserve (Profit on Sale of Investment)

As on 31 March 2008 Rs. 3,00,000

As on 31 March 2009 Rs. 3,50,000

1,50,000 –

2,25,000 5,000

Fixed Assets (Net)

As on 31 March 2008 Rs. 4,00,000

As on 31 March 2009 Rs. 4,75,000

Long-term Investment (at Cost) Stock at Cost

90,000 1,00,000

90,000 1,35,000

Assets

(Continued)

Modified Date: Sat, Jul 03, 2010 12:44:29 PM

Output Date: Tue, Jul 06, 2010 11:47:54 AM

Rev II

Project: Management Accounting_Debarshi Bhattacharyya ACE Pro India Pvt. Ltd. File: X:\Pearson\Management Accounting_Debarshi Bhattacharyya\MAIN\M04\LAYOUT_M04\M04_DEBA_ISBN_EN_SE_C04_III.indd

336

MANAGEMENT ACCOUNTING

Liabilities 15% Debentures Accrued Expenses Creditors Proposed Dividend Provision for Tax

As on 31 March 2008 Rs. 1,50,000 5,000 80,000 15,000 35,000 7,35,000

As on 31 March 2009 Rs. 1,00,000 6,000 1,25,000 17,000 38,000 8,66,000

Assets Debtors Cash

As on 31 March 2008 Rs. 1,12,500 32,500

As on 31 March 2009 Rs. 1,22,500 43,500

7,35,000

8,66,000

Additional Information: i. The balance of accumulated depreciation stood at Rs. 1,00,000 on 31 March 2008 and Rs. 1,25,000 on 31 March 2009. ii. During the year 2008–09, Fixed Assets having a book value of Rs. 5,000 (accumulated depreciation— Rs. 15,000) was sold for Rs. 4,000. iii. During the year 2008–09, investments costing Rs. 40,000 were sold. iv. Debentures were redeemed at a premium of 10%. v. Tax of Rs. 2,75,500 was paid. vi. Dividend proposed in 2007–08 was paid in 2008–09. You are asked to prepare a Cash Flow Statement for the year that ended on 31 March 2009 as per AS-3. [M.Com., Calcutta University—Adapted] Ans.: Net Cash Flow from Operating Activities—Rs. 1,57,000; Net Cash Flow from Investing Activities—Rs. 1,11,000; Net Cash Flow from Financing Activities—Rs. 35,000. 42. Swastik Oils Ltd has furnished the following information for the year that ended on 31 March 2003:

Net Profit Dividend (including Interim Dividend paid) Provision for Income Tax Income Tax paid during the year Loss on Sale of Assets (Net) Book Value of Assets Sold Depreciation charged to Profit & Loss A/c Profit on Sale of Investments Interest Income on Investments Value of Investments Sold Interest Expenses Interest paid during the year Increase in Working Capital (excluding Cash & Bank Balance) Purchase of Fixed Assets Investments on Joint Venture Expenditure on Construction WIP Proceeds from Long-term Borrowings Proceeds from Short-term Borrowings Opening Cash & Bank Balances Closing Cash & Bank Balances

Rs. in Lakhs 37,500 12,000 7,500 6,372 60 277 30,000 150 41,647.50 3,759 15,000 15,780 84,112.50 21,840 5,775 69,480 38,970 30,862.50 11,032.50 2,569

You are required to prepare the Cash Flow Statement for the year that ended on 31 March 2003 as per AS-3 (make assumption wherever necessary). [C.S. (Inter)—June 2004] Ans.: Net Cash Flow from Operating Activities—4,333.50 (Rs. in lakhs); Net Cash Flow from Investing Activities — 51,321.00 (Rs. in lakhs); Net Cash Flow from Financing Activities—42,052.50 (Rs. in lakhs).

Modified Date: Sat, Jul 03, 2010 12:44:29 PM

Output Date: Tue, Jul 06, 2010 11:47:54 AM

Rev II

Project: Management Accounting_Debarshi Bhattacharyya ACE Pro India Pvt. Ltd. File: X:\Pearson\Management Accounting_Debarshi Bhattacharyya\MAIN\M04\LAYOUT_M04\M04_DEBA_ISBN_EN_SE_C04_III.indd

337

CASH FLOW ANALYSIS

43. Financial Statements of Murli Ltd for the year 2007 and 2008 were as follows: Balance Sheets (Rs. in ’000) Liabilities Share Capital General Reserve Profit & Loss A/c Bank Overdraft Sundry Creditors Provision for Tax Proposed Dividend

As on 31 December 2007 800 300 200 300 1,200 300 80 3,180

As on 31 December 2008 900 400 300 464 1,000 400 90 3,554

Assets Fixed Assets Additions Depreciation Investment Stock Debtors

As on 31 December 2007 600 200 (300) 500 200 1,400 1,080 3,180

As on 31 December 2008 800 100 (350) 550

For 2007 430 – –

For 2008 660 30 50

200 630

200 940

1,230 1,774 3,554

Profit & Loss A/c (Rs. in ’000) Particulars To Taxation To Proposed Dividend To Transfer to General Reserve

For 2007 250 80 100

For 2008 450 90 100

200 630

300 940

To Balance c/f

Particulars By Trading Profit By Profit on Sale of Investment By Income Tax Excess provided in the previous year By Balance from last year

Additional Information: i. For the year that ended on 31 December 2008, purchases were Rs. 60 lakhs and sales were Rs. 70 lakhs. ii. Trading profit for the year 2008 was arrived at after charging a depreciation of Rs. 50,000 and directors’ remuneration was Rs. 1,20,000. [M.Com., Madras University—Adapted] Ans.: Net Cash Flow from Operating Activities—314 (Rs. in ’000); Net Cash Flow from Investing Activities—130 (Rs. in ’000); Net Cash Flow from Financing Activities—20 (Rs. in ’000). 44. From the following Balance Sheets as on 31 December 2000 and 31 December 2001 and from the additional information of Kalyani Ltd, you are required to prepare a Cash Flow Statement:

Liabilities Share Capital Profit & Loss A/c Debentures Sundry Creditors Provision for Doubtful Debt

As on 31 December 2000 Rs. 1,82,000 13,040 40,000 14,360 1,000

As on 31 December 2001 Rs. 1,86,000 13,560 – 15,840 1,100

2,50,400

2,16,500

Assets Goodwill Land Plant & Machinery Closing Stock Sundry Debtors Cash in Hand

As on 31 December 2000 Rs. 10,000 40,000 1,00,000 69,200 19,200 12,000 2,50,400

As on 31 December 2001 Rs. 5,000 50,000 66,000 62,700 22,000 10,800 2,16,500

Additional Information: i. A dividend of Rs. 5,000 was paid. ii. Provision for tax made during the year—Rs. 9,000. iii. During the year, a machine costing Rs. 20,000 (accumulated depreciation—Rs. 6,000) was sold for Rs. 11,000. iv. The provision for depreciation against Plant and Machinery as on 31 December 2000 was Rs. 30,000 and on 31 December 2001 was Rs. 44,000. [B.Com. (Hons), Kalyani University—2002]

Modified Date: Sat, Jul 03, 2010 12:44:29 PM

Output Date: Tue, Jul 06, 2010 11:47:54 AM

Rev II

Project: Management Accounting_Debarshi Bhattacharyya ACE Pro India Pvt. Ltd. File: X:\Pearson\Management Accounting_Debarshi Bhattacharyya\MAIN\M04\LAYOUT_M04\M04_DEBA_ISBN_EN_SE_C04_III.indd

338

MANAGEMENT ACCOUNTING

Ans.: Net Cash Flow from Operating Activities—Rs. 38,800; Net Cash Flow from Investing Activities—Rs. 1,000; Net cash flow from Financing Activities—(Rs. 41,000). 45. From the following information, prepare a Cash Flow Statement for the year that ended on 31 March 2009: Balance Sheets (Rs. in Lakhs) Liabilities Equity Share Capital (Rs. 100) Redeemable Preference Shares of Rs. 100, Paid Rs. 50 Securities Premium Capital Redemption Reserve General Reserve Profit & Loss A/c Current Liabilities Provision for Taxation

As on 31 March 2008 10.00 5.00

As on 31 March 2009 15.00 –

0.25 – 10.00 2.75 7.00 3.00 38.00

– 5.00 7.00 3.00 2.00 4.00 36.00

Assets Plant at WDV Stock Debtors Cash Balance Miscellaneous Expenditure

As on 31 March 2008 15.00 6.00

As on 31 March 2009 18.00 3.00

15.00 1.00 1.00

9.00 2.00 4.00

38.00

36.00

Additional Information: i.

During the year, the company paid Rs. 2,00,000 as the equity dividend and Rs. 56,250 as the preference dividend. ii. The company redeemed the preference shares at a premium of 5% after making a call of Rs. 50 per share to make the shares fully paid. iii. During the year, one plant whose book value was Rs. 1,00,000 was sold at a loss of Rs. 25,000 and the company purchased the plant for Rs. 6,00,000. iv. Miscellaneous expenditure included Rs. 5 lakhs as share issue and other expenses paid during the year. v. A sum of Rs. 3,50,000 has been provided for taxation for the year. [B.Com. (Hons), Calcutta University—Adapted] Ans.: Net Cash Flow from Operating Activities—Rs. 14,31,250; Net Cash Flow from Investing Activities— Rs. 5,25,000; Net Cash Flow from Financing Activities—Rs. 8,06,250. 46. The summarized Balance Sheets of Lalgola Ltd as on 31 March 2009 and 31 March 2008 are given as follows:

Liabilities Equity Share Capital (of Rs. 100 each) Reserves & Surplus Secured Loan Sundry Creditors Provision for Tax

As on 31 March 2009 Rs. 2,30,000

As on 31 March 2008 Rs. 1,97,000

3,12,000

1,48,000 87,000 2,51,450 65,000 7,48,450

2,98,000 1,72,000 10,12,000

Assets Fixed Assets Investment Stock Sundry Debtors Bank Prepaid Expenses

As on 31 March 2009 Rs. 6,00,000

As on 31 March 2008 Rs. 3,60,000

10,000 1,96,000 1,40,000 45,000 21,000 10,12,000

11,250 1,42,500 90,700 1,30,000 14,000 7,48,450

Additional Information: i. Investments costing Rs. 5,000 were sold during the year for Rs. 4,800 and government securities with the face value of Rs. 4,000 were purchased during the year for Rs. 3,750.

Modified Date: Sat, Jul 03, 2010 12:44:29 PM

Output Date: Tue, Jul 06, 2010 11:47:54 AM

Rev II

Project: Management Accounting_Debarshi Bhattacharyya ACE Pro India Pvt. Ltd. File: X:\Pearson\Management Accounting_Debarshi Bhattacharyya\MAIN\M04\LAYOUT_M04\M04_DEBA_ISBN_EN_SE_C04_III.indd

CASH FLOW ANALYSIS

ii.

Add: Less:

339

The position of reserves and surplus was as follows: Rs. 1,48,000 1,98,500 3,46,500 34,500 3,12,000

Balance on 1 April 2008 Net Profit for 2008–09 Dividend Balance on 31 March 2009

iii.

Accumulated depreciation on Fixed Assets on 31 March 2009 and on 31 March 2008 were Rs. 1,80,000 and Rs. 1,60,000 respectively. Depreciation provided for 2008–09 amounted to Rs. 30,000. iv. Machinery costing Rs. 20,000, which was one-half depreciated, was discarded and written off in 2008–09. You are asked to prepare a Cash Flow Statement for the year that ended on 31 March 2009 as per AS-3. Ans.: Net Cash Flow from Operating Activities—Rs. 2,82,450; Net Cash Flow from Investing Activities— Rs. 2,78,950; Net Cash Flow from Financing Activities—Rs. 88,500. 47. Financial Statements for the year that ended on 30 September 2008 of Karol Bag Ltd are given as follows: Rs. in ’000 5,684 2,740 1,030 80 76 220 615 4,761 923

Sales (A) Cost of Goods Sold (Including a Depreciation of Rs. 2,80,000) Operating Expenses (Including a Depreciation of Rs. 3,10,000) Interest Expenses Loss on Sale of Equipment Net Loss on Fire Income Tax Total Expenses (B) Net Income (A – B)

Comparative Balance Sheet (Rs. in ’000) As on 30 June 2008 Liabilities Capital Stock, Rs. 10 per Value Premium on Stock Retained Earnings Bonds Payable, Net of Amortized Amount Bank Loan Payable Accounts Payable Accrued Operating Expenses Income Tax Payable

9,600 6,400 2,477 3,765 1,230 1,094 167 370 25,103 As on 30 June 2008

Assets Land Building, Net of Accumulated Depreciation Equipment, Net of Accumulated Depreciation Investment in Hudco Ltd Inventory Accounts Receivable Temporary Investment Cash Prepaid Expenses

Modified Date: Sat, Jul 03, 2010 12:44:29 PM

Output Date: Tue, Jul 06, 2010 11:47:54 AM

2,630 4,360 4,784 3,800 3,872 2,641 150 2,820 46 25,103

As on 30 June 2007 6,800 3,600 1,872 3,750 850 963 152 210 18,197 As on 30 June 2007 1,520 3,810 4,400 0 2,960 2,840 65 2,567 35 18,197

Rev II

Project: Management Accounting_Debarshi Bhattacharyya ACE Pro India Pvt. Ltd. File: X:\Pearson\Management Accounting_Debarshi Bhattacharyya\MAIN\M04\LAYOUT_M04\M04_DEBA_ISBN_EN_SE_C04_III.indd

340

MANAGEMENT ACCOUNTING

Additional Information: i. The investments in Hudco Ltd were acquired upon the issuance of 1,90,000 shares of capital, having a market value of Rs. 20 per share at the time of acquisition. ii. A fire during the financial year destroyed the wing of a building that had a Net Book Value of Rs. 2,60,000 at the time of loss. Equipment having a Net Book Value of Rs. 3,80,000 was also destroyed in the fire. The insurance recovery amounted to Rs. 4,20,000. iii. The Net Book Value of the equipment sold during the financial year was Rs. 2,38,000. iv. Included in the total depreciation charges for the year was depreciation of Rs. 1,60,000 on building. v. No bonds were issued during the financial year. vi. The only entries to retained earnings were those to close out the net income for the year and to record the dividend declared for the year. You are asked to prepare a Cash Flow Statement for the year that ended on 30 June 2008 as per AS-3. [M.Com., Delhi University—Adapted] Ans.: Net Cash Flow from Operating Activities—Rs. 14,71,000; Net Cash Flow from Investing Activities— Rs. 29,30,000; Net Cash Flow from Financing Activities—Rs. 17,97,000.

Modified Date: Sat, Jul 03, 2010 12:44:29 PM

Output Date: Tue, Jul 06, 2010 11:47:54 AM

Rev II

Project: Management Accounting_Debarshi Bhattacharyya ACE Pro India Pvt. Ltd. File: X:\Pearson\Management Accounting_Debarshi Bhattacharyya\MAIN\M05\LAYOUT_M05\M05_DEBA_ISBN_EN_SE_C05_Part-1.indd

Fund Flow Analysis

5

LEARNING OBJECTIVES On completion of the study of the chapter, you should be able to understand: What is the meaning and concept of Fund? What is the meaning and concept of Flow of Fund? What is Fund Flow Statement? Different sources and uses of Fund. How to prepare a Fund Flow Statement. Importance and limitations of a Fund Flow Statement. What is Fund from Operation? Comparison between Fund Flow Statement, Income Statement and Balance Sheet. Comparison between Fund Flow Statement and Cash Flow Statement.

5.1 MEANING AND CONCEPT OF FUND The concept of Fund is explained by different accountants and accounting bodies in different approaches. Accordingly, the word Fund has different meanings as per the interpretations of different accountants and accounting bodies. These different interpretations of the concept of Fund are discussed as follows: i. Cash and Bank: As per the interpretation of some accountants, Fund includes cash and bank of the enterprise only. As per this concept, the inflows and outflows of cash resources alone are considered as Flow of Fund. Accordingly, under this concept, the Fund Flow Statement of an enterprise is prepared taking the inflows and outflows of cash resources alone. ii. Working Capital: As per this interpretation, Fund includes the Working Capital of the enterprise only. The difference between total Current Assets and total Current Liabilities of an enterprise constitutes its Working Capital (i.e., Working Capital = Current Assets − Current Liabilities). As per this concept, the inflows and outflows of the Working Capital elements alone (i.e., Current Assets and Current Liabilities) are considered as the Flow of Fund. Accordingly, under this concept, the Fund Flow Statement of an enterprise is prepared taking the inflows and outflows of Current Assets and Current Liabilities only. iii. All Financial Resources: As per this interpretation, all financial resources of an enterprise are considered as a Fund, irrespective of the fact whether these resources influence the Working Capital of the enterprise or not. As per this concept, the inflows and outflows of all financial resources of the enterprise are considered as Flow of Fund. Accordingly, under this concept, the Fund Flow Statement of an enterprise is prepared taking the inflows and outflows of all the financial resources of the enterprise. 5.2 GENERALLY ACCEPTED CONCEPT OF FUND Although there is no unanimous concept of Fund, yet with reference to the preparation of a Fund Flow Statement the most preferred and accepted concept of Fund is the Working Capital. Working Capital (net) is the excess of Current Assets over Current Liabilities. Hence, Fund Flow Statement is prepared considering the meaning and concept of Fund as the Working Capital. Therefore, Fund items for the purpose of Fund Flow Statement are basically Working Capital items (i.e., current items).

Modified Date: Thu, Jul 01, 2010 03:49:13 PM

Output Date: Tue, Jul 06, 2010 11:48:50 AM

REV-II

Project: Management Accounting_Debarshi Bhattacharyya ACE Pro India Pvt. Ltd. File: X:\Pearson\Management Accounting_Debarshi Bhattacharyya\MAIN\M05\LAYOUT_M05\M05_DEBA_ISBN_EN_SE_C05_Part-1.indd

342

MANAGEMENT ACCOUNTING

5.3 MEANING OF FLOW OF FUND Flow of Fund means the inward and outward movement of a Fund of an enterprise. For the purpose, Fund refers to Working Capital and flow means movement or changes. Therefore, Flow of Fund means movement of or changes in the Working Capital (i.e., current) items. Working Capital items are Current Assets and Current Liabilities. Hence, where there is an inward or outward movement of Current Assets (e.g., debtors, stock, bills receivable, cash, bank) and Current Liabilities (e.g., creditors, bills payable, outstanding expenses), there is a Flow of Fund. In short, the Flow of Fund is identified by the means of inward or outward movement of Current Assets and Current Liabilities.  

When Current Assets increase or Current Liabilities decrease—Inflows of Fund. When Current Assets decrease or Current Liabilities increase—Outflows of Fund.

5.4 WHAT IS FUND FLOW STATEMENT? A Fund Flow Statement is a summarized statement of the movement of Fund (i.e., Working Capital) from different activities of a concern during an accounting period. It is prepared to locate the various sources of Fund inflows into the business and also to identify the various purposes of Fund outflows from the business, during two consecutive Balance Sheet dates. As it is a summarized statement of Fund inflows and Fund outflows from different activities of an enterprise during a particular period, the management gets a vivid picture of the movement of Fund in between two consecutive Balance Sheet dates by preparation of a Fund Flow Statement. One side of the Fund Flow Statement shows the various sources of Fund and the other side shows the various applications of Fund during an accounting period. A Fund Flow Statement acts as an important tool of Financial Analysis to the management. Thus, the management can assess the movement of Fund from different activities of the business and can draw up its future planning. 5.5 IMPORTANCE OR PURPOSES OF FUND FLOW STATEMENT Fund Flow Statement acts as an important tool for Financial Analysis and shows the brief reasons for change in the Working Capital between two Balance Sheet dates. It has more importance from the viewpoint of the management of a concern. It serves the following purposes: i. Fund Flow Statement explains how the financial position has changed from the beginning of an accounting period to the end of that period. ii. It acts as an important instrument for allocation of resources of a concern. It enables the concern for making plans for optimum allocation of resources. iii. It answers many intricate financial queries such as reasons for changes in Working Capital position, various sources of repayment of loans, various sources of acquiring capital, sources of Fund from nonoperating activities, applications of Fund towards various non-operating activities, Fund generation through operating activities and so on. iv. It helps the management in the process of effective use of the Working Capital of a concern. v. As projected Fund Flow Statement estimates the future Fund position of a concern, it helps in framing the rational dividend policy of the concern without any adverse impact on the operating Working Capital. vi. It helps the management in planning the future financial requirements. vii. It projects the future Fund generation, aids in future investment planning and also helps in the Working Capital management of a concern. 5.6 LIMITATIONS OF FUND FLOW STATEMENT Although a Fund Flow Statement has more importance from the view point of the management of a concern, yet it suffers from the following limitations: i. Fund Flow Statement is not a basic Financial Statement, but is a supplementary statement. It does not disclose any new fact which is not reflected in the Income Statement and the Balance Sheet. ii. It provides a partial financial information to the management.

Modified Date: Thu, Jul 01, 2010 03:49:13 PM

Output Date: Tue, Jul 06, 2010 11:48:50 AM

REV-II

Project: Management Accounting_Debarshi Bhattacharyya ACE Pro India Pvt. Ltd. File: X:\Pearson\Management Accounting_Debarshi Bhattacharyya\MAIN\M05\LAYOUT_M05\M05_DEBA_ISBN_EN_SE_C05_Part-1.indd

FUND FLOW ANALYSIS

343

iii. iv. v. vi.

It cannot present the continuous changes in the financial position. It does not indicate the structural change of an Asset or a Liability. It is prepared on the basis of historical data. It exhibits the changes in the Fund position, but does not indicate the changes in the cash position, which is most important for every business concern. vii. Nowadays, Fund Flow Analysis is not as much relevant to the management for Financial Analysis as is Cash Flow Analysis. 5.7 PROFORMA OF PRESENTATION OF FUND FLOW STATEMENT A Fund Flow Statement is generally prepared in an account form, containing therein the sources of Fund on the left-hand side and applications or uses of Fund on the right-hand side. A proforma of presentation of Fund Flow Statement is shown as follows: Fund Flow Statement of for the period Sources of Fund Fund from Operation New Issue of Shares New Issue of Debentures Loan raised Sale of Fixed Assets Income received on Investments Sale of Investments Decrease in Working Capital (Bal. fig.)

Rs. − − − − − − − −

Applications of Fund Redemption of Preference Shares Redemption of Debentures Purchase of Fixed Assets Further investments made Payment of Dividends Payment of Taxes Repayment of Loans Payment of Interest on Debt Increase in Working Capital (Bal. fig.)



Rs. − − − − − − − − − −

5.8 DIFFERENT SOURCES OF FUND Sources of Fund may be classified into two broad categories: Fund from the operating activities and Fund from the non-operating activities. Sources of Fund from the operating activities refer to the Fund that is coming into the business due to the operating activities of the concern. Examples of sources of Fund from such operating activities are: Cash Sale of Goods, Credit Sale of Goods, discount received, commission received and so on. On the other hand, the sources of Fund from the non-operating activities refer to the Fund that is coming into the business due to the various activities of the concern other than the operating activities. Examples of sources of Fund from such operating activities are: Sale of Fixed Assets and long-term investments, proceeds received from the issue of equity, preference shares and debentures, proceeds received from long-term borrowings and so on. 5.9 DIFFERENT APPLICATIONS OF FUND Applications or uses of Fund may also be classified into two broad categories: Fund for operating activities and Fund for non-operating activities. Applications of Fund for operating activities refer to the Fund that is going out of the business due to the operating activities of the concern. Examples of applications of Fund for such operating activities are: Cash Purchase of Goods, Credit Purchase of Goods, Payment of Wages and Salaries, outstanding wages and salaries, and so on. On the other hand, applications of Fund for non-operating activities refer to the Fund that is going out of the business due to various activities of the concern other than the operating activities. Examples of applications of Fund for such operating activities are: purchase of Fixed Assets and long-term investments, redemption of preference shares and debentures, repayment of long-term borrowings, payment of dividend, tax and so on. 5.10 WHAT IS FUND FROM OPERATION? Fund from Operation (or Net Fund Flow from operation) refers to the difference between the inflows of Fund and the outflows of Fund from the operating activities of the concern. It is the net Flow of Fund from the

Modified Date: Thu, Jul 01, 2010 03:49:13 PM

Output Date: Tue, Jul 06, 2010 11:48:50 AM

REV-II

Project: Management Accounting_Debarshi Bhattacharyya ACE Pro India Pvt. Ltd. File: X:\Pearson\Management Accounting_Debarshi Bhattacharyya\MAIN\M05\LAYOUT_M05\M05_DEBA_ISBN_EN_SE_C05_Part-1.indd

344

MANAGEMENT ACCOUNTING

operating activities of the concern. Net Flow of Fund refers to the excess of Fund inflow over the Fund outflow or vice versa. For ascertaining the Fund from Operation, two important points are to be simultaneously considered. They are: (a) There are inflows or outflows of Fund (i.e., Current Assets or Current Liabilities); (b) These inflows or outflows of Fund occur due to the operational activities of the concern. Suppose, the raising of long-term loan reflects the inflow of Fund, still, such inflow of Fund has not occurred due to the operational activities of the concern. Sale of Goods, Purchase of Goods, Wages, Salaries and so on, are the causes of movement of Fund due to the operational activities of the concern. Hence, the Fund from Operation exhibits the Net Flow of Fund coming into or going out of the business due to its operating activities. Illustration Say, there is an inflow of Fund of Rs. 2,40,000 by means of sale of goods and there is an outflow of Fund of Rs. 1,50,000 by means of Purchase of Goods. Therefore, Fund from Operation = Inflows of Fund from operating activities − Outflows of Fund from operating activities = Sale of Goods − Purchase of Goods = Rs. 2,40,000 − Rs. 1,50,000 = Rs. 90,000. 5.11 DIFFERENT APPROACHES OF COMPUTATION OF FUND FROM OPERATION The Fund from Operation (i.e., Net Fund Flow from operational activities) may be computed under two different approaches, such as Direct Approach and Indirect Approach. Depending upon the information given, a particular approach for computation of Fund from Operation is followed. These two different approaches of computation of Fund from Operation are separately explained as follows. 5.11.1 Direct Approach of Computation of Fund from Operation When the Fund from Operation is computed directly by deduction of operating Fund outflows from the operating Fund inflows, such an approach is called ‘direct approach.’ Where the details of the operating incomes and expenses are given and the Net Fund Flow from the Operation is computed by following the Direct Approach. Under this approach, the difference between the Fund inflows and Fund outflows from operation represents the Net Fund Flow from operation. The proforma of computation of Fund from Operation under Direct Approach is shown as follows: Statement showing computation of Fund from Operation for the period Rs. Inflows of Fund from Operating Activities: Cash Sales Credit Sales Other Operating Fund Inflows

Rs.

− − − −

Less:

Outflows of Fund from Operating Activities: Cash Purchases Credit Purchases Total Wages Total Salaries Total General Expenses Other Operating Fund Outflows Net Fund Flow from Operation

− − − − − − − −

5.11.2 Indirect Approach of Computation of Fund from Operation When Fund from the operation is computed by adding back all the Non-Operating and Non-Fund items considered in the Profit & Loss Account (Profit & Loss A/c) to the net profit for the year, such approach is called Indirect Approach of computation of Fund from Operation.

Modified Date: Thu, Jul 01, 2010 03:49:13 PM

Output Date: Tue, Jul 06, 2010 11:48:50 AM

REV-II

Project: Management Accounting_Debarshi Bhattacharyya ACE Pro India Pvt. Ltd. File: X:\Pearson\Management Accounting_Debarshi Bhattacharyya\MAIN\M05\LAYOUT_M05\M05_DEBA_ISBN_EN_SE_C05_Part-1.indd

345

FUND FLOW ANALYSIS

Where the Net Profit along with all the Non-Operating and Non-Fund items considered in the Profit & Loss A/c are given, the Net Fund Flow from the operation is computed by the following Indirect Approach. Where the Balance Sheets of two consecutive years along with the additional information are given, the Net Fund Flow from the operation is also computed by following the Indirect Approach. Indirect approach of computation of Fund from Operation may also be calculated under two different forms such as ‘in statement form’ and ‘in account form.’ Same thing is done under these two alternative formats. Proforma of computation of Fund from Operation under Indirect Approach in statement form is shown as follows: Calculation of Fund from Operation for the Rs. Add:

Less:

Net profit for the year Adjustment for Non-Current and Non-Operating Items debited to Profit & Loss A/c: Loss on Sale of Land Discount on Issue of Share Interest on Debentures Goodwill written off Depreciation General Reserve Provision for Tax Proposed Dividend Interim Dividend Other non-current & non-operating items debited Adjustment for Non-Current and Non-Operating Items credited to Profit & Loss A/c: Interest on Investment Dividend Received Profit on Sale of Plant Interest on Bank Deposit Refund of Tax Other non-current & non-operating items credited Net Fund Flow from Operation

Rs. −

− − − − − − − − − −

− −

− − − − − −

− −

Proforma of computation of Fund from Operation under Indirect Approach in account form is shown below: Adjusted Profit & Loss A/c for the Period Dr. Particulars To Non-Current & Non-Operating items charged: Transfer to General Reserve Proposed Dividend Goodwill Preliminary Expenses Depreciation on Fixed Assets Provision for Taxation Loss on Sale of Investment Interest on Debentures Other Non-Current & Non-Operating Items To Balance c/f

Modified Date: Thu, Jul 01, 2010 03:49:13 PM

Amount (Rs.)

Particulars By Balance b/f

− − − − − − − −

Cr. Amount (Rs.) −

By Non-Current & Non-Operating items credited: Profit on Sale of Machinery Income from Investment Other Non-Current & Non-Operating Items

− − −

By Net Fund Flow from Operation (Bal. fig.)



− − −

Output Date: Tue, Jul 06, 2010 11:48:50 AM



REV-II

Project: Management Accounting_Debarshi Bhattacharyya ACE Pro India Pvt. Ltd. File: X:\Pearson\Management Accounting_Debarshi Bhattacharyya\MAIN\M05\LAYOUT_M05\M05_DEBA_ISBN_EN_SE_C05_Part-1.indd

346

MANAGEMENT ACCOUNTING

5.12 IS DEPRECIATION A SOURCE OF FUND? Depreciation is decrease in the value of Fixed Assets due to wear and tear, lapse of time, exhaustion, obsolescence and so on. It is definitely an operating expense of a concern, but does not involve the Flow of Fund. Accounting entries passed for adjusting depreciation are as follows: i. Depreciation A/c Dr. To Fixed Asset A/c ii. Profit & Loss A/c Dr. To Depreciation A/c From the above entries, it has been transpired that neither Current Asset nor Current Liability (i.e., Fund item) is being involved for adjusting depreciation. More clearly, there is no movement of Fund for adjusting depreciation. Hence, depreciation should not, generally, be regarded as a source (or a use) of Fund. Yet, there are two opinions among the accountants as regards to the consideration whether depreciation is a source of Fund or not. Some accountants opine that depreciation should be regarded as a source of Fund, while some others consider that it is not at all a source of Fund. Arguments advocated by those who opine that depreciation is a source of Fund are as follows: i. Depreciation is an expired cost of Fixed Assets which is recovered in cash by way of its inclusion in the cost of production. Thus, depreciation is basically treated as sale of Fixed Asset in piecemeal. If the sale of Fixed Asset is considered as a source of Fund, then depreciation should also be treated as a source of Fund. ii. Depreciation is charged against revenue as an expense, unlike other operating expenses, with a view to set aside a Fund that is equal to the amount of increase in the Working Capital. iii. By providing depreciation on the Fixed Assets, a Fund equal to the loss in the value of Fixed Assets is set aside as a recovery of capital. iv. While calculating the Net Fund Flow from operation, depreciation is added back to the Net Profit. Had depreciation not been a source of Fund, such addition would not have been allowed. v. By charging depreciation, a concern reduces its Tax Liability and thus saves its Fund. Arguments advocated by those who opine that depreciation is not a source of Fund are as follows: i. From the accounting entries of adjusting depreciation, it is transpired that, though depreciation is an operating expense of a concern, it does not involve any Flow of Fund. ii. Depreciation is charged against revenue as an operating expense, just like other cash operating expenses, with a view to ascertain real profit of the concern. iii. While calculating the Net Fund Flow from operation under Direct Approach, depreciation is neither considered as a source of Fund like Cash Sales, Credit Sales and so on, nor is considered as a use of Fund like purchases, wages and so on. iv. While calculating the Net Fund Flow from operation under Indirect Approach, depreciation is added back to the net profit just to nullify the amount that has already reduced the Fund from Operation by way of charging depreciation. v. If depreciation is really a Source of Fund, a concern could have improved its Fund position merely by charging more depreciation. vi. Sale of Fixed Asset is not, in true sense, a source of augmented Fund, rather is a recovery of capital. vii. If there is no Sale of Goods in any year, there is no inflow of Fund from Operation for that year. Even if there is neither sale Nor Profit during a year, depreciation is charged for that year. Had depreciation been a source of Fund, then there would have been some inflow of Fund from Operation during the year of ‘No Sale of Goods.’ But practically, it does not happen so. From the above discussion, it can be concluded that, though depreciation is an operating expense, it should not, generally, be regarded as a source (or a use) of Fund as it involves no movement of Fund (i.e., Current Asset or Current Liability).

Modified Date: Thu, Jul 01, 2010 03:49:13 PM

Output Date: Tue, Jul 06, 2010 11:48:50 AM

REV-II

Project: Management Accounting_Debarshi Bhattacharyya ACE Pro India Pvt. Ltd. File: X:\Pearson\Management Accounting_Debarshi Bhattacharyya\MAIN\M05\LAYOUT_M05\M05_DEBA_ISBN_EN_SE_C05_Part-1.indd

FUND FLOW ANALYSIS

347

5.13 COMPARISON BETWEEN FUND FLOW STATEMENT, INCOME STATEMENT AND BALANCE SHEET Income Statement and Balance Sheet are two basic Financial Statements. Income Statement ascertains Net Profit or Net Loss of a concern during an accounting period, whereas Balance Sheet exhibits the financial position of the concern at the end of that accounting period. On the basis of the historical data as provided through these basic Financial Statements, a Fund Flow Statement is prepared to show the changes in the financial position (i.e., Fund position) of a concern between two consecutive Balance Sheet dates. Fund Flow Statement is a supplementary Financial Statement which shows he changes in the Fund position (i.e., Working Capital position) from the beginning of the accounting period to its end. A Fund Flow Statement cannot be a substitute of two basic Financial Statements. 5.14 DISTINCTION BETWEEN FUND FLOW STATEMENT AND CASH FLOW STATEMENT Both Cash Flow Statement and Fund Flow Statement are very important tools to the management for Financial Analysis and Decision Making. But, there are some Fundamental differences between these two, which are as follows: Cash Flow Statement 1. Cash Flow Statement exhibits inflows and outflows of cash and cash equivalents only. 2. While preparing a Cash Flow Statement, cash refers to Cash in hand, Cash at bank (demand deposits) and shortterm investments (i.e., cash & cash equivalents only). 3. Cash Flow Statement is prepared on the cash basis of accounting. 4. It is the most important tool for Short-term Financial Analysis. 5. By preparing a Cash Flow Statement, the movement of cash only can be known and identified. 6. Net cash inflows from the different activities of an enterprise, such as operating, investing and financing activities, are separately exhibited through the Cash Flow Statement. 7. It shows the brief reasons for change in cash between two Balance Sheet dates. 8. Cash Flow Statement starts with opening cash and cash equivalents of an accounting period and ends with closing cash and cash equivalents of that accounting period.

Fund Flow Statement 1. Fund Flow Statement exhibits inflows and outflows of Funds (i.e., Working Capital). 2. While preparing a Fund Flow Statement, Fund refers to Working Capital (i.e., Current Assets and Current Liabilities). 3. Fund Flow Statement is prepared on the accrual basis of accounting. 4. It is an important tool for Long-term Financial Analysis. 5. By preparing a Fund Flow Statement, the movement of Working Capital items (i.e., Current Assets and Current Liabilities) can be known and identified. 6. Net Fund Flows from the different activities of an enterprise are not separately exhibited through the Fund Flow Statement. 7. It shows the brief reasons for change in the Working Capital between the two Balance Sheet dates. 8. Fund Flow Statement shows the amount of changes between the opening and closing Working Capital balances of an accounting period.

Tutorial Notes to Students for Solving Problems For preparing a Fund Flow Statement, students are advised to follow the following steps: i. Prepare separate accounts for all non-operating items. If there is no major adjustment in some nonoperating items, it is advised to show their changes in one separate statement. ii. After considering all non-operating items, calculate the Fund from Operation. iii. Prepare a statement showing changes in the Working Capital taking operating Current Assets and Liabilities alone and get the increase or decrease in the Working Capital. iv. Finally, prepare the Fund Flow Statement showing the sources of Fund (i.e., inflows of Fund) on the left-hand side and applications or uses of Fund (i.e., outflows of Fund) on the right-hand side of the statement. Increase or decrease in the Working Capital as computed in the statement showing changes in the Working Capital would appear in the Fund Flow Statement as a balancing figure.

Modified Date: Thu, Jul 01, 2010 03:49:13 PM

Output Date: Tue, Jul 06, 2010 11:48:50 AM

REV-II

Project: Management Accounting_Debarshi Bhattacharyya ACE Pro India Pvt. Ltd. File: X:\Pearson\Management Accounting_Debarshi Bhattacharyya\MAIN\M05\LAYOUT_M05\M05_DEBA_ISBN_EN_SE_C05_Part-1.indd

348

MANAGEMENT ACCOUNTING

5.15 WORKED-OUT PROBLEMS Problem 1 Choose the correct alternative from the following: i. Depreciation on Fixed Assets: (a) causes inflow of Fund; (b) causes outflow of Fund; (c) has no effect on Fund Flow. ii. Interest paid on debentures charged in the Profit & Loss A/c: (a) increases Fund from Operation; (b) decreases Fund from Operation; (c) has no effect on Fund from Operation. iii. Credit Sale of Goods: (a) increases Fund from Operation; (b) decreases Fund from Operation; (c) has no effect on Fund from Operation. iv. Proposed dividend on shares charged in the Profit & Loss A/c: (a) increases Fund from Operation; (b) decreases Fund from Operation; (c) has no effect on Fund from Operation. v. Purchase of Fixed Assets results in: (a) inflow of Fund; (b) outflow of Fund; (c) has no effect on Fund Flow. vi. Sale of long-term investment: (a) causes inflow of Fund; (b) causes outflow of Fund; (c) has no effect on Fund Flow. vii. Loss on sale of Fixed Asset charged in the Profit & Loss A/c: (a) increases Fund from Operation; (b) decreases Fund from Operation; (c) has no effect on Fund from Operation. viii. If the Net Profit of a business concern is Rs. 80,000 and depreciation charged in the Profit & Loss A/c is Rs. 20,000, then Fund from Operation is equal to: (a) Rs. 60,000; (b) Rs. 80,000; (c) Rs. 1,00,000. ix. Provision for Income Tax charged in the Profit & Loss A/c: (a) increases Fund from Operation; (b) decreases Fund from Operation; (c) has no effect on Fund from Operation. x. Credit Purchase of Goods: (a) increases Fund from Operation; (b) decreases Fund from Operation; (c) has no effect on Fund from Operation. xi. Issue of Equity Shares for cash causes: (a) inflow of Fund; (b) outflow of Fund; (c) has no effect on Fund Flow. xii. If the Net Profit of a business concern is Rs. 1,50,000 after crediting an interest on investment of Rs. 10,000 and after transferring Rs. 30,000 in the general reserve, then Fund from the operation is equal to: (i) Rs. 1,10,000; (ii) Rs. 1,30,000; (iii) Rs. 1,70,000. Ans.: i (c); ii (b); iii (a); iv (b); v (b); vi (a); vii (b); viii (c); ix (b); x (b); xi (a); xii (b). Calculation of Fund from Operation Problem 2 From the following information, calculate the Net Fund Flow from operation for the year that ended on 31 March 2010: Profit & Loss A/c for the year that ended on 31 March 2010 Dr. Particulars To Loss on Sale of Land To Discount on Issue of Shares To Interest on Debentures To Depreciation To Goodwill written off To General Reserve To Tax Provision To Proposed Dividend To Interim Dividend To Net Profit

Modified Date: Thu, Jul 01, 2010 03:49:13 PM

Rs. 40,000 10,000 58,000 1,20,000 15,000 75,000 30,000 80,000 20,000 3,52,000 8,00,000

Particulars By Gross Profit By Interest on Investment By Dividend Received By Profit on Sale of Plant By Interest on Bank Deposit By Refund of Tax

Output Date: Tue, Jul 06, 2010 11:48:50 AM

Cr. Rs. 7,10,000 15,000 18,000 20,000 35,000 2,000

8,00,000

REV-II

Project: Management Accounting_Debarshi Bhattacharyya ACE Pro India Pvt. Ltd. File: X:\Pearson\Management Accounting_Debarshi Bhattacharyya\MAIN\M05\LAYOUT_M05\M05_DEBA_ISBN_EN_SE_C05_Part-1.indd

FUND FLOW ANALYSIS

349

Stop and Think As details of the incomes and expenses considered in Profit & Loss A/c along with the Gross Profit are given in the problem, the Net Fund Flow from operation in this case is to be computed by following the Indirect Approach ‘in statement form’ as given in the following solution.

Solution Calculation of Net Fund Flow from operation of Add:

for the year that ended on 31 March 2010

Rs. Net Profit for the year Adjustment for Non-Current and Non-Operating Items Debited to Profit & Loss A/c: Loss on Sale of Land 40,000 Discount on Issue of Share 10,000 Interest on Debentures 58,000 Goodwill written off 15,000 Depreciation 1,20,000 General Reserve 75,000 Tax Provision 30,000 Proposed Dividend 80,000 Interim Dividend 20,000

Less: Adjustment for Non-Current and Non-Operating Items Credited to Profit & Loss A/c: Interest on Investment Dividend Received Profit on Sale of Plant Interest on Bank Deposit Refund of Tax Net Fund Flow from Operation

15,000 18,000 20,000 35,000 2,000

Rs. 3,52,000

4,48,000 8,00,000

90,000 7,10,000

Problem 3 3 From the following particulars, calculate the Fund from Operations: Rs. 1,60,000 2,00,000

Profit & Loss A/c: as on 01 April 2006 as on 31 March 2007 Transactions during the year: Transfer to Revenue Reserve Depreciation on Fixed Assets Underwriting Commission written off Interest Received Interim Dividend Paid Sale of Old Machinery (Book Value − Rs. 48,000)

40,000 16,000 8,000 4,000 16,000 56,000

[B.Com. (Hons), Calcutta University—2008] Stop and Think As the non-operating income and expense considered in the Profit & Loss A/c are given in the problem along with opening and closing balances in Profit & Loss A/c, the Net Fund Flow from operation in this case is to be computed by following the Indirect Approach as given in the following solution.

Modified Date: Thu, Jul 01, 2010 03:49:13 PM

Output Date: Tue, Jul 06, 2010 11:48:50 AM

REV-II

Project: Management Accounting_Debarshi Bhattacharyya ACE Pro India Pvt. Ltd. File: X:\Pearson\Management Accounting_Debarshi Bhattacharyya\MAIN\M05\LAYOUT_M05\M05_DEBA_ISBN_EN_SE_C05_Part-1.indd

350

MANAGEMENT ACCOUNTING

Solution Calculation of Fund from Operations of

for the year that ended on 31 March 2007 Rs.

Less:

Net Profit after Appropriation for the year: Balance of Profit & Loss A/c as on 31 March 2007 Balance of Profit & Loss A/c as on 01 April 2006

Rs.

2,00,000 1,60,000 40,000

Add: Adjustment for Non-Current and Non-Operating Items debited to Profit & Loss A/c: Transfer to Revenue Reserve Depreciation on Fixed Assets Underwriting Commission written off Interim Dividend Paid

40,000 16,000 8,000 16,000 80,000 1,20,000

Less:

Adjustment for Non-Current and Non-Operating Items credited to Profit & Loss A/c: Interest Received Profit on Sale of Old Machinery (56,000 − 48,000)

4,000 8,000 12,000 1,08,000

Fund from Operations

Problem 4 From the following information, calculate the Net Fund Flow from the operation of a company for the year that ended on 31 March 2010: 31 March 2009 (Rs.) 2,20,000 56,000 54,000 48,000 34,000

Balance in Profit & Loss A/c Stock in Trade Sundry Debtors Sundry Creditors Cash at Bank

31 March 2010 (Rs.) 2,50,000 36,000 82,000 72,000 56,000

While ascertaining the net profit for the year that ended on 31 March 2010, the following items were taken into the Profit & Loss A/c for the year: Rs. 40,000 30,000 20,000 10,000 20,000 25,000 30,000 15,000 30,000 10,000

Transfer to General Reserve Proposed Dividend Goodwill written off Preliminary Expenses written off Interest on Debentures Depreciation on Fixed Assets Provision for Taxation Loss on Sale of Investment Profit on Sale of Machinery Income from Investment

Stop and Think As the details of the non-operating income and expense are given in the problem along with the opening and closing balances in Profit & Loss A/c, the Net Fund Flow from operation in this case is to be computed by following the Indirect Approach as given in the following solution. As solution to the previous problem is done in the statement (Continued)

Modified Date: Thu, Jul 01, 2010 03:49:13 PM

Output Date: Tue, Jul 06, 2010 11:48:50 AM

REV-II

Project: Management Accounting_Debarshi Bhattacharyya ACE Pro India Pvt. Ltd. File: X:\Pearson\Management Accounting_Debarshi Bhattacharyya\MAIN\M05\LAYOUT_M05\M05_DEBA_ISBN_EN_SE_C05_Part-1.indd

FUND FLOW ANALYSIS

351

form, under Indirect Approach, solution to the given problem has been done in the account form, under Indirect Approach, as an alternative method for the students.

Solution Adjusted Profit & Loss A/c for the year that ended on 31 March 2010 Dr.

Cr.

Particulars To Non-Current & Non-Operating items charged: Transfer to General Reserve Proposed Dividend Goodwill Preliminary Expenses Depreciation on Fixed Assets Provision for Taxation Loss on Sale of Investment Interest on Debentures To Balance c/f

Amount (Rs.)

Particulars By Balance b/f

40,000 30,000 20,000 10,000 25,000 30,000 15,000 20,000 2,50,000 4,40,000

By Non-Current & Non-Operating Items Credited: Profit on Sale of Machinery Income from Investment

By Net Fund Flow from Operation (Bal. fig.)

Amount (Rs.) 2,20,000

30,000 10,000

1,80,000 4,40,000

Problem 5 From the following information, calculate the Net Fund Flow from operation of a concern for the year that ended on 31 March 2010: Cash Sales for the year Credit Sales for the year Collection from Debtors during the year Cash Purchases for the year Credit Purchases for the year Payment to Creditors during the year Wages Paid during the year Outstanding Wages for the year Salaries Paid during the year Salaries for the year General Expenses Paid during the year Unpaid General Expenses for the year Depreciation on Fixed Assets for the year Proposed Dividend for the year Dividend Paid during the year Interest Received on Investment during the year Payment of Income tax during the year Provision for Income Tax for the year

Rs. 1,20,000 6,60,000 4,40,000 80,000 3,25,000 2,40,000 60,000 18,000 30,000 40,000 20,000 6,000 26,000 28,000 30,000 20,000 32,000 35,000

Stop and Think As details of the operating income and expense are given in the problem, the Net Fund Flow from operation in this case can be computed by following the Direct Approach as given in the following solution.

Modified Date: Thu, Jul 01, 2010 03:49:13 PM

Output Date: Tue, Jul 06, 2010 11:48:50 AM

REV-II

Project: Management Accounting_Debarshi Bhattacharyya ACE Pro India Pvt. Ltd. File: X:\Pearson\Management Accounting_Debarshi Bhattacharyya\MAIN\M05\LAYOUT_M05\M05_DEBA_ISBN_EN_SE_C05_Part-1.indd

352

MANAGEMENT ACCOUNTING

Solution Statement showing computation of Net Fund Flow from Operation of a concern for the year that ended on 31 March 2010 Rs. Inflows of Fund from Operating Activities: Cash Sales Credit Sales

Rs.

1,20,000 6,60,000 7,80,000

Less:

Outflows of Fund from Operating Activities: Cash Purchases Credit Purchases Total Wages (60,000 + 18,000) Total Salaries Total General Expenses (20,000 + 6,000)

80,000 3,25,000 78,000 40,000 26,000 5,49,000 2,31,000

Net Fund Flow from Operation

Problem 6 From the following information, calculate the Net Cash Flow from the operating activities of F Ltd for the year that ended on 31 March 2010: Profit & Loss A/c for the year that ended on 31 March 2010 Dr.

Cr. Rs.

To Opening Stock To Purchases: Cash Credit To Wages: Paid Outstanding To Salaries: Paid Outstanding To Rent Paid To Depreciation on Assets To Preliminary Expenses written off To Interest on Debentures To Provision for Taxation To Net Profit for the year

50,000 1,90,000 55,000 5,000 25,000 2,000

Rs. 40,000

Rs. By Sales: Cash Credit

2,40,000 By Closing Stock By Dividend from Investment

Rs.

70,000 3,80,000 4,12,000 35,000 15,000

60,000

27,000 5,000 18,000 10,000 15,000 25,000 60,000 5,00,000

5,00,000

Stop and Think As the detail of the Profit & Loss A/c is given in the problem, the Net Fund Flow from Operation in this case can be computed by following both Direct Approach as well as Indirect Approach. Here, the solution to the given problem has been done under both the approaches, but students are advised to show either of the two approaches for their examination purpose.

Modified Date: Thu, Jul 01, 2010 03:49:13 PM

Output Date: Tue, Jul 06, 2010 11:48:50 AM

REV-II

Project: Management Accounting_Debarshi Bhattacharyya ACE Pro India Pvt. Ltd. File: X:\Pearson\Management Accounting_Debarshi Bhattacharyya\MAIN\M05\LAYOUT_M05\M05_DEBA_ISBN_EN_SE_C05_Part-1.indd

FUND FLOW ANALYSIS

353

Solution A. Under Direct Approach: Statement showing computation of Net Fund Flow from Operation of F Ltd for the year that ended on 31 March 2010

Less:

Inflows of Fund from Operating Activities: Cash Sales Credit Sales Outflows of Fund from Operating Activities: Cash Purchases Credit Purchases Total Wages (55,000 + 5,000) Total Salaries (25,000 + 2,000) Rent Decrease in Value of Stock (40,000 − 35,000) Net Fund Flow from Operation

Rs.

Rs.

70,000 3,80,000

4,50,000

50,000 1,90,000 60,000 27,000 5,000 5,000 3,37,000 1,13,000

B. Under Indirect Approach: Statement showing computation of Net Fund Flow from Operation of F Ltd for the year that ended on 31 March 2010 Rs. Add:

Net Profit for the year Adjustment for Non-Current and Non-Operating Items debited to Profit & Loss A/c: Depreciation on Assets Preliminary Expenses written off Interest on Debentures Provision for Taxation

Rs. 60,000

18,000 10,000 15,000 25,000 68,000 1,28,000

Less:

Adjustment for Non-Current and Non-Operating Items credited to Profit & Loss A/c: Dividend from Investment Net Fund Flow from Operation

15,000 1,13,000

Problem 7 From the following particulars, calculate the Net Fund Flow from Operation of U Ltd for the year that ended on 31 March 2010. Rs. Balance of Profit & Loss A/c: As on 31 March 2010 As on 31 March 2009 Appropriation of Profit for the year 2009−10: Transfer to General Reserve Proposed Dividend Expenses and Losses for the year 2009−10: Interest on Debentures Depreciation on Fixed Assets Wages & Salaries Provision for Taxation Goodwill written off Loss on Sale of Machinery

3,98,000 2,72,000 24,000 20,000 16,000 18,000 37,000 40,000 14,000 12,000 (Continued)

Modified Date: Thu, Jul 01, 2010 03:49:13 PM

Output Date: Tue, Jul 06, 2010 11:48:50 AM

REV-II

Project: Management Accounting_Debarshi Bhattacharyya ACE Pro India Pvt. Ltd. File: X:\Pearson\Management Accounting_Debarshi Bhattacharyya\MAIN\M05\LAYOUT_M05\M05_DEBA_ISBN_EN_SE_C05_Part-1.indd

354

MANAGEMENT ACCOUNTING

Rs. Income and Gain for the year 2009−10: Profit on Sale of Furniture Income from Investment Expenses Paid During the year 2009−10: Interest on Debentures Wages & Salaries Income Tax

13,000 7,000 18,000 39,000 53,000

Stop and Think As the non-operating income and expenses considered in the Profit & Loss A/c are given in the problem along with the opening and closing balances in the Profit & Loss A/c, the Net Fund Flow from Operation in this case is to be computed by following the Indirect Approach as given in the following solution.

Solution Statement showing computation of Net Fund Flow from Operation of F Ltd for the year that ended on 31 March 2010 Rs.

Less: Add:

Less:

Net Profit after Appropriation for the year: Balance of Profit & Loss A/c as on 31 March 2010 Balance of Profit & Loss A/c as on 31 March 2009 Non-Operating and Non-Current Items debited to Profit & Loss A/c: Transfer to General Reserve Proposed Dividend Interest on Debentures Depreciation on Fixed Assets Provision for Taxation Goodwill written off Loss on Sale of Machinery Non-Operating and Non-Current Items credited to Profit & Loss A/c: Profit on Sale of Furniture Income from Investment Net Fund Flow from Operation

3,98,000 2,72,000 24,000 20,000 16,000 18,000 40,000 14,000 12,000

13,000 7,000

Rs.

1,26,000

1,44,000 2,70,000

20,000 2,50,000

Tutorial Note As wages and salaries are the operating expenses, these are not added back to the Net Profit.

Preparation of Fund Flow Statement Problem 8 The Balance Sheet of Oedrila Ltd for the year that ended on 31 March 2009 and 2010 are as follows: Liabilities Equity Share Capital

31 March 2009 Rs.

31 March 2010 Rs.

2,00,000

3,00,000

Assets Land & Building Plant & Machinery

31 March 2009 Rs. 1,50,000 1,60,000

31 March 2010 Rs. 2,00,000 2,60,000 (Continued)

Modified Date: Thu, Jul 01, 2010 03:49:13 PM

Output Date: Tue, Jul 06, 2010 11:48:50 AM

REV-II

Project: Management Accounting_Debarshi Bhattacharyya ACE Pro India Pvt. Ltd. File: X:\Pearson\Management Accounting_Debarshi Bhattacharyya\MAIN\M05\LAYOUT_M05\M05_DEBA_ISBN_EN_SE_C05_Part-1.indd

355

FUND FLOW ANALYSIS

31 March 2009 Rs. 50,000 60,000 80,000 30,000 5,000 20,000 15,000 4,60,000

Liabilities General Reserve Profit & Loss A/c Debentures Sundry Creditors Bills Payable Proposed Dividend Provision for Tax

31 March 2010 Rs. 60,000 90,000 1,00,000 50,000 10,000 30,000 25,000 6,65,000

Assets Investment Stock in Trade Sundry Debtors Cash & Bank Bills Receivable Preliminary Expenses

31 March 2009 Rs. 30,000 50,000 30,000 20,000 10,000 10,000

31 March 2010 Rs. 50,000 80,000 50,000 15,000 5,000 5,000

4,60,000

6,65,000

Additional information: i. Depreciation of Rs. 20,000 was charged on plant and machinery during the year 2009−10. ii. A machinery costing Rs. 50,000, on which the total depreciation amounting to Rs. 30,000 was provided, was sold at Rs. 25,000 during the year 2009−10. iii. A dividend of Rs. 24,000 was paid during the year 2009−10. Prepare a statement showing the changes in the Working Capital and a Fund Flow Statement for the year that ended on 31 March 2010. Solution Books of Oedrila Ltd Statement showing changes in the Working Capital for the year that ended on 31 March 2010 Elements of Working Capital Current Assets: Stock in Trade Sundry Debtors Cash & Bank Bills Receivable Less:

Current Liabilities: Sundry Creditors Bills Payable Working Capital Increase in Working Capital

As on 31 March 2009 Rs.

As on 31 March 2010 Rs.

50,000 30,000 20,000 10,000 1,10,000

80,000 50,000 15,000 5,000 1,50,000

(30,000) (5,000) 75,000

(50,000) (10,000) 90,000 15,000

Fund Flow Statement for the year that ended on 31 March 2010 Sources of Fund Sale Proceeds of Machinery New Issue of Equity Shares1 New Issue of Debentures3 Net Fund Flow from Operation10

Rs. 25,000 1,00,000 20,000 1,19,000

Applications of Fund Purchase of Land & Building6 Further Investment Made8 Purchase of Plant & Machinery7 Payment of Tax5 Payment of Dividend4 Increase in Working Capital (Bal. fig.)

2,64,000

Modified Date: Thu, Jul 01, 2010 03:49:13 PM

Output Date: Tue, Jul 06, 2010 11:48:50 AM

Rs. 50,000 20,000 1,40,000 15,000 24,000 15,000 2,64,000

REV-II

Project: Management Accounting_Debarshi Bhattacharyya ACE Pro India Pvt. Ltd. File: X:\Pearson\Management Accounting_Debarshi Bhattacharyya\MAIN\M05\LAYOUT_M05\M05_DEBA_ISBN_EN_SE_C05_Part-1.indd

356

MANAGEMENT ACCOUNTING

 Working Notes 1. Dr.

To Balance c/f

Equity Share Capital Account Rs. By Balance b/f By Bank—New Issue of Shares for Cash (Bal. fig.) 3,00,000 3,00,000

Cr. Rs. 2,00,000 1,00,000

3,00,000

2. Dr.

To Balance c/f

General Reserve Account Rs. By Balance b/f By Profit & Loss A/c—Transfer during the year (Bal. fig.) 60,000 60,000

Cr. Rs. 50,000 10,000

Debenture Account Rs. By Balance b/f By Bank—New Issue (Bal. fig.) 1,00,000 1,00,000

Cr. Rs. 80,000 20,000

60,000

3. Dr.

To Balance c/f

1,00,000

4. Dr.

Proposed Dividend Account Rs. To Bank—Dividend Paid during 2009−10 24,000 By Balance b/f By Profit & Loss Appropriation A/c— Dividend Proposed in 2009−10 (Bal. fig.) To Balance c/f 30,000 54,000

Cr. Rs. 20,000 34,000

54,000

5. Dr. To Bank—Tax Paid during 2009−10

To Balance c/f

Provision for Tax Account Rs. 15,000 By Balance b/f By Profit & Loss A/c—Tax Provision made in 2009−10 25,000 40,000

Cr. Rs. 15,000 25,000 40,000

6. Dr.

Land & Building Account Rs. To Balance b/f 1,50,000 To Bank—Purchase in 2009−10 (Bal. fig.) 50,000 By Balance c/f 3,00,000

Modified Date: Thu, Jul 01, 2010 03:49:13 PM

Output Date: Tue, Jul 06, 2010 11:48:50 AM

Cr. Rs. 2,00,000 3,00,000

REV-II

Project: Management Accounting_Debarshi Bhattacharyya ACE Pro India Pvt. Ltd. File: X:\Pearson\Management Accounting_Debarshi Bhattacharyya\MAIN\M05\LAYOUT_M05\M05_DEBA_ISBN_EN_SE_C05_Part-1.indd

FUND FLOW ANALYSIS

357

7. Dr.

Plant & Machinery Account Rs. 1,60,000 By Bank—Sale of Machinery 5,000 By Profit & Loss A/c—Depreciation for 2009−10

To Balance b/f To Profit & Loss A/c—Profit on Sale [25,000 − (50,000 − 30,000)] To Bank—Purchase (Bal. fig.)

1,40,000 3,05,000

By Balance c/f

Cr. Rs. 25,000 20,000 2,60,000 3,05,000

8. Dr.

Investment Account Rs.

To Balance b/f

30,000

To Bank—Further Investment made in 2009−10 (Bal. fig.)

20,000

Cr. Rs.

By Balance c/f

50,000

50,000

50,000

9. Dr.

Preliminary Expense Account Rs. 10,000 By Profit & Loss A/c—written off (Bal. fig.) By Balance c/f 10,000

To Balance b/f

Cr. Rs. 5,000 5,000 10,000

10. Dr.

Adjusted Profit & Loss A/c Rs. By Balance b/f

To Non-Current & Non-Operating Items Charged: Transfer to General Reserve2 Proposed Dividend4 Provision for Tax5 Preliminary Expenses9 Depreciation on Plant & Machinery7 To Balance c/f

10,000 34,000 25,000 5,000 20.000 90,000 1,84,000

Cr. Rs. 60,000

By Non-Current & Non-Operating Items Credited: Profit on Sale of Machinery7 By Net Fund Flow from Operation (Bal. fig.)

5,000 1,19,000 1,84,000

Problem 9 From the following Balance Sheets of Mother India Ltd and additional information, prepare a statement of changes in the Working Capital and Fund Flow Statement for the year that ended on 31 December 2005: Balance Sheet of Mother India Ltd as on Liabilities Equity Share Capital 8% Redeemable Preference Share Profit & Loss A/c General Reserve Capital Reserve Proposed Dividend

2004 (Rs.) 3,00,000 1,50,000 30,000 40,000 − 42,000

2005 (Rs.) 4,00,000 1,00,000 48,000 50,000 20,000 50,000

Assets Goodwill Land & Building Plant Investment Stock Debtors

2004 (Rs.) 1,00,000 2,00,000 80,000 20,000 77,000 1,40,000

2005(Rs.) 80,000 1,70,000 2,00,000 30,000 1,09,000 1,70,000 (Continued)

Modified Date: Thu, Jul 01, 2010 03:49:13 PM

Output Date: Tue, Jul 06, 2010 11:48:50 AM

REV-II

Project: Management Accounting_Debarshi Bhattacharyya ACE Pro India Pvt. Ltd. File: X:\Pearson\Management Accounting_Debarshi Bhattacharyya\MAIN\M05\LAYOUT_M05\M05_DEBA_ISBN_EN_SE_C05_Part-1.indd

358

MANAGEMENT ACCOUNTING

Liabilities Sundry Creditors Bills Payable Liability for Expenses Taxation Provision

2004 (Rs.) 25,000 20,000 30,000 40,000 6,77,000

2005 (Rs.) 47,000 16,000 36,000 50,000 8,17,000

Assets Bills Receivable Bank Cash Preliminary Expenses

2004 (Rs.) 20,000 15,000 10,000 15,000 6,77,000

2005(Rs.) 30,000 10,000 8,000 10,000 8,17,000

Additional information: i. A piece of land was sold in 2005 and the profit on sale was credited to Capital Reserve A/c. ii. A machine (WDV − 12,000) was sold for Rs. 10,000. In 2005, a depreciation of Rs. 10,000 was charged to Plant A/c. iii. The investments are trade investments. Rs. 3,000 is received by way of dividend, including Rs. 1,000 from pre-acquisition profit, which was credited to Investment A/c. iv. An interim dividend of Rs. 20,000 has been paid in 2005. [B.Com. (Hons), Calcutta University—2006] Solution Books of Mother India Ltd Statement showing changes in the Working Capital for the year that ended on 31 December 2005 2004 Rs.

2005 Rs.

77,000 1,40,000 15,000 10,000 20,000 2,62,000

1,09,000 1,70,000 10,000 8,000 30,000 3,27,000

(25,000) (20,000) (30,000) 1,87,000

(47,000) (16,000) (36,000) 2,28,000 41,000

Elements of Working Capital Current Assets: Stock Debtors Bank Cash Bills Receivable Less:

Current Liabilities: Sundry Creditors Bills Payable Liability for Expenses Working Capital Increase in Working Capital

Fund Flow Statement for the year that ended on 31 December 2005 Sources of Fund New Issue of Equity Shares1 Sale Proceeds of Land8 Total Dividend Received on Investment Payment of Tax7 Sale Proceeds of Machine9 Net Fund Flow from Operation14

Rs. 1,00,000 50,000 3,000 40,000 10,000 1,83,000 3,46,000

Applications of Fund Redemption of Preference Shares2 Further Investment Made10 Purchase of Plant & Machinery9 Payment of Proposed Dividend5 Payment of Interim Dividend6 Increase in Working Capital (Bal. fig.)

Rs. 50,000 11,000 1,42,000 42,000 20,000 41,000 3,46,000

 Working Notes 1. Dr.

To Balance c/f

Modified Date: Thu, Jul 01, 2010 03:49:13 PM

Equity Share Capital Account Rs. By Balance b/f By Bank—New Issue of Shares for Cash 4,00,000 (Bal. fig.) 4,00,000

Output Date: Tue, Jul 06, 2010 11:48:50 AM

Cr. Rs. 3,00,000 1,00,000 4,00,000

REV-II

Project: Management Accounting_Debarshi Bhattacharyya ACE Pro India Pvt. Ltd. File: X:\Pearson\Management Accounting_Debarshi Bhattacharyya\MAIN\M05\LAYOUT_M05\M05_DEBA_ISBN_EN_SE_C05_Part-1.indd

FUND FLOW ANALYSIS

359

2. Dr.

8% Preference Share Capital Account Rs. To Bank—Preference Shares Redeemed 50,000 By Balance b/f (Bal. fig.) To Balance c/f 1,00,000 1,50,000

Cr. Rs. 1,50,000

1,50,000

3. Dr.

To Balance c/f

General Reserve Account Rs. By Balance b/f By Profit & Loss A/c—Transfer during the year (Bal. fig.) 50,000 50,000

Cr. Rs. 40,000 10,000

Capital Reserve Account Rs. By Balance b/f By Land & Building A/c—Profit on Sale 20,000 20,000

Cr. Rs. − 20,000

50,000

4. Dr.

To Balance c/f

20,000

5. Dr. To Bank—Proposed Dividend for 2004 paid in 2005

Proposed Dividend Account Rs. 42,000 By Balance b/f By Profit & Loss Appropriation A/c—Dividend Proposed in 2005

To Balance c/f

Cr. Rs. 42,000 50,000

50,000 92,000

92,000

Interim Dividend Account Rs. 20,000 By Profit & Loss Appropriation A/c—Interim Dividend adjusted 20,000

Cr. Rs. 20,000

Taxation Provision Account Rs. 40,000 By Balance b/f 50,000 By Profit & Loss A/c—Tax Provision made in 2005 90,000

Cr. Rs. 40,000 50,000 90,000

6. Dr. To Bank—Interim Dividend paid

20,000

7. Dr. To Bank—Tax Paid during 2005 To Balance c/f

(Continued)

Modified Date: Thu, Jul 01, 2010 03:49:13 PM

Output Date: Tue, Jul 06, 2010 11:48:50 AM

REV-II

Project: Management Accounting_Debarshi Bhattacharyya ACE Pro India Pvt. Ltd. File: X:\Pearson\Management Accounting_Debarshi Bhattacharyya\MAIN\M05\LAYOUT_M05\M05_DEBA_ISBN_EN_SE_C05_Part-1.indd

360

MANAGEMENT ACCOUNTING

8. Dr. To Balance b/f To Capital Reserve4—Profit on Sale

Land & Building Account Rs. 2,00,000 By Bank—Sale Proceeds (Bal. fig.) 20,000 By Balance c/f 2,20,000

Cr. Rs. 50,000 1,70,000 2,20,000

Plant Account Rs. 80,000 By Bank—Sale of Machine By Profit & Loss A/c—Loss on Sale 1,42,000 By Profit & Loss A/c—Depreciation By Balance c/f 2,22,000

Cr. Rs. 10,000 2,000 10,000 2,00,000 2,22,000

9. Dr. To Balance b/f To Bank—Purchase (Bal. fig.)

10. Dr.

To Balance b/f To Bank—Further Investment made in 2005 (Bal. fig.)

Investment Account Rs. By Bank—Pre-Acquisition Dividend received & 20,000 credited 11,000 31,000

By Balance c/f

Cr. Rs. 1,000 30,000 31,000

11. Dr. To Profit & Loss A/c—Post-Acquisition Dividend received & adjusted

Dividend on Investment Account Rs. By Bank—Post-Acquisition Dividend received 2,000 2,000

Cr. Rs. 2,000 2,000

12. Dr. To Balance b/f

Goodwill Account Rs. 1,00,000 By Profit & Loss A/c—Written off (Bal. fig.) By Balance c/f 1,00,000

Cr. Rs. 20,000 80,000 1,00,000

Preliminary Expense Account Rs. 15,000 By Profit & Loss A/c—Written off (Bal. fig.) By Balance c/f 15,000

Cr. Rs. 5,000 10,000 15,000

13. Dr. To Balance b/f

14. Dr. To Non-Current & Non-Operating Items charged: Transfer to General Reserve3 Proposed Dividend5 Taxation Provision7 Interim Dividend6

Adjusted Profit & Loss A/c Rs. By Balance b/f 10,000 50,000 50,000 20,000

By Non-Current & Non-Operating Items credited: Dividend Received on Investment (Post Acquisition)11

Cr. Rs. 30,000

2,000

(Continued)

Modified Date: Thu, Jul 01, 2010 03:49:13 PM

Output Date: Tue, Jul 06, 2010 11:48:50 AM

REV-II

Project: Management Accounting_Debarshi Bhattacharyya ACE Pro India Pvt. Ltd. File: X:\Pearson\Management Accounting_Debarshi Bhattacharyya\MAIN\M05\LAYOUT_M05\M05_DEBA_ISBN_EN_SE_C05_Part-1.indd

FUND FLOW ANALYSIS

Dr.

Adjusted Profit & Loss A/c Rs. 2,000 10,000 By Net Fund Flow from Operation (Bal. fig.) 5,000 20,000 48,000 2,15,000

Loss on Sale of Machine9 Depreciation on Plant9 Preliminary Expenses 13 Goodwill12 To Balance c/f

361

Cr. Rs. 1,83,000

2,15,000

Problem 10 Following are the summarized Balance Sheets of Sonar Bangla Ltd as on 31 March 2002 and 31 March 2003: 31 March 2002 (Rs.) 4,00,000 50,000 1,80,000 1,10,000 1,00,000 1,10,000 70,000 13,000 60,000 40,000 11,33,000

Liabilities Equity Shares of Rs. 10 each Securities Premium General Reserve Profit & Loss A/c 6% Debentures Bank Loan Sundry Creditors Bills Payable Provision for Taxation Proposed Dividend

31 March 2003 (Rs.) 6,00,000 60,000 1,20,000 1,08,000 1,50,000 1,30,000 83,000 15,000 55,000 50,000 13,71,000

Assets Land & Building Plant & Machinery Investments Stock Sundry Debtors Cash at Bank Discount on Debentures

31 March 2002 (Rs.) 4,00,000 4,50,000 40,000 1,20,000 80,000 35,000 8,000

31 March 2003 (Rs.) 5,00,000 5,30,000 50,000 1,40,000 95,000 44,000 12,000

11,33,000

13,71,000

Additional information: i. Depreciation on land and building for the year 2002−03 was Rs. 20,000. ii. Accumulated depreciation on plant and machinery on 31 March 2002 was Rs. 1,50,000 and on 31 March 2003 was Rs. 1,70,000. iii. Machinery costing Rs. 50,000 (written-down value—Rs. 10,000) was sold for Rs. 12,000. iv. 6% Debentures were issued at 10% discount. v. Bonus shares were issued @ one share for every four shares held on 31 March 2002 out of general reserve. vi. Interim dividend paid during the year—Rs. 20,000. You are required to prepare: i. A statement showing the changes in the Working Capital. ii. A Fund Flow Statement for the year that ended on 31 March 2003. [B.Com. (Hons), Calcutta University—2004] Solution Books of Sonar Bangla Ltd. i. Statement showing changes in Working Capital for the year that ended on 31 March 2003 Elements of Working Capital Current Assets: Stock Sundry Debtors Cash at Bank

As on 31 March 2002 (Rs.)

As on 31 March 2003 (Rs.)

1,20,000 80,000 35,000 2,35,000

1,40,000 95,000 44,000 2,79,000 (Continued)

Modified Date: Thu, Jul 01, 2010 03:49:13 PM

Output Date: Tue, Jul 06, 2010 11:48:50 AM

REV-II

Project: Management Accounting_Debarshi Bhattacharyya ACE Pro India Pvt. Ltd. File: X:\Pearson\Management Accounting_Debarshi Bhattacharyya\MAIN\M05\LAYOUT_M05\M05_DEBA_ISBN_EN_SE_C05_Part-1.indd

362

MANAGEMENT ACCOUNTING

Elements of Working Capital Less:

Current Liabilities: Sundry Creditors Bills Payable Working Capital Increase in Working Capital

As on 31 March 2002 (Rs.)

As on 31 March 2003 (Rs.)

(70,000) (13,000) 1,52,000

(83,000) (15,000) 1,81,000 29,000

ii. Fund Flow Statement for the year that ended on 31 March 2003 Sources of Fund New Issue of Equity Shares4 Securities Premium Received5 New Issue of Debentures8 Bank loan Raised (Rs. 1,30,000 − Rs. 1,10,000) Sale Proceeds of Machinery3 Net Fund Flow from Operation12

Rs. 1,00,000 10,000 45,000 20,000 12,000 2,42,000

Applications of Fund Purchase of Land & Building1 Purchase of Plant & Machinery3 Payment of Interim Dividend7 Payment of Tax10 Payment of Proposed Dividend11 Further Investment made (Rs. 50,000 − Rs. 40,000) Increase in Working Capital (Bal. fig.)

4,29,000

Rs. 1,20,000 1,50,000 20,000 60,000 40,000 10,000 29,000 4,29,000

Working Notes 1. Dr. To Balance b/f To Bank—Purchase (Bal. fig.)

Land & Build Account Rs. 4,00,000 By Profit & Loss A/c—Depreciation 1,20,000 By Balance c/f 5,20,000

Cr. Rs. 20,000 5,00,000 5,20,000

2. Dr.

Plant & Machinery (at cost) Account Rs. 6,00,000 By Provision for Depreciation A/c To Balance b/f (4,50,000 + 1,50,000) —Accumulated Depreciation on Machinery Sold To Profit & Loss A/c—Profit on sale of 2,000 By Bank—Sale Proceeds of Machinery Machinery To Bank—New purchase (Bal. fig.) 1,50,000 By Balance c/f (5,30,000 + 1,70,000) 7,52,000

Cr. Rs. 40,000

12,000 7,00,000 7,52,000

3. Dr.

Provision for Depreciation on Plant & Machinery Account Rs. To Plant & Machinery A/c—Accumulated By Balance b/f Depreciation on Machinery Sold 40,000 By Profit & Loss A/c—Depreciation for the year (Bal. fig.) To Balance c/f 1,70,000 2,10,000

Cr. Rs. 1,50,000 60,000

2,10,000 (Continued)

Modified Date: Thu, Jul 01, 2010 03:49:13 PM

Output Date: Tue, Jul 06, 2010 11:48:50 AM

REV-II

Project: Management Accounting_Debarshi Bhattacharyya ACE Pro India Pvt. Ltd. File: X:\Pearson\Management Accounting_Debarshi Bhattacharyya\MAIN\M05\LAYOUT_M05\M05_DEBA_ISBN_EN_SE_C05_Part-1.indd

363

FUND FLOW ANALYSIS

4. Dr.

To Balance c/f

Equity Share Capital Account Rs. By Balance b/f By General Reserve—Bonus Shares issued (1/4 × Rs. 4,00,000) By Bank—New Issue of Shares for Cash (Bal. fig.) 6,00,000 6,00,000

Cr. Rs. 4,00,000 1,00,000

Securities Premium Account Rs. By Balance b/f By Bank—Premium Received on New Issue of Shares (Bal. fig.) 60,000 60,000

Cr. Rs. 50,000

1,00,000 6,00,000

5. Dr.

To Balance c/f

10,000 1,50,000

6. Dr. To Equity Share Capital A/c Bonus Shares issued To Balance c/f

General Reserve Account Rs. 1,00,000 By Balance b/f By Profit & Loss A/c—Transfer made during the year (Bal. fig.) 1,20,000 2,20,000

Cr. Rs. 1,80,000 40,000

2,20,000

7. Dr. To Bank—Interim Dividend paid

Interim Dividend Account Rs. 20,000 By Profit & Loss Appropriation A/c—Interim Dividend adjusted 20,000

Cr. Rs. 20,000 20,000

8. Dr.

6% Debentures Account Rs. By Balance b/f By Bank—New Issue (Bal. fig.) (Rs. 50,000 − Rs. 5,000) By Discount on Debentures (10% on Rs. 50,000)

To Balance c/f

1,50,000 1,50,000

Cr. Rs. 1,00,000 45,000 5,000 1,50,000

9. Dr. To Balance b/f To 6% Debentures A/c—Discount on New Issue

Discount on Debentures Account Rs. 8,000 By Profit & Loss A/c—Written off (Bal. fig.) By Balance c/f 5,000 13,000

Cr. Rs. 1,000 12,000 13,000 (Continued)

Modified Date: Thu, Jul 01, 2010 03:49:13 PM

Output Date: Tue, Jul 06, 2010 11:48:50 AM

REV-II

Project: Management Accounting_Debarshi Bhattacharyya ACE Pro India Pvt. Ltd. File: X:\Pearson\Management Accounting_Debarshi Bhattacharyya\MAIN\M05\LAYOUT_M05\M05_DEBA_ISBN_EN_SE_C05_Part-1.indd

364

MANAGEMENT ACCOUNTING

10. Dr.

Provision for Taxation Account Rs. To Bank—Tax Paid during 2002−03 60,000 By Balance b/f To Balance c/f 55,000 By Profit & Loss A/c—Provision made in 2002−03 1,15,000

Cr. Rs. 60,000 55,000 1,15,000

11. Dr.

Proposed Dividend Account Rs. To Bank—Proposed Dividend for 2001−02 40,000 Paid in 2002−03 By Balance b/f By Profit & Loss Appropriation A/c— Dividend proposed in 2002−03 To Balance c/f 50,000 90,000

Cr. Rs. 40,000 50,000

90,000

12. Dr.

Adjusted Profit & Loss A/c Rs. To Non-Current & Non-Operating Items By Balance b/f charged: 40,000 By Non-Current & Non-Operating Items Transfer to General Reserve6 50,000 Credited: Proposed Dividend11 55,000 Profit on Sale of Machinery2 Provision for Taxation10 20,000 Interim Dividend7 1,000 Discount on Debentures9 60,000 Depreciation on Plant & Machinery3 20,000 Depreciation on Land & Building1 To Balance c/f 1,08,000 By Net Fund Flow from Operation (Bal. fig.) 3,54,000

Cr. Rs. 1,10,000

2,000

2,42,000 3,54,000

Problem 11 The following are the condensed Balance Sheet of B Ltd at the end of 2007 and 2008: Balance Sheets Capital & Liabilities Equity Share Capital Reserve & Surplus 6% Debenture Sundry Creditors Outstanding Expenses Provision for Depreciation Provision for Income Tax Proposed Dividend Provision for Bad Debt

2007 (Rs.) 2,50,000 1,50,000 50,000 79,000 7,000 80,000 30,000 37,500 13,000 6,96,500

2008 (Rs.) 3,50,000 1,40,000 20,000 83,000 15,000 1,00,000 25,000 52,500 18,000 8,03,500

Assets Land & Building Plant & Machinery Debenture Issue Expenses Preliminary Expenses Stock Debtors Bills Receivable Cash in Hand and at Bank

2007 (Rs.) 1,25,000 2,40,000 10,000 15,000 1,90,000 60,000 26,000 30,500

2008 (Rs.) 1,25,000 3,60,000 3,000 12,000 1,93,000 90,000 15,000 5,500

6,96,500

8,03,500

Additional information: i. Income tax paid in 2008 was Rs. 35,000. ii. An old machine was sold for Rs. 44,000, the cost and written-down value of which were Rs. 60,000 and Rs. 40,000, respectively.

Modified Date: Thu, Jul 01, 2010 03:49:13 PM

Output Date: Tue, Jul 06, 2010 11:48:50 AM

REV-II

Project: Management Accounting_Debarshi Bhattacharyya ACE Pro India Pvt. Ltd. File: X:\Pearson\Management Accounting_Debarshi Bhattacharyya\MAIN\M05\LAYOUT_M05\M05_DEBA_ISBN_EN_SE_C05_Part-1.indd

FUND FLOW ANALYSIS

365

iii. Bonus shares at 2 for every 5 equity shares were issued out of accumulated reserves and surplus. iv. Out of the proposed dividend for 2007, only Rs. 30,000 were paid in 2008 and in addition to that, an interim dividend for Rs. 25,000 was paid in the same year. Prepare a Fund Flow Statement and statement of changes in the Working Capital of the company for the year that ended on 31 December 2008. [B.Com. (Hons), Calcutta University—2009] Solution Books of B Ltd Statement showing changes in Working Capital for the year that ended on 31 December 2008 Elements of Working Capital Current Asset: Stock Sundry Debtors Bills Receivable Cash & Bank Less:

Current Liabilities: Sundry Creditors Outstanding Expenses Provision for Bad Debt Working Capital Decrease in Working Capital

As on 31 December 2007 (Rs.)

As on 31 December 2008 (Rs.)

1,90,000 60,000 26,000 30,500 3,06,500

1,93,000 90,000 15,000 5,500 3,03,500

(79,000) (7,000) (13,000) 2,07,500

(83,000) (15,000) (18,000) 1,87,500 20,000

Fund Flow Statement for the year that ended on 31 December 2008 Sources of Fund Sale Proceeds of Machinery6 Fund from Operation9 Decrease in Working Capital (Bal. fig.)

Rs. 44,000 2,36,000 20,000

Applications of Fund Purchase of Plant & Machinery6 Payment of Interim Dividend5 Payment of Proposed Dividend4 Payment of Income Tax3 Redemption of Debenture8

3,00,000

Rs. 1,80,000 25,000 30,000 35,000 30,000 3,00,000

Working Notes 1. Dr.

To Balance c/f

Equity Share Capital Account Rs. By Balance b/f By Reserve & Surplus—Bonus Shares issued (2 ÷ 5 × Rs. 2,50,000) 3,50,000 6,00,000

Cr. Rs. 2,50,000 1,00,000

Reserve & Surplus Account Rs. 1,00,000 By Balance b/f By Profit & Loss A/c—Net Profit for the year (Bal. fig.) 1,40,000 2,40,000

Cr. Rs. 1,50,000 90,000

6,00,000

2. Dr. To Equity Share Capital A/c —Bonus Shares issued To Balance c/f

2,40,000 (Continued)

Modified Date: Thu, Jul 01, 2010 03:49:13 PM

Output Date: Tue, Jul 06, 2010 11:48:50 AM

REV-II

Project: Management Accounting_Debarshi Bhattacharyya ACE Pro India Pvt. Ltd. File: X:\Pearson\Management Accounting_Debarshi Bhattacharyya\MAIN\M05\LAYOUT_M05\M05_DEBA_ISBN_EN_SE_C05_Part-1.indd

366

MANAGEMENT ACCOUNTING

3. Dr.

Provision for Income Tax Account Rs. 35,000 By Balance b/f By Profit & Loss A/c—Provision made in 2008 (Bal. fig.) 25,000 60,000

To Bank—Tax Paid in 2008

To Balance c/f

Cr. Rs. 30,000

30,000 60,000

4. Dr.

Proposed Dividend Account Rs. To Bank—Proposed Dividend for 2007 paid By Balance b/f in 2008 30,000 By Profit & Loss Appropriation A/c —Dividend Proposed in 2008 (Bal. fig.) To Balance c/f 52,500 82,500

Cr. Rs. 37,500

45,000 82,500

5. Dr.

Interim Dividend Account Rs. 25,000 By Profit & Loss Appropriation A/c—Interim Dividend adjusted 25,000

To Bank—Interim Dividend paid

Cr. Rs. 25,000 25,000

6. Dr.

Plant & Machinery (at cost) Account Rs. To Balance b/f 2,40,000 By Provision for Depreciation A/c —Accumulated Depreciation on Machinery Sold To Profit & Loss A/c—Profit on Sale of 4,000 By Bank—Sale Proceeds of Machinery Machinery [44,000 − (60,000 − 20,000)] To Bank—New Purchase (Bal. fig.) 1,80,000 By Balance c/f 4,24,000

Cr. Rs.

20,000 44,000 3,60,000 4,24,000

7. Dr.

Provision for Depreciation on Plant & Machinery Account Rs. To Plant & Machinery A/c—Accumulated By Balance b/f Depreciation on Machinery Sold (60,000 − 40,000) 20,000 By Profit & Loss A/c—Depreciation for the year (Bal. fig.) To Balance c/f 1,00,000 1,20,000

Cr. Rs. 80,000

40,000 1,20,000

8. Analysis of other Non-Current Assets and Liabilities Assets/Liabilities Land & Building Preliminary Expenses Debenture Issue Expense 6% Debenture

Opening Balance Rs. 1,25,000 15,000 10,000 50,000

Closing Balance Rs. 1,25,000 12,000 3,000 20,000

Increase/ Decrease Rs. Nil (−)3,000 (−)7,000 (−)30,000

Analysis No Purchase or Sale Depreciation Written off against Profit & Loss A/c Written off against Profit & Loss A/c Redemption of Debentures (Continued)

Modified Date: Thu, Jul 01, 2010 03:49:13 PM

Output Date: Tue, Jul 06, 2010 11:48:50 AM

REV-II

Project: Management Accounting_Debarshi Bhattacharyya ACE Pro India Pvt. Ltd. File: X:\Pearson\Management Accounting_Debarshi Bhattacharyya\MAIN\M05\LAYOUT_M05\M05_DEBA_ISBN_EN_SE_C05_Part-1.indd

FUND FLOW ANALYSIS

367

9. Calculation of Fund from Operation for the year that ended on 31 December 2008 Rs. Add:

Less:

Net Profit after Appropriation for the year2 Adjustment for Non-Current and Non-Operating items Debited to Profit & Loss A/c: Interim Dividend Paid Proposed Dividend Depreciation on Plant & Machinery Preliminary Expenses written off Debenture-Issue Expense written off Provision for Income Tax

25,000 45,000 40,000 3,000 7,000 30,000

Adjustment for Non-Current and Non-Operating Items credited to Profit & Loss A/c: Profit on Sale of old Machinery Fund from Operation

Rs. 90,000

1,50,000 2,40,000

4,000 2,36,000

Problem 12 Prepare a Fund Flow Statement of Y Ltd from the following: Balance Sheets of Y Ltd as on 31 March 2006 and 31 March 2007 Liabilities Share Capital Capital Reserve General Reserve Profit & Loss A/c Debentures Liabilities for Goods & Services Provision for Income Tax Proposed Dividend Unpaid Dividend

31 March 2006 (Rs.) 6,00,000 − 3,40,000 1,20,000 4,00,000 2,40,000 1,80,000 60,000 − 19,40,000

31 March 2007 (Rs.) 8,00,000 20,000 4,00,000 1,50,000 2,80,000 2,60,000 1,70,000 72,000 8,000 21,60,000

Assets Fixed Assets: At Cost Less: Depreciation Trade Investments Current Assets Preliminary Expenses

31 March 2006 (Rs.)

31 March 2007 (Rs.)

16,00,000 4,60,000 11,40,000 2,00,000 5,60,000 40,000

19,00,000 5,80,000 13,20,000 1,60,000 6,60,000 20,000

19,40,000

21,60,000

Additional information available during the year: i. Sold one machine for Rs. 50,000; the cost of the machine was Rs. 1,28,000 and the depreciation provided for it amounted to Rs. 70,000. ii. Provided Rs. 1,90,000 as depreciation. iii. Redeemed 30% of debentures @ Rs. 103. iv. Some trade investments sold at profit and the profit was credited to the capital reserve. v. Decided to value the stock at cost, whereas previously the practice was to value stock at cost less 10%. The stock according to books on 31 March 2006 was Rs. 1,08,000; the stock on 31 March 2007 was Rs. 1,50,000, which was correctly valued at cost. [B.Com. (Hons), Calcutta University—2008]

Modified Date: Thu, Jul 01, 2010 03:49:13 PM

Output Date: Tue, Jul 06, 2010 11:48:50 AM

REV-II

Project: Management Accounting_Debarshi Bhattacharyya ACE Pro India Pvt. Ltd. File: X:\Pearson\Management Accounting_Debarshi Bhattacharyya\MAIN\M05\LAYOUT_M05\M05_DEBA_ISBN_EN_SE_C05_Part-1.indd

368

MANAGEMENT ACCOUNTING

Solution Books of Y Ltd Fund Flow Statement for the year that ended on 31 March 2007 Sources of Fund New Issue of Equity Shares10 Sale Proceeds of Investment4 Sale Proceeds of Machinery2 Net Fund Flow from Operation12

Rs. 2,00,000 60,000 50,000 5,53,600

Applications of Fund Purchase of Fixed Assets2 Redemption of Debenture9 Payment of Proposed Dividend6 Payment of Tax8 Increase in Working Capital1 (Bal. fig.)

8,63,600

Rs. 4,40,000 1,23,600 52,000 1,80,000 68,000 8,63,600

Working Notes 1. Statement showing changes in Working Capital for the year that ended on 31 March 2007 Elements of Working Capital

Less:

Current Assets [on 31 March 2006: 5,60,000 + Undervaluation of Opening Stock (10 ÷ 90 × 1,08,000)] Current Liabilities: Liabilities for Goods and Services Working Capital Increase in Working Capital

As on 31 March 2006 (Rs.)

As on 31 March 2007 (Rs.)

5,72,000

6,60,000

2,40,000 3,32,000

2,60,000 4,00,000 68,000

2. Dr.

Fixed Assets (at cost) Account Cr. Rs. Rs. 16,00,000 By Provision for Depreciation A/c 70,000 —Accumulated Depreciation on Machine Sold 4,40,000 By Bank—Sale Proceeds of the Machine 50,000 By Profit & Loss A/c—Loss on Sale of the 20,000 Machine [(1,28,000 − 70,000) − 50,000] By Balance c/f 19,00,000 20,40,000 20,40,000

To Balance b/f To Bank—Purchase of Fixed Assets (Bal. fig.)

3. Dr.

Provision for Depreciation Account Rs. 70,000 By Balance b/f

To Fixed Assets A/c—Accumulated Depreciation on Machine Sold

By Profit & Loss A/c—Depreciation for the year To Balance c/f

5,80,000 6,50,000

Cr. Rs. 4,60,000 1,90,000 6,50,000

4. Dr. To Balance b/f To Capital Reserve—Profit on Sale of Investment

Trade Investment Account Rs. 2,00,000 By Bank—Sale proceeds of Investment (Bal. fig.) 20,000 By Balance c/f

Cr. Rs. 60,000 1,60,000

2,20,000

2,20,000 (Continued)

Modified Date: Thu, Jul 01, 2010 03:49:13 PM

Output Date: Tue, Jul 06, 2010 11:48:50 AM

REV-II

Project: Management Accounting_Debarshi Bhattacharyya ACE Pro India Pvt. Ltd. File: X:\Pearson\Management Accounting_Debarshi Bhattacharyya\MAIN\M05\LAYOUT_M05\M05_DEBA_ISBN_EN_SE_C05_Part-1.indd

369

FUND FLOW ANALYSIS

5. Dr.

To Balance c/f

Capital Reserve Account Rs. By Balance b/f 20,000 By Trade Investment A/c—Profit on Sale 20,000

Cr. Rs. − 20,000 20,000

6. Dr.

Proposed Dividend Account Rs. To Bank—Proposed Dividend for 2005−06 By Balance b/f Paid in 2006−07 52,000 By Profit & Loss Appropriation A/c— Dividend Proposed in 2006−07 To Unpaid Dividend A/c—proposed Dividend for 2005−06 remained unpaid 8,000 To Balance c/f 72,000 1,32,000

Cr. Rs. 60,000

72,000

1,32,000

7. Dr.

To Balance c/f

Unpaid Dividend Account Rs. By Balance b/f By Proposed Dividend A/c—Unpaid 8,000 Dividend for 2005−06 8,000

Cr. Rs. − 8,000 8,000

8. Dr. To Bank—Tax Paid in 2005−06 To Balance c/f

Provision for Income Tax Account Rs. 1,80,000 By Balance b/f 1,70,000 By Profit & Loss A/c—Provision made in 2006−07 3,50,000

Cr. Rs. 1,80,000 1,70,000 3,50,000

9. Dr. To Bank—Redeemed [103% of (30% of 4,00,000)] To Balance c/f

Debentures Account Rs. 1,23,600 By Balance b/f 2,80,000

Cr. Rs. 4,00,000

By Profit & Loss A/c—Premium Paid on Redemption adjusted [3% of (30% of 4,00,000)]

4,03,600

3,600 4,03,600

10. Analysis of other Non-Current Assets and Liabilities Assets/Liabilities Share Capital General Reserve Preliminary Expenses

Opening Balance (Rs.) 6,00,000 3,40,000 40,000

Closing Balance (Rs.) 8,00,000 4,00,000 20,000

Increase/ Decrease (Rs.) (+)2,00,000 (+)60,000 (−)20,000

Analysis New Shares Issued for Cash Transfer from Profit & Loss A/c Written off against Profit & Loss A/c (Continued)

Modified Date: Thu, Jul 01, 2010 03:49:13 PM

Output Date: Tue, Jul 06, 2010 11:48:50 AM

REV-II

Project: Management Accounting_Debarshi Bhattacharyya ACE Pro India Pvt. Ltd. File: X:\Pearson\Management Accounting_Debarshi Bhattacharyya\MAIN\M05\LAYOUT_M05\M05_DEBA_ISBN_EN_SE_C05_Part-1.indd

370

MANAGEMENT ACCOUNTING

11. Dr.

Adjusted Profit & Loss A/c Rs. To Non-Current & Non-Operating Items By Balance b/f Charged: 60,000 By Undervaluation of Opening Stock Transfer to General Reserve10 72,000 Proposed Dividend6 (10 ÷ 90 × 1,08,000) 1,70,000 Provision for Income tax8 20,000 Loss on Sale of Machine2 20,000 Preliminary Expenses10 1,90,000 By Net Fund Flow from Operation Depreciation on Plant & Machinery3 3,600 (Bal. fig.) Premium Paid on Redemption Debenture9 To Balance c/f 1,50,000 6,85,600

Cr. Rs. 1,20,000 12,000

5,53,600 6,85,600

Problem 13 Prepare a Fund Flow Statement of DG & Sons Ltd from the following: Balance Sheets of DG & sons Ltd., as on 31 December 2005 and 31 December 2006 Liabilities Equity Share Capital @ Rs. 10 7% Preference Share Capital @ Rs. 100 Each Securities Premium Capital Redemption Reserve Profit & Loss A/c General Reserve Current Liabilities Provision for Taxation Proposed Dividend

31 December 31 December 2005 2006 Rs. in ’000 Rs. in ’000 300 320

100 20

50 17

– 60 80 160

30 75 60 250

30 50 800

40 58 900

Assets Fixed Assets at Cost Less: Accumulated Depreciation Trade Investments Inventories Loans & Advances Receivables Cash at Bank Preliminary Expenses

31 December 31 December 2005 2006 Rs. in ’000 Rs. in ’000 600 680

(140) 460 140 110 20 40 10 20

(180) 500 200 100 30 35 20 15

800

900

Additional information available during the year 2006: i. ii. iii. iv. v.

Furniture costing Rs. 16,000 (fully depreciated) was written off. Tax due on profit estimated to be Rs. 34 000. Old machine costing Rs. 25,000 (30% depreciated) sold for Rs. 20,000. Preference dividend along with 12% dividend on equity was paid for 2005. 500 preference shares were redeemed at 10% premium, 200 equity shares were issued at 10% premium and general reserve was utilized for a redemption purpose. [B.Com. (Hons), Calcutta University—2007]

Modified Date: Thu, Jul 01, 2010 03:49:13 PM

Output Date: Tue, Jul 06, 2010 11:48:50 AM

REV-II

Project: Management Accounting_Debarshi Bhattacharyya ACE Pro India Pvt. Ltd. File: X:\Pearson\Management Accounting_Debarshi Bhattacharyya\MAIN\M05\LAYOUT_M05\M05_DEBA_ISBN_EN_SE_C05_Part-1.indd

371

FUND FLOW ANALYSIS

Solution Books of DG & Sons Ltd Fund Flow Statement for the year that ended on 31 December 2006 Sources of Fund New issue of Equity Shares4 Premium Received on New Issue of Equity Shares7 Sale Proceeds of Machinery2 Net Fund Flow from Operation13 Decrease in Working Capital1 (Bal. fig.)

Rs. in ’000 20 2 20 176 85

Applications of Fund Purchase of Fixed Assets2 Redemption of Preference Shares9

Rs. in ’000 121 50

Premium Paid on Redemption of Preference Shares6 Payment of Tax11 Payment of Proposed Dividend10 Purchase of New Investment12

5 24 43 60 303

303

Working Notes 1. Statement showing changes in Working Capital for the year that ended on 31 December 2006 Elements of Working Capital

As on 31 December 2005 (Rs. ‘000s)

As on 31 December 2006 (Rs. ‘000s)

110 20 40 10 180 160 20

100 30 35 20 185 250 (65) 85

Current Assets: Inventories Loans & Advances Receivables Cash at Bank Less:

Current Liabilities Working Capital Decrease in Working Capital

2. Dr.

Fixed Assets (at cost) Account Rs. in ’000 To Balance b/f 600.00 By Provision for Depreciation A/c —Accumulated Depreciation on Fully Depreciated Furniture To Profit & Loss A/c—Profit on Sale of 2.50 By Provision for Depreciation A/c Machinery [20,000 − (25,000 − 7,500)] —Accumulated Depreciation on Machinery sold To Bank—Purchase of Fixed Assets (Bal. fig.) 121.00 By Bank—Sale Proceeds of Machinery To Balance c/f 723.50

Cr. Rs. in ’000 16.00

7.50

20.00 680.00 723.50

3. Dr.

Provision for Depreciation Account Rs. in ’000 To Fixed Assets A/c—Accumulated By Balance b/f Depreciation on fully depreciated furniture 16.00 To Profit & Loss A/c—Depreciation for the year (Bal. fig.)

Cr. Rs. in ’000 140.50 63.50

(Continued)

Modified Date: Thu, Jul 01, 2010 03:49:13 PM

Output Date: Tue, Jul 06, 2010 11:48:50 AM

REV-II

Project: Management Accounting_Debarshi Bhattacharyya ACE Pro India Pvt. Ltd. File: X:\Pearson\Management Accounting_Debarshi Bhattacharyya\MAIN\M05\LAYOUT_M05\M05_DEBA_ISBN_EN_SE_C05_Part-1.indd

372

MANAGEMENT ACCOUNTING

Dr. To Fixed Assets A/c—Accumulated Depreciation on Machinery Sold (30% of Rs. 25,000) To Balance c/f

Provision for Depreciation Account Rs. in ’000 7.50

180.00 203.50

Cr. Rs. in ’000

203.50

4. Dr.

To Balance c/f

Equity Share Capital Account Rs. in ’000 By Balance b/f By Bank—New Issue of Shares for Cash (Bal. fig.) 320 320

Cr. Rs. in ’000 300 20

320

5. Dr.

7% Preference Share Capital Account Rs. in ’000 To Bank—Preference Shares Redeemed 50 By Balance b/f (Bal. fig.) To Balance c/f 50 100

Cr. Rs. in ’000 100

100

6. Dr.

Premium on Redemption of Pref. Shares Account Rs. in ’000 To Bank—Premium Paid on Redemption 5.00 By Securities Premium A/c—Premium (10% on 50,000) Paid on Redemption Adjusted 5.00

Cr. Rs. in ’000 5.00 5.00

7. Dr.

Securities Premium Account Rs. in ’000 To Premium on Redemption of Preference By Balance b/f Shares A/c—Premium Paid Adjusted 5 To Balance c/f 17 By Bank—Premium Received on New Issue of Equity Shares 22

Cr. Rs. in ’000 20 2 22

8. Rs. in ’000 To Capital Redemption Reserve—Preference Shares Redeemed less New Equity Shares Issued (50,000 − 20,000) To Balance c/f

By Balance b/f 30 60

By Profit & Loss A/c—Transfer during the year (Bal. fig.)

90

Rs. in ’000 80

10 90

9. Dr.

To Balance c/f

Capital Redemption Reserve Account Rs. in ’000 By Balance b/f 30 By General Reserve—Transferred 30

Cr. Rs. in ’000 − 30 30 (Continued)

Modified Date: Thu, Jul 01, 2010 03:49:13 PM

Output Date: Tue, Jul 06, 2010 11:48:50 AM

REV-II

Project: Management Accounting_Debarshi Bhattacharyya ACE Pro India Pvt. Ltd. File: X:\Pearson\Management Accounting_Debarshi Bhattacharyya\MAIN\M05\LAYOUT_M05\M05_DEBA_ISBN_EN_SE_C05_Part-1.indd

373

FUND FLOW ANALYSIS

10. Dr.

Cr.

Proposed Dividend Account Rs. in ’000 43 By Balance b/f

To Bank—Dividend Paid [(Pref. Div.: 7% on 100) + (Eq. Div.: 12% on 300)]

Rs. in ’000 50

By Profit & Loss Appropriation A/c— Dividend Proposed in 2006 (Bal. fig.) To Balance c/f

51

58 101

101

11. Dr.

Provision for Taxation Account Cr. Rs. in ’000 Rs. in ’000 24 By Balance b/f 30 40 By Profit & Loss A/c—Provision Made in 2006 34 64 64

To Bank—Tax Paid in 2006 (Bal. fig.) To Balance c/f

12. Analysis of other non-Current Assets and liabilities Assets/Liabilities Trade Investment Preliminary Expenses

Opening Balance (Rs. in ’000) 140 20

Closing Balance (Rs. in ’000) 200 15

Increase/Decrease (Rs. in ’000) (+) 60 (−) 5

Analysis Purchase for the year written off against Profit & Loss A/c

13. Dr. To Non-Current & Non-Operating Items Charged: Transfer to General Reserve8 Proposed Dividend10 Provision for Taxation11 Preliminary Expenses 12 Depreciation on Plant & Machinery3 To Balance c/f

Adjusted Profit & Loss A/c Rs. in ’000 By Balance b/f 10.00 51.00 34.00 5.00 63.50 75.00 238.50

Cr. Rs. in ’000 60.00

By Profit on Sale of Machinery

2.50

By Net Fund Flow from Operation (Bal. fig.)

176.00

238.50

Problem 14 Balance Sheets of Arzoo Ltd as on 31 March 2009 and 31 March 2010 were as follows: Liabilities Equity Share Capital (Rs.10) 8% Preference Shares of Rs. 100 each, Rs. 75 per Share Called up & Paid up Securities Premium Capital Redemption Reserve General Reserve Profit & Loss A/c Sundry Creditors

As on 31 March 2009 (Rs.) 3,00,000 1,12,500

As on 31 March 2010 (Rs.) 4,00,000 −

20,000 – 30,000 60,000 12,500

7,000 1,10,000 50,000 40,000 50,000

Assets Land & Building Plant & Machinery Furniture Investment Inventory Sundry Debtors Cash & Bank

As on 31 March 2009 (Rs.) 1,30,000 2,00,000

As on 31 March 2010 (Rs.) 1,40,000 2,80,000

80,000 30,000 60,000 40,000 20,000

1,20,000 40,000 50,000 50,000 30,000 (Continued)

Modified Date: Thu, Jul 01, 2010 03:49:13 PM

Output Date: Tue, Jul 06, 2010 11:48:50 AM

REV-II

Project: Management Accounting_Debarshi Bhattacharyya ACE Pro India Pvt. Ltd. File: X:\Pearson\Management Accounting_Debarshi Bhattacharyya\MAIN\M05\LAYOUT_M05\M05_DEBA_ISBN_EN_SE_C05_Part-1.indd

374

MANAGEMENT ACCOUNTING

As on 31 March 2009 (Rs.) 20,000 20,000 5,000 − 5,80,000

Liabilities Proposed Dividend Provision for Taxation Bills Payable Unclaimed Dividend

As on 31 March 2010 (Rs.) 30,000 30,000 − 3,000 7,20,000

Assets Preliminary Expenses

As on 31 March 2009 (Rs.) 20,000

As on 31 March 2010 (Rs.) 10,000

5,80,000

7,20,000

Additional information: i. During the year 2009−10, an old machine, whose book value was Rs. 60,000, was sold at a loss of Rs. 8,000 and was replaced by a new machine costing Rs. 1,60,000. ii. Depreciation on furniture provided for the year 2009−10 amounted to Rs. 15,000. A part of the furniture was sold at a profit of Rs. 3,000 and the cost of new furniture acquired during the year was Rs. 78,000. iii. A preference-share final call @ Rs. 25 per share was made before redeeming the preference shares at a premium of 10%. The Securities Premium Account was utilized to provide the premium paid, and the redemption was made partly out of profit and partly out of new issue of equity shares at 5% premium. iv. A bonus dividend was declared during 2009−10 to the old-equity shareholders, @ one equity share at par for every five equity shares held, out of the General Reserve. v. Before redemption, the preference dividend for the year 2009−10 was paid by the company. Prepare a Statement showing the changes in the Working Capital and a Fund Flow Statement for the year that ended on 31 March 2010.

Solution Books of Arzoo Ltd Fund Flow Statement for the year that ended on 31 March 2010 Sources of Fund New Issue of Equity Shares1 Premium Received on New Issue of Equity Shares4 Proceeds from Sale of Furniture12 Proceeds from Sale of Machinery11 Receipt of final Preferencce Share Call2 Net Fund Flow from Operation14 Decrease in Working Capital1 (Bal. fig.)

Rs. 40,000 2,000 26,000 52,000 37,500 2,89,000 22,500

Applications of Fund Purchase of New Machinery11 Purchase of New Furniture12

Rs. 1,60,000 78,000

Addition to Building13 New Investment Made13 Payment of Tax10 Payment of Preference Dividend7 Payment of Proposed Dividend8 Redemption of Preference Shares2 Premium Paid on redemption of Preference Shares3

10,000 10,000 20,000 9,000 17,000 1,50,000 15,000 4,69,000

4,69,000

Statement showing changes in the Working Capital for the year that ended on 31 March 2010 Elements of Working Capital

As on 31 March 2009 Rs.

Current Assets: Inventory Sundry Debtors Cash & Bank

60,000 40,000 20,000 1,20,000

As on 31 March 2010 Rs. 50,000 50,000 30,000 1,30,000 (Continued)

Modified Date: Thu, Jul 01, 2010 03:49:13 PM

Output Date: Tue, Jul 06, 2010 11:48:50 AM

REV-II

Project: Management Accounting_Debarshi Bhattacharyya ACE Pro India Pvt. Ltd. File: X:\Pearson\Management Accounting_Debarshi Bhattacharyya\MAIN\M05\LAYOUT_M05\M05_DEBA_ISBN_EN_SE_C05_Part-1.indd

375

FUND FLOW ANALYSIS

Elements of Working Capital Less:

Current Liabilities: Sundry Creditors Bills Payable Working Capital Decrease in Working Capital

As on 31 March 2009 Rs. − (12,500) (5,000) 1,02,500

As on 31 March 2010 Rs. (50,000) 80,000 2,500

Working Notes 1. Dr.

Equity Share Capital Account Rs. By Balance b/f By General Reserve—Bonus Shares Issued (1 ÷ 5 × 30,000 = 6,000 Shares of Rs. 10) 4,00,000 By Bank—New Issue of Shares for Cash (Bal. fig.) 4,00,000

To Balance c/f

Cr. Rs. 3,00,000 60,000 40,000 4,00,000

2. Dr.

8% Preference Share Capital Account Rs. To Bank—Preference Shares Redeemed 1,50,000 By Balance b/f By Bank—Final Call on Shares Received (1,500 × Rs. 25) 1,50,000

Cr. Rs. 1,12,500 37,500 1,50,000

Note: No. of Preference Shares = Rs. 1,12,500 ÷ Rs. 75 = 1,500 Shares.

3. Dr. To Bank—Premium Paid on Redemption

Premium on Redemption on Preference Shares Account Rs. 15,000 By Securities Premium A/c—Premium Paid on Redemption Adjusted 15,000

Cr. Rs. 15,000

Securities Premium Account Rs. By Balance b/f By Bank—Premium Received on New Issue of Equity Shares (5% on Rs. 40,000) 15,000 7,000 22,000

Cr. Rs. 20,000

15,000

4. Dr.

To Premium on Redemption on Preference Shares A/c—Premium Paid Adjusted To Balance c/f

2,000 22,000

5. Dr.

To Balance c/f

Capital Redemption Reserve Account Rs. By Balance b/f By Profit & Loss A/c—Difference between the Face Value of Preference Shares Redeemed & Face Value of New Shares Issued for Cash Transferred 1,10,000 1,10,000

Cr. Rs. − 1,10,000

1,10,000 (Continued)

Modified Date: Thu, Jul 01, 2010 03:49:13 PM

Output Date: Tue, Jul 06, 2010 11:48:50 AM

REV-II

Project: Management Accounting_Debarshi Bhattacharyya ACE Pro India Pvt. Ltd. File: X:\Pearson\Management Accounting_Debarshi Bhattacharyya\MAIN\M05\LAYOUT_M05\M05_DEBA_ISBN_EN_SE_C05_Part-1.indd

376

MANAGEMENT ACCOUNTING

6. Dr. To Equity Share Capital A/c— Bonus Dividend declared

General Reserve Account Rs. By Balance b/f 60,000

Cr. Rs. 30,000

To Balance c/f

50,000

80,000

By Profit & Loss A/c—Transfer during the year (Bal. fig.)

1,10,000

1,10,000

7. Dr. To Bank—Preference Dividend Paid (8% on Rs. 1,12,500)

Preference Dividend Account Rs. By Profit & Loss A/c—Preference Dividend declared 9,000 9,000

Cr. Rs. 9,000 9,000

8. Dr. To Bank—Dividend for 2006−07 paid (20,000 − 3,000) To Unpaid Dividend A/c—Dividend not claimed To Balance c/f

Proposed Dividend Account Rs. 17,000 By Balance b/f

3,000 30,000 50,000

By Profit & Loss Appropriation A/c—Dividend Proposed in 2007−08

Cr. Rs. 20,000

30,000 50,000

9. Dr.

To Balance c/f

Unpaid Dividend Account Rs. By Balance b/f 3,000 By Proposed Dividend A/c—Unpaid Dividend for 2006−07 3,000

Cr. Rs. Nil 3,000 3,000

10. Dr. To Bank—Tax for the year 2006−07 Paid in 2007−08 To Balance c/f

Provision for Tax Account Rs. 20,000 By Balance b/f

Cr. Rs. 20,000

30,000 50,000

30,000 50,000

By Profit & Loss A/c—Provision Made for 2007−08

11. Dr. To Balance b/f To Bank—Cost of New Machinery

Plant & Machinery Account Rs. 2,00,000 By Bank—Sale Proceeds of Old Machine (60,000 − 8,000) 1,60,000 By Profit & Loss A/c—Loss on Sale of Old Machinery By Profit & Loss A/c—Depreciation for the year (Bal. fig.) By Balance c/f 3,60,000

Cr. Rs. 52,000 8,000 20,000 2,80,000 3,60,000 (Continued)

Modified Date: Thu, Jul 01, 2010 03:49:13 PM

Output Date: Tue, Jul 06, 2010 11:48:50 AM

REV-II

Project: Management Accounting_Debarshi Bhattacharyya ACE Pro India Pvt. Ltd. File: X:\Pearson\Management Accounting_Debarshi Bhattacharyya\MAIN\M05\LAYOUT_M05\M05_DEBA_ISBN_EN_SE_C05_Part-1.indd

377

FUND FLOW ANALYSIS

12. Dr.

Cr.

Furniture Account Rs. 80,000 78,000 3,000

To Balance b/f To Bank—Cost of New Furniture To Profit & Loss A/c—Profit on Sale of Furniture

By Bank—Sale Proceeds of Furniture (Bal. fig.) By Profit & Loss A/c—Depreciation for the year By Balance c/f

1,61,000

Rs. 26,000 15,000 1,20,000 1,61,000

13. Analysis of other Non-Current Assets and liabilities Assets/Liabilities Land & Building Preliminary Expenses Investment

Opening Balance Rs. 1,30,000 20,000 30,000

Closing Balance Rs. 1,40,000 10,000 40,000

Increase/ Decrease Rs. (+)15,000 (−)10,000 (+)10,000

Analysis Addition to Building Written off Against Profit & Loss A/c New Investment Made

14. Dr. To Non-Current & Non-Operating Items Charged:

Cr.

Adjusted Profit & Loss A/c Rs. By Balance b/f

Capital Redemption Reserve5 General Reserve6 Preference Dividend7 Proposed Dividend 8 Preliminary Expenses 13 Loss on Sale of Machinery11 Depreciation on Plant & Machinery11 Depreciation on Furniture12 Provision for Tax10 To Balance c/f

1,10,000 80,000 9,000 30,000 10,000 8,000 20,000 15,000 30,000 40,000 3,52,000

Rs. 60,000

By Non-Current & Non-Operating Items Credited: By Profit on Sale of Furniture12

3,000

By Net Fund Flow from Operation (Bal. fig.)

2,89,000

3,52,000

Problem 15 From the following information provided by Tatkal Ltd, prepare a statement showing changes in the Working Capital and a Fund Flow Statement for the year that ended on 31 March 2010: Balance Sheets Liabilities Equity Share Capital Securities Premium Profit & Loss A/c 10% Debentures Bank Overdraft Creditors

As on 31 March 2009 (Rs.) 1,00,000 15,000 28,000 70,000 14,000 34,000

As on 31 March 2010 (Rs.) 1,50,000 35,000 70,000 30,000 − 48,000

Assets Freehold Property Plant & Machinery Furniture Stock Debtors Bank

As on 31 March 2009 (Rs.) 1,10,000 1,20,000 24,000 37,000 43,000 −

As on 31 March 2010 (Rs.) 1,30,000 1,51,000 29,000 51,000 44,000 16,000 (Continued)

Modified Date: Thu, Jul 01, 2010 03:49:13 PM

Output Date: Tue, Jul 06, 2010 11:48:50 AM

REV-II

Project: Management Accounting_Debarshi Bhattacharyya ACE Pro India Pvt. Ltd. File: X:\Pearson\Management Accounting_Debarshi Bhattacharyya\MAIN\M05\LAYOUT_M05\M05_DEBA_ISBN_EN_SE_C05_Part-1.indd

378

MANAGEMENT ACCOUNTING

Liabilities Proposed Dividend Provision for Depreciation on: Plant & Machinery Furniture

As on 31 March 2009 (Rs.) 15,000

As on 31 March 2010 (Rs.) 20,000

45,000 13,000 3,34,000

54,000 15,000 4,22,000

Assets Premium on Redemption of Debentures

As on 31 March 2009 (Rs.) −

As on 31 March 2010 (Rs.) 1,000

3,34,000

4,22,000

Additional information: i. There had been no disposal of freehold property in the year. ii. The machine, which had cost Rs. 8,000 and in respect of which Rs. 6,000 depreciation had been provided, was sold for Rs. 3,000, and furniture, which had cost Rs. 5,000 and in respect of which a depreciation of Rs. 2,000 had been provided, was sold for Rs. 1,000. The profits and losses on these transactions had been dealt with through Profit & Loss A/c. iii. Actual premium on the redemption of debentures was Rs. 2,000, of which Rs. 1,000 had been written off to the Profit & Loss A/c. iv. No interim dividend has been paid. [B.Com. (Hons), Calcutta University—Adapted]

Solution Books of Tatkal Ltd Fund Flow Statement for the year that ended on 31 March 2010 Sources of Fund New Issue of Equity Shares8 Premium Received on New Issue of Equity Shares8 Proceeds from Sale of Furniture3 Proceeds from Sale of Machinery1 Net Fund Flow from Operation9

Rs. 50,000 20,000 1,000 3,000 83,000

Applications of Fund Purchase of New Machinery1 Purchase of New Furniture3 Purchase of Freehold Property8 Redemption of Debentures at Premium6 Payment of Proposed Dividend7 Increase in Working Capital (Bal. fig.)

1,57,000

Rs. 39,000 10,000 20,000 42,000 15,000 31,000 1,57,000

Statement showing changes in Working Capital for the year that ended on 31 March 2010 Elements of Working Capital Current Assets: Stock Debtors Bank Less:

Current Liabilities: Creditors Bank overdraft Working Capital Increase in Working Capital

Modified Date: Thu, Jul 01, 2010 03:49:13 PM

Output Date: Tue, Jul 06, 2010 11:48:50 AM

As on 31 March 2009 (Rs.)

As on 31 March 2010 (Rs.)

37,000 43,000 − 80,000

51,000 44,000 16,000 1,11,000

(34,000) (14,000) 32,000

(48,000) − 63,000 31,000

REV-II

Project: Management Accounting_Debarshi Bhattacharyya ACE Pro India Pvt. Ltd. File: X:\Pearson\Management Accounting_Debarshi Bhattacharyya\MAIN\M05\LAYOUT_M05\M05_DEBA_ISBN_EN_SE_C05_Part-1.indd

379

FUND FLOW ANALYSIS

Working Notes 1. Dr.

Plant & Machinery (at cost) Account Rs. To Balance b/f 1,20,000 By Provision for Depreciation A/c —Accumulated Depreciation on To Bank—New Purchase (Bal. fig.) 39,000 Machinery sold To Profit & Loss A/c—Profit on Sale of By Bank—Sale Proceeds of Machinery Machinery [3,000 − (8,000 − 6,000)] 1,000 By Balance c/f 1,60,000

Cr. Rs. 6,000

3,000 1,51,000 1,60,000

2. Dr.

Provision for Depreciation on Plant & Machinery Account Rs. To Plant & Machinery A/c—Accumulated By Balance b/f Depreciation on the Machinery sold 6,000 To Balance c/f 54,000 By Profit & Loss A/c—Depreciation for the year (Bal. fig.) 60,000

Cr. Rs. 45,000 15,000 60,000

3. Dr. To Balance b/f

To Bank—New Purchase (Bal. fig.) To Bank—Sale Proceeds of Furniture

Furniture (at cost) Account Rs. 24,000 By Provision for Depreciation A/c —Accumulated Depreciation on the Furniture Sold 10,000 1,000 By Profit & Loss A/c—Loss on Sale of Furniture [(5,000 − 2,000) − 1,000] By Balance c/f 34,000

Cr. Rs.

2,000

2,000 29,000 34,000

4. Dr.

Provision for Depreciation on Furniture Account Rs. To Furniture A/c—Accumulated By Balance b/f Depreciation on the Furniture Sold 2,000 By Profit & Loss A/c—Depreciation for the year (Bal. fig.) To Balance c/f 15,000 17,000

Cr. Rs. 13,000 4,000

17,000

5. Dr.

Premium on Redemption of Debenture Account Rs. To Balance b/f Nil By Profit & Loss A/c—Premium written off To 10% Debenture A/c—Premium Paid on 2,000 By Balance c/f Redemption 2,000

Cr. Rs. 1,000 1,000 2,000 (Continued)

Modified Date: Thu, Jul 01, 2010 03:49:13 PM

Output Date: Tue, Jul 06, 2010 11:48:50 AM

REV-II

Project: Management Accounting_Debarshi Bhattacharyya ACE Pro India Pvt. Ltd. File: X:\Pearson\Management Accounting_Debarshi Bhattacharyya\MAIN\M05\LAYOUT_M05\M05_DEBA_ISBN_EN_SE_C05_Part-1.indd

380

MANAGEMENT ACCOUNTING

6. Dr.

10% Debenture Account Rs. 42,000 By Balance b/f

To Bank—Debentures Redeemed at Premium (Bal. fig.) To Balance c/f

30,000

Cr. Rs. 70,000

By Premium on Redemption A/c—Premium Paid on Redemption

72,000

2,000 72,000

7. Dr. To Bank—Dividend for 2006−07 paid in 2007−08 To Balance c/f

Proposed Dividend Account Rs. 15,000 By Balance b/f 20,000

Cr. Rs. 15,000

By Profit & Loss Appropriation A/c— Dividend Proposed in 2007−08 (Bal. fig.)

35,000

20,000 35,000

8. Analysis of other Non-Current Assets and Liabilities

Equity Share Capital Securities Premium

Opening Balance Rs. 1,00,000 15,000

Closing Balance Rs. 1,50,000 35,000

Increase/ Decrease Rs. (+)50,000 (+)20,000

Freehold Property

1,10,000

1,30,000

(+)20,000

Assets/Liabilities

Analysis New Issue for Cash Premium Received on New Issue of Equity Shares New Purchase for Cash

9. Dr. To Non-Current & Non-Operating items charged: Proposed Dividend7 Depreciation on Furniture4 Depreciation on Plant & Machinery2 Premium on Redemption of Debentures5 Loss on Sale of Furniture3 To Balance c/f

Adjusted Profit & Loss A/c Rs. By Balance b/f 20,000 4,000 15,000

Cr. Rs. 28,000

By Non-Current & Non-Operating Items Credited: Profit on Sale of Machinery1 By Net Fund Flow from Operation (Bal. fig.)

1,000 2,000 70,000 1,12,000

1,000 83,000

1,12,000

Problem 16 From the following particulars, prepare the trading and Profit & Loss A/c statement showing changes in the Working Capital and a statement showing sources and applications of Funds for the year that ended on 31 December 2009: Balance Sheet of Business Magnet as on 31 December 2008 is as follows: Liabilities Capital Creditors for Goods Outstanding Expenses

(Rs.) 2,00,000 35,000 5,000

Assets Mach inery Furniture Stock

(Rs.) 70,000 15,000 35,000 (Continued)

Modified Date: Thu, Jul 01, 2010 03:49:13 PM

Output Date: Tue, Jul 06, 2010 11:48:50 AM

REV-II

Project: Management Accounting_Debarshi Bhattacharyya ACE Pro India Pvt. Ltd. File: X:\Pearson\Management Accounting_Debarshi Bhattacharyya\MAIN\M05\LAYOUT_M05\M05_DEBA_ISBN_EN_SE_C05_Part-1.indd

FUND FLOW ANALYSIS

Liabilities

(Rs.)

Assets

381

(Rs.) 1,00,000 15,000 5,000 2,40,000

Debtors Bank Cash 2,40,000

Additional information: i. Debtor’s velocity and creditor’s velocity are 2 months and 1 month, respectively, with stock levels uniform throughout the year. ii. Uniform rate of gross profit @ 33.33 %. iii. Sales for the current year are 20% more than the previous year. iv. Sales are 20% in cash and 80% on credit. v. Out of the total receipt from debtors during the year, Rs. 50, 000 was received in cash and the balance by cheque. vi. Following payments are made out of bank: Machinery—Rs. 40,000; Furniture—Rs. 5,000; Investments—Rs. 40,000; Drawings—Rs. 20,000; Business expenditure—Rs. 60,000 and Creditors − ? vii. Following payments are made in cash: Business expenditure—Rs. 90,000 and Cash deposited into bank—Rs. 1,00,000. viii. Depreciate assets @10%. Solution Books of Business Magnet Dr.

Trading and Profit & Loss A/c for the year that ended on 31 December 2009 Amount Particulars Particulars (Rs.) To Opening Stock 35,000 By Sales1 To Purchases (Bal. fig.) 6,00,000 By Closing Stock3 2 3,00,000 To Gross Profit c/d 9,35,000 3,35,000 By Gross Profit b/d To Business Expenses4 11,000 To Depreciation on Machinery [10% on (70,000 + 40,000)] 2,000 To Depreciation on Furniture [10% on (15,000 + 5,000)] To Net Profit 1,42,000 3,00,000

Cr. Amount (Rs.) 9,00,000 35,000 9,35,000 3,00,000

3,00,000

Statement showing changes in Working Capital for the year that ended on 31 December 2009 Elements of Working Capital Current Assets: Stock Debtors Bank Cash Less:

Current Liabilities: Creditors for Goods Outstanding Expenses Working Capital Increase in Working Capital

Modified Date: Thu, Jul 01, 2010 03:49:13 PM

Output Date: Tue, Jul 06, 2010 11:48:50 AM

As on As on 31 December 31 December 2008 Rs. 2009 Rs. 35,000 1,00,000 15,000 5,000 1,55,000

35,000 1,20,0005 15,0009 45,0009 2,15,000

(35,000) (5,000) 1,15,000

(50,0006) − 1,65,000 50,000

REV-II

Project: Management Accounting_Debarshi Bhattacharyya ACE Pro India Pvt. Ltd. File: X:\Pearson\Management Accounting_Debarshi Bhattacharyya\MAIN\M05\LAYOUT_M05\M05_DEBA_ISBN_EN_SE_C05_Part-1.indd

382

MANAGEMENT ACCOUNTING

Statement showing sources and applications of Funds for the year that ended on 31 December 2009 Sources of Fund Fund from Operation

Rs. 1,55,000

Applications of Fund Purchase of Machinery Purchase of Furniture3 Further Investment Made Proprietors' Drawings Increase in Working Capital (Bal. fig.)

1,55,000

Rs. 40,000 5,000 40,000 42,000 50,000 1,55,000

Working Notes 1. Calculation of Sales for the year 2009 Debtors on 31 December 2008 = Rs. 1,00,000 Debtors’ Velocity = 2 months. Debtors Again, Debtors’ Velocity = × 12 Credit Sales 1,00,000 Here, 2 = × 12 Credit Sales Credit Sales for the year 2008 = 1,00,000 ÷ 2 × 12 = Rs. 6,00,000 Cash Sales for the year 2008 = 20% ÷ 80% × 6,00,000 = Rs. 1,50,000 Total Sales for the year 2008 = Rs. 7,50,000 Add: 20% increase in Sales during 2009 = Rs. 1,50,000 [20% of Rs. 7,50,000] Total Sales for the year 2009

= Rs. 9,00,000

Cash Sales for the year 2009 = 20% of Rs. 9,00,000 = Rs. 1,80,000. Credit Sales for the year 2009 = 80% of Rs. 9,00,000 = Rs. 7,20,000. 2. Calculation of Gross Profit for the year 2009 Gross profit for the year 2009 = 33.33 % on sales = 33.33% on Rs. 9,00,000 = Rs. 3,00,000 3. Valuation of Stock as on 31 December 2009 Stock on 31 December 2008 = Rs. 35,000 As Stock levels are uniform throughout the year 2009, Value of Stock as on 31 December 2009 = Rs. 35,000 4. Dr.

Business Expenses Account Rs. To Bank—Expenses Paid by Cheque during the 60,000 By Balance b/f year 2009 To Cash 90,000 By Profit & Loss A/c—Expenses for the —Expenses Paid in Cash during the year 2009 year 2009 1,50,000

Cr. Rs. 5,000 1,45,000 1,50,000

5. Calculation of Balance of Debtors on 31 December 2009 Credit Sales for the year 2009 = Rs. 7,20,000 Debtors’ Velocity = 2 months Debtors Again, Debtors’ Velocity = × 12 Credit Sales Debtors Here, 2 = × 12 7,20,000 Debtors as on 31 December 2009 = 7,20,000 × 2 ÷ 12 = Rs. 1,20,000 (Continued)

Modified Date: Thu, Jul 01, 2010 03:49:13 PM

Output Date: Tue, Jul 06, 2010 11:48:50 AM

REV-II

Project: Management Accounting_Debarshi Bhattacharyya ACE Pro India Pvt. Ltd. File: X:\Pearson\Management Accounting_Debarshi Bhattacharyya\MAIN\M05\LAYOUT_M05\M05_DEBA_ISBN_EN_SE_C05_Part-1.indd

FUND FLOW ANALYSIS

383

6. Calculation of Balance of Creditors on 31 December 2009 Credit Purchases for the year 2009 (ascertained as a balancing figure in Trading A/c) = Rs. 6,00,000 Creditors’ Velocity = 1 month Again, Creditors’ Velocity = Here, 1 =

Creditors × 12 Credit Purchases

Creditors × 12 6,00,000

Creditors as on 31 December 2009 = 6,00,000 × 1 ÷ 12 = Rs. 50,000 7. Dr. To Balance b/f To Sales1—Credit

Total Debtors Account Rs. 60,000 By Cash—Collection in Cash 7,20,000 By Bank—Collection by Cheque (Bal. fig.) By Balance c/f5 8,20,000

Cr. Rs. 50,000 6,50,000 1,20,000 8,20,000

Total Creditors Account Rs. 5,85,000 By Balance b/f By Purchases—Credit 50,000 6,35,000

Cr. Rs. 35,000 6,00,000

8. Dr. To Bank—Payment by Cheque (Bal. fig.) To Balance c/f6

6,35,000

9. Dr. Receipts To Balance b/f To Debtors7 To Cash Sales1 To Cash (C)

Cash Rs. 5,000 50,000 1,80,000 −

2,35,000

Cash & Bank Account Bank Payments Rs. 15,000 By Creditors8 6,50,000 By Machinery − By Furniture 1,00,000 By Bank (C) By Investments By Drawings By Business Expenses By Balance c/f 7,65,000

Cash Rs. − − − 1,00,000 − − 90,000 45,000 2,35,000

Cr. Bank Rs. 5,85,000 40,000 5,000 − 40,000 20,000 60,000 15,000 7,65,000

10. Calculation of Fund from Operation for the year that ended on 31 December 2009 Rs. Add:

Less:

Net Profit as per Profit & Loss A/c Adjustment for Non-Current and Non-Operating items Debited to Profit & Loss A/c: Depreciation on Machinery Depreciation on Furniture

Adjustment for Non-Current and Non-Operating items credited to Profit & Loss A/c Fund from Operation

Modified Date: Thu, Jul 01, 2010 03:49:13 PM

Output Date: Tue, Jul 06, 2010 11:48:50 AM

Rs. 1,42,000

11,000 2,000 13,000 1,55,000 Nil 1,55,000

REV-II

Project: Management Accounting_Debarshi Bhattacharyya ACE Pro India Pvt. Ltd. File: X:\Pearson\Management Accounting_Debarshi Bhattacharyya\MAIN\M05\LAYOUT_M05\M05_DEBA_ISBN_EN_SE_C05_Part-1.indd

384

MANAGEMENT ACCOUNTING

Problem 17 Following were the Balance Sheets of Amulya Ltd as on 31 December 2008 and 31 December 2009: Liabilities Equity Share Capital Preference Share Capital Profit & Loss A/c General Reserve Capital Redemption Reserve 10% Debentures Creditors Proposed Dividend Provision for Tax Provision for Depreciation on Plant & Machinery Bills Payable

2008 Rs. 3,00,000 2,00,000 30,000 60,000 − 1,00,000 30,000 30,000 40,000 40,000 15,000 8,45,000

2009 Rs. 4,00,000 − 70,000 70,000 1,00,000 2,00,000 50,000 40,000 45,000 60,000

Assets Goodwill Land & Building Plant & Machinery Investment Stock Bills Receivable Debtors Cash & Bank Preliminary Expenses

20,000 10,55,000

2008 Rs. 50,000 1,80,000 1,60,000 60,000 1,80,000 35,000 1,10,000 50,000 20,000

2009 Rs. 40,000 2,40,000 2,20,000 1,10,000 1,60,000 45,000 1,40,000 90,000 10,000

8,45,000

10,55,000

Additional information: i. During 2009, a part of machinery costing Rs. 50,000, whose up-to-date depreciation was Rs. 20,000, was sold for Rs. 25,000. ii. During 2009, a part of the building costing Rs. 90,000, whose up-to-date depreciation was Rs. 50,000, was sold for Rs. 70,000. Depreciation on building for 2009 was Rs. 15,000. iii. During 2009, preference shares were redeemed at a premium of 10%, partly out of a new issue of equity shares at par and partly out of profit. iv. Tax paid during 2009 amounted to Rs. 35,000. v. During 2009, an interim dividend of Rs. 20,000 was paid in addition to the proposed dividend. vi. Debentures were issued at a discount of 10%. Prepare a Statement showing the changes in the Working Capital and a Fund Flow Statement of Amulya Ltd for the year that ended on 31 December 2009.

Solution Books of Amulya Ltd Fund Flow Statement for the year that ended on 31 December 2009 Sources of Fund New Issue of Equity Shares1 New Issue of Debentures5 Proceeds from Sale of Machinery9 Proceeds from Sale of Building11 Net Fund Flow from Operation13

Rs. 1,00,000 90,000 25,000 70,000 3,30,000

Applications of Fund Purchase of New Plant & Machinery9 Purchase of New Land & building11 New Investment Made12 Payment of Tax8 Payment of Interim Dividend6 Payment of Proposed Dividend7 Redemption of Preference Shares2 Premium Paid on Redemption of Preference Shares3 Increase in Working Capital (Bal. fig.)

6,15,000

Modified Date: Thu, Jul 01, 2010 03:49:13 PM

Output Date: Tue, Jul 06, 2010 11:48:50 AM

Rs. 1,10,000 1,15,000 50,000 35,000 20,000 30,000 2,00,000 20,000 35,000 6,15,000

REV-II

Project: Management Accounting_Debarshi Bhattacharyya ACE Pro India Pvt. Ltd. File: X:\Pearson\Management Accounting_Debarshi Bhattacharyya\MAIN\M05\LAYOUT_M05\M05_DEBA_ISBN_EN_SE_C05_Part-1.indd

385

FUND FLOW ANALYSIS

Statement showing changes in Working Capital for the year that ended on 31 December 2009 Elements of Working Capital Current Assets: Stock Debtors Bills Receivable Cash & Bank Less:

Current Liabilities: Creditors Bills Payable Working Capital Increase in Working Capital

As on 31 December 2008 Rs.

As on 31 December 2009 Rs.

1,80,000 1,10,000 35,000 50,000 3,75,000

1,60,000 1,40,000 45,000 90,000 4,35,000

(30,000) (15,000) 3,30,000

(50,000) (20,000) 3,65,000 35,000

Working Notes 1. Dr.

To Balance c/f

Equity Share Capital Account Rs. By Balance b/f By Bank—New Issue of Shares for Cash (Bal. fig.) 4,00,000 4,00,000

Cr. Rs. 3,00,000 1,00,000

4,00,000

2. Dr.

Preference Share Capital Account Rs. To Bank—Preference Shares Redeemed 2,00,000 By Balance b/f To Balance c/f Nil 1,50,000

Cr. Rs. 2,00,000 1,50,000

3. Dr.

Premium on Redemption on Preference Shares Account Rs. To Bank—Premium Paid on Redemption 20,000 By Profit & Loss A/c—Premium Paid on (10% of Rs. 2,00,000) Redemption Adjusted 20,000

Cr. Rs. 20,000 20,000

4. Dr.

To Balance c/f

Capital Redemption Reserve Account Rs. By Balance b/f By Profit & Loss A/c—Face Value of Preference Shares Redeemed less Face 1,00,000 Value of New Equity Shares Issued (Rs. 2,00,000 − Rs. 1,00,000) 1,00,000

Cr. Rs. − 1,00,000

1,00,000 (Continued)

Modified Date: Thu, Jul 01, 2010 03:49:13 PM

Output Date: Tue, Jul 06, 2010 11:48:50 AM

REV-II

Project: Management Accounting_Debarshi Bhattacharyya ACE Pro India Pvt. Ltd. File: X:\Pearson\Management Accounting_Debarshi Bhattacharyya\MAIN\M05\LAYOUT_M05\M05_DEBA_ISBN_EN_SE_C05_Part-1.indd

386

MANAGEMENT ACCOUNTING

5. Dr.

To Balance c/f

10% Debentures Account Rs. By Balance b/f By Bank—New Debentures Issued for Cash By Profit & Loss A/c—Discount on Issue of Debentures Adjusted 2,00,000 (10% of Rs. 1,00,000) 2,00,000

Cr. Rs. 1,00,000 90,000 10,000 2,00,000

6. Dr. To Bank—Interim Dividend Paid

Interim Dividend Account Rs. 20,000 By Profit & Loss Appropriation A/c—Interim Dividend Adjusted 20,000

Cr. Rs. 20,000 20,000

7. Dr.

Proposed Dividend Account Rs. To Bank—Dividend Proposed in 2008 Paid By Balance b/f in 2009 30,000 To Balance c/f 40,000 By Profit & Loss Appropriation A/c —Dividend Proposed for 2009 70,000

Cr. Rs. 30,000

40,000 70,000

8. Dr. To Bank—Tax Paid in 2009 To Balance c/f

Provision for Tax Account Rs. 35,000 By Balance b/f 45,000 By Profit & Loss A/c—Provision Made in 2009 (Bal. fig.) 80,000

Cr. Rs. 40,000 40,000 80,000

9. Dr. To Balance b/f

To Bank—New Purchase (Bal. fig.)

Plant & Machinery (at cost) Account Rs. 1,60,000 By Provision for Depreciation A/c —Accumulated Depreciation on Machinery sold 1,10,000 By Bank—Sale Proceeds of Machinery By Profit & Loss A/c—Loss on Sale of Machinery [(50,000 − 20,000) − 25,000] By Balance c/f 2,70,000

Cr. Rs. 20,000

25,000 5,000 2,20,000 2,70,000

10. Dr.

Provision for Depreciation on Plant & Machinery Account Rs. To Plant & Machinery A/c—Accumulated 20,000 By Balance b/f Depreciation on Machinery sold

Cr. Rs. 40,000

(Continued)

Modified Date: Thu, Jul 01, 2010 03:49:13 PM

Output Date: Tue, Jul 06, 2010 11:48:50 AM

REV-II

Project: Management Accounting_Debarshi Bhattacharyya ACE Pro India Pvt. Ltd. File: X:\Pearson\Management Accounting_Debarshi Bhattacharyya\MAIN\M05\LAYOUT_M05\M05_DEBA_ISBN_EN_SE_C05_Part-1.indd

387

FUND FLOW ANALYSIS

Dr.

To Balance c/f

Provision for Depreciation on Plant & Machinery Account Rs. By Profit & Loss A/c—Depreciation for the year (Bal. fig.) 60,000 80,000

Cr. Rs. 40,000

80,000

11. Dr.

Land & Building Account Rs. 1,80,000 By Bank—Sale Proceeds of Building 30,000 By Profit & Loss A/c—Depreciation for the year

To Balance b/f To Profit & Loss A/c—Profit on Sale of Building [70,000 − (90,000 − 50,000)] To Bank—New Purchase of Land & Building (Bal. fig.)

1,15,000 3,25,000

By Balance c/f

Cr. Rs. 70,000 15,000

2,40,000 3,25,000

12. Analysis of other Non-Current Assets and Liabilities Assets/Liabilities General Reserve Goodwill Preliminary Expenses Investment

Opening Balance Rs. 60,000 50,000 20,000 60,000

Closing Balance Rs. 70,000 40,000 10,000 1,10,000

Increase/ Decrease Rs. (+)10,000 (−)10,000 (−)10,000 (+)50,000

Analysis Transfer from Profit & Loss A/c Written off Against Profit & Loss A/c Written off Against Profit & Loss A/c New Investment made

13. Dr.

Adjusted Profit & Loss A/c Rs. By Balance b/f

To Non-Current & Non-Operating Items Charged: Capital Redemption Reserve4 General Reserve12 Interim Dividend6 Proposed Dividend7 Preliminary Expenses 12 Loss on Sale of Machinery9 Depreciation on Plant & Machinery10 Depreciation on Land & building11 Provision for Tax8 Discount on Issue of Debentures12 Goodwill12 Premium on Redemption of Preference Shares3 To Balance c/f

Modified Date: Thu, Jul 01, 2010 03:49:13 PM

1,00,000 10,000 20,000 40,000 10,000 5,000 40,000 15,000 40,000 10,000 10,000 20,000

By Non-Current & Non-Operating items credited: Profit on Sale of Building11

By Net Fund Flow from Operation (Bal. fig.)

70,000 3,90,000

Output Date: Tue, Jul 06, 2010 11:48:50 AM

Cr. Rs. 30,000

30,000

3,30,000

3,90,000

REV-II

Project: Management Accounting_Debarshi Bhattacharyya ACE Pro India Pvt. Ltd. File: X:\Pearson\Management Accounting_Debarshi Bhattacharyya\MAIN\M05\LAYOUT_M05\M05_DEBA_ISBN_EN_SE_C05_Part-2.indd

388

MANAGEMENT ACCOUNTING

Problem 18 Balance Sheet of OP Ltd as on 31 March 2007 and 2008 are as follows: Liabilities Share Capital General Reserve Profit & Loss A/c 10% Debenture Bank loan (Long-term) Creditors Outstanding Expenses Proposed Dividend Provision for Taxation

Amount 31 March 2007 Rs. 20,00,000 4,00,000 2,50,000 10,00,000 5,00,000 4,00,000 20,000 3,00,000 1,00,000 49,70,000

Amount 31 March 2008 Rs. 20,00,000 4,50,000 3,60,000 8,00,000 6,00,000 5,80,000 25,000 3,60,000 1,20,000 52,95,000

Assets Land & Building Plant & Machinery Investment Stock Debtors Prepaid Expenses Cash & Bank

Amount 31 March 2007 Rs. 15,00,000 18,00,000 4,00,000 4,80,000 6,00,000 50,000 1,40,000

Amount 31 March 2008 Rs. 14,00,000 17,50,000 3,72,000 8,50,000 7,98,000 40,000 85,000

49,70,000

52,95,000

Additional information: i. New machinery for Rs. 3,00,000 was purchased but an old machinery costing Rs. 1,45,000 was sold for Rs. 50,000 and accumulated depreciation thereon was Rs. 75,000. ii. 10% debentures were redeemed at 20% premium. iii. Investments were sold for Rs. 45,000, and its profit was transferred to general reserve. iv. Income tax paid during the year 2007–08 was Rs. 80,000. v. An interim dividend of Rs. 1,20,000 has been paid during the year 2007–08. vi. Assume the provision for taxation as Current Liability and proposed dividend as non-Current Liability. vii. Investments are non-trade investment. You are required to prepare: i. Schedule of changes in Working Capital. ii. Fund Flow Statement. [C.A. (PE II)—November 2008] Solution Books of OP Ltd i. Schedule of changes in Working Capital for the year that ended on 31 March 2008 Elements of Working Capital Current Assets: Stock Debtors Prepaid Expenses Cash & Bank Less:

Current Liabilities: Creditors Outstanding Expenses Provision for Taxation Working Capital Increase in Working Capital

Modified Date: Thu, Jul 01, 2010 02:41:25 PM

Output Date: Tue, Jul 06, 2010 11:50:19 AM

As on 31 March 2007 Rs.

As on 31 March 2008 Rs.

4,80,000 6,00,000 50,000 1,40,000 12,70,000

8,50,000 7,98,000 40,000 85,000 17,73,000

(4,00,000) (20,000) (1,00,000) 7,50,000

(5,80,000) (25,000) (1,20,000) 10,48,000 2,98,000

REV-II

Project: Management Accounting_Debarshi Bhattacharyya ACE Pro India Pvt. Ltd. File: X:\Pearson\Management Accounting_Debarshi Bhattacharyya\MAIN\M05\LAYOUT_M05\M05_DEBA_ISBN_EN_SE_C05_Part-2.indd

389

FUND FLOW ANALYSIS

ii. Fund Flow Statement for the year that ended on 31 March 2008 Sources of Fund Proceeds from Sale of Machinery Proceeds from Sale of Investment Fresh Bank Loan Raised7 Net Fund Flow from operation8

Rs. 50,000 45,000 1,00,000 10,63,000

Applications of Fund Purchase of new Plant & Machinery Payment of Proposed Dividend4 Payment of Interim Dividend6 Redemption of Debentures at Premium6 Increase in Working Capital (Bal. fig.)

12,58,000

Rs. 3,00,000 3,00,000 1,20,000 2,40,000 2,98,000 12,58,000

Working Notes 1. Dr.

Plant & Machinery Account Rs. 18,00,000 By Bank—Sale Proceeds of old Machinery 3,00,000 By Profit & Loss A/c—Loss on Sale of old Machinery [(1,45,000 – 75,000) – 50,000] By Profit & Loss A/c—Depreciation for the year (Bal. fig.) By Balance c/f 21,00,000

To Balance b/f To Bank—Cost of new Machinery

Cr. Rs. 50,000 20,000 2,80,000 17,50,000 21,00,000

2. Dr. To Balance b/f To General Reserve—Profit on Sale of Investment (Bal. fig.)

Investment Account Rs. 4,00,000 By Bank—Sale Proceeds of Investment 17,000 4,17,000

By Balance c/f

Cr. Rs. 45,000 3,72,000 4,17,000

3. Dr.

To Balance c/f

General Reserve Account Rs. By Balance b/f By Investment A/c—Profit on Sale By Profit & Loss A/c—Transferred during the year (Bal. fig.) 4,50,000 4,50,000

Cr. Rs. 4,00,000 17,000 33,000 4,50,000

4. Dr.

Proposed Dividend Account Rs.

To Bank—Proposed Dividend for 2006–07 paid in 2007–08

To Balance c/f

Cr.

By Balance b/f

Rs. 3,00,000

By Profit & Loss Appropriation A/c —Dividend proposed in 2007–08

3,60,000

3,00,000

3,60,000 6,60,000

6,60,000

(Continued)

Modified Date: Thu, Jul 01, 2010 02:41:25 PM

Output Date: Tue, Jul 06, 2010 11:50:19 AM

REV-II

Project: Management Accounting_Debarshi Bhattacharyya ACE Pro India Pvt. Ltd. File: X:\Pearson\Management Accounting_Debarshi Bhattacharyya\MAIN\M05\LAYOUT_M05\M05_DEBA_ISBN_EN_SE_C05_Part-2.indd

390

MANAGEMENT ACCOUNTING

5. Dr.

Interim Dividend Account Rs. 1,20,000

To Bank – Interim Dividend Paid

Cr. Rs. 1,20,000

By Profit & Loss Appropriation A/c —Interim Dividend adjusted

1,20,000

1,20,000

6. Dr.

10% Debentures Account

To Bank—Redeemed at 20% Premium [120% of (10,00,000 – 8,00,000)]

Rs. 2,40,000

Cr. Rs. 10,00,000

By Balance b/f By Profit & Loss A/c—Premium Paid on Redemption Adjusted [20% of (10,00,000 – 8,00,000)]

To Balance c/f

8,00,000 10,40,000

40,000

10,40,000

7. Analysis of other non-current assets and liabilities Opening Balance Rs. 5,00,000 15,00,000

Assets/Liabilities Bank Loan Land & Building

Closing Balance Rs. 6,00,000 14,00,000

Increase/ Decrease Rs. (+)1,00,000 (−)1,00,000

Analysis Fresh Loan Taken Depreciation for the Year

8. Dr. To Non-Current & Non-Operating items charged: Transfer to General Reserve3 Proposed Dividend4 Interim Dividend5 Loss on Sale of Machinery1 Depreciation on Plant & Machinery1 Depreciation on Land & Building7 Premium Paid on Redemption Debenture6 To Balance c/f

Adjusted Profit & Loss A/c Rs. By Balance b/f 33,000 3,60,000 1,20,000 20,000 2,80,000 1,00,000

By Net Fund Flow from Operation (Bal. fig.)

40,000 3,60,000 13,13,000

Cr. Rs. 2,50,000

10,63,000

13,13,000

Problem 19 From the following Balance Sheets of Sneha Ltd as on 31 March 2003 and 31 March 2004, prepare a statement of sources and applications of Fund and a schedule of changes in Working Capital for the year that is ending on 31 March 2004:

Modified Date: Thu, Jul 01, 2010 02:41:25 PM

Output Date: Tue, Jul 06, 2010 11:50:19 AM

REV-II

Project: Management Accounting_Debarshi Bhattacharyya ACE Pro India Pvt. Ltd. File: X:\Pearson\Management Accounting_Debarshi Bhattacharyya\MAIN\M05\LAYOUT_M05\M05_DEBA_ISBN_EN_SE_C05_Part-2.indd

391

FUND FLOW ANALYSIS

Balance Sheets Liabilities Equity Share Capital Profit & Loss A/c 10% Debentures Creditors Bills Payable Provision for Tax Dividend Payable

31 March 2003 Rs. 13,00,000 4,90,100 16,25,000 9,00,000 42,500 2,60,000 –

31 March 2004 Rs. 16,90,000 8,77,500 13,00,000 10,00,000 1,70,000 9,75,000 42,250

46,17,600

60,54,750

Assets Goodwill Building Machinery Non-Trade Investments Debtors Stock Cash Prepaid Expenses Debenture Discount

31 March 2003 Rs. 65,000 11,70,000 16,18,500 5,07,000 4,16,000 5,07,000 2,60,000 42,250 31,850 46,17,600

31 March 2004 Rs. 42,500 11,37,500 21,38,500 3,93,250 11,70,000 7,99,500 2,92,500 52,000 29,000 60,54,750

Building Rs. 4,87,500 5,20,000 32,500

Machinery Rs. 15,92,500 15,66,500 1,36,500

i. The following additional information is given:

Accumulated Depreciation on 31 March 2003 Accumulated Depreciation on 31 March 2004 Depreciation for 2003–2004

ii. Profit & Loss A/c for 2003–04 is as follows:

Add:

Balance as on 31 March 2003 Profit for 2003–04

Less:

Dividend

Rs. 4,90,100 4,71,900 9,62,000 84,500 8,77,500

iii. During 2003–04, the machinery costing Rs. 2,92,500 was sold for Rs. 97,500. iv. Investment which were sold for Rs. 1,17,000 had cost Rs. 97,500. v. Provision for taxation and dividend are to be taken as non-current liabilities. [C.A. (Inter)—November 2004] Solution Books of Sneha Ltd i. Schedule of changes in Working Capital for the year that ended on 31 March 2004 Elements of Working Capital Current Assets: Stock Debtors Prepaid Expenses Cash Less:

Current Liabilities: Creditors Bills Payable Working Capital Increase in Working Capital

Modified Date: Thu, Jul 01, 2010 02:41:25 PM

Output Date: Tue, Jul 06, 2010 11:50:19 AM

As on 31 March 2003 Rs.

As on 31 March 2004 Rs.

5,07,000 4,16,000 42,250 2,60,000 12,25,250

7,99,500 11,70,000 52,000 2,92,500 23,14,000

(9,00,000) (42,500) 2,82,750

(10,00,000) (1,70,000) 11,44,000 8,61,250

REV-II

Project: Management Accounting_Debarshi Bhattacharyya ACE Pro India Pvt. Ltd. File: X:\Pearson\Management Accounting_Debarshi Bhattacharyya\MAIN\M05\LAYOUT_M05\M05_DEBA_ISBN_EN_SE_C05_Part-2.indd

392

MANAGEMENT ACCOUNTING

ii. Fund Flow Statement for the year that ended on 31 March 2004 Sources of Fund Proceeds from Sale of Machinery Proceeds from Sale of Investment New Issue of Equity Shares6 Net Fund Flow from Operation7

Rs. 97,500 1,17,000 3,90,000 16,70,500

Applications of Fund Purchase of New Machinery1 Payment of Dividend4 Payment of Tax5 Redemption of Debentures6 Increase in Working Capital (Bal. fig.)

22,75,000

Rs. 7,86,500 42,250 2,60,000 3,25,000 8,61,250 22,75,000

Working Notes 1. Dr. To Balance b/f (16,18,500 + 15,92,500) To Bank—Purchase of New Machinery (Bal. fig.)

Machinery (at cost) Account Rs. 32,11,000 By Bank—Sale Proceeds of old Machinery 7,86,500 By Provision for Depreciation A/c—Accumulated Depreciation on Machinery Sold By Profit & Loss A/c—Loss on Sale of Old Machinery [(2,92,500 – 1,62,500) – 97,500] By Balance c/f (21,38,500 + 15,66,500) 39,97,500

Cr. Rs. 97,500 1,62,500 32,500 37,05,000 39,97,500

2. Provision for Depreciation on Machinery Account Rs. To Machinery A/c—Accumulated By Balance b/f Depreciation on Machinery Sold 1,62,500 (Bal. fig.) To Balance c/f 15,66,500 By Profit & Loss A/c—Depreciation for the year 17,29,000

Dr.

Cr. Rs. 15,92,500

1,36,500 17,29,000

3. Non-trade Investment Account Rs. To Balance b/f 5,07,000 By Bank—Sale Proceeds of Investment To Profit & Loss A/c—Profit on Sale of By Profit & Loss A/c—written off (Bal. fig.) Investment (1,17,000 – 97,500) 19,500 By Balance c/f 5,26,500 Dr.

Cr. Rs. 1,17,000 16,250 3,93,250 5,26,500

4. Dr. To Bank—Dividend Paid in 2003–04 (Bal. fig.)

Dividend Payable Account Rs. 42,250 By Balance b/f By Profit & Loss Appropriation A/c —Dividend Declared in 2003–04

To Balance c/f

Modified Date: Thu, Jul 01, 2010 02:41:25 PM

42,250 84,500

Output Date: Tue, Jul 06, 2010 11:50:19 AM

Cr. Rs.

84,500 84,500

REV-II

Project: Management Accounting_Debarshi Bhattacharyya ACE Pro India Pvt. Ltd. File: X:\Pearson\Management Accounting_Debarshi Bhattacharyya\MAIN\M05\LAYOUT_M05\M05_DEBA_ISBN_EN_SE_C05_Part-2.indd

FUND FLOW ANALYSIS

393

5. Provision for Tax Account Rs. 2,60,000 By Balance b/f By Profit & Loss A/c—Provision made in 2003–04 9,75,000 12,35,000

Dr. To Bank—Tax Paid in 2003–04

To Balance c/f

Cr. Rs. 2,60,000 9,75,000

12,35,000

6. Analysis of other non-Current Assets and liabilities Assets/Liabilities Equity Share Capital 10% Debentures Building Goodwill Debenture Discount

Opening Balance Rs. 13,00,000 16,25,000 11,70,000 65,000 31,850

Closing Balance Rs. 16,90,000 13,00,000 11,37,500 42,500 29,000

Increase/ Decrease Rs. (+)3,90,000 (−)3,25,000 (−)32,500 (−)22,500 (−)2,850

Analysis New Issue of Shares Redemption of Debentures Depreciation for the year Written off against Profit & Loss A/c Written off against Profit & Loss A/c

7. Dr. To Non-Current & Non-Operating items charged: Investment written off3 Dividend Payable4 Provision for Tax5 Loss on Sale of Machinery1 Depreciation on Machinery2 Depreciation on Building6 Goodwill written off6 Debenture Discount written off6 To Balance c/f

Adjusted Profit & Loss A/c Rs. By Balance b/f 16,250 84,500 9,75,000 32,500 1,36,500 32,500 22,500 2,850 8,77,500 21,80,100

Cr. Rs. 4,90,100

By Profit on Sale of Investment3

By Net Fund Flow from Operation (Bal. fig.)

19,500

16,70,500

21,80,100

Problem 20 From the following summarized Balance Sheets of Sunshine Ltd and other relevant information, prepare a Fund Flow Statement and a statement showing changes in Working Capital for the year that ended on 31 March 2010: Liabilities Share Capital Securities premium General Reserve Capital Reserve Profit & Loss A/c 8% Debentures Sundry Creditors Outstanding Expenses Provision for Taxation Proposed Dividend

31 March 2009 (Rs.) 5,00,000 45,000 1,00,000 – 70,000 1,00,000 73,000 10,200 35,000 50,000 9,83,000

Modified Date: Thu, Jul 01, 2010 02:41:25 PM

31 March 2010 (Rs.) 7,00,000 55,000 45,000 15,000 67,500 85,000 61,100 12,700 45,000 70,000 11,56,300

Assets Land & Building Plant & Machinery Stock Sundry Debtors Cash at Bank Prepaid Expenses

Output Date: Tue, Jul 06, 2010 11:50:19 AM

31 March 2009 (Rs.) 5,00,000 2,05,000 98,000 1,02,000 62,500 15,700

31 March 2010 (Rs.) 6,50,000 2,15,500 95,000 1,18,200 68,700 8,900

9,83,000

11,56,300

REV-II

Project: Management Accounting_Debarshi Bhattacharyya ACE Pro India Pvt. Ltd. File: X:\Pearson\Management Accounting_Debarshi Bhattacharyya\MAIN\M05\LAYOUT_M05\M05_DEBA_ISBN_EN_SE_C05_Part-2.indd

394

MANAGEMENT ACCOUNTING

Additional information: i. A plant costing Rs. 50,000 (WDV – Rs. 35,200) has been sold during 2009–10 for Rs. 40,000. Profit on sale has been transferred to capital reserve account. ii. A piece of land has been sold for Rs. 70,000 during the year 2009–10. The profit on sale of such land has also been transferred to capital reserve account. iii. Balance of capital reserve on 31 March 2010 consists of profit on sale of plant and land only. iv. During the year 2009–10, the bonus share of Rs. 1,00,000 has been issued out of general reserve. v. Debentures were redeemed at a premium of 10%. Premium on redemption has been transferred to Profit & Loss A/c. vi. Amount appropriated during 2009–10: for Proposed Dividend—Rs. 70,000 and for Taxation— Rs. 40,000. vii. Depreciation provided during 2009–10: on Plant and Machinery—Rs. 32,800 and on Building— Rs. 22,500 [B.Com. (Hons), Calcutta University—Adapted] Solution Books of Sunshine Ltd Fund Flow Statement for the year that ended on 31 March 2010 Sources of Fund New Issue of Equity Shares1 Securities Premium Received6 Sale Proceeds of Land3 Sale Proceeds of Plant4

Rs. 1,00,000 10,000 70,000 40,000

Net Fund Flow from Operation11

2,09,300

Applications of Fund Purchase of Land & Building3 Purchase of Plant & Machinery4 Redemption of Debentures7 Premium Paid on Redemption of debentures8 Payment of Proposed Dividend10 Payment of Tax9 Increase in Working Capital (Bal. fig.)

4,29,300

Rs. 2,32,300 78,500 15,000 1,500 50,000 30,000 22,000 4,29,300

Statement showing changes in Working capital for the year that ended on 31 March 2010 Elements of Working Capital Current Assets: Stock Debtors Cash PrePaid Expenses Less:

Current Liabilities: Sundry Creditors Outstanding Expenses Working Capital Increase in Working Capital

Modified Date: Thu, Jul 01, 2010 02:41:25 PM

Output Date: Tue, Jul 06, 2010 11:50:19 AM

As on 31 March 2009 Rs.

As on 31 March 2010 Rs.

98,000 1,02,000 62,500 15,700 2,78,200

95,000 1,18,200 68,700 8,900 2,90,800

(73,000) (10,200) 1,95,000

(61,100) (12,700) 2,17,000 22,000

REV-II

Project: Management Accounting_Debarshi Bhattacharyya ACE Pro India Pvt. Ltd. File: X:\Pearson\Management Accounting_Debarshi Bhattacharyya\MAIN\M05\LAYOUT_M05\M05_DEBA_ISBN_EN_SE_C05_Part-2.indd

395

FUND FLOW ANALYSIS

Working Notes 1. Equity Share Capital Account Rs. By Balance b/f By General Reserve—Bonus Shares Issued By Bank—New Issue of Shares for Cash (Bal. fig.) 7,00,000 7,00,000

Dr.

To Balance c/f

Cr. Rs. 5,00,000 1,00,000 1,00,000

7,00,000

2. Dr. To Share Capital—Bonus Shares Issued

To Balance c/f

General Reserve Account Rs. 1,00,000 By Balance b/f By Profit & Loss A/c—Transfer during 2009–10 (Bal. fig.) 45,000 1,45,000

Cr. Rs. 1,00,000 45,000

1,45,000

3. Dr. To Balance b/f To Capital Reserve—Profit on Sale of Land To Bank—Purchase of Land & Building (Bal. fig.)

Land & Building Account Rs. 5,00,000 By Bank—Sale Proceeds of Land 10,200 By Profit & Loss A/c—Depreciation 2,32,300

By Balance c/f

7,42,500

Cr. Rs. 70,000 22,500 6,50,000 7,42,500

4. Dr. To Balance b/f To Capital Reserve—Profit on Sale of Plant (40,000 – 35,200) To Bank—Purchase of Plant & Machinery (Bal. fig.)

Plant & Machinery Account Rs. 2,05,000 By Bank—Sale Proceeds of Plant By Profit & Loss A/c—Depreciation 4,800 By Balance c/f 78,500 2,88,300

Cr. Rs. 40,000 32,800 2,15,500 2,88,300

5. Dr.

To Balance c/f

Modified Date: Thu, Jul 01, 2010 02:41:25 PM

Capital Reserve Account Rs. By Balance b/f By Land & Building A/c—Profit on Sale of Land By Plant & Machinery A/c—Profit on Sale of Plant 15,000 15,000

Output Date: Tue, Jul 06, 2010 11:50:19 AM

Cr. Rs. Nil 10,200 4,800 15,000

REV-II

Project: Management Accounting_Debarshi Bhattacharyya ACE Pro India Pvt. Ltd. File: X:\Pearson\Management Accounting_Debarshi Bhattacharyya\MAIN\M05\LAYOUT_M05\M05_DEBA_ISBN_EN_SE_C05_Part-2.indd

396

MANAGEMENT ACCOUNTING

6. Dr.

To Balance c/f

Securities Premium Account Rs. By Balance b/f By Bank—Premium Received on New Issue of Equity Shares 55,000 55,000

Cr. Rs. 45,000 10,000 55,000

7. Dr. To Bank—Debentures Redeemed (Bal. fig.) To Balance c/f

8% Debenture Account Rs. 15,000 By Balance b/f 85,000 1,00,000

Cr. Rs. 1,00,000

1,00,000

8. Dr.

Premium on Redemption of Debenture Account Rs. To Bank—Premium Paid on By Profit & Loss A/c—Premium Paid Redemption (10% on Rs. 15,000) 1,500 written off 1,500

Cr. Rs. 1,500 1,500

9. Dr. To Bank—Tax for 2008–09 Paid in 2009–10

To Balance c/f

Provision for Taxation Account Rs. By Balance b/f 30,000 By Profit & Loss A/c—Provision Made in 2009–10 (Bal. fig.) 45,000 75,000

Cr. Rs. 35,000 40,000

75,000

10. Dr. To Bank—Proposed Dividend for 2008–09 Paid in 2009–10

To Balance c/f

Proposed Dividend Account Rs. By Balance b/f 50,000 By Profit & Loss Appropriation A/c —Dividend Proposed for 2009–10 70,000 (Bal. fig.) 1,20,000

Cr. Rs. 50,000 70,000

1,20,000

11. Dr. To Non-Current & Non-Operating items charged: Transfer to General Reserve2 Proposed Dividend10 Provision for Taxation9

Adjusted Profit & Loss A/c Rs. By Balance b/f

Cr. Rs. 70,000

45,000 70,000 40,000 (Continued)

Modified Date: Thu, Jul 01, 2010 02:41:25 PM

Output Date: Tue, Jul 06, 2010 11:50:19 AM

REV-II

Project: Management Accounting_Debarshi Bhattacharyya ACE Pro India Pvt. Ltd. File: X:\Pearson\Management Accounting_Debarshi Bhattacharyya\MAIN\M05\LAYOUT_M05\M05_DEBA_ISBN_EN_SE_C05_Part-2.indd

FUND FLOW ANALYSIS

Dr. Premium on redemption of debentures8 Depreciation on Land & Building3 Depreciation on Plant & Machinery4 To Balance c/f

Adjusted Profit & Loss A/c Rs. By Net Fund Flow from Operation 1,500 (Bal. fig.) 22,500 32,800 67,500 2,79,300

397

Cr. Rs.

2,09,300

2,79,300

Problem 21 Following were the Balance Sheets of Labanyalata Ltd as on 31 December 2008 and 31 December 2009: Liabilities Equity Share Capital Profit & Loss A/c General Reserve 10% Debentures Creditors Proposed Dividend Unclaimed Dividend

2008 Rs. 40,000 2,000 8,000 10,000 43,000 8,000 2,000 1,13,000

2009 Rs. 49,000 14,000 17,000 20,000 40,000 10,000 5,000 1,55,000

2008 Rs. 3,000 40,000 20,000 25,000 20,000 5,000

Assets Goodwill Plant & Machinery Investment Stock Cash & Bank Preliminary Expenses Discount on Issue of Shares

1,13,000

2009 Rs. 4,000 62,000 30,000 28,000 30,000 − 1,000 1,55,000

Additional information: i. Depreciation on machinery for the year 2009 amounted to Rs. 7,000. ii. In 2009, the company acquired some assets and liabilities from Z Ltd for Rs. 13,000 and paid the purchase consideration by fully-paid equity shares of the company of Rs. 9,000 and the balance paid in cash. Assets and liabilities acquired from Z Ltd were: Goodwill—Rs. 3,000; Machinery—Rs. 8,000; Stock—Rs. 7,000 and Creditors—Rs. 5,000. iii. Dividend on investment of Rs. 5,000 received during 2009, which included pre-acquisition dividend of Rs. 2,000. iv. Debentures were issued at a discount of 10% and debenture interest of Rs. 1,200 was paid during 2009. v. Stock on 31 December 2008 of Rs. 25,000 was valued at 20% below cost, but stock on 31 December 2009 of Rs. 28,000 was correctly valued at cost. vi. Unclaimed dividend as on 31 December 2008 was fully claimed by respective shareholders during 2009 and was duly paid off. vii. In addition, an interim dividend of Rs. 9,000 was paid during 2009. Prepare a statement showing changes in Working Capital as on 31 December 2009 and a Fund Flow Statement for the year that ended on 31 December 2009. Solution Books of Labanyalata Ltd Schedule of changes in Working Capital for the year that ended on 31 December 2009 Elements of Working Capital Current Assets: Stock (25,000 + 6,65012) Cash & Bank

As on 31 December 2008 Rs. 31,250 20,000 51,250

As on 31 December 2009 Rs. 28,000 30,000 58,000 (Continued)

Modified Date: Thu, Jul 01, 2010 02:41:25 PM

Output Date: Tue, Jul 06, 2010 11:50:19 AM

REV-II

Project: Management Accounting_Debarshi Bhattacharyya ACE Pro India Pvt. Ltd. File: X:\Pearson\Management Accounting_Debarshi Bhattacharyya\MAIN\M05\LAYOUT_M05\M05_DEBA_ISBN_EN_SE_C05_Part-2.indd

398

MANAGEMENT ACCOUNTING

Elements of Working Capital Less:

Current Liabilities: Creditors Working Capital Increase in Working Capital

As on 31 December 2008 Rs.

As on 31 December 2009 Rs.

(43,000) 8,250

(40,000) 18,000 9,750

Fund Flow Statement for the year that ended on 31 December 2009 Sources of Fund Total Dividend from Investment Received (2,000 + 3,000) 7

Rs. 5,000

9,000 7,000

New Issue of Debentures Stock Taken Over from Z Ltd1 Net Fund Flow from Operation12

46,950

Applications of Fund Purchase of new Machinery2

Rs. 21,000

Further Investment Made3 Payment of Debenture Interest6 Paid to Z Ltd in consideration of Net Assets taken over1 Payment of Total Dividend (3,0008 + 2,0009 + 9,00010) Creditors taken over from Z Ltd1 Increase in Working Capital (Bal. fig.)

12,000 1,200

67,950

5,000 14,000 5,000 9,750 67,950

Working Notes 1. Treatment of Assets and Liabilities acquired from Z Ltd Rs. Sundry Assets taken over: Goodwill Machinery Stock Less:

Sundry Liabilities taken over: Creditors Net Assets taken over

3,000 8,000 7,000 18,000 5,000 13,000

This Rs. 13,000 is discharged by the company partly by issue of equity shares of Rs. 9,000 and the balance in cash. Here, the balances of Equity Share Capital as on 31 December 2008 and 31 December 2009 are Rs. 40,000 and Rs. 49,000, respectively. Therefore, the equity shares of Rs. 9,000 (those issued to Z Ltd against the net assets taken over) only have been issued during the year 2009. Again, the balances of discount, on issue of shares as on 31 December 2008 and 31 December 2009, are nil and Rs. 1,000 respectively. Therefore, this discount of Rs. 1,000 has been allowed while issuing the equity shares of Rs. 9,000 to Z Ltd during the year 2009.

Net assets taken over from Z Ltd Purchase consideration discharged by: Issue of Equity Shares at Discount (Rs. 9,000 – Rs. 1,000) Rest Amount Paid in Cash

Rs. 13,000 8,000 5,000 (Continued)

Modified Date: Thu, Jul 01, 2010 02:41:25 PM

Output Date: Tue, Jul 06, 2010 11:50:19 AM

REV-II

Project: Management Accounting_Debarshi Bhattacharyya ACE Pro India Pvt. Ltd. File: X:\Pearson\Management Accounting_Debarshi Bhattacharyya\MAIN\M05\LAYOUT_M05\M05_DEBA_ISBN_EN_SE_C05_Part-2.indd

399

FUND FLOW ANALYSIS

Dr.

Z Ltd Account Rs. 5,000 By Goodwill 9,000 By Machinery 5,000 By Stock By Discount on Issue of Shares 19,000

To Creditors To Equity Share Capital To Bank (Bal. fig.)

Cr. Rs. 3,000 8,000 7,000 1,000 19,000

2. Dr. To Balance b/f To Z Ltd A/c—Acquired To Bank—Purchase of New Machinery for Cash (Bal. fig.)

Plant & Machinery Account Rs. 40,000 By Profit & Loss A/c—Depreciation for 2009 8,000

Cr. Rs. 7,000

21,000 By Balance c/f 69,000

62,000 69,000

3. Dr. To Balance b/f To Bank—Further Investment made in 2009 (Bal. fig.)

Investment Account Rs. 20,000 By Bank—Pre-Acquisition Dividend received & credited By Balance c/f 12,000 32,000

Cr. Rs. 2,000 30,000 32,000

4. Dr.

(Post-Acquisition) Dividend on Investment Account Rs. To Profit & Loss A/c—Post-Acquisition By Bank—Post-Acquisition Dividend Dividend received & adjusted 3,000 received (5,000 – 2,000) 3,000

Cr. Rs. 3,000 3,000

5. Dr. To Balance b/f To Z Ltd—Acquired

Goodwill Account Rs. 3,000 By Profit & Loss A/c—Written off (Bal. fig.) 3,000 By Balance c/f 6,000

Cr. Rs. 2,000 4,000 6,000

6. Dr. To Bank—Debenture Interest paid

Debenture Interest Account Rs. 1,200 By Profit & Loss A/c—Debenture Interest adjusted 1,200

Cr. Rs. 1,200 1,200

7. Dr.

10% Debentures Account Rs. By Balance b/f By Bank—New Issue (Bal. fig.) (Rs. 10,000 – Rs. 1,000)

Cr. Rs. 10,000 9,000

(Continued)

Modified Date: Thu, Jul 01, 2010 02:41:25 PM

Output Date: Tue, Jul 06, 2010 11:50:19 AM

REV-II

Project: Management Accounting_Debarshi Bhattacharyya ACE Pro India Pvt. Ltd. File: X:\Pearson\Management Accounting_Debarshi Bhattacharyya\MAIN\M05\LAYOUT_M05\M05_DEBA_ISBN_EN_SE_C05_Part-2.indd

400

MANAGEMENT ACCOUNTING

Dr.

10% Debentures Account Rs. By Profit & Loss A/c: Discount on Issue Adjusted (10% on Rs. 10,000) 20,000 20,000

To Balance c/f

Cr. Rs. 1,000

20,000

8. Proposed Dividend Account Rs. 3,000 By Balance b/f

Dr. To Bank—Dividend for 2008 paid (8,000 – 5,000) To Unclaimed Dividend A/c—Dividend for 2008 not claimed To Balance c/f

5,000 10,000 18,000

Cr. Rs. 8,000

By Profit & Loss Appropriation A/c —Dividend Proposed in 2009

10,000 18,000

9. Dr. To Bank—Unclaimed Dividend as on 31 December 2008 paid

To Balance c/f

Unclaimed Dividend Account Rs. By Balance b/f 2,000 By Proposed Dividend A/c—Unclaimed Dividend for 2008 5,000 7,000

Cr. Rs. 2,000

5,000 7,000

10. Interim Dividend Account Rs. 9,000 By Profit & Loss Appropriation A/c —Interim Dividend adjusted 9,000

Dr. To Bank—Interim Dividend paid

Cr. Rs. 9,000 9,000

11. Analysis of other Non-Current Assets and Liabilities Assets/Liabilities Equity Share Capital General Reserve Preliminary Expense Discount on Issue of Shares

Opening Balance Rs. 40,000 8,000 5,000

Closing Balance Rs. 49,000 17,000 – 1,000

Increase/ Decrease Rs. (+) 9,000 (+) 9,000 (−) 5,000 (+) 1,000

Analysis New Issue to Z Ltd Transfer from Profit & Loss A/c Written off against Profit & Loss A/c Discount allowed on New Issue

12. Dr. To Non-Current & Non-Operating Items Charged: Transfer to General Reserve11 Proposed Dividend8 Interim Dividend10

Adjusted Profit & Loss A/c Rs. By Balance b/f 9,000

By Undervaluation of Opening Stock (20 ÷ 80 × 25,000)

Cr. Rs. 2,000 6,250

10,000 9,000 (Continued)

Modified Date: Thu, Jul 01, 2010 02:41:25 PM

Output Date: Tue, Jul 06, 2010 11:50:19 AM

REV-II

Project: Management Accounting_Debarshi Bhattacharyya ACE Pro India Pvt. Ltd. File: X:\Pearson\Management Accounting_Debarshi Bhattacharyya\MAIN\M05\LAYOUT_M05\M05_DEBA_ISBN_EN_SE_C05_Part-2.indd

FUND FLOW ANALYSIS

Dr. Debenture Interest6 Discount on Issue of Debentures7 Depreciation on Plant & Machinery2 Goodwill written off5 Preliminary Expense written off11 To Balance c/f

Adjusted Profit & Loss A/c Rs. 1,200 By Dividend (Post-Acquisition) Received on Investment4 1,000 7,000 2,000 By Net Fund Flow from Operation 5,000 (Bal. fig.) 14,000 58,200

401

Cr. Rs. 3,000

46,950 58,200

Problem 22 From the following details provided as on 31 December 2008 and 31 December 2009, prepare statements showing (a) changes in Working Capital during the year 2009 and (b) a statement of sources and applications of Funds during the same period.

Less: Less: Add:

Less:

11% Preference Shares of Rs. 100 each fully called up Calls-in-Arrear (Final call of Rs. 10 per share) Equity Shares of Rs. 10 each fully called up Calls-in-Arrear (Final Call of Rs. 2.50 each) Forfeited Shares A/c: Reserves & Surplus: Capital Reserve Profit & Loss A/c 15% Debentures Fixed Assets (at Cost) Depreciation to Date Investment at Cost Current Assets: Stock Debtors Cash at Bank

31 December 2008 Rs. Rs. 2,00,000 8,000 1,92,000 6,00,000 25,000 5,75,000

31 December 2009 Rs. Rs. 2,00,000 2,000 1,98,000 7,40,000 – 15,000 7,55,000

20,000 1,20,000 5,00,000 14,07,000

1,20,000 1,50,000 3,50,000 15,73,000

40,00,000 35,00,000

5,00,000 2,00,000

4,50,000 8,00,000 50,000

41,00,000 35,20,000

8,00,000 7,50,000 56,000 13,00,000 20,00,000

Less:

Current Liabilities: Sundry Creditors Proposed Dividend

5,80,000 50,000

4,93,000 1,00,000

16,06,000 22,36,000 5,03,000 1,60,000

5,93,000 14,07,000

6,63,000 15,73,000

Stocks as on 31 December 2008 were valued at 10% below the cost, on 31 December 2009 at the cost. Equity shares at default on 31 December 2009 were forfeited. Some of the Forfeited Shares were reissued and the profit on reissue was credited in the capital reserve A/c. The only other credit in capital reserve A/c was the profit on the sale of investments – Rs. 65,000. [M.Com., Calcutta University—Adapted]

Modified Date: Thu, Jul 01, 2010 02:41:25 PM

Output Date: Tue, Jul 06, 2010 11:50:19 AM

REV-II

Project: Management Accounting_Debarshi Bhattacharyya ACE Pro India Pvt. Ltd. File: X:\Pearson\Management Accounting_Debarshi Bhattacharyya\MAIN\M05\LAYOUT_M05\M05_DEBA_ISBN_EN_SE_C05_Part-2.indd

402

MANAGEMENT ACCOUNTING

Solution Books of Statement showing changes in Working Capital for the year that ended on 31 December 2009 Elements of Working Capital Current Assets: Stock (4,50,000 × 100 ÷ 90) Debtors Cash at Bank Less:

Current Liabilities: Sundry Creditors Working Capital Increase in Working Capital

As on 31 December 2008 Rs.

As on 31 December 2009 Rs.

5,00,000 8,00,000 50,000 13,50,000

8,00,000 7,50,000 56,000 16,06,000

(4,93,000) 8,57,000

(5,03,000) 11,03,000 2,46,000

Fund Flow Statement for the year that ended on 31 December 2009 Sources of Fund Proceeds Received from Reissue of Forfeited Shares4 New Issue of Equity Shares7 Calls-in-arrear Received from preference Shares7 Sale Proceeds of Investment5 Net Fund Flow from Operation12

Rs. 55,000 1,60,000 6,000 2,15,000 1,60,000 5,96,000

Applications of Fund Purchase of Fixed Assets9

Rs. 1,00,000

Redemption of Debentures8 Payment of Proposed Dividend11

1,50,000 1,00,000

Increase in Working Capital (Bal. fig.)

2,46,000 5,96,000

Working Notes 1. Calculation of equity shares forfeited, reissue of Forfeited Shares and transfer of profit on reissue. Final call @ Rs. 2.50 per share due from = Rs. 25,000 ÷ Rs. 2.50 = 10,000 equity shares. Therefore, 10,000 equity shares were forfeited. Entry for forfeiture of equity shares: Equity Share Capital A/c Dr. 1,00,000 (10,000 × Rs. 10) To Calls-in-arrear A/c (10,000 × Rs. 2.50) To Forfeited Shares A/c (10,000 × Rs. 7.50)

25,000 75,000

Amount transferred to Capital Reserve as a profit on reissue: Rs.

Less:

Balance of Capital Reserve on 31 December 2009 Balance of Capital Reserve on 31 December 2008 Transfer of profit during the year 2009 Profit on sale of investment transferred Profit on reissue transferred to Capital Reserve

1,20,000 20,000 1,00,000 65,000 35,000

Discount on reissue adjusted against Forfeited Shares A/c: Rs. Total amount forfeited from 10,000 Equity Shares Less: Amount transferred to Capital Reserve as a profit on reissue

75,000 35,000 40,000 (Continued)

Modified Date: Thu, Jul 01, 2010 02:41:25 PM

Output Date: Tue, Jul 06, 2010 11:50:19 AM

REV-II

Project: Management Accounting_Debarshi Bhattacharyya ACE Pro India Pvt. Ltd. File: X:\Pearson\Management Accounting_Debarshi Bhattacharyya\MAIN\M05\LAYOUT_M05\M05_DEBA_ISBN_EN_SE_C05_Part-2.indd

403

FUND FLOW ANALYSIS

Less:

Amount kept in Forfeited Shares A/c as a provision for discount on non-reissued Forfeited Shares Discount on reissue adjusted against Forfeited Shares A/c

15,000 25,000

No. of Forfeited Shares reissued and not yet reissued: Total amount forfeited from 10,000 equity shares = 75,000 Amount forfeited per equity share = Rs. 75,000 ÷ 10,000 = Rs. 7.50 Maximum discount may be offered for reissue of non-reissued Forfeited Shares = Rs. 7.50 per share Again, the amount kept in Forfeited Shares A/c as a provision for discount on non-reissued Forfeited Shares = Rs. 15,000 No. of Forfeited Shares not yet reissued = Rs. 15,000 ÷ Rs. 7.50 = 2,000 shares No. of Forfeited Shares already reissued = 10,000 – 2,000 = 8,000 shares Adjusting Journal Entries for reissue of Forfeited Shares and transfer of profit on reissue: (a) For reissue of Forfeited Shares: Bank A/c ................................................................. Dr. Forfeited Shares A/c .......................................... Dr. To Equity Share Capital A/c (8,000 × Rs. 10) (b) For transferring profit on reissue: Forfeited Shares A/c .......................................... Dr. To Capital Reserve A/c

55,000 25,000 80,000

35,000 35,000

2. Dr.

Calls-in-arrear (on Equity Shares) Account Rs. To Balance b/f (10,000 eq. Shares @ Rs. 2.50) 25,000 By Equity Share Capital A/c 25,000

Cr. Rs. 25,000 25,000

3. Dr.

Forfeited Shares Account Rs. To Equity Share Capital A/c 25,000 By Equity Share Capital A/c—Amount —Discount on Reissue Adjusted (Bal. fig.) Forfeited [(10,000 × Rs. 10) – To Capital Reserve A/c—Profit on Reissue 35,000 Rs. 25,000] To Balance c/f 15,000 75,000

Cr. Rs. 75,000

75,000

4. Dr.

Equity Share Capital Account Rs. To Forfeited Shares A/c—Amount Forfeited 75,000 By Balance b/f To Calls-in-arrear A/c 25,000 By Bank—Proceeds received from reissue of Shares By Forfeited Shares A/c—Discount on reissue adjusted By Bank—New Issue of Shares for Cash (Bal. fig.) To Balance c/f 7,40,000 8,40,000

Cr. Rs. 6,00,000 55,000 25,000 1,00,000

8,40,000 (Continued)

Modified Date: Thu, Jul 01, 2010 02:41:25 PM

Output Date: Tue, Jul 06, 2010 11:50:19 AM

REV-II

Project: Management Accounting_Debarshi Bhattacharyya ACE Pro India Pvt. Ltd. File: X:\Pearson\Management Accounting_Debarshi Bhattacharyya\MAIN\M05\LAYOUT_M05\M05_DEBA_ISBN_EN_SE_C05_Part-2.indd

404

MANAGEMENT ACCOUNTING

5. Dr. To Balance b/f To Capital Reserve—Profit on Sale of Investment

Investment (at cost) Account Rs. 2,00,000 By Bank—Sale Proceeds of Investment (Bal. fig.) By Balance c/f 65,000 2,65,000

Cr. Rs. 2,15,000 50,000 2,65,000

6. Dr.

Capital Reserve Account Rs. By Balance b/f By Investment A/c—Profit on Sale By Forfeited Shares A/c—Profit on reissue transferred (Bal. fig.) 1,20,000 1,20,000

To Balance c/f

Cr. Rs. 20,000 65,000 35,000 1,20,000

7. Dr.

Calls-in-Arrear (on preference shares) Account Rs. 8,000 By Bank A/c—Calls-in-arrear partly received (Bal. fig.) By Balance c/f 8,000

To Balance b/f

Cr. Rs. 6,000 2,000 8,000

8. Dr.

15% Debentures Account Rs. 1,50,000 By Balance b/f 3,50,000 5,00,000

To Bank—Redeemed (Bal. fig.) To Balance c/f

9. Dr. To Balance b/f To Bank—Purchase of New Fixed Assets (Bal. fig.)

Fixed Assets (at cost) Account Rs. 40,00,000 1,00,000 By Balance c/f 41,00,000

Cr. Rs. 5,00,000 5,00,000

Cr. Rs. 41,00,000 41,00,000

10. Dr.

To Balance c/f

Accumulated Depreciation on Fixed Assets Account Rs. By Balance b/f By Profit & Loss A/c—Depreciation for the year (Bal. fig.) 35,20,000 35,20,000

Cr. Rs. 35,00,000 20,000

35,20,000 (Continued)

Modified Date: Thu, Jul 01, 2010 02:41:25 PM

Output Date: Tue, Jul 06, 2010 11:50:19 AM

REV-II

Project: Management Accounting_Debarshi Bhattacharyya ACE Pro India Pvt. Ltd. File: X:\Pearson\Management Accounting_Debarshi Bhattacharyya\MAIN\M05\LAYOUT_M05\M05_DEBA_ISBN_EN_SE_C05_Part-2.indd

405

FUND FLOW ANALYSIS

11. Dr.

Proposed Dividend Account Rs. To Bank—Proposed Dividend for 2008 paid By Balance b/f in 2009 1,00,000 By Profit & Loss Appropriation A/c— Dividend Proposed for 2009 To Balance c/f 1,60,000 2,60,000

Cr. Rs. 1,00,000

1,60,000 2,60,000

12. Dr. To Non-Current & Non-Operating Items charged: Depreciation on Fixed Assets10 Proposed Dividend11 To Balance c/f

Adjusted Profit & Loss A/c Rs. By Balance b/f 20,000 1,60,000 1,50,000 3,30,000

By Undervaluation of Opening Stock (10 ÷ 90 × 4,50,000) By Net Fund Flow from Operation (Bal. fig.)

Cr. Rs. 1,20,000 50,000 1,60,000 3,30,000

CHAPTER REVIEW SUMMARY  The concept of Fund is explained by different accountants and accounting bodies in different approaches such as cash and bank, Working Capital, all financial resources and so on. With reference to the preparation of Fund Flow Statement, the most preferred and accepted concept of Fund is Working Capital.  Flow of Fund means movement of or changes in Working Capital (i.e., current) items. It is identified by means of inward or outward movement of Current Assets and Current Liabilities.  A Fund Flow Statement is a summarized statement of movement of Fund (i.e., Working Capital) from different activities of a concern during an accounting period. It is prepared to locate various sources of Fund inflows into the business and also to identify various purposes of Fund outflows from the business during two consecutive balance sheet dates.  Fund Flow Statement acts as an important tool for financial analysis and shows the brief reasons for change in the Working Capital between two balance sheet dates.  Fund Flow Statement is not a basic Financial Statement, but is a supplementary statement. It does not disclose any new fact which is not reflected in the Income Statement and the Balance Sheet. Nowadays, Fund Flow Analysis is not that much relevant to the management for financial analysis as the Cash Flow Analysis.  Both sources and applications of Fund may be classified into two broad categories: Fund from operating activities and Fund from non-operating activities.  Fund from operation (or net Fund Flow from operation) refers to the difference between the inflows of Fund and outflows of Fund from the operating activities of the concern. It is the net Flow of Fund from operating activities of the concern.  Fund from operation (i.e., net Fund Flow from operational activities) may be computed under two different approaches, such as Direct Approach and Indirect Approach.  There are two opinions among the accountants as regards to the consideration whether depreciation is a source of Fund or not. As there is no movement of Fund for adjusting the depreciation, though it is an operating expense, yet it should not, generally, be regarded as a source (or a use) of Fund.  Income Statement and Balance Sheet are two basic Financial Statements whereas Fund Flow Statement is a supplementary statement which shows the changes in the financial position (i.e., Fund position) between two consecutive balance sheet dates.

Modified Date: Thu, Jul 01, 2010 02:41:25 PM

Output Date: Tue, Jul 06, 2010 11:50:19 AM

REV-II

Project: Management Accounting_Debarshi Bhattacharyya ACE Pro India Pvt. Ltd. File: X:\Pearson\Management Accounting_Debarshi Bhattacharyya\MAIN\M05\LAYOUT_M05\M05_DEBA_ISBN_EN_SE_C05_Part-2.indd

406

MANAGEMENT ACCOUNTING

CHAPTER REVIEW QUIZ 1. Classify the following items into: (a) Fund involving as well as operating items; (b) Fund involving but nonoperating items and (c) Non-Fund involving but operating items: (i) Purchase of machinery; (ii) Purchase of goods; (iii) Sale of long-term investments; (iv) Sale of goods; (v) Issue of equity shares for cash; (vi) Redemption of preference shares in cash; (vii) Receipt of interest on investment; (viii) Payment of interest on debentures; (ix) Payment of dividend; (x) Issue of debentures for cash; (xi) Repayment of long-term loan; (xii) Depreciation on Fixed Assets; (xiii) Payment of administration expenses; (xiv) Payment of wages; (xv) Sale of building; (xvi) Payment of salaries. Ans.: (a) (ii), (iv), (xiii), (xiv), (xvi); (b) (i), (iii), (v), (vi), (vii), (viii), (ix), (x), (xi), (xv); (c) (xii). 2. How would you deal with the following items (already considered in the Profit & Loss A/c) while calculating ‘Net Fund Flow from operation’ from the given figure of Net Profit earned during a year: a. Goodwill written off. b. Interest received from investment. c. Depreciation on Fixed Assets. d. Payment of interest on debentures. e. Payment of wages. f. Profit on sale of building. g. Credit sale of goods. h. Cash purchase of goods. i. Proposed dividend. j. Undervaluation of opening stock. k. Preliminary expense written off. l. Outstanding salaries. Ans.: To be added to the net profit: (a), (c), (d), (i), (k); To be deducted from the net profit: (b), (f), ( j); No adjustment required: (e), (g), (h), (l). 3. Choose the correct alternative from the following: a. Purchase of machinery results in: (i) inflow of Fund; (ii) outflow of Fund; (iii) has no effect on Fund Flow. b. Sale of patents: (i) causes inflow of Fund; (ii) causes outflow of Fund; (iii) has no effect on Fund Flow. c. Loss on sale of furniture charged in the Profit & Loss A/c: (i) increases Fund from operation; (ii) decreases Fund from operation; (iii) has no effect on Fund from operation. d. Depreciation on building: (i) causes inflow of Fund; (ii) causes outflow of Fund; (iii) has no effect on Fund Flow. e. Credit purchase of goods: (i) increases Fund from operation; (ii) decreases Fund from operation; (iii) has no effect on Fund from operation. f. Credit sale of goods: (i) increases Fund from operation; (ii) decreases Fund from operation; (iii) has no effect on Fund from operation. g. Interim dividend on shares charged to the Profit & Loss A/c: (i) increases Fund from operation; (ii) decreases Fund from operation; (iii) has no effect on Fund from operation. h. Issue of preference shares for cash causes: (i) inflow of Fund; (ii) outflow of Fund; (iii) has no effect on Fund Flow. i. Interest paid on loan charged to the Profit & Loss A/c: (i) increases Fund from operation; (ii) decreases Fund from operation; (iii) has no effect on Fund from operation. j. Provision for income tax charged to the Profit & Loss A/c: (i) increases Fund from operation; (ii) decreases Fund from operation; (iii) has no effect on Fund from operation. k. If the net profit of a business concern is Rs. 3,20,000 after crediting an interest on investment of Rs. 30,000 and after transferring Rs. 50,000 to general reserve, then Fund from the operation is equal to: (i) Rs. 3,00,000; (ii) Rs. 2,40,000; (iii) Rs. 3,40,000.

Modified Date: Thu, Jul 01, 2010 02:41:25 PM

Output Date: Tue, Jul 06, 2010 11:50:19 AM

REV-II

Project: Management Accounting_Debarshi Bhattacharyya ACE Pro India Pvt. Ltd. File: X:\Pearson\Management Accounting_Debarshi Bhattacharyya\MAIN\M05\LAYOUT_M05\M05_DEBA_ISBN_EN_SE_C05_Part-2.indd

FUND FLOW ANALYSIS

407

l. If the net profit of a business concern is Rs. 2,30,000, and depreciation and interest on debentures charged to the Profit & Loss A/c are Rs. 30,000 and Rs. 25,000 respectively, then the Fund from operation is equal to: (i) Rs. 1,75,000; (ii) Rs. 2,35,000; (iii) Rs. 2,25,000. Ans.: (a) (ii); (b) (ii); (c) (ii); (d) (iii); (e) (ii); (f) (i); (g) (ii); (h) (i); (i) (ii); ( j) (ii); (k) (iii); (l) (i). 4. State whether the following statements are true or false: a. Fund Flow Statement is a part of Financial Statements. b. Fund Flow Statement is a statement of sources and applications of Fund during a particular period of time. c. Fund Flow Statement is a substitute of the Income Statement. d. Difference between sources and applications of Fund may increase or decrease in the Working Capital. e. Cash Flow Statement presents a more complete picture than the Fund Flow Statement. f. ‘Fund from operation’ means the net Flow of Fund from the operating activities of a concern. Ans.: True: (a), (b), (d), (f); False: (c), (e). EXERCISE I. Theoretical Questions A. Short Answer Type Questions:

1. 2. 3. 4. 5. 6. 7. 8. 9.

What is meant by Fund? What is the generally accepted concept of Fund? What is Flow of Fund? What is Fund Flow Statement? What is Fund from Operation? Give three examples of source of Fund? Give three examples of use of Fund? Give the proforma of computation of Fund from operation under Direct Approach. Give the proforma of computation of Fund from operation under Indirect Approach.

B. Essay Type Questions:

1. 2. 3. 4. 5. 6.

Discuss the different concepts of Fund? What is the generally accepted concept of Fund? What do you mean by Flow of Fund? Give five examples each of inflows and outflows of Fund. What is Fund Flow Statement? Draw up a proforma of Fund Flow Statement. What are the importance and limitations of Fund Flow Statement? What are the different sources and applications of Fund? What do you mean by Fund from Operation? What are the different approaches of computation of Fund from operation? 7. Is depreciation a source of Fund? Discuss. 8. Compare between Income Statement, Balance Sheet and Fund Flow Statement. 9. Distinguish between Cash Flow Statement and Fund Flow Statement. II. Practical Problems A. Calculation of Fund Flow from Operation

1. From the following information, calculate the net Fund Flow from operation for the year that ended on 31 March 2010: Profit & Loss A/c for the year that ended on 31 March 2010 Dr. Particulars To Loss on Sale of Plant To Preliminary Expenses To Interest on Debentures To Depreciation

Rs. 30,000 12,000 25,000 80,000

Particulars By Gross Profit By Interest on Investment By Dividend Received By Profit on Sale of Building

Cr. Rs. 6,40,000 10,000 12,000 22,000 (Continued)

Modified Date: Thu, Jul 01, 2010 02:41:25 PM

Output Date: Tue, Jul 06, 2010 11:50:19 AM

REV-II

Project: Management Accounting_Debarshi Bhattacharyya ACE Pro India Pvt. Ltd. File: X:\Pearson\Management Accounting_Debarshi Bhattacharyya\MAIN\M05\LAYOUT_M05\M05_DEBA_ISBN_EN_SE_C05_Part-2.indd

408

MANAGEMENT ACCOUNTING

Profit & Loss A/c for the year that ended on 31 March 2010 Dr. Particulars To Goodwill written off To General Reserve To Provision for Tax To Proposed Dividend To Net Profit

Rs. 20,000 50,000 40,000 33,000 4,10,000 7,00,000

Particulars By Interest on Bank Deposit By Refund of Tax

Cr. Rs. 15,000 1,000

7,00,000

Ans.: Rs. 6,40,000. 2. From the following particulars, calculate the Fund from operations: Rs. 1,10,000 2,30,000

Profit & Loss A/c: as on 01 April 2009 as on 31 March 2010 Transactions during the year: Transfer to General Reserve Depreciation on Fixed Assets Underwriting Commission written off Interest received Interim Dividend Paid Sale of Fixed Assets (Book Value – Rs. 32,000) Provision for Tax

30,000 22,000 5,000 7,000 20,000 40,000 36,000

[B.Com. (Hons), Calcutta University—Adapted] Ans.: Rs. 2,18,000. 3. From the following information, calculate the net Fund Flow from operation of a company for the year that ended on 31 March 2010: 31 March 2009 Rs. 1,70,000 18,000

Balance in Profit & Loss A/c Cash at bank

31 March 2010 Rs. 2,30,000 35,000

While ascertaining the net profit for the year that ended on 31 March 2010, the following items were taken into the Profit & Loss A/c for the year: Rs. 35,000 28,000 12,000 15,000 14,000 24,000 30,000 35,000 19,000 23,000 15,000

Transfer to General Reserve Proposed Dividend Interim Dividend Goodwill written off Preliminary Expenses written off Interest on Debentures Depreciation on Fixed Assets Provision for Taxation Loss on Sale of Investment Profit on Sale of Machinery Income from Investment

Ans.: 2,34,000. 4. From the following information, calculate the net Fund Flow from operation of a concern for the year that ended on 31 March 2010: Rs. 1,70,000 5,30,000

Cash Sales for the year Credit Sales for the year

(Continued)

Modified Date: Thu, Jul 01, 2010 02:41:25 PM

Output Date: Tue, Jul 06, 2010 11:50:19 AM

REV-II

Project: Management Accounting_Debarshi Bhattacharyya ACE Pro India Pvt. Ltd. File: X:\Pearson\Management Accounting_Debarshi Bhattacharyya\MAIN\M05\LAYOUT_M05\M05_DEBA_ISBN_EN_SE_C05_Part-2.indd

409

FUND FLOW ANALYSIS

Rs. 3,80,000 1,10,000 2,80,000 2,32,000 75,000 15,000 36,000 40,000 22,000 8,000 32,000 30,000 34,000 22,000 27,000 32,000

Collection from Debtors during the year Cash Purchases for the year Credit Purchases for the year Payment to Creditors during the year Wages Paid during the year Outstanding Wages for the year Salaries Paid during the year Salaries for the year General Expenses Paid during the year Unpaid General Expenses for the year Depreciation on Fixed Assets for the year Proposed Dividend for the year Dividend Paid during the year Interest received on Investment during the year Payment of income tax during the year Provision for income tax for the year

Ans.: Rs. 1,50,000. 5. From the following information, calculate the Fund from operation of Tulip Ltd for the year that ended on 31 March 2010: Dr.

Profit & Loss A/c for the year that ended on 31 March 2010 Rs. Rs. 62,000 By Sales: Cash 60,000 Credit 2,23,000 2,83,000 By Closing Stock 62,000 By Interest from Investment 6,000 68,000

To Opening Stock To Purchases: Cash Credit To Wages: Paid Outstanding To Salaries: Paid Outstanding To Rent Paid To Depreciation on Assets To General Expenses To Interest on Debentures To Preliminary Expenses written off To Loss on Sale of Machinery To Capital Redemption Reserve

32,000 3,000

Cr. Rs.

Rs.

93,000 3,94,000 4,87,000 52,000 11,000

35,000 8,000 16,000 15,000 20,000 7,000 12,000 48,000 5,74,000

By Net Loss for the year

24,000 5,74,000

Ans.: Rs. 68,000. 6. From the following particulars, calculate the Fund from operation of Rini Ltd for the year that ended on 31 March 2010: Rs. Balance of Profit & Loss A/c: As on 31 March 2010 As on 31 March 2009

4,60,000 3,10,000 (Continued)

Modified Date: Thu, Jul 01, 2010 02:41:25 PM

Output Date: Tue, Jul 06, 2010 11:50:19 AM

REV-II

Project: Management Accounting_Debarshi Bhattacharyya ACE Pro India Pvt. Ltd. File: X:\Pearson\Management Accounting_Debarshi Bhattacharyya\MAIN\M05\LAYOUT_M05\M05_DEBA_ISBN_EN_SE_C05_Part-2.indd

410

MANAGEMENT ACCOUNTING

Rs. Appropriation of Profit for the year 2009–10: Transfer to General Reserve Proposed Dividend Interim Dividend Expenses and Losses for the year 2009–10: Interest on Debentures Depreciation on Fixed Assets Wages & Salaries Trade Expenses Provision for Taxation Goodwill written off Loss on Sale of Machinery Incomes and Gains for the year 2009–10: Profit on Sale of Furniture Income from Investment Expenses Paid during the year 2009–10: Interest on Debentures Wages & Salaries Income Tax

50,000 35,000 15,000 24,000 28,000 52,000 12,000 32,000 16,000 23,000 18,000 21,000 20,000 45,000 30,000

Ans.: Rs. 3,34,000. B. Preparation of Fund Flow Statement

7. The Balance Sheets of Tune-up Ltd were as follows: Liabilities Equity Share Capital General Reserve Profit & Loss A/c Creditors Bills Payable Proposed Dividend Provision for Taxation

2008 Rs. 4,50,000 40,000 30,000 55,000 20,000 42,000 40,000 6,77,000

2009 Rs. 5,00,000 70,000 48,000 83,000 16,000 50,000 50,000 8,17,000

Assets Goodwill Land & Building Plant Stock Debtors Bills Receivable Cash

2008 Rs. 1,15,000 2,00,000 80,000 77,000 1,60,000 20,000 25,000 6,77,000

2009 Rs. 90,000 1,70,000 2,00,000 1,09,000 2,00,000 30,000 18,000 8,17,000

Additional information: i. Depreciation of Rs. 15,000 and Rs. 25,000 have been charged on plant and land and building, respectively. ii. An interim dividend of Rs. 20,000 has been paid in 2009. iii. An income tax of Rs. 35,000 was paid during 2009. Prepare a Fund Flow Statement for the year 2009. Ans.: Increase in Working Capital—Rs. 51,000; Fund from operation—Rs. 2,33,000; Fund Flow Statement total—Rs. 2,83,000. 8. Balance Sheets of Raj & Joy Co. are given as follows: Liabilities Current Liabilities Loan from Raj Bank Loan Capital

Modified Date: Thu, Jul 01, 2010 02:41:25 PM

Year 2007 Rs. 1,20,000 – 1,80,000 2,40,000

Year 2008 Rs. 1,50,000 70,000 1,20,000 4,20,000

5,40,000

7,60,000

Assets Cash Debtors Stock Land Building Machinery

Output Date: Tue, Jul 06, 2010 11:50:19 AM

Year 2007 Rs. 50,000 50,000 60,000 1,00,000 1,20,000 1,60,000 5,40,000

Year 2008 Rs. 30,000 70,000 80,000 1,60,000 2,00,000 2,20,000 7,60,000

REV-II

Project: Management Accounting_Debarshi Bhattacharyya ACE Pro India Pvt. Ltd. File: X:\Pearson\Management Accounting_Debarshi Bhattacharyya\MAIN\M05\LAYOUT_M05\M05_DEBA_ISBN_EN_SE_C05_Part-2.indd

FUND FLOW ANALYSIS

411

During the year 2008, Raj & Joy introduced an additional capital of Rs. 50,000 and drew Rs. 70,000. Provision for depreciation on machinery: Opening balance—Rs. 60,000 and Closing balance—Rs. 90,000. No depreciation was provided on the other assets. The value of building was increased by Rs. 20,000 and the same was adjusted with the capital account. Prepare the Fund Flow Statement of Raj & Joy Co. for the year 2008. [B.Com. (Hons), Calcutta University—Adapted] Ans.: Decrease in Working Capital—Rs. 10,000; Net profit for 2008—Rs. 1,80,000; Fund from Operation— Rs. 2,10,000; Fund Flow Statement total—Rs. 3,40,000. 9. From the following information, prepare the Fund Flow Statement for the year that ended on 31 March 2009: Balance Sheets Liabilities

Share Capital Profit & Loss A/c Bank Loan Creditors Bills Payable

As on 31 March 2008 Rs. 2,00,000 40,000 1,00,000 50,000 40,000 4,30,000

As on 31 March 2009 Rs. 3,00,000 90,000 60,000 75,000 25,000 5,50,000

Assets

Land & Building Machinery Stock Debtors Cash

As on 31 March 2008 Rs. 1,20,000 1,60,000 60,000 50,000 40,000 4,30,000

As on 31 March 2009 Rs. 1,70,000 2,40,000 40,000 50,000 50,000 5,50,000

Additional information: i. Net profit for the year 2008–09 amounted to Rs. 50,000. ii. During the year 2008–09, a machine costing Rs. 50,000 (accumulated depreciation – Rs. 20,000) was sold for Rs. 25,000. The provision for depreciation against machinery as on 31 March 2008 was Rs. 30,000 and on 31 March 2009 was Rs. 70,000. [B.Com. (Hons), Calcutta University—Adapted] Ans.: Decrease in the Working Capital—Rs. 20,000; Fund from Operation—Rs. 1,15,000; Fund Flow Statement total— Rs. 2,60,000. 10. From the following Balance Sheets as on 31 December 2008 and 31 December 2009 and from the additional information of Kalyani Ltd, you are required to prepare a Fund Flow Statement for the year that ended on 31 December 2008: Liabilities

Share Capital Profit & Loss A/c Debentures Sundry Creditors Provision for Doubtful Debt

As on 31 December 2008 Rs. 1,82,000 13,040 40,000 14,360 1,000 2,50,400

As on 31 December 2009 Rs. 1,86,000 13,560 – 15,840 1,100

Assets

Goodwill Land Plant & Machinery Closing Stock Sundry Debtors Cash-in-hand

2,16,500

As on 31 December 2008 Rs. 10,000 40,000 1,00,000 69,200 19,200 12,000 2,50,400

As on 31 December 2009 Rs. 5,000 50,000 66,000 62,700 22,000 10,800 2,16,500

Additional information: i. Dividend of Rs. 5,000 was paid. ii. Provision for the tax made during the year—Rs. 9,000. iii. During the year, a machine costing Rs. 20,000 (accumulated depreciation—Rs. 6,000) was sold for Rs. 11,000.

Modified Date: Thu, Jul 01, 2010 02:41:25 PM

Output Date: Tue, Jul 06, 2010 11:50:19 AM

REV-II

Project: Management Accounting_Debarshi Bhattacharyya ACE Pro India Pvt. Ltd. File: X:\Pearson\Management Accounting_Debarshi Bhattacharyya\MAIN\M05\LAYOUT_M05\M05_DEBA_ISBN_EN_SE_C05_Part-2.indd

412

MANAGEMENT ACCOUNTING

iv.

The provision for depreciation against plant and machinery as on 31 December 2008 was Rs. 30,000 and 31 December 2009 was Rs. 44,000. [B.Com. (Hons), Kalyani University—Adapted] Ans.: Decrease in Working Capital—Rs. 6,480; Fund from Operation—Rs. 42,520; Fund Flow Statement total— Rs. 64,000. 11. The Balance Sheets of Tollywood Ltd as on 31 March 2008 and 31 March 2009 are as follows: Balance Sheets (Rs. in ’000) Liabilities Equity Share Capital General Reserve Profit & Loss A/c Secured Loan Sundry Creditors Bills Payable Outstanding Expenses Unpaid Dividend

As on 31 March 2008 350 20 40 – 30 50 10 10 510

As on 31 March 2009 400 – – 180 45 25 30 – 680

Assets Fixed Assets Stock Debtors Bills Receivable Investment Cash Profit & Loss A/c

As on 31 March 2008 210 90 60 50 70 30 –

As on 31 March 2009 320 140 44 75 40 31 30

510

680

Accumulated depreciation was Rs. 60,000 on 31 March 2008 and on 31 March 2009, it was Rs. 57,000. Machinery costing Rs. 1,70,000, having WDV of Rs. 80,000, was sold for Rs. 70,000. Depreciation on the fixed asset for the year is Rs. 17,000. Prepare a Fund Flow Statement for the year that ended on 31 March 2009. Ans.: Increase in Working Capital—Rs. 50,000; Fund from operation—Rs. 63,000; Fund Flow Statement total— Rs. 3,30,000. 12. Prepare a Fund Flow Statement of Rongpo Ltd for the year that ended on 31 December 2009 from the following particulars: Balance Sheets as on Liabilities Equity Share Capital 9% Preference Share Capital Profit & Loss A/c 10% Debentures Creditors Proposed Dividend Provision for Taxation

31 December 2008 Rs. 2,00,000 1,00,000

31 December 2009 Rs. 2,50,000 1,50,000

1,40,000 1,50,000 50,000 30,000 50,000 7,20,000

2,20,000 1,00,000 40,000 40,000 40,000 8,40,000

Assets Goodwill Building Machinery Stock Debtors Bank Preliminary Expenses

31 December 2008 Rs. 80,000 2,50,000

31 December 2009 Rs. 50,000 3,30,000

1,60,000 70,000 1,10,000 40,000 10,000 7,20,000

2,35,000 50,000 1,40,000 30,000 5,000 8,40,000

Additional information: i. A building having the book value of Rs. 40,000 was sold for Rs. 50,000. Depreciation on the building provided for 2009 was Rs. 60,000. ii. A machinery having the book value of Rs. 45,000 was sold for Rs. 35,000. Depreciation provided on the machinery for the year 2009 amounted to Rs. 40,000. iii. Tax paid during the year 2009 was Rs. 45,000. iv. 10% debentures were redeemed at 10% premium. Ans.: Increase in Working Capital—Rs. 10,000; Fund from operation—Rs. 2,95,000; Fund Flow Statement total— Rs. 4,80,000.

Modified Date: Thu, Jul 01, 2010 02:41:25 PM

Output Date: Tue, Jul 06, 2010 11:50:19 AM

REV-II

Project: Management Accounting_Debarshi Bhattacharyya ACE Pro India Pvt. Ltd. File: X:\Pearson\Management Accounting_Debarshi Bhattacharyya\MAIN\M05\LAYOUT_M05\M05_DEBA_ISBN_EN_SE_C05_Part-2.indd

FUND FLOW ANALYSIS

413

13. From the following particulars, prepare a Fund Flow Statement of Tendulkar Ltd for the year that ended on 31 March 2010: Dr.

Profit & Loss A/c for the year that ended on 31 March 2010 Rs. Particulars 45,000 By Sales (Less Return) 3,60,000 By Closing Stock 1,75,000 2,40,000 8,20,000 To Office Expenses 50,000 By Gross Profit b/d To Selling Expenses 30,000 By Interest on Deposit with Bank To Depreciation on Fixed Asset 40,000 To Income Tax 50,000 To Net Profit c/d 80,000 2,50,000 To Dividend 30,000 By Balance b/d To Balance c/d 70,000 By Net Profit b/d 1,00,000

Cr. Rs. 7,50,000 70,000

Particulars To Opening Stock To Purchases (Less Return) To Wages To Gross Profit c/d

8,20,000 2,40,000 10,000

2,50,000 20,000 80,000 1,00,000

Balance Sheets as on 31 March 2009 and 31 March 2010 Liabilities

Share Capital Profit & Loss A/c Loan from Bank Sundry Creditors Bills Payable Outstanding Expenses Tax Liability

As on 31 March 2009 Rs. 3,00,000 20,000 1,00,000 60,000 30,000 10,000 15,000

As on 31 March 2010 Rs. 3,80,000 70,000 1,25,000 55,000 40,000 15,000 25,000

5,35,000

7,10,000

Assets

Plant & Machinery (net) Investment Deposit with Bank Stock Sundry Debtors Bills Receivable Cash Prepaid Expenses

As on 31 March 2009 Rs. 3,15,000 – 50,000 45,000 65,000 40,000 17,000 3,000 5,35,000

As on 31 March 2010 Rs. 4,60,000 50,000 – 70,000 42,500 45,000 35,000 7,500 7,10,000

[B.Com. (Hons), Kalyani University—Adapted] Ans.: Increase in Working Capital—Rs. 20,000; Fund from operation—Rs. 1,60,000; Fund Flow Statement total— Rs. 3,25,000. 14. The summarized Balance Sheets of Chandana Ltd., as on 31 March 2009 and 31 March 2010, were given as follows: Liabilities

Share Capital General Reserve Profit & Loss A/c Mortgage Loan Sundry Creditors Provision for Tax

As on 31 March 2009 Rs. 4,00,000 40,000 1,00,000 60,000 30,000 10,000 6,40,000

Modified Date: Thu, Jul 01, 2010 02:41:25 PM

As on 31 March 2010 Rs. 4,80,000 70,000 1,80,000 50,000 40,000 15,000 8,35,000

Assets

Fixed Assets Investment Stock Sundry Debtors Bills Receivable Bank

Output Date: Tue, Jul 06, 2010 11:50:19 AM

As on 31 March 2009 Rs. 3,40,000 1,25,000 45,000 65,000 25,000 40,000 6,40,000

As on 31 March 2010 Rs. 4,60,000 1,50,000 70,000 80,000 30,000 45,000 8,35,000

REV-II

Project: Management Accounting_Debarshi Bhattacharyya ACE Pro India Pvt. Ltd. File: X:\Pearson\Management Accounting_Debarshi Bhattacharyya\MAIN\M05\LAYOUT_M05\M05_DEBA_ISBN_EN_SE_C05_Part-2.indd

414

MANAGEMENT ACCOUNTING

Additional information: i. An investment costing Rs. 35,000 were sold during the year for Rs. 45,000. ii. Provision for tax made during the year—Rs. 12,000. iii. Dividend paid during the year amounted to Rs. 20,000. iv. During the year, part of the Fixed Assets having WDV of Rs. 10,000 was sold for Rs. 12,000 and the depreciation provided on the Fixed Assets for the year 2009–10 was Rs. 25,000. You are asked to prepare a Fund Flow Statement for the year that ended on 31 March 2010. Ans.: Increase in Working Capital—Rs. 40,000; Fund from Operation—Rs. 1,55,000; Fund Flow Statement total— Rs. 2,92,000. 15. The following are the Balance Sheets of a company as on 31 March 2008 and 31 March 2009:

Liabilities Equity Share Capital 8% Preference Share Capital General Reserve Capital Reserve Securities Premium Profit & Loss A/c Sundry Creditors Bills Payable Provision for Tax Proposed Dividend

Rs. in ’000 As on As on 31 March 31 March 2008 2009 300 350 150 100 40 75 – 20 20 25 30 73 55 83 20 16 40 50 42 50 697 842

Assets Goodwill Land & Building (at cost) Plant & Machinery (Net) Investment Stock Sundry Debtors Bills Receivable Cash & Bank Preliminary Expenses

Rs. in ’000 As on As on 31 March 31 March 2009 2008 100 89 200 170 80 200 20 35 77 100 140 170 20 30 25 18 35 30 697

842

Additional information: i. One piece of land was sold at a profit and the profit was transferred to Capital Reserve. ii. One machine was sold for Rs. 15,000, the written-down value of which on the date of sale was Rs. 18,000. iii. Depreciation charged on plant and machinery amounted to Rs. 16,000. iv. A dividend of Rs. 4,000 was received from the investment, of which Rs. 2,000 was credited to investment A/c, being the dividend declared from the pre-acquisition profit. v. The actual amount of dividend and tax paid were Rs. 35,000 and Rs. 38,000, respectively. Prepare a Fund Flow Statement for the year that ended on 31 March 2009. Ans.: Increase in Working Capital—Rs. 32,000; Fund from Operation – Rs. 2,02,000; Fund Flow Statement total— Rs. 3,26,000. 16. The Balance Sheets of Rajapaksha Ltd as on 31 March 2008 and 31 March 2009 are as follows: Liabilities Share Capital General Reserve Profit on Sale of Investment Profit & Loss A/c 7% Debentures Creditors for Goods Creditors for Expenses

31 March 2008 Rs. 12,00,000 4,00,000

31 March 2009 Rs. Rs. 14,00,000 5,00,000

– 2,00,000 6,00,000 3,20,000 20,000

20,000 4,00,000 4,00,000 5,00,000 24,000

Assets Fixed Assets Less: Accumulated Depreciation Investment at cost Stock (at cost) Sundry debtors (Less Provision for Bad Debt of Rs. 40,000 and Rs. 50,000, respectively)

31 March 2008 Rs. 20,00,000

31 March 2009 Rs. 24,00,000

4,00,000 16,00,000 3,60,000 4,00,000 4,50,000

5,00,000 19,00,000 3,60,000 5,40,000 4,90,000

(Continued)

Modified Date: Thu, Jul 01, 2010 02:41:25 PM

Output Date: Tue, Jul 06, 2010 11:50:19 AM

REV-II

Project: Management Accounting_Debarshi Bhattacharyya ACE Pro India Pvt. Ltd. File: X:\Pearson\Management Accounting_Debarshi Bhattacharyya\MAIN\M05\LAYOUT_M05\M05_DEBA_ISBN_EN_SE_C05_Part-2.indd

FUND FLOW ANALYSIS

Liabilities Proposed Fividend Provision for Tax

31 March 2008 Rs. 60,000 1,40,000

31 March 2009 Rs. Rs. 70,000 1,50,000

Assets

Bills Receivable Prepaid Expenses Miscellaneous Expenditure 29,40,000

34,64,000

415

31 March 2008 Rs.

31 March 2009 Rs.

80,000 20,000 30,000

1,30,000 24,000 20,000

29,40,000

34,64,000

Additional information: i. During the current year, the Fixed Assets (valued at Rs. 20,000 and depreciation written off Rs. 60,000) was sold for Rs. 16,000. ii. Proposed dividend for the last year paid in the current year. iii. During the current year, the investments costing Rs. 1,60,000 were sold and later in the year, investments of the same cost were purchased. iv. Debentures were redeemed at a premium of 10%. v. Liability for tax for the last year came to Rs. 1,10,000. vi. During the current year, bad debts written off were Rs. 30,000 against provision. You are asked to prepare a Fund Flow Statement for the year that ended on 31 March 2009. Ans.: Increase in Working Capital—Rs. 50,000; Fund from Operation—Rs. 6,44,000; Fund Flow Statement total— Rs. 10,40,000. 17. Following are the Balance Sheets as on 31 March 2008 and 31 March 2009: Liabilities

Equity Share Capital General Reserve Profit & Loss A/c Mortgage Loan (Against Plant & Machinery) Sundry Creditors Provision for Tax Bills Payable Bank Overdraft

As on 31 March 2008 Rs. 1,00,000 60,000 5,000 –

As on 31 March 2009 Rs. 1,50,000 10,000 30,000 40,000

30,000 10,000 10,000 – 2,15,000

20,000 15,000 30,000 65,000 3,60,000

Assets

Land & Building Plant & Machinery Furniture & Fittings Investment Stock Sundry Debtors Cash Preliminary Expenses

As on 31 March 2008 Rs. 80,000 42,000 7,000 6,000

As on 31 March 2009 Rs. 75,000 85,000 6,000 12,000

27,500 46,500 2,000 4,000

94,500 77,200 7,300 3,000

2,15,000

3,60,000

During the year that ended on 31 March 2009, the following transactions took place: i. Bonus shares have been issued at one for every two held out of general reserve. ii. The company purchased plant and machinery for Rs. 60,000, out of which Rs. 20,000 was paid in cash and for the rest plant and machinery was mortgaged to the seller. iii. Dividend paid was Rs. 15,000. iv. Furniture (Book value—Rs. 2,100) was sold for Rs. 3,045. v. An investment costing of Rs. 3,000 written off in 2002 was sold for Rs. 5,000 in 2008–09. vi. Furniture purchased during the year was Rs. 1,500. vii. Net profit for the year, after charging depreciation on land and building, plant and machinery, furniture and fittings, and Rs. 21,000 as provision for tax. Prepare a Fund Flow Statement for the year that ended on 31 March 2009. [B.Com., Bombay University—Adapted] Ans.: Increase in Working Capital—Rs. 28,000; Fund from Operation—Rs. 78,455; Fund Flow Statement total— Rs. 86,500.

Modified Date: Thu, Jul 01, 2010 02:41:25 PM

Output Date: Tue, Jul 06, 2010 11:50:19 AM

REV-II

Project: Management Accounting_Debarshi Bhattacharyya ACE Pro India Pvt. Ltd. File: X:\Pearson\Management Accounting_Debarshi Bhattacharyya\MAIN\M05\LAYOUT_M05\M05_DEBA_ISBN_EN_SE_C05_Part-2.indd

416

MANAGEMENT ACCOUNTING

18. The Balance Sheets of Bharbi Ltd as on 31 March 2008 and 31 March 2009 were given as follows: Liabilities

Share Capital General Reserve Profit & Loss A/c Capital Reserve Debentures Provision for Tax Proposed Dividend Unpaid Dividend Current Liabilities

As on 31 March 2008 Rs. 3,00,000 1,70,000 60,000 – 2,00,000 90,000 30,000 – 1,20,000 9,70,000

As on 31 March 2009 Rs. 4,00,000 2,00,000 75,000 10,000 1,40,000 85,000 36,000 4,000 1,30,000 10,80,000

Assets

Less:

Fixed Assets at Cost Depreciation Trade Investment Current Assets Bank Preliminary Expenses

As on 31 March 2008 Rs. 8,00,000 2,30,000 5,70,000 1,00,000 2,00,000 80,000 20,000

As on 31 March 2009 Rs. 9,50,000 2,90,000 6,60,000 80,000 3,00,000 30,000 10,000

9,70,000

10,80,000

During the year 2008–09, the company: i. Sold one machine for Rs. 25,000, the cost of which was Rs. 50,000 and the depreciation provided on it was Rs. 21,000. ii. Provided Rs. 95,000 as depreciation. iii. Redeemed 30% of the debentures @103. iv. Sold some trade investment at a profit which was credited to capital reserve. v. Decided to value the stock at cost whereas previously the practice was to value the stock at a cost less 10%. The stock according to books on 31 March 2008 was Rs. 54,000. The stock on 31 March 2009 was correctly valued at cost Rs. 75,000. vi. Decided to write off Fixed Assets costing Rs. 14,000 (fully depreciated). Prepare a Fund Flow Statement for the year that ended on 31 March 2009. Ans.: Increase in Working Capital—Rs. 34,000; Fund from operation—Rs. 2,70,800; Fund Flow Statement total— Rs. 4,25,800. 19. The following are the summarized Balance Sheets of a company as on 31 March 2009 and 31 March 2010: Rs. in ’000 Liabilities

Share Capital General Reserve Profit & Loss A/c Bank Loan Sundry creditors Provision for Tax

As on 31 March 2009 100.00 25.00 15.25 35.00 75.00 15.00

As on 31 March 2010 125.00 30.00 15.30 – 67.60 17.50

265.25

255.40

Rs. in ’000 Assets

Land & Building Plant & Machinery Stock Sundry Debtors Cash Bank Goodwill

As on 31 March 2009 100.00 75.00 50.00 40.00 0.25 – – 265.25

As on 31 March 2010 95.00 84.50 37.00 32.10 0.30 4.00 2.50 255.40

Additional information: i. Dividends of Rs. 11,500 were paid. ii. The following assets of another company were purchased for a consideration of Rs. 25,000 paid for in shares: Stock—Rs. 10,000 and Machinery—Rs. 12,500. iii. Machinery was further purchased for cash Rs. 12,500. iv. Depreciation written off: On building—Rs. 5,000 and On machinery—Rs. 7,000. v. Income tax paid during the year—Rs. 14,000. vi. Net profit for the year was Rs. 33,050. Prepare a Fund Flow Statement for the year that ended on 31 March 2010. [B.Com. (Hons), Calcutta University—Adapted]

Modified Date: Thu, Jul 01, 2010 02:41:25 PM

Output Date: Tue, Jul 06, 2010 11:50:19 AM

REV-II

Project: Management Accounting_Debarshi Bhattacharyya ACE Pro India Pvt. Ltd. File: X:\Pearson\Management Accounting_Debarshi Bhattacharyya\MAIN\M05\LAYOUT_M05\M05_DEBA_ISBN_EN_SE_C05_Part-2.indd

417

FUND FLOW ANALYSIS

Ans.: Decrease in Working Capital—Rs. 9,450; Fund from Operation—Rs. 45,050; Fund Flow Statement total— Rs. 73,000. 20. Financial Statements of R. Parvathi Ltd for the year 2007 and 2008 were as follows: Balance Sheets (Rs. in ’000) Liabilities Share Capital General Reserve Profit & Loss A/c Bank Overdraft Sundry Creditors Provision for Tax Proposed Dividend

As on As on 31 December 31 December 2007 2008 800 900 300 400 200 300 300 464 1,200 1,000 300 400 80 90 3,180 3,554

Assets Fixed Assets Additions Depreciation Investment Stock Debtors

As on As on 31 December 31 December 2007 2008 600 800 200 100 (300) (350) 500 550 200 1,400 1,230 1,080 1,774 3,180 3,554

Profit & Loss A/c (Rs. in ’000) Particulars To Taxation To Proposed Dividend

For 2007 250 80

For 2008 450 90

To Transfer to General Reserve

100

100

To Balance c/f

200

300

630

940

Particulars By Trading profit By Profit on Sale of Investment By Income Tax Excess Provided in the Previous year By Balance from Last year

For 2007 430 –

For 2008 660 30



50

200 630

200 940

Additional information: i. For the year that ended on 31 March 2008, purchases were Rs. 60 lakhs and sales were Rs. 70 lakhs. ii. Trading profit for the year 2008 was arrived at after charging a depreciation of Rs. 50,000 and directors’ remuneration of Rs. 1,20,000. [M.Com., Madras University—Adapted] Ans.: Increase in Working Capital—560 (Rs. in ’000); Fund from operation—830 (Rs. in ’000); Fund Flow Statement total—1,160 (Rs. in ’000). 21. Following are the Balance Sheets of a company as on 31 March 2008 and 31 March 2009: As on 31 March 2008 Rs. 3,00,000

As on 31 March 2009 Rs. 3,50,000

General Reserve Capital Reserve (Profit on Sale of Investment)

1,50,000 –

2,25,000 5,000

15% Debentures Accrued Expenses Creditors Proposed Dividend Provision for Tax

1,50,000 5,000 80,000 15,000 35,000 7,35,000

1,00,000 6,000 1,25,000 17,000 38,000 8,66,000

Liabilities

Equity Share Capital (Rs. 10 each fully Paid)

As on 31 March 2008 Rs. 4,00,000

As on 31 March 2009 Rs. 4,75,000

90,000

90,000

Stock at Cost

1,00,000

1,35,000

Debtors Cash

1,12,500 32,500

1,22,500 43,500

7,35,000

8,66,000

Assets

Fixed Assets (Net) Long-term Investment (At Cost)

Modified Date: Thu, Jul 01, 2010 02:41:25 PM

Output Date: Tue, Jul 06, 2010 11:50:19 AM

REV-II

Project: Management Accounting_Debarshi Bhattacharyya ACE Pro India Pvt. Ltd. File: X:\Pearson\Management Accounting_Debarshi Bhattacharyya\MAIN\M05\LAYOUT_M05\M05_DEBA_ISBN_EN_SE_C05_Part-2.indd

418

MANAGEMENT ACCOUNTING

Additional information: i. The balance of accumulated depreciation stood at Rs. 1,00,000 on 31 March 2008 and Rs. 1,25,000 on 31 March 2009. ii. During the year 2008–09, Fixed Assets having a book value of Rs. 5,000 (accumulated depreciation— Rs. 15,000) was sold for Rs. 4,000. iii. During the year 2008–09, investments costing Rs. 40,000 were sold. iv. Debentures were redeemed at a premium of 10%. v. Tax of Rs. 27,000 was paid. vi. Dividend proposed in 2007–08 was paid in 2008–09. You are asked to prepare a Fund Flow Statement for the year that ended on 31 March 2009. Ans.: Increase in Working Capital—Rs. 10,000; Fund from operation—Rs. 1,68,000; Fund Flow Statement total— Rs. 2,67,000. 22. From the following information, prepare a Fund Flow Statement for the year that ended on 31 March 2009: Balance Sheets (Rs. in Lakhs) Liabilities Equity Share Capital (Rs. 100) Redeemable Preference Shares of Rs. 100, Paid Rs. 50

As on 31 March 2008 10.00 5.00

Securities Premium Capital Redemption Reserve General Reserve Profit & Loss A/c Current Liabilities Provision for Taxation

0.25 – 10.00 2.75 7.00 3.00 38.00

As on 31 March 2009 15.00 –

– 5.00 7.00 3.00 2.00 4.00 36.00

Assets Plant at WDV Stock Debtors Cash Balance Miscellaneous Expenditure

As on 31 March 2008 15.00 6.00

As on 31 March 2009 18.00 3.00

15.00 1.00 1.00

9.00 2.00 4.00

38.00

36.00

Additional information: i. During the year, the company paid Rs. 2,00,000 as equity dividend and Rs. 56,250 as preference dividend. ii. The company redeemed the preference shares at a premium of 5% after making a call of Rs. 50 per share to make the shares fully paid. iii. During the year, one plant whose book value was Rs. 1,00,000 was sold at a loss of Rs. 25,000 and the company purchased plant for Rs. 6,00,000. iv. Miscellaneous expenditure included Rs. 5 lakhs as share issue and other expenses paid during the year. v. A sum of Rs. 3,50,000 has been provided for taxation for the year. [B.Com. (Hons), Calcutta University—Adapted] Ans.: Decrease in Working Capital—Rs. 3,00,000; Fund from operation—Rs. 12,81,250; Fund Flow Statement total—Rs. 26,56,250. 23. From the following Balance Sheets of Sundries Ltd, prepare: i. Statement of changes in Working Capital; ii. Fund Flow Statement for the year that ended on 31 March 2010.

Modified Date: Thu, Jul 01, 2010 02:41:25 PM

Output Date: Tue, Jul 06, 2010 11:50:19 AM

REV-II

Project: Management Accounting_Debarshi Bhattacharyya ACE Pro India Pvt. Ltd. File: X:\Pearson\Management Accounting_Debarshi Bhattacharyya\MAIN\M05\LAYOUT_M05\M05_DEBA_ISBN_EN_SE_C05_Part-2.indd

FUND FLOW ANALYSIS

419

Balance Sheets Liabilities

Equity Share Capital General Reserve Profit & Loss A/c 6% Debentures Unclaimed Dividend Sundry Creditors Provision for Tax Bills Payable

As on 31 March 2009 Rs. 20,000 2,500 3,000 5,000 300 4,500 2,500 1,700 39,500

As on 31 March 2010 Rs. 27,500 4,000 2,500 10,000 250 7,250 8,500 2,500 62,500

Assets

Goodwill Land & Building Plant & Machinery Bills Receivable Stock Sundry Debtors Cash & Bank Discount on Issue of Shares

As on 31 March 2009 Rs. 3,500 7,500 9,000 3,250 10,000 4,000 2,250 – 39,500

As on 31 March 2010 Rs. 2,500 10,000 22,500 4,200 11,500 3,500 7,800 500 62,500

Additional information: i. Depreciation on plant and machinery and building for the year 2009–10 amounted to Rs. 2,500 and Rs. 1,500 respectively. ii. The above company acquired the assets of a small company for Rs. 10,000 and paid the purchase consideration by issuing the fully paid shares amounting to Rs. 7,500 and paying the balance in cash. The assets consisted of plant and machinery for Rs. 5,500, stock for Rs. 2,500 and goodwill for Rs. 2,000. iii. Interim dividend paid during 2009–10 was Rs. 5,000. iv. Income tax paid during 2009–10 amounted to Rs. 3,500. v. Debentures were issued at a discount of Rs. 1,000. vi. Decided to value the stock at cost, whereas previously the practice was to value the stock at cost less 20%. The stock according to books as on 31 March 2009 was Rs. 10,000. The stock on 31 March 2010 of Rs. 11,500 was correctly valued at cost. [B.Com. (Hons), Calcutta University—Adapted] Ans.: Increase in Working Capital—Rs. 1,500; Fund from operation—Rs. 21,000; Fund Flow Statement total— Rs. 27,600.

Modified Date: Thu, Jul 01, 2010 02:41:25 PM

Output Date: Tue, Jul 06, 2010 11:50:19 AM

REV-II

Project: Management Accounting_Debarshi Bhattacharyya ACE Pro India Pvt. Ltd. File: X:\Pearson\Management Accounting_Debarshi Bhattacharyya\MAIN\M06\LAYOUT_M06\M06_DEBA_ISBN_EN_SE_C06.indd

Working Capital

6

LEARNING OBJECTIVES On completion of the study of the chapter, you should be able to understand: What is Working Capital? Need and importance of Working Capital. Different concepts and classification of Working Capital. Sources of Working Capital. Determinants of Working Capital. Components of Working Capital. Working Capital or Operating Cycle. Forecasting or Estimation of Working Capital. Valuation of work-in-process.

6.1 MEANING AND CONCEPT OF WORKING CAPITAL Working Capital refers to the portion of capital which is employed in the business to carry on its day-to-day activities. It is used by the business to perform its operating activities. Therefore, the capital invested into the business to carry on the continuous operational activities of a concern is called the ‘Working Capital.’ Working Capital of a business is invested in its Current Assets and Current Liabilities. Working Capital is the life-blood of every business concern. Without adequate Working Capital, a concern cannot perform its day-to-day operating activities smoothly. Lack of Working Capital in a business results in interruption or discontinuation of its day-to-day operational activities. 6.2 IMPORTANCE OF WORKING CAPITAL As mentioned earlier, Working Capital is the life-blood of every business concern. Accordingly, it has immense importance to every business concern. It helps in the following business activities of every concern: i. Smooth running of operating activities. ii. A continuous or uninterrupted flow of production. iii. Ensures continuous supply of raw materials in time at a least price. iv. Regularity in payment to suppliers of raw materials and in payment of wages and overhead. v. Efficient use of Fixed Assets. vi. Easy availability of bank overdraft or short-term loans. vii. Regularity in payment of interest on loan and of dividend to shareholders. 6.3 NEED FOR WORKING CAPITAL The cash invested in raw materials, wages and overheads are brought back into the business only after realization from the sale of goods. There is a time lag between such cash engagement and its recovery through sales. To carry on a continuous production process, Working Capital is essentially required to every business concern till

Modified Date: Tue, Jul 06, 2010 05:14:52 PM

Output Date: Tue, Jul 06, 2010 05:15:41 PM

Rev-II

Project: Management Accounting_Debarshi Bhattacharyya ACE Pro India Pvt. Ltd. File: X:\Pearson\Management Accounting_Debarshi Bhattacharyya\MAIN\M06\LAYOUT_M06\M06_DEBA_ISBN_EN_SE_C06.indd

WORKING CAPITAL

421

sales realization occurs. Working Capital of a business provides a continuous finance for purchasing raw materials and payment of wages and overheads till the cash is brought back into the business by way of sales realization. Once the cash is realized from the sales, it is reinvested into the business for purchasing raw materials and payment of wages and overheads. The process goes on similarly. 6.4 DIFFERENT CONCEPTS AND CLASSIFICATION OF WORKING CAPITAL From the different basis or viewpoints, Working Capital may be classified in different ways. Different classifications of Working Capital are first shown and thereafter these classifications are discussed in Figure 6.1. Classification of Working Capital

On the basis of time

On the basis of concept

Gross Working Capital

Net Working Capital

Positive Working Capital

Permanent Working Capital

Temporary Working Capital

Negative Working Capital

Figure 6.1

Classification of Working Capital

6.4.1 Classification of Working Capital on the Basis of Concept From the conceptual point of view, Working Capital is classified into two parts. They are: (i) Gross Working Capital; and (ii) Net Working Capital. (i) Gross Working Capital: This is a wider concept of Working Capital. Under this concept, the capital invested in the total Current Assets alone is considered as the Working Capital. Therefore, Gross Working Capital refers to the capital invested in the total Current Assets of a business. This concept of Working Capital is called ‘Balance Sheet approach of Working Capital.’ As the Gross Working Capital is represented by the sum of total Current Assets, it always becomes a positive value. Gross Working Capital = Total Current Assets. If the Gross Working Capital is defined from the viewpoint of a complete Balance Sheet, it may be expressed as follows: Gross Working Capital = Proprietors’ Fund + Total Debt − Fixed Assets. Where, Proprietors’ Fund = Share Capital + Reserves & Surplus − Miscellaneous Expenditure and Total Debt = Long-term Loan + Current Liabilities. (ii) Net Working Capital: This is a narrower concept of Working Capital. Under this concept, the capital invested in the total Current Assets less the total Current Liabilities is considered as the Working Capital. Therefore, Net Working Capital refers to the difference between the total Current Assets and the total Current Liabilities of a business. This is the most popular concept of the Working Capital. This concept of Working Capital is also called ‘Balance Sheet approach of Working Capital.’ Net Working Capital = Total Current Assets − Total Current Liabilities. If the Net Working Capital is defined from the viewpoint of a complete Balance Sheet, it may be expressed as follows: Net Working Capital = Proprietors’ Fund + Long-term Loan − Fixed Assets.

Modified Date: Sat, Jun 26, 2010 02:55:02 PM Output Date: Tue, Jul 06, 2010 12:17:35 PM

Rev-II

Project: Management Accounting_Debarshi Bhattacharyya ACE Pro India Pvt. Ltd. File: X:\Pearson\Management Accounting_Debarshi Bhattacharyya\MAIN\M06\LAYOUT_M06\M06_DEBA_ISBN_EN_SE_C06.indd

422

MANAGEMENT ACCOUNTING

Again, Net Working Capital is sub-classified into two parts. These are: (a) Positive Working Capital and (b) Negative Working Capital. (a) Positive Working Capital: If the total Current Assets of a concern are more than its total Current Liabilities, the difference between them is called the positive Net Working Capital. Therefore, it is nothing but the excess of total Current Assets of a concern over its total Current Liabilities. It indicates a favourable shortterm solvency position of a concern. (b) Negative Working Capital: If the total Current Liabilities of a concern is more than its total Current Assets, then the difference between them is called the negative Net Working Capital. Therefore, it is nothing but the excess of total Current Liabilities of a concern over its total Current Assets. It indicates an adverse short-term solvency position of a concern. 6.4.2 Classification of Working Capital on the Basis of Time From the point of view of time span, Working Capital is classified into two parts. They are: (i) Permanent Working Capital; and (ii) Temporary Working Capital. (i) Permanent Working Capital: Permanent Working Capital is the portion of the Working Capital which remains permanently invested into the business. It is the minimum amount of Working Capital which is always kept ready in the form of Current Assets to carry on operational activities of a concern uninterruptedly throughout the year. It constitutes the minimum amount of Working Capital locked up in Current Assets in the form of stock of raw materials, work-in-process (WIP), finished goods, loose tools, spare parts, cash in hand and at bank and so on, throughout the year, so that the normal operational activities of the concern could never be discontinued. This type of Working Capital is also known as Core Current Assets or Core Working Capital. As this type of Working Capital is required to be invested permanently in the Current Assets of a concern, it is generally financed out of long-term sources of capital of the concern, such as issue of shares, dentures and so on. (ii) Temporary Working Capital: Permanent Working Capital is the portion of Working Capital which is required for a short period over and above the permanent Working Capital. It is the additional Working Capital that a concern needs in the form of inventories, receivable, cash and so on, during different times of the year. This type of Working Capital always changes its forms from cash to inventory, inventory to receivables and receivables to cash again. This type of Working Capital is also called Variable Working Capital or Seasonal Working Capital. 6.5 SOURCES OF WORKING CAPITAL Every concern has to finance its Working Capital out of various sources. Fixed Working Capital of the business is financed out of the long-term capital employed in the business, whereas temporary Working Capital of the business is arranged out of short-term capital employed in the business. Different sources of permanent and temporary Working Capital are written as follows: 6.5.1 Sources of Permanent Working Capital (i) External sources: (a) Issue of shares; (b) Issue of debentures; and (c) Raising of long-term loans. (ii) Internal sources: (a) Ploughing back of profit or reinvestment of profit. 6.5.2 Sources of Temporary Working Capital (i) External sources: (a) Trade creditors; (b) Advance from customers; (c) Short-term borrowings; (d) Bank overdraft; (e) Outstanding wages and expenses; and (f) Short-term public deposits. (ii) Internal sources: (a) Provision for Depreciation and (b) Provision for Taxation.

Modified Date: Sat, Jun 26, 2010 02:55:02 PM

Output Date: Tue, Jul 06, 2010 12:17:35 PM

Rev-II

Project: Management Accounting_Debarshi Bhattacharyya ACE Pro India Pvt. Ltd. File: X:\Pearson\Management Accounting_Debarshi Bhattacharyya\MAIN\M06\LAYOUT_M06\M06_DEBA_ISBN_EN_SE_C06.indd

WORKING CAPITAL

423

6.6 DETERMINANTS OF WORKING CAPITAL Following are the factors that are to be considered in determining the Working Capital Requirements: i. Nature of the business. ii. Size of the business. iii. Volume of production and sales. iv. Length of operating cycle. v. Production policy. vi. Credit policy. vii. Operational efficiency. viii. Inventory policy. ix. Seasonal variation. x. Expansion and growth of the business. xi. Depreciation policy. xii. Level of taxes. xiii. Dividend policy. xiv. Cash Reserve requirement. xv. Price-level changes. 6.7 COMPONENTS OF WORKING CAPITAL As Working Capital is the difference between the Current Assets and Current Liabilities, the components of Working Capital are Current Assets and Current Liabilities. Current assets are those assets which can be converted into cash within one accounting year in the ordinary course of business. These assets are easily convertible into cash. Examples of currents assets are: stock of Raw Materials, WIP, Finished Goods, Sundry Debtors, Cash in hand, Cash at bank, Bills Receivable, Prepaid Expenses, Accrued Incomes and so on. On the other hand, Current Liabilities are those liabilities which are intended to be repaid within one accounting year in the ordinary course of business. These liabilities are readily payable in cash. Examples of Current Liabilities are: Sundry Creditors, Bills Payable, Outstanding Expenses, Pre-received Incomes and so on. Above all, Current Assets and Current Liabilities are related to the operational activities of the concern and are termed as Operating Current Assets and Operating Current Liabilities. Provision for taxation, proposed dividend and outstanding interest on loan or debentures are also Current Liabilities, but these are not directly related to the operational activities of the concern and are termed as non-operating Current Liabilities. 6.8 POSITIVE AND NEGATIVE WORKING CAPITAL Whenever the Working Capital is determined on the basis of Gross Working Capital concept, it always becomes a positive Working Capital, as the Gross Working Capital represents the sum of all Current Assets only. But, on the other hand, whenever the Working Capital is determined on the basis of Net Working Capital concept, it may be positive- or negative Working Capital, as Net Working Capital represents the difference between the Current Assets and Current Liabilities. Whenever the total Current Assets exceed the total Current Liabilities, it results in a positive Working Capital. Whenever the total Current Liabilities exceed the total Current Assets, it results in a negative Working Capital. 6.9 WORKING CAPITAL OR OPERATING CYCLE Working Capital Cycle or Operating Cycle refers to the period that an enterprise takes in converting the cash back into the business from the cash initially invested in various operating activities of the enterprise. The said cycle starts from the cash blocked by the way of purchase of raw materials and ends with the realization of cash out of sales. Therefore, the said cycle is the time lag between the cash investment in purchase of raw materials and

Modified Date: Sat, Jun 26, 2010 02:55:02 PM

Output Date: Tue, Jul 06, 2010 12:17:35 PM

Rev-II

Project: Management Accounting_Debarshi Bhattacharyya ACE Pro India Pvt. Ltd. File: X:\Pearson\Management Accounting_Debarshi Bhattacharyya\MAIN\M06\LAYOUT_M06\M06_DEBA_ISBN_EN_SE_C06.indd

424

MANAGEMENT ACCOUNTING

the recovery of cash by the way of sale of goods. In the mean period, the cash engaged in operation is being gradually converted into different forms of Working Capital till its recovery from the sales. Cash, thus, engaged in the operating activities is being blocked in the following forms of Working Capital: i. Cash blocked in purchase of raw materials—thus the cash is being converted into the stock of raw materials. ii. From the production process, the further cash is blocked by way of payment of wages and overheads along with the cash blocked in the raw materials—thus the stock of raw materials and cash paid for wages and overheads are being blocked into the stock of WIP. iii. Amount incurred for raw materials, wages and overheads are further blocked in the finished goods— thus the stock of WIP is being converted into the stock of finished goods. iv. If the finished goods are sold on credit, then the amount invested in raw materials, wages and overheads, along with profit, are further blocked in debtors—thus the stock of finished goods is being converted into debtors. v. Cash is brought back into the business by way of realization of sales from debtors—thus the amount blocked in debtors is being converted into cash. 6.10 ESTIMATION OR FORECASTING OF WORKING CAPITAL Forecasting of Working Capital refers to the estimation of Working Capital Requirement for a certain future period. It is made so as to indicate the amount of Working Capital required for the future period and the sources from which the required amount of Working Capital can be financed. Following factors should be taken into consideration for the estimation of Working Capital Requirement: i. Expected annual production quantity. ii. Cost of raw materials, wages and overheads per unit of the product. iii. Raw materials holding period in store before use. iv. Production-processing period. v. Finished goods storage period before sale. vi. Credit period allowed to sundry debtors for credit sales. vii. Credit period received from the sundry creditors against the credit purchases of raw materials. viii. Lag in the payment of wages and overheads. ix. Management policy regarding inclusion of profit element into Working Capital blocked in the amount due from debtors. x. Amount of cash to be kept in hand for contingencies. 6.11 PROCEDURE FOR ESTIMATION/FORECASTING OF WORKING CAPITAL The following steps are to be followed for the estimation or forecasting of Working Capital Requirement: i. Calculation of average production: The average estimated production per period (i.e., per day or per week or per month) is to be calculated first. ii. Calculation of average cost and sales: Secondly, the average cost of each element of cost as well as the total cost per period is to be calculated—that is, the cost of material, wages and overhead per day or per week or per month and the total cost per month to be calculated. The profit per period is to be added to the total cost per period so as to get the sales per period. iii. Calculation of period of block: The period of block of each element of cost, such as materials, wages and overheads in the operating cycle, are to be separately calculated, that is, the cost of materials blocked in the store, in the production process, in the finished goods and in the debtors; wages and overhead costs blocked in the production process, in the finished goods and in the debtors; and the profit blocked in the debtors, if the profit element of Working Capital is considered.

Modified Date: Sat, Jun 26, 2010 02:55:02 PM

Output Date: Tue, Jul 06, 2010 12:17:35 PM

Rev-II

Project: Management Accounting_Debarshi Bhattacharyya ACE Pro India Pvt. Ltd. File: X:\Pearson\Management Accounting_Debarshi Bhattacharyya\MAIN\M06\LAYOUT_M06\M06_DEBA_ISBN_EN_SE_C06.indd

425

WORKING CAPITAL

iv. Calculation of Working Capital Requirement: Firstly, the Working Capital Requirement for each element of cost (i.e., for materials, labour and overheads) is to be separately calculated at different stages of the operating cycle (i.e., in store, production process, finished goods and debtors) by multiplying their respective average cost per period with the period of block at the respective stages of the operating cycle. If any credit period is allowed by the suppliers of materials and if there is any time lag for the payment of wages and overheads, then the Working Capital not required for such credit period is to be deducted from the Working Capital Requirement for materials, wages and overheads, respectively. Finally, the Working Capital Requirements for each element of cost as calculated in the above are to be added to get the total Working Capital Requirement without any profit. If the profit blocked in debtors is considered, then the amount of profit blocked per period is to be calculated by multiplying the amount of profit blocked per period with the credit period allowed to debtors, which is to be added to the total Working Capital Requirement without any profit as calculated in the above to get the total Working Capital Requirement with the profit. If any cash balance is required to be kept in hand, then that amount is to be added to the total Working Capital Requirement with profit, as calculated in the above to get the ‘Net Working Capital Requirement with profit and cash balance.’ 6.12 VALUATION OF STOCK OF WORK-IN-PROGRESS (WIP) As the degree of completion of the stock of WIP cannot be specifically known, it is valued on the assumption that it is completed up to 100% of the raw materials, 50% of the wages and 50% the of overheads. As the stock of WIP as mentioned above, 50% of the amount blocked per period (i.e., per day or per week or per month) in wages and overheads are to be taken for ascertaining the Working Capital that is required for wages and overheads, considering the period of block per period in wages and overheads in full. Alternatively, the periods of block for wages and overheads could be taken at 50% of the normal period, considering the amount blocked per period in wages and overheads in full. But the Working Capital required for materials in WIP is to be taken as usual, that is, the full period of block of materials with the full amount blocked in the materials per period. 6.13 DIFFERENT APPROACHES OF ESTIMATION/FORECASTING OF WORKING CAPITAL Working Capital Requirement for a future period may be estimated or forecasted in different approaches as shown in the following: First Approach: Statement showing the requirement of Working Capital (with profit) for the period Elements of Working Capital Current Assets: Stock of Raw Materials Stock of WIP Stock of Finished Goods Sundry Debtors Cash & Bank Less:

Rs.

– – – – – –

Current Liabilities: Sundry Creditors Outstanding Wages Outstanding Overheads Working Capital Requirement

Modified Date: Sat, Jun 26, 2010 02:55:02 PM

Rs.

Output Date: Tue, Jul 06, 2010 12:17:35 PM

– – –

– –

Rev-II

Project: Management Accounting_Debarshi Bhattacharyya ACE Pro India Pvt. Ltd. File: X:\Pearson\Management Accounting_Debarshi Bhattacharyya\MAIN\M06\LAYOUT_M06\M06_DEBA_ISBN_EN_SE_C06.indd

426

MANAGEMENT ACCOUNTING

Second Approach: Statement showing the requirement of Working Capital (with profit) for the period

Particulars

Period of Block

Amount Blocked per Period Rs.

Amount Rs.

Total Rs.

Raw Materials: In Stores In WIP In Finished Stock In Credit to Debtors Less: Credit from Creditors Net Block Wages: In WIP In Finished Stock In Credit to Debtors Less: Lag in Payment Net Block Overheads: In WIP In Finished Stock In Credit to Debtors Less: Lag in payment Net Block Profit: In Credit to Debtors Working Capital Requirement (without cash) Add: Required cash in hand Working Capital Requirement (with cash)





– – – – –

Third Approach: Statement showing the requirement of Working Capital (with profit) for the period

Particulars

Less:

Amount Blocked Raw Materiper Period als WIP Period of Rs. Rs. Rs. Block

Outstanding OutSundry Finished Sundry Credi- standing Overheads Total Wages tors Goods Debtors Rs.

Raw Materials: In Stores In WIP In Finished Stock In Debtors In Creditors Net Block

Rs.

Rs.

Rs.

Rs.

Rs.



Wages: In WIP In Finished Stock

In Debtors (Continued)

Modified Date: Sat, Jun 26, 2010 02:55:02 PM

Output Date: Tue, Jul 06, 2010 12:17:35 PM

Rev-II

Project: Management Accounting_Debarshi Bhattacharyya ACE Pro India Pvt. Ltd. File: X:\Pearson\Management Accounting_Debarshi Bhattacharyya\MAIN\M06\LAYOUT_M06\M06_DEBA_ISBN_EN_SE_C06.indd

WORKING CAPITAL

Particulars

Amount Blocked Raw Materiper Period als WIP Period of Rs. Rs. Rs. Block

427

Outstanding OutSundry Finished Sundry Credi- standing Overheads Total Wages tors Goods Debtors Rs.

Rs.

Rs.

Rs.

Rs.

Rs.

Less: Lag in payment Net Block



Overheads: In WIP In Finished Stock In Debtors Less: Lag in payment Net Block



Profit: In Debtors Working Capital (without cash) Add: Required cash in hand Working Capital (with cash)

– – – –

Tutorial Notes to Students for Solving Problems For estimating the Working Capital Requirement, students are advised to follow the following steps: i. Convert the expected annual production into production per month, week or day, depending upon the suitability of the given case. ii. Prepare an estimated cost sheet for a month or week or day, as per the production that is converted under 1 above, showing therein the element-wise costs (i.e., materials, wages and overheads), total cost, profit and the selling price. iii. Compute the period of block, if not given. iv. Finally, prepare a statement showing the Working Capital Requirement with suitable columns and calculate the amount of Working Capital blocked in materials, wages and overheads. The profit element may or may not be considered, if not otherwise stated. Get the amount of ‘Working Capital Requirement without cash’ adding up to the Working Capital blocked in each element of cost and profit. v. If the concern desires to maintain a cash balance for contingencies, then add the amount of cash balance that is required to be kept with the amount of Working Capital Requirement without cash as ascertained under 4 above and get the amount of Working Capital Requirement with cash.

6.14 WORKED-OUT PROBLEMS Problem 1 The following are a trading company’s forecast sales and other particulars: Forecast Annual Sales Net Profit on the Cost of Sales Average Credit allowed to Debtors Average Credit allowed by Creditors Average Stock carrying (to meet sales)

Modified Date: Sat, Jun 26, 2010 02:55:02 PM

Output Date: Tue, Jul 06, 2010 12:17:35 PM

Rs. 1,30,000 25% 8 weeks 6 weeks 4 weeks

Rev-II

Project: Management Accounting_Debarshi Bhattacharyya ACE Pro India Pvt. Ltd. File: X:\Pearson\Management Accounting_Debarshi Bhattacharyya\MAIN\M06\LAYOUT_M06\M06_DEBA_ISBN_EN_SE_C06.indd

428

MANAGEMENT ACCOUNTING

Determine the forecast Working Capital of the company. The estimated cash on hand and at bank is Rs. 5,000. [B.Com. (Hons), Calcutta University—2007] Solution Statement showing computation of Forecast Working Capital (with profit) of the company Elements of Working Capital Current Assets: Average Stock (Rs. 2,000 × 4) Average Debtors (Rs. 2,500 × 8) Less:

Add:

Rs. 8,000 20,000 28,000

Current Liabilities: Average Creditors (Rs. 2,000 × 6)

18,000 10,000 5,000 15,000

Estimated cash on hand and at bank Forecast Working Capital

Working Notes 1. Projected Annual Sales = Rs. 1,30,000. Projected Weekly Sales = Rs. 1,30,000 ÷ 52 = Rs.2,500 Projected Weekly Cost = Weekly sales – Weekly Profit = Rs. 2,500 – (25% on cost) = Rs. 2,500 – (1/4 on cost) = Rs. 2,500 – (1/5 on sales) = Rs. 2,500 – (1/5 on Rs. 2,500) = Rs. 2,500 – Rs. 500 = Rs. 2,000. 2. Debtors are valued at sales, but stocks and creditors are valued at cost.

Problem 2 From the following data, compute the money-block period of the Working Capital: Rs. in ’000 20 14 21 96 140 160 32 16

Stock: Raw Materials WIP Finished Goods Purchases Cost of Goods Sold Sales Debtors Creditors 1 year = 360 days

[B.Com. (Hons), Calcutta University—2008] Solution Computation of Money-Block Period 1. Money-block period of raw materials in store =

Value of Stock of Raw Materials × 360 days Purchase of Raw Materials during the year

Modified Date: Sat, Jun 26, 2010 02:55:02 PM

Output Date: Tue, Jul 06, 2010 12:17:35 PM

Rev-II

Project: Management Accounting_Debarshi Bhattacharyya ACE Pro India Pvt. Ltd. File: X:\Pearson\Management Accounting_Debarshi Bhattacharyya\MAIN\M06\LAYOUT_M06\M06_DEBA_ISBN_EN_SE_C06.indd

WORKING CAPITAL

=

429

Rs. 20,000 × 360 = 75 days Rs. 96,000

2. Money-block period of WIP =

Value of Stock of WIP × 360 days 50% of Cost of Goods Sold during the year

=

Rs. 14,000 × 360 = 72 days 50% of Rs. 1,40,000

3. Money-block period of finished goods in godown =

Value of Stock of Finished Goods × 360 days Total Cost of production for the year

=

Rs. 21,000 × 360 = 54 days Rs. 1,40,000

4. Money-block period of indebtors =

Value of Sundry Debtors × 360 days Credit Sales for the year

=

Rs. 32,000 × 360 days = 72 days Rs. 1,60,000

5. Money-block period of creditors for goods =

Value of Sundry Creditors × 360 days Credit Purchases of Materials for the year

=

Rs. 16,000 × 360 = 60 days Rs. 96,000

Notes While calculating the Working Capital Requirement for the year 2010, the following points have been considered: i. It has been assumed that the entire purchases and sales are made on credit. ii. Stock of WIP is valued on the assumption that it is completed to the extent of (i.e., it consists of) 50% only and the stock of finished goods is valued at the total cost. Problem 3 From the following information as furnished by Libra Ltd, calculate the average Working Capital Requirement for the year 2010: Average Credit Period 2 months 1 month

Purchase of Raw Materials Wages

Estimated Amount for 2010 (Rs.) 3,00,000 1,20,000 (Continued)

Modified Date: Sat, Jun 26, 2010 02:55:02 PM

Output Date: Tue, Jul 06, 2010 12:17:35 PM

Rev-II

Project: Management Accounting_Debarshi Bhattacharyya ACE Pro India Pvt. Ltd. File: X:\Pearson\Management Accounting_Debarshi Bhattacharyya\MAIN\M06\LAYOUT_M06\M06_DEBA_ISBN_EN_SE_C06.indd

430

MANAGEMENT ACCOUNTING

Average Credit Period Overheads: Rent & Rates Salaries to Staff Insurance Travelling Expenses Depreciation on Machinery Other Overheads Sales: Cash Credit Average Amount of Stock & WIP Average Amount of Undrawn Profit

Estimated Amount for 2010 (Rs.)

4 weeks 1.5 months 6 weeks 5 weeks – 6 weeks

26,000 96,000 12,000 10,400 30,000 52,000

– 13 weeks – –

2,00,000 6,00,000 60,000 20,000

Sales are made evenly throughout the year. An overdraft limit of Rs. 23,000 has been arranged with the banker. Solution Statement showing the Average Working Capital Requirement of Libra Ltd for the year 2010 Elements of Working Capital Current Assets: Average Stock and WIP Average Debtors (Rs. 6,00,000 × 13 ÷ 52) Less:

Less:

Current Liabilities: Sundry Creditors for Materials (Rs. 3,00,000 × 2 ÷ 12) Outstanding Wages (Rs. 1,20,000 × 1 ÷ 12) Outstanding Rent & Taxes (Rs. 26,000 × 4 ÷ 52) Outstanding Salaries (Rs. 96,000 × 1.5 ÷ 12) Outstanding Insurance (Rs. 12,000 × 6 ÷ 12) Outstanding Travelling Expenses (Rs. 10,400 × 5 ÷ 52) Outstanding Other Overheads (Rs. 52,000 × 6 ÷ 52) Average Undrawn Profit Maximum Bank Overdraft Limit Average Working Capital Requirement

Rs.

Rs. 60,000 1,50,000 2,10,000

50,000 10,000 2,000 12,000 6,000 1,000 6,000 20,000 23,000

87,000 1,23,000 43,000 80,000

Note: 1 year = 52 weeks. Problem 4 Calculate the Working Capital Requirement from the following information: Debtors Velocity Stock Velocity Creditors Velocity Gross Profit Ratio Sales Credit Purchases Cash & Bank Balance

60 days 90 days 75 days 25% Rs. 20,00,000 1/3 of the Cost of Goods Sold 2.5% of Sales

The company expects a 50% sales increment during the next year. Assume 1 year = 360 days. [C.A. (Inter)—Adapted]

Modified Date: Sat, Jun 26, 2010 02:55:02 PM

Output Date: Tue, Jul 06, 2010 12:17:35 PM

Rev-II

Project: Management Accounting_Debarshi Bhattacharyya ACE Pro India Pvt. Ltd. File: X:\Pearson\Management Accounting_Debarshi Bhattacharyya\MAIN\M06\LAYOUT_M06\M06_DEBA_ISBN_EN_SE_C06.indd

431

WORKING CAPITAL

Solution Statement showing Working Capital Requirement (with profit) of

for the period

Elements of Working Capital Current Assets: Stock3 Debtors4 Cash & Bank (2.5% of Rs. 30,00,000) Less:

Rs. 5,62,500 5,00,000 75,000 11,37,500

Current Liabilities: Creditors4 Working Capital Requirement

1,56,250 9,81,250

Working Notes 1. Calculation of Expected Sales and Cost of Goods Sold for the Next Year Existing sales Add: 50% Expected Sales increment during the next year (50% of 20 lakhs) Expected Sales for the next year Less: Gross Profit (25% on sales = 25% on 30,00,000) Estimated Cost of Goods Sold

Rs. 20,00,000 10,00,000 30,00,000 7,50,000 22,50,000

2. Calculation of Credit Purchases Credit Purchases = 1/3 of the Cost of Goods Sold = 1/3 of Rs. 22,50,000 = Rs. 7,50,000. 3. Value of Stock Stock Velocity = 90 days. 360 days Stock Velocity Ratio = =4 90 days Stock Velocity Ratio =

or 4 = ∴ Average Stock =

Cost of Goods Sold Average Stock

22,50,0001 Average Stock 22,50,000 = Rs. 5,62,500 4

4. Value of Debtors Debtors Velocity = 60 days. Debtors Velocity Ratio =

360 days =6 60 days

Debtors Velocity Ratio =

Credit Sales Average Debtors

or 6 =

30,00,0001 Average Debtors

∴ Average Debtors =

Modified Date: Sat, Jun 26, 2010 02:55:02 PM

30,00,000 = Rs. 5,00,000 6

Output Date: Tue, Jul 06, 2010 12:17:35 PM

Rev-II

Project: Management Accounting_Debarshi Bhattacharyya ACE Pro India Pvt. Ltd. File: X:\Pearson\Management Accounting_Debarshi Bhattacharyya\MAIN\M06\LAYOUT_M06\M06_DEBA_ISBN_EN_SE_C06.indd

432

MANAGEMENT ACCOUNTING

5. Value of Creditors Creditors Velocity = 75 days. Creditors Velocity Ratio =

360 days = 4.8 75 days

Creditors Velocity Ratio =

Credit Purchases Average Creditors

or 4.8 =

7,50,0002 Average Creditors

∴ Average Creditors =

7,50,000 = Rs. 1,56,250 4.8

Note: Debtors are valued at sales, but Stocks and Creditors are valued at cost. Problem 5 Missionji Ltd has a production capacity to produce 80,000 units per annum. In 2010, the company has decided to operate at 75% of its capacity at the following Cost Price structure: Rs. 6.00 4.00 3.00 2.00 15.00

Materials per unit Wages per unit Overhead per unit Profit per unit Selling Price per unit

Following are the other information in relation to the production and sales of the company: i. Raw materials are kept in stores for 2 months, the processing time is 1 month and the finished goods are sold after 1 month. ii. Debtors are allowed a credit period of 8 weeks and a credit of 6 weeks is received from the creditors. iii. Lag in the payment of wages and overheads are 1 month and 15 days, respectively. Prepare a statement showing the Working Capital Requirement for the year 2010. Solution Alternative 1: Statement showing the requirement of Working Capital (with profit) for the year 2010 Elements of Working Capital Current Assets: Stock of Raw Materials (Rs. 30,000 × 2) Stock of WIP3 Stock of Finished Goods (Rs. 65,000 × 1) Sundry Debtors (Rs. 75,000 × 20) Cash & Bank Less: Current Liabilities: Sundry Creditors (Rs. 30,000 × 1.50) Outstanding Wages (Rs. 20,000 × 1) Outstanding Overheads (Rs. 15,000 × 0.50) Working Capital Requirement

Modified Date: Sat, Jun 26, 2010 02:55:02 PM

Output Date: Tue, Jul 06, 2010 12:17:35 PM

Rs.

Rs. 60,000 47,500 65,000 1,50,000 Nil 3,22,500

45,000 20,000 7,500

72,500 2,50,000

Rev-II

Project: Management Accounting_Debarshi Bhattacharyya ACE Pro India Pvt. Ltd. File: X:\Pearson\Management Accounting_Debarshi Bhattacharyya\MAIN\M06\LAYOUT_M06\M06_DEBA_ISBN_EN_SE_C06.indd

433

WORKING CAPITAL

Alternative 2: Statement showing the requirement of Working Capital (with profit) for the year 2010 Particulars

Period of Block (Months)

Amount Blocked per Month (Rs.)

2 1 1 2 6 1.5 4.5

30,000 30,000 30,000 30,000

1 1 2 4 1 3

10,000 20,000 20,000

1 1 2 4 0.5 3

7,500 15,000 15,000

2

10,000

Raw Materials: In Stores In WIP In Finished Stock In Credit to Debtors Less: Credit from Creditors Net Block Wages: In WIP In Finished Stock In Credit to Debtors Less: Lag in payment Net Block Overheads: In WIP In Finished Stock In Credit to Debtors Less: Lag in payment Net Block Profit: In Credit to Debtors Working Capital Requirement

Amount (Rs.)

Total (Rs.)

60,000 30,000 30,000 60,000 1,80,000 (45,000)

30,000

1,35,000 10,000 20,000 40,000 70,000 (20,000)

20,000

50,000 7,500 15,000 30,000 52,500 (7,500)

15,000

45,000 20,000

20,000 2,50,000

Alternative 3: Statement showing the requirement of Working Capital (with profit) for the year 2010

Particulars

Raw Materials: In Stores In WIP In Finished Stock In Debtors Less: In Creditors Net Block Wages: In WIP In Finished Stock In Debtors

Amount Blocked Raw Mateper Period of Block Month rials Rs. Rs. (months) 2 1 1 2 6 1.5 4.5

30,000 30,000 30,000 30,000

1 1

10,000 20,000

2 4

20,000

OutOut- standing Sundry Finished Sundry Credi- standing Overheads Total Wages WIP Goods Debtors tors Rs.

Rs.

Rs.

Rs.

Rs.

Rs.

Rs.

60,000 30,000 30,000 60,000

30,000

(45,000) 1,35,000 10,000 20,000 40,000

(Continued)

Modified Date: Sat, Jun 26, 2010 02:55:02 PM

Output Date: Tue, Jul 06, 2010 12:17:35 PM

Rev-II

Project: Management Accounting_Debarshi Bhattacharyya ACE Pro India Pvt. Ltd. File: X:\Pearson\Management Accounting_Debarshi Bhattacharyya\MAIN\M06\LAYOUT_M06\M06_DEBA_ISBN_EN_SE_C06.indd

434

MANAGEMENT ACCOUNTING

Particulars

Less: Lag in payment Net Block Overheads: In WIP In Finished Stock In Debtors Less: Lag in payment Net Block Profit: In Debtors Working Capital

Amount Blocked Raw Mateper Period of Block Month rials Rs. Rs. (months) 1 3

20,000

1 1 2 4 0.5 3.5

7,500 15,000 15,000

2

10,000

OutOut- standing Sundry Finished Sundry Credi- standing Overheads Total Wages WIP Goods Debtors tors Rs.

Rs.

Rs.

Rs.

Rs.

Rs.

Rs.

(20,000) 50,000 7,500 15,000 30,000

15,000

(7,500) 45,000

60,000 47,500

65,000

20,000 1,50,000

(45,000)

(20,000)

20,000 (7,500) 2,50,000

Working Notes 1. Full production capacity (i.e., 100%) = 80,000 units per year. Estimated production for the year 2010 at 75% capacity level = 80,000 × 75 ÷ 100 = 60,000 units Estimated monthly production in the year 2010 = 60,000 ÷ 12 = 5,000 units 2. Estimated Cost Sheet for a Month of 2010 Raw Materials (5,000 units × Rs. 6) Wages (5,000 units × Rs. 4) Overheads (5,000 units × Rs. 3) Total Cost Profit (5,000 units × Rs. 2) Selling Price (5,000 units × Rs. 15)

Rs. 30,000 20,000 15,000 65,000 10,000 75,000

3. Valuation of Stock of WIP Raw Materials (100% of Rs. 30,000) Wages (50% of Rs. 20,000) Overheads (50% of Rs. 15,000) Value of Stock of WIP

Rs. 30,000 10,000 7,500 47,500

Notes While calculating the Working Capital Requirement for the year 2010, the following points have been considered: 1. 2. 3. 4.

It has been assumed that the production and sales are made evenly throughout the year. Here, the Working Capital Requirement for the year 2010 with profit has been ascertained. It has also been assumed that the entire purchases and sales have been made on credit. Stock of WIP is valued on the assumption that it is completed to the extent of (i.e., it consists of ) 100% of raw materials, 50% of wages and 50% of overheads.

Modified Date: Sat, Jun 26, 2010 02:55:02 PM

Output Date: Tue, Jul 06, 2010 12:17:35 PM

Rev-II

Project: Management Accounting_Debarshi Bhattacharyya ACE Pro India Pvt. Ltd. File: X:\Pearson\Management Accounting_Debarshi Bhattacharyya\MAIN\M06\LAYOUT_M06\M06_DEBA_ISBN_EN_SE_C06.indd

435

WORKING CAPITAL

Stop and Think 1. For the solution made under Alternative 1, Working Note 3 is only to be made. For solutions made under Alternatives 1 and 2, Working Notes 2 and 3 are to be made. 2. As the stock of WIP is valued at 100% of raw materials, 50% of wages and 50% of overheads, 50% of the amount blocked per month in wages and overheads have been taken for ascertaining the Working Capital Requirement for wages and overheads, considering the period of block per month in the wages and overheads in full. Alternatively, the periods of block per month for wages and overheads could have been taken at 50% of the normal period, considering the amount blocked per month in wages and overheads in full. 3. Alternatively, the Working Capital Requirement for the year 2010 without any profit could have been ascertained. In such a case, the profit element of the Working Capital is to be ignored. 4. Students are advised to follow any one of the different alternative solutions for their examination purpose, if no specific approach is asked to follow in the question.

Problem 6 Kalyan Ltd normally produces and sells 1,04,000 units of a product annually. In 2010, the company decides to operate at 50% of its capacity at the following Cost Price structure: Rs. 5.00 3.00 2.00 2.00

Materials per unit Wages per unit Overhead per unit Profit per unit

Following are the other information in relation to the production and sales of the company: i. Raw materials are kept in stores for 8 weeks, the processing time is 4 weeks and the finished goods are sold after 1 month. ii. Debtors are allowed a credit period of 1.5 months and a credit of 4 weeks is received from the creditors. iii. Lag in the payment of wages and overheads are 21 days and 15 days respectively. iv. Cash in hand is expected to be 20% of the Net Working Capital. Prepare a statement showing the Working Capital Requirement for the year 2010. Solution Statement showing the requirement of Working Capital (without profit) for the year 2010

Particulars Raw Materials: In Stores In WIP In Finished Stock In Credit to Debtors

Period of Block (Weeks) (Rs.)

Amount Blocked per Week (Rs.)

8 4 4 6 22

5,000 5,000 5,000 5,000

Amount (Rs.)

Total (Rs.)

40,000 20,000 20,000 30,000 1,10,000 (Continued)

Modified Date: Sat, Jun 26, 2010 02:55:02 PM

Output Date: Tue, Jul 06, 2010 12:17:35 PM

Rev-II

Project: Management Accounting_Debarshi Bhattacharyya ACE Pro India Pvt. Ltd. File: X:\Pearson\Management Accounting_Debarshi Bhattacharyya\MAIN\M06\LAYOUT_M06\M06_DEBA_ISBN_EN_SE_C06.indd

436

MANAGEMENT ACCOUNTING

Particulars Less:

Less:

Less:

Add:

Credit from Creditors Net Block Wages: In WIP In Finished Stock In Credit to Debtors

Period of Block (Weeks) (Rs.) 4 18

Amount Blocked per Week (Rs.) 5,000

4 4 6 14 3 11

1,500 3,000 3,000

4 4 6 14 2 12

1,000 2,000 2,000

Lag in payment Net Block Overheads: In WIP In Finished Stock In Credit to Debtors Lag in payment Net Block

Amount (Rs.) (20,000)

Total (Rs.) 90,000

3,000

6,000 12,000 18,000 36,000 (9,000) 27,000

2,000

4,000 8,000 12,000 24,000 (4,000) 20,000 1,37,000 34,250 1,71,250

Cash in hand (20/80 × 100) Working Capital Requirement

Working Notes 1. Full production capacity (i.e., 100%) = 1,04,000 units per year. Estimated production for the year 2010 at 50% capacity level = 1,04,000 ¥ 50 ÷ 100 = 52,000 units Estimated weekly production in the year 2010 = 52,000 ÷ 52 = 1,000 units 2. Estimated Cost Sheet for a Week of 2010

Raw Materials (1,000 units × Rs. 5) Wages (1,000 units × Rs. 3) Overheads (1,000 units × Rs. 2) Total Cost Profit (1,000 units × Rs. 2) Selling Price (1,000 units × Rs. 12)

Rs. 5,000 3,000 2,000 10,000 2,000 12,000

Notes While calculating the Working Capital Requirement for the year 2010, the following points have been considered: 1. It has been assumed that the production and sales are made evenly throughout the year. 2. Here, the Working Capital Requirement for the year 2010 without any profit has been ascertained. 3. It has also been assumed that the entire purchases and sales have been made on credit. 4. Stock of WIP is valued on the assumption that it is completed to the extent of (i.e., it consists of ) 100% of raw materials, 50% of wages and 50% of overheads.

Modified Date: Sat, Jun 26, 2010 02:55:02 PM

Output Date: Tue, Jul 06, 2010 12:17:35 PM

Rev-II

Project: Management Accounting_Debarshi Bhattacharyya ACE Pro India Pvt. Ltd. File: X:\Pearson\Management Accounting_Debarshi Bhattacharyya\MAIN\M06\LAYOUT_M06\M06_DEBA_ISBN_EN_SE_C06.indd

WORKING CAPITAL

437

Stop and Think 1. Alternatively, the Working Capital Requirement for the year 2010 with profit could have been ascertained. In such a case, the Working Capital blocked in profit is to be considered. 2. As the stock of WIP is valued at 100% of raw materials, 50% of wages and 50% of overheads, the 50% of the amounts blocked per month in wages and overheads have been taken for ascertaining the Working Capital Requirement for wages and overheads, considering the period of block per month in wages and overheads in full. Alternatively, the periods of block per month for wages and overheads could have been taken at 50% of the normal period, considering the amount blocked per month in wages and overheads in full.

Problem 7 Tata Manufacturing Co. started for production of NANO cars at Calcutta in March 2008 and purchased a land for Rs. 10,00,000 and incurred Rs. 5,00,00,000 on its factory construction. However, before the production was started, due to labour problems, the company has shifted its factory to Maharashtra, where it had the benefit of low overheads. Overheads are 50% of the labour expenses in Maharashtra. Following is the Cost Structure per car in Maharashtra for the year 2009–10: Steel Spare parts Engine Labour charges

50 kg @ Rs. 1,000 per kg 10 kg @ Rs. 200 per kg 1 engine @ Rs. 20,000 per engine 100 hrs @ Rs. 20 per hour

From the following additional information, calculate the Working Capital Requirement for the company that is to be started in Maharashtra for the year 2009–10: i. Steel remains in stock for 2 months, spare parts for 1 month and engine for 6 months. ii. Suppliers of steel allow credit for 1 month, suppliers of spare parts for 15 days and suppliers of engine for 2 months. iii. Time lag for payment of labour and overheads is 1 month. iv. Cars will be in stock for 15 days after production. v. Production cycle is for 1 month. vi. Estimated production during the year 2009–10 will be 5,000 NANO cars. [B.Com. (Hons), Mumbai University—April 2009] Solution Statement showing the computation of Working Capital requirement (without profit) for the production of NANO cars during the year 2009–10

Particulars Materials: (i) Steel: In Stock In WIP In Finished Stock

Period of Block (Months)

Amount Blocked per Month (Rs. in ’000)

2.00 1.00 0.50 3.50

20,850 20,850 20,850

Amount (Rs. in ’000)

Total (Rs. in ’000)

41,700 20,850 10,425 72,975 (Continued)

Modified Date: Sat, Jun 26, 2010 02:55:02 PM

Output Date: Tue, Jul 06, 2010 12:17:35 PM

Rev-II

Project: Management Accounting_Debarshi Bhattacharyya ACE Pro India Pvt. Ltd. File: X:\Pearson\Management Accounting_Debarshi Bhattacharyya\MAIN\M06\LAYOUT_M06\M06_DEBA_ISBN_EN_SE_C06.indd

438

MANAGEMENT ACCOUNTING

Period of Block (Months) 1.00 2.50

Particulars Less:

Less:

Less:

Less:

Less:

Credit from Creditors Net Block (ii) Spare Parts: In Stock In WIP In Finished Stock

Amount Blocked per Month (Rs. in ’000) 20,850

1.00 1.00 0.50 2.50 0.50 2.50

Credit from Creditors Net Block (iii) Engine: In Stock In WIP In Finished Stock

6.00 1.00 0.50 7.50 2.00 5.50

Credit from Creditors Net Block Labour: In WIP In Finished Stock Lag in payment Net Block Overheads: In WIP In Finished Stock Lag in payment Net Block Working Capital Requirement

Amount (Rs. in ’000) (20,850)

Total (Rs. in ’000) 51,125

834 834 834

834 834 417 2,085 (417)

834

1,668 8,340 8,340 8,340

50,040 8,340 4,170 62,550 (16,680)

8,340

45,870

0.50 0.50 1.00 1.00 Nil

834 834

0.50 0.50 1.00 1.00 Nil

417 417

417 417 834 (834)

834

Nil 208.50 208.50 417 (417)

417

Nil 98,663

Working Notes 1. Estimated Annual Production of NANO cars for the year 2009–10 = 5,000 units. Estimated Monthly Production of NANO cars for the year 2009–10 = 5,000 units ÷ 12 = 417 units. 2. Estimated Cost Sheet of NANO cars for a month of 2009–10 Materials; Steel (50 kg × Rs. 1,000 × 417) Spare parts (10 kg × Rs. 200 × 417) Engine (Rs. 20,000 × 417) Labour (100 hrs × Rs. 20 × 417) Overheads (50% of labour = 50% of 834) Total Cost

Modified Date: Sat, Jun 26, 2010 02:55:02 PM

Output Date: Tue, Jul 06, 2010 12:17:35 PM

Rs. in ’000 20,850 834 8,340 30,024 834 417 31,275

Rev-II

Project: Management Accounting_Debarshi Bhattacharyya ACE Pro India Pvt. Ltd. File: X:\Pearson\Management Accounting_Debarshi Bhattacharyya\MAIN\M06\LAYOUT_M06\M06_DEBA_ISBN_EN_SE_C06.indd

WORKING CAPITAL

439

Notes While computing the Working Capital Requirement of NANO cars for the year 2009–10, the following points have been considered: 1. It has been assumed that the production is made evenly throughout the year. 2. Here, the Working Capital Requirement has been ascertained without considering the profit element. 3. Stock of WIP is valued on the assumption that it is completed to the extent of (i.e., it consists of ) 100% of raw materials, 50% of labour and 50% of overheads.

Stop and Think 1. Alternatively, the Working Capital Requirement for the year 2010 with profit could have been ascertained. In such a case, the Working Capital blocked in profit is to be considered. 2. As the stock of WIP is valued at 100% of raw materials, 50% of labour and 50% of overheads, 50% of the period of block at WIP stage of labour and overheads have been taken for ascertaining the Working Capital Requirement for labour and overheads, considering the amount blocked per month for labour and overheads in full.

Problem 8 From the following information of Sunshine Paints Ltd, you are required to determine the Working Capital Requirement: (i) Annual (expected) Sales (ii) Analysis of Sales: Raw Materials Expenses Profit (iii) Credit allowed to Debtors Credit allowed by Creditors Raw Materials in Store Processing Period Finished Goods in Store Bank Overdraft Cash in Hand for Contingencies

Rs. 3,60,000 50% 30% 20% 2.5 months 1.5 months 1 month 2 months 4 months Rs. 1,00,000 Rs. 6,000

Production is carried out evenly during the year, and expenses too accrue similarly. [B.Com. (Hons), Calcutta University—1995] Solution Statement showing the requirement of Working Capital (with profit) for the year Particulars Raw Materials: In Stores In WIP

Period of Block (Months)

Amount Blocked per Month

1 2

15,000 15,000

Raw Materials

WIP

Finished Goods

Sundry Debtors

Sundry Creditors

Total

Rs.

Rs.

Rs.

Rs.

Rs.

Rs.

15,000 30,000 (Continued)

Modified Date: Sat, Jun 26, 2010 02:55:02 PM

Output Date: Tue, Jul 06, 2010 12:17:35 PM

Rev-II

Project: Management Accounting_Debarshi Bhattacharyya ACE Pro India Pvt. Ltd. File: X:\Pearson\Management Accounting_Debarshi Bhattacharyya\MAIN\M06\LAYOUT_M06\M06_DEBA_ISBN_EN_SE_C06.indd

440

MANAGEMENT ACCOUNTING

Particulars In Finished Stock In Debtors Less: In Creditors Net Block Expenses: In WIP In Finished Stock In Debtors Net Block Profit: In Debtors

Period of Block (Months) 4

Amount Blocked per Month 15,000

2.5 9.5 1.5 8

15,000

2

4,500

4 2.5 8.5

9,000 9,000

2.5

6,000

Raw Materials

WIP

Rs.

Rs.

Finished Goods

Sundry Debtors

Sundry Creditors

Total

Rs. 60,000

Rs.

Rs.

Rs.

37,500

15,000

(22,500) 1,20,000 9,000 36,000 22,500 67,500

15,000

39,000

96,000

Add: Cash in hand Less: Bank Overdraft Working Capital

15,000 75,000

(22,500)

15,000 2,02,500 6,000 2,08,500 1,00,000 1,08,500

Working Notes 1. Annual Expected Sales = Rs. 3,60,000. Expected Monthly Sales = 3,60,000 ÷ 12 = Rs. 30,000. 2. Estimated Cost Sheet for a Month Raw Materials (50% of Rs. 30,000) Expenses (30% of Rs. 30,000) Total Cost Profit (20% of Rs. 30,000) Selling Price

Rs. 15,000 9,000 24,000 6,000 30,000

Notes 1. Here, the Working Capital Requirement with profit has been ascertained. 2. Stock of WIP is valued on the assumption that it is completed to the extent of (i.e., it consists of ) 100% of raw materials and 50% of expenses. That is why, the amount of expenses blocked per month at the WIP stage is taken at 50% (i.e., instead of Rs. 9,000, taken at Rs. 4,500).

Problem 9 From the following information, prepare a statement showing the estimated Working Capital Requirement: i. Projected annual sales 36,000 units ii. Analysis of sales: Rs. Raw Materials 6 per unit

Modified Date: Sat, Jun 26, 2010 02:55:02 PM

Output Date: Tue, Jul 06, 2010 12:17:35 PM

Rev-II

Project: Management Accounting_Debarshi Bhattacharyya ACE Pro India Pvt. Ltd. File: X:\Pearson\Management Accounting_Debarshi Bhattacharyya\MAIN\M06\LAYOUT_M06\M06_DEBA_ISBN_EN_SE_C06.indd

WORKING CAPITAL

441

Labour 4 per unit Overheads 3 per unit Profit 2 per unit Selling Price 15 per unit iii. Additional information: a. Raw Materials in Stock 1 month b. Production Process 2 months c. Finished Goods in Store 3 months d. Credit allowed to Debtors 4 months e. Credit allowed by Suppliers 2 monthsw f. Monthly Wages and Expenses are paid twice on 1st and 16th of each month. g. Production is carried out evenly during the year, and Expenses and Wages accrue similarly. iv. Cash is to be kept at 10% of the Net Working Capital. [B.Com. (Hons), Calcutta Univeristy—2006] Solution Statement showing the estimated Working Capital (with profit) requirement

Particulars

Raw Materials: In Stores In WIP In Finished Stock In Debtors Less: In Creditors Net Block Wages: In WIP In Finished Stock In Debtors Less: Lag in payment Net Block Overheads: In WIP In Finished Stock In Debtors Less: Lag in payment Net Block Profit: In Debtors

Amount Raw Blocked Materiper Period als of Block Month Rs. Rs. (Months) 1 2 3

18,000 18,000 18,000

4 10 2 8

18,000

2 3

6,000 12,000

4 9 0.5 8.5

12,000

2 3

4,500 9,000

4 9 0.5 8.5

9,000

4

6,000

Outstanding OutFinished Sundry Sundry standing Overheads Total WIP Goods Debtors Creditors Wages Rs.

Rs.

Rs.

Rs.

Rs.

Rs.

Rs.

18,000 36,000 54,000 72,000

18,000

(36,000) 1,44,000 12,000 36,000 48,000

12,000

(6,000) 90,000 9,000 27,000 36,000

9,000

(4,500) 67,500

18,000

57,000

1,17,000

24,000 1,80,000

Add: Cash in hand (10 ÷ 90 × 3,25,500) Net Working Capital

Modified Date: Sat, Jun 26, 2010 02:55:02 PM

(36,000)

(6,000)

(4,500)

24,000 3,25,500 36,167 3,61,667

Output Date: Tue, Jul 06, 2010 12:17:35 PM

Rev-II

Project: Management Accounting_Debarshi Bhattacharyya ACE Pro India Pvt. Ltd. File: X:\Pearson\Management Accounting_Debarshi Bhattacharyya\MAIN\M06\LAYOUT_M06\M06_DEBA_ISBN_EN_SE_C06.indd

442

MANAGEMENT ACCOUNTING

 Working Notes 1. Projected Annual Sales = 36,000 units. Projected Monthly Sales = 36,000 ÷ 12 = 3,000 units. 2. Estimated Cost Sheet for a Month Raw Materials (3,000 × Rs. 6) Labour (3,000 × Rs. 4) Overheads (3,000 × Rs. 3) Total Cost Profit (3,000 × Rs. 2) Selling Price (3,000 × Rs. 15)

Rs. 18,000 12,000 9,000 39,000 6,000 45,000

Notes 1. Here, the Working Capital Requirement with profit has been ascertained. 2. Stock of WIP is valued on the assumption that it is completed to the extent of (i.e., it consists of ) 100% of raw materials, 50% of wages and 50% of overheads. That is why, the amount of wages and overheads blocked per month at WIP stage are taken at 50% (i.e., instead of Rs. 12,000, taken at Rs. 6,000, and instead of Rs. 9,000, taken at Rs. 4,500, respectively)

Problem 10 The production capacity of Quick Profit Ltd is 5,20,000 units per annum. Due to power crisis, the company can operate at 80% of the capacity level. You are asked to ascertain the Working Capital Requirement at the current level of operation. Add 10% to your computed figure to allow for contingencies. Other information: i. Selling Price – Rs. 20 per unit. ii. Elements of Cost (per unit): a. Raw Materials – 40% of selling price. b. Labour – 30% of selling price. c. Budgeted Overhead – Rs. 32,000 per week. Overhead includes depreciation of Rs. 8,000 per week. iii. Planned Stock will include 24,000 units of the finished goods. iv. Time-lag Information: a. Raw Materials in stores – 3 weeks. b. Materials will stay in Process – 2 weeks. c. Credit allowed to Debtors – 5 weeks. d. Credit allowed by Creditors – 1 month. e. Lag in Payment of Wages and Overhead – 15 days. 25% of sales may be considered to be for cash. Assume that the production is carried on evenly throughout the year, and wages and overhead accrue similarly. A time period of 52 weeks is equivalent to a year and a month comprises 4 weeks. [B.Com. (Hons), Calcutta University—2004]

Modified Date: Sat, Jun 26, 2010 02:55:02 PM

Output Date: Tue, Jul 06, 2010 12:17:35 PM

Rev-II

Modified Date: Sat, Jun 26, 2010 02:55:02 PM

Output Date: Tue, Jul 06, 2010 12:17:35 PM

Add:

10% for contingencies (10% of 10,78,000) Net Working Capital

Less: Lag in payment Net Block Profit: In Debtors

Less: Lag in payment Net Block Overheads: In WIP In Finished Stock In Debtors

Less: In Creditors Net Block Wages: In WIP In Finished Stock In Debtors

Raw Materials: In Stores In WIP In Finished Stock In Debtors

Particulars

Solution

(24,000)

(12,000)

Rs.

5

12,000

24,000

24,000 24,000 18,000

48,000

48,000 48,000 36,000

64,000

1,92,000

2,00,000

24,000

48,000

4,08,000

72,000

1,44,000

60,000 5,70,000

90,000

1,80,000

(2,56,000)

(24,000)

(12,000)

11,85,800

60,000 10,78,000 1,07,800

1,74,000

(2,56,000)

Rs.

1 3 5 9 0.5 8.5

2,40,000

Rs.

Total

3,48,000

1,92,000

Rs.

Rs.

Rs.

Outstand- Outstanding ing Overheads Wages

1 3 5 9 0.5 8.5

1,28,000

Sundry Creditors

Sundry Debtors

Finished Goods

4,96,000

1,92,000

Rs.

Rs.

Rs. 64,000 64,000 64,000 48,000

WIP

Raw Materials

Amount Blocked per Week

3 2 3 5 13 4 9

Period of Block (weeks)

Statement showing the Working Capital (with profit) Requirement

Project: Management Accounting_Debarshi Bhattacharyya ACE Pro India Pvt. Ltd. File: X:\Pearson\Management Accounting_Debarshi Bhattacharyya\MAIN\M06\LAYOUT_M06\M06_DEBA_ISBN_EN_SE_C06.indd

Rev-II

Project: Management Accounting_Debarshi Bhattacharyya ACE Pro India Pvt. Ltd. File: X:\Pearson\Management Accounting_Debarshi Bhattacharyya\MAIN\M06\LAYOUT_M06\M06_DEBA_ISBN_EN_SE_C06.indd

444

MANAGEMENT ACCOUNTING

 Working Notes 1. Full production capacity (i.e., 100%) = 5,20,000 units per year. Estimated annual production at 80% current capacity level = 5,20,000 × 80 ÷ 100 = 4,16,000 units. Estimated weekly production at the current level of operation = 4,16,000 ÷ 52 = 8,000 units. 2. Estimated Cost Sheet for a Week

Raw Materials [8,000 × (40% of Rs. 20)] Labour [8,000 × (30% of Rs. 20)] Overheads: Depreciation Other than depreciation (32,000 − 8,000) Total Cost Profit (bal. fig.) Selling Price (8,000 × Rs. 20)

For Total Sales Rs. Rs. 64,000 48,000 8,000 24,000

32,000 1,44,000 16,000 1,60,000

For 75% Credit Sales Rs. Rs. 48,000 36,000 6,000 18,000

24,000 1,08,000 12,000 1,20,000

3. Calculation of Finished Goods Storage Period Stock of Finished Goods in units Production per week 24,000 units = 8,000 units = 3 weeks

Finished Goods Storage Period =

Alternative Calculation: Total Cost for production of 8,000 units per week = Rs. 1,44,000. Total Cost per unit = Rs. 1,44,000 ÷ 8,000 = Rs. 18. Value of Stock of Finished Goods of 24,000 units = 24,000 × Rs. 18 = Rs. 4,32,000. Stock of Finished Goods Total Cost per week Rs. 4,32,000 = Rs. 1,44,000 = 3 weeks

Finished Goods Storage Period =

Notes 1. Here, the Working Capital Requirement with profit has been determined. 2. Stock of WIP is valued on the assumption that it is completed to the extent of (i.e., it consists of) 100% of raw materials, 50% of wages and 50% of overheads. That is why, the period of block for wages and overheads at WIP stage are taken at 50% (i.e., instead of 2 weeks, taken at 1 week), considering the amount blocked per month in full.

Problem 11 A company is presently operating at 60% of its capacity of production and produces 45,000 units of a product annually. In 2010, the company decides to operate at 50% of its capacity at the following Cost Price structure and periods of block:

Modified Date: Sat, Jun 26, 2010 02:55:02 PM

Output Date: Tue, Jul 06, 2010 12:17:35 PM

Rev-II

Project: Management Accounting_Debarshi Bhattacharyya ACE Pro India Pvt. Ltd. File: X:\Pearson\Management Accounting_Debarshi Bhattacharyya\MAIN\M06\LAYOUT_M06\M06_DEBA_ISBN_EN_SE_C06.indd

WORKING CAPITAL

i. Estimated Selling Price per unit Raw Materials per unit Wages per unit Overhead per unit

445

Rs. 20.00 Rs. 7.00 Rs. 4.00 Rs. 4.00

ii. 80% of sales are made on credit and 25% of purchases of materials are made in cash. iii. Raw materials are kept in stores for 2 months, the processing time is 8 weeks and the finished goods are sold after 1 month. iv. Debtors are allowed a credit period of 90 days and a credit of 60 days is received from the creditors. v. Lag in payment of wages is 1 month and overhead is 2 weeks. Prepare a statement showing the Working Capital Requirement for the year 2010. Solution Statement showing the requirement of Working Capital (with profit) for the year 2010 Particulars Raw Materials: In Stores In WIP In Finished Stock In Credit to Debtors Less:

Less:

Less:

Credit from Creditors Net Block Wages: In WIP In Finished Stock In Credit to Debtors Lag in payment Net Block Overheads: In WIP In Finished Stock In Credit to Debtors Lag in payment Net Block Profit: In Credit to Debtors Working Capital Requirement

Period of Block (Months)

Amount Blocked per Month (Rs.)

2 2 1 3 8 2 6

35,000 35,000 35,000 28,000

1 1 3 5 1 4

20,000 20,000 16,000

1 1 3 5 0.5 4.5

20,000 20,000 16,000

3

20,000

26,250

Amount (Rs.)

Total (Rs.)

70,000 70,000 35,000 84,000 2,59,000 (52,500) 2,06,500

20,000

20,000 20,000 48,000 88,000 (20,000) 68,000

20,000

20,000 20,000 48,000 88,000 (10,000) 78,000 60,000

60,000 4,12,500

Working Notes 1. Present production at 60% capacity = 45,000 units per year. Estimated annual production at 80% capacity level = 45,000 × 80 ÷ 60 = 60,000 units. Estimated monthly production at 80% capacity level = 60,000 ÷ 12 = 5,000 units. (Continued)

Modified Date: Sat, Jun 26, 2010 02:55:02 PM

Output Date: Tue, Jul 06, 2010 12:17:35 PM

Rev-II

Project: Management Accounting_Debarshi Bhattacharyya ACE Pro India Pvt. Ltd. File: X:\Pearson\Management Accounting_Debarshi Bhattacharyya\MAIN\M06\LAYOUT_M06\M06_DEBA_ISBN_EN_SE_C06.indd

446

MANAGEMENT ACCOUNTING

2. Estimated Cost Sheet for a Month in 2010 For Total Sales Rs. 35,000 20,000 20,000 75,000 25,000 1,00,000

Raw Materials (5,000 × Rs. 7) Wages (5,000 × Rs. 4) Overheads (5,000 × Rs. 4) Total Cost Profit (bal. fig.) Selling Price (5,000 × Rs. 20)

For 80% Credit For 75% Credit Sales Purchases Rs. Rs. 28,000 26,250 16,000 16,000 60,000 20,000 80,000

1. Here, the Working Capital Requirement with profit has been determined.

Stop and Think 2. Stock of WIP is valued on the assumption that it is completed to the extent of (i.e., it consists of ) 100% of raw materials, 50% of wages and 50% of overheads. That is why, the period of block for wages and overheads at WIP stage are taken at 50% (i.e., instead of 2 weeks, taken at 1 week), considering the amount blocked per month in full. 3. As 80% of sales are made on credit, the Working Capital blocked in debtors is taken at 80%. 4. As 75% of purchases are made on credit, the credit received from creditors is taken at 75%.

Problem 12 P&G Ltd furnishes as follows its cost and other data: Unit Cost

Rs. 30 20 ? ? 100

Raw Materials Direct labour Overheads Profit (25% on Selling Price) Unit selling price

Additional information are as follows: Average raw material: – in stock 1 month – in process 15 days Credit allowed by suppliers 1.5 months Credit allowed to debtors 3 months Time lag in payment: Wages 15 days Overhead 1 month Cash balance is assumed to be Rs. 1,00,000 and 70% of sales are credit sales. Assuming that production is carried on evenly throughout the year, you are required to determine the Working Capital Requirement of P&G Ltd, to achieve an output level of 1,20,000 units. [B.Com. (Hons), Calcutta University—2008]

Modified Date: Sat, Jun 26, 2010 02:55:02 PM

Output Date: Tue, Jul 06, 2010 12:17:35 PM

Rev-II

Project: Management Accounting_Debarshi Bhattacharyya ACE Pro India Pvt. Ltd. File: X:\Pearson\Management Accounting_Debarshi Bhattacharyya\MAIN\M06\LAYOUT_M06\M06_DEBA_ISBN_EN_SE_C06.indd

WORKING CAPITAL

447

Solution Statement showing the requirement of Working Capital (with profit) for the year 2010 Particulars Raw Materials: In Stores In WIP In Credit to Debtors Less:

Less:

Less:

Add:

Credit from Creditors Net Block Direct Labour: In WIP In Credit to Debtors Lag in payment Net Block Overheads: In WIP In Credit to Debtors Lag in payment Net Block Profit: In Credit to Debtors

Period of Block (Months)

Amount Blocked per Month (Rs.)

1 0.5 3 4.5 1.5 3

3,00,000 3,00,000 2,10,000

0.5 3 3.5 0.5 3

1,00,000 1,40,000

0.5 3 3.5 1 2.5

1,25,000 1,75,000

3

1,75,000

3,00,000

Amount (Rs.)

Total (Rs.)

3,00,000 1,50,000 6,30,000 10,80,000 (4,50,000) 6,30,000

2,00,000

50,000 4,20,000 4,70,000 (1,00,000) 3,70,000

2,50,000

62,500 5,25,000 5,87,500 (2,50,000) 3,37,500

Cash Balance Working Capital Requirement

5,25,000

5,25,000 18,62,500 1,00,000 19,62,500

Working Notes 1. Estimated annual production = 1,20,000 units. Estimated monthly production = 1,20,000 ÷ 12 = 10,000 units. 2. Estimated Cost Sheet for a Month

Raw Materials (10,000 × Rs. 30) Direct Labour (10,000 × Rs. 20) Overheads (10,000 × Rs. 25) (bal. fig.) Total Cost Profit (25% on Rs. 10,00,000) Selling Price (10,000 × Rs. 100)

For Total Sales Rs. 3,00,000 2,00,000 2,50,000 7,50,000 2,50,000 10,00,000

For 70% Credit Sales Rs. 2,10,000 1,40,000 1,75,000 5,25,000 1,75,000 7,00,000

Stop and Think 1. Here, the Working Capital Requirement with profit has been determined. 2. Stock of WIP is valued on the assumption that it is completed to the extent of (i.e., it consists of ) 100% of raw materials, 50% of wages and 50% of overheads. That is why, the amount of wages and overheads blocked (Continued)

Modified Date: Sat, Jun 26, 2010 02:55:02 PM

Output Date: Tue, Jul 06, 2010 12:17:35 PM

Rev-II

Project: Management Accounting_Debarshi Bhattacharyya ACE Pro India Pvt. Ltd. File: X:\Pearson\Management Accounting_Debarshi Bhattacharyya\MAIN\M06\LAYOUT_M06\M06_DEBA_ISBN_EN_SE_C06.indd

448

MANAGEMENT ACCOUNTING

per month at WIP stage are taken at 50% (i.e., instead of Rs. 1,00,000, taken at Rs. 2,00,000, and instead of Rs. 2,50,000, taken at Rs. 1,25,000, respectively), considering the full period of block at this stage. 3. As 70% of sales are made on credit, the Working Capital blocked in debtors is taken at 70%.

Problem 13 Determine the Working Capital Requirement from the following particulars: Annual Budget Figures: Rs. in Lakhs 480 240 180 900 1,000

Raw Materials Direct Wages Overheads Sales

Additional information: i. Average stock level of raw materials – 18 days. ii. Credit sales – 20 days’ credit is normal. iii. Finished goods are held in stock for a period of 10 days before they are released for sale. iv. Process period is for 12 days. v. The company enjoys 30 days’ credit facilities for the purchase. vi. Estimated cash and bank balance: 10% of the total Working Capital. Assumptions: i. 1 year = 360 days. ii. Raw materials are introduced at the beginning of the manufacturing process, and labour and overhead accrue evenly. [B.Com. (Hons), Calcutta University—2009] Solution Statement showing the requirement of Working Capital (with profit) for the year 2010 Particulars Raw Materials: In Stores In WIP In Finished Stock In Credit to Debtors Less:

Credit from Creditors Net Block Wages: In WIP In Finished Stock In Credit to Debtors Net Block

Amount Blocked per Day

Amount

Total

(Rs. in Lakhs)

(Rs. in Lakhs)

(Rs. in Lakhs)

18 12 10 20 60 30 30

1.33 1.33 1.33 1.33

23.94 15.96 13.30 26.60 79.80 (39.90)

6 10 20 36

0.67 0.67 0.67

Period of Block (days)

1.33

39.90 4.02 6.70 13.40 24.12 (Continued)

Modified Date: Sat, Jun 26, 2010 02:55:02 PM

Output Date: Tue, Jul 06, 2010 12:17:35 PM

Rev-II

Project: Management Accounting_Debarshi Bhattacharyya ACE Pro India Pvt. Ltd. File: X:\Pearson\Management Accounting_Debarshi Bhattacharyya\MAIN\M06\LAYOUT_M06\M06_DEBA_ISBN_EN_SE_C06.indd

WORKING CAPITAL

Particulars

Add:

Amount Blocked per Day

Amount

Total

(Rs. in Lakhs)

(Rs. in Lakhs)

(Rs. in Lakhs)

6 10 20 36

0.50 0.50 0.50

3.00 5.00 10.00

20

0.27

Period of Block (days)

Overheads: In WIP In Finished Stock In Credit to Debtors Net Block Profit: In Credit to Debtors Working Capital (without cash) Estimated Cash & Bank Balance (10 ÷ 90 × 87.42) Total Working Capital Requirement

449

18.00 5.40

5.40 87.42 9.71 97.13

Working Notes 1. Estimated Cost Sheet For the Year Rs. in Lakhs 480 240 180 900 100 1,000

Raw Materials Direct Wages Overheads Total Cost Profit (bal. fig.) Sales

Per Day of the Year Rs. in Lakhs 1.33 0.67 0.50 2.50 0.27 2.77

Stop and Think 1. Here, the Working Capital Requirement with profit has been determined. 2. Stock of WIP is valued on the assumption that it is completed to the extent of (i.e., it consists of ) 100% of raw materials, 50% of wages and 50% of overheads. That is why, the period of block for wages and overheads at WIP stage are taken at 50% (i.e., instead of 12 days, taken at 6 days), considering the full amount blocked per day.

Problem 14 A company is currently operating at 60% of its capacity producing 36,000 units during the year 2009 at the following Cost Price structure: Rs. per unit 4.00 3.00 3.00 12.00

Materials Labour Overheads (60% variable) Selling Price

Modified Date: Sat, Jun 26, 2010 02:55:02 PM

Output Date: Tue, Jul 06, 2010 12:17:35 PM

Rev-II

Project: Management Accounting_Debarshi Bhattacharyya ACE Pro India Pvt. Ltd. File: X:\Pearson\Management Accounting_Debarshi Bhattacharyya\MAIN\M06\LAYOUT_M06\M06_DEBA_ISBN_EN_SE_C06.indd

450

MANAGEMENT ACCOUNTING

On 31 December 2009, the company has the following Current Assets and liabilities: Rs. 24,000 21,000 60,000 90,000 30,000 9,000 4,500

Stock of Raw Materials Stock of WIP Stock of Finished Goods Sundry Debtors Sundry Creditors Outstanding Wages Outstanding Overheads

In the year 2010, the company wishes to operate at 80% of its capacity at the same Cost Price structure, period of block and selling price of 2009. Calculate the additional Working Capital Requirement for the year 2010. [B.Com. (Hons), Calcutta University—Adapted] Solution Statement showing the requirement of Working Capital (with profit) for the year 2010

Particulars Raw Materials: In Stores In WIP In Finished Stock In Credit to Debtors Less:

Credit from Creditors Net Block Wages: In WIP In Finished Stock In Credit to Debtors

Less:

Lag in payment Net Block Overheads: In WIP In Finished Stock In Credit to Debtors

Less:

Lag in payment Net Block

Period of Block (Months) 2 1 2 2.5 7.5 2.5 5

16,000 16,000 16,000 16,000

1 2 2.5 5.5 1 4.5

6,000 12,000 12,000

1 2 2.5 5.5 0.5 5

5,400 10,800 10,800

2.5

9,200

Profit: In Credit to Debtors Working Capital Requirement

Modified Date: Sat, Jun 26, 2010 02:55:02 PM

Amount Blocked per Month (Rs.)

Output Date: Tue, Jul 06, 2010 12:17:35 PM

16,000

Amount (Rs.)

Total (Rs.)

32,000 16,000 32,000 40,000 1,20,000 (40,000) 80,000

12,000

6,000 24,000 30,000 60,000 (12,000) 48,000

10,800

5,400 21,600 27,000 54,000 (5,400) 48,600 23,000

23,000 1,99,600

Rev-II

Project: Management Accounting_Debarshi Bhattacharyya ACE Pro India Pvt. Ltd. File: X:\Pearson\Management Accounting_Debarshi Bhattacharyya\MAIN\M06\LAYOUT_M06\M06_DEBA_ISBN_EN_SE_C06.indd

WORKING CAPITAL

451

Statement showing the computation of additional Working Capital requirement for the year 2010

Less:

Rs. 1,99,600 1,51,500 48,100

Working Capital Requirement for the year 2010 as per above statement Working Capital as on 31 December 20093 Additional Working Capital Requirement for the year 2010

Working Notes 1. Annual production for the year 2009 at 60% capacity level = 36,000 units. Monthly production for the year 2009 at 60% capacity level = 36,000 ÷ 12 = 3,000 units. 2. Cost Sheet for a Month of 2009 Rs. Raw Materials (3,000 units × Rs. 4) Labour (3,000 units × Rs. 3) Overheads: Fixed [3,000 units × (40% of Rs. 3)] Variable [3,000 units × (60% of Rs. 3)] Total Cost Profit (bal. fig.) Selling Price (3,000 units × Rs. 12)

3,600 5,400

Rs. 12,000 9,000

9,000 30,000 6,000 36,000

3. Computation of Working Capital as on 31 December 2009 and Period of Block for 2009 Elements of Working Capital Current Assets as on 31 December 2009: Stock of Raw Materials

Rs.

Rs.

24,000

Period of Block

Value of stock of raw materials Raw materials consumed per month Rs. 24,000 = 2 months Rs. 12,000

= Stock of WIP

21,000

Value of stock of WIP 100% of raw materials + 50% of labour & overheads per month =

Rs. 21,000 100% of Rs. 12,000 + 50% of Rs. 9,000 + 50% of Rs. 9,000

= 1 month Stock of Finished Goods

60,000

Value of stock of finished goods Total cost of production per month =

Sundry Debtors

90,000

Rs. 60,000 = 2 months Rs. 30,000

Value of sundry debtors Credit sales per month =

Rs. 90,000 = 2.5 months Rs. 36,000

1,95,000 (Continued)

Modified Date: Sat, Jun 26, 2010 02:55:02 PM

Output Date: Tue, Jul 06, 2010 12:17:35 PM

Rev-II

Project: Management Accounting_Debarshi Bhattacharyya ACE Pro India Pvt. Ltd. File: X:\Pearson\Management Accounting_Debarshi Bhattacharyya\MAIN\M06\LAYOUT_M06\M06_DEBA_ISBN_EN_SE_C06.indd

452

MANAGEMENT ACCOUNTING

Elements of Working Capital Less: Current Liabilities as on 31 December 2009: Sundry Creditors

Rs.

Rs.

30,000

Period of Block

Value of sundry creditors Credit purchases of materials per month =

Rs. 30,000 = 2.5 months Rs. 12,000

Outstanding Wages

9,000

Outstanding wages Rs. 9,000 = = 1 month Wages per month Rs. 9,000

Outstanding Overheads

4,500

Outstanding overheads Rs. 4,500 = = 0.5 month Total overheads per month Rs. 9,000 43,500

Working Capital

1,51,500

4. Monthly production for the year 2010 at 80% capacity level = 3,000 × 80% / 60% = 4,000 units. 5. Estimated Cost Sheet for a Month of 2010 Rs. Raw Materials (4,000 units × Rs. 4) Labour (4,000 units × Rs. 3) Overheads: Fixed (same as in 2009) Variable [4,000 units × (60% of Rs. 3)] Total Cost Profit (bal. fig.) Selling Price (4,000 units × Rs. 12)

3,600 7,200

Rs. 16,000 12,000

10,800 38,800 9,200 48,000

Stop and Think 1. Here, the Working Capital requirement with profit has been determined. 2. Stock of WIP is valued on the assumption that it is completed to the extent of (i.e., it consists of ) 100% of raw materials, 50% of wages and 50% of overheads. That is why, the amount of wages and overheads blocked per month at the WIP stage are taken at 50% (i.e., instead of Rs. 1,00,000, taken at Rs. 2,00,000, and instead of Rs. 2,50,000, taken at Rs. 1,25,000, respectively), considering the full period of block at this stage.

Problem 15 A company is currently operating at 60% of its capacity producing 24,000 units during the year 2009 at the following Cost Price structure: Rs. Raw Materials per unit Wages per unit Overheads per unit: Variable Fixed Profit per unit Selling Price per unit

Modified Date: Sat, Jun 26, 2010 02:55:02 PM

2.00 1.00

Output Date: Tue, Jul 06, 2010 12:17:35 PM

Rs. 5.00 3.00

3.00 2.00 13.00

Rev-II

Project: Management Accounting_Debarshi Bhattacharyya ACE Pro India Pvt. Ltd. File: X:\Pearson\Management Accounting_Debarshi Bhattacharyya\MAIN\M06\LAYOUT_M06\M06_DEBA_ISBN_EN_SE_C06.indd

WORKING CAPITAL

453

On 31 December 2009, the company has the following Current Assets and liabilities: Rs. 20,000 8,000 33,000 78,000 30,000 3,000 6,000

Stock of Raw Materials—4,000 units at cost Stock of WIP—1,000 units at cost Stock of Finished Goods—3,000 units at cost Sundry Debtors Sundry Creditors Liability for Wages Liability for Overheads

In view of the increased demand for the product, it has been decided to operate at 90% capacity on and from 01 January 2010 at the same Cost Price structure, period of block and Selling Price of 2009. Calculate the additional Working Capital Requirement for the year 2010. [B.Com. (Hons), Calcutta University—Adapted] Solution Statement showing the requirement of Working Capital (with profit) for the year 2010 Particulars Raw Materials: In Stores In WIP In Finished Stock In Credit to Debtors Less:

Less:

Less:

Credit from Creditors Net Block Wages: In WIP In Finished Stock In Credit to Debtors Lag in payment Net Block Overheads: In WIP In Finished Stock In Credit to Debtors Lag in payment Net Block Profit: In Credit to Debtors Working Capital Requirement

Period of Block (Months)

Amount Blocked per Month (Rs.)

2 0.5 1.5 3 7 3 4

15,000 15,000 15,000 15,000

0.25 1.5 3 4.75 0.5 4.25

9,000 9,000 9,000

0.25 1.5 3 4.75 1 3.75

8,000 8 ,000 8,000

3

7,000

15,000

Amount (Rs.)

Total (Rs.)

30,000 7,500 22,500 45,000 1,05,000 (45,000) 60,000

9,000

2,250 13,500 27,000 42,750 (4,500) 38,250

8,000

2,000 12,000 24,000 38,000 (8,000) 30,000 21,000

21 ,000 1,49,250

Statement showing the computation of additional Working Capital requirement for the year 2010 Working Capital Requirement for the year 2010 as per above statement Less: Working Capital as on 31 December 20093 Additional Working Capital Requirement for the year 2010

Modified Date: Sat, Jun 26, 2010 02:55:02 PM

Output Date: Tue, Jul 06, 2010 12:17:35 PM

Rs. 1,49,250 1,00,000 49,250

Rev-II

Project: Management Accounting_Debarshi Bhattacharyya ACE Pro India Pvt. Ltd. File: X:\Pearson\Management Accounting_Debarshi Bhattacharyya\MAIN\M06\LAYOUT_M06\M06_DEBA_ISBN_EN_SE_C06.indd

454

MANAGEMENT ACCOUNTING

Working Notes 1. Annual production for the year 2009 at 60% capacity level = 24,000 units. Monthly production for the year 2009 at 60% capacity level = 24,000 ÷ 12 = 2,000 units. 2. Cost Sheet for a Month of 2009 Rs. Raw Materials (2,000 units × Rs. 5) Wages (2,000 units × Rs. 3) Overheads: Fixed (2,000 units × Re. 1) Variable (2,000 units × Rs. 2) Total Cost Profit (2,000 × Rs. 2) Selling Price (2,000 units × Rs. 13)

2,000 4,000

Rs. 10,000 6,000

6,000 22,000 4,000 26,000

3. Computation of Working Capital as on 31 December 2009 and period of block for 2009 Elements of Working Capital Current Assets as on 31 December 2009:

Rs.

Period of Block

20,000

Units of stock of raw materials Monthly production units 4,000 units = 2,000 units = 2 months

8,000

Units of stock of WIP Monthly production units 1,000 units = 2,000 units = 0.5 month.

Stock of Finished Goods

33,000

Units of stock of finished goods Monthly production units 3,000 units = 2,000 units = 1.5 months.

Sundry Debtors

78,000

Value of sundry debtors Credit sales per month Rs. 78,000 = Rs. 26,000 = 3 months.

Stock of Raw Materials

Stock of WIP

Rs.

1,39,000 (Continued)

Modified Date: Sat, Jun 26, 2010 02:55:02 PM

Output Date: Tue, Jul 06, 2010 12:17:35 PM

Rev-II

Project: Management Accounting_Debarshi Bhattacharyya ACE Pro India Pvt. Ltd. File: X:\Pearson\Management Accounting_Debarshi Bhattacharyya\MAIN\M06\LAYOUT_M06\M06_DEBA_ISBN_EN_SE_C06.indd

WORKING CAPITAL

Elements of Working Capital Less: Current Liabilities as on 31 December 2009: Sundry Creditors

Rs.

Rs.

455

Period of Block

30,000

Value of sundry creditors Credit purchases of materials per month Rs. 30,000 = Rs. 10,000 = 3 months.

Liability for Wages

3,000

Liability for wages Rs. 3,000 = = 0.5 month Wages per month Rs. 6,000

Liability for Overheads

6,000

Liability for overheads Rs. 6,000 = Total overheads per month Rs. 6,000 = 1 month. 39,000

Working Capital

1,00,000

4. Monthly production for the year 2010 at 90% capacity level = 2,000 ×

90% = 3,000 units. 60%

5. Estimated Cost Sheet for a Month of 2010 Rs. Raw Materials (3,000 units × Rs. 5) Wages (3,000 units × Rs. 3) Overheads: Fixed (same as in 2009) Variable (3,000 units × Rs. 2) Total Cost Profit (bal. fig.) Selling Price (3,000 units × Rs. 13)

2,000 6,000

Rs. 15,000 9,000

8,000 32,000 7,000 39,000

Stop and Think 1. Here, the Working Capital Requirement with profit has been determined. 2. Stock of WIP is valued on the assumption that it is completed to the extent of (i.e., it consists of) 100% of raw materials, 50% of wages and 50% of overheads. That is why, the period of block for wages and overheads at WIP stage are taken at 50% (i.e., instead of 0.50 month, taken at 0.25 month), considering the full amount blocked per month.

Modified Date: Sat, Jun 26, 2010 02:55:02 PM

Output Date: Tue, Jul 06, 2010 12:17:35 PM

Rev-II

Project: Management Accounting_Debarshi Bhattacharyya ACE Pro India Pvt. Ltd. File: X:\Pearson\Management Accounting_Debarshi Bhattacharyya\MAIN\M06\LAYOUT_M06\M06_DEBA_ISBN_EN_SE_C06.indd

456

MANAGEMENT ACCOUNTING

Problem 16 Jadukadai Ltd has been operating on a single shift working till 31 December 2009 at the following Cost Price structure: Rs. per unit 6.00 5.00 5.00 2.00 18.00

Materials Wages (40% fixed) Overheads (20% variable) Profit Selling Price

Sales during the year 2009 amount to Rs. 4,32,000. On 31 December 2009, the company has the following Current Assets: Rs. 36,000 22,000 80,000 1,08,000

Stock of Raw Materials at Cost Stock of WIP at Cost Stock of Finished Goods at Cost Sundry Debtors

In view of the increased demand, it is proposed to double the production in the year 2010 by working an extra shift. It is expected that a 10% discount will be available from the suppliers of raw materials. There will be no change in the selling price per unit and variable cost per unit. The credit availed from suppliers and the lag in payment of wages and overheads will continue to remain at 2 months, 15 days and 15 days, respectively. Other period of block will also continue to remain at the present level. You are required to calculate the additional Working Capital Requirement if the policy to increase the output is implemented in the year 2010. Solution Statement showing the requirement of Working Capital (with profit) for the year 2010 Particulars Raw Materials: In Stores In WIP In Finished Stock In Credit to Debtors Less:

Less:

Credit from Creditors Net Block Wages: In WIP In Finished Stock In Credit to Debtors Lag in payment Net Block

Period of Block (Months)

Amount Blocked per Month (Rs.)

Amount (Rs.)

3 1 2.5 3

21,600 10,800 21,600 21,600

64,800 10,800 54,000 64,800

9.5 2 7.5

21,600

1,94,400 (43,200)

0.5 2.5 3

8,000 16,000 16,000

4,000 40,000 48,000

6 0.5 5.5

16,000

92,000 (8,000)

Total (Rs.)

1,51,200

84,000 (Continued)

Modified Date: Sat, Jun 26, 2010 02:55:02 PM

Output Date: Tue, Jul 06, 2010 12:17:35 PM

Rev-II

Project: Management Accounting_Debarshi Bhattacharyya ACE Pro India Pvt. Ltd. File: X:\Pearson\Management Accounting_Debarshi Bhattacharyya\MAIN\M06\LAYOUT_M06\M06_DEBA_ISBN_EN_SE_C06.indd

WORKING CAPITAL

Particulars Overheads: In WIP In Finished Stock In Credit to Debtors Less:

Lag in payment Net Block Profit: In Credit to Debtors Working Capital Requirement

Period of Block (Months)

Amount Blocked per Month (Rs.)

0.5 2.5 3 6 0.5 5.5

6,000 12,000 12,000

3

22,400

12,000

Amount (Rs.)

457

Total (Rs.)

3,000 30,000 36,000 69,000 (6,000) 63,000 67,200

67,200 3,65,400

Statement showing the computation of additional Working Capital requirement for the year 2010 Rs. 3,65,400 2,12,000 1,53,400

Working Capital Requirement for the year 2010 as per above statement Less: Working Capital as on 31 December 20093 Additional Working Capital Requirement for the year 2010

 Working Notes 1. Production and sales quantity for the year 2009 = Rs. 4,32,000 ÷ Rs. 18 = 24,000 units. Monthly production and sales for the year 2009 = 24,000 ÷ 12 = 2,000 units. 2. Cost Sheet for a Month of 2009 Rs. Raw Materials (2,000 units × Rs. 6) Wages: Fixed [2,000 units × (40% of Rs. 5)] Variable [2,000 units × (60% of Rs. 5)] Overheads: Fixed [2,000 units × (80% of Rs. 5)] Variable [2,000 units × (20% of Rs. 5)] Total Cost Profit (2,000 units × Rs. 2) Selling Price (2,000 units × Rs. 18)

4,000 6,000 8,000 2,000

Rs. 12,000

10,000

10,000 32,000 4,000 36,000

3. Computation of Working Capital as on 31 December 2009 and period of block for 2009 Elements of Working Capital Current Assets as on 31 December 2009: Stock of Raw Materials

Rs.

Rs. 36,000

Period of Block

Value of stock of raw materials Raw materials per month Rs. 36,000 = 3 months = Rs. 12,000

(Continued)

Modified Date: Sat, Jun 26, 2010 02:55:02 PM

Output Date: Tue, Jul 06, 2010 12:17:35 PM

Rev-II

Project: Management Accounting_Debarshi Bhattacharyya ACE Pro India Pvt. Ltd. File: X:\Pearson\Management Accounting_Debarshi Bhattacharyya\MAIN\M06\LAYOUT_M06\M06_DEBA_ISBN_EN_SE_C06.indd

458

MANAGEMENT ACCOUNTING

Elements of Working Capital Stock of WIP

Rs.

Rs. 22,000

Period of Block

Value of stock of WIP 100% of raw materials + 50% of labour & overheads per month Rs. 22,000 = [(100 % of Rs. 12,000) + (50 % of Rs. 10,000 + (50 % of Rs. 10,000)] = 1month

Stock of Finished Goods

80,000 Value of stock of finished goods Total cost of production per month Rs. 80, 000 = = 2.5 months Rs. 32, 000

Sundry Debtors

1,08,000

Value of sundry debtors Credit sales per month Rs. 1,08,000 = = 3 months Rs. 36,000

2,46,000 Less:

Current Liabilities as on 31 December 2009: Sundry Creditors

Period of block × Credit purchases of materials per month = 2 × Rs. 12,000 = Rs. 24,000. Period of block × Total wages per month = 0.5 × Rs. 10,000 = Rs. 5,000. Period of block × Total overheads per month = 0.5 × Rs. 10,000 = Rs. 5,000.

24,000

Outstanding Wages

5,000

Outstanding Overheads

5,000 34,000 2,12,000

Working Capital

4. In 2010, it is proposed to double the production by working an extra shift. Estimated production for the year 2010 = Actual production for 2009 × 2 = 24,000 × 2 = 48,000 units. Estimated monthly production in the year 2010 = 48,000 ÷ 12 = 4,000 units.

5. Estimated Cost Sheet for a Month of 2010 Total Rs. Raw Materials [4,000 units × {Rs. 6 – (10% of Rs. 6)}] Wages: Fixed (same as in 2009) Variable [4,000 units × (60% of Rs. 5)] Overheads: Fixed (same as in 2009) Variable [4,000 units × (20% of Rs. 5)] Total Cost Profit (2,000 units × Rs. 2) Selling Price (4,000 units × Rs. 18)

4,000 12,000 8,000 4,000

Per Unit Rs. 21,600

Rs. 5.40

16,000

4.00

12,000 49,600 22,400 72,000

3.00 12.40 5.60 18.00 (Continued)

Modified Date: Sat, Jun 26, 2010 02:55:02 PM

Output Date: Tue, Jul 06, 2010 12:17:35 PM

Rev-II

Project: Management Accounting_Debarshi Bhattacharyya ACE Pro India Pvt. Ltd. File: X:\Pearson\Management Accounting_Debarshi Bhattacharyya\MAIN\M06\LAYOUT_M06\M06_DEBA_ISBN_EN_SE_C06.indd

WORKING CAPITAL

459

6. Treatment of WIP for a Month of 2010 As the second working shift of a day is started immediately after completion of the first working shift of the day, WIP of the first shift is being converted into the Finished Goods in the second shift of the same day. At the end of a day, only the WIP of the second working shift remains incomplete and not the WIP of the first shift. Therefore, due to working an extra shift in a day, the quantity of WIP does not become double, rather it remains the same as in the case of single working shift. Hence, by working an extra shift in a month of 2010, other things such as Finished Goods and sales, become double (i.e., 2,000 units per month to 4,000 units per month), but WIP remains unchanged (i.e., 2,000 units). Value of Raw Materials included in WIP (2,000 units × Rs. 5.40)

= Rs. 10,800.

Value of wages included in WIP (2,000 units × Rs. 4.00)

= Rs. 8,000.

Value of overheads included in WIP (2,000 units × Rs. 3.00)

= Rs. 6,000.

Stop and Think 1. Here, the Working Capital Requirement with profit has been determined. 2. Stock of WIP is valued on the assumption that it is completed to the extent of (i.e., it consists of ) 100% of raw materials, 50% of wages and 50% of overheads. That is why, the period of block for wages and overheads at WIP stage are taken at 50% (i.e., instead of 1 month, taken at 15 days), considering the full amount blocked per month.

CHAPTER REVIEW SUMMARY  Working Capital refers to the portion of the capital which is employed in the business to carry on its day-to-day activities. It is used by the business to perform its operating activities.  Working Capital is the life-blood of every business concern and accordingly, it has immense importance to every business concern.  To carry on a continuous production process, the Working Capital is essentially required to every business concern till sales realization occurs. Working Capital of a business provides continuous finance for purchasing raw materials and payment of wages and overheads till cash is brought back into the business by way of sales realization.  From the different basis or viewpoints, Working Capital may be classified in different ways, such as gross Working Capital and Net Working Capital; positive Working Capital and negative Working Capital; and permanent Working Capital and temporary Working Capital.  Fixed Working Capital of the business is financed out of the long-term capital employed in the business, whereas temporary Working Capital of the business is arranged out of short-term capital employed in the business.  Nature of the business, size of the business, volume of production and sales, length of operating cycle, production policy, credit policy, and operational efficiency, among other factors, are the determinants of Working Capital Requirement.  Components of Working Capital are Current Assets and Current Liabilities.  Gross Working Capital always becomes positive Working Capital, but Net Working Capital may either be positive or negative Working Capital.  Working Capital cycle or Operating cycle refers to the period that an enterprise takes in converting the cash back into the business from the cash initially invested in various operating activities of the enterprise. The said cycle starts from the cash blocked by way of purchase of raw materials and ends with realization of cash out of sales.  Forecasting of Working Capital refers to the estimation of Working Capital Requirement for a certain future period.

Modified Date: Sat, Jun 26, 2010 02:55:02 PM

Output Date: Tue, Jul 06, 2010 12:17:35 PM

Rev-II

Project: Management Accounting_Debarshi Bhattacharyya ACE Pro India Pvt. Ltd. File: X:\Pearson\Management Accounting_Debarshi Bhattacharyya\MAIN\M06\LAYOUT_M06\M06_DEBA_ISBN_EN_SE_C06.indd

460

MANAGEMENT ACCOUNTING

CHAPTER REVIEW QUIZ 1. Classify the following items into: (i) components of operating Working Capital; (ii) components of nonoperating Working Capital; and (iii) not-at-all Working Capital: a. Stock of WIP. b. Proposed Dividend. c. Outstanding Wages. d. Prepaid Salaries. e. Provision for Taxation. f. Trade Mark. g. Bills Receivable. h. Outstanding Interest on Debentures. i. Bills Payable. j. Pre-received Rent. k. Sundry Debtors. l. Sundry Creditors. m. General Reserve. n. Patents. Ans.: (i): (a), (c), (d), (g), (i), ( j) , (k), (l); (ii): (b), (e), (h); (iii): (f), (m), (n). 2. State whether the following statements are true or false: a. Working Capital refers to the difference between Current Assets and Current Liabilities. b. There is no difference between the concepts of Gross Working Capital and Net Working Capital. c. Working Capital represents that amount of capital of a business which is employed in the business to perform its day-to-day activities. d. Gross Working Capital may be positive or negative. e. Net Working Capital always becomes positive. f. Operating cycle represents the time lag between the cash investment in operating activities and the cash recovery from operating activities. Ans.: True: (a), (c), (f); False: (b), (d), (e). EXERCISE I. Theoretical Questions A. Short Answer Type Questions

1. 2. 3. 4. 5. 6. 7. 8.

What is meant by Working Capital? What are the components of Working Capital? Name five components of Working Capital. What is Gross Working Capital? What is Net Working Capital? What is positive Working Capital? What is negative Working Capital? What is Operating Cycle?

B. Essay Type Questions

1. 2. 3. 4. 5. 6. 7.

What do you mean by Working Capital? What are its needs and importance? Classify Working Capital as per different viewpoints. What are the different sources of Working Capital? What are the determinants of Working Capital? What are the components of Working Capital? What do you mean by Working Capital cycle? What do you mean by estimation of Working Capital? What factors are to be considered for the estimation of Working Capital? 8. What are the procedures for the estimation of Working Capital?

Modified Date: Sat, Jun 26, 2010 02:55:02 PM

Output Date: Tue, Jul 06, 2010 12:17:35 PM

Rev-II

Project: Management Accounting_Debarshi Bhattacharyya ACE Pro India Pvt. Ltd. File: X:\Pearson\Management Accounting_Debarshi Bhattacharyya\MAIN\M06\LAYOUT_M06\M06_DEBA_ISBN_EN_SE_C06.indd

WORKING CAPITAL

461

II. Practical Problems

1. From the following information of Akashbani Ltd, you are required to determine the Working Capital Requirement: Projected annual sales % of profit on cost of sales Average credit period allowed to Debtors Average credit period allowed by Creditors Average stock carrying (in terms of sales requirement) Add 10% to computed figures to allow for contingencies.

Rs. 5,20,000 25% 8 weeks 5 weeks 10 weeks

[M.Com, Calcutta University—Adapted] Ans.: Working Capital—Rs. 1,32,000. 2. GX Ltd is about to commence a new business for which you are asked to calculate its average Working Capital Requirement for the first year with the help of the following information:

Purchase of Raw Materials Wages Overheads: Rent & Rates Salaries to Staff Insurance Selling Expenses Other Overheads Sales: Cash Credit Average amount of Stock & WIP Average amount of undrawn Profit

Average Credit Period 6 weeks 1.5 weeks

Estimated Amount for 1st year (Rs.) 26,00,000 19,50,000

6 months 1 months 3 months 2 weeks 2 months

1,00,000 3,60,000 2,00,000 4,55,000 6,00,000

– 7 weeks – –

1,40,000 65,00,000 3,00,000 3,10,000

Sales are made at an even rate for the year. An overdraft limit of Rs. 1,50,000 has been arranged with the banker. [B.Com. (Hons), Calcutta University—Adapted] Ans.: Net Working Capital—Rs. 1,11,250. 3. From the following information as furnished by Gokul Ltd, forecast the Working Capital Requirement for the year 2010: a. Estimated annual production 60,000 units b. Estimated Selling Price per unit Rs. 20 c. Ratio of Cost to Selling price: Materials 40% Wages 30% Overhead 20% d. Raw materials are kept in stores for 2 months, the processing time is 2 months and the finished goods are sold after 1 month. e. Debtors are allowed a credit period of 3 months and a credit of 8 weeks is received from the creditors. f. Lag in payment of wages and overheads are 4 weeks and 15 days, respectively. Ans.: Working Capital: Without Profit—Rs. 4,50,000; With Profit—Rs. 4,80,000.

Modified Date: Sat, Jun 26, 2010 02:55:02 PM

Output Date: Tue, Jul 06, 2010 12:17:35 PM

Rev-II

Project: Management Accounting_Debarshi Bhattacharyya ACE Pro India Pvt. Ltd. File: X:\Pearson\Management Accounting_Debarshi Bhattacharyya\MAIN\M06\LAYOUT_M06\M06_DEBA_ISBN_EN_SE_C06.indd

462

MANAGEMENT ACCOUNTING

4. Prepare a columnar statement to determine the Working Capital Requirement from the following information: a. Expected Sales b. Analysis of Selling Price: Raw Materials Labour Expenses Profit Selling Price c. Raw Materials in Store d. Processing time e. Finished Goods in Store f. Credit allowed to Debtors g. Credit allowed by Creditors h. Lag in payment of wages and Expenses j. Production is carried on evenly during the year, and Wages and Expenses accrue in the same day itself

7,800 units p.a. Rs. 7 per unit Rs. 5 per unit Rs. 4 per unit Rs. 4 per unit Rs. 20 per unit 3 weeks 2 weeks 4 weeks 2 weeks

[B.Com. (Hons), Calcutta University—1997] Ans.: Working Capital: Without Profit—Rs. 17,250; With Profit—Rs. 19,650. 5. From the following information, ascertain the Working Capital Requirement: (i) Annual production (ii) Analysis of Sales per unit: Raw Materials Wages Overheads Profit Sales (iii) Time lag: Credit allowed to Debtors Credit allowed by Creditors Raw Materials in Store Processing Period Finished Goods in Store Payment of Wages and Overhead (iv) Production has been carried on evenly during the year, and wages and overheads accrue similarly

15,600 units Rs. 5 Rs. 4 Rs. 2 Re. 1 Rs. 12 4 weeks 3 weeks 3 weeks 4 weeks 5 weeks 2 weeks

[B.Com. (Hons), Calcutta University—1989] Ans.: Working Capital: Without Profit—Rs. 35,700; With Profit—Rs. 36,900. 6. With the following information, prepare a statement showing the Working Capital required to finance a level of activity of 10,400 units per annum: a. Selling Price per unit is Rs. 5. b. Expected Ratio of cost to Selling Price: Raw Materials 40% Direct Wages 10% Overhead 30% Profit 20% c. Raw materials are expected to remain in the store for an average period of 2 months before being issued for production, and materials are in process on an average period of 6 weeks. d. Finished goods will stay in the store approximately for 6 weeks before getting dispatched to customers. e. Credit allowed to debtors is for a period of 2 months.

Modified Date: Sat, Jun 26, 2010 02:55:02 PM

Output Date: Tue, Jul 06, 2010 12:17:35 PM

Rev-II

Project: Management Accounting_Debarshi Bhattacharyya ACE Pro India Pvt. Ltd. File: X:\Pearson\Management Accounting_Debarshi Bhattacharyya\MAIN\M06\LAYOUT_M06\M06_DEBA_ISBN_EN_SE_C06.indd

WORKING CAPITAL

463

f. Credit allowed by creditors is for a period of 2 months. g. Lag in payment of wages and overheads are for a period of 2 weeks each. h. Cash in hand and at bank is expected to be Rs. 10,000. It may be noted that production has been carried on evenly during the year, and wages and overheads accrue similarly. Assume there are 4 weeks in a month. [B.Com. (Hons), Calcutta University—1984] Ans.: Working Capital: Without Profit—Rs. 24,000; With Profit—Rs. 25,600. 7. From the following information, prepare a statement showing the estimated Working Capital Requirement: i. Projected Annual Sales ii. Selling Price per unit iii. Analysis of Selling Price: Raw Materials Labour Overheads Profit iv. Time lag (on average): Raw Materials in Stock Production Process Credit allowed to Debtors Credit from Suppliers Finished Goods in Warehouse Payment of Wages and Overhead v. Cash in hand is expected to be 10% of Net Working Capital

26,000 units Rs. 60 40% 30% 20% 10% 3 weeks 4 weeks 5 weeks 3 weeks 2 weeks 2 weeks

[B.Com. (Hons), Calcutta University—1986] Ans.: Working Capital (with profit) including cash balance—Rs. 2,80,000; Expected cash in hand—Rs. 28,000. 8. From the following information of Bright Ltd, you are required to determine the Working Capital Requirement: i. Annual (expected) Sales ii. Analysis of Sales: Materials Expenses Profit iii. Credit allowed to Debtors Credit allowed by Creditors Raw Materials in Store Processing Period Finished Goods in Store Bank Overdraft Cash in hand for Contingencies Production is carried out evenly during the year, and expenses accrue similarly

Rs. 1,20,000 60% 15% 25% 2.5 months 1.5 months 1 month 2 months 3 months Rs. 80,000 Rs. 5,000

[B.Com. (Hons), Calcutta University—1998] Ans.: Working Capital (without profit) excluding cash—Rs. 5,17,500; Net Working Capital Requirement including cash—Rs. 4,42,500. 9. From the following particulars of Sun Ltd, determine the Working Capital Requirement: a. Monthly Sales (expected): 16,000 units of Rs. 10 each. b. Analysis of Sales: Raw Materials 40% Labour 30% Overhead Rs. 8,000 per week c. Stock includes raw materials for Rs. 48,000 and 8,000 units of finished goods. d. Processing Period 2 weeks Credit to Debtors 5 weeks

Modified Date: Sat, Jun 26, 2010 02:55:02 PM

Output Date: Tue, Jul 06, 2010 12:17:35 PM

Rev-II

Project: Management Accounting_Debarshi Bhattacharyya ACE Pro India Pvt. Ltd. File: X:\Pearson\Management Accounting_Debarshi Bhattacharyya\MAIN\M06\LAYOUT_M06\M06_DEBA_ISBN_EN_SE_C06.indd

464

MANAGEMENT ACCOUNTING

Credit from Creditors 1 month Lag in payment of Overhead 2 weeks e. Production is carried on evenly during the year, and expenses accrue similarly. [B.Com. (Hons), Calcutta University—1990] Ans.: Working Capital: Without Profit—Rs. 2,72,000; With Profit—Rs. 2,92,000. 10. From the following information given by Tata Ltd, estimate the Working Capital Requirement for the year ending 31 March 2009; Estimated production: 120 NANO cars per year Per Car 1,000 kg 20 kg 1 50 hrs Rs. 20,000

Steel Spares Engine Labour Overhead

Rate Rs. 70 per kg Rs. 60 per kg Rs. 20,000 per engine Rs. 100 per hr

a. Steel remains in stock for 2 months, spares remain in stock for 15 days and engine for 1 month. b. Suppliers of steel allow credit for 2 months, Suppliers of spare parts for 1 month and Suppliers of engine for 15 days. c. Time lag for payment of labour and overheads is one month. d. Car (finished goods) remains in stock for 1 month. e. Production process takes 15 days. f. Activity is spread evenly throughout the year. [B.Com. (Hons), Mumbai University—April 2008] Ans.: Working Capital Requirement (without profit) Rs. 15,18,500. 11. From the following details concerning a manufacturing enterprise, estimate the amount of Working Capital needed to finance a level of activity of 50%. The capacity of the concern is to produce 2,40,000 units per annum. Expected Selling Price Raw Materials Direct Labour Overhead (including depreciation of Rs. 50,000)

Rs. 10 per unit Rs. 3 per unit Rs. 2.50 per unit Rs. 2,50,000 p.a.

Raw materials are in stock on an average for 1 month. Materials are in process on an average for 2 months. Finished goods are in stock on an average for 2 months. Credit allowed to debtors is 3 months and that received from suppliers of raw materials is 1 month. Lag in payment of wages is half a month and of overhead is 1 month. Cash in hand and at bank is 10% of Net Working Capital. You may assume that production is carried on evenly throughout the year, and wages and overheads accrue similarly. One-fourth of the output is sold against cash. [B.Com. (Hons), Calcutta University—1992] Ans.: Working Capital (without profit) excluding cash—Rs. 4,02,085; Net Working Capital Requirement (without profit) including cash—Rs. 4,46,761. 12. From the following information presented by a manufacturing company, prepare a Working Capital Requirement forecast statement for the coming year: a. Monthly Sales (expected): 32,000 units of Rs. 10 each. b. The anticipated ratios of Cost to Selling Price: Raw Materials 40% Labour 30% Overhead Rs. 16,000 per week c. Overhead expenses include a depreciation of Rs. 4,000 per week. c. Planned Stock includes raw materials for Rs. 96,000 and 16,000 units of finished goods. e. Materials stay in process 2 weeks Credit to Debtors 5 weeks

Modified Date: Sat, Jun 26, 2010 02:55:02 PM

Output Date: Tue, Jul 06, 2010 12:17:35 PM

Rev-II

Project: Management Accounting_Debarshi Bhattacharyya ACE Pro India Pvt. Ltd. File: X:\Pearson\Management Accounting_Debarshi Bhattacharyya\MAIN\M06\LAYOUT_M06\M06_DEBA_ISBN_EN_SE_C06.indd

WORKING CAPITAL

Credit from Creditors Lag in payment of Overhead

465

1 month 2 week

f. 25% of sales may be assumed against cash and cash in hand is expected to be Rs. 25,000. Assume that the production is carried on evenly throughout the year, and wages and overheads accrue similarly. A time period of 4 weeks is equivalent to a month. [B.Com. (Hons), Calcutta University—1987] Ans.: Working Capital Requirement without profit including cash—Rs. 4,60,000. 13. The components of Current Assets of a new trading concern are Stock, Debtors and Cash. The only component of its Current Liability is Creditors. Ascertain the concern’s Working Capital Requirement for the first year from the following information: a. Expected Sales at a Profit of 20% on Cost (20% cash) Rs. 9,00,000 b. Expected Stock Turnover 6 times c. Expected Credit Period to Debtors 2 months d. Expected Credit Period from Creditors 1.5 months e. The concern intends to maintain a Cash Balance equal to 1 month’s Cash Sales. [B.Com. (Hons), Calcutta University—2001] Ans.: Stock—Rs. 1,25,000; Debtors—Rs. 1,20,000; Cash—Rs. 15,000; Creditors—Rs. 93,750; Working Capital— Rs. 1,66,250. 14. From the following particulars, prepare a Working Capital Requirement forecast statement for the year 2010: a. Estimated Annual Production: 31,200 units. b. Estimated Selling Price: Rs. 10 per unit. c. Ratios of Cost to Selling Price: Raw Materials Wages Overhead

40% 30% 20%

d. Overheads include a monthly depreciation of Rs. 2,400. e. 25% of sales are made in cash. f. Raw materials are kept in store for 4 weeks, the processing time is 6 weeks and the finished goods are kept in stores for 2 months. g. Debtors are allowed a credit period of 8 weeks and a credit period of 1 month is received from the creditors. h. Lag in payment of wages and overheads are 15 days and 3 weeks, respectively. i. Cash in hand is expected to be 20% of the Net Working Capital. Ans.: Working Capital Requirement with profit including cash—Rs. 1,13,250; Cash in hand—Rs. 22,650. 15. A company is currently operating at 80% of its capacity producing 48,000 units per year at the following Cost Price structure: Materials Labour Overheads (including depreciation of Rs. 500 per week) Selling Price

Rs. per unit 5.00 3.00 2.00 12.00

Other Information: a. 80% of sales are made on credit and 40% of purchases are made in cash. b. Materials are kept in store for 6 weeks, the processing time is 8 weeks and the finished goods are stored in godown for 90 days. c. Debtors are allowed a credit period of 10 weeks and a credit period of 45 days is received from the creditors. d. Lag in payment of wages and overheads are 3 weeks and 15 days, respectively. Prepare the Working Capital forecasting statement of the company. Ans.: Working Capital Requirement with Profit—Rs. 2,68,000.

Modified Date: Sat, Jun 26, 2010 02:55:02 PM

Output Date: Tue, Jul 06, 2010 12:17:35 PM

Rev-II

Project: Management Accounting_Debarshi Bhattacharyya ACE Pro India Pvt. Ltd. File: X:\Pearson\Management Accounting_Debarshi Bhattacharyya\MAIN\M06\LAYOUT_M06\M06_DEBA_ISBN_EN_SE_C06.indd

466

MANAGEMENT ACCOUNTING

16. A company is currently operating at 60% of its capacity producing 36,000 units during the year 2009 at the following Cost Price structure: Rs. per unit 4.00 3.00 3.00 2.00 12.00

Materials Labour Overheads (60% variable) Profit Selling Price

On 31 December 2009, the company has the following Current Assets and liabilities: Rs. 12,000 21,000 60,000 72,000 18,000 9,000 4,500

Stock of Raw Materials Stock of WIP Stock of Finished Goods Sundry Debtors Sundry Creditors Outstanding Wages Outstanding Overheads

In the year 2010, the company wishes to operate at 80% of its capacity at the same Cost Price structure and selling price of 2009. Calculate the additional Working Capital Requirement in the year 2010. Ans.: Working Capital Requirement with profit for 2010—Rs. 1,75,700; Additional Working Capital Requirement for 2010—Rs. 42,100. 17. A company is currently operating at 75% of its capacity producing 60,000 units during the year 2009 at the following Cost Price structure: Rs. Raw Materials per unit Wages per unit Overheads per unit: Variable Fixed Profit per unit Selling Price per unit

2.00 2.00

Rs. 5.00 4.00

4.00 2.00 15.00

On 31 December 2009, the company has the following Current Assets and liabilities: Stock of Raw Materials – 10,000 units at cost Stock of WIP – 5,000 units at cost Stock of Finished Goods – 15,000 units at cost Sundry Debtors Sundry Creditors Liability for Wages Liability for Overheads

Rs. 50,000 45,000 1,95,000 1,50,000 50,000 20,000 10,000

In view of the increased demand for the product, it has been decided to operate at 90% capacity on and from 01 January 2010 at the same Cost Price structure, period of block and Selling Price of 2009. Calculate the additional Working Capital Requirement for the year 2010. [B.Com. (Hons), Calcutta University—Adapted] Ans.: Working Capital Requirement with profit for 2010—Rs. 4,26,000; Additional Working Capital Requirement for 2010—Rs. 66,000.

Modified Date: Sat, Jun 26, 2010 02:55:02 PM

Output Date: Tue, Jul 06, 2010 12:17:35 PM

Rev-II

Project: Management Accounting_Debarshi Bhattacharyya ACE Pro India Pvt. Ltd. File: X:\Pearson\Management Accounting_Debarshi Bhattacharyya\MAIN\M07\LAYOUT_M07\M07_DEBA_ISBN_EN_SE_C07 I.indd

Budget and Budgetary Control

7

LEARNING OBJECTIVES On completion of the study of the chapter, you should be able to understand: What is Budget and Budgeting? What is Budgetary Control? Objectives, advantages and limitations of Budgetary Control. Comparison between Budget and Budgetary Control. Comparison between Standard Costing and Budgetary Control. Concept of Zero-based Budgeting. Classification of Budget.

7.1 MEANING OF BUDGET According to CIMA, London, Budget is defined as ‘a financial and/or quantitative statement prepared and approved prior to a defined period of time, of the policy to be pursued during that period for the purpose of attaining a given objective. It may include income, expenditure and the employment of capital.’ In other words, Budget refers to a plan covering all the sectors of operations expressed in monetary and/or quantitative terms for a definite future period of time. Budget exhibits managerial plans and policies, for the organization as a whole, or a part thereof, to achieve business goals and objectives in quantitative terms for a definite future period. 7.2 MEANING OF BUDGETING According to J. Batty, ‘the entire process of preparing the Budgets is known as Budgeting.’ Therefore, the term Budgeting refers to the act of preparing Budgets. It is the managerial action of formulating Budgets. 7.3 FEATURES OF BUDGET A Budget must have the following features: i. It should reflect the managerial plans and policies to achieve business goals and objectives. ii. It is expressed either in monetary terms or quantitative terms or both. iii. It is a comprehensive plan for a definite future period. iv. Though it is basically an instrument of planning, it still provides the basis for performance evaluation and control. 7.4 BUDGETARY CONTROL According to CIMA, London, ‘Budgetary Control is the establishment of Budgets relating to the responsibilities of executives of a policy and the continuous comparison of the actual with the budgeted results, either to secure by individual action the objective of the policy or to provide a basis for its revision.’

Modified Date: Thu, Jul 01, 2010 02:46:45 PM

Output Date: Tue, Jul 06, 2010 12:18:54 PM

Rev II

Project: Management Accounting_Debarshi Bhattacharyya ACE Pro India Pvt. Ltd. File: X:\Pearson\Management Accounting_Debarshi Bhattacharyya\MAIN\M07\LAYOUT_M07\M07_DEBA_ISBN_EN_SE_C07 I.indd

468

MANAGEMENT ACCOUNTING

In other words, Budgetary Control is a technique of control under which Budgets are prepared at first for all business activities of an organization and actual performances of those business activities are compared with the respective budgeted data so that remedial measures can be taken for any adverse deviation from the Budget. Therefore, Budgetary Control refers to the application of a comprehensive system of Budgeting in the organization to assist the management in the process of its planning, coordinating, controlling and performance evaluation. It is an effective tool to the management to achieve the business goals and objectives of the organization. 7.5 OBJECTIVES OF BUDGETARY CONTROL Following are the main objectives of a Budgetary Control System: i. Performance Evaluation: It is the most effective tool to the management for the performance evaluation of all the business activities of the organization. ii. Planning: It is a very effective tool of planning for all business activities. iii. Defining Responsibilities: One of the important objectives of Budgetary Control is to define the responsibility of the concerned executive who is engaged in different business activities. iv. Coordinating: It helps in coordinating various divisional or sectional activities with a view to attain the business goals and objectives successfully. v. Communicating: It acts as an effective communicative device of the business objectives among the different levels of employees of an organization. vi. Motivating: It also acts as a useful motivating device to perform clearly the defined responsibilities of different executives of the organization. vii. Cost Control: It is used as a very powerful tool for controlling the costs of an organization. 7.6 ADVANTAGES OF BUDGETARY CONTROL Several advantages that accrue to Budgetary Control are as follows: i. It acts as a very useful and effective tool for controlling costs. ii. It provides yardsticks for evaluation of actual performance. iii. It clearly defines the areas of responsibility of all concerned executives who are engaged in various business activities. iv. It points out the efficiency of various business activities. v. It increases the operational efficiency of all business activities. vi. It helps the management in the process of its planning in respect of various business activities. vii. It coordinates various activities of different sections, divisions or departments of the organization. viii. It facilitates the effective utilizations of all resources of the organization. ix. It creates the habit of planning in the minds of the employees. x. It serves as a communicating device in the organization. 7.7 LIMITATIONS OF BUDGETARY CONTROL In spite of having many advantages of Budgetary Control, it suffers from the following limitations: i. Budget Plans are based on estimates which may not be accurate in all cases. ii. It plays a limited role in the process of controlling various business activities. iii. Budgetary Control System introduced in an organization may be resisted by some employees who are not as much efficient as others. iv. Introduction of Budgetary Control System in an organization is an expensive programme. v. Though it acts as an effective tool of the management, it is not a substitute of the management. vi. It loses its usefulness if it is not revised with the changing circumstances.

Modified Date: Thu, Jul 01, 2010 02:46:45 PM

Output Date: Tue, Jul 06, 2010 12:18:54 PM

Rev II

Project: Management Accounting_Debarshi Bhattacharyya ACE Pro India Pvt. Ltd. File: X:\Pearson\Management Accounting_Debarshi Bhattacharyya\MAIN\M07\LAYOUT_M07\M07_DEBA_ISBN_EN_SE_C07 I.indd

BUDGET AND BUDGETARY CONTROL

469

7.8 COMPARISON BETWEEN BUDGET AND BUDGETARY CONTROL Budget is a quantitative plan of action for a future period. On the other hand, Budgetary Control is a system of controlling costs and performances of various business activities through preparation of Budget, assigning responsibilities, evaluation of actual performances by comparing the actual results with the Budgeted data and taking remedial measures in case of any adverse deviation observed. Therefore, Budget is a planning tool that estimates the future courses of action, whereas Budgetary Control is a controlling system that uses Budget and ensures an evaluation of the actual performance. Although Budget is an integral part of Budgetary Control system, both are interrelated and dependent on each other. Preparation of Budget becomes effectively useful if Budgetary Control System is introduced in the organization. On the other hand, application of Budgetary Control system primarily depends on the preparation of Budget. 7.9 COMPARISON BETWEEN STANDARD COSTING AND BUDGETARY CONTROL Standard Costing and Budgetary Control have the common objective of Cost Control by establishing predetermined costs. Although both of them follow certain common basic principles, such as establishment of predetermined targets of performance, measurement of actual performance, comparison of actual performance with the predetermined performance, analysis of the differences between predetermined and actual performance, adoption of remedial measures—wherever necessary and so on, there are a number of fundamental differences between Standard Costing and Budgetary Control. These differences are as follows: Standard Costing 1. Standard Costing is based on technical assessment. 2. It is the method of preparation of standard cost and application of those for measuring the efficiency of the Actual Cost. 3. It is a projection of cost accounts. 4. It is mainly concerned with the ascertainment and control of costs. 5. Standard is set mainly in respect of manufacturing functions such as different elements of cost and sales. 6. Standard Costing sets the standard which should be maintained in the actual performance. 7. It cannot be applied without Budget. 8. It is more an intensive technique of controlling cost. 9. It cannot be applied in part. 10. Under Standard Costing, variances are revealed through different accounts.

Budgetary Control 1. Budgetary Control is based on past performance adjusted with future trend. 2. It is the method of forecasting cost by preparation of a Budget prior to a definite period to attain the given objectives. 3. Budget is a projection of Financial Accounts. 4. It is mainly concerned with the profitability and financial position of a concern. 5. It is prepared in respect of different functions of the business, such as purchase, wages, production and sales. 6. It sets up maximum limits of expenses, beyond which the actual expenses should not normally exceed. 7. It can be operated without standard. 8. It is more an extensive approach which covers all the business functions of a concern. 9. It can be applied in part. 10. In Budgetary Control, variances are used as a statistical information only.

7.10 ZERO-BASED BUDGETING (ZBB) According to the Official Terminology of CIMA, London, Zero-based Budgeting is defined as ‘a method of Budgeting whereby all activities are re-evaluated each time a Budget is formulated. Each functional Budget starts with the assumption that the function does not exist and is at zero cost. Increments of cost are compared with increments of benefits culminating in the planned maximum benefit given the Budgeted cost.’ Zero-based Budgeting (ZBB) is an alternative budgeting system that starts with zero base. It is a newly invented Budget technique where executives are required to start at Zero Budget level every year and justify all the costs of the existing function in comparison with all present and future functions. It attempts to review and defend all functions and costs every year.

Modified Date: Thu, Jul 01, 2010 02:46:45 PM

Output Date: Tue, Jul 06, 2010 12:18:54 PM

Rev II

Project: Management Accounting_Debarshi Bhattacharyya ACE Pro India Pvt. Ltd. File: X:\Pearson\Management Accounting_Debarshi Bhattacharyya\MAIN\M07\LAYOUT_M07\M07_DEBA_ISBN_EN_SE_C07 I.indd

470

MANAGEMENT ACCOUNTING

7.11 CLASSIFICATION OR TYPES OF BUDGET Budget is generally classified on the basis of the needs of the respective organization. Preparation of Budget may be required by an organization for the purpose of its flexibility of production or for the purpose of its functions involved or for the purpose of its period covered. Therefore, Budget is usually classified into many parts and sub-parts on the basis of its nature of requirement by the organization. Classification or types of Budget is shown in Figure 7.1. Classification/Types of Budget

On the basis of period

Long-term Budget

Short-term Budget

On the basis of flexibility of production

Current Budget

Fixed Budget

Sales Budget

Material Budget

Labour Budget

On the basis of function or coverage

Flexible Budget

Operating Budget

Production Budget

Production Overhead Budget

Administration Overhead Budget

Selling & Distribution Overhead Budget

Financial Budget

Cost Budget

Research & Development Cost Budget

Cash Budget

Figure 7.1

Master Budget

Capital Expenditure Budget

Classification of Budget

7.11.1 Classification on the Basis of Period On the basis of the period or time covered in the Budget period, Budget is classified into two parts: (i) Short-term Budget; and (ii) Long-term Budget. i. Long-term Budget: When a Budget is prepared for a business activity covering a period of more than 1 year, then it is called Long-term Budget. Long-term Budgets are generally prepared for long-term planning of the organization. Industries involved in producing goods or providing services during a longer generation period like electricity, engineering goods, sugar and so on, quite often use Long-term Budget. ii. Short-term Budget: When a Budget is prepared covering a period of 1 year or less, it is called Shortterm Budget. An organization prefers to prepare a Short-term Budget for sales, cash and so on. Consumer-goods producing industries such as textile, cotton, sugar and so on, quite often use Shortterm Budget.

Modified Date: Thu, Jul 01, 2010 02:46:45 PM

Output Date: Tue, Jul 06, 2010 12:18:54 PM

Rev II

Project: Management Accounting_Debarshi Bhattacharyya ACE Pro India Pvt. Ltd. File: X:\Pearson\Management Accounting_Debarshi Bhattacharyya\MAIN\M07\LAYOUT_M07\M07_DEBA_ISBN_EN_SE_C07 I.indd

BUDGET AND BUDGETARY CONTROL

471

7.11.2 Classification on the Basis of Flexibility of Production On the basis of flexibility of production capacity, Budget is classified into three parts: (i) Fixed Budget; (ii) Flexible Budget; and (iii) Current Budget. i. Fixed Budget: Fixed Budget is prepared for a particular level of production. This type of Budget is suitable for a static production level condition. According to ICWA, London, ‘Fixed Budget is a Budget which is designed to remain unchanged irrespective of the level of activity actually attained.’ ii. Flexible Budget: A Flexible Budget consists of a series of Budgets prepared in respect of different levels of activity during a Budget period. A Flexible Budget is a series of Fixed Budgets prepared for different levels of production. It is useful in case of those concerns whose level of activity rapidly changes due to unforeseen changes in the business condition. According to ICWA, London, ‘Flexible Budget is a Budget which, by recognizing the difference between Fixed, Semi-Fixed and Variable Costs, is designed to change in relation to the level of activity attained.’ iii. Current Budget: This Budget relates to the current business activities of a concern and is prepared under current conditions for a very short period. According to ICWA, London, ‘Current Budget is a Budget which is established for use over a short period of time and is related to Current Conditions.’ 7.11.3 Classification on the Basis of Function or Coverage On the basis of the function area of a concern, Budget is classified into three parts: (i) Operating Budget; (ii) Financial Budget; and (iii) Master Budget. i. Operating Budget: This Budget relates to the different activities or operations of a concern. It is a plan of the expected revenues and costs of the concern. It is ordinarily set out to cover each financial year. As every concern has different sections of its operations, Operating Budget is prepared for each of such operational activity section. Accordingly, it is classified into three parts: (a) Production Budget; (b) Cost Budget; and (c) Sales Budget. a. Production Budget: Production Budget is prepared in relation to the Sales Budget for the Budgeted period of the concern. It establishes the level of production planned for the Budget period. It is prepared for the number of units that are to be produced in conformity with the sales as projected by the Sales Budget. It is expressed both in units as well as values. b. Cost Budget: Production Budget estimates the number of units to be produced during the Budget period. Cost Budget is the budget for different elements of cost to be incurred for producing the output as estimated in the Production Budget. It estimates the total cost that is to be incurred for the Budgeted output. Therefore, Cost Budget consists of a series of Budgets for every element of cost, materials, labour, overheads and so on. It comprises the following sub-budgets: 1. Material Budget: It is concerned with the estimation of quantity (and also value) of materials required for the product-wise budgeted production. It exhibits per unit as well as the total consumption of materials and cost of materials that are required for the budgeted production. 2. Labour Budget: Labour Budget is concerned with the Estimation of Direct Labour hours (and also labour cost) required for the product-wise budgeted production. It exhibits per unit as well as total labour hours and labour cost that are required for the budgeted production. 3. Production Overhead Budget: As we know, Production Overhead consists of Indirect Materials, Indirect Labour and Indirect Expenses that are required for the production. Therefore, it is concerned with the estimation of indirect materials, indirect labour and indirect expenses required for the product-wise Budgeted production. It exhibits per unit as well as total Production Overhead Cost required for the Budgeted production. In a Production Overhead Budget, generally, Production Overhead Cost is classified into Fixed Cost, Semi-Variable Cost and Variable Cost.

Modified Date: Thu, Jul 01, 2010 02:46:45 PM

Output Date: Tue, Jul 06, 2010 12:18:54 PM

Rev II

Project: Management Accounting_Debarshi Bhattacharyya ACE Pro India Pvt. Ltd. File: X:\Pearson\Management Accounting_Debarshi Bhattacharyya\MAIN\M07\LAYOUT_M07\M07_DEBA_ISBN_EN_SE_C07 I.indd

472

MANAGEMENT ACCOUNTING

4. Administration Overhead Budget: It estimates all office- and administration overhead costs that are to be incurred for the Budgeted production. It exhibits per unit as well as the total of such cost that are to be incurred for the Budgeted production. 5. Selling and Distribution Overhead Budget: It estimates all Selling and Distribution Overhead costs that are to be incurred for the budgeted production. It exhibits per unit as well as the total of such cost that are to be incurred for the budgeted production. 6. Research and Development Cost Budget: Nowadays, generally, every manufacturing concern runs a research and development wing for improving and upgrading the quality of its products, for which the concern spends a reasonable amount every year. Research and Development Budget estimates all the costs that are to be incurred for research and development activities of the concern during the budgeted period. Such cost is charged to the production units for the Budgeted period. Accordingly, it exhibits per unit as well as the total of such cost that are to be incurred for the budgeted production. c. Sales Budget: It is the key Budget that leads to the preparation of all other functional or Operating Budgets. A Sales Budget is an estimate of the expected sales during a Budget period that is expressed in terms of both Sales Units as well as Sales Values. A Sales Budget is the starting point, on the basis of which all other functional or Operating Budgets are prepared. ii. Financial Budget: Financial Budget relates to all expected financial transactions that are to be incurred during the Budget period. It may, generally, be of two types: (a) Cash Budget; and (b) CapitalExpenditure Budget. a. Cash Budget: A Cash Budget is an estimate of the expected cash receipts and the expected cash payments that may take place during the Budget period. It includes all cash receipts and cash payments, whether of revenue or capital nature, of operating or non-operating nature during the Budget period. It is, therefore, a summary of the future cash book. Finally, it discloses both cash in hand as well as at bank, at the end of the Budget period. b. Capital Expenditure Budget: A Capital Expenditure Budget lays down a plan of the estimated future expenditure that is to be incurred by the concern during the Budget period on Fixed Assets. This Budget is based on the Forecast of Capital Expenditure that is required for the various divisions of a concern during a Budget period. iii. Master Budget: Master Budget is the summary of all the Functional Budgets. It is the overall Budget of a concern which includes the summary of all the functional activities of the concern. According to ICWA, London, ‘Master Budget is the Summary Budget, incorporating its component Functional Budgets and which is finally approved, adopted and employed.’ It coordinates the various activities of the business. A Master Budget generally includes: (a) Sales Budget; (b) Production Budget; (c) Cost Budget; (d) Cash Budget; (e) Projected Income Statement; and (f) Projected Balance Sheet.

Tutorial Notes to Students for Solving Problems How do you identify the nature of costs? i. Variable Cost: Per unit cost remains constant at different levels of output, although the total cost changes with the change in the output level. ii. Fixed Cost: The Total Cost remains constant at different levels of output, though the per unit cost changes with the change in the output level. iii. Semi-Variable Cost: Neither the Total Cost nor the per unit cost remains constant at different levels of output, i.e., both Total Cost as well as per unit cost vary at different output levels.

Modified Date: Thu, Jul 01, 2010 02:46:45 PM

Output Date: Tue, Jul 06, 2010 12:18:54 PM

Rev II

Project: Management Accounting_Debarshi Bhattacharyya ACE Pro India Pvt. Ltd. File: X:\Pearson\Management Accounting_Debarshi Bhattacharyya\MAIN\M07\LAYOUT_M07\M07_DEBA_ISBN_EN_SE_C07 I.indd

BUDGET AND BUDGETARY CONTROL

473

WORKED-OUT PROBLEMS I. Production Budget Problem 1 From the following particulars, prepare a Purchase Budget for the year 2001 when the estimated price per kg of materials are: A—Re. 1, B—Rs. 2 and C—Rs. 3: Materials A B C

Estimated Consumption of Materials (kg) 2,00,000 3,00,000 4,00,000 Estimated Stock (kg) On 1 January 2001 20,000 40,000 50,000

A B C

On 31 December 2001 25,000 30,000 40,000

[B.Com. (Hons), Calcutta University—2002] Solution Materials Purchase Budget for the year 2001

Add: Less:

Estimated Consumption for the year Estimated Closing Stock [on 31 December 2001] Estimated Opening Stock [on 1 January 2001] Estimated Purchase for the year Estimated Price per kg ∴ Estimated Cost of Purchase for the year [Estimated Purchase × Estimated Price per kg]

Material A (kg) 2,00,000 25,000 2,25,000 20,000 2,05,000 Re. 1 Rs. 2,05,000

Material B (kg) 3,00,000 30,000 3,30,000 40,000 2,90,000 Rs. 2 Rs. 5,80,000

Material C (kg) 4,00,000 40,000 4,40,000 50,000 3,90,000 Rs. 3 Rs. 11,70,000

Total (kg) 9,00,000 95,000 9,95,000 1,10,000 8,85,000 Rs. 19,55,000

Problem 2 From the following information, prepare a Production Budget of Rajanikant Ltd for the year that ended on 31 March 2010: Products X Y Z

Sales as Per Sales Budget (units) 60,000 80,000 50,000

Estimated Stock on 1 April 2009 (units) 10,000 20,000 15,000

Estimated Stock on 31 March 2010 (units) 20,000 10,000 5,000

Solution Production (Quantity) Budget of Rajanikant Ltd for the year that ended on 31 March 2010

Add: Less:

Estimated Sales as per Sales Budget Estimated Closing Stock [on 31 March 2010] Estimated Opening Stock [on 1 April 2009] Estimated Production for the year

Modified Date: Thu, Jul 01, 2010 02:46:45 PM

X (units) 60,000 20,000 80,000 10,000 70,000

Output Date: Tue, Jul 06, 2010 12:18:54 PM

Products Y (units) 80,000 10,000 90,000 20,000 70,000

Z (units) 50,000 5,000 55,000 15,000 40,000

Total (units) 1,90,000 35,000 2,25,000 45,000 1,80,000

Rev II

Project: Management Accounting_Debarshi Bhattacharyya ACE Pro India Pvt. Ltd. File: X:\Pearson\Management Accounting_Debarshi Bhattacharyya\MAIN\M07\LAYOUT_M07\M07_DEBA_ISBN_EN_SE_C07 I.indd

474

MANAGEMENT ACCOUNTING

Problem 3 The following are the estimated sales of Philips Co. for about 8 months that is ending on 30 November 2008: 2008 April May June July

Units 12,000 13,000 9,000 8,000

2008 August September October November

Units 10,000 12,000 14,000 12,000

As a matter of policy, the company maintains the Closing Balance of Finished Goods and Raw Materials as follows: i. Finished Goods—The Closing Stock of a month will be 50% of the estimated sales for the next month. ii. Raw Material—The Closing Stock of a month will be equal to the estimated consumption for the next month. Each unit of production consumes 2 kg of raw material costing Rs. 6 per kg. Prepare the following Budgets for the half-year that is ending on 30 September 2008: i. Production Budget (month-wise in units). ii. Raw Material Purchase Budget (month-wise in units and cost). [B.Com. (Hons), Delhi University—2008] Solution Production Budget of Philips Co. for the half-year that is ending on 30 September 2008

Add:

Less:

Sales Closing Stock of Finished Goods [50% of next months’ sales]1 Opening Stock of Finished Goods [50% of current months’ sales]2 Estimated Production

April

May

June

July

(units) 12,000 6,500

(units) 13,000 4,500

(units) 9,000 4,000

(units) 8,000 5,000

August September (units) 10,000 6,000

(units) 12,000 7,000

(units) 64,000 33,000

Total

18,500 6,000

17,500 6,500

13,000 4,500

13,000 4,000

16,000 5,000

19,000 6,000

97,000 32,000

12,500

11,000

8,500

9,000

11,000

13,000

65,000

Raw Material Purchase Budget of Philips Co. for the half-year that is ending on 30 September 2008

Consumption of material [@ 2 kg per unit of estimated production] Closing Stock of Raw Material [@ 2 kg per unit of next months’ estimated production]3

April (kg) 25,000

May (kg) 22,000

June (kg) 17,000

July (kg) 18,000

22,000

17,000

18,000

22,000

August September Total (kg) (kg) (kg) 22,000 26,000 1,30,000 26,000

26,000

1,31,000

47,000 39,000 35,000 40,000 48,000 25,000 22,000 17,000 18,000 22,000 Less: Opening Stock of Raw Material [@ 2 kg per unit of current months’ estimated production]4 Estimated Purchases 22,000 17,000 18,000 22,000 26,000 Estimated Cost of Purchases @ Rs. 6 per kg (Rs.) 1,32,000 1,02,000 1,08,000 1,32,000 1,56,000

52,000 26,000

2,61,000 1,30,000

26,000

1,31,000

1,56,000

7,86,000

Add:

Modified Date: Thu, Jul 01, 2010 02:46:45 PM

Output Date: Tue, Jul 06, 2010 12:18:54 PM

Rev II

Project: Management Accounting_Debarshi Bhattacharyya ACE Pro India Pvt. Ltd. File: X:\Pearson\Management Accounting_Debarshi Bhattacharyya\MAIN\M07\LAYOUT_M07\M07_DEBA_ISBN_EN_SE_C07 I.indd

475

BUDGET AND BUDGETARY CONTROL

Tutorial Note i. It is stated in the problem that the estimated Closing Stock of finished goods at the end of each month is equal to the estimated sales for the next month. Therefore, sales for the month of May represent the Closing Stock of finished goods of April. Similarly, sales for the month of June represent the Closing Stock of finished goods of May and so on. ii. Sales for the month of May represent the Closing Stock of finished goods of April, which is also the Opening Stock of May. Similarly, sales for the month of April represent the Closing Stock of finished goods of March, which is also the Opening Stock for April. The Opening Stocks of finished goods for the subsequent months have been computed in this way. iii. It is stated in the problem that the estimated Closing Stock of raw materials at the end of each month is equal to the estimated consumption (i.e., the estimated production) for the next month. Therefore, the production for the month of May (i.e., 11,000 units × 2 kg) represents the Closing Stock of raw materials of April. Similarly, the production for the month of June represents the Closing Stock of raw materials of May and so on. iv. Production for the month of May represents the Closing Stock of raw materials of April, which is also the Opening Stock of May. Similarly, the production for the month of April represents the Closing Stock of raw materials of March, which is also the Opening Stock for April. Opening Stocks of raw materials for the subsequent months have been computed in this way.

Problem 4 From the following information as furnished by Maocaro Ltd, prepare a month-wise Production Quantity Budget and a summarized Production Cost Budget for the 6 months that ended on 31 December 2009: i. Estimated units to be sold: July 2009 August 2009 September 2009 October 2009

units 4,000 5,000 7,000 8,000

units 9,000 11,000 10,000

November 2009 December 2009 January 2010

ii. Estimated Finished Stock at the end of each month is equal to half of the estimated sales for the next month. iii. Budgeted units to be produced during the year 2009 and details of production cost for the year that ended on 31 December 2009 are as follows: Estimated Production for the year Direct Material Cost per unit Direct Wage Cost per unit Variable Factory Overhead cost per unit Total Fixed Factory Overhead for the year

1,00,000 units Rs. 20 Rs. 12 Rs. 6 Rs. 5,00,000

Solution Production Quantity Budget of Maocaro Ltd for 6 Months that ended on 31 December 2009

Add:

Sales Closing Stock1 [1/2 of next months’ sales]

July (units) 4,000 2,500

August (units) 5,000 3,500

6,500

8,500

September October November (units) (units) (units) 7,000 8,000 9,000 4,000 4,500 5,500 11,000

12,500

14,500

December (units) 11,000 5,000 16,000

Total (units) 44,000 25,000 69,000 (Continued)

Modified Date: Thu, Jul 01, 2010 02:46:45 PM

Output Date: Tue, Jul 06, 2010 12:18:54 PM

Rev II

Project: Management Accounting_Debarshi Bhattacharyya ACE Pro India Pvt. Ltd. File: X:\Pearson\Management Accounting_Debarshi Bhattacharyya\MAIN\M07\LAYOUT_M07\M07_DEBA_ISBN_EN_SE_C07 I.indd

476

Less:

MANAGEMENT ACCOUNTING

Opening Stock2 [1/2 of current months’ sales] Estimated Production

July (units) 2,000

August (units) 2,500

4,500

6,000

September October November (units) (units) (units) 3,500 4,000 4,500 7,500

8,500

December (units) 5,500

10,000

10,500

Total (units) 22,000 47,000

Summarized Production Cost Budget of Maocaro Ltd for 6 months that ended on 31 December 2009 Estimated output: 47,000 units Per unit Rs. Direct Materials Direct Wages Factory Overhead: Variable Factory Overhead Fixed Factory Overhead3 [1/2 of Rs. 5,00,000]

Total Rs. 20 12

6 5.32

Rs.

2,82,000 2,50,000 11.32 43.32

Total Production Cost

Rs. 9,40,000 5,64,000

5,32,000 20,36,000

Tutorial Note 1. It is stated in the problem that the estimated Finished Stock at the end of each month is equal to half of the estimated sales for the next month. Therefore, half of the sales for August represent the Closing Stock at the end of July. Similarly, half of the sales for September represent the Closing Stock at the end of August and so on. 2. Half of the sales for August represent the Closing Stock at the end of July, which is also the Opening Stock for August. Similarly, half of the sales for July represent the Closing Stock at the end of June, which is also the Opening Stock for July. Opening Stocks for the subsequent months have been computed in this way. 3. Total Fixed Overheads for 12 months of 2009 is estimated at Rs. 5,00,000. Therefore, the Total Fixed Overheads for 6 productive months of 2009 = 1/2 of Rs. 5,00,000 = Rs. 2,50,000

Problem 5 From the following information, prepare the following Budgets for the year 2009: (a) Production Budget; (b) Material usage Budget; and (c) Material purchase Budget. P 10,000 2,000 7,000

Sales Finished Stock on 1 January 2009 Finished Stock on 31 December 2009

Stock of Materials on 1 January 2009 (kg) Stock of Materials on 31 December 2009 (kg) Quantities used in (kg): Product P Product Q Product R Rate of Materials per kg (Rs.)

Modified Date: Thu, Jul 01, 2010 02:46:45 PM

Output Date: Tue, Jul 06, 2010 12:18:54 PM

Product (units) Q 20,000 6,000 1,000

R 15,000 5,000 10,000

Materials used in Production S2 S3 S1 20,000 50,000 20,000 60,000 35,000 55,000 3 7 3 5

5 8 6 4

5 4 9 6

Rev II

Project: Management Accounting_Debarshi Bhattacharyya ACE Pro India Pvt. Ltd. File: X:\Pearson\Management Accounting_Debarshi Bhattacharyya\MAIN\M07\LAYOUT_M07\M07_DEBA_ISBN_EN_SE_C07 I.indd

BUDGET AND BUDGETARY CONTROL

477

Solution i. Production Budget for the year 2009

Add:

Less:

Estimated sales for the year Estimated Closing Stock of Finished Goods [on 31 December 2009] Estimated Opening Stock of Finished Goods [on 1 January 2009] Estimated production for the year

Product P (units) 10,000

Product Q (units) 20,000

Product R (units) 15,000

Total (units) 45,000

7,000 17,000

1,000 21,000

10,000 25,000

18,000 63,000

2,000 15,000

6,000 15,000

5,000 20,000

13,000 50,000

ii. Material Usage Budget for the year 2009 Material S1 Qty Required for Per unit of Production (kg) For Production of 15,000 units of product P For Production of 15,000 units of product Q For Production of 20,000 units of product R Estimated usage of materials for the year

Material S2

Total (kg)

Qty Required for Per unit of Production (kg)

Material S3

Total (kg)

Qty Required for Per unit of Production (kg)

Total (kg)

Total (kg)

3

45,000

5

75,000

5

75,000 1,95,000

7

1,05,000

8

1,20,000

4

60,000 2,85,000

3

60,000

6

1,20,000

9

1,80,000 3,60,000

2,10,000

3,15,000

3,15,000 8,40,000

iii. Material Purchase Budget for the year 2009

Add: Less:

Estimated Usages of Materials for the year [as computed in (b) above] Estimated Closing Stock of Materials Estimated Opening Stock of Materials Estimated Purchase Quantity of materials for the year Rate of materials per kg ∴ Estimated Cost of Purchase for the year [Estimated Purchase Quantity × Rate per kg]

Material S1 (kg)

Material S2 (kg)

Material S3 (kg)

Total (kg)

2,10,000

3,15,000

3,15,000

8,40,000

60,000 2,70,000 20,000 2,50,000 Rs. 5 12,50,000

35,000 3,50,000 50,000 3,00,000 Rs. 4 12,00,000

55,000 3,70,000 20,000 3,50,000 Rs. 6 21,00,000

1,50,000 9,90,000 90,000 9,00,000 45,55,000

Problem 6 The following information relating to the 3rd and last quarter of 2003–04 is furnished by a company which manufactures and sells a single product: 3rd Quarter (Actual) Rs. 6,24,000 Opening Balance Closing Balance

Sales Inventory of Raw Materials and Finished Goods: Raw Material A (kg) Raw Material B (kg) Finished Goods (units)

Modified Date: Thu, Jul 01, 2010 02:46:45 PM

25,000 12,650 670

Output Date: Tue, Jul 06, 2010 12:18:54 PM

23,500 13,400 700

Last Quarter (Estimate) Rs. 6,60,000 Closing Balance 25,000 15,000 1,000

Rev II

Project: Management Accounting_Debarshi Bhattacharyya ACE Pro India Pvt. Ltd. File: X:\Pearson\Management Accounting_Debarshi Bhattacharyya\MAIN\M07\LAYOUT_M07\M07_DEBA_ISBN_EN_SE_C07 I.indd

478

MANAGEMENT ACCOUNTING

Unit Cost data: Raw Material A = 10 kg @ Rs. 3 = Rs. 30 Raw Material B = 5 kg @ Rs. 2 = Rs. 10 Direct Labour: Machine shop [Machine time of 5 hrs @ Rs. 4] = Rs. 20 Assembly [Labour time of 2 hrs @ Rs. 5] = Rs. 10 Production Overheads: Machine shop @ Rs. 12 per machine hour Assembly @ Rs. 10 per labour hour Selling and Administration Overheads: 20% of production cost Profit Margin: 10% on selling price Production and sales occur evenly during the Budget period. You are required to prepare for the last quarter of the year: (a) Production Budget (in units); (b) Purchase Budget (in quantity and value); and (c) Production Cost Budget. [I.C.W.A. (Inter)—June 2004] Solution i. Production Budget (in units) for the last quarter of 2003–04

Add: Less:

Units 3,300 1,000 4,300 700 3,600

Sales for the last quarter of 2003–04 [Rs. 6,60,000 ÷ Rs. 200] Closing Stock of Finished Goods at the end of the last quarter Opening Stock of Finished Goods at the beginning of the last quarter Production (in units) for the last quarter of 2003–04

ii. Purchase Budget (in quantity & in value) for the last quarter of 2003–04

Add:

Less:

Estimated consumption for the last quarter: Material A: 3,600 units × 10 kg Material B: 3,600 units × 5 kg Estimated Closing Stock of materials at the end of last quarter Opening Stock of materials at the beginning of last quarter Estimated purchase [in quantity] for the last quarter Price of materials per kg Estimated purchase [in value] for the last quarter [Estimated purchase quantity × Price per kg]

Material A (kg) 36,000

Material B (kg) 18,000

Total (kg) 54,000

25,000 61,000

15,000 33,000

40,000 94,000

23,500 37,500 Rs. 3 1,12,500

13,400 19,600 Rs. 2 39,200

36,900 57,100 1,51,700

iii. Production Cost Budget for the last quarter of 2003–04 Amount Rs. Raw Materials: A: 36,000 kg × Rs. 3 B: 18,000 kg × Rs. 2

Amount Rs.

1,08,000 36,000 1,44,000 (Continued)

Modified Date: Thu, Jul 01, 2010 02:46:45 PM

Output Date: Tue, Jul 06, 2010 12:18:54 PM

Rev II

Project: Management Accounting_Debarshi Bhattacharyya ACE Pro India Pvt. Ltd. File: X:\Pearson\Management Accounting_Debarshi Bhattacharyya\MAIN\M07\LAYOUT_M07\M07_DEBA_ISBN_EN_SE_C07 I.indd

BUDGET AND BUDGETARY CONTROL

Direct Labour: Machine shop: 5 hrs × 3,600 units × Rs. 4 Assembly : 2 hrs × 3,600 units × Rs. 5

479

72,000 36,000 1,08,000 2,52,000

Prime Cost Production Overheads: Machine shop: 5 hrs × 3,600 units × Rs. 12 Assembly: 2 hrs × 3,600 units × Rs. 10

2,16,000 72,000 2,88,000 5,40,000

Estimated Production Cost for the last quarter of 2003–04

 Working Note Calculation of Selling Price per unit of the Product

Add:

Raw Materials: A: 10 kg × Rs. 3 B: 5 kg × Rs. 2 Direct Labour: Machine shop: 5 hrs × Rs. 4 Assembly: 2 hrs × Rs. 5 Production Overheads: Machine shop: 5 hrs × Rs. 12 Assembly: 2 hrs × Rs. 10 Production Cost Selling & Administration Overheads: 20% of Rs. 150 Total cost: Profit Margin [10% on Selling Price = 1/10 on Selling Price = 1/9 on Total Cost = 1/9 on Rs. 180.] Selling Price per unit

Rs.

Rs.

30 10

40

20 10

30

60 20

80 150 30 180 20 200

Problem 7 A company is engaged in manufacturing two products X and Y. Product X uses one unit of component P and two units of component Q. Product Y uses two units of component P, one unit of component Q and two units of component R. Component R which is assembled in the factory uses one unit of component Q. Components P and Q are purchased from the market. The company has prepared the following forecast of sales and inventory for the next year: Product X 80,000 10,000 30,000

Sales (in units) At the end of the year At the beginning of the year

Product Y 1,50,000 20,000 50,000

The production of both the products and the assembling of the component R will be spread out uniformly throughout the year. The company at present orders its inventory of P and Q in quantities equivalent to 3 months’ production. The company has compiled the following data related to two components: P 20 1,500 20%

Price per unit (Rs.) Order Placing Cost per order (Rs.) Carrying Cost per annum

Modified Date: Thu, Jul 01, 2010 02:46:45 PM

Output Date: Tue, Jul 06, 2010 12:18:54 PM

Q 8 1,500 20%

Rev II

Project: Management Accounting_Debarshi Bhattacharyya ACE Pro India Pvt. Ltd. File: X:\Pearson\Management Accounting_Debarshi Bhattacharyya\MAIN\M07\LAYOUT_M07\M07_DEBA_ISBN_EN_SE_C07 I.indd

480

MANAGEMENT ACCOUNTING

Required: i. Prepare a Budget of production and requirements of components during next year. ii. Suggest the optimal order quantity of components P and Q. [C.A. (PE-II)—May 2006] Solution i. Production Budget for the next year of product X and product Y

Add: Less:

Product X units 80,000 10,000 90,000 30,000 60,000

Estimated sales quantity for the year Inventory at the end of the year Inventory at the beginning of the year Production for the year

Product Y units 1,50,000 20,000 1,70,000 50,000 1,20,000

Total units

1,80,000

Budgeted requirement of component P, Q and R for the next year Component P For production of 6,000 units of product X: P: 60,000 × 1 unit Q: 60,000 × 2 units For production of 1,20,000 units of product Y: P: 1,20,000 × 2 units Q: 1,20,000 × 1 unit R: 1,20,000 × 2 units For production of 2,40,000 units of component R: Q: 2,40,000 × 1 unit Total requirement of components

Component Q

Component R

Total

60,000 1,20,000 2,40,000 1,20,000 2,40,000

3,00,000

2,40,000 4,80,000

2,40,000

10,20,000

ii. The company should purchase components P and Q as per Economic Order Quantity (EOQ) to minimize its Cost of Materials, which are calculated as follows: (i) Annual Requirements (U) (ii) Ordering Cost per order (O) (iii) Carrying Cost p.a. (C) (20% of per unit of material cost) ⎛ ∴ EOQ ⎜ ⎝

(2 × O × U) ⎞ ⎟ C ⎠

Component P 3,00,000 units Rs. 1,500 20% of Rs. 20 = Rs. 4

Component Q 4,80,000 units Rs. 1,500 20% of Rs. 8 = Rs. 1.60

(2 × 1,500 × 3, 00, 000) 4 = 15,000 units

(2 × 1,500 × 4, 80, 000) 1.60 = 30,000 units

Problem 8 A single-product company estimated its sales for the next year quarter-wise as follows: Quarter I II III IV

Modified Date: Thu, Jul 01, 2010 02:46:45 PM

Sales units 30,000 37,500 41,250 45,000

Output Date: Tue, Jul 06, 2010 12:18:54 PM

Rev II

Project: Management Accounting_Debarshi Bhattacharyya ACE Pro India Pvt. Ltd. File: X:\Pearson\Management Accounting_Debarshi Bhattacharyya\MAIN\M07\LAYOUT_M07\M07_DEBA_ISBN_EN_SE_C07 I.indd

481

BUDGET AND BUDGETARY CONTROL

The Opening Stock of Finished Goods is 10,000 units and the company expects to maintain the Closing Stock of Finished Goods at 16,250 units at the end of the year. The production pattern in each quarter is based on 80% of the sales of the current quarter and 20% of the sales of the next quarter. The Opening Stock of Raw Materials in the beginning of the year is 10,000 kg and the Closing Stock at the end of the year is required to be maintained at 5,000 kg. Each unit of finished output requires 2 kg of Raw Materials. The company proposes to purchase the entire annual requirement of Raw Materials in the first three quarters in the proportion and at the prices given as follows: Purchase of Materials in % to Total Annual Requirement in Quantity 30% 50% 20%

Quarter I II III

Price Per kg 2 3 4

The value of the Opening Stock of Raw Materials in the beginning of the year is Rs. 20,000. You are required to present the following for the next year, quarter-wise: i. Production Budget in units. ii. Raw Material Consumption Budget in quantity. iii. Raw Material Purchase Budget in quantity and value. [C.A. (Inter)—Adapted] Solution i. Production Budget (in units) for the next year

Add:

Sales for the current quarter Estimated production for the current quarter [80% of the sales of current quarter] Estimated additional production for the next quarter [20% of the sales of next quarter] Estimated Total Production

Quarter I (units) 30,000 24,000

Quarter II (units) 37,500 30,000

Quarter III (units) 41,250 33,000

Quarter IV (units) 45,000 36,000

Total (units) 1,53,750 1,23,000

7,500

8,250

9,000

12,2503

37,000

31,500

38,250

42,000

48,2502

1,60,0001

ii. Raw Material Consumption Budget (in quantity) for the next year

(A) Raw Materials required to produce one unit of the finished product (B) Estimated production of finished product [as computed in (i) above] ∴ Estimated consumption of Raw Materials (A × B)

Quarter I

Quarter II

Quarter III

Quarter IV

2 kg 31,500 units 63,000 kg

2 kg 38,250 units 76,500 kg

2 kg 42,000 units 84,000 kg

2 kg 48,250 units 96,500 kg

Total

3,20,000 kg

iii. Raw Material Purchase Budget (in quantity) for the next year Quarter I

Quarter II

Quarter III

Total

Estimated purchase of raw materials = 30% of annual requirement = 30% of 3,15,000 kg4 = 94,500 kg

Estimated purchase of Raw Materials = 50% of annual requirement = 50% of 3,15,000 kg4 = 1,57,500 kg

Estimated purchase of raw materials = 20% of annual requirement = 20% of 3,15,000 kg4 = 63,000 kg

3,15,000 kg4

Modified Date: Thu, Jul 01, 2010 02:46:45 PM

Output Date: Tue, Jul 06, 2010 12:18:54 PM

Rev II

Project: Management Accounting_Debarshi Bhattacharyya ACE Pro India Pvt. Ltd. File: X:\Pearson\Management Accounting_Debarshi Bhattacharyya\MAIN\M07\LAYOUT_M07\M07_DEBA_ISBN_EN_SE_C07 I.indd

482

MANAGEMENT ACCOUNTING

iv. Raw Material Purchase Budget (in value) for the next year (A)

Estimated purchase quantities of Raw Materials [as computed in (iii) above] Rate of Raw Materials per kg ∴ Estimated cost of raw material purchase (A × B)

(B)

Quarter I 94,500 kg

Quarter II 1,57,500 kg

Quarter III 63,000 kg

Total 3,15,000 kg

Rs. 2 Rs. 1,89,000

Rs. 3 Rs. 4,72,500

Rs. 4 Rs. 2,52,000

Rs. 9,13,500

Working Notes 1. Calculation for Estimated Total Annual Production for the Next Year

Add: Less:

units 1,53,750 16,250 1,70,000 10,000 1,60,000

Sales in 4 quarters of the next year Closing Stock of Finished Goods Opening Stock of Finished Goods Estimated Total Annual Production in 4 quarters of the next year

2. Calculation of Estimated Production in Units in Quarter IV of the Next Year units Less:

Total annual production in 4 quarters Production in: Quarter I Quarter II Quarter III

units 1,60,000

31,500 38,250 42,000

Estimated Total Production in Quarter IV

1,11,750 48,250

3. Calculation of Estimated Additional Production in Quarter IV

Less:

Estimated Total Production in Quarter IV Estimated Production for the Current Quarter [80% of the sales of current quarter = 80% of 45,000 units] Estimated Additional Production in Quarter IV [for next quarter]

units 48,250 36,000 12,250

4. Calculation of Estimated Total Annual Purchase of Raw Materials (in Kg) for the Next Year

Add: Less:

Estimated Total Consumption of Raw Materials [as computed in (ii)] Closing Stock of Raw Materials Opening Stock of Raw Materials Estimated Total Annual Purchase

Kg 3,20,000 5,000 3,25,000 10,000 3,15,000

II. Sales Budget Problem 9 Rahul & Co. manufactures two products ‘Phool’ and ‘Kante’ and operates two sales divisions for selling them in the market. From the following information, prepare a Sales Budget for the year 2010 which is to be presented to the Budget committee:

Modified Date: Thu, Jul 01, 2010 02:46:45 PM

Output Date: Tue, Jul 06, 2010 12:18:54 PM

Rev II

Project: Management Accounting_Debarshi Bhattacharyya ACE Pro India Pvt. Ltd. File: X:\Pearson\Management Accounting_Debarshi Bhattacharyya\MAIN\M07\LAYOUT_M07\M07_DEBA_ISBN_EN_SE_C07 I.indd

BUDGET AND BUDGETARY CONTROL

Product Phool Kante Phool Kante

Market Division I Division II

Budgeted Sales for the Year 2009 2,000 units @ Rs. 4 each 1,500 units @ Rs. 6 each 3,000 units @ Rs. 4 each 2,500 units @ Rs. 6 each

483

Actual Sales for the Year 2009 2,500 units @ Rs. 4 each 1,000 units @ Rs. 6 each 3,500 units @ Rs. 4 each 2,000 units @ Rs. 6 each

At a meeting of the Divisional Sales Managers, the following decisions have been taken: i. The Sales Managers contend that there is a high potential demand for product Phool and this product has been under-priced. Therefore, the price of product Phool should be increased by Rs. 2. ii. Product Kante is not selling at the expected rate as it is over-priced. Therefore, if the selling price of this product is reduced by Re. 1, then it is expected that the market would absorb more. On the basis of these price changes and impact of additional advertisement, the Divisional Sales Managers have made the following estimates: Market

Product

Division I

Phool Kante Phool Kante

Division II

% Increase in Sales for Price Changes 15% 5% 20% 10%

% Increase in Sales for Additional Advertisement 5% 5% 5% 10%

Solution Sales Budget of Rahul & Co. for the year 2010 Division

Product

I

(a) Phool (b) Kante

II

(c) Phool (d) Kante

Total Sales

Phool (a + c) Kante (b + d)

Budgeted Sales for the Year 2009 Qty Rate Amount (Units) (Rs.) (Rs.) 2,000 4 8,000 1,500 6 9,000 3,500 17,000 3,000 4 12,000 2,500 6 15,000 5,500 27,000 5,000 20,000 4,000 24,000 9,000 44,000

Actual Sales for the Year 2009 Qty Rate Amount (Units) (Rs.) (Rs.) 2,500 4 10,000 1,000 6 6,000 3,500 16,000 3,500 4 14,000 2,000 6 12,000 5,500 26,000 6,000 24,000 3,000 18,000 9,000 42,000

Budgeted Sales for the Year 2010 Qty. Rate Amount (Units) (Rs.) (Rs.) 2,400 6 14,400 1,650 5 8,250 4,050 22,650 3,900 6 23,400 2,875 5 14,375 6,775 37,775 6,300 37,800 4,525 22,625 10,825 60,425

Working Notes 1. Budget Sales for the year 2010 in Division I

Add: Add:

Product Budgeted Sales for the year 2009 Proposed increase in sales for price changes in 2010 Proposed increase in sales for additional advertisement in 2010 Budgeted Total Sales for the year 2010

Phool (units) 2,000 (15% of 2,000) 300 (5% of 2,000) 100 2,400

Kante (units) 1,500 (5% of 1,500) 75 (5% of 1,500) 75 1,650

Phool (units) 3,000 (20% of 3,000) 600 (10% of 3,000) 300 3,900

Kante (units) 2,500 (5% of 2,500) 125 (10% of 2,500) 250 2,875

2. Budgeted Sales for the Year 2010 in Division II

Add: Add:

Budgeted Sales for the year 2009 Proposed increase in sales for price changes in 2010 Proposed increase sales for additional advertisement in 2010 Budgeted Total Sales for the year 2010

Modified Date: Thu, Jul 01, 2010 02:46:45 PM

Output Date: Tue, Jul 06, 2010 12:18:54 PM

Rev II

Project: Management Accounting_Debarshi Bhattacharyya ACE Pro India Pvt. Ltd. File: X:\Pearson\Management Accounting_Debarshi Bhattacharyya\MAIN\M07\LAYOUT_M07\M07_DEBA_ISBN_EN_SE_C07 I.indd

484

MANAGEMENT ACCOUNTING

Problem 10 A company manufactures three products, namely, A, B and C. The current pattern of sales of A, B and C is in the ratio of 8:2:1 respectively. The relevant data are as follows: Products Selling Price per unit (Rs.) Raw Materials per unit (kg) Direct Materials per unit (kg) Skilled Labour hours ÷ unit Semi-skilled Labour hours per unit Variable Overheads (Rs. per unit)

A 130 0.50 0.25 4 2 20

B 230 1.2 − 6 2 40

C 417 2.5  8 3 80

The price of raw materials and direct materials, respectively, are Rs. 100 and Rs. 40 per kg. The wage rates of skilled and semi-skilled labour, respectively, are Rs. 6 and Rs. 5. Each operator works for 8 hours a day for 25 days in a month. The positions of inventories are as follows:

Opening Closing

Raw Materials kg 600 650

Direct Materials kg 400 260

A units

B units

C units

400 200

100 300

50 50

The fixed overheads amount to Rs. 2,00,000 per month and the company desires a profit of Rs. 1,20,000 per month. You are required to prepare the following for a month: i. Sales Budget in quantity and value. ii. Production Budget showing the quantity to be manufactured. iii. Purchase Budget showing the quantity and value. iv. Direct Labour Budget showing the number of workers and wages. [C.A. (PE II)—May 2004] Solution i. Sales Budget for the Month (in quantity and value) A. B.

C. D. E. F. G. H.

Product A 130

Selling Price per unit (Rs.) Variable Cost per unit: Raw Materials (Rs.) Direct Materials (Rs.) Skilled Labour (Rs.) Semi-skilled Labour (Rs.) Variable Overhead (Rs.) Contribution per unit (Rs.) (A − B) Ratio of Sales for the month Contribution from Total Sales for the month (C × D) (Rs.) Total Fixed Overheads for the month (Rs.) Total Profit for the month (Rs.) Total Contribution for the month (Rs.) [Fixed Cost + Profit, i.e., F + G] ∴ Sales units for the month [(H ÷ E) × D] ∴ Sales value for the month (Rs.) [Sales units × Selling Price p.u.]

Modified Date: Thu, Jul 01, 2010 02:46:45 PM

Product B 230

Product C 417

50 10 24 10 20 114 16 8 128

120 − 36 10 40 206 24 2 48

250 − 48 15 80 393 24 1 24

12,800 16,64,000

3,200 7,36,000

1,600 6,67,200

Output Date: Tue, Jul 06, 2010 12:18:54 PM

Total

11 200 2,00,000 1,20,000 3,20,000 17,600 30,67,200

Rev II

Project: Management Accounting_Debarshi Bhattacharyya ACE Pro India Pvt. Ltd. File: X:\Pearson\Management Accounting_Debarshi Bhattacharyya\MAIN\M07\LAYOUT_M07\M07_DEBA_ISBN_EN_SE_C07 I.indd

BUDGET AND BUDGETARY CONTROL

485

ii. Production Budget for the month

Add: Less:

Product A (units) 12,800 200 13,000 400 12,600

Sales units Closing Stock units Opening Stock units Production units

Product B (units) 3,200 300 3,500 100 3,400

Product C (units) 1,600 50 1,650 50 1,600

Total (units) 17,600 550 18,150 550 17,600

iii. Purchase Budget for the month (in quantity & value) Raw Direct Materials (kg) Materials (kg) Materials required for production: A: [12,600 units × 0.50 kg] [12,600 units × 0.25 kg] B: [3,400 units × 1.2 kg] C: [1,600 units × 2.5 kg]

Total (kg)

6,300 3,150

Add:

Closing Stock of Materials

Less:

Opening Stock of Materials Materials required to be purchased [in quantity] Materials required to be purchased [in value] [14,430 kg × Rs. 100] [3,010 kg × Rs. 40]

4,080 4,000 14,380 650 15,030 600 14,430 Rs. 14,43,000

3,150 260 3,410 400 3,010 Rs. 1,20,400

17,440 Rs. 15,63,400

iv. Direct Labour Budget for the month (a) (b) (c) (d) (e) (f ) (g)

Production for the month [units] Skilled labour hours per unit Total skilled labour hours [a × b] Semi-skilled labour hours per unit Total semi-skilled labour hours [a × d] Wage rate per hour of skilled labour Wage rate per hour of semi-skilled labour

∴Total Wages paid to skilled labour [c × f ] ∴Total Wages paid to semi-skilled labour [e × g] Total Wages (h) Total labour hours in a month per labour [25 days × 8 hrs] ∴ No. of skilled workers required [c ÷ h] ∴ No. of semi-skilled workers required [e ÷ h] Total no. of workers required

Product A 12,600 4 hrs 50,400 hrs 2 hrs 25,200 hrs Rs. 6 Rs. 5 Rs. 3,02,400 1,26,000 4,28,400

Product B 3,400 6 hrs 20,400 hrs 2 hrs 6,800 hrs Rs. 6 Rs. 5 Rs. 1,22,400 34,000 1,56,400

Product C 1,600 8 hrs 12,800 hrs 3 hrs 4,800 hrs Rs. 6 Rs. 5 Rs. 76,800 24,000 1,00,800

200 hrs 252 126 378

200 hrs 102 34 136

200 hrs 64 24 88

Total 17,600 83,600 hrs 36,800 hrs

Rs. 5,01,600 1,84,000 6,85,600

418 184 602

III. Flexible Budget Problem 11 Jharna Ltd Co. is currently operating at 50% capacity and produces 10,000 units at a cost of Rs. 180 per unit, the details of which are as follows: Rs. 100 30

Materials per unit Wages per unit

(Continued)

Modified Date: Thu, Jul 01, 2010 02:46:45 PM

Output Date: Tue, Jul 06, 2010 12:18:54 PM

Rev II

Project: Management Accounting_Debarshi Bhattacharyya ACE Pro India Pvt. Ltd. File: X:\Pearson\Management Accounting_Debarshi Bhattacharyya\MAIN\M07\LAYOUT_M07\M07_DEBA_ISBN_EN_SE_C07 I.indd

486

MANAGEMENT ACCOUNTING

Rs. 30 20 200

Factory Overheads [40% fixed] per unit Administration Overheads [50% variable] per unit Current Selling Price per unit

At 60% capacity, the Material Cost per unit increases by 2% and the Selling Price per unit falls by 2%. At 80% capacity, the Material Cost per unit increases by 5% and the Selling Price per unit falls by 5%. Prepare a Marginal Cost Statement showing the Total Cost and Profit at 50%, 60% and 80% capacity of production. Also give your comments on the profitability at these levels of performance. [I.C.W.A. (Inter)—Adapted] Solution Marginal Cost Statement of Jharna Ltd Co. for the period

Particulars

A. Variable Costs2: Materials Wages Factory Overheads [60% variable = 60% of Rs. 30 = Rs. 18] Administration Overheads [50% variable = 50% of Rs. 20 = Rs. 10] B. Fixed Costs: Factory Overheads [40% fixed = 40% of Rs. 30 = Rs. 12] Administration Overheads [50% fixed = 50% of Rs. 20 = Rs. 10] C. Total Cost (A + B) D. Sales E. Contribution4 (D − A) F. Profit (D − C) or (E − B) G. % of Profit on Sales

At 50% Capacity (Production: 10,000 Units) Per Unit Total (Rs.) (Rs.)

At 60% Capacity (Production: 12,0001 Units) Per Unit Total (Rs.) (Rs.)

At 80% Capacity (Production: 16,0006 Units) Per Unit Total (Rs.) (Rs.)

100 30 18

10,00,000 3,00,000 1,80,000

102 30 18

12,24,000 3,60,000 2,16,000

105 30 18

16,80,000 4,80,000 2,88,000

10

1,00,000

10

1,20,000

10

1,60,000

158

15,80,000

160

19,20,000

163

26,08,000

12

1,20,000

1,20,000

1,20,000

10

1,00,000

1,00,000

1,00,000

200 42

2,20,000 18,00,000 20,00,000 4,20,000 2,00,000 10%

2,20,000 21,40,000 23,52,000 4,32,000 2,12,000 9.10%

2,20,000 28,28,000 30,40,000 4,32,000 2,12,000 6.97%

196 36

190 27

F ⎡ Profit ⎤ ⎢ Sales × 100 = D × 100 ⎥ ⎣ ⎦

Comments i. Total profit increases from Rs. 2,00,000 to Rs. 2,12,000 when production level increases from 10,000 units to 12,000 units. But it does not increase further when production level increases from 12,000 units to 16,000 units. ii. As the level of production increases, percentage of profit on sales gradually decreases from 10% to 9.10% and 9.10% to 6.97%. iii. It is most profitable to raise the production up to 12,000 units as, at this level of production, highest profit of Rs. 2,12,000 is earned. It is not profitable to raise the production level up to 16,000 units as it does not provide any additional profit.

Modified Date: Thu, Jul 01, 2010 02:46:45 PM

Output Date: Tue, Jul 06, 2010 12:18:54 PM

Rev II

Project: Management Accounting_Debarshi Bhattacharyya ACE Pro India Pvt. Ltd. File: X:\Pearson\Management Accounting_Debarshi Bhattacharyya\MAIN\M07\LAYOUT_M07\M07_DEBA_ISBN_EN_SE_C07 I.indd

487

BUDGET AND BUDGETARY CONTROL

Working Notes 1. Here given, Production at 50% capacity = 10,000 units ∴ Production at 60% capacity = 60% ÷ 50% × 10,000 = 12,000 units, and production at 80% capacity = 80% ÷ 50% × 10,000 = 16,000 units 2. Variable Costs increase with the increase in output. 3. Fixed Costs do not increase with the increase in output. They remain same at different levels of output. 4. As per Marginal Costing Techniques, Contribution = Sales – Variable Cost

Problem 12 Delta Engineering Ltd produces a uniform type of product and has a manufacturing capacity of 3,000 units per week of 48 hours. From the cost records of the company, the following data are available relating to output and cost for three consecutive weeks: Weeks 1 2 3

Units Manufactured & Sold 1,200 units 1,600 units 1,800 units

Direct Materials (Rs.)

Direct Labour (Rs.)

Overhead (Rs.)

9,000 12,000 13,500

3,600 4,800 5,400

31,000 33,000 34,000

Assuming that the company charges a profit of 20% on sales, find out the selling price per unit when the weekly production and sales is 2,000 units. [B.Com. (Hons), Calcutta University—2009]

Solution Books of Delta Engineering Ltd Flexible Budget of Total Cost and Sales for 2,000 units of output for the period Particulars

Per unit (Rs.) 7.50 3.00 10.50 17.50 28.00 7.00 35.00

Direct Materials1 Direct Labour2 Prime Cost Overhead3 Total Cost Profit [20% on sales = 1/5 on sales = 1/4 on cost = 1/4 on Rs. 56,000] Selling Price

Total (Rs.) 15,000 6,000 21,000 35,000 56,000 14,000 70,000

Working Notes 1. Direct Materials Total Direct Materials cost ∴ Direct Materials Cost per unit

At 1,200 units (Rs.) 9,000 9,000 ÷ 1,200 = 7.50

At 1,600 units (Rs.) 12,000 12,000 ÷ 1,600 = 7.50

At 1,800 units (Rs.) 13,500 13,500 ÷ 1,800 = 7.50

Here, it has been observed that direct materials cost per unit remains unchanged at different levels of production, but the total costs are different at different levels of production. ∴ Direct Materials Cost is definitely a Variable Cost ∴ Total Direct Materials Cost at 2,000 units of output = 2,000 units × Rs. 7.50 = Rs. 15,000 (Continued)

Modified Date: Thu, Jul 01, 2010 02:46:45 PM

Output Date: Tue, Jul 06, 2010 12:18:54 PM

Rev II

Project: Management Accounting_Debarshi Bhattacharyya ACE Pro India Pvt. Ltd. File: X:\Pearson\Management Accounting_Debarshi Bhattacharyya\MAIN\M07\LAYOUT_M07\M07_DEBA_ISBN_EN_SE_C07 I.indd

488

MANAGEMENT ACCOUNTING

2. Direct Labour At 1,200 units (Rs.) 3,600 3,600 ÷ 1,200 = 3

Total Direct Labour Cost ∴ Direct Labour cost per unit

At 1,600 units (Rs.) 4,800 4,800 ÷ 1,600 = 3

At 1,800 units (Rs.) 5,400 5,400 ÷ 1,800 = 3

Here, it has been observed that Direct Labour Cost per unit remains unchanged at different levels of production, but the Total Costs are different at different levels of production. ∴ Direct Labour Cost is definitely a Variable Cost. ∴ Total Direct Labour Cost at 2,000 units of output = 2,000 units × Rs. 3 = Rs. 6,000 3. Overhead At 1,200 units (Rs.) 31,000 31,000 ÷ 1,200 = 25.83

Total Overhead Cost ∴ Overhead Cost per unit

At 1,600 units (Rs.) 33,000 33,000 ÷ 1,600 = 20.625

At 1,800 units (Rs.) 34,000 34,000 ÷ 1,800 =18.88

Here, it has been observed that neither the total overhead cost nor the per unit overhead cost remains unchanged at different levels of output. ∴ Overhead Cost is a Semi-Variable Cost.

At 1,600 units of output, the total overhead cost and, = Rs. 33,000 at 1,200 units of output, the total overhead cost. = Rs. 31,000 Increase in output = 400 units Increase in cost = Rs. 2,000

This amount of increase in cost is due to the variable portion of the expenses included in it. ∴ Variable Overhead Cost per unit = Rs. 2,000 ÷ 400 = Rs. 5 Now, at 1,200 units of output: Total Overhead Cost Variable Overhead Cost [1,200 units × Rs. 5] Fixed Overhead Cost ∴ At 2,000 units of output: Variable Overhead Cost [2,000 units × Rs. 5] Fixed Overhead Cost [same as at 1,200 units] Total Overhead Cost at 2,000 units of output

Rs. 31,000 Rs. 6,000 Rs. 25,000 Rs. 10,000 Rs. 25,000 Rs. 35,000

∴ Overhead Cost per unit at 2,000 units of output = Rs. 35,000 ÷ 2,000 units = Rs. 17.50

Problem 13 A company incurs the following expenses to produce 1,000 units of an article: Rs. 30,000 15,000 10,000 8,000 6,000 12,000

Direct Materials Direct Labour Power [20% fixed] Repairs & Maintenance [15% fixed] Depreciation (40% variable) Administrative Expenses [100% fixed]

Prepare a Flexible Budget showing the individual expenses of production levels at 1,500 units and 2,000 units. [B.Com. (Hons), Calcutta University—1998]

Modified Date: Thu, Jul 01, 2010 02:46:45 PM

Output Date: Tue, Jul 06, 2010 12:18:54 PM

Rev II

Project: Management Accounting_Debarshi Bhattacharyya ACE Pro India Pvt. Ltd. File: X:\Pearson\Management Accounting_Debarshi Bhattacharyya\MAIN\M07\LAYOUT_M07\M07_DEBA_ISBN_EN_SE_C07 I.indd

489

BUDGET AND BUDGETARY CONTROL

Solution Flexible Budget of a company for the period At 1,500 units Per Unit Total (Rs.) (Rs.) 30.00 45,000 15.00 22,500 9.33 14,000 7.60 11,400 4.80 7,200 8.00 12,000 74.73 1,12,100

Particulars Direct Materials1 Direct Labour 2 Power3 Repairs & Maintenance4 Depreciation5 Administrative Expenses6 Total Cost

At 2,000 units Per Unit Total (Rs.) (Rs.) 30 60,000 15 30,000 9 18,000 7.40 14,800 4.20 8,400 6.00 12,000 71.60 1,43,200

Working Notes 1. At the production level of 1,000 units, Total Direct Materials = Rs. 30,000 ∴ Direct Materials per unit = Rs. 30,000 ÷ 1,000 = Rs. 30 As Direct Material is a Variable Cost, At the production level of 1,500 units, Total Direct Materials = 1,500 units × Rs. 30 = Rs. 45,000, and at the 2,000 units level, Total Direct Materials = 2,000 units × Rs. 30 = Rs. 60,000 2. At the production level of 1,000 units, Total Direct Labour = Rs. 15,000 ∴ Direct Labour per unit = Rs. 15,000 ÷ 1,000 = Rs. 15 As Direct Labour is a Variable Cost, At the production level of 1,500 units, Total Direct Labour = 1,500 units × Rs. 15 = Rs. 22,500, and at the 2,000 units level, Total Direct Labour = 2,000 units × Rs. 15 = Rs. 30,000 3. At the production level of 1,000 units, Power = Rs. 10,000, which includes 20% fixed portion and balance 80% variable portion. Then, at the production level of 1,000 units, Fixed Power [20% of Rs. 10,000] Variable Power [80% of Rs. 10,000] Total Power

Rs. 2,000 8,000 10,000

∴ Variable Power per unit = Rs. 8,000 ÷ 1,000 = Rs. 8 Now, at the production level of 1,500 units, Variable Power [1,500 units × Rs. 8] Fixed Power [as it was at 1,000 units] Total Power at 1,500 units of production

Rs. 12,000 2,000 14,000

Again, at the production level of 2,000 units, Variable Power [2,000 units × Rs. 8] Fixed Power [as it was at 1,000 units] Total Power at 2,000 units of production

Rs. 16,000 2,000 18,000

4. At the production level of 1,000 units, repairs and maintenance is Rs. 8,000, which includes 15% of fixed portion and balance 85% of variable portion. (Continued)

Modified Date: Thu, Jul 01, 2010 02:46:45 PM

Output Date: Tue, Jul 06, 2010 12:18:54 PM

Rev II

Project: Management Accounting_Debarshi Bhattacharyya ACE Pro India Pvt. Ltd. File: X:\Pearson\Management Accounting_Debarshi Bhattacharyya\MAIN\M07\LAYOUT_M07\M07_DEBA_ISBN_EN_SE_C07 I.indd

490

MANAGEMENT ACCOUNTING

Fixed Repairs & Maintenance [15% of Rs. 8,000] Variable Repairs & Maintenance [85% of Rs. 8,000] Total Repairs & Maintenance

Rs. 1,200 6,800 8,000

∴ Variable Repairs and Maintenance per unit = Rs. 6,800 ÷ 1,000 = Rs. 6.80 Now, at the production level of 1,500 units, Variable Repairs & Maintenance [1,500 units × Rs. 6.80] Fixed Repairs & Maintenance [as it was at 1,000 units] Total Repairs & Maintenance at 1,500 units of production

Rs. 10,200 1,200 11,400

Again, at the production level of 2,000 units, Variable Repairs & Maintenance [2,000 units × Rs. 6.80] Fixed Repairs & Maintenance [as it was at 1,000 units] Total Repairs & Maintenance at 2,000 units of production

Rs. 13,600 1,200 14,800

5. At the production level of 1,000 units, depreciation is Rs. 6,000, which includes 40% of variable portion and balance 60% of fixed portion. Fixed Depreciation [60% of Rs. 6,000] Variable Depreciation [40% of Rs. 6,000] Total Depreciation

Rs. 3,600 2,400 6,000

∴ Variable Depreciation per unit = Rs. 2,400 ÷ 1,000 = Rs. 2.40 Now, at the production level of 1,500 units, Variable Depreciation [1,500 units × Rs. 2.40] Fixed Depreciation [as it was at 1,000 units] Total Depreciation at 1,500 units of production

Rs. 3,600 3,600 7,200

Again, at the production level of 2,000 units, Variable Depreciation [2,000 units × Rs. 2.40] Fixed Depreciation [as it was at 1,000 units] Total Depreciation at 2,000 units of production

Rs. 4,800 3,600 8,400

6. At the production level of 1,000 units, Administrative Expenses is Rs. 12,000, which is 100% fixed. ∴ At the production level of 1,500 units, Administrative Expenses is Rs. 12,000, and at the production level of 2,000 units too, Administrative Expenses is Rs. 12,000 only.

Problem 14 From the following information, prepare a Flexible Budget for overheads at 50%, 60% and 70% capacity levels of production and also ascertain the overhead rate at these levels: At 60% Capacity Rs. Fixed Overheads: Depreciation Salaries Insurance

20,000 40,000 10,000 (Continued)

Modified Date: Thu, Jul 01, 2010 02:46:45 PM

Output Date: Tue, Jul 06, 2010 12:18:54 PM

Rev II

Project: Management Accounting_Debarshi Bhattacharyya ACE Pro India Pvt. Ltd. File: X:\Pearson\Management Accounting_Debarshi Bhattacharyya\MAIN\M07\LAYOUT_M07\M07_DEBA_ISBN_EN_SE_C07 I.indd

BUDGET AND BUDGETARY CONTROL

Semi-Variable Overheads: Repairs & Maintenance (20% variable) Electricity (50% fixed) Variable Overheads: Indirect Material Indirect Wages Total Overheads Estimated Direct Labour Hours

491

30,000 24,000 36,000 48,000 2,08,000 2,86,500

Solution Flexible Budget for the Overheads of

for the period At 50% Capacity (Rs.)

At 60% Capacity (Rs.)

At 70% Capacity (Rs.)

30,000 40,000

36,000 48,000

42,000 56,000

29,000 22,000

30,000 24,000

31,000 26,000

20,000 40,000 10,000 1,91,000 2,38,750

20,000 40,000 10,000 2,08,000 2,86,500

20,000 40,000 10,000 2,25,000 3,34,250

Re. 0.80

Re. 0.7260

Re. 0.6731

Variable Overheads: Indirect Material1 Indirect Wages2 Semi-Variable Overheads: Repairs & Maintenance3 Electricity4 Fixed Overheads: Depreciation5 Salaries5 Insurance5 Total Overheads (A) Estimated Direct Labour Hours6 (B) ∴ Overhead rate per direct labour hour Total overhead A⎤ ⎡ ⎢ Estimated direct-labour hours = B ⎥ ⎣ ⎦

Working Notes 1. Calculation of Indirect Materials at 50% and 70% Capacity Here, given indirect Materials at 60% capacity = Rs. 36,000 ∴ Indirect Materials at 50% capacity = 50% / 60% × 36,000 = Rs. 30,000, and Indirect Materials at 70% capacity = 70% / 60% × 36,000 = Rs. 42,000 2. Indirect Wages at 50% and 70% Capacity Here, given Indirect Wages at 60% capacity = Rs. 48,000 ∴ Indirect Wages at 50% capacity = 50% / 60% × 48,000 = Rs. 40,000, and Indirect Wages at 70% capacity = 70% / 60% × 48,000 = Rs. 56,000 3. Repairs and Maintenance at 50% and 70% Capacity Here, given repairs and maintenance at 60% capacity = Rs. 30,000, which includes 20% of variable portion. ∴ Fixed portion of this cost = 100 – 20 = 80% Here, at 60% Capacity, Variable Repair and Maintenance [20% of Rs. 30,000] Fixed Repairs and Maintenance [80% of Rs. 30,000] Total Repairs and Maintenance

Modified Date: Thu, Jul 01, 2010 02:46:45 PM

= Rs. 6,000 = Rs. 24,000 = Rs. 30,000

Output Date: Tue, Jul 06, 2010 12:18:54 PM

Rev II

Project: Management Accounting_Debarshi Bhattacharyya ACE Pro India Pvt. Ltd. File: X:\Pearson\Management Accounting_Debarshi Bhattacharyya\MAIN\M07\LAYOUT_M07\M07_DEBA_ISBN_EN_SE_C07 I.indd

492

MANAGEMENT ACCOUNTING

Then, at 50% capacity, Variable Repairs and Maintenance [50% ÷ 60% × Rs. 6,000] Fixed Repairs and Maintenance [same as at 60% capacity] Total Repairs and Maintenance at 50% capacity Again, at 70% capacity, Variable Repairs and Maintenance [70% ÷ 60% × Rs. 6,000] Fixed Repairs and Maintenance [same as at 60% capacity] Total Repairs and Maintenance at 70% capacity 4. Electricity at 50% and 70% Capacity. At 60% capacity: Variable Electricity [50% of Rs. 24,000] Fixed Electricity [50% of Rs. 24,000] Total Electricity at 60% capacity Now, at 50% capacity: Variable Electricity [50% ÷ 60% × Rs. 12,000] Fixed Electricity [same as at 60% capacity] Total Electricity at 50% capacity Again, at 70% capacity: Variable Electricity [70% ÷ 60% × Rs. 12,000] Fixed Electricity [same as at 60% capacity] Total Electricity at 70% capacity

= Rs. 5,000 = Rs. 24,000 = Rs. 29,000 = Rs. 7,000 = Rs. 24,000 = Rs. 31,000

= Rs. 12,000 = Rs. 12,000 = Rs. 24,000 = Rs. 10,000 = Rs. 12,000 = Rs. 22,000 = Rs. 14,000 = Rs. 12,000 = Rs. 26,000

5. Fixed Overheads at 50% and 70% Capacity. Here, the given Fixed Costs at 60% capacity: Depreciation = Rs. 20,000 Salaries = Rs. 40,000 Insurance = Rs. 10,000 As these are Fixed Costs, they remain unchanged at 50% and 70% capacity. Depreciation at 50% and 70% capacity = Rs. 20,000 Salaries at 50% and 70% capacity = Rs. 40,000 Insurance at 50% and 70% capacity = Rs. 10,000 6. Estimated Direct Labour hours at 50% capacity = 50% ÷ 60% × 2,86,500 = 2,38,750 hrs Estimated Direct Labour hours at 70% capacity = 70% ÷ 60% × 2,86,500 = 3,34,250 hrs

Problem 15 The information relating to the Budget prepared for two levels of capacity utilization is given as follows: Capacity Output (units) Direct Materials Direct Wages Production Overhead Administrative Overhead Selling Overhead

60% 36,000 Rs. 3,60,000 2,16,000 5,40,000 1,80,000 1,44,000

100% 60,000 Rs. 6,00,000 3,60,000 7,56,000 1,80,000 1,92,000

Prepare a Flexible Budget for 70%, 80% and 90% capacity utilization showing clearly the Unit Fixed Cost, Unit Variable Cost and Total Cost. [B.Com. (Hons), Calcutta University—2006]

Modified Date: Thu, Jul 01, 2010 02:46:45 PM

Output Date: Tue, Jul 06, 2010 12:18:54 PM

Rev II

Project: Management Accounting_Debarshi Bhattacharyya ACE Pro India Pvt. Ltd. File: X:\Pearson\Management Accounting_Debarshi Bhattacharyya\MAIN\M07\LAYOUT_M07\M07_DEBA_ISBN_EN_SE_C07 I.indd

493

BUDGET AND BUDGETARY CONTROL

Solution Flexible Budget for 70%, 80% and 90% capacity utilization At 70% Capacity (42,000 Units) Per unit Total Rs. Rs.

Particulars A. Variable Cost: Direct Materials Direct Wages Variable Production Overheads Variable Selling Overheads B. Fixed cost: Administrative Overheads Fixed Production Overheads Fixed Selling Overheads C. Total Cost (A + B)

At 80% Capacity (49,000 Units) Per unit Total Rs. Rs.

At 90% Capacity (54,000 Units) Per unit Total Rs. Rs.

10 6 9 2 27

4,20,000 2,52,000 3,78,000 84,000 11,34,000

10 6 9 2 27

4,80,000 2,88,000 4,32,000 96,000 12,96,000

10 6 9 2 27

5,40,000 3,24,000 4,86,000 1,08,000 14,58,000

4.29 5.14 1.71 11.14 38.14

1,80,000 2,16,000 72,000 4,68,000 16,02,000

3.75 4.50 1.50 9.75 36.75

1,80,000 2,16,000 72,000 4,68,000 17,64,000

3.33 4.00 1.33 8.66 35.66

1,80,000 2,16,000 72,000 4,68,000 19,26,000

Notes: Production at 100% capacity = 60,000 units ∴ Production at 70% capacity = 70% of 60,000 = 42,000 units ∴ Production at 80% capacity = 80% of 60,000 = 48,000 units ∴ Production at 90% capacity = 90% of 60,000 = 54,000 units Working Notes 1. Analysis of Costs

(a)

Total Cost at 36,000 units of output [i.e., at 60% capacity] (b) ∴Per unit cost at 36,000 units of output [(a) ÷ 36,000] (c) Total Cost at 60,000 units of output [i.e., at 100% capacity] (d) ∴Per unit cost at 60,000 units of output [(c) ÷ 60,000] ∴Nature of Cost

Direct Materials Rs. 3,60,000

Direct Wages Rs. 2,16,000

Production Overheads Rs. 5,40,000

Administrative Overheads Rs. 1,80,000

Selling Overheads Rs. 1,44,000

10

6

15

5

4

3,60,000

7,56,000

1,80,000

1,92,000

6

12.60

3

3.20

6,00,000

10

Variable [as per unit cost unchanged]

Variable [as per unit cost unchanged]

Semi- variable [as neither total cost nor per unit cost un changed]

Fixed [as total cost unchanged]

Semi- variable [as neither total cost nor per-unit cost unchanged] (Continued)

Modified Date: Thu, Jul 01, 2010 02:46:45 PM

Output Date: Tue, Jul 06, 2010 12:18:54 PM

Rev II

Project: Management Accounting_Debarshi Bhattacharyya ACE Pro India Pvt. Ltd. File: X:\Pearson\Management Accounting_Debarshi Bhattacharyya\MAIN\M07\LAYOUT_M07\M07_DEBA_ISBN_EN_SE_C07 I.indd

494

MANAGEMENT ACCOUNTING

2. Calculation of Fixed and Variable Production Overheads At 60,000 units of output, Total Production Overheads At 36,000 units of output, Total Production Overheads Increase in Output = 24,000 units

Increase in Cost

Rs. 7,56,000 5,40,000 Rs. 2,16,000

∴Variable Production Overhead per unit = Rs. 2,16,000 ÷ 24,000 units = Rs. 9 Now, at 60,000 units of output,

Less:

Rs. 7,56,000 5,40,000 2,16,000

Total Production Overheads Variable Production Overheads [60,000 units × Rs. 9] ∴Fixed Production Overheads

3. Calculation of Fixed and Variable Selling Overheads At 60,000 units of output, Total Selling Overheads At 36,000 units of output, Total Selling Overheads Increase in Output = 24,000 units

Increase in Cost

Rs. 1,92,000 1,44,000 Rs. 48,000

∴ Variable Selling Overheads per unit = Rs. 48,000 ÷ 24,000 units = Rs. 2 Now, at 60,000 units of output:

Less:

Total Selling Overheads Variable Selling Overheads [60,000 units × Rs. 2] ∴ Fixed Selling Overheads

Rs. 1,92,000 1,20,000 72,000

Problem 16 Rangan Ltd manufactures a single product for which the market demand exists for an additional quantity. Present sale of Rs. 60,000 per month utilizes only 60% capacity of the plant. The sales manager assures that with a reduction of 10% in the price, he would be in a position to increase the sales by about 25% to 30%. The following data are available: i. Selling Price: Rs. 10 per unit. ii. Variable Cost: Rs. 3 per unit. iii. Semi-Variable Cost: Rs. 6,000 fixed plus Re. 0.50 per unit. iv. Fixed Cost: Rs. 20,000 at present level estimated to be Rs. 24,000 at 80% output. You are required to submit the following statements to the board showing: i. The Operating Profits at 60%, 70% and 80% levels at the Current Selling Price and at the proposed Selling Price. ii. The percentage increase in the present output which will be required to maintain the present profit margin at the proposed Selling Price. [I.C.W.A. (Inter)—Adapted]

Modified Date: Thu, Jul 01, 2010 02:46:45 PM

Output Date: Tue, Jul 06, 2010 12:18:54 PM

Rev II

Project: Management Accounting_Debarshi Bhattacharyya ACE Pro India Pvt. Ltd. File: X:\Pearson\Management Accounting_Debarshi Bhattacharyya\MAIN\M07\LAYOUT_M07\M07_DEBA_ISBN_EN_SE_C07 I.indd

495

BUDGET AND BUDGETARY CONTROL

Solution Books of Rangan Ltd i. Statement showing operating profits at the Current Selling Price and proposed Selling Price

(a)

Output [units]

(b) Variable Cost [Rs. 3 × a] (c) Semi-variable Cost: Fixed Portion Variable Portion [Re. 0.50 × a] (d) Fixed Cost (e) Total Cost (f ) Sales at Current Price [Rs. 10 × a] (g) Sales at Proposed Price [{Rs. 10 − (10% of Rs. 10)} × a] ∴ Operating Profit at Current Selling Price [f − e] ∴ Operating Profit at Proposed Selling Price [g − e]

At 60% capacity 6,000 Rs. 18,000

At 70% capacity 7,000 Rs. 21,000

At 80% capacity 8,000 Rs. 24,000

6,000 3,000 20,000 47,000 60,000 54,000 13,000 7,000

6,000 3,500 20,000 50,500 70,000 63,000 19,500 12,500

6,000 4,000 24,000 58,000 80,000 72,000 22,000 14,000

ii. Statement showing percentage increase in the present output to maintain the present profit margin at the proposed Selling Price Rs. Present Profit Present Total Fixed Cost: Fixed Cost Fixed Portion of Semi-Variable Cost

Less:

(a) Total contribution required to maintain present Profit Proposed Selling Price per unit [Rs. 10 − (10% of Rs. 10)] Total Variable Cost per unit: Variable Cost per unit Variable Portion of Semi-Variable Cost per unit (b) Contribution per unit from the Proposed Selling Price ∴

Less:

Output required to maintain the present profit at the proposed Selling Price [a ÷ b = Rs. 39,000 ÷ Rs. 5.50] Present output Increase in output ∴ % increase in the present output to maintain the present profit margin at the Proposed Selling Price ⎡⎛ Increase in output ⎞ ⎤ ⎛ 1, 091 ⎞ ⎢⎜ ⎟ × 100 = ⎜⎝ 6, 000 ⎟⎠ × 100 ⎥ ⎣⎝ Present output ⎠ ⎦

Rs. 13,000

20,000 6,000 26,000 39,000 9.00 3.00 0.50 3.50 5.50 units 7,091 6,000 1,091

18.18%

Problem 17 The following data are available in a manufacturing company for the year 2010: Rs. (Lakhs) Fixed Expenses: Wages & Salaries Rent, Rates & Taxes

9.5 6.6 (Continued)

Modified Date: Thu, Jul 01, 2010 02:46:45 PM

Output Date: Tue, Jul 06, 2010 12:18:54 PM

Rev II

Project: Management Accounting_Debarshi Bhattacharyya ACE Pro India Pvt. Ltd. File: X:\Pearson\Management Accounting_Debarshi Bhattacharyya\MAIN\M07\LAYOUT_M07\M07_DEBA_ISBN_EN_SE_C07 I.indd

496

MANAGEMENT ACCOUNTING

Depreciation Sundry Administrative Expenses Semi-variable Expenses [at 50% capacity]: Repairs & Maintenance Indirect Labour Sales Department Salaries Sundry Administrative expenses Variable Expenses [at 50% capacity]: Materials Labour Other Expenses

7.4 6.5 3.5 7.9 3.8 2.8 21.7 20.4 7.9 98.00

Assume that the Fixed Expenses remain constant for all levels of production, and Semi-Variable Expenses remain constant between 45% and 60% of capacity, increasing by 10% between 65% and 80% capacity and by 20% between 80% and 100% capacity. Sales at various levels are as follows: Rs. (Lakhs) 100 120 150 180 200

50% Capacity 60% Capacity 75% Capacity 90% Capacity 100% Capacity

Prepare a Flexible Budget for the year and Forecast the Profit at 50%, 60%, 75%, 90% and 100% of capacity. [I.C.W.A. (Inter)—Adapted] Solution Flexible Budget of

A.

B.

C.

D. E. F.

Particulars Capacity Levels Fixed Expenses1 Wages & Salaries Rent, Rates & Taxes Depreciation Sundry Administrative Expenses Semi-Variable Expenses2 Repairs & Maintenance Indirect Labour Sales Departments’ Salaries Sundry Administrative Expenses Variable Expenses3 Materials Labour Other Expenses Total Cost of Production [A + B + C] Sales Profit [E – D]

Modified Date: Thu, Jul 01, 2010 02:46:45 PM

for the year 2010 50%

60%

Rs. (Lakhs) 75%

90%

100%

9.50 6.60 7.40 6.50 30.00

9.50 6.60 7.40 6.50 30.00

9.50 6.60 7.40 6.50 30.00

9.50 6.60 7.40 6.50 30.00

9.50 6.60 7.40 6.50 30.00

3.50 7.90 3.80 2.80 18.00

3.50 7.90 3.80 2.80 18.00

3.85 8.69 4.18 3.08 19.80

4.20 9.48 4.56 3.36 21.60

4.20 9.48 4.56 3.36 21.60

21.70 20.40 7.90 50.00 98.00 100 2.00

26.04 24.48 9.48 60.00 108.00 120 12.00

32.55 30.60 11.85 75.00 124.80 150 25.20

39.06 36.72 14.22 90.00 141.60 180 38.40

43.40 40.80 15.80 100.00 151.60 200 48.40

Output Date: Tue, Jul 06, 2010 12:18:54 PM

Rev II

Project: Management Accounting_Debarshi Bhattacharyya ACE Pro India Pvt. Ltd. File: X:\Pearson\Management Accounting_Debarshi Bhattacharyya\MAIN\M07\LAYOUT_M07\M07_DEBA_ISBN_EN_SE_C07 II.indd

BUDGET AND BUDGETARY CONTROL

497

Working Notes 1. As these are Fixed Expenses, they do not vary at different levels of production. 2. Semi-Variable Expenses remain unchanged at 60% capacity of production. But, Semi-Variable Expenses at 75% capacity = Semi-Variable Expenses at 50% capacity + 10%, and SemiVariable Expenses at 90% and 100% capacity = Semi-Variable Expenses at 50% capacity + 20%. 3. Variable Expenses vary with the change in the production levels. At 50% production capacity, Materials Cost = 21.70 (Rs. in lakhs) ∴ At 60% production capacity, Materials Cost = 60% ÷ 50% × 21.70 = 26.04 (Rs. in lakhs) ∴ At 75% production capacity, Materials Cost = 75% ÷ 50% × 21.70 = 32.55 (Rs. in lakhs) ∴ At 90% production capacity, Materials Cost = 90% ÷ 50% × 21.70 = 39.06 (Rs. in lakhs) ∴ At 100% production capacity, Materials Cost = 100% ÷ 50% × 21.70 = 43.40 (Rs. in lakhs) Again, at 50% production capacity, Labour Cost = 20.40 (Rs. in lakhs) ∴ At 60% production capacity, Labour Cost = 60% ÷ 50% × 20.40 = 24.48 (Rs. in lakhs) ∴ At 75% production capacity, Labour Cost = 75% ÷ 50% × 20.40 = 30.60 (Rs. in lakhs) ∴ At 90% production capacity, Labour Cost = 90% ÷ 50% × 20.40 = 36.72 (Rs. in lakhs) ∴ At 100% production capacity, Labour Cost =100% ÷ 50% × 20.40 = 40.80 (Rs. in lakhs) Again, at 50% production capacity, Other Expenses = 7.90 (Rs. in lakhs) ∴ At 60% production capacity, Other Expenses = 60% ÷ 50% × 7.90 = 9.48 (Rs. in lakhs) ∴ At 75% production capacity, Other Expenses = 75% ÷ 50% × 7.90 = 11.85 (Rs. in lakhs) ∴ At 90% production capacity, Other Expenses = 90% ÷ 50% × 7.90 = 14.22 (Rs. in lakhs) ∴ At 100% production capacity, Other Expenses = 100% ÷ 50% × 7.90 = 15.80 (Rs. in lakhs)

Problem 18 The Budget manager of Jhumpa Engineering Ltd is preparing a Flexible Budget for the accounting year commencing from 1April 2009. The company produces a single product ‘Dimpu.’ Direct materials cost Rs. 7 per unit. Direct Labour averages Rs. 2.50 per hour and requires 1.60 hours to produce one unit of Dimpu. Salesmen are paid a commission of Re.1 per unit sold. Fixed Selling and Administrative Expenses amount to Rs. 85,000 per year. Manufacturing Overheads under specified conditions of volume have been estimated as follows: Volume of Production (units) Expenses: Indirect Materials Indirect Labour Inspection Maintenance Supervision Depreciation—Plant & Equipment Engineering Services Total Manufacturing Overheads

1,20,000 Rs. 2,64,000 1,50,000 90,000 84,000 1,98,000 90,000 94,000 9,70,000

1,50,000 Rs. 3,30,000 1,87,500 1,12,500 1,02,000 2,34,000 90,000 94,000 11,50,000

Normal capacity of production of the company is 1,25,000 units. Prepare a Budget of Total Cost at 1,40,000 units of output. [C.S. (Inter)—Adapted]

Modified Date: Thu, Jul 01, 2010 02:49:57 PM

Output Date: Tue, Jul 06, 2010 12:19:29 PM

Rev II

Project: Management Accounting_Debarshi Bhattacharyya ACE Pro India Pvt. Ltd. File: X:\Pearson\Management Accounting_Debarshi Bhattacharyya\MAIN\M07\LAYOUT_M07\M07_DEBA_ISBN_EN_SE_C07 II.indd

498

MANAGEMENT ACCOUNTING

Solution Books of Jhumpa Engineering Ltd Flexible Budget of total cost at 1,40,000 units of Dimpu for the year that ended on 31 March 2010 Per unit Rs. 7.00 4.00 11.00

Elements of Cost Direct Materials Direct Labour [1.60 hours × Rs. 2.50] Prime cost Manufacturing Overheads: Indirect Materials Indirect Labour Inspection Maintenance Supervision Depreciation—Plant & Equipment Engineering Services Factory Cost Selling & Administrative Overheads: Fixed Selling & Administrative Expenses Salesmen’s commission [1,40,000 units × Re.1] Total Cost

Total Rs. 9,80,000 5,60,000 15,40,000

2.20 1.25 0.75 0.69 1.58 0.64 0.67 18.75

3,08,000 1,75,000 1,05,000 96,000 2,22,000 90,000 94,000 26,30,000

0.61 1.00 20.39

85,000 1,40,000 28,55,000

Working Notes 1. Indirect Materials Total Indirect Material Cost (Rs.) ∴ Indirect Material Cost per unit (Rs.)

At 1,20,000 units 2,64,000 2,64,000 ÷1,20,000 = Rs. 2.20

At 1,50,000 units 3,30,000 3,30,000 ÷ 1,50,000 = Rs. 2.20

Here, it has been observed that cost of indirect materials per unit remains unchanged at different levels of production, but total costs are different at different levels of production. ∴ Indirect Material Cost is definitely a variable expense ∴ Total Indirect Material Cost at 1,40,000 units of output = 1,40,000 units × Rs. 2.20 = Rs. 3,08,000 2. Indirect Labour Total Indirect Material Cost (Rs.) ∴ Indirect Material Cost per unit (Rs.)

At 1,20,000 units 1,50,000 1,50,000 ÷1,20,000 = Rs. 1.25

At 1,50,000 units 1,87,500 1,87,500 ÷1,50,000 = Rs. 1.25

Here, it has been observed that per unit indirect labour cost remains unchanged at different levels of output, but total costs are different at different levels of output. ∴ Indirect Labour Cost is a variable cost. ∴ Total Indirect Labour Cost at 1,40,000 units of output = 1,40,000 units × Rs. 1.25. = Rs. 1,75,000 3. Inspection Total Inspection Cost (Rs.) ∴ Inspection Cost per unit (Re.)

At 1,20,000 units 90,000 90,000 ÷1,20,000 = 0.75

At 1,50,000 units 1,12,500 1,12,500 ÷1,50,000 = 0.75

Here, it has been observed that per-unit Inspection Cost remains unchanged at different levels of output, but Total costs are different at different levels of output. ∴ Inspection Cost is a Variable Cost. ∴ Total Inspection Cost at 1,40,000 units of output = 1,40,000 units × Re. 0.75 = Rs. 1,05,000 (Continued)

Modified Date: Thu, Jul 01, 2010 02:49:57 PM

Output Date: Tue, Jul 06, 2010 12:19:29 PM

Rev II

Project: Management Accounting_Debarshi Bhattacharyya ACE Pro India Pvt. Ltd. File: X:\Pearson\Management Accounting_Debarshi Bhattacharyya\MAIN\M07\LAYOUT_M07\M07_DEBA_ISBN_EN_SE_C07 II.indd

499

BUDGET AND BUDGETARY CONTROL

4. Maintenance At 1,20,000 units 84,000 84,000 ÷1,20,000 = 0.70

Total Maintenance Cost (Rs.) ∴ Maintenance Cost per unit (Rs.)

At 1,50,000 units 1,02,000 1,02,000 ÷1,50,000 = 0.68

Here, it has been observed that neither the total maintenance cost nor the per unit maintenance cost remain unchanged at different levels of output. ∴ Maintenance Cost is a Semi-Variable Cost. At 1,50,000 units of output, Total Maintenance Cost At 1,20,000 units of output, Total Maintenance Cost Increase in Cost Increase in output = 30,000 units

Rs. 1,02,000 84,000 18,000

This amount of increase in cost is due to the variable portion of the expenses included in it. ∴ Variable Maintenance Cost per unit = Rs. 80,000 ÷ 30,000 = Re. 0.60 Now, at 1,20,000 units of output, Rs. 84,000 72,000 12,000

Total Maintenance Cost Variable Maintenance Cost [1,20,000 units × Re. 0.60] Fixed Maintenance Cost

∴ At 1,40,000 units of output, Rs. 84,000 12,000 96,000

Variable Maintenance Cost [1,40,000 units × Re. 0.60] Fixed Maintenance Cost [same as at 1,20,000 units] Total Maintenance Cost at 1,40,000 units of output

∴ Maintenance Cost per unit at 1,40,000 units of output = Rs. 96,000 ÷ 1,40,000 units = Re. 0.69 5. Supervision At 1,20,000 units At 1,50,000 units (Rs.) (Rs.) 1,98,000 2,34,000 1,98,000 ÷1,20,000 = 1.65 2,34,000 ÷1,50,000 = 1.56

Total Supervision Cost ∴ Supervision Cost per unit

Here, it has been observed that neither the total supervision cost nor the per unit supervision cost remain unchanged at different levels of output. ∴ Supervision Cost is a Semi-Variable Cost. At 1,50,000 units of output, Total Supervision Cost At 1,20,000 units of output, Total Supervision Cost Increase in output = 30,000 units

Increase in cost

Rs. 2,34,000 1,98,000 36,000

∴ Variable Supervision Cost per unit = Rs. 36,000 ÷ 30,000 units = Rs. 1.20 Now, at 1,50,000 units of output, Total Supervision Cost Variable Supervision Cost [1,50,000 units × Rs. 1.20] Fixed Supervision Cost

Rs. 2,34,000 1,80,000 54,000

∴ At 1,40,000 units of output, Variable Supervision Cost [1,40,000 units × Rs. 1.20] Fixed Supervision Cost [same as at 1,50,000 units] Total Supervision Cost at 1,40,000 units

Rs. 1,68,000 54,000 2,22,000 (Continued)

Modified Date: Thu, Jul 01, 2010 02:49:57 PM

Output Date: Tue, Jul 06, 2010 12:19:29 PM

Rev II

Project: Management Accounting_Debarshi Bhattacharyya ACE Pro India Pvt. Ltd. File: X:\Pearson\Management Accounting_Debarshi Bhattacharyya\MAIN\M07\LAYOUT_M07\M07_DEBA_ISBN_EN_SE_C07 II.indd

500

MANAGEMENT ACCOUNTING

∴ Supervision Cost per unit at 1,40,000 units of output = Rs. 2,22,000 ÷ 1,40,000 units = Rs. 1.58 6. Depreciation—Plant and Equipment At 1,20,000 units 90,000

Total Depreciation Cost (Rs.)

At 1,50,000 units 90,000

Here, it has been observed that the total depreciation cost at different levels of output remains unchanged. ∴ Depreciation is a Fixed Cost which does not vary with the change in output. ∴ Total Depreciation at 1,40,000 units of output = Rs. 90,000 Then, Depreciation per unit at 1,40,000 units of output = Rs. 90,000 ÷ 1,40,000 units = Re. 0.64 7. Engineering Service At 1,20,000 units 94,000

Total Engineering Service Cost (Rs.)

At 1,50,000 units 94,000

Here, it has been observed that the total engineering service cost remain unchanged at different levels of output. ∴ Engineering Service is a Fixed Cost. ∴ Total Engineering Service at 1,40,000 units of output = Rs. 94,000 ∴ Engineering Service per unit at 1,40,000 units of output = Rs. 94,000 ÷ 1,40,000 units = Re. 0.67 IV. Cash Budget

Problem 19 From the following information, compute the collection from Debtors for the month of June, July and August: April 1,00,000

Sales (Rs.)

May 1,20,000

June 90,000

July 1,05,000

August 1,25,000

20% of sales are made in cash. Debtors are allowed a 2-month credit. They will receive 5% discount on sale if they pay off their dues within 1 month from the date of sale. About four-fifth of the Debtors normally clear their dues to avail the Cash Discount, but the remaining Debtors pay on the due date only. [B.Com. (Hons), Calcutta University—2008] Solution Statement showing computation of Collection from Debtors for the month of June, July and August Cash Sales [20% of current month’s Sales] Collection from Debtors: 1/5th of the Credit Sales for April [1/5 of (80% of Rs. 1,00,000)] 4/5th of the Credit Sales for May [{4/5 of (80% of Rs. 1,20,000)} − 5% discount] 1/5th of the Credit Sales for May [1/5 of (80% of Rs. 1,20,000)] 4/5th of the Credit Sales for June [{4/5 of (80% of Rs. 90,000)} − 5% discount] 1/5th of the Credit Sales for June [1/5 of (80% of Rs. 90,000)] 4/5th of the Credit Sales for July [{4/5 of (80% of Rs. 1,50,000)} − 5% discount] Total Collection from Debtors

Modified Date: Thu, Jul 01, 2010 02:49:57 PM

Output Date: Tue, Jul 06, 2010 12:19:29 PM

June Rs. 18,000

July Rs. 21,000

August Rs. 25,000

16,000 72,960 19,200 54,720 14,400 63,840 88,960

73,920

78,240

Rev II

Project: Management Accounting_Debarshi Bhattacharyya ACE Pro India Pvt. Ltd. File: X:\Pearson\Management Accounting_Debarshi Bhattacharyya\MAIN\M07\LAYOUT_M07\M07_DEBA_ISBN_EN_SE_C07 II.indd

BUDGET AND BUDGETARY CONTROL

501

Problem 20 From the following information, prepare the Cash Budget for the 3 months—September, October and November of 2004: Month

Sales Rs. 50,000 65,000 70,000 60,000 84,000

July August September October November

Purchase Rs. 28,000 32,500 37,200 29,500 39,700

Wages Rs. 3,400 3,700 3,900 4,200 4,500

Factory Expenses Rs. 3,200 3,300 3,450 3,600 3,700

Office Expenses Rs. 4,000 4,200 4,350 4,420 4,800

Additional Information: i. A Sales Commission at 5% on sales, which is due in the month following the month in which the sales dues are collected, is payable in addition to the Office Expenses. ii. The period of credit allowed to Debtors is 2 months. iii. A month’s credit is obtained from the Creditors. iv. Wages are paid on the 1st and 16th of each month in respect of dues for a period of 15 days, preceding those days. v. Other Expenses are paid in the month in which they are due. vi. For purchase of an asset under instalment-payment system, an instalment of Rs. 1,250 per month is paid. vii. Bank Balance on 1st September is Rs. 15,200. [B.Com. (Hons), Calcutta University (Part II)—2005] Solution Cash Budget for the 3 months—September, October and November 2004 Particulars A. Opening Bank Balance B. Cash Receipts during the month: Collection from Debtors1 C.

D.

September Rs. 15,200

Cash Payments during the month: Payment to Creditors2 Payment of Wages4 Payment of Factory Expenses3 Payment of Office Expenses3 Payment of Sales Commission5 Payment of Instalment for the Purchase of Asset Closing Bank Balance (A + B – C)

October Rs. 19,850

November Rs. 31,830

50,000 50,000

65,000 65,000

70,000 70,000

32,500 3,800 3,450 4,350 – 1,250 45,350 19,850

37,200 4,050 3,600 4,420 2,500 1,250 53,020 31,830

29,500 4,350 3,700 4,800 3,250 1,250 46,850 54,980

Working Notes 1. Collection from Debtors As the period of Credit allowed to Debtors is 2 months, the Sales for July, August and September will be collected in September, October and November respectively. (Continued)

Modified Date: Thu, Jul 01, 2010 02:49:57 PM

Output Date: Tue, Jul 06, 2010 12:19:29 PM

Rev II

Project: Management Accounting_Debarshi Bhattacharyya ACE Pro India Pvt. Ltd. File: X:\Pearson\Management Accounting_Debarshi Bhattacharyya\MAIN\M07\LAYOUT_M07\M07_DEBA_ISBN_EN_SE_C07 II.indd

502

MANAGEMENT ACCOUNTING

2. Payment to Creditors As 1 month’s Credit is obtained from the Creditors, the Purchases for August, September and October will be paid in September, October and November respectively. 3. Payment of Factory Expenses and Office Expenses Factory and Office Expenses will be paid in the month during which these expenses are incurred. 4. Payment of Wages As Wages are paid on 1st and 16th of each month, the Wages for the first 15 days of a month will be paid on 16th of that month and the Wages for the last 15 days of that month will be paid on 1st of the next month as follows: September Rs. 1,850 1,950 – – 3,800

Wages for August [1/2 of Rs. 3,700] Wages for September [1/2 of Rs. 3,900] Wages for October [1/2 of Rs. 4,200] Wages for November [1/2 of Rs. 4,500] Total Wages Paid

October Rs. – 1,950 2,100 – 4,050

November Rs. – – 2,100 2,250 4,350

5. Sales Commission September 5% on Sales for June = Sales for June is not given in the problem

Sales Commission payable

October 5% on Sales for July = 5% on Rs. 50,000 = Rs. 2,500

November 5% on Sales for August = 5% on Rs. 65,000 = Rs. 3,250

Problem 21 From the following information, prepare a cash Budget for the quarter that is ending on 30 June 2000: Sales Rs. 1,20,000 1,30,000 80,000 1,16,000 88,000

Month February March April May June

Purchases Rs. 84,000 1,00,000 1,04,000 1,06,000 80,000

Wages Rs. 10,000 12,000 8,000 10,000 8,000

Misc. Exp. Rs. 7,000 8,000 6,000 12,000 6,000

Additional Information: i. Cash on hand on 1 April 2000 is Rs. 5,000. ii. Sales—20% realized in the month of sale, discount allowed is 2% and balance realized is after 2 months. iii. Purchases are paid 1 month after. iv. Wages—25% in arrear paid in the following month. v. Other Expenses are paid at a lag of 1 month. vi. Income Tax of Rs. 25,000 due on or before 30 June 2000. [B.Com. (Hons), Calcutta University—2000] Solution Cash Budget for the quarter ending on 30 June 2000 Particulars [A] Opening Cash & Bank Balance [B] Cash Receipts during the month: Cash Sales1 Collection from Debtors2

April Rs. 5,000 15,680 96,000 1,11,680

May Rs. (–)3206 22,736 1,04,000 1,26,736

June Rs. 6,9166 17,248 64,000 81,248 (Continued)

Modified Date: Thu, Jul 01, 2010 02:49:57 PM

Output Date: Tue, Jul 06, 2010 12:19:29 PM

Rev II

Project: Management Accounting_Debarshi Bhattacharyya ACE Pro India Pvt. Ltd. File: X:\Pearson\Management Accounting_Debarshi Bhattacharyya\MAIN\M07\LAYOUT_M07\M07_DEBA_ISBN_EN_SE_C07 II.indd

BUDGET AND BUDGETARY CONTROL

[C]

[D]

Cash Payments during the month: Payment to Creditors for Goods3 Payment of Wages4 Payment of Other Expenses5 Income Tax Payment Closing Cash & Bank Balance [A + B – C]

1,00,000 9,000 8,000 – 1,17,000 (–)320

1,04,000 9,500 6,000 – 1,19,500 6,916

503

1,06,000 8,500 12,000 25,000 1,51,500 (–) 63,336

Working Notes 1. Receipts from Cash Sales In April = (20% of Rs. 80,000) – {2% of (20% of Rs. 80,000)} = Rs. 15,680 In May = (20% of Rs. 1,16,000) – {2% of (20% of Rs 1,16,000)} = Rs. 22,736 In June = (20% of Rs. 88,000) – {2% of (20% of Rs. 88,000)} = Rs. 17,248 2. Collection from Debtors 20% of the Total Sales for each month represent Cash Sales. ∴ Credit Sales = 80% of the Total Sales Credit Sales are to be realized after 2 months. ∴ Collection from Debtors in April = 80% of Total Sales in February = 80% of Rs. 1,20,000 = Rs. 96,000 Collection from Debtors in May = 80% of Total Sales in March = 80% of Rs. 1,30,000 = Rs. 1,04,000 Collection from Debtors in June = 80% of Total Sales in April = 80% of Rs. 80,000 = Rs. 64,000 3. Payment to Creditors for Goods Credit allowed by the creditors for Goods is 1 month. ∴ Payment to Creditors in April = Payment for Purchases in March = Rs. 1,00,000 Payment to Creditors in May = Payment for Purchases in April = Rs. 1,04,000 Payment to Creditors in June = Payment for Purchases in May = Rs. 1,06,000 4. Payment of Wages 25% of the Wages payable for each month to be paid in the subsequent month. ∴ Wages paid in each month = 25% of Wages payable for the previous month + 75% of the Wages payable for the current month. Rs. ∴Payment of Wages in April: 25% of Wages of March [i.e., 25% of Rs. 12,000] 75% of Wages of April [i.e., 75% of Rs. 8,000] Payment of Wages in May: 25% of Wages of April [i.e., 25% of Rs. 8,000] 75% of Wages of May [i.e., 75% of Rs. 10,000] Payment of Wages in June: 25% of Wages of May [i.e., 25% of Rs. 10,000] 75% of Wages of June [i.e., 75% of Rs. 8,000]

3,000 6,000 9,000 2,000 7,500 9,500 2,500 6,000 8,500

5. Payment of Other Expenses A lag in the payment of other expenses is 1 month. Therefore, the other expenses payable for each month to be paid in the subsequent month. ∴ Payment of Other Expenses in April = Other Expenses of March = Rs. 8,000 Payment of Other Expenses in May = Other Expenses of April = Rs. 6,000 Payment of Other Expenses in June = Other Expenses of May = Rs. 12,000 6. Closing Cash & Bank Balance of April = Opening Cash & Bank Balance of May And, Closing Cash & Bank Balance of May = Opening Cash & Bank Balance of June

Modified Date: Thu, Jul 01, 2010 02:49:57 PM

Output Date: Tue, Jul 06, 2010 12:19:29 PM

Rev II

Project: Management Accounting_Debarshi Bhattacharyya ACE Pro India Pvt. Ltd. File: X:\Pearson\Management Accounting_Debarshi Bhattacharyya\MAIN\M07\LAYOUT_M07\M07_DEBA_ISBN_EN_SE_C07 II.indd

504

MANAGEMENT ACCOUNTING

Problem 22 i. Prepare a Cash Budget for the 3 months that is ending on 30 June 2009 from the following information: Month

Sales Rs. 14,000 15,000 16,000 17,000 18,000

February March April May June

Materials Rs. 9,600 9,000 9,200 10,000 10,400

Wages Rs. 3,000 3,000 3,200 3,600 4,000

Overheads Rs. 1,700 1,900 2,000 2,200 2,300

ii. 10% of the sales are in cash. iii. Credit terms are: Debtors: 50% of the Credit Sales are collected next month and the balance in the following month. Creditors: For Materials – 2 months; For Wages – 1/4 month; For Overheads – 1/2 month. iv. Plant & Machinery will be installed in February 2009 at a cost of Rs. 96,000. The monthly instalment of Rs. 2,000 is payable from April onwards. v. A Dividend @ 5% on Preference Share Capital of Rs. 2,00,000 will be paid on 1 June. vi. Advance to be received for sale of vehicles of Rs. 9,000 in June. vii. Dividend from investments amounting to Rs. 1,000 is expected to be received in June. viii. Income Tax (advance) to be paid in June of Rs. 2,000. ix. Cash and Bank Balance on 1 April 2002 is expected to be Rs. 6,000. [I.C.W.A. (Inter)—Adapted] Solution Cash Budget for the 3 Months that is ending on 30 June 2009 Particulars [A] Opening Cash & Bank Balance [B] Cash Receipts during the month: Cash Sales1 Collection from Debtors2 Advance for sale of vehicle Dividend from investment [C] Cash Payments during the month: Payment to Creditors for Goods3 Payment of Wages4 Payment of Overheads5 Instalment for Purchase of Plant & Machinery Dividend on Preference Shares (5% on Rs. 2,00,000) Advance Income Tax [D] Closing Cash & Bank Balance [A + B – C]

April Rs. 6,000

May Rs. 3,9506

June Rs. 3,0006

1,600 13,050 – – 14,650

1,700 13,950 – – 15,650

1,800 14,850 9,000 1,000 26,650

9,600 3,150 1,950 2,000 –

9,000 3,500 2,100 2,000 –

9,200 3,900 2,250 2,000 10,000

– 16,700 3,950

– 16,600 3,000

2,000 29,350 300

Working Notes 1. Calculation of Cash Sales In April = 10% of Rs. 16,000 = Rs. 1,600 In May = 10% of Rs. 17,000 = Rs. 1,700 In June = 10% of Rs. 18,000 = Rs. 1,800 (Continued)

Modified Date: Thu, Jul 01, 2010 02:49:57 PM

Output Date: Tue, Jul 06, 2010 12:19:29 PM

Rev II

Project: Management Accounting_Debarshi Bhattacharyya ACE Pro India Pvt. Ltd. File: X:\Pearson\Management Accounting_Debarshi Bhattacharyya\MAIN\M07\LAYOUT_M07\M07_DEBA_ISBN_EN_SE_C07 II.indd

BUDGET AND BUDGETARY CONTROL

505

2. Collection from Debtors Cash Sales for each month = 10% of Total Sales for that month ∴ Credit Sales for each month = 90% of Total Sales for that month 50% of the Credit Sales for each month is realized in the subsequent month and the Balance 50% is realized next to the subsequent month. Collection from Debtors in April: 50% of (90% of Total Sales of February), i.e., [50% of (90% of Rs. 14,000)] 50% of (90% of Total Sales of March), i.e., [50% of (90% of Rs. 15,000)] Total collection from Debtors in April Again, Collection from Debtors in May: 50% of (90% of Total Sales of March), i.e., [50% of (90% of Rs. 15,000)] 50% of (90% of Total Sales of April), i.e., [50% of (90% of Rs. 16,000)] Total collection from Debtors in May Again, Collection from Debtors in June: 50% of (90% of Total Sales of April), i.e., [50% of (90% of Rs. 16,000)] 50% of (90% of Total Sales of May), i.e., [50% of (90% of Rs. 17,000)] Total collection from Debtors in June

Rs. 6,300 6,750 13,050 6,750 7,200 13,950 7,200 7,650 14,850

3. Payment to Creditors for Goods Credit allowed by the Creditors is 2 months. Therefore, Creditors are to be paid off after 2 months, immediately succeeding the month of Purchase. ∴ Payment to Creditors in April = Purchase of Materials in the month of February = Rs. 9,600 Payment to Creditors in May = Purchase of Materials in the month of March = Rs. 9,000 Payment to Creditors in June = Purchase of Materials in the month of April = Rs. 9,200 4. Payment of Wages A lag in the payment of Wages is 1/4 month. Again, 1 month = 4 weeks. Therefore, 1/4 month = 1 week. Hence, the lag in payment of Wages = 1 week, that is, the wages for the first week of a month is paid in the second week of that month, wages for the second week is paid in the third week of that month, wages for the third week is paid in the fourth week of that month and wages for the fourth week is paid in the first week of the subsequent month. ∴ Payment of wages for each month = 1/4th of the wages for the previous month + 3/4th of the wages for the current month Consequently, payment of wages in April = 1/4 of wages for March + 3/4 of wages in April = 1/4 of Rs. 3,000 + 3/4 of Rs. 3,200 = Rs. 3,150 Payment of wages in May = 1/4 of wages for April + 3/4 of wages for May = 1/4 of Rs. 3,200 + 3/4 of Rs. 3,600 = Rs. 3,500 Payment of wages in June = 1/4 of wages for May + 3/4 of wages for June = 1/4 of Rs. 3,600 + 3/4 of Rs. 4,000 = Rs. 3,900 5. Payment of Overheads Lag in the payment of overheads is 1/2 month, that is, the overheads for the first half of a month is paid during the second half of that month and the overheads for the second half of a month is paid during the first half of the next month. ∴ Payment of Overheads for each month =1/2 of the overheads for the previous month + 1/2 of the overheads for the current month ∴ Payment of Overheads in April = 1/2 of overheads for March + 1/2 of overheads for April = 1/2 of Rs. 1,900 + 1/2 of Rs. 2,000 = Rs. 1,950 Payment of Overheads in May = 1/2 of overheads for April + 1/2 of overheads for May = 1/2 of Rs. 2,000 + 1/2 of Rs. 2,200 = Rs. 2,100 Payment of Overheads in June = 1/2 of overheads for May + 1/2 of overheads for June = 1/2 of Rs. 2,200 + 1/2 of Rs. 2,300 = Rs. 2,250 6. Closing Cash and Bank Balance at the end of April represent the Opening Cash and Bank Balance at the beginning of May. Similarly, the Closing Cash and Bank Balance at the end of May represent the cash and balance at the beginning of June.

Modified Date: Thu, Jul 01, 2010 02:49:57 PM

Output Date: Tue, Jul 06, 2010 12:19:29 PM

Rev II

Project: Management Accounting_Debarshi Bhattacharyya ACE Pro India Pvt. Ltd. File: X:\Pearson\Management Accounting_Debarshi Bhattacharyya\MAIN\M07\LAYOUT_M07\M07_DEBA_ISBN_EN_SE_C07 II.indd

506

MANAGEMENT ACCOUNTING

Problem 23 From the following particulars, prepare the monthly Cash Budget of the Sunclear Ltd for October, November and December of 1996: Month (1996) July August September October November December

Purchases Rs. 40,000 60,000 50,000 70,000 80,000 60,000

Sales Rs. 60,000 80,000 70,000 90,000 1,00,000 1,20,000

Wages Rs. 8,000 10,500 17,500 17,100 12,000 12,000

Expenses Rs. 10,000 12,000 12,500 11,600 11,800 12,300

It is expected that 50% of sales will be in cash and 25% of the purchases can be made on credit. Debtors are allowed a 2-months credit, but will receive a cash discount of 5% if they will pay off their dues within the month next to the month of sale. About 80% of the Debtors normally clear their dues at the end of that period to avail the cash discount. But the remaining 20% of the Debtors pay on the due date only. About 4/5th of the credit purchase is paid after 1 month of that purchase and next to that month, the balance 1/5th is paid. Wages are paid within fifth of the following month. Expenses include Selling and Distribution Expenses which are 10% of the sales. Any deficiency in cash at the end of a month will be met by taking a short-term loan for 2 months from the bank. At the end of September 1996, the Sunclear Ltd had Rs. 40,000 cash in hand. [B.Com. (Hons), Calcutta University—1997] Solution Cash Budget of Sunclear Ltd for the 3 months ending on 31 December 1996 Particulars [A] Opening Cash Balance [B] Cash Receipts during the month: Cash Sales1 Collection from Debtors2 [C] Cash Payments during the month: Cash Purchases of Goods3 Payment to Creditors for Goods4 Payment of Wages5 Payment of Expenses6 [D] Closing Cash Balance [A + B – C]

October Rs. 40,000

November Rs. 25,0007

December Rs. 10,8007

45,000 34,600 79,600

50,000 41,200 91,200

60,000 47,000 1,07,000

52,500 13,000 17,500 11,600 94,600 25,000

60,000 16,500 17,100 11,800 1,05,400 10,800

45,000 19,500 12,000 12,300 88,800 29,000

Working Notes 1. Cash Sales October = 50% of Rs. 90,000 = Rs. 45,000 November = 50% of Rs. 1,00,000 = Rs. 50,000 December = 50% of Rs. 1,20,000 = Rs. 60,000 (Continued)

Modified Date: Thu, Jul 01, 2010 02:49:57 PM

Output Date: Tue, Jul 06, 2010 12:19:29 PM

Rev II

Project: Management Accounting_Debarshi Bhattacharyya ACE Pro India Pvt. Ltd. File: X:\Pearson\Management Accounting_Debarshi Bhattacharyya\MAIN\M07\LAYOUT_M07\M07_DEBA_ISBN_EN_SE_C07 II.indd

507

BUDGET AND BUDGETARY CONTROL

2. Collection from Debtors Cash Sales for each month = 50% of Total Sales for that month ∴ Credit Sales for each month = Remaining 50% of Total Sales for that month Generally, 2 months’ Credit is allowed to the Debtors. But if they pay off their dues within the next month to the month of sale, a discount of 5% is offered. Here, out of the Total Debtors of each month, 80% of them have cleared their dues in the next month to avail a 5% discount offer and the remaining 20% of the debtors pay their dues after 2 months only. ∴ Collection from Debtors in October: 20% of Credit Sales in August, i.e., {20% of (50% of Rs. 80,000)} 80% of Credit Sales in September less 5% discount i.e., [{80% of (50% of Rs. 70,000)} – 5%] ∴ Collection from Debtors in November: 20% of Credit Sales in September, i.e., {20% of (50% of Rs. 70,000)} 80% of Credit Sales in October less 5% discount, i.e., [{80% of (50% of Rs. 90,000)} – 5%] ∴ Collection from Debtors in December: 20% of Credit Sales in October, i.e., 20% of (50% of Rs. 90,000) 80% of Credit Sales in November less 5% discount, i.e., [{80% of (50% of Rs. 1,00,000)} – 5%]

Rs. 8,000 26,600 34,600 7,000 34,200 41,200 9,000 38,000 47,000

3. Cash Purchase of Goods Rs. Credit Purchases for each month = 25% of the Total Purchases ∴ Cash Purchases for each month = 75% of the Total Purchases ∴ Cash Purchases in September = 75% of Rs. 70,000 ∴ Cash Purchases in October = 75% of Rs. 80,000 ∴Cash Purchases in November = 75% of Rs. 60,000

52,500 60,000 45,000

4. Payment to Creditors for Goods ∴ Payment to Creditors in October: 1/5th of the Credit Purchase of August, i.e., 1/5th of [25% of Rs. 60,000] 4/5th of Credit Purchase of September, i.e., 4/5 of [25% of Rs. 50,000] ∴ Payment to Creditors in November: 1/5th of Credit Purchase of September, i.e., 1/5 of [25% of Rs. 50,000] 4/5th of Credit Purchase of October, i.e., 4/5 of [25% of Rs. 70,000] ∴ Payment to Creditors in December: 1/5th of Credit Purchase of October, i.e., 1/5 of [25% of Rs. 70,000] 4/5th of Credit Purchase of November i.e., 4/5 of [25% of Rs. 80,000]

Rs. 3,000 10,000 13,000 2,500 14,000 16,500 3,500 16,000 19,500

5. Payment of Wages Wages for each month is paid within fifth of the next month. ∴ A lag in the Payment of Wages = 1 month ∴ Payment of Wages in October = Payment of Wages for September = Rs. 17,500 ∴ Payment of Wages in November = Payment of Wages for October = Rs. 17,100 ∴ Payment of Wages in December = Payment of Wages for November = Rs. 12,000 6. Payment of Expenses As no information is given in the problem regarding the lag in the payment of expenses, it is assumed that the expenses payable for each month is paid in that month only. (Continued)

Modified Date: Thu, Jul 01, 2010 02:49:57 PM

Output Date: Tue, Jul 06, 2010 12:19:29 PM

Rev II

Project: Management Accounting_Debarshi Bhattacharyya ACE Pro India Pvt. Ltd. File: X:\Pearson\Management Accounting_Debarshi Bhattacharyya\MAIN\M07\LAYOUT_M07\M07_DEBA_ISBN_EN_SE_C07 II.indd

508

MANAGEMENT ACCOUNTING

Again, these expenses include both selling and distribution expenses also. Therefore, no separate payment is required for selling and distribution expenses. 7. Cash Balance at the end of October = Opening Cash Balance of November And, Cash Balance at the end of November = Cash Balance at the beginning of December. On the other hand, as there is no deficit in cash in every month, the requirement of raising a short-term loan is not required.

Problem 24 From the following information, prepare a Cash Budget of DB Ltd for 4 months that ended on 31 December 2006: Sales Rs. 60,000 90,000 75,000 90,000 1,05,000 1,20,000

Months July August September October November December

Purchases Rs. 40,000 60,000 50,000 60,000 70,000 80,000

Wages Rs. 8,000 10,500 17,500 17,100 12,000 12,000

Other Expenses Rs. 9,000 8,850 8,000 7,850 7,300 7,050

Additional Information: i. Cash in hand on 01 September 2006 – Rs. 47,000. ii. It is expected that 50% of the sales will be in cash and 25% of the purchases will be made on credit. iii. 75% of the Credit Sales is realized 1 month after the sales, 20% realized is after 2 months of sale and the balance is bad debt. iv. 70% of the credit purchases is paid after 1 month and the remaining 30% is paid after 2 months of purchase. v. Wages are paid on the 3rd day of the following month. vi. Other Expenses include a depreciation of Rs. 2,000 p.m. vii. Commission @ 5% on the total sales payable after 1 month is not included in the other expenses. viii. Machinery is purchased @ Rs. 60,000 and the payment is to be made in September. ix. A Dividend on investments of Rs. 10,250 to be received in November. x. The company has 10% Debentures of Rs. 1,00,000. The interest is payable quarterly in March, June, September and December. xi. In case of deficit in cash in any month, the company can arrange an overdraft from the bank for that month, and such overdraft is to be repaid as early as possible out of the Surplus Cash of the subsequent month. Ignore the interest on the overdraft. [B.Com. (Hons), Calcutta University—2007] Solution Cash Budget of DB Ltd for 4 months that ended on 31 December 2006 Particulars A. Opening Cash Balance B. Receipts during the month: Cash Sales [50% of current months’ Sales]

September Rs. 47,000

October Rs. Nil

November Rs. Nil

December Rs. Nil

37,500

45,000

52,500

60,000 (Continued)

Modified Date: Thu, Jul 01, 2010 02:49:57 PM

Output Date: Tue, Jul 06, 2010 12:19:29 PM

Rev II

Project: Management Accounting_Debarshi Bhattacharyya ACE Pro India Pvt. Ltd. File: X:\Pearson\Management Accounting_Debarshi Bhattacharyya\MAIN\M07\LAYOUT_M07\M07_DEBA_ISBN_EN_SE_C07 II.indd

509

BUDGET AND BUDGETARY CONTROL

Particulars Collection from Debtors1 Dividend on Investment Total Receipts C. Payments during the month: Cash Purchases [75% of current months’ Purchases] Payment to Creditors2 Wages [Last months’ Wages] Other Expenses [Current months’ expenses less depreciation per month] Commission on Sales [5% of last months’ Sales] Payment for Machinery Purchased Interest on Debentures [10% on Rs. 1,00,000 for 3 months] Total Payments D. Surplus/Deficit of Cash [A + B – C] E. Overdraft taken from bank F. Overdraft paid off G. Closing Cash Balance

September Rs. 39,750 – 77,250

October Rs. 37,125 – 82,125

November Rs. 41,250 10,250 1,04,000

December Rs. 48,375 – 1,08,375

37,500

45,000

52,500

60,000

13,500 10,500

13,250 17,500

14,250 17,100

16,750 12,000

6,000

5,850

5,300

5,050

4,500

3,750

4,500

5,250

60,000 2,500





2,500

1,34,500 (10,250)

85,350 (3,225)

93,650 10,350

1,01,550 6,825

10,250 – Nil

3,225 – Nil

– (10,350) Nil

– (3,125) 3,700

Note The bank overdraft repaid in December = Overdraft taken in September and October less the overdraft repaid in November = Rs. 10,250 + Rs. 3,225 – Rs. 10,350 = Rs. 3,125

Working Notes 1. Computation of Collection from Debtors

From Credit Sales of July [20% of (50% of 60,000)] From Credit Sales of August [75% of (50% of 90,000)] [20% of (50% of 90,000)] From Credit Sales of September [75% of (50% of 75,000)] [20% of (50% of 75,000)] From Credit Sales of October [75% of (50% of 90,000)] [20% of (50% of 90,000)] From Credit Sales of November [75% of (50% of 1,05,000)] Collection from Debtors

September Rs. 6,000

October Rs.

November Rs.

December Rs.

33,750 9,000 28,125 7,500 33,750 9,000

39,750

37,125

41,250

39,375 48,375 (Continued)

Modified Date: Thu, Jul 01, 2010 02:49:57 PM

Output Date: Tue, Jul 06, 2010 12:19:29 PM

Rev II

Project: Management Accounting_Debarshi Bhattacharyya ACE Pro India Pvt. Ltd. File: X:\Pearson\Management Accounting_Debarshi Bhattacharyya\MAIN\M07\LAYOUT_M07\M07_DEBA_ISBN_EN_SE_C07 II.indd

510

MANAGEMENT ACCOUNTING

2. Computation of Payments to Creditors September Rs. For Credit Purchases of July [30% of (25% of 40,000)] For Credit Purchases of August [70% of (25% of 60,000)] [30% of (25% of 60,000)] For Credit Purchases of September [70% of (25% of 50,000)] [30% of (25% of 50,000)] For Credit Purchases of October [70% of (25% of 60,000)] [30% of (25% of 60,000)] For Credit Purchases of November [70% of (25% of 70,000)] Payment to Creditors

October Rs.

November Rs.

December Rs.

3,000 10,500 4,500 8,750 3,750 10,500 4,500

13,500

13,250

14,250

12,250 16,750

Problem 25 i. Prepare a Cash Budget for 3 months ending on 30 September 2000 from the following information: Month May June July August September

Sales Rs. 56,000 60,000 64,000 68,000 72,000

Materials Rs. 19,200 18,000 18,400 20,000 20,800

Wages Rs. 6,000 6,000 6,400 7,200 8,000

Overheads Rs. 3,400 3,800 4,000 4,400 4,600

ii. Credit Terms are: Sales/Debtors: 10% sales are on cash. 50% of the Credit Sales are collected in the next month and the balance in the following month. Creditors: Raw Materials – 1 month; Wages – 1/2 month; Overheads – 1/4 month. iii. Other Relevant Information: (a) Rent from a godown let out on 1 January 1999 at a yearly contract of Rs. 2,25,000 with a condition of 12% increase in each year is receivable in equal instalments in each month. (b) Interest @ 15% p.a. on an amount of Rs. 2,00,000 borrowed on 1 January 1998 is to be paid each month starting from 1 March 2000. Interest for the period from 1 January 1998 to 29 February 2000 is to be paid in 10 equal monthly instalments starting from 1 July 2000. Interest for the month of April 2000 remains unpaid. This unpaid amount is to be paid in the month of September 2000 with a penal interest of Rs. 100. (c) An advance of Rs. 50,000 will be paid to a contractor for the construction of a building in September 2000. (d) Adequate overdraft facility may be arranged for, if needed. [B.Com. (Hons), Calcutta University—2001]

Modified Date: Thu, Jul 01, 2010 02:49:57 PM

Output Date: Tue, Jul 06, 2010 12:19:29 PM

Rev II

Project: Management Accounting_Debarshi Bhattacharyya ACE Pro India Pvt. Ltd. File: X:\Pearson\Management Accounting_Debarshi Bhattacharyya\MAIN\M07\LAYOUT_M07\M07_DEBA_ISBN_EN_SE_C07 II.indd

511

BUDGET AND BUDGETARY CONTROL

Solution Cash Budget for 3 months ending on 30 September 2000 Particulars A. Opening Cash & Bank Balance B. Receipts during the month: Cash Sales [10% of Total Sales in each month] Collection from Debtors1 Instalment of Rent from Godown2 C. Payments during the month: Creditors for Raw Material3 Wages4 Overheads5 Interest on Loan6 Advance for Construction of Building D. Closing Cash & Bank Balance [A + B – C]

July Rs. Nil

August Rs. 42,450

September Rs. 87,550

6,400

6,800

7,200

52,200 21,000 79,600

55,800 21,000 83,600

59,400 21,000 87,600

18,000 6,200 3,950 9,000 – 37,150 42,450

18,400 6,800 4,300 9,000 – 38,500 87,550

20,000 7,600 4,550 11,600 50,000 93,750 81,400

 Working Notes 1. Collection from Debtors Collection from Debtors in July: 50% of the Credit Sales of May i.e., 50% of [90% of Rs. 56,000] 50% of the Credit Sales of June i.e., 50% of [90% of Rs. 60,000]

Rs. 25,200 27,000 52,200

Collection from Debtors in August: 50% of the Credit Sales of June i.e., 50% of [90% of Rs. 60,000] 50% of the Credit Sales of July i.e., 50% of [90% of Rs. 64,000]

27,000 28,800 55,800

Collection from Debtors in September: 50% of the Credit Sales of July i.e., 50% of [90% of Rs. 64,000] 50% of the Credit Sales of August i.e., 50% of [90% of Rs. 68,000]

28,800 30,600 59,400

2. Instalment of Rent from Godown Yearly Rent from Godown in 1999 Add: 12% increase in next year [12% of Rs. 2,25,000] Yearly Rent from Godown for the year 2000

Rs. 2,25,000 27,000 2,52,000

∴ Monthly instalment receivable as Rent of Godown in 2000 = Rs. 2,52,000 ÷ 12 = Rs. 21,000 (Continued)

Modified Date: Thu, Jul 01, 2010 02:49:57 PM

Output Date: Tue, Jul 06, 2010 12:19:29 PM

Rev II

Project: Management Accounting_Debarshi Bhattacharyya ACE Pro India Pvt. Ltd. File: X:\Pearson\Management Accounting_Debarshi Bhattacharyya\MAIN\M07\LAYOUT_M07\M07_DEBA_ISBN_EN_SE_C07 II.indd

512

MANAGEMENT ACCOUNTING

3. Creditors for Raw Materials Payment to Creditors for Raw Materials in July = Purchase of Raw Materials in June = Rs. 18,000 Payment to Creditors for Raw Materials in August = Purchase of Raw Materials in July = Rs. 18,400 Payment to Creditors for Raw Materials in September = Purchase of Raw Materials in August = Rs. 20,000 4. Payment of Wages Rs. ∴ Payment of Wages in July: 50% of Wages for June i.e., 50% of Rs. 6,000 50% of the Wages for July i.e., 50% of Rs. 6,400

3,000 3,200 6,200

∴ Payment of Wages in August: 50% of the Wages for July i.e., 50% of Rs. 6,400 50% of the Wages for August i.e., 50% of Rs. 7,200

3,200 3,600 6,800

∴ Payment of Wages in September: 50% of the Wages for August i.e., 50% of Rs. 7,200 50% of the Wages for September i.e., 50% of Rs. 8,000

3,600 4,000 7,600

5. Payment of Overheads Rs. ∴ Payment of Overheads in July: 1/4th of the Overheads of June i.e., 1/4 of Rs. 3,800 3/4th of the Overheads of July i.e., 3 /4 of Rs. 4,000

950 3,000 3,950

∴ Payment of Overheads in August: 1/4th of the Overheads of July i.e., 1/4 of Rs. 4,000 3/4th of the Overheads of August i.e., 3/4 of Rs. 4,400

1,000 3,300 4,300

∴ Payment of Overheads in September: 1/4th of the Overheads of August i.e., 1/4 of Rs. 4,400 3/4th of the Overheads of September i.e., 3/4 of Rs. 4,600

1,100 3,450 4,550

6. Payment of Interest on Loan: Total Arrear Interest from 1 January 1998 to 29 February 2000 = 15% on Rs. 2,00,000 for 26 months = Rs. 65,000 This Total Arrear Interest is payable in 10 equal monthly instalments commencing from 1 July 2000. ∴ Arrear Interest paid in each month = Rs. 65,000 ÷ 10 = Rs. 6,500 (Continued)

Modified Date: Thu, Jul 01, 2010 02:49:57 PM

Output Date: Tue, Jul 06, 2010 12:19:29 PM

Rev II

Project: Management Accounting_Debarshi Bhattacharyya ACE Pro India Pvt. Ltd. File: X:\Pearson\Management Accounting_Debarshi Bhattacharyya\MAIN\M07\LAYOUT_M07\M07_DEBA_ISBN_EN_SE_C07 II.indd

BUDGET AND BUDGETARY CONTROL

513

Rs. ∴ Payment of Interest in July: Interest for the month of July [15% on Rs. 2,00,000 for 1 month] Instalment for Arrear Interest

2,500 6,500 9,000

∴ Payment of Interest in August: Interest for the month of August [15% on Rs. 2,00,000 for 1 month] Instalment for Arrear Interest

2,500 6,500 9,000

Payment of Interest in September: Interest for the month of September [15% on Rs. 2,00,000 for 1 month] Instalment for Arrear Interest Interest for the month of April [15% on Rs. 2,00,000 for 1 month] Penal Interest paid for the late payment of interest for the month of April

2,500 6,500 2,500 100 11,600

Problem 26 Prepare a Cash Budget for the 3 months that ended on 30 June 2010 on the basis of the following information: i. Estimated Sales and Expenses are as follows: January Rs. 1,20,000 24,000 21,000

Sales Wages & Salaries Miscellaneous Expenses

February Rs. 1,00,000 24,000 30,000

March Rs. 1,50,000 24,000 24,000

April Rs. 2,40,000 30,000 27,000

May Rs. 2,00,000 27,000 34,000

June Rs. 2,00,000 27,000 17,000

ii. 20% of the sales are in cash and the balance on credit. iii. The firm has a Gross Margin of 25% on sales. iv. 50% of the Credit Sales are collected in the month following the sales, 30% in the second month and balance 20% in the third month. v. Material for the sale of each month is purchased 1 month in advance on a credit for 2 months. vi. Time lag in the payment of wages and salaries is one-third of a month and of Miscellaneous Expenses is 1 month. vii. Debentures worth Rs. 40,000 are sold in April 2010. viii. The firm maintains a minimum cash balance of Rs. 40,000. Funds can be borrowed @ 12% p.a. in the multiples of Rs. 1,000, the interest being payable on monthly basis. ix. Cash Balance at the end of March 2010 is Rs. 64,800. Solution Cash Budget for the 3 months ended on 30 June 2010 Particulars A. Opening Cash Balance B. Cash Receipts during the month: Cash Sales [20% of Sales of each month]

April Rs. 64,800

May Rs. 91,500

June Rs. 40,500

48,000

40,000

40,000 (Continued)

Modified Date: Thu, Jul 01, 2010 02:49:57 PM

Output Date: Tue, Jul 06, 2010 12:19:29 PM

Rev II

Project: Management Accounting_Debarshi Bhattacharyya ACE Pro India Pvt. Ltd. File: X:\Pearson\Management Accounting_Debarshi Bhattacharyya\MAIN\M07\LAYOUT_M07\M07_DEBA_ISBN_EN_SE_C07 II.indd

514

MANAGEMENT ACCOUNTING

Particulars 1

Collection from Debtors Sales proceeds of Debentures C. Cash Payments during the month: Payment to Creditors for Materials2 Payment of Wages & Salaries3 Payment of Miscellaneous Expenses D. E. F. G. H. I.

Balance [A + B – C] Minimum Cash Balance to be maintained Loan to be raised [if D < E only] [in multiples of Rs. 1,000] Repayment of Principal of Loan4 Repayment of Interest on Loan4 Closing Cash Balance [D + F – G – H]

April Rs. 1,03,200 40,000 1,91,200

May Rs. 1,48,000 – 1,88,000

June Rs. 1,61,600 – 2,01,600

1,12,500 28,000 24,000 1,64,500 91,500 40,000 Nil – – 91,500

1,80,000 28,000 34,000 2,42,000 37,500 40,000 3,000 – – 40,500

1,50,000 28,000 17,000 1,95,000 47,100 40,000 Nil 3,000 30 44,070

Working Notes 1. Collection from Debtors April Rs. Out of Credit Sales for January: [20% of (80% of Rs. 1,20,000)] Out of Credit Sales for February: [30% of (80% of Rs. 1,00,000)] [20% of (80% of Rs. 1,00,000)] Out of Credit Sales for March: [50% of (80% of Rs. 1,50,000)] [30% of (80% of Rs. 1,50,000)] [20% of (80% of Rs. 1,50,000)] Out of Credit Sales for April: [50% of (80% of Rs. 2,40,000)] [30% of (80% of Rs. 2,40,000)] Out of Credit Sales for May: [50% of (80% of Rs. 2,00,000)] Total Collection

May Rs.

June Rs.

19,200 24,000 16.000 60,000 36,000 24,000 96,000 57,600

1,03,200

1,48,000

80,000 1,61,600

May Rs. 2,00,000 50,000 1,50,000

June Rs. 2,00,000 50,000 1,50,000

2. Calculation of Purchase of Materials and payment to Creditors for Materials

Less:

Sales Gross Margin or Sales [25% on Sales] Purchases of Materials

March Rs. 1,50,000 37,500 1,12,500

April Rs. 2,40,000 60,000 1,80,000

Now, Payment to Creditors for Materials: Payment to Creditors

April For Purchases for March bought in February = Rs. 1,12,500

May For Purchases for April bought in March = Rs. 1,80,000

June For Purchases for May bought in April = Rs. 1,50,000 (Continued)

Modified Date: Thu, Jul 01, 2010 02:49:57 PM

Output Date: Tue, Jul 06, 2010 12:19:29 PM

Rev II

Project: Management Accounting_Debarshi Bhattacharyya ACE Pro India Pvt. Ltd. File: X:\Pearson\Management Accounting_Debarshi Bhattacharyya\MAIN\M07\LAYOUT_M07\M07_DEBA_ISBN_EN_SE_C07 II.indd

BUDGET AND BUDGETARY CONTROL

515

3. Payment of Wages and Salaries April Rs. 8,000

1/3rd of Salaries & Wages for March [1/3 of Rs. 24,000] 2/3rd of Salaries & Wages for April [2/3 of Rs. 30,000] 1/3rd of Salaries & Wages for April [1/3 of Rs. 30,000] 2/3rd of Salaries & Wages for May [2/3 of Rs. 27,000] 1/3rd of Salaries & Wages for May [1/3 of Rs. 27,000] 2/3rd of Salaries & Wages for June [2/3 of Rs. 27,000] Payment of Wages & Salaries

May Rs.

June Rs.

20,000 10,000 18,000 9,000 18,000 28,000

28,000

27,000

4. Repayment of Principal amount of Loan and Interest thereon in June It is assumed that a Loan of Rs. 3,000 will be taken at the end of May, and as the fund permits, it will be repaid at the end of June together with the Interest thereon. ∴ Interest payable on Loan = 12% on Rs. 3,000 for 1 month = Rs. 30

Problem 27 From the following information, prepare a Cash Budget of a company for four quarters of a year:

Opening Cash Balance Collection from Customers Payments: Purchase of Materials Other Expenses Salaries & Wages Income Tax Purchase of Machinery

1st Quarter Rs. 10,000 1,25,000

2nd Quarter Rs.

3rd Quarter Rs.

4th Quarter Rs.

1,50,000

1,60,000

2,21,000

20,000 25,000 90,000 5,000 –

35,000 20,000 95,000 – –

35,000 20,000 95,000 – –

54,200 17,000 1,09,200 – 20,000

The company desires to maintain a cash balance of Rs. 15,000 at the end of each quarter. Cash can be borrowed at an interest of 10% p.a. in multiples of Rs. 500; and when it will be repaid, the same too to be made in multiples of Rs. 500 only. Management does not want to borrow cash more than what is necessary and wants to repay as early as possible. In any event, loan cannot be extended beyond the four quarters. Interest is computed and paid once the principal is repaid. Assume that borrowings take place at the beginning of the quarter and repayments are made at the end of the quarter. Solution Cash Budget for four quarters of a year Particulars A. Opening Cash Balance B. Receipt during the quarter excluding borrowing: Collection from Customers

1st Quarter Rs. 10,000

2nd Quarter Rs. 15,000

3rd Quarter Rs. 15,000

4th Quarter Rs. 15,325

1,25,000

1,50,000

1,60,000

2,21,000 (Continued)

Modified Date: Thu, Jul 01, 2010 02:49:57 PM

Output Date: Tue, Jul 06, 2010 12:19:29 PM

Rev II

Project: Management Accounting_Debarshi Bhattacharyya ACE Pro India Pvt. Ltd. File: X:\Pearson\Management Accounting_Debarshi Bhattacharyya\MAIN\M07\LAYOUT_M07\M07_DEBA_ISBN_EN_SE_C07 II.indd

516

MANAGEMENT ACCOUNTING

Particulars C. Payments during the quarter excluding repayment of Loan: Purchase of Raw Materials Other Expenses Salaries & Wages Income Tax Purchase of Machinery D. Closing Surplus/Deficit of Cash [A + B – C – 15,000] E. Loan taken1 F. Repayment2: Principal of Loan Interest on Loan G. Closing Cash Balance

1st Quarter Rs. 1,25,000

2nd Quarter Rs. 1,50,000

20,000 25,000 90,000 5,000 – 1,40,000 (–) 5,000

35,000 20,000 95,000 – – 1,50,000 Nil

20,000



– – 15,000

– – 15,000

3rd Quarter Rs. 1,60,000

4th Quarter Rs. 2,21,000

35,000 20,000 95,000 – – 1,50,000 (+) 25,000

54,200 17,000 1,09,200 – 20,000 2,00,400 (+) 35,925

– (–) 9,000 (–) 675 15,325

– (–) 11,000 (–) 1,100 23,825

Working Notes 1. Loan taken in 1st quarter Deficit in Cash at the end of 1st quarter Minimum Closing Cash Balance to be maintained Loan taken in 1st quarter

Rs. 5,000 15,000 20,000

2. Repayment of Loan in 3rd quarter and 4th quarter In 3rd quarter: Surplus in Cash at the end of 3rd quarter Less: Minimum Closing Cash Balance to be maintained Amount available for repayment of Loan & its Interest

Rs. Rs. 25,000 Rs. 15,000 Rs. 10,000

Here, the cash available to pay off the principal of loan and its interest is Rs. 10,000. As the principal of the loan is repayable together with interest, this entire cash of Rs. 10.000 cannot be utilized for the payment of principal alone. Again, the principal is payable in multiples of Rs. 500. ∴ Payments of Principal of the Loan together with Interest are made in the following way:

Add:

∴ Repayment of Loan in 3rd quarter Interest paid in 3rd quarter [10% on Rs. 9,000 for 9 months (i.e., 3 quarters)] Payment of Principal & Interest on Loan paid in 3rd Quarter

Rs. 9,000 675 9,675

In 4th quarter:

Add:

Repayment of Balance of Principal of Loan taken (Rs. 20,000 – 9,000) Interest on above Loan paid [10% of Rs. 11,000 for 12 months (i.e., for 4 quarters)] Payment of Principal & Interest on Loan in 4th quarter

Modified Date: Thu, Jul 01, 2010 02:49:57 PM

Output Date: Tue, Jul 06, 2010 12:19:29 PM

Rs. 11,000 1,100 Rs. 12,100

Rev II

Project: Management Accounting_Debarshi Bhattacharyya ACE Pro India Pvt. Ltd. File: X:\Pearson\Management Accounting_Debarshi Bhattacharyya\MAIN\M07\LAYOUT_M07\M07_DEBA_ISBN_EN_SE_C07 II.indd

BUDGET AND BUDGETARY CONTROL

517

V. Master Budget

Problem 28 From the following particulars, prepare a Forecasted Profit & Loss A/c (P & L A/c) for the year that ended on 31 December 2009, a Forecasted Balance Sheet as on that date, a Purchase Budget for the year 2009, a Sales Budget for the year 2009 and a Cash Budget for the year 2009: i. Profit & Loss A/c for the year that ended on 31 December 2008 Rs. 50,000 30,000 20,000 20,000 20,000

To Materials Consumed To Wages [direct] To Factory Overheads [60% variable] To Administrative Overheads [fixed] To Selling & Distribution Overheads [60% fixed] To Net Profit before Tax c/d

60,000 2,00,000 30,000 30,000 60,000 10,000 20,000 30,000

To Income Tax To Net Profit after Tax c/d To Dividend To Balance c/f

By Sales [1,000 units]

Rs. 2,00,000

By Net Profit before Tax b/d

2,00,000 60,000

By Net Profit after Tax b/d

60,000 30,000 30,000

ii. Balance Sheet as on 31 December 2008 Liabilities Share Capital: 6,000 Equity Shares of Rs. 10 each fully paid up Reserve & Surplus Sundry Creditors Provision for Tax Proposed Dividend

Rs. 60,000 40,000 20,000 30,000 10,000 1,60,000

Assets Fixed Assets [at cost less depreciation] Stock of Raw Materials Sundry Debtors Cash & Bank Balance

Rs. 1,00,000 20,000 30,000 10,000 1,60,000

Additional Information: i. The present level of activity is 50%. In the year 2009, it is expected to operate at 75% capacity. However, in order to sell the additional production in the market, the Selling Price per unit is to be reduced by 5% in the year 2009. ii. Market Price forecasts indicate that cost of material, labour and variable overheads are likely to increase by 4%, 5% and 5% respectively. Fixed Costs (other than depreciation) are also expected to go up by 5% due to annual increments of salaries. iii. Fixed Costs include depreciation, which is a Fixed Instalment of Rs. 5,000 p.a., charged in full to production overhead. iv. Three months’ requirements of raw materials are to be held in stock. The FIFO method is generally used in pricing out the issues. v. All units started for production are expected to be completed and sold in the Budget period. vi. Sales and purchases are generally made on 2 months’ credit. Wages and expenses are paid within the period. vii. Machinery Costing Rs. 25,000 is expected to be purchased for cash in December 2009. viii. Rate of Income Tax is 50% and, if profit permits, a dividend of 20% may be proposed.

Modified Date: Thu, Jul 01, 2010 02:49:57 PM

Output Date: Tue, Jul 06, 2010 12:19:29 PM

Rev II

Project: Management Accounting_Debarshi Bhattacharyya ACE Pro India Pvt. Ltd. File: X:\Pearson\Management Accounting_Debarshi Bhattacharyya\MAIN\M07\LAYOUT_M07\M07_DEBA_ISBN_EN_SE_C07 II.indd

518

MANAGEMENT ACCOUNTING

Solution Sales Budget for the year 2009 Add:

Actual Sales in units at 50% capacity for the year 2008 Expected Increase in Sales due to Increase in activity level [1,000 units × 25% / 50%] ∴ Budgeted Sales in units at 75% capacity for the year 2009 Actual Selling Price per unit for the year 2008

1,000 units 500 units 1,500 units Rs. 200

⎡ Total Sales in 2008 Rs. 2, 00, 000 ⎤ = ⎢ ⎥ Units sold 1, 000 ⎣ ⎦ Less:

Expected reduction in Selling Price per unit in the year 2002 [5% of Rs. 200] Budget Selling Price per unit for the year 2009 ∴ Budgeted Total Sales in value for the year 2009 [Budgeted Selling Units × Budgeted Selling Price per unit = 1,500 units × Rs. 190]

Rs. 10 Rs. 190 Rs. 2,85,000

Purchase Budget for the year 2009

Add:

Add:

Less: Add:

Consumption of Raw Materials at 50% capacity level for the year 2008 Expected Increase in consumption of Raw Materials due to an increase in the activity level [Rs. 50,000 × 25%/50%]. Cost of Raw Materials to be consumed in the year 2009 [at last year’s price] Expected Stock of Raw Materials required at the end of 2009 [at last year’s price] [3 months÷12 months × Rs. 75,000] Opening Stock of Raw Materials as per Balance Sheet as on 31 December 2008 Net Purchase of Raw Materials during the year 2009 [at last year’s price] Expected Increase in price of Raw Materials in 2009 [4% of Rs. 73,750] Budgeted Purchase of Raw Materials for the year 2009 [at the price of the year 2009]

Rs. 50,000 25,000 75,000 18,750 93,750 20,000 73,750 2,950 76,700

Cash Budget for the year that ended on 31 December 2009 Rs. Add: Less:

Opening Cash & Bank Balance Receipts during the year: Collection from Debtors 6 Payments during the year: Payments to Creditors 7 Payment of Wages2 Payment of Factory Overheads excluding Depreciation3 [Rs. 39,650 – Rs. 5,000] Payment of Administrative Overheads4 Payment of Selling & Distribution Overheads5 Payment of Income Tax8 Payment of Dividend9 Purchase of Machinery

Rs. 10,000 2,67,500 2,77,500

83,917 47,250 34,650 21.000 25,200 30,000 10,000 25,000 2,77,017 483

Closing Cash & Bank Balance

Forecasted Profit & Loss A/c for the year that ended on 31 December 2009 Dr. Particulars To Raw Materials consumed: Opening Stock

Rs. 20,000

Rs.

Particulars By Sales [as per Sales Budget]

Rs.

Cr. Rs 2,85,000 (Continued)

Modified Date: Thu, Jul 01, 2010 02:49:57 PM

Output Date: Tue, Jul 06, 2010 12:19:29 PM

Rev II

Project: Management Accounting_Debarshi Bhattacharyya ACE Pro India Pvt. Ltd. File: X:\Pearson\Management Accounting_Debarshi Bhattacharyya\MAIN\M07\LAYOUT_M07\M07_DEBA_ISBN_EN_SE_C07 II.indd

519

BUDGET AND BUDGETARY CONTROL

Dr.

Cr. Particulars Add: Purchase of Raw Materials [as per Purchase Budget]

Rs. 76,700 96,700 19,500

Less: Closing Stock1 To Wages (direct)2 To Factory Overheads3 To Administrative Overheads4 To Selling & Distribution Overheads5 To Net Profit before Tax c/d

Rs.

37,350 74,700 12,000

To Proposed Dividend [20% on paid-up Share Capital = 20% of Rs. 60,000] To Balance c/f

Rs.

Rs

77,200 47,250 39,650 21,000 25,200 74,700 2,85,000 37,350

To Income Tax [50% of Rs. 74, 700] To Net Profit after Tax c/d

Particulars

2,85,000 74,700

By Net Profit before Tax b/d

74,700 37,350

By Net Profit after Tax b/d

25,350 37,350

37,350

Forecasted Balance Sheet as on 31 December 2009 Liabilities Issued & Subscribed Share Capital 6,000 Equity Shares of Rs. 10 each fully paid up

Add:

Reserves & Surplus As per last Balance Sheet Net Profit for the year Current liabilities & provisions Creditors7 Provision for Tax Proposed Dividend

Rs.

Rs.

60,000

40,000 25,350

65,350

12,783 37,350 12,000

Assets Fixed Assets Opening Balance Less: Depreciation for the year Add: Addition during the year Current Assets, Loans & Advances Stock of Raw Materials1 Debtors6 Cash & Bank Balance [as per Cash Budget]

Rs.

Rs.

1,00,000 5,000 95,000 25,000

1,87,483

1,20,000

19,500 47,500 483 1,87,483

Working Notes 1. Expected Stock of Raw Materials on 31 December 2009

Add:

Expected Stock of Raw Materials required on 31 December 2009 [at last year’s price] as per Purchase Budget 4% Increase in Cost of Raw Materials in 2009 [4% of Rs. 18,750] Expected Stock of Raw Materials on 31 December 2009 [at the price of 2009]

Rs. 18,750 750 19,500 (Continued)

Modified Date: Thu, Jul 01, 2010 02:49:57 PM

Output Date: Tue, Jul 06, 2010 12:19:29 PM

Rev II

Project: Management Accounting_Debarshi Bhattacharyya ACE Pro India Pvt. Ltd. File: X:\Pearson\Management Accounting_Debarshi Bhattacharyya\MAIN\M07\LAYOUT_M07\M07_DEBA_ISBN_EN_SE_C07 II.indd

520

MANAGEMENT ACCOUNTING

2. Budgeted Wages (Direct) for the year 2009

Add:

Wages per unit produced & sold in the year 2008 [Rs. 30,000 ÷ 1,000 units] 5% Increase in the rate of Wages in the year 2009 [5% of Rs. 30] Budgeted Wages per unit for the year 2009 Budgeted units to be produced & sold in the year 2009 [1,000 units × 75% ÷ 50%] ∴ Budgeted Total Wages (direct) for the year 2009 [1,500 units × Rs. 31.50]

Rs. 30.00 Rs. 1.50 Rs. 31.50 1,500 units Rs. 47,250

3. Budgeted Factory Overheads for the year 2009 Rs.

Add:

Factory Overheads for the year 2008: Variable Factory Overheads [60% of Rs. 20,000] Fixed Factory Overheads [40% of Rs. 20,000] Total Factory Overheads for 2008 ∴ Variable Factory Overheads per unit for the year 2008 [Rs. 20,000 ÷ 1,000 units] 5% Increase in Cost of Variable Overhead in 2009 [5% of Rs. 20] Budgeted Variable Factory Overheads per unit for the year 2009 ∴ Budgeted Factory Overheads for the year 2009: Variable Factory Overheads [1,500 units × Rs. 21] Fixed-Factory Overheads: Depreciation Other Fixed Factory Overheads [(Rs. 8,000 – Rs. 5,000) + 5% increase] Budgeted Total Factory Overheads for the year 2009

Rs. 12,000 8,000 20,000 20 1 21 31,500

5,000 3,150 8,150 39,650

4. Budgeted Administrative Overheads for the year 2009

Add:

Administrative Overheads for the year 2008 5% Increase in Fixed Costs in the year 2009 [5% of Rs. 20,000] Budgeted Total Administrative Overheads for the year 2009

Rs. 20,000 1,000 21,000

5. Budgeted Selling and Distribution Overheads for the year 2009 Selling & Distribution Overheads for the year 2008: Fixed Selling & Distribution Overheads [60% of Rs. 20,000] Variable Selling & Distribution Overheads [40% of Rs. 20,000] Total Selling & Distribution Overheads in 2008

Add:

∴ Variable Selling & Distribution Overheads per unit in 2008 5% Increase in the Cost of Variable Overheads in 2009 [5% of Rs. 8] Budgeted Variable Selling & Distribution Overheads per unit for the year 2009 ∴ Budgeted Selling & Distribution Overheads for the year 2009: Variable Selling & Distribution Overheads [1,500 units ÷ Rs. 8.40] Fixed Selling & Distribution Overheads [Rs. 12,000 + 5% increase] Budgeted Total Selling & Distribution Overheads for the year 2009

Rs. 12,000 8,000 20,000 Rs. 8.00 0.40 8.40 Rs. 12,600 12,600 25,200 (Continued)

Modified Date: Thu, Jul 01, 2010 02:49:57 PM

Output Date: Tue, Jul 06, 2010 12:19:29 PM

Rev II

Project: Management Accounting_Debarshi Bhattacharyya ACE Pro India Pvt. Ltd. File: X:\Pearson\Management Accounting_Debarshi Bhattacharyya\MAIN\M07\LAYOUT_M07\M07_DEBA_ISBN_EN_SE_C07 III.indd

521

BUDGET AND BUDGETARY CONTROL

6. Receipts from Debtors during 2009 and Balance of Debtors on 31 December 2009

Add: Less:

Opening Balance of Debtors as per last Balance Sheet Budgeted Sales for the year 2009 Expected Debtors’ Balance on 31 December 2009 [2 months’ sale = 2 ÷ 12 × Rs. 2,85,000] Budgeted Receipts from Debtors during the year 2009

Rs. 30,000 2,85,000 3,15,000 47,500 2,67,500

7. Payments to Creditors during 2009 and Balance of Creditors on 31 December 2009

Add: Less:

Opening Balance of Creditors as per last Balance Sheet Budgeted Purchase for the year 2009 Expected Creditors’ Balance on 31 December 2009 [2 months’ Purchase = 2 ÷ 12 × Rs. 76,700] Budgeted Payments to Creditors during the year 2009

Rs. 20,000 76,700 96,700 12,783 83,917

8. Payment of Tax during the year 2009 Provision for Income Tax for the year 2008 (as per last Balance Sheet) = Rs. 30,000 As nothing is otherwise stated in the problem, it is assumed that the provision for Tax for the year 2008 is maintained in the year 2009. ∴ Provision for Tax for the year 2009 = Rs. 30,000 9. Payment of Dividend during the year 2009 Proposed Dividend for the year 2008 (as per last Balance Sheet) = Rs. 10,000 As nothing is otherwise stated in the problem, it is assumed that the proposed Dividend for the year 2008 is maintained in the year 2009. ∴ Proposed Dividend for the year 2009 = Rs. 10,000

CHAPTER REVIEW SUMMARY Budget refers to a plan covering all the sectors of operations that are expressed in monetary and/or quantitative terms for a definite future period of time.  Budgeting refers to the act of preparing Budgets. It is the managerial action of formulating Budgets.  Features of Budget are as follows: (i) It should reflect the managerial plans and policies to achieve business goals and objectives; (ii) It is expressed either in monetary terms or quantitative terms or both; (iii) It is a comprehensive plan for a definite future period; and (iv) Though it is basically an instrument of planning, it provides the basis for performance evaluation and control.  Budgetary Control refers to the application of a comprehensive system of Budgeting in the organization to assist the management in the process of its planning, coordinating, controlling and performance evaluation. It is an effective tool to the management to achieve the business goals and objectives of the organization.  Objectives of Budgetary Control are: (a) Performance evaluation; (b) Planning; (c) Defining responsibilities; (d) Coordinating; (e) Communicating; (f) Motivating; and (g) Cost Control.  Advantages of Budgetary Control are: (a) It acts as a very useful and effective tool of controlling the costs; (b) It provides yardsticks for the evaluation of actual performance; (c) It clearly defines the areas of responsibility of all the concerned executives who are engaged in various business activities; (d) It points out the efficiency of various business activities; and (e) It increases the operational efficiency of all business activities. 

Modified Date: Mon, Jun 28, 2010 06:20:11 PM

Output Date: Tue, Jul 06, 2010 12:20:11 PM

Rev II

Project: Management Accounting_Debarshi Bhattacharyya ACE Pro India Pvt. Ltd. File: X:\Pearson\Management Accounting_Debarshi Bhattacharyya\MAIN\M07\LAYOUT_M07\M07_DEBA_ISBN_EN_SE_C07 III.indd

522 









MANAGEMENT ACCOUNTING

Limitations of Budgetary Control are: (a) Budget Plans are based on estimates which may not be accurate in all cases; (b) It plays a limited role in the process of controlling various business activities; (c) Budgetary Control system introduced in an organization may be resisted by some employees who are not as much efficient as others; and (d) Introduction of Budgetary Control system in an organization is an expensive programme. Budget is a quantitative plan of action for a future period, whereas Budgetary Control is a system of controlling costs and performances of various business activities through preparation of Budget, assigning responsibilities, evaluation of actual performances by comparing the actual results with the Budgeted data and taking remedial measures in case of any adverse deviation observed. Distinction between Standard Costing and Budgetary Control are as follows: (a) Standard Costing is based on technical assessment, whereas Budgetary Control is based on past performance adjusted with future trend; (b) Standard Costing is the method of preparation of Standard Cost and application of those for measuring the efficiency of the Actual Cost, whereas Budgetary Control is the method of Forecasting Cost by preparation of a Budget prior to a definite period to attain the given objectives; (c) Standard Cost is a projection of Cost Accounts, whereas Budget is a projection of Financial Accounts; and (d) Standard Costing is mainly concerned with the ascertainment and control of costs, whereas Budgetary Control is mainly concerned with the profitability and financial position of a concern. Zero-based Budgeting (ZBB) is an alternative Budgeting system that starts with the zero base. It is a newly invented Budget technique where executives are required to start at the Zero-Budget Level every year and justify all the costs of the existing function in comparison with all the present and future functions. Budget is usually classified into many parts and sub-parts on the basis of its nature of requirement to the organization. Classification or types of Budget are: (a) Classification on the basis of period; (b) Classification on the basis of production flexibility; and (c) Classification on the basis of function or coverage.

CHAPTER REVIEW QUIZ 1. Fill in the blanks of the following statements: is a summary of all Functional Budgets. a. Budget. b. Cash Budget is a is a Budgeting process whereby each item of the Budget has to be justified in its entirety, c. whether it is an existing item or new. determines the priorities of Functional Budgets. d. is the most important Budget among all Budgets. e. and . f. Budgetary Control helps the management in its process of is a Budget by which the Budgeted Costs are ascertained for different activity levels. g. shows the expected inflows and outflows of cash. h. Ans.: (a) Master Budget; (b) short-term; (c) ZBB; (d) Principal Budget Factors; (e) Sales Budget; (f) planning, controlling; (g) Flexible Budget; (h) Cash Budget. 2. State whether the following statements are true or false: a. The principal Budget factor constitutes the starting point for the preparation of various Budgets. b. Fixed Budgets are most suitable for Fixed Costs. c. There is no difference between Budget and forecast. d. Production Budget is the basis for Sales Budget. e. Sales Budget is the basis for preparation of all other Operating Budgets. f. All Functional Budgets should be coordinated with Sales Budget as it is prepared first. g. Flexible Budget is generally prepared by a business whose demand of goods is stable. h. A Master Budget consists of Operating Budgets, Capital Budget and Estimated Financial Statements. i. A system by which Budgets are used as a means of planning and controlling all aspects of a business is called Budgetary Control. j. A system of Budgetary Control cannot be used in an organization where Standard Costing is in use. Ans.: True: (a), (b), (e), (h), (i); False: (c), (d), (f), (g), ( j).

Modified Date: Mon, Jun 28, 2010 06:20:11 PM

Output Date: Tue, Jul 06, 2010 12:20:11 PM

Rev II

Project: Management Accounting_Debarshi Bhattacharyya ACE Pro India Pvt. Ltd. File: X:\Pearson\Management Accounting_Debarshi Bhattacharyya\MAIN\M07\LAYOUT_M07\M07_DEBA_ISBN_EN_SE_C07 III.indd

523

BUDGET AND BUDGETARY CONTROL

EXERCISE I. Theoretical Questions A. Short Answer Type Questions:

1. 2. 3. 4. 5. 6. 7. 8. 9.

What is a Budget? What is a Master Budget? What is a Flexible Budget? What is Cash Budget? What is Fixed Budget? What is Current Budget? What is Operating Budget? What is Budgetary Control? List the important Functional Budgets prepared by a business.

B. Essay Type Questions:

1. 2. 3. 4. 5. 6.

What is a Budget? What are its features? What is Budgeting? What do you mean by Budgetary Control? What are its objectives? What are the advantages and limitations of Budgetary Control? Explain various classifications of Budget. What do you mean by Zero-based Budgeting (ZBB)? Compare the following: (a) Budget and Budgetary Control; (b) Standard Costing and Budgetary Control; (c) Fixed Budget and Flexible Budget; (d) Conventional Budget and Zero-based Budget. 7. Write notes on the following: (a) Budgetary Control; (b) Master Budget; (c) Flexible Budget; (d) Zero-based Budgeting; (e) Capital Expenditure Budget; (f) Production Budget; (g) Sales Budget; (h) Cash Budget; and (i) Operating Budget. II.

Practical Problems

I.

Production Budget

1. From the following information, prepare a Production Budget for the year that ended on 31 March 2010: Products

Sales as Per Sales Budget (units)

P Q R S

61,000 46,875 75,000 50,000

Estimated Stock (units) On 1 April 2009 On 31 March 2010 4,000 5,500 8,125 11,250 6,250 12,500 1,250 7,500

Ans.: Budgeted Production: P—62,500 units; Q—50,000 units; R—81,250 units; S—50,000 units. 2. From the following information, prepare a Production Budget for the year 2010: Stock on 31 December 2009 Rs.

Budgeted Sales for 2010 Rs.

Standard Cost Per unit Rs.

10,000 12,000 20,000

40,000 50,000 1,00,000

5 7 3

2,000 4,000

20,000 10,000

10 12

Product Group ‘Jaya’: X Y Z Product Group ‘Pati’: M N

Stock of Y and Z is to be maintained at 10% above the existing level to sustain the Budgeted Sales, whereas the stock level of X may be reduced by 20%. Stock of M is proposed to be raised by 6,000 units.

Modified Date: Mon, Jun 28, 2010 06:20:11 PM

Output Date: Tue, Jul 06, 2010 12:20:11 PM

Rev II

Project: Management Accounting_Debarshi Bhattacharyya ACE Pro India Pvt. Ltd. File: X:\Pearson\Management Accounting_Debarshi Bhattacharyya\MAIN\M07\LAYOUT_M07\M07_DEBA_ISBN_EN_SE_C07 III.indd

524

MANAGEMENT ACCOUNTING

Ans.: Budgeted Production: X—38,000 units; Y—51,200 units; Z—1,02,000 units; M—20,000 units; N—16,000 units. Budgeted Production Cost: X—Rs. 1,90,000; Y—Rs. 3,58,400; Z—Rs. 3,06,000; M—Rs. 2,00,000; N—Rs. 1,92,000. 3. The following are the estimated Sales of a company for 8 months ending 30 November 2010: Months (2010) April May June July August September October November

Estimated Sales (units) 12,000 13,000 9,000 8,000 10,000 12,000 14,000 12,000

As a matter of policy, the company maintains the Closing Balances of Finished Goods and Raw Materials as follows: Stock Item Finished Goods Raw Materials Every unit of production requires 2 kg of Raw Material

Closing Balance of Month 50% of the estimated Sales for the next month Estimated consumption for the next month Costing Rs. 5 per kg.

Prepare a Production Budget (in units) and Raw Material Purchase Budget (in units and cost) of the company for the half-year that is ending on 30 September 2010. [I.C.W.A. (Inter)—Adapted] Ans.: Total Production—65,000 units; Purchase (total)—1,31,000 kg, Rs. 6,55,000. 4. The sales director of a manufacturing company reports that he expects to sell 54,000 units of a certain product in the next year. The production manager consults the store-keeper and casts his figures as follows: Two types of raw materials, P and Q, are required for manufacturing the product. Each unit of the product requires two units of P and three units of Q. The estimated stocks at the commencement of the next year are: Finished Products—10,000 units; Raw Material P—12,000 units; Raw Material Q—15,000 units. The desirable Closing Stocks at the end of the next year are: Finished products—14,000 units; Raw Material P—13,000 units; Raw Material Q—16,000 units. Draw up a material purchase Budget for the next year. [M.Com. C.U.—Adapted] Ans.: Budget Purchases: X—1,17,000 units; Y—1,75,000 units. 5. Prepare a Production Budget for each month and a Summarized Production Cost Budget for the 6-month period ending on 31 December 2010, from the following data: (a) Months July 2010 August 2010 September 2010 October 2010 November 2010 December 2010 January 2011

Modified Date: Mon, Jun 28, 2010 06:20:11 PM

Output Date: Tue, Jul 06, 2010 12:20:11 PM

Budgeted units to be sold 1,100 1,100 1,700 1,900 2,500 2,300 2,000

Rev II

Project: Management Accounting_Debarshi Bhattacharyya ACE Pro India Pvt. Ltd. File: X:\Pearson\Management Accounting_Debarshi Bhattacharyya\MAIN\M07\LAYOUT_M07\M07_DEBA_ISBN_EN_SE_C07 III.indd

525

BUDGET AND BUDGETARY CONTROL

(b) Estimated finished stock at the end of each month is equal to half of the estimated Sales for the next month. (c) Budgeted production and production cost for the year ending 31 December 2010 are as follows: Production Direct Materials per unit Direct Wages per unit Total Factory Overheads for the year (100% variable)

22,000 units Rs. 10 Rs. 4 Rs. 88,000

Ans.: Budgeted Production: July—1,100 units; August—1,400 units; September—1,800 units; October—2,200 units; November—2,400 units; December—2,150 units; Total Budgeted Production Cost—Rs. 1,98,900. 6. A company is drawing its production plan for the year 2009–10 in respect of two of its products ‘Gamma’ and ‘Delta.’ The company’s policy is not to carry any closing WIP (work-in-progress) at the end of any month. However, its policy is to hold a Closing Stock of Finished Goods at 50% of the anticipated quantity of sales of the succeeding month. For the year 2009–10, the company’s Budgeted Production is 20,000 units of ‘Gamma’ and 25,000 units of ‘Delta.’ The following is the estimated cost data:

Direct Materials per unit Direct Labour per unit Other Manufacturing Expenses apportionable to each type of product, based on Production

Gamma Rs. 50 20

Delta Rs. 80 30

2,00,000

3,75,000

The estimated units to be sold in the first 7 months of the year 2009–10 are as follows: Gamma Delta

April 900 2,900

May 1,100 2,900

June 1,400 2,500

July 1,800 2,100

August 2,200 1,700

September 2,200 1,700

October 1,800 1,900

You are required to: a. Prepare a Production Budget showing month-wise number of units to be manufactured. b. Present a summarized Cost Budget for the half-year ending on 30 September 2009. [I.C.W.A. (Inter)—Adapted] Ans.: (a) April—2,900 units; May—2,700 units; June—2,300 units; July—1,900 units; August—1,700 units; September—1,800 units; (b) Gamma—Rs. 8,04,000, Delta—Rs. 16,62,500. 7. Lodha & Co. manufactures two products, R & S. A forecast of the number of units to be sold in the first 7 months of a year is given as follows: Product R units 1,000 1,200 1,600 2,000 2,400 2,400 2,000

Months January February March April May June July

Product S units 2,800 2,800 2,400 2,000 1,600 1,600 1,800

It is anticipated that: i. There will be no WIP at the end of any month. ii. Finished units equal to half the sales for the next month will be in stock at the end of each month.

Modified Date: Mon, Jun 28, 2010 06:20:11 PM

Output Date: Tue, Jul 06, 2010 12:20:11 PM

Rev II

Project: Management Accounting_Debarshi Bhattacharyya ACE Pro India Pvt. Ltd. File: X:\Pearson\Management Accounting_Debarshi Bhattacharyya\MAIN\M07\LAYOUT_M07\M07_DEBA_ISBN_EN_SE_C07 III.indd

526

MANAGEMENT ACCOUNTING

iii. Budgeted Production and Production Cost for the whole Budgeted year are as follows: Production Direct Materials per unit Direct Wages per unit Total Factory Overhead apportioned to each type of product [100% variable]

Product R 22,000 units Rs. 12.50 Rs. 4.50

Product S 24,000 units Rs. 19.00 Rs. 7.00

Rs. 66,000

Rs. 96,000

Prepare a Production Budget for each month and a summarized Production Cost Budget for the 6 months that ends on 30th June of the year. Ans.: Budgeted Cost of Production: R—Rs. 4,40,000; S—7,20,000. 8. A company manufactures two products A and B by making use of two types of materials, namely, X and Y. Product A requires 10 units of X and 3 units of Y. Product B requires 5 units of X and 2 units of Y. The price of X is Rs. 2 per unit and that of Y is Rs. 3 per unit. Standard hours allowed per product are 4 and 3, respectively. Budgeted wage rate is Rs. 8 per hour. Overtime premium is 50% and is payable, if a worker works for more than 40 hours a week. There are 150 workers. The Sales Manager has estimated the sales of product A to be 5,000 units and product B 10,000 units. The target productivity ratio (or efficiency ratio) for the productive hours worked by the direct worker in actually manufacturing the product is 80%; in addition, the non-productive downtime is Budgeted at 20% of the productive hours worked. In the Budget period, 5-day weeks are 12; and it is anticipated that sales and production will occur evenly throughout the whole period. It is anticipated that the stock at the beginning of the period will be: Product A—800 units and Product B—1,680 units. The targeted Closing Stock expressed in terms of anticipated activity during the Budget period are: Product A—12-day sales and Product B—18-day sales. The Opening and Closing Stock of Raw Material of X and Y will be maintained according to the requirement of stock position for Product A and B. You are required to prepare the following for the next period: i. Material Usage and Material Purchase Budget in terms of quantities and values. ii. Production Budget. iii. Wages Budget for the direct workers. [C.A. (PE II)—May 2005] Ans.: Material Usage [units] Material Purchase [units] Material Cost [Rs.] Production [units] Normal Labour Hours required for production Overtime Hours Total Hours for Production Total Wages [Rs.]

Material X 1,00,000 1,08,600 2,17,200 5,200

Material Y 35,000 38,240 1,14,720 11,320

Total

72,000 10,140 82,140 6,97,680

II. Sales Budget

9. A manufacturing company submits the following information for the first quarter of 2010: Sales (in units): January February March Selling price per unit [Rs.]

Product K

Product L

Product M

25,000 20,000 30,000 10

30,000 25,000 35,000 20

10,000 10,000 10,000 40 (Continued)

Modified Date: Mon, Jun 28, 2010 06:20:11 PM

Output Date: Tue, Jul 06, 2010 12:20:11 PM

Rev II

Project: Management Accounting_Debarshi Bhattacharyya ACE Pro India Pvt. Ltd. File: X:\Pearson\Management Accounting_Debarshi Bhattacharyya\MAIN\M07\LAYOUT_M07\M07_DEBA_ISBN_EN_SE_C07 III.indd

527

BUDGET AND BUDGETARY CONTROL

Product K Target for 1st quarter of 2010: Sales quantity increase [%] Selling Price increase [%] Stock position on 1 January 2010: % of January-2010 Sales [%] Stock position on 31 March 2010 Stock position at the end of January & February 2010: % of subsequent months’ Sales [%]

Product L

Product M

20 Nil

20 10

10 25

50 20,000

50 25,000

50 5,000

50

50

50

You are required to prepare the Sales and Production Budget for the first quarter of 2010. Ans.: Sales (units): January February March Sales (Rs.): January February March Production (units): January February March

K

L

M

30,000 24,000 36,000

33,000 27,500 38,500

11,000 11,000 11,000

3,00,000 2,40,000 3,60,000

7,26,000 6,05,000 8,47,000

5,50,000 5,50,000 5,50,000

27,000 30,000 38,000

30,250 33,000 44,250

11,000 11,000 10,500

III. Flexible Budget

10. A factory engaged in manufacturing plastic buckets is working @ 40% capacity and produces 10,000 buckets per month. The present cost break-up for one bucket is as follows: Rs. 10 3 5

Material Labour Overhead [60% fixed]

The Selling Price is Rs. 20 per bucket. If it is decided to work the factory at 50% capacity, the Selling Price falls by 3%. At 90% capacity, the Selling Price falls by 5%, accompanied by a similar fall in the price of material. You are required to prepare a statement showing the profit at 50% and 90% capacities and also to calculate the break-even points at each of these production levels. [I.C.W.A. (Inter)—June 1993] Ans.: At 40% 20,000 6,000

Profit [Rs.] Break-even point [units]

At 50% 25,000 6,818

At 90% 71,250 6,667

11. A company is currently running at 50% capacity and produces 5,000 units at a cost of Rs. 90 per unit as per the given following details: Material Labour Factory Overheads [Rs. 6 fixed] Administrative Overheads [Rs. 5 variable] Current Selling Price

Modified Date: Mon, Jun 28, 2010 06:20:11 PM

Output Date: Tue, Jul 06, 2010 12:20:11 PM

Rs. 50 15 15 10 100

Rev II

Project: Management Accounting_Debarshi Bhattacharyya ACE Pro India Pvt. Ltd. File: X:\Pearson\Management Accounting_Debarshi Bhattacharyya\MAIN\M07\LAYOUT_M07\M07_DEBA_ISBN_EN_SE_C07 III.indd

528

MANAGEMENT ACCOUNTING

At 60% Working Capacity, the Material Cost per unit increases by 2% and the Selling Price per unit falls by 2%. At 80% Working Capacity, the Material Cost per unit increases by 5% and the Selling Price per unit falls by 5%. Estimate the Profits of the factory at 50%, 60% and 80% Working Capacity and offer your comments. [I.C.W.A. (Inter)—June 1990] Ans.: Profits: At 50%—Rs. 50,000; At 60%—Rs. 53,000; At 80%—Rs. 53,000. 12. From the following data, prepare a Flexible Budget for production of 40,000 units, 60,000 units and 75,000 units of product X, distinctly showing the variable and Fixed Cost as well as the Total Cost. Also indicate the element-wise cost per unit. 1,00,000 units Per-unit Cost Rs. 90 45 10 40 5 5 10 15

Budget Output Direct Material Direct Labour Direct Variable Expenses Manufacturing Variable Overheads Fixed Production Overheads Administration Overheads [fixed] Selling Overheads [10% fixed] Distribution Overheads [20% fixed]

[I.C.W.A. (Inter)—December 1994] Ans.: At 40,000 units, 60,000 units and 75,000 units: Total Cost (lakhs)—Rs. 96.40, Rs. 137.60 and Rs. 168.50, respectively. 13. From the following data, prepare a Flexible Budget for production of 40,000 units, 60,000 units and 75,000 units of product Z, distinctly showing the variable and Fixed Cost as well as the Total Cost. Also indicate the element-wise cost per unit. 1,00,000 Units Rs.

Budgeted Output Budgeted Cost Per Unit: Direct Material Direct Labour Direct Variable Expenses Manufacturing Variable Overheads Fixed-Production Overheads Administrative Overheads [fixed] Selling Overheads [10% fixed] Distribution Overheads [20% fixed]

90 45 10 40 5 5 10 15

[I.C.W.A. (Inter)—Adapted] Ans.:

Budgeted Total Cost Budgeted Cost per unit

units 40,000 Rs. 96,40,000 241.00

units 60,000 Rs. 1,37,60,000 229.33

units 75,000 Rs. 1,68,50,000 224.67

14. Rohini Ltd prepared the Budget for the production of 1,00,000 units of the only commodity manufactured by it for the year 2010 as follows: Rs. 2.52 0.75

Raw Material per unit Direct Labour per unit

(Continued)

Modified Date: Mon, Jun 28, 2010 06:20:11 PM

Output Date: Tue, Jul 06, 2010 12:20:11 PM

Rev II

Project: Management Accounting_Debarshi Bhattacharyya ACE Pro India Pvt. Ltd. File: X:\Pearson\Management Accounting_Debarshi Bhattacharyya\MAIN\M07\LAYOUT_M07\M07_DEBA_ISBN_EN_SE_C07 III.indd

BUDGET AND BUDGETARY CONTROL

529

Rs. 0.10 2.50 0.40 0.20

Direct Expenses per unit Factory Overheads per unit [60% fixed] Administrative Overheads per unit [80% fixed] Selling Overheads per unit [50% fixed]

The Actual Production during the period was 60,000 units. Calculate the Budgeted Total Cost and per unit cost for the production of 60,000 units. Ans.: Budgeted Total Cost = Rs. 4,65,000; Budgeted Cost per unit = Rs. 7.75 15. The monthly Budget for manufacturing the Overhead of a concern for two levels of activity were as follows: Capacity [%] Budgeted Production [units]

60 600 Rs. 1,200 900 1,1,00 1,600 4,000 1,000 9,800

Wages Consumable Stores Maintenance Power and Fuel Depreciation Insurance

100 1,000 Rs. 2,000 1,500 1,500 2,000 4,000 1,000 12,000

You are required to: (i) Indicate which of the items are fixed, variable and semi-variable. (ii) Prepare a Budget for 80% capacity. (iii) Find the Total Cost, both fixed and variable, per unit of output at 60%, 80% and 100% capacity. [C.S. (Inter)—December 1990] Ans.: (i) Fixed Cost—Depreciation and Insurance; Variable Cost—Wages and Consumable stores; Semi-Variable Cost—Maintenance and Power and Fuel. (ii) Total Cost—Rs. 10,900. (iii) Rs. 16.33, Rs. 13.63 and Rs. 12. 16. The Budgeted Cost of a factory specializing in the production of a single product at the optimum capacity of 6,400 units p.a. is Rs. 7,04,192, the details of which is as follows: Rs. Fixed Cost Variable Cost: Power Repairs Miscellaneous Expenses Direct Materials Direct Labour

Rs. 82,752

5,760 6,800 2,160 1,97,120 4,09,600 6,21,440 7,04,192

Budgeted Total Cost

Having regard to a possible impact on the turnover by market trends, the company decides to have a Flexible Budget with a production target of 3,200 units and 4,800 units at 50% and 75% capacity of production respectively. Assume that the Selling Price per unit is maintained at Rs. 160 at present. Administration and Selling and Distribution Expenses continue to remain at Rs. 14,400. Prepare a Flexible Budget at 50% and 75% capacity and indicate the effect on the Net Profit.

Modified Date: Mon, Jun 28, 2010 06:20:11 PM

Output Date: Tue, Jul 06, 2010 12:20:11 PM

Rev II

Project: Management Accounting_Debarshi Bhattacharyya ACE Pro India Pvt. Ltd. File: X:\Pearson\Management Accounting_Debarshi Bhattacharyya\MAIN\M07\LAYOUT_M07\M07_DEBA_ISBN_EN_SE_C07 III.indd

530

MANAGEMENT ACCOUNTING

Ans.: At 50% Capacity p.u. Total Rs. Rs. 160 5,12,000 97.10 3,10,120 30.36 97,152 32.54 1,04,128

Sales Variable Cost Fixed Cost Profit

At 75% Capacity p.u. Total Rs. Rs. 160 7,68,000 97.10 4,66,080 20.24 97,152 42.66 2,04,768

At 100% Capacity p.u. Total Rs. Rs. 160 10,24,000 97.10 6,21,440 15.18 97,152 47.72 3,05,408

17. Draw up a Flexible Budget for the Overhead Expenses on the basis of the following data and determine the Overhead rates at 70%, 80% and 90% plant capacity levels: At 80% Capacity Rs. Variable Overheads: Indirect Labour Indirect Material Semi-variable Overheads: Power [30% fixed] Repairs and maintenance [40% variable] Fixed Overheads: Depreciation Insurance Others Total Overheads Estimated Direct Labour hour work

12,000 4,000 20,000 2,000 11,000 3,000 10,000 62,000 1,24,000 hrs

[I.C.W.A. (Inter)—Adapted] Ans.: Total Overheads: At 70%—Rs. 58,150; At 80%—Rs. 62,000; At 90%—Rs. 65,850. Overheads Rates: At 70%—Re. 0.5359; At 80%—Re. 0.50; At 90%—Re. 0.4720. 18. A factory can produce 60,000 units per year at its optimum (100%) capacity. The estimated costs of production are as follows: Direct Material Direct Labour Indirect Expenses: Fixed Variable Semi-variable

Rs. 3 per unit Rs. 2 per unit Rs. 1,50,000 p.a. Rs. 5 per unit Rs. 50,000 p.a. up to 50% capacity and an extra expense of Rs. 10,000 for every 25% increase in capacity or part thereof

The factory produces only against order (and not for its own stock). If the production programme of the factory is as indicated in the following and the management desires to ensure a profit of Rs. 1,00,000 for the year, work out the average Selling Price at which each unit should be quoted: First 3 months of the year 50% of the capacity Remaining 9 months of the year 80% of the capacity Ignore Selling, Distribution and Administration Overhead Ans.: Average Selling Price per unit = Rs. 17.25 Hints to Answer: Cost of 7,500 units produced in first 3 months Cost of 36,000 units produced in remaining 9 months

Rs. 1,25,000 5,25,000 (Continued)

Modified Date: Mon, Jun 28, 2010 06:20:11 PM

Output Date: Tue, Jul 06, 2010 12:20:11 PM

Rev II

Project: Management Accounting_Debarshi Bhattacharyya ACE Pro India Pvt. Ltd. File: X:\Pearson\Management Accounting_Debarshi Bhattacharyya\MAIN\M07\LAYOUT_M07\M07_DEBA_ISBN_EN_SE_C07 III.indd

531

BUDGET AND BUDGETARY CONTROL

Rs. 6,50,000 1,00,000 7,50,000

Total Cost of 43,500 units produced during the year Add: Desired Profit Estimated Selling Price of 43,500 units produced

19. SV Ltd manufactures a single product. The Selling Price of the product is Rs. 95 per unit. The following are the results obtained by the company during the last two quarters: Quarter 1 5,100 5,500 Rs. 66,000 55,000 1,56,750 86,000 79,000

Sales units Production units Direct Materials A Direct Materials B Manufacturing Wages Factory Overheads Selling Overheads

Quarter 2 4,800 4,500 Rs. 54,000 45,000 1,38,000 83,000 73,000

The company estimates its sales for the next quarter to range between 5,500 units and 6,500 units, the most likely volume being 6,000 units. The manufacturing programme will match with the sales quantity such that no increase in the inventory of finished goods is contemplated in the next quarter. The following price and cost changes will, however, apply to the next quarter. The price of direct material B will increase by 10%. There will be no change in the price of direct material A. The wage rates will go up by 8%. If the production volume increases beyond 5,500 units, the overtime premium of 50% is payable on the increased volume due to the overtime working to be done by the variable labour complement.  The fixed factory and selling expenses will increase by 20% and 25%, respectively.  A discount in the Selling Price of 2% is allowed on all sales made at 6,500 units level of output. The Selling Price, however, will remain unaltered, if the volume of output is below 6,500 units. While operating at a volume of output of 6,500 units in the next quarter, the company intends to quote for an additional volume of 2,000 units to be supplied to a government department for its captive consumption. The working capital requirement of this order is estimated at 80% of the sales value of the government order. The company desires a return of 20% on the capital employed in respect of this order.  

Required: i. Prepare a flexible Budget for the next quarter at 5,500, 6,000 and 6,500 unit levels and determine the profit at the respective volumes. ii. Calculate the lowest price per unit to be quoted in respect of the government order for 2,000 units. [C.A. (Inter)—May 2003] Ans.: (i) Total Cost (Rs.) Profit/Loss (ii) Lowest price to be quoted is Rs. 91 per unit (approx.)

5,500 units 6,01,940 (79,440)

6,000 units 6,40,128 (70,128)

6,500 units 6,78,315 (73,165)

IV. Cash Budget

20. From the following information, compute the collection from Debtors for the months of June, July and August: Sales (Rs.)

April 1,00,000

May 1,20,000

June 90,000

July 1,05,000

August 1,25,000

20% of sales are made in cash. Debtors are allowed a 2 months’ credit, but will receive a 5% cash discount if they pay off their dues within the month next to the month of sale. About 4/5th of the Debtors normally clear their dues to avail the Cash Discount. Remaining Debtors pay on the due date. [B.Com. (Hons), C.U. (Part II)—2004] Ans.: June—Rs. 88,960; July—Rs. 73,920; August—Rs. 78,240.

Modified Date: Mon, Jun 28, 2010 06:20:11 PM

Output Date: Tue, Jul 06, 2010 12:20:11 PM

Rev II

Project: Management Accounting_Debarshi Bhattacharyya ACE Pro India Pvt. Ltd. File: X:\Pearson\Management Accounting_Debarshi Bhattacharyya\MAIN\M07\LAYOUT_M07\M07_DEBA_ISBN_EN_SE_C07 III.indd

532

MANAGEMENT ACCOUNTING

21. From the following information, prepare a Cash Budget for the 3 months that is ending on 30 November 2009: Sales Rs. 25,000 24,000 22,950 23,400 27,000 28,500

Months June July August September October November

Purchases Rs. 11,800 12,000 12,600 11,550 11,250 13,200

Wages Rs. 1,600 1,680 1,740 1,740 1,770 1,770

Factory Expenses Rs. 1,200 1,170 1,230 1,260 1,530 1,800

Office Expenses Rs. 2,800 3,000 3,600 4,200 4,800 3,900

The Bank Balance on 1 September 2009 is Rs. 3,000. A sales commission @ 5% on sales, which is due in the month following the month in which the sales dues are collected, is payable in addition to the office expenses. Fixed Assets worth Rs. 19,500 will be purchased in September 2009 to be paid in October 2009. Rs. 5,000 in respect of debenture interest will be paid in October 2009. The period of credit allowed to customers is 2 months and 1 month’s credit is obtained from the suppliers of goods. Wages are paid on an average, fortnightly on the 1st and 16th of each month. Expenses are paid in the month in which they are due. [M.Com., C.U.—Adapted] Ans.: Closing Cash Balance: September 2001—Rs. 5,950; October 2001—(−) Rs. 16,435; November 2001—(−) Rs. 12,902.50. Sales Commission: September 2001—Rs. 1,250; October 2001—Rs. 1,200; November 2001—Rs. 1,147.50. 22. From the following information supplied by Bright Ltd, prepare a Cash Budget for the period from 1 September 1989 to 31 December 1989: Months July August September October November December

Credit Purchase Rs. 85,000 92,000 1,00,000 1,20,000 90,000 98,000

Credit Sales Rs. 1,60,000 1,85,000 2,10,000 2,45,000 1,78,000 1,82,000

Wages Rs. 32,000 37,000 42,000 49,000 35,500 36,000

Selling Expenses Rs. 8,000 9,500 10,500 12,500 8,900 9,000

Overheads Rs. 10,000 11,500 13,000 14,500 10,500 11,000

Additional Information: a. Expected Cash Balance on 1 September—Rs. 10,500. b. Period of credit allowed to Debtors—2 months. c. Period of credit allowed by Creditors—1 month. d. Lag in payment of wages, selling expenses and overheads—1 month. e. Selling commission @ 2% on sales is payable for 1 month after sales. f. Expenditure on machinery worth Rs. 50,000 is payable in October. g. Expected Cash Sales per month is Rs. 15,000. No commission is payable on Cash Sales. [B.Com. (Hons), C.U.—1989] Ans.: Expected Closing Cash Balance: September—Rs. 31,800; October—Rs. 12,100; November—Rs. 36,200; December—Rs. 1,47,740. 23. From the following information, prepare the Cash Budget for the year ending on 30 June 2005: Months (2005) February March April May June

Modified Date: Mon, Jun 28, 2010 06:20:11 PM

Sales Rs. 2,40,000 2,60,000 1,60,000 2,32,000 1,76,000

Purchases Rs. 1,68,000 2,00,000 2,08,000 2,12,000 1,60,000

Output Date: Tue, Jul 06, 2010 12:20:11 PM

Wages Rs. 20,000 24,000 16,000 20,000 16,000

Other Expenses Rs. 14,000 16,000 12,000 24,000 12,000

Rev II

Project: Management Accounting_Debarshi Bhattacharyya ACE Pro India Pvt. Ltd. File: X:\Pearson\Management Accounting_Debarshi Bhattacharyya\MAIN\M07\LAYOUT_M07\M07_DEBA_ISBN_EN_SE_C07 III.indd

533

BUDGET AND BUDGETARY CONTROL

Additional Information: i. Cash in hand on 01 April 2005—Rs. 10,000. ii. Sales @ 20% realized in the month of sale and discount allowed is 2%, with the balance realized after 2 months of sales. iii. About 4/5th of the Credit Purchase is paid after 1 month of that purchase and next to that month, the balance 1/5th is paid. iv. Wages are paid 3/4th on the due date while 1/4th during the next month. v. Other Expenses are paid at a lag of 1 month. vi. Income Tax of Rs. 25,000 is due on or before 30 June 2005. [B.Com. (Hons), C.U.—2005] Ans.: Closing Cash Balance: April—Rs. 5,760; May—Rs. 21,832; June—Rs. 92,782. 24 From the following information, prepare a Cash Budget for 3 months that ended on 30 June 2010: Months February March April May June

Sales Rs. 1,20,000 1,30,000 80,000 1,16,000 88,000

Purchases Rs. 84,000 1,00,000 1,04,000 1,06,000 80,000

Wages Rs. 10,000 12,000 8,000 10,000 8,000

Misc. Exp. Rs. s,000 8,000 6,000 12,000 6,000

Additional information: a. Sales: 20% realized in the month of sale, discount allowed 2%. Balance realized equally in two subsequent months. b. Purchases: These are paid for in the month following the month of supply. c. Wages: 25% paid in arrear the following month. d. Miscellaneous Expenses; Paid a month in arrear. e. Rent: Rs. 1,000 per month and quarterly in advance, due in April. f. Income Tax: First instalment of advance Tax—Rs. 25,000 due on 15 June. g. Cash in hand: Rs. 5,000 on 1 April 2010. Ans.: Closing Cash Balance: April—Rs. 5,200; May—Rs. 8,260; June—Rs. 57,330. 25. Prepare a Cash Budget for the 3 months that ended on 31 March 2010, on the basis of the following information: Rs. 25,000 10,000 5,000

Cash Balance on 1 January 2010 Estimated monthly salaries and wages Interest payable in February 2010 Estimated Cash Sales Credit Sales Purchases Other Expenses

December Rs. − 1,00,000 1,60,000 −

January Rs. 1,40,000 80,000 1,70,000 20,000

February Rs. 1,52,000 1,40,000 2,40,000 22,000

March Rs. 1,21,000 1,20,000 1,80,000 21,000

Cash Sales are collected 50% in the month in which the sales are made and 50% in the following month. Collections from the Credit Sales are subject to 5% discount if the payment is received during the month of sale and 2 12 % if the payment is received in the following month. Creditors are paid either on a ‘prompt’ or 30-day basis. It is estimated that 10% of Creditors are in the ‘prompt’ category. [C.A.I.I.B.—Adapted] Ans.: Closing Cash Balance: January 2010—Rs. 60,750; February 2002—Rs. 1,04,250; March 2002—Rs. 85,500.

Modified Date: Mon, Jun 28, 2010 06:20:11 PM

Output Date: Tue, Jul 06, 2010 12:20:11 PM

Rev II

Project: Management Accounting_Debarshi Bhattacharyya ACE Pro India Pvt. Ltd. File: X:\Pearson\Management Accounting_Debarshi Bhattacharyya\MAIN\M07\LAYOUT_M07\M07_DEBA_ISBN_EN_SE_C07 III.indd

534

MANAGEMENT ACCOUNTING

26. From the following particulars given, prepare a Cash Budget for 4 months from July to October 2005: i. Bank Balance on 1 July is expected to be Rs. 50,000. ii. Expected capital expenditure: (a) Plant and Machinery to be purchased in September at a cost of Rs. 50,000 payable in October; (b) Extension of office building costing Rs. 40,000 is to be paid in four equal instalments starting from August and (c) Hire-Purchase instalment payment of Rs. 5,000 is to be made in each month. iii. Cash Sales of Rs. 5,000 per month are expected. iv. Forecast of monthly recurring incomes and expenditure: Months May June July August September October

Credit Sales Rs. 25,000 20,000 24,000 26,000 28,000 27,000

Credit Purchase Rs. 15,000 12,000 14.000 16,000 15,000 18,000

Work Expenses Rs. 1,200 1,500 1,400 1,600 1,800 1,700

Wages Rs. 1,900 1,800 1,780 1,800 1,790 1,800

Office Expenses Rs. 3,500 3,800 3,400 3,900 4,000 3,600

Marketing Expenses Rs. 1,000 800 1,100 1,500 1,800 1,700

v. Other information available: (a) A Sales commission of 4% on Credit Sales is to be paid to the salesman in the month in which the dues are collected; (b) The period of credit allowed to customers is 2 months and that allowed for suppliers is 1 month; (c) Wages are paid fortnightly on the 1st & 16th of each month and (d) Other Expenses are paid in the month in which they become due. [B.Com. (Hons), Kalyani University (Part II)—2005] 27. ABC Company Ltd has given the following particulars. You are required to prepare a Cash Budget for the 3 months that is ending on 31 December 1994: (a) Sales Rs. 20,000 21,000 23,000 25,000 30,000

Months August September October November December

Materials Rs. 10,200 10,000 9,800 10,000 10,800

Wages Rs. 3,800 3,800 4,000 4,200 4,500

Overheads Rs. 1,900 2,100 2,300 2,400 2,500

(b) Credit terms are: Sales/Debtors—10% Sales are on cash basis. 50% of the Credit Sales are collected next month and the balance in the following month. Creditors:

Materials Wages Overheads

2 months 1/2 month 1/5 month

a. Cash Balance on 1 October 1994 is expected to be Rs. 8,000. b. A machinery will be installed in August 1992 at a cost of Rs. 1,00,000. The monthly instalment of Rs. 5,000 is payable from October onwards. c. A Dividend at 10% on Preference Share Capital of Rs, 3,00,000 will be paid on 1 December 1994. d. Advance to be received for the sale of vehicle is Rs. 20,000 in December. e. Income Tax (advance) to be paid in December—Rs. 5,000. [C.S. (Inter)—December 1995] Ans.: Closing Cash Balance: October—Rs. 7,390; November—Rs. 8,180; December (−) Rs. 3,910.

Modified Date: Mon, Jun 28, 2010 06:20:11 PM

Output Date: Tue, Jul 06, 2010 12:20:11 PM

Rev II

Project: Management Accounting_Debarshi Bhattacharyya ACE Pro India Pvt. Ltd. File: X:\Pearson\Management Accounting_Debarshi Bhattacharyya\MAIN\M07\LAYOUT_M07\M07_DEBA_ISBN_EN_SE_C07 III.indd

BUDGET AND BUDGETARY CONTROL

535

28. From the following information of Moon Ltd, prepare a Cash Budget for the 3 months commencing on 1 June 2010, when the Bank Balance was Rs. 10,000: Sales Rs. 1,00,000 1,20,000 1,40,000 1,60,000 1,80,000

Months April May June July August

Purchases Rs. 70,000 80,000 90,000 1,00,000 1,10,000

Wages Rs. 8,500 9,500 9,500 12,000 14,000

Selling Expenses Rs. 3,500 3,500 3,500 3,500 3,500

Overheads Rs. 4,000 4,500 6,000 6,500 7,000

A commission of 5% on the sales due of 2 months after the sales is payable in addition to the above selling expenses. Credit terms of sale are—payment by the end of the month following the month of supply. On an average, one-half of sales is paid on the due date, while the other half is paid during the next month. Creditors are paid during the month following the month of supply. The plant is purchased in June. Rs. 78,000 is payable on delivery. Rs. 48,000 and the balance in two equal monthly instalments in July and in August. A dividend of Rs. 30,000 will be paid in September. Wages are paid 3/4th on due date while 1/4th on during the next month. The lag in payment of Selling Expenses and Overheads is 1 month. [B.Com. (Hons), C.U.—Adapted] Ans.: June Rs. (−) 30,500 5,000 1,10,000

Closing Cash Balance Sales Commission Collection from Debtors

July Rs. (−) 32,375 6,000 1,30,000

August Rs. (−) 27,875 7,000 1,50,000

29. Prepare a Cash Budget for 6 months that ended on 30 June 2010, on the basis of the following information: a. The estimated Sales and Expenses are as follows:

Sales Wages & Salaries Misc. Expenses

Nov. ’09 Rs. 2,00,000 30,000

Dec. ’09 Rs. 2,20,000 30,000

Jan. ’10 Rs. 1,20,000 24,000

Feb. ’10 Rs. 1,00,000 24,000

March ’10 Rs. 1,50,000 24,000

April ’10 Rs. 2,40,000 30,000

May ’10 Rs. 2,00,000 27,000

June ’10 Rs. 2,00,000 27,000

27,000

27,000

21,000

30,000

24,000

27,000

27,000

27,000

b. 20% of the sales are in cash and the balance on credit. c. The firm has a gross margin of 25% on sales. d. 50% of the Credit Sales are collected in the month following the sales, 30% in the second month and the balance 20% in the third month. e. Material for the sale of each month is purchased 1 month in advance on a credit for 2 months. f. The time lag in the payment of wages and salaries is 1/3 of a month and of Miscellaneous Expenses is 1 month. g. Debentures worth Rs. 40,000 are sold in January 2010. h. The firm maintains a minimum Cash Balance of Rs. 40,000. Funds can be borrowed @ 12% p.a. in the multiples of Rs. 1,000, the interest being payable on a monthly basis. i. Cash Balance at the end of December 2009 is Rs. 60,000. [C.A. (Final)—Adapted] Ans.: Closing Cash Balance: January ’10—Rs. 42,000; February ’10—Rs. 59,800; March ’10—Rs. 64,800; April ’10— Rs. 51,500; May ’10—Rs. 40,500; June ’10—Rs. 40,740.

Modified Date: Mon, Jun 28, 2010 06:20:11 PM

Output Date: Tue, Jul 06, 2010 12:20:11 PM

Rev II

Project: Management Accounting_Debarshi Bhattacharyya ACE Pro India Pvt. Ltd. File: X:\Pearson\Management Accounting_Debarshi Bhattacharyya\MAIN\M07\LAYOUT_M07\M07_DEBA_ISBN_EN_SE_C07 III.indd

536

MANAGEMENT ACCOUNTING

30. From the following details of a company, prepare its Cash Budget for 3 months that ended on 30 September 2008: a. Balance on 30 June 2008: Cash in hand and at bank—Rs. 13,000; Debtors—Rs. 1,80,000 (Rs. 1,00,000 for sales in May and Rs. 80,000 for sales in June); Creditors—Rs. 65,000 (for purchases in June); Stock in trade—Rs. 45,000. b. Budgeted Sales: July—Rs. 1,00,000; August—Rs. 1,20,000; September—Rs. 1,40,000. c. All sales are at a profit of 25% on cost (being cost of goods sold only). d. Debtors: Credit allowed—2 months; 50% of credit allowed is expected to be received on time, balance in the subsequent month. e. Creditors: Credit received—1 month. f. Time lag: Wages—1/2 month; Other Expenses—1/4 month. g. Stock level to be raised to Rs. 50,000 in July and to Rs. 55,000 in August. h. Actual and Budgeted Wages: June—Rs. 8,000 (actual); July—Rs. 9,000; August—Rs. 9,000; September—Rs. 10,000. i. Actual and Budgeted Expenses: June—Rs. 6,000 (actual); July—Rs. 6,000; August—Rs. 5,000; September— Rs. 7,000. j. Advance Income Tax in September—Rs. 7,000 and Purchase of machinery in August—Rs. 10,000. k. 10% of Total Sales and 20% of Total Purchases are on cash basis. Ans.: Closing Cash Balance: July—Rs. 23,500; August—Rs. 34,700; September—Rs. 60,400. 31. From the following information, draw up a Cash Budget for the 3 months that is ending on 31 March 2010: i. Cash and Bank Balance on 1 January 2010—Rs. 1,00,000. ii. Purchases: Actual

Rs. 2,00,000 2,40,000 2,25,000

Estimated January 2010 February 2010 March 2010

Rs. 2,40,000 2,00,000 2,50,000

Actual

Rs. 3,25,000 3,50,000 3,75,000

Estimated January 2010 February 2010 March 2010

Rs. 4,00,000 4,10,000 4,45,000

Actual

Rs. 75,000 75,000

Estimated January 2010 February 2010 March 2010

Rs. 90,000 90,000 1,00,000

Rs.

Estimated January 2010 February 2010 March 2010

Rs. 30,000 40,000 40,000

October 2009 November 2009 December 2009

iii. Sales: October 2009 November 2009 December 2009

iv. Wages: November 2009 December 2009

v.

Expenses: Actual

November 2009 December 2009

v. vi. vii. viii.

25,000 30,000

10% of purchases and sales are on cash basis. Advance payment of Income Tax in March 2010—Rs. 25,000. Plant to be purchased for Rs. 50,000 and price paid in January 2010. Time lag:

Credit Purchases Credit Sales Wages Expenses

2 months 1 month 1/2 month 1/4 month

Ans.: Closing Cash Balance: January 2010—Rs. 66,000; February 2010—Rs. 81,000; March 2010—Rs. 1,20,500.

Modified Date: Mon, Jun 28, 2010 06:20:11 PM

Output Date: Tue, Jul 06, 2010 12:20:11 PM

Rev II

Project: Management Accounting_Debarshi Bhattacharyya ACE Pro India Pvt. Ltd. File: X:\Pearson\Management Accounting_Debarshi Bhattacharyya\MAIN\M08\LAYOUT_M08\M08_DEBA_ISBN_EN_SE_C08.indd

Standard Costing and Variance Analysis

8

LEARNING OBJECTIVES On completion of the study of the chapter, you should be able to understand: What is Standard Cost? What is Standard Costing? Features, advantages and limitations of Standard Costing. Preliminary steps for establishing Standard Costing System. Types of standards. Distinction between Standard Cost, Estimated Cost and Actual/Historical Cost. Distinction between Standard Costing and Budgetary Control. Meaning of Variance and Variance Analysis. Classification of Variances.

8.1 WHAT IS STANDARD COST? The meaning of the term ‘Standard Cost’ is predetermined cost scientifically computed for measuring the efficiency of the actual performance of an entity. According to CIMA, London, Standard Cost is ‘a predetermined calculation of how much costs should be under specified working conditions.’ Therefore, Standard Cost is the predetermined cost which should have been under specified operating conditions. It is the yardstick of measuring the efficiency of the actual cost incurred. Before incurring the actual cost of production, an enterprise determines the Standard Cost of each function production on the basis of its working conditions. After the production, the concern judges the efficiency of its actual costs of production with the predetermined Standard Cost and takes appropriate remedial measures after identifying the deviation from the Standard Cost, if any. Standard Cost is ideal in nature and related to future. 8.2 WHAT IS STANDARD COSTING? According to CIMA, London, Standard Costing is ‘the preparation and the use of standard costs, their comparison with actual costs and analysis of variances to their causes and points of incidence.’ Therefore, Standard Costing is the method of setting up of standard costs and applying them to measure the deviations from the actual costs and analysing the causes of such deviation with a view to maximize the efficiency of the production functions. It is a technique of costing which uses standards for costs and revenues for the purpose of control through Variance Analysis. Standard Costing is not a distinct system of accounting, but is only a technique which can be applied to all types of costing like job costing or process costing. 8.2.1 Features of Standard Costing i. Predetermination of appropriate standards for each element of cost and sales. ii. Comparison of actual performance with the standards and any deviation of actual performance from the standard is being identified as a Variance.

Modified Date: Mon, Jul 05, 2010 05:03:14 PM

Output Date: Tue, Jul 06, 2010 12:21:13 PM

Rev II

Project: Management Accounting_Debarshi Bhattacharyya ACE Pro India Pvt. Ltd. File: X:\Pearson\Management Accounting_Debarshi Bhattacharyya\MAIN\M08\LAYOUT_M08\M08_DEBA_ISBN_EN_SE_C08.indd

538

MANAGEMENT ACCOUNTING

iii. Analysis of variances in order to assess the reasons of variances. iv. Reporting to the management for taking corrective action for any off-standard variance. 8.3 PRELIMINARY STEPS FOR ESTABLISHING STANDARD COSTING SYSTEM For establishing a Standard Costing System in an organization, the following preliminary steps are to be adopted: i. Establishment of Cost Centres: Establishment of cost centres with clearly defined areas of responsibility is the first step for establishing a Standard Costing System. ii. Classification of Accounts: Classification of accounts for the purpose of identifying each expense and revenue by function and deciding the responsibility of such expenses and revenues. iii. Types of Standard: Standards may be set out as per the situation and according to the suitability of their achievement. In this context, generally, five types of standard are available, namely, Basic Standard, Current Standard, Ideal Standard, Normal Standard and Expected/Practical Standard. 8.4 TYPES OF STANDARDS Standard may be classified into the following five types: i. Basic Standard: Basic standard is a standard which is established for use over a long period of time. This type of standard remains constant over a long period of time. In this type of standard, a base year is chosen for comparison purpose. ii. Current Standard: Current standard is established for a short period and for current condition. iii. Ideal Standard: Ideal standard is a standard which may be attained under most favourable conditions. This standard is based on the best possible operating conditions. iv. Normal Standard: Normal standard is a standard which can be achieved under normal operating conditions. This standard is difficult to set as it requires a significant degree of forecasting. v. Expected or Practical Standard: Expected or practical standard is a standard which is based on the expected operating performance after making a reasonable allowance for unavoidable losses. This is an attainable and realistic standard. 8.5 DISTINCTION BETWEEN STANDARD COST, ESTIMATED COST AND ACTUAL/HISTORICAL COST Standard Cost refers to the predetermined cost that should have been under the specified operating conditions of an enterprise. Standard Cost is the predetermined cost scientifically calculated on the basis of the actual output of the enterprise. It is ideal in nature and related to future. Estimated Costs are the future costs assumed at present. It is also a predetermined cost, but computed with the help of historic data. It is probable in nature and related to future. Actual or Historical Costs are the costs actually incurred in the past and ascertained after they have been incurred. It is actual in nature and related to past. 8.6 ADVANTAGES OF STANDARD COSTING Standard Costing System has the following advantages: i. It facilitates an effective cost control and provides necessary information for cost control. ii. It identifies the areas of strength and weakness of production functions. iii. It helps in measuring the efficiency of the actual performance. iv. It helps in formulation of future production and pricing policies. v. It is useful for budgeting and planning. vi. It provides useful information for price quotation of the products.

Modified Date: Mon, Jul 05, 2010 05:03:14 PM

Output Date: Tue, Jul 06, 2010 12:21:13 PM

Rev II

Project: Management Accounting_Debarshi Bhattacharyya ACE Pro India Pvt. Ltd. File: X:\Pearson\Management Accounting_Debarshi Bhattacharyya\MAIN\M08\LAYOUT_M08\M08_DEBA_ISBN_EN_SE_C08.indd

STANDARD COSTING AND VARIANCE ANALYSIS

539

vii. It increases the cost consciousness among the employees, as through Variance Analysis—the responsibility for favourable or unfavourable performance is assigned to them. viii. It increases the efficiency and productivity of workers through motivation for achieving a better performance. ix. It facilitates the use of Management by Exception (MBE) principle as the management needs to concentrate on the area required for corrective action alone. x. It simplifies the procedure of valuation of inventory as the inventory is valued at Standard Cost under Standard Costing System. 8.7 LIMITATIONS OF STANDARD COSTING In spite of having immense important advantages, yet, Standard Costing System suffers from the following limitations: i. Standard Costing System is not effective for the organizations which deal with non-standardized products and jobs which change according to the customer’s specification. ii. Sometimes, it becomes difficult, in practise, to establish standards as it needs a high degree of technical skill. iii. If standards are inaccurately established, wrong and misleading conclusions may be drawn. iv. Installation of Standard Costing System in an organization is a very costly one and accordingly, small business organizations cannot afford to install this system. v. Standard Costing is a tool of management and is not a substitute of management. vi. Organizations, where technological changes take place at frequent intervals, find difficulty to adopt this system. vii. Standards have to be revised on regular intervals owing to the changing conditions, which is an expensive and difficult task for the organization. viii. Non-achievement of unrealistic standards may lead to an adverse effect on the morale and motivation of the employee. 8.8 COMPARISON BETWEEN STANDARD COSTING AND BUDGETARY CONTROL Standard Costing and Budgetary Control have the common objective of cost control by establishing the predetermined costs. Although both of these follow certain common basic principles, such as establishment of predetermined targets of performance, measurement of actual performance, comparison of actual performance with the predetermined performance, analysis of the differences between the predetermined and actual performance, adoption of remedial measures—wherever necessary, and so on, there are a number of fundamental differences between Standard Costing and Budgetary Control. These differences are as follows: Standard Costing 1. Standard Costing is based on technical assessment. 2. It is the method of preparation of Standard Cost and application of those for measuring the efficiency of the actual cost. 3. It is a projection of cost accounts. 4. It is mainly concerned with the ascertainment and control of costs. 5. Standard is set mainly in respect of manufacturing functions such as different elements of cost and sales.

Budgetary Control 1. Budgetary Control is based on the past performance adjusted with the future trend. 2. It is the method of forecasting cost by preparation of budget prior to a definite period to attain the given objectives. 3. It is a projection of financial accounts. 4. It is mainly concerned with the profitability and financial position of a concern. 5. Budget is prepared in respect of different functions of the business such as purchase, wages, production and sales. (Continued)

Modified Date: Mon, Jul 05, 2010 05:03:14 PM

Output Date: Tue, Jul 06, 2010 12:21:13 PM

Rev II

Project: Management Accounting_Debarshi Bhattacharyya ACE Pro India Pvt. Ltd. File: X:\Pearson\Management Accounting_Debarshi Bhattacharyya\MAIN\M08\LAYOUT_M08\M08_DEBA_ISBN_EN_SE_C08.indd

540

MANAGEMENT ACCOUNTING

Standard Costing 6. Standard Costing sets the standard that should be maintained in the actual performance. 7. It cannot be applied without budget. 8. It is more an intensive technique of controlling cost. 9. It cannot be applied in part. 10. Under Standard Costing, variances are revealed through different accounts.

Budgetary Control 6. Budget sets up the maximum limits of expenses, beyond which the actual expenses should not normally exceed. 7. It can be operated without standard. 8. It is more an extensive approach which covers all the business functions of a concern. 9. It can be applied in part. 10. In Budgetary Control, variances are used as a statistical information only.

8.9 ANALYSIS OF VARIANCE The deviation of actual performance from the standard set out is called ‘variance.’ As per the official terminology of CIMA, London, variance is ‘the difference between planned, budgeted or Standard Cost and Actual Cost.’ Variance Analysis is the process of analysing the reasons behind the variances occurred by sub-dividing them in such a manner so that the management can be reported for adopting remedial measures for any off-standard performance. According to CIMA, London, Variance Analysis is ‘the process of computing the amount of variance and isolating the cause of variance between actual and standard.’ Therefore, Variance Analysis is a process of calculating the total variance by comparing the actual performance with the standard set out and all its sub-variances that may be practised for identifying the causes for such variance and reporting to the management for taking the corrective action for any off-standard performance of the manufacturing function. Hence, Variance Analysis involves three functional actions: (i) Ascertainment of individual variances; (ii) Determination of causes of variance; and (iii) Reporting to the management for taking corrective action for any off-standard performance. 8.9.1 Direction of Variances In case of cost variances, the excess of Standard Cost over the actual cost represents the positive or favourable variance, which indicates that the actual cost is less than the Standard Cost. On the other hand, the excess of actual cost over the Standard Cost represents the negative or unfavourable or adverse variance, which indicates that the actual cost is more than the Standard Cost. In case of sales variances, the excess of actual sales over budgeted sales represents the positive or favourable variance, which indicates that the actual sales is more than the budgeted sales. On the other hand, the excess of budgeted sales over the actual sales represents the negative or unfavourable or adverse variance, which indicates that actual sales is lesser than the budgeted sales. 8.9.2 Nature of Variances Nature of variances may be of two types: (a) Controllable Variances; and (ii) Uncontrollable Variances. Controllable variances are those which arise due to controllable causes, that is, which can be controlled by taking an appropriate corrective action, for example, use of defective or inferior quality of materials, engagement of inefficient workers and so on. Uncontrollable variances are those which arise due to uncontrollable causes, that is, on which control can be exercised, for example, change in the market price of materials, change in the market rate of labour and so on. 8.9.3 Purposes of Variance Analysis Variance Analysis serves the following purposes: i. To assess the overall, departmental and individual efficiency of the concern. ii. To act as an instrument of cost control and cost reduction. iii. To enable the management to utilize men, materials and machines effectively. iv. To provide the basis for framing future plans of action and formulating managerial policies.

Modified Date: Mon, Jul 05, 2010 05:03:14 PM

Output Date: Tue, Jul 06, 2010 12:21:13 PM

Rev II

Project: Management Accounting_Debarshi Bhattacharyya ACE Pro India Pvt. Ltd. File: X:\Pearson\Management Accounting_Debarshi Bhattacharyya\MAIN\M08\LAYOUT_M08\M08_DEBA_ISBN_EN_SE_C08.indd

STANDARD COSTING AND VARIANCE ANALYSIS

541

8.10 CLASSIFICATION OF VARIANCES Broadly, variances are classified into two parts—Cost Variances and Sales Variances. Again, cost variances are classified into four parts for each of the cost element, such as Material Cost Variances, Labour Cost Variances, Variable Overhead Variances and Fixed Overhead Variances. Classification of variances is explained in a chart format in Figure 8.1. Variance

Cost Variance

Materials Variance

Labour Variance

Variable Overhead Variance

Figure 8.1

Sales Variance

Fixed Overhead Variance

Sales Value Variance

Sales Margin Variance

Classification of Variance

8.10.1 Material Variances Material Variances are classified as shown in Figure 8.2. Material Cost Variance (MCV)

Material Price Variance (MPV)

Material Usage/Quantity Variance (MUV/MQV)

Material Mix Variance (MMV)

Figure 8.2

Material Yield Variance (MYV)

Classification of Material Variance

i. Material Cost Variance (MCV): It is the difference between the standard material cost for the actual output and actual material cost for the actual output. Material Cost Variance (MCV) = Standard materials cost for the actual output –Actual materials cost for the actual output = SP × SQ × AO − AP × AQ × AO Where, SP = Standard price of materials per unit AP = Actual price of materials per unit SQ = Standard quantity of materials required to produce one unit of output AQ = Actual quantity of materials used to produce one unit of output AO = Actual output ii. Material Price Variance (MPV): It is the difference between the Standard Cost of actual quantity of materials used for the actual output and actual cost of actual quantity of materials used for the actual output.

Modified Date: Mon, Jul 05, 2010 05:03:14 PM

Output Date: Tue, Jul 06, 2010 12:21:13 PM

Rev II

Project: Management Accounting_Debarshi Bhattacharyya ACE Pro India Pvt. Ltd. File: X:\Pearson\Management Accounting_Debarshi Bhattacharyya\MAIN\M08\LAYOUT_M08\M08_DEBA_ISBN_EN_SE_C08.indd

542

MANAGEMENT ACCOUNTING

Material Price Variance (MPV) = Standard Cost of actual quantity of material used for the actual output − Actual cost of actual quantity of materials used for the actual output = SP × AQ × AO − AP × AQ × AO = (SP − AP) AQ × AO iii. Material Usage/Quantity Variance (MUV/MQV): It is the difference between the Standard Cost of standard quantity of material required for the actual output and Standard Cost of actual quantity of material used for the actual output. Material Usage/Quantity Variance (MUV/MQV) = Standard Cost of standard quantity of material required for the actual output − Standard Cost of actual quantity of materials used for the actual output = SP × SQ × AO − SP × AQ × AO = SP [(SQ × AO) − (AQ × AO)] iv. Material Mix Variance (MMV): This variance is applied where more than one type of material is used to produce the output. It is the difference between the Standard Cost of standard mix/proportion of a material required in the actual input of total materials and Standard Cost of actual mix/proportion of that material used in the actual input of total materials. Material Mix Variance (MMV) = Standard mix/proportion of a material required in the actual input of total materials − Standard Cost of actual mix/proportion of that material used in the actual input of total materials = SP [Standard mix of a material in the actual input − Actual mix of that material in the actual input] v. Material Yield Variance (MYV): This variance is also applied where more than one type of material is used to produce the output. It is the difference between the Standard Cost of the actual yield/output for the actual input of materials and Standard Cost of the standard yield/output for the actual input of materials. Material Yield Variance (MYV) = Standard Cost of actual yield/output for the actual input of materials − Standard Cost of standard yield/output for the actual input of materials = Standard yield rate [Actual yield for actual input − Standard yield for actual input] Tutorial Notes 1. ‘SQ’ alone represents ‘Standard quantity of materials required to produce one unit of output,’ whereas ‘SQ × AO’ refers to ‘Standard quantity of materials required to produce the total actual output.’ Similarly, ‘AQ’ alone represents ‘Actual quantity of materials used to produce one unit of output,’ whereas ‘AQ × AO’ refers to ‘Actual quantity of materials used to produce total actual output.’ 2. ‘Mix’ refers to ‘proportion of materials,’ whereas ‘Yield’ refers to ‘output from those materials.’ 3. Where one type of material is used to produce output, the question of computation of MMV and MYV does not arise at all, that is, in such a case, MCV, MPV and MUV alone to be computed. 4. Where more than one type of material is used to produce a single output, MMV and MYV are to be computed (along with MCV, MPV and MUV). 5. Standard Yield Rate refers to Standard Output Rate. 6. Standard Yield Rate =

Standard Material Cost of standard output Standard output

7. Reconciliation or Test for Material Variances; MCV = MPV + MUV MUV = MMV + MYV

Modified Date: Mon, Jul 05, 2010 05:03:14 PM

Output Date: Tue, Jul 06, 2010 12:21:13 PM

Rev II

Project: Management Accounting_Debarshi Bhattacharyya ACE Pro India Pvt. Ltd. File: X:\Pearson\Management Accounting_Debarshi Bhattacharyya\MAIN\M08\LAYOUT_M08\M08_DEBA_ISBN_EN_SE_C08.indd

STANDARD COSTING AND VARIANCE ANALYSIS

543

8.10.2 Labour Variances Labour variances are classified in several parts as shown in Figure 8.3. Labour Cost Variance (LCV)

Labour Rate/P Variance (LRV/LPV)

Labour Idle Time Variance (LITV)

Labour Mix Variance (LMV)

Figure 8.3

Labour Efficiency Variance (LEV)

Labour Yield Variance (LYV)

Classification of Labour Variance

i. Labour Cost Variance (LCV): It is the difference between the standard labour cost for the actual output and actual labour cost for the actual output. Labour Cost Variance (LCV) = Standard labour cost for the actual output − Actual labour cost for the actual output = SR × SH × AO − AR × AH × AO Where, SR = Standard labour rate per hour AR = Actual labour rate per hour SH = Standard hours required to produce one unit of output AH = Actual hours taken to produce one unit of output AO = Actual output SH × AO = Total standard hours required to produce the total actual output AH × AO = Total actual hours taken to produce the total actual output (including abnormal idle hours) ii. Labour Rate/Price Variance (LRV/LPV): It is the difference between the Standard Cost of actual labour hours taken for the actual output and actual cost of actual labour hours taken for the actual output. Material Price Variance (MPV) = Standard Cost of actual labour hours taken for the actual output − Actual cost of actual labour hours taken for the actual output = SR × AH × AO − AR × AH × AO = (SR – AR) AH × AO iii. Labour Idle Time Variance (LITV): It represents the standard labour cost for the time during which no work has been done, but workers are duly paid for this period. It happens due to some abnormal circumstances (i.e., abnormal idle time) like strikes, lock-out, long period of power failure, machine break-down and so on. LITV will always be negative or adverse. Labour Idle Time Variance (LITV) = SR × Abnormal idle hours iv. Labour Efficiency Variance (LEV): It is the difference between the standard labour cost of standard hours for the actual output and standard labour cost of effective actual hours taken by the worker for producing the actual output. Here, ‘effective actual hours taken for the actual output’ refers to the total actual hours taken for the actual output less abnormal idle hours. Labour Efficiency Variance (LEV) = Standard labour cost of standard hours for the actual output − Standard labour cost of effective actual hours taken for the actual output = SR × SH × AO − SR × Effective AH × AO = SR [SH × AO − Effective AH × AO]

Modified Date: Mon, Jul 05, 2010 05:03:14 PM

Output Date: Tue, Jul 06, 2010 12:21:13 PM

Rev II

Project: Management Accounting_Debarshi Bhattacharyya ACE Pro India Pvt. Ltd. File: X:\Pearson\Management Accounting_Debarshi Bhattacharyya\MAIN\M08\LAYOUT_M08\M08_DEBA_ISBN_EN_SE_C08.indd

544

MANAGEMENT ACCOUNTING

v. Labour Mix Variance (LMV): This variance is applied where workers of different efficiency levels are engaged to produce the output. It is the difference between the standard labour cost of actual hours for the actual output allocated between the different classes of workers on the basis of standard proportion and standard labour cost of effective actual hours taken by the worker for producing the actual output. This standard labour cost of actual hours for the actual output allocated between the different classes of workers on the basis of standard proportion is called ‘revised standard hours’ (RSH). RSH = Total actual hours in standard ratio Standard hours required for a particular class of labour = Total actual hours taken × Total standard hours required for all classes of labour Labour Mix Variance (LMV) = SR × RSH − SR × Effective AH × AO = SR [RSH – Effective AH × AO] vi. Labour Yield Variance (LYV): This variance is also applied where workers of different efficiency levels are engaged to produce the output. It is the difference between the standard labour cost of standard hours for the actual output and standard labour cost of actual hours for the actual output allocated between the different classes of workers on the basis of standard proportion (i.e., RSH). Labour Yield Variance (LYV) = SR × SH × AO − SR × RSH = SR [SH × AO – RSH] Tutorial Notes 1. Effective AH × AO = Effective actual hours taken to produce the total actual output (excluding abnormal idle hours), i.e., [(AH × AO) – Abnormal idle hours] RSH = Total actual hours in standard ratio = Total actual hours taken ×

Standard hours required for a particular class of labour Total standard hours required for all classes of labour

2. While calculating LEV and LMV, Effective (AH × AO) is to be taken, i.e., Total actual hours taken for the total actual output excluding abnormal idle hours, i.e., (AH × AO) – Abnormal idle hours. 3. Where workers having the same efficiency are engaged in production, only LCV, LRV, LITV and LEV are to be computed, i.e., LMV and LYV need not be computed. Where workers having different efficiency levels are engaged in the production process, LMV and LYV are also to be computed along with LCV, LRV, LITV and LEV. 4. LITV will always be negative or adverse. 5. Reconciliation or Test for Labour Variances: LCV = LRV + LITV + LEV LEV = LMV + LYV

8.10.3 Variable Overhead Variances Variable Overhead Variances are classified in several parts as shown in Figure 8.4. Variable Overhead Cost Variance (VOCV)

Variable Overhead Expenditure Variance (VOExp.V)

Figure 8.4

Modified Date: Mon, Jul 05, 2010 05:03:14 PM

Variable Overhead Efficiency Variance (VOEff.V)

Classification of Variable Overhead Variance

Output Date: Tue, Jul 06, 2010 12:21:13 PM

Rev II

Project: Management Accounting_Debarshi Bhattacharyya ACE Pro India Pvt. Ltd. File: X:\Pearson\Management Accounting_Debarshi Bhattacharyya\MAIN\M08\LAYOUT_M08\M08_DEBA_ISBN_EN_SE_C08.indd

STANDARD COSTING AND VARIANCE ANALYSIS

545

i. Variable Overhead Cost Variance (VOCV): It is the difference between the standard variable overhead cost for the actual output and actual variable-overhead cost for the actual output. Variable Overhead Cost Variance (VOCV) = Standard variable overhead cost for actual output − Actual variable overhead cost for the actual output On the basis of hours: VOCV = SRH × SH × AO − ARH × AH × AO On the basis of units: VOCV = SRU × AO − ARU × AO ii. Variable Overhead Expenditure Variance (VOExp.V): It is the difference between the budgeted variable overrohead cost and actual variable overhead cost. Variable Overhead Expenditure Variance (VOExp.V) = Budgeted variable overhead cost—Actual variable overhead cost On the basis of hours: VOExp.V = (SRH − ARH) AH × AO On the basis of units: VOExp.V = SRU × SO − ARU × AO iii. Variable Overhead Efficiency Variance (VOEff.V): It is the difference between the standard variable overhead cost and budgeted variable overhead cost. Variable Overhead Efficiency Variance (VOEff.V) = Standard variable overhead cost − Budgeted variable overhead cost On the basis of hours: = SRH [SH × AO − AH × AO] On the basis of units: = SRU × AO − SRU × SO Tutorial Notes 1. SRH = Standard variable overhead rate per labour hour ARH = Actual variable overhead rate per labour hour SH = Standard labour hours required to produce one unit of output AH = Actual labour hours worked to produce one unit of output AO = Actual output SH × AO = Total standard labour hours required to produce the total actual output AH × AO = Total actual labour hours worked to produce the total actual output SRU = Standard variable overhead rate per unit of production ARU = Actual variable overhead rate per unit of production SO = Standard output Budgeted variable overhead 2. Standard variable overhead rate per labour hour ( SRH) = Budgeted hours 3. Standard variable overhead rate per unit of production ( SRU) =

Budgeted variable overhead Budgeted output

4. Reconciliation or Test of Variable Overhead Variances: VOCV = VOExp.V + VOEff.V

Modified Date: Mon, Jul 05, 2010 05:03:14 PM

Output Date: Tue, Jul 06, 2010 12:21:13 PM

Rev II

Project: Management Accounting_Debarshi Bhattacharyya ACE Pro India Pvt. Ltd. File: X:\Pearson\Management Accounting_Debarshi Bhattacharyya\MAIN\M08\LAYOUT_M08\M08_DEBA_ISBN_EN_SE_C08.indd

546

MANAGEMENT ACCOUNTING

8.10.4 Fixed Overhead Variances Fixed overhead variances are classified in several parts as shown in Figure 8.5. Fixed Overhead Cost Variance (FOCV)

Fixed Overhead Expenditure Variance (FOExp.V)

Fixed Overhead Efficiency Variance (FOEff.V)

Figure 8.5

Fixed Overhead Volume Variance (FOVol.V)

Fixed Overhead Capacity Variance (FOCap.V)

Fixed Overhead Calender Variance (FOCal.V)

Classification of Fixed Overhead Variance

i. Fixed Overhead Cost Variance (FOCV): It is the difference between the standard fixed overhead for the actual output and the actual fixed overhead for the actual output. Fixed Overhead Cost Variance (FOCV) = Standard fixed overhead for the actual output − Actual fixed overhead for the actual output On the basis of hours: = SRH × SH × AO − ARH × AH × AO On the basis of units: = SRU × AO − ARU × AO ii. Fixed Overhead Expenditure Variance (FOExp.V): It is the difference between the budgeted fixed overhead for the budgeted output and the actual fixed overhead for the actual output. Fixed Overhead Expenditure Variance (FOExp.V) = Budgeted fixed overhead for the budgeted output − Actual fixed overhead for the actual output On the basis of hours: FOExp.V = SRH × SH × BO − ARH × AH × AO On the basis of units: FOExp.V = SRU × BO − ARU × AO iii. Fixed Overhead Volume Variance (FOVol.V): It is the difference between the standard fixed overhead for the actual output and budgeted fixed overhead for the budgeted output. Fixed Overhead Volume Variance (FOVol.V) = Standard fixed overhead for the actual output − Budgeted fixed overhead for the budgeted output On the basis of hours: FOVol.V = SRH × SH × AO − SRH × SH × BO On the basis of units: FOVol.V = SRU × AO − SRU × BO iv. Fixed Overhead Efficiency Variance (FOEff.V): On the basis of hours: FOEff.V = Standard fixed overhead for the actual output − Actual fixed overhead for the actual output at standard rate = SRH × SH × AO − SRH × AH × AO

Modified Date: Mon, Jul 05, 2010 05:03:14 PM

Output Date: Tue, Jul 06, 2010 12:21:13 PM

Rev II

Project: Management Accounting_Debarshi Bhattacharyya ACE Pro India Pvt. Ltd. File: X:\Pearson\Management Accounting_Debarshi Bhattacharyya\MAIN\M08\LAYOUT_M08\M08_DEBA_ISBN_EN_SE_C08.indd

STANDARD COSTING AND VARIANCE ANALYSIS

547

On the basis of units: FOEff.V = Standard fixed overhead for the actual output—Standard fixed overhead for the standard output = SRU × AO − SRU × SO v. Fixed Overhead Capacity Variance (FOCap.V): On the basis of hours: FOCap.V = Actual fixed overhead for AO at standard rate − Standard fixed overhead for RSH = SRH × AH × AO − SRH × RSH = SRH (AH × AO − RSH) On the basis of units: FOCap.V = Standard fixed overhead for the standard output − Budgeted fixed overhead for the budgeted output = SRU × SO − SRU × BO vi. Fixed Overhead Calendar Variance (FOCal.V): On the basis of hours: FOCal.V = Standard fixed overhead for RSH − Budgeted fixed overheads for the budgeted output = SRH × RSH − SRH × SH × BO = SRH [RSH − (SH × BO)] On the basis of units: FOCal.V = Standard fixed overhead for the actual days − Budgeted fixed overhead for the budgeted days = SRD × Actual days − SRD × Budgeted days = SRD [Actual days − Budgeted days]

Tutorial Notes 1. SRH = Standard fixed overhead rate per hour ARH = Actual fixed overhead rate per hour SRU = Standard fixed overhead rate per unit ARU = Actual fixed overhead rate per unit SRD = Standard fixed overhead rate per day SH = Standard hours required for one unit of output AH = Actual hours taken for one unit of output RSH = =

Total budgeted Standard hours in actual days Total standard Actual days × Budgeted hours Budgeted days

AO = Actual output BO = Budgeted output SO = Standard output = Budgeted output for actual days BH = Budgeted hours for budgeted output Budgeted fixed overhead 2. Standard fixed overhead rate per hour ( SRH) = Budgeted hours

Modified Date: Mon, Jul 05, 2010 05:03:14 PM

Output Date: Tue, Jul 06, 2010 12:21:13 PM

Rev II

Project: Management Accounting_Debarshi Bhattacharyya ACE Pro India Pvt. Ltd. File: X:\Pearson\Management Accounting_Debarshi Bhattacharyya\MAIN\M08\LAYOUT_M08\M08_DEBA_ISBN_EN_SE_C08.indd

548

MANAGEMENT ACCOUNTING

3. Standard fixed overhead rate per unit ( SRU) =

Budgeted fixed overhead Budgeted output

4. ‘Budgeted’ refers to ‘Budgeted cost for budgeted output’. 5. ‘Standard’ refers to ‘Budgeted cost for actual output’. 6. If the fixed overhead variances are calculated ‘on the basis of units’, either FOCap.V or FOCal.V is to be ascertained. But if fixed overhead variances are calculated ‘on the basis of hours’, both these variances are to be ascertained. 7. Reconciliation/Test of Fixed Overhead Variances: On the basis of hours: FOCV = FOExp.V + FOVol.V FOVol.V = FOEff.V + FOCap.V + FOCal.V On the basis of units: FOCV = FOExp.V + FOVol.V FOVol.V = FOEff.V + FOCap.V or FOCal.V

8.10.5 Sales Value Variances Sales Value Variances are classified in several parts as shown in Figure 8.6. Total Sales Value Variance (TSVV)

Sales Price/Rate Variance (SPV/SRV)

Sales Volume Variance (SVV)

Sales Mix Variance (SMV)

Figure 8.6

Sales Quantity Variance (SQV)

Classification of Sales Value Variance

i. Total Sales Value Variance (TSVV): It is the difference between the actual value of sales and the budgeted value of sales. Total Sales Value Variance (TSVV) = Actual value of sales − Budgeted value of sales = AQ × AP − BQ × SP ii. Sales Price/Rate Variance (SPV/SRV): It is the difference between the actual sales and the standard sales. Sales Price/Rate Variance (SPV/SRV) = Actual price of sales − Standard price of sales = Actual sales − Standard sales = AQ × AP − AQ × SP = AQ (AP − SP) iii. Sales Volume Variance (SVV): It is the difference between the standard sales and the budgeted sales. Sales Volume Variance (SVV) = Actual sales quantity at the standard rate − Budgeted quantity of sales at the standard rate = Standard sales − Budgeted sales = AQ × SP − BQ × SP = SP (AQ − BQ) iv. Sales Mix Variance (SMV): It is the difference between the standard sales and the revised standard sales.

Modified Date: Mon, Jul 05, 2010 05:03:14 PM

Output Date: Tue, Jul 06, 2010 12:21:13 PM

Rev II

Project: Management Accounting_Debarshi Bhattacharyya ACE Pro India Pvt. Ltd. File: X:\Pearson\Management Accounting_Debarshi Bhattacharyya\MAIN\M08\LAYOUT_M08\M08_DEBA_ISBN_EN_SE_C08.indd

STANDARD COSTING AND VARIANCE ANALYSIS

549

Sales Mix Variance (SMV) = Actual sales quantity at the standard rate − Revised standard sales quantity at the standard rate = Standard sales − Revised standard sales = AQ × SP − RSQ × SP = SP (AQ − RSQ) v. Sales Quantity Variance (SQV): It is the difference between the revised standard sales and the budgeted sales. Sales Quantity Variance (SQV) = Revised standard sales quantity at the standard rate − Budgeted sales quantity at the standard rate = Revised standard sales − Budgeted sales = RSQ × SP − BQ × SP = SP (RSQ − BQ)  Tutorial Notes 1. AQ = Actual sales quantity BQ = Budgeted sales quantity RSQ = Revised standard sales quantity = Total actual sales quantity in the budgeted sales quantity ratio = Total actual sales quantity × SP =

Budgeted sales quantity of a particular product Total budgeted sales quantity

Budgeted Standard selling price per unit

AP = Actual selling price per unit 2. Reconciliation or Test of Sales Value Variances: TSVV = SPV /SRV + SVV SVV = SMV + SQV

8.10.6 Sales Margin (Profit) Variances Sales margin/Profit variances are classified in several parts as shown in Figure 8.7. Total Sales Margin Variance (TSMV)

Sales Margin Price Variance (SMPV)

Sales Margin Volume Variance (SMVV)

Sales Margin Mix Variance (SMMV)

Figure 8.7

Sales Margin Quantity Variance (SMQV)

Classification of Sales Margin Variance

i. Total Sales Margin Variance (TSMV): It is the difference between the actual profit and the budgeted profit.

Modified Date: Mon, Jul 05, 2010 05:03:14 PM

Output Date: Tue, Jul 06, 2010 12:21:13 PM

Rev II

Project: Management Accounting_Debarshi Bhattacharyya ACE Pro India Pvt. Ltd. File: X:\Pearson\Management Accounting_Debarshi Bhattacharyya\MAIN\M08\LAYOUT_M08\M08_DEBA_ISBN_EN_SE_C08.indd

550

MANAGEMENT ACCOUNTING

ii.

iii.

iv.

v.

Total Sales Margin Variance (TSMV) = Actual profit − Budgeted profit = AQ × AR − BQ × SR Sales Margin Price Variance (SMPV): It is the difference between the actual profit and the standard profit. Sales Margin Price Variance (SMPV) = Actual profit for the actual sales quantity − Budgeted profit for the actual sales quantity = Actual profit − Standard profit = AQ × AR − AQ × SR = AQ (AR − SR) Sales Margin Volume Variance (SMVV): It is the difference between the standard profit and the budgeted profit. Sales Margin Volume Variance (SMVV) = Budgeted profit for the actual sales quantity − Budgeted profit for the budgeted sales quantity = Standard profit − Budgeted profit = AQ × SR − BQ × SR = SR (AQ − BQ) Sales Margin Mix Variance (SMMV): It is the difference between the standard profit and the revised standard profit. Sales Margin Mix Variance (SMMV) = Budgeted profit for the actual sales quantity − Budgeted profit for the revised standard sales quantity = Standard profit − Revised standard profit = AQ × SR − RSQ × SR = SR (AQ − RSQ) Sales Margin Quantity Variance (SMQV): It is the difference between the revised standard profit and the budgeted profit. Sales Margin Quantity Variance (SMQV) = Budgeted profit for the revised standard sales quantity − Budgeted profit for the budgeted sales quantity = Revised standard profit − Budgeted profit = RSQ × SR − BQ × SR = SR (RSQ − BQ)

Tutorial Notes 1. AQ = Actual sales quantity BQ = Budgeted sales quantity RSQ = Revised standard sales quantity = Total actual sales quantity in budgeted sales quantity ratio = Total actual sales quantity ×

Budgeted sales quantity of a particular product Total budgeted sales quantity

AR = Actual rate of profit Budgeted SR = Standard rate of profit 2. Reconciliation or Test of Sales Margin Variances: TSMV = SMPV + SMVV SMVV = SMQV + SMMV

Modified Date: Mon, Jul 05, 2010 05:03:14 PM

Output Date: Tue, Jul 06, 2010 12:21:13 PM

Rev II

Project: Management Accounting_Debarshi Bhattacharyya ACE Pro India Pvt. Ltd. File: X:\Pearson\Management Accounting_Debarshi Bhattacharyya\MAIN\M08\LAYOUT_M08\M08_DEBA_ISBN_EN_SE_C08.indd

STANDARD COSTING AND VARIANCE ANALYSIS

551

8.11 FORMULAE OF VARIOUS VARIANCES AT A GLANCE I. Cost Variances 1. Material Variances: i. Material Cost Variance (MCV) = Standard material cost for the actual output − Actual material cost for the actual output = SP × SQ × AO − AP × AQ × AO ii. Material Price Variance (MPV) = (SP – AP) AQ × AO iii. Material Usage/Quantity Variance (MUV/MQV) = SP [SQ × AO – AQ × AO] iv. Material Mix Variance (MMV) = SP [Standard mix of material in the actual input − Actual mix of that material in the actual input] v. Material Yield Variance (MYV) = Standard yield rate [Actual yield for the actual input − Standard yield for the actual input] Stop and Think SP = Standard price of materials per unit AP = Actual price of materials per unit SQ = Standard quantity of materials required to produce one unit of output AQ = Actual quantity of materials used to produce one unit of output AO = Actual output Standard yield rate =

Standard material cost of the standard output Standard output

Reconciliation or Test of Material Variances: MCV = MPV + MUV /MQV MUV /MQV = MMV + MYV

2. Labour Variances: i. Labour Cost Variance (LCV) = Standard labour cost for the actual output – Actual labour cost for the actual output = SR × SH × AO − AR × AH × AO ii. Labour Rate/Price Variance (LRV/LPV) = (SR – AR) AH × AO iii. Labour Idle-Time Variance (LITV) = SR × Abnormal idle hours iv. Labour Efficiency Variance (LEV) = SR [SH × AO – Effective AH × AO] v. Labour Mix Variance (LMV) = SR [RSH – Effective AH × AO] vi. Labour Yield Variance (LYV) = SR [SH × AO – RSH]

Modified Date: Mon, Jul 05, 2010 05:03:14 PM

Output Date: Tue, Jul 06, 2010 12:21:13 PM

Rev II

Project: Management Accounting_Debarshi Bhattacharyya ACE Pro India Pvt. Ltd. File: X:\Pearson\Management Accounting_Debarshi Bhattacharyya\MAIN\M08\LAYOUT_M08\M08_DEBA_ISBN_EN_SE_C08.indd

552

MANAGEMENT ACCOUNTING

Stop and Think SR = Standard labour rate per hour AR = Actual labour rate per hour SH = Standard hours required to produce one unit of output AH = Actual hours taken to produce one unit of output AO = Actual output SH × AO = Total standard hours required to produce the total actual output AH × AO = Total actual hours taken to produce the total actual output (including abnormal idle hours) Effective AH × AO = Effective actual hours taken to produce the total actual output (excluding abnormal idle hours), i.e., [(AH × AO) – Abnormal idle hours] RSH = Revised Standard Hours = Total actual hours in the standard ratio Standard hours required for a particular class of labour = Total actual hours taken × Total standard hours required for all classes of labour Reconciliation or Test of Labour Variances: LCV = LRV /LPV + LITV + LEV LEV = LMV + LYV

3. Variable Overhead Variances: i. Variable Overhead Cost Variance (VOCV) = Standard variable overhead cost for the actual output − Actual variable overhead cost for the actual output On the basis of hours: = SRH × SH × AO − ARH × AH × AO On the basis of units: = SRU × AO − ARU × AO ii. Variable-Overhead-Expenditure Variance (VOExp.V) = Budgeted variable overhead cost − Actual variable overhead cost On the basis of hours: = (SRH − ARH) AH × AO On the basis of units: = SRU × SO − ARU × AO iii. Variable Overhead Efficiency Variance (VOEff.V) = Standard variable overhead cost − Budgeted variable overhead cost On the basis of hours: = SRH [SH × AO − AH × AO] On the basis of units: = SRU × AO − SRU × SO Stop and Think SRH = Standard variable overhead rate per labour hour ARH = Actual variable overhead rate per labour hour SH = Standard hours required to produce one unit of output AH = Actual hours worked to produce one unit of output AO = Actual output SH × AO = Total standard hours required to produce the total actual output (Continued)

Modified Date: Mon, Jul 05, 2010 05:03:14 PM

Output Date: Tue, Jul 06, 2010 12:21:13 PM

Rev II

Project: Management Accounting_Debarshi Bhattacharyya ACE Pro India Pvt. Ltd. File: X:\Pearson\Management Accounting_Debarshi Bhattacharyya\MAIN\M08\LAYOUT_M08\M08_DEBA_ISBN_EN_SE_C08.indd

STANDARD COSTING AND VARIANCE ANALYSIS

553

AH × AO = Total actual hours worked to produce the total actual output SRU = Standard variable overhead rate per unit of production ARU = Actual variable overhead rate per unit of production SO = Standard output Reconciliation or Test of Variable Overhead Variances: VOCV = VOExp.V + VOEff.V

4. Fixed Overhead Variances: i. Fixed Overhead Cost Variance (FOCV) = Standard fixed overhead for the actual output − Actual fixed overhead for the actual output On the basis of hours: = SRH × SH × AO − ARH × AH × AO On the basis of units: = SRU × AO − ARU × AO ii. Fixed Overhead Expenditure Variance (FOExp.V) = Budgeted fixed overhead for the budgeted output − Actual fixed overhead for the actual output On the basis of hours: = SRH × SH × BO − ARH × AH × AO On the basis of units: = SRU × BO − ARU × AO iii. Fixed Overhead Volume Variance (FOVol.V) = Standard fixed overhead for the actual output − Budgeted fixed overhead for the budgeted output On the basis of hours: = SRH × SH × AO − SRH × SH × BO On the basis of units: = SRU × AO − SRU × BO iv. Fixed Overhead Efficiency Variance (FOEff.V) On the basis of hours: = Standard fixed overhead for the actual output − Actual fixed overhead for the actual output at standard rate = SRH × SH × AO − SRH × AH × AO On the basis of units: = Standard fixed overhead for the actual output − Standard fixed overhead for the standard output = SRU × AO − SRU × SO v. Fixed Overhead Capacity Variance (FOCap.V) On the basis of hours: = Actual fixed overhead for AO at standard rate − Standard fixed overhead for RSH = SRH × AH × AO − SRH × RSH = SRH (AH × AO − RSH) On the basis of units: = Standard fixed overhead for the standard output − Budgeted fixed overhead for the budgeted output = SRU × SO − SRU × BO vi. Fixed Overhead Calendar Variance (FOCal.V) On the basis of hours: = Standard fixed overhead for RSH − Budgeted fixed overheads for the budgeted output = SRH × RSH − SRH × SH × BO = SRH [RSH − (SH × BO)]

Modified Date: Mon, Jul 05, 2010 05:03:14 PM

Output Date: Tue, Jul 06, 2010 12:21:13 PM

Rev II

Project: Management Accounting_Debarshi Bhattacharyya ACE Pro India Pvt. Ltd. File: X:\Pearson\Management Accounting_Debarshi Bhattacharyya\MAIN\M08\LAYOUT_M08\M08_DEBA_ISBN_EN_SE_C08.indd

554

MANAGEMENT ACCOUNTING

On the basis of units: = Standard fixed overhead for the actual days − Budgeted fixed overhead for the budgeted days = SRD × Actual days − SRD × Budgeted days = SRD [Actual days − Budgeted days] Stop and Think i. SRH = Standard fixed overhead rate per hour ARH = Actual fixed overhead rate per hour SRU = Standard fixed overhead rate per unit ARU = Actual fixed overhead rate per unit SRD = Standard fixed overhead rate per day SH = Standard hours required for one unit of output AH = Actual hours taken for one unit of output RSH = Total budgeted/standard hours in actual days = Total budgeted/standard hours × AO BO SO BH

Actual days Budgeted days

= Actual output = Budgeted output = Standard output = Budgeted output for actual days = Budgeted hours for the budgeted output

ii. Standard fixed overhead rate per hour ( SRH) = iii. Standard fixed overhead rate per unit ( SRU) =

Budgeted fixed overhead Budgeted hours

Budgeted fixed overhead Budgeted output

iv. ‘Budgeted’ refers to ‘Budgeted cost for the budgeted output.’ v. ‘Standard’ refers to ‘Budgeted cost for the actual output.’ vi. Standard output ( SO) = Budgeted output for the actual hours/days vii. If fixed overhead variances are calculated ‘on the basis of units,’ either FOCap.V or FOCal.V is to be ascertained. But if the fixed overhead variances are calculated ‘on the basis of hours,’ both these variances are to be ascertained. viii. Reconciliation/Test of Fixed Overhead Variances: On the basis of hours: FOCV = FOExp.V + FOVol.V FOVol.V = FOEff.V + FOCap.V + FOCal.V On the basis of units: FOCV = FOExp.V + FOVol.V FOVol.V = FOEff.V + FOCap.V or FOCal.V

II. Sales Variances: 1. Sales Value Variances: i. Total Sales Value Variance (TSVV) = Actual value of sales − Budgeted value of sales = AQ × AP − BQ × SP

Modified Date: Mon, Jul 05, 2010 05:03:14 PM

Output Date: Tue, Jul 06, 2010 12:21:13 PM

Rev II

Project: Management Accounting_Debarshi Bhattacharyya ACE Pro India Pvt. Ltd. File: X:\Pearson\Management Accounting_Debarshi Bhattacharyya\MAIN\M08\LAYOUT_M08\M08_DEBA_ISBN_EN_SE_C08.indd

STANDARD COSTING AND VARIANCE ANALYSIS

555

ii. Sales Price/Rate Variance (SPV/SRV) = Actual price of sales − Standard price of sales = AQ × AP − AQ × SP = AQ (AP − SP) iii. Sales Volume Variance (SVV) = Actual sales quantity at standard rate − Budgeted quantity of sales at standard rate = AQ × SP − BQ × SP = SP (AQ − BQ) iv. Sales Mix Variance (SMV) = Actual sales quantity at standard rate − Revised standard sales quantity at standard rate = AQ × SP − RSQ × SP = SP (AQ − RSQ) v. Sales Quantity Variance (SQV) = Revised standard sales quantity at standard rate − Budgeted sales quantity at standard rate = RSQ × SP − BQ × SP = SP (RSQ − BQ) Stop and Think i. AQ = Actual sales quantity BQ = Budgeted sales quantity RSQ = Revised standard sales quantity = Total actual sales quantity in the budgeted sales quantity ratio Budgeted sales quantity of a particular product = Total actual sales quantity × Total budgeted sales quantity SP = Budgeted/Standard selling price per unit AP = Actual selling price per unit ii. Reconciliation or Test of Sales Value Variances: TSVV = SPV/SRV + SVV SVV = SMV + SQV

2. Sales Margin Variances: i. Total Sales Margin Variance (TSMV) = Actual profit − Budgeted profit = AQ × AR − BQ × SR ii. Sales Margin Rate/Price Variance (SMRV/SMPV) = Actual profit for actual sales quantity − Budgeted profit for actual sales quantity = AQ × AR − AQ × SR = AQ (AR − SR) iii. Sales Margin Volume Variance (SMVV) = Budgeted profit for actual sales quantity − Budgeted profit for budgeted sales quantity = AQ × SR − BQ × SR = SR (AQ − BQ)

Modified Date: Mon, Jul 05, 2010 05:03:14 PM

Output Date: Tue, Jul 06, 2010 12:21:13 PM

Rev II

Project: Management Accounting_Debarshi Bhattacharyya ACE Pro India Pvt. Ltd. File: X:\Pearson\Management Accounting_Debarshi Bhattacharyya\MAIN\M08\LAYOUT_M08\M08_DEBA_ISBN_EN_SE_C08.indd

556

MANAGEMENT ACCOUNTING

iv. Sales Margin Mix Variance (SMMV) = Budgeted profit for actual sales quantity − Budgeted profit for revised standard sales quantity = AQ × SR − RSQ × SR = SR (AQ − RSQ) v. Sales Margin Quantity Variance (SMQV) = Budgeted profit for revised standard sales quantity − Budgeted profit for budgeted sales quantity = RSQ × SR − BQ × SR = SR (RSQ − BQ) Stop and Think i. AQ = Actual sales quantity BQ = Budgeted sales quantity RSQ = Revised standard sales quantity = Total actual sales quantity in budgeted sales quantity ratio Budgeted sales quantity of a particular product = Total actual sales quantity × Total budgeted sales quantity AR = Actual rate of profit SR = Budgeted/Standard rate of profit ii. Reconciliation or Test of Sales Margin Variances: TSMV = SMRV/SMPV + SMVV SMVV = SMQV + SMMV

8.12 WORKED-OUT PROBLEMS A. Material Variances: Problem 1 From the following information, calculate (i) MCV; (ii) MPV; and (iii) MUV: Standard Price of material per kg Actual Price of material per kg Standard Quantity of material required to produce 1 unit of output Actual Quantity of Material used to produce 1 unit of output Actual Output

Rs. 10 Rs. 8 20 kg 24 kg 20,000 units

Solution SP = Standard price of material per kg = Rs. 10 AP = Actual price of material per kg = Rs. 8 SQ = Standard quantity of material required to produce one unit of output = 20 kg AQ = Actual quantity of material used to produce one unit of output = 24 kg AO = Actual output = 20,000 units i. Material Cost Variance (MCV) = SP × SQ × AO − AP × AQ × AO = 10 × 20 × 20,000 − 8 × 24 × 20,000 = 40,00,000 − 38,40,000 = Rs. 1,60,000 (F) [Here, F = Favourable or Positive (+)]

Here,

Modified Date: Mon, Jul 05, 2010 05:03:14 PM

Output Date: Tue, Jul 06, 2010 12:21:13 PM

Rev II

Project: Management Accounting_Debarshi Bhattacharyya ACE Pro India Pvt. Ltd. File: X:\Pearson\Management Accounting_Debarshi Bhattacharyya\MAIN\M08\LAYOUT_M08\M08_DEBA_ISBN_EN_SE_C08.indd

STANDARD COSTING AND VARIANCE ANALYSIS

557

ii. Material Price Variance (MPV) = (SP − AP) AQ × AO = (10 − 8) × 24 × 20,000 = 2 × 24 × 20,000 = Rs. 9,60,000 (F) iii. Material Usage Variance (MUV) = SP [SQ × AO − AQ × AO] = 10 [20 × 20,000 − 24 × 20,000] = 10 (4,00,000 − 4,80,000) = Rs. 8,00,000 (A) [Here, A = Adverse or Negative (–)] Test MCV = MPV + MUV Here, Rs. 1,60,000 (F) = Rs. 9,60,000 (F) + Rs. 8,00,000 (A) = Rs. 1,60,000 (F), checked Problem 2 Standard Cost of material per kg Standard Quantity of material required to produce one unit of output Actual Quantity of material used for actual output of 20,000 units Actual Cost of material for actual output

Rs. 20 8 kg 2,00,000 kg Rs. 48,00,000

Compute Materials Cost Variances.

Solution Material Cost Variance (MCV) = Standard Cost of material for the actual output − actual cost of material for the actual output = Rs. 32,00,0001 − Rs. 48,00,0002 = Rs. 16,00,000 (A) Material Price Variance (MPV) = (SP − AP) AQ × AO = (201 − 244) × 2,00,0003 = Rs. 8,00,000 (A) Material Quantity Variance (MQV) = SP [ SQ × AO − AQ × AO] = 201 [(81 × 20,000) – 2,00,0003] = 20 × (–) 40,000 = Rs. 8,00,000 (A) Test MCV = MPV + MQV Here, Rs. 16,00,000 (A) = Rs. 8,00,000 (A) + Rs. 8,00,000 (A) = Rs. 16,00,000 (A), checked. Working Notes 1. Here, Standard Cost of material per kg = SP = Rs. 20 Standard quantity of material required to produce one unit of output (SQ) = 8 kg Actual output = AO = 20,000 units ∴ Standard Cost of material for the actual output = SP × SQ × AO = 20 × 8 × 20,000 = Rs. 32,00,000 (Continued)

Modified Date: Mon, Jul 05, 2010 05:03:14 PM

Output Date: Tue, Jul 06, 2010 12:21:13 PM

Rev II

Project: Management Accounting_Debarshi Bhattacharyya ACE Pro India Pvt. Ltd. File: X:\Pearson\Management Accounting_Debarshi Bhattacharyya\MAIN\M08\LAYOUT_M08\M08_DEBA_ISBN_EN_SE_C08.indd

558

MANAGEMENT ACCOUNTING

2. Actual cost of material for the actual output (i.e., AP × AQ × AO) = Rs. 48,00,000 3. Actual quantity of material used for the actual output (i.e., AQ × AO) = 2,00,000 kg 4. Actual price of material per kg = AP = Rs. 48,00,000 ÷ 2,00,000 kg = Rs. 24

Problem 3 Calculate MCV, MPV and MUV from the following data: Standard − For 5 units of Product A. Materials − 80 kg @ Rs. 20.00 per kg. Actual production − 800 units. Materials − 13,000 kg @ Rs. 19.50 per kg. [B.Com. (Hons), Mumbai University—April 2009] Solution SP = Standard price of material per kg = Rs. 20 AP = Actual price of material per kg = Rs. 19.50 SQ = Standard quantity of material required to produce one unit of output = 80 kg ÷ 5 = 16 kg AQ = Actual quantity of material used to produce one unit of output = 13,000 kg ÷ 800 = 16.25 kg AO = Actual output = 800 units Material Cost Variance (MCV) = SP × SQ × AO − AP × AQ × AO = 20 × 16 × 800 − 19.50 × 16.25 × 800 = 2,56,000 − 2,53,500 = Rs. 2,500 (F) [Here, F = Favourable or Positive (+)] Material Price Variance (MPV) = (SP − AP) AQ × AO = (20 − 19.50) × 16.25 × 800 = 0.50 × 16.25 × 800 = Rs. 6,500 (F) Material Usage Variance (MUV) = SP [SQ × AO − AQ × AO] = 20 [16 × 800 − 16.25 × 800] = 20 (12,800 − 13,000) = Rs. 4,000 (A) [Here, A = Adverse or Negative (–)]

Here,

Test MCV = MPV + MUV Here, Rs. 2,500 (F) = Rs. 6,500 (F) + Rs. 4,000 (A) = Rs. 2,500 (F), checked Problem 4 Standard Quantity of material required for 100 units of output Standard Price of material per kg Actual Quantity of material used for actual output Actual Cost of material for actual output Actual Output

800 kg Rs. 10 50,000 kg Rs. 6,00,000 10,000 units

Calculate Materials Cost Variances.

Modified Date: Mon, Jul 05, 2010 05:03:14 PM

Output Date: Tue, Jul 06, 2010 12:21:13 PM

Rev II

Project: Management Accounting_Debarshi Bhattacharyya ACE Pro India Pvt. Ltd. File: X:\Pearson\Management Accounting_Debarshi Bhattacharyya\MAIN\M08\LAYOUT_M08\M08_DEBA_ISBN_EN_SE_C08.indd

STANDARD COSTING AND VARIANCE ANALYSIS

559

Solution Material Cost Variance (MCV) = Standard Cost of material for actual output − Actual cost of material for actual output = Rs. 8,00,0001 − Rs. 6,00,0002 = Rs. 2,00,000 (F) Material Price Variance (MPV) = (SP − AP) AQ × AO = (101 − 122) × 50,000 = Rs. 1,00,000(A) Material Quantity Variance (MQV) = SP [ SQ × AO − AQ × AO] = 101 [(8 × 10,000)1 − 50,0002] = 10 × 30,000 = Rs. 3,00,000 (F) Test MCV = MPV + MQV Here, 2,00,000 (F) = 1,00,000 (A) + 3,00,000 (F) = 2,00,000 (A), checked Working Notes 1. Standard price of material per kg (i.e., SP) = Rs. 10 Standard quantity of material required to produce one unit of output (i.e., SQ) = 800 kg ÷ 100 units = 8 kg Actual output (AO) = 10,000 units ∴ Standard Cost of material for the actual output = SP × SQ × AO = 10 × 8 × 10,000 = Rs. 8,00,000 2. Actual quantity of material used for the actual output (i.e., AQ × AO) = 50,000 kg Actual price of material per kg (i.e., AP) = Rs. 6,00,000 ÷ 50,000 kg = Rs. 12 ∴ Actual cost of material for the actual output = AP × AQ × AO = 12 × 50,000 = Rs. 6,00,000

Problem 5 The standard raw material cost for producing 300 units of Product A were as follows: 600 units of raw materials @ Rs. 50 per unit = Rs. 30,000 But, the cost of actual raw materials producing 300 units of Product A were: 1,000 units of raw materials @ Rs. 40 per unit = Rs. 40,000 Determine: i. Raw Materials Cost Variance. ii. Raw Materials Price Variance. iii. Raw Materials Usage Variance. [B.Com. (Hons), Calcutta University—1998] Solution i. Raw Material Cost Variance (RMCV) = Standard Cost of raw material for actual output − Actual cost of raw material for actual output = Rs. 30,0001 – Rs. 40,0002 = Rs. 10,000(A) ii. Raw Material-Price Variance (RMPV) = (SP − AP) AQ × AO = (501 − 402) × 1,0002 = Rs. 10,000 (F)

Modified Date: Mon, Jul 05, 2010 05:03:14 PM

Output Date: Tue, Jul 06, 2010 12:21:13 PM

Rev II

Project: Management Accounting_Debarshi Bhattacharyya ACE Pro India Pvt. Ltd. File: X:\Pearson\Management Accounting_Debarshi Bhattacharyya\MAIN\M08\LAYOUT_M08\M08_DEBA_ISBN_EN_SE_C08.indd

560

MANAGEMENT ACCOUNTING

iii. Raw Material Usage Variance (RMUV) = SP [SQ × AO − AQ × AO] = 501 [6001 − 10002] = Rs. 20,000 (A) Test RMCV = RMPV + RMUV Here, Rs. 10,000 (A) = Rs. 10,000 (F) + Rs. 20,000 (A) = Rs. 10,000 (A), checked Working Notes 1. Here, Actual output (AO) = 300 units Standard quantity of raw materials required for the actual output (i.e., SQ × AO) = 600 units Standard price of raw materials per unit (SP) = Rs. 50 ∴ Standard Cost of raw materials for the actual output = SP × SQ × AO = Rs. 50 × 600 units = Rs. 30,000 2. But, the Actual quantity of raw materials used for the actual output of 300 units (i.e., AQ × AO) = 1,000 units Actual price of raw material per unit (AP) = Rs. 40 ∴ Actual cost of raw material for the actual output = AP × AQ × AO = Rs. 40 × 1,000 units = Rs. 40,000

Problem 6 From the following particulars, compute the material variances: Quantity of materials purchased Value of materials purchased Standard quantity of materials required per tonne of output Standard Rate of material per unit Opening Stock of materials Closing Stock of materials Output during the period

3,000 units Rs. 9,000 30 units Rs. 2.50 Nil 500 units 80 tonnes

[B.Com. (Hons), Delhi University—Adapted] Solution Materials Consumed = Opening Stock + Purchase − Closing Stock Here, Materials Consumed = Nil + 3,000 units – 500 units = 2,500 units Standard rate of materials per unit = SP = Rs. 2.50 Actual rate of materials per unit = AR = Rs. 9,000 ÷ 3,000 units = Rs. 3.00 Standard quantity of materials required to produce one tonne of output = SQ = 30 units Actual output = AO = 80 tonnes Actual quantity of materials consumed for the actual output = AQ × AO = 2,500 units ∴ Material Cost Variance (MCV) = SP × SQ × AO − AP × AQ × AO = 2.50 × 30 × 80 − 3 × 2,500 = 6,000 − 7,500 = Rs. 1,500 (A) ∴ Material Price Variance (MPV) = (SP – AP) AQ × AO = (2.50 – 3) × 2,500 = Rs. 1,250 (A)

Modified Date: Mon, Jul 05, 2010 05:03:14 PM

Output Date: Tue, Jul 06, 2010 12:21:13 PM

Rev II

Project: Management Accounting_Debarshi Bhattacharyya ACE Pro India Pvt. Ltd. File: X:\Pearson\Management Accounting_Debarshi Bhattacharyya\MAIN\M08\LAYOUT_M08\M08_DEBA_ISBN_EN_SE_C08.indd

STANDARD COSTING AND VARIANCE ANALYSIS

561

∴ Material Usage Variance (MUV) = SP [SQ × AQ − AQ × AO] = 2.50 [(30 × 80) – 2,500] = Rs. 250 (A) Test MCV = MPV + MUV Here, Rs. 1,500 (A) = Rs. 1,250 (A) + Rs. 250 (A) = Rs. 1,500 (A), checked Problem 7 1 tonne of material input yields a standard output of 1,00,000 units. The standard price of material is Rs. 20 per kg. The actual quantity of material used is 10 tonnes and actual price paid is Rs. 21 per kg. Actual output obtained is 9,00,000 units. Compute Material Variances. [B.Com. (Hons), Calcutta University—2005] Solution Here, Standard price of material per kg = SP = Rs. 20 Standard quantity of materials required for standard output of 1,00,000 units = 1 tonne = 1,000 kg Actual output = AO = 9,00,000 units ∴ Standard quantity of material required for actual output of 9,00,000 units = SQ × AO =

1,000 kg × 9,00,000units = 9,000 kg 1,00,000 units

Actual price of material per kg = AP = Rs. 21 Actual quantity of material used for actual output of 9,00,000 units = AQ × AO = 10 tonnes = 10 × 1,000 kg = 10,000 kg i. Material Cost Variance (MCV) = SP × SQ × AO – AP × AQ × AO = Rs. 20 × 9,000 kg − Rs. 21 × 10,000 kg = Rs. 1,80,000 – Rs. 2,10,000 = Rs. 30,000 (A) ii. Material Price Variance (MPV) = (SP – AP) AQ × AO = (Rs. 20 – Rs. 21) × 10,000 kg = Rs. 10,000 (A) iii. Material Usage Variance (MUV) = SP [SQ × AO – AQ × AO] = Rs. 20 [9,000 kg – 10,000 kg] = Rs. 20,000 (A) Test MCV = MPV + MUV Here, Rs. 30,000 (A) = Rs. 10,000 (A) + Rs. 20,000 (A) = Rs. 30,000 (A), checked Problem 8 Material M N Less:

Loss

Standard Qty (units) 60 50 110 10 100

Rate (Rs.) 5 6

Actual Amount (Rs.) 300 300 600 – 600

Qty (units) 500 600 1,100 100 1,000

Rate (Rs.) 7 5

Amount (Rs.) 3,500 3,000 6,500 – 6,500

From the above information, calculate the Material Variances.

Modified Date: Mon, Jul 05, 2010 05:03:14 PM

Output Date: Tue, Jul 06, 2010 12:21:13 PM

Rev II

Project: Management Accounting_Debarshi Bhattacharyya ACE Pro India Pvt. Ltd. File: X:\Pearson\Management Accounting_Debarshi Bhattacharyya\MAIN\M08\LAYOUT_M08\M08_DEBA_ISBN_EN_SE_C08.indd

562

MANAGEMENT ACCOUNTING

Solution i. Material Cost Variance (MCV) = Standard Cost of material for actual output – Actual cost of material for actual output = (Rs. 600 ÷ 100 × 1,000)1 – Rs. 6,5001 = Rs. 6,000 – Rs. 6,500 = Rs. 500(A) ii. Material Price Variance (MPV) = (SP – AP) AQ × AO Material M: (5 – 7) × 500 = Rs. 1,000 (A) Material N: (6 – 5) × 600 = Rs. 600 (F) MPV Rs. 400 (A) iii. Material Usage Variance (MUV) = SP [SQ × AO – AQ × AO] Material M: 5 [(60 ÷ 100 × 1,000)2 – 500] = Rs. 500 (F) Material N: 6 [(50 ÷ 100 × 1,000)2 – 600] = Rs. 600 (A) MUV Rs. 100 (A) iv. Material Mix Variance (MMV) = SP[Standard proportion/Mix of materials for actual input − Actual proportion/Mix of materials in actual input] Material M: 5 {(60 ÷ 110 × 1,100)3 – 500} = Rs. 500 (F) Material N: 6 {(50 ÷ 110 × 1,100)2 – 600} = Rs. 600 (A) MMV Rs. 100 (A) v. Material Yield Variance (MYV) = Standard yield rate [Actual yield for actual input – Standard yield for actual input] = (Rs. 600 ÷ 100)4 [ 1,0004 − (100 ÷ 110 × 1,100)4] = 6 (1,000 – 1,000) = Nil Test i. MCV = MPV + MUV Here, Rs. 500 (A) = Rs. 400 (A) + Rs. 100 (A) = Rs. 500 (A), checked ii. Again, MUV = MMV + MYV Here, Rs. 100 (A) = 100 (A) + Nil = Rs. 100 (A), checked Working Notes 1. Total standard input = 110 units Total standard output = 100 units Standard Cost of material for 100 units of standard output = Rs. 600 Again, Total actual input = 1,100 units Total actual output = 1,000 units Actual cost of material for 1,000 units of output (i.e., for actual output) = AP × AQ × AO = Rs. 6,500 ∴ Total Standard Cost of material for actual output = SP × SQ × AO = Rs. 600 ÷ 100 units × 1,000 units = Rs. 6,000 2. Standard quantity of Material M required for 100 units of output = 60 units ∴ Standard quantity of Material M required for actual output of 1,000 units (i.e., SQ × AO for Material M) = 60 ÷ 100 × 1,000 = 600 units Again, Standard quantity of Material N required for 100 units of output = 50 units ∴ Standard quantity of Material N required for actual output of 1,000 units (i. e. SQ × AO for Material N) = 50 ÷ 100 × 1,000 = 500 units (Continued)

Modified Date: Mon, Jul 05, 2010 05:03:14 PM

Output Date: Tue, Jul 06, 2010 12:21:13 PM

Rev II

Project: Management Accounting_Debarshi Bhattacharyya ACE Pro India Pvt. Ltd. File: X:\Pearson\Management Accounting_Debarshi Bhattacharyya\MAIN\M08\LAYOUT_M08\M08_DEBA_ISBN_EN_SE_C08.indd

STANDARD COSTING AND VARIANCE ANALYSIS

563

3. Standard proportion/mix of Material M for standard input of 110 units = 60 units ∴ Standard proportion/mix of Material M for actual input of 1,100 units = Again,

60 × 1,100 = 600 units 100

Standard proportion/mix of Material N for standard input of 110 units = 50 units

∴Standard proportion/mix of Material N for actual input of 1,100 units =

50 × 1,100 = 500 units 110

4. Standard yield rate = Standard output rate =

Standard cost of standard output Rs. 600 = = Rs. 6 Standard output 100

Again, Actual yield for actual input = Actual output for actual input of 1,100 units = 1,000 units Again, Standard yield for actual input = Standard output for actual input of 1,100 units Here, Standard output for standard input of 110 units = 100 units ∴ Standard output for actual input of 1,100 units =

100 × 1,100 = 1,000 units 110

Problem 9 Material A B C Less:

Loss

Standard Qty (units) 500 400 700 1,600 100 1,500

Rate (Rs.) 6 9 7

Actual Amount (Rs.) 3,000 3,600 4,900 11,500 – 11,500

Qty (units) 7,000 5,000 10,000 22,000 2,000 20,000

Rate (Rs.) 5 10 6

Amount (Rs.) 35,000 50,000 60,000 1,45,000 – 1,45,000

From the above information, calculate the material variances. Solution i. Material Cost Variance (MCV) = Standard Cost of material for actual output – Actual cost of material for actual output = Rs. 1,53,3331 – Rs. 1,45,0001 = Rs. 8,333 (F) ii. Material Price Variance (MPV) = (SP – AP) AQ × AO Material A: (6 – 5) × 7,000 = Rs. 7,000 (F) Material B: (9 – 10) × 5,000 = Rs. 5,000 (A) Material C: (7 – 6) × 10,000 = Rs. 10,000 (F) Rs. 12,000 (F) iii. Material Quantity Variance (MQV) = SP [ SQ × AO – AQ × AO] Material A: 6 [(500 ÷ 1,500 × 20,000)2 − 7,000] = Rs. 2,000 (A) Material B: 9 [(400 ÷ 1,500 × 20,000)2 − 5,000] = Rs. 3,000 (F)

Modified Date: Mon, Jul 05, 2010 05:03:14 PM

Output Date: Tue, Jul 06, 2010 12:21:13 PM

Rev II

Project: Management Accounting_Debarshi Bhattacharyya ACE Pro India Pvt. Ltd. File: X:\Pearson\Management Accounting_Debarshi Bhattacharyya\MAIN\M08\LAYOUT_M08\M08_DEBA_ISBN_EN_SE_C08.indd

564

MANAGEMENT ACCOUNTING

Material C: 7 [(700 ÷ 1,500 × 20,000)2 − 10,000]

= Rs. 4,667 (A) Rs. 3,667 (A)

iv. Material Mix Variance (MMV) = SP [Standard mix of material in actual input – Actual mix of material in actual input] Material A: 6 [(500 ÷ 1,600 × 22,000)3 − 7,000] = Rs. 750 (A) Material B: 9 [(400 ÷ 1,600 × 22,000)3 − 5,000] = Rs. 4,500 (F) Material C: 7 [(700 ÷ 1,600 × 22,000)3 − 10,000] = Rs. 2,625 (A) Rs. 1,125 (F) v. Material Yield Variance (MYV) = Standard yield rate[Actual yield/output for actual input − Standard yield/output for actual input] = (Rs. 11,500 ÷ 1,500)4 [ 20,000 − (1,500 ÷ 1,600 × 22,000)4] = Rs. 4,792 (A) Test i. MCV = MPV + MQV Here, Rs. 8,333 (F) = Rs. 12,000 (F) + Rs. 3,667 (A) = Rs. 8,333 (F), checked ii. MQV = MMV + MYV Here, Rs. 3,667 (A) = Rs. 1,125 (F) + Rs. 4,792 (A) = Rs. 3,667 (A), checked

Working Notes 1. Total standard input = 1,600 units Total standard output = 1,500 units Standard Cost of material for 1,500 units of standard output = Rs. 11,500 Again, Total actual input = 22,000 units Total actual output (i.e., AO) = 20,000 units Actual cost of material for 20,000 units of actual output (i.e., AP × AQ × AO) = Rs. 1,45,000 ∴ Standard Cost of material for the actual output of 20,000 units (i.e., SP × SQ × AO) = Rs. 11,500 × (20,000 ÷1,500) = Rs. 1,53,333 2. Standard quantity of Material A required for the standard output of 1,500 units = 500 units ∴ Standard quantity of Material A required for the actual output of 20,000 units (i.e., SQ × AO for Material A) = 500 ÷1,500 × 20,000 = 6,667 units Again, Standard quantity Material B required for the standard output of 1,500 units = 400 units ∴ Standard quantity of Material B required for the actual output of 20,000 units (i.e., SQ × AO for Material B) = 400 ÷1,500 × 20,000 = 5,333 units Again, the standard quantity of Material C required for the standard output of 1,500 units = 700 units ∴ Standard quantity of Material C required for the actual output of 20,000 units (i.e., SQ × AO for Material C) = 700 ÷1,500 × 20,000 = 9,333 units 3. Standard mix/proportion of Material A for standard input of 1,600 units = 500 units ∴ Standard mix of Material A for actual input of 22,000 units = 500 ÷1,600 × 22,000 = 6,875 units Again, Standard proportion of Material B for standard input of 1,600 units = 400 units ∴ Standard mix of Material B for the actual input of 22,000 units = 400 ÷1,600 × 22,000 = 5,500 units Again, Standard proportion of Material C for standard input of 1,600 units = 700 units ∴ Standard mix of Material C for the actual input of 22,000 units = 700 ÷ 1,600 × 22,000 = 9,625 units (Continued)

Modified Date: Mon, Jul 05, 2010 05:03:14 PM

Output Date: Tue, Jul 06, 2010 12:21:13 PM

Rev II

Project: Management Accounting_Debarshi Bhattacharyya ACE Pro India Pvt. Ltd. File: X:\Pearson\Management Accounting_Debarshi Bhattacharyya\MAIN\M08\LAYOUT_M08\M08_DEBA_ISBN_EN_SE_C08.indd

STANDARD COSTING AND VARIANCE ANALYSIS

4. Standard Yield Rate =

565

Standard cost of material for standard output Standard output

= Rs. 11,500 ÷1,500 = Rs. 7.6667 Again, Standard yield (i.e., output) for the actual input of 1,600 units = 1,500 units ∴ Standard yield/output for actual input of 22,000 units =

1,500 × 22,000 = 20,625 units 1,600

Problem 10 From the following data, compute the Material Cost Variances: Standard Material A Material B

Qty (units) 500 1,000 1,500

Rate (Rs.) 10 5

Actual Amount (Rs.) 5,000 5,000 10,000

Qty (units) 600 1,200 1,800

Rate (Rs.) 8 6

Amount (Rs.) 4,800 7,200 12,000

Solution i. Material Cost Variance (MCV) = Standard Cost of material for actual output – Actual cost of material for actual output = 10,0001 – 12,000 = Rs. 2,000 (A) ii. Material Price Variance (MPV) = (SP – AP) AQ × AO Material A: (10 – 8) × 6002 = Rs. 1,200 (F) Material B: (5 – 6) × 1,2002 = Rs. 1,200 (A) Nil iii. Material Usage Variance (MUV) = SP [ SQ × AO – AQ × AO] Material A: 10 [5003 – 6002] = Rs. 1,000 (A) Material B: 5 [1,0003 – 1,2003] = Rs. 1,000 (A) Rs. 2,000 (A) iv. Material Mix Variance (MMV) = SP [Standard mix of material for actual input – Actual mix of material for actual input] Material A: 10 [(500 ÷ 1,500 × 1,800)4 − 600] = Nil Material B: 5 [(1,000 ÷ 1,500 × 1,800)4 − 1200] = Nil Nil v. Material Yield Variance (MYV) = Standard yield rate [Actual yield for actual input – Standard yield for actual input] As in this problem, the actual output is not given, the computation of MYV is not possible. In such a situation, MYV may be computed by considering the actual input as the actual output and the actual output as the standard output or without considering this ‘material usage/quantity other causes variance’ (MUOCV/ MQOCV) may be computed as follows: First Alternative If it is considered that: Actual input = Actual output

Modified Date: Mon, Jul 05, 2010 05:03:14 PM

Output Date: Tue, Jul 06, 2010 12:21:13 PM

Rev II

Project: Management Accounting_Debarshi Bhattacharyya ACE Pro India Pvt. Ltd. File: X:\Pearson\Management Accounting_Debarshi Bhattacharyya\MAIN\M08\LAYOUT_M08\M08_DEBA_ISBN_EN_SE_C08.indd

566

MANAGEMENT ACCOUNTING

Actual input = 1,800 units = Actual output Actual output = 1,800 units = Standard output Material Yield Variance (MYV) = Rs. 10,000 ÷ 1,800 [1,800 − (1,800 ÷ 1,500 × 1,800)] = 5.5555 [ 1,800 – 2,160] = Rs. 2,000 (A) Test i. MCV = MPV + MUV Here, Rs. 2,000 (A) = Nil + Rs. 2,000 (A) = Rs. 2,000 (A), checked ii. MUV = MMV + MYV Here, Rs. 2,000 (A) = Nil + Rs. 2,000 (A) = Rs. 2,000 (A), checked Then,

Second Alternative If it is considered that: Actual input = Actual output Then, ‘material usage other causes variance’ (MUOCV) = SP [Standard mix of material for standard input − Standard mix of material for actual input] Material A: 10 [500 − (500 ÷ 1,500 × 1,800)] = Rs. 1,000 (A) Material B: 5 [1,000 − (1,000 ÷ 1,500 × 1,800)] = Rs. 1,000 (A) Rs. 2,000 (A) Test In this case, MUV = MMV + MUOCV Here, Rs. 2,000 (A) = Nil + Rs. 2,000 (A) = Rs. 2,000 (A), checked Note: For examination purpose, a student may follow any one of the above two alternatives.

Working Notes 1. In this problem, the actual output is not given ∴ Standard Cost of material for actual output = Rs. 10,000 2. AQ × AO = Actual quantity of material used for actual output Here, AQ × AO for Material A = 600 units and AQ × AO for Material B = 1,200 units 3. SQ × AO = Standard quantity of materials required for actual output Here, SQ × AO for Material A = 500 units and SQ × AO for Material B = 1000 units [Standard data are given in respect of the actual output.] 4. Actual total input = 1,800 units Standard total input = 1,500 units Standard input of Material A = 500 units Standard input of Material B = 1000 units Standard proportion/mix of material for actual input = Total actual input × Here,

Standard input of a particular material Total standard input

Standard mix of Material A for actual input = 1,800 × (500 ÷1,500) = 600 units and Standard mix of Material B for actual input = 1,800 × (1,000 ÷1,500) = 1,200 units

Modified Date: Mon, Jul 05, 2010 05:03:14 PM

Output Date: Tue, Jul 06, 2010 12:21:13 PM

Rev II

Project: Management Accounting_Debarshi Bhattacharyya ACE Pro India Pvt. Ltd. File: X:\Pearson\Management Accounting_Debarshi Bhattacharyya\MAIN\M08\LAYOUT_M08\M08_DEBA_ISBN_EN_SE_C08.indd

STANDARD COSTING AND VARIANCE ANALYSIS

567

Problem 11 Determine the MMV from the following information: Standard Mix Raw Material

Qty (units) 100 200 300

A B C

Price per unit (Rs.) 5 6 4

Actual Mix Qty (units) 150 250 400

Price per unit (Rs.) 5.50 6.00 3.50

Due to a shortage of Material B, it was decided to reduce the consumption of B by 5% and increase that of A by 10%. [B.Com. (Hons), Calcutta University—2007] Solution Revised standard mix of materials: Material A: 100 units + 10% of 100 units Material B: 200 units – 5% of 200 units Material C:

= 110 units = 190 units = 300 units

In such a case, given information may be rearranged as follows: Standard Material A Material B Material C

Qty (units) 110 190 300 600

Rate (Rs.) 5.00 6.00 4.00

Actual Amount (Rs.) 550 1,140 1,200 2,890

Qty (units) 150 250 400 800

Rate (Rs.) 5.50 6.00 3.50

Amount (Rs.) 825 1,500 1,400 3,725

Here, it is to be assumed that: Actual input = Actual output = 800 units, and Standard output = Actual output = 800 units Material Mix Variance (MMV) = SR [Standard mix of materials for actual input – Actual mix of materials for actual input] ∴ Material A = 5 [(110 ÷ 600 × 800) − 150] = Rs. 15 (A) Material B = 6 [(190 ÷ 600 × 800) − 250] = Rs. 18 (F) Material C = 4 [(300 ÷ 600 × 800) − 400] = Nil MMV = Rs. 3 (F) Problem 12 The following information has been extracted from the records of a chemical company: Standard Price: Raw material A: Rs. 2 per kg Raw material B: Rs. 10 per kg Standard Mix: A: 75% and B: 25% (By weight) Standard Yield: 90% In a period, the actual costs, usage and output were as follows: Used: 2,200 kg of A, costing Rs. 4,650 800 kg of B, costing Rs. 7,850 Output: 2,850 kg of products Calculate Materials Cost Variances. [C.S. (Inter)—June 1994]

Modified Date: Mon, Jul 05, 2010 05:03:14 PM

Output Date: Tue, Jul 06, 2010 12:21:13 PM

Rev II

Project: Management Accounting_Debarshi Bhattacharyya ACE Pro India Pvt. Ltd. File: X:\Pearson\Management Accounting_Debarshi Bhattacharyya\MAIN\M08\LAYOUT_M08\M08_DEBA_ISBN_EN_SE_C08.indd

568

MANAGEMENT ACCOUNTING

Solution Firstly, standard and actual data are rearranged as follows: Standard

Less:

Material A (75% of 3,000 kg) Material B (25% of 3,000 kg) Input Loss (bal. fig.) Output (90% of 3,000 kg)

Qty (units) 2,250 750 3,000 300 2,700

Rate (Rs.) 2 10

Actual Amount (Rs.) 4,500 7,500 12,000 − 12,000

Qty (units) 2,200 800 3,000 150 2,850

Rate (Rs.) 2.1136 9.8125

Amount (Rs.) 4,650 7,850 12,500 − 12,500

Here, Actual Input is given (2,200 kg + 800 kg) = 3,000 kg. It is assumed that Standard input = Actual Input = 3,000 kg i. Material Cost Variance (MCV) = Standard Cost of material for actual output – Actual cost of material for actual output = (Rs. 12,000 × 2,850 ÷ 2,700) – Rs. 12,500 = Rs. 167 (F) ii. Material Price Variance (MPV) = (SP – AP) AQ × AO Material A = (2 – 2.1136) × 2,200 = Rs. 250 (A) Material B = (10 – 9.8125) × 800 = Rs. 150 (F) Rs. 100 (A) iii. Material Usage Variance (MUV) = SP [SQ × AO – AQ × AO] Material A = 2 [(2,250 × 2,850 ÷ 2,700) − 2,200] = Rs. 350 (F) Material B = 10 [(2,250 × 2,850 ÷ 2,700) − 800] = Rs. 83 (A) Rs. 267 (F) Test MCV = MPV + MUV Here, Rs. 167 (F) = Rs. 100 (A) + Rs. 267 (F) = Rs. 167 (F), checked iv. Material-Mix Variance (MMV) = SP [Standard mix of material for actual input – Actual mix of material for actual input] Material A = 2 [(2,250 × 3,000 ÷ 3,000) − 2,200] = Rs. 100 (F) Material B = 10 [(750 × 3,000 ÷ 3,000) − 800] = Rs. 500 (A) Rs. 400 (A) v. Material Yield Variance (MYV) = Standard yield rate [Actual yield for actual input – Standard yield for actual input] = (Rs. 12,000 ÷ 2,700 kg) [2,850 − (2,700 ÷ 3,000 × 3,000)] = 4.4444 [2,850 – 2,700] = Rs. 667 (F) Test MUV = MMV + MYV Here, Rs. 267 (F) = Rs. 400 (A) + Rs. 667 (F) = Rs. 267 (F), checked

Modified Date: Mon, Jul 05, 2010 05:03:14 PM

Output Date: Tue, Jul 06, 2010 12:21:13 PM

Rev II

Project: Management Accounting_Debarshi Bhattacharyya ACE Pro India Pvt. Ltd. File: X:\Pearson\Management Accounting_Debarshi Bhattacharyya\MAIN\M08\LAYOUT_M08\M08_DEBA_ISBN_EN_SE_C08.indd

STANDARD COSTING AND VARIANCE ANALYSIS

569

Problem 13 The standard mixture of Material X and Material Y for the production of Product Z is as follows: Material X: 40% @ Rs. 400 per kg. Material Y: 60% @ Rs. 600 per kg. A standard loss of 10% is normally anticipated in production. The following actual data are available for the month of January, 2010: 180 kg of Material X have been used @ Rs. 360 per kg. 220 kg of Material Y have been used @ Rs. 680 per kg. The actual production of Product Z during January 2010 was 369 kg. Calculate the Material Variances. Solution Here,

Actual production during the month = 369 kg. Standard loss = 10% of the input. Standard input for actual production = 100 ÷ 90 × 369 kg = 410 kg.

Now, the information has been rearranged as follows: Standard

Less:

Material X Material Y Input Loss Output

Qty (units) 164 246 410 41 369

Rate (Rs.) 400 600

Actual Amount (Rs.) 65,600 1,47,600 2,13,200 − 2,13,200

Qty (units) 180 220 400 31 369

Rate (Rs.) 360 680

Amount (Rs.) 64,800 1,49,600 2,14,400 − 2,14,400

Material Cost Variance (MCV) = Standard material cost of actual output – Actual material cost of actual output = Rs. 2,13,200 – Rs. 2,14,400 = Rs. 1,200 (A) Material Price Variance (MPV) = (SP – AP) AQ × AO Material X: (400 – 360) × 180 = Rs. 7,200 (F) Material Y: (600 – 680) × 220 = Rs. 17,600 (A) Rs. 10,400 (A) Material Usage Variance (MUV) = SP (SQ × AO – AQ × AO) Materials X: 400 [164 – 180] = Rs. 6,400 (A) Materials Y: 600 [246 – 220] = Rs 15,400 (F) Rs. 9,200 (F) Material Mix Variance (MMV) = SP [Standard mix for actual input – Actual mix for actual input] Material X: 400 [(164 ÷ 410 × 400) − 180] = Rs. 8,000 (A) Material Y: 600 [(246 ÷ 410 × 400) − 220] = Rs. 12,000 (F) Rs. 4,000 (F) Material Yield Variance (MYV) = Standard yield rate [Actual output for actual input – Standard output for actual input]

Modified Date: Mon, Jul 05, 2010 05:03:14 PM

Output Date: Tue, Jul 06, 2010 12:21:13 PM

Rev II

Project: Management Accounting_Debarshi Bhattacharyya ACE Pro India Pvt. Ltd. File: X:\Pearson\Management Accounting_Debarshi Bhattacharyya\MAIN\M08\LAYOUT_M08\M08_DEBA_ISBN_EN_SE_C08.indd

570

MANAGEMENT ACCOUNTING

= (Rs. 2,13,200 ÷ 369 kg) [369 − (369 ÷ 410 × 400)] = Rs. 5,200 (F) Test i. MCV = MPV + MUV Here, Rs. 1,200 (A) = Rs. 10,400 (A) + Rs. 9,200 (F) = Rs. 1,200 (F), checked ii. MUV = MMV + MYV Here, Rs. 9,200 (F) = Rs. 4,000 (F) + Rs. 5,200 (F) = Rs. 9,200 (F), checked Problem 14 The standard material cost for a normal mix of one tonne Chemical S is based on: Chemical P Q R

Usage (kg) 240 400 640

Price Per kg Rs. 6 12 10

Usage (tones) 1.6 2.4 4.5

Cost (Rs.) 11,200 30,000 47,250

During a month, 6.25 tonnes of Chemical S were produced from: Chemical P Q R

Calculate the Material Variances. Solution This problem has been arranged as follows: Standard

Less:

Chemical P Chemical Q Chemical R Input Loss Output

Qty (units) 240 400 640 1,280 280 1,000

Rate (Rs.) 6 12 10

Actual Amount (Rs.) 1,440 4,800 6,400 12,640 − 12,640

Qty (units) 1,600 2,400 4,500 8,500 2,250 6,250

Rate (Rs.) 7.00 12.50 10.50

Amount (Rs.) 11,200 30,000 47,250 88,450 − 88,450

1 Tonne = 1,000 kg. i. Material Cost Variance (MCV) = Standard Cost of actual output – Actual cost of actual output = (Rs. 12,640 × 6,250 ÷ 1,000) – Rs. 88,450 = Rs. 79,000 – Rs. 88,450 = Rs. 9,450 (A) ii. Material Price Variance (MPV) = (SP – AP) AQ × AO Chemical P = (6 – 7) × 1,600 = Rs. 1,600 (A) Chemical Q = (12 – 12.50) × 2,400 = Rs. 1,200 (A) Chemical R = (10 – 10.50) × 4,500 = Rs. 2,250 (A) Rs. 5,050 (A)

Modified Date: Mon, Jul 05, 2010 05:03:14 PM

Output Date: Tue, Jul 06, 2010 12:21:13 PM

Rev II

Project: Management Accounting_Debarshi Bhattacharyya ACE Pro India Pvt. Ltd. File: X:\Pearson\Management Accounting_Debarshi Bhattacharyya\MAIN\M08\LAYOUT_M08\M08_DEBA_ISBN_EN_SE_C08.indd

571

STANDARD COSTING AND VARIANCE ANALYSIS

iii. Material Usage Variance (MUV) = SP (SQ × AQ – AQ × AO) Chemical P = 6 [(240 × 6,250 ÷ 1,000) − 1,600] Chemical Q = 12 [(400 × 6,250 ÷ 1,000) − 2,400] Chemical R = 10 [(640 × 6,250 ÷ 1,000) − 4,500]

= Rs. 600 (A) = Rs. 1,200 (F) = Rs. 5,000 (A) Rs. 4,400 (A)

iv. Material Mix Variance (MMV) = SP [Standard mix for actual input – Actual mix for actual input] Chemical P = 6 [(240 × 8,500÷1,280) − 1,600] = Rs. 37.50 (A) Chemical Q = 12 [(400 × 8,500÷1,280) − 2,400] = Rs. 3,075 (F) Chemical R = 10 [(640 × 8,500÷1,280) − 4,500] = Rs. 2,500 (A) Rs. 537.50 (F) v. Material Yield Variance (MYV) = Standard yield rate [Actual yield for actual input − Standard yield for actual input] = (Rs. 12,640 ÷ 1,000) [6,250 − (1,000 ÷ 1,280 × 8,500)] = Rs. 4,937.50 (A) Test i. MCV = MPV + MUV Here, Rs. 9,450(A) = Rs. 5,050(A) + Rs. 4,400(A) = Rs. 9,450 (A), checked ii. MUV = MMV + MYV Here, Rs. 4,400 (A) = Rs. 537.50(F) + Rs. 4,937.50(A) = Rs. 4,400 (A), checked Problem 15 Modern Tiles Ltd makes plastic tiles of standard size of 6⬙ × 6⬙ × 1/8⬙. From the information given ahead, you are required to calculate: i. Total MCV ii. MPV iii. MUV iv. MMV v. MYV A standard mix of the compound required to produce an output of 20,000 sq. ft. of tiles of 1/8⬙ thickness is as follows: Quantity Kg 600 400 500

Direct Material A B C

Price per Kg Rs. 9.00 6.50 4.00

For the month of March 2008, the actual production of plastic tiles was 6,20,000 units. The actual data for the direct material consumed was as follows: Quantity Consumed Kg 5,000 2,900 4,400

Direct Material A B C

Price per Kg Rs. 8.50 6.00 4.50

[B.Com. (Hons), Delhi University—2009]

Modified Date: Mon, Jul 05, 2010 05:03:14 PM

Output Date: Tue, Jul 06, 2010 12:21:13 PM

Rev II

Project: Management Accounting_Debarshi Bhattacharyya ACE Pro India Pvt. Ltd. File: X:\Pearson\Management Accounting_Debarshi Bhattacharyya\MAIN\M08\LAYOUT_M08\M08_DEBA_ISBN_EN_SE_C08.indd

572

MANAGEMENT ACCOUNTING

Solution i. Material Cost Variance (MCV) = Standard material cost of actual output – Actual material cost of actual output = (Rs. 10,000 ÷ 80,000 × 6,20,000) – Rs. 79,700 = Rs. 77,500 – Rs. 79,700 = Rs. 2,200 (A) ii. Material Price Variance (MPV) = (SP – AP) AQ × AO Material A = (9 – 8.50) × 5,000 Material B = (6.50 – 6) × 2,900 Material C = (4 – 4.50) × 4,400

= Rs. 2,500 (F) = Rs. 1,450 (F) = Rs. 2,200 (A) Rs. 1,750 (F)

iii. Material Usage Variance (MUV) = SP (SQ × AQ – AQ × AO) Material A = 9 [(600 ÷ 80,000 × 6,20,000) − 5,000] Material B = 6.50 [(400 ÷ 80,000 × 6,20,000) − 2,900] Material C = 4 [(500 ÷ 80,000 × 6,20,000) − 4,400]

= Rs. 3,150 (A) = Rs. 1,300 (F) = Rs. 2,100 (A) Rs. 3,950 (A)

iv. Material Mix Variance (MMV) = SP [Standard mix for actual input – Actual mix for actual input] Material A = 9 [(600 ÷ 1,500 × 12,300) − 5,000] = Rs. 720 (A) Material B = 6.50 [(400 ÷1,500 × 12,300) − 2,900] = Rs. 2,470 (F) Material C = 4 [(500 ÷ 1,500 × 12,300) − 4,400] = Rs. 1,200 (A) Rs. 550 (F) v. Material Yield Variance (MYV) = Standard yield rate [Actual yield for actual input – Standard yield for actual input] = Re. 0.125 [6,20,000 − (80,000 ÷ 1,500 × 12,300)] = Rs. 4,500 (A) Test i. MCV = MPV + MUV Here, Rs. 2,200 (A) = Rs. 1,750 (F) + Rs. 3,950 (A) = Rs. 2,200 (A), checked ii. MUV = MMV + MYV Here, Rs. 3,950 (A) = Rs. 550 (F) + Rs. 4,500 (A) = Rs. 3,950 (A), checked

Working Notes Area of one unit of plastic tile of 1/8⬙ thickness = (6⬙ × 6⬙) ÷ (12 × 12) = 0.25 sq. ft ∴ Standard output of 20,000 sq. ft. of tiles of 1/8⬙ thickness contain = 80,000 units of tiles Here, Standard input = 600 kg + 400 kg + 500 kg = 1,500 kg (of Materials A, B and C) Standard output = 80,000 units Actual input = 5,000 kg + 2,900 kg + 4,400 kg = 12,300 kg (of Materials A, B and C) Actual output = 6,20,000 units Standard yield rate (i.e., Standard Cost per unit of standard output) = Rs. 10,000 ÷ 80,000 units = Re. 0.125

Modified Date: Mon, Jul 05, 2010 05:03:14 PM

Output Date: Tue, Jul 06, 2010 12:21:13 PM

Rev II

Project: Management Accounting_Debarshi Bhattacharyya ACE Pro India Pvt. Ltd. File: X:\Pearson\Management Accounting_Debarshi Bhattacharyya\MAIN\M08\LAYOUT_M08\M08_DEBA_ISBN_EN_SE_C08.indd

STANDARD COSTING AND VARIANCE ANALYSIS

573

B. Labour Variances Problem 16 From the following particulars, compute labour variances: Standard Rate of labour per hour Actual Rate of labour per hour Standard hours required to produce one unit of output Actual hours taken to produce one unit of output Actual Output

Rs. 50 Rs. 60 10 hrs 8 hrs 2,000 units

Solution i. Labour Cost Variance (LCV) = Standard labour cost for actual output – Actual labour cost for actual output = SR × SH × AO – AR × AH × AO = 50 × 10 × 2,000 – 60 × 8 × 2,000 = Rs. 40,000 (F) ii. Labour Rate Variance (LRV) = (SR – AR) AH × AO = (50 – 60) 8 × 2,000 = Rs. 1,60,000 (A) iii. Labour Efficiency Variance (LEV) = SR [SH × AO – AH × AO] = 50 [(10 × 2000) – (8 × 2,000)] = Rs. 2,00,000 (F) Test LCV = LRV + LEV [∴ Here, LITV is not applicable] Here, Rs. 40,000 (F) = Rs. 1,60,000 (A) + Rs. 2,00,000 (F) = Rs. 40,000 (F), checked Working Notes SR = Standard rate of labour per hour = Rs. 50 AR = Actual rate of labour per hour = Rs. 60 SH = Standard hours required to produce one unit of output = 10 hrs AH = Actual hours taken to produce one unit of output = 8 hrs AO = Actual output = 2,000 units ∴ SH × AO = Total standard hours required to produce the actual output = 10 hrs × 2,000 = 20,000 hrs and AH × AO = Total actual hours taken to produce the actual output = 8 hrs × 2,000 = 16,000 hrs

Here,

Problem 17 From the following information of Orient Manufacturing Co. Ltd, determine (i) LCV; (ii) LRV; and (iii) LEV: i. Standard labour cost per unit of production is Rs. 15. ii. Time allotted per unit is 30 hrs. iii. During the month of March 1996, 3,000 units are produced in 75,000 hrs. iv. Actual payment of wages for the month is Rs. 45,000. [B.Com. (Hons), Calcutta University—1996]

Modified Date: Mon, Jul 05, 2010 05:03:14 PM

Output Date: Tue, Jul 06, 2010 12:21:13 PM

Rev II

Project: Management Accounting_Debarshi Bhattacharyya ACE Pro India Pvt. Ltd. File: X:\Pearson\Management Accounting_Debarshi Bhattacharyya\MAIN\M08\LAYOUT_M08\M08_DEBA_ISBN_EN_SE_C08.indd

574

MANAGEMENT ACCOUNTING

Solution i. Labour Cost Variance (LCV) = Standard labour cost for actual output – Actual labour cost for actual output = SR × SH × AO – AR × AH × AO = (0.50 × 30 × 3,000)1 – 45,0002 = 45,000 – 45,000 = Nil ii. Labour Rate Variance (LRV) = (SR – AR) AH × AO = (0.501 – 0.602) × 75,0002) = Rs. 7,500 (A) iii. Labour Efficiency Variance (LEV) = SR [SH × AO – AH × AO] = 0.50 [(30 × 3000)2 – 75,0002] = Rs. 7,500 (F) Test LCV = LRV + LEV Here, Nil = Rs. 7,500 (A) + Rs. 7,500 (F) = Nil, checked Working Notes 1. Standard labour cost per unit of production = Rs. 15 Standard hours required to produce one unit of output (i.e., SH) = 30 hrs ∴ Standard labour rate for 30 hrs = Rs. 15 ∴ Standard labour rate per hour (i.e., SR) = Rs. 15 ÷ 30 = Re. 0.50 Again given, Actual output (i.e., AO) = 3,000 units ∴ Standard labour cost for actual output = SR × SH × AO = 0.50 × 30 × 3,000 = Rs. 45,000 2. Actual labour cost for actual output (i.e., AR × AH × AO) = Rs. 45,000 and Actual hours taken for actual output (i.e., AH × AO) = 75,000 hrs ∴ Actual labour rate per hour (i.e., AR) = Rs. 45,000 ÷ 75,000 hrs = Rs. 0.60 Again, Standard hours required for one unit of output (i.e., SH) = 30 hrs ∴ Standard hours required for actual output (i.e., SH × AO) = 30 × 3,000 = 90,000 hrs

Problem 18 Tridhara Ltd furnished the following information before you: Standard Wage rate per hour Actual Wages paid for actual production Actual hours taken for actual output (including abnormal idle time of 10 hrs) Actual Output Standard Output per hour

Re. 0.90 Rs. 76 80 hrs 1,640 units 20 units

Calculate the Labour Variances. Solution i. Labour Cost Variance (LCV) = Standard labour cost for actual output – Actual labour cost for actual output = SR × SH × AO – AR × AH × AO = [Re. 0.90 × (1 ÷ 20) × 1,640 units] − Rs. 76 = Rs. 2.20 (A)

Modified Date: Mon, Jul 05, 2010 05:03:14 PM

Output Date: Tue, Jul 06, 2010 12:21:13 PM

Rev II

Project: Management Accounting_Debarshi Bhattacharyya ACE Pro India Pvt. Ltd. File: X:\Pearson\Management Accounting_Debarshi Bhattacharyya\MAIN\M08\LAYOUT_M08\M08_DEBA_ISBN_EN_SE_C08.indd

STANDARD COSTING AND VARIANCE ANALYSIS

575

ii. Labour Rate Variance (LRV) = (SR – AR) AH × AO = (Re. 0.90 – Re. 0.95) × 80 hrs = Rs. 4 (A) iii. Labour Idle Time Variance (LITV) = SR × Abnormal idle hours = Re. 0.90 × 10 hrs = Rs. 9 (A) iv. Labour Efficiency Variance (LEV) = SR [SH × AO – AH × AO] = Re. 0.90 [(1÷20 × 1640) − (80 − 10)] = Rs. 10.80 (F) Test LCV = LRV + LITV + LEV Here, Rs. 2.20 (A) = Rs. 4(A) + Rs. 9(A) + Rs. 10.80 (F) = Rs. 2.20 (A), checked Working Notes 1. Standard output per hour = 20 units Standard hours required for one unit of output (SH) = 1 ÷ 20 hr 2. Actual labour rate per hour (AR) = Rs. 76 ÷ 80 hrs = Re. 0.95

Problem 19 From the following information, calculate the LCVs: Standard labour hour required for each unit of output Standard labour rate per hour Actual hours taken for actual output of 4,000 units (including 200 idle hours) Actual labour cost for 4,000 units of output

1.50 hrs Rs. 2 9,000 hrs Rs. 19,800

Solution i. Labour Cost Variance (LCV) = Standard labour cost for actual output – Actual labour cost for actual output = Rs. 12,0001 – Rs. 19,800 = Rs. 7,800 (A) ii. Labour Rate Variance (LRV) = (SR – AR) AH × AO = (2 – 2.23) × 9,000 = Rs. 1,800 (A) iii. Labour Idle Time Variance (LITV) = SR × Abnormal idle hours = 2 × 200 = Rs. 400 (A) [∴ LITV always be negative] iv. Labour Efficiency Variance (LEV) = SR [SH × AO – AH × AO] = 2[6,0004 – 8,8005] = Rs. 5,600 (A). Test LCV = LRV + LITV + LEV Here, Rs. 7,800 (A) = Rs. 1,800 (A) + Rs. 400 (A) + Rs. 5,600 (A) = Rs. 7,800 (A), checked

Modified Date: Mon, Jul 05, 2010 05:03:14 PM

Output Date: Tue, Jul 06, 2010 12:21:13 PM

Rev II

Project: Management Accounting_Debarshi Bhattacharyya ACE Pro India Pvt. Ltd. File: X:\Pearson\Management Accounting_Debarshi Bhattacharyya\MAIN\M08\LAYOUT_M08\M08_DEBA_ISBN_EN_SE_C08.indd

576

MANAGEMENT ACCOUNTING

Working Notes 1. Here, given: Standard labour hours required for each unit of output (i.e., SH) = 1.50 hrs Standard labour rate per hour (i.e., SR) = Rs. 2 Actual output (i.e., AO) = 4,000 units ∴ Standard labour cost for actual output = SR × SH × AO = 2 × 1.50 × 4,000 = Rs. 12,000 2. Again, given: Actual labour cost for actual output of 4,000 units = AR × AH × AO = Rs. 19,800 3. Actual hours taken for actual output of 4,000 units (i.e., AH × AO) = 9,000 hrs Again, Actual labour cost for actual output of 4,000 units (i.e., AR × AH × AO) = Rs. 19,800 ∴ Actual labour rate per hour (i.e., AR) = Rs. 19,800 ÷ 9,000 = Rs. 2.20 4. Standard hours required for the actual output of 4,000 units = SH × AO = 1.50 hrs × 4,000 = 6,000 hrs 5. Actual hours taken for actual output of 4,000 units (i.e., AH × AO) = 9,000 hrs This 9,000 hrs includes 200 idle hours. For ascertaining LEV, effective actual hours to be taken. ∴ For LEV, Effective AH × AO = 9,000 – 200 = 8,800 hrs

Problem 20 The following data are available from the records of XYZ Ltd, where Standard Costing is followed: Actual output in the month of April ’09 = 52,000 units Actual wages paid = Rs. 1,25,000 Number of days worked in April ’09 = 25 days Idle time paid and included in above = 1 day Number of workers = 100 Standard wage rate = Rs. 40 per day Standard daily output per worker = 20 units Calculate: i. LCV; ii. LRV; iii. LEV; and iv. LITV. [B.Com. (Hons), Calcutta University—2009] Solution i. Labour Cost Variance (LCV) = Standard labour cost for actual output – Actual labour cost for actual output = SR × SMD × AO – AR × AMD × AO = (Rs. 40 × 2,6001) – (Rs. 50 × 2,500)2 = Rs. 21,000 (A) ii. Labour Rate Variance (LRV) = (SR – AR) AMD × AO = (Rs. 40 – Rs. 50) × 2,500 = Rs. 25,000 (A) iii. Labour Idle Time Variance (LITV) = SR × Idle man-days = Rs. 40 × 100 = Rs. 4,000 (A)

Modified Date: Mon, Jul 05, 2010 05:03:14 PM

Output Date: Tue, Jul 06, 2010 12:21:13 PM

Rev II

Project: Management Accounting_Debarshi Bhattacharyya ACE Pro India Pvt. Ltd. File: X:\Pearson\Management Accounting_Debarshi Bhattacharyya\MAIN\M08\LAYOUT_M08\M08_DEBA_ISBN_EN_SE_C08.indd

STANDARD COSTING AND VARIANCE ANALYSIS

577

iv. Labour Efficiency Variance (LEV) = SR [SMD × AO – Effective AMD × AO] = Rs. 40 [2,600 − (2,500 − 100)] = Rs. 8,000 (F) Test LCV = LRV + LITV + LEV Here, Rs. 21,000 (A) = Rs. 25,000 (A) + Rs. 4,000 (A) + Rs. 8,000 (F) = Rs. 21,000 (A), checked Working Notes 1. Standard wage rate per worker per day (i.e., per man-day) (SR) = Rs. 40 Again, Actual output (AO) = 52,000 units and Standard daily output per worker = 20 units ∴ Standard man-days required to produce an actual output of 52,000 units (SMD × AO) = 52,000 ÷ 20 = 2,600 man-days 2. Actual man-days taken to produce an actual output of 52,000 units (AMD × AO) = 100 × 25 days = 2,500 man-days Again, Actual wages paid for an actual output of 52,000 units (AR × AMD × AO) = Rs. 1,25,000 ∴ Actual wage rate per worker per day (i.e., per man-day) (AR) = Rs. 1,25,000 ÷ 2,500 man-days = Rs. 50

Problem 21 Standard hours for producing two products A and B are 15 hours and 20 hours per unit, respectively. Both products require an identical type of labour and the standard wage rate is Rs. 5 per hour. In a year, 10,000 units of A and 15,000 units of B were produced. The total labour hours actually worked were 4,50,500 and actual wages bill came to Rs. 23,00,000. This included 12,000 hours paid for at Rs. 7 per hour and 9,400 hours paid for @ Rs. 7.50 per hour, the balance having been paid at Rs. 5 per hour. You are required to compute LCV, LRV and LEV. [B.Com. (Hons), Delhi University—2008] Solution i. Labour Cost Variance (LCV) = Standard labour cost for actual output – Actual labour cost for actual output = Rs. 22,50,0001 − Rs. 23,00,0002 = Rs. 50,000 (A) ii. Labour Rate Variance (LRV) = (SR – AR) AMD × AO = (Rs. 5 – Rs. 7) × 12,000 = Rs. 24,000 (A) = (Rs. 5 – Rs. 7.50) × 9,400 = Rs. 23,500 (A) = (Rs. 5 – Rs. 5) × 4,29,100 = Nil Rs. 47,500 (A) iii. Labour Efficiency Variance (LEV) = SR [SH × AO – AH × AO] = Rs. 5 [4,50,0003 − 4,50,5004] = Rs. 2,500 (F) Test LCV = LRV + LEV Here, Rs. 50,000 (A) = Rs. 47,500 (A) + Rs. 2,500 (A) = Rs. 50,000 (A), checked

Modified Date: Mon, Jul 05, 2010 05:03:14 PM

Output Date: Tue, Jul 06, 2010 12:21:13 PM

Rev II

Project: Management Accounting_Debarshi Bhattacharyya ACE Pro India Pvt. Ltd. File: X:\Pearson\Management Accounting_Debarshi Bhattacharyya\MAIN\M08\LAYOUT_M08\M08_DEBA_ISBN_EN_SE_C08.indd

578

MANAGEMENT ACCOUNTING

Working Notes 1. Standard labour cost for the actual output (SR × SH × AO): A: Rs. 5 × 15 hrs × 10,000 = Rs. 7,50,000 B: Rs. 5 × 20 hrs × 15,000 = Rs. 15,00,000 Rs. 22,50,000 2. Actual labour cost for actual output (AR × AH × AO) = Rs. 23,00,000 3. Standard labour hours required for actual output (SH × AO): A: 15 hrs × 10,000 B: 20 hrs × 15,000

= 1,50,000 hrs = 3,00,000 hrs 4,50,000 hrs

4. Actual labour hours taken for actual output (AH × AO) = 4,50,500 hrs

Problem 22 Standard labour hours and rates for production of each unit is given as follows: Hours 5 8 4 17

Skilled Worker Unskilled Worker Semi-skilled Worker

Rate Rs. 1.50 0.50 0.75

Total Rs. 7.50 4.00 3.00 14.50

Actual data for actual production of 1,000 units are given as follows: Skilled Worker Unskilled Worker Semi-skilled Worker

4,500 10,000 4,200 18,700

2.00 0.45 0.75

9,000 4,500 3,150 16,650

Calculate Labour Cost Variances. Solution i. Labour Cost Variance (LCV) = Standard labour cost for actual output – Actual labour cost for actual output = 14,5001 – 16,6501 = Rs. 2,150 (A) ii. Labour Rate Variance (LRV) = (SR – AR) AH × AO Skilled Worker = (1.50 – 2) × 4,5002 = Rs. 2,250 (A) Unskilled Worker = (0.50 – 0.45) × 10,0002 = Rs. 500 (F) Semi-skilled Worker = (0.75 – 0.75) × 4,2002 = Nil Rs. 1,750 (A) iii. Labour Efficiency Variance (LEV) = SR [SH × AO – AH × AO] Skilled Worker = 1.50 (5,0003 – 4,5002) = Rs. 750 (F) Unskilled Worker = 0.50 (8,0003 – 10,0002) = Rs. 1,000 (A) Semi-skilled Worker = 0.75 (4,0003 – 4,2002) = Rs. 150 (A) Rs. 400 (A)

Modified Date: Mon, Jul 05, 2010 05:03:14 PM

Output Date: Tue, Jul 06, 2010 12:21:13 PM

Rev II

Project: Management Accounting_Debarshi Bhattacharyya ACE Pro India Pvt. Ltd. File: X:\Pearson\Management Accounting_Debarshi Bhattacharyya\MAIN\M08\LAYOUT_M08\M08_DEBA_ISBN_EN_SE_C08.indd

STANDARD COSTING AND VARIANCE ANALYSIS

iv. Labour Mix Variance (LMV) = SR [RSH – (AH × AO)] Skilled Worker = 1.50 (5,5004 – 4,5002) Unskilled Worker = 0.50 (8,8004 – 10,0002) Semi-skilled Worker = 0.75 (4,4004 – 4,2002)

579

= Rs. 1,500 (F) = Rs. 600 (A) = Rs. 150 (F) Rs. 1.050 (F)

v. Labour Yield Variance (LYV) = SR [(SH × AO) – RSH] Skilled Worker = 1.50 (5,0003 – 5,5004) = Rs. 750 (A) Unskilled Worker = 0.50 (8,0003 – 8,8004) = Rs. 400 (A) Semi-skilled Worker = 0.75 (4,0003 – 4,4004) = Rs. 300 (A) Rs. 1,450 (A) Test i. LCV = LRV + LEV Here, Rs. 2,150 (A) = Rs. 1,750 (A) + Rs. 400 (A) = Rs. 2,150 (A), checked ii. LEV = LMV + LYV Here, Rs. 400 (A) = Rs. 1,050 (F) + Rs. 1,450 (A) = Rs. 400 (A), checked

Working Notes 1. Actual output (i.e., AO) = 1,000 units Standard labour cost for one unit of output = Rs. 14.50 ∴ Standard labour cost for actual output (i.e., SR × SH × AO) = 1,000 × 14.50 = Rs. 14,500 Again given, Actual labour cost for actual output of 1,000 units (i.e., AR × AH × AO) = Rs. 16,650 2. AH × AO = Actual labour hours required for actual output of 1,000 units Here, given: Hours 4,500 10,000 4,200

Skilled Worker Unskilled Worker Semi-skilled Worker

3. Standard hours required for one unit of output (i.e., SH): Hours 5 8 4

Skilled Worker Unskilled Worker Semi-skilled Worker

And, Actual output (i.e., AO) = 1,000 units ∴ SH × AO = Standard hours required for the actual output: Skilled Worker = 5 × 1,000 = 5,000 hrs Unskilled Worker = 8 × 1,000 = 8,000 hrs Semi-Skilled Worker = 4 × 1,000 = 4,000 hrs 4. Revised standard hours (RSH) = Total actual hours in standard ratio = Total actual hours ×

Standard hours for a labour Total standard hours

Skilled Worker = 18,700 × 5 ÷ 17 = 5,500 hrs Unskilled Worker = 18,700 × 8 ÷ 17 = 8,800 hrs Semi-skilled Worker = 18,700 × 4 ÷ 17 = 4,400 hrs

Modified Date: Mon, Jul 05, 2010 05:03:14 PM

Output Date: Tue, Jul 06, 2010 12:21:13 PM

Rev II

Project: Management Accounting_Debarshi Bhattacharyya ACE Pro India Pvt. Ltd. File: X:\Pearson\Management Accounting_Debarshi Bhattacharyya\MAIN\M08\LAYOUT_M08\M08_DEBA_ISBN_EN_SE_C08.indd

580

MANAGEMENT ACCOUNTING

Problem 23 From the following information, calculate the LMV: Standard Mix 100 skilled workers @ Rs. 300 per month 200 semi-skilled workers @ Rs. 200 per month

Actual Mix 110 skilled workers @ Rs. 350 per month 340 semi-skilled workers @ Rs. 225 per month

Due to a shortage of skilled workers, it was decided to reduce the number of skilled workers by 10% and increase that of semi-skilled workers by 5%.

Solution New standard Mix of Labour: Skilled Workers = 100 – 10% of 100 = 90 Semi-skilled Workers = 200 + 5% of 200 = 210 Under such a circumstance, the given information may be rearranged as follows: Skilled Workers Semi-skilled Workers

Standard 90 workers @ Rs. 300 p.m. = Rs. 27,000 210 workers @ Rs. 200 p.m. = Rs. 42,000 300

Actual 110 workers @ Rs. 350 p.m. = Rs. 38,500 340 workers @ Rs. 225 p.m. = Rs. 76,500 450

∴ Labour Mix Variance (LMV) = SR [Revised Standard Mix – Actual Mix ] Skilled Workers = Rs. 300 [(450 × 90 ÷ 300) − 110] = Rs. 7,500 (F) Semi-skilled Worker = Rs. 200 [(450 × 210 ÷ 300) − 340]= Rs. 5,000 (A) LMV = Rs. 2,500 (F) Note Revised Standard Mix = Total Actual Mix in Standard Ratio

Problem 24 Gajna Ltd operates Standard Costing and Budgetary Control system. The budgeted labour mix for 10,000 units of output is as follows: Skilled labour: 10,000 labour hours @ Rs. 2.50 per hr Unskilled labour: 10,000 labour hours @ Rs. 1.50 per hr During April 2010, a shortage of skilled labour is expected. Accordingly, the budgeted labour mix for the production of 10,000 units of output during April 2010 is revised as follows: 20% less employment of skilled labour than the original budget. 20% more employment of unskilled labour than the original budget. The actual results for April 2010 showed the following: Actual production: 12,000 units. Actual labour mix: Skilled labour − 9,200 hrs @ Rs. 2.80 per hr Unskilled labour − 15,800 hrs @ Rs. 1.30 per hr During April 2010, idle time for unskilled workers accounted for 200 hours due to power failure and machine breakdown. Calculate the Labour Variances.

Modified Date: Mon, Jul 05, 2010 05:03:14 PM

Output Date: Tue, Jul 06, 2010 12:21:13 PM

Rev II

Project: Management Accounting_Debarshi Bhattacharyya ACE Pro India Pvt. Ltd. File: X:\Pearson\Management Accounting_Debarshi Bhattacharyya\MAIN\M08\LAYOUT_M08\M08_DEBA_ISBN_EN_SE_C08.indd

STANDARD COSTING AND VARIANCE ANALYSIS

581

Solution The given problem has been rearranged in the following manner: For a standard output of 10,000 units: Skilled Labour (10,000 hrs – 20%) Unskilled Labour (10,000 hrs + 20%)

Hours 8,000 12,000 20,000

Rate Rs. 2.50 1.50

Amount Rs. 20,000 18,000 38,000

Hours 9,200 15,800 25,000

Rate Rs. 2.80 1.30

Amount Rs. 25,760 20,540 46,300

For an actual output of 12,000 units: Skilled Labour Unskilled Labour

i. Labour Cost Variance (LCV) = Standard labour cost for actual output – Actual labour cost for actual output = (Rs. 38,000 × 12,000 ÷ 10,000) – Rs. 46,300 = Rs. 45,600 – Rs. 46,300 = Rs. 700 (A) ii. Labour Rate Variance (LRV) = (SR – AR) AH × AO Skilled labour = (Rs. 2.50 – Rs. 2.80) × 9,200 = Rs. 2,760 (A) Unskilled labour = (Rs. 1.50 – Rs. 1.30) × 15,800 = Rs. 3,160 (F) Rs. 400 (F) iii. Labour Idle Time Variance (LITV) = SR × Abnormal idle hours Unskilled labour = Rs. 1.50 × 200 hrs = Rs. 300 (A) Note: LITV always be negative or adverse. iv. Labour Efficiency Variance (LEV) = SR [SH × AO – AH × AO] Skilled labour = Rs. 2.50 [(8,000 × 12,000 ÷ 10,000) − 9,200] = Rs. 1,000 (F) Unskilled labour = Rs. 1.50 [(12,000 × 12,000 ÷ 10,000) − (15,800 − 200)] = Rs. 1,800 (A) Rs. 800 (A) Note: While calculating LEV, Effective actual hours worked [i.e., Gross actual hours for actual output (AH × AO) – Abnormal idle hours] to be taken as actual hours for actual output. v. Labour Mix Variance (LMV) = SR [RSH – (AH × AO)] Skilled labour = Rs. 2.50 (10,000 – 9,200) = Rs. 2,000 (F) Unskilled labour = Rs. 1.50 (15,000 – 15,600) = Rs. 900 (A) Rs. 1,100 (F) Note: Here, AH × AO = Gross actual hours for AO – Abnormal idle hours vi. Labour-Yield Variance (LYV) = SR [(SH × AO) – RSH] Skilled labour = Rs. 2.50 [(8,000 × 12,000 ÷ 10,000) − 10,200] = Rs. 1,000 (A) Unskilled labour = Rs. 1.50 [(12,000 × 12,000÷10,000) − 15,000] = Rs. 900 (A) Rs. 1,900 (A)

Modified Date: Mon, Jul 05, 2010 05:03:14 PM

Output Date: Tue, Jul 06, 2010 12:21:13 PM

Rev II

Project: Management Accounting_Debarshi Bhattacharyya ACE Pro India Pvt. Ltd. File: X:\Pearson\Management Accounting_Debarshi Bhattacharyya\MAIN\M08\LAYOUT_M08\M08_DEBA_ISBN_EN_SE_C08.indd

582

MANAGEMENT ACCOUNTING

Test i. LCV = LRV + LITV + LEV ∴ Rs. 700(A) = Rs. 400(F) + Rs. 300(A) + Rs. 800(A) = Rs. 700(A), checked ii. LEV = LMV + LYV ∴ Rs. 800(A) = Rs. 1,100(F) + Rs. 1,900(A) = Rs. 800 (A), checked Working Notes Revised standard hours (RSH) = Total actual hours in standard ratio = Total actual hours ×

Standard hours for a labour Total standard hours

Skilled labour = 25,000 × 8,000 ÷ 20,000 = 10,000 hrs Unskilled labour = 25,000 × 12,000 ÷ 20,000 = 15,000 hrs

Problem 25 A gang of workers normally consists of 30 men, 15 women and 10 boys. They are paid at standard rates per hour as follows: Men—Re. 0.80; Women—Re. 0.60 and Boys—Re. 0.40. In a normal working week of 40 hours, the gang is expected to produce 2,000 units of output. During the week that ended on 31 December 2004, the gang that consisted of 40 men, 10 women and 5 boys were paid wages per hour @ Re. 0.70, Re. 0.65 and Re. 0.30, respectively. During the week, they produced 1,600 units and 4 hours were lost due to abnormal idle time. Calculate Labour Variances. Solution i. Labour Cost Variance (LCV) = Standard labour cost for actual output – Actual labour cost for actual output = SR × SH × AO – AR × AH × AO Men: Re. 0.80 × 960 hrs − Re. 0.70 × 1,600 hrs = Rs. 352 (A) Women: Re. 0.60 × 480 hrs − Re. 0.65 × 400 hrs = Rs. 28 (F) Boys: Re. 0.40 × 320 hrs − Re. 0.30 × 200 hrs = Rs. 68 (F) Rs. 256 (A) ii. Labour Rate Variance (LRV) = (SR – AR) AH × AO Men: (Re. 0.80 – Re. 0.70) × 1,600 hrs = Rs. 160 (F) Women: (Re. 0.60 – Re. 0.65) × 400 hrs = Rs. 20 (A) Boys: (Re. 0.40 – Re. 0.30) × 200 hrs = Rs. 20 (F) Rs. 160 (F) iii. Labour Idle Time Variance (LITV) = SR × Abnormal idle hours Men: Re. 0.80 × 160 hrs = Rs. 128 (A) Women: Re. 0.60 × 40 hrs = Rs. 24 (A) Boys: Re. 0.40 × 20 hrs = Rs. 8 (A) Rs. 160 (A)

Modified Date: Mon, Jul 05, 2010 05:03:14 PM

Output Date: Tue, Jul 06, 2010 12:21:13 PM

Rev II

Project: Management Accounting_Debarshi Bhattacharyya ACE Pro India Pvt. Ltd. File: X:\Pearson\Management Accounting_Debarshi Bhattacharyya\MAIN\M08\LAYOUT_M08\M08_DEBA_ISBN_EN_SE_C08.indd

STANDARD COSTING AND VARIANCE ANALYSIS

583

iv. Labour Efficiency Variance (LEV) = SR [(SH × AO) – (Effective AH × AO)] Men: Re. 0.80 [960 hrs – 1,440 hrs] = Rs. 384 (A) Women: Re. 0.60 [480 hrs – 360 hrs] = Rs. 72 (F) Boys: Re. 0.40 [320 hrs – 180 hrs] = Rs. 56 (F) Rs. 256 (A) v. Labour Mix Variance (LMV) = SR [RSH – (Effective AH × AO)] Men: Re. 0.80 [1080 hrs – 1,440 hrs] = Rs. 288 (A) Women: Re. 0.60 [540 hrs – 360 hrs] = Rs. 108 (F) Boys: Re. 0.40 [360 hrs – 180 hrs] = Rs. 72 (F) Rs. 108 (A) vi. Labour Yield Variance (LYV) = SR [(SH × AO) – RSH] Men: Re. 0.80 [960 hrs – 1,080 hrs] Women: Re. 0.60 [480 hrs – 540 hrs] Boys: Re. 0.40 [320 hrs – 360 hrs]

= Rs. 96 (A) = Rs. 36 (A) = Rs. 16 (A) Rs. 148 (A)

Test i. LCV = LRV + LITV + LEV Here, Rs. 256(A) = Rs. 160(F) + Rs. 160(A) + Rs. 256(A) = Rs. 256(A), checked ii. LEV = LMV + LYV Here, Rs. 256(A) = Rs. 108(A) + Rs. 148(A) = Rs. 256(A), checked

Working Notes 1. Standard labour hours required for an actual output of 1,600 units in a week (SH ¥ AO) Men: [(30 × 40 hrs) ÷ 2,000 units] × 1,600 units = 960 hrs Women: [(15 × 40 hrs) ÷ 2,000 units] × 1,600 units = 480 hrs Boys: [(10 × 40 hrs) ÷ 2,000 units] × 1,600 units = 320 hrs 2. Actual labour hours worked for actual output (AH ¥ AO) Men: 40 × 40 hrs = 1,600 hrs Women: 10 × 40 hrs = 400 hrs Boys: 5 × 40 hrs = 200 hrs 3. Idle hours in a week Men: 40 × 4 hrs = 160 hrs Women: 10 × 4 hrs = 40 hrs Boys: 5 × 4 hrs = 20 hrs 4. Effective actual hours worked for actual output (Effective AH ¥ AO) Men: 1,600 hrs – 160 hrs = 1,440 hrs Women: 400 hrs – 40 hrs = 360 hrs Boys: 200 hrs – 20 hrs = 180 hrs 1,980 hrs (Continued)

Modified Date: Mon, Jul 05, 2010 05:03:14 PM

Output Date: Tue, Jul 06, 2010 12:21:13 PM

Rev II

Project: Management Accounting_Debarshi Bhattacharyya ACE Pro India Pvt. Ltd. File: X:\Pearson\Management Accounting_Debarshi Bhattacharyya\MAIN\M08\LAYOUT_M08\M08_DEBA_ISBN_EN_SE_C08.indd

584

MANAGEMENT ACCOUNTING

5. Revised Standard Hours [i.e., Effective AH ¥ AO in standard ratio] (RSH) Men: 1,980 hrs ×

30 × 40 hrs = 1,080 hrs 30 + ( 15 + 10) × 40 hrs

Women: 1,980 hrs × Boys: 1,980 hrs ×

15 × 40hrs

(30 + 15 + 10) × 40 hrs 10 × 40hrs

(30 + 15 + 10) × 40 hrs

= 540 hrs

= 360 hrs

Problem 26 The standard labour and the actual labour employment in a week for a job are as follows: Standard number of workers in the gang Standard wage rate per hour (Rs.) Actual number of workers employed in the gang during the week Actual wage rate per hour (Rs.)

Skilled Workers 32 3 28

Semi-skilled Workers 12 2 18

Unskilled Workers 6 1 4

4

3

2

During the 40-hour working week, the gang produced 1,800 standard labour hours of work. Calculate the Labour Variances. [C.A. (Inter)—Adapted] Solution This problem may be rearranged as follows: Standard Hour No. of Per Workers Week Skilled Worker 32 40 Semi-skilled Worker 12 40 Unskilled Worker 6 40 Less:

Loss (bal. fig.)

Total Hours 1,280 480 240 2,000 – 2,000

Actual Rate Rs. 3 2 1

Amount Rs. 3,840 960 240 5,040 – 5,040

No. of Workers 28 18 4

Hour Per Total Rate Amount Week Hours Rs. Rs. 40 1,120 4 4,480 40 720 3 2,160 40 160 2 320 2,000 6,960 200 – 1,800 6,960

i. Labour Cost Variance (LCV) = Standard labour cost for actual hours worked – Actual labour cost for actual hours worked = (Rs. 5,040 × 1,800 ÷ 2,000) – Rs. 6,960 = Rs. 2,424 (A) ii. Labour Rate Variance (LRV) = (SR – AR) AH × AO. Skilled Worker = (Rs. 3 – Rs. 4) × 1,120 = Rs. 1,120 (A) Semi-skilled Worker = (Rs. 2 – Rs. 3) × 720 = Rs. 720 (A) Unskilled Worker = (Re. 1 – Rs. 2) × 160 = Rs. 160 (A) Rs. 2,000 (A)

Modified Date: Mon, Jul 05, 2010 05:03:14 PM

Output Date: Tue, Jul 06, 2010 12:21:13 PM

Rev II

Project: Management Accounting_Debarshi Bhattacharyya ACE Pro India Pvt. Ltd. File: X:\Pearson\Management Accounting_Debarshi Bhattacharyya\MAIN\M08\LAYOUT_M08\M08_DEBA_ISBN_EN_SE_C08.indd

STANDARD COSTING AND VARIANCE ANALYSIS

585

iii. Labour Efficiency Variance (LEV) = SR [SH × AO – AH × AO] Skilled Worker = Rs. 3 [(1,280 ÷ 2,000 × 1,800) − 1,120] = Rs. 96 (F) Semi-skilled Worker = Rs. 2 [(480 ÷ 2,000 × 1,800) − 720] = Rs. 576 (A) Unskilled Worker = Re. 1 [(240 ÷ 2,000 × 1,800) − 160] = Rs. 56 (F) Rs. 424 (A) iv. Labour Mix Variance (LMV) = SR [Standard labour hours for actual input hours – Actual labour hours for actual input hours] Skilled Worker = Rs. 3 [(1,280 ÷ 2,000 × 2,000) – 1,120] = Rs. 480 (F) Semi-skilled Worker = Rs. 2 [(480 ÷ 2,000 × 2,000) – 720] = Rs. 480 (A) Unskilled Worker = Re. 1 [(240 ÷ 2,000 × 2,000) − 160] = Rs. 80 (F) Rs. 80 (F) v. Labour Yield Variance (LYV) = Standard Yield Rate [Actual labour hours for actual input hours – Standard labour hours for actual input hours] = (Rs. 5,040 ÷ 2,000) [1,800 − (2,000 ÷ 2,000 × 2,000)] = Rs. 504 (A) Test (a) LCV = LRV + LEV Here, Rs. 2,424(A) = Rs. 2,000(A) + Rs. 424(A) = Rs. 2,424 (A), checked (b) LEV = LMV + LEV Here, Rs. 424(A) = Rs. 80(F) + Rs. 504(A) = Rs. 424(A), checked C. Material and Labour Variances combined: Problem 27 The following details are related to the Product A for the month of March 2000. You are required to compute MCVs and LCVs from the given details: Actual Production

100 Units

Standard Cost per unit: Material Labour Actual Cost for the month: Material Labour

50 kg @ Rs. 40 per kg 400 hrs @ Re. 1 per hr 4,900 kg @ Rs. 42 per kg 39,600 hrs @ Rs. 1.10 per hr

[B.Com. (Hons), Calcutta University—2002] Solution A. Material Variances i. Material Cost Variance (MCV) = Standard material cost for actual output − Actual material cost for actual output = (50 kg × Rs. 40 × 100 units) − (4,900 kg × Rs. 42) = Rs. 2,00,000 − Rs. 2,05,800 = Rs. 5,800 (A) ii. Material Price Variance (MPV) = (SP – AP) AQ × AO = (40 – 42) × 4,900 = Rs. 9,800 (A). [∴ Given actual quantity of material used for actual output (i.e., AQ × AO) = 4,900 kg]

Modified Date: Mon, Jul 05, 2010 05:03:14 PM

Output Date: Tue, Jul 06, 2010 12:21:13 PM

Rev II

Project: Management Accounting_Debarshi Bhattacharyya ACE Pro India Pvt. Ltd. File: X:\Pearson\Management Accounting_Debarshi Bhattacharyya\MAIN\M08\LAYOUT_M08\M08_DEBA_ISBN_EN_SE_C08.indd

586

MANAGEMENT ACCOUNTING

iii. Material Quantity Variance (MQV) = SR [ SQ × AO – AQ × AO] = 40 [(50 kg × 100 units) – 4,900] = Rs. 4,000 (F). [∴ Given SQ = 50 kg and AO = 100 units ∴ SQ × AO = 50 × 100 = 5,000 kg] Test MCV = MPV + MQV Here, Rs. 5,800 (A) = Rs. 9,800 (A) + Rs. 4,000 (F) = Rs. 5,800 (A), checked B. Labour Variances i. Labour Cost Variance (LCV) = Standard labour cost for actual output – Actual labour cost for actual output = (400 hrs × Re. 1 × 100 units) – (39,600 hrs × Rs. 1.10) = Rs. 40,000 – Rs. 43,560 = Rs. 3,560 (A) ii. Labour-Rate Variance (LRV) = (SR – AR) AH × AO = (1 – 1.10) × 39,600 = Rs. 3,960 (A) [∴ Given actual labour hours taken for actual output (i.e., AH × AO) = 39,600 hours] iii. Labour-Efficiency Variance (LEV) = SR [SH × AO – AH × AO] = 1 [400 hrs × 100 units) – 39,600] [∴ Given SH = 400 hrs and AO = 100 units] = Rs. 400 (F) Test LCV = LRV + LEV Here, as 3,560 (A) = Rs. 3,960 (A) + Rs. 400 (F) = Rs. 3,560 (A), checked Note As there is no abnormal idle hours, LITV need not be computed in this case. Problem 28 From the following particulars, calculate MCVs and LCVs:

Standard Cost per unit: Materials Labour

Qty/Hours

Rate Rs.

Amount Rs.

10 kg 20 hrs

10 4

100 80

For the production of 2,000 units: Material consumed: 24,000 kg @ Rs. 9 per kg. Labour hours taken: 38,200 hrs @ Rs. 5 per hour, including 200 idle hours due to machine break down. Solution A. Material Cost Variances: i. Material Cost Variance (MCV) = Standard material cost for actual output – Actual material cost for actual output = SP × SQ × AO – AP × AQ × AO

Modified Date: Mon, Jul 05, 2010 05:03:14 PM

Output Date: Tue, Jul 06, 2010 12:21:13 PM

Rev II

Project: Management Accounting_Debarshi Bhattacharyya ACE Pro India Pvt. Ltd. File: X:\Pearson\Management Accounting_Debarshi Bhattacharyya\MAIN\M08\LAYOUT_M08\M08_DEBA_ISBN_EN_SE_C08.indd

STANDARD COSTING AND VARIANCE ANALYSIS

587

= (Rs. 10 × 10 kg × 2,000 units) – (Rs. 9 × 24,000 kg) = Rs. 2,00,000 – Rs. 2,16,000 = Rs. 16,000 (A) ii. Material Price Variance (MPV) = (SP – AP) AQ × AO = (10 – 9) × 24,000 = Rs. 24,000 (F) iii. Material Quantity Variance (MQV) = SP [SQ × AO – AQ × AO] = 10 [(10 kg × 2,000 units) – 24,000 kg] = Rs. 40,000 (A) Test MCV = MPV + MQV Here, Rs. 16,000 (A) = Rs. 24,000 (F) + Rs. 40,000 (A) = Rs. 16,000 (A), checked B. Labour Cost Variances: i. Labour Cost Variance (LCV) = Standard labour cost for actual output –Actual labour cost for actual output = SR × SH × AO – AR × AH × AO = (Rs. 4 × 20 hrs × 2,000 units) – (Rs. 5 × 38,200 hrs) = Rs. 1,60,000 – Rs. 1,91,000 = Rs. 31,000 (A) ii. Labour-Rate Variance (LRV) = (SR – AR) AH × AO = (4 – 5) × 38,200 = Rs. 38,200 (A) iii. Labour Idle Time Variance (LITV) = SR × Abnormal idle hours = 4 × 200 = Rs. 800 (A) [∴ LITV always be negative] iv. Labour Efficiency Variance (LEV) = SR [SH × AO – AH × AO] = 4 [(20 hrs × 2,000 units) – (38,200 – 200)] = 4 (40,000 – 38,000) = Rs. 8,000 (F) [∴ For ascertaining LEV, actual effective hours to be considered. Therefore, in this case, AH × AO = Actual hours for AO – Idle hours] Test LCV = LRV + LITV + LEV Here, Rs. 31,000 (A) = Rs. 38,200 (A) + Rs. 800 (A) + Rs. 8,000 (F) = Rs. 31,000 (A), checked Working Notes Here, Standard price of material per kg = SP = Rs. 10 Standard quantity of materials required for one unit of output = SQ = 10 kg Actual output = AO = 2,000 units ∴ Standard quantity of material required for actual output = SQ × AO = 10 kg × 2,000 units = 20,000 kg Actual quantity of materials used for actual output = AQ × AO = 24,000 kg Actual price of material per kg = AP = Rs. 9 Again, Standard hours required for one unit of output = SH = 20 hrs (Continued)

Modified Date: Mon, Jul 05, 2010 05:03:14 PM

Output Date: Tue, Jul 06, 2010 12:21:13 PM

Rev II

Project: Management Accounting_Debarshi Bhattacharyya ACE Pro India Pvt. Ltd. File: X:\Pearson\Management Accounting_Debarshi Bhattacharyya\MAIN\M08\LAYOUT_M08\M08_DEBA_ISBN_EN_SE_C08.indd

588

MANAGEMENT ACCOUNTING

∴ Standard hours required for actual output = SH × AO = 20 hrs × 2,000 units = 4,000 hrs Actual hours taken for actual output = AH × AO = 38,200 hours (including idle hours) Standard labour rate per hour = SR = Rs. 4 Actual labour rate per hour = AR = Rs. 5

Problem 29 From the following records of a company, calculate MCVs and LCVs: 1 tonne of material input yields a standard output of 1 lakh units. No. of employees is 200. Standard wage rate per employee per day is Rs. 6. Standard price of materials is Rs. 20 per kg. Actual quantity of materials issued by production department is 10 tonnes. Actual price of materials is Rs. 21 per kg. Actual output is 9 lakh units. Actual wage rate per employee per day is Rs. 6.50. Standard daily output per employee is 100 units. Total number of days worked is 50. Idle time paid for and included above is half day. [C.A. (Inter)—Adapted] Solution Standard price of materials per kg = SP = Rs. 20 Actual price of materials per kg = AP = Rs. 21 Standard quantity of materials required to produce one unit of output = SQ = 1 tonne ÷ 1,00,000 units = 1,000 kg ÷ 1,00,000 units = 0.01 kg Actual quantity of materials used to produce actual output = AQ × AO = 10 tonnes = 10 × 1,000 kg = 10,000 kg Actual output = AO = 9,00,000 units Standard wage rate per employee per day = SR = Rs. 6 Standard daily output per employee = 100 units Total number of days worked = 50 ∴ Actual daily output = AO ÷ 50 = 9,00,000 ÷ 50 = 18,000 units No. of employees = 200 ∴ Standard man-days required to produce actual output = SMD × AO = 9,00,000 ÷ 100 = 9,000 man-days Actual man-days taken to produce actual output = AMD × AO = 200 × 50 = 10,000 man-days Actual wage rate per employee per day = AR = Rs. 6.50 1 Idle time man-days = × 200 workers = 100 man-days 2 A. Material Variances: i. Material Cost Variance (MCV) = SP × SQ × AO – AP × AQ × AO = (20 × 0.01 × 9,00,000) – (21 × 10,000) = Rs. 30,000 (A)

Modified Date: Mon, Jul 05, 2010 05:03:14 PM

Output Date: Tue, Jul 06, 2010 12:21:13 PM

Rev II

Project: Management Accounting_Debarshi Bhattacharyya ACE Pro India Pvt. Ltd. File: X:\Pearson\Management Accounting_Debarshi Bhattacharyya\MAIN\M08\LAYOUT_M08\M08_DEBA_ISBN_EN_SE_C08.indd

STANDARD COSTING AND VARIANCE ANALYSIS

589

ii. Material Price Variance (MPV) = (SP – AP) AQ × AO = (20 – 21) × 10,000 = Rs. 10,000 (A) iii. Material Usage Variance (MUV) = SP [SQ × AO – AQ × AO] = 20 [(0.01 × 9,00,000) – 10,000] = Rs. 20,000 (A) Test MCV = MPV + MUV Here, Rs. 30,000 (A) = Rs. 10,000 (A) + Rs. 20,000 (A) = Rs. 30,000 (A), checked B. Labour Variances: i. Labour Cost Variance (LCV) = SR × SMD × AO – AR × AMD × AO = (6 × 9,000) – (6.50 × 10,000) = Rs. 11,000 (A) ii. Labour Rate Variance (LRV) = (SR – AR) AMD × AO = (6 – 6.50) × 10,000 = Rs. 5,000 (A) iii. Labour Idle Time Variance (LITV) = SR × Idle man-days = 6 × 100 = Rs. 600 (A) iv. Labour Efficiency Variance (LEV) = SR [SMD × AO – Effective AMD × AO] = 6 [9,000 – (10,000 – 100)] = Rs. 5,400 (A) Test LCV = LRV + LITV + LEV Here, Rs. 11,000 (A) = Rs. 5,000 (A) + Rs. 600 (A) + Rs. 5,400 (A) = Rs. 11,000 (A), checked Problem 30 From the following information, calculate MCVs and LCVs: Standard Direct Cost per unit Material (5 kg @ Rs. 10 per kg) Labour (20 hrs @ Re. 0.50 per hr)

Rs. 50 10 60

For production of 500 units, materials consumption amounted to Rs. 25,650 against 2.7 metric tonne. Wage payment was Rs. 6,050 for 11,000 hours including 20 hours idle time due to machine break down. Solution Standard Cost per unit of output: Material: 5 kg @ Rs. 10 per kg = Rs. 50 Labour: 10 hrs @ Re. 0.50 per hr = Rs. 10

Modified Date: Mon, Jul 05, 2010 05:03:14 PM

Output Date: Tue, Jul 06, 2010 12:21:13 PM

Rev II

Project: Management Accounting_Debarshi Bhattacharyya ACE Pro India Pvt. Ltd. File: X:\Pearson\Management Accounting_Debarshi Bhattacharyya\MAIN\M08\LAYOUT_M08\M08_DEBA_ISBN_EN_SE_C08.indd

590

MANAGEMENT ACCOUNTING

Actual cost for actual output of 500 units: Material: 2,700 kg @ Rs. 9.50 per kg = Rs. 25,650 Labour: 11,000 hrs @ Re. 0.55 per hr = Rs. 6,050 Again, 1 M.T. = 1,000 kg ∴ 2.7 M.T. = 2700 kg ∴ Actual price of material per kg = Rs. 25,650 ÷ 2,700 kg = Rs. 9.50 ∴ Actual rate of labour per hour = Rs. 6,050 ÷ 11,000 hrs = Re. 0.55 A. Material Variances: i. Material Cost Variance (MCV) = Standard Cost of material for actual output − Actual cost of material for actual output = (500 units × Rs. 50) – Rs. 25,650 = Rs. 650(A) or = SP × SQ × AO – AP × AQ × AO = (Rs. 10 × 5 kg × 500 units) – (Rs. 9.50 × 2,700 kg) = Rs. 650 (A) ii. Material Price Variance (MPV) = (SP – AP) AQ = (Rs. 10 – Rs. 9.50) × 2,700 kg = Rs. 1,350 (F) iii. Material Usage Variance (MUV) = SP [SQ × AO – AQ × AO] = Rs. 10 [(5 kg × 500 units) – 2,700 kg] = Rs. 2,000 (A) Test MCV = MPV + MUV Here, Rs. 650(A) = Rs. 1,350(F) + Rs. 2,000 (A) = Rs. 650(A), checked B. Labour Variances: i. Labour Cost Variance (LCV) = Standard labour cost for actual output – Actual labour cost for actual output = (Rs. 10 × 500 units) – Rs. 6,050 = Rs. 1.050 (A) or = SR × SH × AO – AR × AH × AO = (Re. 0.50 × 20 hrs × 5 units) – (Re. 0.55 × 11,000 hrs) = Rs. 1,050 (A) ii. Labour Rate Variance (LRV) = (SR – AR) AH × AO = (Re. 0.50 – Re. 0.55) × 11,000 hrs = Rs. 550 (A) iii. Labour Idle Time Variance (LITV) = SR × Abnormal idle hours = Re. 0.50 × 20 hrs = Rs. 10(A) iv. Labour Efficiency Variance (LEV) = SR [SH × AO – Effective AH × AO] = Re. 0.50 [(20 hrs × 500 units) – (11.000 hrs – 20 hrs)] = Rs. 490 (A)

Modified Date: Mon, Jul 05, 2010 05:03:14 PM

Output Date: Tue, Jul 06, 2010 12:21:13 PM

Rev II

Project: Management Accounting_Debarshi Bhattacharyya ACE Pro India Pvt. Ltd. File: X:\Pearson\Management Accounting_Debarshi Bhattacharyya\MAIN\M08\LAYOUT_M08\M08_DEBA_ISBN_EN_SE_C08.indd

STANDARD COSTING AND VARIANCE ANALYSIS

591

Test LCV = LRV + LITV + LEV Here, Rs. 1,050(A) = Rs. 550(A) + Rs. 10(A) + Rs. 490(A) = Rs. 1,050(A), checked Problem 31 Following information are given for a certain period of a company: Output Wages Paid Materials Consumed

6,250 units Rs. 33,680 for 10,400 hrs 3850 kg for Rs. 17,059

The Variance Analysis revealed the following variances: Wage rate variance MUV LEV MPV

Rs. 1,750 (adverse) Rs. 890 (favourable) Rs. 570 (favourable) Rs. 1,400 (favourable)

Calculate the prime cost per unit. Solution Standard Labour Cost per unit:

Less: Add:

Actual Wages Paid Adverse Wage Rate Variance Favourable LEV Standard Labour Cost for 6,250 units Standard Labour Cost per unit (Rs. 32,500 ÷ 6,250)

Rs. 33,680 1,750 31,930 570 32,500 Rs. 5.20

Standard Material Cost per unit:

Add: Add:

Actual consumption of materials MUV (F) MPV (F) Standard Material Cost for 6,250 units Standard Material Cost per unit (Rs. 19,349 ÷ 6,250)

Rs. 17,059 890 1,400 19,349 Rs. 3.10

Computation of Prime Cost per unit: Rs. 3.10 5.20 8.30

Material Cost per unit Labour Cost per unit Prime Cost per unit

D. Overhead Variances: Problem 32 A factory produces two products P and Q. P takes 10 hours to produce and Q requires 16 hours as per the budget. Budgeted production for the month is 600 units of P and 250 units of Q. A month has 25 budgeted days of 8 hours each. During the month, 500 units of P and 400 units of Q were produced. The factory employs 50 workers. They actually worked for 9 hours daily for 24 days. Calculate: (i) Efficiency ratio; (ii) Activity ratio; (iii) Capacity ratio; and (iv) Calendar ratio. [B.Com. (Hons), Delhi University—2007 (External)—Adapted]

Modified Date: Mon, Jul 05, 2010 05:03:14 PM

Output Date: Tue, Jul 06, 2010 12:21:13 PM

Rev II

Project: Management Accounting_Debarshi Bhattacharyya ACE Pro India Pvt. Ltd. File: X:\Pearson\Management Accounting_Debarshi Bhattacharyya\MAIN\M08\LAYOUT_M08\M08_DEBA_ISBN_EN_SE_C08.indd

592

MANAGEMENT ACCOUNTING

Solution Budgeted days for actual production Actual days for actual production Budgeted hours for budgeted production: 50 × 25 days × 8 hrs or (600 units × 10 hrs) + (250 units × 16 hrs) Actual hours for actual production = 50 × 24 days × 9 hrs Standard hours for actual production: Product P = 500 × 10 hrs Product Q = 400 × 16 hrs i. Efficiency Ratio =

= 25 days = 24 days = 10,000 hrs = 10,000 hrs = 10,800 hrs = 5,000 hrs = 6,400 hrs = 11,400 hrs

Standard hours for actual production

Actual hours for actual production 11,400 = × 100 = 105.55% 10,800

ii. Activity Ratio =

× 100

Standard hours for actual production

Budgeted hours for budgeted production 11,400 = × 100 = 114% 10,000

iii. Capacity Ratio =

× 100

Actual hours for actual production

Budgeted hours for budgeted production 10,800 = × 100 = 108% 10,000

iv. Calendar Ratio =

Actual days for actual production

Budgeted days for actual production 24 = × 100 = 96% 25

× 100

× 100

Problem 33 Normal Capacity Budged Variable Production Overhead Actual Production Variable Production Overhead Incurred Actual direct Labour hours worked Labour hours required to produce one unit of output

9,600 labour hours Rs. 3 per labour hour 5,000 units Rs. 28,900 9,300 labour hours 2 labour hours

Calculate Variable Overhead Variances. Solution i. Variable Overhead Cost Variance (VOCV) = SR × SH × AO − AR × AH × AO = (3 × 2 × 5,000) – 28,900 = Rs. 1,100 (F) ii. Variable Overhead Expenditure Variance (VOExp.V) = (SR – AR) AH × AO = (3 – 3.1075) × 9,300 = Rs. 1,000 (A)

Modified Date: Mon, Jul 05, 2010 05:03:14 PM

Output Date: Tue, Jul 06, 2010 12:21:13 PM

Rev II

Project: Management Accounting_Debarshi Bhattacharyya ACE Pro India Pvt. Ltd. File: X:\Pearson\Management Accounting_Debarshi Bhattacharyya\MAIN\M08\LAYOUT_M08\M08_DEBA_ISBN_EN_SE_C08.indd

593

STANDARD COSTING AND VARIANCE ANALYSIS

iii. Variable Overhead Efficiency Variance (VOEff.V) = SR (SH × AO – AH × AO) = 3 [(2 × 5,000) – 9,300] = Rs. 2,100 (F) Test VOCV = VOExp.V + VOEff.V Here, Rs. 1,100 (F) = Rs, 1,000 (A) + Rs, 2,100 (F) = Rs. 1,100 (F), checked Working Notes Standard Variable Overhead Rate per hour (SR) = Rs. 3 Actual variable overhead Actual Variable Overhead Rate per hour (AR ) = Actual labour hours worked Rs. 28,900 = = Rs. 3.1075 9,300 Standard labour hours required to produce one unit of output (SH) = 2 labour hours Actual labour hours worked for actual output (AH × AO) = 9,300 labour hours Actual output (AO) = 5,000 units Actual variable overhead incurred for actual output (AR × AH × AO) = Rs. 28,900

Problem 34 Budget Activity Actual Activity Actual Production Actual Variable Overhead Budget of Variable Overhead (for each 5% variation in activity)

5,000 units 80% 3,750 units Rs. 85,000 Rs. 5,000

Calculate Variable Overhead Variances. Solution i. Variable Overhead Cost Variance (VOCV) = SR × AO – AR × AO = (Rs. 20 × 3750) – Rs. 85,000 = Rs. 10,000 (A) ii. Variable Overhead Expenditure Variance (VOExp.V) = SR × SO – AR × AO = (Rs. 20 × 4,000) – (Rs. 22.6667 × 3,750) = Rs. 5,000 (A) iii. Variable Overhead Efficiency Variance (VOEff.V) = SR × AO – SR × SO = (Rs. 20 × 3,750) – (Rs. 20 × 4,000) = Rs. 5,000 (A) Test VOCV = VOExp.V + VOEff.V Here, Rs. 10,000 (A) = Rs. 5,000 (A) + Rs. 5,000 (A) = Rs. 10,000 (A), checked  Working Notes Standard output (SO) = 80% of 5,000 = 4,000 units Actual output (AO) = 3,750 units Budgeted variable overhead at 100% capacity = 100 ÷ 5 × Rs. 5,000 = Rs. 1,00,000 (Continued)

Modified Date: Mon, Jul 05, 2010 05:03:14 PM

Output Date: Tue, Jul 06, 2010 12:21:13 PM

Rev II

Project: Management Accounting_Debarshi Bhattacharyya ACE Pro India Pvt. Ltd. File: X:\Pearson\Management Accounting_Debarshi Bhattacharyya\MAIN\M08\LAYOUT_M08\M08_DEBA_ISBN_EN_SE_C08.indd

594

MANAGEMENT ACCOUNTING

Standard variable overhead rate per unit (SR) Budgeted variable overhead at 100% capacity Rs. 1,00,000 = = Rs. 20 Budgeted output at 100% capacity 5,000 Actual variable overhead incurred Actual variable overheadrate per unit (AR ) = Actual output Rs. 85,000 = = Rs. 22.6667 3,750 =

Actual variable overhead for actual output (AR × AO) = Rs. 85,000

Problem 35 Standard 30,000 30,000 45,000 25

Output (units) Hours Worked (hours) Fixed Overhead (Rs.) Working Days

Actual 32,500 33,000 50,000 26

Calculate Fixed Overhead Variances. Solution Budgeted Output (BO) = 30,000 units Actual Output (AO) = 32,500 units Standard Output (SO) = Budgeted output for actual days = (30,000 units ÷ 25 days) × 26 days = 31,200 units Budgeted fixed overhead Budgeted output Rs. 45,000 = = Rs. 1.50 30,000 Budgeted fixed overhead Standard Fixed Overhead Rate per day (SRD) = Budgeted days Rs. 45,000 = = Rs. 1,800 25

Standard Fixed Overhead Rate per unit (SRU ) =

Actual fixed overhead Actual output Rs. 50,000 = = Rs. 1.53846 32,500

Actual Fixed Overhead Rate per unit ( ARU ) =

i. Fixed Overhead Cost Variance (FOCV) = Standard fixed overhead for actual output – Actual fixed overhead for actual output = SRU × AO – ARU × AO = Rs. 1.50 × 32,500 – Rs. 1.53846 × 32,500 = Rs. 1,250 (A) ii. Fixed Overhead Expenditure Variance (FOExp.V) = Budgeted fixed overhead for budgeted output − Actual fixed overhead for actual output = SRU × BO – ARU × AO = Rs. 1.50 × 30,000 – Rs. 1.53846 × 32,500 = Rs. 5,000 (A)

Modified Date: Mon, Jul 05, 2010 05:03:14 PM

Output Date: Tue, Jul 06, 2010 12:21:13 PM

Rev II

Project: Management Accounting_Debarshi Bhattacharyya ACE Pro India Pvt. Ltd. File: X:\Pearson\Management Accounting_Debarshi Bhattacharyya\MAIN\M08\LAYOUT_M08\M08_DEBA_ISBN_EN_SE_C08.indd

STANDARD COSTING AND VARIANCE ANALYSIS

595

iii. Fixed Overhead Volume Variance (FOVol.V) = Standard fixed overhead for actual output − Budgeted fixed overhead for budgeted output = SRU × AO – SRU × BO = Rs. 1.50 × 32,500 – Rs. 1.50 × 30,000 = Rs. 3,750 (F) iv. Fixed Overhead Efficiency Variance (FOEff.V) = Standard fixed overhead for actual output − Standard fixed overhead for standard output = SRU × AO – SRU × SO = Rs. 1.50 × 32,500 – Rs. 1.50 × 31,200 = Rs. 1,950 (F) v. Fixed Overhead Capacity Variance (FOCap.V) = Standard fixed overhead for standard output – Budgeted fixed overhead for budgeted output = SRU × SO − SRU × BO = Rs. 1.50 × 31,200 – Rs. 1.50 × 30,000 = Rs. 1,800 (F) or Fixed Overhead Calendar Variance (FOCal.V) = Standard fixed overhead for actual days − Budgeted fixed overhead for budgeted days = SRD × Actual days − SRD × Budgeted days = Rs. 1,800 × 26 – Rs. 1,800 × 25 = Rs. 1,800 (F) Test vi. FOCV = FOExp.V + FOVol.V Here, Rs. 1,250 (A) = Rs. 5,000 (A) + Rs. 3,750 (F) = Rs. 1,250 (A), checked vii. FOVol.V = FOEff.V + FOCap.V or FOCal.V Here, Rs. 3,750 (F) = Rs. 1,950 (F) + Rs. 1,800 (F) = Rs. 3.750 (F), checked Alternative Solution (on the basis of hours) The alternative solution is done on the basis of hours instead of units as done earlier. Budgeted fixed overhead Budgeted hours Rs. 45,000 = = Rs. 1.50 30,000

Standard Fixed Overhead Rate per hour (SRH ) =

Budgeted hours Budgeted output 30,000 hrs = = 1 hr 30,000

Standard hours required per unit of output (SH) =

Actual fixed overhead Actual hours Rs. 50,000 = = Rs. 1.5151 33,000

Actual Fixed Overhead Rate per hour ( ARH ) =

Actual hours taken Actual output 33,000 hrs = = 1.01538 hrs 32,500

Actual hours taken per unit of output ( AH ) =

Modified Date: Mon, Jul 05, 2010 05:03:14 PM

Output Date: Tue, Jul 06, 2010 12:21:13 PM

Rev II

Project: Management Accounting_Debarshi Bhattacharyya ACE Pro India Pvt. Ltd. File: X:\Pearson\Management Accounting_Debarshi Bhattacharyya\MAIN\M08\LAYOUT_M08\M08_DEBA_ISBN_EN_SE_C08.indd

596

MANAGEMENT ACCOUNTING

Revised standard hours (RSH) = Total budgeted hours in actual days = 30,000 hrs × 26 ÷ 25 = 31,200 hrs Budgeted output (BO) = 30,000 units Actual output (AO) = 32,500 units i. Fixed Overhead Cost Variance (FOCV) = SRH × SH × AO – ARH × AH × AO = Rs. 1.50 × 1 hr × 32,500 – Rs. 1.5151 × 1.01538 hrs × 32,500 = Rs. 1,250 (A) ii. Fixed Overhead Expenditure Variance (FOExp.V) = Budgeted fixed overheads for budgeted output – Actual fixed overheads for actual output = SRH × SH × BO – ARH × AH × AO = Rs. 1.50 × 1 hr × 30,000 – Rs. 1.5151 × 1.01538 hrs × 32,500 = Rs. 5,000 (A) iii. Fixed Overhead Volume Variance (FOVol.V) = Standard fixed overheads for actual output – Budgeted fixed overheads for budgeted output = SRH × SH × AO – SRH × SH × BO = Rs. 1.50 × 1 hr. × 32,500 – Rs. 1.50 × 1 hr. × 30,000 = Rs. 3,750 (F) iv. Fixed Overhead Efficiency Variance (FOEff.V) = Standard fixed overheads for actual output − Actual fixed overheads for actual output at standard rate = SRH × SH × AO – SRH × AH × AO = Rs. 1.50 × 1 hr × 32,500 – Rs. 1.50 × 1.01538 hrs × 32,500 = Rs. 750 (A) v. Fixed Overhead Capacity Variance (FOCap.V) = Actual fixed overhead for AO at standard rate − Standard fixed overhead for RSH = SRH × AH × AO – SRH × RSH = Rs. 1.50 × 1.01538 hrs × 32,500 – Rs. 1.50 × 31,200 hrs = Rs. 2,700 (F) vi. Fixed Overhead Calendar Variance (FOCal.V) = Standard fixed overhead for RSH − Budgeted fixed overheads for budgeted output = SRH × RSH – SRH × SH × BO = Rs. 1.50 × 31,200 hrs – Rs. 1.50 × 1 hr × 30,000 = Rs. 1,800 (F) Test i. FOCV = FOExp.V + FOVol. V Here, Rs. 1,250 (A) = Rs. 5,000 (A) + Rs. 3,750 (F) = Rs. 1,250 (A), checked ii. FOVol.V = FOEff.V + FOCap.V + FOCal.V Here, Rs. 3,750 (F) = Rs. 750 (A) + Rs. 2,700 (F) + Rs. 1,800 (F) = Rs. 3,750 (F), checked Problem 36 6 Standard 4,000 units 20 days Rs. 40,000

Production Working Days Fixed Overheads

Actual 3,800 units 21 days Rs. 39,000

Calculate Fixed Overhead Variances. Solution Budgeted output (BO) = 4,000 units Actual output (AO) = 3,800 units Standard output (SO) = Budgeted output for actual days = (4,000 units ÷ 20 days) × 21 days = 4,200 units

Modified Date: Mon, Jul 05, 2010 05:03:14 PM

Output Date: Tue, Jul 06, 2010 12:21:13 PM

Rev II

Project: Management Accounting_Debarshi Bhattacharyya ACE Pro India Pvt. Ltd. File: X:\Pearson\Management Accounting_Debarshi Bhattacharyya\MAIN\M08\LAYOUT_M08\M08_DEBA_ISBN_EN_SE_C08.indd

STANDARD COSTING AND VARIANCE ANALYSIS

Standard Fixed Overhead Rate per unit (SRU ) = = Standard Fixed Overhead Rate per day (SRD) = = Actual Fixed Overhead Rate per unit ( ARU ) = =

597

Budgeted fixed overhead Budgeted output Rs. 40,000 = Rs. 10 4,000 Budgeted fixed overhead Budgeted days Rs. 40,000 = Rs. 2,000 20

Actual fixed overhead Actual output Rs. 39,000 = Rs. 10.2631 3,800

i. Fixed Overhead Cost Variance (FOCV) = Standard fixed overhead for actual output − Actual fixed overhead for actual output = SRU × AO – ARU × AO = Rs. 10 × 3,800 – Rs. 10.2631 × 3,800 = Rs. 1,000 (A) ii. Fixed Overhead Expenditure Variance (FOExp.V) = Budgeted fixed overhead for budgeted output − Actual fixed overhead for actual output = SRU × BO – ARU × AO = Rs. 10 × 4,000 – Rs. 10.2631 × 3,800 = Rs. 1,000 (F) iii. Fixed Overhead Volume Variance (FOVol.V) = Standard fixed overhead for actual output − Budgeted fixed overhead for budgeted output = SRU × AO – SRU × BO = Rs. 10 × 3,800 – Rs. 10 × 4,000 = Rs. 2,000 (A) iv. Fixed Overhead Efficiency Variance (FOEff.V) = Standard fixed overhead for actual output − Standard fixed overhead for standard output = SRU × AO – SRU × SO = Rs. 10 × 3,800 – Rs. 10 × 4,200 = Rs. 4,000 (A) v. Fixed Overhead Capacity Variance (FOCap.V) = Standard fixed overhead for standard output – Budgeted fixed overhead for budgeted output = SRU × SO – SRU × BO = Rs. 10 × 4,200 – Rs. 10 × 4,000 = Rs. 2,000 (F) or Fixed Overhead Calendar Variance (FOCal.V) = Standard fixed overhead for actual days – Budgeted fixed overhead for budgeted days = SRD × Actual days – SRD × Budgeted days = Rs. 2,000 × 21 – Rs. 2,000 × 20 = Rs. 2,000 (F) Test i. FOCV = FOExp.V + FOVol.V Here, Rs. 1,000 (A) = Rs. 1,000 (F) + Rs. 2,000 (A) = Rs. 1,000 (A), checked ii. FOVol. V = FOEff.V + FOCap.V or FOCal.V Here, Rs. 2,000 (A) = Rs. 4,000 (A) + Rs. 2,000 (F) = Rs. 2,000 (A), checked

Modified Date: Mon, Jul 05, 2010 05:03:14 PM

Output Date: Tue, Jul 06, 2010 12:21:13 PM

Rev II

Project: Management Accounting_Debarshi Bhattacharyya ACE Pro India Pvt. Ltd. File: X:\Pearson\Management Accounting_Debarshi Bhattacharyya\MAIN\M08\LAYOUT_M08\M08_DEBA_ISBN_EN_SE_C08.indd

598

MANAGEMENT ACCOUNTING

E. Sales Variances Problem 37 From the following information, calculate the sales value variances: Budgeted Qty (units) Rate (Rs.) 900 12

Product D

Actual Qty (units) 1,000

Rate (Rs.) 11

Solution i. Total Sales Value Variance (TSVV) = Actual value of sales − Budgeted value of sales = AQ × AR – BQ × SR = 1,000 × Rs. 11 − 900 × Rs. 12 = Rs. 200 (F) ii. Sales Price Variance (SPV) = Actual price of sales − Standard price of sales = AQ × AR − AQ × SR = AQ (AR – SR) = 1,000 (Rs. 11 – Rs. 12) = Rs. 1,000 (A) iii. Sales-Volume Variance (SVV) = SR (AQ – BQ) = Rs. 12 (1,000 – 900) = Rs. 1,200 (F) Test TSVV = SPV + SVV Here, Rs. 200 (F) = Rs. 1,000 (A) + Rs. 1,200 (F) = Rs. 200 (F), checked Working Notes AR = Actual selling price per unit = Rs. 11 SR = Standard selling price per unit = Rs. 12 AQ = Actual sales quantity = 1,000 units BQ = Budgeted sales quantity = 900 units

Problem 38 From the following information, calculate sales profit margin variances:

Product

Qty (units) 2,000

Budgeted Selling Price per unit (Rs.) 10

Cost per unit (Rs.) 8

Qty (units) 1,500

Actual Selling Price Per Unit (Rs.) 12

Cost per unit (Rs.) 9

Solution i. Total Sales Margin Variance (TSMV) = Actual profit − Budgeted profit = AQ × Actual rate of profit − BQ × Standard rate of profit = 1,500 × Rs. 3 − 2,000 × Rs. 2 = Rs. 500 (F)

Modified Date: Mon, Jul 05, 2010 05:03:14 PM

Output Date: Tue, Jul 06, 2010 12:21:13 PM

Rev II

Project: Management Accounting_Debarshi Bhattacharyya ACE Pro India Pvt. Ltd. File: X:\Pearson\Management Accounting_Debarshi Bhattacharyya\MAIN\M08\LAYOUT_M08\M08_DEBA_ISBN_EN_SE_C08.indd

STANDARD COSTING AND VARIANCE ANALYSIS

599

ii. Sales Margin Price Variance (SMPV) = AQ [Actual rate of profit − Standard rate of profit] = 1,500 (Rs. 3 – Rs. 2) = Rs. 1,500 (F) iii. Sales Margin Volume Variance (SMVV) = Standard rate of profit [AQ – BQ] = Rs. 2 (1,500 – 2,000) = Rs. 1,000 (A) Test TSMV = SMPV + SMVV Here, Rs. 500 (F) = Rs. 1,500 (F) + Rs. 1,000 (A) = Rs. 500 (F), checked Working Notes Actual sales quantity = AQ = 1,500 units Budgeted sales quantity = BQ = 2,000 units Actual rate of profit = Actual selling price per unit − Actual cost price per unit = Rs. 12 – Rs. 9 = Rs. 3 Standard rate of profit = Standard selling price per unit − Standard Cost price per unit = Rs. 10 – Rs. 8 = Rs. 2

Problem 39 Budgeted and actual sales for a period of two products are given as follows:

Product P Product Q

Qty (kg) 1,000 1,200 2,200

Budgeted Rate (Rs.) Amount (Rs.) 5 5,000 10 12,000 17,000

Qty (kg) 1,200 1,110 2,310

Actual Rate (Rs.) 6 9

Amount (Rs.) 7,200 9,990 17,190

Standard Cost per unit of product P and Q were Rs. 4 and Rs. 7, respectively. Calculate SVVs and SMVs. Solution A. Sales Value Variances (SVV): i. Total Sales Value Variance (TSVV) = Actual value of sales − Budgeted value of sales = Rs. 17,190 – Rs. 17,000 = Rs. 190 (F) ii. Sales Price/Rate Variance (SPV/SRV) = AQ (AR – SR) Product P: 1,200 (Rs. 6 – Rs. 5) Product Q: 1,110 (Rs. 9 – Rs. 10)

= Rs. 1,200 (F) = Rs. 1,110 (A) Rs. 90 (F)

iii. Sales Volume Variance (SVV) Product P: Rs. 5 (1,200 – 1,000) Product Q: Rs. 10 (1,110 – 1,200)

= SR (AQ – BQ) = Rs. 1,000 (F) = Rs. 900 (A) Rs. 100 (F)

iv. Sales Quantity Variance (SQV) Product P: Rs. 5 (1,050 – 1,000)

= SR (RSQ – BQ) = Rs. 250 (F)

Modified Date: Mon, Jul 05, 2010 05:03:14 PM

Output Date: Tue, Jul 06, 2010 12:21:13 PM

Rev II

Project: Management Accounting_Debarshi Bhattacharyya ACE Pro India Pvt. Ltd. File: X:\Pearson\Management Accounting_Debarshi Bhattacharyya\MAIN\M08\LAYOUT_M08\M08_DEBA_ISBN_EN_SE_C08.indd

600

MANAGEMENT ACCOUNTING

Product Q: Rs. 10 (1,260 – 1,200)

= Rs. 600 (F) Rs. 850 (F)

v. Sales Mix Variance (SMV) = SR [AQ – RSQ] Product P: Rs. 5 (1,200 – 1,050) = Rs. 750 (F) Product Q: Rs. 10 (1,110 – 1,260) = Rs. 1,500 (A) Rs. 750 (A) Test i. TSVV = SPV + SVV Here, Rs. 190 (F) = Rs. 90 (F) + Rs. 100 (F) = Rs. 190 (F), checked ii. SVV = SQV + SMV Here, Rs. 100 (F) = Rs. 850 (F) + Rs. 750 (A) = Rs. 100 (F), checked Working Notes

Actual Sales Quantity (AQ) Budgeted Sales Quantity (BQ) Actual Selling Price per kg (AR) Standard Selling Price per kg (SR) Revised Standard Quantity (RSQ) (i.e., Total Actual Sales Quantity in Budgeted Ratio)

Product P 1,200 kg 1,000 kg Rs. 6 Rs. 5 2310 × 1,000 ÷ 2,200 = 1,050 kg

Product Q 1,110 kg 1,200 kg Rs. 9 Rs. 10 2,310 × 1,200 ÷ 2,200 = 1,260 kg

B. Sales Margin Variances (SMV): i. Total Sales Margin Variance (TSMV) = Actual profit – Budgeted profit = AQ × Actual rate of profit − BQ × Standard rate of profit Product P: (1,200 × Rs. 2) – (1,000 × Re. 1) = Rs. 1,400 (F) Product Q: (1,110 × Rs. 2) – (1,200 × Rs. 3) = Rs. 1,380 (A) Rs. 20 (F) ii. Sales Margin Price Variance (SMPV) = AQ [Actual rate of profit − Standard rate of profit] Product P: 1,200 (Rs. 2 – Re. 1) = Rs. 1,200 (F) Product Q: 1,110 (Rs. 2 – Rs. 3) = Rs. 1,110 (A) Rs. 90 (F) iii. Sales Margin Volume Variance (SMVV) = Standard rate of profit (AQ – BQ) Product P: Re. 1 (1,200 – 1,000) = Rs. 200 (F) Product Q: Rs. 3 (1,110 – 1,200) = Rs. 270 (A) Rs. 70 (A) iv. Sales Margin Quantity Variance (SMQV) = Standard rate of profit (RSQ – BQ) Product P: Re. 1 (1,050 – 1,000) = Rs. 50 (F) Product Q: Rs. 3. (1,260 – 1,200) = Rs. 180 (F) Rs. 230 (F)

Modified Date: Mon, Jul 05, 2010 05:03:14 PM

Output Date: Tue, Jul 06, 2010 12:21:13 PM

Rev II

Project: Management Accounting_Debarshi Bhattacharyya ACE Pro India Pvt. Ltd. File: X:\Pearson\Management Accounting_Debarshi Bhattacharyya\MAIN\M08\LAYOUT_M08\M08_DEBA_ISBN_EN_SE_C08.indd

601

STANDARD COSTING AND VARIANCE ANALYSIS

v. Sales Margin Mix Variance (SMMV) = Standard rate of profit (AQ – RSQ) Product P: Re. 1 (1,200 – 1,050) Product Q: Rs. 3 (1,110 – 1,260)

= Rs. 150 (F) = Rs. 450 (A) Rs. 300 (A)

Test i. TSMV = SMPV + SMVV Here, Rs. 20 (F) = Rs. 90 (F) + Rs. 70 (A) = Rs. 20 (F), checked ii. SMVV = SMQV + SMMV Here, Rs. 70 (A) = Rs. 230 (F) + Rs. 300 (A) = Rs. 70 (A), checked

Working Notes

Actual Sales Quantity (AQ) Budgeted Sales Quantity (BQ) Actual Rate of Profit Standard Rate of Profit Revised Standard Quantity (RSQ) (i.e., Total Actual Sales Quantity in Budgeted Ratio)

Product P 1,200 kg 1,000 kg Rs. 6 – Rs. 4 = Rs. 2 Rs. 5 – Rs. 4 = Re. 1 2,310 × 1,000 ÷ 2,200 = 1,050 kg

Product Q 1,110 kg 1,200 kg Rs. 9 – Rs. 7 = Rs. 2 Rs. 10 – Rs. 7=Rs. 3 2,310 × 1,200 ÷ 2,200 = 1,260 kg

Problem 40 Compute the missing data from the following information: Sales Quantity: Standard (units) Actual (units) Price per kg: Standard (Rs.) Actual (Rs.) Sales Price Variance (Rs.) Sales Volume Variance (Rs.) Total Sales Value Variance

Product X

Product Y

? 500

400 ?

12 15 ? 1,200 F ?

15 20 ? ? ?

Sales Mix Variance for both the products together was Rs. 450 F. Solution i. Sales Volume Variance (SVV) = SR (AQ – BQ). Here, for Product X, 1,200 = Rs. 12 (500 – BQ) or 12 BQ = 6,000 – 1,200 ∴BQ = 4,800 ÷ 12 = 400 units ∴Standard sales quantity of Product X = 400 units

Modified Date: Mon, Jul 05, 2010 05:03:14 PM

Output Date: Tue, Jul 06, 2010 12:21:13 PM

Rev II

Project: Management Accounting_Debarshi Bhattacharyya ACE Pro India Pvt. Ltd. File: X:\Pearson\Management Accounting_Debarshi Bhattacharyya\MAIN\M08\LAYOUT_M08\M08_DEBA_ISBN_EN_SE_C08.indd

602

MANAGEMENT ACCOUNTING

ii. Let the actual sales quantity of Product Y be ‘a’ units. We know, Sales Mix Variance (SMV) = SR [AQ – RSQ] RSQ = Total actual sales quantity in standard ratio ∴RSQ for Product X = {400 ÷ (400 + 400)} × (500 + a) = (500 + a) ÷ 2 units ∴RSQ for product Y = {400 ÷ (400 + 400)} × (500 + a) = (500 + a) ÷ 2 units ∴SMV for Product X = 12 [500 − {(500 + a) ÷ 2}] = 12 {(500 − a) ÷ 2} = 3,000 – 6a ∴SMV for Product Y = 15 [a − {(500 + a) ÷ 2}] = 15 {(a − 500) ÷ 2} = 7.5a – 3,750 SMV for both the products = 1.5a – 750 ∴By condition, 1.5a – 750 = 450 or 1.5a = 450 + 750 or a = 1,200 ÷ 1.5 = 800 ∴ Actual sales quantity of Product Y = 800 units iii. Sales Price Variance (SPV) = AQ (AR – SR) ∴ SPV for Product X = 500 (Rs. 15 – Rs. 12) = Rs. 1,500 (F) ∴ SPV for Product Y = 800 (Rs. 20 – Rs. 15) = Rs. 4,000 (F) iv. Sales Volume Variance (SVV) = SR (AQ – BQ) ∴ SVV for Product Y = Rs. 15 (800 – 400) = Rs. 6,000 (F) v. Total Sales Value Variance (TSVV) = AQ × AR – BQ × SR ∴ TSVV for Product X = 500 × Rs. 15 – 400 × Rs. 12 = Rs. 2,700 (F) ∴ TSVV for Product Y = 800 × Rs. 20 – 400 × Rs. 15 = Rs. 10,000 (F) CHAPTER REVIEW SUMMARY  Standard Cost is the predetermined cost what should have been under the specified operating conditions. It is the yardstick of measuring the efficiency of the actual cost incurred.  Standard Costing is the method of setting up of standard costs and applying them to measure the deviations from the actual costs and analysing the causes of such deviation with a view to maximize the efficiency of the production functions.  Steps adopted for establishing a Standard Costing System in an organization are: (a) Establishment of cost centres; (b) Classification of accounts; and (c) Types of standard.  Standard may be classified into the following five types: (a) Basic standard; (b) Current standard; (c) Ideal standard; (d) Normal standard; and (e) Expected or practical standard.  Standard Cost refers to the scientifically calculated predetermined cost that should have been under the specified conditions. It is ideal in nature and related to future. Estimated Cost is also a predetermined cost, but computed with the help of historic data. It is probable in nature and related to future. Actual or Historical Cost is the cost actually incurred in the past and ascertained after it is incurred. It is actual in nature and related to past.  Advantages of Standard Costing are: (a) It facilitates an effective cost control and provides necessary information for cost control; (b) It identifies the areas of strength and weakness of production functions; (c) It helps in measuring the efficiency of the actual performance; (d) It helps in formulation of future production and pricing policies; (e) It is useful for budgeting and planning; and (f) It provides useful information for price quotation of the products.  Limitations of Standard Costing are: (a) Standard Costing System is not effective for the organizations which deal with non-standardized products and jobs which change according to customer’s specification; (b) Sometimes, it becomes difficult, in practise, to establish standards as it needs a high degree of technical skill;

Modified Date: Mon, Jul 05, 2010 05:03:14 PM

Output Date: Tue, Jul 06, 2010 12:21:13 PM

Rev II

Project: Management Accounting_Debarshi Bhattacharyya ACE Pro India Pvt. Ltd. File: X:\Pearson\Management Accounting_Debarshi Bhattacharyya\MAIN\M08\LAYOUT_M08\M08_DEBA_ISBN_EN_SE_C08.indd

STANDARD COSTING AND VARIANCE ANALYSIS







603

(c) If standards are inaccurately established, wrong and misleading conclusions may be drawn; (d) Installation of Standard Costing System in an organization is a very costly one and accordingly, small business organizations cannot afford to install this system; and (e) Standard Costing is only a tool of management and not a substitute of management. Distinction between Standard Costing and Budgetary Control are: (a) Standard Costing is based on technical assessment whereas Budgetary Control is based on past performance adjusted with future trend; (b) Standard Costing is the method of preparation of Standard Cost and application of those for measuring the efficiency of the actual cost whereas Budgetary Control is the method of forecasting cost by preparation of budget prior to a definite period to attain the given objectives; (c) Standard Cost is a projection of cost accounts whereas budget is a projection of financial accounts; and (d) Standard Costing is mainly concerned with the ascertainment and control of costs whereas Budgetary Control is mainly concerned with the profitability and financial position of a concern. The deviation of actual performance from the standard set out is called ‘variance.’  Variance Analysis is the process of analysing reasons behind the variances occurred by sub-dividing them in such a manner so that the management can be reported for adopting remedial measures for any off-standard performance. It involves three functional actions, which are: (i) Ascertainment of individual variances; (ii) Determination of causes of variance; and (iii) Reporting to the management for taking a corrective action for any off-standard performance. Broadly, variances are classified into two parts—Cost variances and Sales variances. Again, cost variances are classified into four parts for each of the cost element, such as MCVs, LCV, VOV (Variable Overhead Variance) and FOV (Fixed Overhead Variance). Sales variances are classified into two parts, such as SVV and SPMV. Each of the variance is again sub-divided into various sub-variances.

CHAPTER REVIEW QUIZ 1. Fill in the blank/s of the following statements: cost. a. Standard cost is a b. The deviation of actual cost or sales or profit from the standard cost or sales or profit is called . c. When the material actually consumed is 1,100 kg at Rs. 8 per kg, the MPV is Rs. 2,200 (F) and MUV is kg and standard Rs. 1,000 (A), then the standard quantity for the actual output is per kg. price is Rs. d. When the standard hours required for the actual output is 5,000 hours at Rs. 20 per hour, the LRV is Rs. 20,000 (F) and LEV is Rs. 20,000 (F), then the actual hours taken for the actual output is hours and the actual labour rate is Rs. per hour. and . e. SVV is the difference between the of the actual output and . f. MCV is the difference between the + g. FOCV = + h. VOCV = Ans.: (a) predetermined; (b) variance; (c) 1,000 kg, Rs. 10; (d) 4,000 hrs, Rs. 15; (e) actual sales, budgeted sales; (f) standard material cost, actual material cost of actual output; (g) FOExp.V, FOVol.V; (h) VOExp.V, VOEff.V. 2. Choose the correct alternative from the following: a. An adverse MPV occurs because of: (i) price increase in material; (ii) price decrease in material; (iii) more than anticipated normal wastage of material. b. A favourable MUV occurs because of: (i) over consumption of material; (ii) under consumption of material; (iii) price decrease in material. c. MYV = (SH × AO − AH × AO) X: (i) AR; (ii) Standard yield rate; (iii) SR. d. Standard Costing helps in: (i) reducing losses; (ii) measuring the actual performance; (iii) controlling prices. e. Basic standard is prepared for a: (i) short period; (ii) long period; (iii) current period. f. Standard Costing involves in determination of: (i) Standard Cost; (ii) Estimated Cost; (iii) budgeted cost.

Modified Date: Mon, Jul 05, 2010 05:03:14 PM

Output Date: Tue, Jul 06, 2010 12:21:13 PM

Rev II

Project: Management Accounting_Debarshi Bhattacharyya ACE Pro India Pvt. Ltd. File: X:\Pearson\Management Accounting_Debarshi Bhattacharyya\MAIN\M08\LAYOUT_M08\M08_DEBA_ISBN_EN_SE_C08.indd

604

MANAGEMENT ACCOUNTING

g. If MCV is Rs. 10,000 (F) and MPV is Rs. 15,000 (A), then MUV is equal to: (i) Rs. 5,000 (A); (ii) Rs. 25,000 (F); (iii) Rs. 5,000 (F). h. If LCV is Rs. 20,000 (A) and LEV is Rs. 12,000 (F), then LPV is equal to: (i) Rs. 32,000 (A); (ii) Rs. 8,000 (F); (iii) Rs. 32,000 (F). Ans.: (a) (i); (b) (ii); (c) (iii); (d) (ii); (e) (iii); (f) (i); (g) (ii); (h) (i); 3. State whether the following statements are true or false: a. MUV indicates the difference between the standard price of material and the actual price of material. b. LITV always represents an unfavourable variance. c. A cost variance is said to be favourable if the actual cost is more than the Standard Cost. d. While calculating LEV, the actual hours attended by workers less idle hours are to be taken as the actual hours taken for the actual output. e. MMV is to be calculated where more than one type of material is used to produce the output. f. A favourable sales variance occurs when the actual sales revenue is less than the expected sales. g. Variable Overhead Cost Variance (VOCV) is always favourable. h. If LMV exists, then LYV is bound to exist. i. MYV arises even if only one type of material is used to produce the output. j. Variable Overhead Expenditure Variance (VOExp.V) = Budgeted variable overhead cost − Actual variable overhead cost Ans.: True: (b), (d), (e), (i), ( j); False: (a), (c), (f), (g), (h).

EXERCISE I. Theoretical Questions A. Short Answer Type Questions:

1. 2. 3. 4. 5. 6. 7. 8. 9. 10. 11. 12. 13. 14. 15.

What is Standard Cost? What is Standard Costing? What is Estimated Cost? What is Historical cost? What are the features of Standard Costing? Name the different types of Standard. What is Variance? What is Variance Analysis? What do you mean by Favourable Cost Variance? What do you mean by Adverse Cost Variance? What do you mean by Ideal Standard and Normal Standard? Write the formula of Material Usage Variance. Write the formula of Labour Efficiency Variance. Write the formula of Variable Overhead Expenditure Variance. Write the formula of Sales Value Volume Variance.

B. Essay Type Questions:

1. Define Standard Cost, Estimated Cost and Historical Cost. Distinguish between them. 2. What is Standard Costing System? What are its features? What are the advantages and limitations of this system? 3. What are the preliminary steps to be taken for the establishment of Standard Costing System in a concern? 4. Distinguish between Standard Costing and Budgetary Control. 5. What is a Variance? What is Variance Analysis? What is its nature and purpose? 6. Discuss the different types of standard.

Modified Date: Mon, Jul 05, 2010 05:03:14 PM

Output Date: Tue, Jul 06, 2010 12:21:13 PM

Rev II

Project: Management Accounting_Debarshi Bhattacharyya ACE Pro India Pvt. Ltd. File: X:\Pearson\Management Accounting_Debarshi Bhattacharyya\MAIN\M08\LAYOUT_M08\M08_DEBA_ISBN_EN_SE_C08.indd

STANDARD COSTING AND VARIANCE ANALYSIS

605

7. Classify Cost Variances. 8. Classify Sales Variances. 9. Explain the following; (a) Material Price Variance; (b) Material Mix Variance; (c) Labour Efficiency Variance; (d) Labour Yield Variance; (e) Variable Overhead Efficiency Variance; (f) Fixed Overhead Volume Variance; (g) Fixed Overhead Capacity Variance; (h) Sales Mix Variance; and (i) Sales Margin Quantity Variance. II. Practical Problems A. Material Variance:

1. Standard raw material cost of producing 4,000 units of Product P was as follows: 10,000 units of raw materials @ Rs. 8 per unit. But the actual raw material cost of producing 4,000 units of Product P was: 12,000 units of raw materials @ Rs. 6 per unit. Determine: (a) RMCV; (b) RMPV and (c) RMUV. Ans.: MCV = Rs. 8,000 (F); MPV = Rs. 24,000 (F); MUV = Rs. 16,000 (A). 2. Standard material cost of manufacturing 1,000 units of output is 400 kg of material at Rs. 2.50 per kg. When 2,000 units are produced, it is found that the actual cost is 825 kg of material at Rs. 2.70 per kg. Calculate MCV, MPV and MUV.

[B.Com. (Hons), Delhi University—2008]

Ans.: MCV = Rs. 227.50 (A); MPV = Rs. 165 (A); MUV = Rs. 62.50 (A).

3. Standard Cost material per kg Standard quantity of material required to produce one unit of output Actual quantity of material used for actual output of 20,000 units Actual cost of material for actual output Ans.: MCV = Rs. 40,000 (F); MPV=Rs. 40,000; MUV=Rs. 80,000 (F).

= Rs. 4 = 3 kg = 40,000 kg = Rs. 2,00,000

4. The standard material cost of producing each unit of product K is as follows: 4.8 kg of material @ Rs. 5 per kg. Actual material cost of producing 200 units of product K is as follows: 1,200 kg of material costing Rs. 4,800. Compute the Material Variances. [B.Com. (Hons), Calcutta University—Part II—2004] Ans.: MCV = Nil; MPV = Rs. 1,200 (F); MUV = Rs. 1,200 (A). 5. From the following information, calculate (a) MCV, (b) MPV and (c) MUV: Standard price of material per kg Actual price of material per kg Standard quantity of material required to produce 1 unit of output Actual quantity of material used to produce 1 unit of output Actual output Ans.: MCV = Rs. 1,60,000 (F); MPV = Rs. 9,60,000 (F); MUV = Rs. 8,00,000 (A).

Rs. 10 Rs. 8 20 kg 24 kg 20,000 units

6. 80 kg of Material A at a standard price of Rs. 2 per kg and 40 kg of Material B at a standard price of Rs. 5 per kg were to be used to manufacture a 100 kg of a chemical. During a month, 70 kg of Material A priced at Rs. 2.10 per kg and 50 kg of Material B priced at Rs. 4.50 per kg were actually used and the output of the chemical was 102 kg. Find out the Material Variances. Ans.: MCV = Rs. 4.80 (A); MPV = Rs. 18 (F); MUV = Rs. 22.80 (A); MMV = Rs. 30 (A); MYV = Rs. 7.20 (F).

Modified Date: Mon, Jul 05, 2010 05:03:14 PM

Output Date: Tue, Jul 06, 2010 12:21:13 PM

Rev II

Project: Management Accounting_Debarshi Bhattacharyya ACE Pro India Pvt. Ltd. File: X:\Pearson\Management Accounting_Debarshi Bhattacharyya\MAIN\M08\LAYOUT_M08\M08_DEBA_ISBN_EN_SE_C08.indd

606

MANAGEMENT ACCOUNTING

7. Standard

Material

Qty (Units) 220 330 550 50 500

A B Less: Loss

Actual

Rate (Rs.) 10 5

Amount (Rs.) 2,200 1,650 3,850

Qty (units) 440 660 1,100 100 1,000

− 3,850

Rate (Rs.) 8 8

Amount (Rs.) 3,520 5,280 8,800

− 8,800

From the above information, calculate MCVs. Ans.: MCV = Rs. 1,100 (A); MPV = Rs. 1,100 (A); MUV = Nil; MMV = Nil; MYV = Nil. 8. X Ltd manufactures a simple product, the standard mix of which is: Material A: 120 kg @ Rs. 20 per kg. Material B: 80 kg @ Rs. 10 per kg. Normal loss in production is 20% of input. Actual inputs for February 2005 were: Material A: 105 kg @ Rs. 20 per kg. Material B: 95 kg @ Rs. 9 per kg. Actual output for February 2005 was 164 kg. Calculate: i. MPV; ii. MUV; iii. MMV; iv. MYV. [B.Com. (Hons), Kalyani University—2005] Ans.: (i) MPV = Rs. 95 (F); (ii) MUV = Rs. 230 (F); (iii) MMV = Rs. 150 (F); (iv) MYV = Rs. 80 (F). 9. Bogula Company Ltd provides you the following information: For making a 10 kg of a product, the standard material requirement is: Material X Y

Quantity (kg) 8 4

Rate Per kg (Rs.) 6.00 4.00

During March 2010, 1,000 kg of the product were produced. The actual consumption of materials is as follows: Material X Y

Quantity (kg) 750 500

Rate Per kg (Rs.) 7.00 5.00

Calculate MCVs. [C.A. (Inter)—Adapted] Ans.: MCV = Rs. 1,350 (A); MPV = Rs. 1,250 (A); MUV = Rs. 100 (A); MMV = Rs. 167 (F); MYV = Rs. 267 (A). 10. A company manufactures a Product R by mixing three raw materials. For every 100 kg of R, 125 kg of raw materials are used. In April 2010, there was an output of 5,600 kg of R. The standard and actual particulars of April 2010 are as follows: Standard Raw material: A

Actual

Mix (%)

Price Per kg (Rs.)

Mix (%)

Price Per kg (Rs.)

50

40

60

42

(Continued)

Modified Date: Mon, Jul 05, 2010 05:03:14 PM

Output Date: Tue, Jul 06, 2010 12:21:13 PM

Rev II

Project: Management Accounting_Debarshi Bhattacharyya ACE Pro India Pvt. Ltd. File: X:\Pearson\Management Accounting_Debarshi Bhattacharyya\MAIN\M08\LAYOUT_M08\M08_DEBA_ISBN_EN_SE_C08.indd

STANDARD COSTING AND VARIANCE ANALYSIS

Standard Mix (%) 30 20

B C

607

Actual

Price Per kg (Rs.) 20 10

Mix (%) 20 20

Price Per kg (Rs.) 16 12

Calculate MCVs. [I.C.W.A. (Inter)—Adapted] Ans.: MCV = Rs. 19,600 (A); MPV = Rs. 5,600 (A); MUV = Rs. 14,000 (A); MMV = Rs. 14,000 (A); MYV = Nil. 11. The standard mixture of Material P and Material Q for a certain period is specified as follows: Material P: 120 tonnes @ Rs. 5 per tonne. Material Q: 80 tonnes @ Rs. 10 per tonne. A loss of 10% is expected. During the period, 275 tonnes of output were produced. Due to a shortage of Material Q, it was used less in proporti on as specified in standard. Actual use of materials for that period was as follows: Material P: 200 tonnes @ Rs. 6. Material Q: 100 tonnes @ Rs. 8. Calculate MCVs. Ans.: MCV = Rs. 139 (F); MPV = Nil; MUV = Rs. 139 (F); MMV = Rs. 100 (F); MYV = Rs. 39 (F). 12. From the following data, compute the MCVs: Name of the Material Zee Wee Tee

Standard Qty (units) 3,500 1,500 1,000

Price (Rs.) 10 21 33

Actual Qty (units) 3,700 1,650 1,250

Price (Rs.) 12 20 36

[I.C.W.A. (Inter)—December 1994] Ans.: MCV = Rs. 22,900 (A); MPV = Rs. 9,500 (A); MUV = Rs. 13,400 (A); MMV = Rs. 3,450 (A); MYV = Rs. 9,950 (A). 13. From the following information, calculate the MMV: Material X Y

Standard 200 units @ Rs. 12 100 units @ Rs. 10

Actual 160 units @ Rs. 13 140 units @ Rs. 10

Due to shortage of Material X, it was decided to reduce the consumption of X by 15% and increase that of Y by 30%. Ans.: MMV = Rs. 20 (F). 14. The standard material cost for 100 kg of Chemical D is made up of: Chemical A Chemical B Chemical C

30 kg @ Rs. 4.00 per kg 40 kg @ Rs. 5.00 per kg 80 kg @ Rs. 6.00 per kg

In a batch, 500 kg of Chemical D is produced from a mix of: Chemical A Chemical B Chemical C

Modified Date: Mon, Jul 05, 2010 05:03:14 PM

140 kg at a cost of Rs. 588 220 kg at a cost of Rs. 1,056 440 kg at a cost of Rs. 2,860

Output Date: Tue, Jul 06, 2010 12:21:13 PM

Rev II

Project: Management Accounting_Debarshi Bhattacharyya ACE Pro India Pvt. Ltd. File: X:\Pearson\Management Accounting_Debarshi Bhattacharyya\MAIN\M08\LAYOUT_M08\M08_DEBA_ISBN_EN_SE_C08.indd

608

MANAGEMENT ACCOUNTING

Compute the following material variances contributing to the actual cost of 100 kg of Chemical D over the Standard Cost: (i) MCV; (ii) MPV; (iii) MUV; (iv) MMV; (v) MYV. [B.Com. (Hons), Delhi University] Ans.: (i) MCV = Rs. 100.80 (A); (ii) MPV = Rs. 40.80 (A); (iii) MUV = Rs. 60 (A); (iv) MMV = Rs. 6.67 (A); (v) MYV = Rs. 53.33 (A). 15. The Standard Cost of a certain chemical mixture is: 40% Material A @ Rs. 20 per kg. 60% Material B @ Rs. 30 per kg. A standard loss of 10% is expected in production. During a period, the actual cost is: 90 kg of Material A @ Rs. 18 per kg. 110 kg of Material B @ Rs. 34 per kg. The actual output is 182 kg. Calculate: (a) MPV; (b) MMV; (c) MYV; (d) MCV. [B.Com. (Hons), Calcutta University—2008] Ans.: (a) Rs. 260 (A); (b) Rs. 98 (F); (c) Rs. 52 (F); (d) Rs. 110 (A). 16. Lodha Ltd has established the following standard mix for producing 9 gallons of Product P: Rs. 35 15 4 54

5 gallons of Material A @ Rs. 7 per gallon 3 gallons of Material B @ Rs. 5 per gallon 2 gallons of Material C @ Rs. 2 per gallon

A standard loss of 10% of input is expected to occur. Actual input was as follows: 53,000 gallons of Material A @ Rs. 7 per gallon. 28,000 gallons of Material B @ Rs. 5.30 per gallon. 19,000 gallons of Material C @ Rs. 2.20 per gallon. Actual output for a period was 92,700 gallons of Product P. Compute the Material Variances. [B.Com. (Hons), Delhi University—Adapted] Ans.: MCV = Rs. 5,000 (A); MPV = Rs. 12,200 (A); MUV = Rs. 7,200 (F); MMV = Rs. 9,000 (A); MYV = Rs. 16,200 (F). 17. The standard material input required for 1,000 kg of a finished product are given as follows: Material P Q R Standard Loss Standard Output

Qty (kg) 450 400 250 1,100 100 1,000

Standard Rate per kg (Rs.) 20 40 60

Actual production in a period was 20,000 kg of the finished product for which the actual quantities of material used and the prices paid thereof are as follows: Material P Q R

Quantity Used (kg) 10,000 8,500 4,500

Purchase Price Per kg (Rs.) 19 42 65

Calculate: (i) MCV; (ii) MPV; (iii) MUV; (iv) MMV; (v) MYV. [I.C.W.A. (Inter)—June 1996]

Modified Date: Mon, Jul 05, 2010 05:03:14 PM

Output Date: Tue, Jul 06, 2010 12:21:13 PM

Rev II

Project: Management Accounting_Debarshi Bhattacharyya ACE Pro India Pvt. Ltd. File: X:\Pearson\Management Accounting_Debarshi Bhattacharyya\MAIN\M08\LAYOUT_M08\M08_DEBA_ISBN_EN_SE_C08.indd

STANDARD COSTING AND VARIANCE ANALYSIS

609

Ans.: (i) MCV = Rs. 39,500 (A); (ii) MPV = Rs. 29,500 (A); (iii) MUV = Rs. 10,000 (A); (iv) MMV = Rs. 26,363 (F); (v) MYV = Rs. 36,363 (A). B. Labour Variances:

18. From the following information, calculate the labour variances: Standard for 100 units:

500 labour hours Rate @ Rs. 24 per hr 1,000 units were produced Total wages paid @ Rs. 1,30,000 for 5,200 hrs

Actual Production:

[B.Com. (Hons), Mumbai University—October 2006] Ans.: LCV = Rs. 10,000 (A); LRV = Rs. 5,200 (A); LEV = Rs. 4,800 (A). 19. From the following information, calculate the LCV, LRV and LEV: Standard Rate per hour Standard time per unit of output Units Produced Actual hours worked Actual Labour Cost

Rs. 4.00 20 hrs 500 12,000 Rs. 38,400

[B.Com. (Hons), Calcutta University—2006] Ans.: LCV = Rs. 1,600 (F); LRV = Rs. 9,600 (F); LEV = Rs. 8,000 (A). 20. From the following particulars, compute the labour variances: Standard Labour Rate per hour Actual Labour Rate per hour Standard hours required to produce one unit of output Actual hours taken to produce one unit of output Actual output

Rs. 100 Rs. 120 20 hrs 16 hrs 5,000 units

Ans.: LCV = Rs. 4,00,000 (F); LRV = Rs. 16,00,000 (A); LEV = Rs. 20,00,000 (F). 21. A company manufactured 820 units of a product per week. The standard wage rate per hour was Re.0.90. The standard output was 20 units per hour. During a week, the actual wages paid amounted to Rs. 38 for 40 hours, of which 5 hours remained idle. Calculate LCVs. Ans.: LCV = Rs. 1.10 (A); LRV = Rs. 2 (A); LITV = Rs. 4.50 (A); LEV = Rs. 5.40 (F). 22. From the following information, calculate LCVs: Standard Labour hours required for each unit of output Standard Labour Rate per hour Actual hours taken for actual output of 5,000 units (including 500 idle hours) Actual Labour Cost for 5,000 units of output

3 hrs Rs. 4 10,000 hrs Rs. 50,000

Ans.: LCV = Rs. 10,000 (F); LRV = Rs. 10,000 (A); LITV = Rs. 2,000 (A); LEV = Rs. 22,000 (F). 23. Following are the particulars furnished by KJ Ltd for the production of Product G.: Standard Labour Rate per hour Standard Labour hours required per unit of output Actual Labour hours for actual production of 4,000 units (including 200 hours for power failure) Actual Labour Rate per hour

Rs. 2 20 hrs 76,000 hrs Rs. 2.10

Calculate the Labour Variances.

Ans.: LCV = Rs. 400 (F); LRV = Rs. 7,600 (A); LITV = Rs. 400(A); LEV = Rs. 8,400 (F).

Modified Date: Mon, Jul 05, 2010 05:03:14 PM

Output Date: Tue, Jul 06, 2010 12:21:13 PM

Rev II

Project: Management Accounting_Debarshi Bhattacharyya ACE Pro India Pvt. Ltd. File: X:\Pearson\Management Accounting_Debarshi Bhattacharyya\MAIN\M08\LAYOUT_M08\M08_DEBA_ISBN_EN_SE_C08.indd

610

MANAGEMENT ACCOUNTING

24. The standard output of Product S is 25 units per hour of a factory where 100 workers are employed. Standard wage rate per labour hour is Rs. 6. In a 42-hour week, the factory produced 1,040 units of Product S despite the loss of 5% of the time paid due to abnormal reasons. The hourly rates actually paid were Rs. 6.20, Rs. 6 and Rs. 5.70, respectively, to 10, 30 and 60 workers. Compute various Labour Variances. Ans.: LCV = Rs. 432 (F); LRV = Rs. 672 (F); LITV = Rs. 1,260 (A); LEV = Rs. 1,020 (F). 25. From the following information, calculate labour variances for each department: Dept I 2,808 8,640 0.30 8,200

Wages Paid (Rs.) Standard hours produced Standard Rate per hour (Re.) Actual hours worked

Dept II 1,937 6,015 0.33 6,345

Ans.: Dept I: LCV—Rs. 216 (A); LRV—Rs. 348 (A); LEV—Rs. 132 (F). Dept II: LCV—Rs. 48 (F); LRV—Rs. 157 (F); LEV—Rs. 109 (A). 26. From the following information as furnished by a manufacturing company, calculate the labour variances: Standard Average Wages per worker per month (Rs.) No. of workers No. of working days in a month Output (units)

Actual 528 1,100 24 56,000

500 1,200 25 60,000

Ans.: LCV = Rs. 20,800 (A); LRV = Rs. 52, 800 (A); LEV = Rs. 32.000 (F). 27. Standard labour cost of producing 40 units of a product is 30-hour work by skilled workers at a standard rate of Rs. 60 per hour and 90-hour work by unskilled workers at standard rate of Rs. 20 per hour. 40 units of the product were produced for which the skilled workers were paid for 20 hours at Rs. 55 per hour and unskilled workers were paid for 130 hours at Rs. 24 per hour. Due to a machine break-down, both skilled as well as unskilled workers lost 9 hours each. They were paid even for this time. Calculate: (i) LCV; (ii) LRV; (iii) LEV; (iv) LITV; (v) LMV; (vi) LYV. [B.Com. (Hons), Delhi University—2007 (Ext.)] Ans.: (i) LCV—Rs. 620 (A); (ii) LRV—Rs. 420 (A); (iii) LEV—Rs. 520 (F); (iv) LITV—Rs. 720 (A); (v) LMV—Rs. 880 (F); LYV—Rs. 360 (A). 28. Standard labour hours and rate for production of each unit are given as follows: Skilled Worker Unskilled Worker Semi-skilled Worker

Hours 5 10 8 23

Rate (Rs.) 6 4 5

Total (Rs.) 30 40 40 110

Rate Rs. 7 3 6

Total Rs. 28,000 36,000 42,000 1,06,000

Actual data for actual production of 1,000 units are given as follows: Skilled Worker Unskilled Worker Semi-skilled Worker

Hours 4,000 12,000 7,000 23,000

Calculate LCVs. Ans.: LCV—Rs. 4,000 (F); LRV—Rs. 1,000 (F); LEV—Rs. 3,000 (F); LMV—Rs. 3,000 (F); LYV—Nil.

Modified Date: Mon, Jul 05, 2010 05:03:14 PM

Output Date: Tue, Jul 06, 2010 12:21:13 PM

Rev II

Project: Management Accounting_Debarshi Bhattacharyya ACE Pro India Pvt. Ltd. File: X:\Pearson\Management Accounting_Debarshi Bhattacharyya\MAIN\M08\LAYOUT_M08\M08_DEBA_ISBN_EN_SE_C08.indd

STANDARD COSTING AND VARIANCE ANALYSIS

611

29. A gang of workers usually consists of 10 men, 50 women and 5 boys in a factory. They are paid at standard hourly rates of Rs. 1.25, Re. 0.80 and Re. 0.70, respectively. In a normal working week of 40 hours, the gang is expected to produce 1,000 units of output. In a certain week, the gang consisted of 13 men, 4 women and 3 boys. Actual wage rates per hour were paid @ Rs. 1.20, Re. 0.85 and Re 0.65, respectively. Two hours per week were lost due to abnormal idle time and 960 units of output were produced. Calculate various Labour Variances. Ans.: LCV—Rs. 70 (A); LRV—Rs. 24 (F); LITV—Rs. 43.10 (A); LEV—Rs. 50.90 (A); LMV—Rs. 58.90 (A); LYV—Rs. 8 (F). C. MCVs and LCVs combined:

30. The following details relating to the Product A during the month of March 2002 are available. You are required to compute the MCVs and LCVs and also to reconcile the standard and the actual cost with the help of such variances. Standard Cost per unit: Materials Labour Actual Cost for the month: Materials Labour

50 kg @ Rs. 40 per kg 400 hrs @ Re. 1 per hr 4,900 kg @ Rs. 42 per kg 39,600 hrs @ Rs. 1.10 per hr

Actual production for the month is 100 units.

Ans.: MCV—Rs. 5,800 (A); MPV—Rs. 9,800 (A); MUV—Rs. 4,000 (F); LCV—Rs. 3,560 (A); LRV—Rs. 3,960 (A); LEV—Rs. 400 (F). 31. The standard cost per unit of a product are as follows: Material cost – 5 kg @ Rs. 10 per kg Labour cost – 4 hrs @ Rs. 5 per hr The Actual Cost emerged from the business operation are as follows: Production Materials Consumed: 30,000 kg @ Rs. 8 per kg Wages Paid: 25,000 hrs @ Rs. 6 per hr

Rs. 50 20 5,000 units Rs. 2,40,000 Rs. 1,50,000

Calculate MCVs and LCVs. Ans.: MCV – Rs. 10,000 (F); MPV – Rs. 60,000 (F); MUV – Rs. 50,000 (A); LCV – Rs. 50,000 (A); LRV – Rs. 25,000 (A); LEV – Rs. 25,000 (A). 32. The following standards have been set to manufacture a product: Rs. Direct Materials: 2 units of P at Rs. 4 per unit 3 units of Q at Rs. 3 per unit 15 units of R at Re. 1 per unit

8.00 9.00 15.00 32.00 24.00 56.00

Direct Labour – 3 hrs @ Rs. 8 per hr Total Standard Prime Cost

The company manufactured and sold 6,000 units of the product during the year. Direct materials cost is as follows: 12,500 units of P at Rs. 4.40 per unit. 18,000 units of Q at Rs. 2.80 per unit. 88,500 units of R at Rs. 1.20 per unit.

Modified Date: Mon, Jul 05, 2010 05:03:14 PM

Output Date: Tue, Jul 06, 2010 12:21:13 PM

Rev II

Project: Management Accounting_Debarshi Bhattacharyya ACE Pro India Pvt. Ltd. File: X:\Pearson\Management Accounting_Debarshi Bhattacharyya\MAIN\M08\LAYOUT_M08\M08_DEBA_ISBN_EN_SE_C08.indd

612

MANAGEMENT ACCOUNTING

The company worked for 17,500 direct labour hours during the year. For 2,500 of these hours, the company paid at Rs. 12 per hour, while for the remaining the wages were paid at the standard rate. Calculate material price and usage variances, and labour rate and efficiency variances. [C.A. (Inter)—May 1986] Ans.: MPV—Rs. 19,100 (A); MUV—Rs. 500 (A); LRV—Rs. 10,000 (A); LEV—Rs. 4,000(F). 33. From the following particulars given, compute: MPV, MUV, LRV, LITV and LEV with full working details: 1 tonne of material input yields a standard output of 1,00,000 units. The standard price of material is Rs. 20 per kg. Number of employees engaged is 200. The standard wage rate per employee per day is Rs. 6. The standard daily output per employee is 100 units. The actual quantity of materials used is 10 tonnes and the actual price paid is Rs. 21 per kg. Actual output obtained is 9,00,000 units. Actual number of days worked is 50 and actual rate of wages paid is Rs. 6.50 per day. Idle time paid for and included in the above time is half day. [C.A. (Inter) – May 1996] Ans.: MPV—Rs. 10,000 (A); MUV—Rs. 20,000 (A); LRV—Rs. 5,000 (A); LITV—Rs. 600 (A); LEV—Rs. 6,000 (A). D. Overhead Variances:

34. Budgeted Production Standard hours per unit Actual Production Actual Working hours

88 units 10 75 units 600

From the above figures, calculate: (i) Efficiency ratio; (ii) Activity ratio; (iii) Capacity ratio. [B.Com. (Hons), Delhi University—Model Paper—2002] Ans.: (i) 125%; (ii) 85.23%; (iii) 68.18%. 35. A company has a normal capacity of 120 machines working 8 hours per day of 25 days in a month. The fixed overheads are budgeted at Rs. 1,44,000 per month. The standard time required to manufacture one unit of production is 4 hours. In April 2006, the company worked for 24 days of 840 machine hours per day and produced 5,305 units of output. The actual fixed overheads were Rs. 1,42,000. Compute: (i) Efficiency variance; (ii) Capacity variance; (iii) Calendar variance. [B.Com. (Hons), Delhi University—Model Paper—2005] Ans.: (i) Rs. 6,360 (F); (ii) Rs. 17,280 (A); (iii) Rs. 5,760 (A). 36. For the Department Y of a plant, the following data are submitted for the week that ended on 31 March 2010: Standard Output for 40 hours per week Budgeted Fixed Overheads Actual Output Actual hours Worked Actual Fixed Overheads

1,400 units Rs. 1,400 1,200 units 32 units Rs. 1,500

You are required to calculate the relevant variances. Ans.: FOExp.V—Rs. 100 (A); FOCap.V—Rs. 280 (A); FOEff.V—Rs. 80 (F); FOVol.V—Rs. 200 (A). 37. AB Co. Ltd is having a Standard Costing System in operation for quite some time. The following data relating to the month of April 1994 are available from the cost records: Output (units) Operating hours Fixed Overheads (Rs.) Variable Overheads (Rs.) Working Days

Modified Date: Mon, Jul 05, 2010 05:03:14 PM

Budgeted 30,000 30,000 45,000 60,000 25

Output Date: Tue, Jul 06, 2010 12:21:13 PM

Actual 32,500 33,000 50,000 68,000 26

Rev II

Project: Management Accounting_Debarshi Bhattacharyya ACE Pro India Pvt. Ltd. File: X:\Pearson\Management Accounting_Debarshi Bhattacharyya\MAIN\M08\LAYOUT_M08\M08_DEBA_ISBN_EN_SE_C08.indd

STANDARD COSTING AND VARIANCE ANALYSIS

613

You are required to workout the relevant variances. [I.C.W.A. (Inter) – June 1994] Ans.: VOVs: Exp.—Rs. 2,000 (A); Eff.—Rs. 1,000 (A). FOVs: Exp.—Rs. 5,000 (A); Vol.—Rs. 3,750 (F); Cap.—Rs. 2,700 (F); Cal.—Rs. 1,800 (F); Eff.—Rs. 750 (A); Cost—Rs. 1,250 (A). 38. Following information is available from the records of a company: Budget 10,000 2,000 10 −

Fixed Overhead for March (Rs.) Production in March (units) Standard Time per unit (hours) Actual hours Worked in March

Actual 12,000 2,100 − 22,000

Compute: (i) FOCV; (ii) Exp.V; (iii) Vol.V; (iv) Cap.V; (v) Eff.V. Ans.: (i) Rs. 1,500 (A); (ii) Rs. 2,000 (A); (iii) Rs. 500 (F); (iv) Rs. 1,000 (F); (v) Rs. 500 (A). 39. Overhead Cost Variance Overhead Volume Variance Budgeted hours Budgeted Overhead Actual Rate of Recovery of Overhead

Rs. 1,400 (A) Rs. 1,000 (A) 1,200 hrs Rs. 6,000 Rs. 8 per hr

Calculate: (a) Overhead expenditure variance. (b) Actual overhead incurred. (c) Actual hours taken for actual production. (d) Overhead capacity variance. (e) Overhead efficiency variance. (f) Standard hours for actual production. Ans.: (a) Rs. 400 (A); (b) Rs. 6,400; (c) 800 hrs; (d) Rs. 2,000; (A); (e) Rs. 1,000; (f) 1,000 hrs. E. Sales Variances:

40. From the following information about sales, calculate: (a) Total sales variance, (b) Sales price variance, (c) Sales volume variance, (d) Sales mix variance, (e) Sales quantity variance. Product

Units

A B C

5,000 4,000 3,000

Standard Rate (Rs.) 5 6 7

Units 6,000 5,000 4,000

Actual Rate (Rs.) 6 5 8

[B.Com. (Hons), Delhi University—2008] Ans.: (a) Rs. 23,000 (F); (b) Rs. 5,000 (F); (c) Rs. 18,000 (F); (d) Rs. 500 (F); (e) Rs. 17,500 (F). 41. From the following information of a company, calculate: (a) Sales variance; (b) Sales volume variance; (c) Sales price variance. Budgeted Sales Value Units (Rs.) 100 1,200 50 600 100 900 75 450 325 3,150

Product P Q R S

Actual Sales Value Units (Rs.) 100 1,100 50 600 200 1,700 50 300 400 3,700

Ans.: (a) Rs. 550 (F); (b) Rs. 750 (A); (c) Rs. 200 (A).

Modified Date: Mon, Jul 05, 2010 05:03:14 PM

Output Date: Tue, Jul 06, 2010 12:21:13 PM

Rev II

Project: Management Accounting_Debarshi Bhattacharyya ACE Pro India Pvt. Ltd. File: X:\Pearson\Management Accounting_Debarshi Bhattacharyya\MAIN\M08\LAYOUT_M08\M08_DEBA_ISBN_EN_SE_C08.indd

614

MANAGEMENT ACCOUNTING

42. Budgeted and actual sales for the month of April 2005 of two products X and Y of a company were as follows: Units

Product X

6,000

Y

10,000

Budgeted Price per unit (Rs.) 5.00 2.00

units 5,000 1,500 7,500 1,750

Actual Price per unit (Rs.) 5.00 4.75 2.00 1.90

Budgeted cost for products X and Y were Rs. 4.00 and Rs. 1.50 per unit, respectively. Calculate the following variances: (a) Sales value variance; (b) Sales volume variance; (c) Sales price variance; (d) Sales mix variance; (e) Sales quantity variance. Ans.: (a) Rs. 450 (F); (b) Rs. 1,000 (F); (c) Rs. 550 (A); (d) Rs. 1,781 (F) (e) Rs. 781 (F). 43. X Ltd had budgeted the following sales for the month of August 2004: Product A: 800 units @ Rs. 100 per unit. Product B: 700 units @ Rs. 200 per unit. The actual sales for the month were as follows: Product A: 900 units @ Rs. 110 per unit. Product B: 800 units @ Rs. 180 per unit. The costs per unit of products A and B were Rs. 80 and Rs. 170 respectively. You are required to compute the different variances to explain the difference between the budgeted and actual profits. [CIMA—London—Adapted] Ans.: TSMV—Rs. 2,000 (A); SMPV—Rs. 7,000 (A); SMVV—Rs. 5,000 (F); SMMV—Rs. 68 (F); SMQV—Rs. 4,932 (F). 44. A company uses Standard Costing System. The sales data for a period are as follows:

A

Budgeted Sales (units) 1,280

Budgeted Selling Price per unit (Rs.) 20

Annual Sales (units) 650

Actual Sales Value (Rs.) 12,350

B C

3,200 1,920

12 6

3,900 1,950

50,700 29,250

A Rs. 16 18

B Rs. 10 12

C Rs. 13 13

Product

The cost data are as follows: Standard Cost per unit Actual Cost per unit

You are required to calculate the following for the period: (a) Gross margin total sales variance; (b) Gross margin sales volume variance; (c) Gross margin sales mix variance; (d) Gross margin sales quantity variance; (e) Sales price variance; (f) Total cost variance. [C.A. (Inter)—May 2005] Ans.: (a) Rs. 270 (F); (b) Rs. 1,030 (A); (c) Rs. 1,300 (A); (d) Rs. 270 (F); (e) Rs. 1,300 (F); (f) Rs. 9,100 (A).

Modified Date: Mon, Jul 05, 2010 05:03:14 PM

Output Date: Tue, Jul 06, 2010 12:21:13 PM

Rev II

Project: Management Accounting_Debarshi Bhattacharyya ACE Pro India Pvt. Ltd. File: X:\Pearson\Management Accounting_Debarshi Bhattacharyya\MAIN\M09\LAYOUT_M09\M09_DEBA_ISBN_EN_SE_C09.indd

Absorption Costing and Variable Costing

9

LEARNING OBJECTIVES On completion of the study of the chapter, you should be able to understand: What is Absorption Costing? Features, advantages and limitations of Absorption Costing. What is Variable Costing? Features, advantages and limitations of Variable Costing. Distinction between Absorption Costing and Variable Costing. How profit is determined under Absorption Costing and Variable Costing?

9.1 ABSORPTION COSTING Absorption Costing may be defined as the technique which takes into account both Fixed and Variable Costs for determining unit cost of the goods produced or the operation carried on. The Chartered Institute of Management Accountants (CIMA), United Kingdom, defines Absorption Costing as ‘the practice of charging all costs, both variable and fixed, to operations, processes or products’. Under this technique, the cost per unit of the goods produced represents the Variable Cost per unit plus the allocated share of Fixed Cost per unit. Here, ‘Absorbed Cost’ is the total of direct cost and overheads cost. Accordingly, Absorption Costing is also called Full or Total Costing. Wherever this technique of costing is applied, the cost per unit remains the same only when the level of output remains constant. When the level of output changes, the cost per unit also changes because of the existence of Fixed Cost in the unit cost. The Fixed Cost remains constant even when there is a change in the output level. Under this technique, closing inventory is also valued at total cost, which includes Variable Cost as well as Fixed Cost. 9.2 FEATURES OF ABSORPTION COSTING Distinguished features of Absorption Costing are as follows: i. Under this technique, both Fixed as well as Variable Costs are allocated to the Cost Unit. ii. Under it, the overhead costs are allocated to Cost Unit with the help of the overhead absorption rate. iii. Under it, the closing inventories are also valued at both Fixed as well as Variable Costs. iv. Under this technique, the periodic profit is affected by a change in the inventory as well as in the volume of sales. Accordingly, the profit may be shifted from one accounting period to another by increasing or decreasing the Inventories. v. Under it, the profit becomes a function of production instead of sales.

Modified Date: Tue, Jun 29, 2010 05:09:32 PM

Output Date: Tue, Jul 06, 2010 12:22:53 PM

Rev II

Project: Management Accounting_Debarshi Bhattacharyya ACE Pro India Pvt. Ltd. File: X:\Pearson\Management Accounting_Debarshi Bhattacharyya\MAIN\M09\LAYOUT_M09\M09_DEBA_ISBN_EN_SE_C09.indd

616

MANAGEMENT ACCOUNTING

9.3 ADVANTAGES OF ABSORPTION COSTING Absorption Costing Technique has the following advantages: i. It recognizes the importance of incorporating the Fixed Cost into the Unit Cost of production. ii. Unit price as determined under Absorption Costing technique ensures that all costs are included in it. iii. It reduces the work-burden of segregation of cost into fixed and variable parts. iv. It confirms the accrual concepts by matching the costs with the revenue for a particular period. v. Under this costing technique, the inventory can be valued at the total cost. vi. Identification of an effective utilization of production resources is possible by application of Absorption Costing. vii. It helps to calculate Gross Profit and Net Profit separately in the Income Statement for an accounting period. 9.4 LIMITATIONS OF ABSORPTION COSTING In spite of having a number of important advantages, Absorption Costing technique suffers from the following limitations: i. It can be effectively used only at a constant level of output to get a uniform unit cost. Under this technique, unit costs would be different for different levels of output. ii. As the unit costs as determined by this technique under different levels of output are different, the process of cost comparison and cost control becomes difficult. iii. In Absorption Costing, a portion of Fixed Cost is carried forward to the next accounting period as the closing inventory is valued at the total cost which includes a proportionate Fixed Cost. iv. Most of the accountants opine that Fixed Costs are period costs and they do not produce future benefits of the concerns and hence, should not be included in the unit cost of production. v. Under this costing technique, a portion of Fixed Cost is not charged against the revenue of the period in which they are incurred due to the inclusion of Fixed Cost in the closing inventory. Thus, the profit for an accounting period cannot be accurately computed under this technique. vi. Absorption Cost deals with production cost alone and ignores administration, selling and distribution costs. vii. It is not helpful for managerial decision making process, such as selection of suitable product mix, temporary shut down of plant, suspension of production activity if required or not, choice between various alternatives or pricing decision. 9.5 VARIABLE COSTING Variable Costing is also called ‘Marginal Costing’. It may be defined as the technique which charges only the Variable Cost to the cost unit. According to the Chartered Institute of Management Accountants (CIMA), United Kingdom, Marginal Costing is ‘the technique of ascertainment of Marginal Cost and of the effect on profit of changes in volume by differentiating between Fixed and Variable Costs.’ CIMA also defines Marginal Cost as ‘the cost for producing one additional unit of the product.’ Generally, the cost of a product includes the Fixed and Variable Costs. If one unit of output of the product is increased, only the Variable Cost increases, as the Fixed Cost remains constant even in the change of output level. Therefore, the additional cost incurred for producing one additional unit represents the Variable Cost only. This increase in cost (i.e., the Variable Cost) due to increase in one additional unit of output is called Marginal Cost. Hence, Marginal Cost is nothing but Variable Cost. That is why Variable Costing is also called Marginal Costing.

Modified Date: Tue, Jun 29, 2010 05:09:32 PM

Output Date: Tue, Jul 06, 2010 12:22:53 PM

Rev II

Project: Management Accounting_Debarshi Bhattacharyya ACE Pro India Pvt. Ltd. File: X:\Pearson\Management Accounting_Debarshi Bhattacharyya\MAIN\M09\LAYOUT_M09\M09_DEBA_ISBN_EN_SE_C09.indd

ABSORPTION COSTING AND VARIABLE COSTING

617

9.6 FEATURES OF VARIABLE COSTING Distinguished features of Variable Costing are as follows: i. Under Variable Costing technique, only Variable Costs are charged to the cost unit. Fixed Costs are recovered from contribution. ii. Under this costing technique, all costs including Semi-variable Costs are divided into two parts, such as fixed and variable, on the basis of their variability with respect to the change in the levels of output. iii. Under this technique, the closing inventories are valued at Variable Cost only. iv. Under it, the selling price is based on the Variable Cost plus contribution. v. Under this technique, profit is calculated in two parts: (a) Contribution = Sales – Variable Cost; and (b) Profit = Contribution – Fixed Cost. vi. It is an important managerial technique for decision-making and cost control. vii. Break-even Analysis and Cost–Volume–Profit (CVP) analysis are integral parts of this costing technique. 9.7 ADVANTAGES OF VARIABLE COSTING Variable Costing technique has the following advantages: i. It provides useful data for the managerial decision-making. ii. It is a very effective tool of profit planning. iii. It facilitates control over Variable Costs by avoidance of arbitrary apportionment or allocation of Fixed Costs. iv. Under this technique, the problems on computation of accurate fixed factory overhead rate can be avoided as fixed overheads are charged against contribution. v. Under this technique, as the valuation of closing inventory is done at a Variable Cost only, the problem of illogically carrying forward the Fixed Cost of one period to the next period through closing inventories can be avoided. vi. It provides useful information to the management for pricing the product of the enterprise. Since the Variable Cost per unit is constant from period to period within a short span of time, the decision on pricing the product can easily be taken by the management. vii. It helps the management in the process of cost control by concentrating on the Variable Cost alone, as Fixed Costs are non-controllable in the short span of time. viii. It provides the management with many useful techniques for decision-making, like Break-even Analysis, Profit–Volume (PV) Ratio, Variable Cost Ratio. ix. It ensures a greater control over the cost through Standard Costing and budgetary control. x. Use of Variable Cost technique is very simple and easy to understand and operate. 9.8 LIMITATIONS OF VARIABLE COSTING In spite of having important advantages, Variable Costing technique suffers from the following limitations: i. It assumes that Semi-Variable Costs can be segregated into two parts, fixed and variable elements. In practice, however, such segregation of Semi-Variable Costs is a very difficult task. ii. It excludes Fixed Cost for decision-making, which sometimes may lead to a wrong conclusion. iii. With the development of technology and application of automation in the industry, Fixed Costs have substantially increased and their impact on production cost is no longer ignorable nowadays. Thus, the Variable Costing technique fails to reflect the impact of these costs on the production cost, as it focuses a greater emphasis on the Variable Cost only in determining the Product Cost. iv. Variable Cost technique cannot be successfully applied in ‘Cost Plus Contract’ unless an adequate percentage over the Variable Cost is charged from the contractee to cover the Fixed Cost and profit.

Modified Date: Tue, Jun 29, 2010 05:09:32 PM

Output Date: Tue, Jul 06, 2010 12:22:53 PM

Rev II

Project: Management Accounting_Debarshi Bhattacharyya ACE Pro India Pvt. Ltd. File: X:\Pearson\Management Accounting_Debarshi Bhattacharyya\MAIN\M09\LAYOUT_M09\M09_DEBA_ISBN_EN_SE_C09.indd

618

MANAGEMENT ACCOUNTING

v. As the Variable Costing technique does not provide any specific parameter for control, cost control can be better achieved with the help of Standard Costing and Budgetary Control than with the Variable Costing technique. vi. It is not very useful for valuation of inventory in case of abnormal loss and such valuation is not accepted from the point of view of the income tax authorities and auditors. vii. It does not resolve the difficulties that are involved in the apportionment of variable overheads. viii. It does not reflect the impact of time factor and the value of investment that is made for the determination of Product Cost. 9.9 INCOME DETERMINATION UNDER ABSORPTION COSTING AND VARIABLE COSTING Net income (i.e., Net Profit) as computed under Variable Costing technique always moves in the same direction as sales volume. On the other hand, the net income as computed under Absorption Costing technique does not move in the same direction as sales volume, but at times it moves in the opposite direction from Sales. This happens due to the impact of existence of inventories as valued under the Absorption Costing technique in the Income Statement. The net income under both the techniques becomes equal when the sales volume is equal to the production volume, that is, there is no opening or closing inventories. If the sales volume is not equal to the production volume, the net income as computed under these two techniques would not be equal. The relationship between the net income as determined under these two techniques may be summarized as follows: i. When the production quantity is equal to the sales quantity (i.e., no opening or closing inventories), then the net income computed under Variable Costing and Absorption Costing techniques would be the same. ii. When there is opening inventory only and no closing inventory (i.e., the situation when the sales volume exceeds the production volume), the net income as computed under Variable Costing technique becomes higher than the net income as ascertained under the Absorption Costing technique. iii. When there is closing inventory only and no opening inventory (i.e., the situation when the production volume exceeds the sales volume), the net income as computed under the Absorption Costing technique becomes higher than the net income as ascertained under the Variable Costing technique. iv. When there are both opening and closing inventories and the volume of the opening inventory is more than that of the closing inventory (i.e., the situation when the sales volume exceeds the production volume), the net income as computed under Variable Costing technique becomes higher than the net income as ascertained under the Absorption Costing technique, if both opening and closing inventories are valued at a uniform price. v. When there are both opening and closing inventories and the volume of the closing inventory is more than that of the opening inventory (i.e., the situation when the production volume exceeds the sales volume), then the net income as computed under the Absorption Costing technique becomes higher than the net income as ascertained under the Variable Costing technique, if both opening and closing inventories are valued at a uniform price.

Stop and Think The summarized analysis of net income under Absorption Costing and Variable Costing is as follows: i. When Production = Sales: Net income under Absorption Costing = Net income under Variable Costing. ii. When Production > Sales: Net income under Absorption Costing > Net income under Variable Costing. iii. When Production < Sales: Net income under Absorption Costing < Net income under Variable Costing.

Modified Date: Tue, Jun 29, 2010 05:09:32 PM

Output Date: Tue, Jul 06, 2010 12:22:53 PM

Rev II

Project: Management Accounting_Debarshi Bhattacharyya ACE Pro India Pvt. Ltd. File: X:\Pearson\Management Accounting_Debarshi Bhattacharyya\MAIN\M09\LAYOUT_M09\M09_DEBA_ISBN_EN_SE_C09.indd

619

ABSORPTION COSTING AND VARIABLE COSTING

9.9.1 Income Determination under Absorption Costing Technique Rs. Less:

Sales Factory Cost of Production: Direct Materials Direct Labour Direct Expenses Factory Overheads: Fixed (on Actual Production Basis) Variable

Add:

Opening Stock of Finished Goods

Less:

Closing Stock of Finished Goods

– – – – –

Add/Less: Under- or Over-Absorption of Fixed Factory Overheads Less:

Rs. –

Gross Profit Office & Administration and Selling & Distribution Overheads: Fixed Variable Net Profit

– –

– – – – – – – –

– –

9.9.2 Income Determination under Variable Costing Technique Rs. Less:

Sales Variable Cost

Less:

Fixed Cost

Contribution Profit

– – – – –

9.10 DISTINCTION BETWEEN ABSORPTION COSTING AND VARIABLE COSTING Absorption Costing 1. Both fixed and Variable Costs are charged to the product cost. 2. As fixed overheads are also charged to the product cost, these are arbitrarily apportioned to various products. 3. Closing inventories are valued at the total cost, which includes both fixed as well as Variable Cost. Accordingly, the value of inventory under Absorption Costing is higher than that under the Variable Costing. 4. Different unit product costs are obtained at different levels of output, even if the price level remains constant over the years. 5. The difference between the sales and the total cost represents profit. 6. Managerial decisions are guided by relative profit made by products.

Variable Costing 1. Only Variable Costs are charged to the product cost. Fixed Costs are recovered from the contribution. 2. As fixed overheads are not charged to the product cost, there would be no under- or over-recovery of fixed overheads. 3. Closing inventories are valued at variable or marginal cost. Accordingly, the value of the inventory under Variable Costing is lower than that under the Absorption Costing. 4. If the price level remains constant over the years, the unit product cost also remains constant at different levels of output. 5. The difference between the sales and the Variable Cost represents contribution and the difference between the contribution and the Fixed Cost represents profit. 6. Managerial decisions are guided by relative contribution made by products. (Continued)

Modified Date: Tue, Jun 29, 2010 05:09:32 PM

Output Date: Tue, Jul 06, 2010 12:22:53 PM

Rev II

Project: Management Accounting_Debarshi Bhattacharyya ACE Pro India Pvt. Ltd. File: X:\Pearson\Management Accounting_Debarshi Bhattacharyya\MAIN\M09\LAYOUT_M09\M09_DEBA_ISBN_EN_SE_C09.indd

620

MANAGEMENT ACCOUNTING

Absorption Costing 7. Costs are classified according to their functional elements as production cost, office and administration cost, and selling and distribution cost. 8. Absorption costing fails to establish the relationship between cost, volume and profit.

Variable Costing 7. Costs are classified according to their behaviour such as Fixed Cost and Variable Cost. 8. Variable Costing establishes a relationship between cost, volume and profit, which is an integral part of the Variable Costing itself.

Tutorial Notes to Students for Solving Problems i. Students shall be mainly asked to determine the profit by means of preparation of income statement as per Absorption Costing technique and Variable Costing technique, and a comparative study between the two profits has to be made. ii. It is to be noted that the Opening and Closing Stock of finished goods are to be valued, as per Absorption Costing technique, at the total production cost (i.e., fixed as well as variable production cost). On the other hand, the Opening and Closing Stock of finished goods are to be valued, as per Variable Costing technique, at a variable production cost only. iii. If not otherwise stated, the Closing Stock of finished goods is to be valued based on FIFO (First-in First-out). iv. Net income under these two costing techniques will always be different unless the production volume is equal to the sales volume. In other words, the profits calculated under Absorption Costing and Variable Costing become equal only when the production quantity is equal to the sales quantity, that is, when there is no Opening or Closing Stock of finished goods. v. If the production volume is higher than the sales volume, the profit as per Absorption Costing technique will be higher than that under the Variable Costing technique. On the other hand, if the sales volume is higher than the production volume, the profit as per Variable Costing technique will be higher than that under Absorption Costing technique.

9.11 WORKED-OUT PROBLEMS Problem 1 The following data have been extracted from the books of a company: Production & Sales Volume Selling Price per unit Materials per unit Labour per unit Production Overheads:

50,000 units Rs. 20 Rs. 8 Rs. 4

Variable Fixed Other Fixed Overheads

Rs. 3 per unit Rs. 75,000 Rs. 95,000

Prepare an Income Statement under: (i) Absorption Costing technique; and (ii) Variable Costing technique. Solution i.

Income Statement under Absorption Costing Rs.

Less:

Sales (50,000 × Rs. 20) Factory Cost of Production: Materials (50,000 × Rs. 8)

Rs. 10,00,000

4,00,000 (Continued)

Modified Date: Tue, Jun 29, 2010 05:09:32 PM

Output Date: Tue, Jul 06, 2010 12:22:53 PM

Rev II

Project: Management Accounting_Debarshi Bhattacharyya ACE Pro India Pvt. Ltd. File: X:\Pearson\Management Accounting_Debarshi Bhattacharyya\MAIN\M09\LAYOUT_M09\M09_DEBA_ISBN_EN_SE_C09.indd

621

ABSORPTION COSTING AND VARIABLE COSTING

Labour (50,000 × Rs. 4) Variable Production Overheads (50,000 × Rs. 3) Fixed Production Overheads Gross Income Other Fixed Overheads Net Income

Less:

ii.

Rs. 2,00,000 1,50,000 75,000

Rs.

8,25,000 1,75,000 95,000 80,000

Income Statement under Variable Costing Rs. Sales (50,000 × Rs. 20) Variable Cost: Materials (50,000 × Rs. 8) Labour (50,000 × Rs. 4) Variable Production Overheads (50,000 × Rs. 3)

Less:

4,00,000 2,00,000 1,50,000 Contribution

Less:

Fixed Cost: Fixed Production Overheads Other Fixed Overheads

75,000 95,000 Net Income

Rs. 10,00,000

7,50,000 2,50,000

1,70,000 80,000

Stop and Think In the given problem, as the production volume is equal to the sales volume (i.e., there is no Opening or Closing Stock), there is no difference between the net income as ascertained under both the costing techniques.

Problem 2 The following data have been obtained from the books of a company: Production Volume Sales Volume Selling Price per unit Materials per unit Labour per unit Production Overheads: Variable Fixed Administration & Selling Overheads: Variable Fixed

50,000 units 48,000 units Rs. 50 Rs. 20 Rs. 10 Rs. 6 per unit Rs. 2,00,000 Rs. 3 per unit Rs. 1,56,000

Prepare an Income Statement under: (i) Absorption Costing technique; and (ii) Marginal Costing technique. Solution i.

Income Statement under Absorption Costing Rs.

Less:

Sales (48,000 × Rs. 50) Factory Cost of Production: Materials (50,000 × Rs. 20)

Rs. 24,00,000

10,00,000 (Continued)

Modified Date: Tue, Jun 29, 2010 05:09:32 PM

Output Date: Tue, Jul 06, 2010 12:22:53 PM

Rev II

Project: Management Accounting_Debarshi Bhattacharyya ACE Pro India Pvt. Ltd. File: X:\Pearson\Management Accounting_Debarshi Bhattacharyya\MAIN\M09\LAYOUT_M09\M09_DEBA_ISBN_EN_SE_C09.indd

622

MANAGEMENT ACCOUNTING

Labour (50,000 × Rs. 10) Variable Production Overheads (50,000 × Rs. 6) Fixed Production Overheads Less:

Closing Stock of Finished Goods [ Rs. 20,00,000 × 2,000 units / 50,000 units ]

Rs. 5,00,000 3,00,000 2,00,000 20,00,000 80,000

19,20,000 4,80,000

Gross Income Less:

Administration & selling overheads: Variable (48,000 × Rs. 3) Fixed

1,44,000 1,56,000 Net Income

ii.

Rs.

3,00,000 1,80,000

Income Statement under Marginal Costing Rs.

Less:

Sales (48,000 × Rs. 50) Variable Cost: Materials (50,000 × Rs. 20) Labour (50,000 × Rs. 10) Variable Production Overheads (50,000 × Rs. 6) Less:

Closing Stock of Finished Goods [ Rs. 18,00,000 × 2,000 units / 50,000 units ]

Add:

Variable Administration & Selling Overheads (48,000 × Rs. 3)

10,00,000 5,00,000 3,00,000 18,00,000 72,000 17,28,000 1,44,000 18,72,000 5,28,000

Contribution Less:

Fixed Production Overheads Fixed Administration & Selling Overheads

2,00,000 1,56,000 Net Income

Rs. 24,00,000

3,56,000 1,72,000

Stop and Think In the given problem, as the production volume is more than the sales volume (i.e., there is no Opening Stock but only Closing Stock), the net income as ascertained under Absorption Costing is higher than that under the Variable Costing. This happens due to the overvaluation of Closing Stock under Absorption Costing (or undervaluation of Closing Stock under Variable Costing). Under Absorption Costing, the Closing Stock of finished goods is valued at the total factory cost (i.e., the fixed factory cost plus the variable factory cost), whereas under Variable Costing, it is valued at variable factory cost only, which results in the difference in net income under these two costing techniques.

Problem 3 The following data have been obtained from the books of a company: Production Volume Sales Volume Opening Stock Volume Value of Opening Stock

20,000 units 25,000 units 10,000 units Rs. 95,000 (including Variable Cost Rs. 70,000) Rs. 20

Selling Price per unit

(Continued)

Modified Date: Tue, Jun 29, 2010 05:09:32 PM

Output Date: Tue, Jul 06, 2010 12:22:53 PM

Rev II

Project: Management Accounting_Debarshi Bhattacharyya ACE Pro India Pvt. Ltd. File: X:\Pearson\Management Accounting_Debarshi Bhattacharyya\MAIN\M09\LAYOUT_M09\M09_DEBA_ISBN_EN_SE_C09.indd

623

ABSORPTION COSTING AND VARIABLE COSTING

Variable Production Cost per unit Fixed Production Overheads Administration & Selling Overheads: Variable Fixed

Rs. 12 Rs. 60,000 Re. 1 per unit Rs. 55,000

Prepare an Income Statement under: (i) Absorption Costing technique; and (ii) Variable Costing technique. Solution i.

Income Statement under Absorption Costing Rs.

Less:

Sales (25,000 × Rs. 20) Factory Cost of Production: Variable Production Cost (20,000 × Rs. 12) Fixed Production Overheads Add:

Opening Stock of Finished Goods3

Less:

Closing Stock of Finished Goods2 [Rs. 3,00,000 × 5,000 units1 / 20,000 units]

2,40,000 60,000 3,00,000 95,000 3,95,000 75,000 3,20,000 1,80,000

Gross Income Less:

Administration & Selling Overheads: Variable (25,000 × Rs. 1) Fixed

25,000 55,000 Net Income

ii.

Sales (25,000 × Rs. 20) Variable Production Cost (20,000 × Rs. 12) Add: Opening Stock of Finished Goods3 Less:

Add:

Rs. 5,00,000

Closing Stock of Finished Goods2 [Rs. 2,40,000 × 5,000 units1 / 20,000 units]

2,40,000 70,000 3,10,000 60,000 2,50,000 25,000

Variable Administration & Selling Overheads (25,000 × Re. 1) Contribution

Less:

80,000 1,00,000

Income Statement under Variable Costing Rs.

Less:

Rs. 5,00,000

Fixed Production Overheads Fixed Administration & Selling Overheads

60,000 55,000 Net Income

2,75,000 2,25,000 – 1,15,000 1,10,000

 Working Notes 1. Closing Stock of Finished Goods (in Volume) = Production Volume + Opening Stock Volume – Sales Volume = 20,000 + 10,000 – 25,000 = 5,000 units. 2. Under both the above costing techniques, the closing stock is valued based on FIFO. 3. Under Absorption Costing, the Opening Stock is taken at the total cost, but under Variable Costing, it is taken at variable cost only.

Modified Date: Tue, Jun 29, 2010 05:09:32 PM

Output Date: Tue, Jul 06, 2010 12:22:53 PM

Rev II

Project: Management Accounting_Debarshi Bhattacharyya ACE Pro India Pvt. Ltd. File: X:\Pearson\Management Accounting_Debarshi Bhattacharyya\MAIN\M09\LAYOUT_M09\M09_DEBA_ISBN_EN_SE_C09.indd

624

MANAGEMENT ACCOUNTING

Stop and Think In the given problem, as the sales volume is more than the production volume and the Opening and Closing Stock are taken at different values under Absorption Costing and Variable Costing techniques, the net income as ascertained under these two techniques become different. This happens due to the overvaluation of Opening and Closing stock under Absorption Costing (or undervaluation of Opening and Closing Stock under Variable Costing). Under Absorption Costing, the Opening and Closing Stock of Finished Goods are valued at total factory cost (i.e., fixed factory cost plus variable factory cost), whereas under Variable Costing, these are valued at only variable factory cost, which results in the difference in the net income under these two costing techniques.

Problem 4 A company is having a production capacity of 1,00,000 units per year. The normal capacity utilization is reckoned at 80%. The standard variable production costs are Rs. 15 per unit and fixed production costs are Rs. 3,20,000 per year. The variable selling costs are Rs. 4 per unit and fixed selling costs are Rs. 1,60,000 per year. The unit selling price is Rs. 30. During the year that ended on 31 March 2009, 75,000 units were produced and 70,000 units were sold. The closing inventory on 31 March 2009 was 10,000 units. The actual variable production cost for the year was Rs. 75,000 which was higher than the standard. Prepare an income statement under: (i) Absorption Costing technique; and (ii) Variable Costing technique. [B. Com. (Hons.), Delhi University—Adapted] Solution i. Income Statement under Absorption Costing Rs. Less:

Sales (70,000 × Rs. 30) Factory Cost of Production: Variable Production Cost (75,000 × Rs. 15) Increase in Variable Production Cost Fixed Production Cost (75,000 × Rs. 4) Add:

Opening Stock of Finished Goods1

Less:

Closing Stock of Finished Goods2 [ Rs. 15,00,000 × 10,000 units / 75,000 units ]

Add:

Less:

Under-Absorption of Fixed Production Cost (Rs. 3,20,000 – Rs. 3,00,000) Gross Income Selling Cost: Variable (70,000 × Rs. 4) Fixed Net Income

Rs. 21,00,000

11,25,000 75,000 3,00,000 15,00,000 95,000 15,95,000 2,00,000 13,95,000 20,000 14,15,000 6,85,000 2,80,000 1,60,000

4,40,000 2,45,000

ii. Income Statement under Variable Costing Rs. Less:

Sales (70,000 × Rs. 30) Variable Costs: Variable Production Cost (75,000 × Rs. 15) Increase in Variable Production Cost

Rs. 21,00,000

11,25,000 75,000 12,00,000 (Continued)

Modified Date: Tue, Jun 29, 2010 05:09:32 PM

Output Date: Tue, Jul 06, 2010 12:22:53 PM

Rev II

Project: Management Accounting_Debarshi Bhattacharyya ACE Pro India Pvt. Ltd. File: X:\Pearson\Management Accounting_Debarshi Bhattacharyya\MAIN\M09\LAYOUT_M09\M09_DEBA_ISBN_EN_SE_C09.indd

625

ABSORPTION COSTING AND VARIABLE COSTING

Add: Opening Stock of Finished Goods (5,000 units × Rs. 15)

Rs. 75,000

1

12,75,000 1,60,000

Less: Closing Stock of Finished Goods2 [ Rs. 12,00,000 × 10,000 units1/75,000 units ] Add:

11,15,000 2,80,000

Variable Selling Cost (70,000 × Rs. 4)

13,95,000 7,05,000

Contribution Less:

Rs.

Fixed Costs: Fixed Production Cost Fixed Selling Cost

3,20,000 1,60,000 Net Income

4,80,000 2,25,000

Working Notes 1. Opening Stock of Finished Goods (in volume) = Sales Volume + Closing Stock Volume – Production Volume = 70,000 + 10,000 – 75,000 = 5,000 units Fixed Production Cost per unit at Normal Capacity Utilization = Rs. 3,20,000 / (80% of 1,00,000 units) = Rs. 4 Value of Opening Stock under Absorption Costing = 5,000 units × (Rs. 15 + Rs. 4) = Rs. 95,000 Value of Opening Stock under Variable Costing = 5,000 units × Rs. 15 = Rs. 75,000 Under Absorption Costing, the Opening Stock is taken at the Total Cost, but under Variable Costing, it is taken at the Variable Cost only. 2. Under both the above costing techniques, the Closing Stock is valued based on FIFO. Under Absorption Costing, the Closing Stock is taken at the total cost, but under Variable Costing, it is taken at the variable cost only.

Stop and Think In the given problem, as the production volume is more than the sales volume and the Opening and Closing Stocks are taken at different values under Absorption Costing and Variable Costing techniques, the net Income as ascertained under these two techniques become different. This happens due to the overvaluation of Opening and Closing Stocks under Absorption Costing (or under-valuation of Opening and Closing Stocks under Variable Costing). Under Absorption Costing, the Opening and Closing Stocks of finished goods are valued at the total factory cost (i.e., fixed factory cost plus variable factory cost), whereas under Variable Costing, these are valued at variable factory cost only, which results in the difference in net income under these two costing techniques.

Problem 5 The following data relate to a company which makes and sells computers. Production Volume Sales Volume Selling Price per unit Variable Production Cost per unit Fixed Production Overheads Incurred Fixed Production Overheads per unit, being predetermined Overhead Absorption rate Selling, Distribution & Administration Cost (all fixed)

Modified Date: Tue, Jun 29, 2010 05:09:32 PM

Output Date: Tue, Jul 06, 2010 12:22:53 PM

March 2009 10,000 units 5,000 units Rs. 100 Rs. 50 Rs. 1,00,000 Rs. 10

April 2009 5,000 units 10,000 units Rs. 100 Rs. 50 Rs. 1,00,000 Rs. 10

Rs. 50,000

Rs. 50,000

Rev II

Project: Management Accounting_Debarshi Bhattacharyya ACE Pro India Pvt. Ltd. File: X:\Pearson\Management Accounting_Debarshi Bhattacharyya\MAIN\M09\LAYOUT_M09\M09_DEBA_ISBN_EN_SE_C09.indd

626

MANAGEMENT ACCOUNTING

You are required to present a Comparative Profit Statement for each month using: (i) Absorption Costing technique; and (ii) Marginal Costing technique. Comment on the following statement using the figures contained in your answer: (iii) Marginal Costing rewards sales whereas Absorption Costing rewards production. Solution i.

Less:

Income Statement under Absorption Costing

Sales (5,000 × Rs. 100) & (10,000 × Rs. 100) Factory Cost of Production: Variable Production Cost (10,000 × Rs. 50) & (5,000 × Rs. 50) Fixed Production Overheads (10,000 × Rs. 10) & (5,000 × Rs. 10) Add: Opening Stock of Finished Goods1 (Rs. 6,00,000 × 5,000 units / 10,000 units) Less: Closing Stock of Finished Goods1 [Rs. 6,00,000 × 5,000 units1 / 10,000 units]

Less:

ii.

Less:

Add: Under-Absorption of Fixed Production Overhead (Rs.1,00,000 – Rs.1,00,000) & (Rs.1,00,000 – Rs.50,000) Gross Income Selling, Distribution & Administration Costs Net Income

March 2009 Rs. in ‘000 Rs. in ‘000 500

500

250

100

50

600 –

300 300

600 300

600 –

300 –

600 50 300 200 50 150

650 350 50 300

Income Statement under Marginal Costing

Sales (5,000 × Rs. 100) & (10,000 × Rs. 100) Variable Production Cost (10,000 × Rs. 50) & (5,000 × Rs. 50) Add: Opening Stock of Finished Goods1 [Rs. 5,00,000 × 5,000 units1 / 10,000 units] Less: Closing Stock of Finished Goods1 [Rs. 5,00,000 × 5,000 units1 / 10,000 units]

March 2009 Rs. in ‘000 Rs. in ‘000 500

Fixed Costs: Fixed Production Overheads Fixed Selling, Distribution & Administration Costs

April 2009 Rs. in ‘000 Rs. in ‘000 1,000

500

250



250

500 250

500 – 250 250

Contribution Less:

April 2009 Rs. in ‘000 Rs. in ‘000 1,000

100 50

Net Income

500 500 100 50

150 100

150 350

iii. From the above calculation, it has been clearly observed that in April 2009, a higher profit is ascertained under Marginal Costing technique than that under Absorption Costing technique, and in March 2009, a higher profit is ascertained under Absorption Costing technique than that under Marginal Costing

Modified Date: Tue, Jun 29, 2010 05:09:32 PM

Output Date: Tue, Jul 06, 2010 12:22:53 PM

Rev II

Project: Management Accounting_Debarshi Bhattacharyya ACE Pro India Pvt. Ltd. File: X:\Pearson\Management Accounting_Debarshi Bhattacharyya\MAIN\M09\LAYOUT_M09\M09_DEBA_ISBN_EN_SE_C09.indd

ABSORPTION COSTING AND VARIABLE COSTING

627

technique. It may be seriously noted that in April 2009, when the sales quantity exceeds the production quantity, a higher profit is ascertained under Marginal Costing technique and a lower profit is ascertained under Absorption Costing technique. Again in March 2009, when the production quantity exceeds the sales quantity, a higher profit is ascertained under Absorption Costing technique and a lower profit is ascertained under Marginal Costing technique. Therefore, on the basis of the above information, it can be concluded that when sales are in excess of production, Marginal Costing produces higher profit and when production exceeds sales, Absorption Costing produces higher profit.  Working Notes 1. Closing Stock of Finished Goods of March 2009 (in volume) = Production Volume – Sales Volume = 10,000 – 5,000 = 5,000 units. 2. Opening Stock of Finished Goods of April 2009 (in volume) = Closing Stock of Finished Goods of March 2009 = 5,000 units. 3. Under Absorption Costing, the stock is valued at the total cost, but under Variable Costing, it is taken at the variable cost only.

CHAPTER REVIEW SUMMARY  Absorption Costing may be defined as ‘the technique which takes into account both fixed and variable costs for determining unit cost of the goods produced or the operation carried on’. It is also called Full Costing or Total Costing.  Distinguished features of Absorption Costing are: (a) Both fixed and variable costs are allocated to cost unit; (b) Overhead costs are allocated to cost unit with the help of Overhead Absorption Rate; and (c) The closing inventories are also valued at both Fixed and Variable Costs.  Absorption Costing has many advantages: (a) It recognizes the importance of incorporating fixed cost into the unit cost of production; (b) It reduces the work-burden of segregation of cost into fixed and variable parts; (c) Inventory can be valued at total cost; and (d) It helps to calculate Gross Profit and Net Profit separately in the income statement for an accounting period.  Limitations of Absorption Costing are: (a) It can be effectively used only at a constant level of output to get a uniform unit cost. The unit costs would be different for different levels of output; (b) A portion of fixed cost is carried forward to the next accounting period as the closing inventory is valued at the total cost which includes a proportionate fixed cost; (c) Most of the accountants opine that such fixed costs do not produce future benefits and hence, should not be included in the unit cost of production; and (d) It is not helpful for any managerial decision making process.  Variable Costing is also called Marginal Costing. It may be defined as ‘the technique which charges only the variable cost to the cost unit.’ The additional cost incurred for producing one additional unit represents the variable cost only. This increase in cost (i.e., variable cost) due to an increase in one additional unit of output is called marginal cost. Hence, marginal cost is nothing but variable cost.  Distinguished features of Variable Costing are: (a) Only variable costs are charged to the cost unit. fixed costs are recovered from contribution; (b) All costs including semi-variable costs are divided into two parts — fixed and variable; (c) closing inventories are valued at variable cost only; and (d) Break-even analysis and CVP analysis are integral parts of this costing technique.  Variable Costing technique has many advantages: (a) It provides useful data for managerial decision-making; (b) It is a very effective tool of profit planning; (c) It facilitates control over variable costs by avoidance of arbitrary apportionment or allocation of fixed costs; (d) Problems on computation of accurate fixed factory overhead rate can be avoided as fixed overheads are charged against contribution; and (h) It provides the management with many useful techniques for decision making like Break-even analysis.  Limitations of Variable Costing are: (a) It assumes that the semi-variable costs can be segregated into two parts—fixed and variable elements. In practice, however, such segregation of semi-variable costs is very difficult; (b) It excludes fixed cost for decision making, which sometimes may lead to wrong conclusions;

Modified Date: Tue, Jun 29, 2010 05:09:32 PM

Output Date: Tue, Jul 06, 2010 12:22:53 PM

Rev II

Project: Management Accounting_Debarshi Bhattacharyya ACE Pro India Pvt. Ltd. File: X:\Pearson\Management Accounting_Debarshi Bhattacharyya\MAIN\M09\LAYOUT_M09\M09_DEBA_ISBN_EN_SE_C09.indd

628





MANAGEMENT ACCOUNTING

(c) It fails to reflect the impact of increased fixed costs due to the development of technology on Production cost; and (d) Variable Cost technique cannot be successfully applied in ‘Cost Plus Contract.’ Net income under these two costing techniques becomes equal when the sales volume is equal to the production volume, i.e., there is no opening or closing inventories. If sales volume is not equal to the production volume, then the net incomes as computed under these two techniques would not be equal. Distinction between Absorption Costing and Variable Costing are: (a) Both fixed and variable costs are charged to the product cost under absorption costing whereas only variable costs are charged to the Product cost under variable costing; (b) Closing inventories are valued at total cost under absorption costing whereas these are valued at variable cost only under variable costing; (c) Costs are classified according to their functional elements under absorption costing whereas costs are classified according to their behaviour under variable costing; and (d) Absorption Costing fails to establish the relationship between cost, volume, and profit, whereas variable costing establishes a relationship between cost, volume, and profit, which is an integral part of variable costing.

CHAPTER REVIEW QUIZ 1. State whether the following statements are true or false: a. Absorption Costing is also called Marginal Costing. b. Marginal Cost is nothing but Variable Cost. c. Absorption Costing fails to establish a relationship between cost, volume, and profit. d. Variable Costing is also called Direct Costing. e. Under Variable Costing, the stock is valued at a Marginal Cost only. f. Under Absorption Costing, the stock is valued at the Total Cost. g. Under Absorption Costing, only Variable Costs are charged to the Product Cost. h. For a decision-making purpose, Variable Costing is more effective than Absorption Costing. i. Net income as determined under Absorption Costing and Variable Costing techniques will always be different unless the production volume is equal to the sales volume. j. In Absorption Costing, the profit becomes a function of sales instead of production. Ans.: True: (b), (c), (e), (f), (h), (i); False: (a), (d), (g), (j) . 2. Indicate the effect on net income as determined under Absorption Costing and Variable Costing in the following situations: a. When the Production Quantity is equal to the Sales Quantity. b. When the Sales Volume exceeds the Production Volume. c. When the Production Volume exceeds the Sales Volume. Ans.: (a) Net incomes are equal under both the techniques; (b) Net income under Variable Costing Technique is higher; (c) Net income under Absorption Costing technique is higher. 3. Choose the correct alternative from the following: a. Absorption Costing is also called: (i) Variable Costing; (ii) Marginal Costing; (iii) Total Costing. b. Under Absorption Costing, the Closing Stock is valued at: (i) Total Cost; (ii) Variable Cost; (iii) Fixed Cost. c. Marginal Cost is nothing but: (i) Fixed Cost; (ii) Total Cost; (iii) Variable Cost. d. In Absorption Costing, the unit Product Cost includes: (i) Total Cost; (ii) Variable Cost only; (iii) Fixed Cost only. e. Fixed Costs are recovered from contribution under: (i) Absorption Costing; (ii) Variable Costing; (iii) None of these. f. Costs are classified according to their functional elements under: (i) Absorption Costing; (ii) Variable Costing; (iii) None of these. g. Unit Product Cost remains constant at different levels of output under: (i) Absorption Costing; (ii) Variable Costing; (iii) None of these. h. Profit becomes a function of production instead of sales under: (i) Absorption Costing; (ii) Variable Costing; (iii) None of these. Ans.: (a) (iii); (b) (i); (c) (iii); (d) (i); (e) (ii); (f) (i); (g) (ii); (h) (i).

Modified Date: Tue, Jun 29, 2010 05:09:32 PM

Output Date: Tue, Jul 06, 2010 12:22:53 PM

Rev II

Project: Management Accounting_Debarshi Bhattacharyya ACE Pro India Pvt. Ltd. File: X:\Pearson\Management Accounting_Debarshi Bhattacharyya\MAIN\M09\LAYOUT_M09\M09_DEBA_ISBN_EN_SE_C09.indd

ABSORPTION COSTING AND VARIABLE COSTING

629

4. Fill in the blank of the following statements: a. Cost–volume–profit analysis is an integral part of . b. The additional cost incurred for producing one additional unit is called . c. Under , the unit product costs are different at different levels of the output. is also called Full Costing. d. e. Under , all costs including Semi-Variable Costs are divided into two parts, Fixed and Variable Costs. f. Net income ascertained under Absorption Costing and Variable Costing becomes equal only when the the sales quantity. Production quantity is Ans.: (a) Variable Costing; (b) Marginal Cost; (c) Absorption Costing; (d) Absorption Costing; (e) Variable Costing; (f) equal to.

EXERCISE I. Theoretical Questions A. Short Answer Type Questions

1. 2. 3. 4. 5. 6. 7. 8.

What is Absorbed Cost? What is Marginal Cost? What is Absorption Costing? What is Marginal Costing? Give two distinguished features of Absorption Costing. Give two distinguished features of Marginal Costing. Give two points of distinction between Absorption Costing and Marginal Costing. Distinguish between Absorbed Cost and Marginal Cost.

B. Essay Type Questions

1. 2. 3. 4. 5. 6. 7.

What is Absorption Costing? What are its distinguished features? What are the advantages and limitations of Absorption Costing? What is Marginal Costing? What are its distinguished features? What are the advantages and limitations of Marginal Costing? Distinguish between Absorption Costing and Marginal Costing. Discuss how the net income is determined under Absorption Costing and Marginal Costing. Give the proforma of Income Statement prepared under Absorption Costing and Marginal Costing.

II. Practical Problems

1. The following data have been extracted from the books of a company: Production and Sales Volume Selling Price per unit Materials per unit Labour per unit Production Overheads: Variable Fixed Other Fixed Overheads

20,000 units Rs. 100 Rs. 40 Rs. 25 Rs. 15 per unit Rs. 80,000 Rs. 1,00,000

Prepare an Income Statement under: (i) Absorption Costing Technique; and (ii) Variable Costing Technique. Ans.: Net income under: (i) Absorption Costing is Rs. 2,20,000; and (ii) Variable Costing is Rs. 2,20,000. 2. The following data have been extracted from the books of a company: Production and Sales Selling Price per unit Materials and Labour per unit

Modified Date: Tue, Jun 29, 2010 05:09:32 PM

35,000 units Rs. 20 Rs. 12

Output Date: Tue, Jul 06, 2010 12:22:53 PM

Rev II

Project: Management Accounting_Debarshi Bhattacharyya ACE Pro India Pvt. Ltd. File: X:\Pearson\Management Accounting_Debarshi Bhattacharyya\MAIN\M09\LAYOUT_M09\M09_DEBA_ISBN_EN_SE_C09.indd

630

MANAGEMENT ACCOUNTING

Production Overheads: Variable Fixed Administration and Selling Overheads (fixed)

Rs. 3 per unit Rs. 1,20,000 Rs. 1,05,000

Prepare an Income Statement under: (i) Absorption Costing technique; and (ii) Marginal Costing technique. Ans.: Net income under: (i) Absorption Costing is Rs. 50,000; and (ii) Marginal Costing is Rs. 50,000. 3. The following data have been obtained from the books of a company: Production Volume Sales Volume Selling Price per unit Materials per unit Labour per unit Production Overheads: Variable Fixed Administration and Selling Overheads: Variable Fixed

40,000 units 35,000 units Rs. 80 Rs. 30 Rs. 20 Rs. 5 per unit Rs. 2,00,000 Rs. 8 per unit Rs. 2,20,000

Prepare an Income Statement under: (i) Absorption Costing technique; and (ii) Variable Costing technique. Ans.: Net income under: (i) Absorption Costing is Rs. 2,00,000; and (ii) Variable Costing is Rs. 1,75,000. 4. The following data have been extracted from the books of a company: Production Sales Selling Price per unit Materials and labour per unit Production Overheads: Variable Fixed Administration and Selling Overheads (Fixed)

20,000 units 17,000 units Rs. 35 Rs. 18 Rs. 6 per unit Rs. 1,20,000 Rs. 75,000

Prepare an Income Statement under: (i) Absorption Costing technique; and (ii) Marginal Costing technique. Ans.: Net income under: (i) Absorption Costing is Rs. 10,000; and (ii) Marginal Costing is Rs. 8,000. 5. Prepare an Income Statement under Absorption Costing and Variable Costing from the following information relating to the year 2008–09: Opening Stock Fixed Cost Variable Cost Production Sales Stocks are valued based on FIFO

1,000 units valued at Rs. 70,000 Including the Variable Cost of Rs. 50 per unit Rs. 1,20,000 Rs. 60 per unit 10,000 units 7,000 units @ Rs. 100 per unit

[B. Com. (Hons.), Delhi University—Adapted] Ans.: Net income under: (i) Absorption Costing is Rs. 1,98,000; and (ii) Variable Costing is Rs. 1,70,000.

Modified Date: Tue, Jun 29, 2010 05:09:32 PM

Output Date: Tue, Jul 06, 2010 12:22:53 PM

Rev II

Project: Management Accounting_Debarshi Bhattacharyya ACE Pro India Pvt. Ltd. File: X:\Pearson\Management Accounting_Debarshi Bhattacharyya\MAIN\M09\LAYOUT_M09\M09_DEBA_ISBN_EN_SE_C09.indd

ABSORPTION COSTING AND VARIABLE COSTING

631

6. The following data have been obtained from the books of a company: Opening Stock Fixed Production Cost Variable Production Cost Production Sales Administration and Selling Overheads (fixed): Rs. 1,30,000

6,000 units valued at Rs. 90,000 including the Variable Cost of Rs. 10 per unit Rs. 70,000 Rs. 15 per unit 14,000 units 20,000 units @ Rs. 30 per unit

Prepare an Income Statement under: (i) Absorption Costing Technique; and (ii) Variable Costing Technique. Ans.: Net income under: (i) Absorption Costing is Rs. 1,00,000; and (ii) Variable Costing is Rs. 1,30,000. 7. The following data have been obtained from the books of a company: Production Volume Sales Volume Opening Stock Volume Value of Opening Stock Selling Price per unit Variable Production Cost per unit Fixed Production Overheads Administration and Selling Overheads: Variable Fixed Stocks are Valued Based on FIFO

30,000 units 40,000 units 20,000 units Rs. 2,00,000 (including the Variable Cost Rs. 1,20,000) Rs. 25 Rs. 15 Rs. 1,50,000 Rs. 2 per unit Rs. 1,20,000

Prepare an Income Statement under: (i) Absorption Costing technique; and (ii) Variable Costing technique. Ans.: Net income under: (i) Absorption Costing is Rs. 2,00,000; and (ii) Variable Costing is Rs. 2,30,000. 8. Your company has a production capacity of 12,500 units and the normal capacity utilization is 80%. The opening inventory of finished goods on 1 April 2008 was 1,000 units. During the year that ended on 31 March 2009, it produced 11,000 units, though it could sell only 10,000 units. The standard Variable Cost per unit is Rs. 6.50 and the Standard Fixed Factory Cost per unit is Rs. 1.50. The total Fixed Selling and Administration Overheads amounted to Rs. 10,000. The company sells its product at Rs. 10 per unit. Prepare Income Statements under Absorption Costing and Variable Costing. Explain the reasons for the difference in profit, if any. [B. Com. (Hons.), Delhi University—Adapted] Ans.: Profit under: (i) Absorption Costing is Rs. 11,500; and (ii) Variable Costing is Rs. 10,000.

Modified Date: Tue, Jun 29, 2010 05:09:32 PM

Output Date: Tue, Jul 06, 2010 12:22:53 PM

Rev II

Project: Management Accounting_Debarshi Bhattacharyya ACE Pro India Pvt. Ltd. File: X:\Pearson\Management Accounting_Debarshi Bhattacharyya\MAIN\M10\LAYOUT_M10\M10_DEBA_ISBN_EN_SE_C10.indd

Marginal Costing and Cost–Volume–Profit Analysis

10

LEARNING OBJECTIVES On completion of the study of the chapter, you should be able to understand: What is Marginal Cost? What is Marginal Costing? Features, advantages and limitations of Marginal Costing. Concept of Cost–Volume–Profit (CVP) Analysis. Features and objectives of CVP Analysis. Classification of cost under Marginal Costing Technique. Elements of CVP Analysis. Marginal Cost equation. Concept of contribution, Break-Even (BE) Point and Margin of Safety. Concept of Profit–Volume Ratio. Concept of BE Analysis. Concept of BE Chart and Angle of Incidence. Development of different formulae under CVP Analysis.

10.1 CONCEPT OF MARGINAL COST According to the Chartered Institute of Management Accountants (CIMA), London, Marginal Cost is defined as ‘the amount at any given volume of output by which aggregate costs are changed if the volume of output is increased or decreased by one unit. In practice, this is measured by the total Variable Cost attributable to one unit.’ Analysing the above definition, it can be concluded that Marginal Cost is nothing but the additional cost incurred for an increase in one additional unit of output (or decremental cost for decrease in one unit of output). Cost generally consists of two parts—Fixed Cost and Variable Cost. While Fixed Cost remains unchanged in the short run with the change in the output level, the Variable Cost varies with the change in the output level. Accordingly, for an increase in one additional unit of output, only Variable Cost increases and Fixed Cost remains unchanged. Therefore, an additional cost incurred for an increase in one additional unit of output is nothing but the Variable Cost only. Hence, Marginal Cost is nothing but Variable Cost. Illustration Say, the Variable Cost per unit of a product is Rs. 4 and the total Fixed Cost for the product is Rs. 5,000. Therefore, the Total Cost of producing 1,000 units of output = Variable Cost + Fixed Cost = (1,000 units × Rs. 4) + Rs. 5,000 = Rs. 9,000. Now, if one additional unit is produced beyond 1,000 units (i.e., 1,001 units), then the Total Cost for producing 1,001 units = Variable Cost + Fixed Cost = (1,001 units × Rs. 4) + Rs. 5,000 = Rs. 9,004. Hence, the additional cost incurred for producing one additional unit = Rs. 9,004 − Rs. 9,000 = Rs. 4.

Modified Date: Mon, Jul 05, 2010 05:08:16 PM

Output Date: Tue, Jul 06, 2010 12:23:48 PM

Rev II

Project: Management Accounting_Debarshi Bhattacharyya ACE Pro India Pvt. Ltd. File: X:\Pearson\Management Accounting_Debarshi Bhattacharyya\MAIN\M10\LAYOUT_M10\M10_DEBA_ISBN_EN_SE_C10.indd

MARGINAL COSTING AND COST–VOLUME–PROFIT ANALYSIS

633

This additional cost of Rs. 4 incurred for producing one additional unit is the Marginal Cost, which is nothing but the Variable Cost per unit. 10.2 DEFINITION OF MARGINAL COSTING As per CIMA, London, Marginal Costing is defined as ‘the ascertainment of Marginal Cost and of the effect on profit of changes in volume or type of output by differentiating between Fixed Costs and Variable Costs.’ CIMA, London, further defines Marginal Costing as ‘the accounting system in which Variable Costs are charged to the cost units and Fixed Costs of the period are written off in full against the aggregate contribution.’ Analysing the above, it can be said that Marginal Costing is the method of ascertaining Marginal Cost and it evaluates the effect of fixed and Variable Costs on profit due to change in the volume of production. 10.3 FEATURES OF MARGINAL COSTING Distinguished features of Marginal Costing are as follows: i. Under Marginal Costing alone, Variable Costs are charged to the cost unit. Fixed Costs are recovered from the contribution. ii. Under this costing technique, all costs including semi-variable costs are divided into two parts, such as fixed and variable, on the basis of their variability with respect to the change in the levels of output. iii. Under this costing technique, the closing inventories are valued at Variable Cost only. iv. Under this costing technique, Selling Price is based on the Variable Cost plus contribution. v. Under this costing technique, profit is calculated in two parts: (a) Contribution = Sales − Variable Cost; and (b) Profit = Contribution − Fixed Cost. vi. It is an important managerial technique for Decision Making and cost control. vii. Break-Even (BE) Analysis and Cost–Volume–Profit (CVP) Analysis are integral parts of this costing technique. 10.4 ADVANTAGES OF MARGINAL COSTING Marginal Costing Technique has the following advantages: i. It provides useful data for managerial Decision Making. ii. It is a very effective tool of profit planning. iii. It facilitates control over Variable Costs by avoidance of arbitrary apportionment or allocation of Fixed Costs. iv. Under this technique, problems on computation of accurate, fixed factory overhead rate can be avoided as fixed overheads are charged against contribution. v. As, under this technique, the valuation of closing inventory is done at Variable Cost only, the problem of illogical carry forward of Fixed Cost of one period to the next period through closing inventories can be avoided. vi. It provides useful information to the management for pricing the product of the enterprise. As the Variable Cost per unit is constant from period to period within a short span of time, the decision on pricing the product can easily be taken by the management. vii. It helps the management in the process of cost control by concentrating on Variable Cost alone, as Fixed Costs are non-controllable in such a short span of time. viii. It provides many useful techniques to the management for Decision Making like BE Analysis, Profit–Volume (P/V) Ratio, Variable Cost (VC) Ratio and so on. ix. It ensures a greater control over the cost through Standard Costing and Budgetary Control. x. Use of Marginal Cost Technique is very simple as it is very easy to understand and very simple to operate.

Modified Date: Mon, Jul 05, 2010 05:08:16 PM

Output Date: Tue, Jul 06, 2010 12:23:48 PM

Rev II

Project: Management Accounting_Debarshi Bhattacharyya ACE Pro India Pvt. Ltd. File: X:\Pearson\Management Accounting_Debarshi Bhattacharyya\MAIN\M10\LAYOUT_M10\M10_DEBA_ISBN_EN_SE_C10.indd

634

MANAGEMENT ACCOUNTING

10.5 LIMITATIONS OF MARGINAL COSTING In spite of having immense important advantages, Marginal Costing Technique suffers from the following limitations too: i. Marginal Costing Technique assumes that semi-variable costs can be segregated into two parts—fixed and variable elements. In practice, however, such segregation of semi-variable costs is a very difficult task. ii. It excludes the Fixed Cost for Decision Making, which sometimes may lead to a wrong conclusion. iii. With the development of technology and application of automation in the industry, Fixed Costs have substantially increased and their impact on production cost is no longer ignorable nowadays. Thus, Marginal Costing Technique fails to reflect the impact of these costs on Production Cost, as it gives a greater emphasis on Variable Cost only in determination of the Product Cost. iv. This technique cannot be successfully applied in ‘Cost plus Contract’ unless an adequate percentage over the Variable Cost is charged from the contractee to cover the Fixed Cost and profit. v. As this technique does not provide any specific parameter for control, cost control can be better achieved with the help of Standard Costing and Budgetary Control instead of relying on this technique. vi. It is not very useful for the valuation of inventory in case of abnormal loss, and such valuation is not accepted from the view point of the income tax authorities and auditors. vii. Marginal Costing Technique does not resolve the difficulties involved in the apportionment of variable overheads. viii. It ignores to reflect the impact of time factor and value of investment made for the determination of Product Cost. 10.6 MEANING OF COST–VOLUME–PROFIT (CVP) ANALYSIS As per the official terminology of CIMA, London, Cost–Volume–Profit (CVP) Analysis is ‘the study of the effects on future profit of changes in Fixed Cost, Variable Cost, sale price, quantity and mix.’ There is a direct relationship between the cost, volume of output and profit. The Total Cost of a product depends on its volume of the output. On the other hand, profit from the product depends on its Total Sales and Total Cost. Therefore, profitability of a product depends on its Total Cost and the volume of production. CVP Analysis examines the relationship of costs and profit to the volume of production to maximize the profit of the firm. The method of studying the relationship between the cost, volume of production, sales and their impact on profit is called CVP Analysis. It is a logical extension of Marginal Costing and is used as a very powerful tool by the management in the process of budgeting and profit planning. It is also useful for short-term Decision Making. BE Analysis is a widely used technique to study CVP relationship. 10.7 FEATURES OF CVP ANALYSIS CVP Analysis has the following distinguished features: i. It evaluates the behaviour of cost in relation to production or sales volume. ii. It exhibits the effect on profit due to the changes in cost and volume of output. iii. It evaluates the amount of projected profit for a projected sales value or volume. iv. It evaluates the amount and quantity of production and sales needed to achieve a target profit level. v. It evaluates the value and volume of sales needed to achieve BE. 10.8 OBJECTIVES OF CVP ANALYSIS CVP Analysis has the following objectives: i. It helps to forecast the profit fairly and accurately. ii. It acts as an effective tool of profit planning to the management.

Modified Date: Mon, Jul 05, 2010 05:08:16 PM

Output Date: Tue, Jul 06, 2010 12:23:48 PM

Rev II

Project: Management Accounting_Debarshi Bhattacharyya ACE Pro India Pvt. Ltd. File: X:\Pearson\Management Accounting_Debarshi Bhattacharyya\MAIN\M10\LAYOUT_M10\M10_DEBA_ISBN_EN_SE_C10.indd

MARGINAL COSTING AND COST–VOLUME–PROFIT ANALYSIS

635

iii. It helps in ascertaining the BE Point (BEP) of the product produced and sold. iv. It is very much useful in setting up the flexible budget, which ascertains cost, profit and sales at different levels of activity. v. It assists the management in the process of performance evaluation for the purpose of control. vi. It helps in formulating the price policies by projecting the effect of different price structures on the costs and profits. vii. It helps in determining the amount of overhead cost to be charged to the product at different levels of operation, as overhead rates are generally predetermined on the basis of a selected volume of production. viii. It helps in determining the optimum product mix in case where multiple products are produced and sold by a firm. 10.9 ASSUMPTIONS UNDERLYING CVP ANALYSIS The CVP Analysis is based on the following assumptions: i. Total Cost consists of two components—Fixed Cost and Variable Cost. ii. Variable Cost varies with the change in the output level whereas Fixed Cost remains constant even at different levels of output. iii. Selling Price per unit remains constant at different volumes of sales. iv. Only one product is sold by the concern or if it sells multiple products, the sales mix remains constant at different volumes of sales. v. Volume of production is equal to the sales volume, that is, there would be no opening or closing inventory during a period. vi. The efficiency and productivity level is constant at different levels of output. vii. There would be no change in the price of material, rate of wages, and so on, at all the levels of production. 10.10 CLASSIFICATION OF COST UNDER CVP ANALYSIS In CVP Analysis, costs are classified into two parts—Fixed Cost and Variable Cost. Fixed Cost: It remains fixed at different levels of production. Fixed Cost does not change with the change in the output level, for example, rent, salary to staff, insurance premium and so on. Variable Cost: It does not remain fixed at different levels of production. Variable Cost varies with the change in the output level, for example, materials, wages, power and so on. There is another type of cost called Semi-Variable Cost. It remains fixed with the change in the output up to a certain level, but it varies with the change in the output beyond that level of output, for example, repairs and maintenance. Semi-Variable Cost consists of both fixed as well as variable elements in it. It is not separately recognized in CVP Analysis. A fixed portion of the Semi-Variable Cost is clubbed with the Fixed Cost and its variable portion is clubbed with the Variable Cost. Therefore, in CVP Analysis, costs are divided into two parts only—Fixed Cost and Variable Cost. 10.11 ELEMENTS OF CVP ANALYSIS CVP Analysis establishes a relationship between cost, volume of output and profit. It evaluates the effect on profit due to changes in cost and volume of output. This analysis consists of several integral parts or components which are as follows: i. Marginal Cost Equation. ii. Contribution. iii. Profit–Volume (P/V) Ratio.

Modified Date: Mon, Jul 05, 2010 05:08:16 PM

Output Date: Tue, Jul 06, 2010 12:23:48 PM

Rev II

Project: Management Accounting_Debarshi Bhattacharyya ACE Pro India Pvt. Ltd. File: X:\Pearson\Management Accounting_Debarshi Bhattacharyya\MAIN\M10\LAYOUT_M10\M10_DEBA_ISBN_EN_SE_C10.indd

636

MANAGEMENT ACCOUNTING

iv. Break-Even Point (BEP). v. Margin of Safety (MS). Each element of the CVP Analysis is explained separately as follows. i. Marginal Cost Equation Marginal Cost equation exhibits the relationship between contribution, Fixed Cost and profit. It explains that the excess of sales over the Variable Cost is the contribution towards Fixed Cost and profit. Marginal Cost equation can be developed as follows: Sales (S) = Total Cost + Profit (P) Again, Total Cost = Fixed Cost (F) + Variable Cost (V) ∴ S = F + V + P or S – V = F + P Now, Contribution (C) = Excess of Sales over Variable Cost, i.e., C = S – V ∴C=F+P ii. Contribution Contribution is the excess of sales over Variable Cost. More clearly, contribution is that portion of sales which remains after recovering the Variable Cost to that extent of sales. This contribution is available towards Fixed Cost and profit. Mathematically, contribution can be expressed as follows: Contribution (C) = Excess of Sales over Variable Cost ∴C=S–V Again, from Marginal Cost Equation, we get: S=F+P+V or S−V=F+P ∴C=F+P Therefore, generally, contribution is equal to the Fixed Cost plus profit. Illustration Say, 2,000 units of a product are sold at Rs. 10 per unit. Variable Cost of manufacturing the product is Rs. 6 per unit and the total Fixed Cost is Rs. 5,000. Solution

Less:

Contribution

Rs. 20,000 12,000 8,000

Contribution

Rs. 5,000 3,000 8,000

Sales (S) (2,000 × Rs. 10) Variable Cost (V) (2,000 × Rs. 6)

Alternatively:

Add:

Fixed Cost Profit (S − V − F = 20,000 − 12,000 − 5,000)

Difference Between Contribution and Profit Contribution is the excess of sales over Variable Cost alone, whereas profit is the excess of sales over variable as well as Fixed Costs. Generally, contribution should be higher than the profit as it includes Fixed Cost and profit, that is, C = F + P. At the MS level of sales alone, contribution is equal to the profit, that is, C = P.

Modified Date: Mon, Jul 05, 2010 05:08:16 PM

Output Date: Tue, Jul 06, 2010 12:23:48 PM

Rev II

Project: Management Accounting_Debarshi Bhattacharyya ACE Pro India Pvt. Ltd. File: X:\Pearson\Management Accounting_Debarshi Bhattacharyya\MAIN\M10\LAYOUT_M10\M10_DEBA_ISBN_EN_SE_C10.indd

637

MARGINAL COSTING AND COST–VOLUME–PROFIT ANALYSIS

Therefore, contribution includes the total profit, but the profit does not include the total contribution. Profit includes a portion of contribution that has been arrived at by the deduction of Fixed Cost from the total contribution, that is, C − F = P. iii. Profit–Volume Ratio (P/V Ratio) Profit–Volume (P/V) Ratio is the ratio of contribution and sales. It is generally expressed in percentage. It exhibits the percentage of contribution included in sales. It indicates the effect on the profit for a given change in sales. Mathematically, P/V Ratio can be expressed as follows: P/V Ratio =

C × 100 S

Mathematically, P/V Ratio may be expressed in different forms as shown in the following: P/V Ratio =

S−V × 100 S

[As C = S − V]

P/V Ratio =

F+P × 100 P

[As C = F + P]

P/V Ratio =

Change in Profit × 100 Change in Sales

Illustration Say, 5,000 units of a product are sold at Rs. 10 per unit. The Variable Cost of manufacturing the product is Rs. 6 per unit and the total Fixed Cost is Rs. 15,000. Solution

Less:

Sales (S) [5,000 × Rs. 10] Variable Cost (V) [5,000 × Rs. 6]

Less:

Fixed Cost

Contribution Profit

Rs. 50,000 30,000 20,000 15,000 5,000

From the above calculation, P/V Ratio may be computed in the following ways: 1. P/V Ratio =

C 20,000 × 100 = × 100 = 40% S 50,000

2. P/V Ratio =

S−V 10 − 6 × 100 = × 100 = 40% S 10

or

P/V Ratio =

3. P/V Ratio =

S−V 50,000 − 30,000 × 100 = × 100 = 40% S 50,000

F+P 15,000 + 5,000 × 100 = × 100 = 40% S 50,000

Modified Date: Mon, Jul 05, 2010 05:08:16 PM

Output Date: Tue, Jul 06, 2010 12:23:48 PM

Rev II

Project: Management Accounting_Debarshi Bhattacharyya ACE Pro India Pvt. Ltd. File: X:\Pearson\Management Accounting_Debarshi Bhattacharyya\MAIN\M10\LAYOUT_M10\M10_DEBA_ISBN_EN_SE_C10.indd

638

MANAGEMENT ACCOUNTING

Features of P/V Ratio P/V Ratio is an indicator of the rate at which the profit is being earned by a concern. It can be improved either by increasing the Selling Price per unit or by reducing the Variable Cost per unit. It possesses the following distinguished features: i. It exhibits the percentage of contribution included in sales. ii. It shows the rate of profit that can be earned by a concern after achieving its BEP. iii. Higher P/V Ratio indicates a higher profitability rate of the concern. iv. It does not change with the change in the output level. v. It changes either due to the change in the Selling Price per unit or Variable Cost per unit or both. vi. It is useful to calculate the BEP. vii. It is useful to ascertain the MS. viii. It is useful to calculate profit at the given level of sales. ix. It is useful to calculate the sales that is required to earn a given profit. Stop and Think i. P/V Ratio changes if either the Selling Price per unit or the Variable Cost per unit or both changes. ii. P/V Ratio does not change at different levels of output (even in different years), if there is no change in either the Selling Price per unit or the Variable Cost per unit or both. iii. P/V Ratio does not change due to a change in the Fixed Cost.

iv. Break-Even Point (BEP) Break-Even Point (BEP) is that level of sales where there is neither profit nor loss. At BEP, the total sales revenue is equal to the Total Cost. Any sales above this BEP, a concern earns profit. On the other hand, any sale below this BEP, the concern suffers loss. At BEP, the total Fixed Cost and the Variable Cost up to that level of sales have been recovered from the sales. Generally, at any other point of sales, the contribution from sales is available towards the Fixed Cost and profit. But as there is no profit or loss at BEP, the contribution from sales at BEP is available towards the Fixed Cost only, that is, at BEP, C = F. Mathematically BEP can be expressed as follows: As per Marginal Cost Equation: S=F+P+V At BEP, there is neither profit nor loss, i.e., P or L = Nil ∴S=F+V or S−V=F ∴C=F ∴ At BEP, Contribution = Fixed Cost Illustration Say, 7,000 units of a product are sold at Rs. 10 per unit. The Variable Cost of manufacturing the product is Rs. 7 per unit and the total Fixed Cost is Rs. 21,000. Solution

Less:

Sales (S) (7,000 × Rs. 10) Variable Cost (V) (7,000 × Rs. 7) Contribution

Rs. 70,000 49,000 21,000 (Continued)

Modified Date: Mon, Jul 05, 2010 05:08:16 PM

Output Date: Tue, Jul 06, 2010 12:23:48 PM

Rev II

Project: Management Accounting_Debarshi Bhattacharyya ACE Pro India Pvt. Ltd. File: X:\Pearson\Management Accounting_Debarshi Bhattacharyya\MAIN\M10\LAYOUT_M10\M10_DEBA_ISBN_EN_SE_C10.indd

639

MARGINAL COSTING AND COST–VOLUME–PROFIT ANALYSIS

Less:

Fixed Cost Profit/Loss

Rs. 21,000 Nil

This sale of Rs. 70,000 is the BEP sales (BEPS) as there is neither profit nor loss, that is, the total sale is equal to the Total Cost. At this BEPS, the contribution is equal to the Fixed Cost (i.e., Rs. 21,000).

Stop and Think i. ii. iii. iv.

BEPS increases due to the increase in Fixed Cost or Variable Cost per unit or both. BEPS decreases due to the increase in Selling Price per unit. BEPS changes due to the change in P/V Ratio. BEPS does not change due to the change in sales quantity.

v. Margin of Safety (MS) Margin of Safety (MS) is the level of sales made above the BEP. In other words, MS is the excess of actual sales over BEPS. As the total Fixed Cost for the actual sales has already been recovered at BEP, no further Fixed Cost is to be borne at the MS level of sales. Generally, at any point of sales, the contribution from sales is available towards the Fixed Cost and profit. But as the total Fixed Cost has already been recovered at BEP, the contribution from sales at MS is available towards profit only, that is, at MS, C = P. At the MS level of sales, the excess of sales over Variable Cost only represents the profit. Mathematically, MS can be expressed as follows: Margin of Safety (MS) = Actual Sales (S) − BEPS ∴ MS = S – BEPS Generally, at any point of sales other than BEP: C=F+P At BEP, the total Fixed Cost has been recovered for sales. As MS is that level of sales achieved above BEP, no further Fixed Cost is to be borne at the MS level of sales. ∴ At MS, Contribution = Profit [as at MS, no further Fixed Cost is incurred] ∴ At MS, C = P Illustration Say, 9,000 units of a product are sold at Rs. 10 per unit. The Variable Cost of manufacturing the product is Rs. 7 per unit and the BEP has been achieved at a sale of 5,000 units. Solution

Less:

Total Sales (S) (9,000 × Rs. 10) BEPS (5,000 × Rs. 10) MS

Rs. 90,000 50,000 40,000

∴ MS in units = Total Sales in units − BEPS in units = 9,000 − 5,000 = 4,000 units Again, Contribution per unit = S − V = Rs. 10 − Rs. 7 = Rs. 3 ∴ Contribution at MS = 4,000 units × Rs. 3 = Rs. 12,000 This Contribution at MS of Rs. 12,000 is equal to the profit earned from the total sale of 9,000 units.

Modified Date: Mon, Jul 05, 2010 05:08:16 PM

Output Date: Tue, Jul 06, 2010 12:23:48 PM

Rev II

Project: Management Accounting_Debarshi Bhattacharyya ACE Pro India Pvt. Ltd. File: X:\Pearson\Management Accounting_Debarshi Bhattacharyya\MAIN\M10\LAYOUT_M10\M10_DEBA_ISBN_EN_SE_C10.indd

640

MANAGEMENT ACCOUNTING

Stop and Think i. MS increases due to an increase in the Selling Price per unit or sales volume or both. ii. MS decreases due to an increase in the Fixed Cost or Variable Cost per unit or both. iii. MS changes due to the change in P/V Ratio.

10.12 BREAK-EVEN ANALYSIS (BE ANALYSIS) CVP Analysis is popularly known as Break-Even (BE) Analysis, even though there exists a narrow difference between the terms CVP Analysis and BE Analysis. CVP Analysis refers to the study of the effect on profit due to the changes in cost and volume of output, whereas BE Analysis refers to the study of determination of that level of activity where the total sales is equal to the Total Cost and also the study of determination of profit at any level of activity. BE Analysis is an integral part of CVP Analysis. CVP Analysis includes the entire study of profit planning, whereas BE Analysis is one of the techniques used in the study of profit planning by CVP Analysis. However, the technique of BE Analysis is so popular for studying CVP Analysis that these two terms are generally used synonymously. 10.13 BREAK-EVEN CHART (BE CHART) Break-Even Chart (BE Chart) is the graphical representation of BE Analysis. It depicts the relationship between costs, sales and profits. According to CIMA, London, BE Chart is ‘a chart which shows profit or loss at various levels of activity, the level at which neither profit nor loss is shown being termed as Break-Even Point.’ Therefore, BE chart graphically shows the profit or loss at various levels of activity and also shows the level of activity where there is no profit or no loss (i.e., Total Cost equals total sales). 10.13.1 Presentation of BE Chart On the X-axis (horizontal line) of the graph, the volume of production or sales is measured. On the Y-axis (vertical line) of the graph, the values of cost and sales are measured. The Fixed Cost line is drawn parallel to the X-axis as Fixed Cost does not change due to the change in production or sales level. The Total Cost (i.e., Fixed Cost plus Variable Cost) line is drawn from the point of intersection of the Y-axis and Fixed Cost line upward to the right. Sales line is drawn from the origin upward to the right. The BE Chart is shown in Figure 10.1.

Break-Even Sales

P ea Ar ss o L

L

O

Margin of Safety

Values of Cost and Sales (in rupees)

Margin of Safety

ce en t d n i i c Po In en of le Ev g k An ea Br

rea fit A Pro

S

M

N

Break-Even Sales Volume of Production or Sales (in units)

Figure 10.1

Break-Even Chart (BE Chart)

In this figure, OS shows the sales line, LN shows the Fixed Cost line and LM shows the Total Cost line. At point P, Total Cost line intersects the total sales line (i.e., point of intersection of LM and OS) where there is no

Modified Date: Mon, Jul 05, 2010 05:08:16 PM

Output Date: Tue, Jul 06, 2010 12:23:48 PM

Rev II

Project: Management Accounting_Debarshi Bhattacharyya ACE Pro India Pvt. Ltd. File: X:\Pearson\Management Accounting_Debarshi Bhattacharyya\MAIN\M10\LAYOUT_M10\M10_DEBA_ISBN_EN_SE_C10.indd

MARGINAL COSTING AND COST–VOLUME–PROFIT ANALYSIS

641

profit or no loss. Hence, Point P shows the BEP. If there is any sale above this Point P, a concern earns profit. Therefore, the area above the Point P exhibits MS of the concern. On the other hand, if there is any sale below this Point P, the concern suffers loss. Profit or loss at any level of sales can be known by taking any point on the sales line OS. The angle created by intersection of sales line OS and Total Cost line LM at BEP P is called the Angle of Incidence, which indicates the rate at which the profit is being made by the concern. 10.14 ANGLE OF INCIDENCE Angle of Incidence is the angle formed by the intersection of sales line and Total Cost line at BEP in the BE chart (in Figure 10.1, angle SPM). This angle exhibits the rate at which the profits are being earned by the concern after reaching the BEP. It shows the profit-earning capacity of the concern. Wider angle of incidence exhibits a higher profit-earning capacity of the concern or vice-versa. The management of every concern should aim to have as wide the angle as possible. 10.15 MATHEMATICAL REPRESENTATION OF CVP ANALYSIS  Sales (S) = Cost + Profit (P) or Cost − Loss (L) Again, Cost = Fixed Cost (F) + Variable Cost (V) ∴ S = F + V + P (or − L) or S – V = F + P (or − L) 

Now, Contribution (C) = Excess of Sales over Variable Cost i.e., C = S – V ∴ C = F + P (or − L)



BEP = Where there is no profit, no loss i.e., Total Sales = Total Cost i.e., S = F + V ∴ At BEP: C = F (as here, P or L = Nil)



P/V Ratio = % of Contribution included in Sales C ∴ P/V Ratio = × 100 S S−V or P/V Ratio = × 100 [ As C = S − V] S F+P or P/V Ratio = × 100 [ As C = F + P] S Change in Profit or P/V Ratio = × 100 Change in Sales



P/V Ratio at BEP: C P/V Ratio = × 100 S At BEP: F P/V Ratio = × 100 BEPS F ∴ BEPS = × 100 P/V Ratio

Modified Date: Mon, Jul 05, 2010 05:08:16 PM

Output Date: Tue, Jul 06, 2010 12:23:48 PM

Rev II

Project: Management Accounting_Debarshi Bhattacharyya ACE Pro India Pvt. Ltd. File: X:\Pearson\Management Accounting_Debarshi Bhattacharyya\MAIN\M10\LAYOUT_M10\M10_DEBA_ISBN_EN_SE_C10.indd

642

MANAGEMENT ACCOUNTING



MS = Excess of actual sales over BEPS i.e., MS = S – BEPS ∴ S = BEPS + MS As, S = F + V + P ∴ F + V + P = BEPS + MS Let us consider V = V1 + V2 Where, V1 = Variable Cost for BEPS V2 = Variable Cost for MS ∴ F + (V1 + V2) + P = BEPS + MS or P = (BEPS – V1 – F) + (MS – V2) or P = Nil + (MS – V2) [∴ at BEP, S – V – F = Nil] or MS – V2 = P ∴C=P [∴at S – V = C] ∴ Contribution at MS = Profit



P/V Ratio at MS: C P/V Ratio = × 100 S At MS: P × 100 MS P ∴ MS = × 100 P/V Ratio P/V Ratio =

[ ∴ at MS, C = P]

10.16 DIFFERENT FORMULAE OF CVP ANALYSIS AT A GLANCE 1. Contribution (C): i. Generally, C = S – V = F + P ii. At BEP, C = F [∴ at BEP, P = Nil] iii. At MS, C = P [∴ at MS, C = P] 2. Break-Even Point (BEP): At BEP, Total Sales = Total Cost [∴at BEP, P = Nil] i.e., S = F + V F BEPS in value = × 100 P/V Ratio 3. Margin of Safety (MS): Margin of Safety = Actual Sales — BEPS i.e., MS = S – BEPS. P MS = × 100 P/V Ratio 4. Profit–Volume Ratio (P/V Ratio) or Contribution–Sales Ratio: S=F+V+P or S = (F + P) + V ∴S=C+V (∴at C = F + P) ∴ Sales include Contribution and Variable Cost.

Modified Date: Mon, Jul 05, 2010 05:08:16 PM

Output Date: Tue, Jul 06, 2010 12:23:48 PM

Rev II

Project: Management Accounting_Debarshi Bhattacharyya ACE Pro India Pvt. Ltd. File: X:\Pearson\Management Accounting_Debarshi Bhattacharyya\MAIN\M10\LAYOUT_M10\M10_DEBA_ISBN_EN_SE_C10.indd

MARGINAL COSTING AND COST–VOLUME–PROFIT ANALYSIS

643

P/V Ratio = % of Contribution included in Sales C i. Generally, P/V Ratio = × 100t S S−V = × 100 S F + P = × 100 S Change in Profit × 100 = Change in Sales F × 100 BEPS P iii. At MS, P/V Ratio = × 100 MS ii. At BEP, P/V Ratio =

5. Variable-Cost (VC) Ratio: VC Ratio = % of Variable Cost included in Sales V ∴ VC Ratio = × 100 S Note P/V Ratio = 1 – VC Ratio and VC Ratio = 1 – P/V Ratio 6. Calculation of Number Units (i.e., in volume): i. Sales (in units) =

Total Contribution F + P = Contribution per unit Contribution per unit

ii. MS (in units) =

Total Contribution F = Contribution per unit Contribution per unit

iii. MS (in units) =

Total Contribution F = Contribution per unit Contribution per unit

7. Sales volume at which two different companies earn an equal profit or incur an equal loss: Sales level (in units) at which two different companies Difference in Fixed Cost = earn an equal profit or incur an equal loss Difference in Contribution per unit

Tutorial Notes to Students for Solving Problems i. For solving problems under CVP Analysis, it is very much essential to ascertain the P/V Ratio at first (if not given in the problem). ii. P/V Ratio remains unchanged at different levels/volume of sales, if the Selling Price per unit and Variable Cost per unit remain changed. iii. P/V Ratio remains unaltered even in different years, if the Selling Price per unit and Variable Cost per unit remain unchanged in those years. iv. P/V Ratio changes either due to a change in the Selling Price per unit or Variable Cost per unit or both. v. Use different formulae of P/V Ratio when you are asked to calculate the Fixed Cost, BE sales (BES), profit at a given sales, sales required to earn a given profit and so on.

Modified Date: Mon, Jul 05, 2010 05:08:16 PM

Output Date: Tue, Jul 06, 2010 12:23:48 PM

Rev II

Project: Management Accounting_Debarshi Bhattacharyya ACE Pro India Pvt. Ltd. File: X:\Pearson\Management Accounting_Debarshi Bhattacharyya\MAIN\M10\LAYOUT_M10\M10_DEBA_ISBN_EN_SE_C10.indd

644

MANAGEMENT ACCOUNTING

10.17 WORKED-OUT PROBLEMS Problem 1 P/V Ratio = 30% Fixed Cost = Rs. 1,20,000 Find out: i. BES ii. Sales when profit is Rs. 1,20,000 iii. Profit when sales is Rs. 6,00,000 Solution i. We know, at BEP: Fixed Cost P/V Ratio = × 100 BEPS Fixed Cost or BEPS = × 100 P/V Ratio 1,20,000 × 100 or BEPS = 30 ∴ BEPS = Rs. 4,00,000 ii. We know: F+P × 100 S Here, P/V Ratio = 30 F = Rs. 1,20,000 P = Rs. 1,20,000 S=? P/V Ratio =

1,20,000 + 1,20,000 × 100 S 2,40,000 S= × 100 30

∴ 30 = or

∴ Required sales when profit is Rs. 1,20,000 (S) = Rs. 8,00,000 iii. Again, we know: F+P × 100 S Here, P/V Ratio = 30 F = Rs. 1,20,000 S = Rs. 6,00,000 ∴P=? ∴ 30 = {(1,20,000 + P) ÷ 6,00,000} × 100 or 1,20,000 + P = 1,80,000 or P = 1,60,000 – 1,20,000 = 60,000 ∴ Required profit when sales is Rs. 6,00,000 = Rs. 60,000 P/V Ratio =

Modified Date: Mon, Jul 05, 2010 05:08:16 PM

Output Date: Tue, Jul 06, 2010 12:23:48 PM

Rev II

Project: Management Accounting_Debarshi Bhattacharyya ACE Pro India Pvt. Ltd. File: X:\Pearson\Management Accounting_Debarshi Bhattacharyya\MAIN\M10\LAYOUT_M10\M10_DEBA_ISBN_EN_SE_C10.indd

MARGINAL COSTING AND COST–VOLUME–PROFIT ANALYSIS

645

Note For all the above-mentioned cases, P/V Ratio remains the same as there is no change in the Selling Price per unit or Variable Cost per unit. Problem 2 Following particulars are available in respect of A Ltd and B Ltd: Particulars

A Ltd Rs. 6,00,000 25% Rs. 90,000

Sales (Rs. 10/unit) P/V Ratio Fixed Cost

B Ltd Rs. 6,00,000 20% Rs. 80,000

Calculate: i. BEP of both companies. ii. MS of both companies. iii. Sales required to earn a profit of Rs. 90,000 by each company. [B. Com. (Hons), Mumbai University—April 2009] Solution i. BEPS (Fixed Cost/P/V Ratio) ii. MS [Actual Sales − BEPS] iii. We know, P/V Ratio =

F+P F+P × 100; or S = × 100 S P/V Ratio

∴ Required Sales (S)

A Ltd 90,000 ÷ 25% = Rs. 3,60,000 6,00,000 − 3,60,000 = Rs. 2,40,000

B Ltd 80,000 ÷ 20% = Rs. 4,00,000 6,00,000 − 4,00,000 = Rs. 2,00,000

(90,000 + 90,000) ÷ 25%

(80,000 + 90,000) ÷ 20%

= Rs. 7,20,000

= Rs. 8,50,000

Problem 3 From the following records of Disha Ltd, calculate: i. BEP in rupees. ii. Sales required to earn a profit of Rs. 72,000. iii. Profit when sales are Rs. 4,75,000. Rs. 2,40,000 8.00 20.00

Fixed Cost Variable Cost per unit Selling Price per unit

[B. Com. (Hons), Mumbai University—October 2008] Solution S−V × 100 S 20 − 8 Here, P/V Ratio = × 100 20 ∴ P/V Ratio = 60%

We know that P/V Ratio =

Modified Date: Mon, Jul 05, 2010 05:08:16 PM

Output Date: Tue, Jul 06, 2010 12:23:48 PM

Rev II

Project: Management Accounting_Debarshi Bhattacharyya ACE Pro India Pvt. Ltd. File: X:\Pearson\Management Accounting_Debarshi Bhattacharyya\MAIN\M10\LAYOUT_M10\M10_DEBA_ISBN_EN_SE_C10.indd

646

MANAGEMENT ACCOUNTING

Fixed Cost × 100 P / V Ratio 2,40,000 Here, BEPS = × 100 = Rs. 4,00,000 60 F + P ii. We know, P/V Ratio = × 100 S 2,40,000 + 72,000 Here, 60 = × 100 S or 60 S = 3,12,000 × 100 i. We know, BEPS =

3,12,00,000 60 ∴ Required Sales = Rs. 5,20,000.

or

S=

iii. We know that P/ V ratio =

F + P × 100 S

2,40,000 + P × 100 4,75,000 or 2,40,000 + P = 60 × 4,750 or P = 2,85,000 − 2,40,000 ∴ Profit = Rs. 45,000. Here, 60 =

Alternative Approach: MS = Actual sales − BEPS Here, MS = 4,75,000 − 4,00,000 = Rs. 75,000 Again, P/V Ratio = Contribution–Sales Ratio = 60% We know, Profit = Contribution at MS Here, Profit = 60% of 75,000 = Rs. 45,000 Problem 4 X Ltd made sales during a certain period for Rs. 1,00,000. The Net Profit for the same period was Rs. 10,000 and the fixed overheads were Rs. 15,000. Find out: i. P/V Ratio. ii. Required sales to earn a profit of Rs. 15,000. iii. Net Profit from sales of Rs. 1,50,000. iv. BEPS [B.Com. (Hons), Calcutta University—1995] Solution F + P × 100 S Here, F = Rs. 15,000; P = Rs. 10,000; and S = Rs. 1,00,000 ∴ P/V Ratio = [(15,000 + 10,000) ÷ 1,00,000] × 100 ∴ P/V Ratio = 25%

i. P/V Ratio =

Modified Date: Mon, Jul 05, 2010 05:08:16 PM

Output Date: Tue, Jul 06, 2010 12:23:48 PM

Rev II

Project: Management Accounting_Debarshi Bhattacharyya ACE Pro India Pvt. Ltd. File: X:\Pearson\Management Accounting_Debarshi Bhattacharyya\MAIN\M10\LAYOUT_M10\M10_DEBA_ISBN_EN_SE_C10.indd

MARGINAL COSTING AND COST–VOLUME–PROFIT ANALYSIS

647

F + P × 100 S 15,000 + 15,000 Here 25 = × 100 [ ∵ given, Profit = Rs. 15,000] S or S = (30,000 ÷ 25) × 100 ∴ Sales = Rs. 1,20,000 ∴ Sales required to earn a profit of Rs. 15,000 = Rs. 1,20,000

ii. P/V Ratio =

iii. When Sales = Rs. 1,50,000, then Profit = ? F + P P/V Ratio = × 100 S 15,000 + P Here 25 = × 100 [ ∵ given, Sales = Rs. 1,50,000] 1,50,000 or 15,000 + P = 1,50,000 × 25 ÷ 100 or 15,000 + P = 37,500 ∴ Profit = 37,500 – 15,000 = Rs. 22,500 ∴ Net Profit from sales of Rs. 1,50,000 = Rs. 22,500 iv. We know, at BEP: F P/V Ratio = × 100 BEPS 15,000 or 25 = × 100 BEPS 15,000 or BEPS = × 100 = 60,000 25 ∴ BEPS = Rs. 60,000 Problem 5 A company budgets for a production of 1,50,000 units. The Variable Cost per unit is Rs. 15 and Fixed Cost is Rs. 3,00,000. The company fixes its Selling Price to fetch a profit of 25% on cost. i. What is the BEP (in units)? ii. What is the P/V Ratio? [B. Com. (Hons), Kalyani University—2009] Solution

Add:

Variable Cost for Budgeted Production of 1,50,000 units (1,50,000 × Rs. 15) Total Fixed Cost for Budgeted Production of 1,50,000 units Total Cost of Budgeted Production of 1,50,000 units Budgeted Profit (25% on 25,50,000) Selling Price of Budgeted Production of 1,50,000 units

Rs. 22,50,000 3,00,000 25,50,000 6,37,500 31,87,500

∴ Selling Price per unit = Rs. 31,87,500 ÷ 1,50,000 = Rs. 21.25 ∴ Contribution per unit = S − V = Rs. 21.25 − Rs. 15 = Rs. 6.25

Modified Date: Mon, Jul 05, 2010 05:08:16 PM

Output Date: Tue, Jul 06, 2010 12:23:48 PM

Rev II

Project: Management Accounting_Debarshi Bhattacharyya ACE Pro India Pvt. Ltd. File: X:\Pearson\Management Accounting_Debarshi Bhattacharyya\MAIN\M10\LAYOUT_M10\M10_DEBA_ISBN_EN_SE_C10.indd

648

MANAGEMENT ACCOUNTING

i. BEP (in units ) = ii. P/V Ratio =

Fixed Cost 3,00,000 = = 48,000 units. Contribution per unit 6.25

C 6.25 × 100 = × 100 = 29.4117% S 21.25

Problem 6 From the following particulars, find out the BEP: Rs. 7.50 27,000 10

Variable Cost per unit Fixed Expenses Selling Price per unit

What should be the Selling Price per unit, if the BEP should be brought down to 6,000 units? [B.B.M., Bangalore University—December 2007] Solution Existing Contribution per unit = S − V = Rs. 10 − Rs. 7.50 = Rs. 2.50 Fixed Cost Rs. 27,000 ∴ Existing BEP in units = = = 10,800 units Contribution per unit Rs. 2.50

(

)

Now, if the BEP is brought down to 6,000 units, then Total Variable Cost at new BEP (6,000 × Rs. 7.50) = Rs. 45,000 Total Fixed Cost at new BEP (same as before) = Rs. 27,000 Total Cost at new BEP = Rs. 72,000 Again, at BEP, Total Sales = Total Cost ∴ Total Sales at new BEP = Total Cost at new BEP = Rs. 72,000 ∴ New Selling Price per unit = Rs. 72,000 ÷ 6,000 = Rs. 12 Problem 7 Selling Price per unit = Rs. 100 Variable Cost per unit = Rs. 60 Total Fixed Cost = Rs. 40,000 Calculate: i. BEPS in value. ii. BEPS in volume. iii. Sales in value and volume when a profit of Rs. 1,20,000 is earned. Solution S−V × 100 S Where S = Total Selling Price or Selling Price per unit V = Total Variable Cost or Variable Cost per unit Here, S = Rs. 100 and V = Rs. 60 100 − 60 ∴ P/ V Ratio = × 100 = 40% 100

i. We know that P/V Ratio =

Modified Date: Mon, Jul 05, 2010 05:08:16 PM

Output Date: Tue, Jul 06, 2010 12:23:48 PM

Rev II

Project: Management Accounting_Debarshi Bhattacharyya ACE Pro India Pvt. Ltd. File: X:\Pearson\Management Accounting_Debarshi Bhattacharyya\MAIN\M10\LAYOUT_M10\M10_DEBA_ISBN_EN_SE_C10.indd

MARGINAL COSTING AND COST–VOLUME–PROFIT ANALYSIS

Again, at BEP, P/V Ratio = or or ∴

ii.

649

F × 100 BEPS

40,000 × 100 BEPS 40,000 BEPS = × 100 = Rs. 1,00,000 40 BES (in value) = Rs. 1,00,000 40 =

Selling Price = Rs. 100 unit Variable Cost = Rs. 60 unit Contribution per unit = Rs. 40

[∵ C = S – V] Fixed Cost We know, BEPS (in volume ) = Contribution per unit ∴ BEPS (in volume) = Rs. 40,000 ÷ Rs. 40 = 1,000 units iii. When Profit = Rs. 1,20,000 F + P Then P/V Ratio = × 100 S 40,000 + 1,20,000 or 40 = × 100 S 1,60,000 S= × 100 = Rs. 4,00,000 or 40 ∴ Sales (in value) = Rs. 4,00,000 ∴ Sales (in volume ) =

F + P Contribution per unit

40,000 + 1,20,000 40 ∴ Sales (in volume) = 4,000 units Here, Sales (in volume ) =

Problem 8 In a purely competitive market, 10,000 pocket transistors can be manufactured and sold and a certain profit can be generated. It is estimated that 2,000 pocket transistors need to be manufactured and sold in a monopoly market to earn the same profit. Profit under both the conditions is targeted at Rs. 2,00,000. The Variable Cost per transistor is Rs. 100 and the total Fixed Cost is Rs. 37,000. You are required to find out the unit Selling Prices, both under monopoly as well as competitive conditions. [B.Com. (Hons), Delhi University]

Modified Date: Mon, Jul 05, 2010 05:08:16 PM

Output Date: Tue, Jul 06, 2010 12:23:48 PM

Rev II

Project: Management Accounting_Debarshi Bhattacharyya ACE Pro India Pvt. Ltd. File: X:\Pearson\Management Accounting_Debarshi Bhattacharyya\MAIN\M10\LAYOUT_M10\M10_DEBA_ISBN_EN_SE_C10.indd

650

MANAGEMENT ACCOUNTING

Solution Under monopoly condition: Total Variable Cost (2,000 × Rs. 100) Total Fixed Cost Total Cost Target Profit Total Sales

Add:

Rs. 2,00,000 37,000 2,37,000 2,00,000 4,37,000

∴ Selling Price per unit under monopoly condition = Rs. 4,37,000 ÷ 2,000 = Rs. 218.50 Under competitive condition: Total Variable Cost (10,000 × Rs. 100) Total Fixed Cost Total Cost Target Profit Total Sales

Add:

Rs. 10,00,000 37,000 10,37,000 2,00,000 12,37,000

∴ Selling Price per unit under competitive condition = Rs. 12,37,000 ÷ 10,000 = Rs. 123.70 Problem 9 Alpha Ltd furnished the following information: Selling Price—Rs. 10 per unit Variable Cost—Rs. 6 per unit It is expected that Variable Cost will increase by 20%. What will be the Selling Price per unit if the company wishes to maintain the same P/V Ratio. [B.Com. (Hons), Calcutta University—2004] Solution Present P/ V Ratio =

S−V 10 − 6 × 100 = × 100 = 40% S 10

It is expected that: Variable Cost (V) per unit = Rs. 6 + (20% of Rs. 6) = Rs. 7.20, and P/V Ratio = 40%. Then New Selling Price (S) per unit = ? S−V Now P/ V Ratio = × 100 S S − 7.20 40 = × 100 or S 40 ÷ 100 = S − 7.20 or S or 0.40 S = S – 7.20 or S − 0.40 S = 7.20 or 0.60 S = 7.20 7.20 ∴ S= = 12 0.60 ∴ New Selling Price per unit will be Rs. 12 so as to maintain the present P/V Ratio

Modified Date: Mon, Jul 05, 2010 05:08:16 PM

Output Date: Tue, Jul 06, 2010 12:23:48 PM

Rev II

Project: Management Accounting_Debarshi Bhattacharyya ACE Pro India Pvt. Ltd. File: X:\Pearson\Management Accounting_Debarshi Bhattacharyya\MAIN\M10\LAYOUT_M10\M10_DEBA_ISBN_EN_SE_C10.indd

MARGINAL COSTING AND COST–VOLUME–PROFIT ANALYSIS

651

Problem 10 You are given the following data: Fixed expenses = Rs. 4,000 BEP = Rs. 10,000 Calculate: i. P/V Ratio. ii. Profit when sales are Rs. 20,000. iii. New BEP, if the Selling Price is reduced by 20%. [B.Com. (Hons), Delhi University] Solution i. We know that at BEP: P/V Ratio =

Fixed Cost × 100 BEPS

Here, P/V Ratio =

4,000 × 100 10,000

∴ P/V Ratio = 40% ii. We know that P/V Ratio = F + P × 100 S 4,000 + P Here, 40 = × 100 20,000 or 4,000 + P = 8,000 or P = 8,000 − 4,000 ∴ Required Profit = Rs. 4,000 iii. We know, P/V Ratio =

C × 100 S

Here, P/V Ratio = 40% ∴ If Sales (S) = 100, then Contribution (C) = 40 ∴ Variable Cost (V) = S − C = 100 − 40 = 60 [As S = C + V, then V = S − C] Now, if the Selling Price is reduced by 20%, then New Selling Price = 100 − (20% of 100) = 80 Less: Variable Cost = 60 [As Variable Cost is not changing] New Contribution = 20 New Contribution 20 × 100 = × 100 = 25% New Sales 80 Fixed Cost ∴ New BEPS = × 100 [as Fixed Cost does not change] New P/ V Ratio

∴ New P/ V Ratio =

or

4,000 × 100 25 ∴ New BEPS = Rs. 16,000 New BEPS =

Modified Date: Mon, Jul 05, 2010 05:08:16 PM

Output Date: Tue, Jul 06, 2010 12:23:48 PM

Rev II

Project: Management Accounting_Debarshi Bhattacharyya ACE Pro India Pvt. Ltd. File: X:\Pearson\Management Accounting_Debarshi Bhattacharyya\MAIN\M10\LAYOUT_M10\M10_DEBA_ISBN_EN_SE_C10.indd

652

MANAGEMENT ACCOUNTING

Problem 11 P/V Ratio = 50% MS = 40% Actual Sales = Rs. 5,00,000 i. ii. iii. iv.

Find out: MS at Actual Sales. BEPS. Profit at Actual Sales. Fixed Cost.

Solution i. Here, MS = 40% It means, if Actual Sales = 100, then MS = 40 Actual Sales = Rs. 5,00,000 ∴ MS = 40% of Rs. 5,00,000 = Rs. 2,00,000 ii. Again, if Actual Sales = 100, then MS = 40 Now, BEPS = Actual Sales − MS ∴ BEPS = 100 – 40 = 60% ∴ BES = 60% of Actual Sales = 60% of Rs. 5,00,000 = Rs. 3,00,000 iii. Here, MS = Rs. 2,00,000, and P/V Ratio = 50% C ⎡ ⎤ ∴ Contribution at MS = 50% of Rs. 2,00,000 = Rs. 1,00,000 ⎢∵ P/V Ratio = × 100⎥ S ⎣ ⎦ Again, Contribution at MS = Profit ∴ Profit at Actual Sales = Rs. 1,00,000 iv. BEPS = Rs. 3,00,000, and P/V Ratio = 50% Then, F = ? We know, at BEP: F P/V Ratio = × 100 BEPS F or 50 = × 100 3,00,000 (3,00,000 × 50) or F = 100 ∴ Fixed Cost = Rs. 1,50,000 Problem 12 A Ltd maintains an MS of 37.5% with an overall Contribution–Sales ratio of 40%. If the Fixed Costs amount to Rs. 5 lakhs, calculate the following: (a) BES; (b) Total Sales; (c) Total Variable Cost; (d) Current Profit; (e) MS; (f) What would be the impact of change in Contribution–Sales ratio to 50%; and (g) New MS if the Sales Value is increased by 7.5%. [B.B.M., Bangalore University—December 2008]

Modified Date: Mon, Jul 05, 2010 05:08:16 PM

Output Date: Tue, Jul 06, 2010 12:23:48 PM

Rev II

Project: Management Accounting_Debarshi Bhattacharyya ACE Pro India Pvt. Ltd. File: X:\Pearson\Management Accounting_Debarshi Bhattacharyya\MAIN\M10\LAYOUT_M10\M10_DEBA_ISBN_EN_SE_C10.indd

653

MARGINAL COSTING AND COST–VOLUME–PROFIT ANALYSIS

Solution i. Here, the overall Contribution–Sales Ratio (i.e., P/V Ratio) = 40%, and Fixed Cost = Rs. 5,00,000 F 5,00,000 ∴ BES = × 100 = × 100 = Rs. 12,50,000 P/V ratio 40 ii. Here, MS = 37.5%. It means that if the Total Sales = 100, then MS = 37.50 ∴ BEPS = Total Sales − MS = 100 − 37.50 = 62.50 Here, BEPS [as computed in (a) above] = Rs. 12,50,000 ∴ Total Sales = 100 ÷ 62.50 = Rs. 12,50,000 = Rs. 20,00,000 C × 100 = 40% S It means that if Sales = 100, then Contribution = 40 ∴ Variable Cost = S − C = 100 − 40 = 60 [As C = S − V] Here, Total Sales = Rs. 20,00,000 60 ∴ Total Variable Cost = × 20,00,000 = Rs. 12,00,000 100

iii. Given P/V Ratio (Contribution − Sales Ratio) =

iv. Current profit = Total Sales − Total Variable Cost − Total Fixed Cost Here, Current Profit = 20,00,000 − 12,00,000 − 5,00,000 = Rs. 3,00,000 v. MS = 37.50% of Total sales = 37.50% of Rs. 20,00,000 = Rs. 7,50,000 vi. If Contribution–Sales Ratio (i.e., P/V Ratio) is changed to 50%, then F 5,00,000 New BEPS = × 100 = × 100 = Rs. 10,00,000 P/Vratio 50 vii. New Sales = 20,00,000 + (7.5% of 20,00,000) = Rs. 21,50,000 ∴ New MS = New Sales − BEPS = 21,50,000 − 12,50,000 = Rs. 9,00,000 Problem 13 From the following information, compute: i. Fixed Cost. ii. BEP. Units 4,000 5,000

Profit/Loss (Rs.) 20,000 20,000

[B.Com. (Hons), Kalyani University—2004] Solution i. Contribution per unit = =

Change in Profit Change in Sales Volume Rs. 20,000 − ( − Rs.20,000)

Modified Date: Mon, Jul 05, 2010 05:08:16 PM

5,000 − 4,000

=

Rs. 40,000 = Rs. 40 1,000

Output Date: Tue, Jul 06, 2010 12:23:48 PM

Rev II

Project: Management Accounting_Debarshi Bhattacharyya ACE Pro India Pvt. Ltd. File: X:\Pearson\Management Accounting_Debarshi Bhattacharyya\MAIN\M10\LAYOUT_M10\M10_DEBA_ISBN_EN_SE_C10.indd

654

MANAGEMENT ACCOUNTING

∴ At 5,000 units of output: Rs. 2,00,000 20,000 1,80,000

Contribution (5,000 units × Rs. 40) Profit Fixed Cost

Less:

ii. BEPS (in units ) =

Fixed Cost

Contribution per unit = Rs. 1,80,000 ÷ Rs. 40 = 4,500 units

Problem 14 A company has a Fixed Cost of Rs. 3,00,000. On a sale of 15,000 units which equal to 40% MS, it earned a profit of Rs. 60,000. Calculate the following: i. BEP in units. ii. Total present sales in units. iii. Total units sold at which it suffered a loss of Rs. 62,492. iv. If the present Fixed Cost increases by 15%, what is the revised BEP in units and how many units should be sold to earn a profit of Rs. 1,15,000? [B.Com. (Hons), Mumbai University—October 2007] Solution i. Present MS in units = 15,000 units Given, Present Profit = Rs. 60,000 Again, Contribution at MS = Profit = Rs. 60,000 Contribution at MS Rs. 60,000 = = Rs. 4 MS in units 15,000 Fixed Cost Rs. 3,00,000 ∴ BEPS (in units ) = = = 75,000 units Contribution per unit Rs. 4 ∴ Contribution per unit =

ii. Present MS in units = 15,000 = 40% (of present total sales) 100 ∴ Total Present Sales in units = × 15,000 = 37,500 units 40 iii. We know that Sales in units =

F+P Contribution per unit

3,00,000 + ( − 62492)

2,37,508 4 4 ∴ Required Total Sales in units = 59,377 units or

Sales in units =

=

iv. Revised Fixed Cost = 3,00,000 + (15% of 3,00,000) = Rs. 3,45,000 Revised Fixed Cost Rs. 3,45,000 ∴ Revised BEPS (in units ) = = = 86,250 units Contribution per unit Rs. 4 ∴ Required Sales in units =

Modified Date: Mon, Jul 05, 2010 05:08:16 PM

F+P Rs. 3,45,000 + Rs. 1,15,000 = = 1,15,000 units Contribution per unit Rs. 4

Output Date: Tue, Jul 06, 2010 12:23:48 PM

Rev II

Project: Management Accounting_Debarshi Bhattacharyya ACE Pro India Pvt. Ltd. File: X:\Pearson\Management Accounting_Debarshi Bhattacharyya\MAIN\M10\LAYOUT_M10\M10_DEBA_ISBN_EN_SE_C10.indd

MARGINAL COSTING AND COST–VOLUME–PROFIT ANALYSIS

655

Problem 15 X Ltd sells 8,000 units of its products at a loss of Rs. 16,000. Variable Cost per unit is Rs. 12 and the Total Fixed Cost is Rs. 48,000. Calculate: i. P/V Ratio. ii. The number of units to be sold to earn a profit of Rs. 10,000. iii. The amount of profit from a sale of 20,000 units. [B.Com. (Hons), Calcutta University—1999] Solution i. P/V Ratio =

F + P ( or − L ) S

× 100

F−L × 100 [∵ Loss has been incurred ] S 48,000 − 16,000 or P/ V Ratio = × 100 1,28,000 32,000 or P/ V Ratio = × 100 1,28,000 ∴ P/V Ratio = 25% In the given case, P/V Ratio =

ii. This part of the problem may be solved by either of the following two approaches: First Alternative: Rs. 16 12 4

Selling Price per unit Variable Cost per unit Contribution per unit

We know that Sales (in units ) =

F+P Contribution per unit

48,000 + 10,000 = 14,500 units 4 ∴ To earn a profit of Rs. 10,000, 14,500 units are to be sold Here, Sales (in units ) =

Second Alternative: F+P × 100 S 48,000 + 10,000 Here, 25 = × 100 S 58,000 or S = × 100 = Rs. 2,32,000 25 ∴ Sales (in value) to earn a profit of Rs. 10,000 = Rs. 2,32,000 ∴ Sales (in volume) to earn a profit of Rs. 10,000 = Rs. 2,32,000 ÷ Rs. 16 = 14,500 units Again, P/V Ratio =

Modified Date: Mon, Jul 05, 2010 05:08:16 PM

Output Date: Tue, Jul 06, 2010 12:23:48 PM

Rev II

Project: Management Accounting_Debarshi Bhattacharyya ACE Pro India Pvt. Ltd. File: X:\Pearson\Management Accounting_Debarshi Bhattacharyya\MAIN\M10\LAYOUT_M10\M10_DEBA_ISBN_EN_SE_C10.indd

656

MANAGEMENT ACCOUNTING

iii. Here, Given Sales (in units) = 20,000 units ∴ Sales (in value) = 20,000 units × Rs. 16 = Rs. 3,20,000 We know that: F+P P/V Ratio = × 100 S 48,000 + P or 25 = × 100 3,20,000 3,20,000 × 25 or 48,000 + P = 100 or 48,000 + P = 80,000 ∴ P = 80,000 – 48,000 = 32,000 ∴ At a sale of 20,000 units, the profit earned is Rs. 32,000 Working Notes We know that S = F + V + P or – L Here, as loss has been incurred, S = F + V – L Total Fixed Cost Total Variable Cost at 8,000 units (8,000 units × Rs. 12) Total Cost at 8,000 units Loss Total Selling Price at 8,000 units

Less:

∴ Selling Price per unit =

Rs. 48,000 96,000 1,44,000 16,000 1,28,000

Rs. 1,28,000 = Rs. 16 8,000 units

Problem 16 Sale of product amounts to 200 units per month at Rs. 10 per unit. Fixed overhead is Rs. 400 per month and Variable Cost is Rs. 6 per unit. There is a proposal to reduce the Selling Price by 10%. Calculate: (a) Present P/V Ratio and BEP in units; (b) Future P/V Ratio and BEP in units; and (c) How many units to be sold to earn the present total profit? [B.B.M., Bangalore University—December 2006] Solution i. Present Contribution per unit (C) = S − V = Rs. 10 − Rs. 6 = Rs. 4 C 4 ∴ Present P/ V Ratio = × 100 = × 100 = 40% S 10 F Rs. 400 ∴ Present BEP in units = = = 100 units Contribution per unit Rs. 4 ii. Future Selling Price per unit = Rs. 10 − (10% of Rs. 10) = Rs. 9 ∴ Future Contribution per unit (C) = S − V = Rs. 9 − Rs. 6 = Rs. 3 ∴ Future P/ V Ratio =

C 3 × 100 = × 100 = 33.33% S 9

Modified Date: Mon, Jul 05, 2010 05:08:16 PM

Output Date: Tue, Jul 06, 2010 12:23:48 PM

Rev II

Project: Management Accounting_Debarshi Bhattacharyya ACE Pro India Pvt. Ltd. File: X:\Pearson\Management Accounting_Debarshi Bhattacharyya\MAIN\M10\LAYOUT_M10\M10_DEBA_ISBN_EN_SE_C10.indd

MARGINAL COSTING AND COST–VOLUME–PROFIT ANALYSIS

∴ Future BEP in units =

657

F Rs. 400 = = 133 units Contribution per unit Rs. 3

iii. Present Total Profit = S − V − F = (200 × Rs. 10) − (200 × Rs. 6) − Rs. 400 = Rs. 400 ∴ No. of units to be sold at reduced selling F+P = price to earn the above present total profit Contribution per unit Rs. 400 + Rs. 400 = = 267 units Rs. 3 Problem 17 Following are the information available in respect of a product at an activity level of 10,000 units: Rs. 6 4 2 3 15 20 5

Direct Materials per unit Direct Labour per unit Variable Overheads per unit Fixed Overheads per unit Total Cost per unit Selling Price per unit Profit per unit

How many units to be sold for the BE?

Solution Rs. Less:

Selling Price per unit Variable Cost per unit: Direct Materials Direct Labour Variable Overhead Contribution per unit

6 4 2

Rs. 20

12 8

Total Fixed Cost at an activity level of 10,000 units = Rs. 30,000 (10,000 units × Rs. 3) Total Fixed Cost 30,000 ∴ BES (in units ) = = = 3,750 units Contribution per unit 8 Problem 18 A company budgets for a production of 1,50,000 units. The Variable Cost per unit is Rs. 14 and Fixed Cost is Rs. 2 per unit. The company fixes its Selling Price to fetch a profit of 15% on cost. i. What is the BEP? ii. What is the P/V Ratio? iii. If it reduces its Selling Price by 5%, how much does the revised Selling Price affect the BEP and the P/V Ratio? iv. If a profit increase of 10% is desired more than the budget, what should be the sales at the reduced prices? [B.Com. (Hons), Delhi University]

Modified Date: Mon, Jul 05, 2010 05:08:16 PM

Output Date: Tue, Jul 06, 2010 12:23:48 PM

Rev II

Project: Management Accounting_Debarshi Bhattacharyya ACE Pro India Pvt. Ltd. File: X:\Pearson\Management Accounting_Debarshi Bhattacharyya\MAIN\M10\LAYOUT_M10\M10_DEBA_ISBN_EN_SE_C10.indd

658

MANAGEMENT ACCOUNTING

Solution Variable Cost for Budgeted Production of 1,50,000 units (1,50,000 × Rs. 14) Fixed Cost for Budgeted Production of 1,50,000 units (1,50,000 × Rs. 2) Total Cost of Budgeted Production of 1,50,000 units Budgeted Profit (15% on 24,00,000) Selling Price of Budgeted Production of 1,50,000 units

Add:

∴ Selling Price per unit =

Rs. 21,00,000 3,00,000 24,00,000 3,60,000 27,60,000

Rs. 27,60,000 = Rs. 18.40 1,50,000

∴ Contribution per unit = S − V = Rs. 18.40 − Rs. 14 = Rs. 4.40 i. BEP (in units ) =

Fixed Cost 3,00,000 = = 68,182 units Contribution per unit 4.40

BEP (in value) = 68,182 units × Rs. 18.40 = Rs. 12,54,549 C 4.40 × 100 = 23.91% ii. P/ V Ratio = × 100 = S 18.40 iii. Current Selling Price per unit Reduction in Selling Price (5% of 18.40) New Selling Price per unit Variable Cost per unit New Contribution per unit

Less: Less:

∴ New BEPS (in units ) = ∴ New P/ V Ratio =

Rs. 18.40 0.92 17.48 14.00 3.48

Fixed Cost 3,00,000 = = 86,207 units New Contribution per unit 3.48

New Contribution per unit 3.48 × 100 = × 100 = 19.91% New Selling Price per unit 17.48

iv. Budgeted Profit [as computed above] Desired Increase in Profit (10% of 3,60,000) Desired Profit

Add:

Rs. 3,60,000 36,000 3,96,000

∴ Sales (in units) required to achieve the Desired Profit at the reduced Selling Price =

F+P 3,00,000 + 3,96,000 = = 2,00,000 units New Contribution per unit 3.48

∴ Sales (in value) required to achieve the desired profit at the reduced Selling Price = 2,00,000 units × Rs. 17.48 = Rs. 34,96,000

Problem 19 On the basis of the following information, you are required to ascertain: (a) Sales to BE; and (b) Sales to earn a profit of Rs. 60,000, if the Selling Price per unit is reduced by Re. l:

Modified Date: Mon, Jul 05, 2010 05:08:16 PM

Output Date: Tue, Jul 06, 2010 12:23:48 PM

Rev II

Project: Management Accounting_Debarshi Bhattacharyya ACE Pro India Pvt. Ltd. File: X:\Pearson\Management Accounting_Debarshi Bhattacharyya\MAIN\M10\LAYOUT_M10\M10_DEBA_ISBN_EN_SE_C10.indd

MARGINAL COSTING AND COST–VOLUME–PROFIT ANALYSIS

659

Rs. 1,60,000 96,000 48,000

Sales (10,000 units) Variable Cost Fixed Cost

[B.Com. (Hons), Calcutta University—2001] Solution i. This part of the problem can be solved by either of the following two approaches: First Alternative:

Less:

Sales for 10,000 units Variable Costs for 10,000 units Contribution for 10,000 units

Rs. 1,60,000 96,000 64,000

Rs. 64,000 = Rs. 6.40 10,000 units Fixed Cost We know that BEPS (in units ) = Contribution per unit

∴ Contribution per unit =

48,000 = 7,500 units 6.40 Rs. 1,60,000 Again, Selling Price per unit = = Rs. 16 10,000 units ∴ BES (in value) = BEPS (in units) × Selling Price per unit = 7,500 units × Rs. 16 = Rs. 1,20,000 ∴ BEPS (in units ) =

Second Alternative: S−V × 100 S 1,60,000 − 96,000 Here, P/ V Ratio = × 100 1,60,000 We know that P/ V Ratio =

∴ P/V Ratio = 40% We know that at BEP: F P/V Ratio = × 100 BEPS 48,000 or 40 = × 100 BEPS 100 or BEPS = 48,000 × 40 ∴ BES (in value) = Rs. 1,20,000 Again, Selling Price per unit = Rs. 1,60,000 ÷ 10,000 units = Rs. 16 ∴ BES (in units) = Rs. 1,20,000 ÷ Rs. 16 = Rs. 7,500 units ii. Existing Selling Price per unit = Rs. 16 [as ascertained in (i) above] ∴New Selling Price per unit = Rs. 16 – Re.1 = Rs. 15

Modified Date: Mon, Jul 05, 2010 05:08:16 PM

Output Date: Tue, Jul 06, 2010 12:23:48 PM

Rev II

Project: Management Accounting_Debarshi Bhattacharyya ACE Pro India Pvt. Ltd. File: X:\Pearson\Management Accounting_Debarshi Bhattacharyya\MAIN\M10\LAYOUT_M10\M10_DEBA_ISBN_EN_SE_C10.indd

660

MANAGEMENT ACCOUNTING

Existing Variable Cost per unit = Rs. 96,000 ÷ 10,000 units = Rs. 9.60 As not otherwise stated in this regard, New Variable Cost per unit = Rs. 9.60 As Selling Price per unit is reduced by Re. 1, S−V 15 − 9.60 ∴ New P/ V Ratio = × 100 = × 100 = 36% S 15 Again, we know F+P P/V Ratio = × 100 S 48,000 + 60,000 Here, 36 = × 100 S 1,08,000 or S = × 100 36 ∴ Sales = Rs. 3,00,000 ∴ Required sales to earn a profit of Rs. 60,000 = Rs. 3,00,000 Problem 20 Rainbow Ltd sold goods for Rs. 30,00,000 in a year. In that year, the Variable Costs were Rs. 6,00,000 and Fixed Costs were Rs. 8,00,000. Find out: i. P/V Ratio. ii. BES. iii. BES if the Selling Price was reduced by 10% and Fixed Costs were increased by Rs. 1,00,000. [B.Com. (Hons), Calcutta University—2000]

Solution S−V × 100 S Where, S = Selling Price and V = Variable Cost 30,00,000 − 6,00,000 ∴ Here, P/ V Ratio = × 100 30,00,000 ∴ P/V Ratio = 80% ii. Again, at BEP, F P/V Ratio = × 100 BEPS Where, F = Fixed Cost BEPS = BEPS 8,00,000 ∴ Here, 80 = × 100 BEPS 8,00,000 or BEPS = × 100 80 ∴ BES = Rs. 10,00,000 i. P/V Ratio =

Modified Date: Mon, Jul 05, 2010 05:08:16 PM

Output Date: Tue, Jul 06, 2010 12:23:48 PM

Rev II

Project: Management Accounting_Debarshi Bhattacharyya ACE Pro India Pvt. Ltd. File: X:\Pearson\Management Accounting_Debarshi Bhattacharyya\MAIN\M10\LAYOUT_M10\M10_DEBA_ISBN_EN_SE_C10.indd

MARGINAL COSTING AND COST–VOLUME–PROFIT ANALYSIS

661

Note P/V Ratio remains the same as it was, because neither the Selling Price per unit nor the Variable Cost per unit changes. ∴ In this case also, P/V Ratio = 80% iii. In this case, the Selling Price per unit decreases by 10%. Consequently, P/V Ratio will be changed and now new P/V Ratio is to be computed. ∴ New Total Selling Price = Rs. 30,00,000 – 10% of Rs. 30,00,000 = Rs. 27,00,000 New Total Fixed Cost = Rs. 8,00,000 + Rs. 1,00,000 = Rs. 9,00,000 S−V × 100 S 27,00,000 − 6,00,000 or New P/ V Ratio = × 100 27,00,000 ∴ New P/V Ratio = 77.7777% ∴ New P/ V Ratio =

Note As the number of units produced remains the same and Variable Cost per unit also remains the same, the Total Variable Cost also remains the same, that is, Rs. 6,00,000. We know that at BEP, F P/V Ratio = × 100 BEPS 9,00,000 or 77.7777 = × 100 BEPS 9,00,000 ∴ BEPS = × 100 = Rs. 11,57,144 77.7777 ∴ New BES = Rs. 11,57,144 Problem 21 A company had incurred fixed expenses of Rs. 4,50,000 with a sales of Rs. 15,00,000 and earned a profit of Rs. 3,00,000 during the first half-year. In the second half, it suffered a loss of Rs. 1,50,000. Calculate: i. P/V Ratio, BEP and MS for the first half-year. ii. Sales for the second half-year assuming that the Selling Price and fixed expenses remained unchanged during the second half-year. iii. The BEP and MS for the whole year. [C.S. (Inter)—Adapted] Solution i. P/V Ratio =

F+P × 100 S

Modified Date: Mon, Jul 05, 2010 05:08:16 PM

Output Date: Tue, Jul 06, 2010 12:23:48 PM

Rev II

Project: Management Accounting_Debarshi Bhattacharyya ACE Pro India Pvt. Ltd. File: X:\Pearson\Management Accounting_Debarshi Bhattacharyya\MAIN\M10\LAYOUT_M10\M10_DEBA_ISBN_EN_SE_C10.indd

662

MANAGEMENT ACCOUNTING

Here, for the first half-year: 4,50,000 + 3,00,000 P/V Ratio = × 100 = 50% 15,00,000 F 4,50,000 Then, BEP in value = × 100 = × 100 = Rs. 9,00,000 P/V ratio 50 ∴ MS = Total Sales − BEPS = Rs. 15,00,000 − Rs. 9,00,000 = Rs. 6,00,000 ii. P/V Ratio =

F−L × 100 S

Here, in the second half-year: 4,50,000 − 1,50,000 50 = × 100 S 3,00,000 × 100 or S = = 6,00,000 50 ∴ Sales for the second half-year = Rs. 6,00,000 iii. Total Sales for the whole year = Rs. 15,00,000 + Rs. 6,00,000 = Rs. 21,00,000 Total Fixed Cost for the whole year = Rs. 4,50,000 + Rs. 4,50,000 = Rs. 9,00,000 P/V Ratio remains constant throughout the year as there is no change either in the Selling Price per unit or in the Variable Cost per unit during both the half-years. ∴ BEP for the whole year =

F 9,00,000 × 100 = × 100 = Rs. 18,00,000 P/V ratio 50

∴ MS for the whole year = Total Sales for the whole year − BEPS for the whole year = Rs. 21,00,000 − Rs. 18,00,000 = Rs. 6,00,000 Problem 22 A company earned a profit of Rs. 30,000 during the year 1994–95. If the Marginal Cost and Selling Price of a product are Rs. 8 and Rs. 10 per unit, respectively, find out the amount of ‘MS.’ [C.A. (Inter)—May 1995] Solution

Less:

Here, Selling Price per unit Marginal Cost per unit (i.e., Variable Cost per unit) Contribution per unit (C)

Rs. 10 8 2

C 2 × 100 = × 100 = 20% S 10 Again, we know that Contribution at MS = Profit Here, Profit = Rs. 30,000 ∴ Contribution at MS = Rs. 30,000 ∴ At MS, Contribution at MS C ⎡ ⎤ P/V Ratio = × 100 ⎢∵ P / V Ratio = × 100⎥ MS S ⎣ ⎦ ∴ P/ V Ratio =

Modified Date: Mon, Jul 05, 2010 05:08:16 PM

Output Date: Tue, Jul 06, 2010 12:23:48 PM

Rev II

Project: Management Accounting_Debarshi Bhattacharyya ACE Pro India Pvt. Ltd. File: X:\Pearson\Management Accounting_Debarshi Bhattacharyya\MAIN\M10\LAYOUT_M10\M10_DEBA_ISBN_EN_SE_C10.indd

MARGINAL COSTING AND COST–VOLUME–PROFIT ANALYSIS

663

30,000 × 100 MS 30,000 or MS = × 100 = 1,50,000 20

Here, 20 =

∴ MS = Rs. 1,50,000 Problem 23 A company has a P/V Ratio of 40%. By what percentage must sales be increased to offset: i. 10% reduction in Selling Price? ii. 20% reduction in Selling Price? [B.Com. (Hons), Delhi University] Solution C × 100 S Existing P/V Ratio = 40% i.e., If Selling Price (S) = 100, then Contribution (C) = 40 Again, S − C = V [as C = S − V] ∴ Existing Variable Cost (V) = 100 − 40 = 60 P/V Ratio =

i. Now, if there is a reduction in the Selling Price by 10%, then: New Selling Price (S) = 100 – (10% of 100) = 90 New Variable Cost (V) (remains same) = 60 New Contribution (C) = 30 Now, in order to maintain the existing contribution of 40 under the changed scenario, the Sales Volume must be increased as the Variable Cost also remains the same. 40 × 90 = 120 30 ∴ Increase in Sales Volume required = 120 − 90 = 30 ∴ % increase in Sales Volume required to Increase in Sales Volume = × 100 offset the 10% reduction in the Selling Price Original Sales Volume under the changed scenario ∴ New Sales Volume =

=

30 × 100 = 33.33% 90

ii. Now, if there is a reduction in the Selling Price by 20%, then, New Selling Price (S) = 100 − (20% of 100) = 80 New Variable Cost (V) (remains same) = 60 New Contribution (C) = 20 Now, in order to maintain the existing contribution of 40 under the changed scenario, the sales volume must be increased as the Variable Cost also remains the same. 40 ∴ New Sales Volume = × 80 = 160 20 ∴ Increase in Sales Volume required = 160 − 80 = 80

Modified Date: Mon, Jul 05, 2010 05:08:16 PM

Output Date: Tue, Jul 06, 2010 12:23:48 PM

Rev II

Project: Management Accounting_Debarshi Bhattacharyya ACE Pro India Pvt. Ltd. File: X:\Pearson\Management Accounting_Debarshi Bhattacharyya\MAIN\M10\LAYOUT_M10\M10_DEBA_ISBN_EN_SE_C10.indd

664

MANAGEMENT ACCOUNTING

∴ % increase in the Sales Volume Required to Increase in Sales Volume = × 100 offset the 20% reduction in the Selling Price Original Sales Volume under the changed scenario

=

80 × 100 = 100% 80

Problem 24 The following figures are available from the books of Makalu Ltd: Year Sales Loss Profit

2008 Rs. 2,00,000 20,000 –

2009 Rs. 3,00,000 – 30,000

Calculate: i. ii. iii. iv. v.

Fixed Cost. BEPS. MS for 2009. Sales required to earn a profit of Rs. 60,000. Profit or Loss at a sales of Rs. 2,50,000.

Solution Change in Profit × 100 Change in Sales Profit for 2009 − Profit for 2008

i. We know that P/V Ratio = Here, P/V Ratio = or

× 100 Sales for 2009 − Sales for 2008 30,000 − ( − 20,000) P/V Ratio = × 100 [∵ Profit for 2008 = (−) 20,000] 3,00,000 − 2,00,000

30,000 + 20,000 × 100 1,00,000 ∴ P/V Ratio = 50% F+P Again, P/V Ratio = × 100 S In 2009, F + 30,000 P/ V Ratio = × 100 [∵ In 2009, Profit = Rs. 30,000 and Sales = Rs. 3,00,000] 3,00,000 F + 30,000 or 50 = × 100 [as P / V Ratio = 50% calculated under working note] 3,00,000 50 or F + 30,000 = 3,00,000 × 100 or F + 30,000 = 1,50,000 or F = 1,50,000 – 30,000 ∴ Fixed Cost = Rs. 1,20,000 or P/V Ratio =

Modified Date: Mon, Jul 05, 2010 05:08:16 PM

Output Date: Tue, Jul 06, 2010 12:23:48 PM

Rev II

Project: Management Accounting_Debarshi Bhattacharyya ACE Pro India Pvt. Ltd. File: X:\Pearson\Management Accounting_Debarshi Bhattacharyya\MAIN\M10\LAYOUT_M10\M10_DEBA_ISBN_EN_SE_C10.indd

MARGINAL COSTING AND COST–VOLUME–PROFIT ANALYSIS

665

ii. We know that at BEP, F P/V Ratio = × 100 BEPS 1,20,000 or 50 = × 100 BEPS 1,20,000 or BEPS = × 100 50 ∴ BEPS = Rs. 2,40,000 iii. We know Actual Sales = BEPS + MS ∴ MS = Actual Sales – BEPS ∴ MS in 2009 = Actual Sales in 2009 – BEPS or MS in 2009 = 3,00,000 – 2,40,000 ∴ MS in 2009 = Rs. 60,000 iv. Here, if Profit = Rs. 60,000, Sales = ? F+P × 100 S 1,20,000 + 60,000 or 50 = × 100 S 1,80,000 or S = × 100 50 ∴ Sales = Rs. 3,60,000 ∴ Sales required to earn a profit of Rs. 60,000 = Rs. 3,60,000 P/V Ratio =

v. Here, Sales = Rs. 2,50,000, Profit or Loss = ? F+P × 100 S 1,20,000 + P or 50 = × 100 2,50,000 2,50,000 × 50 or 1,20,000 + P = 100 or 1,20,000 + P = 1,25,000 ∴ Profit = 1,25,000 – 1,20,000 = 5,000 ∴ Profit at a Sales of Rs. 2,50,000 = Rs. 5,000 P/V Ratio =

Problem 25 A company has annual Fixed Costs of Rs. 14,00,000. In 2009, the sales amounted to Rs. 60,00,000 as compared with Rs. 45,00,000 in 2008 and the profit in 2009 was Rs. 4,20,000 which was higher than in 2008. i. At what level of sales does the company BE? ii. Determine the profit or loss on sales of Rs. 80,00,000. iii. If there is a reduction in the Selling Price in 2010 by 10% and the company desires to earn the same profit as in 2009, what would be the required sales? [B.Com. (Hons), Delhi University]

Modified Date: Mon, Jul 05, 2010 05:08:16 PM

Output Date: Tue, Jul 06, 2010 12:23:48 PM

Rev II

Project: Management Accounting_Debarshi Bhattacharyya ACE Pro India Pvt. Ltd. File: X:\Pearson\Management Accounting_Debarshi Bhattacharyya\MAIN\M10\LAYOUT_M10\M10_DEBA_ISBN_EN_SE_C10.indd

666

MANAGEMENT ACCOUNTING

Solution i. P/V Ratio =

Change in Profit × 100 Change in Sales

4,20,000 × 100 = 28% 60,00,000 − 45,00,000 Fixed Cost 14,00,000 ∴ BES = × 100 = × 100 = Rs. 50,00,000 P/V Ratio 28 Here, P/V Ratio =

F+P × 100 S 14,00,000 + P Here, 28 = × 100 80,00,000 28 × 80,00,000 or 14,00,000 + P = 100 or 14,00,000 + P = 22,40,000 ∴ Required Profit = 22,40,000 − 14,00,000 = Rs. 8,40,000

ii. P/V Ratio =

F+P × 100 S Now, in 2009, 14,00,000 + P 28 = × 100 60,00,000 28 × 60,00,000 or 14,00,000 + P = 100 or 14,00,000 + P = 16,80,000 ∴ Profit earned in 2009 = 16,80,000 − 14,00,000 = Rs. 2,80,000 Now, till 2009, P/V Ratio = 28% C ⎡ ⎤ i.e., if the Selling Price (S) = 100, then Contribution (C) = 28 ⎢ as P/V ratio = × 100⎥ S ⎣ ⎦ Then, Variable Cost (V) = S − C = 100 − 28 = 72 [as S − V = C] Now, in 2010, S = 100 − (10% of 100) = 90 V (same as before) = 72 ∴ C = S − V = 90 − 72 = 18 C 18 ∴ P/ V Ratio in 2010 = × 100 = × 100 = 20% S 90 If the company desires to earn the same profit as in 2009 (i.e., Rs. 2,80,000) in the year 2010 under the reduced Selling Price, then: F+P P/V Ratio = × 100 S 14,00,000 + 2,80,000 or 20 = × 100 S 16,80,000 × 100 or S = 20 ∴ Required Sales in 2010 = Rs. 84,00,000

iii. P/V Ratio =

Modified Date: Mon, Jul 05, 2010 05:08:16 PM

Output Date: Tue, Jul 06, 2010 12:23:48 PM

Rev II

Project: Management Accounting_Debarshi Bhattacharyya ACE Pro India Pvt. Ltd. File: X:\Pearson\Management Accounting_Debarshi Bhattacharyya\MAIN\M10\LAYOUT_M10\M10_DEBA_ISBN_EN_SE_C10.indd

MARGINAL COSTING AND COST–VOLUME–PROFIT ANALYSIS

667

Problem 26 A company manufactures a single product having a Marginal Cost of Re. 0.75 a unit. Fixed Costs are Rs. 12,000. The market is such that up to 40,000 units can be sold at Rs. 1.50 a unit, but any additional sales must be made at Re. 1 a unit only. There is a planned profit of Rs. 20,000. How many units to be made and sold? [B.Com. (Hons), Delhi University]

Solution Firstly, let us see to what extent of the planned profit can be achieved by sale of the first 40,000 units. Contribution per unit from the sale of first 40,000 units of output = S − V = Rs. 1.50 − Re. 0.75 = Re. 0.75 F+P We know that sales (in units) = Contribution per unit ∴ For sale of 40,000 units: 40,000 = (12,000 + P) ÷ 0.75 or 12,000 + P = 30,000 ∴ P = 18,000 ∴ Out of the total planned profit of Rs. 20,000, a profit of Rs. 18,000 is earned from the sale of first 40,000 units. ∴ Balance of the planned profit, i.e., Rs. 20,000 − Rs. 18,000 = Rs. 2,000, which is to be earned from the additional sales over 40,000 units. As the Fixed Cost of Rs. 12,000 has already been covered from the sale of first 40,000 units, the contribution received (i.e., Sales − Variable Cost) from the additional sales over 40,000 units is equal to profit. Again, the contribution per unit from the additional sales over 40,000 units = Re. 1 − Re. 0.75 = Re. 0.25, and the total contribution required from the additional sales over 40,000 units = Rs. 2,000 ∴ Additional sales required to earn the Total Contribution = balance of Rs. 2,000 of the planned profit Contribution per unit Rs. 2,000 = = 8,000 units Re. 0.25 ∴ Total number of units to be made and sold to earn a planned profit of Rs. 20,000 = 40,000 + 8,000 = 48,000 units. Problem 27 The ratio of Variable Cost to sales is 70%. The BEP occurs at 60% of the capacity sales. Find the capacity sales when the Fixed Costs are Rs. 90,000. Also compute the profit at 75% of the capacity. [C.A. (Inter)—Adapted] Solution Variable Cost = 70% = 70/100 Sales ∴ If the Sales (S) = 100, then the Variable Cost (V) = 70 Therefore, the Contribution (C) = Sales – Variable Cost = 100 – 70 = 30

i. Here, given Variable Cost to sales =

∴ P/ V Ratio =

C 30 × 100 = × 100 = 30% S 100

Modified Date: Mon, Jul 05, 2010 05:08:16 PM

Output Date: Tue, Jul 06, 2010 12:23:48 PM

Rev II

Project: Management Accounting_Debarshi Bhattacharyya ACE Pro India Pvt. Ltd. File: X:\Pearson\Management Accounting_Debarshi Bhattacharyya\MAIN\M10\LAYOUT_M10\M10_DEBA_ISBN_EN_SE_C10.indd

668

MANAGEMENT ACCOUNTING

Again we know, at BEP, F × 100 [∵ At BEP, C = F] BEPS 90,000 Here, 30 = × 100 [∵ The given Fixed Cost = Rs. 90,000] BEPS 90,000 ∴ BEPS = × 100 = Rs. 3,00,000 30 Again, it is given in the problem that 60% of the Capacity Sales = BEPS ∴ 60% of Capacity Sales = BEPS = Rs. 3,00,000 If 60% of the Capacity Sales = Rs. 3,00,000, then Full Capacity Sales = 3,00,000 × 100 ÷ 60 = Rs. 5,00,000 P/V Ratio =

ii. Sales at 75% of capacity = 75% of Rs. 5,00,000 = Rs. 3,75,000 Again, we know, Sales (S) = Fixed Cost (F) + Variable Cost (V) + Profit (P) ∴P=S–F–V Here, Profit = 3,75,000 – 90,000 – (70% of 3,75,000) [∵ the given Variable Cost = 75% of Sales and Fixed Cost = Rs. 90,000] or Profit = 3,75,000 – 90,000 – 2,62,500 ∴ Profit at 75% of the capacity = Rs. 22,500

Problem 28 Selling Price per unit Trade Discount Direct Material Cost per unit Direct Labour Cost per unit Fixed Overheads Variable Overheads on Direct Labour Cost

Rs. 10.00 5% Rs. 3.00 Rs. 2.00 Rs. 10,000 100%

If sales are 10% and 15% above the BE, then determine the Net Profits.

Solution Rs. Less: Less:

Selling Price per unit Trade Discount (5% of Rs. 10) Net Selling Price per unit Variable Cost per unit: Direct Material Direct Labour Variable Overhead (100% of Rs. 2)

3.00 2.00 2.00 Contribution per unit

Rs. 10.00 0.50 9.50

7.00 2.50

Total Fixed Cost = Rs. 10.000 ∴ BES (in units ) =

Total Fixed Cost 10,000 = = 4,000 units Contribution per unit 2.50

∴ BES (in value) = 4,000 units × Rs. 9.50 = Rs. 38,000

Modified Date: Mon, Jul 05, 2010 05:08:16 PM

Output Date: Tue, Jul 06, 2010 12:23:48 PM

Rev II

Project: Management Accounting_Debarshi Bhattacharyya ACE Pro India Pvt. Ltd. File: X:\Pearson\Management Accounting_Debarshi Bhattacharyya\MAIN\M10\LAYOUT_M10\M10_DEBA_ISBN_EN_SE_C10.indd

MARGINAL COSTING AND COST–VOLUME–PROFIT ANALYSIS

669

When sales are 10% above BE, then: Sales (in units) = 4,000 units + 10% = 4,400 units

Less:

Total Contribution at this Level of Sales (4,400 units × Rs. 2.50) Fixed Cost Profit

Rs. 11,000 10,000 1,000

When sales are 15% above BE, then:

Less:

Total Contribution at this Level of Sales (4,600 units × Rs. 2.50) Fixed Cost Profit

Rs. 11,500 10,000 1,500

Problem 29 Fixed Cost Variable Cost Selling Price Output

Rs. 1,20,000 Rs. 3 per unit Rs. 7 per unit 50,000 units

Determine the profit for each of the following situation: i. With the above data. ii. With a 10% increase in output and sales. iii. With a 10% increase in the Fixed Costs. iv. With a 10% increase in the Variable Costs. v. With a 10% increase in the Selling Price.

Solution i.

Less: Less:

Selling Price at Present Level (50,000 units × Rs. 7) Variable Cost at Present Level (50,000 units × Rs. 3) Contribution at Present Level Fixed Cost Profit at Present Level

Rs. 3,50,000 1,50,000 2,00,000 1,20,000 80,000

ii. New output and sales = 50,000 units + 10% = 55,000 units

Less:

Contribution at this Level of Sales [(Rs. 7 – Rs. 3) × 55,000 units] Fixed Cost Profit at this Level of Output and Sales

Rs. 2,20,000 1,20,000 1,00,000

iii. New Fixed Cost = Rs. 1,20,000 + 10% = Rs. 1,32,000

Less:

Contribution at the Existing Level of Output & Sales [(Rs. 7 – Rs. 3) × 50,000 units] New Fixed Cost Profit

Modified Date: Mon, Jul 05, 2010 05:08:16 PM

Output Date: Tue, Jul 06, 2010 12:23:48 PM

Rs. 2,00,000 1,32,000 68,000

Rev II

Project: Management Accounting_Debarshi Bhattacharyya ACE Pro India Pvt. Ltd. File: X:\Pearson\Management Accounting_Debarshi Bhattacharyya\MAIN\M10\LAYOUT_M10\M10_DEBA_ISBN_EN_SE_C10.indd

670

MANAGEMENT ACCOUNTING

iv.

Less:

Less:

Selling Price per unit New Variable Cost per unit (Rs. 3 + 10% of Rs. 3) New Contribution per unit New Total Contribution at Present Level of Output & Sales (50,000 units × Rs. 3.70) Fixed Cost Profit

Rs. 7.00 3.30 3.70 1,85,000 1,20,000 65,000

New Selling Price per unit (Rs. 7 + 10% of Rs. 7) Variable Cost per unit New Contribution per unit New Total Contribution at Present Level of Output & Sales (50,000 units × Rs. 4.70) Fixed Cost Profit

Rs. 7.70 3.00 4.70 2,35,000 1,20,000 1,15,000

v.

Less:

Less:

Combined effect of these four changes on Profit: New Output and Sales = 50,000 units + 10% = 55,000 units New Total Fixed Cost = Rs. 1,20,000 + 10% = Rs. 1,32,000 New Variable Cost per unit = Rs. 3 + 10% = Rs. 3.30 New Selling Price per unit = Rs. 7 + 10% = Rs. 7.70

Less: Less:

Rs. 4,23,500 1,81,500 2,42,000 1,32,000 1,10,000

Selling Price (55,000 units × Rs. 7.70) Variable Cost (55,000 units × Rs. 3.30) Contribution Fixed Cost Profit

Problem 30 DB Ltd furnished the following information: 2004–05 Rs. 2,00,000 30,000

Sales (Rs. 10/unit) Profit

2005–06 Rs. 2,50,000 50,000

You are required to compute: i. P/V Ratio. ii. BEP. iii. Total Variable Cost for 2004–05 and 2005–06. iv. Sales required to earn a profit of Rs. 60,000. v. Profit/Loss when sales are Rs. 1,00,000. vi. MS when profit is Rs. 80,000. vii. During 2006–07, due to an increase in cost, the Variable Cost is expected to rise to Rs. 7/unit and Fixed Cost to Rs. 55,000. If the Selling Price cannot be increased, what will be the amount of sales to maintain the profit of 2005–06? [B.Com. (Hons), Calcutta University—2007]

Modified Date: Mon, Jul 05, 2010 05:08:16 PM

Output Date: Tue, Jul 06, 2010 12:23:48 PM

Rev II

Project: Management Accounting_Debarshi Bhattacharyya ACE Pro India Pvt. Ltd. File: X:\Pearson\Management Accounting_Debarshi Bhattacharyya\MAIN\M10\LAYOUT_M10\M10_DEBA_ISBN_EN_SE_C10.indd

MARGINAL COSTING AND COST–VOLUME–PROFIT ANALYSIS

671

Solution i. P/V Ratio =

Change in Profit × 100 Change in Sales

Here, P/ V Ratio =

50,000 − 30000 × 100 = 40% 2,50,000 − 20,00,000

F+P × 100 S In the year 2005–06, F + 50,000 P/V Ratio = × 100 2,50,000 F + 50,000 or 40 = 2,500 or F + 50,000 = 1,00,000 ∴ Fixed Cost = Rs. 50,000 Fixed Cost Now, BEPS = × 100 P/V Ratio

ii. P/V Ratio =

∴ BEPS =

50,000 × 100 = Rs. 1,25,000 40

S−V × 100 S In the year 2004–05, 2,00,000 − V 40 = × 100 2,00,000 or 80,000 = 2,00,000 – V or V = 2,00,000 – 80,000 ∴ Total Variable Cost for 2004–05 = Rs. 1,20,000 In the year 2005–06, 2,50,000 − V 40 = × 100 2,50,000 or 1,00,000 = 2,50,000 – V or V = 2,50,000 – 1,00,000 ∴ Total Variable Cost for 2005–06 = Rs. 1,50,000

iii. P/V Ratio =

F+P × 100 S 50,000 + 60,000 Here, 40 = × 100 S 1,10,000 or S = × 100 40 ∴ Required Sales = Rs. 2,75,000

iv. P/V Ratio =

v. P/V Ratio =

F+P × 100 S

Modified Date: Mon, Jul 05, 2010 05:08:16 PM

Output Date: Tue, Jul 06, 2010 12:23:48 PM

Rev II

Project: Management Accounting_Debarshi Bhattacharyya ACE Pro India Pvt. Ltd. File: X:\Pearson\Management Accounting_Debarshi Bhattacharyya\MAIN\M10\LAYOUT_M10\M10_DEBA_ISBN_EN_SE_C10.indd

672

MANAGEMENT ACCOUNTING

50,000 + P × 100 1,00,000 or 40,000 = 50,000 + P ∴ P = (10,000) ∴ Loss = Rs. 10,000 Here, 40 =

vi. MS =

P × 100 P/V Ratio

80,000 × 100 40 ∴ MS = Rs. 2,00,000 Here, MS =

vii. New Contribution per unit = Rs. 10 – Rs. 7 = Rs. 3 C 3 ∴ New P/ V Ratio = × 100 = × 100 = 30% S 10 Desired Profit = Rs. 50,000 F+P Now, P/V Ratio = × 100 S 55,000 + 50,000 Here, 30 = × 100 S 1,05,000 ∴ S= × 100 = Rs. 3,50,000 30 ∴ Required sales to maintain the desired profit = Rs. 3,50,000 Problem 31 A company produces a single product which sells for Rs. 20 per unit. The Variable Cost is Rs. 15 per unit and the fixed overhead for the year is Rs. 6,30,000. Required: i. Calculate the sales value needed to earn a profit of 10% on sales. ii. Calculate the sales price per unit to bring the BEP down to 1,20,000 units. iii. Calculate the MS sales, if the profit is Rs. 60,000. [C.A. (P.C.C)—November 2007] Solution S−V × 100 S 20 − 15 Here, P/V Ratio = × 100 = 25% 20 Let the required sales to earn a profit of 10% on sales be x. ∴ Profit = 10% of x = 0.x F+P Again, P/V Ratio = × 100 S

i. P/V Ratio =

Modified Date: Mon, Jul 05, 2010 05:08:16 PM

Output Date: Tue, Jul 06, 2010 12:23:48 PM

Rev II

Project: Management Accounting_Debarshi Bhattacharyya ACE Pro India Pvt. Ltd. File: X:\Pearson\Management Accounting_Debarshi Bhattacharyya\MAIN\M10\LAYOUT_M10\M10_DEBA_ISBN_EN_SE_C10.indd

MARGINAL COSTING AND COST–VOLUME–PROFIT ANALYSIS

673

6,30,000 + 0.x × 100 x or 0.25x – 0.x = 6,30,000 or 0.15x = 6,30,000 ∴ x = 630000 ÷ 0.15 = 42,00,000 ∴ Required sales to earn a profit of 10% on sales = Rs. 42,00,000 Here, 25 =

ii. BEPS (in units ) = Here, 1,20,000 =

Fixed Cost Contribution per unit

6,30,000 Contribution per unit

∴ Contribution per unit =

6,30,000 = Rs. 5.25 1,20,000

Again, at BEP, C = S − V ∴ 5.25 = S – 15 or S = 15 + 5.25 ∴ S = 20.25 ∴ Selling Price per unit to bring the BEP down to 1,20,000 units = Rs. 20.25 iii. MS =

Profit × 100 P/V Ratio

60,000 × 100 25 ∴ MS = Rs. 2,40,000 ∴ MS Sales = Rs. 2,40,000 Here, MS =

Problem 32 Material per unit Labour per unit Variable Overhead per unit Selling Price per unit Total Fixed Overhead

Rs. 50 Rs. 80 75% of Labour Cost Rs. 250 Rs. 2,40,000

Find out the following: i. BEPS in value and in volume. ii. What would be the value and volume of sales, if products are sold to make a profit of Rs. 1,20,000? iii. If the Selling Price per unit is reduced by Rs. 20, what would be the BEPS in value and in volume? Solution i. Rs. Less:

Existing Selling Price per unit Existing Variable Costs per unit: Material

Rs. 250

50 (Continued)

Modified Date: Mon, Jul 05, 2010 05:08:16 PM

Output Date: Tue, Jul 06, 2010 12:23:48 PM

Rev II

Project: Management Accounting_Debarshi Bhattacharyya ACE Pro India Pvt. Ltd. File: X:\Pearson\Management Accounting_Debarshi Bhattacharyya\MAIN\M10\LAYOUT_M10\M10_DEBA_ISBN_EN_SE_C10.indd

674

MANAGEMENT ACCOUNTING

Labour Overhead (75% of Rs. 80) Contribution per unit

Rs. 80 60

Rs. 190 60

C 60 × 100 = × 100 S 250 ∴ Existing P/V Ratio = 24% Again, we know, at BEP, F P/V Ratio = × 100 BEPS 2,40,000 or 24 = × 100 BEPS 2,40,000 or BEPS = × 100 24 ∴ BEPS (in value) = Rs. 10,00,000 Rs. 10,00,000 ∴ BEPS (in volume ) = = 4,000 units Rs. 250 ∴ Existing P/V Ratio =

ii. Here, if Profit = Rs. 1,20,000, then Sales = ? F+P P/V Ratio = × 100 S 2,40,000 + 1,20,000 Here, 24 = × 100 S 3,60,000 or S = × 100 24 ∴ Sales (in value) = Rs. 15,00,000 Rs. 15,00,000 ∴ Sales (in volume ) = = 6,000 units Rs. 250 iii. As the Selling Price per unit is decreased by Rs. 20, the P/V Ratio will be changed. New Selling Price per unit (Rs. 250 – Rs. 20) = Rs. 230 Less: Variable Cost per unit = Rs. 190 [same as before] New contribution per unit = Rs. 40 Fixed Cost ∴ New BEPS (in volume ) = Contribution per unit Rs. 2,40,000 = 6,000 units Rs. 40 ∴ New BEPS (in value) = 6,000 units × Rs. 230 = Rs. 13,80,000 ∴ New BEPS (in volume ) =

Problem 33 A business produces 200 units of a product by making the following expenditure: (i) Materials—Rs. 30,000; (ii) Labour—Rs. 20,000; (iii) Factory Overheads—Rs. 4,000; (iv) Administrative Overheads—Rs. 5,754; and (v) Selling and Distribution Overheads—Rs. 1,500. The products are sold at a price of Rs. 400 per unit.

Modified Date: Mon, Jul 05, 2010 05:08:16 PM

Output Date: Tue, Jul 06, 2010 12:23:48 PM

Rev II

Project: Management Accounting_Debarshi Bhattacharyya ACE Pro India Pvt. Ltd. File: X:\Pearson\Management Accounting_Debarshi Bhattacharyya\MAIN\M10\LAYOUT_M10\M10_DEBA_ISBN_EN_SE_C10.indd

675

MARGINAL COSTING AND COST–VOLUME–PROFIT ANALYSIS

The above expenditures are classified into fixed and variable types as follows: Expenditure i. ii. iii. iv. v.

Fixed Nil 50% 25% 100% 60%

Materials Labour Factory Overheads Administrative Overheads Selling & Distribution Overheads

Variable 100% 50% 75% Nil 40%

From the above information, determine the following: i. Total Variable Costs and Fixed Costs. ii. Contribution. iii. P/V Ratio. iv. BEPS in units and in value. [B.Com. (Hons), Calcutta University—1997] Solution i. Calculation of Total Variable Costs and Total Fixed Costs Costs Materials Labour Factory Overheads Administrative Overheads Selling & Distribution Overheads Total

Variable Costs (100% of Rs. 30,000) (50% of Rs. 20,000) (75% of Rs. 4,000) (40% of Rs. 1,500)

Fixed Costs Rs. 30,000 10,000 3,000 Nil 600 43,600

(50% of Rs. 20,000) (25% of Rs. 4,000) (100% of Rs. 5,754) (60% of Rs. 1,500)

Rs. Nil 10,000 1,000 5,754 900 17,654

ii.

Less:

Total Selling Price at 200 units of output (200 units × Rs. 400) Total Variable Cost at 200 units of output [as ascertained in (i) above] Total Contribution at 200 units of output

Rs. 80,000 43,600 36,400

Rs. 36,400 Total Contribution = = Rs. 182 ∴ Contribution per unit = 200 Sales in units C iii. P/V Ratio = × 100 S 36,400 Here, P/V Ratio = × 100 80,000 ∴ P/V Ratio = 45.50%. iv. We know that Total Fixed Cost BEPS (in value) = × 100 P/V Ratio 17,654 Here, BEPS (in value) = × 100. [ ∵ as ascertained in (i) above, Fixed Cost = Rs. 17,654] 45.50 ∴BES (in value) = Rs. 38,800. Total Fixed Cost Again, BEPS (in units) = Contribution per unit 17,654 ⎡ Contribution ⎤ Here, BEPS (in units) = = Rs. 182⎥ ⎢∵ 182 unit ⎣ ⎦ ∴ BES (in units) = 97 units.

Modified Date: Mon, Jul 05, 2010 05:08:16 PM

Output Date: Tue, Jul 06, 2010 12:23:48 PM

Rev II

Project: Management Accounting_Debarshi Bhattacharyya ACE Pro India Pvt. Ltd. File: X:\Pearson\Management Accounting_Debarshi Bhattacharyya\MAIN\M10\LAYOUT_M10\M10_DEBA_ISBN_EN_SE_C10.indd

676

MANAGEMENT ACCOUNTING

Problem 34 A company sells its products at Rs. 15 per unit. In a period, if it produces and sells 8,000 units, it incurs a loss of Rs. 5 per unit. If the volume is raised to 20,000 units, it earns a profit of Rs. 4 per unit. Calculate BEP, both in terms of rupees as well as units. [C.A. (Inter)—November 1996] Solution Sales Volume 20,000 units 8,000 units

We know that P/V Ratio =

Sales Value (Rs.) 20,000 units × Rs. 15 = Rs. 3,00,000 8,000 units × Rs. 15 = Rs. 1,20,000 Change in Sales Value = Rs. 1,80,000

Profit/ Loss (Rs.) 20,000 units × Rs. 4 = Rs. 80,000 8,000 units × Rs. 5 = Rs. 40,000 Change in Profit = Rs. 1,20,000

Change in Profit 1,20,000 × 100 = × 100 = 66.6666% Change in Sales 1,80,000

Now, when 20,000 units are sold out, then F+P P/V Ratio = × 100 S [ ∵ At 20,000 units of sale, the Total Profit = Rs. 80,000 F + 80,000 or 66.6666 = × 100 and Sales Value = Rs. 3,00,000] 3,00,000 30,000 × 66.6666 100 ∴ F = 2,00,000 – 80,000 = Rs. 1,20,000 Again we know that at BEP, F P/V Ratio = × 100 BEPS 1,20,000 ∴ 66.6666 = × 100 BEPS ∴ BEPS (in rupees) = Rs. 1,80,000 Rs.1,80,000 ∴ BEPS (in units) = = 12,000 units Rs. 15 or F + 80,000 =

Problem 35 Fill in the blanks for each of the following independent situations: Situation Selling Price per unit Variable Cost as a percentage of Selling Price No. of units sold Contribution Fixed Cost Profit / Loss

P ? (i) 60 10,000 Rs. 20,000 Rs. 12,000 ? (ii)

Q Rs. 50 ? (iii) 4,000 Rs. 80,000 ? (iv) Rs. 20,000

R Rs. 20 75 ? (v) ? (vi) Rs. 1,20,000 Rs. 30,000

S ? (vii) 75 6,000 Rs. 25,000 Rs. 10,000 ? (viii)

T Rs. 30 ? (ix) 5,000 Rs. 50,000 ? (x) Rs. 15,000

[B. Com. (Hons), Calcutta University—2009—Adapted] Solution i. Here, the Variable Cost as a percentage of Selling Price = 60% If the Selling Price = 100, then the Variable Cost = 60

Modified Date: Mon, Jul 05, 2010 05:08:16 PM

Output Date: Tue, Jul 06, 2010 12:23:48 PM

Rev II

Project: Management Accounting_Debarshi Bhattacharyya ACE Pro India Pvt. Ltd. File: X:\Pearson\Management Accounting_Debarshi Bhattacharyya\MAIN\M10\LAYOUT_M10\M10_DEBA_ISBN_EN_SE_C10.indd

MARGINAL COSTING AND COST–VOLUME–PROFIT ANALYSIS

677

∴ Contribution = S − V = 100 − 60 = 40 ∴ When Contribution = 40, then Selling Price = 100 ∴ When the Total Contribution = Rs. 20,000 at a sales volume of 10,000 units, then— 100 ∴ Total Selling Price = × 20,000 = Rs. 50,000 40 Rs. 50,000 ∴ Selling Price per unit = = Rs. 5 10,000 units ii. Contribution = Fixed Cost + Profit Here, 20,000 = 12,000 + Profit ∴ Profit = Rs. 8,000 iii. Total Contribution = Rs. 80,000 at a sales volume of 4,000 units ∴ Contribution per unit (C) = 8,000 ÷ 4,000 = Rs. 20 Again, C = S − V ∴ 20 = 50 − V ∴ Variable Cost per unit (V) = 50 − 20 = Rs. 30 V 30 ∴ Variable Cost as a percentage of Selling Price = × 100 = × 100 = 60% S 50 iv. Contribution = Fixed Cost + Profit Here, 80,000 = Fixed Cost + 20,000 ∴ Fixed Cost = 80,000 − 20,000 = Rs. 60,000 v. Contribution per unit = S − V = Rs. 20 − (75% of Rs. 20) = Rs. 20 − Rs. 15 = Rs. 5 F+P 1,20,000 + 30,000 ∴ Sales in units = = = 30,000 units Contribution per unit 5 vi. Total Contribution = 30,000 units × Rs. 5 = Rs. 1,50,000 or = F + P = 1,20,000 + 30,000 = Rs. 1,50,000 vii. Here, the Variable Cost as a percentage of Selling Price = 75% If the Selling Price = 100, then the Variable Cost = 75 ∴ Contribution = S − V = 100 − 75 = 25 ∴ When Contribution = 25, then Selling Price = 100 ∴ When Total Contribution = Rs. 25,000 at a sales volume of 6,000 units, then— 100 Total Selling Price = × 25,000 = Rs. 1,00,000 25 Rs. 1,00,000 ∴ Selling Price per unit = = Rs. 16.67 6,000 units viii. Contribution = Fixed Cost + Profit Here, 25,000 = 10,000 + Profit ∴ Profit = Rs. 15,000 ix. Total Contribution = Rs. 50,000 at a sales volume of 5,000 units ∴ Contribution per unit (C) = 50,000 ÷ 5,000 = Rs. 10 Again, C = S − V Here, 10 = 30 − V ∴ Variable Cost per unit (V) = 30 − 10 = Rs. 20

Modified Date: Mon, Jul 05, 2010 05:08:16 PM

Output Date: Tue, Jul 06, 2010 12:23:48 PM

Rev II

Project: Management Accounting_Debarshi Bhattacharyya ACE Pro India Pvt. Ltd. File: X:\Pearson\Management Accounting_Debarshi Bhattacharyya\MAIN\M10\LAYOUT_M10\M10_DEBA_ISBN_EN_SE_C10.indd

678

MANAGEMENT ACCOUNTING

∴ Variable cost as a percentage of Selling Price =

V 20 × 100 = × 100 = 66.67% S 30

x. Contribution = Fixed Cost + Profit Here, 50,000 = Fixed Cost + 15,000 ∴ Fixed Cost = 50,000 − 15,000 = Rs. 35,000 Problem 36 i. When the volume is 3,000 units, the average cost is Rs. 4 per unit. When the volume is 4,000 units, the average is Rs. 3.50. The BEP is 5,000 units. Find the P/V Ratio. ii. ABC Ltd has a Fixed Cost of Rs. 2,00,000. It has two products that it can sell, A and B. The company sells these products at a rate of 2 units of A to 1 unit of B. The unit contribution is Re. 1 per unit for A and Rs. 2 per unit for B. How many units of A and B would be sold at the BEP? iii. If MS is 40% of sales, find the Fixed Cost when profit is Rs. 20,000. [I.C.W.A. (Inter)—Adapted] Solution i. Rs. Total Cost at 4,000 units = 4,000 units × Rs. 3.50 Total Cost at 3,000 units = 3,000 units × Rs. 4.00 Increase in output = 1,000 units

∴ Variable Cost per unit =

14,000 12,000 Increase in Cost = Rs. 2,000

Increase in Cost Rs. 2,000 = = Rs. 2 Increase in output 1,000

∴ At 4,000 units of output: Variable Cost = 4,000 units × Rs. 2 Fixed Cost (bal. fig.) Total Cost

Now, Contribution per unit =

ii.

Rs. 8,000 6,000 14,000

Fixed Cost Rs. 6,000 = = Rs. 1.20 BEPS in units 5,000 units

Again, S – V = C Here, S – 2 = 1.20 or S = 1.20 + 2 = Rs. 3.20 ∴ Selling Price per unit = Rs. 3.20 C 1.20 ∴ P/V Ratio = × 100 = × 100 = 37.50% S 3.20

Contribution from 2 units of A = 2 × Re. 1 Contribution from 1 unit of B = 1 × Rs. 2 Contribution from 3 units

∴ Total BEPS (in units ) =

Modified Date: Mon, Jul 05, 2010 05:08:16 PM

Rs. 2 2 4

Total Fixed Cost 2,00,000 = = 2,00,000 × ( 3 ÷ 4 ) = 1,50,000 units Contribution per unit 4÷3

Output Date: Tue, Jul 06, 2010 12:23:48 PM

Rev II

Project: Management Accounting_Debarshi Bhattacharyya ACE Pro India Pvt. Ltd. File: X:\Pearson\Management Accounting_Debarshi Bhattacharyya\MAIN\M10\LAYOUT_M10\M10_DEBA_ISBN_EN_SE_C10.indd

MARGINAL COSTING AND COST–VOLUME–PROFIT ANALYSIS

679

∴ BE units for A = ( 2 ÷ 3) × 1,50,000 = 1,00,000 units ∴ BE units for B = (1 ÷ 3) × 1,50,000 = 50,000 units iii. Here, MS = 40% of Sales ∴ BEPS = 100 – 40 = 60% of Sales Profit P/V ratio 20,000 ............................................................................... (i) Here, 40% of Sales = P/V ratio Again, we know that =

Again, we know that BEPS = Here, 60% of Sales =

Fixed Cost P/V Ratio

F ............................................................................ . (ii) P/V Ratio

Now, dividing (ii) by (i), we get: P/V Ratio 60% of Sales F = × 40% of Sales P/V Ratio 20,000 60 F or = 40 20,000 60 × 20,000 ∴ Fixed Cost = = Rs. 30,000 40 Problem 37 Raj Ltd manufactures three products, X, Y and Z. The unit Selling Prices of these products are Rs. 100, Rs. 160 and Rs. 75 respectively. The corresponding unit Variable Costs are Rs. 50, Rs. 80 and Rs. 30 respectively. The proportions (quantity-wise) in which these products are manufactured and sold are 20%, 30% and 50% respectively. The total Fixed Costs are Rs. 14,80,000. Calculate the overall BE quantity and the product-wise break-up of such quantity. [C.A. (Inter)—May 1999] Solution Product Less:

Selling Price per unit Variable Cost per unit Contribution per unit Proportion of the Products Manufactured

X Rs. 100 50 50 20%

Y Rs. 160 80 80 30%

Z Rs. 75 30 45 50%

∴ Overall Contribution per unit = (20% of Rs. 50) + (30% of Rs. 80) + (50% of Rs. 45) = Rs. 10 + Rs. 24 + Rs. 22.50 = Rs. 56.50 Overall Fixed Cost = Rs. 14,80,000 ∴ Overall BE quantity = Rs. 14,80,000 ÷ Rs. 56.50 = 26,195 units

Modified Date: Mon, Jul 05, 2010 05:08:16 PM

Output Date: Tue, Jul 06, 2010 12:23:48 PM

Rev II

Project: Management Accounting_Debarshi Bhattacharyya ACE Pro India Pvt. Ltd. File: X:\Pearson\Management Accounting_Debarshi Bhattacharyya\MAIN\M10\LAYOUT_M10\M10_DEBA_ISBN_EN_SE_C10.indd

680

MANAGEMENT ACCOUNTING

∴ Product-wise break-up of BE quantity = 26,195 units in 20%, 30% & 50% ∴ BE quantity for product X = 20% of 26,195 = 5,239 units BE quantity for product Y = 30% of 26,195 = 7,858 units BE quantity for product Z = 50% of 26,195 = 13,098 units Problem 38 M & Co. and N & Co. furnished the following details: M & Co. 10,000 units Rs. 40,000 Rs. 40,000

Annual Production Fixed Costs Profit at the Above Level of Production

N & Co. 10,000 units Rs. 36,000 Rs. 24,000

Find out: i. BEP units of each company. ii. Level of sales (in units) at which both the companies earn an equal profit or incur an equal loss.

Solution i. Total Contribution at the Present Level of Production (F + P) No. of Units Presently Produced & Sold ∴ Contribution per unit ⎛ ⎞ F ∴BEPS in units ⎜ ⎝ Contribution per unit ⎟⎠

M & Co. 40,000 + 40,000 = Rs. 80,000

N & Co. 36,000 + 24,000 = Rs. 60,000

10,000 units Rs. 80,000 ÷ 10,000 = Rs. 8

10,000 units Rs. 60,000 ÷ 10,000 = Rs. 6

Rs. 40,000 ÷ Rs. 8 = 5,000 units Rs. 36,000 ÷ Rs. 6 = 6,000 units

ii. Level of sales (in units) at which both the companies earn an equal profit or incur an equal loss Difference in the Fixed Cost = Difference in the Contribution per unit Here, the level of sales (in units) at which both the companies can earn an equal profit or incur an equal loss Rs. 40,000 − Rs.36,000 4,000 = = = 2,000 units Rs. 8 − Rs. 6 2 As the BEP of both the companies is higher than the above 2,000 units, it is concluded that at this 2,000 units of sales, both the companies incur an equal loss. Problem 39 Zebra Ltd has furnished the following data for 2 years: Sales P/V Ratio MS (Sales as a Percentage of Total Sales)

Modified Date: Mon, Jul 05, 2010 05:08:16 PM

Output Date: Tue, Jul 06, 2010 12:23:48 PM

2007–08 Rs. 8,00,000 50% 40%

2008–09 ? 37.5% 21.875%

Rev II

Project: Management Accounting_Debarshi Bhattacharyya ACE Pro India Pvt. Ltd. File: X:\Pearson\Management Accounting_Debarshi Bhattacharyya\MAIN\M10\LAYOUT_M10\M10_DEBA_ISBN_EN_SE_C10.indd

MARGINAL COSTING AND COST–VOLUME–PROFIT ANALYSIS

681

There has been substantial savings in the Fixed Cost in the year 2008–09 due to the restructuring process. The company could maintain its sales-quantity level of 2007–08 in the year 2008–09 by reducing the Selling Price. You are required to calculate the following: i. Sales for 2008–09 in rupees. ii. BES for 2008–09 in rupees. iii. Fixed Cost for 2008–09. Solution C S−V × 100 = × 100 S S Now, in 2007–08, 8,00,000 − V 50 = × 100 8,00,000 50 × 8,00,000 or = 8,00,000 − V 100 or 4,00,000 = 8,00,000 − V or V = 8,00,000 − 4,00,000 = 4,00,000 ∴ Total Variable Cost in 2007–08 = Rs. 4,00,000

i. P/V Ratio =

Here, the sales quantity in both the given years is the same. Nothing is specifically stated in the problem regarding the change in the Variable Cost in 2 years. Hence, it may be presumed that the Variable Cost per unit in both the years remains unchanged. As the sales quantity and Variable Cost per unit for both the years are the same, therefore, the total Variable Cost in the both the years is the same. ∴ Total Variable Cost in 2008–09 = Rs. 4,00,000 Now, in 2008–09, S−V × 100 S S − 4,00,000 or 37.5 = × 100 S 37.5 × S or = S − 4,00,000 100 or 0.375S = S − 4,00,000 or S − 0.375S = 4,00,000 or 0.625S = 4,00,000. ∴ S = 4,00,000 ÷ 0.625 = 6,40,000 ∴ Sales for 2008–09 = Rs. 6,40,000 P/V Ratio =

ii. In 2008–09, MS = 21.875% (on sales) ∴ BEPS = Total sales − MS = 100 − 21.875 = 78.125% (of sales) ∴ BEPS = 78.125% of Rs. 6,40,000 = Rs. 5,00,000 Fixed Cost × 100 BEPS F Now, in 2008 −09, 37.5 = × 100 5,00,000

iii. At BEP, P/V Ratio =

Modified Date: Mon, Jul 05, 2010 05:08:16 PM

Output Date: Tue, Jul 06, 2010 12:23:48 PM

Rev II

Project: Management Accounting_Debarshi Bhattacharyya ACE Pro India Pvt. Ltd. File: X:\Pearson\Management Accounting_Debarshi Bhattacharyya\MAIN\M10\LAYOUT_M10\M10_DEBA_ISBN_EN_SE_C10.indd

682

MANAGEMENT ACCOUNTING

37.5 × 5,00,000 100 or F = 1,87,500 ∴ Fixed Cost for 2008–09 = Rs. 1,87,500. or F =

Problem 40 A pharmaceutical company produces formulations having a shelf life of 1 year. The company has an opening stock of 30,000 boxes on 1 January 2005 and is expected to produce 1,30,000 boxes as was in the just-ended year of 2004. The expected sale would be 1,50,000 boxes. The costing department has worked out an escalation in the cost by 25% on the Variable Cost and 10% on the Fixed Cost. While the Fixed Cost for the year 2004 is Rs. 40 per unit, the new price announced for 2005 is Rs. 100 per box. The Variable Cost on opening stock is Rs. 40 per box. You are required to compute the BE volume for the year 2005. [C.A. (PE-II)—November 2005] Solution

Less:

Selling Price per unit Variable Cost per unit (Rs. 40 + 25% of Rs. 40) Contribution per unit Total Fixed Cost

Opening Stock (i.e., from Production of 2004)

Production for 2005

Rs. 100 40 60 1,30,000 × Rs. 40 = Rs. 52,00,000

Rs. 100 50 50 Rs. 52,00,000 + 10% of Rs. 52,00,000 = Rs. 57,20,000

Now, BEPS for 2005 (in volume) = Opening Stock units + = 30,000 units +

Total Fixed Cost in 2005 − Contribution from Opening Stock Contribution per unit in 2005

Rs. 57,20,000 − 30,000 × Rs. 60 Rs. 50

= 30,000 units + 78,400 units = 1,08,400 units ∴ BEPS for 2005 (in value) = Value of Opening Stock of 30,000 units + Value of 78,400 units produced in 2005 = (30,000 × Rs. 100) + (78,400 × Rs. 100) = Rs. 1,08,40,000 Problem 41 K1 and K2 are two similar plants under the same management who want them to be merged for better operation. The details of K1 and K2 are as follows: K1 90% Rs. 6,30,000 Rs. 3,60,000 Rs. 1,20,000

Capacity Utilization Sales Variable Cost Fixed Cost

Modified Date: Mon, Jul 05, 2010 05:08:16 PM

Output Date: Tue, Jul 06, 2010 12:23:48 PM

K2 60% Rs. 3,60,000 Rs. 2,40,000 Rs. 80,000

Rev II

Project: Management Accounting_Debarshi Bhattacharyya ACE Pro India Pvt. Ltd. File: X:\Pearson\Management Accounting_Debarshi Bhattacharyya\MAIN\M10\LAYOUT_M10\M10_DEBA_ISBN_EN_SE_C10.indd

MARGINAL COSTING AND COST–VOLUME–PROFIT ANALYSIS

683

Find out: i. BEPS of the merged plant and capacity utilization for achieving BEPS. ii. Profitability of the merged plant at 80% capacity. iii. Sales of the merged plant to earn a profit of Rs. 1,00,000. Solution As plants K1 and K2 are presently operating at different capacities, let us firstly convert them into a 100% capacity and thereafter they can be merged. K1 Rs. 6,30,000 × 100 ÷ 90 = 7,00,000 3,60,000 × 100 ÷ 90 = 4,00,000 3,00,000 1,20,000

Sales at 100% capacity (a) Variable Cost at 100% capacity (b) Contribution at 100% capacity (a − b) Fixed Cost at 100% capacity (Same as at 90% or 60% capacity)

∴ P/V Ratio of the Merged Plant = =

K2 Rs. 3,60,000 × 100 ÷ 60 = 6,00,000 2,40,000 × 100 ÷ 60 = 4,00,000 2,00,000 80,000

Merged Plant Rs. 13,00,000 8,00,000 5,00,000 2,00,000

Contribution of Merged Plant × 100 Sales of Merged Plant

5,00,000 × 100 = 38.4615% 13,00,000

i. We know that at BEP, Fixed Cost P/V Ratio = × 100 BEPS ∴ In case of Merged Plant, 2,00,000 38.4615 = × 100 BEPS 2,00,000 or BEPS = × 100 38.4615 ∴ BEPS of the Merged Plant = Rs. 5,20,000 ∴ Capacity Utilization of the Merged Plant at BEPS =

100 100 × BEPS = × 5,20,000 = 40% Sales at 100% capacity 13,00,000

ii. Sales of the Merged Plant at 80% capacity = 80% of sales (at 100% capacity) = 80% of Rs. 13,00,000 = Rs. 10,40,000 F+P × 100 S 2,00,000 + P Here, 38.4615 = × 100 10,40,000 38.4615 × 10,40,000 or 20,000 + P = 100 Now, P/V Ratio =

Modified Date: Mon, Jul 05, 2010 05:08:16 PM

Output Date: Tue, Jul 06, 2010 12:23:48 PM

Rev II

Project: Management Accounting_Debarshi Bhattacharyya ACE Pro India Pvt. Ltd. File: X:\Pearson\Management Accounting_Debarshi Bhattacharyya\MAIN\M10\LAYOUT_M10\M10_DEBA_ISBN_EN_SE_C10.indd

684

MANAGEMENT ACCOUNTING

or 2,00,000 + P = 4,00,000 or P = 4,00,000 – 2,00,000 = Rs. 2,00,000 ∴ Profit at 80% capacity of the Merged Plant = Rs. 2,00,000 iii. In this case, if Profit = Rs. 1,00,000, then Sales = ? F+P P/V Ratio = × 100 S 2,00,000 + 1,00,000 Here, 38.4615 = × 100 S 3,00,000 or S = × 100 = Rs. 7,80,000 38.4615 ∴ Sales of the Merged Plant to earn a Profit of Rs. 1,00,000 = Rs. 7,80,000. Problem 42 Company H produces and sells a product. The machines are rented from the manufacturer. The following expenses and revenue relationship exist at present: Rs. Fixed Monthly Expenses: Machine Rent Room Rent Service Charge for Machine Other Fixed Costs

1,535 288 727 50 2,600 Rs. per unit 50 40 10

Other Data: Selling Price Variable Cost Contribution

Consider each question independently and then give your answer: i. What is the monthly BEP (in units and in rupees)? ii. If 20,000 units are sold during the year, what will be the company’s net income? iii. If the room rental is doubled, what will be the monthly BEP (in units and in rupees)? iv. In addition to the fixed machine rent, if the company pays the machine manufacturer an additional 50 paise per unit of goods sold, what will be the monthly BEP (in units and in rupees)? v. In addition to the fixed room rent, if the company pays the landlord an additional 10 paise per unit of goods sold in excess of BEP as per (i) above, what will be the annual income of the company at a sales level of 20,000 units for the year? [B.Com. (Hons), Kalyani University—2004] Solution C 10 × 100 = × 100 = 20% S 50 Total Fixed Cost for the month = Rs. 2,600

i. P/V Ratio =

∴ BEP (in rupees ) =

F 2,600 × 100 = × 100 = Rs. 13,000 P/V Ratio 20

Modified Date: Mon, Jul 05, 2010 05:08:16 PM

Output Date: Tue, Jul 06, 2010 12:23:48 PM

Rev II

Project: Management Accounting_Debarshi Bhattacharyya ACE Pro India Pvt. Ltd. File: X:\Pearson\Management Accounting_Debarshi Bhattacharyya\MAIN\M10\LAYOUT_M10\M10_DEBA_ISBN_EN_SE_C10.indd

MARGINAL COSTING AND COST–VOLUME–PROFIT ANALYSIS

∴ BEP (in units ) =

685

F 2,600 = = 260 units Contribution per unit 10

ii. Fixed Cost for the year = Rs. 2,600 × 12 = Rs. 31,200 Sales Value for 20,000 units = 20,000 units × Rs. 50 = Rs. 10,00,000 F+P We know that P/V Ratio = × 100 S 31,200 + P Here, 20 = × 100 10,00,000 20 × 10,00,000 or 31,200 + P = 100 ∴ P = 2,00,000 – 31,200 = Rs. 1,68,800 ∴ Company’s Net Income for the year = Rs. 1,68,800 Alternatively:

Less:

Rs. 2,00,000 31,200 1,68,800

Contribution for 20,000 Units of Sales = 20,000 × Rs. 10 Fixed Cost for the Year (Rs. 2,600 × 12) Net Income for the Year

iii. New Fixed Cost for the month:

Add:

Rs. 2,600 288 2,888

Existing Fixed Cost for the Month Increase in the Room Rent New Monthly Fixed Cost

∴ BEP for the month (in rupees ) =

F

(P / V Ratio)

× 100 =

2,888 × 100 = Rs. 14,440 20

Rs. 14,440 = 289 units ( approx ) Rs. 50 iv. New Variable Cost and New Contribution per unit: ∴ BEP for the month (in units ) =

Add:

Existing Variable Cost per unit Additional Payment to Machine Manufacturer New Variable Cost per unit Selling Price per unit New Contribution per unit

Rs. 40.00 0.50 40.50 50.00 9.50

C 9.50 × 100 = × 100 = 19% S 50 F 2,600 ∴ Monthly BEP (in rupees ) = × 100 = × 100 = Rs. 13,684 P/V Ratio 19 F 2,600 ∴ Monthly BEP (in units ) = = = 274 units ( approx.) Contribution per unit 9.50 ∴ New P/ V Ratio =

v. As per (i) above, the company achieves BEP at 260 units of sales. ∴ BEPS for the year = 260 units × 12 = 3,120 units Here, the actual sales for the year = 20,000 units

Modified Date: Mon, Jul 05, 2010 05:08:16 PM

Output Date: Tue, Jul 06, 2010 12:23:48 PM

Rev II

Project: Management Accounting_Debarshi Bhattacharyya ACE Pro India Pvt. Ltd. File: X:\Pearson\Management Accounting_Debarshi Bhattacharyya\MAIN\M10\LAYOUT_M10\M10_DEBA_ISBN_EN_SE_C10.indd

686

MANAGEMENT ACCOUNTING

∴ MS = Actual sales – BEPS = 20,000 – 3,120 = 16,880 units At this MS level: Existing Variable Cost per unit Additional Payment of Rent in Excess of BEP New Variable Cost per unit Selling Price per unit Contribution per unit at MS

Add:

Rs. 40.00 0.10 40.10 50.00 9.90

We know that Contribution at MS = Profit ∴ Annual income (i.e., profit) at a sales level of 20,000 units = 20,000 units × Rs. 9.90 = Rs. 1,98,000 Problem 43 A company manufactures a product, currently utilizing 80% capacity with a turnover of Rs. 8,00,000 at Rs. 25 per unit. The cost data are as follows: Material Cost is Rs. 7.50 per unit, Labour Cost is Rs. 6.25 per unit, Semi-Variable Cost (including the Variable Cost of Rs. 3.75 per unit) is Rs. 1,80,000. The Fixed Cost is Rs. 90,000 up to 80% level of the output; and beyond this, an additional amount of Rs. 20,000 will be incurred. Calculate: i. ii. iii. iv.

Activity level at BEP. Number of units to be sold to earn a net income of 8% of sales. Activity level needed to earn a profit of Rs. 95,000. What should be the Selling Price per unit, if BEP is to be brought down to 40% activity level? [C.A. (Inter)—November 2000]

Solution Material Cost Labour Cost Variable Portion within Semi-Variable Cost Variable Cost per unit Selling Price per unit Contribution per unit

Rs. 7.50 6.25 3.75 17.50 25.00 7.50

C 7.50 × 100 = × 100 = 30% S 25 Number of units produced and sold Rs. 8,00,000 = = 32,000 units currently utilizing 80% capacity Rs. 25

∴ P/ V Ratio =

∴ Fixed portion included in the Semi-Variable Cost:

Less:

Total Semi-Variable Cost at 32,000 units of output Variable Portion of Semi-Variable Cost at 32,000 Units of Output (32,000 units × Rs. 3.75) Fixed Portion of Semi-Variable Cost at 32,000 Units of Output (i.e., at 80% Capacity)

Rs. 1,80,000 1,20,000 60,000 (Continued)

Modified Date: Mon, Jul 05, 2010 05:08:16 PM

Output Date: Tue, Jul 06, 2010 12:23:48 PM

Rev II

Project: Management Accounting_Debarshi Bhattacharyya ACE Pro India Pvt. Ltd. File: X:\Pearson\Management Accounting_Debarshi Bhattacharyya\MAIN\M10\LAYOUT_M10\M10_DEBA_ISBN_EN_SE_C10.indd

MARGINAL COSTING AND COST–VOLUME–PROFIT ANALYSIS

687

Rs. ∴ Total Fixed Cost up to 80% capacity: Fixed Cost Fixed Portion of Semi-Variable Cost Total Fixed Cost up to 80% capacity

Add:

90,000 60,000 1,50,000

Rs.1,50,000 = 20,000 units Rs. 7.50 80 ∴ Activity level at BEP = × 20,000 = 50% 32,000

i. BEPS (in units ) =

ii. Let the Sales be Rs. x. ∴ Profit = 8% of Rs. x = 0.08x F+P We know that P/V ratio = × 100 S Here, 30 = or or or

1,50,000 + 0.08x × 100 x

30x = 1,50,000 + 0.08x 100 0.3x – 0.08x = 1,50,000 0.22x = 1,50,000

∴x =

1,50,000 = 6,81,818 0.22

∴ Required Sales Value = Rs. 6,81,818 ∴ Required number of units to be sold to earn a net income of 8% of sales = Rs. 6,81,818 ÷ Rs. 25 = 27,273 units (approx.) iii. As the profit of Rs. 95,000 is higher than the existing profit, this amount of profit will be achieved by operating beyond the mentioned 80% capacity. ∴ New Fixed Cost = Existing Fixed Cost + Rs. 20,000 = Rs. 1,50,000 + Rs. 20,000 = Rs. 1,70,000 F+P 1,70,000 + 95,000 ∴ Required Sales = = = 35,333 units Contribution per unit 7.50 ∴ Activity level needed to earn a profit of Rs. 95,000 =

80 × 35,333 = 88.33% 32,000

iv. Output at 40% activity level = 32,000 × 40 ÷ 80 = 16,000 units ∴ Variable Cost at 40% Activity Level (Rs. 17.50 × 16,000 units) ∴ Fixed Cost at 40% Activity Level Total Cost at 40% Activity Level

Rs. 2,80,000 1,50,000 4,30,000

As it is the BEP, therefore, here, Total Cost = Total Sales = Rs. 4,30,000 ∴ Selling Price per unit = Rs. 4,30,000 ÷ 16,000 units = Rs. 26.875

Modified Date: Mon, Jul 05, 2010 05:08:16 PM

Output Date: Tue, Jul 06, 2010 12:23:48 PM

Rev II

Project: Management Accounting_Debarshi Bhattacharyya ACE Pro India Pvt. Ltd. File: X:\Pearson\Management Accounting_Debarshi Bhattacharyya\MAIN\M10\LAYOUT_M10\M10_DEBA_ISBN_EN_SE_C10.indd

688

MANAGEMENT ACCOUNTING

Problem 44 A company producing a single product sells it at Rs. 50 per unit. The unit Variable Cost is Rs. 35 and Fixed Cost amounts to Rs. 12 lakhs per annum. With this data, you are required to calculate the following, treating each as independent of the other: i. ii. iii. iv.

P/V Ratio and BES. New BES, if the Variable Cost increases by Rs. 3 per unit, without any increase in the Selling Price. Increase in the sales required if profits are to be increased by Rs. 2.4 lakhs. Percentage increase/decrease in the sales volume units to offset: a. An increase of Rs. 3 in the Variable Cost per unit. b. A 10% increase in the Selling Price without affecting the existing profit quantum. v. Quantum of advertisement expenditure permissible to increase the sales by Rs. 1.2 lakhs, without affecting the existing profit quantum. [I.C.W.A—(Inter)]

Solution i. Less:

Rs. 50 35 15

Selling Price per unit Variable Cost per unit Contribution per unit

C 15 × 100 = × 100 = 30% S 20 F 12,00,000 ∴ BEPS = × 100 = × 100 = Rs. 40 lakhs P/V Ratio 24 ∴ P/V Ratio =

ii. Less:

Selling Price per unit New Variable Cost per unit (Rs. 35 + Rs. 3) New Contribution per unit

Rs. 50 38 12

C 12 × 100 = × 100 = 24% S 50 F 12,00,000 ∴ New BEPS = × 100 = × 100 = Rs. 50 lakhs P/V Ratio 24 ∴ New P/V Ratio =

iii. Profits to be increased by Rs. 2.4 lakhs (over the existing situation). This is a level after achieving BEP (i.e., MS). At this level, Contribution = Profit. ∴ New contribution required = Rs. 2.4 lakhs Again, Existing P/V Ratio = 30% ∴ Increase in sales required to increase the profits by Rs. 2.4 lakhs = (2,40,000 / 30) × 100 = Rs. 8 lakhs iv. New Variable Cost per unit = Rs. 35 + Rs. 3 = Rs. 38 ∴ New Contribution per unit = Rs. 50 – Rs. 38 = Rs. 12 C 12 ∴ New P/V Ratio = × 100 = × 100 = 24% S 50

Modified Date: Mon, Jul 05, 2010 05:08:16 PM

Output Date: Tue, Jul 06, 2010 12:23:48 PM

Rev II

Project: Management Accounting_Debarshi Bhattacharyya ACE Pro India Pvt. Ltd. File: X:\Pearson\Management Accounting_Debarshi Bhattacharyya\MAIN\M10\LAYOUT_M10\M10_DEBA_ISBN_EN_SE_C10.indd

MARGINAL COSTING AND COST–VOLUME–PROFIT ANALYSIS

689

∴ Loss of Contribution per unit = Rs. 15 – Rs. 12 = Rs. 3 ∴ Increase in Sales required to regain C 3 = × 100 = × 100 = Rs.12.50 this loss of Contribution of Rs. 3 P/V Ratio 24 Again, Selling Price per unit = Rs. 50 12.50 = 0.25 unit 50 ∴ % increase in the Sales units required = 0.25 ÷ 1 × 100 = 25% ∴ Increase in the Sales units required =

v. New Selling Price per unit = Rs. 50 + 10% of Rs. 50 = Rs. 55 Less: Variable Cost per unit = Rs. 35 New Contribution per unit = Rs. 20 C 20 ∴ New P/V Ratio = × 100 = × 100 = 36.36% S 55 ∴ Increase in Contribution per unit = Rs. 20 – Rs. 15 = Rs. 5 ∴ Decrease in Sales required to off-set this increase in the Contribution of Rs. 5 C 5 = × 100 = × 100 = Rs. 13.75 P/V Ratio 36.36 Again, Selling Price per unit = Rs. 55 ∴ Decrease in Sales units required to maintain the existing quantum of Profits = 13.75 ÷ 55 = 0.25 unit ∴ % decrease in the sales units required = 0.25 ÷ 1 × 100 = 25% vi. Existing P/V Ratio = 30% ∴ Increase in Contribution due to the increase in Sales by Rs. 1.2 lakhs P/V Ratio × S 30 × 1,20,000 = = = Rs. 36,000 100 100 ∴ This Rs. 36,000 should be the maximum permissible advertisement expenditure to set off an increase in sales by Rs. 1.2 lakhs without affecting the existing profits. CHAPTER REVIEW SUMMARY  Marginal Cost is the additional cost incurred for an increase in one additional unit of output. It is nothing but the Variable Cost.  Marginal Costing is the method of ascertaining Marginal Cost and it evaluates the effect of Fixed and Variable Costs on profit due to a change in the volume of production.  Distinguished features of Marginal Costing are as follows: (a) Only Variable Costs are charged to the cost unit. Fixed Costs are recovered from contribution; (b) All costs including semi-variable costs are divided into two parts—fixed and variable; (c) Closing inventories are valued at Variable Cost only; and (d) BE Analysis and CVP Analysis are integral parts of this costing technique.  Marginal Costing Technique has many advantages, such as: (a) It provides useful data for managerial Decision Making; (b) It is a very effective tool of profit planning; (c) It facilitates control over Variable Costs by avoidance of arbitrary apportionment or allocation of Fixed Costs; (d) Problems on computation of accurate Fixed Factory Overhead Rate can be avoided as fixed overheads are charged against the contribution; and (e) It provides the management with many useful techniques for a Decision Making like BE Analysis.  Limitations of Marginal Costing are: (a) It assumes that the semi-variable costs can be segregated into two parts —fixed and variable elements. In practice, however, such segregation of semi-variable costs is very difficult; (b) It excludes the Fixed Cost for Decision Making, which sometimes may lead to a wrong conclusion;

Modified Date: Mon, Jul 05, 2010 05:08:16 PM

Output Date: Tue, Jul 06, 2010 12:23:48 PM

Rev II

Project: Management Accounting_Debarshi Bhattacharyya ACE Pro India Pvt. Ltd. File: X:\Pearson\Management Accounting_Debarshi Bhattacharyya\MAIN\M10\LAYOUT_M10\M10_DEBA_ISBN_EN_SE_C10.indd

690









 

 











MANAGEMENT ACCOUNTING

(c) It fails to reflect the impact of increased Fixed Costs due to the development of technology on Production Cost; and (d) Variable-Cost Technique cannot be successfully applied in ‘cost-plus contract.’ CVP Analysis examines the relationship of costs and profit to the volume of production to maximize the profit of the firm. It is the method of studying the relationship between the cost, volume of production, sales and their impact on profit. CVP Analysis is a logical extension of Marginal Costing and is used as a very powerful tool by the management in the process of budgeting and profit planning. Objectives of CVP Analysis are: (a) It helps to forecast profit fairly and accurately; (b) It acts as an effective tool of profit planning to the management; (c) It helps in ascertaining the BEP of the product produced and sold; (d) It is very much useful in setting up the flexible budget; (e) It assists the management in the process of performance evaluation for the purpose of control; and (f) It helps in formulating the price policies by projecting the effect of different price structures on costs and profits. Underlying assumptions of CVP Analysis are: (a) Total Cost consists of two components—Fixed Cost and Variable Cost; (b) Selling Price per unit remains constant at different volumes of sales; (c) Only one product is sold by the concern; or, if it sells multiple products, then the sales mix remains constant at different volumes of sales; and (d) Volume of production is equal to the sales volume. In CVP Analysis, costs are classified into two parts—Fixed Cost and Variable Cost. Semi-Variable Cost is not separately recognized in CVP Analysis. Fixed portion of the Semi-Variable Cost is clubbed with the Fixed Cost and its variable portion is clubbed with the Variable Cost. Elements of CVP Analysis are: (a) Marginal Cost equation; (b) Contribution; (c) P/V Ratio; (d) BEP; and (e) MS. Marginal Cost Equation exhibits the relationship between Contribution, Fixed Cost and Profit. It explains that the excess of sales over the Variable Cost is the contribution towards the Fixed Cost and profit, that is, S − V = F + P. Contribution is the excess of sales over Variable Cost, that is, C = S – V. This contribution is available towards Fixed Cost and profit, that is, C = F + P. Profit–Volume ratio (P/V Ratio) is the ratio of contribution and sales. It is generally expressed in percentage. It exhibits a percentage of contribution included in sales, that is, P/V Ratio = C/S × 100. It indicates the effect on profit for a given change in sales. Break-Even Point (BEP) is that level of sales where there is no profit or no loss. At BEP, the total sales revenue is equal to the Total Cost. If there are any sales above this BEP, a concern earns profit; whereas any sales below this BEP, the concern suffers loss. At BEP, the total Fixed Cost and Variable Cost up to that level of sales have been recovered from the sales. Generally, at any other point of sales, the contribution from sales is available towards the Fixed Cost and profit. But as there is no profit or loss at BEP, contribution from sales at BEP is available towards the Fixed Cost only, that is, at BEP, C = F. Margin of Safety (MS) is the level of sales made above the BEP. In other words, MS is the excess of actual sales over BEPS. Generally, at any point of sales, the contribution from sales is available towards the Fixed Cost and profit. But as the total Fixed Cost has already been recovered at BEP, the contribution from sales at MS is available towards profit only, that is, at MS, C = P. CVP Analysis is popularly known as BE Analysis, although there exists a narrow difference between these two terms. CVP Analysis refers to the study of the effect on the profit due to changes in the cost and volume of output, whereas BE Analysis refers to the study of determination of that level of activity where the total sales is equal to the Total Cost and also the study of determination of profit at any level of activity. However, the technique of BE Analysis is so popular for studying CVP Analysis that these two terms are generally used synonymously. Break-Even Chart (BE Chart) is the graphical representation of BE Analysis. It depicts the relationship between costs, sales and profits. BE chart graphically shows the profit or loss at various levels of activity and also shows the level of activity where there is no profit no loss (i.e., Total Cost equals total sales). Angle of Incidence is the angle formed by the intersection of sales line and Total Cost line at BEP in the BE chart. This angle exhibits the rate at which profits are being earned by a concern after reaching the BEP. It shows the profit-earning capacity of a concern. Wider angle of incidence exhibits higher profit-earning capacity of the concern or vice-versa.

Modified Date: Mon, Jul 05, 2010 05:08:16 PM

Output Date: Tue, Jul 06, 2010 12:23:48 PM

Rev II

Project: Management Accounting_Debarshi Bhattacharyya ACE Pro India Pvt. Ltd. File: X:\Pearson\Management Accounting_Debarshi Bhattacharyya\MAIN\M10\LAYOUT_M10\M10_DEBA_ISBN_EN_SE_C10.indd

MARGINAL COSTING AND COST–VOLUME–PROFIT ANALYSIS

691

CHAPTER REVIEW QUIZ 1. State the effects of the following changes on P/V Ratio: a. Increase in the unit Selling Price. b. Decrease in the unit Variable Cost. c. Increase in the total Fixed Cost. d. Decrease in the Sales Volume. Ans.: (i) P/V Ratio increases; (ii) P/V Ratio increases; (iii) P/V Ratio does not change; (iv) P/V Ratio does not change. 2. State the effects of the following changes on BEP: a. Increase in the unit Selling Price. b. Increase in the unit Variable Cost. c. Increase in the total Fixed Cost. d. Increase in the Sales Volume. Ans.: (i) BEP decreases; (ii) BEP increases; (iii) BEP increases; (iv) BEP does not change. 3. State the effects of the following changes on margin of safety (MS): a. Increase in the unit Selling Price. b. Increase in the unit Variable Cost. c. Increase in the total Fixed Cost. d. Increase in the Sales Volume. Ans.: (i) MS increases; (ii) MS decreases; (iii) MS decreases; (iv) MS increases. 4. Fill in the blanks of the following statements: . a. CVP Analysis is an integral part of . b. Additional cost incurred for producing one additional unit is called . c. Generally, contribution = . d. At BEP, contribution = . e. At MS, contribution = . f. P/V Ratio exhibits the percentage of contribution included in line and g. Angle of incidence is the angle formed by intersection of line at BEP in the BE chart. is equal to the volume h. Under CVP Analysis, it is assumed that the volume of . of Ans.: (a) Marginal Costing; (b) Marginal Cost; (c) Fixed Cost plus profit; (d) Fixed Cost; (e) profit; (f) sales; (g) Sales, Total Cost; (h) production, sales. 5. Choose the correct alternative from the following: a. BEP is that level of sales where: (i) the Fixed Cost is equal to the Variable Cost; (ii) total sales is equal to the Total Cost; (iii) contribution is equal to the profit. b. Contribution at BEP is equal to the: (i) Fixed Cost; (ii) Variable Cost; (iii) profit. c. Contribution at MS is equal to the: (i) Fixed Cost; (ii) Variable Cost; (iii) profit. d. P/V Ratio does not change with the change in: (i) Selling Price per unit; (ii) sales volume; (iii) Variable Cost per unit. e. If total sales is Rs. 5,00,000 and MS is 40%, then BEP is equal to: (i) Rs. 2,00,000; (ii) Rs. 3,00,000; (iii) Rs. 2,50,000. f. If P/V Ratio is 40% and Fixed Cost is Rs. 2,00,000, then BEP is equal to: (i) Rs. 5,00,000; (ii) Rs. 80,000; (iii) Rs. 1,20,000. g. If Fixed Cost is Rs. 4,00,000 and contribution per unit is Rs. 20, then BEP is equal to: (i) 40,000 units; (ii) 30,000 units; (iii) 20,000 units. h. If profit is Rs. 1,00,000 and contribution per unit is Rs. 20, then MS is equal to: (i) 2,000 units; (ii) 5,000 units; (iii) 1,000 units. i. If Contribution–Sales ratio is 30% and MS is Rs. 2,00,000, then profit is equal to (i) Rs. 20,000; (ii) Rs. 60,000; (iii) Rs. 1,40,000. j. If profit is Rs. 80,000 and MS is Rs. 2,00,000, then P/V Ratio is equal to: (i) 40%; (ii) 30%; (iii) 50%. Ans.: (a) (ii); (b) (i); (c) (iii); (d) (ii); (e) (ii); (f) (i); (g) (iii); (h) (ii); (i) (ii); (j) (i).

Modified Date: Mon, Jul 05, 2010 05:08:16 PM

Output Date: Tue, Jul 06, 2010 12:23:48 PM

Rev II

Project: Management Accounting_Debarshi Bhattacharyya ACE Pro India Pvt. Ltd. File: X:\Pearson\Management Accounting_Debarshi Bhattacharyya\MAIN\M10\LAYOUT_M10\M10_DEBA_ISBN_EN_SE_C10.indd

692

MANAGEMENT ACCOUNTING

6. State whether the following statements are true or false: a. Marginal Cost is nothing but the Variable Cost. b. At BEP, the Total Cost is higher than the total sales. c. At BEP, the contribution is equal to the Variable Cost. d. P/V Ratio does not change due to the change in Fixed Cost. e. P/V Ratio changes with the change in the sales volume. f. Contribution at MS level is equal to the profit. g. P/V Ratio changes with the change in the Variable Cost per unit. h. P/V Ratio remains unchanged with the change in the Selling Price per unit. i. Generally, contribution is equal to the sum of Fixed Cost and profit. j. Difference between total sales and BEPS represents MS. k. A concern suffers loss at any sales level below the BES. l. Profit changes in the same proportion of the change in contribution. Ans.: True: (a), (d), (f), (g), (i), (j), (k); False: (b), (c), (e), (h), (l).

EXERCISE I. Theoretical Questions A. Short Answer Type Questions:

1. 2. 3. 4. 5. 6. 7. 8. 9. 10. 11.

What is Marginal Cost? What is Marginal Costing? What is Cost–Volume–Profit Analysis? What is the Marginal Cost equation? What is Contribution? What is Profit–Volume Ratio? What is Break-Even Point? What is Margin of Safety? What is Break-Even Chart? What is Angle of Incidence? Is contribution equal to profit?

B. Essay Type Questions

1. Define Marginal Costing. What are its features? 2. What are the advantages and limitations of Marginal Costing? 3. What is Cost–Volume–Profit Analysis? What are the underlying assumptions behind CVP Analysis? What are its objectives? 4. How is cost classified under CVP Analysis? What are the elements of CVP Analysis? 5. What is Marginal Cost Equation? How is it developed? 6. Write a note on P/V Ratio. What are its features? 7. What is Contribution? State the difference between contribution and profit. 8. Write notes on the following: 9. (a) P/V Ratio; (b) BEP; (c) MS; (d) Angle of Incidence. 10. What is BE Chart? What does it indicate? How is it presented? II. Practical Problems

1. P/V Ratio = 40% Fixed Cost = Rs. 1,00,000 Find out: a. Sales when profit is Rs. 80,000.

Modified Date: Mon, Jul 05, 2010 05:08:16 PM

Output Date: Tue, Jul 06, 2010 12:23:48 PM

Rev II

Project: Management Accounting_Debarshi Bhattacharyya ACE Pro India Pvt. Ltd. File: X:\Pearson\Management Accounting_Debarshi Bhattacharyya\MAIN\M10\LAYOUT_M10\M10_DEBA_ISBN_EN_SE_C10.indd

MARGINAL COSTING AND COST–VOLUME–PROFIT ANALYSIS

693

b. BES. c. Profit when Sales is Rs. 6,00,000. Ans.: (a) Rs. 4,50,000; (b) Rs. 2,50,000; (c) Rs. 1,40,000. 2. P/V Ratio = 25%. MS = 40%. Actual sales = Rs. 4,00,000. Find out: a. MS at Actual Sales. b. BEPS. c. Profit at Actual Sales. d. Fixed Cost. Ans.: (a) Rs. 1,60,000; (b) Rs. 2,40,000; (c) Rs. 40,000; (d) Rs. 60,000. 3. Selling Price per unit = Rs. 200. Variable Cost per unit = Rs. 100. Total Fixed Cost = Rs. 96,000. Calculate: a. BEPS in value. b. BEPS in volume. c. Sales to earn a profit of Rs. 20 per unit. [B.Com., Delhi University—1994] Ans.: (a) Rs. 1,92,000; (b) 960 units; (c) Rs. 2,40,000; 1,200 units. 4. P/V Ratio of a company is 50%, while its MS is 40%. If the sales volume of the company is Rs. 50 lakhs, find out its BEP and Net Profit. [B.Com.(Hons), Delhi University—1999] Ans.: BEP: Rs. 30,00,000; Net Profit: Rs. 10,00,000. 5. Sales = Rs. 1,00,000 Fixed Cost = Rs. 24,000 Variable Cost = Rs. 50,000 Find out: a. P/V Ratio. b. BEPS. c. MS at this level of sales. Ans.: (a) 50%; (b) Rs. 48,000; (c) Rs. 52,000. 6. Sales = Rs. 8,00,000 Total Fixed Cost = Rs. 2,00,000 Profit = Rs. 2,00,000 Calculate: a. P/V Ratio. b. BEPS. c. MS. d. Sale to earn a profit of Rs. 3,00,000. Ans.: (a) 50%; (b) Rs. 4,00,000; (c) Rs. 4,00,000; (d) Rs. 10,00,000. 7. Level of Sales = Rs. 90,000 Profit at this level of sales = Rs. 16,000 Fixed Overheads = Rs. 20,000 Calculate: a. P/V Ratio. b. BES.

Modified Date: Mon, Jul 05, 2010 05:08:16 PM

Output Date: Tue, Jul 06, 2010 12:23:48 PM

Rev II

Project: Management Accounting_Debarshi Bhattacharyya ACE Pro India Pvt. Ltd. File: X:\Pearson\Management Accounting_Debarshi Bhattacharyya\MAIN\M10\LAYOUT_M10\M10_DEBA_ISBN_EN_SE_C10.indd

694

MANAGEMENT ACCOUNTING

c. Profit at the sales of Rs. 2,00,000. d. Sales required to earn a profit of Rs. 30,000. Ans.: (a) 40%; (b) Rs. 50,000; (c) Rs. 60,000; (d) Rs. 1,25,000. 8. BEPS = Rs. 1,20,000 Total Fixed Cost = Rs. 30,000 Find out: a. P/V Ratio. b. Profit when sales is Rs. 2,00,000. c. New BEP, if sales is reduced by 10%. [B.Com. (Hons), Delhi University—Adapted] Ans.: (a) 25%; (b) Rs. 20,000; (c) Rs. 1,80,000. 9. P/V Ratio = 30% MS = 20% Total Fixed Cost = Rs. 60,000 Calculate: a. BEPS. b. Total Sales. c. Profit at Total Sales. d. Total Variable Cost for Total Sales. Ans.: (a) Rs. 2,00,000; (b) Rs. 2,50,000; (c) Rs. 15,000; (d) Rs. 1,75,000. 10. Find out: (i) P/V Ratio; (ii) BEPS; (iii) Net Profit from Rs. 75,000 sales; and (iv) Sales required for a profit of Rs. 7,500. If the sales amount to Rs. 50,000 and the Net Profit amounts to Rs. 5,000 and fixed overheads are Rs. 7,500. [B.Com. (Hons), Kalyani University (N.P.)—2007] Ans.: (i) 25%; (ii) Rs. 30,000; (iii) Rs. 11,250; (iv) Rs. 60,000. 11. A company producing a single product sells each unit at Rs. 10. The Marginal Cost is Rs. 6. The Fixed Costs are Rs. 3,00,000. The annual production is 1,50,000 units. Calculate: (a) P/V Ratio and BES; and (b) Sales required to make an additional profit of Rs. 1,00,000 maintaining the Selling Price at Rs. 10. [B.Com.(Hons), Kalyani University—2006] Ans.: (a) 40%, Rs. 7,50,000; (b) Rs. 17,50,000. 12. Z Ltd produces and sells a single article at Rs. 10 each. The Marginal Cost of production is Rs. 6 and the Fixed Cost is Rs. 400 per annum. Calculate: (a) P/V Ratio; (b) BES (in Rs. and Nos.); (c) Sales to earn a profit of Rs. 500; (d) Profit at a sale of Rs. 3,000; (e) New BEP if the Selling Price is reduced by 10%; (f) MS at a sale of Rs. 1,500; and (g) Selling Price per unit if the BEP is reduced to 80 units. [B.Com. (Hons), Mumbai University—October 2004] Ans.: (a) 40%; (b) Rs. 1,000; (c) Rs. 2,250; (d) Rs. 800; (e) Rs. 1,200; (f) Rs. 300 [as per new BEP as ascertained under (e)]; (g) Rs. 11. 13. From the following details, find out the BES and Fixed Cost and the required sales to earn a profit of Rs. 3,00,000. Sales MS P/V Ratio

Rs. 9,00,000 40% 2/3

[B.Com. (Hons), Calcutta University—2008] Ans.: BEPS – Rs. 5,40,000; Fixed Cost – Rs. 3,60,000; Required sales – Rs. 9,90,000.

Modified Date: Mon, Jul 05, 2010 05:08:16 PM

Output Date: Tue, Jul 06, 2010 12:23:48 PM

Rev II

Project: Management Accounting_Debarshi Bhattacharyya ACE Pro India Pvt. Ltd. File: X:\Pearson\Management Accounting_Debarshi Bhattacharyya\MAIN\M10\LAYOUT_M10\M10_DEBA_ISBN_EN_SE_C10.indd

695

MARGINAL COSTING AND COST–VOLUME–PROFIT ANALYSIS

14. i. Find out the BEPS if the budgeted output is 80,000 units, the Fixed Cost is Rs. 4,00,000, the Selling Price per unit is Rs. 20 and Variable Cost per unit is Rs. 10. ii. Calculate the Selling Price, if the Marginal Cost is Rs. 2,400 and P/V Ratio is 20%. iii. Find out the MS if the profit is Rs. 20,000 and P/V Ratio is 40%. [B.Com., Delhi University—1997] Ans.: (i) Rs. 8,00,000, 4,000 units; (ii) Rs. 3,000; (iii) Rs. 50,000. 15. M/S M Ltd made sales of Rs. 2,50,000 during a certain period. The Net Profit for the same period was Rs. 24,000 and the fixed overheads were Rs. 38,000. Find out: (i) BEPS; and (ii) Volume of sales to earn a profit of Rs. 40,000. [B.Com.(Hons), Calcutta University—2005] Ans.: (i) Rs. 1,53,226; (ii) Rs. 3,14,516. 16. Following are the information available from the records of a company: Rs. in Lakhs 2,000 1,200 600

Sales Marginal Cost Fixed Cost

Determine the BES and MS. Ans.: 1,500 (Rs. in lakhs); 500 (Rs. in lakhs). 17. From the following data, calculate: i. BEP expressed in amount of sales in rupees. ii. How many units to be sold to earn a net income of 10% of sales? Selling Price per unit = Rs. 20; Variable Cost per unit = Rs. 12; Fixed Cost = Rs. 2,40,000. [B.Com., Delhi University—1991] Ans.: (i) Rs. 6,00,000; (ii) 40,000 units. 18. During 2009, a company earned a profit of Rs. 1,80,000 on a sale of Rs. 30,00,000. The Variable Costs were Rs. 21,00,000. Calculate: a. BEPS at present. b. BEPS if Variable Costs increase by 5%. c. Sales required to maintain the profit as at present if the Selling Price is reduced by 5%. Ans.: (a) Rs. 24,00,000; (b) Rs. 27,16,981; (c) Rs. 34,20,752. 19. The following figures relate to M/S Deepak Industries: Rs. 2,40,000 4,00,000 3,00,000 8,00,000 20,00,000

Fixed Overheads Variable Overheads Direct Wages Direct Materials Sales

Calculate: i. P/V Ratio; (ii) BEP; (iii) MS. [B.Com.(Hons), Mumbai University—April 2008] Ans.: (i) 25%; (ii) Rs. 9,60,000; (iii) Rs. 10,40,000. 20. Rs. 50.00 20.00 10.00

Selling Price per unit Direct Materials per unit Direct Labour per unit

(Continued)

Modified Date: Mon, Jul 05, 2010 05:08:16 PM

Output Date: Tue, Jul 06, 2010 12:23:48 PM

Rev II

Project: Management Accounting_Debarshi Bhattacharyya ACE Pro India Pvt. Ltd. File: X:\Pearson\Management Accounting_Debarshi Bhattacharyya\MAIN\M10\LAYOUT_M10\M10_DEBA_ISBN_EN_SE_C10.indd

696

MANAGEMENT ACCOUNTING

Rs. 5.00 5.00 1,00,000

Direct Expenses per unit Variable Overheads per unit Total Fixed Overheads

Calculate: a. BES in value and in units. b. Profit if sales are 50% above the BEP. Ans.: (a) Rs. 5,00,000, 10,000 units; (b) Rs. 50,000. 21. From the following data, you are required to calculate: (a) P/V Ratio; (b) BES with the help of P/V Ratio; (c) Sales required to earn a profit of Rs. 4,50,000. Fixed Expenses Selling Price per unit Direct Material per unit Direct Labour per unit Overheads

Rs. 90,000 Rs. 12 Rs. 5 Rs. 2 100% of Labour

[B.Com. (Hons), Kalyani University—2007 (N.S.)] Ans.: (a) 25%; (b) Rs. 3,60,000, 30,000 units; (c) Rs. 21,60,000. 22. Material per unit Labour per unit Variable Overhead per unit

Rs. 50 Rs. 80 75% of Labour cost Rs. 2,40,000 Rs. 230

Fixed Overheads for the Year Selling Price per unit

Calculate: a. BEPS in value and in volume. b. Required sales to earn a profit of Rs. 1,00,000. c. If the Selling Price is reduced by Rs. 15 each, how many units have to be sold for BE? [I.C.W.A. (Inter)—Adapted] Ans.: (a) Rs. 13,80,000, 6,000 units; (b) Rs. 19,55,000; (c) 9,600 units. 23. From the following particulars, calculate: i. BES in terms of sales value and in units. ii. Number of units that must be sold to earn a profit of Rs. 90,000. Rs. 60,000 12,000 12 3 24

Fixed Factory Overhead Cost Fixed Selling Overhead Cost Variable Manufacturing Cost per unit Variable Selling Cost per unit Selling Price per unit

[B.B.M., Bangalore University—December 2006] Ans.: (i) Rs. 1,92,000, 8,000 units; (ii) 18,000 units. 24. From the following information, calculate: (a) P/V Ratio and (b) BEP: Year 2004 2005

Sales (Rs.) 4,80,000 5,60,000

Profit (Rs.) 32,000 52,000

[B.Com. (Hons), Calcutta University—2006] Ans.: (a) 25%; (b) Rs. 3,52,000.

Modified Date: Mon, Jul 05, 2010 05:08:16 PM

Output Date: Tue, Jul 06, 2010 12:23:48 PM

Rev II

Project: Management Accounting_Debarshi Bhattacharyya ACE Pro India Pvt. Ltd. File: X:\Pearson\Management Accounting_Debarshi Bhattacharyya\MAIN\M10\LAYOUT_M10\M10_DEBA_ISBN_EN_SE_C10.indd

MARGINAL COSTING AND COST–VOLUME–PROFIT ANALYSIS

697

25. The operating result of two companies for the last 2 years are as follows: Year

Sales (Rs.) 1,35,000 1,50,000

2006 2007

Profit (Rs.) 3,000 7,500

The cost structure and the Selling Price remain the same in both the years. Calculate: (a) P/V Ratio; (b) Fixed Cost; (c) BEPS; (d) MS at a profit of Rs. 12,000. [B.Com. (Hons), Kalyani University—2008 (N.P.)] Ans. (a) 30%; (b) Rs. 37,500; (c) Rs. 1,25,000; (d) Rs. 40,000. 26. The XL Ltd furnishes the following information: 1st Period (Rs.) 2,00,000 20,00,000

Profit Sales

2nd Period (Rs.) 4,00,000 30,00,000

From the above, calculate the following: a. P/V Ratio. b. Fixed Cost. c. BEPS. d. Sales to earn a profit of Rs. 5,00,000. e. Profit when sales are Rs. 15,00,000. [B.Com. (Hons), Mumbai University—October 2006] Ans.: (a) 20%; (b) Rs. 2,00,000; (c) Rs. 10,00,000; (d) Rs. 35,00,000; (e) Rs. 1,00,000. 27. Year Sales Loss Profit

2008 (Rs.) 1,00,000 10,000 –

2009 (Rs.) 2,00,000 – 40,000

Find out: a. P/V Ratio. b. Fixed Cost. c. BEPS. d. MS for 2009. e. Required sales to incur a loss of Rs. 20,000. Ans.: (a) 50%; (b) Rs. 60,000; (c) Rs. 1,20,000; (d) Rs. 80,000; (e) Rs. 80,000. 28. The sales and profits of X Ltd for two successive years are given as follows: Year 2003–04 2004–05

Sales (Rs.) 5,00,000 7,00,000

Profit/Loss (Rs.) 20,000 30,000

You are requested to determine: (i) P/V Ratio; (ii) BEP; (iii) Sales to earn a profit of Rs. 55,000; (iv) Estimated profit for 2005–06 if the sales are expected to be Rs. 9,00,000; (v) MS if the profit is Rs. 25,000. [B.B.A., Calcutta University—2005] Ans.: (i) 25%; (ii) Rs. 5,80,000; (iii) Rs. 8,00,000; (iv) Rs, 80,000; (v) Rs. 1,00,000. 29. A company earned a profit of Rs. 2,00,000 on a sale volume of Rs. 14,00,000 during the first half of a year, the Fixed Cost being Rs. 5,00,000. However, during the second-half of the year, it incurred a loss of Rs. 1,00,000, although the unit Variable Cost, the Selling Price and the Fixed Cost remained the same.

Modified Date: Mon, Jul 05, 2010 05:08:16 PM

Output Date: Tue, Jul 06, 2010 12:23:48 PM

Rev II

Project: Management Accounting_Debarshi Bhattacharyya ACE Pro India Pvt. Ltd. File: X:\Pearson\Management Accounting_Debarshi Bhattacharyya\MAIN\M10\LAYOUT_M10\M10_DEBA_ISBN_EN_SE_C10.indd

698

MANAGEMENT ACCOUNTING

Required: i. ii. iii.

P/V Ratio, BEP and MS for the first-half of the year. Sales volume for the second-half. BEP and MS for the whole year.

[I.C.W.A. (Inter)—June 2003] Ans.: (i) 50%, Rs. 10,00,000, Rs. 4,00,000; (ii) Rs. 8,00,000; (iii) Rs. 20,00,000, Rs. 2,00,000. 30. The following figures relating to the performance of a company at the year I and II are available. Assuming that (i) the ratio of Variable Costs to sales; and (ii) the Fixed Costs, are the same for both the years, ascertain: a. The P/V Ratio. b. The amount of the Fixed Costs. c. The BEP. d. The budgeted profit for the year III, if the budgeted sales for that year are Rs. 1 crore. Total Sales (Rs. in ‘000) 7,000 9,000

Year I Year II

Total Costs (Rs. in ‘000) 5,800 6,600

[I.C.W.A (Inter)—June 1991] Ans.: (a) 60%; (b) Rs. 30,00,00; (c) Rs. 50,00,000; (d) Rs. 30,00,000. 31. Sales, Marginal Cost at this sales and BES of a company are Rs. 5,00,000, Rs. 2,00,000, and Rs. 3,00,000, respectively. Find out: a. Fixed Cost. b. Profit/Loss at a sale of Rs. 2,00,000. Ans.: (a) Rs. 1,20,000; (b) Rs. 40,000. 32. G Ltd maintains an MS of 33.33% with an overall contribution–sales ratio of 60%. If the Fixed Costs amount to Rs. 3 lakhs, calculate the following: (a) BES; (b) Total sales; (c) Total Variable Cost; (d) Current profit; (e) MS; and (f) New MS, if the sales value is increased by 20%. [B.B.M., Bangalore University—Adapted] Ans.: (a) Rs. 5,00,000; (b) Rs. 7,50,000; (c) Rs. 3,00,000; (d) Rs. 1,50,000; (e) Rs. 2,50,000; (f) Rs. 4,00,000. 33. Production and Sales for the Year Selling Price per unit Variable Cost per unit Fixed Cost for the Year

40,000 units Rs. 25 Rs. 13 Rs. 3,36,000

Find out: a. BEPS in value and in units. b. Number of units to be sold to earn a profit of Rs. 30,000. c. Number of units to be sold to earn a profit of 20% on sales. Ans.: (a) Rs. 7,00,000, 28,000 units; (b) 30,500 units; (c) 48,000 units. 34. Rajapaksha Ltd furnishes the following information: Selling Price per unit Direct Materials Cost per unit Direct Labour Cost per unit Variable Overhead per unit Budgeted Level of Output & Sales Budgeted Recovery Rate of Fixed Overhead Cost per unit

Modified Date: Mon, Jul 05, 2010 05:08:16 PM

Output Date: Tue, Jul 06, 2010 12:23:48 PM

Rs. 20 Rs. 5 Rs. 3 Rs. 2 80,000 units Rs. 5

Rev II

Project: Management Accounting_Debarshi Bhattacharyya ACE Pro India Pvt. Ltd. File: X:\Pearson\Management Accounting_Debarshi Bhattacharyya\MAIN\M10\LAYOUT_M10\M10_DEBA_ISBN_EN_SE_C10.indd

MARGINAL COSTING AND COST–VOLUME–PROFIT ANALYSIS

699

You are required to calculate: a. BES. b. New BES: i. If the Selling Price per unit is increased by 30%. ii. If the Selling Price per unit is decreased by 10%. Ans.: (a) Rs. 8,00,000; (b) (i) Rs. 6,50,000; (ii) Rs. 9,00,000. 35. The following figures are extracted from the books of a manufacturing concern for the year 1990–91: Rs. 2,05,000 75,000 60,000 1,00,000 5,00,000

Direct Materials Direct Labour Fixed Overheads Variable Overheads Sales

a. Calculate the BEP. b. What will be the effect on BEP, if there is an increase of 10% in: i. Fixed expenses. ii. Variable expenses. [C.A. (Inter)—June 1992] Ans.: (a) Rs. 2,50,000; (b) (i) Rs. 2,75,000; (ii) Rs. 3,65,854. 36. Units Produced and Sold for the Year Selling Price Fixed Expenses for the Year Variable Cost per unit

15,000 units Rs. 1,50,000 Rs. 34,000 Rs. 6

Calculate: a. P/V Ratio, BEP and MS. b. Revised P/V Ratio, BEP and MS in each of the following cases: i. Decrease of 10% in the Selling Price. ii. Increase of 10% in the Variable Cost. iii. Increase of sales volume by 2,000 units. iv. Increase of Rs. 6,000 in the Fixed Costs. v. Increase of 20% in the Selling Price accompanied by an increase of Rs. 10,000 in the Fixed Costs and a decrease of 10% in the Variable Costs. Ans.: (a) 40%, Rs. 85,000 and Rs. 65,000. (b) (i) 33.33%, Rs. 1,02,000 and Rs. 33,000. (ii) 34%, Rs. 1,00,000 and Rs. 50,000. (iii) 40%, Rs. 85,000 and Rs. 85,000. (iv) 40%, Rs. 1,00,000 and Rs. 50,000. (v) 55%, Rs. 80,000 and Rs. 1,00,000. 37. Indian Plastics Ltd makes plastic buckets. An analysis of their accounting reveals: Variable Cost per Bucket Fixed Cost Capacity Selling Price per Bucket

Rs. 20 Rs. 50,000 per year 2,000 Buckets per year Rs. 70

Required: i. Find the BEP. ii. Find the number of buckets to be sold to earn a profit of Rs. 30,000.

Modified Date: Mon, Jul 05, 2010 05:08:16 PM

Output Date: Tue, Jul 06, 2010 12:23:48 PM

Rev II

Project: Management Accounting_Debarshi Bhattacharyya ACE Pro India Pvt. Ltd. File: X:\Pearson\Management Accounting_Debarshi Bhattacharyya\MAIN\M10\LAYOUT_M10\M10_DEBA_ISBN_EN_SE_C10.indd

700

MANAGEMENT ACCOUNTING

iii.

If the company can manufacture 600 buckets more with a fixed additional cost of Rs. 2,000, what should be the Selling Price to maintain the profit per bucket as at (ii) above? [B.Com. (Hons), Kalyani University—2008 (Sup.)] Ans.: (i) 71.4285%; (ii) 1,600 buckets; (iii) Rs. 62.3863. 38. From the following information, calculate: (i) Fixed Cost; and (ii) BEP. Units

Profit/Loss (Rs.) 3,00,000 3,00,000

10,000 30,000

[B.Com. (Hons), Kalyani University—Adapted] Ans.: (i) Rs. 6,00,000; (ii) 20,000 units. 39. E & Co. and F & Co. furnished the following details: Annual Production Fixed Costs Profit at the Above Level of Production

E & Co. 20,000 units Rs. 60,000 Rs. 40,000

F & Co. 20,000 units Rs. 40,000 Rs. 40,000

Find out: a. BEP units of each company. b. Level of sales (in units) at which both the companies earn an equal profit or incur an equal loss. Ans.: (a) 12,000 units, 10,000 units; (b) 20,000 units. 40. The particulars of two plants producing an identical product with the same Selling Price are as follows: Capacity Utilization Sales Variable Cost Fixed Cost

Plant A 70% (Rs. in lakhs) 150 105 30

Plant B 60% (Rs. in lakhs) 90 75 20

It has been decided to merge Plant B with Plant A. The additional fixed expenses involved in the merger amount to Rs. 2,00,000. You are required to find out: a. The BEP of Plant A and Plant B before merger and the BEP of the merged plant. b. The capacity utilization of the integrated plant required to earn a profit of Rs. 18 lakhs. [B.Com. (Hons), Calcutta University—2008] Ans.: (a) BEP: of Plant A—100 (Rs. in lakhs), of Plant B—120 (Rs. in lakhs), of the Merged Plant—204 (Rs. in lakhs); (b) 76·16%. 41. P, Q and R are three similar plants under the same management who wants them to be merged for better operation. The details of these three plants are as follows: Capacity Utilization Turnover (Rs. in lakhs) Variable Cost Fixed Cost

Plant P 100% 300 200 70

Plant Q 70% 280 210 50

Plant R 50% 150 75 62

Find out: a. The capacity utilization of the merged plant for BE. b. The profit at 75% capacity of the merged plant. c. The turnover of the merged plant to earn a profit of Rs. 28 lakhs. Ans.: (a) 52%; (b) Rs. 80.50 lakhs; (c) Rs. 600 lakhs.

Modified Date: Mon, Jul 05, 2010 05:08:16 PM

Output Date: Tue, Jul 06, 2010 12:23:48 PM

Rev II

Project: Management Accounting_Debarshi Bhattacharyya ACE Pro India Pvt. Ltd. File: X:\Pearson\Management Accounting_Debarshi Bhattacharyya\MAIN\M10\LAYOUT_M10\M10_DEBA_ISBN_EN_SE_C10.indd

MARGINAL COSTING AND COST–VOLUME–PROFIT ANALYSIS

701

42. An analysis of Sultan Manufacturing Co. Ltd led to the following information: Cost Elements Direct Materials Direct Labour Factory Overheads Distribution Overheads General Administration Overheads

Variable Cost (% of Sales) 32.8 28.4 12.6 4.1 1.1

Fixed Cost Rs.

1,89,900 58,400 66,700

Budgeted sales are Rs. 18,50,000. You are required to determine: a. BES volume. b. Profit at the budgeted sales volume. c. Profit if the actual size: (i) drop by 10%; (ii) increase by 5% from the budgeted sales. [B.Com. (Hons), Delhi University] Ans.: (a) Rs. 15,00,000; (b) Rs. 73,500; (c) (i) Rs. 34,650; (ii) Rs. 92,925. 43. Md. Alfazuddin, the manufacturer of shoes, sells his product at Rs. 20 each and makes a profit of Rs. 5 per shoe. During the year 2009, he produces 50,000 shoes at 50% capacity of his machinery, the cost of each shoe is given as follows: Rs. 6 2 5 2

Direct Materials Direct Labour Work Overheads (50% fixed) Selling Expenses (25% variable)

His anticipation for the next year is that the cost will go up as follows: % 10 5 20

Fixed Expenses Direct Materials Direct Labour

There will be no change in the Selling Price in the next year. There is an additional order for 20,000 shoes in the next year. What is the lowest price he can quote so that he can earn the same profit as the current year? [C.A. (Final) – Adapted] Ans.: Rs. 14.25. 44. X Ltd manufactures and sells a product, the Selling Price and raw material cost of which have remained unchanged during the past 2 years. The following are the relevant data: Particulars Quantity Sold (kg.) Sales Value Raw Materials Direct Wages Factory Overheads Profit

Year 1 100 Rs. 20,000 10,000 3,000 5,000 2,000

Year 2 150 Rs. ? ? ? 5,700 2,250

During Year 2, the direct wage rate was increased by 50%, but there was a saving of Rs. 300 in the fixed factory overheads.

Modified Date: Mon, Jul 05, 2010 05:08:16 PM

Output Date: Tue, Jul 06, 2010 12:23:48 PM

Rev II

Project: Management Accounting_Debarshi Bhattacharyya ACE Pro India Pvt. Ltd. File: X:\Pearson\Management Accounting_Debarshi Bhattacharyya\MAIN\M10\LAYOUT_M10\M10_DEBA_ISBN_EN_SE_C10.indd

702

MANAGEMENT ACCOUNTING

What quantity (in kg) the company should have produced and sold in Year 2 in order to maintain the same amount of profit per kg, as it earned during Year 1? [B.Com. (Hons), Calcutta University—Part II—2003] Ans.: 201 kg. 45. The Variable Cost structure of a product manufactured by a company during the current year is as follows: Rs. per unit 120 30 12

Materials Labour Overheads

The Selling Price per unit is Rs. 270 and the Fixed Cost and sales during the current year are Rs. 14 lakhs and Rs. 40.5 lakhs, respectively. During the forthcoming year, the direct workers will be entitled to a wage increase of 10% from the beginning of the year, and the material cost, variable overheads and fixed overheads are expected to increase by 7.5%, 5% and 3% respectively. The following are required to be computed: a. New sale price in the forthcoming year if the current P/V Ratio is to be maintained. b. Number of units that would require to be sold during the forthcoming year so as to yield the same amount of profit in the current year, assuming that the Selling Price per unit will not be increased. [I.C.W.A. (Inter)—June 1997] Ans.: (a) Rs. 291; (b) 17,422 units.

Modified Date: Mon, Jul 05, 2010 05:08:16 PM

Output Date: Tue, Jul 06, 2010 12:23:48 PM

Rev II

Project: Management Accounting_Debarshi Bhattacharyya ACE Pro India Pvt. Ltd. File: X:\Pearson\Management Accounting_Debarshi Bhattacharyya\MAIN\M11\LAYOUT_M11\M11_DEBA_ISBN_EN_SE_C11.indd

Decision Making

11

LEARNING OBJECTIVES On completion of the study of the chapter, you should be able to understand: What is Decision Making? Steps in Decision Making. Different cost concepts in Decision Making. Concept of Relevant Cost and Relevant Revenue in Decision Making. Concept of Differential Cost. Comparison of Differential Cost Analysis and Marginal Costing. Concept of key or limiting factor in Decision Making. Mostly occurred situations of Decision Making.

11.1 CONCEPT OF DECISION MAKING Decision Making is one of the most important functions of the management. Success of business planning highly depends on the Decision Making capacity of the management. Decision Making means the process of choosing the best one among the various alternative actions. For any business planning and control, the management is engaged in search of the most appropriate course of action. Among the various alternatives feasible to the management, it has to choose the best one from the point of view of the business concern. However, choosing the best alternative among the various options requires an evaluation of all feasible alternatives for which relevant quantitative and qualitative information are essentially available to the management. On the basis of these information, the management takes a decision on the concerned issue which deals with the future. Such decision should also affect the cost and other business factors. 11.2 STEPS IN DECISION MAKING For appropriate and effective Decision Making, the following steps are to be essentially considered: i. Clearly defining the concerned problem. ii. Identifying and describing the available alternatives. iii. Evaluating the Relevant Cost and revenue (i.e., the quantitative factors). iv. Evaluating the non-cost factors (i.e., qualitative factors). v. Selecting the best one among the identified various alternatives. vi. Evaluating the performance of the decision and providing feedback. 11.3 COST CONCEPTS IN DECISION MAKING There are certain concepts of cost which are used in the process of cost analysis for Decision Making, which are discussed as follows: i. Marginal Cost: Marginal Cost is the Variable Cost of producing a product. It includes direct and indirect Variable Costs (i.e., prime cost and overheads) incurred for the product, but it does not include

Modified Date: Sat, Jun 26, 2010 03:08:44 PM

Output Date: Tue, Jul 06, 2010 12:30:28 PM

Rev-II

Project: Management Accounting_Debarshi Bhattacharyya ACE Pro India Pvt. Ltd. File: X:\Pearson\Management Accounting_Debarshi Bhattacharyya\MAIN\M11\LAYOUT_M11\M11_DEBA_ISBN_EN_SE_C11.indd

704

MANAGEMENT ACCOUNTING

any Fixed Cost. While determining the cost of a product under Marginal Cost technique, only the Variable Costs incurred for the product are taken into consideration and Fixed Costs are ignored. ii. Differential Cost: Differential Cost refers to the difference in the cost between various alternative courses of action. More clearly, it means changes in costs due to a change in the output level, a change in the sales volume, a change in the product/sales mix, the make or buy decision, an alternative method of production and so on. In case of such change in cost due to a change in the output level, the change in cost due to an increase in the output level is called ‘incremental cost’ and the change in cost due to a decrease in the output level is called ‘decremental cost’. Both incremental as well as decremental costs are called ‘Differential Cost.’ For achieving business objectives to maximize the profit (or minimize the loss), the management should evaluate these Differential Costs with the differential revenues of the identified various alternatives to choose the best alternative which will provide the maximum return to the business. Most of the time, Differential Costs are considered by the management in the course of Decision Making. Illustration

iii.

iv.

v.

vi.

vii.

Say, the company is presently manufacturing 20,000 units of a component for its product at an unit cost of Rs. 10. If the company buys the component from the market, it would cost Rs. 8 per unit. Therefore, in this case, there would be a Differential Cost of Rs. 40,000 (i.e., Rs. 2,00,000−Rs. 1,60,000) by way of decrease in the cost if the company decides to buy the component from the market instead of manufacturing the same. This decrease in the cost by Rs. 40,000 would ultimately increase the profit of the company by Rs. 40,000. Sunk Cost: Sunk cost is caused due to a complete abandonment of a plant. It is equivalent to the written-down value of the plant abandoned less its salvage value. It is a historical cost incurred in the past and is an irrecoverable cost too. Sunk costs are not affected due to a change in the production volume. Out of Pocket Cost: It is that portion of the cost incurred for which payment was made to outsiders. Out of pocket cost represents the cost which involves payment to outsiders, for example, purchases of materials, payment of wages, overheads and so on. It does not include those costs for which the payment was not made to outsiders, for example, depreciation and so on. Opportunity Cost: Opportunity cost is nothing but the cost involved for the next best alternative foregone. More clearly, it represents the maximum possible income lost for not choosing the next best alternative course of action. It is the net cash flow that could be earned if the resource had been used in its most desirable alternative. It is the measure of the benefit of opportunity foregone. Opportunity costs are not real recorded costs in accounting. Imputed Cost: Imputed costs are those costs which are notionally considered as cost, but do not involve any cash outlay. These costs are not actually paid, but charged notionally in the accounts as cost, for example, the interest on capital for which actually no interest has been paid, notional rent charged on the business premises owned by the proprietor and so on. That is why imputed cost is also called ‘notional cost.’ Replacement Cost: It is the current cost of replacing the old asset with a new asset of identical nature. It represents the current market price of the replaced plant of an identical nature in place of the old plant.

11.4 CONCEPT OF RELEVANT COST AND RELEVANT REVENUE In the process of Decision Making, the management should consider the Relevant Costs and Relevant Revenues only and not the costs and revenues which are irrelevant for Decision Making. A cost (or revenue) that is essentially important and relevant to a Decision Making process is called Relevant Cost (or Relevant Revenue). Relevant Costs and revenues are those expected future costs and future revenues which differ under alternative courses of action being considered in Decision Making process. Relevant Costs (or revenues) are pertinent costs (or revenues) as they guide the management for an effective Decision Making. All costs and revenues, such

Modified Date: Sat, Jun 26, 2010 03:08:44 PM

Output Date: Tue, Jul 06, 2010 12:30:28 PM

Rev-II

Project: Management Accounting_Debarshi Bhattacharyya ACE Pro India Pvt. Ltd. File: X:\Pearson\Management Accounting_Debarshi Bhattacharyya\MAIN\M11\LAYOUT_M11\M11_DEBA_ISBN_EN_SE_C11.indd

DECISION MAKING

705

as sunk cost, absorbed Fixed Cost and so on, are not relevant for Decision Making. Only Relevant Costs and revenues are taken into consideration for Decision Making. Marginal Cost, Differential Cost, opportunity cost, replacement cost, Out of pocket cost and so on are the examples of Relevant Cost. Relevant Costs and revenues posses the following features: i. These costs and revenues must relate to the future. ii. They must differ under alternative courses of action. 11.5 COMPARISON OF DIFFERENTIAL COST ANALYSIS AND MARGINAL COSTING Although Differential Cost Analysis is often confused with Marginal Costing, they are not synonymous. Differential Cost Analysis has some similarities and some dissimilarities as well with Marginal Costing. A comparative analysis between Differential Cost Analysis and Marginal Costing is done as follows: Similarities i. Both Differential Costing as well as Marginal Costing are techniques of cost analysis. ii. Both the techniques are used by the management in its process of Decision Making. iii. Both the techniques are based on the classification of costs into Fixed Cost and Variable Cost. When there is no change in the Fixed Cost due to a change in the volume of production or sales, Differential Cost becomes equal to the Marginal Cost. Dissimilarities i. In Marginal Costing, Profit–Volume Ratio (P/V Ratio), contribution margin, contribution per unit of limiting factor and so on are the main parameters for evaluation of performance and Decision Making. On the other hand, in the Differential Cost Analysis, Differential Costs are compared with differential revenues to select the best alternative in the process of Decision Making. ii. Marginal Costing excludes all Fixed Costs from its analysis, whereas Differential Cost Analysis includes all identifiable Fixed Costs. iii. Marginal Cost may be incorporated into the system of accounting, whereas Differential Costs are separately calculated as an aid to the management for a Decision Making purpose. 11.6 CONCEPT OF KEY OR LIMITING FACTOR A key or limiting factor is a factor which limits or restricts the production or sales level at a point of time due to scarcity of a factor of production. That is why the key or limiting factor is also termed as ‘scarce factor’ or ‘governing factor.’ According to CIMA, United Kingdom, key factor is the factor which, at a particular time or over a period, will limit the activities of an undertaking. The limiting factor is usually the level of demand for the product or services of the undertaking, but it could be a shortage of one of the productive resources, e.g. skilled labour, raw material or machine capacity. Therefore, key factor is a limitation of constraint, such as availability of raw material, labour, machine/ plant capacity, capital, market demand and so on, on the activities of a business. Where a concern produces two or more products, and there exists a key or limiting factor, the decision must be taken by the management as to which product should be produced more so as to utilize the limiting factor in the best possible way for maximizing the profit for the concern. In such a situation, the contribution per unit of the limiting factor should be the criterion to assess the profitability of a product; and accordingly, all the products of the concern are to be taken into consideration for the purpose. The product which gives the highest contribution per unit of the limiting factor should be produced first and the product which gives the next highest contribution per unit of the limiting factor should be produced thereafter, if the balance of the limiting factor permits, and so on.

Modified Date: Sat, Jun 26, 2010 03:08:44 PM

Output Date: Tue, Jul 06, 2010 12:30:28 PM

Rev-II

Project: Management Accounting_Debarshi Bhattacharyya ACE Pro India Pvt. Ltd. File: X:\Pearson\Management Accounting_Debarshi Bhattacharyya\MAIN\M11\LAYOUT_M11\M11_DEBA_ISBN_EN_SE_C11.indd

706

MANAGEMENT ACCOUNTING

Illustration Say, a company produces three products X, Y and Z. The company has a limiting factor of 10,000 labour hours. In such a situation, the company has calculated contribution per labour hour (i.e., per unit of limiting factor) as Rs. 20 for X, Rs. 50 for Y and Rs. 30 for Z. As product Y gives the highest contribution per unit of limiting factor (i.e. labour hour), the company firstly produces 10,000 units of Y using 5,000 labour hours. Thereafter, the company produces 6,000 units of Z using 4,000 labour hours, as product Z gives the next best contribution per unit of the limiting factor. The company lastly produces product X, as it gives the lowest contribution per unit of the limiting factor, using the remaining available 1,000 labour hours (i.e., 10,000− 5,000−4,000 = 1,000 labour hours). Say, per unit of product X requires 2 labour hours. Hence, in such a case, the number of units of product X to be produced = 1,000 hrs/2 hrs = 500 units. 11.7 DIFFERENT SITUATIONS OF DECISION MAKING Decision Making is an action of selecting the best one from different alternatives. Therefore, all decisions are alternate-choice decisions. There are many situations in a business when the management is required to choose the best one from different alternatives. Following are some frequently applicable situations of Decision Making. 11.7.1 Determination of Most Profitable Product/Sales Mix Product/sales mix signifies the mix or proportion in which various products are produced/sold. Most profitable product/sales mix signifies the best mix or proportion in which various products are produced/sold for maximizing the profit. In case where a concern produces and sells two or more products simultaneously, it needs to choose the most profitable product/sales mix among it products for maximizing the profit. Where there is no change in the Fixed Cost due to a change in the product/sales mix, the selection of product/sales mix is done on the basis of contribution per unit of each product. The product having the highest contribution should be produced/sold first (i.e., to be given the first priority) and the product having the lowest contribution should be produced/sold last (i.e., to be given the least priority). The product giving negative contribution should be discontinued unless there are other reasons to continue its production. Where there is a change in the Fixed Cost due to a change in the product/sales mix, selection of product/sales mix is done on the basis of the highest profitability from the mix, that is, the product/sales mix which provides the highest profit should be chosen as the most profitable product/sales mix. 11.7.2 Accept or Reject Decision Whenever a concern receives special orders for supply of its product from its regular or new customers in the home market or in the foreign market, the concern should consider whether acceptance of such offers utilizing its spare production capacity is profitable or not. Generally, such special orders are offered at a price less than the normal selling price. Acceptance or rejection of such special orders is based entirely on the profitability of the concern. As the products against the special order are to be produced utilizing the spare production capacity of the concern, no extra Fixed Cost is normally to be borne by the concern for such special order. Accordingly, if the special order price is more than the Marginal Cost of the product, the concern should accept the special order. In such a case, the contribution represents the profit from the special order. However, any extra cost for the special order, such as special packing cost, freight and insurance charges and so on, is to be deducted from the contribution to arrive at the net profit from the special order. 11.7.3 Make or Buy Decision A concern may manufacture any component for its product by itself or it may buy the component from an outside supplier. Decision to make or buy the component is based entirely on saving in cost by the concern. This saving in cost refers to the saving in the Marginal Cost, as no saving can generally be effected in the Fixed Cost.

Modified Date: Sat, Jun 26, 2010 03:08:44 PM

Output Date: Tue, Jul 06, 2010 12:30:28 PM

Rev-II

Project: Management Accounting_Debarshi Bhattacharyya ACE Pro India Pvt. Ltd. File: X:\Pearson\Management Accounting_Debarshi Bhattacharyya\MAIN\M11\LAYOUT_M11\M11_DEBA_ISBN_EN_SE_C11.indd

DECISION MAKING

707

If the Marginal Cost of producing the component is lower than the selling price of the component offered by the outside supplier, it should be decided to buy the component from the outside supplier. Otherwise, the concern should opt for making the component by itself, if it has the spare production capacity. 11.7.4 Operate or Shut-Down Decision In such a situation, the concern has to take a decision whether it will continue its plant operation in future or it will shut down its plant operation right then. If it is observed that the concern will suffer lesser amount of loss if it shuts down the plant operation right then than the amount of loss that it will suffer if it continues the plant operation, then the decision should be taken in favour of shut-down of the plant operation to minimize the loss. In such a circumstance, the Fixed Costs play the most significant role as a concern has to bear some Fixed Cost even if it shuts down the plant operation. Guidance Notes to Students for Solving Problems Students are advised to go through the language of the problem very carefully and identify the purpose of Decision Making with the key or limiting factor, if any.

11.8 WORKED-OUT PROBLEMS Problem 1 The Income Statement of a company for the year that ended on 31 March 2010 reveals the following: Rs. 36,00,000 22,00,000 10,50,000 3,50,000

Sales @ Rs. 90 per unit Variable Costs Fixed Cost Net Profit

The Board of Directors are dissatisfied with the result and plans. Implementation of a new marketing programme would increase the Fixed Costs by Rs. 1,92,000 and Variable Costs by Rs. 10 per unit and increase the selling price by 5%. This will increase the sales units by 15%. You are required to calculate new and old break-even units and also evaluate whether the proposed change is acceptable or not. Solution Comparative Analysis of the proposal A. Sales in units B. Selling Price per unit (Rs.) C. Sales Value [A × B] (Rs.) D. Variable Cost per unit (Rs.) E. Total Variable Cost [A × D] (Rs.) F. Contribution [B − E](Rs.) G. Fixed Cost (Rs.)

Existing Rs. 36,00,000/Rs. 90

40,000 + 15%

Proposed

= 40,000 units 90.00 36,00,000 Rs. 22,00,000 ÷ 40,000 = 55.00 22,00,000 14,00,000

= 46,000 units 90 + 5% = 94.50 43,47,000 55 + 10 = 65.00 29,90,000 13,57,000

10,50,000

10,50,000 + 1,92,000 = 12,42,000 (Continued)

Modified Date: Sat, Jun 26, 2010 03:08:44 PM

Output Date: Tue, Jul 06, 2010 12:30:28 PM

Rev-II

Project: Management Accounting_Debarshi Bhattacharyya ACE Pro India Pvt. Ltd. File: X:\Pearson\Management Accounting_Debarshi Bhattacharyya\MAIN\M11\LAYOUT_M11\M11_DEBA_ISBN_EN_SE_C11.indd

708

MANAGEMENT ACCOUNTING

Existing 3,50,000 35 10,50,000/35 = 30,000 units

∴ Net Profit [F − G] (Rs.) Contribution per unit [B − D] (Rs.) ∴ Break-Even Point in units [Fixed Cost/Contribution per unit]

Proposed 1,15,000 29.50 12,42,000/29.50 = 42,102 units

Above analysis reveals that the proposal is not acceptable as it gives lower profit than the existing situation. Problem 2 Quality Product Ltd has drawn up the following budget for the year 1998–99: Rs. 20,00,000 6,00,000 7,00,000 4,00,000 3,00,000 40,00,000 50,00,000 10,00,000

Raw Material Labour, Store, Power and other Variable Costs Fixed Manufacturing Overheads Packing and Variable Distribution Cost Fixed General Overheads Including Selling Sales Revenue @ Rs. 50 per unit Budgeted Profit

The General Manager suggests to reduce the selling price by 5% and expects to achieve an additional volume of 5%. The more intensive manufacturing programme will involve additional costs of Rs. 50,000 for production planning. It will also be necessary to open an additional sales office at the cost of Rs. 1,00,000 per annum. The Sales Manager, on the other hand, suggests to increase the selling price by 10% which, it is estimated, will reduce the sales volume by 10%. At the same time, a saving in the manufacturing overheads and general overheads of Rs. 50,000 and Rs. 1,00,000 per annum, respectively, is expected on this reduced volume. Which of these two proposals would you accept and why? Show the complete working. [B.Com. (Hons), Delhi University—1998] Solution Comparative Analysis of various proposals Budget for 1998–99 A. Budgeted Sales Quantity (units) [Rs.50,00,000/Rs. 50] [1,00,000 + 5%] [1,00,000 − 10%] B. Selling Price per unit (Rs.) [Rs.50 − 5%] & [Rs.50 + 10%] C. Raw Materials per unit (Rs.) [Rs.20,00,000/1,00,000 units] D. Labour, Store, Power and Other Variable Costs per unit (Rs.) [Rs.6,00,000/1,00,000 units] E. Packing and Variable Distribution Cost per unit (Rs.) [Rs.4,00,000/1,00,000 units]

1,00,000

Proposal of General Manager 1,05,000

Proposal of Sales Manager 90,000

50

47.50

55

20

20

20

6

6

6

4

4

4 (Continued)

Modified Date: Sat, Jun 26, 2010 03:08:44 PM

Output Date: Tue, Jul 06, 2010 12:30:28 PM

Rev-II

Project: Management Accounting_Debarshi Bhattacharyya ACE Pro India Pvt. Ltd. File: X:\Pearson\Management Accounting_Debarshi Bhattacharyya\MAIN\M11\LAYOUT_M11\M11_DEBA_ISBN_EN_SE_C11.indd

DECISION MAKING

Rs. 50,00,000

Proposal of General Manager Rs. 49,87,500

Proposal of Sales Manager Rs. 49,50,000

20,00,000 6,00,000 4,00,000 30,00,000 20,00,000

21,00,000 6,30,000 4,20,000 31,50,000 18,37,500

18,00,000 5,40,000 3,60,000 27,00,000 22,50,000

7,00,000 3,00,000 10,00,000 10,00,000

7,50,000 4,00,000 11,50,000 6,87,500

6,50,000 2,00,000 8,50,000 14,00,000

Budget for 1998–99 F. Sales [A × B] G. Variable Cost: Raw Materials (A × C) Labour, Store, Power and other Variable Costs (A × D) Packing and Variable Distribution Cost (A × E) H. Contribution (F − G) I. Fixed Cost: Fixed Manufacturing Overheads Fixed General Overheads including Selling ∴ Profit (H − I)

709

CONCLUSION From the above comparative analysis of different proposals, it has been observed that the highest profit (i.e., Rs.14,00,000) can be achieved if the proposal of the Sales Manager (i.e., by increasing the selling price by 10% and reducing the volume of sales by 10%) is accepted. Therefore, the proposal of Sales Manager should be accepted. Problem 3 Mr X has Rs. 2,00,000 as the investment in his business firm. He wants a 15% return on his money. From an analysis of the recent cost figures, he finds that his Variable Cost of operating is 60% of the sales, while his Fixed Costs are Rs. 80,000 per year. Show the computation to answer the following questions: i. What sales volume must be obtained to reach the Break-Even Point (BEP)? ii. What sales volume must be obtained to get a 15% return on the investment? iii. Mr X estimates that even if he closed the doors of his business, he would incur Rs. 25,000 as expenses per year. At what sales would he be better off by locking his business up? [C.A. (Inter)—Adapted] Solution i. Let the Selling Price (S) be 100. Then, the Variable Cost (V) = 60% of 100 ∴ Contribution (C) = S − V = 100 − 60 ∴ P/V Ratio = C/S × 100 = 40/100 × 100 Given Fixed Cost (F) ∴ BEP Sales = F/(P/V Ratio) × 100 = 80000/40 × 100 ii. Desired Profit (P) = 15% on Rs. 2,00,000 Now, P/V Ratio = {(F + P)/S} × 100 Here, 40 = {(80,000 + 30,000)/S} × 100 or 40 S = 1,10,000 × 100 ∴ S = (1,10,000 × 100)/40 ∴ Required Sales

Modified Date: Sat, Jun 26, 2010 03:08:44 PM

= 60 = 40 = 40% = Rs. 80,000 = Rs. 2,00,000 = Rs. 30,000

= 2,75,000 = Rs. 2,75,000

Output Date: Tue, Jul 06, 2010 12:30:28 PM

Rev-II

Project: Management Accounting_Debarshi Bhattacharyya ACE Pro India Pvt. Ltd. File: X:\Pearson\Management Accounting_Debarshi Bhattacharyya\MAIN\M11\LAYOUT_M11\M11_DEBA_ISBN_EN_SE_C11.indd

710

MANAGEMENT ACCOUNTING

iii. Fixed Cost to be incurred even if Mr X shuts down his business = Rs. 25,000. To cover this minimum amount of Fixed Cost, the minimum sales needed by Mr X = F/(P/V Ratio) × 100 = 25,000/40 × 100 = Rs. 62,500. ∴ Mr X would be better off by locking his business up at any sales volume more than Rs. 62,500. Problem 4 Alaska Paints, working at a normal capacity, manufactures 4,00,000 tins per year. The cost of manufacturing per tin is as follows: Rs. 15.60 4.20 5.00 8.00 32.80

Consumption of Materials Direct Wages Variable Factory Overheads Fixed Overheads

Variable selling and administrative expenses amount to Rs. 1.25 per tin. Each tin is sold for Rs. 45.00. During the next quarter, only 20,000 tins can be sold. The management plans to shut down the plant estimating that the manufacturing Fixed Costs can be reduced by Rs.1,48,000 for the quarter. While the plant is in operation, fixed overheads are incurred at a uniform rate. You are required to suggest: (a) whether the plant should be shut down at this level; and (b) at what level of activity per quarter, the plant should be shut down. Solution Rs. Variable Cost per tin: Raw Materials Direct Wages Variable Factory Overheads Variable Selling & Administrative Expenses Selling Price per tin Contribution per tin Fixed Overheads for the year (4,00,000 × Rs. 8) ∴ Fixed Overheads for the quarter (Rs. 32,00,000/4) Now, during the next quarter when 2,000 tins can be sold: Contribution from 20,000 tins (20,000 × Rs.18.95) Less: Fixed Overheads for the quarter Possible Loss

15.60 4.20 5.00 1.25 26.05 45.00 18.95 32,00,000 8,00,000 Rs. 3,79,000 8,00,000 4,21,000

i. If the plant operation is shut down, the company has to bear a fixed overhead cost of (Rs. 8,00,000 – 1,48,000) Rs. 6,52,000. Therefore, the shut-down of the plant will result is higher loss than the production of 20,000 units during the next quarter. ii. As the shut-down-point loss is calculated at Rs. 6,52,000, it is advised to the company to down its plant operation when the quarterly contribution falls below (Rs. 8,00,000 – Rs. 6,52,000) Rs. 1,48,000. ∴ The suggested shut-down point of the plant is at a production of (Rs. 1,48,000 ÷ Rs. 18.95) 7,810 tins per quarter.

Modified Date: Sat, Jun 26, 2010 03:08:44 PM

Output Date: Tue, Jul 06, 2010 12:30:28 PM

Rev-II

Project: Management Accounting_Debarshi Bhattacharyya ACE Pro India Pvt. Ltd. File: X:\Pearson\Management Accounting_Debarshi Bhattacharyya\MAIN\M11\LAYOUT_M11\M11_DEBA_ISBN_EN_SE_C11.indd

711

DECISION MAKING

Problem 5 Your company produces two products P and Q. The relevant data per unit of output are given as follows: P (Rs.) 28.00 15.00 25.00 10.00 14.00 92.00 100.00 8.00

Cost of Direct Material Direct Labour Variable Factory Overheads Fixed Factory Overheads Variable Selling Expenses Total Cost Selling Price Profit

Q (Rs.) 13.00 25.00 12.50 5.00 10.00 65.50 70.00 4.50

Factory overheads are applied on the basis of machine hour. The existing plant and infrastructure will allow production and sale of either P or Q. Both the products are processed through the same production centre. You are required to suggest which product should be processed and sold. Solution Product P Rs. Selling Price Less: Variable Costs: Direct Material Direct Labour Variable Overheads Selling Expenses Contribution per unit Variable Factory Overheads per unit Fixed Factory Overheads per unit Total Factory Overheads per unit

28.00 15.00 25.00 14.00

Rs. 100.00

82.00 18.00 25.00 10.00 35.00

Product Q Rs. Rs. 70.00 13.00 25.00 12.50 10.00

60.50 9.50 12.50 5.00 17.50

Since factory overheads are applied on the basis of machine hours and machine hours are indicative of available infrastructure, as they constitute the key or limiting factor for the products. From the above calculation, it has been observed that the total factory overheads per unit for Product P is nearly double than that of Product Q, This indicates that Product Q requires half the machine hours when compared to Product P (or Product P requires double the machine hours when compared to Product Q). Therefore, the rate of contribution per machine hour from Product P = Rs. 18 × 1 = Rs. 18 and from Product Q = Rs. 9.50 × 2 = Rs. 19 . Therefore, Product Q should be processed and sold as it gives a higher contribution per machine hour than the Product P. Problem 6 Product X takes 20 hours to process on Machine 99. It has a selling price of Rs. 100 and Marginal Cost of Rs. 60. Y (a component part used in production) could be made on Machine 99 in 3 hours for a Marginal Cost of Rs. 5. The supplier’s price is Rs. 10. Should one make or buy Y? Discuss in both situations (i) when Machine 99 is working at full capacity; and (ii) when there is idle capacity. What are other non-cost considerations to be kept in mind in such cases? [B.Com. (Hons.), Delhi University]

Modified Date: Sat, Jun 26, 2010 03:08:44 PM

Output Date: Tue, Jul 06, 2010 12:30:28 PM

Rev-II

Project: Management Accounting_Debarshi Bhattacharyya ACE Pro India Pvt. Ltd. File: X:\Pearson\Management Accounting_Debarshi Bhattacharyya\MAIN\M11\LAYOUT_M11\M11_DEBA_ISBN_EN_SE_C11.indd

712

MANAGEMENT ACCOUNTING

Solution (i) Situation when Machine 99 is working at full capacity When Machine 99 is working at full capacity, the production of Product X is to be curtailed to some extent to produce Component Part Y, as a consequence of which the manufacturer has to bear some loss of contribution from Product X. Now, in the case of production of Product X, Selling Price per unit = Rs. 100 Less: Marginal Cost per unit = Rs. 60 Contribution per unit = Rs. 40 Again, Product X takes 20 hours to produce one unit, that is, time taken to produce 1 unit of Product X = 20 hrs. ∴ Contribution per hour from Product X = Rs. 40/20 hrs = Rs. 2 Now, if Component Part Y is produced in this situation, then Additional cost per unit to be incurred to produce Y = Marginal Cost per unit + Contribution Lost per unit from X = Rs. 5 + (3 hrs × Rs. 2) = Rs. 11 ∴ Additional cost per unit to be incurred for producing Component Y when Machine 99 is working at full capacity = Rs. 11 On the other hand, Component Y can be bought from the market at a cost of Rs. 10 per unit. ∴ One should buy Component part Y as it is cheaper than making when Machine 99 is working at full capacity. (ii) Situation when Machine 99 has idle capacity When Machine 99 remains idle after production of Product X, the question of curtailing the production of Product X for production of Component Part Y does not arise. As a consequence, there will be no loss of contribution from Product X due to production of Component Part Y. In such a situation, the manufacturer has to bear the Marginal Cost only for the production of Component Part Y. As the Marginal Cost per unit for the production of Component Part Y (i.e., Rs. 5) is lower than the supplier’s price (i.e., Rs. 10), it is profitable to make the Component Part Y than to buy it. Other non-cost considerations: i. Expected future market price of Component Y. ii. Availability of Y in the market. iii. Assurance of continuous supply of Y, if it is bought from outside. iv. Assurance of no change in price during the period of agreement, if it is bought from outside. Problem 7 Two firms A & Co. and B & Co. sell the same type of product in the same market. Their budgeted Profit & Loss account (Profit & Loss A/c) for the year that ends on 31 March 1996 is as follows: A & Co. Rs. Sales Less: Variable Cost Fixed Cost

Rs. 5,00,000

4,00,000 30,000

Modified Date: Sat, Jun 26, 2010 03:08:44 PM

Output Date: Tue, Jul 06, 2010 12:30:28 PM

Rs. 6,00,000

4,00,000 70,000 4,30,000 70,000

Net Profit

B & Co. Rs.

4,70,000 1,30,000

Rev-II

Project: Management Accounting_Debarshi Bhattacharyya ACE Pro India Pvt. Ltd. File: X:\Pearson\Management Accounting_Debarshi Bhattacharyya\MAIN\M11\LAYOUT_M11\M11_DEBA_ISBN_EN_SE_C11.indd

713

DECISION MAKING

Required:

i. Calculate at which sales both the firms will earn an equal profit. ii. State which firm is likely to earn greater profit in condition of: a. Heavy demand for the product. b. Low demand for the product. [C.A. (Inter)—November1995] Solution

Less:

Sales Variable Cost

Less:

Fixed Cost

Contribution Profit ∴ P/V Ratio [Contribution/Sales × 100] ∴ BEP Sales [Fixed Cost/P/V ratio × 100]

A & Co. Rs. 5,00,000 4,00,000 1,00,000 30,000 70,000 1,00,000/5,00,000 × 100 = 20% 30,000/20 × 100 = Rs. 1,50,000

B & Co. Rs. 6,00,000 4,00,000 2,00,000 70,000 1,30,000 2,00,000/6,00,000 × 100 = 33.33% 70,000/33.33 × 100 = Rs. 2,10,000

(1) We know that sales at which both the firms = (Difference in Fixed Cost/Difference in P/V Ratio) × 100 earn equal profit = [(Rs. 70,000 − Rs. 30,000)/(33.33 − 20)] × 100 = 40,000/13.33 × 100 = Rs. 3,00,000 (2) i. In case of heavy demand, that company is more profitable which has higher P/V ratio. As there is a heavy demand for the product in the market, BEP sales can be achieved easily. After achieving BEP sales, the total amount of contribution earned is the profit (as at the Margin of safety, Contribution = Profit). Therefore, after achieving BEP sales that company will earn a greater profit whose P/V ratio is higher. Hence, B & Co. will earn a greater profit in case of heavy demand as its P/V ratio is higher than that of A & Co. ii. In case of low demand, that company is more profitable whose BEP is lower. As the demand for the product in the market is very low, the company should try first not to incur any loss. Therefore, the company should try to achieve BEP as soon as possible to avoid any loss. The company having lower BEP will avoid loss at a faster rate than the other company whose BEP is higher. Hence, in this case, A & Co. is more profitable as its BEP is lower than that of B & Co. Problem 8 The cost per unit of the three products A, B and C of a concern are as follows: Particulars Variable Cost Fixed Cost Total Cost Profit Selling Price No. of units Produced

A (Rs.) 20 3 23 9 32 10,000

B (Rs.) 20 3 23 7 30 5,000

C (Rs.) 18 2 20 6 26 8,000

Production arrangements are that if one product is given up, the production of the others can be raised by 50%. The directors propose that C should be given up because the contribution in that case is the lowest. Do you agree? What other non-cost considerations should be kept in mind before taking any decision in such a situation? [B.Com. (Hons), Delhi University—1995]

Modified Date: Sat, Jun 26, 2010 03:08:44 PM

Output Date: Tue, Jul 06, 2010 12:30:28 PM

Rev-II

Project: Management Accounting_Debarshi Bhattacharyya ACE Pro India Pvt. Ltd. File: X:\Pearson\Management Accounting_Debarshi Bhattacharyya\MAIN\M11\LAYOUT_M11\M11_DEBA_ISBN_EN_SE_C11.indd

714

MANAGEMENT ACCOUNTING

Solution Statement showing computation of present profit Sales (units) Sales Value Less: Variable Cost Contribution Less: Fixed Cost Profit

Product A 10,000 Rs. 3,20,000 2,00,000 1,20,000 30,000 90,000

Product B 8,000 Rs. 1,50,000 1,00,000 50,000 15,000 35,000

Product C 5,000 Rs. 2,08,000 1,44,000 64,000 16,000 48,000

Total 23,000 Rs. 6,78,000 4,44,000 2,34,000 61,000 1,73,000

Statement showing computation of profit if Product A is given up Sales [Present + 50%] (units)

Less:

Contribution B – 7,500 × (Rs. 30 − Rs. 20) C – 12,000 × (Rs. 26 − Rs.18) Fixed Cost Profit

Product B 7,500 Rs. 75,000

Product C 12,000 Rs. 96,000

Total 19,500 Rs. 1,71,000

61,000 1,10,000

Statement showing computation of profit if Product B is given up Sales [Present + 50%] (units)

Less:

Contribution A – 15,000 × (Rs. 32 − Rs. 20) C – 12,000 × (Rs. 26 − Rs. 18) Fixed Cost Profit

Product A 15,000 Rs. 1,80,000

Product C 12,000 Rs. 96,000

Total 37,000 Rs. 2,76,000

61,000 2,15,000

Statement showing computation of profit if Product C is given up Sales [Present + 50%] (units)

Less:

Contribution A – 15,000 × (Rs. 32 − Rs. 20) B – 7,500 × (Rs. 30 − Rs. 20) Fixed Cost Profit

Product A 15,000 Rs. 1,80,000

Product B 7,500 Rs. 75,000

Total 22,500 Rs. 2,55,000

61,000 1,94,000

CONCLUSION The above calculation shows that the overall profit is maximum (i.e., Rs. 2,15,000) when Product B is given up. Therefore, the proposal of directors to give up the production of Product B is not acceptable as it does not yield the maximum overall profit. Non-cost considerations: i. Expected market demand for the Products A, B and C in future. ii. Expected future selling prices of the products. iii. Evaluation of the fact whether the Products A, B and C are complementary to each other in the market. iv. Position of similar competitive products in the market.

Modified Date: Sat, Jun 26, 2010 03:08:44 PM

Output Date: Tue, Jul 06, 2010 12:30:28 PM

Rev-II

Project: Management Accounting_Debarshi Bhattacharyya ACE Pro India Pvt. Ltd. File: X:\Pearson\Management Accounting_Debarshi Bhattacharyya\MAIN\M11\LAYOUT_M11\M11_DEBA_ISBN_EN_SE_C11.indd

715

DECISION MAKING

Problem 9 A company is operating at 50% capacity producing 2,000 units of output which are totally sold in the domestic market at a price of Rs. 100 per unit. The cost per unit of the product is given as follows: Direct materials Direct Wages Manufacturing Overheads (20% fixed) Administration Overheads (100% fixed) Selling and Distribution Overheads (50% variable) Total Cost

Rs. Per Unit 30 20 10 8 12 80

The company receives an order from the foreign market for 2000 units at a price of Rs. 80 per unit. The additional distribution cost for export is estimated at Rs. 2 per unit. Is the offer of the foreign market accepted by utlizing the spare capacity? Solution The company is presently operating at 50% capacity producing 2000 units for the domestic market. ∴ The company still has the remaining 50% spare capacity to produce additional 2,000 units for the foreign market. If the company accepts the offer of the foreign market, the position will be as follows: Rs. Selling Price per unit Less: Variable Costs per unit: Direct Materials Direct Wages Variable Manufacturing Overheads (80% of Rs. 10) Variable Selling and Distribution Overheads [(50% of Rs. 12) + Rs. 2] Contribution per unit

30 20 8 8

Rs. 80

66 14

If the offer of the foreign market is accepted, then the company earns an additional contribution of Rs. 14 per unit. Currently, the company has been earning a profit of (Rs. 100 – Rs. 80) = Rs. 20 per unit. As at the present situation, the company has an already earned profit, it indicates that the BEP sale has already been achieved by it. As soon as the BEP sale is achieved, the total Fixed Cost and the Variable Cost up to that level of sales have already been covered. Now, if the sales are increased beyond BEP, no additional Fixed Cost is to be incurred, rather only Variable Costs are to be incurred. Therefore, after achieving BEP at any point of sales, Contribution = Profit. Here, as BEP has already been achieved by the company, no additional Fixed Cost to be incurred for producing additional 2000 units for the foreign market and the contribution would be equal to the additional profit. Therefore, by utilizing the remaining 50% spare capacity, if the company produces the additional 2000 units for the foreign market, it will earn an additional profit of (2000 units ∞ Rs. 14) Rs. 28,000. Hence, the offer of the foreign market is acceptable. Problem 10 An umbrella manufacturer makes an average profit of Rs. 2.50 per unit on a selling price of Rs. 14.30 by producing and selling 60,000 units at 60% of the potential capacity.

Modified Date: Sat, Jun 26, 2010 03:08:44 PM

Output Date: Tue, Jul 06, 2010 12:30:28 PM

Rev-II

Project: Management Accounting_Debarshi Bhattacharyya ACE Pro India Pvt. Ltd. File: X:\Pearson\Management Accounting_Debarshi Bhattacharyya\MAIN\M11\LAYOUT_M11\M11_DEBA_ISBN_EN_SE_C11.indd

716

MANAGEMENT ACCOUNTING

His cost of sales per unit is as follows: Rs. 3.50 1.25 6.25 (50% fixed) 0.80 (25% variable)

Direct Material Direct Wages Factory Overheads Sales Overheads

During the current year, he intends to produce the same number but estimates that his Fixed Cost would go up by 10% while the rates of direct wages and direct materials will increase by 8% and 6%, respectively. However, the selling price cannot be changed. Under this situation, he obtains an offer for a further 20% of his potential capacity. What minimum price would you recommend for acceptance of the offer to ensure the manufacturer with an overall profit of Rs. 1,67,300? [B.Com. (Hons), Delhi University] Solution Statement of Marginal Cost and Profit for the current year utilizing 60% of potential capacity

A. Sales B. Variable Cost: Direct Materials (Rs. 3.50 + 6%) Direct Wages (Rs. 1.25 + 8%) Variable Factory Overheads (50% of Rs. 6.25) Variable Sales Overheads (25% of Re. 0.80) C. Contribution (A − B) D. Fixed Cost1 ∴ Profit (C − D)

Per Unit Rs. 14.30 3.71 1.35 3.125 0.20 8.385 5.915

For 60,000 Units Rs. 8,58,000 2,22,600 81,000 1,87,500 12,000 5,03,100 3,54,900 2,45,850 1,09,050

Now, if the new offer is accepted for a further 20% of the potential capacity, then the additional number of units to be produced = 60,000 × 20%/60% = 20,000 units. Statement showing computation of recommended minimum price for the offer to ensure an overall profit of Rs. 1,67,300 Variable Cost to be incurred (20,000 units × Rs. 8.385) Add: Additional Profit to be earned to ensure an Overall Profit (Rs. 1,67,300 − Rs. 1,09,050) Recommended total minimum Selling Price for additional 20,000 units ∴ Recommended minimum Selling Price per unit (Rs. 2,25,950/20,000)

Rs. 1,67,700 58,250 2,25,950 11.2975

Working Notes 1. Calculation of Total Fixed Cost for the Current Year Fixed Factory Overheads [(50% of Rs. 6.25) × 60,000] Fixed Sales Overheads [(75% of Re. 0.80) × 60,000] Add:

10% in Current Year (10% of Rs. 2,23,500) Total Fixed Cost

Modified Date: Sat, Jun 26, 2010 03:08:44 PM

Output Date: Tue, Jul 06, 2010 12:30:28 PM

Rs. 1,87,500 36,000 2,23,500 22,350 2,45,850

Rev-II

Project: Management Accounting_Debarshi Bhattacharyya ACE Pro India Pvt. Ltd. File: X:\Pearson\Management Accounting_Debarshi Bhattacharyya\MAIN\M11\LAYOUT_M11\M11_DEBA_ISBN_EN_SE_C11.indd

DECISION MAKING

717

Problem 11 Diwana Ltd manufactures automobile accessories and parts. The following are the total costs and also the unit costs of processing a component, DIL-2010: Total Cost for 1,00,000 units Rs. 5,00,000 8,00,000 6,00,000 5,00,000 24,00,000

Direct Material Direct Labour Variable Factory Overheads Fixed Factory Overheads

Per unit Cost Rs. 5 8 6 5 24

Another manufacturer has offered to sell the same part to Diwana Ltd for Rs. 22 each. The fixed overheads would continue to be incurred even when the component is bought out, although there would be a reduction to the extent to Rs. 1,50,000 following the savings in the salaries of supervisory personnel that could be avoided if the company opts to ‘buy’ rather than ‘make.’ a. Should the part be made or bought considering that the present facility when released following a buying decision would remain idle? b. In case the released facility can be rented to another manufacturer for Rs. 50,000, as there is a good demand for spare facility, what would be the position? [I.C.W.A. (Final)—Adapted] Solution Comparative statement showing computation of total costs under different alternatives

Fixed Cost Less: Reduction in Fixed Cost if the component is bought from outside Add:

Variable Cost Total Cost Less: Reduction in cost due to the revenue earned for rented out spare facility Net Total Cost

Make Rs. 5,00,000

Buy and Spare Facility Idle Rs. 5,00,000

Buy and Spare Facility Rented Out Rs. 5,00,000

– 5,00,0001 19,00,000 24,00,000

1,50,000 3,50,0002 22,00,000 25,50,000

1,50,000 3,50,0002 22,00,000 25,50,000

– 24,00,000

– 25,50,000

50,000 25,00,000

From the above statement, it is observed that the Net Total Cost is the minimum when the company manufactures the component DIL-2010. Therefore, it is profitable to continue making the component DIL-2010. Working Notes 1. Total Variable Costs for production of 1,00,000 units of component DIL-2010 Rs. Variable Cost per unit: Direct Material Direct Labour Variable Factory Overheads

Modified Date: Sat, Jun 26, 2010 03:08:44 PM

5 8 6 19

Output Date: Tue, Jul 06, 2010 12:30:28 PM

Rev-II

Project: Management Accounting_Debarshi Bhattacharyya ACE Pro India Pvt. Ltd. File: X:\Pearson\Management Accounting_Debarshi Bhattacharyya\MAIN\M11\LAYOUT_M11\M11_DEBA_ISBN_EN_SE_C11.indd

718

MANAGEMENT ACCOUNTING

∴ Total Variable Cost for production of 1,00,000 units of component DIL-2010 = 1,00,000 units × Rs. 19 = Rs. 19,00,000 2. Total Variable Cost if 1,00,000 units of component DIL-2010 are bought from outside = 1,00,000 units × Rs. 22 = Rs. 22,00,000

Problem 12 Following are the budgeted information of a company for the next year’s operation: Capacity Output

2,50,000 units Rs. 23,00,000 13,80,000 5,75,000 3,45,000

Budgeted Sales (2,30,000 units) Cost of Sales Selling and Distribution Expenses Budgeted Profit

The Fixed Costs included in the cost of sales amount to Rs. 4,60,000. Variable selling and distribution expenses comprise of 10% commission on sales and Re. 0.50 per unit to be paid as the license fee to the designer of the product. A new customer has approached to buy 10,000 units of the product at a price of Rs. 7 per unit. The budgeted sales have been projected on the basis of orders already at hand and the offer is an addition to that. You are required to determine: (a) the net profit that will result if this offer is accepted; (b) the lowest price per unit that the company could quote for this offer and yet maintain the budgeted level of profit; and (c) whether it is profitable if the order is for 40,000 units from the new customer, instead of 10,000 units. Solution Rs. 4,60,000 2,30,000 6,90,000 Rs. 9,20,000 2,30,000 1,15,000 12,65,000 = Rs. 5.50

Fixed Costs included Cost of Sales Fixed Selling and Distribution Expenses (Rs. 5,75,000 − Rs. 2,30,000 − Rs. 1,15,000) Budgeted Total Fixed Cost under the existing output Variable Costs included cost of Sales (Rs. 13,80,000 − Rs. 4,60,000) Variable Selling and Distribution Expenses – Commission on Sales Variable Selling and Distribution Expenses – license fee to the designer Budgeted Total Variable Cost under the existing output ∴ Variable Cost per unit = Rs. 12,65,000/2,30,000 units

Comparative profitability statement under different situations Existing Normal output Order from new customer

Less:

2,30,000 units – Rs.

Contribution: From normal output @ (10 − 5.50) = Re. 0.50 per unit From the order of new customer @ (7 – 5.50) = Rs. 1.50 Total Contribution Fixed Costs Net Profit

Modified Date: Sat, Jun 26, 2010 03:08:44 PM

Proposal of Additional 10,000 units 2,30,000 units 10,000 units Rs.

Proposal of Additional 40,000 units 2,10,000 units 40,000 units Rs.

10,35,000

10,35,000

9,45,000

10,35,000 6,90,000 3,45,000

15,000 10,50,000 6,90,000 3,60,000

60,000 10,55,000 6,90,000 3,65,000

Output Date: Tue, Jul 06, 2010 12:30:28 PM

Rev-II

Project: Management Accounting_Debarshi Bhattacharyya ACE Pro India Pvt. Ltd. File: X:\Pearson\Management Accounting_Debarshi Bhattacharyya\MAIN\M11\LAYOUT_M11\M11_DEBA_ISBN_EN_SE_C11.indd

DECISION MAKING

719

a. The offer from the new customer for sale of 10,000 units should be accepted as it gives higher profit (i.e., Rs. 3,60,000) than the existing situation (i.e., Rs. 3,45,000). b. To maintain the budgeted level of profit of Rs. 3,45,000 with the additional sales of 10,000 units to the new customer, the company needs a total contribution of Rs. 10,35,000 (i.e., Profit of Rs. 3,45,000 + Fixed Cost of Rs. 6,90,000) from the normal output of 2,30,000 units and additional output of 10,000 units. As from the normal output of 2,30,000 units, the company has already achieved a contribution of Rs. 10,35,000, and, so, the company can sell the additional 10,000 units to the new customer after covering the Variable Cost for those 10,000 units only. Therefore, the minimum price could be quoted @ Rs. 5.50 per unit, being the Marginal Cost per unit, for those 10,000 units to the new customer for maintaining the budgeted level of profit. c. If the order from the new customer is for 40,000 units instead of 10,000 units, the company has to curtail its normal output from the existing level of 2,30,000 units to 2,10,000 units, as the full capacity of production is restricted to 2,50,000 units. In this situation, the company earns a total profit of Rs. 3,65,000, which is higher than the existing and proposed alternative situations. Therefore, if the order from the new customer is for 40,000 units instead of 10,000 units, then that would be more profitable to the company. Problem 13 Although Kaloo Ltd has the capacity to produce 16,000 units per month, it currently produces and sells only 10,000 units per month at Rs. 15 each. Following are the details of costs per unit at the current level of production: Rs. 5.00 3.00 0.75 1.50 0.25 1.00 11.50

Direct Material Direct Labour Variable Factory Overheads Fixed Factory Overheads Variable Selling Expenses Fixed Administrative Expenses

i. Should the company accept a special order of 4,000 units at Rs. 10 per unit? ii. What is the maximum price the company should be willing to pay to the outside supplier who is interested in manufacturing this product? iii. What would be the effect on the monthly contribution margin if the selling price is reduced to Rs. 14 each, resulting in a 10% increase in the sales volume? [I.C.W.A. (Final)—Adapted] Solution i. If the company desires to accept a special order of 4,000 units together with the existing production of 10,000 units, then the following additional Variable Costs are to be incurred: Direct Materials per unit Direct Labour per unit Variable Factory Overheads per unit Variable Cost of Manufacturing per unit Price per unit of the special order Contribution per unit from special order ∴ Total Contribution from 4000 units of special order (4000 units × Rs. 1.25) Less: Variable Selling Expense (4,000 units × Re. 0.25) Net Contribution from special order

Modified Date: Sat, Jun 26, 2010 03:08:44 PM

Output Date: Tue, Jul 06, 2010 12:30:28 PM

Rs. 5.00 3.00 0.75 8.75 10.00 1.25 5,000 1,000 4,000

Rev-II

Project: Management Accounting_Debarshi Bhattacharyya ACE Pro India Pvt. Ltd. File: X:\Pearson\Management Accounting_Debarshi Bhattacharyya\MAIN\M11\LAYOUT_M11\M11_DEBA_ISBN_EN_SE_C11.indd

720

MANAGEMENT ACCOUNTING

As the company has already achieved the BEP, this net contribution represents profit from the special order of 4,000 units. As there is enough spare capacity to produce the additional 4,000 units for accepting the special order (actual spare capacity available is 6,000 units.) and gives a profit of Rs. 4,000 too, the company should accept their special order. ii. As the Variable Cost of manufacturing per unit of this product is Rs. 8.75, the company should be willing to pay to the outside supplier the maximum of Rs. 8.75 per unit and not more than that. iii. Rs. Existing Selling Price per unit Existing Variable Cost per unit: Cost of Manufacture Selling Expense Existing Contribution per unit Existing Monthly Sales Volume ∴ Existing Monthly Contribution (10,000 units × Rs. 6)

8.75 0.25

Rs. 15

9 6 10,000 units Rs. 60,000

If the selling price is reduced to Rs. 14 per unit resulting in a 10% increase in the sales volume, then Rs. 14 9 5 11,000 units Rs. 55,000

Proposed Selling Price per unit Less: Variable Cost per unit Proposed Contribution per unit Proposed Monthly Sales Volume (10,000 units + 10%) ∴ Proposed Monthly Contribution (11,000 units × Rs. 5)

∴ Reduction in monthly contribution as per new proposal = Rs. 60,000 – Rs. 55,000 = Rs. 5,000 Problem 14 Normal capacity of a company is 2,40,000 machine hours, but at present the company utilizes 40% of its capacity. In this situation, the monthly performance of the company is as follows: Rs. in Lakhs 30.00 15.00 6.00 5.00 3.50 0.50

Sales Consumption of Materials Wages (including Rs. 1 lakh for security guards) Factory Overheads (60% fixed) Other Overheads (10% variable) Net Profit

The following two alternative proposals are under consideration of the company: a. The factory will utilize about 50% of the unutilized capacity by accepting the contract work, by charging Rs. 3.50 per machine hour. b. Discontinue the own production completely and lease out the entire facility on a monthly rental of Rs. 6.85 lakhs. You are required to comment on the action that is to be taken. Solution Comparative Analysis of Profitability of Various Proposals Available Capacity (hours) Utilized Capacity on own Production (hours)

Existing 2.40,000 96,000

Proposal A 2,40,000 96,000

Proposal B 2,40,000 – (Continued)

Modified Date: Sat, Jun 26, 2010 03:08:44 PM

Output Date: Tue, Jul 06, 2010 12:30:28 PM

Rev-II

Project: Management Accounting_Debarshi Bhattacharyya ACE Pro India Pvt. Ltd. File: X:\Pearson\Management Accounting_Debarshi Bhattacharyya\MAIN\M11\LAYOUT_M11\M11_DEBA_ISBN_EN_SE_C11.indd

721

DECISION MAKING

Existing –

Proposal A 72,000

Proposal B –

Rs. in lakhs 30.00 – – 30.00

Rs. in lakhs 30.00 2.52 – 32.52

Rs. in lakhs – – 6.85 6.85

15.00 5.00 2.00 0.35 22.35 7.65

15.00 5.00 2.00 0.35 22.35 10.17

– 6.85

1.00 3.00 3.15 7.15 0.50

1.00 3.00 3.15 7.15 3.02

1.00 3.00 3.15 7.15 (0.30)

Capacity to be given for the outside contract work (hours) [50% of (60 % of 2,40,000 machine hours)] Sales Revenue from outside contract (72,000 × Rs. 3.50) Rent from leasing the factory A. Total Revenue B. Variable Costs: Consumption of Materials Wages (Rs. 6 lakhs − Rs. 1 lakh) Factory Overheads (40% of Rs. 5 lakhs) Other Overheads (10% of Rs. 3.5 lakhs) C. Contribution (A − B) D. Fixed Costs: Security-staff Wages Factory Overheads (60% of Rs. 5 lakhs) Other Overheads (90% of Rs. 3.5 lakhs) Net Profit /(Net Loss) [C − D]

From the above analysis, it is transparent that Proposal A would be most profitable to the company as it gives the highest net profit among the various alternatives. Therefore, Proposal A should be accepted. Problem 15 The following data are available with respect to the coffee shop of a company for a week: BEP Profits when 2,40,000 units are sold Variable Cost to Sales Ratio

2,00,000 units Rs. 80,000 60%

Coffee shop remains open from 8 a.m. to 8 p.m. everyday. The manager is considering an extension of operations under two distinct proposals. Proposal I suggests operations till mid-night and this will increase the sales by Rs. 24,000 per week and involve additional costs on waiters, clerks and so on of Rs. 8,000 per week. The other alternative is to operate 24 hours a day. This will add to sales by Rs. 40,000 per week and involve additional costs on staff of Rs. 18,000 per week. Variable Costs (only on provisions) approximate about 40% of the sales price. You are asked to evaluate whether the shop should remain open from 8 a.m. to 8 p.m. or 8 a.m. to midnight or 24 hours a day from the view point of profitability. Solution Existing output (i.e., when the shop remains open from 8 a.m. to 8 p.m.) = 2,40,000 units Existing BEP = 2,00,000 units ∴ Existing Margin of Safety (MS) = 2,40,000 − 2,00,000 = 40,000 units We know, Contribution on MS = Profit Here, the Existing Profit = Rs. 80,000 Contribution on MS of 40,000 units = Rs. 80,000 ∴ Contribution per unit = Rs. 80,000/40,000 units = Rs. 2.00 Now, we know, BEP Sales in units = Fixed Cost/Contribution per unit Here, 2,00,000 = Fixed Cost/2

Modified Date: Sat, Jun 26, 2010 03:08:44 PM

Output Date: Tue, Jul 06, 2010 12:30:28 PM

Rev-II

Project: Management Accounting_Debarshi Bhattacharyya ACE Pro India Pvt. Ltd. File: X:\Pearson\Management Accounting_Debarshi Bhattacharyya\MAIN\M11\LAYOUT_M11\M11_DEBA_ISBN_EN_SE_C11.indd

722

MANAGEMENT ACCOUNTING

∴ Existing Fixed Cost = 2,00,000 × 2 Again, Variable Cost to Sales Ratio ∴ Contribution to Sales Ratio As the Existing Contribution per unit ∴ Existing Selling Price per unit = 100/40 × Rs. 2 ∴ Existing Sales = 2,40,000 units × Rs. 5

= Rs. 4,00,000 = 60% = 1 − 60% = 40% = Rs. 2 = Rs. 5 = Rs. 12,00,000

Comparative analysis of profitability of alternative proposals Operating Time

8 a.m. to 8 p.m. Rs. 12,00,000 7,20,000 4,80,000 4,00,000 80,000

Sales Less: Variable Cost @ 60% of Sales Contribution Less: Fixed Cost Profit

8 a.m. to Midnight Rs. 12,24,000 7,34,400 4,89,600 4,08,000 81,600

24 Hours Rs. 12,40,000 7,44,000 4,96,000 4,18,000 78,000

From the above analysis, it can be concluded that the shop should be open from 8 a.m. to midnight as it gives the highest profit among the different alternatives. Problem 16 A company produces and sells two products A and B. The company incurs Rs. 1,00,000 per annum towards fixed overheads and has provided the following further information: Product A Rs. per Unit 200 100 100 520

Direct Materials @ Rs. 50 per kg Direct Wages @ Rs. 10 per hour Variable Overheads Selling Price

Product B Rs. per Unit 250 150 150 715

Depending on the availability of raw materials and labour hours, the company considers the following two alternative sales mixes: a. 1,000 units of A and 600 units of B. b. 600 units of A and 1,000 units of B. Recommend which of the sales mixes should be adopted by the company. [B.Com. (Hons), Calcutta University—2006] Solution Comparative Statement of two sales mixes

Sales Less: Variable Cost: Direct Materials Direct Wages Variety Overheads Contribution Less: Fixed Cost Profit

Modified Date: Sat, Jun 26, 2010 03:08:44 PM

Sales Mix (a) (i.e., 1,000 units of A & 600 units of B) Product A Product B Total (Rs.) (Rs.) (Rs.) 5,20,000 4,29,000 9,49,000

Sales Mix (b) (i.e., 600 units of A & 1,000 units of B) Product A Product B Total (Rs.) (Rs.) (Rs.) 3,12,000 7,15,000 10,27,000

(2,00,000) (1,00,000) (1,00,000) 1,20,000

(1,20,000) (60,000) (60,000) 72,000

(1,50,000) (90,000) (90,000) 99,000

(3,50,000) (1,90,000) (1,90,000) 2,19,000 1,00,000 1,19,000

Output Date: Tue, Jul 06, 2010 12:30:28 PM

(2,50,000) (1,50,000) (1,50,000) 1,65,000

(3,70,000) (2,10,000) (2,10,000) 2,37,000 1,00,000 1,37,000

Rev-II

Project: Management Accounting_Debarshi Bhattacharyya ACE Pro India Pvt. Ltd. File: X:\Pearson\Management Accounting_Debarshi Bhattacharyya\MAIN\M11\LAYOUT_M11\M11_DEBA_ISBN_EN_SE_C11.indd

723

DECISION MAKING

From the above statement, it has been observed that the profit from sales mix (b) is higher than that from the sales mix (a). Therefore, it is recommended to the company to adopt Sales Mix (b) (i.e., 600 units of Product A and 1,000 units of Product B) as it is giving a higher net profit to the company. Problem 17 X Ltd manufactures three products in respect of which the following information is available: Product Cost of Materials per unit Direct Skilled Labour per unit Direct Unskilled Labour per unit Variable Cost per unit Selling Price per unit

Fine Rs. 100 24 8 132 240

Superfine Rs. 16 18 16 50 140

Deluxe Rs. 124 16 40 180 200

Fixed overheads are budgeted at Rs. 6,00,000. Skilled labour is paid at the rate of Rs. 12 per hour and unskilled labour is paid at Rs. 6 per hour. The marketing manager has estimated the quantity of sales as follows at the current price. Product

Sales in Quantity 20,000 units 28,000 units 12,000 units

Fine Superfine Deluxe

The availability of skilled labour is a constraint. Only 80,000 skilled labour hours would be available. You are required to work out the optimum product mix and ascertain the profit/loss expected at that level of output. Solution Statement showing determination of ranking of contribution per labour hour Product Selling Price Less: Variable Cost per unit Contribution per unit (a) Skilled Labour hours required per unit (b) (Skilled Labour rate per unit ÷ Rate per hour) Contribution per Skilled Labour hour [(a)/(b)] (Rs.) Ranking (on the basis of the key factor of Skilled Labour hours)

Fine Rs. 240 132 108 2.00

Superfine Rs. 140 50 90 1.50

Deluxe Rs. 200 180 20 1.33

54 II

60 I

15 III

Hours per Unit 1.5 2.0

Total Hours 42,000 38,000 80,000

Statement showing computation of optimum product mix Quantity Superfine Fine Total

28,000 19,000 47,000

Statement of profitability of optimum product mix Contribution from Superfine (28,000 units × Rs. 90) Contribution from Fine (19,000 units × Rs. 108) Total Contribution Less: Fixed Overheads Total Profit

Modified Date: Sat, Jun 26, 2010 03:08:44 PM

Output Date: Tue, Jul 06, 2010 12:30:28 PM

Rs. 25,20,000 20,52,000 45,72,000 6,00,000 39,72,000

Rev-II

Project: Management Accounting_Debarshi Bhattacharyya ACE Pro India Pvt. Ltd. File: X:\Pearson\Management Accounting_Debarshi Bhattacharyya\MAIN\M11\LAYOUT_M11\M11_DEBA_ISBN_EN_SE_C11.indd

724

MANAGEMENT ACCOUNTING

Problem 18 From the following particulars, find the most profitable product mix and prepare a statement of profitability of that product mix: Units Budgeted to be Produced and Sold Selling Price per unit (Rs.) Direct Material Required per unit (kg) Direct Labour per unit (hrs) Variable Overheads (Rs.) Fixed Overheads (Rs.) Cost of Direct Material per kg (Rs.) Direct Labour hour rate (Rs.) Maximum Possible units of Sale

Product A 1,800 60 5 4 7 10 4 2 4,000

Product B 3,000 55 3 3 13 10 4 2 5,000

Product C 1,200 50 4 2 8 10 4 2 1,500

All the three products are produced from the same direct material using the same type of machines and labour. Direct labour which is the key factor is limited to 18,600 hours. [B.Com. (Hons), Delhi University] Solution Statement showing determination of ranking of contribution per labour hour A. Selling Price per unit (Rs.) B. Variable Cost per unit (Rs.): Direct Materials (Rs.) (5 kg × Rs. 4), (3 kg × Rs. 4), (4 kg × Rs. 4) Direct Labour (Rs.) (4 hrs × Rs. 2), (3 hrs × Rs. 2), (2 hrs × Rs. 2) Variable Overheads (Rs.) C. Contribution per unit (Rs.) [A − B] D. Labour hours per unit (hrs) ∴ Contribution per Labour hour [C/D] (Rs.) Ranking (on the basis of the key factor of labour hours)

Product A 60

Product B 55

20 8 7 35 25 4 6.25 III

Product C 50

12 6 13 31 24 3 8 II

16 4 8 28 22 2 11 I

Statement showing computation of most profitable product mix Product C (1,500 units × 2 hrs) Product B (5,000 units × 3 hrs) Product A [{18,600 hrs − (3,000 hrs + 15,000 hrs)}/4 hrs] Total

Labour Hours 3,000 15,000 Balance 600 18,600

Units 1,500 5,000 150 6,650

Statement of profitability of most profitable product mix Rs. Contribution from Product C (1,500 units × Rs. 22) Contribution from Product B (5,000 units × Rs. 24) Contribution from Product A (150 units × Rs. 25) Total Contribution Less: Fixed Cost: Product C (1,200 units × Rs. 10) Product B (3,000 units × Rs. 10) Product A (1,800 units × Rs. 10) Total Profit

Modified Date: Sat, Jun 26, 2010 03:08:44 PM

Output Date: Tue, Jul 06, 2010 12:30:28 PM

Rs. 33,000 1,20,000 3,750 1,56,750

12,000 30,000 18,000 60,000 96,750

Rev-II

Project: Management Accounting_Debarshi Bhattacharyya ACE Pro India Pvt. Ltd. File: X:\Pearson\Management Accounting_Debarshi Bhattacharyya\MAIN\M11\LAYOUT_M11\M11_DEBA_ISBN_EN_SE_C11.indd

DECISION MAKING

725

Problem 19 ABC Ltd, which produces three products, furnishes the following data for the year 1998: Alfa 100 10% 40,000 50%

Selling Price per unit (Rs.) P/V Ratio Maximum Sales Potential (units) Raw Material as a % of Variable Cost

Products Beta 75 20% 25,000 50%

Gama 50 40% 10,000 50%

The company uses the same raw material for all the three products. Raw material is in short supply and the company has a quota for supply of raw material of the value of Rs. 18,00,000 for the year 1998 for manufacture of its products to meet its sales. The total Fixed Cost is Rs. 6,80,000. You are required to: i. Determine a sales mix which will give the maximum overall profit keeping in view the short supply of raw material. ii. Compute the maximum profit. [B.Com. (Hons), Delhi University—1999]

Solution Statement showing computation of comparative contribution against per unit of raw material cost A. Selling Price per unit (Rs.) B. P/V Ratio C. Variable Cost Ratio [1 − P/V ratio] D. Variable Cost per unit [C × A] (Rs.) E. Raw Material as a % of Variable Cost F. Raw Material Cost per unit [E × D] (Rs.) G. Contribution per unit [A − D] (Rs.) ∴ Contribution per unit of Raw Material Cost (Rs.) [H/G] Ranking (on the basis of the key factor of Raw Material Cost)

Alfa 100 10% 90% 90 50% 45 10 0.22 III

Beta 75 20% 80% 60 50% 30 15 0.50 II

Gamma 50 40% 60% 30 50% 15 20 1.33 I

Statement showing computation of most profitable sales mix

Gamma (10,000 units × Rs. 15) Beta (25,000 units × Rs. 30) Alfa [{Rs.18,00,000 − (Rs. 1,50,000 + Rs. 7,50,000)}/Rs. 45] Total

Raw Materials

Sales Quantity

Rs. 1,50,000 7,50,000 Balance 9,00,000 18,00,000

Rs. 10,000 25,000 20,000 55,000

Statement of profitability of most profitable sales mix Contribution from Gamma (10,000 units × Rs. 20) Contribution from Beta (25,000 units × Rs. 15) Contribution from Alfa (20,000 units × Rs. 10) Total Contribution Less: Total Fixed Cost Total Profit

Modified Date: Sat, Jun 26, 2010 03:08:44 PM

Output Date: Tue, Jul 06, 2010 12:30:28 PM

Rs. 2,00,000 3,75,000 2,00,000 7,75,000 6,80,000 95,000

Rev-II

Project: Management Accounting_Debarshi Bhattacharyya ACE Pro India Pvt. Ltd. File: X:\Pearson\Management Accounting_Debarshi Bhattacharyya\MAIN\M11\LAYOUT_M11\M11_DEBA_ISBN_EN_SE_C11.indd

726

MANAGEMENT ACCOUNTING

Problem 20 ABC Ltd manufactures three products A, B and C using the same machine which has an annual working capacity of 70,000 hours. The details of costs and selling price of these products are as follows: Sales price per unit Variable Cost per unit: Direct Material Direct Wages (@ Rs. 16 per machine hour) Variable Overheads Total Variable Cost per unit Maximum Market Demand (in units)

A (Rs.) 400

B (Rs.) 316

140 96 72 308 6,000

80 64 80 224 5,000

C (Rs.) 448 160 112 84 356 10,000

Total Fixed Cost of the company amount to Rs. 3,50,000 per annum. The company could purchase similar products from an assembly centre at the following costs: A Rs. 350 per unit B Rs. 280 per unit C Rs. 400 per unit You are required to recommend which products the company should manufacture and purchase in what quantity to maximize the company profit and also compute the overall profit of the company as per your recommended optimum production mix. Solution Statement showing determination of ranking of contribution per working hour Selling Price per unit Variable Cost per unit Contribution per unit (a) Hours of Work Required per unit (b) (Direct Wages ÷ Rs. 16) Contribution per Working hour [a/b] Ranking

A (Rs.) 400 308 92 6 Rs.15.33 II

B (Rs.) 316 224 92 4 Rs. 23 I

C (Rs.) 448 356 92 7 Rs.13.14 III

Statement showing computation of optimum production mix Product B (5,000 units × 4 hrs) Product A (6,000 units × 6 hrs) Product C [{70,000 hrs − (20,000 hrs + 36,000 hrs)}/7 hrs] Total

Working Hours 20,000 36,000 Balance 14,000 70,000

Units 5,000 6,000 2,000 13,000

The company may now purchase 8,000 units of product C from the assembly shop at a cost of Rs. 400 per unit. Problem 21 X Ltd manufactures a semi-conductor for which the cost and price structure is given as follows: Rs. per Unit 500 150 100 50

Selling Price Direct Materials Direct Labour Variable Overheads Fixed Cost

Modified Date: Sat, Jun 26, 2010 03:08:44 PM

Rs. 2,00,000

Output Date: Tue, Jul 06, 2010 12:30:28 PM

Rev-II

Project: Management Accounting_Debarshi Bhattacharyya ACE Pro India Pvt. Ltd. File: X:\Pearson\Management Accounting_Debarshi Bhattacharyya\MAIN\M11\LAYOUT_M11\M11_DEBA_ISBN_EN_SE_C11.indd

DECISION MAKING

727

The product is manufactured by a machine, whose spare part costing Rs. 2,000 needs a replacement after every 100 pieces of output. This is in addition to the above costs. Assume that no defectives are produced and that the spare parts are readily available in the market at all times at Rs. 2,000. i. Prepare the profitability statement for the production levels of 2,000 units and 3,000 units, when the Fixed Cost = Rs. 1,00,000. ii. What is the BEP for the above data? iii. Comment on the BEP, if the Fixed Cost can be reduced to Rs. 1,80,000 from the existing level of Rs. 2,00,000. [C.A. (PE-II)—November 2006] Solution i.

Profitability statement for production levels of 2,000 units and 3,000 units At 2,000 Units (Rs. in ’000) A. Variable Costs: 300 Direct Materials (Rs. 150 × production units) 200 Direct Labour (Rs. 100 × production units) 100 Variable Overheads (Rs. 50 × production units) 600 B. Fixed Cost 100 40 C. Spare Parts Cost (Rs. 2000 × 2000/100) & (Rs. 2000 × 3000/100) 740 D. Total Cost (A + B + C) 1,000 E. Sales (Rs. 500 × 2,000) & (Rs. 500 × 3,000) 260 ∴ Profit (E – D) ii.

At 3,000 Units (Rs. in ’000) 450 300 150 900 100 60 1,060 1,500 440

Spare Parts Cost is basically Step Fixed Cost, but is considered as a Variable Cost for the limited purpose of computing the range in which the BEP occurs.

∴ Spare Parts Cost per unit = Rs. 2,000/100 = Rs. 20

Selling Price per unit Less: Variable Cost per unit: Direct Materials Direct Labour Variable Overheads Spare Parts Contribution per unit (considering Spare Parts Cost as Variable Cost) Fixed Cost (without considering the Spare Parts Cost as Fixed Cost) ∴ BEP (Fixed Cost/Contribution per unit) ∴ Range of units in which BEP occurs (a) General Fixed Cost (b) Spare Parts Cost (now treating it as Fixed Cost) ∴ Total Fixed Cost (a + b) Real contribution per unit (i.e., not considering the spare parts cost as a Variable Cost) ∴ Actual BEP (in units) (Total Fixed Cost/Contribution per unit) ∴ Actual BEP (in value)

Modified Date: Sat, Jun 26, 2010 03:08:44 PM

Output Date: Tue, Jul 06, 2010 12:30:28 PM

When Fixed Cost is Rs. 1,00,000, Rs. 500

When Fixed Cost is Rs. 2,00,000, Rs. 500

(150) (100) (50) (20) 180 Rs. 1,00,000 1,00,000/180 = 555 units 501–600 units Rs. 1,00,000 6 × Rs. 2,000 = Rs. 12,000 Rs. 1,12,000 Rs. 500 – Rs. 300 = Rs. 200 Rs. 1,12,000/ Rs. 200 = 560 units 560 × Rs. 500 = Rs. 2,80,000

(150) (100) (50) (20) 180 Rs. 2,00,000 2,00,000/180 = 1,111 units 1,101–1,200 units Rs. 2,00,000 12 × Rs. 2,000 = Rs. 24,000 Rs. 2,24,000 Rs. 500 – Rs. 300 = Rs. 200 Rs. 2,24,000/ Rs. 200 1,120 units 1,120 × Rs. 500 = Rs. 5,60,000

Rev-II

Project: Management Accounting_Debarshi Bhattacharyya ACE Pro India Pvt. Ltd. File: X:\Pearson\Management Accounting_Debarshi Bhattacharyya\MAIN\M11\LAYOUT_M11\M11_DEBA_ISBN_EN_SE_C11.indd

728

MANAGEMENT ACCOUNTING

iii. If the Fixed Cost is reduced to Rs. 1,80,000, then there would be a new BEP. Then, the range of units in which the BEP occurs

= Fixed Cost/Contribution per unit (considering spare parts cost as a variable) = Rs. 1,80,000/Rs. 180 = 1,000 units ∴ Range of BEP = 901 to 1,000 units Since the new BEP of 1,000 units falls on the uppermost limit of the range of 901 to 1,000 units, there would be one more BEP in the subsequent range of 1,001 to 1100. Range of BEP (a) General Fixed Cost (b) Spare Parts Cost ∴ Total Fixed Cost (a + b) ∴ Contribution per unit (treating Spare Parts Cost as a Variable Cost) ∴ BEP (in units) [Total Fixed Cost/Contribution per unit] ∴ BEP (in value)

901 to 1,000 units Rs. 1,80,000 10 × Rs. 2,000 = Rs. 20,000 Rs. 2,00,000 Rs. 200 Rs. 2,00,000/ Rs. 200 = 1,000 units 1,000 × Rs.500 = Rs. 5,00,000

1001 to 1100 units Rs. 1,80,000 11 × Rs. 2,000 = Rs. 22,000 Rs. 2,02,000 Rs. 200 Rs. 2,02, 000/ Rs. 200 = 1,010 units 1,010 × Rs.500 = Rs. 5,05,000

Problem 22 The Manager of Division X of a company has given a production budget of 2,00,000 units of components, to be manufactured at a price which will provide a return of 25% on the average assets employed in the division. Following are the relevant data in relation thereto: Variable Cost Fixed Overheads Average Assets Employed: Stocks Debtors Fixed Assets

Re. 1 per Unit Rs. 4,00,000 Rs. 6,00,000 Rs. 2,00,000 Rs. 4,00,000

However, the marketing department of the company considers that the maximum units of the components the market can take at the proposed price is 1,40,000 only. The production manager of Division Y is ready to purchase 60,000 units of the component at a price of Rs. 2.25 per unit, as he feels that the component can be manufactured in his division at that price. The manager of Division X feels that rather than selling at Rs. 2.25 per unit, he would restrict the production in his division to 1,40,000 units only. By this, he feels that he could reduce Rs. 80,000 in stocks, Rs. 30,000 in debtors and Rs. 90,000 in plant and also reduce selling expenses by Rs. 40,000. You are required to work out whether 60,000 units should be produced for a transfer to Division Y at Rs. 2.25. Solution Total target selling price from 2,00,000 units is as follows: Variable Cost Fixed Overheads Return on Assets Employed (25% on Rs. 12,00,000) Total Selling Price

Rs. 2,00,000 4,00,000 3,00,000 9,00,000

∴ Selling Price per unit = Rs. 9,00,000 ÷ 2,00,000 = Rs. 4.50

Modified Date: Sat, Jun 26, 2010 03:08:44 PM

Output Date: Tue, Jul 06, 2010 12:30:28 PM

Rev-II

Project: Management Accounting_Debarshi Bhattacharyya ACE Pro India Pvt. Ltd. File: X:\Pearson\Management Accounting_Debarshi Bhattacharyya\MAIN\M11\LAYOUT_M11\M11_DEBA_ISBN_EN_SE_C11.indd

DECISION MAKING

1,40,000 – 1,40,000 Rs.

Proposal of Production Manager of Division Y 1,40,000 60,000 2,00,000 Rs.

5,20,000 1,70,000 3,10,000 10,00,000 6,30,000 – 6,30,000 1,40,000 4,90,000 3,60,000 1,30,000 13%

6,00,000 2,00,000 4,00,000 12,00,000 6,30,000 1,35,000 7,65,000 2,00,000 5,65,000 4,00,000 1,65,000 13.75%

Proposal of Manager of Division X Production for Sale in the market (units) Production for a transfer to Division Y (units) Total Sales and transfer (in units) Average Assets Employed: Stocks Debtors Fixed Assets Assets Employed (a) Revenue from Sale in the market @ Rs. 4.50 per unit Revenue from a transfer to Division Y @ Rs. 2.25 per unit Total Revenue from Sale and transfer Less: Variable Cost @ Re.1 per unit Contribution Less: Fixed Cost Net Profit (b) Return on Assets Employed [b/a × 100]

729

From the above calculation, it has been observed that the overall profit as per the proposal of Production Manager of Division Y (i.e., Rs. 1,65,000) is higher than that of as per the proposal of Manager of Division X (i.e., Rs. 1,30,000). Therefore, it may be concluded that the production of 1,40,000 units for sale in the market @ Rs. 4.50 per unit and the production of 40,000 units for a transfer to Division Y @ Rs. 2.25 per unit are advised to Division X, as this combination gives a higher overall profit to the division. Problem 23 A company is currently operating at 75% level of activity and it produces and sells two products X and Y. The cost sheets of these two products are as follows: Units Produced & Sold Direct Materials Direct Labour Factory Overheads (40% fixed) Administration & Selling Overheads (60% fixed) Total Cost per unit Selling Price per unit

X 3,000 Rs. per unit 10 20 25 40 95 115

Y 2,000 Rs. per unit 20 20 15 25 80 95

Factory overheads are absorbed on the basis of machine hour which is the limiting factor. The machine hour rate is Rs. 10 per hour. The company receives an offer from Japan for the purchase of product X at the price of Rs. 87.50 per unit. Alternatively, the company has another offer from Bangkok for the purchase of product Y at the price of Rs. 77.50 per unit. In both the cases, a special packing charge of Rs. 2.50 per unit has to be borne by the company. The company can accept either of the two export orders by utilizing the balance of 25% of its capacity. Advise the company of which proposal should be accepted and prepare a statement showing the overall profitability of the company after incorporating the export proposal suggested by you. [B.Com. (Hons), Delhi University]

Modified Date: Sat, Jun 26, 2010 03:08:44 PM

Output Date: Tue, Jul 06, 2010 12:30:28 PM

Rev-II

Project: Management Accounting_Debarshi Bhattacharyya ACE Pro India Pvt. Ltd. File: X:\Pearson\Management Accounting_Debarshi Bhattacharyya\MAIN\M11\LAYOUT_M11\M11_DEBA_ISBN_EN_SE_C11.indd

730

MANAGEMENT ACCOUNTING

Solution i.

Statement showing contribution per unit in the home market

Direct Materials per unit Direct Labour per unit Variable Factory Overheads per unit (60%) Variable Administration & Selling overheads per unit (40%) Total Variable Cost per unit Selling Price per unit Contribution per unit ii.

X Rs. 10 20 15

Y Rs. 20 20 9

16 61 115 54

10 59 95 36

Rs.

Rs.

Statement showing Total Fixed Cost

Fixed Factory Overheads (40%): X − (40% of Rs. 25) × 3,000 units Y − (40% of Rs. 15) × 2,000 units

30,000 12,000 42,000

Fixed Administration & Selling Overheads (60%): X − (60% of Rs. 40) × 3,000 units Y − (60% of Rs. 25) × 2,000 units

72,000 30,000 1,02,000 1,44,000

Total Fixed Cost iii.

Computation of machine hours required per unit X Rs. 25 Rs. 10 2.5 hrs

Factory Overheads per unit Machine hour rate Machine hours required per unit iv.

Computation of machine hours available for export production

Machine hours required to produce 3,000 units of X (3,000 units × 2.50 hrs) Machine hours required to produce 2,000 units of Y (2,000 units × 1.5 hrs) Total machine hours utilized at 75% capacity level) ∴ Total machine hours at 100% capacity level [100%/75% × 10,500 hrs] Less: Total machine hours utilized for the Production of X & Y to be sold in the home market Machine hours available for being utilized for Production of X & Y for export v.

Add:

Y Rs. 15 Rs. 10 1.5 hrs

7,500 hrs 3,000 hrs 10,500 hrs 14,000 hrs 10,500 hrs 3,500 hrs

Comparative Statement of two export proposals

Existing Variable Cost per unit as ascertained in (i) above Special packing charge per unit Total Variable Cost per unit for export order (a) Export Price per unit (b) Contribution per unit (b – a) Machine hours required per unit ∴ Contribution per machine hour (Rs.) Ranking on the basis of Contribution per machine hour

Modified Date: Sat, Jun 26, 2010 03:08:44 PM

Output Date: Tue, Jul 06, 2010 12:30:28 PM

Offer from Japan for Product X (Rs.) 61.00 2.50 63.50 87.50 24.00 2.5 hrs Rs. 24/ 2.5 = 9.60 II

Offer from Bangkok for Product Y (Rs.) 59.00 2.50 61.50 77.50 16.00 1.5 hrs Rs. 16/ 1.5 = 10.67 I

Rev-II

Project: Management Accounting_Debarshi Bhattacharyya ACE Pro India Pvt. Ltd. File: X:\Pearson\Management Accounting_Debarshi Bhattacharyya\MAIN\M11\LAYOUT_M11\M11_DEBA_ISBN_EN_SE_C11.indd

731

DECISION MAKING

As machine hours is the limiting factor, the remaining 25% capacity for export purpose is to be utilized to produce that proposal which yields a maximum contribution per machine hour. Therefore, here the offer from Bangkok for Product Y to be accepted by utilizing the remaining 25% capacity. ∴ No. of additional units of Product Y to be produced for export purpose by utilizing the remaining 25% capacity of 3,500 hrs = 3,500 hrs ÷ 1.5 hrs = 2,333 units vi.

Statement showing overall profitability Rs. Contribution from home market: X – 3,000 units × Rs. 54 Y – 2,000 units × Rs. 36

Rs.

1,62,000 72,000 2,34,000

Contribution from export market: Y – 2,333 units × Rs. 16 Total Contribution Less: Total Fixed Cost Total Profit

37,238 2,71,238 1,44,000 1,27,238

Then the position of production and purchase of the company will be as follows: A 6,000 –

Units to be produced Units to be purchased vii.

B 5,000 –

C 2,000 8,000

Total 13,000 8,000

Rs.

Rs.

Statement of profitability of most profitable product mix

Contribution from production: From Product A (6,000 units × Rs. 92) From Product B (5,000 units × Rs. 92) From Product C (2,000 units × Rs. 92) Contribution from purchase: From Product C [(Rs. 448 − Rs. 400) × 8,000 units] Total Contribution Less: Fixed Cost Total Profit

5,52,000 4,60,000 1,84,000

11,96,000 3,84,000 15,80,000 3,50,000 12,30,000

Problem 24 A company has an installed production capacity of 1,00,000 units and presently it is working at 70% capacity utilization. As the production capacity utilization increases, the cost per unit decreases as follows: Capacity Utilization 70 % 80 % 90 % 100 %

Cost per Unit (Rs.) 97 92 87 82

The company has received three export orders from different sources as follows: Source A 5,000 units at Rs. 55 per unit Source B 10,000 units at Rs. 52 per unit Source C 10,000 units at Rs. 51 per unit Advise the company whether any or all of the export orders should be accepted or not. [B.Com. (Hons), Delhi University]

Modified Date: Sat, Jun 26, 2010 03:08:44 PM

Output Date: Tue, Jul 06, 2010 12:30:28 PM

Rev-II

Project: Management Accounting_Debarshi Bhattacharyya ACE Pro India Pvt. Ltd. File: X:\Pearson\Management Accounting_Debarshi Bhattacharyya\MAIN\M11\LAYOUT_M11\M11_DEBA_ISBN_EN_SE_C11.indd

732

MANAGEMENT ACCOUNTING

Solution Statement showing computation of Differential Cost at different capacity levels Production (Units) (a) 70,000 80,000 90,000 1,00,000

Capacity 70% 80% 90% 100%

Cost per Unit (Rs.) (b) 97 92 87 82

Total Cost (Rs.) (a ¥ b) 67,90,000 73,60,000 78,30,000 82,00,000

Differential Cost Total (Rs.) – 5,70,000 4,70,000 3,70,000

Differential Cost per Unit (Rs.) – 57 47 37

Statement showing computation of Differential Cost at different capacity levels Export Order Source A Source B

Quantity (Units) 5,000 10,000

Capacity Utilization 75% 85%

Source C

10,000

95%

Differential Cost Per Unit (Rs.) Total (Rs.) 57 2,85,000 5,000 @ 57 5,20,000 5,000 @ 47 5,000 @ 47 4,20,000 5,000 @ 37

Selling Price Per Unit (Rs.) Total (Rs.) 55 2,75,000 52 5,20,000 51

5,10,000

Profit/ Loss (Rs.) (10,000) Nil 90,000

CONCLUSION From the above, it is observed that the acceptance of export order from Source A gives a loss of Rs. 10,000, acceptance of export order from Source B does not add any profit and acceptance of export order from Source C gives only a profit of Rs. 90,000. On the other hand, the overall profit from all the three orders comes to Rs. 80,000 (i.e., Rs. 90,000 − Rs. 10,000). Therefore, the company should accept the export offer from Source C only. If the three export orders are complementary to each other, then the company may accept all the three orders as this combination yields an overall profit of Rs. 80,000. CHAPTER REVIEW SUMMARY  Decision Making means the process of choosing of the best one among the various alternative actions. For any business planning and control, the management is engaged in search of the most appropriate course of action. Among the various alternatives feasible to the management, it has to choose the best one from the point of view of the business concern.  For an effective Decision Making, the following steps are to be considered: (i) Clearly defining the concerned problem; (ii) Identifying and describing the available alternatives; (iii) Evaluating relevant quantitative and qualitative factors; (v) Selecting the best one among the identified various alternatives; and (vi) Evaluating the performance of the decision and providing feedback.  Certain concepts of cost which are used in the process of cost analysis for Decision Making are: (a) Marginal Cost; (b) Differential Cost; (c) Sunk cost; (d) Out of pocket cost; (e) Opportunity cost; (f) Imputed cost; and (g) Replacement cost.  A cost (or revenue) that is essentially important and relevant to a Decision Making process is called Relevant Cost (or Relevant Revenue). Relevant Costs and revenues are those expected future costs and future revenues which differ under alternative courses of action being considered in the Decision Making process.  A comparison of Differential Cost Analysis with Marginal Costing shows the following facts: (a) Both Differential Costing as well as Marginal Costing are techniques of cost analysis; (b) Both the techniques are used by the management in its process of Decision Making; (c) Both the techniques are based on the classification of costs into Fixed Cost and Variable Cost; (d) When there is no change in the Fixed Cost due to a change in the volume of production or sales, then Differential Cost becomes equal to the Marginal Cost; and (e) In Marginal Costing, P/V ratio, contribution margin, contribution per unit of limiting factor and so on are the main parameters for evaluation of performance and Decision Making. On the other hand, in Differential Cost Analysis, Differential Costs are compared with the differential revenues to select the best alternative in the process of Decision Making.

Modified Date: Sat, Jun 26, 2010 03:08:44 PM

Output Date: Tue, Jul 06, 2010 12:30:28 PM

Rev-II

Project: Management Accounting_Debarshi Bhattacharyya ACE Pro India Pvt. Ltd. File: X:\Pearson\Management Accounting_Debarshi Bhattacharyya\MAIN\M11\LAYOUT_M11\M11_DEBA_ISBN_EN_SE_C11.indd

DECISION MAKING 

 

733

A Key or Limiting Factor is a factor which limits or restricts the production or sales level at a point of time due to scarcity of a factor of production. Key factor is a limitation of constraints such as availability of raw material, labour, machine/plant capacity, capital, market demand and so on on the activities of a business. Following are some frequently applicable situations of Decision Making: (a) Determination of most profitable product/sales mix; (b) Accept or reject decision; (c) Make or buy decision; (d) Operate or shut-down decision.

CHAPTER REVIEW QUIZ 1. State whether the following statements are true or false: a. In Differential Cost Analysis, managerial decisions are based on contribution and P/V ratio. b. Differential Costing can be used in Absorption Costing as well as Marginal Costing. c. Contribution per unit of key factor is the basis of ranking products on profitability. d. Sunk cost is a Relevant Cost for Decision Making. e. Differential Cost becomes equal to the Marginal Cost if there is no change in the Fixed Cost. f. Opportunity Cost is an Irrelevant Cost for Decision Making. Ans.: True: (b), (c), (e); False: (a), (d), (f). 2. Fill in the blanks of the following statements: among the various alternative a. Decision Making means the process of choosing of the actions. b. A cost that is essentially important and relevant to a Decision Making process is called c. When there is no change in the Fixed Cost due to a change in the volume of production or sales, becomes equal to the Marginal Cost. d. A key or limiting factor is a factor which limits or restricts the production or the sales level at a point of . time due to scarcity of a . e. Both Differential Costing as well as Marginal Costing are techniques of f. For an effective Decision Making, evaluations of relevant and factors are essentially required. Ans.: (a) best one; (b) Relevant Cost; (c) Differential Cost; (d) factor of production; (e) cost analysis; (f) quantitative and qualitative. EXERCISE I. Theoretical Questions A. Short Answer Type Question

1. 2. 3. 4. 5. 6. 7. 8. 9. 10.

What is meant by Decision Making? What are the steps involved in Decision Making? What is Relevant Cost? What is Relevant Revenue? What is Differential Cost? Is Differential Cost equal to Marginal Cost? When is the Differential Cost equal to the Marginal Cost? What is the Key or Limiting Factor in Decision Making? What is Sunk Cost? What is Out of Pocket Cost?

B. Essay Type Questions

1. 2. 3. 4. 5.

What do you mean by the term ‘Decision Making’? What are the steps involved in Decision Making? What are the different concepts of cost used in the cost analysis for Decision Making? What is Differential Cost Analysis? Compare Differential Cost Analysis with Marginal Costing. What is Key or Limiting Factor? How is it applied in Decision Making? Briefly explain three situations where Decision Making is required.

Modified Date: Sat, Jun 26, 2010 03:08:44 PM

Output Date: Tue, Jul 06, 2010 12:30:28 PM

Rev-II

Project: Management Accounting_Debarshi Bhattacharyya ACE Pro India Pvt. Ltd. File: X:\Pearson\Management Accounting_Debarshi Bhattacharyya\MAIN\M11\LAYOUT_M11\M11_DEBA_ISBN_EN_SE_C11.indd

734

MANAGEMENT ACCOUNTING

II. Practical Problems

1. A machine manufactures 10,000 units of a part at a total cost of Rs. 21, of which Rs. 18 is a variable. This part is readily available in the market at Rs. 19 per unit. If the part is purchased from the market, then the machine can either be utilized to manufacture a component in the same quantity contributing Rs. 2 per component or it can be hired out at Rs. 21,000. Recommend which of the alternatives is profitable? Ans.: It is recommended to purchase the part from the market and the machine time is hired out, as this alternative is the least costly of the three alternatives. 2. Java Ltd manufactures and markets a single product. The following information is available: Materials per unit Conversion Costs (variable) per unit Dealer’s Margin per unit Selling Price per unit Fixed Cost Present Sales Capacity Utilization

Rs. 8.00 Rs. 6.00 Rs. 2.00 Rs. 20.00 Rs. 2,50,000 80,000 units 60%

There is acute competition. Extra efforts are necessary to sell. Suggestions have been made for increasing sales: (i) by reducing the sales price by 5% and (ii) by increasing the dealer’s margin by 25% over the existing rate. Which of the two suggestions would you recommend if the company desires to maintain the present profit? [C.S. (Inter)—Adapted] Ans.: Under proposal (i): Contribution per unit – Rs. 3.10; Sales volume required – 1,03,226 units. Under proposal (ii): Contribution per unit – Rs. 3.50; Sales volume required – 91,429 units. Proposal (ii) is recommended. 3. A company producing 24,000 units provides you the following information: Direct Materials Direct Wages Variable Overheads Semi-variable Overheads Fixed Overheads Total Cost

Rs. 1,20,000 84,000 48,000 28,000 80,000 3,60,000

The product is sold at Rs. 20 per unit. The management proposes to increase the production by 3,000 units for sales in the foreign market. It is estimated that the semi-variable overheads will increase by Rs. 1,000. But the product will be sold at Rs. 14 per unit in the foreign market. However, no additional capital expenditure will be incurred. The management seeks your advice as a Cost Accountant. What will you advise them? [B.Com. (Hons), Delhi University] Ans.: Export order should be accepted as it yields an additional profit of Rs. 9,500. 4. A manufacturer has planned his level of operations at 50% of his plant capacity of 30,000 units. His expenses are estimated as follows, at 50% of the plant capacity utilization: Direct Materials Direct Wages Variable and other Manufacturing Expenses Total Fixed Expenses Irrespective of Capacity Utilization

Rs. 8,280 11,160 3,960 6,000

The expected selling price in the domestic market is Rs. 2 per unit. Recently, the manufacturer has received a trade enquiry from an overseas organization that is interested in purchasing 6,000 units at a price of Rs. 1.45 per unit.

Modified Date: Sat, Jun 26, 2010 03:08:44 PM

Output Date: Tue, Jul 06, 2010 12:30:28 PM

Rev-II

Project: Management Accounting_Debarshi Bhattacharyya ACE Pro India Pvt. Ltd. File: X:\Pearson\Management Accounting_Debarshi Bhattacharyya\MAIN\M11\LAYOUT_M11\M11_DEBA_ISBN_EN_SE_C11.indd

DECISION MAKING

735

As a professional Management Accountant what would be your suggestion regarding acceptance or rejection of the offer? Support your suggestion with suitable quantitative information. [I.C.W.A. (Inter)—Adapted] Ans.: Export order is not acceptable as it yields a decrease in the total profit by Rs. 60. 5. The cost per unit of the three Products X, Y and Z are given as follows: Particulars

X (Rs.) 20 12 8 6 46 18 64 10,000

Direct Material Direct Labour Variable Overheads Fixed Expenses Profit Selling Price No. of units Produced

Y (Rs.) 16 14 10 6 46 14 60 5,000

Z (Rs.) 18 12 6 4 40 12 52 8,000

Production arrangements are such that if one product is given up, the production of the others can be raised by 50%. The directors propose that Product Z should be given up because the contribution from the product is the lowest. Present a suitable analysis of the data indicating whether the proposal should be accepted or not. [B.Com. (Hons), Delhi University—1998] Ans.: Proposal of the directors is not acceptable. Profit (Rs. 4,30,000) would be maximum if Product Y is given up. 6. A company manufactures electric motors at a price of Rs. 6,900 each, which is made up as follows: Rs. 3,200 400 1,000 200 200 100 200 1,000 6,300 600 6,900

Direct Material Direct Labour Variable Overheads Fixed Overheads Depreciation Variable Selling Overheads Royalty Profit Central Excise Duty

i.

A foreign buyer has offered to buy 200 such motors at Rs. 5,000 each. As a Cost Accountant of the company, would you advise acceptance of the offer? ii. What should the company quote for a motor to be purchased by a company under the same management if it should be at cost? [B.Com. (Hons), Delhi University] Ans.: (i) Offer from the foreign buyer is acceptable as it is expected to yield an incremental profit of Rs. 40,000. (ii) Rs. 5,200 (without charging an excise duty). 7. Two competing companies Heera Ltd and Panna Ltd sell the same type of product in the same market. Their forecasted Profit & Loss A/c for the year ending 31 March 2010 are as follows: Heera Ltd Rs. Sales Less: Variable Costs Fixed Costs Forecasted Net Profit before Tax

Modified Date: Sat, Jun 26, 2010 03:08:44 PM

4,00,000 50,000

Output Date: Tue, Jul 06, 2010 12:30:28 PM

Rs. 5,00,000 4,50,000 50,000

Panna Ltd Rs. Rs. 5,00,000 3,00,000 1,50,000 4,50,000 50,000

Rev-II

Project: Management Accounting_Debarshi Bhattacharyya ACE Pro India Pvt. Ltd. File: X:\Pearson\Management Accounting_Debarshi Bhattacharyya\MAIN\M11\LAYOUT_M11\M11_DEBA_ISBN_EN_SE_C11.indd

736

MANAGEMENT ACCOUNTING

You are required to state which company is likely to earn a greater profit in the conditions of: a. Low Demand. b. High Demand. [I.C.W.A. (Inter)—Adapted] Ans.: (a) Heera Ltd and (b) Panna Ltd 8. A mechanical toy factory presents the following information for the year 2009: Material Cost Labour Cost Fixed Overheads Variable Overheads Units Produced Selling Price per unit

Rs. 1,20,000 Rs. 2,40,000 Rs. 1,20,000 Rs. 60,000 12,000 Rs. 50

The available capacity is a production of 20,000 units per year. The firm has an offer for the purchase of 5,000 additional units at a price of Rs. 40 per unit. It is expected that by accepting this offer there will be a saving of rupee one per unit in the material cost on all the units manufactured, the fixed overheads will increase by Rs. 35,000 and the overall efficiency will drop by 2% on all production. State whether the offer is acceptable or not. [C.S. (Inter)—Adapted] Ans.: P/V Ratio increases from 25% to 26.9% and the fixed overheads increase from Rs. 1,20,000 to Rs. 1,55,000 if the offer is accepted. The offer is, therefore, not very attractive. 9. Wanna manufacturing company sells the following three products: X @ Rs. 8 per unit, Y @ Rs. 2 per unit and Z @ Rs. 3 per unit. While Product X contributes 20% its revenue in Fixed Cost and profit, Product Y contributes 10% and product Z contributes 60%. The company earned a net profit of Rs. 50,000 last year by selling 50,000 units of X, 1,50,000 units of Y and 60,000 units of Z. It is believed that the profit picture can be improved by eliminating Product Y and concentrating the sales efforts on Products X and Z. There is an opportunity to increase the sale of Product X to 70,000 units, but product Z will probably be sold at the same volume next year. As a Cost Accountant of the company, give your suggestion regarding the elimination of Product Y. Ans.: Elimination of Product Y is recommended as it will increase the net profit by Rs. 2,000 and the total Fixed Cost is Rs. 1,68,000. 10. King Parts Ltd has an annual production of 90,000 units for a motor component. The component’s cost structure is as follows: Rs. Material per unit Labour per unit (25% fixed) Expenses per unit: Variable Fixed

90 135 Total Cost per unit

Rs. 270 180

225 675

a. The purchase manager has an offer from a supplier who is willing to supply the component at Rs. 540. Should the component be purchased and production stopped? b. Assume the resources now used for this component’s manufacture are to be used to produce another new product for which the selling price is Rs. 485. In the latter case, the material price will be Rs. 200 per unit. 90,000 units of this product can be produced at the same cost basis as above for labour and expenses. Discuss whether it would be advisable to divert the resources to manufacture that new product, on the footing that the component presently being produced would instead of being produced, be purchased from the market. [C.A. (Inter)—Adapted]

Modified Date: Sat, Jun 26, 2010 03:08:44 PM

Output Date: Tue, Jul 06, 2010 12:30:28 PM

Rev-II

Project: Management Accounting_Debarshi Bhattacharyya ACE Pro India Pvt. Ltd. File: X:\Pearson\Management Accounting_Debarshi Bhattacharyya\MAIN\M11\LAYOUT_M11\M11_DEBA_ISBN_EN_SE_C11.indd

737

DECISION MAKING

Ans.: a. The company should not stop production of the component. b. It is advised to divert the resources to manufacture the new product as there will be a saving of Rs. 15 per unit. 11. Firdous Ltd operating at 75% level of activity produces and sells two products K and L. The cost sheets of these two products are as follows: Product K 600 Rs. 2 4 5 8 19 23

Units Produced and Sold Direct Materials Direct Labour Factory Overheads (40% fixed) Selling & Administration Overheads (60% fixed) Total Cost per unit Selling Price per unit

Product L 400 Rs. 4 4 3 5 16 19

Factory overheads are absorbed on the basis of machine hour which is the limiting (key) factor. The machine hour rate is Rs. 2 per hour. The company receives an offer from Italy for the purchase of Product K at a price of Rs. 17.50 per unit. Alternatively, the company has another offer from Japan for the purchase of Product L at a price of Rs. 15.50 per unit. In both the cases, a special packing charge of Re. 0.50 per unit has to be borne by the company. The company can accept either of the two export orders and in either case, the company can supply such quantities as may be possible to produce by utilizing the balance of 25% of its capacity. You are required to prepare: i.

a statement showing the economies of the two export proposals giving your recommendations as to which proposal should be accepted. ii. a statement showing the overall profitability of the company after incorporating the export proposal recommended by you. [C.A. (Inter)—Adapted] Ans.: Contribution per machine hour: K – Rs. 1.92 and L – Rs. 2.13; No. of export order of L – 4,607 units; Overall profit – Rs. 5,095; and Total Fixed Cost – Rs. 5,760. 12. A company has a plant capacity of 19,800 machine hours. The plant can either produce Product M or Product N or a mixture of both. Following is the relevant information given: Product M 20 30 5

Selling Price per unit (Rs.) Variable Cost per unit (Rs.) Machine hours required per unit (hrs)

Product N 30 18 3

Market conditions are such that not more than 5,000 units of M and 4,000 units of N can be sold in a year. Fixed Costs are Rs. 60,000. Compute the product mix that will maximize the net income and find that maximum income. [I.C.W.A. (Inter)—Adapted] Ans.: Most profitable product mix: 4,000 units of N and 1,560 units of M; Net income as per most profitable mix is Rs. 18,720. 13. From the following information, state which of the alternative sales mixes you would recommend to the management and why: Selling Price per unit (Rs.) Direct Materials per unit (Rs.) Direct Wages

Modified Date: Sat, Jun 26, 2010 03:08:44 PM

Product R 25 8 24 hours @ 25 paise per hour

Output Date: Tue, Jul 06, 2010 12:30:28 PM

Product S 20 6 16 hours @ 25 paise per hour

Rev-II

Project: Management Accounting_Debarshi Bhattacharyya ACE Pro India Pvt. Ltd. File: X:\Pearson\Management Accounting_Debarshi Bhattacharyya\MAIN\M11\LAYOUT_M11\M11_DEBA_ISBN_EN_SE_C11.indd

738

MANAGEMENT ACCOUNTING

Fixed overheads — Rs. 750; Variable overheads — 150% of direct wages. Alternative sales mix: a. 250 units of R and 250 units of S. b. Nil units of R and 400 units of S. c. 400 units of R and 100 units of S. [I.C.W.A. (Inter)—Adapted] Ans.: Recommended sales mix is (b) as it yields the highest overall profit. 14. A company producing two Products P and Q using a single production process furnishes the following cost data: Product P 20 11 1 1,00,000

Selling Price per unit (Rs.) Variable Cost per unit (Rs.) Machine hours required per unit of production (hrs) Market Limitation (units)

Product Q 30 16 2 2,50,000

Total machine hours available — 4,00,000; Fixed Cost per annum — Rs. 26,00,000. Considering the limiting factors of machine hours and market limitations, you are required to: a. Indicate the best combination of products to give an optimum contribution. b. Show the additional machinery requirement to be augmented on rental basis at an annual rent of Rs. 1,50,000 per machine to provide an additional capacity of 30,000 hours per machine.

c. Change in number of machines to be rented if the annual rental charges reduce to Rs. 1,25,000 per machine. [I.C.W.A. (Inter)—Adapted] Ans.: (a) Best combination of products: 1,00,000 units of P and 1,50,000 units of Q; Total contribution as per suggested combination is Rs. 30,00,000. (b) Six machines should be taken on rent. (c) Seven machines could be taken on rent. 15. XY Ltd is manufacturing three household Products A, B and C and selling them in a competitive market. Details of the current demand, selling price and cost structure are given as follows: Expected Demand (units) Selling Price per unit (Rs.) Variable Cost per unit (Rs.): Direct Materials (Rs. 10/kg) Direct Labour (Rs. 15/hr.) Variable Overheads Fixed Overheads per unit (Rs.)

A 10,000 20

B 12,000 16

C 20,000 10

6 3 2 5

4 3 1 4

2 1.50 1 2

The company is frequently affected by an acute scarcity of raw material and high labour turnover. During the next period it is expected to have one of the following situations: a. Raw materials available will be only 12,100 kg. b. Direct labour hours available will be only 5,000 hrs. c. It may be possible to increase the sales of any one product by 25% without any additional Fixed Costs, but by spending Rs. 20,000 on advertisement. There will be no shortage of materials or labour. Suggest the best production plan in each case and the resultant profit that the company would earn according to your suggestion. [I.C.W.A. (Inter)—June 1998] Ans.: Profit: (a) Rs. 1,17,500; (b) Rs. 1,02,000; and (c) Rs. 1,65,500.

Modified Date: Sat, Jun 26, 2010 03:08:44 PM

Output Date: Tue, Jul 06, 2010 12:30:28 PM

Rev-II

Project: Management Accounting_Debarshi Bhattacharyya ACE Pro India Pvt. Ltd. File: X:\Pearson\Management Accounting_Debarshi Bhattacharyya\MAIN\M11\LAYOUT_M11\M11_DEBA_ISBN_EN_SE_C11.indd

739

DECISION MAKING

16. Three products X, Y and Z are made and sold by a company. The relevant information is given as follows: Products Standard Costs per unit: Direct Materials (Rs.) Variable Overheads (Rs.) Direct Labour Department A Department B Department C

Rate per hour Rs. 5 Rs. 6 Rs. 4

X

Y

Z

50 12 Hrs 14 4 8

120 7 Hrs 8 3 4

90 16 Hrs 15 5 15

Total fixed overheads for the year are Rs. 3,00,000. The budget for the current financial year, which is prepared for a recessionary period, is based on the following sales: Products X Y Z

Sales (Units) 7,500 6,000 6,000

Selling Price per Unit (Rs.) 210 220 300

You are required to show in the form of a statement to the management, the unit Variable Cost of the three products and the total profit expected for the current year. Which of these products is the most profitable? Rank the products. [C.S. (Inter)—Adapted] Ans.: X 188 II

Unit Variable Cost (Rs.) Ranking of Profitability Total Profit — Rs. 1,53,000.

Y 201 III

Z 271 I

17. A company can produce three different products from the same raw material using same production facilities. The requisite labour is available in plenty at Rs. 8 per hour for all products. The supply of raw material, which is imported at Rs. 8 per kg, is limited to 10,400 kg for the budget period. The variable overheads are Rs. 5.60 per hour. The fixed overheads are Rs. 50,000. The selling commission is 10% on sales. a. From the following information, you are required to suggest the most suitable sales mix, which will maximize the company’s profit. Also determine the profit that will be earned at that level.

Product X Y Z

Market Demand Required per Unit (Units) 8,000 6,000 5,000

Selling Price per Unit (Rs.) 30 40 50

Labour Hours per Unit (Hrs) 1 2 1.5

Raw Materials Required per Unit (Kg) 0.7 0.4 1.5

b. Assume, in the above situation, if an additional 4,500 kg of raw material is made available for production, should the company go in for further production, if it will result in additional fixed overheads of Rs. 20,000 and a 25% increase in the rates per hour for labour and variable overheads ? [B.Com. (Hons.), Delhi University] Ans.: (a) Most suitable sales mix: 6,000 units of Y, 8,000 units of X and 1,600 units of Z; Total profit as per suggested sales mix is Rs. 66,144. (b) The company should go in for an additional production of 3,000 units of Z as it would yield an additional profit of Rs. 2,500.

Modified Date: Sat, Jun 26, 2010 03:08:44 PM

Output Date: Tue, Jul 06, 2010 12:30:28 PM

Rev-II

Project: Management Accounting_Debarshi Bhattacharyya ACE Pro India Pvt. Ltd. File: X:\Pearson\Management Accounting_Debarshi Bhattacharyya\MAIN\M11\LAYOUT_M11\M11_DEBA_ISBN_EN_SE_C11.indd

740

MANAGEMENT ACCOUNTING

18. Neem Agro Ltd, engaged in agricultural activities, has 500 hectares of virgin land, which can be based for growing jointly or individually tea, coffee and cardamom. The yield per hectare of the different crops and selling prices per kg are as follows: Yield (kg) 2,500 625 125

Tea Coffee Cardamom

Selling Price (Rs. per kg) 25 50 300

The Relevant Cost data are given as follows: 1. Variable Cost per kg:

Labour Charges Packing Materials Other Costs

Tea Rs. 10.00 2.50 5.00 17.50

Coffee Rs. 12.50 2.50 1.25 16.25

Cardamom Rs. 150.00 12.50 25.00 187.50

2. Fixed Cost per annum: Rs. 16,00,000 4,50,000 2,75,000 5,00,000 6,75,000 35,00,000

Cultivation and Growing Cost Administrative Cost Land Revenue Repairs and Maintenance Other Costs

The policy of the company is to produce and sell all the three kinds of products and the maximum and minimum area to be cultivated per product are as follows: Maximum Area (Hectares) 320

Tea Coffee Cardamom

Minimum Area (Hectares) 240

100

60

60

20

Calculate the priority of production, the most profitable product mix and the maximum profit which can be achieved. [C.S. (Inter)—Adapted] Ans.: Priority of production (on the basis of contribution per hectare of land): coffee – I, tea – II and cardamom – III; Most profitable product mix: coffee in 100 hectares, tea in 320 hectares and cardamom in 60 hectares; Maximum profit as per most profitable product mix is Rs. 54,53,125. 19. Stirling Industries Ltd manufactures a Product Z by making and assembling three components A, B and C. The components are made in a machine shop using three identical machines each of which can make any of the three components. However, the total capacity of the three machines is only 12,000 machine-hours per month and is just sufficient to meet the current demand. Labour for assembling is available according to requirements. Further details are given as follows:

Modified Date: Sat, Jun 26, 2010 03:08:44 PM

Output Date: Tue, Jul 06, 2010 12:30:28 PM

Rev-II

Project: Management Accounting_Debarshi Bhattacharyya ACE Pro India Pvt. Ltd. File: X:\Pearson\Management Accounting_Debarshi Bhattacharyya\MAIN\M11\LAYOUT_M11\M11_DEBA_ISBN_EN_SE_C11.indd

DECISION MAKING

Components

Machine-Hours Required per Unit

A B C Assembling

4 5 6 –

Variable Cost per Unit Rs. 48 60 80 30 (per unit of Z)

741

Market Price at Which the Component can be Purchased if Required Rs. 64 75 110 –

Fixed Costs per month amount to Rs. 50,000. Product Z is sold at Rs. 300 per unit. From the next month onwards, the company expects the demand for Z to rise by 25%. As the machine capacity is limited, the company wants to meet the increase in demand by buying such numbers of A, B or C which is most profitable. You are asked to find out the following: a. Current demand and profit made by the company. b. Which component and how many units of the same should be bought from the market to meet the increase in demand?

c. Profit made by the company if the suggestion in (b) is accepted. [I.C.W.A. (Inter)—December 1998] Ans.: (a) 800 units, Profit Rs. 15,600; (b) 600 units of B; (c) Rs. 23,000. 20. A company produces 30,000 units of product A and 20,000 units of a product B per annum. The sales value and costs of the two products are as follows: Rs. 7,60,000 1,40,000 1,90,000 1,90,000 1,20,000

Sales Value Direct Material Direct Labour Factory Overheads Administrative & Selling Overheads

50% of factory overheads are variable and 50% of administrative and selling overheads are fixed. The selling price of A is Rs. 12 per unit and B is Rs. 20 per unit. The direct material and labour ratio for product A is 2:3 and for B is 4:5. For both the products, the selling price is 400% of direct labour. The factory overheads are charged in the ratio of direct labour, and administrative and selling overheads are recovered at a flat rate of Rs. 2 per unit of A and Rs. 3 per unit of B. Due to a fall in the demand of the above products, the company has a plan to diversify and make the product C using 40% capacity. It has been estimated that for C the direct material and direct labour will be Rs. 2.50 and Rs. 3 per unit, respectively. Other Variable Costs will be the same as applicable to product A. The selling price of product C is Rs. 14 per unit and the production will be 30,000 units. Assuming that 60% capacity is used for manufacturing A and B: a. Calculate the present cost and profit. b. Calculate the costs and profit after diversification. c. Give your recommendation as to whether to diversify or not. [I.C.W.A. (Inter)—June 1999] Ans.: (a) Net profit – Rs. 1,20,000; (b) Net profit – Rs. 1,90,000; (c) To diversify.

Modified Date: Sat, Jun 26, 2010 03:08:44 PM

Output Date: Tue, Jul 06, 2010 12:30:28 PM

Rev-II

Project: Management Accounting_Debarshi Bhattacharyya ACE Pro India Pvt. Ltd. File: X:\Pearson\Management Accounting_Debarshi Bhattacharyya\MAIN\M11\LAYOUT_M11\M11_DEBA_ISBN_EN_SE_C11.indd

742

MANAGEMENT ACCOUNTING

21. ABC Ltd is at present operating at 80% capacity level, the production being 15,000 units per annum. It operates a flexible budgetary control system. The following Relevant Cost data are obtained from the company’s budget at different capacity utilization levels: Capacity Utilization Level 80% 100% 15,000 18,750 Rs. Rs. 20,00,000 25,00,000 2,25,000 2,50,000 1,05,000 1,11,000 4,00,000 4,70,000

Output in units Sales Variable Overheads Semi-variable Overheads Fixed Overheads

Material and labour cost per unit are constant under the present conditions. The management expects a profit margin of 10% on sales. You are required to compute the Differential Cost of producing the additional 3,750 units, by increasing the capacity utilization level to 100%. [B.Com. (Hons), Delhi University] Ans.: Differential Cost: Rs. 3,68,500.

Modified Date: Sat, Jun 26, 2010 03:08:44 PM

Output Date: Tue, Jul 06, 2010 12:30:28 PM

Rev-II

Project: Management Accounting_Debarshi Bhattacharyya ACE Pro India Pvt. Ltd. File: X:\Pearson\Management Accounting_Debarshi Bhattacharyya\MAIN\M12\LAYOUT_M12\M12_DEBA_ISBN_EN_SE_C12.indd

Capital Budgeting

12

LEARNING OBJECTIVES On completion of the study of the chapter, you should be able to understand: What is Capital Budgeting? Features and importance of Capital Budgeting. Different types of investments. Different types of Capital Budgeting decisions. Evaluation criteria of Capital Budgeting decisions. Concept and determination of present value and discounted cash flow. Different Capital Budgeting techniques.

12.1 CONCEPT OF CAPITAL BUDGETING Capital Budgeting is the technique of long-term planning and analysis of outflows and inflows of funds relating to some investment objectives. It is the planning of capital expenditure which provides return over a number of years. According to Charles T. Horngreen, ‘Capital Budgeting is long-term planning for making and financing proposed capital outlay.’ Capital Budgeting is the process of making investment decisions in the capital expenditure which are essentially long-term projects, the benefits of which are expected to be received over a number of years. Therefore, it is the decision-making process by which firms evaluate their future long-term investments like purchase of new fixed assets, new investments to be made outside the firm and so on. 12.2 FEATURES OF CAPITAL BUDGETING Capital Budgeting technique possesses the following distinguished features: i. Investment proposal for which the Capital Budgeting technique is to be applied should be of a longterm nature. ii. Proposed investment is to be made during the current period, but return from the investment will be obtained over a number of years in the future period. iii. Expenditure for the proposed investment and return from such investment should be measured in terms of cash flow, that is, cash outflow and cash inflow respectively. iv. Investment decision may be taken for a single project proposal, or for two or more mutually exclusive project proposals. v. Acceptance or rejection of an investment proposal should be based on the maximization of value of the firm. vi. Capital Budgeting involves a higher degree of risk as it is a planning of log-term investments.

Modified Date: Tue, Jul 06, 2010 12:32:27 PM

Output Date: Tue, Jul 06, 2010 06:05:04 PM

Rev II

Project: Management Accounting_Debarshi Bhattacharyya ACE Pro India Pvt. Ltd. File: X:\Pearson\Management Accounting_Debarshi Bhattacharyya\MAIN\M12\LAYOUT_M12\M12_DEBA_ISBN_EN_SE_C12.indd

744

MANAGEMENT ACCOUNTING

12.3 IMPORTANCE OF CAPITAL BUDGETING Capital Budgeting has an immense importance to every concern for its long-term decision-making due to the following reasons: i. Capital Budgeting decisions involve a huge investment. As the resources are limited, it is very important to judge the viability of the proposed capital expenditure. They provide a necessary plan and control for the proposed capital expenditure. ii. While capital investment involves a long-term investment of fund with the risk factor, Capital Budgeting provides the necessary plan and action for minimizing the risk involved in the long-term investment. iii. Capital Budgeting decisions are, in most cases, of irreversible nature due to the involvement of a huge fund for a long-term period. Hence, proper care should be taken through the Capital Budgeting for implementing such an investment. iv. They not only effect on the present earning of the firm but also effect on the future profitability of the firm. v. Capital Budgeting decisions involve complexity due to a high degree of risk, future uncertainties and long-time involvement associated with them. vi. They influence on the financing and dividend decisions of the firm. vii. They involve employment, economic growth and development, economic activities and so on; and thus, they also have a social and national importance. 12.4 TYPES OF INVESTMENTS Under Capital Budgeting technique, investments are classified into two parts as follows: Expansion projects: In this segment, investments are made to new projects with the view to generate an additional revenue of the firm. Such investment in new project is often referred to as ‘revenue expansion investment.’ Replacement projects: In this segment, investments are made to the existing projects with the view to increase the earnings of the firm by reduction of cost through replacement of the existing project. Such investment for replacement of the existing project is often referred to as ‘cost reduction investment.’ 12.5 TYPES OF CAPITAL BUDGETING DECISIONS Capital Budgeting decisions may be applied for different long-term investment decisions. Such investment decisions are generally classified into the following three categories: Accept–reject decision: This implies either the acceptance of the investment proposal or rejection of the proposal. Under this category, the investment proposal that has been considered for evaluation is being evaluated on the basis of certain evaluation criteria for a Capital Budgeting decision. If the investment proposal satisfies all those evaluation criteria for a Capital Budgeting decision, it is accepted by the firm; otherwise, the proposal is rejected. Mutually exclusive decision: This implies the acceptance of the best proposal and rejection of other proposals. Under this category, all investment proposals are considered to be independent of each other and any two proposals cannot be accepted simultaneously. Each of these investment proposals is being evaluated separately on the basis of certain evaluation criteria for a Capital Budgeting decision. Among the various alternative investment proposals, the proposal which shows the best result on the basis of those evaluation criteria for Capital Budgeting decision is accepted by the firm and other proposals are rejected. Capital-rationing decision: It implies the decision of distribution of limited funds among different accepted investment proposals. This is applied in a situation when a number of alternative investment proposals have been selected for investments, but the firm does not have sufficient funds to invest in all these selected

Modified Date: Tue, Jul 06, 2010 12:32:27 PM

Output Date: Tue, Jul 06, 2010 06:05:04 PM

Rev II

Project: Management Accounting_Debarshi Bhattacharyya ACE Pro India Pvt. Ltd. File: X:\Pearson\Management Accounting_Debarshi Bhattacharyya\MAIN\M12\LAYOUT_M12\M12_DEBA_ISBN_EN_SE_C12.indd

CAPITAL BUDGETING

745

investment proposals. In this situation, the firm should rank the different investment proposals according to their return from the highest to the lowest. Thereafter, a cut-off point is selected for investment on the basis of the available funds. Investment proposals above the cut-off point will be selected for investment and investment proposals below the cut-off point will be rejected. 12.6 EVALUATION CRITERIA OF CAPITAL BUDGETING DECISIONS For evaluating the acceptability of a long-term investment proposal under Capital Budgeting technique, the following three steps are to be followed: Step 1: Determination of the future expected cash flows from the investment proposals. Step 2: Determination of the required expected rate of return by the investors (i.e., cost of capital). Step 3: Application of Capital Budgeting evaluation criteria to evaluate the investment proposals. 12.7 CONCEPT OF PRESENT VALUE (PV) In the context of Capital Budgeting technique of an investment decision, Present Value (PV) is the current value of a future receivable or payable amount. Such future receivable or payable amount expressed in the future price level is being converted at its PV with the help of a discounting factor or PV factor. Say, an income of Rs. 40,000 will be earned from a project after 4 years. This Rs. 40,000 represents the future value which will be earned after 4 years. This future value is to be converted at today’s value with the help of discounting or PV factor; and thus, the PV of Rs. 40,000 could be obtained. Hence, the PV is today’s value of a future income or expense. 12.8 DETERMINATION OF PV FACTOR OR DISCOUNTING FACTOR The Present Value of a future income or expense is to be determined with the help of a PV or discounting factor. This PV or discounting factor is to be computed with the help of the following formula: PV factor or Discounting factor of Re. 1 = 1/(1 + k)n Where, k = PV or Discounting rate and n = Future period of income or expense Thus, the PV of Re. 1 at 10% discounting rate for the next 3 years would be as follows: For the 1st year: PV = 1/(1 + 0.10)1 = Re. 0.909 For the 2nd year: PV = 1/(1 + 0.10)2 = Re. 0.826 For the 3rd year: PV = 1/(1 + 0.10)3 = Re. 0.751 Now, if Rs. 20,000, Rs. 30,000 and Rs. 40,000 are the values that will be earned after 1st, 2nd and 3rd years, respectively, at a 10% discounting rate, then their PVs would be as follows: PV of Rs. 20,000 = Rs. 20,000 × 0.909 = Rs. 18,180 PV of Rs. 30,000 = Rs. 30,000 × 0.826 = Rs. 24,780 PV of Rs. 40,000 = Rs. 40,000 × 0.751 = Rs. 30,040 12.9 CONCEPT OF DISCOUNTED CASH FLOW When cash is going out of the firm, it is called ‘cash outflow’ and when cash is coming into the firm, it is called ‘cash inflow.’ For an investment project proposal, the cash outflow refers to estimated amount of cash required to be invested into the project. On the other hand, in this context, the cash inflow refers to the expected amount of cash that would be earned from the project during its life. When any future expected cash inflow or outflow is being converted at its PV with the help of the PV or the discounting factor, such converted PV of cash inflow or outflow is called the ‘discounted cash flow.’ Hence, the discounted cash flow is the PV of the ‘future cash flow.’

Modified Date: Tue, Jul 06, 2010 12:32:27 PM

Output Date: Tue, Jul 06, 2010 06:05:04 PM

Rev II

Project: Management Accounting_Debarshi Bhattacharyya ACE Pro India Pvt. Ltd. File: X:\Pearson\Management Accounting_Debarshi Bhattacharyya\MAIN\M12\LAYOUT_M12\M12_DEBA_ISBN_EN_SE_C12.indd

746

MANAGEMENT ACCOUNTING

12.10 CAPITAL BUDGETING TECHNIQUES Capital Budgeting techniques are broadly classified into two categories: (i) non-discounted technique; and (ii) discounted technique. Each category is again classified under different methods as follows: i. Non-discounted techniques (i.e., without considering the time value of money): a. Pay Back Period (PBP) method. b. Pay Back Profitability method. c. Average Rate of Return (ARR) method. ii. Discounted cash-flow techniques (i.e., time-adjusted techniques): a. Discounted PBP method. b. Profitability Index (PI) method. c. Net Present Value (NPV) method. d. Internal Rate of Return (IRR) method. e. Terminal Value (TV) method. 12.10.1 Non-discounted Techniques (Without Considering the Time Value of Money) Non-discounted technique of Capital Budgeting refers to the technique or method of investment decision where it is considered that there is no change in the price level between the date of initial investment made and the date of last return from the investment. Under this technique, the future value of money is considered at the current value. 12.10.1.1 Pay Back Period (PBP) Method Pay Back Period refers to the period (generally number of years) of an investment project proposal at which the firm expects to recover its initial investment to the project proposal. More clearly, PBP is the period at which the total cash inflows from a project equals to the initial investment (i.e., cash outflows) made to the project. Computation of PBP i. When the Cash Inflows after Tax (CFAT) are constant every year: PBP =

Initial investment Constant CFAT p.a.

ii. When CFAT are not constant every year, the following steps are to be followed: Step 1: Calculate the CFAT of each year. Step 2: Compute the cumulative CFAT. Step 3: The time (i.e., year) when the cumulative CFAT becomes equal to the initial investment would be the PBP or else, use the method of interpolation. Decision-making criterion Under this method, the project having the shortest PBP should be undertaken. 12.10.1.2 Pay Back Profitability Method Pay Back Profitability is the amount of cash flows earned from a project after its Pay Back Period. More clearly, it is the excess of cash inflows from a project over the initial investment into the project. It represents the net cash inflows from the project proposal. Computation of Pay Back Profitability i. When the CFAT p.a. are constant: Pay Back Profitability = (Expected life of the project – PBP) × CFAT p.a. ii. When CFAT p.a. are not constant: Pay Back Profitability = Total CFAT – Initial investment

Modified Date: Tue, Jul 06, 2010 12:32:27 PM

Output Date: Tue, Jul 06, 2010 06:05:04 PM

Rev II

Project: Management Accounting_Debarshi Bhattacharyya ACE Pro India Pvt. Ltd. File: X:\Pearson\Management Accounting_Debarshi Bhattacharyya\MAIN\M12\LAYOUT_M12\M12_DEBA_ISBN_EN_SE_C12.indd

CAPITAL BUDGETING

747

Decision-making criterion Under this method, the project having the highest pay-back profitability should be undertaken. Note: Pay Back Profitability method is superior to PBP method. 12.10.1.3 Average or Accounting Rate of Return (ARR) Method Average or Accounting Rate of Return is the average rate of profit to be earned from a project proposal. It is expressed as a percentage of average Profit after Tax (PAT) earned from a project on the average investment made to the project. Under ARR method, net PAT expected to be earned from the project is being considered and not the cash inflow from the project. PAT = CFAT + Non-cash expenses like depreciation ARR =

Average annual Profit After Tax (i.e., avg. PAT) Average investment in the project

where Average PAT = Average investment =

Total PAT during the life of the project Life of the project in years (Initial investment – Scrap value) + Scrap value + Additional networking capital 2

Decision-making criterion Under this method, the project having the highest ARR should be undertaken. 12.10.2 Discounted Cash Flow Techniques (Time-adjusted Techniques) Discounted technique of Capital Budgeting refers to the technique or method of investment decision where the incidence of change in the price level between the date of initial investment made and the date of last return from the investment is being considered. In case of long-term investment, there must be a long time gap between the date of initial investment made and the dates of return from the investment. During this long time interval, the price level cannot remain constant. This change in the price level is considered under different discounted techniques of Capital Budgeting decision. Under this technique, future value of money is being converted at its PV with the help of PV or discounting factor. 12.10.2.1 Discounted Pay Back Period (Discounted PBP) Method Discounted PBP refers to the period (generally, number of years) of an investment project proposal at which the firm expects to recover its initial investment made to the project proposal after converting the future expected cash inflows from the project into their PVs. More clearly, the discounted PBP is the period at which the PV of the total cash inflows (i.e. Total PV of CFAT) from a project equals to the PV of the total cash outflows (i.e., the initial investment made to the project). Following steps are to be followed for determining the discounted PBP: Step 1: Calculate the CFAT of each year. Step 2: Multiply the CFAT of each year by the respective PV factor and get the discounted CFAT or PV of CFAT. Step 3: Compute the cumulative discounted CFAT. Step 4: Get the discounted PBP where the discounted CFAT is equal to the total investment. If such a figure (i.e., the value of total investment) cannot be found out from the discounted CFAT, then apply the method of interpolation. Decision-making criterion Under this method, the project having the shortest discounted PBP should be undertaken.

Modified Date: Tue, Jul 06, 2010 12:32:27 PM

Output Date: Tue, Jul 06, 2010 06:05:04 PM

Rev II

Project: Management Accounting_Debarshi Bhattacharyya ACE Pro India Pvt. Ltd. File: X:\Pearson\Management Accounting_Debarshi Bhattacharyya\MAIN\M12\LAYOUT_M12\M12_DEBA_ISBN_EN_SE_C12.indd

748

MANAGEMENT ACCOUNTING

12.10.2.2 Net Present Value (NPV) Method Net Present Value (NPV) is the difference between the PV of the total cash inflows from a project and the PV of the total cash outflows for the project. NPV = Total of PV of cash inflows – Total of PV of cash outflows (i.e., the total of discounted cash inflows – the total of discounted cash outflows) Following steps are to be followed for determining the NPV: Step 1: Calculate the CFAT of each year. Step 2: Multiply the CFAT of each year by the respective PV factor and get the discounted CFAT or PV of CFAT. Step 3: Compute the cumulative discounted CFAT. Step 4: Compute the PV of cash outflows. Step 5: Calculate the NPV by deducting the PV of the total cash outflows from the PV of the total cash inflows. Decision-making criterion i. In case of a single project proposal, accept it if NPV > 0; else, reject it. ii. In case of two mutually exclusive project proposals, accept the project having the higher NPV. 12.10.2.3 Profitability Index (PI) Method Profitability Index (PI) is the ratio of the PV of the total cash inflows from a project and the PV of the total cash outflows for the project. PI is also called the ‘benefit-cost ratio.’ PI =

PV of total cash inflows PV of total cash outflows

Decision-making criterion i. If PI > 1 — Accept the project proposal. ii. If PI < 1 — Reject the project proposal. iii. If PI = 1 — The management may be indifferent in accepting or rejecting the project proposal. Generally, the project proposal is rejected in such a case as the firm does not get the net benefit from it. 12.10.2.4 Internal Rate-of-Return (IRR) Method Internal Rate of Return (IRR) is the rate of return which equates the PV of the total cash inflows with the PV of the total cash outflows. Therefore, IRR is the rate of return (i.e., the discounting factor) which makes the NPV zero. At the IRR, PV of total cash inflows = PV of total cash outflows. Hence, PI = 1. Decision-making criterion i. In case of a single project proposal, accept it if the IRR exceeds the cost of capital. ii. In case of two mutually exclusive project proposals, accept the project having the higher IRR. Note: NPV method is superior to IRR method. 12.10.2.5 Terminal Value (TV) Method Net Terminal Value (NTV) refers to the difference between the total PV of the compounded reinvested cash inflows from a project and the PV of the cash outflows for the project. Under this method, the cash inflows from a project proposal are compounded forward rather than discounting them backward as in the case of NPV method. Here, it is assumed that each cash inflow from a project is reinvested in another project at a

Modified Date: Tue, Jul 06, 2010 12:32:27 PM

Output Date: Tue, Jul 06, 2010 06:05:04 PM

Rev II

Project: Management Accounting_Debarshi Bhattacharyya ACE Pro India Pvt. Ltd. File: X:\Pearson\Management Accounting_Debarshi Bhattacharyya\MAIN\M12\LAYOUT_M12\M12_DEBA_ISBN_EN_SE_C12.indd

CAPITAL BUDGETING

749

certain rate of interest, till the termination of the project; and thus, the cash inflows from the project are compounded forward. These compounded values are then discounted at an appropriate discounting rate (generally at the rate of cost of capital for the project) to get the PV of reinvested cash flows, as follows. NTV = Total PV of compounded reinvested cash inflows – Total PV of cash outflows. Decision-making criterion i. In case of a single project proposal, accept it if the total PV of the compounded reinvested cash inflows is greater than the PV of the cash outflows. Otherwise, reject the project proposal. ii. In case of two mutually exclusive project proposals, accept the project having the higher total PV of the compounded reinvested cash inflows. Tutorial Notes to Students for Solving Problems i. Firstly, students are advised to go through the problem very carefully. ii. Except accounting or ARR method, all other Capital Budgeting techniques require CFAT. In case of ARR alone, PAT is required. iii. Profit before Tax (PBT) = Profit before depreciation and tax (PBDT) – Depreciation. PAT = PBT – Tax. CFAT = PAT + Depreciation. iv. PAT = CFAT – Depreciation. v. Students are advised to use PV table for getting the required PV or discounting factor, if not given in the problem.

12.11 WORKED-OUT PROBLEMS Problem 1 The cost of a project is estimated at Rs. 5,20,000 with an estimated life of 5 years, at the end of which the project would realize Rs. 20,000 as scrap. Annual PBDT from the project is estimated at Rs. 2,00,000. Calculate the annual cash flow from the project assuming the tax rate at 40%. Solution Statement showing computation of Annual Cash Flow from the project

Less: Less: Add:

Annual PBDT as given Depreciation [(Rs. 5,20,000 – Rs. 20,000)/5] Annual PBT Tax @40% on PBT Annual PAT Non-cash Items Charged to the Profit & Loss A/c: Depreciation Annual CFAT

Rs. 2,00,000 1,00,000 1,00,000 40,000 60,000 1,00,000 1,60,000

Problem 2 From the following information, calculate the PAT and annual CFAT for the year: Rs. 1,50,000 5,00,000 4,50,000

Cash Sales Credit Sales Collection from Credit Sales

(Continued)

Modified Date: Tue, Jul 06, 2010 12:32:27 PM

Output Date: Tue, Jul 06, 2010 06:05:04 PM

Rev II

Project: Management Accounting_Debarshi Bhattacharyya ACE Pro India Pvt. Ltd. File: X:\Pearson\Management Accounting_Debarshi Bhattacharyya\MAIN\M12\LAYOUT_M12\M12_DEBA_ISBN_EN_SE_C12.indd

750

MANAGEMENT ACCOUNTING

Rs. 2,00,000 1,20,000 20,000 @40%

Cash Expenses Debited to the Profit & Loss A/c Non-cash Expenses Debited to the Profit & Loss A/c Other Non-cash Income Credited to the Profit & Loss A/c Tax Rate

Solution Statement showing computation of PAT and CFAT for the year

Cash Sales Credit Sales Collection from Credit Sales Less:

Cash Expenses Non-cash Expenses

Add:

Other Non-cash Incomes PBT Cash Flow before Tax (CFBT) Tax @40% on PBT PAT CFAT

Less:

Profit Rs. 1,50,000 5,00,000 – 6,50,000 (2,00,000) (1,20,000) 3,30,000 20,000 3,50,000 1,40,000 2,10,000

Cash Flow Rs. 1,50,000 – 4,50,000 6,00,000 (2,00,000) – 4,00,000 – 4,00,000 1,40,000 2,60,000

Problem 3 Following are the details of two mutually exclusive project proposals of a company: Project D Rs. 2,90,000 30,000 1,00,000 40,000 1,80,000 20,000 40,000

Cost of Plant Installation Charges of Plant Cost of Building Working Capital Requirement Annual PBDT Scrap Value of Plant Disposal Value of Building

Project S Rs. 4,70,000 60,000 2,00,000 75,000 2,50,000 30,000 70,000

Life of each project is 10 years and depreciation has been charged on fixed assets under straight line basis. Income tax rate is 40%. Determine (a) Total cash outflow for each project; (b) Total cash inflow from each project; and (c) Terminal cash inflow from each project. Solution i. Determination of Total Cash Outflow for the projects Project D Rs. 2,90,000 30,000

Cost of Plant Installation Cost for Plant

Project S Rs. 4,70,000 60,000 (Continued)

Modified Date: Tue, Jul 06, 2010 12:32:27 PM

Output Date: Tue, Jul 06, 2010 06:05:04 PM

Rev II

Project: Management Accounting_Debarshi Bhattacharyya ACE Pro India Pvt. Ltd. File: X:\Pearson\Management Accounting_Debarshi Bhattacharyya\MAIN\M12\LAYOUT_M12\M12_DEBA_ISBN_EN_SE_C12.indd

CAPITAL BUDGETING

Cost of Building Working Capital Requirement Total Cash Outflow

751

Project D Rs. 1,00,000 40,000 4,60,000

Project S Rs. 2,00,000 75,000 8,05,000

Project D Rs. 1,80,000

Project S Rs. 2,50,000

(30,000)

(50,000)

(10,000)

(20,000)

1,40,000 56,000 84,000 30,000 10,000 1,24,000 11,24,000

1,80,000 72,000 1,08,000 50,000 20,000 1,78,000 17,80,000

Project D Rs. 40,000 20,000 40,000 1,00,000

Project S Rs. 75,000 30,000 70,000 1,75,000

ii. Determination of Total Cash Inflow from the projects

Less:

Less: Add:

PBDT Depreciation: On Plant [(2,90,000 + 30,000 – 20,000) ÷ 10] [(4,70,000 + 60,000 – 30,000) ÷ 10] On Building [1,00,000 ÷ 10] [2,00,000 ÷ 10] PBT Tax @40% on PBT PAT Depreciation Added Back: On Plant On Building Annual CFAT Total CFAT [CFAT × 10]

iii. Determination of Terminal Cash Inflow from the projects

Recovery of Working Capital Salvage Value of Plant Disposal Value of Building Terminal Cash Inflow

Problem 4 Project X requires an initial investment of Rs. 5,00,000, whereas project Y requires an initial investment of Rs. 6,00,000. Project X gives a CFAT of Rs. 1,25,000 p.a. over its estimated economic life of 6 years, whereas project Y gives a CFAT of Rs. 1,00,000 p.a. for 10 years. i. Compute the PBP of each project. ii. Compute the Pay Back Profitability of each project. iii. Which project should be accepted under these methods? iv. What is your ultimate suggestion as regards to the acceptance of a project? Solution Determination of PBP and Pay Back Profitability of Project X and Project Y (a) Initial Investment (Rs.) (b) CFAT p.a. (Rs.)

Project X 5,00,000 1,25,000

Project Y 6,00,000 1,00,000 (Continued)

Modified Date: Tue, Jul 06, 2010 12:32:27 PM

Output Date: Tue, Jul 06, 2010 06:05:04 PM

Rev II

Project: Management Accounting_Debarshi Bhattacharyya ACE Pro India Pvt. Ltd. File: X:\Pearson\Management Accounting_Debarshi Bhattacharyya\MAIN\M12\LAYOUT_M12\M12_DEBA_ISBN_EN_SE_C12.indd

752

MANAGEMENT ACCOUNTING

(c) Estimated Economic Life of the Project (i) PBP (a ÷ b) [Initial Investment/CFAT p.a.] (ii) Pay Back Profitability [CFAT p.a. × (Estimated life − PBP)

Project X 6 years 5,00,000 ÷ 1,25,000 = 4 years 1,25,000 × (6 – 4) = Rs. 2,50,000

Project Y 10 years 6,00,000 ÷ 1,00,000 = 6 years 1,00,000 × (10 – 6) = Rs. 4,00,000

iii. On the basis of PBP, project X should be accepted as its PBP is shorter (i.e., 6 years) than that of project Y (i.e., 10 years). On the other hand, on the basis of pay-back profitability, project Y should be accepted as its Pay Back Profitability is more (i.e., Rs. 4,00,000) than that of project X (i.e., Rs. 2,50,000). iv. As Pay Back Profitability method is more effective to a business concern than PBP method, project Y should be accepted here, as it is recommended as per Pay Back Profitability method. Problem 5 A company requests you to compute the PBP and Pay Back Profitability of project Z from the following information: Initial Investment Year

Rs. 27,00,000 CFAT (Rs.) 4,00,000 4,40,000 5,00,000 6,00,000 7,60,000 8,00,000 10,00,000 12,00,000

1 2 3 4 5 6 7 8

What would have been your answer had the initial investment been Rs. 30,00,000 instead of Rs. 27,00,000? Stop and Think In the given problem, CFAT from the project are not constant every year. Hence, in such a case, the cumulative CFAT are to be calculated adding every year’s CFAT. The year in which the cumulative CFAT would be equal to the initial investment is the PBP for the project. If the cumulative CFAT from the project is not exactly equal to the initial investment, then the PBP for the project is to be calculated with the help of either the ‘forward method’ or the ‘backward method’ of ‘interpolation.’

Solution Calculation of Cumulative CFAT Year 1 2 3 4 5 6 7 8

Modified Date: Tue, Jul 06, 2010 12:32:27 PM

CFAT (Rs.) 4,00,000 4,40,000 5,00,000 6,00,000 7,60,000 8,00,000 10,00,000 12,00,000

Output Date: Tue, Jul 06, 2010 06:05:04 PM

Cumulative CFAT (Rs.) 4,00,000 8,40,000 13,40,000 19,40,000 27,00,000 35,00,000 45,00,000 57,00,000

Rev II

Project: Management Accounting_Debarshi Bhattacharyya ACE Pro India Pvt. Ltd. File: X:\Pearson\Management Accounting_Debarshi Bhattacharyya\MAIN\M12\LAYOUT_M12\M12_DEBA_ISBN_EN_SE_C12.indd

753

CAPITAL BUDGETING

Here, the cumulative CFAT is equal to the initial investment of Rs. 27,00,000 of the 5th year. ∴ PBP of project Z = 5 years. ∴ Pay Back Profitability of project Z = Total of CFAT – Initial investment = Rs. 57,00,000 – Rs. 27,00,000 = Rs. 30,00,000 Had the initial investment been Rs. 30,00,000, instead of Rs. 27,00,000, then: From the above calculation, it could be observed that the initial investment of Rs. 30,00,000 lies in between the cumulative CFAT of Rs. 27,00,000 and Rs. 35,00,000, which would be recovered in between the period of 5th and 6th year. In such a situation, PBP is to be calculated with the help of either the forward method or backward method of interpolation as used as follows: Using Forward Method of Interpolation: PBP of project Z = 5 years + [{(3000000 – 2700000)/(3500000 – 2700000)} × 1] = 5 + 0.375 = 5.375 years Using Backward Method of Interpolation: PBP of project Z = 6 years – [{(3500000 – 3000000)/(3500000 – 2700000)} × 1] = 6 – 0.625 = 5.375 years Then, the Pay-back profitability of project Z = Total of CFAT – Initial investment = Rs. 57,00,000 – Rs. 30,00,000 = Rs. 27,00,000 Problem 6 Compute the PBP for the Project End of the Year Book Value of Fixed Assets (Rs. in lakhs) PAT (Rs. in lakhs)

1 90 20

2 80 22

3 70 24

4 60 26

5 50 28

[B.Com. (Hons), Calcutta University—2006] Solution Here, PAT for useful years are given, but for the purpose of computation of PBP, CFAT for the useful years are needed. Therefore, CFAT of the useful years are to be calculated first for the purpose. For ascertaining the CFAT from PAT, non-cash expenses like depreciation and so on are to be added to the PAT. ∴ CFAT = PAT + Depreciation Here, Depreciation on the fixed assets for each year = 10 (Rs. in lakhs), as the book values of fixed assets are reducing by 10 (Rs. in lakhs) every year. ∴ Initial investment in the fixed assets at the beginning of the 1st year = 90 + 10 = 100 (Rs. in lakhs). Year 1 2 3 4 5

Profit After Tax (Rs. Lakhs) 20 22 24 26 28

Depreciation (Rs. Lakhs) 10 10 10 10 10

Net CFAT (Rs. Lakhs) 30 32 34 36 38

Cumulative Net CFAT (Rs. Lakhs) 30 62 96 132 170

From the above calculation, it has been observed that the initial investment of 100 (Rs. in lakhs) lies in between the cumulative CFAT of 96 (Rs. in lakhs) and 132 (Rs. in lakhs), which could be recovered in between

Modified Date: Tue, Jul 06, 2010 12:32:27 PM

Output Date: Tue, Jul 06, 2010 06:05:04 PM

Rev II

Project: Management Accounting_Debarshi Bhattacharyya ACE Pro India Pvt. Ltd. File: X:\Pearson\Management Accounting_Debarshi Bhattacharyya\MAIN\M12\LAYOUT_M12\M12_DEBA_ISBN_EN_SE_C12.indd

754

MANAGEMENT ACCOUNTING

the period of 3rd and 4th year. In such a situation, PBP is to be calculated with the help of either the forward method or the backward method of interpolation as used as follows: Using Forward Method of Interpolation: PBP of fixed assets = 3 years + [{(100 – 96)/(132 – 96)} × 1] = 3 + 0.11 = 3.11 years Problem 7 A project requires an initial investment of Rs. 4,00,000. The estimated life of the project is 5 years and its estimated scrap value at the end of its life would be Rs. 80,000. Additional working capital required for the project is Rs. 20,000. The expected annual profits before depreciation and tax from the project in 5 years would be Rs. 60,000, Rs. 1,40,000, Rs. 1,60,000, Rs. 1,60,000 and Rs. 40,000, respectively. If the tax rate is 40% and depreciation is charged under the straight-line basis, calculate the ARR of the project. If the minimum required rate of return is 10%, should the project be accepted? Solution Calculation of Average Annual PAT

Less: Less:

Average Annual PBDT [(60,000 + 1,40,000 + 1,60,000 + 1,60,000 + 40,000) ÷ 5] Depreciation per Year [(4,00,000 – 80,000) ÷ 5] Average Annual PBT Tax @ 40% on PBT Average Annual PAT

Rs. 1,12,000 64,000 48,000 19,200 28,800

Calculation of Average Investment Average investment = {(Initial investment – Salvage value) ÷ 2} + Salvage value + Additional working capital = {(4,00,000 – 80,000) ÷ 2} + 80,000 + 20,000 = Rs. 2,60,000 \ Average Rate of Return (ARR) = (Average annual PAT ÷ Average investment) × 100 = (28,800 ÷ 2,60,000) × 100 = 11.077% As the ARR (i.e., 11.077%) is higher than the minimum required rate of return (i.e., 10%), the project should be accepted. Problem 8 Raj and Co. intends to invest Rs. 10 lakhs in a project having a life of 4 years. The cash inflows from the project at the end of 1st year to 4th year are expected to be Rs. 3,00,000, Rs. 4,20,000, Rs. 4,00,000 and Rs. 3,30,000 before charging depreciation and tax. You are required to calculate the accounting rate of return of the project and comment on the use of the rate of return. [B.Com. (Hons), Calcutta University—2007] Solution Calculation of Average Annual PAT

Less:

Average Annual Profit before Depreciation & Tax (PBDT) [(3,00,000 + 4,20,000 + 4,00,000 + 3,30,000) ÷ 4] Depreciation per Year [10,00,000 ÷ 4]

Rs. 3,62,500 2,50,000 (Continued)

Modified Date: Tue, Jul 06, 2010 12:32:27 PM

Output Date: Tue, Jul 06, 2010 06:05:04 PM

Rev II

Project: Management Accounting_Debarshi Bhattacharyya ACE Pro India Pvt. Ltd. File: X:\Pearson\Management Accounting_Debarshi Bhattacharyya\MAIN\M12\LAYOUT_M12\M12_DEBA_ISBN_EN_SE_C12.indd

CAPITAL BUDGETING

Less:

755

Rs. 1,12,500 56,250 56,250

Average Annual PBT Tax @50% on PBT Average Annual PAT

Calculation of Average Investment Average investment = {(Initial investment – Salvage value) ÷ 2} + Salvage value + Additional working capital = {(10,00,000 – 0) ÷ 2} + 0 + 0 = Rs. 5,00,000 ∴Accounting Rate of Return (ARR) = (Average annual PAT ÷ Average investment) × 100 = (56,250 ÷ 5,00,000) × 100 = 11.25% If the rate of cost of capital for the proposed project is less than the ARR (i.e., 11.25%), then the project proposal should be accepted; otherwise, it should be rejected. Thus, the ARR is used for assessing the profit worthiness of the proposed project through its comparison with the rate of cost of capital for the project. Problem 9 Determine the ARR from the following information as furnished in respect of two machines and comment on the results: Cost (Rs.) Estimated CFAT (Rs.): 1st Year 2nd Year 3rd Year 4th Year 5th Year Estimated Life (Years) Estimated Salvage Value (Rs.) Additional Investment in Net Working Capital (Rs.) Rate of Tax (%)

Machine X 2,20,000

Machine Y 2,20,000

40,000 60,000 90,000 1,20,000 1,60,000 4,70,000 5 20,000 28,000 50

1,00,000 1,20,000 1,50,000 1,10,000 80,000 5,60,000 5 40,000 30,000 50

Depreciation has been charged on the straight-line basis. Solution ARR = (Average annual PAT ÷ Average investment) × 100 Now, the Average annual PAT = Total PAT during the estimated life of the project ÷ Estimated life of the project in years. And, Average investment = {(Initial investment – Salvage value) ÷ 2} + Salvage value + Additional working capital requirement. Here, CFAT for the expected useful years are given, but for the purpose of computation of ARR, PAT for the expected useful years are needed. Therefore, PAT for the expected useful years are to be calculated first for the purpose. For ascertaining the PAT from CFAT, non-cash expenses like depreciation and so on are to be deducted from the given CFAT.

Modified Date: Tue, Jul 06, 2010 12:32:27 PM

Output Date: Tue, Jul 06, 2010 06:05:04 PM

Rev II

Project: Management Accounting_Debarshi Bhattacharyya ACE Pro India Pvt. Ltd. File: X:\Pearson\Management Accounting_Debarshi Bhattacharyya\MAIN\M12\LAYOUT_M12\M12_DEBA_ISBN_EN_SE_C12.indd

756

MANAGEMENT ACCOUNTING

For Machine X: Calculation of Average Annual PAT

Add: Less: Less:

Average Annual CFAT [Rs. 4,70,000 ÷ 5] Tax Deducted @50% [50/50 × CFAT] Cash Flow before Tax (CFBT) Depreciation per year [(2,20,000 – 20,000) ÷ 5] Average Annual PBT Tax @50% on PBT Average Annual PAT

Rs. 94,000 94,000 1,88,000 40,000 1,48,000 74,000 74,000

Calculation of Average Investment Average investment = {(Initial investment – Salvage value) ÷ 2} + Salvage value + Additional working capital = {(2,20,000 – 20,000) ÷ 2} + 20,000 + 28,000 = Rs. 1,48,000 \ Average Rate of Return (ARR) = (Average annual PAT ÷ Average investment) × 100 = (74,000 ÷ 1,48,000) × 100 = 50% For Machine Y: Calculation of Average Annual PAT

Add: Less: Less:

Average Annual CFAT [Rs. 5,60,000 ÷ 5] Tax Deducted @50% [50/50 × CFAT] Cash Flow before Tax (CFBT) Depreciation per Year [(2,20,000 – 40,000) ÷ 5] Average Annual PBT Tax @50% on PBT Average Annual PAT

Rs. 1,12,000 1,12,000 2,24,000 45,000 1,79,000 89,500 89,500

Calculation of Average Investment Average investment = {(Initial investment – Salvage value) ÷ 2} + Salvage value + Additional working capital = {(2,20,000 – 40,000) ÷ 2} + 40,000 + 30,000 = Rs. 1,60,000 \ Average Rate of Return (ARR) = (Average annual PAT ÷ Average investment) × 100 = (89,500 ÷ 1,60,000) × 100 = 55.9375% From the above calculation, it has been observed that ARR for machine Y is higher than that of the machine X. Hence, the investment proposal in machine Y is more profitable as it gives a higher return than machine X. Problem 10 Using the following information, compute the PBP under discounted PBP method and comment. Initial Outlay Estimated Life PAT: End of Year 1 2 3 4 5

Modified Date: Tue, Jul 06, 2010 12:32:27 PM

Rs. 80,000 5 years Rs. 6,000 Rs. 14,000 Rs. 24,000 Rs. 16,000 Nil

Output Date: Tue, Jul 06, 2010 06:05:04 PM

Rev II

Project: Management Accounting_Debarshi Bhattacharyya ACE Pro India Pvt. Ltd. File: X:\Pearson\Management Accounting_Debarshi Bhattacharyya\MAIN\M12\LAYOUT_M12\M12_DEBA_ISBN_EN_SE_C12.indd

757

CAPITAL BUDGETING

Depreciation has been calculated under straight-line method. The cost of capital may be taken at 20% p.a. and the PV of Re. 1 at 20% p.a. is given as follows: Year PV Factor

1

2

3

4

5

0.83

0.69

0.58

0.48

0.40

[B.Com. (Hons), Calcutta University—2008] Solution Here, PAT for useful years are given, but for the purpose of computation of PBP, CFAT for the useful years are needed. Therefore, CFAT of useful years are to be calculated first for the purpose. For ascertaining CFAT from PAT, non-cash expenses like depreciation and so on are to be added to the PAT. ∴ CFAT = PAT + Depreciation Here, Depreciation on the project per year = Rs. 80,000 ÷ 5 = Rs. 16,000 Year 1 2 3 4 5

PAT (a) Rs. 6,000 14,000 24,000 16,000 0

Depreciation (b) Rs. 16,000 16,000 16,000 16,000 16,000

CFAT (c = a + b) Rs. 22,000 30,000 40,000 32,000 16,000

PV Factor @20% (d) 0.83 0.69 0.58 0.48 0.40

PV of CFAT (c × d) Rs. 18,260 20,700 23,200 15,360 6,400

Cumulative PV of CFAT (Rs.) 18,260 38,960 62,160 77,520 83,920

From the above calculation, it has been observed that the initial investment of Rs. 80,000 lies in between the cumulative PV of CFAT of Rs. 77,520 and Rs. 83,920, which could be recovered in between the period of 4th and 5th year. In such a situation, the discounted PBP is to be calculated with the help of either the forward method or backward method of interpolation as used as follows: Using Forward Method of Interpolation: Discounted PBP of the project = 4 years + [{(80,000 – 77,520) ÷ (83,920 – 77,520)} × 1] = 4 + 0.3875 = 4.3875 years Problem 11 A company has a proposal of investing Rs. 2,00,000 into a machinery, the estimated useful life of which is 5 years. The expected output from the machine during its estimated useful life are as follows: Year Output (units)

1 10,000

2 15,000

3 20,000

4 17,000

5 12,000

The selling price per unit of the product is Rs. 10. The variable cost to sales ratio is 40%. The fixed operating cost other than depreciation is Rs. 20,000. The estimated scrap value of the machine at the end of its useful life is Rs. 20,000. The cost of capital of the project is 10%. Evaluate the investment proposal using NPV method. The PVs of Re. 1 at a 10% discounting rate are as follows: Year PV Factor @10% p.a.

Modified Date: Tue, Jul 06, 2010 12:32:27 PM

1 0.909

2 0.826

Output Date: Tue, Jul 06, 2010 06:05:04 PM

3 0.751

4 0.683

5 0.621

Rev II

Project: Management Accounting_Debarshi Bhattacharyya ACE Pro India Pvt. Ltd. File: X:\Pearson\Management Accounting_Debarshi Bhattacharyya\MAIN\M12\LAYOUT_M12\M12_DEBA_ISBN_EN_SE_C12.indd

758

MANAGEMENT ACCOUNTING

Solution Calculation of CFAT from the project

Less: Less:

Less: Less: Add: Add:

Year Sales (Rs. 10 × units) Variable Cost (40% of Sales) Contribution Operating Fixed Cost other than Depreciation PBDT Depreciation [(200000 – 20000) ÷ 5] PBT Tax @50% PAT Depreciation Scrap Realization CFAT

1 1,00,000 40,000 60,000 20,000

2 1,50,000 60,000 90,000 20,000

3 2,00,000 80,000 1,20,000 20,000

4 1,70,000 68,000 1,02,000 20,000

5 1,20,000 48,000 72,000 20,000

40,000 36,000 4,000 2,000 2,000 36,000 – 38,000

70,000 36,000 34,000 17,000 17,000 36,000 – 53,000

1,00,000 36,000 64,000 32,000 32,000 36,000 – 68,000

82,000 36,000 46,000 23,000 23,000 36,000 – 59,000

52,000 36,000 16,000 8,000 8,000 36,000 20,000 64,000

Calculation of NPV of the project Year 1 2 3 4 5 Less:

CFAT (Rs.) (a) 38,000 53,000 68,000 59,000 64,000 Total PV of CFAT Cash Outflows NPV

PV Factor @10% (b) 0.909 0.826 0.751 0.683 0.621

PV of CFAT (Rs.) (a × b) 34,542 43,778 51,068 40,297 39,744 2,09,429 2,00,000 9,429

From the above calculation, it is observed that the NPV is + Rs. 9,429. As the NPV is greater than zero, the project proposal should be accepted. Problem 12 Following are the details of two mutually exclusive project proposals of a company: Project R Rs. in Lakhs 700 100 200 300 450 600

Project Cost Cash Inflows: Year 1 Year 2 Year 3 Year 4 Year 5

Project S Rs. in Lakhs 700 500 400 200 100 100

Assume no residual values at the end of the year. The company’s cost of capital is 10%. You are asked to calculate the following for each of the two projects: i. NPV using a 10% discounting rate. ii. PI. Suggest which project proposal should be accepted by the company.

Modified Date: Tue, Jul 06, 2010 12:32:27 PM

Output Date: Tue, Jul 06, 2010 06:05:04 PM

Rev II

Project: Management Accounting_Debarshi Bhattacharyya ACE Pro India Pvt. Ltd. File: X:\Pearson\Management Accounting_Debarshi Bhattacharyya\MAIN\M12\LAYOUT_M12\M12_DEBA_ISBN_EN_SE_C12.indd

CAPITAL BUDGETING

759

The PVs of Re. 1 at a 10% discounting rate are as follows: Year

1 0.909

PV Factor @10% p.a.

2 0.826

3 0.751

4 0.683

5 0.621

Solution i. Computation of NPV Year 1 2 3 4 5 Less:

PV Factor @10% (a)

Project R CFAT (b) Rs. Lakhs

0.909 0.826 0.751 0.683 0.621 Total PV of CFAT Total Cash Outflows NPV

100 200 300 450 600

PV of CFAT (a × c) Rs. Lakhs 90.90 165.20 225.30 307.35 372.60 1,161.35 700.00 461.35

Project S CFAT (c) Rs. Lakhs 500 400 200 100 100

PV of CFAT (a × b) Rs. Lakhs 454.50 330.40 150.20 68.30 62.10 1,065.50 700.00 365.50

ii. Computation of PI PI = Total PV of cash inflows/Total PV of cash outflows. For Project R: PI = 1,161.35 (Rs. in lakhs)/700 (Rs. in lakhs) = 1.659 For Project S: PI = 1,065.50 (Rs. in lakhs)/700 (Rs. in lakhs) = 1.522 Comment From the above calculation, it is transpired that both NPV and PI are higher in the case of project R than in the case of project S. As project R shows higher values under both the methods, it should be accepted by the company. Problem 13 R Ltd is presently considering two machines for a possible purchase. Other details related to the machines are as follows: Purchase Price (Rs.) Estimated Life Method of Depreciation Estimated Scrap Value (Rs.) CFBDT (Rs.): 1st Year 2nd Year 3rd Year 4th Year

Machine 1 50,000 4 years Straight line Nil

Machine 2 60,000 4 years Straight line Nil

25,000 25,000 25,000 25,000

45,000 19,000 25,000 25,000

Rate of tax is 40%. Compute the NPV of each machine assuming the cost of capital is 8%. Which machine the company should buy? The PVs of Re. 1 at 8% are as follows: Y1 – 0.926; Y2 – 0.857; Y3 – 0.794; and Y4 – 0.735 [B.Com. (Hons), Calcutta University—2007]

Modified Date: Tue, Jul 06, 2010 12:32:27 PM

Output Date: Tue, Jul 06, 2010 06:05:04 PM

Rev II

Project: Management Accounting_Debarshi Bhattacharyya ACE Pro India Pvt. Ltd. File: X:\Pearson\Management Accounting_Debarshi Bhattacharyya\MAIN\M12\LAYOUT_M12\M12_DEBA_ISBN_EN_SE_C12.indd

760

MANAGEMENT ACCOUNTING

Solution Here, the Cash Flow before Depreciation and Tax (CFBDT) is given, but for the purpose of computation of NPV, CFAT is needed. So, to get the CFAT from the CFBDT, the following adjustments are to be made: CFAT = PAT + Depreciation Again, PAT = CFBDT – Depreciation – Tax Here, depreciation per year is: For Machine 1 = Rs. 50,000 ÷ 4 = Rs. 12,500 For Machine 2 = Rs. 60,000 ÷ 4 = Rs. 15,000 Computation of NPV of Machine 1 Year 1 2 3 4 Less:

CFBDT (a) Rs.

Depreciation (b) Rs.

25,000 12,500 25,000 12,500 25,000 12,500 25,000 12,500 Total PV of Cash Inflows Total Cash Outflows NPV

PBT (c = a – b) Rs. 12,500 12,500 12,500 12,500

Tax @40% (d) Rs. 5,000 5,000 5,000 5,000

PAT (e = c − d) Rs. 7,500 7,500 7,500 7,500

CFAT (f = e + b) Rs. 20,000 20,000 20,000 20,000

PV Factor (g) 0.926 0.857 0.794 0.735

PV of CFAT (f × g) Rs. 18,520 17,140 15,880 14,700 66,240 50,000 16,240

Computation of NPV of Machine 2 Year 1 2 3 4 Less:

CFBDT (a) Rs.

Depreciation (b) Rs.

45,000 15,000 19,000 15,000 25,000 15,000 27,000 15,000 Total PV of Cash Inflows Total Cash Outflows NPV

PBT (c = a – b) Rs. 30,000 4,000 10,500 12,200

Tax @40% (d) Rs. 12,000 1,600 4,000 7,200

PAT (e = c − d) Rs. 18,000 2,400 6,000 7,200

CFAT (f = e + b) Rs. 33,000 17,400 21,000 22,000

PV Factor (g) 0.926 0.857 0.794 0.735

PV of CFAT (f × g) Rs. 30,558 14,912 16,674 16,317 78,461 60,000 18,461

Comment Here, both the machines are having positive NPV and thus qualify for acceptance of both the investment proposals. But, if these two investment proposals are mutually exclusive, then the investment in machine 2 should be chosen as the NPV of machine 2 is higher than that of machine 1. Problem 14 The following information is available in respect of a project proposal: Initial Outlay Estimated Life Cash Inflows p.a. Cost of Capital

Rs. 1,00,000 4 years Rs. 50,000 10%

Expected interest rates at which the cash inflows will be reinvested: End of Year 1 2 3 4

Modified Date: Tue, Jul 06, 2010 12:32:27 PM

Rate of Interest 6% 7% 8% 9%

Output Date: Tue, Jul 06, 2010 06:05:04 PM

Rev II

Project: Management Accounting_Debarshi Bhattacharyya ACE Pro India Pvt. Ltd. File: X:\Pearson\Management Accounting_Debarshi Bhattacharyya\MAIN\M12\LAYOUT_M12\M12_DEBA_ISBN_EN_SE_C12.indd

CAPITAL BUDGETING

761

You are asked to evaluate the project proposal using Terminal Value method. Compounded value of Re. 1: At a 6% rate for 3 years – 1.191. At a 7% rate for 2 years – 1.145. At a 8% rate for 1 year – 1.080. At a 9% rate for 0 years – 0.000. PV of Re. 1 at a 10% discounting factor for 4 years – 0.683. Solution Statement showing computation of compounded values of the Reinvested Cash Inflows Year

Cash Inflows Rs.

1 2 3 4

50,000 50,000 50,000 50,000

Rate of Interest 6% 7% 8% 9%

Period of Investment Years 3 2 1 0

Compounding Factor 1.191 1.145 1.080 1.000

Compounded Value Rs. 59,550 57,250 54,000 50,000 2,20,800

Now, the PV of the total compounded value of the reinvested cash inflows is to be computed at a 10% discounting factor (i.e., at the rate of the cost of capital) for 4 years. The PV of the total compounded value of the reinvested cash inflows at the 10% discounting factor for 4 years = Total compounded value of the reinvested cash inflows × PV of Re. 1 at 10% for 4 years = Rs. 2,20,800 × 0.6830 = Rs. 1,50,806 Net Terminal Value (NTV) = PV of the total compounded value of the reinvested cash inflows – PV of the total cash outflows = Rs. 1,50,806 – Rs. 1,00,000 = Rs. 50,806 As the PV of the total compounded value of the reinvested cash inflows is greater than the PV of the total cash outflows (thus, NTV becomes positive), the given project proposal should be accepted. Problem 15 A company proposes to install a machine involving a capital cost of Rs. 3,60,000. The life of the machine is 5 years and its salvage value at the end of the life is nil. The machine will produce the net operating income after a depreciation of Rs. 68,000 p.a. The company’s tax rate is 45%. The NPV factors for 5 years are as follows: Year

1 14 3.43

Discounting Rate Cumulative PV Factor

2 15 3.35

3 16 3.27

4 17 3.20

5 18 3.13

You are required to calculate the IRR of the proposal. [B.Com. (Hons), Calcutta University—2002] Solution

Less: Add:

Computation of Annual CFAT

Net Operating Income after Depreciation Tax @45% PAT Depreciation [Rs. 3,60,000 ÷ 5] Annual CFAT

Modified Date: Tue, Jul 06, 2010 12:32:27 PM

Output Date: Tue, Jul 06, 2010 06:05:04 PM

Rs. 68,000 30,600 37,400 72,000 1,09,400

Rev II

Project: Management Accounting_Debarshi Bhattacharyya ACE Pro India Pvt. Ltd. File: X:\Pearson\Management Accounting_Debarshi Bhattacharyya\MAIN\M12\LAYOUT_M12\M12_DEBA_ISBN_EN_SE_C12.indd

762

MANAGEMENT ACCOUNTING

Calculation of Internal Rate of Return (IRR) Internal Rate of Return is the rate of return which equates the total PV of cash inflows with the total PV of cash outflows, that is, which makes the NPV zero. To find out that discounting factor, let us start the solution on a trial-and-error basis. Firstly, let us calculate the average PBP for obtaining a quotient as follows: Average PBP = Initial investment/Average annual CFAT = Rs. 3,60,000/Rs. 1,09,400 = 3.29 From the given PV factor of the annuity table for 5 years, we find: Sum of the PV at a 15% discounting rate for 5 years = 3.35 Sum of the PV at a 16% discounting rate for 5 years = 3.27 Here, the quotient of average PBP (i.e., 3.29) lies in between 3.35 and 3.27 as calculated earlier. Therefore, the IRR may lie in between the discounting factors at 15% and 16% and the picture becomes as follows: Discounting Rate (%) 15 IRR 16

PV Factor 3.35 3.29 3.27

Now, IRR is calculated using the method of interpolation as follows: IRR = 15% + [{(3.35 – 3.29)/(3.35 – 3.27)} × (16% – 15%)] = 15% + 0.75% = 15.75% Problem 16 For a project having an expected useful life of 5 years, an initial investment of Rs. 60,000 is required. It is expected to generate cash inflows of Rs. 12,000, Rs. 16,000, Rs. 20,000, Rs. 24,000 and Rs. 8,000, respectively, from the project at the end of each year of its useful life. If the cost of the capital is assumed at 10% p.a., then calculate the IRR from the project and make your comment on the acceptability of the project. The PV of Re. 1 at 10% and 11% discounting rates are as follows: Year PV Factor @10% p.a. PV Factor @11% p.a.

1 0.909 0.901

2 0.826 0.812

3 0.751 0.731

4 0.683 0.659

5 0.621 0.593

Solution IRR is the rate of return which equates the total PV of cash inflows with the total PV of cash outflows, that is, which makes the NPV zero. To find out that discounting factor, let us start the solution on a trial-and-error basis. Firstly, let us calculate the average PBP for obtaining a quotient as follows: Average PBP = Initial investment/Average annual cash inflow = Rs. 60,000/{(Rs. 12,000 + Rs. 16,000 + Rs. 20,000 + Rs. 24,000 + Rs. 8,000) ÷ 5} = Rs. 60,000/Rs. 16,000 = 3.75 From the PV factor of the annuity table for 5 years, we get: Sum of the PV at a 10% discounting rate for 5 years = 3.791 Sum of the PV at a 11% discounting rate for 5 years = 3.696 Here, the quotient of average PBP (i.e., 3.75) lies in between 3.791 and 3.696 as calculated earlier. Therefore, the IRR may lie in between the discounting factors at 10% and 11%. Let us calculate the total PV of cash inflows at the discounting factors of 10% as well as 11%.

Modified Date: Tue, Jul 06, 2010 12:32:27 PM

Output Date: Tue, Jul 06, 2010 06:05:04 PM

Rev II

Project: Management Accounting_Debarshi Bhattacharyya ACE Pro India Pvt. Ltd. File: X:\Pearson\Management Accounting_Debarshi Bhattacharyya\MAIN\M12\LAYOUT_M12\M12_DEBA_ISBN_EN_SE_C12.indd

CAPITAL BUDGETING

763

Computation of the total PVs at the PV factor of 10% and 11% At 10% PV Factor Cash Inflows Rs. (a)

Year

PV Factor (b)

1 2 3 4 5

12,000 0.909 16,000 0.826 20,000 0.751 24,000 0.683 8,000 0.621 Total of PV of Cash Inflows

At 11% PV Factor

PV of Cash Inflows (Rs.) (a × b) 10,908 13,216 15,020 16,392 4,968 60,504

PV Factor (c) 0.901 0.812 0.731 0.659 0.593

PV of Cash Inflows (Rs.) (a × c) 10,812 12,992 14,620 15,816 4,744 58,984

From the above calculation, it has been transpired that the initial cash outflow of Rs. 60,000 lies in between the total PV of cash inflows of Rs. 60,504 at the 10% discounting rate and Rs. 58,984 at the 11% discounting rate. Therefore, the IRR lies in between the discounting factors at 10% and 11%. Now, IRR is calculated using the method of interpolation as follows: IRR = 10% + [{(Rs. 60,504 – Rs. 60,000)/(Rs. 60,504 – Rs. 58,984)} × (11% – 10%)] = 10% + 0.3315% = 10.3315% Problem 17 A company has to select any one of the two alternative projects whose particulars are given as follows: Projects

Net Cash Flow (Rs.)

Initial Outlay Rs.

I II

11,872 10,067

Y1 10,000 1,000

Y2 2,000 1,000

Y3 1,000 2,000

Y4 1,000 10,000

3 0.794 0.751 0.712 0.675

4 0.735 0.683 0.636 0.592

The company can arrange a fund at 8%. Compute the NPV and IRR of each project and comment on the result. The PV of Re. 1 at different cost of capitals are given as follows: Year PV Factor @8% p.a. PV Factor @10% p.a. PV Factor @12% p.a. PV Factor @14% p.a.

1 0.926 0.909 0.893 0.877

2 0.857 0.826 0.797 0.770

[B.Com. (Hons), Calcutta University—2006] Solution i. Computation of NPV of the projects

Year

PV Factor @8% (a)

1 2 3

0.926 0.857 0.794

Project I CFAT (b) Rs. 10,000 2,000 1,000

Project II PV of CFAT (a × b) Rs. 9,260 1,714 794

CFAT (c) Rs. 1,000 1,000 2,000

PV of CFAT (a × c) Rs. 926 857 1,588 (Continued)

Modified Date: Tue, Jul 06, 2010 12:32:27 PM

Output Date: Tue, Jul 06, 2010 06:05:04 PM

Rev II

Project: Management Accounting_Debarshi Bhattacharyya ACE Pro India Pvt. Ltd. File: X:\Pearson\Management Accounting_Debarshi Bhattacharyya\MAIN\M12\LAYOUT_M12\M12_DEBA_ISBN_EN_SE_C12.indd

764

Year 4 Less:

MANAGEMENT ACCOUNTING

PV Factor @8% (a) 0.735 Total PV of CFAT Total Cash Outflows NPV

Project I CFAT (b) Rs. 1,000

Project II PV of CFAT (a × b) Rs. 735 12,503 11,872 631

CFAT (c) Rs. 10,000

PV of CFAT (a × c) Rs. 7,350 10,721 10,067 654

Comment Under NPV Method Here, both the projects are having a positive NPV and thus qualify for acceptance of both the investment proposals. But if these two investment proposals are mutually exclusive, then the investment in Project II should be chosen as the NPV of Project II is higher than that of Project I. ii. Computation of IRR for the Projects As both the projects are resulting in a marginally positive NPV at an 8% discounting rate, NPV would be zero at a higher discounting rate (i.e., IRR). We need to find out that discounting factor at which NPV = 0 (i.e., IRR) on a trial-and-error basis. Firstly, let us consider the discounting rate at 10% and accordingly the calculations can be made as follows for both the projects: Year 1 2 3 4 Less:

PV Factor @10% (a) 0.909 0.826 0.751 0.683 Total PV of CFAT Total Cash Outflows NPV

Project I CFAT (b) Rs. 10,000 2,000 1,000 1,000

PV of CFAT (a × b) Rs. 9,090 1,652 751 683 12,176 11,872 304

Project II CFAT (c) (a × c) Rs. 1,000 1,000 2,000 10,000

PV of CFAT Rs. 909 826 1,502 6,830 10,067 10,067 Nil

From the above calculation, it is transpired that NPV becomes zero for Project II, at a discounting factor of 10%. Therefore, IRR of Project II is 10%. On the other hand, NPV is still marginally positive for Project I, even at the discounting factor of 10%. For Project I, NPV would be zero at a higher discounting rate. Let us consider the discounting rate at 12% and accordingly, calculations can be made as follows for Project I: PV Factor @12% (a) 0.893 0.797 0.712 0.636

Year 1 2 3 4 Less:

Total PV of CFAT Total Cash Outflows NPV

CFAT (b) Rs. 10,000 2,000 1,000 1,000

PV of CFAT (a × b) Rs. 8,930 1,594 712 636 11,872 11,872 Nil

From the above calculation, it is transpired that NPV becomes zero for Project I at a discounting factor of 12%. Therefore, IRR of Project I is 12%. Comment Under IRR Method Under IRR method, the investment in Project I should be chosen, as its IRR (12%) is higher than that of Project II (10%).

Modified Date: Tue, Jul 06, 2010 12:32:27 PM

Output Date: Tue, Jul 06, 2010 06:05:04 PM

Rev II

Project: Management Accounting_Debarshi Bhattacharyya ACE Pro India Pvt. Ltd. File: X:\Pearson\Management Accounting_Debarshi Bhattacharyya\MAIN\M12\LAYOUT_M12\M12_DEBA_ISBN_EN_SE_C12.indd

765

CAPITAL BUDGETING

Overall Comment As per NPV method, Project II should be selected for the investment. To the contrary, as per IRR method, Project I should be selected for the investment. As the NPV method is superior than the IRR method, finally Project II should be chosen for the investment. Problem 18 A company is thinking of investing Rs. 20 lakhs in a project. The life of the project is 5 years at the end of which the estimated scrap value from the project will be zero. The tax rate is @50% and the straight-line method of depreciation is followed. The cost of capital may be taken at 10% p.a. The expected CFBDT are as follows: Year Estimated CFBDT (Rs. in lakhs)

1 4

2 6

3 8

4 8

5 10

2 0.826 0.812 0.797

3 0.751 0.731 0.712

4 0.683 0.659 0.636

5 0.621 0.593 0.567

You are asked to calculate: i. ii. iii. iv. v. vi.

PBP. ARR. Discounted PBP. NPV. PI. IRR.

The PVs of Re. 1 at different discounting rates are as follows: Year

1 0.909 0.901 0.893

This page is intentionally left blank

PV Factor @10% p.a. PV Factor @11% p.a. PV Factor @12% p.a.

Solution Depreciation of the project per year = 20 (Rs. in lakhs) ÷ 5 years = 4 (Rs. in lakhs) Statement showing computation of PAT and CFAT Year

CFBDT (a) Rs. Lakhs

1 2 3 4 5

4 6 8 8 10

Depreciation p.a. (b) Rs. Lakhs 4 4 4 4 4

PBT (c = a – b) Rs. Lakhs Nil 2 4 4 6

Tax @50% on PBT (d) Rs. Lakhs Nil 1 2 2 3

PAT (e = c – d) Rs. Lakhs

CFAT (e + b) Rs. Lakhs

Cum. CFAT Rs. Lakhs

Nil 1 2 2 3

4 5 6 6 7

4 9 15 21 28

8

i. PBP From the above calculation, it has been observed that the initial investment of 20 (Rs. in lakhs) lies in between the cumulative CFAT of 15 (Rs. in lakhs) and 21 (Rs. in lakhs), which could be recovered in between the period of 4th and 5th year. In such a situation, PBP is to be calculated with the help of either the forward method or backward method of interpolation as used in the following: Using Forward Method of Interpolation: PBP of fixed assets = 4 years + [{(20 – 15)/(21 – 15)} × 1] = 4 + 0.8333 = 4.8333 years

Modified Date: Tue, Jul 06, 2010 12:32:27 PM

Output Date: Tue, Jul 06, 2010 06:05:04 PM

Rev II

Project: Management Accounting_Debarshi Bhattacharyya ACE Pro India Pvt. Ltd. File: X:\Pearson\Management Accounting_Debarshi Bhattacharyya\MAIN\M12\LAYOUT_M12\M12_DEBA_ISBN_EN_SE_C12.indd

766

MANAGEMENT ACCOUNTING

ii. ARR Here, as per the earlier calculation, the Average Annual PAT = 8 ÷ 5 = 1.6 (Rs. in lakhs) And, Average investment = {(Initial investment – Salvage value) ÷ 2} + Salvage value + Additional working capital = {(20 – 0) ÷ 2} + 0 + 0 = 10 (Rs. in lakhs) ∴ ARR = (Average annual PAT ÷ Average investment) × 100 = (1.6 ÷ 10) × 100 = 16% Calculation of NPV at 10% discounting factor Year 1 2 3 4 5

CFAT (a) Rs. Lakhs 4 5 6 6 7 Total PV of CFAT

PV Factor @10% (b) 0.909 0.826 0.751 0.683 0.621

PV of CFAT (a × b) Rs. Lakhs 3.636 4.130 4.506 4.098 4.347 20.717

Cumulative PV of CFAT Rs. Lakhs 3.636 7.766 12.272 16.370 20.717

iii. Discounted PBP From the above calculation, it has been observed that the initial investment of 20 (Rs. in lakhs) lies in between the cumulative PV of CFAT of 16.37 (Rs. in lakhs) and 20.717 (Rs. in lakhs), which could be recovered in between the period of 4th and 5th year. In such a situation, the discounted PBP is to be calculated with the help of either the forward method or the backward method of interpolation as used as follows: Using Forward Method of Interpolation: Discounted PBP of the project = 4 years + [{(20 – 16.37) ÷ (20.717 – 16.37)} × 1] = 4 + 0.835 = 4.835 years iv. NPV NPV = Total PV of cash inflows – Total PV of cash outflows = 20.717 (Rs. in lakhs) – 20 (Rs. in lakhs) = 0.717 (Rs. in lakhs) v. PI PI = Total PV of cash inflows/Total PV of cash outflows = 20.717 (Rs. in lakhs)/20 (Rs. in lakhs) = 1.03585 vi. IRR IRR is the rate of return which equates the total PV of cash inflows with the total PV of cash outflows, that is, which makes the NPV zero. To find out that discounting factor, let us start the solution on a trial-and-error basis. Firstly let us calculate the average PBP for obtaining a quotient as follows: Average PBP = Initial investment/Average annual cash inflow = 20 (Rs. in lakhs)/{28 (Rs. in lakhs) ÷ 5} = 20 (Rs. in lakhs)/5.6 (Rs. in lakhs) = 3.571 From the PV factor of the annuity table for 5 years, we get: Sum of the PV at a 12% discounting rate for 5 years = 3.605. Sum of the PV at a 11% discounting rate for 5 years = 3.696. As the NPV at the 10% discounting factor shows a reasonable amount of positive value, the IRR (i.e., where NPV = 0) should be at a higher discounting factor. Here, the quotient of average PBP (i.e., 3.571) is nearer to 3.605 and 3.696 at 12% and 11% discounting factors, respectively, as calculated earlier. Therefore, the IRR may exist nearer to the discounting factors of 12% and 11%. Let us calculate total PV of cash inflows at the discounting factors of 12% and 11%.

Modified Date: Tue, Jul 06, 2010 12:32:27 PM

Output Date: Tue, Jul 06, 2010 06:05:04 PM

Rev II

Project: Management Accounting_Debarshi Bhattacharyya ACE Pro India Pvt. Ltd. File: X:\Pearson\Management Accounting_Debarshi Bhattacharyya\MAIN\M12\LAYOUT_M12\M12_DEBA_ISBN_EN_SE_C12.indd

CAPITAL BUDGETING

767

Computation of total PVs at the PV factors of 10% and 11% At 12% PV Factor Year

Less:

1 2 3 4 5 Total PV of Cash Inflows Total PV of Cash Outflows NPV

Cash Inflows Rs. Lakhs (a) 4 5 6 6 7

PV Factor (b) 0.893 0.797 0.712 0.636 0.567

PV of Cash Inflows Rs. Lakhs (a × b) 3.572 3.985 4.272 3.816 3.969 19.614 20.000 (0.386)

At 11% PV Factor PV Factor (c) 0.901 0.812 0.731 0.659 0.593

PV of Cash Inflows Rs. Lakhs (a × c) 3.604 4.062 4.386 3.954 4.151 20.155 20.000 0.155

From the above calculation, it has been transpired that the initial cash outflow of 20 (Rs. in lakhs) lies in between the total PV of cash inflows of 19.614 (Rs. in lakhs) at the 12% discounting rate and 20.155 (Rs. in lakhs) at the 11% discounting rate. Therefore, the IRR lies in between the discounting factors at 12% and 11% and the picture becomes as follows: Total PV of Cash Inflows Rs. in Lakhs 19.614 20 20.155

Discounting Rate (%) 12 IRR 11

Now, IRR is calculated using the backward method of interpolation as follows: IRR = 12 – [{(20 – 19.614)/(20.155 – 19.614)} × (12% − 11%)] = 12 – (0.386/0.541) × 1% = (12 – 0.7135) × 1% = 11.2865% CHAPTER REVIEW SUMMARY 





 

Capital Budgeting is the technique of long-term planning and analysis of outflows and inflows of funds relating to some investment objectives. It is the planning of capital expenditure which provides a return over a number of years. Features of Capital Budgeting are as follows: (a) It is of a long-term nature; (b) It is a long-time gap between the initial investment and the return from the investment; (c) An investment decision may be taken for a single project proposal, or for two or more mutually exclusive project proposals; and (d) Acceptance or rejection of an investment proposal should be based on the maximization of value of the firm. Capital Budgeting has more importance due to the following reasons: (a) Capital Budgeting decisions involve a huge investment; (b) Capital investment involves a long-term investment of fund with the risk factor; (c) Capital Budgeting decisions are, in most cases, of irreversible nature due to an involvement of huge fund for a long-term period; (d) Capital Budgeting decision not only effects on the present earning of the firm, but also has an effect on the future profitability of the firm. Under Capital Budgeting technique, investments are classified into two parts, such as: (a) Revenue-expansion investment and (b) Cost-reduction investment. Capital Budgeting decisions are generally classified into the following three categories, such as: (a) Accept– reject decision; (b) Mutually exclusive decision; and (c) Capital-rationing decision.

Modified Date: Tue, Jul 06, 2010 12:32:27 PM

Output Date: Tue, Jul 06, 2010 06:05:04 PM

Rev II

Project: Management Accounting_Debarshi Bhattacharyya ACE Pro India Pvt. Ltd. File: X:\Pearson\Management Accounting_Debarshi Bhattacharyya\MAIN\M12\LAYOUT_M12\M12_DEBA_ISBN_EN_SE_C12.indd

768 

MANAGEMENT ACCOUNTING

For evaluating the acceptability of a long-term investment proposal under Capital Budgeting technique, the following three steps are to be followed: Step 1: Determination of the future expected cash flows from the investment proposals. Step 2: Determination of the required expected rate of return by the investors (i.e., cost of capital). Step 3: Application of the Capital Budgeting evaluation criteria to evaluate the investment proposals.

 

 

PV is the current value of a future receivable or payable amount. Such future receivable or payable amount expressed in the future price level is being converted at its PV with the help of the discounting factor or PV factor. When any future expected cash inflow or outflow is being converted at its PV with the help of the PV or discounting factor, then such converted PV of cash inflow or outflow is called the ‘discounted cash flow.’ Hence, the discounted cash flow is the PV of the future cash flow. Capital Budgeting techniques are broadly classified into two categories: (i) Non-discounted technique; and (ii) Discounted technique. Each of such category is again classified under different methods as discussed in the following i. Non-discounted techniques (i.e., without considering the time value of money): a. PBP method. b. Pay Back Profitability method. c. ARR method. ii. Discounted cash flow techniques (i.e., time-adjusted techniques): a. Discounted PBP method. b. PI method. c. NPV method. d. IRR method. e. TV method.

CHAPTER REVIEW QUIZ 1. State whether the following statements are true or false: a. In case of two mutually exclusive investment proposals, the proposal having a longer PBP should be accepted. b. In case of two mutually exclusive investment proposals, the proposal having the higher NPV should be rejected. c. In case of single-investment proposal, accept it if PI > 1. d. In case of single-investment proposal, accept it if NPV < 0. e. NPV method is superior to IRR method for a Capital Budgeting decision. f. In case of two mutually exclusive investment proposals, the proposal having a greater IRR should be accepted. g. Discounted PBP method ignores the time value of money. h. IRR is the discounting rate at which the total PV of cash inflows equals the total PV of cash outflows. Ans.: True: (c), (e), (f), (h); False: (a), (b), (d), (g). 2. Choose the correct alternative from the following: a. Annual CFAT: (i) Net PBDT – Depreciation; (ii) Net PAT + Depreciation; (iii) Net PBDT – tax. b. If the cash inflow p.a. from a project having an initial cash outlay of Rs. 1,50,000 is Rs. 50,000, then its PBP is: (i) 4 years; (ii) 2 years; (iii) 3 years. c. If the expected life a project is 7 years, during which it is expected to generate cash inflows of Rs. 30,000 p.a. and its PBP is 3 years, then its Pay Back Profitability is: (i) Rs. 1,20,000; (ii) Rs. 90,000; (iii) Rs. 2,10,000. d. If the total PV of cash inflows and cash outflows for a project are Rs. 1,42,000 and Rs. 1,20,000, respectively, the NPV of the project is: (i) 1.1833; (ii) Rs. 2,62,000; (iii) Rs. 22,000. e. If the total PV of cash inflows and cash outflows for a project are Rs. 1,50,000 and Rs. 1,00,000, respectively, the PI of the project is: (i) 1.50; (ii) 0.67; (iii) Rs. 50,000. f. PV of Re. 1 for 3rd at a 10% discounting rate is: (i) 0.917; (ii) 0.751; (iii) 0.826. Ans.: (a) (ii); (b) (iii); (c) (i); (d) (iii); (e) (i); (f) (ii).

Modified Date: Tue, Jul 06, 2010 12:32:27 PM

Output Date: Tue, Jul 06, 2010 06:05:04 PM

Rev II

Project: Management Accounting_Debarshi Bhattacharyya ACE Pro India Pvt. Ltd. File: X:\Pearson\Management Accounting_Debarshi Bhattacharyya\MAIN\M12\LAYOUT_M12\M12_DEBA_ISBN_EN_SE_C12.indd

CAPITAL BUDGETING

769

EXERCISE I. Theoretical Questions A. Short Answer Type Questions:

1. 2. 3. 4. 5. 6. 7. 8. 9. 10. 11. 12. 13. 14. 15. 16.

What do you mean by Capital Budgeting? Name the different types of Capital Budgeting decisions. Write the evaluation criteria of a Capital Budgeting decision. Write the name of different non-discounted Capital Budgeting technique. Write the name of different discounted Capital Budgeting technique. What is Present Value? What is Discounted Present Value? What is Net Present Value? What is Net Terminal Value? What is Internal Rate of Return? What is PBP? What is Discounted PBP? What is Accounting Rate of Return? What is Profitability Index? What is capital-rationing decision? Mention the classification of investments under Capital Budgeting technique.

B. Essay Type Questions

1. What do you mean by Capital Budgeting? What are its features and importance? 2. How are investments classified under Capital Budgeting technique? Discuss the classification of Capital Budgeting decisions? 3. Discuss the evaluation criteria of Capital Budgeting decision. 4. What is Present Value? What is discounted cash flow? 5. Discuss the various techniques of Capital Budgeting decisions. 6. What is Net Present Value? How are Capital Budgeting decisions taken under this method? 7. What is Net Terminal Value? How are Capital Budgeting decisions taken under this method? 8. What is Internal Rate of Return? How are Capital Budgeting decisions taken under this method? 9. What is PBP? How does it differ from the Pay Back Profitability? 10. What is Discounted PBP? How are Capital Budgeting decisions taken under this method? II.

Practical Problems

1. The cost of a project is estimated at Rs. 4,10,000 with an estimated life of 5 years, at the end of which the project would realize Rs. 10,000 as scrap. The annual PBDT from the project is estimated at Rs. 1,80,000.

Calculate the annual cash flow from the project assuming the tax rate at 40%. Ans.: Rs. 1,40,000. 2. From the following information, calculate PAT and annual CFAT for the year: Cash Sales Credit Sales

Rs. 1,20,000 4,30,000

Collection from Credit Sales Cash Expenses Debited to the Profit & Loss A/c Non-cash Expenses Debited to the Profit & Loss A/c Other Non-cash Income Credited to the Profit & Loss A/c Tax Rate

4,00,000 1,60,000 1,00,000 30,000 @40%

Ans.: PAT: Rs. 1,92,000; CFAT: Rs. 2,16,000.

Modified Date: Tue, Jul 06, 2010 12:32:27 PM

Output Date: Tue, Jul 06, 2010 06:05:04 PM

Rev II

Project: Management Accounting_Debarshi Bhattacharyya ACE Pro India Pvt. Ltd. File: X:\Pearson\Management Accounting_Debarshi Bhattacharyya\MAIN\M12\LAYOUT_M12\M12_DEBA_ISBN_EN_SE_C12.indd

770

MANAGEMENT ACCOUNTING

3. Following are the details of two mutually exclusive project proposals of a company: Project A Rs. 3,00,000 10,000 50,000 50,000 80,000 10,000 20,000

Cost of Plant Installation Charges of Plant Cost of Building Working Capital Requirement Annual PBDT Scrap Value of Plant Disposal Value of Building

Project L Rs. 4,00,000 15,000 1,00,000 70,000 95,000 20,000 50,000

Life of each project is 10 years and depreciation has been charged on the fixed assets under straight-line basis. Income tax rate is 35%. Determine (a) Total cash outflow for each project; (b) Total cash inflow from each project; and (c) Terminal cash inflow from each project. Ans.: Project A: (a) Rs. 4,10,000; (b) Rs. 6,42,500; (c) Rs. 80,000; Project L: (a) Rs. 5,85,000; (b) Rs. 7,90,750; (c) Rs. 1,70,000. 4. (i) A company requests you to compute the PBP and Pay Back Profitability of project K from the following information: Initial Investment Year 1 2 3 4 5

Rs. 5,00,000 CFAT (Rs.) 1,00,000 1,40,000 2,60,000 1,80,000 1,50,000

(ii) What would have been your answer had the initial investment been Rs. 6,00,000 instead of Rs. 5,00,000? Ans.: (i) PBP: 3 years; Pay Back Profitability: Rs. 3,30,000. (iii) PBP: 3.44 years; Pay Back Profitability: Rs. 2,30,000. 5. From the following information of a project proposal, compute its PBP and Pay Back Profitability: Initial investment: Rs. 3,00,000. Estimated life of the project: 5 years. PBDT; Year 1 2 3 4 5

PBDT (Rs.) 80,000 1,20,000 1,80,000 2,00,000 2,40,000

Assume that the tax rate is 40%. Ans.: PBP: 3 years; Pay Back Profitability: Rs. 3,12,000. 6. A company has decided to purchase a machine. There are three machines under consideration of the company, the details of which are given as follows: Machine 1 Rs. 3,00,000 5,00,000

Initial Investment Required Estimated Annual Sales Estimated Cost of Production: Direct Materials Direct Labour

40,000 50,000

Machine 2 Rs. 3,00,000 4,00,000 50,000 30,000

Machine 3 Rs. 3,00,000 4,50,000 48,000 36,000 (Continued)

Modified Date: Tue, Jul 06, 2010 12:32:27 PM

Output Date: Tue, Jul 06, 2010 06:05:04 PM

Rev II

Project: Management Accounting_Debarshi Bhattacharyya ACE Pro India Pvt. Ltd. File: X:\Pearson\Management Accounting_Debarshi Bhattacharyya\MAIN\M12\LAYOUT_M12\M12_DEBA_ISBN_EN_SE_C12.indd

CAPITAL BUDGETING

Machine 1 Rs. 60,000 20,000 10,000

Factory Overheads Administration Cost Selling & Distribution Cost

Machine 2 Rs. 50,000 10,000 10,000

771

Machine 3 Rs. 58,000 15,000 10,000

All sales are made in cash. Corporate tax rate is 40%. The economic life of machine 1 is 2 years, while it is 3 years for the other two. The scrap values from the machines are Rs. 40,000, Rs. 25,000 and Rs. 30,000, respectively. You are required to find out the most profitable investment based on the PBP method. [I.C.W.A. (Inter)—Adapted] Ans.: PBP for: Machine 1: 1.23 years; Machine 2: 1.61; Machine 3: 1.46 years; Machine 1 should be accepted. 7. The cost of a plant is Rs. 30,000. The expected life of the plant is 3 years. It is expected to generate EBDIT (Earnings before depreciation, interest and tax) of Rs. 13,000, Rs. 15,000 and 17,000, respectively. Compute the ARR assuming a 50% tax and a straight-line method of depreciation. [B.Com. (Hons), Calcutta University—2009] Ans.: 8.33%. 8. Determine the ARR from the following information as furnished in respect of two machines and comment on the results: Cost (Rs.) Estimated PBT (Rs.): 1st Year 2nd Year 3rd Year Estimated Life (Years) Estimated Salvage Value (Rs.) Rate of Tax (%)

Machine P 2,00,000

Machine Q 2,00,000

90,000 1,20,000 1,90,000 4,00,000 4 Nil 40

1,50,000 2,20,000 1,30,000 5,00,000 4 Nil 40

Depreciation has been charged on the straight-line basis. Ans.: ARR for machine P: 15%; ARR for machine Q: 22.50%. Investment in machine Q is more profitable as it gives a higher return than machine P. 9. From the following information of a project proposal, compute its discounted PBP: Cost of the Project Estimated Life of the Project Cost of Capital Estimated Annual CFAT

Rs. 7,50,000 4 years 12%

Year 1 2 3 4

CFAT (Rs.) 6,00,000 2,00,000 1,00,000 5,00,000

The PVs of Re. 1 at a 12% discounting rate for 4 years are as follows: Year PV Factor @12% p.a.

1 0.893

2 0.797

3 0.712

4 0.636

Ans.: 2.77 years.

Modified Date: Tue, Jul 06, 2010 12:32:27 PM

Output Date: Tue, Jul 06, 2010 06:05:04 PM

Rev II

Project: Management Accounting_Debarshi Bhattacharyya ACE Pro India Pvt. Ltd. File: X:\Pearson\Management Accounting_Debarshi Bhattacharyya\MAIN\M12\LAYOUT_M12\M12_DEBA_ISBN_EN_SE_C12.indd

772

MANAGEMENT ACCOUNTING

10. A project requires an initial cash outlay of Rs. 3,00,000 with an expected life of 4 years. It is expected that the project will generate the following cash inflows during its expected life: Year

1 75,000

Cash Inflows (Rs.)

2 1,20,000

3 1,20,000

4 1,50,000

Assuming the cost of capital @12%, evaluate the project under NPV and PI method. Ans.: NPV: Rs. 43,455; PI: 1.145; Project should be accepted. 11. A project requires an initial cash outlay of Rs. 2,00,000 with an expected life of 5 years. It is expected that the project will generate the following cash inflows during its expected life: Year

1 40,000

Cash Inflows (Rs.)

2 50,000

3 60,000

4 70,000

5 80,000

Assuming the cost of capital @10%, calculate the NPV and PI of the project and evaluate the project proposal. Ans.: NPV: Rs. 20,210; PI: 1.10; Project should be accepted. 12. A project requires an initial cash outlay of Rs. 5,00,000 with an expected life of 5 years. It is expected that the project will generate the following cash inflows during its expected life: Year PBDT (Rs.)

1 2,00,000

2 3,00,000

3 4,00,000

4 2,00,000

5 1,00,000

Depreciation has been charged under the straight-line basis. Tax rate is 50%. Assuming the cost of capital @10%, calculate the NPV and PI of the project and evaluate the project proposal. Ans.: NPV: Rs. 1,53,850; PI: 1.3077; Project should be accepted. 13. A company proposes to install a machine involving a capital cost of Rs. 3,60,000. The life of the machine is 5 years and its salvage value at the end of the life is nil. The machine will produce the net operating income after a depreciation of Rs. 68,000 p.a. The company’s tax rate is 45%. The NPV factors for 5 years are as follows: Discounting Factor Cumulative Factor

14 3.43

15 3.35

16 3.27

17 3.20

18 3.13

You are required to calculate the IRR of the proposal. [C.A. (Inter)—Adapted] Ans.: 15.75%. 14. From the following particulars, calculate the IRR of the project: Initial Outlay Estimated Life CFAT: End of Year 1 2 3 4

Rs. 20,000 4 years Rs. 5,000 Rs. 8,000 Rs. 10,000 Rs. 4,000

No realizable scrap value at the end of the project life.

PV of Re. 1 receivable at the end of the year 1, 2, 3 and 4 are: Year PV Factor 12% 13% 14%

1

2

3

4

0.892 0.885 0.877

0.797 0.783 0.770

0.712 0.693 0.675

0.636 0.613 0.592 (Continued)

Modified Date: Tue, Jul 06, 2010 12:32:27 PM

Output Date: Tue, Jul 06, 2010 06:05:04 PM

Rev II

Project: Management Accounting_Debarshi Bhattacharyya ACE Pro India Pvt. Ltd. File: X:\Pearson\Management Accounting_Debarshi Bhattacharyya\MAIN\M12\LAYOUT_M12\M12_DEBA_ISBN_EN_SE_C12.indd

773

CAPITAL BUDGETING

Year PV Factor 15% 16%

1

2

3

4

0.867 0.862

0.756 0.743

0.658 0.641

0.572 0.552

[M.Com., Calcutta University—Adapted] Ans.: 13.1740%. 15. A company is considering a project having an initial outlay of Rs. 10,00,000. The expected life of the project is 5 years and no salvage value is expected at the end of its life. The applicable tax rate is 40%. Depreciation is charged under the straight-line method. It is expected that the project will generate the following profits before depreciation and tax during its expected life: Year

1 2,00,000

PBT (Rs.)

2 2,15,000

3 2,55,000

4 2,70,000

5 4,10,000

Determine (a) PBP; (b) ARR; (c) Discounted PBP at a 10% discounting rate; (d) NPV at a 10% discounting rate; (e) PI at a 10% discounting rate; (f) IRR; and (g) NTV, if the compounding rate for the first two years is 8% and for the next three years is 12% and the appropriate discount rate is 10%. Ans.: (a) 4.36 years; (b) 8.40%; (c) Not reached; (d) Rs. 1,02,851; (e) 0.90; (f) 6.19%; and (g) Rs. 1,15,378. 16. The following information is available in respect of a project proposal: Initial outlay Rs. 1,20,000 Estimated life 5 years Cost of capital 12% Expected cash inflows and interest rates at which the cash inflows will be reinvested are given as follows: End of Year 1 2 3 4 5

Cash Inflows (Rs.) 60,000 50,000 40,000 30,000 20,000

Rate of Interest 8% 8% 10% 10% 10%

You are required to evaluate the project proposal using the TV method. Ans.: NTV: Rs. 19,596; Proposal should be accepted. 17. A company has been considering two mutually exclusive projects A and B. Following details are made available to you:

Project Cost Cash Inflows: Year 1 Year 2 Year 3 Year 4 Year 5 Scrap Value at the end of the 5th year

Rs. in Lakhs Project A Project B 700 700 100 200 300 450 550 50

500 400 200 100 75 25’

Assuming that the firm’s cost of capital is 10%, indicate which project would be accepted by the firm using the following methods: (i) NPV at a 10% discounting; (ii) IRR; and (iii) PI.

Modified Date: Tue, Jul 06, 2010 12:32:27 PM

Output Date: Tue, Jul 06, 2010 06:05:04 PM

Rev II

Project: Management Accounting_Debarshi Bhattacharyya ACE Pro India Pvt. Ltd. File: X:\Pearson\Management Accounting_Debarshi Bhattacharyya\MAIN\M12\LAYOUT_M12\M12_DEBA_ISBN_EN_SE_C12.indd

774

MANAGEMENT ACCOUNTING

The PVs of Re. 1 at different discounting rates are as follows: Year

1 0.909 0.787 0.781 0.730 0.725

PV Factor @10% p.a. PV Factor @27% p.a. PV Factor @28% p.a. PV Factor @37% p.a. PV Factor @38% p.a.

2 0.826 0.620 0.610 0.533 0.525

3 0.751 0.488 0.477 0.389 0.381

4 0.683 0.384 0.373 0.284 0.276

5 0.621 0.303 0.291 0.207 0.200

[I.C.W.A. (Inter)—Adapted] Ans.: Project A: NPV: 461.35 (Rs. in lakhs); IRR: 27.21%; PI: 1.659. Project B: NPV: 365.50 (Rs. in lakhs); IRR: 37.63%; PI: 1.522. 18. A company has to select one of the two alternative projects whose particulars are given as follows: Initial Outlay Rs. in Lakhs 25 40

Projects D A

Net Cash Inflow (Rs. in Lakhs) Y2 Y3 Y4 5 20 14 14 16 17

Y1 – 10

Y5 14 15

The company’s cost of capital is 10%. Compute the NPV, PI, PBP and discounted PBP of each project and comment on the result. The PVs of Re. 1 at a 10% discount rate are given as follows: Year PV Factor

0

1

2

3

4

5

1

0.91

0.83

0.75

0.68

0.62

[C.A. (Inter)—Adapted] Ans.: Project D: NPV: 12.35 (Rs. in lakhs); PI: 1.494; PBP: 3 years; DPBP: 3.6145 years. Project A: NPV: 13.58 (Rs. in lakhs); PI: 1.3395; PBP: 3 years; DPBP: 3.6298 years. Project A should be accepted on the basis of superior NPV method.

Modified Date: Tue, Jul 06, 2010 12:32:27 PM

Output Date: Tue, Jul 06, 2010 06:05:04 PM

Rev II

Project: Management Accounting_Debarshi Bhattacharyya ACE Pro India Pvt. Ltd. File: X:\Pearson\Management Accounting_Debarshi Bhattacharyya\MAIN\M13\LAYOUT_M13\M13_DEBA_ISBN_EN_SE_C13.indd

Responsibility Accounting

13

LEARNING OBJECTIVES On completion of the study of the chapter, you should be able to understand: What is the meaning and concept of Responsibility Accounting? Prerequisites and assumptions of Responsibility Accounting. Advantages and limitations of Responsibility Accounting. Meaning and concept of Responsibility Centres. Different types of Responsibility Centre. Concepts of Controllable and Non-controllable Costs. What is Responsibility Reporting? Different types of Responsibility Reporting.

13.1 MEANING AND CONCEPT OF RESPONSIBILITY ACCOUNTING An organization uses various techniques of costing such as Standard Costing, Budgetary Control for Control of Costs and so on. Under these costing techniques, focus is given on the cost and not on the person who has the authority to control the costs. In every well-structured organization, the responsibilities of every person’s actions are clearly defined and a manager is engaged in every section of actions of the organization. Every such person is accountable to his/her superior authority for the responsibility assigned to him/her. Responsibility Accounting may be defined as a system of control where a responsibility is assigned to different executives of a concern for control of cost or increase of revenue. It is one of the basic components of a good control system. In this system, an executive is held responsible only for those activities for which he/she has been delegated a responsibility. Therefore, unlike other techniques of costing like standard costing and budgetary control where the emphasis is given on the devices of control of costs, Responsibility Accounting is a system of control where the focus is given on different persons who are responsible for the control of cost within the periphery of the responsibilities assigned to them. Say, the Production Manager of a concern prepares Materials Usage Budget and assigns the responsibility to Supervisor II of the factory for the control of usage of materials as per the material usage budget. In case of overconsumption of materials, the workers working under Supervisor II are made responsible for such an action and answerable to Supervisor II, who, in turn, is made responsible and answerable to the Production Manager, who is ultimately made responsible overall for such an overconsumption of materials and is answerable to his higher management. Hence, under Responsibility Accounting, every individual working in a concern is assigned some responsibilities, and he/she is accountable to his/her superior authority for any dereliction of responsibilities vested upon them. According to the Institute of Cost and Works Accountants of India (ICWAI), Responsibility Accounting is ‘a system of Management Accounting under which accountability is established according to the responsibility delegated to various levels of management and a management information and reporting system instituted to give adequate feedback in terms of delegated responsibility. Under this system, divisions or units of an

Modified Date: Tue, Jul 06, 2010 12:33:14 PM

Output Date: Tue, Jul 06, 2010 05:12:04 PM

Rev II

Project: Management Accounting_Debarshi Bhattacharyya ACE Pro India Pvt. Ltd. File: X:\Pearson\Management Accounting_Debarshi Bhattacharyya\MAIN\M13\LAYOUT_M13\M13_DEBA_ISBN_EN_SE_C13.indd

776

MANAGEMENT ACCOUNTING

organization under a specified authority in a person are developed as Responsibility Centres and evaluated individually for their performance.’ According to Charles T. Horngreen, ‘Responsibility Accounting is a system of accounting that recognizes various responsibility centres throughout the organization and reflects the plans and actions of each of these centres by assigning particular revenues and costs to the one having the pertinent responsibility. It is also called Profitability Accounting or Activity Accounting.’ 13.2 PREREQUISITES OF RESPONSIBILITY ACCOUNTING Following are the prerequisites of Responsibility Accounting: i. Activities of an organization are to be segmented in different Responsibility Centres. ii. Clear-cut distinction between Controllable and Non-controllable Costs of the concerned Responsibility Centre should be made. iii. Areas of responsibility of each Responsibility Centre in the organization should be clearly defined. iv. The charge of each Responsibility Centre is to be given to a manager. v. A plan of objective should be set up for each Responsibility Centre. vi. The concerned manager must be motivated to attain the objectives of the Responsibility Centre. vii. Adequate device should exist in the organization to ascertain and evaluate the actual performance of the Responsibility Centre. viii. An adequate device should exist in the organization to provide Feedback Report to the management in respect of the actual performance of the Responsibility Centres. ix. An adequate device should exist in the organization for adopting corrective measures on the basis of the Feedback Report of the actual performance of the Responsibility Centres. 13.3 ASSUMPTIONS OF RESPONSIBILITY ACCOUNTING The development of a sound Responsibility Accounting System is based on the following assumptions: i. Goals and responsibilities set out by the management are attainable with efficient and effective performance. ii. Employees of the organization give their best effort to achieve the goals and responsibilities delegated upon them. iii. Employees are held responsible for the areas over which they exercise control. iv. Performance of the employees is evaluated by the higher management through Feedback Reports at regular intervals. v. Performance evaluation process of the employees is based on reward-providing nature. 13.4 ADVANTAGES OR BENEFITS OR PURPOSES OF RESPONSIBILITY ACCOUNTING Responsibility Accounting has the following advantages or purposes: i. It emphasizes the need of clearly defining and communicating the goals and objectives of the organization and its employees. ii. Under this system, the responsibility of each employee is clearly identified. iii. It compels the management to set out a definite plan and budget for each segment of organizational activities. iv. It facilitates a stronger control on cost, revenue and uses of assets. v. It evaluates the actual performance of each employee of the organization by comparing with the standards set out as the yardstick. vi. It provides greater motivation to the employees to work more harder to perform responsibilities vested upon them. vii. It helps to improve the quality of decision-making through focusing on the principle of ‘management by exception’ (MBE).

Modified Date: Tue, Jul 06, 2010 12:33:14 PM

Output Date: Tue, Jul 06, 2010 05:12:04 PM

Rev II

Project: Management Accounting_Debarshi Bhattacharyya ACE Pro India Pvt. Ltd. File: X:\Pearson\Management Accounting_Debarshi Bhattacharyya\MAIN\M13\LAYOUT_M13\M13_DEBA_ISBN_EN_SE_C13.indd

RESPONSIBILITY ACCOUNTING

777

13.5 LIMITATIONS OF RESPONSIBILITY ACCOUNTING Responsibility Accounting suffers from the following limitations: i. Making of a clear-cut distinction between the Controllable and Non-controllable Costs of the concerned Responsibility Centre is really a difficult task. ii. It may fail in case of lack of coordination between the different employees and segments of the organization. iii. Employees may feel demotivated if unrealistic standards are set out for performance evaluation. iv. Preparation of Responsibility Accounting Report is not an easy task. v. Any delay in the submission of Responsibility Accounting Report to the management may deteriorate the existing problem. vi. Individual Responsibility Centres may give more emphasis on its own areas of responsibility rather than on the organizational goals. 13.6 RESPONSIBILITY CENTRE A Responsibility Centre is an area of responsibility of a concern delegated to and controlled by an individual who is accountable to his higher authority for such responsibility. Every activity of a concern is divided into a separate Responsibility Centre. The overall responsibility of each such a Responsibility Centre is assigned to a manager. Therefore, a Responsibility Centre is a segment of a concern over which a manager exercises responsibility, and he, in turn, is accountable to his higher authority for the activities of the Responsibility Centre. The basic feature of Responsibility Centres is that each of them uses inputs, such as materials, labour and overheads, to produce and sell the output in the form of goods. The inputs of Responsibility Centres are measured as costs and the outputs of Responsibility Centres are measured as revenues. Hence, in every Responsibility Centre, there should be an identifiable responsibility for cost or revenue or resource of the concern. 13.7 TYPES OF RESPONSIBILITY CENTRES For an effective control, Responsibility Centres are generally classified into three classes. These are: (a) Cost Centre; (b) Profit Centre; and (c) Investment Centre. Each of the above classes of Responsibility Centres is separately explained as follows. 13.7.1 Cost Centre A Cost Centre is a segment of an organization in which all costs are accumulated, and the manager of this segment is held responsible for the costs incurred though he has no responsibility for revenue generation. In a manufacturing concern, the production and service departments are classified as Cost Centres. Responsibility in a Cost Centre is restricted to cost only. A Cost Centre is not held directly responsible for revenue generation in the organization. Cost Centre managers have a control over some or all of the costs accumulated at the Cost Centre. Therefore, a Cost Centre is a responsibility level where the employees of that centre are concerned with the cost management alone. A Cost Centre is also called Expense Centre. In this segment of the Responsibility Centre, only the inputs are measured in monetary terms. The outputs of a Cost Centre are not measured in monetary terms, as services rendered by some departments of a Cost Centre (such as legal department) are not measurable in monetary value. The performance of a Cost Centre is measured in terms of quantity of inputs used for producing a given output. The performance of the manager of a Cost Centre is evaluated by comparing the costs incurred with the budgeted costs. 13.7.2 Profit Centre A Profit Centre is a segment of an organization in which both cost and revenue are measured in terms of monetary units and the manager of this segment is held responsible for both cost as well as revenue. In this Responsibility Centre, the inputs are taken as costs and the outputs are taken as revenue. Here, both inputs

Modified Date: Tue, Jul 06, 2010 12:33:14 PM

Output Date: Tue, Jul 06, 2010 05:12:04 PM

Rev II

Project: Management Accounting_Debarshi Bhattacharyya ACE Pro India Pvt. Ltd. File: X:\Pearson\Management Accounting_Debarshi Bhattacharyya\MAIN\M13\LAYOUT_M13\M13_DEBA_ISBN_EN_SE_C13.indd

778

MANAGEMENT ACCOUNTING

and outputs are measured in terms of monetary units. The difference between the output (i.e., revenue) and input (i.e., cost) represents the profit earned. The manager of the Profit Centre is held responsible for the amount of profit earned by that Profit Centre. The Income Statement of a Profit Centre is used as a control device. A Profit Centre provides a more effective assessment of the manager’s performance, as both costs and revenues are measured in monetary terms in this segment of Responsibility Centre. Therefore, a Profit Centre is a responsibility level where both cost and revenue are measured in terms of monetary units, in which the performance is measured in terms of Budgeted Profit and the manager is held responsible for both cost and revenue. 13.7.3 Investment Centre An Investment Centre is a responsibility level of an organization where the manager is concerned not only with cost management and revenue generation, but also held responsible for the investment in assets used by the centre. Here, the manager is held responsible for both profit as well as the assets that are under his control. The manager of an investment centre possesses more authority and responsibility than the manager of either a Cost Centre or a Profit Centre. The performance of an Investment Centre is measured in terms of the Return on Investment (ROI) of the centre. The investment made in each centre and profit earned by each centre is separately ascertained, on the basis of which ROI of each centre is measured for the evaluation of the performance. For this purpose, the ROI of each centre is to be calculated in the following manner: Return on Investment (ROI) = Profit of the Investment Centre/Assets used in the Investment Centre × 100 13.8 CONTROLLABLE AND NON-CONTROLLABLE COSTS Making a distinction between Controllable and Non-controllable Costs is one of the prerequisites of the Responsibility Accounting System. Controllable Cost may be defined as the cost which can be influenced by the action of a specific employee of a Responsibility Centre. This type of cost can be changed by the head of a Responsibility Centre. He has the control to alter the price or the quantity or the both, within the area of his Responsibility Centre by his managerial action. Generally, all variable direct costs are controllable, such as cost of direct materials, direct labour and so on. But some direct costs like depreciation on plant and machinery are not controllable by the head of the Responsibility Centre as these costs are determined as per the policy of the organization. On the other hand, Non-controllable Costs are those which cannot be influenced by the action of a specific employee of a Responsibility Centre. This type of cost cannot be changed by the head of a Responsibility Centre. The manager of a Responsibility Centre has no control to alter this type of costs as these costs are set out by the top management as per the policy of the organization. Generally, most of the fixed indirect costs are noncontrollable. 13.9 RESPONSIBILITY REPORTING Responsibility Reporting implies the preparation of reports showing therein the performance of each Responsibility Centre. It includes the performance of managers who are accountable for that specific Responsibility Centre under their control. In short, reports prepared under Responsibility Accounting are called Responsibility Reporting. Responsibility Reporting is also called Responsibility Performance Reporting or Feedback Reporting. Responsibility Reports are prepared for the following purposes: i. To evaluate the level of performance in the area of responsibility of the concerned employees and the Responsibility Centre as a whole. ii. To formulate the remedial measures for improvement of the performance of the employees of the concerned Responsibility Centre.

Modified Date: Tue, Jul 06, 2010 12:33:14 PM

Output Date: Tue, Jul 06, 2010 05:12:04 PM

Rev II

Project: Management Accounting_Debarshi Bhattacharyya ACE Pro India Pvt. Ltd. File: X:\Pearson\Management Accounting_Debarshi Bhattacharyya\MAIN\M13\LAYOUT_M13\M13_DEBA_ISBN_EN_SE_C13.indd

RESPONSIBILITY ACCOUNTING

779

13.10 TYPES OF RESPONSIBILITY REPORTS Responsibility Reports can be classified into four types. These are: i. Enumerative Reports: Enumerative Report is a type of Responsibility-Accounting Feedback Report presented in forms of lists, charts, schedules, tabulations and so on, where performance information is provided in a raw form without any comparison. ii. Analytical and Interpretative Reports: Analytical and Interpretative Report is a type of ResponsibilityAccounting Feedback Report generally presented in a diagrammatical format, where the performance information is provided through analysis and interpretation. iii. Comparative and Evaluative Reports: Comparative and Evaluative Report is a type of ResponsibilityAccounting Feedback Report where the performance of the Responsibility Centre is provided with comparative and evaluative information. iv. Problem-solving Reports: Problem-solving Report is a type of Responsibility-Accounting Feedback Report where the information is provided with problem-solving measures of the Responsibility Centre. CHAPTER REVIEW SUMMARY  Responsibility Accounting is a system of control where responsibility is assigned upon different executives of a concern for control of cost or increase of revenue. It is one of the basic components of a good control system. In this system, an executive is held responsible only for those activities for which he has been delegated Responsibility.  Prerequisites of Responsibility Accounting are: (i) Activities of an organization are segmented in different Responsibility Centres; (ii) Clear-cut distinction between Controllable and Non-controllable Costs is done; (iii) Areas of responsibilities of each Responsibility Centre are clearly defined; (iii) The charge of each Responsibility Centre is given to a manager; and (iv) A plan of objective is set up for each Responsibility Centre.  Assumptions of Responsibility Accounting are: (i) Goals and responsibilities set out by the management are attainable; (ii) Employees of the organization give their best effort to achieve the goals and responsibilities; (iii) Employees are held responsible for the areas under their control; and (iv) Performances are evaluated by the higher management through Feedback Reports at regular intervals.  Advantages or Benefits or Purposes of Responsibility Accounting are: (i) Responsibility of each employee is clearly identified; (ii) It compels the management to set out definite plan and budget for each segment; (iii) It facilitates stronger control on cost, revenue and uses of assets; and (iv) It evaluates the actual performance by comparing with the standards set out as the yardstick.  Limitations of Responsibility Accounting are: (i) Clear-cut distinction between Controllable and Non-controllable Costs of the concerned Responsibility Centre is really a difficult task; (ii) Employees may feel demotivated if unrealistic standards are set out for performance evaluation; and (iii) Preparation of Responsibility Accounting Report is not an easy task.  A Responsibility Centre is an area of responsibility of a concern delegated to and controlled by an individual who is accountable to his higher authority for such responsibility. It is a segment of a concern over which a manager exercises responsibility and he is accountable to his higher authority for the activities of the Responsibility Centre.  For an effective control, Responsibility Centres are generally classified into three classes, which are: (a) Cost Centre; (b) Profit Centre; and (c) Investment Centre.  A Cost Centre is a segment of an organization in which costs are accumulated. Though the manager of this segment is held responsible for the costs incurred, he has no responsibility for the revenue generation.  A Profit Centre is a segment of an organization in which both cost as well as revenue are measured in terms of monetary units and the manager of this segment is held responsible for both cost and revenue.  An Investment Centre is a responsibility level of an organization where the manager is concerned not only with cost management and revenue generation, but also held responsible for the investment in assets used by the centre.  Controllable Cost may be defined as the cost which can be influenced by the action of a specific employee of a Responsibility Centre. Generally, all variable direct costs are controllable.

Modified Date: Tue, Jul 06, 2010 12:33:14 PM

Output Date: Tue, Jul 06, 2010 05:12:04 PM

Rev II

Project: Management Accounting_Debarshi Bhattacharyya ACE Pro India Pvt. Ltd. File: X:\Pearson\Management Accounting_Debarshi Bhattacharyya\MAIN\M13\LAYOUT_M13\M13_DEBA_ISBN_EN_SE_C13.indd

780   

MANAGEMENT ACCOUNTING

Non-controllable Costs are those which cannot be influenced by the action of a specific employee of a Responsibility Centre. Generally, most of the fixed indirect costs are non-controllable. Responsibility Reporting implies the preparation of reports showing therein the performance of each Responsibility Centre. Responsibility Reports can be classified into four types. These are: (a) Enumerative Reports; (b) Analytical and Interpretative Reports; (c) Comparative and Evaluative Reports; and (d) Problem-solving Reports.

CHAPTER REVIEW QUIZ 1. State whether the following statements are true or false: a. A cost manager is concerned not only with cost management but also with revenue creation. b. Responsibility Accounting is generally classified into four classes. c. Responsibility Accounting is used as a control device. d. Whether a cost is controllable or non-controllable is directly related to whether or not the cost is fixed or variable. e. Responsibility Accounting is also known as Profitability Accounting. f. All controllable costs are direct costs. g. An Investment Centre manager is concerned with investment decisions of the company. h. Profit Centre is established where the responsible manager can influence both cost as well as revenue. i. Return on investment is a function of the income earned and the assets used in order to earn that income. j. Indirect Costs are common to more than one segment of the organization. k. The performance report for a Responsibility Centre should include those costs which are of both controllable and non-controllable nature. Ans.: True: (c), (e), (f), (h), (i), ( j); False: (a), (b), (d), (g), (k). 2. Choose the correct alternative from the following: a. The manager who is concerned not only with cost management but also with revenue generation and manager: (i) a Profit Centre; investment decisions in a responsibility level is called (ii) an Investment Centre; (iii) a Cost Centre. b. The manager who is concerned not only with cost management but also with revenue generation in manager: (i) a Profit Centre; (ii) an Investment Centre; a responsibility level is called (iii) a Cost Centre. : (i) reducing cost; (ii) increasing sales; c. The return on investment may be improved by (iii) both of these. d. Depreciation on Plant which is traceable to a production manager’s Responsibility Centre is : (i) a controllable cost; (ii) direct cost; (iii) an indirect cost. e. The manager who is concerned with cost management in a responsibility level is called manager: (i) a Profit Centre; (ii) an Investment Centre; (iii) a Cost Centre. should be held f. If the actual usage of materials is more than the standard usage, the directly responsible for that variance: (i) production manager; (ii) purchase manager; (iii) sales manager. Ans.: (a) (ii); (b) (i); (c) (iii); (d) (ii); (e) (iii); (f) (i). EXERCISE A. Short Answer Type Questions

1. 2. 3. 4. 5. 6. 7. 8. 9.

What is Responsibility Accounting? What is Responsibility Centre? Name the various classes of Responsibility Centres. What is Cost Centre? What is Profit Centre? What is Investment Centre? What is Controllable Cost? What is Non-controllable Cost? What is Feedback Report?

Modified Date: Tue, Jul 06, 2010 12:33:14 PM

Output Date: Tue, Jul 06, 2010 05:12:04 PM

Rev II

Project: Management Accounting_Debarshi Bhattacharyya ACE Pro India Pvt. Ltd. File: X:\Pearson\Management Accounting_Debarshi Bhattacharyya\MAIN\M13\LAYOUT_M13\M13_DEBA_ISBN_EN_SE_C13.indd

RESPONSIBILITY ACCOUNTING

781

B. Essay Type Questions

1. 2. 3. 4. 5. 6.

Define Responsibility Accounting. What are its prerequisites? Explain the different assumptions of Responsibility Accounting. What are its benefits? What are the advantages and limitations of Responsibility Accounting? ‘Responsibility Accounting is an important device of control.’ Discuss. What is meant by Responsibility Centre? How is it classified? How far are these centres helpful to the management? Write short notes on the following: (a) Responsibility Accounting; (b) Responsibility Centre; (c) Cost Centre; (d) Profit Centre; (e) Investment Centre; (f) Controllable Cost; and (g) Non-controllable Cost. 7. What do you mean by Responsibility Accounting Report? What are the different types of Responsibility Accounting Report? 8. Distinguish between the following: (a) Cost Centre and Profit Centre; (b) Profit Centre and Investment Centre; and (c) Controllable Cost and Non-controllable Cost.

Modified Date: Tue, Jul 06, 2010 12:33:14 PM

Output Date: Tue, Jul 06, 2010 05:12:04 PM

Rev II

This page intentionally left blank

Year 1 2 3 4 5 6 7 8 9 10 11 12 13 14 15 16 17 18 19 20 21 22 23 24 25 30 35 40 45 50

Table 1 1% 1.010 1.020 1.030 1.041 1.051 1.062 1.072 1.083 1.094 1.105 1.116 1.127 1.138 1.149 1.161 1.173 1.184 1.196 1.208 1.220 1.232 1.245 1.257 1.270 1.282 1.348 1.417 1.489 1.565 1.645

2% 1.020 1.040 1.061 1.082 1.104 1.126 1.149 1.172 1.195 1.219 1.243 1.268 1.294 1.319 1.346 1.373 1.400 1.428 1.457 1.486 1.516 1.546 1.577 1.608 1.641 1.811 2.000 2.208 2.438 2.691

Compounded Sum of One Rupee

Appendix 3% 1.030 1.061 1.093 1.126 1.159 1.194 1.230 1.267 1.305 1.344 1.384 1.426 1.469 1.513 1.558 1.605 1.653 1.702 1.753 1.806 1.860 1.916 1.974 2.033 2.094 2.427 2.814 3.262 3.781 4.384

4% 1.040 1.082 1.125 1.170 1.217 1.265 1.316 1.359 1.423 1.480 1.539 1.601 1.665 1.732 1.801 1.873 1.948 2.026 2.107 2.191 2.279 2.370 2.465 2.563 2.666 3.243 3.946 4.801 5.841 7.106

5% 1.050 1.102 1.158 1.216 1.276 1.340 1.407 1.477 1.551 1.629 1.710 1.796 1.886 1.980 2.079 2.183 2.292 2.407 2.527 2.653 2.786 2.925 3.071 3.225 3.386 4.322 5.516 7.040 8.985 11.467

6% 1.060 1.124 1.191 1.262 1.338 1.419 1.504 1.594 1.689 1.791 1.898 2.012 2.133 2.261 2.397 2.540 2.693 2.854 3.026 3.207 3.399 3.603 3.820 4.049 4.292 5.743 7.686 10.285 13.764 18.419

7% 1.070 1.145 1.225 1.311 1.403 1.501 1.606 1.718 1.838 1.967 2.105 2.252 2.410 2.579 2.759 2.952 3.159 3.380 3.616 3.870 4.140 4.430 4.740 5.072 5.427 7.612 10.676 14.974 21.002 29.456

Modified Date: Tue, Jul 06, 2010 12:34:49 PM

Output Date: Tue, Jul 06, 2010 05:11:36 PM

(Continued)

8% 1.080 1.166 1.260 1.360 1.469 1.587 1.714 1.851 1.999 2.159 2.332 2.518 2.720 2.937 3.172 3.426 3.700 3.996 4.316 4.661 5.034 5.436 5.871 6.341 6.848 10.062 14.785 21.724 31.920 46.900

Project: Management Accounting_Debarshi Bhattacharyya File: X:\Pearson\Management Accounting_Debarshi Bhattacharyya\MAIN\Z01\Z01_DEBA_ISBN_EN_SE_APPENDIX.indd ACE Pro India Pvt. Ltd.

Rev-II

Year 1 2 3 4 5 6 7 8 9 10 11 12 13 14 15 16 17 18 19 20 21 22 23 24 25 30 35 40 45 50

Table 1

9% 1.090 1.188 1.295 1.412 1.539 1.677 1.828 1.993 2.172 2.367 2.580 2.813 3.066 3.342 3.642 3.970 4.328 4.717 5.142 5.604 6.109 6.658 7.258 7.911 8.623 13.267 20.413 31.408 48.325 74.354

10% 1.110 1.210 1.331 1.464 1.611 1.772 1.949 2.144 2.358 2.594 2.853 3.138 3.452 3.797 4.177 4.595 5.054 5.560 6.118 5.727 7.400 8.140 8.954 9.850 10.834 17.449 28.102 45.258 72.888 117.386

Compounded Sum of One Rupee 11% 1.110 1.232 1.368 1.518 1.685 1.870 2.076 2.305 2.558 2.839 3.152 3.498 3.883 4.310 4.785 5.311 5.895 6.543 7.263 8.062 8.949 9.933 11.026 12.239 13.585 22.892 38.574 64.999 109.527 184.559

12% 1.120 1.254 1.405 1.574 1.762 1.974 2.211 2.476 2.773 3.106 3.479 3.896 4.363 4.887 5.474 6.130 6.866 7.690 8.613 9.646 10.804 12.100 12.552 15.178 17.000 29.960 52.799 93.049 163.985 288.996

13% 1.130 1.277 1.443 1.630 1.842 2.082 2.353 2.658 3.004 3.395 3.836 4.334 4.898 5.535 6.254 7.067 7.986 9.024 10.197 11.523 13.021 14.713 16.626 18.788 21.230 39.115 72.066 132.776 244.629 450.711

14% 1.140 1.300 1.482 1.689 1.925 2.195 2.502 2.853 3.252 3.707 4.226 4.818 5.492 6.261 7.138 8.137 9.276 10.575 12.055 13.743 15.667 17.861 20.361 23.212 26.461 50.949 98.097 188.876 363.662 700.197

15% 1.150 1.322 1.521 1.749 2.011 2.313 2.660 3.059 3.518 4.046 4.652 5.350 6.153 7.076 8.137 9.358 10.761 12.375 14.232 16.366 18.821 21.644 24.891 28.625 32.918 66.210 133.172 267.856 538.752 1,083.619

Modified Date: Tue, Jul 06, 2010 12:34:49 PM

Output Date: Tue, Jul 06, 2010 05:11:36 PM

(Continued)

16% 1.160 1.346 1.561 1.811 2.100 2.436 2.826 3.278 3.803 4.411 5.117 5.936 6.886 7.987 9.265 10.748 12.468 14.462 16.776 19.461 22.574 26.186 30.376 35.236 40.874 85.849 180.311 378.715 795.429 1,670.669

Project: Management Accounting_Debarshi Bhattacharyya File: X:\Pearson\Management Accounting_Debarshi Bhattacharyya\MAIN\Z01\Z01_DEBA_ISBN_EN_SE_APPENDIX.indd ACE Pro India Pvt. Ltd.

Rev-II

Year 1 2 3 4 5 6 7 8 9 10 11 12 13 14 15 16 17 18 19 20 21 22 23 24 25 30 35 40 45 50

Table 1

17% 1.170 1.369 1.602 1.874 2.192 2.565 3.001 6.511 4.108 4.807 5.624 6.580 7.699 9.007 10.539 12.330 14.426 16.879 19.748 23.105 27.033 31.629 37.005 43.296 50.658 111.061 243.495 533.846 1,170.425 2,566.080

18% 1.180 1.392 1.643 1.939 2.288 2.700 3.185 3.759 4.435 5.234 6.176 7.288 8.599 10.147 11.974 14.129 16.672 19.673 23.214 27.393 32.323 38.141 45.007 53.108 62.667 143.367 327.988 750.353 1,716.619 3,927.189

Compounded Sum of One Rupee 19% 1.190 1.416 1.685 2.005 2.386 2.840 3.379 4.021 4.785 5.695 6.777 8.064 9.596 11.420 13.589 16.171 19.244 22.900 27.251 32.429 38.591 45.923 54.648 65.031 77.387 184.672 440.691 1,051.642 2,509.583 5,988.730

20% 1.200 1.440 1.728 2.074 2.488 2.986 3.583 4.300 5.160 6.192 7.430 8.916 10.699 12.839 15.407 18.488 22.186 26.623 31.948 38.337 46.005 55.205 66.247 79.496 95.395 237.373 590.657 1,469.740 3,657.176 9,100.191

21% 1.210 1.464 1.772 2.144 2.594 3.138 3.797 4.595 5.560 6.727 8.140 9.850 11.918 14.421 17.449 21.113 25.547 30.912 37.404 45.258 54.762 66.262 80.179 97.017 117.388 304.417 789.716 2,048.309 5,312.758 13,779.844

22% 1.220 1.488 1.816 2.215 2.703 3.297 4.023 4.908 5.987 7.305 8.912 10.872 13.264 16.182 19.742 24.089 29.384 35.848 43.735 53.357 65.095 79.416 96.887 118.203 144.210 389.748 1,053.370 2,846.941 7,694.418 20,795.680

23% 1.230 1.513 1.861 2.289 2.815 3.463 4.259 5.239 6.444 7.926 9.749 11.991 14.749 18.141 22.314 27.446 33.759 41.523 51.074 62.821 77.269 95.041 116.901 143.788 176.859 497.904 1,401.749 3,946.340 11,110.121 31,278.301

Modified Date: Tue, Jul 06, 2010 12:34:49 PM

Output Date: Tue, Jul 06, 2010 05:11:36 PM

(Continued)

24% 1.240 1.538 1.907 2.364 2.932 3.635 4.508 5.589 6.931 8.594 10.657 13.215 16.386 20.319 25.195 31.242 38.740 48.038 59.567 73.863 91.591 113.572 140.831 174.630 261.542 634.810 1,861.020 5,455.797 15,994.316 46,889.207

Project: Management Accounting_Debarshi Bhattacharyya File: X:\Pearson\Management Accounting_Debarshi Bhattacharyya\MAIN\Z01\Z01_DEBA_ISBN_EN_SE_APPENDIX.indd ACE Pro India Pvt. Ltd.

Rev-II

Year 1 2 3 4 5 6 7 8 9 10 11 12 13 14 15 16 17 18 19 20 21 22 23 24 25 30 35 40 45 50

Table 1

25% 1.250 1.562 1.953 2.441 3.052 3.815 4.768 5.960 7.451 9.313 11.642 14.552 18.190 22.737 28.422 35.527 44.409 55.511 69.389 86.736 108.420 135.525 169.407 211.758 264.698 807.793 2,465.189 5,723.156 22,958.844 70,064.812

26% 1.260 1.588 2.000 2.520 3.176 4.001 5.042 6.353 8.004 10.086 12.708 16.012 20.175 25.420 32.030 40.357 50.850 64.071 80.730 101.720 128.167 161.490 203.477 256.381 323.040 1,025.904 3,258.053 10,364.879 32,859.457 1,04,354.562

Compounded Sum of One Rupee 27% 1.270 1.613 2.048 2.601 3.304 4.196 5.329 6.767 8.595 10.915 13.862 17.605 22.359 28.395 36.062 45.799 58.165 73.869 93.813 119.143 151.312 192.165 244.050 309.943 393.628 1,300.477 4,296.547 14,195.051 46,897.973 1,54,942.687

28% 1.280 1.638 2.097 2.684 3.436 4.398 5.629 7.206 9.223 11.806 15.112 19.343 24.759 31.691 40.565 51.923 66.461 85.070 108.890 139.379 178.405 228.358 292.298 374.141 478.901 1,645.488 5,653.840 19,426.418 66,748.500 2,29,345.875

29% 1.290 1.664 2.147 2.769 3.572 4.608 5.945 7.669 9.893 12.761 16.462 21.236 27.395 35.339 45.587 58.808 75.862 97.862 126.242 162.852 210.079 271.002 349.592 450.974 581.756 2,078.208 7,423.988 26,520.723 94,739.937 3,38,440.000

30% 1.300 1.690 2.197 2.856 3.713 4.827 6.275 8.157 10.604 13.876 17.921 23.298 30.287 39.373 51.185 66.541 86.503 112.454 146.190 190.047 247.061 321.178 417.531 542.791 705.627 2,619.936 9,727.598 36,117.754 1,34,102.187 4,97,910.125

31% 1.310 1.716 2.248 2.945 3.858 5.054 6.621 8.673 11.362 14.884 19.498 25.542 33.460 43.832 57.420 75.220 98.539 129.086 169.102 221.523 290.196 380.156 498.044 652.385 854.623 3,297.081 12,719.918 49,072.621

Modified Date: Tue, Jul 06, 2010 12:34:49 PM

Output Date: Tue, Jul 06, 2010 05:11:36 PM

(Continued)

32% 1.320 1.742 2.300 3.036 4.007 5.290 6.983 9.217 12.166 16.060 21.199 27.982 36.937 48.756 64.358 84.953 112.138 148.022 195.389 257.913 340.446 449.388 593.192 783.504 1,033.577 4,142.008 16,598.906 66,519.313

Project: Management Accounting_Debarshi Bhattacharyya File: X:\Pearson\Management Accounting_Debarshi Bhattacharyya\MAIN\Z01\Z01_DEBA_ISBN_EN_SE_APPENDIX.indd ACE Pro India Pvt. Ltd.

Rev-II

Year 1 2 3 4 5 6 7 8 9 10 11 12 13 14 15 16 17 18 19 20 21 22 23 24 25 30 35 40

Table 1

33% 1.330 1.769 2.353 3.129 4.162 5.535 7.361 9.791 13.022 17.319 23.034 30.635 40.745 54.190 72.073 95.857 127.490 169.561 225.517 299.937 398.916 530.558 705.642 938.504 1,248.210 5,194.516 21,617.363 89,962.188

34% 1.340 1.769 2.406 3.224 4.320 5.789 7.758 10.395 13.930 18.666 25.012 33.516 44.912 60.181 80.643 108.061 144.802 194.035 260.006 348.408 466.867 625.601 838.305 1,123.328 1,505.528 6,503.285 28,097.695 1,21,388.437

Compounded Sum of One Rupee 35% 1.350 1.822 2.460 3.312 4.484 6.053 8.172 11.032 14.894 20.106 27.144 36.644 49.469 66.784 90.158 121.713 164.312 221.822 299.459 404.270 545.764 736.781 994.653 1,342.781 1,812.754 8,128.426 36,448.051 1,63,433.875

36% 1.360 1.850 2.515 3.421 4.653 6.328 8.605 11.703 15.917 21.646 29.439 40.037 54.451 74.053 100.712 136.968 186.277 253.337 344.537 468.571 637.256 866.668 1,178.668 1,602.988 2,180.063 10,142.914 47,190.727 2,19,558.625

37% 1.370 1.877 2.571 3.523 4.826 6.612 9.058 12.410 17.001 23.292 31.910 43.716 59.892 82.051 112.410 154.002 210.983 289.046 395.993 542.511 743.240 1,018.238 1,394.986 1,911.129 2,618.245 12,636.086 60,983.836 2,94,317.937

38% 1.380 1.904 2.628 3.627 5.005 6.907 9.531 13.153 18.151 25.049 34.567 47.703 65.830 90.845 125.366 173.005 238.747 329.471 454.669 627.443 865.781 1,194.900 1,648.961 2,275.564 3,140.275 15,716.703 78,660.188 3,93,684.687

39% 1.390 1.932 2.686 3.733 5.189 7.213 10.025 13.935 19.370 26.924 37.425 52.020 72.308 100.509 139.707 194.192 269.927 375.198 521.525 724.919 1,007.637 1,400.615 1,946.854 2,706.125 3,761.511 19,517.969 1,01,276.125 5,25,508.312

40% 1.400 1.960 2.744 3.842 5.378 7.530 10.541 14.758 20.661 28.925 40.495 56.694 79.371 111.119 155.567 217.793 304.911 426.875 597.625 836.674 1,171.343 1,639.878 2,295.829 3,214.158 3,499.816 24,201.043 1,30,158.687 7,00,022.688

Project: Management Accounting_Debarshi Bhattacharyya File: X:\Pearson\Management Accounting_Debarshi Bhattacharyya\MAIN\Z01\Z01_DEBA_ISBN_EN_SE_APPENDIX.indd

Modified Date: Tue, Jul 06, 2010 12:34:49 PM

Output Date: Tue, Jul 06, 2010 05:11:36 PM

ACE Pro India Pvt. Ltd.

Rev-II

Year 1 2 3 4 5 6 7 8 9 10 11 12 13 14 15 16 17 18 19 20 21 22 23 24 25 30 35 40 45 50

Table 2 1% 0.990 0.980 0.971 0.961 0.951 0.942 0.933 0.923 0.914 0.905 0.896 0.887 0.879 0.870 0.861 0.853 0.844 0.836 0.828 0.820 0.811 0.803 0.795 0.788 0.780 0.742 0.706 0.672 0.639 0.806

2% 0.980 0.961 0.942 0.924 0.906 0.888 0.871 0.853 0.837 0.820 0.804 0.789 0.773 0.758 0.743 0.728 0.714 0.700 0.686 0.673 0.660 0.647 0.634 0.622 0.610 0.552 0.500 0.453 0.410 0.372

The Present Value of One Rupee 3% 0.971 0.943 0.915 0.888 0.863 0.837 0.813 0.789 0.766 0.744 0.722 0.701 0.681 0.661 0.642 0.623 0.605 0.587 0.570 0.554 0.538 0.522 0.507 0.492 0.478 0.412 0.355 0.307 0.264 0.228

4% 0.962 0.925 0.889 0.855 0.822 0.790 0.760 0.731 0.703 0.676 0.650 0.625 0.601 0.577 0.555 0.534 0.513 0.494 0.475 0.456 0.439 0.422 0.406 0.390 0.375 0.308 0.255 0.203 0.171 0.141

5% 0.952 0.907 0.864 0.823 0.784 0.746 0.711 0.677 0.645 0.614 0.585 0.557 0.530 0.505 0.481 0.458 0.436 0.416 0.396 0.377 0.359 0.342 0.326 0.310 0.295 0.231 0.181 0.142 0.111 0.087

6% 0.943 0.890 0.840 0.792 0.747 0.705 0.665 0.627 0.592 0.558 0.527 0.497 0.469 0.442 0.417 0.394 0.371 0.350 0.331 0.312 0.294 0.278 0.262 0.247 0.233 0.174 0.130 0.097 0.073 0.054

7% 0.935 0.873 0.816 0.763 0.713 0.666 0.623 0.582 0.544 0.508 0.475 0.444 0.415 0.388 0.362 0.339 0.317 0.296 0.227 0.258 0.242 0.226 0.211 0.197 0.184 0.131 0.094 0.067 0.048 0.034

8% 0.926 0.857 0.794 0.735 0.681 0.630 0.583 0.540 0.500 0.463 0.429 0.397 0.368 0.340 0.315 0.292 0.270 0.250 0.232 0.215 0.199 0.184 0.170 0.158 0.146 0.099 0.068 0.046 0.031 0.021

9% 0.917 0.842 0.772 0.708 0.650 0.596 0.547 0.502 0.460 0.422 0.388 0.356 0.326 0.299 0.275 0.252 0.231 0.212 0.194 0.178 0.164 0.150 0.138 0.126 0.116 0.075 0.049 0.032 0.021 0.013

Modified Date: Tue, Jul 06, 2010 12:34:49 PM

Output Date: Tue, Jul 06, 2010 05:11:36 PM

(Continued)

10% 0.909 0.826 0.751 0.683 0.621 0.564 0.513 0.467 0.424 0.386 0.350 0.319 0.290 0.263 0.239 0.218 0.198 0.180 0.164 0.149 0.135 0.123 0.112 0.102 0.092 0.057 0.036 0.022 0.014 0.009

Project: Management Accounting_Debarshi Bhattacharyya File: X:\Pearson\Management Accounting_Debarshi Bhattacharyya\MAIN\Z01\Z01_DEBA_ISBN_EN_SE_APPENDIX.indd ACE Pro India Pvt. Ltd.

Rev-II

Year 1 2 3 4 5 6 7 8 9 10 11 12 13 14 15 16 17 18 19 20 21 22 23 24 25 30 35 40 45 50

Table 2 11% 0.901 0.812 0.731 0.659 0.593 0.535 0.482 0.434 0.391 0.352 0.317 0.286 0.258 0.232 0.209 0.188 0.170 0.153 0.138 0.124 0.112 0.101 0.091 0.082 0.074 0.044 0.026 0.015 0.009 0.005

12% 0.893 0.797 0.712 0.636 0.567 0.507 0.452 0.404 0.361 0.322 0.287 0.257 0.229 0.205 0.183 0.163 0.146 0.130 0.116 0.104 0.093 0.083 0.074 0.066 0.059 0.033 0.019 0.011 0.006 0.003

The Present Value of One Rupee 13% 0.885 0.783 0.693 0.613 0.543 0.480 0.425 0.376 0.333 0.295 0.261 0.231 0.204 0.181 0.160 0.141 0.125 0.111 0.098 0.087 0.077 0.068 0.060 0.053 0.047 0.026 0.014 0.008 0.004 0.002

14% 0.877 0.769 0.675 0.592 0.519 0.456 0.400 0.351 0.308 0.270 0.237 0.208 0.182 0.160 0.140 0.123 0.108 0.095 0.083 0.073 0.064 0.056 0.049 0.043 0.038 0.020 0.010 0.005 0.003 0.001

15% 0.870 0.756 0.658 0.572 0.497 0.432 0.376 0.327 0.284 0.247 0.215 0.187 0.163 0.141 0.123 0.107 0.093 0.081 0.070 0.061 0.053 0.046 0.040 0.035 0.030 0.015 0.008 0.004 0.002 0.001

16% 0.862 0.743 0.641 0.552 0.476 0.410 0.354 0.305 0.263 0.227 0.195 0.168 0.145 0.125 0.108 0.093 0.080 0.069 0.060 0.051 0.044 0.038 0.033 0.028 0.024 0.012 0.006 0.003 0.001 0.001

17% 0.855 0.731 0.624 0.534 0.456 0.390 0.333 0.285 0.243 0.208 0.178 0.152 0.130 0.111 0.095 0.081 0.069 0.059 0.051 0.043 0.037 0.032 0.027 0.023 0.020 0.009 0.004 0.002 0.001 0.000

18% 0.847 0.718 0.609 0.516 0.437 0.370 0.314 0.266 0.225 0.191 0.162 0.137 0.116 0.099 0.084 0.071 0.060 0.051 0.043 0.037 0.031 0.026 0.022 0.019 0.016 0.007 0.003 0.001 0.001 0.000

19% 0.840 0.706 0.593 0.499 0.419 0.352 0.296 0.249 0.209 0.176 0.148 0.124 0.104 0.088 0.074 0.062 0.044 0.044 0.037 0.031 0.026 0.022 0.018 0.015 0.013 0.005 0.002 0.001 0.000 0.000

Modified Date: Tue, Jul 06, 2010 12:34:49 PM

Output Date: Tue, Jul 06, 2010 05:11:36 PM

(Continued)

20% 0.833 0.694 0.579 0.482 0.402 0.335 0.279 0.233 0.194 0.162 0.135 0.112 0.093 0.078 0.065 0.054 0.038 0.038 0.031 0.026 0.022 0.018 0.015 0.013 0.010 0.004 0.002 0.001 0.000 0.000

Project: Management Accounting_Debarshi Bhattacharyya File: X:\Pearson\Management Accounting_Debarshi Bhattacharyya\MAIN\Z01\Z01_DEBA_ISBN_EN_SE_APPENDIX.indd ACE Pro India Pvt. Ltd.

Rev-II

Year 1 2 3 4 5 6 7 8 9 10 11 12 13 14 15 16 17 18 19 20 21 22 23 24 25 30 35 40 45 50

Table 2 21% 0.826 0.683 0.564 0.467 0.386 0.319 0.263 0.218 0.180 0.149 0.123 0.102 0.084 0.069 0.057 0.047 0.039 0.032 0.027 0.022 0.018 0.015 0.012 0.010 0.009 0.003 0.001 0.000 0.000 0.000

22% 0.820 0.672 0.551 0.451 0.370 0.303 0.249 0.204 0.167 0.137 0.112 0.092 0.075 0.062 0.051 0.042 0.034 0.028 0.023 0.019 0.015 0.013 0.010 0.008 0.007 0.003 0.001 0.000 0.000 0.000

The Present Value of One Rupee 23% 0.813 0.661 0.537 0.437 0.355 0.289 0.235 0.191 0.155 0.126 0.103 0.083 0.068 0.055 0.045 0.036 0.030 0.024 0.020 0.016 0.013 0.011 0.009 0.007 0.006 0.002 0.001 0.000 0.000 0.000

24% 0.806 0.650 0.524 0.423 0.341 0.275 0.222 0.179 0.114 0.116 0.094 0.076 0.061 0.049 0.040 0.032 0.026 0.021 0.017 0.014 0.011 0.009 0.007 0.006 0.005 0.002 0.001 0.000 0.000 0.000

25% 0.800 0.640 0.512 0.410 0.328 0.262 0.210 0.168 0.134 0.107 0.086 0.069 0.055 0.044 0.035 0.028 0.023 0.018 0.014 0.012 0.009 0.007 0.006 0.005 0.004 0.001 0.000 0.000 0.000 0.000

26% 0.794 0.630 0.500 0.397 0.315 0.250 0.198 0.157 0.125 0.099 0.079 0.062 0.050 0.039 0.031 0.025 0.020 0.016 0.012 0.010 0.008 0.006 0.005 0.004 0.003 0.001 0.000 0.000 0.000 0.000

27% 0.787 0.620 0.488 0.384 0.303 0.238 0.188 0.148 0.116 0.092 0.072 0.057 0.045 0.035 0.028 0.022 0.017 0.014 0.011 0.008 0.007 0.005 0.004 0.003 0.003 0.001 0.000 0.000 0.000 0.000

28% 0.781 0.610 0.477 0.373 0.291 0.227 0.178 0.139 0.108 0.085 0.066 0.052 0.040 0.032 0.025 0.019 0.015 0.012 0.009 0.007 0.006 0.004 0.003 0.003 0.002 0.001 0.000 0.000 0.000 0.000

29% 0.775 0.601 0.466 0.361 0.280 0.217 0.168 0.130 0.101 0.078 0.061 0.047 0.037 0.028 0.022 0.017 0.013 0.010 0.008 0.006 0.005 0.004 0.003 0.002 0.002 0.000 0.000 0.000 0.000 0.000

Modified Date: Tue, Jul 06, 2010 12:34:49 PM

Output Date: Tue, Jul 06, 2010 05:11:36 PM

(Continued)

30% 0.769 0.592 0.455 0.350 0.269 0.207 0.159 0.123 0.094 0.073 0.056 0.043 0.033 0.025 0.020 0.015 0.012 0.009 0.007 0.005 0.004 0.003 0.002 0.002 0.001 0.000 0.000 0.000 0.000 0.000

Project: Management Accounting_Debarshi Bhattacharyya File: X:\Pearson\Management Accounting_Debarshi Bhattacharyya\MAIN\Z01\Z01_DEBA_ISBN_EN_SE_APPENDIX.indd ACE Pro India Pvt. Ltd.

Rev-II

Year 1 2 3 4 5 6 7 8 9 10 11 12 13 14 15 16 17 18 19 20 21 22 23 24 25 30 35 40 45 50

Table 2 31% 0.763 0.583 0.445 0.340 0.259 0.198 0.151 0.115 0.088 0.067 0.051 0.039 0.030 0.023 0.017 0.013 0.010 0.008 0.006 0.005 0.003 0.003 0.002 0.002 0.001 0.000 0.000 0.000 0.000 0.000

32% 0.758 0.574 0.435 0.329 0.250 0.189 0.143 0.108 0.082 0.062 0.047 0.036 0.027 0.021 0.016 0.012 0.009 0.007 0.005 0.004 0.003 0.002 0.002 0.001 0.001 0.000 0.000 0.000 0.000 0.000

The Present Value of One Rupee 33% 0.752 0.565 0.425 0.320 0.240 0.181 0.136 0.102 0.077 0.058 0.043 0.033 0.025 0.018 0.014 0.010 0.008 0.006 0.004 0.003 0.003 0.002 0.001 0.001 0.001 0.000 0.000 0.000 0.000 0.000

34% 0.746 0.557 0.416 0.310 0.231 0.173 0.129 0.096 0.072 0.054 0.040 0.030 0.022 0.017 0.012 0.009 0.007 0.005 0.004 0.003 0.002 0.002 0.001 0.001 0.001 0.000 0.000 0.000 0.000 0.000

35% 0.741 0.549 0.406 0.301 0.223 0.165 0.122 0.091 0.067 0.050 0.037 0.027 0.020 0.015 0.011 0.008 0.006 0.005 0.003 0.002 0.002 0.001 0.001 0.001 0.001 0.000 0.000 0.000 0.000 0.000

36% 0.735 0.541 0.398 0.292 0.215 0.158 0.116 0.085 0.063 0.046 0.034 0.025 0.018 0.014 0.010 0.007 0.005 0.004 0.003 0.002 0.002 0.001 0.001 0.001 0.001 0.000 0.000 0.000 0.000 0.000

37% 0.730 0.533 0.389 0.284 0.207 0.151 0.110 0.081 0.059 0.043 0.031 0.023 0.017 0.012 0.009 0.006 0.005 0.003 0.003 0.002 0.001 0.001 0.001 0.001 0.000 0.000 0.000 0.000 0.000 0.000

38% 0.725 0.525 0.381 0.276 0.200 0.145 0.105 0.076 0.055 0.040 0.029 0.021 0.015 0.011 0.008 0.006 0.004 0.008 0.002 0.002 0.001 0.001 0.001 0.000 0.000 0.000 0.000 0.000 0.000 0.000

39% 0.719 0.518 0.372 0.268 0.193 0.139 0.100 0.072 0.052 0.037 0.027 0.019 0.014 0.010 0.007 0.005 0.004 0.003 0.002 0.001 0.001 0.001 0.001 0.000 0.000 0.000 0.000 0.000 0.000 0.000

40% 0.714 0.510 0.364 0.260 0.186 0.133 0.095 0.068 0.048 0.035 0.025 0.018 0.013 0.009 0.006 0.005 0.003 0.002 0.002 0.001 0.001 0.001 0.001 0.000 0.000 0.000 0.000 0.000 0.000 0.000

Project: Management Accounting_Debarshi Bhattacharyya File: X:\Pearson\Management Accounting_Debarshi Bhattacharyya\MAIN\Z01\Z01_DEBA_ISBN_EN_SE_APPENDIX.indd

Modified Date: Tue, Jul 06, 2010 12:34:49 PM

Output Date: Tue, Jul 06, 2010 05:11:36 PM

ACE Pro India Pvt. Ltd.

Rev-II